Sei sulla pagina 1di 918

2017-18 100 &

op kers
Class 12 T
By E ran culty
-JE Fa r
IIT enior emie .
S fP r es
o titut
Ins

MATHEMATICS
FOR JEE MAIN & ADVANCED
SECOND
EDITION

Exhaustive Theory
(Now Revised)

Formula Sheet
9000+ Problems
based on latest JEE pattern

2500 + 1000 (New) Problems


of previous 35 years of
AIEEE (JEE Main) and IIT-JEE (JEE Adv)

5000+Illustrations and Solved Examples


Detailed Solutions
of all problems available

Topic Covered Plancess Concepts


Tips & Tricks, Facts, Notes, Misconceptions,
Matrices Key Take Aways, Problem Solving Tactics

PlancEssential
Questions recommended for revision
16. M AT R I C E S

1. INTRODUCTION
A rectangular array of m x n numbers (real or complex) in the form of m horizontal lines (called rows) and n vertical
lines (called columns), is called a matrix of order m by n, written as m × n matrix. Such an array is enclosed by [ ] or
( ) or || ||. An m × n matrix is usually written as
 a11 a12 ...... a1n 
 
a a22 ...... a2n 
A =  21
     
 
am1 am2 ...... amn 

In brief, the above matrix is represented by A = [aij]m×n. The number a11, a12, …… etc., are known as the elements of
the matrix A, where aij belongs to the ith row and jth column and is called the (i, j)th element of the matrix A = [aij].

2. ORDER OF A MATRIX
3 −1 5 
A matrix which has m rows and n columns is called a matrix of order m × n E.g. the order of   matrix
is 2 × 3.  6 2 −7 

Note: (a) The difference between a determinant and a matrix is that a determinant has a certain value, while the
matrix has none. The matrix is just an arrangement of certain quantities.
(a) The elements of a matrix may be real or complex numbers. If all the elements of a matrix are real, then the matrix
is called a real matrix.
(a) An m × n matrix has m.n elements.

Illustration 1: Construct a 3×4 matrix A = [aij], whose elements are given by aij = 2i + 3j.  (JEE MAIN)

Sol: In this problem, i and j are the number of rows and columns respectively. By substituting the respective values of
rows and columns in aij = 2i + 3j we can construct the required matrix.
 a11 a12 a13 a14 
 
We have A = a21 a22 a23 a24  ; ∴ a11 = 2 × 1 + 3 × 1 = 5; a12 = 2 × 1 + 3 × 2 = 8
a a32 a33 a34 
 31
Similarly, a13 = 11, a14 = 14, a21 = 7, a22 = 10, a23 = 13, a24 = 16, a31 = 9, a32 = 12, a33 = 15, a34 = 18

5 8 11 14 
∴ A = 7 10 13 16 
9 12 15 18 
 
1 6 . 2 | Matrices

1
Illustration 2: Construct a 3 × 4 matrix, whose elements are given by: aij = | –3i + j |  (JEE MAIN)
2
Sol: Method for solving this problem is the same as in the above problem.
1
Since aij = | –3i + j | we have
2
1 1 1 2
a11 = | –3(1) + 1 | = | – 3 + 1 | = | – 2 | = = 1
2 2 2 2
1 1 1 1
a12 = | –3(1) + 2 | = | – 3 + 2 | = | – 1 | =
2 2 2 2
1 1 1
a13 = | –3(1) + 3 | = | – 3 + 3 | = (0) = 0
2 2 2
1 1 1 1 1 5
a14 = | –3(1) + 4 | = | – 3 + 4 | = ; a21 = | –3(2) + 1 | = | – 6 + 1 | =
2 2 2 2 2 2
1 1 4 1 1 3
a22 = | –3(2) + 2 | = | – 6 + 2 | = = 2; a23 = | –3(2) + 3 | = | – 6 + 3 | =
2 2 2 2 2 2
1 1 2 7 5
a24 = | –3(2) + 4 | = | – 6 + 4 | = = 1; Similarly a31 = 4, a32 = , a33 = 3, a34 =
2 2 2 2 2
 1 1
1 0 
 2 2
5 3
Hence, the required matrix is given by A =  2 1 
2 2
 
4 7 3 5
 2 2 

3. TYPES OF MATRICES

3.1 Row Matrix


A matrix having only one row is called a row matrix. Thus A = [aij]m×n is a row matrix if m = 1;
E.g. A = [1 2 4 5] is row matrix of order 1 × 4.

3.2 Column Matrix


A matrix having only one column is called a column matrix. Thus A = [aij]m×n is a column matrix if
 −1 
 
2
n = 1; E.g. A =   is column matrix of order 4 × 1.
 −4 
 
 5 

3.3 Zero or Null Matrix


If in a matrix all the elements are zero then it is called a zero matrix and it is generally denoted by 0.Thus, A = [aij]
0 0 0 
m×n
is a zero matrix if aij = 0 for all i and j; E.g.   is a zero matrix of order 2 × 3.
0 0 0 
0 0  0 0 0 
   
A = 0 0  is a 3 × 2 null matrix & B = 0 0 0  is 3 × 3 null matrix.
0 0  0 0 0 
   
M a them a ti cs | 16.3

3.4 Singleton Matrix


If in a matrix there is only one element then it is called singleton matrix. Thus, A = [aij]m×n is a singleton matrix if m
= n = 1. E.g. [2], [3], [a], [–3] are singleton matrices.

3.5 Horizontal Matrix


1 2 3 4 
A matrix of order m × n is a horizontal matrix if n > m; E.g.  
2 5 1 1 

3.6 Vertical Matrix 2 5


 
1 1
A matrix of order m × n is a vertical matrix if m > n; E.g. 
3 6
 
2 4 
3.7 Square Matrix
If the number of rows and the number of columns in a matrix are equal, then it is called a square matrix.
 a11 a12 a13 
 
Thus, A = [aij]m×n is a square matrix if m = n; E.g. a21 a22 a23  is a square matrix of order 3 × 3.
a a32 a33 
 31
The sum of the diagonal elements in a square matrix A is called the trace of matrix A, and which is denoted by
n
tr(A); tr(A) = ∑ aii = a 11
+ a22 + ….. ann.
i=1

3.8 Diagonal Matrix


If all the elements, except the principal diagonal, in a square matrix are zero, it is called a diagonal matrix.
2 0 0 
 
Thus, a square matrix A = [aij] is a diagonal matrix if aij = 0, when i ≠ j; E.g. 0 3 0  is a diagonal matrix of order
3 × 3, which can also be denoted by diagonal [2 3 4]. 0 0 4 
 

3.9 Scalar Matrix


If all the elements in the diagonal of a diagonal matrix are equal, it is called a scalar matrix. Thus, a square matrix A
0, i ≠ j
= [aij]n×n is a scalar matrix if aij =  where k is a constant.
k, i = j
 −7 0 0 
 
E.g.  0 −7 0  is a scalar Matrix.
 0 0 −7 
 

3.10 Unit Matrix


If all the elements of a principal diagonal in a diagonal matrix are 1, then it is called a unit matrix. A unit matrix of
order n is denoted by In. Thus, a square matrix A = [aij]n×n is a unit matrix if
1 0 0 
1, i = j  
aij =  E.g. I3 = 0 1 0 
0, i ≠ j 0 0 1 
 
Note: Every unit matrix is a scalar matrix.
1 6 . 4 | Matrices

3.11 Triangular Matrix


A square matrix is said to be a triangular matrix if the elements above or below the principal diagonal are zero.
There are two types:

3.11.1 Upper Triangular Matrix


A square matrix [aij] is called an upper triangular matrix, If aij = 0, when i > j.

3 1 2 
 
E.g. 0 4 3  is an upper uriangular matrix of order 3 × 3.
0 0 6 
 

3.11.2 Lower Triangular Matrix


A square matrix is called a lower triangular matrix, if aij = 0 when i < j.

1 0 0 
 
E.g.  2 3 0  is a lower triangular matrix of order 3 × 3.
 4 5 2
 

3.12 Singular Matrix


Matrix A is said to be a singular matrix if its determinant | A | = 0, otherwise a non-singular matrix, i.e.
If det | A | = 0 ⇒ Singular and det | A | ≠ 0 ⇒ non-singular

PLANCESS CONCEPTS

• A triangular matrix A = [aij]n×n is called strictly triangular if aij = 0 ∀ i=j


• The multiplication of two triangular matrices is a triangular matrix.
 very row matrix is also a horizontal matrix but not the converse. Similarly every column matrix is also
• E
a vertical matrix but not the converse.
Vaibhav Gupta (JEE 2009 AIR 54)

3.13 Symmetric and Skew Symmetric Matrices


Symmetric Matrix: A square matrix A = [aij] is called a symmetric matrix if aij = aji, for all i,j values;

1 2 3
 
E.g. A =  2 4 5  is symmetric, because a12 = 2 = a21, a31 = 3 = a13 etc.
3 5 2
 
Note: A is symmetric ⇔ A = A ’(where A’ is the transpose of matrix)
Skew-Symmetric Matrix: A square matrix A = [aij] is a skew-symmetric matrix if aij = –aji, for all values of i,j.
 aij = –aji, for all i,j. ⇒ aii = –aii, [putting j = i] ⇒ 2aii = 0 ⇒ aii = 0
0 2 1
   0 2
Thus, in a skew-symmetric matrix all diagonal elements are zero; E.g. A =  −2 0 −3 , B =   are skew-
symmetric matrices.  −2 0 
 −1 3 0 
 
Note: A square matrix A is a skew-symmetric matrix ⇔ A’ = –A.
M a them a ti cs | 16.5

Few results:
(a) If A is any square matrix, then A + A’ is a symmetric matrix and A – A’ is a skew-symmetric matrix.
(b) Every square matrix can be uniquely expressed as the sum of a symmetric matrix and a skew-symmetric
1 1 1
matrix. A = (A + A’) + (A – A’) = (B + C), where B is symmetric and C is a skew symmetric matrix.
2 2 2
(c) If A and B are symmetric matrices, then AB is symmetric ⇔ AB = BA, i.e. A & B commute.
(d) The matrix B’AB is symmetric or skew-symmetric in correspondence if A is symmetric or skew-symmetric.
(e) All positive integral powers of a symmetric matrix are symmetric.
(f) Positive odd integral powers of a skew-symmetric matrix are skew-symmetric and positive even integral
powers of a skew-symmetric matrix are symmetric.

PLANCESS CONCEPTS

Elements of the main diagonal of a skew-symmetric matrix are zero because by definition aii = – aii ⇒ 2aii =
0 or aii = 0 for all values of i.
Trace of a skew symmetric matrix is always 0. The sum of symmetric matrices is symmetric.
Every square matrix can be uniquely expressed as the sum of a symmetric matrix and a skew-symmetric
1 1 1
matrix A = ( A + A’) + ( A – A’) = (B + C), where B is symmetric and C is a skew symmetric matrix.
2 2 2
If A and B are symmetric matrices, then AB is symmetric ⇔ AB = BA, i.e. A & B commute. The matrix
B’AB is symmetric or skew-symmetric accordingly when A is symmetric or skew symmetric. All positive
integral powers of a symmetric matrix are symmetric. Positive odd integral powers of a skew-symmetric
matrix are skew-symmetric and positive even integral powers of a skew-symmetric matrix are symmetric.
Chen Reddy Sandeep Reddy (JEE 2012 AIR 62)

3.14 Hermitian and Skew-Hermitian Matrices


A square matrix A = [aij] is said to be a Hermitian matrix if aij= aji ∀ i, j; i.e. A = Aq

 3 3 − 4i 5 + 2i 
 a b + ic   
E.g.   . 3 + 4i 5 −2 + i are Hermitian matrices
b − ic d 
5 − 2i −2 − i 2 

Note: (a) If A is a Hermitian matrix then aii = aii ⇒ aii is real ∀ i, thus every diagonal element of a Hermitian matrix
must be real.
(b) If a Hermitian matrix over the set of real numbers is actually a real symmetric matrix; and A a square matrix,
A = [aij] is said to be a skew-Hermitian if aij = – aji , ∀ i, j;

 3i −3 + 2i −1 − i 
 0 −2 + i  
i.e. A = – A; E.g. 
θ
.  3 − 2i −2i −2 − 4i  are skew-Hermitian matrices.
2 − i 0   1 + i 2 + 4i 0 

(c) If A is a skew-Hermitian matrix then aii =– aii ⇒ aii + a = 0


ii

i.e. aii must be purely imaginary or zero.

(d) A skew-Hermitian matrix over the set of real numbers is actually is real skew-symmetric matrix.
1 6 . 6 | Matrices

4. TRACE OF A MATRIX
Let A = [aij]n×n and B = [bij]n×n and λ be a scalar,
(i) tr(λA) = λ tr(A) (ii) tr(A + B) = tr(A) + tr(B) (iii) tr(AB) = tr(BA)

Square Matrix

Triangular Matrix Diagonal Matrix


Atleast one, aii ≠ 0 and aij = 0 if i ≠ j
d1 0 0 
Upper triangular Lower triangular 0 d
2 0 
Matrix Matrix 
If aij = 0 ∀ i > j If aij = 0 ∀ i < j  0 0 d3 
x x x x 0 0 |
, Abbreviated as dia (d1, d2, d3, …….dn)
A = 0 x x  A = x x 0 |
0 0 x  x x x 
Scalar Matrix Unit Matrix
If d1=d2=d3 ….= a≠0 If d1=d2=d3 ….= 1
a 0 0 1 0 0
0 a 0 0 1 0 =I
    3
0 0 a 0 0 1 

5. TRANSPOSE OF A MATRIX
The matrix obtained from a given matrix A by changing its rows into columns or columns into rows is called the
transpose of matrix A and is denoted by AT or A’. From the definition it is obvious that if the order of A is m × n,
then the order of AT becomes n × m; E.g. transpose of matrix
 a1 b1 
 a1 a2 a3   

b b b
 is a2 b2 
 1 2 3  2×3 a b 
 3 3  3×2

5.1 Properties of Transpose of Matrix


(i) (AT)T = A (ii) (A ± B)T = AT ± BT (iii) (AB)T = BTAT (iv) (kA)T = k(A)T
(v) (A1 A2 A3 ..........An−1 An )T = AnT AnT−1 …….. A3T A2T A1T (vi) IT = I (vii) tr(A) = tr(AT)

 1 3
 1 −2 3  
Illustration 3: If A =   and B =  −1 0  then prove that (AB) = B A . 
T T T
(JEE MAIN)
 −4 2 5   2 4
 
Sol: By obtaining the transpose of AB i.e. (AB)T and multiplying BT and AT we can easily get the result.

 1 3
 1 −2 3    1(1) − 2( −1) + 3(2) 1(3) − 2(0) + 3(4)   9 15
Here, AB =    −1 0  =  = 
 −4 2 5   2 4   −4(1) + 2( −1) + 5(2) −4(3) + 2(0) + 5(4)  4 8 
 
 1 −4 
 9 4 1 −1 2     1(1) − 1( −2) + 2(3) 1( −4) − 1(2) + 2(5)   9 4 
∴ (AB)T =   ; B A
T T
=    −2 2  =   =  = (AB)
T

15 8  3 0 4   3 5  3(1) + 0( −2) + 4(3) 3( −4) + 0(2) + 4(5) 15 8 


 
M a them a ti cs | 16.7

5 −1 3 0 2 3 
Illustration 4: If A = 0 1 2 and B =   then what is (AB’)’ is equal to?  (JEE MAIN)
  1 −1 4 

Sol: In this problem, we use the properties of the transpose of matrix to get the required result.

 5 0
0 2 3     7 8
We have (AB′)’ = (B’)’ A’ = BA’ =    −1 1  =  
1 −1 4   3 2  18 7 
 

3 − x 2 2 
 
Illustration 5: If the matrix A =  2 4−x 1  is a singular matrix then find x. Verify whether AAT = I for that
value of x.   −2 −4 −1 − x  (JEE ADVANCED)

Sol: Using the condition of singular matrix, i.e. | A | = 0, we get the value of x and then substituting the value of x in
matrix A and multiplying it to its transpose we will obtain the required result.

3−x 2 2
Here, A is a singular matrix if | A | = 0, i.e., 2 4−x 1 =0
−2 −4 −1 − x

3−x 2 2 3−x 0 2
or 2 4 − x 1 = 0, using R3 → R3 + R2 or 2 3 − x 1 = 0, using C2 → C2 – C3
0 −x −x 0 0 −x

or – x(3 – x)2 = 0, ∴ x = 0, 3.

3 2 2 3 2 2 3 2 −2 
     
When x = 0, A =  2 4 1  ; ∴ AA =  2 4 1 
T
2 4 −4 
 −2 −4 −1  −2 −4 −1 2 1 −1 
     

 17 16 −16 
 
=  16 21 −21  ≠ I
 −16 −21 21 
 

0 2 2 0 2 2 0 2 −2   8 4 −16 
       
When x = 3,A =  2 1 1  ∴ AAT = 2 1 1 2 1 −4  =  4 6 −12  ≠ I
 −2 −4 −4   −2 −4 −4  2 1 −4   −16 −12 36 
       

Note: A simple way to solve is that if A is a singular matrix then |A| = 0 and |AT| = 0. But |I| is 1. Hence, AAT ≠ I if
|A| = 0.

a b c 
 
Illustration 6: If the matrix A = b c a  where a, b, c, are positive real numbers such that abc = 1 and ATA = I
 c a b
 
then find the value of a + b + c3.
3 3
(JEE ADVANCED)

a b c  a b c 
   
Sol: Here, A = b c a  . So, AT = b c a  , interchanging rows and columns.
 c a b  c a b
   
1 6 . 8 | Matrices

2
a b c 
 
∴ A A = b c a  = A2 ∴ | ATA | = | A2 |; But ATA = I (given). ∴ | I | = | A |2 ⇒ 1 = | A |2
T

 c a b
 

a b c 1 1 1
Now, | A | = b c a = (a + b + c) b c a , R1 → R1 + R2 + R3
c a b c a b

1 0 0
C → C2 − C1
= (a + b + c) b c − b a − b , 2
C3 → C3 − C1
c a−c b−c

= (a + b + c) {(c – b) (b – c) – (a – b) (a – c)} = (a + b + c) (– b2 – c2 + 2bc – a2 + ac + ab – bc)


= – (a + b + c) (a2 + b2 + c2 – bc – ca – ab) = – (a3 + b3 + c3 – 3 abc)
= – (a3 + b3 + c3 – 3) ( abc = 1) ∴ | A |2 = 1 ⇒ (a3 + b3 + c3 – 3)2 = 1  … (i)

a3 + b3 + c3 3 3 3 3
As a, b, c, are positive, > abc (∵ abc = 1); ∴ a3 + b3 + c3 > 3
3
∴ (i) ⇒ a3 + b3 + c3 – 3 = 1 ∴ a3 + b3 + c3 = 4

6. MATRIX OPERATIONS

6.1 Equality of Matrices

Two matrices A and B are said to be equal if they are of the same order and their corresponding elements are equal,
i.e. Two matrices A = [aij]m×n and B=[bij]r×s are equal if
(a) m = r i.e. the number of rows in A = the number of rows in B.
(b) n = s, i.e. the number of columns in A = the number of columns in B
(c) aij = bij, for i = 1, 2, …., m and j = 1, 2, ….., n, i.e. the corresponding elements are equal;

0 0  0 0 0 
E.g. Matrices   and   are not equal because their orders are not the same.
0 0  0 0 0 

1 6 3  a1 a2 a3 
E.g. If A =   and B=   are equal matrices then,
5 2 1  b1 b2 b3 

a1 = 1, a2 = 6, a3 = 3, b1 = 5, b2 = 2, b3 = 1.

6.2 Addition of Matrices


If A[aij]m×n and B[bij]m×n are two matrices of the same order then their sum A + B is a matrix, and each element of
that matrix is the sum of the corresponding elements. i.e. A + B = [aij + bij]m×n

Properties of Matrix Addition: If A, B and C are matrices of same order, then


(a) A + B = B + A (Commutative law),
(b) (A + B) + C = A + (B + C) (Associative law),
(c) A + O = O + A = A, where O is zero matrix which is additive identity of the matrix,
M a them a ti cs | 16.9

(d) A + (–A) = 0 = (–A) + A, where (–A) is obtained by changing the sign of every element of A which is additive
inverse of the matrix,

A + B = A + C
(e)   ⇒ B = C
B + A = C + A

(f) tr (A ± B) = tr (A) ± tr (B)


(g) Additive Inverse: If A + B = 0 = B + A, then B is called additive inverse of A and also A is called the additive
inverse of A.
(h) Existence of Additive Identity: Let A = [aij] be an m × n matrix and O be an m × n zero matrix, then
A + O = O + A = A. In other words, O is the additive identity for matrix addition.

6.3 Subtraction of Matrices


If A and B are two matrices of the same order, then we define A – B = A + (– B).

6.4 Scalar Multiplication of Matrices


If A=[aij]m×n is a matrix and k any number, then the matrix which is obtained by multiplying the elements of A by k
is called the scalar multiplication of A by k and it is denoted by k A thus if A = [aij]m×n
Then kAm×n = Am×nk = [kai×j]

Properties of Scalar Multiplication: If A, B are matrices of the same order and λ, µ are any two scalars then

(a) λ (A + B) = λA + λB (b) (λ + µ)A = λA + µA (c) λ(µA) = (λ µA) = µ(λA)


(d) (–λ A) = – (λ A) = λ(–A) (e) tr (kA) = k tr (A)

6.5 Multiplication of Matrices


If A and B be any two matrices, then their product AB will be defined only when the number of columns in A is
equal to the number of rows in B. If A[aij]m×n and B[bij]n×p then their product AB = C = [cij], will be a matrix of order
n
m × p, where (AB)ij = Cij = ∑ airbrj
r =1

Proof: Let A=[aij] be an m × n matrix and B = [bij] be an n × p matrix. Then the m × p matrix C = [cij] is called
the product if Cij=AiBj Where Ai is the ith row of A and Bj is the jth column of B. Thus the product AB is obtained as
following:

A=m×n B=n×p
C1 C2 Cj Cp

↓ ↓ ↓ ↓
a11 a12 a13 ……… a1j ……… a1n b11 b12 ……… b1j ……… b1p
R1 →
a21 a22 a23 ……… a2j ……… a2n b21 b22 ……… b2j ……… b2p
R2 →
……………………………………………….……… × ………………………………………………
Ri →
ai1 ai2 ai3 ……… aij ……… ain bi1 bi2 ……… bij ……… bip
Rm →
am1 am2 am3 ……… amj ……… amn bn1 bn2 ……… bnj ……… bnpn
1 6 . 1 0 | Matrices

R1C1 R1C2 ………. R1Cj R1Cp


R2C1 R2C2 ………. R2Cj R2Cp
………. ………. ………. ……….……….
= Thus (AB)ij = AiBj
RiC1 RiC2 ………. RiCj RiCp
………. ………. ………. ……….……….
RmC1 RmC2 ………. RmCj RmCp

b1 j 
 
b2 j 
n
 
(AB)ij = [ai1 ai2 …….. aij……. ain]  ...  = [ai1 b1j + ai2 b2j + …….. + ainbnj] ; (AB)ij = ∑ (air .brj )
b  r =1
 ij 
 .... 
 
 bnj 

Properties of matrix multiplication:


(a) Matrix multiplication is not commutative in general, i.e. in general AB ≠ BA.
(b) Matrix multiplication is associative, i.e. (AB)C = A(BC).
(c) Matrix multiplication is distributive over matrix addition, i.e. A.(B + C) = A.B + A.C and (A + B)C = AC + BC.
(d) If A is an m × n matrix, then ImA = A = AIn .
(e) The product of two matrices can be a null matrix while neither of them is null, i.e. if AB = 0, it is not necessary
that either A = 0 or B = 0.
(f) If A is an m × n matrix and O is a null matrix then Am×n. On×p = Om × p, i.e. the product of the matrix with a null
matrix is always a null matrix.
(g) If AB = 0 (It does not mean that A = 0 or B = 0, again the product of two non-zero matrices may be a zero
matrix).
(h) If AB = AC ⇒ B ≠ C (Cancellation Law is not applicable).
(i) tr(AB) = tr(BA).

 2 1 3 1 −2
   
Illustration 7: If A =  3 −2 1  and B =  2 1  find AB and BA if possible  (JEE MAIN)
 −1 0 1   4 −3
   

Sol: Using matrix multiplication. Here, A is a 3 × 3 matrix and B is a 3 × 2 matrix, therefore, A and B are conformable
for the product AB and it is of the order 3 × 2 such that

1 
 
(AB)11 = (First row of A) (First column of B) = [2 1 3]  2  = 2 × 1 + 1 × 2 + 3 × 4 = 16
4
 
 −2
 
(AB)12 = (First row of A) (Second column of B) = [2 1 3]  1  = 2 × (– 2) + 1 × 1 + 3 × (– 3) = –12
 −3
 
M a them a ti cs | 16.11

1 
 
(AB)21 = (Second row of A) (First column of B)= [3 –2 1]  2  = 3 × 1 + (–2) × 2 + 1 × 4 = 3
4
 
16 −12
 
Similarly (AB)22 = – 11, (AB)31 = 3 and (AB)32 = –1; ∴ AB =  3 −11
 3 −1 
 
BA is not possible since number of columns of B ≠ number of rows of A.

1 3  y 0  5 6 
Illustration 8: Find the value of x and y if 2  +  =    (JEE MAIN)
0 x  1 2  1 8 

Sol: Using the method of multiplication and addition of matrices, then equating the corresponding elements of
L.H.S. and R.H.S., we can easily get the required values of x and y.

1 3  y 0  5 6  2 6   y 0  5 6  2 + y 6 + 0  5 6 
We have, 2  +  =   ⇒  + =  ⇒  =  
0 x  1 2  1 8  0 2x  1 2  1 8   0 + 1 2x + 2 1 8 
Equating the corresponding elements, a11 and a22, we get
2 + y = 5 ⇒ y = 3; 2x + 2 = 8 ⇒ 2x = 6 ⇒ x = 3;
Hence x = 3 and y = 3.

 a − b 2a + c   −1 5 
Illustration 9: Find the value of a, b, c and d, if  =   (JEE MAIN)
2a − b 3c + d  0 13

Sol: As the two matrices are equal, their corresponding elements are equal. Therefore, by equating the corresponding
elements of given matrices we will obtain the value of a, b, c and d.

 a − b 2a + c   −1 5 
 =   (given)
2a − b 3c + d  0 13
a – b =−1  … (i)
2a + c = 5  … (ii)
2a – b = 0  … (iii)
3c + d = 13  … (iv)
Subtracting equation (i) from (iii), we have a = 1;
Putting the value of a in equation (i), we have 1 – b = – 1 ⇒ b = 2;
Putting the value of a in equation (ii), we have 2 + c = 5 ⇒ c = 3;
Putting the value of c in equation (iv), we find 9 + d = 13 ⇒ d =
Hence a = 1, b = 2, c = 3, d = 4.

2 3  2 −2 
Illustration 10: Find x and y, if 2x + 3y =   and 3x + 2y =    (JEE MAIN)
4 0  −1 5 
Sol: Solving the given equations simultaneously, we will obtain the values of x and y.
2 3
We have 2x + 3y =   … (i)
4 0
1 6 . 1 2 | Matrices

 2 −2 
3x + 2y =    … (ii)
 −1 5 
 6 9
Multiplying (i) by 3 and (ii) by 2, we get 6x + 9y =    … (iii)
12 0 
 4 −4 
6x + 4y =    … (iv)
 −2 10 
 6 − 4 9 + 4   2 13 
Subtracting (iv) from (iii), we get 5y =  = 
12 + 2 0 − 10  14 −10 
2 13  2 13 
5 5  5 5
⇒y=   ⇒y=  
 14 −10   14 −2 
 5 5   5 
2 13 
5 5 2 3
Putting the value of y in (iii), we get 2x + 3  =  
 14 −2  4 0
 5 
6 39   6 39   4 24   2 12 
2 3  5 5   2− 5 3− 5   5 −
5   5 −
5
⇒ 2x =   –   =  =   ⇒ x =  
 4 0   42 −6   4 − 42 0 + 6   − 22 6   − 11 3 
 5   5   5   5 

 2 12  2 13 
 5 −
5 5 5
Hence x=   and y =  
 − 11 3   14 −2 
 5   5 

 x + 3 z + 4 2y − 7   0 6 3y − 2
   
Illustration 11: If  −6 a − 1 0  =  −6 −3 2c + z  then find the values of a, b, c, x, y and z.
b − 3 −21 0  2b + 4 −21 0 
   (JEE ADVANCED)
Sol: As the two matrices are equal, their corresponding elements are also equal. Therefore, by equating the
corresponding elements of the given matrices, we will obtain the values of a, b, c, x, y and z.

 x + 3 z + 4 2y − 7   0 6 3y − 2
   
 − 6 a − 1 0  =  −6 −3 2c + z 
b − 3 −21 0  2b + 4 −21 0 
 

Comparing both sides, we get x + 3 = 0 ⇒ x = – 3  … (i)


and z + 4 = 6 ⇒ z = 6 – 4 ⇒ z = 2  … (ii)
and 2y – 7 = 3y – 2 ⇒ 2y – 3y = – 2 + 7 ⇒ –y = 5 ⇒ y = – 5  … (iii)
and a – 1 = –3 ⇒ a = –3 + 1 ⇒ a = –2  … (iv)
and b – 3 = 2b + 4 ⇒ b – 2b = 4 + 3 ⇒ – b = 7 ⇒ b = – 7  … (v)
−2
and 2c + 2 = 0 ⇒ 2c + 2 = 0 ⇒ 2c = – 2 ⇒c= ⇒c=–1 … (vi)
2
[from(2)] Thus; a = –2, b = –7, c = –1, x =–3, y = –5 and z = 2
M a them a ti cs | 16.13

7. RANK OF A MATRIX
If A = (aij)m×n is a matrix, and B is its sub-matrix of order r, then | B |, the determinant is called r-rowed minor of A.

Definition: Let A = (aij)m×n be a matrix. A positive integer r is said to be a rank of A if

(a) A possesses at least one r-rowed minor which is different from zero; and
(b) Every (r + 1) rowed minor of A is zero.

From (ii), it automatically follows that all minors of higher order are zeros. We denote rank of A by ρ(A)

Note: The rank of a matrix does not change when the following elementary row operations are applied to the matrix:
(a) Two rows are interchanged (Ri ↔ Rj);
(b) A row is multiplied by a non-zero constant, (Ri → kRi , with k ≠ 0);
(c) A constant multiple of another row is added to a given row (Ri → Ri, + kRj) where i ≠ j.

Note: The arrow → means “replaced by”.


Note that the application of these elementary row operations does not change a singular matrix to a non-singular
matrix nor does a non-singular matrix change to a singular matrix. Therefore, the order of the largest non-singular
square sub-matrix is not affected by the application of any of the elementary row operations. Thus, the rank of a
matrix does not change by the application of any of the elementary row operations. A matrix obtained from a given
matrix by applying any of the elementary row operations is said to be equivalent to it. If A and B are two equivalent
matrices, we write A ~ B. Note that if A ~ B, then ρ(A) = ρ(B)
By using the elementary row operations, we shall try to transform the given matrix in the following

1 * * *
 
0 1 * *
0 0 1 *
 
form  . . . .
. . . .
 
. . . .
 
0 0 0... *

Where * stands for zero or non-zero element. That is, we shall try to make aii as 1 and all the elements below aij as zero.

PLANCESS CONCEPTS

A non zero matrix A is said to have rank r, if


• Every square sub-matrix of order (r + 1) or more is singular;
• There exists at least one square sub-matrix or order r which is non singular.
B Rajiv Reddy (JEE 2012 AIR 11)

3 + x 5 2 
 
Illustration 12: For what values of x does the matrix  1 7+x 6  have the rank 2?  (JEE ADVANCED)
 2 5 3 + x 

Sol: The given matrix has only one 3rd-order minor. In order that the rank arrive at 2, we must bring about its
determinant to zero. Hence, by applying the invariance method we can obtain values of x.
1 6 . 1 4 | Matrices

3+x 5 2
1 7+x 6 = 0  … (i)
2 5 3+x

Now, using R1 → R1 – R3

3+x 5 2 1+x 0 −1 − x 1+x 0 0


1 7+x 6 = 1 7+x 6 ; using C3 → C3 + C1 = 1 7+x 7
2 5 3+x 2 5 3+ x 2 5 5+x

7+x 7
= (1 + x) = (1+ x) [(7 + x ( 5 + x) – 35] = (1 + x) (x2 + 12x) = x(1+ x) (x + 12)
5 5+x

\ (i) holds for x = 0, –1, –12

3 5 2 
  3 5 
When x = 0, the matrix = 1 7 6  Clearly, a minor   ≠ 0, So, the rank = 2
2 5 3  1 7 
 

2 5 2 
  2 5 
When x = –1, the matrix = 1 6 6  Clearly, a minor   ≠ 0, So, the rank = 2
2 5 2  1 6 
 

 −9 5 2
   −9 5 
When x = –12, the matrix =  1 −5 6  Clearly, a minor   ≠ 0, So, the rank = 2
 2 5 −9   1 −5
 
∴ The matrix has the rank 2 if x = 0, –1, –12.

8. POSITIVE INTEGRAL POWERS OF A SQUARE MATRIX


The positive integral powers of a matrix A are defined only when A is a square matrix.
Also then, A2 = A.A; A3 = A.A.A = A2A. Also for any positive integers m, n:

(a) AmAn = Am+n (b) (Am)n = Amn = (An)m


(c) In = I, Im= I (d) A0 = In

Matrix polynomial: If f(x) = a0xn + a1xn–1 + a2xn–2 +………. + anx0, then we define a matrix polynomial a,b
f(A) = a0An + a1An–1 + a2An–2 +………. + anIn where A is the given square matrix. If f(A) is a null matrix, then A is called
the zero or root of the matrix polynomial f(A)

9. SPECIAL MATRICES
(a) Idempotent Matrix: A square matrix is idempotent, provided A2 = A. For an idempotent matrix A, An = A ∀
n>2, n ∈ N ⇒ An = A, n ≥ 2.
For an idempotent matrix A, det A = 0 or 1 A2, ‫ ׀‬A ‫ ׀‬2 = ‫ ׀‬A ‫)׀‬.
(b) Nilpotent Matrix: A nilpotent matrix is said to be nilpotent of index p, (p ∈ N), if Ap = O, Ap-1 ≠ O, i.e. if p is
the least positive integer for which Ap = O, then A is said to be nilpotent of index p.
M a them a ti cs | 16.15

(c) Periodic Matrix: A square matrix which satisfies the relation AK+1 = A, for some positive integer K, then A is
periodic with period K, i.e. if K is the least positive integer for which AK+1 = A, and A is said to be periodic with
period K. If K =1 then A is called idempotent.

 2 −3 −5 
 
E.g. the matrix  −1 4 5  has the period 1.
 1 −3 −4 
 
Note: (i) Period of a square null matrix is not defined. (ii) Period of an idempotent matrix is 1.

(d) Involutary Matrix: If A2 = I, the matrix is said to be an involutary matrix. An involutary matrix its own inverse

0 1  0 1  1 0 
E.g. (i) A =     = 
1 0  1 0  0 1 

PLANCESS CONCEPTS

Two matrices cannot be added if they are of different order


1 1
If A is an involutary matrix, then (I + A) and (I – A) are idempotent and (I + A)(I – A) = 0
2 2
Chinmay S Purandare (JEE 2012 AIR 698)

 2 0 1
 
Illustration 13: Let A =  2 1 3  and f(x) = x2 – 5x + 6I3. Find f(A). (JEE MAIN)
 −1 −1 0 
 

Sol: By using methods of multiplication and addition of matrices we will obtain the required result. Here f(A) =
A2 – 5A + 6I3
2
 2 0 1  2 0 1 1 0 0 
     
=  2 1 3 – 5  2 1 3 + 6 0 1 0 
 −1 −1 0   −1 −1 0  0 0 1 
     

 2 0 1  2 0 1  10 0 5  6 0 0 
       
=  2 1 3  ×  2 1 3  – 10 5 15 + 0 6 0 
 −1 −1 0   −1 −1 0   −5 −5 0  0 0 6 
       

 2 × 2 + 0 × 2 + 1(–1) 2 × 0 + 0 × 1 + 1(–1) 2 × 1 + 0 × 3 + 1 × 0   6 − 10 0−0 0 −5 


   
=  2 × 2 + 1 × 2 + 3(–1) 2 × 0 + 1 × 1 + 3(–1) 2 × 1 + 1 × 3 + 3 × 0  +  0 − 10 6 −5 0 − 15 
( −1)2 + ( −1)2 + 0(–1) ( −1)0 + ( −1)1 + 0(–1) ( −1)1 + ( −1)3 + 0 × 0  0 − ( −5) 0 − ( −15) 6 − 0 
   

 3 −1 2   −4 0 −5   3 − 4 −1 + 0 2 − 5   −1 −1 −3 
       
=  3 −2 5  +  −10 1 −15 =  3 − 10 −2 + 1 5 − 15  =  −7 −1 −10 
 −4 −1 −4   5 5 6   −4 + 5 −1 + 5 −4 + 6  1 4 2 
      

a b
Illustration 14: Let A =   be such that A3 = 0, but A ≠ 0, then (JEE MAIN)
 c d 

Sol: (a) As A3 = 0, we get | A3 | = 0; | A3 | = 0 ⇒ | A | = 0 ⇒ ad – bc = 0


1 6 . 1 6 | Matrices

In this problem, A3 = 0 means | A | also is equal to 0; therefore, by calculating A2 we can obtain the result.
(a) A2 = 0 (b) A2 = A (c) A2 = I – A (d) None of these

 a2 + bc (a + d)b   a2 + ad (a + d)b 
Also, A2 =   =   = (a + d) A
 (a + d)c bc + d2   (a + d)c ad + d2 
   

If a + d = 0, we get A2 = 0. But, if a + d ≠ 0, then A3 = A2A = (a + d) A2 ⇒ 0 = (a + d) A2 ⇒ A2 = 0

1 0  1 0 
Illustration 15: If A =   and I =   then which one of the following holds for all n > 1, by the principle
1 1  0 1 
of mathematical induction. (JEE MAIN)
(a) An = nA + (n – 1) I (b) An = 2n–1 A + (n – 1) I
(c) An = nA – (n – 1) I (d) An = 2n–1 A – (n – 1) I

Sol: By substituting n = 2 we can determine the correct answer.

1 0  3 0 
For n = 2, A2 =   For n = 2, RHS of (a) = 2A + I = 3  ≠ A
2

0 1   2 3 
For n = 2, RHS of (b) = 2A + I ≠ A2 So possible answer is (c) or (d)

1 0 
In fact An =   which equals nA – (n – 1) I;
n 1 
0 0 
Alternatively. Write A = I + B Where B =  
1 0 
As B2 = 0, we get Br = 0 ∀r > 2

By the binomial theorem, An = I + nB = I + n(A – I) = nA – (n –1)I

10. ADJOINT OF A MATRIX


Let the determinant of a square matrix A be | A |

 a11 a12 a13  a11 a12 a13


 
If A = a21 a22 a23  Then | A | = a21 a22 a23
a a33 
 31 a32 a31 a32 a33
 A11 A12 A13 
 
The matrix formed by the cofactors of the elements in | A | is  A21 A22 A23 
A A32 A33 
 31
a22 a23
Where A11 = (–1)1+1 = a22 a33 – a23. a32
a32 a33

a21 a23 a a22


A12 = (–1)1+2 = –a21. a33 + a23. a31; A13 = (–1)1+3 21 = a21 a32 – a22 a31;
a31 a33 a31 a32

a12 a13 a a13


A21 = (–1)2+1 = –a12 a33 + a13. a32; A22 = (–1)2+2 11 = a11 a33 – a13. a31;
a32 a33 a31 a33
M a them a ti cs | 16.17

a11 a12 a12 a13


A23 = (–1)2+3 = –a11 a32 + a12. a31; A31 = (–1)3+1 = a12 a23 – a13. a22;
a31 a32 a22 a23

a11 a13 a a12


A32 = (–1)3+2 = –a11 a23 + a13 a21; A33 = (–1)3+3 11 = a11 a22 – a12. a21;
a21 a23 a21 a22

Then the transpose of the matrix of co-factors is called the adjoint of the matrix A and is written as

 A11 A21 A31 


 
adj A. adj A=  A12 A22 A32 
A A23 A33 
 13
The product of a matrix A and its adjoint is equal to unit matrix multiplied by the determinant A.
Let A be a square matrix, then (Adjoint A). A = A. (Adjoint A) = | A |. I

 a11 a12 a13   A11 A21 A31 


   
Let A = a21 a22 a23  and adj A =  A12 A22 A32 
a a32 a33  A A23 A33 
 31  13
 a11 a12 a13   A11 A21 A31 
   
A. (adj. A) = a21 a22 a23  ×  A12 A22 A32 
a a32 a33  A A23 A33 
 31  13
 a11 A11 + a12 A12 + a13 A13 a11 A21 + a12 A22 + a13 A23 a11 A31 + a12 A32 + a13 A33 
 
= a21 A11 + a22 A12 + a23 A13 a21 A21 + a22 A22 + a23 A 23 a21 A31 + a22 A32 + a23 A33 
a A + a A + a A a31 A21 + a32 A22 + a33 A 23 a31 A31 + a32 A32 + a33 A33 
 31 11 32 12 33 13

| A | 0 0  1 0 0 
   
=  0 | A | 0  = | A | 0 1 0  = |A| I.
 0 0 | A | 0 0 1 
  

x 3 2
 
Illustration 16: If A =  −3 y −7  and A = – A’, then x + y is equal to
 −2 7 0 
 
(a) 2 (b) –1 (c) 0 (d) 12 (JEE MAIN)

Sol: (c) A = – A’; ⇔ A is skew-symmetric matrix; ⇒ diagonal elements of A are zeros


⇒ x = 0, y = 0; ∴ x + y = 0

Illustration 17: If A and B are two skew-symmetric matrices of order n, then,  (JEE MAIN)
(a) AB is a skew-symmetric matrix (b) AB is a symmetric matrix
(c) AB is a symmetric matrix if A and B commute (d) None of these

Sol: (c) We are given A’ = – A and B’ = – B; Now, (AB)’ = B’A’ = (–B) (–A) = BA = AB if A and B commute.

Illustration 18: Let A and B be two matrices such that AB’ + BA’ = O. If A is skew symmetric ,then BA  (JEE MAIN)
(a) Symmetric (b) Skew symmetric (c) Invertible (d) None of these

Sol: (c) we have, (BA)’ = A’B’ = – AB’ [ A is skew symmetric]; = BA’ = B(–A) = – BA ⇒ BA is skew symmetric.
1 6 . 1 8 | Matrices

1 2 3 
 
Illustration 19: Let A = 1 3 4  , then the co-factors of elements of A are given by - (JEE MAIN)
1 4 3 
 
Sol: Co-factors of the elements of any matrix are obtain by eliminating all the elements of the same row and
column and calculating the determinant of the remaining elements.
3 4
A11 = =3×3–4×4=–7
4 3

1 4 1 3 2 3 1 3
A12 = – = 1, A13 = = 1; A21 = – = 6, A22 = =0
1 3 1 4 4 3 1 3

1 2 2 3 1 3 1 2
A23 = – = – 2, A31 = = – 1; A32 = – = – 1, A33 = =1
1 4 3 4 1 4 1 3

−7 6 −1
∴ Adj A = 1 0 −1
1 −2 1

Illustration 20: Which of the following statements are false – (JEE MAIN)
(a) If | A | = 0, then | adj A | = 0;
(b) Adjoint of a diagonal matrix of order 3 × 3 is a diagonal matrix;
(c) Product of two upper triangular matrices is a upper triangular matrix;
(d) adj (AB) = adj (A) adj (B);

Sol: (d) We have, adj (AB) = adj (B) adj (A) and not adj (AB) = adj (A) adj (B)

11. INVERSE OF A MATRIX


If A and B are two square matrices of the same order, such that AB = BA = I (I = unit matrix)
Then B is called the inverse of A, i.e. B = A–1 and A is the inverse of B. Condition for a square matrix A to possess an inverse
is that the matrix A is non-singular, i.e., | A | ≠ 0. If A is a square matrix and B is its inverse then AB = I. Taking determinant
of both sides | AB | = | I | or | A | | B | = I. From this relation it is clear that | A | ≠ 0, i.e. the matrix A is non-singular.
To find the inverse of matrix by using adjoint matrix:
A.(AdjA)
We know that, A. (Adj A) = | A | I or = I (Provided | A | ≠ 0)
|A|
1
and A. A–1 = I; \ A–1 = (Adj. A)
|A|

1 0 −1 
 
Illustration 21: Let A = 3 4 5  . What is inverse of A ? (JEE MAIN)
0 −6 −7 
 
adj A
Sol: By using the formula A–1 = we can obtain the value of A–1.
|A|
M a them a ti cs | 16.19

4 5 3 5 
We have A11 =   = 2 A12 = –   = 21
 −6 −7  0 −7 

And similarly A13 = –18, A31 = 4, A32 = –8, A33 = 4, A21 = +6, A22 = - 7, A23 = 6

 2 6 4 1 0 −1
 
∴ adj A =  21 −7 −8  Also | A | = 3 4 5 = {4×(–7) – (–6)×5 – 3×(–6)}
 −18 6 4  0 −6 −7

 2 6 4
adj A 1  
= – 28 + 30 + 18 = 20 \ A = –1
=  21 −7 −8 
|A| 20
 −18 6 4 

1 1  3 2
Illustration 22: If the product of a matrix A and   is the matrix   , then A is given by: (JEE MAIN)
–1

 2 0  1 1 
0 −1   0 −1  0 1 
(a)   (b)   (c)   (d) None of these
2 −4   −2 −4  2 −4 

Sol: (a) We know if AB = C, then B–1 A–1 = C–1 ⇒ A–1 = BC–1 by using this formula we will get value of A–1 in the above
problem.
−1
1 1  3 2 1 1  3 2 1 1   1 −2  0 1 
Here, A   =   ⇒ A = 
–1
  =   = 
2 0  1 1  2 0  1 1  2 0   −1 3  2 −4 

2 1 −1  1 2 5
   
Illustration 23: Let A = 0 1 0  and B =  2 3 1 . Prove that (AB)–1 = B–1A–1.  (JEE ADVANCED)
1 3 −1  −1 1 1 
   
adj AB
Sol: By obtaining | AB | and adj AB we can obtain (AB)–1 by using the formula (AB)–1 = . Similarly we can also
| AB |
obtain the values of B–1 and A–1.Then by multiplying B–1 and A–1 we can prove the given problem.

2 1 −1  1 2 5  2 + 2 + 1 4 + 3 − 1 10 + 1 − 1 5 6 10 
       
Here, AB = 0 1 0   2 3 1  = 0 + 2 + 0 0 + 3 + 0 0 + 1 + 0  =  2 3 1 
1 3 −1  −1 1 1  1 + 6 + 1 2 + 9 − 1 5 + 3 − 1  8 10 7 
       

5 6 10
Now, | AB | = 2 3 1 = 5(21 – 10) – 6(14 – 8) + 10(20 – 24) = 55 – 36 – 40 = – 21.
8 10 7

 3(7) − 1(10) −{2(7) − 8(1)} 2(10) − 3(8)   11 −6 −4 


   
The matrix of cofactors of | AB | is =  −{6(7) − 10(10)} 5(7) − 8(10) −{5(10 − 6(8)}=   58 −45 −2 
 5(3) − 6(2)   3 
 6(1) − 10(3) −{5(1) − 2(10)}  −24 15

11 58 −24  11 58 −24 


adj AB −1  
∴ adj AB =  −6 −45 
15  ; ∴ (AB)–1 = = −6 −45 15 
| AB | 21 
 −4
 −2 3   −4 −2 3 
1 6 . 2 0 | Matrices

1 2 5
Next, | B | = 2 3 1 = 1(3 – 1) – 2(2 + 1) + 5(2 + 3) = 21
−1 1 1

 2 3 −13 2 1 −1
adj B 1    
∴B = –1
=  −3 6 9  ; | A | = 0 1 0  = 1 (–2 + 1) = – 1
|B | 21
 5 −3 −1  1 3 −1
   

 −1 −2 1   2 3 −13  −1 −2 1 
adj A 1   1    
∴ A–1 = = 0 −1 0 ; ∴ B A
–1 –1
= – −3 6 9   0 −1 0 
|A| −1   21 
 −1 −5 2   5 −3 −1   −1 −5 2 
     

11 58 −24 
1  
=– −6 −45 15  Thus, (AB)–1 = B–1 A–1
21 
 −4 −2 3 

0 2y z 
 
Illustration 24: If A =  x y −z  satisfies A’ = A–1, then (JEE ADVANCED)
 x −y z 
 
(a) x = ± 1/ 6 , y = ± 1/ 6 , z = ± 1/ 3 (b) x = ± 1/ 2 , y = ± 1/ 6 , z = ± 1/ 3

(c) x = ± 1/ 6 , y = ± 1/ 2 , z = ± 1/ 3 (d) x = ± 1/ 2 , y = ± 1/3, z = ± 1/ 2

Sol: (b) Given that A’ = A–1 and we know that AA–1= I and therefore AA’ = I. Using the multiplication method we can
obtain values of x, y and z.
A’ = A–1 ⇔ AA’ = I
0 2y z   0 x x  4y 2 + z 2 2y 2 − z 2 −2y 2 + z 2 
     
Now, AA’ =  x y −z  2y y − y  =  2y 2 − z 2 x2 + y 2 + z2 x2 − y 2 − z2 
 x − y z   z −z z   2 2 
     −2y + z x2 − y 2 − z2 x2 + y 2 + z2 

Thus, AA’ = I ⇒ 4y2 + z2=1, 2y2 – z2 = 0, x2 + y2 + z2 =1, x2 – y2 – z2 = 0

\ x = ± 1/ 2 , y = ± 1/ 6,z=±1/ 3

0 1 2  1 / 2 −1 / 2 1 / 2
   
Illustration 25: If A = 1 2 3  and A–1 =  −4 3 y  , then (JEE ADVANCED)
3 x 1  5 / 2 −3 / 2 1 / 2
   
(a) x = 1, y = –1 (b) x = –1, y = 1
1
(c) x = 2, y = –1/2 (d) x = 1/2, y =
2
Sol: (a) We know AA–1= I, hence by solving it we can obtain the values of x and y.
We have

1 0 0  0 1 2  1 / 2 −1 / 2 1 / 2  1 0 y +1 
       
0 1 0  = AA = 1 2 3   −4 3 y  =  0 1 2(y + 1)
–1

0 0 1  3 x 1  5 / 2 −3 / 2 1 / 2  4(1 − x) 3(x − 1) 2 + xy 
       

⇒ 1 – x = 0, x – 1 = 0; y + 1 = 0, y + 1 = 0, 2 + xy = 1; \ x = 1, y = – 1
M a them a ti cs | 16.21

12. SYSTEM OF LINEAR EQUATIONS


Let the equations be
a1x + a2y + a3z = d1
b1x + b2y + b3z = d2
c1x + c2y + c3z = d3
We write the above equations in the matrix form as follows

 a1 x + a2 y + a3 z  d1   a1 a2 a3  x d1 


         
b1 x + b2 y + b3 z  = d2  ⇒ b1 b2 b3  y  = d2  ⇒ AX = B  … (i)
c x +c y +c z  d  c c2 c3  z  d 
 1 2 3   3  1    3
 a1 a2 a3  x d1 
     
Where, A = b1 b2 b3  , X =  y  and B = d2 
c c2 c3  z  d 
 1    3

Multiplying (i) by A–1, we get A–1AX = A–1B ⇒ I. X = A–1B ⇒ X = A–1B

12.1 Solution to a System of Equations


A set of values of x, y, z which simultaneously satisfy all the equations is called a solution to the system of equations.
Consider, x + y + z = 9 2x – y + z = 5 4x + y – z = 7
Here, the set of values – x = 2, y =3, z = 4, is a solution to the system of linear equations.
Because, 2 + 3 + 4 = 9 4 – 3 + 4 = 5 8+3–4=7

12.2 Consistent Equations


If the system of equations has one or more solution, then it is said to be a consistent system of equations, otherwise
it is an inconsistent system of equations. For example, the system of linear equations x + 3y = 5 x – y = 1 is
consistent, because x = 2, y = 1 is a solution to it. However, the system of linear equations x + 3y = 5 2x + 6y = 8
is inconsistent, because there is no set of values of x and y which may satisfy the two equations simultaneously.
Condition for consistency of a system of linear equation AX = B
(a) If |A| ≠ 0, then the system is consistent and has a unique solution, given by X = A–1B
(b) If |A| = 0, and (Adj A) B ≠ 0 then the system is inconsistent.
(c) If |A| = 0, and (Adj A) B = 0, then the system is consistent and has infinitely many solutions.
Note, AX = 0 is known as homogeneous system of linear equations, here B = 0. A system of homogeneous
equations is always consistent.
The system has non-trivial solution (non-zero solution), if | A | = 0

Theorem 1: Let AX = B be a system of linear equations, where A is the coefficient matrix. If A is invertible then the
system has a unique solution, given by X = A–1 B

Proof: AX = B; Multiplying both sides by A–1. Since A–1 exists ⇒ | A | ≠ 0


⇒ A–1 AX = A–1B ⇒ IX = A–1B ⇒ X = A–1B
Thus, the system of equations AX = B has a solution given by X = A–1B

Uniqueness: If AX = B has two sets of solutions X1 and X2, then


AX1 = B and AX2 = B (Each equal to B) ⇒ AX1 = AX2
1 6 . 2 2 | Matrices

By cancellation law, A being invertible ⇒ X1 = X2


Hence, the given system AX = B has a unique solution. Proved
Note: A homogeneous system of equations is always consistent.

x + y y  2 3 
Illustration 26: Let A =   , B =   and C =   . If AB = C. Then find the matrix A (JEE MAIN)
2

 2x x − y   −1 2 
Sol: By solving AB = C we get the values of x and y. Then by substituting these values in A we obtain A2.

x + y y  2 3   2(x + y) – y  3 
Here     =   ⇒   =   ⇒ 2(x + y) – y = 3 and 4x – (x – y) = 2
 2x x − y −1
  2
  2x .2 – (x − y) 2 
⇒ 2x + y = 3 and 3x + y= 2 Subtracting the two equations, we get, x = –1, So, y = 5.

 −1 + 5 5  4 5 4 5 4 5
∴A=   =   ; ∴ A2 =    
 2( −1) −1 − 5  −2 −6   −2 −6   −2 −6 

 4 × 4 + 5(–2) 4 × 5 + 5( −6)   6 −10 


=   =  
 −2 × 4 + ( −6)( −2) −2 × 5 + ( −6)( −6)  4 26 

Illustration 27: Solve the following equations by matrix inversion


2x + y + 2z = 0 2x – y + z = 10 x + 3y – z = 5 (JEE ADVANCED)

Sol: The given equation can be written in a matrix form as AX = D and then by obtaining A–1 and multiplying it on
both sides we can solve the given problem.

2 1 2  x 0 2 1 2  x 0


           
2 −1 1   y  = 10  ∴ AX = D where A = 2 −1 1  , X =  y  , D= 10 
1 3 −1 z  5 1 3 −1 z  5
           
⇒ A–1(AX) = A–1D ⇒ (A–1A)X = A–1D ⇒ IX = A–1D ⇒ X = A–1D …(i)

2 1 2
adj A
Now A = –1
; |A| = 2 −1 1 = 2(1 – 3) – 1 (– 2 –1) + 2(6 + 1) = 13
|A|
1 3 −1

 −2 3 7  −2 7 3
   
The matrix of co-factors of | A | is  7 −4 −5  . So, adj A =  3 −4 2 
 3 2 −4   7 −5 −4 
   

 −2 7 3  −2 7 3 0
1   1    
∴A = –1
3 −4 2  . ∴ from (i), X = 3 −4 2  10 
13  13 
 7 −5 −4   7 −5 −4  5
     

 0 + 70 + 15   85 / 13  x  85 / 13 
1         85 −30 −70
= 0 − 40 + 10 = −30 / 13 ; ∴ y  =  −30 / 13 ⇒ x = 13 , y = 13 , z = 13
13    
 0 − 50 − 20   −70 / 13 z   −70 / 13
       
M a them a ti cs | 16.23

2 1   −3 2  1 0 
Illustration 28: If 7 4  A   =   , then matrix A equals:
   5 −3 0 1 

 7 5 2 1   7 1  5 3
(a)   (b)   (c)   (d)    (JEE ADVANCED)
 −11 −8  5 3 34 5 13 8 

Sol: (a) We know that if XAY = I, then A = X–1 Y–1 = (YX)–1.


−1
 −3 2  2 1   8 5  8 5  7 5
In this case YX =     =   ; ∴ A =   =  
 5 −3 7 4   −11 −7   −11 −7  −11 −8 

 3 −2 1   x  b
     
Illustration 29: The system of equations  5 −8 9   y  =  3  has no solution if a and b are
2 1 a  z   −1 
     
(a) a = –3, b ≠ 1/3 (b) a = 2/3, b ≠ 1/3
(c) a ≠ 1/4, b = 1/3 (d) a ≠ –3, b ≠ 1/3 (JEE ADVANCED)

Sol: By applying row operation in the given matrices and comparing them we can obtain the required result.

 3 −2 1 b 
 
(a) The augmented matrix is given by (A|B) =  5 −8 9 3 
 2 1 a −1 
 
 1 4 −7 2b − 3 
 
Applying R1 →2R1 – R2, we get (A|B) ~  5 −8 9 3 
2 1 a −1 

1 4 −7 2b − 3 
 
Applying R2 →R2 – 5R1, R3 → R3 – 2R1, we get (A|B) ~  0 −28 44 18 − 10b 
 0 −7 a + 14 5 − 4b 
 
−28 44 18 − 10b
The system of equations will have no solution if = ≠
−7 a + 14 5 − 4b
⇒ a + 14 = 11 and 20 – 16b ≠ 18 – 10b
⇒ a = –3 and b ≠ – 1/3.

1 0 0 1 
   
Illustration 30: Let A =  2 1 0  . If u1 and u2 are column matrices such that Au1 =  0  and
3 2 1 0
   
0
 
Au2 =  1  , then u1 + u2 equals:
0
 

 −1   −1  1   −1 
       
(a)  1  (b)  −1  (c)  −1  (d)  1  (JEE ADVANCED)
 −1  0  −1  0
       
1 6 . 2 4 | Matrices

Sol: (c) Adding Au1 and Au2 we get A(u1 + u2). Then using the invariance method we obtain u1 + u2.

1  0 1 
     
By adding, we have A(u1 + u2) = Au1 + Au2 =  0  + 1  = 1 
0 0 0
     
1 0 0 1 
 
We then solve the above equation for u1 + u2, if we consider the augmented matrix (A|B) =  2 1 0 1 
3 2 1 0
 
1 0 0 1  1 
   
Applying R3 →R3 – 2R2 + R1 and R2 →R2 – 2R1, we get (A|B) ~  0 1 0 −1  ⇒ u1 + u2 =  −1 
 0 0 1 −1   −1 
   

PROBLEM-SOLVING TACTICS

If A, B are square matrices of order n, and In is a corresponding unit matrix, then

(a) A(adj.A) = | A | In = (adj A) A

(b) | adj A | = | A |n-1 (Thus A (adj A) is always a scalar matrix)

(c) adj (adj.A) = | A |n-2 A


2
(d) | adj (adj.A) | = | A |(n-1)

(e) adj (AT) = (adj A)T

(f) adj (AB) = (adj B) (adj A)

(g) adj (Am) = (adj A)m, m ∈ N

(h) adj (kA) = kn–1 (adj. A) , k ∈ R

(i) adj (In) = In

( j) adj 0 = 0

(k) A is symmetric ⇒ adj A is also symmetric

(l) A is diagonal ⇒ adj A is also diagonal

(m) A is triangular ⇒ adj A is also triangular

(n) A is singular ⇒ | adj A | = 0


M a them a ti cs | 16.25

FORMULAE SHEET

(a) Types of matrix:


(i) Symmetric Matrix: A square matrix A = [aij] is called a symmetric matrix if aij = aji, for all i,j.
(ii) Skew-Symmetric Matrix: when aij = –aji
(iii) Hermitian and skew – Hermitian Matrix: A = Aθ (Hermitian matrix)
Aθ = – A (skew-Hermitian matrix )
(iv) Orthogonal matrix: if AAT = In = ATA
(v) Idempotent matrix: if A2 = A
(vi) Involuntary matrix: if A2 = I or A–1 = A
(vii) Nilpotent matrix: if ∃ p ∈ N such that AP = 0

(b) Trace of matrix:


(i) tr(λA) = λ tr(A)
(ii) tr(A + B) = tr(A) + tr(B)
(iii) tr(AB) = tr(BA)
(c) Transpose of matrix:
(i) (AT)T = A (ii) (A ± B)T = AT ± BT (iii) (AB)T = BTAT (iv) (kA)T = k(A)T

(v) (A1 A2 A3 ..........An−1 An )T = AnT AnT−1 …….. A3T A2T A1T (vi) IT = I (vii) tr(A) = tr(AT)

(d) Properties of multiplication:


(i) AB ≠ BA (ii) (AB)C = A(BC) (iii) A.(B + C) = A.B + A.C

(e) Adjoint of a Matrix:


(i) A(adj A) = (adj A)A = | A | In (ii) | adj A | = | A |n–1
(iii) (adj AB) = (adj B) (adj A) (iv) adj (adj A) = | A |n–2
(v) (adj KA) = Kn–1(adj A)

(e) Inverse of a matrix: A–1 exists if A is non singular i.e. | A | ≠ 0


1
(i) A–1 = (Adj. A) (ii) A–1A = In = AA–1
|A|

(iii) (AT)–1 = (A–1)T (iv) ( A–1 )–1 = A

(v) | A–1 | = | A |–1 = 1


|A|
1 6 . 2 6 | Matrices

Solved Examples

JEE Main/Boards Similarly,

0 0 0 1 1 3
 x−y 2x + z   −1 5     
Example 1: If   =  , ∴ A = A . A = 3 3 9 × 5 2 6  =
3 2

3x + y 3z + 4w   5 25  −1 −1 −3  −2 −1 −3


   
find x, y, z, w.
0 0 0 
 
Sol. We know that in equal matrices the corresponding 0 0 0 
elements are equal. Therefore, by equating the elements 0 0 0 
 
of these two matrices which have the same number of
rows and columns, we get the value of x, y, z and w. ⇒ A3 = 0.

 x−y 2x + z   −1 5  i.e. Ak = 0
Given  =  
3x + y 3z + 4w   5 25 Here k = 3
x – y = –1, Hence, A is nilpotent matrix of index 3
2x + z = 5;
3x + y = 5, Example 3: Solve the following system of homogeneous
equations:
3z + 4w = 25
2x + 3y – z = 0, x – y – 2z = 0 and
By solving these equations, we get
3x + y + 3z = 0
x = 1, y = 2, z = 3, w = 4
Sol: In this problem we can write the given homogeneous
Example 2: Show that the matrix equations in a matrix form, i.e. [A][X] = [O] and then by
calculating the determinant of matrix A we can find if
1 1 3 that given system has a trivial solution or not.
 
 5 2 6  is a nilpotent matrix of index 3 The given system can be written as
 −2 −1 −3
 
2 3 −1  x  0 
     
Sol: Value of the index at which all elements of the 1 −1 −2  y  = 0  or AX = O
matrix become 0, i.e. null matrix, is called the nilpotent 3 1 3   z  0 
     
matrix of that index. Here we calculate the nth power of
the matrix, where n =1, 2, 3, …. The value of n at which 2 3 −1 x 0 
the matrix becomes null matrix is the index value.      
Where, A = 1 −1 −2 X =  y  and O = 0 
1 1 3 3 1 3  z  0 
     
 
Given A =  5 2 6 
 −2 −1 −3 2 3 −1
  Now, | A | = 1 −1 −2
1 1 3 1 1 3 3 1 3
   
⇒ A = A×A=  5 2 6  ×  5 2 6  =
2
= 2(– 3 + 2) – 3 (3 + 6) – 1(1 + 3)
 −2 −1 −3  −2 −1 −3
   
= –2 – 27 – 4 = –33 ≠ 0
0 0 0
  Thus | A |≠ 0.
3 3 9
 −1 −1 −3 So the given system has only the trivial solution given
  by x = y = z = 0
M a them a ti cs | 16.27

Example 4: Find x, y, z and a for which 4 5


C31 = = ( 24 − 10 ) =
14 ,
 x + 3 2y + x  0 −7  2 6
 =  
 z − 1 4a − 6  3 2a  1 5
C32 = – = − ( 6 − 15 ) =9,
Sol: We know that, in equal matrices the corresponding 3 6
elements are equal. Therefore by equating the elements 1 4
of these two matrices which have the same number of C33 = = ( 2 − 12 ) =
–10,
3 2
rows and columns we get the values of x, y, z and w.
T
 x + 3 2y + x  0 −7   −6 0 3   −6 5 14 
Given,  =      
 z − 1 4a − 6  3 2a  ∴ adj A =  5 0 −1  =  0 0 9 
14 9 −10   3 −1 −10 
We know, that for equal matrices the corresponding    
elements are equal, therefore
x+3 = 0; cos θ sin θ  1 n
Example 6: If A =   , Then find nlim A.
2y + x =−7;  sin θ cos θ  →∞ n

z −1 = 3;
Sol: For this problem, we first have to calculate the nth
4a − 6 = 2a; power of matrix A, i.e. An, and multiply the matrix An
By solving these equations, we get 1
by .
n
∴ x = – 3, z = 4, y = – 2, a = 3.
Then, by with the given limit we can find the solution
of this problem.
Example 5: Compute the adjoint of the matrix
cos θ sin θ 
1 4 5  Given A =  
   sin θ cos θ 
A = 3 2 6 
0 1 0  cos θ sin θ 
n
 cosnθ sinnθ 
 
A =
n
 =  
Sol: For this problem, we use the formula to get the co-  sin θ cos θ   − sinnθ cosnθ 
factors of all the elements of matrix A. Then by taking  cosnθ sinnθ 
the transpose of the co-factor matrix we can get the 1 n  n 
⇒ A =  n 
adjoint of matrix A.
n  − sinnθ cosnθ 
Consider Cij be a co-factor of aij in matrix A.  n n 
Then the co-factors of the elements of A are given by But – 1 ≤ cos nθ, sin nθ ≤ 1;
2 6 cos nθ
C11 = =0 – 6 = − 6 , ∴ lim = 0,
1 0 n→∞ n

3 6 sinnθ
lim = 0,
C12 = – = 0, n→∞ n
0 0
3 2 1 n 0 0 
∴ lim A =  
C13 = = 3 – 0 = 3, n→∞ n 0 0 
0 1
4 5
C21 = – = − (0 − 5) =
5 Example 7: A trust fund has Rs. 50,000 that is to be
1 0 invested into two types of bonds. The first bond pays 5%
1 5 interest per year and the second bond pays 6% interest
C22 = = 0, per year. Using matrix multiplication determine how to
0 0
divide by Rs, 50,000 among the two types of bonds so as
1 4 to obtain an annual total interest of Rs. 2,780.
C23 = – = –(1 – 0) = −1 ,
0 1
Sol: In this problem, investment amounts can be written
in the form of a row matrix and interest amounts can
1 6 . 2 8 | Matrices

be written in the form of column matrix. By multiplying  cos γ sin γ 


these two matrix we will get the equation for annual f ( γ ) = 
interest rates. By equating this to the given annual  − sin γ cos γ 
interest value we will get the required answer.  cos α sin α   cos β sin β 
f(α)f(β) =    
Consider investment of first type of bond = Rs. x  − sin α cos α   − sin β cos β 
And second type of bond = Rs. 50,000 – x  cos α cos β − sin α sin β cos α sin β + sin α cos β 
= 
These amounts can be written in the form of a row  − sin α cos β − cos α sin β − sin α sin β + cos α cos β 
matrix A which is given by  cos(α + β) sin(α + β) 
=  
A =  x 50000 − x   − sin(α + β) cos(α + β)
1×2
Similarly f(α) f(β) f(γ)
The interest amounts per rupee, per year from the two
5 6  cos(α + β + γ ) sin(α + β + γ ) 
bonds are Rs. and which can be written in the =  
100 100  − sin(α + β + γ ) cos(α + β + γ )
form of a column matrix B which is given by
 cos π sin π 
=   and as α + β + γ = p
 5   − sin π cos π 
 
B =  100   −1 0 
= 
1 0 
 6   =–   = – I2
 0 −1  0 1 
 100  2×1
∴ The total interest per year is given by cos α − sin α 0 
 
 5  Example 9: If M(α) =  sin α cos α 0 
   0 0 1 
A.B = [x 50,000 – x] ×  100  
 6 
 100   cos β 0 sin β 
 
M(β) =  0 1 0 
= [x. 5/100 + (50,000 – x). 6/100]
 − sin β 0 cos β 
 
= [3000 – x/100]
then prove that [M(α) M (β)]–1 = M ( −β ) M ( −α )
Since the required total annual interest is
= Rs.2,780. ∴ [3000 – x/100] = [2780] Sol: In this problem, by finding the inverse of the matrix
we can easily get the required answer.
⇒ 3000 – x/100 = 2780
[M(α) M (β)]–1 = M(β)–1 M(α)–1
⇒ x = 100(3000 – 2780) = 22,000
cos α − sin α 0 
Hence the required amounts to be invested in the two  
bonds are Rs. 22,000 and Rs. (50,000 – 22,000), i.e. Rs. Given M(α) =  sin α cos α 0 
22,000 and Rs. 28,000 respectively.  0 0 1 

T
cos α − sin α 0 
 cos α sin α   
Example 8: If f(α) =   and if α, β, γ are M(α) =  sin α cos α 0 
–1

 − sin α cos α   0
 0 1 
the angles of a triangle, then prove that f(α). f(β) ,
f(γ) = –I2  cos α sin α 0 
 
=  − sin α cos α 0 
Sol: In this problem, by the methods of substitution  0 0 1 
and multiplication of matrices we can easily prove the 
given equation. We can also write this in the form
 cos α sin α 
Given that f(α) =   cos( −α ) − sin( −α ) 0 
 − sin α cos α   
 sin( −α ) cos( −α ) 0  = M(– α)
 cos β sin β   0 0 1 
∴ f(β)=  
 and
 − sin β cos β 
M a them a ti cs | 16.29

Similarly, 1 −2 x


where A =  ,X=  
 cos β 0 − sin β  1 1  y 
 
M(β) =  0
–1
1 0 
 −k  1 −2
 sin β 0 cos β  and B =   Now, | A | = = 3 ≠ 0.
  k 1 1
 
 cos( −β) 0 sin( −β)   1 2
  So A–1 exists; We have, adj A =  
=  0 1 0  = M(–β)
 −1 1 
 − sin(–β) 0 cos( −β)
  1  1 2
1
So, A–1 = adj A ⇒ A–1 =  
∴ [M(α) M (β)] = M(–β) M(–α)
–1
|A| 3  −1 1 

Now X = A–1B
Example 10: Show that the homogeneous system of
equations x – 2y + z = 0, x + y – z = 0, 3x + 6y – 5z = 0  x  1  1 2  −k   k / 3 
has a non-trivial solution, Also, find the solution. ⇒  =    =  
 y  3  −1 1   k  2k / 3
Sol: In this problem we can write the given homogeneous ⇒ x = k/3, y = 2k/3
equations in a matrix form, i.e. [A][X] = [O] and then by
These values of x, y and z also satisfy the third equation.
calculating the determinant of matrix A we can find if
Hence x = k/3, y = 2k/3 and z = k, where k is any real
that given system has a non- trivial solution or not.
number and which satisfy the given system of equations.
The given equations are
x – 2y + z = 0,
JEE Advanced/Boards
x + y – z = 0,
3x + 6y – 5z = 0, Example 1: Let A and B be symmetric matrices of the
same order. Then show that
We can write these equations in the form of matrices
as shown below (i) A + B is symmetric

1 −2 1   x  0  (ii) AB – BA is skew-symmetric
     
1 1 −1   y  = 0  or AX = O, where (iii) AB + BA is symmetric
3 6 −5   z  0 
     
Sol: In this problem, by using the conditions for
1 −2 1  x 0  symmetric and skew-symmetric matrices we can get
      the required result.
A = 1 1 −1  , X =  y  and O = 0 
3 6 −5  z  0  As given, A and B are symmetric.
     
1 −2 1 ∴ A’ = A and B’ = B
Now, | A | = 1 1 −1 (i) (A + B)’ = A’ + B’ = A + B
3 6 −5 ∴ A + B is symmetric
= 1(– 5 + 6) + 2 (– 5 + 3) + 1(6 – 3) = 0 (ii) (AB – BA)’ = (A’B)’ – (BA)’
Thus, | A | = 0 = B’A’ – A’B’ [by reversal law]
Hence, the given system of equations has a non-trivial = BA – AB [A’ = A, B’ = B]
solution.
∴ AB – BA is skew-symmetric
To find the solution, we take z = k in the first two
(iii) (AB + BA)’ = (AB)’ + (BA)’
equations and write them as follows:
= B’A’ + A’B’= BA + AB = AB + BA
x – 2y = – k and x + y = k
∴ AB + BA is symmetric.
1 −2  x   −k 
or     =   or AX = B,
1 1   y   k 
1 6 . 3 0 | Matrices

Example 2: Solve the following equations: 1 2 2 


 
2x – 3y + z = 9 x + y + z = 6 x – y + z = 2 Example 3: Let A = 2 1 2 ,
2 2 1 
 
Sol: In this problem, we can write the given
homogeneous equations in a matrix form, i.e. [A][X] = prove that A – 4A – 5I = 0, hence obtain A–1:
2

[O]. Then, by calculating the determinant of matrix A


Sol: In this problem, by using a simple multiplication
and adjoint of A, we get an inverse of matrix A, i.e. A −1 .
method we can get the matrix A2, then by substituting
By multiplying this into [A][X] = [O] we get the required
these in the given equation we will easily obtain the
values of x, y, and z.
required result.
We can also find if that given system has a trivial
1 2 2  1 2 2 
solution or not.    
A = A. A = 2 1 2 2 1 2
2

As given, 2x – 3y + z = 9 2 2 1  2 2 1 
   
x+y+z=6
1 + 4 + 4 2 + 2 + 4 2 + 4 + 2  9 8 8
x–y+z=2    
= 2 + 2 + 4 4 + 1 + 4 4 + 2 + 2  =  8 9 8 
This system can be written as AX = B, 2 + 4 + 2 4 + 2 + 2 4 + 4 + 1 8 8 9
   
2 −3 1  x Now A2 – 4A – 5I
   
Where, A = 1 1 1 X = y  9 8 8   4 8 8  5 0 0 
1 −1 1 z       
    = 8 9 8  −  8 4 8  − 0 5 0 
9  8 8 9   8 8 4  0 0 5 
     
 
and B = 6  9 − 4 − 5 8 − 8 − 0 8 − 8 − 0  0 0 0 
2     
  = 8 − 8 − 0 9 − 4 − 5 8 − 8 − 0  =0 0 0 
   
|A|=2(2)+3(0)+1(-2) = 2 8 − 8 − 0 8 − 8 − 0 9 − 4 − 5  0 0 0 

 1 −1 1 1 1 1 
T = 0 [Here 0 is the zero matrix]
 − 
 −1 1 1 1 1 −1  Thus A2 – 4A – 5I = O
 −3 1 2 1 2 −3  ∴ A–1 A2 – 4A–1 A – 5A–1I = A–1O = O
−
Adj A = − 
 −1 1 2 1 1 −1  or (A–1A)A – 4(A–1A) – 5A–1I = O;
 
 −3 1 − 2 1 2 −3 
or IA – 4I – 5A–1 = O; ∴ 5A–1 = A – 4I
 1 1 1 1 1 1 

 1 2 2   4 0 0   −3 2 2 
 2 2 −4       
  =  2 1 2  − 0 4 0  =
 2 −3 2 
=  0 1 −1   2 2 1   0 0 4   2 2 −3
 −2 −1 5       
 
 −3 2 2   −3 / 5 2 / 5 2 / 5 
1 1    
∴ A −1 = Adj A ∴ A =  2 −3 2  =  2 / 5 −3 / 5 2 / 5 
–1

|A| 5
 2 2 −3  2 / 5 2 / 5 −3 / 5
   
Now, X = A −1B
Example 4: Find the product of two matrices
 2 2 −4  9   22 
1   1   −5 1 3
=  0 1 −1  6  =  4   
2 2 A and B where A =  7 1 −5 
 −2 −1 5  2   −14 
      1 −1 1 
1 1 2  
∴ x =11, y=2, z=-7 is the solution.  
B =  3 2 1  and use it for solving the equations
2 1 3
 
x + y + 2z =1, 3x + 2y + z = 7 and 2x + y + 3z = 2
M a them a ti cs | 16.31

Sol: As the given system of equations is in the form Sol: Pre-multiplying both sides by B–1 and Post-
BX = C, multiplying it by B–1, which is obtained by the multiplying both sides by A–1 in
multiplication of AB, we can get the required result.
1 0 1 
 −5 1 3  1 1 2  BPA =   we can find P.
    0 1 0 
AB =  7 1 −5 3 2 1 
 1 −1 1  2 1 3  1 0 1 
    Given BPA =  
 −5 + 3 + 6 −5 + 2 + 3 −10 + 1 + 9  4 0 0 0 1 0 
   
=  7 + 3 − 10 7 + 2 − 5 14 + 1 − 15  =  0 4 0  1 0 1  –1
 1−3+2 B–1BPA A–1= B–1   A
 1−2+1 2 − 1 + 3  0 0 4 
  0 1 0 

Also the given system of equations in matrix form is 1 0 1  –1


⇒ IPI = B–1   A
BX = C  … (ii) 0 1 0 
x 1 
    1 0 1  –1
Where X =  y  and C = 7  ⇒ P = B–1   A  …(i)
z  2  0 1 0 
   
From (ii), X = B–1C 2 3 
To find B–1, B=  
[Multiplying both sides of (ii) by B–1 3 4 

∴ B–1 B = I 2 3
|B|= = 8 – 9 = –1 ≠ 0
A 3 4
From (1), AB = 4I3 ∴ . B = I3
4 Let C be the matrix of co-factors of elements in | B |;
 −5 / 4 1 / 4 3/4  C
A   C12 
∴ B–1 = =  7 / 4 1 / 4 −5 / 4  C =  11 
4
 1 / 4 −1 / 4 1 / 4  C21 C22 
 
∴ C11 = 4 C12 = –3 C21 = – 3 C22 = 2
x
 
∴  y  = X = B–1C  4 −3
∴C=  
z   −3 2 
 

 −5 / 4 1 / 4 3/4  1  adjB C'


∴ B–1 = = = – C’
    |B | −1
=  7 / 4 1 / 4 −5 / 4  7 
 1 / 4 −1 / 4 1 / 4  2   4 −3  −4 3 
   
=–  =    … (ii)
 5 7 6  −3 2   3 −2
− + + 
 4 4 4 2 1 1 1 
 7 7 10     
= + − =  1  ∴ x = 2, y = 1, z = –1 To Find A–1, Since A = 2 4 1
4 4 4 
   −1 2 3 1 
 1–7+2     
 4 4 4 
 
∴ | A | = 1(4 – 3) – 1(2 – 2) + 1(6 – 8)
1 1 1 
  2 3  =1–0–2=–1≠0
Example 5: Given A = 2 4 1 , B =  
2 3 1  3 4  Let C be the matrix of co-factors of elements in | A |;
 
C11 C12 C13 
1 0 1   
Find P such that BPA =   C = C21 C22 C23 
0 1 0  C C32 C33 
 31
1 6 . 3 2 | Matrices

 4 1 2 1 2 4  cos x − sinx 0  cos y − sin y 0 


 −     
 3 1 2 1 2 3  F(x) F(y) =  sinx cos x 0   sin y cos y 0 
 1  0 0 1   0 0 1 
1 1 1 1 1 
= − − 
 3 1 2 1 2 3
  cos x cos y − sinx sin y − cos x sin y − sinx cos y 0 
 1 1

1 1 1 1  
= sinx cos y + cos x sin y − sinx sin y + cos x cos y 0 

 4 1 2 1 2 4 
  0 0 1 

 1 0 −2  1 2 −3 cos(x + y) − sin(x + y) 0 


     
=  2 −1 −1 ; ∴ C’ = =  sin(x + y) cos(x + y) 0  = F(x +y)
 0 −1 1 
 −3 1 2   −2 −1 2   0 0 1 
    
i.e. F(x).F(y) = F(x +y)  … (i)
 1 2 −3
  2nd part.
Adj A =  0 −1 1 
 −2 −1 2  As we know that F(x) [F(x)]–1= I  … (ii)
 
Adj A Replacing y by –x in (i),
A–1 = = – Adj, A
|A| we get F(x). F(–x) = F(x – x) = F(0)

 −1 −2 3  cos0 − sin0 0  1 0 0 
     
=  0 1 −1 … (iii) =  sin0 cos0 0  = 0 1 0 
 2 1 −2  0 0 1  0 0 1 
    
Substituting eq. (ii) and (iii) in eq. (i), we get i.e. F(x) F(–x) = I  … (iii)
therefore from (ii) and (iii)
 −1 −2 3 
 −4 3  1 0 1    ⇒ [F(x)]–1 = F(–x).
P=   ×   ×  0 1 −1
 3 −2   0 1 0   2 1 −2
 
Example 7: Show that every square matrix A can
 −1 −2 3  be uniquely expressed as P + iQ where P and Q are
 −4 3 −4   
P=   ×  0 1 −1 Hermitian matrices.
 3 −2 3   2 1 −2
  1
Sol: By considering P = (A + Aθ)
2
 4 + 0 − 8 8 + 3 − 4 −12 − 3 + 8  1
P=   And Q = (A –Aθ) we get A = P + iQ
 −3 − 0 + 6 −6 − 2 + 3 9 + 2 − 6  2i
Then, using the property of a Hermitian matrix we can
 −4 7 −7  prove the above problem.
P = 
 3 −5 5 
θ
1  1
Now P =  (A + A θ ) = (A + Aθ)q
θ

2  2
cos α − sin α 0 
  1 θ 1 1
Example 6: If F(α) =  sin α cos α 0  then = {A + (Aθ)q} = (Aθ + A) = (A + Aθ) = P
 0 0 1  2 2 2

show that F(x). F(y) = F( x+ y). ∴ P = Pθ , hence P is a Hermitian matrix.
Hence, prove that [F(x)]–1 = F(– x). Similarly
θ
1   1 
Sol: By substituting x and y in place of α in given Qθ =  (A − A θ ) =   (A – Aθ)q
matrices we will get F(x) and F(y) respectively and then  2i   2i 
by multiplying them we will get the required result. 1 θ 1 1
=– {A – (Aθ)q} = – (Aθ – A) = (A – Aθ) = Q
2i 2i 2i
∴ Q is also Hermitian matrix,
M a them a ti cs | 16.33

Therefore A can be expressed as P + iQ ,where P and Q Example 9: If the non-singular matrix A is symmetric,
are Hermitian matrices. then prove that A–1 is also symmetric.
Let A = R + iS where R and S are both Hermitian matrices
Sol: By using the conditions of non-singular and
We have Aθ = (R + iS)θ = Rθ + (iS)q symmetric matrix we can easily find the required result.
= Rθ + iSθ = Rθ – iSθ= R – iS As given matrix A is a non-singular symmetric matrix.
(since R and S are both Hermitian) ∴ | A | ≠ 0 and AT = A,

∴ A + Aθ = (R + iS) + (R – iS) = 2R So, A–1 exists

1 Now, AA–1 = I = A–1 A


⇒R= (A + Aθ) = P
2 ⇒ (AA–1)T = (I)T = (A–1A)T
Also A – Aθ = (R + iS) – (R – iS) = 2iS ⇒ (A–1)T AT= I = AT(A–1)T
1 ⇒ (A–1)T A = I = A(A–1)T [ AT = A]
⇒S= (A – Aθ) = Q
2i ⇒ A–1 = (A–1)T
Hence expression (1) for A is unique
⇒ A–1 is symmetric.

Example 8: If A is Hermitian such that A2 = 0, show that


Example 10: Find the rank of the matrix
A = 0,
2 3 −1 −1 
Sol: As A is a Hermitian matrix therefore Aθ = A. By  
1 −1 −2 −4 
considering A = [aij]n×n to be a Hermitian matrix of order A= 
3 1 3 −2 
n and as given A2 = 0, we can solve given problem as  
follows. 6 3 0 −7 
 a11 a12 ...... a1n 
  Sol: given
a a22 ...... a2n 
A =  21 and 1 −1 −2 −4 
 ...... ...... ...... ...... 
   
2 3 −1 −1 
 an1 an2 ...... ann  A= 
3 1 3 −2 
 a11 a21 ...... an1   
  6 3 0 −7 
a a22 ...... an2 
A =  12
θ
R 2 → R 2 – 2R1 ,R 3 → R 3 − 3R1 
...... ...... ...... ...... 
   
 and R 4 → R 4 – 6R1 
 a1n a2n ...... ann 

Since A2 = 0; 1 −1 −2 −4 
 
0 5 3 7
Let AAθ = [bij]n×n ⇒ AAθ = 0 A= 
0 4 9 10 
Then each element of AAθ is zero and so all the principal  
0 9 12 17 
diagonal elements of AAθ are zero
∴ bii = 0 for all i = 1, 2, ……, n [Applying R4 → R4 - R2 - R3]
Now, bii = ai1 a i1+ ai2 a i2 + … + ain a in 1 −1 −2 −4 
 
= |ai1| + |ai2| + …… |ain| ∴ bii = 0
2 2 2 0 5 3 7
= 
0 4 9 10 
⇒ |ai1|2 + |ai2|2 + …… + |ain|2 = 0  
⇒ |ai1| = |ai2| = …… = |ain| = 0
0 0 0 0 

⇒ ai1 = ai2 = …… = ain = 0 [Applying R2 → R2 – R3]

⇒ each element of the ith row of A is zero, but bii = 0 ∀ 1 −1 −2 −4 


i = 1, ……. n  
0 1 −6 −3 
= 
∴ Each element of each row of A is zero. Hence, A = O 0 4 9 10 
 
0 0 0 0 
1 6 . 3 4 | Matrices

[Applying R3 → R3 – 4R2] [Applying R3 → 1/11 R3]

1 −1 −2 −4  1 −1 −2 −4 
   
0 1 −6 −3  0 1 −6 −3 
=  ~ 
0 0 33 22  0 0 3 2 
   
0 0 0 0  0 0 0 0 
Since the equivalent matrix is in echelon form having
three non-zero rows. Hence, r(A) = 3

JEE Main/Boards

Exercise 1 Q.11 If f(x) = 3x2 – 9x + 7, then for a square matrix A,


write f(A).
 2x − 1   3 
Q.1 Find x and y, if   =  
 5  x + y Q.12 If A, B and AB are symmetric matrices, then what
is the relation between AB and BA?
Q.2 A matrix has 2 rows and 3 columns. How many 1 
elements a matrix has? Find the number of elements of  
Q.13 If A = 2  B, [2 –2 4], find AB.
a matrix if it has 3 rows and 2 columns.
3 
 
Q.3 Order of matrix A is 2 × 2 and order of matrix B is 2 1 2 0
1 2   
× 3. Find the order of AB and BA, if defined. Q.14 Are the matrix   and  3 4 0  equal?
Give reasons. 3 4  0 0 0
 
Q.4 Given a matrix A = [aij], 1 ≤ i ≤3 and 1 ≤ j ≤ 3, where
aij = i + 2j. Write the element  2 −1
Q.15 Given a matrix A =  .
(i) a11 (ii) a32 (iii) a23 (iv) a34 4 2 
1
Find matrix kA, where k = –
Q.5 A matrix has 18 elements. Write the possible orders 2
of matrix.
sec θ tan θ 
Q.16 Simplify: tan θ  
Q.6 Give an example of a diagonal matrix, which is not tan θ − sec θ 
a scalar matrix. Also give an example of a scalar matrix.
 − tan θ − sec θ 
+ sec θ  .
Q.7 For the matrix A, show that A + AT is a symmetric  − sec θ tan θ 
matrix.
Q.17 If Xm×3YP×4 = Z2×b, for three matrices X, Y, Z, find the
values of m, p and b.
Q.8 For the matrix A, Show that A – AT is a skew-
symmetric matrix.  0 −1 2 
 
Q.18 Is matrix A =  1 0 −3 symmetric or skew-
Q.9 The total number of elements in a matrix represents  −2 3 0 
 
a prime number. How many possible orders a matrix
can have ? symmetric? Give reasons.

 x   −1  0 cos θ sin θ 


Q.10 Find x and y, if   +   =   π
Q.19 If R (θ) =   , write (i) R   , (ii) R(x+y)
 2y   4  3  sin θ − cos θ  2
M a them a ti cs | 16.35

Q.20 For a skew-symmetric matrix A = [aij], what is the  cos α sin α 


nature of elements aij, if i = j. Q.34 If A =   , then show that
 − sin α cos α 
a 0  cos2α sin2α 
Q.21 If A =   , find A16. A2 =  
0 0  − sin2α cos2α 
 1 0  x 
Q.22 Find x, if [x 1]     =0 0 1  1 0 
 −2 −3 3 Q.35 If A =   and I =  ,
0 0  0 1 
 2 −1  prove that (aI + bA)3 = a3I + 3a2bA.
Q.23 Find the sum of matrix A =   and its
additive inverse. 4 6 
 0 1
Q.36 If A =   , find the values of
2 −1 2 4   −1 1
Q.24 Find X, if X +  =  
3 −1 5 0  p and q such that (pI + qA)2 = A.

 sin2 θ 1   cos2 θ  2 3 −4  5 1 2
0   0 −1    
Q.25 Evaluate,  +  +   Q.37 If A =  1 0 6  and B = 6 −1 4
cot2 θ 0   − cosec2 θ 1   −1 0  
 −2 1 5  5 3 −4 
   
Q.26 If A and B are symmetric matrices, show that AB find 2A – 3B.
is symmetric.
Q.38 Construct a 3 × 3 matrix [aij], whose elements are
Q.27 If a matrix has 8 elements, what the possible given by aij = 2i – 3j.
orders it can have ? What if it has 5 elements?
 x 3x − y  3 2
Q.39 If  =   , find x, y, z, w.
Q.28 Evaluate the following: 2x + z 3y − w  4 7
a
  Q.40 Find matrices X and Y, if
c  b
[a, b]   + [a b c d]   5 2  3 6 
d c  X+Y=   and X – Y =  
  0 9  0 −1
d

0 0 1   cos2 θ cos θ sin θ 


  Q.41 If A =  ;
Q.29 If A = 0 1 0  , find A2. Hence find A6  cos θ sin θ sin2 θ 

1 0 0 
 
 cos2 φ cos φ sin φ 
B =  , then show that AB is zero
 cos φ sin φ sin2
φ 
Q.30 Show that the element of the main diagonal of a  
skew-symmetric matrix are all zeros. matrix, provided (θ – φ) is an odd multiple of π/2.

0 −4  5 −7   −1 1 −1  0 4 3
Q.31 Find AB, if A =   and B =      
0 −3  0 0  Q.42 If A =  1 −3 3  and B =  1 −3 −3  ,
 5 −5 5   −1 4 4 
   
 4 3
Q.32 If A =   , find values of x and y such that compute A2B2.
 2 5
A2 – xA + yI = O where I is a 2×2 unit matrix and O is a
Q.43 Find the matrix X such that,
2 × 2 zero matrix.
 2 −1   −1 −8 −10 
 1 3 5  4 5 −9     
Q.33 If A =   and A – 3B =  , 0 1 X+ 3 4 0 
find B.  −2 5 7  1 2 3   −2 4  10 20 10 
   
1 6 . 3 6 | Matrices

Exercise 2 Q.9 If A is a skew symmetric matrix such that ATA = I,


then A4n–1(n∈N) is equal to
Single Correct Choice Type (A) –AT (B) I (C) – I (D) AT

Q.1 If number of elements in a matrix is 60 then how Q.10 A and B are 2 × 2 matrices satisfying det
many dimensions of matrix are possible A = det B and tr(A) = tr(B), further A2 – 3A + 14I = 0 and
(A) 12 (B) 6 (C) 24 (D) None of these B2 – λB + µI = 0, then µ is equal to
(A) 3 (B) 11 (C) –11 (D) 14
Q.2 Matrix A has x rows and x + 5 columns. Matrix B has
y rows and 11 – y columns. Both AB and BA exist, then Q.11 The false statement is -
(A) x = 3, y = 4 (B) x = 4, y = 3 (A) The adjoint of a scalar matrix is scalar matrix.
(C) x = 3, y = 8 (D) x = 8, y = 3 (B) The adjoint of upper triangular matrix is lower
triangular matrix.
Q.3 If A is square invertible matrix such that A2 = A, (C) The adjoint of upper triangular matrix is upper
then det.(A2 – I) is triangular matrix.
(A) 1 (B) 2 (C) 3 (D) None of these (D) adj(adj A) = A, A is a square matrix of order 2.

Q.4 Number of distinct matrices that can be formed Q.12 If the matrices A, B, (A + B) are non-singular, then
using all the 143 distinct elements is [A(A + B)–1B]–1 is equal to
(A) 4! (B) 4(143)! (C) 2(143)! (D) (143)! (A) A + B (B) A–1 + B–1
(C) (A + B)–1 (D) None of these
Q.5 If A = A, then (I + A) is equal to
2 4

(A) I + A (B) I + 4A Q.13 If A is an orthogonal matrix | A | = – 1, then AT is


equal to
(C) I + 15A (D) None of these
(A) – A (B) A
 α β (C) – (adj A) (D) (adj A)
Q.6 If A =   is an orthogonal matrix, where α,
 −β α 
β and the roots other than the common root of the Q.14 If A and B are square matrices of order 3, then
equations x2 – px + q = 0 & x2 + px – q = 0, then (A) adj(AB) = adj A+adj B
1 1 1 (B) (A + B)–1 = A–1 + B–1
(A) p=± , q=± (B) p = 0, q = ±
2 2 2
(C) AB = 0 ⇒ | A | = 0 or | B | = 0
1
(C) p = ± , q = 0 (D) None of these (D) AB = 0 ⇒ | A | = 0 and B = 0
2
a 0 0
Q.7 A is a square matrix of order n and (det A) = 3. If  
Q.15 If A = 0 a 0  , then | A | | adj A | is equal to
det (λA) = 81; where λ ∈ N, then possible value of n is
0 0 a 
 
(A) 3 (B) 5 (C) 2 (D) 7
(A) a25 (B) a27
(C) a81 (D) None of these
1 2 1+x
Q.8 If A =   and f(x) = , then f(A)is
2 1  1−x
 1 tanx 
Q.16 If A =   , then the value of | A A | is
T –1
1 1  −1 −1 − tanx 1
(A)   (B)    
1 1   −1 −1
(A) cos 4x (B) sec2x
2 2  (C) – cos 4x (D) 1
(C)   (D) None of these
2 2 
M a them a ti cs | 16.37

2 3   a2 ab ac 
Q.17 If A =   , then 19A–1 is equal to  
5 − 2  B = ab b2 bc  then AB is equal to
1  
(A) AT (B) 2A (C) A (D) A ac bc c2 
2  
(A) A3 (B) B2 (C) O (D) I
Q.18 If P is a two-rowed matrix satisfying PT= P–1, then
P is Q.25 If A, B, C are square matrices of same order & AB
 cos θ − sin θ   cos θ sin θ  = BA, C2 = B, then (A–1CA)2 is equal to
(A)   (B)  
 − sin θ cos θ   − sin θ cos θ  (A) B2 (B) A2 (C) C2 (D) C

 − cos θ sin θ  Q.26 A is a diagonal matrix of order 3, and tr(A) = 12.


(C)   (D) None of these
 sin θ − cos θ  If all diagonal entries are positive then maximum value
of det (A) is
Q.19 If A and B are two non-singular matrices of the (A) 8 (B) 16 (C) 32 (D) 64
same order such that Br = I, for some positive integer
r > 1, then A–1Brr–1 A A–1B–1 A is equal to. Q.27 If A and B are two matrix such that AB = B and
(A) 0 (B) I BA = A, then A2 + B2 is equal to

(C) A–1 (D) None of these (A) 2AB (B) 2BA (C) A + B (D) AB

Q.20 If A and B are orthogonal matrices of same order, 0 1 


Q.28 A =   and B is column matrix such
then: 3 0 
(A) A + B is also orthogonal. 0 
that (A8 + A6 + A4 + A2 + I), B =   where I
(B) A – B is also orthogonal. 1 
is a unit matrix of order 2 ×2, then B is equal to
(C) AB is also orthogonal.
    1
(D) AB + BA is also orthogonal. 0    
0    0  11 
(A)   (B)   (C)   (D)  
Q.21 If C is an orthogonal matrix and A is a square 1  2 1 1
   
matrix of same order then, trace of CTAC is equal to  11  2
 
 
   11 
(A) Trace of C (B) Trace of AC
(C) Trace of A (D) None of these matrix
Q.29 If A and B are square matrices of same order such
that AB = BA and A2 = I, then ABA is equal to
Q.22 Let A and B are idempotent matrices such that
A.B = BA and A – B is non singular then | A + B | is equal (A) (AB)2 (B) I (C) B (D) B2
to
(A) 0 (B) –1 (C) 1 (D) ±1
Previous Years’ Questions
Q.23 If A and B are square matrices of same order and
AAT = I, then (ATBA)10 is equal to Q.1 The parameter, on which the value of the

(A) AB10AT (B) ATB10A 1 a a2


(C) A10B10(AT)10 (D) 10ATBA determinant cos(p − d)x cospx cos(p + d)x
sin(p − d)x sinpx sin(p + d)x
 0 c −b 
  does not depend upon, is  (1997)
Q.24 If A =  −c 0 a  and
 b −a 0  (A) a (B) p (C) d (D) x
 
1 6 . 3 8 | Matrices

1 x x +1 bc ca ab
Q.2 If f(x) = 2x x(x − 1) (x + 1)x Q.9 Find the value of the determinant p q r
3x(x − 1) x(x − 1)(x − 2) (x + 1)x(x − 1) 1 1 1
then f(100) is equal to  (1999) where a, b, and c are respectively the pth, qth and rth
(A) 0 (B) 1 (C) 100 (D) – 100 terms of a harmonic progression (1987)

Q.10 Suppose, f(x) is a function satisfying the following


Q.3 The number of distinct real roots of
conditions:  (1998)
sinx cos x cos x (a) f(0) = 2, f(1) =1
cos x sinx cos x = 0 in the interval 5
(b) f has a minimum value at x = , and
cos x cos x sinx 2
π π (c) For all x,
– ≤x≤ is  (2001)
4 4 2ax 2ax − 1 2ax + b + 1
f’(x)= b b +1 −1
(A) 0 (B) 2 (C) 1 (D) 3
2(ax + b) 2ax + 2b + 1 2ax + b
Q.4 The number of values of k for which the system of where a, b are some constants. Determine the constants
equations  (2004) a, b and the function f(x)
(k + 1)x + 8y = 4k, kx + (k + 3)y = 3k – 1
(A) 0 (B) 1 (C) 2 (D) –1 Q.11 Prove that for all values of θ, (2000)
sin θ cos θ sin2θ
Q.5 Given, 2x – y + 2z = 2, x – 2y + z = –4, x +y + lz =
4, then the value of λ such that the given system of  2π   2π   4π 
sin  θ +  cos  θ +  sin  2θ + 
equations has no solution is  (2004)  3   3   3  =0

(A) 3 (B) 1 (C) 0 (D) –3


 2π   2π   4π 
sin  θ −  cos  θ −  sin  2θ − 
 3   3   3 
 3 /2 1/2  1 1
Q.6 If P =  ,A=   and
 −1 / 2 3 / 2  0 1  Q.12 If A is an 3 × 3 non – singular matrix such that
AA' = A' A and B = A −1 A then BB’ equals:  (2014)
Q= PAPT, then PTQ2005P is  (2005)
(A) I + B (B) I (C) B−1 (D) (B−1 )
1 2005  1 2005
(A)   (B)  
0 1  2005 1 
1 2 2 
 
 1 0 1 0  =
Q.13 If A 2 1 −2 is a matrix satisfying the
(C)   (D)   a 2 b 
2005 1  0 1   
equation AA T = I where I is 3 3 × 3 identity matrix, then
1 0 0  the ordered pair (a, b) is equal to:  (2015)
 
Q.7 If A = 0 1 1  , 6A–1 = A2 + cA + dI, then
(A) ( 2, −1 ) (B) ( −2,1 )
0 −2 4 
 
(C) ( 2,1 ) (D) ( −2, −1 )
(c, d) is (2005)

(A) (–6, 11) (B) (–11, 6) (C) (11, 6) (D) (6, 11)
5a −b  T
Q.14 If A =   and A adjA = AA , then 5a + b is
 3 2 
Q.8 Let α1, α2, β1, β2 be the roots of ax2 + bx + c = 0 and
px2 + qx + r = 0 respectively. If the system of equations equal to  (2016)
α1y + α2z = 0 and β1y + β2z = 0 has a non-trivial (A) -1 (B) 5 (C) 4 (D) 13
2
b ac
solution. Then prove that =  (1987)
2 pr
q
M a them a ti cs | 16.39

JEE Advanced/Boards

Exercise 1 idempotent matrix. Find the absolute value of sum of


the products of elements of the set S taken two at a
time.
Q.1 (a) A3×3 is a matrix such that | A | = a, B = (adj A)
such that | B | = b. Find the value of cos x − sinx 0 
 
1 a a2 a3 Q.6 If F(x) =  sinx cos x 0  then show that F(x).
(ab2 + a2b + I) S Where S= + + +  0 0 1 
2 b b3 b5 
F(y) = F(x + y).
…… up to ∞, and a = 3
Hence, Prove that [F(x)]–1 = F(–x).
(b) If A and B are square matrices of order 3, where | A |
= –2 and | B | = I, then find (A–1) adj (B–1) adj(2A–1)
Q.7 Let An and Bn be square matrices of order 3, which
are defind as An = [aij] and Bn = [bij]
Q.2 Let A be the 2 × 2 matrices given by A = [aij] where
aij ∈ {0, 1, 2, 3, 4} such that a11 + a12 + a21 + a22 = 4 2i + j 3i – j
where aij = and bij = for all i and j, 1 ≤ i.,j ≤ 3.
2n
(i) Find the number of matrices A Such that the trace of 3 22n
A is equal to 4. If l = lim Tr (3A1 + 32A2 + 33A3 + …. 3nAn) and
n→∞
(ii) Find the number of matrices A such that A is
invertible. m = lim Tr(2B1 + 22B2 + 23B3 + …. 2nBn)
n→∞
(iii) Find the absolute value of the difference between
maximum value and minimum value of det (A). Then find the value of ( l + m).

(iv) Find the number of matrices A such that A is either [Note: Tr (P) denotes the trace of matrix P]
symmetric or skew-symmetric or both and det (A) is
divisible by 2. Q.8 Let A be a 3 × 3 matrix such that a11 = a33 = 2 and
all the other aij = 1.
 4 −4 5 
  Let A–1 = xA2 + yA + zI then find the value of (x + y + z)
Q.3 For the matrix A =  −2 3 −3 find A2.
where I is a unit matrix of order 3.
 3 −3 4 
 
1 2 2
1 1 1   
2 3  Q.9 Given that A = 2 2 3 ,
 
Q.4 (a) Given A = 2 4 1 ,B =  , 1 −1 3 
2 3 1  3 4   
 
2 1 1  10 
1 0 1     
C = 2 2 1  , D = 13 and that Cb = D.
Find P such that BPA =  
0 1 0  1 1 1 
 
9
 
(b) Find the matrix A satisfying the matrix Solve the matrix equation Ax = b.
2 1  3 2  2 4 
equation   . A.   =   2 0 7   −x 14x 7x 
3 2  5 −3 3 −1    
Q.10 Let A = 0 1 0  and B = 0 1 0  are
1 −2 1   x −4x −2x 
Q.5 Let S be the set which contains all possible values    
of I, m, n, p, q, r for which two matrices such that AB = (AB)–1 and AB ≠ 1 (where I
is an identity matrix of order 3 × 3).
l2 − 3 p 0 
  Find the value of Tr (AB + (AB)2 + (AB)3 + …… + (AB)100)
A =  0 m2 − 8 q  Be a non singular
  where Tr.(A) denotes the trace of matrix A.
 r 0 n2 − 15 

1 6 . 4 0 | Matrices

Q.11 Let Mn = [mij] denotes a square matrix of order n Q.17 The set of natural numbers is divided into arrays
with entries as follows. of rows and columns in the form of matrices as A1 = (1),
For 1 ≤ i ≤ n , mii = 10; For 1 ≤ i ≤ n – 1, mi+1, i = mi, i + 1 = 3; 6 7 8
 2 3  
And all other entries in Mn are zero. Let Dn be the A2 =   , A3 =  9 10 11  …………… So on.
 4 5  12 13 14 
determinant of matrix Mn, then find the value of  
(D3 – 9D2).
Find the value of Tr(A10).
[Note: Tr(A) denotes trace of A]
Q.12 Find the product of two matrices A & B,

 −5 1 3  1 1 2  I
    xn
where A =  7 1 −5 & B= 3 2 1  and use it to Q.18 Consider In.m = ∫ xm − 1 dx and J n.m
0
 1 −1 1  2 1 3  I
    xn
solve the following system of linear equations ∫ xm + 1 dx∀ n > m and n, m ∈ N.
0

x + y + 2z = 1; 3x + 2y + z =7; 2x + y + 3z = 2 (a) Consider a matrix A = [aij]3×3,

I –I , i= j
Q.13 Determine the values of a and b for which the where aij =  6 +i,3 i+3,3 . Then find trace (A–1).
 0, i≠ j
3 −2 1   x   b 
      [Note: Trace of a square matrix is sum of the diagonal
system 5 −8 9   y  =  3  elements.]
2 1 a   z   −1
     
 J6,5 72 J11,5  I6,5 72 I11,5 
(i) Has a unique solution;    
(b) Let A =  J7,5 63 J12,5  and B = I7,5 63 I12,5 
(ii) Has no solution and    
 J8,5 56 J13,5  I8,5 56 I13,5 
(iii) Has infinitely many solutions.
then find the value of det (A) – det (B)

1 2  3 1  1 2  3 −4 
Q.14 If A =  ;B=  ;C=   Q.19 Consider the matrices A = 
3 4  1 0  2 4   and
1 −1 
x x2  a b
and X =  1 B=   and let P be any orthogonal matrix and
 then solve the following
 x3 x4  0 1 
Q = PAPT and R = PTQKP also S = PBPT and T = PTSKP
matrix equations.
(a) AX = B – 1 (b) (B – 1) X = IC Column I Column II
(c) CX = A (A) If we vary K from 1 to n (p) G.P. with common
then the first row first column ratio a
Q.15 If A is an orthogonal matrix and B = AP where P elements of R will form
is a non singular matrix, then show that the matrix PB–1 (B) If we vary K from 1 to n then (q) A.P. with volume
is also orthogonal. the 2nd row 2nd column elements difference 2
of R will form
Q.16 Let M be a 2 × 2 matrix such that M (C) If we vary K from 1 to n (r) G.P. with common
1 then the first row first column ratio b
1  −1 1  elements of T will form
  =   and M  −1 =   . If x1 and
2
−1
  2
    0  (D) If we vary K from 3 to n (s) A.P. with volume
x2 (x1 > x2) are the two values x for which det (M – xI) = 0, then the first row 2nd column difference –2
where I is an identity matrix of order 2, then find the value elements of T will represent the
of (5x1 + 2x2). sum of
M a them a ti cs | 16.41

Q.20 Consider a square matrix A of order 2 which has Q.5 If α, β, γ are the real numbers and
its elements as 0, 1, 2 and 4. Let N denote the number
of such matrices, all elements of which are distinct.  1 cos(α − β) cos(α − γ ) 
 
A =  cos(β − α ) 1 cos(β − γ )  then
Column I Column II  cos( γ − α ) cos( γ − β) 1 
 
(A) Possible non-negative value of det (A) is (p) 2
(A) A is skew symmetric
(B) Sum of values of determinants (q) 4
corresponding to N matrices is (B) A is invertible

(C) If absolute value of (det(A)) is least, then (r) – 2 (C) A is non singular
possible value of |adj(adj(adj A))| (D) | A | = 0
(D) If det (A) is algebraically least, then (s) –2
possible value of det (4A–1) is Q.6 The values of x for which the matrix
(t) 8
x + a b c 
 
 a x +b c  is non-singular are
 a b x + c 

Exercise 2
(A) R – {0}
Single Correct Choice Type (B) R – { – (a + b + c)}

Q.1 Let A, B be two square matrices of the same (C) R – { 0, – (a + b + c)}


dimension and let [A, B] = AB – BA, then for three 2 × (D) None of these
2 matrices
A, B, C, [[A, B],C] + [[B, C],A] + [[C, A],B] = Q.7 Let A is a skew symmetric matrix such A2 = A, and B
is a square matrix such that BTB = B; | B | ≠0. If X = (A +
(A) 1 (B) 0
B) (A – B), then XTX is
(C) ABC – CBA (D) None of these
(A) A – I (B) I – A
1 2 3 (C) A (D) None of these
Q.2 A = 4 5 7 & f(x) = x3 – 8x2 + bx + γ. If A
2 3 α Q.8 For two uni-modular complex numbers z1 and z2,
satisfies f(x) = 0, then ordered pair (α, γ) is  z1 −z2 
−1
 z1 z2 
−1

    equal to
(A) (2, –7) (B) (–2, 7)  z2 z1   − z2 z1 
(C) (2, 7) (D) (–2, –7)
z z2  1 0 
(A)  1  (B)  
α β  z1 z2  0 1 
Q.3 If   is a square root of the two rowed unit
 γ δ 1 / 2 0 
(C)   (D) None of these
matrix, then δ is equal to  0 1 / 2
(A) a (B) β
−2
(C) g (D) None of these 1 / 25 0   5 0
Q.9 If   =  ,
 x 1 / 25  −a 5 
4 2i then the value of x is
Q.4 For A = , (A – 2I) (A – 3I) is a
i 1 a 2a
(A) (B)
125 25
(A) Null-matrix (B) Hermitian matrix
2a
(C) Unit matrix (D) None of these (C) (D) None of these
125
1 6 . 4 2 | Matrices

Q.10 If A is square matrix such that A2 = I, |A| = 1 and = Ax + B Where A and B are determinants of order 3
B = (adj A)–1 then incorrect statement is not involving x.  (1982)
(A) AB = BA (B) AB = I
Q.4 Show that the system of equations 3x – y + 4z = 3, x +
(C) A = B (D) B = I
2y – 3z = – 2, 6x + 5y + lz = – 3 has at least one solution
for any real number λ ≠ –5. Find the set of solutions, if
Q.11 If A and B are square matrices of order 3 and adj λ = – 5.  (1983)
A = B, then adj (3AB) is equal to
(A) 3 | B |2 I3 (B) 9 | B | I3 Q.5 Consider the system of linear equations in x, y, z
(sin 3θ) x – y + z = 0, (cos 2θ) x + 4y + 3z = 0 and 2x +
(C) 3 | A |2 I3 (D) 9 | A | I3
7y + 7z = 0. Find the values of θ for which this system
has non-trivial solution.  (1986)
Q.12 Let A and B are square matrices of order n such
that AT + B = O, O is a null matrix, A = adj B, tr (A) = –1 a−1 n 6
and A2 = A then tr {adj(ATB)} is equal to
Q.6 Let Da = (a − 1)2 2n2 4n − 2
(A) (– 1)n–1 (B) 1 3 8
(a − 1) 3n 3n2 − 3n
(C) (– 1) n
(D) None of these n
Show that ∑ ∆a = c ∈ constant.  (1989)
a=1
Q.13 If A is a non-singular matrix such that C = A + B,
| C |2 = | A |2 | I – (A–1 B)2| and AB =BA, then
p b c
(A) B is null matrix (B) A is null matrix Q.7 If a ≠ p, b ≠ q, c ≠ r and a q c = 0
(C) | C | = | A – B | (D) | A | = | B | a b r
p q r
Then, find the value of + +  (1991)
p−a q−b r −c
Previous Years’ Questions
Q.8 For a fixed positive integer n, if
Q.1 Let ω ≠ 1 be a cube root of unity and S be the set of
n! (n + 1)! (n + 2)!
all non-singular matrices of the
D = (n + 1)! (n + 2)! (n + 3)! , then show that
1 a b (n + 2)! (n + 3)! (n + 4)!
 
form  ω 1 c  , where each of a, b or c is
ω2  D 
 ω 1   3 − 4  is divisible by n.  (1992)
 (n!) 
either ω and w . Then, the number of distinct matrices
2

in the set S is  (2011)


Q.9 Let λ and α be real. Find the set of all values of λ
(A) 2 (B) 6 (C) 4 (D) 8
for which the system of linear equations lx + (sin α)y +
(cos α)z = 0 x + (cos α)y + (sin α)z = 0 and –x + (sin α)
Q.2 Let M and N be two 3 × 3 non-singular skew- y – (cos α)z = 0 has a non-trivial solution for λ = 1, find
symmetric matrices such that MN = NM. If PT denotes all values of α.  (1993)
the transpose of P, then M2N2(MTN)–1(MN–1)T is equal to
 (2011)
Q.10 Let a, b, c be real numbers with a2 + b2 + c2 = 1.
(A) M 2
(B) –N 2
(C) – M 2
(D) MN Show that the equation

ax − by − c bx + ay cx + a
Q.3 Without expanding a determinant at any
bx + ay −ax + by − c cy + b =0
x2 + x x +1 x−2 cx + a cy + b −ax – by + c
2
stage, show that 2x + 3x − 1 3x 3x − 3 represents a straight line.  (2001)
2
x + 2x + 3 2x − 1 2x − 1
M a them a ti cs | 16.43

U is the zero matrix.


Q.11 Let ω ≠ 1 be a cube root of unity and S be the set
of all non−singular matrices of the form
(C) determinant of (M2+MN2) ≥ 1 .
1 a b
  (D) for a 3 × 3 matrix U, if (M2+MN2) U equals the zero
ω 1 c  where each of a, b and c is either ω or ω2 .
ω2 ω 1  matrix the U is the zero matrix.

Then the number of distinct matrices in the set S is
Q.17 The quadratic equation p(x)=0 with real coefficients
 (2011)
(A) 2 (B) 6 (C) 8 (D) 4 has purely imaginary roots. Then the equation p(p(x))=0
has
Q.12 Let M be a 3 × 3 matrix satisfying
(A) Only purely imaginary roots.
0   −1   1  1 1   0  (B) All real roots.
           
M=
1   2  ,M =
 −1   1  and M 1  =  0  (C) Two real and two purely imaginary roots.
0  3 0  −1  1  12 
            (D) Neither real nor purely imaginary roots.
Then the sum of the diagonal entries of M is  (2011)
−1 + 3i
(A) 5 (B) 6 (C) 9 (D) 8 Q.18 Let z = , where −1 ,and r,s ∈ {1,2,3} .
2
 −z r z 2s 
Q.13 If P is 3 × 3 matrix such that PT = 2P + I where PT is Let P = ( ) and I be the identity matrix of
he transpose of P and I is the 3 × 3 identity matrix, then  z 2s zr 

there exists a column matrix
order 2. Then the total number of or dered pairs (r, s)
 x  0 
   
X  y  ≠ 0  such that 
= (2012)
 z  0  for which P2 = − I is
   
3 −1 −2
0   
  Q.19
= Let P 2 0 α  where α ∈ R .
(A) PX 0  (B) PX = X (C) PX = 2X (D) PX = − X
3 5 0 
0   
 
Suppose Q = qij  is a matrix such that PQ = kI
 
Q.14 Let P = aij  is 3 × 3 matrix and let Q = bij  , where R,k ≠ 0 and I is the identity matrix of order 3. If
   
k k2
where bij = 2i+ j aij 1 ≤ i, j ≤ 3 .If the determinant of P is 2, q23 = − and det ( Q ) = , then
8 2
then the determinant of the matrix Q is  (2012)
α 0,k
(A) = = 8 (B) 4α − k + 8 = 0
(A) 210 (B) 211 (C) 212 (D) 213
( )
(C) det Padj ( Q ) = 29

( )
(D) det Padj (P ) = 213
Q.15 Let ω be a complex cube root of unity with ω ≠ 1
 1 0 0
and P = pij  be a n × n matrix with Pij = ωi+ j Then  
  Q.20 Let P =  4 1 0  and I be the identity matrix
P2 ≠ 0 , when n =  (2013) 16 4 1 
 
(A) 57 (B) 55 (C) 58 (D) 56
of order 3. If Q = qij  is a matrix such that p50 − Q =I
 
Q.16 Let M and N be two 3 × 3 matrices such that MN = q + q32
then 31 equals  (2016)
NM. Further, if M ≠ N2 and M2 = N4 then  (2014) q21

(A) 52 (B) 103 (C) 201 (D) 205


(A) determinant of (M2+MN2) is 0.
(B) there is a 3×3 non−zero matrix U such that (M2+MN2)
1 6 . 4 4 | Matrices

PlancEssential Questions
JEE Main/Boards JEE Advanced/Boards
Exercise 1 Exercise 1
Q.7 Q.8 Q.17 Q.7 Q.10 Q.13
Q.23 Q.32 Q.35 Q.18 Q.19 Q.20
Q.38 Q.41 Q.44 Q.17

Exercise 2 Exercise 2
Q.4 Q.11 Q.14 Q.2 Q.5 Q.8
Q.19 Q.22 Q.26 Q.12

Previous Years’ Questions Previous Years’ Questions


Q.1 Q.2 Q.6 Q.2 Q.4 Q.11
Q.10 Q.13 Q.12

Answer Key

JEE Main/Boards
Exercise 1

Q.1 x = 2, y = 3 Q.2 6; 6
Q.3 Order of AB is 2 × 3; order of BA is not defined Q.4 (i) 3 (ii) 7 (iii) 8 (iv) 11

2 0   −1 0 
Q.5 1 × 18, 2 × 9, 3 × 6, 6 × 3, 9 × 2, 18× 1 Q.6    
0 3   0 −1 
1
Q.9 Two Q.10 x = 1, y = –
2
Q.11 f(A) = 3A2 – 9S + 7I Q.12 AB = BA

 2 −3 4 
 
Q.13  4 −6 8  Q.14 No
 6 −9 12 
 

 −1 1 / 2  0 −1 
Q.15   Q.16  
 −2 −1   −1 0 
M a them a ti cs | 16.45

Q.17 m = 2, p = 3, b = 4 Q.18 Skew-symmetric

0 1  cos(x + y) sin(x + y) 
Q.19 (i)   (ii)   Q.20 Each element is zero
1 0   sin(x + y) − cos(x + y)

0 0
Q.21   Q.22 1 ± 10
0 0

0 0 0 5 
Q.23   Q.24  
0 0 2 1 

 1 0
Q.25   Q.27 1 × 8, 2 × 4, 4 × 2, 8 × 1; 1 × 1, 5 × 1
 −2 1 
1 0 0 
  6
Q.28 [ac + bd + a + b + c + d ]
2 2 2 2
Q.29 0 1 0  ; A = A
2

0 0 1 
 
0 0 
Q.31   Q.32 x = 9m, y = 14
0 0 

1  −2 1 19  1 1
Q.33 B =   Q.36 P ± ,q±
3  −5 8 11  3 3

 −11 3 −14   −1 −4 −7 
   
Q.37  −16 3 0  Q.38  1 −2 −5 
 −19 −7 22   3 0 −3
   
4 4 1 −2
Q.39 x = 3, y = 7, z = –2, w = 14 Q.40 X =  ,Y=  
0 4  0 5 

1 −1 −5
Q.42A2 = A, B2 = I; A2B2 = AI =A Q.43 X =  
3 4 0 

Exercise 2
Single Correct Choice Type

Q.1 A Q.2 C Q.3 D Q.4 B Q.5 C Q.6 C


Q.7 A Q.8 B Q.9 D Q.10 D Q.11 B Q.12 B
Q.13 C Q.14 C Q.15 D Q.16 D Q.17 D Q.18 B
Q.19 A Q.20 C Q.21 C Q.22 C Q.23 B Q.24 C
Q.25 C Q.26 D Q.27 C Q.28 C Q.29 C

Previous Years’ Questions


Q.1 B Q.2 A Q.3 C Q.4 B Q.5 B

1 5 1 2 5
Q.6 A Q.7 A Q.9 0 Q.10 a = ,b= f (x) = x − x+2
4 4 4 4
Q.12 B Q.13 D Q.14 B
1 6 . 4 6 | Matrices

JEE Advanced/Boards
Exercise 1
Q.1 (a) 225(b) –8 Q.2 (i) 5 (ii) 18 (iii) 8 (iv) 5

 17 4 −19 
   −4 7 −7  1  48 −25
Q.3  −10 0 13  Q.4 (a)   (b)  
 3 −5 5  19  −70 42 
 −21 −3 25 
 
Q.5 29 Q.7 21
Q.8 1 Q.9 x1 = 1, x2 = –1, x3 = 1
Q.10 100 Q.11 1
Q.12 x = 2, y = 1, z = – 1
Q.13 (i) a ≠ –3, b ∈ R    (ii) a = –3 and b ≠ 1/3   (iii) a = –3, b = 1/3
 −3 −3
1 2
Q.14 (a) X =  5     (b) X =
     (c) No solution
 2  −1 −2 
 2 
Q.16 8` Q.17 3355
Q.18 (a) 18   (b) 0 Q.19 A→q; B → s; C→ p; D→ p
Q.20 A → p, q, t; B → s; C→ p, r; D → r

Exercise 2
Single Correct Choice Type

Q.1 B Q.2 A Q.3 A Q.4 A Q.5 D Q.6 C


Q.7 B Q.8 C Q.9 C Q.10 D Q.11 B Q.12 C
Q.13 C

Previous Years' Questions


π
Q.1 A Q.2 C Q.4 ∆ = ∆1 = ∆2 = ∆3 = 0 Q.5 θ = nπ, nπ + (–1)n ,n∈Z
6
n
Q6. ∑ ∆a = c Q.7 2 Q.8 2n(n2 + 4n + 5) Q.9 α = nπ or nπ + π/4
a=1
Q.10 0 Q.11 A Q.12 9 Q.13 D Q.14 D Q.15 B, C, D

Q.16 A, B Q.17 D Q.18 A Q.19 B, C Q.20 B


M a them a ti cs | 16.47

Solutions

JEE Main/Boards Sol 7: Matrix A = [aij] assume


AT = [aji]
Exercise 1 So A + AT = [aij + aji] = [bij] assume
2x − 1  3  Here bij = aij = aji
Sol 1:   =  
 5  x + y  bji = aij + aji
2x – 1 = 3 Here bij = bji
2x = 4 ⇒ x = 4/2 = 2 Matrix is symmetric.
5=x+y=2+y
Sol 8: A – AT
y=5–2=3
⇒ [aij] – [aji] = [bij] assume
(x, y) = (2, 3)
bij = aij – aji
Sol.2 row = n bji = aji – aij
Column = m ⇒ bij = – bij
Then total elements = mn This matrix is known as symmetric matrix.
if (n, m) = (2, 3) ⇒ nm = 2.3 = 6
Sol 9: A matrix → row n, column = m
if (n, m) = (3, 2) ⇒ mn = 3.2 = 6
Total element = mn
Sol 3: A2×2 and B2×3 mn is prime no.
so mn could be → 2, 5, 7, 11
For AB ⇒ order will be ⇒ 2 × 3 factor of 2 = 1 × 2 or 2 × 1
For BA ⇒ row of B ≠ column of A
So for any prime no. of only 2 order
So, BA does not exist
⇒ 1 × n and n × 1(n ∈ prime no. )

Sol 4: A = [aij], 1 ≤ i ≤ 3, i ≤ j ≤ 3
 x   −1 0 
Sol 10:   +   =  
aij = i + 2j 2y   4  3 
(i) a11 = 1 + 2 = 3 (ii) a32 = 3 + 2(2) = 3 + 4 = 7
 x − 1  0 
(iii) a23 = 2 + 3(2) = 8 (iv) a34 ⇒ not a element = i ≤ j ≤ 3 ⇒  = 
2y + 4  3 
but here 4 > 3
x–1=0⇒x=1
Sol 5: Total element = 18 2y + 4 = 3 ⇒ 2y = 3 – 4 = – 1
Assume no of row = n 1
Y=–
And no. of column = m 2
so n × m = 18 = 1 × 18 = 2 × 9 = 6 × 3 = 3 × 6 = 9 ×
2 × 18 × 1 Sol 11: f(x) = 3x2 – 9x + 7
f(A) ⇒ if A is a matrix
2 0 
Sol 6: Diagonal matrix =  
0 3  f(A) = 3A2 – 9A + 7I
3 0  A is a square matrix so A2 is possible.
scalar matrix =  
0 3 
1 6 . 4 8 | Matrices

Sol 12: A, B and AB are symmetric matrices Sol 17: Xm×3 Yp×4 = Z2×b
A = aij
Column of x = row of y ⇒ 3 = p and 2 × b = (m × 4)
B = bij
So m = 2; b = 4
AB = Aij Bji= Cij
BA = Bij . Aji = dij  0 −1 2 
 
Sol18: A =  1 0 −3
but Bij = B ji 
 −2 3 0 
 symmetric matrix’s property  
and Aij = A ji 

a12 = – a21, a13 = – a31
∴ AB = Aij Bij = Aij . Bij = BA
a23 = – a32,
AB = BA
so A is skew symmetric.

1 
 
Sol 13: A = 2  = B 2 −2 4  cos θ sin θ 
1×3 Sol 19: R(θ) =  
3   sin θ − cos θ 
 3×1
1   2 −2 4   π π 
     cos sin 
AB = 2  2 −2 4 1×3 =  4 −4 8 
  π
R  =  2 2  = 0 1 
3   6 −6 12  
 3×1   2 sin π π
− cos  
1 0
 12 2

1 2 0 
cos(x + y) sin(x + y) 
R(x + y)=  
1 2   
Sol 14:   and 3 4 0   sin(x + y) − cos(x + y)
3 4  2×2 0 0 0 
 3×3
Sol 20: Skew symmetric A = [aij]
Both have different orders. So they are not same.
For all skew symmetric Matrix dia. l element (aij) are
zero so aij = 0 & when i = j
 2 −1  1
Sol 15: A =  ,K=–
4 2  2
a 0
Sol 21: A =  
 2 −1  1  2 −1 0 0 
KA = K   =–  
 4 2  2 4 2 
a 0 a 0 a2 0
A2 =     ⇒A =
2
 
 2 1 
 1 0 0  0 0   0 0 
 − − ( −1)
2 2  −1 
=  = 2 a2
 − 1 (4) − 1 (2)   0 a2 0  a4 0
 −2 −1 A2 × A2 =     ⇒ A4 =  
 2 2   0 0   0 0   0 0 

a16 0
sec θ tan θ   − tan θ − sec θ  ⇒ A16=  
Sol 16: tanθ   + secθ    0 0 
tan θ − sec θ   − sec θ tamθ 

tan θ sec θ − tan θ sec θ tan2 θ − sec2 θ  1 0 X 


=  Sol 22: [X 1]1×2     =0
2 2
 tan θ − sec θ − tan θ sec θ + cos θ sec θ   −2 −3 2×2  3  2×1

 sin2 θ − 1  X  X 
 0  ⇒ [ x − 2 0 − 3]1×2   [x – 2–3]   = 0
=  cos2 θ  =  0 −1   3  2×1 3
 sin2 θ − 1   
 −1 0 
 0 
 cos2 θ  [(x – 2)x –3(3)] = 0 ⇒ x2 – 2x – 9 = 0
M a them a ti cs | 16.49

2 ± 22 − 4( −9) 0 0 1 
x= =1± 10  
2 Sol 29: A = 0 1 0
1 0 0
 2 −1
Sol 23: A =   0 0 1  0 0 1 
 4 6  2×2    
A2 =  0 1 0   0 1 0 
Additive inverse B which is – A 1 0 0  1 0 0 
   
So, A + B =A – A = 0
1 0 0 
 
A2 =  0 1 0  = I
2 −1 2 4 
Sol 24: x +  =  0 0 1 
 
3 −1 5 0 
A6 = [A2]3 = [I]3 = I
x x2 
Assume x =  1  A6 = I = A2
 x3 x4 

 x + 2 x2 − 1  2 4  Sol 30: Properties of skew – symmetric matrix [a, j]


⇒  1  =  
 x3 + 3 x 4 − 1  5 0  ⇒ All diagonal element are zero

⇒ x1 + 2 = 2 ⇒ x1 = 0 ⇒ aij = – aji

⇒ x2 – 1 = 4 ⇒ x2 = 1 + 4 = 5
0 −4  5 −7 
⇒ x3 + 3 = 5 ⇒ x3 = 5 – 3 = 2 Sol 31: A =  . B= 
0 −3  0 0 
⇒ x4 – 1 = 0 ⇒ x4 = 1
0 −4  5 −7 
x x2  0 5  AB =    
x=  1  =   0 −3  0 0 
 x3 x4  2 1 
0.5 − 4.0 0( −7) 0 0 
=   =  
Sol 25:  0(5) 0( −3)  0 0 
 sin2 θ 1   cos2 θ 0  0 −1 
  +   +    4 3
 −1 0 
2 2
cot θ 0   − cosec θ 1  Sol 32: A =  
 2 5
 sin2 θ + cos2 θ + 0 1 + 0 − 1 
=   A2 – XA +YI = 0
cot2 θ − cosec2 θ − 1 0 + 1 + 0 
 4 3  4 3  42 + 3.2 4.3 + 3.5
 1 0  1 0 A2 =     =  
=  =    2 5  2 5 2.4 + 5.2 2.3 + 5.2 
 −1 − 1 1   −2 1 
22 27 
A2 =  
Sol 27: Matrix has 8 element 18 31 
m×n=8=1×8=8×1=2×4=4×2 A2 – XA + YI = 0
if m × n = 5 = 1 × 5 = 5 × 1 (only 2 possible order) 22 27   4 3 0 0 
1
⇒   ×   + =  
a 18 31   2 5 1 0  0 0 
 
c 
 
b 0 1 
Sol 28: a b    × a b c   
   
22 − 4x + y
 c 27 − 3x  0 0 
d ⇒   =  
 
d  18 − 2x + x 31 − 5x + y  0 0 
[ac + bd] × [a2 + b2 + c2 + d2] ⇒ Compare elements
⇒ [a2 + b2 + c2 + d2 + ac + bd] 27 – 3x = 0
1 6 . 5 0 | Matrices

27 0 1  1 0 
3x = 27 ⇒ x = = 9 ⇒ y = 45 – 31 = 14 Sol 35: A =  , F = 
3 0 0 
  0 1 
(x, y) = (9, 14)
1 0  a 0
For (aI + bA)3; aI = a   =  
 1 3 5  0 1  0 a 
Sol 33: A =  
 −2 5 7  0 1  0 b 
bA = b   =  
 4 5 −9  0 0  0 0 
2A – 3B =  
1 2 3  a 0 0 b  a b
aI + bA =   +   =  
b b2 b3  0 a  0 0  0 a 
Assume B =  1 
b 4 b5 b6 
a b a b a b
(aI + bA)3 =      
2.1 − 3b1 2.3 − 3b2 2 × 5.3b3  0 a  0 a  0 a 
⇒ 2A – 3B =  
 −4 − 3b 4 2.5 − 3b5 2.7 − 3b6 
a2 ab + ba a b a3 a2b + 2a2b 
=     =  
 4 5 −9   0 a2  0 a   0 a3 
=  
1 2 3  a3 3a2b 
=  
4−2 2  0 a3 
2 – 3b1 = 4 → b1 = =–
−3 3
and R. H. S. = a3I + 3a2bA
⇒ 6 – 3b2 = 1
1 0  0 1 
⇒ 3b2 = 6 – 5 = 1 = a3   + 3a b 0 0 
2

0 1   
1
⇒ b2 =
3 a3 0 0 3a2b  a3 3a2b 
19 =  +   =  
Same as b3 =  0 a3  0 0   0 a3 
3
5 8 11 L. H. S. = R. H. S
b4 = – , b5 = , b6 =
3 3 3
 0 1
1  −2 1 19  Sol 36: A =  
So B =    −1 1
3  −5 8 11 
(pI + qA)2 = A
 cos α sin α  p 0   0 1  0 q
Sol 34: A =   pI = 
 − sin α cos α   , qA = q   =  
 0 p   −1 1   −q q
 cos α sin α   cos α sin α  p 0   0 q p q 
A2 =     pI + qA = 
 − sin α cos α   − sin α cos α  +  =  
0 p   −q q  −q p + q

 cos2 α − sin2 α cos α sin α + sin α cos α  p q  p q 


A2 =   (pI + qA)2 =    
 − sin α cos α − sin α cos α − sin2 α + cos2 α   −q p + q  −q p + q

we know → cos2α – sin2α = cos2α  p2 − q2 pq + q(p + q) 


=   = A (given)
2 2
and 2cosα sinα = sin2α  −pq − q(p + q) −q + (q + q) 

 0 1
2  cos2α sin2α  A=  
so A =    −1 1
 − sin2α cos2a
So p2 – q2 = 0 ⇒ p2 = q2 ⇒ p = ± q
M a them a ti cs | 16.51

pq + qp + q2 = 1 5 2  3 6 
Sol 40: X + Y =  ,X–Y=  
q2 + 2qp = q2 ± 2q2 = 1 0 9  0 −1
–ve → q2 – 2q2 = 1 ⇒ q2 = 1 not possible sum of X + Y, X – Y
+ve → q + 2q = 3q = 1 ⇒ q = 1/3
2 2 2 2
⇒ X + Y + X – Y = 2X
1
So p = q = ± 5 2  3 6  5 + 3 2 + 6 
3 =   +   =  
0 9  0 −1  0 9 − 1
 2 3 −4 
  8 8  4 4
Sol 37: A =  1 0 6  2X =   = 2 
 −2 1 5  0 8  0 4 
 
4 4
5 1 2 X=  
  0 4 
B = 6 −1 4 
5 3 −4 
  5 2  5 2  4 4
Y=   –X=   –  
 2 3 −4  5 1 2 0 9  0 9  0 4 
   
2A – 3B = 2  1 0 6  – 3 6 −1 4 
 −2 1 5  5 3 −4  5 − 4 2 − 4  1 −2
    Y=   = 
 0 9 − 4  0 5 
 −11 3 −14 
 
=  −16 3 0   cos2 θ cos θ sin θ 
 −19 −7 22 
  Sol 41: A =  
cos θ sin θ sin2 θ 

Sol 38: A3×3 = [aij]  cos2 φ cos φ sin φ 


B=  
aij = 2i – 3j cos φ sin φ sin2 θ 
\a11 = 2(1) – 3(1) = – 1, a12 = 2(1) – 3(2) = – 4
AB = (Cij)
a13 = 2(1) – 3(3) = – 7, a21 = 2(2) – 3(1) = 1
C11 = cos2θ cos2φ + cosθ sinθ cosφ sinφ
a22 = 2(3) – 3(2) = – 2, a23 = 2(2) – 3(3) = – 5
C11 = cosθ cosφ (cosθ cosφ + sinθ sinφ)
a31 = 2(3) – 3(1) = 3, a32 = 2(3) – 3(2) = 0
C11 = cosθ cosφ cos(θ – φ) = 0
a33 = 2(3) – 3(3) = – 3
Similarly C12, C21 and C22 will also be zero
 a11 a12 a13   −1 −4 −7 
    0 0 
So A = a21 a22 a23  =  1 −2 −5  So AB =  
a
 31 a32 a33   3 0 −3
  0 0 

 x 3x − y  3 2   −1 1 −1 0 4 3
Sol 39:  =      
Sol 42: A =  3 −3 3  , B=  1 −3 −3
2x + z 3y − w  1 7 
 5 −5 3   −1 4 4 
   
Compare elements
 −1 1 −1  −1 1 −1
x=3    
A =  3 −3 3   3 −3 3 
2

3x – y = 3(3) – y = 9 – y = 2  5 −5 3   5 −5 3 
   
y=9–2=7
 1 + 3 −5 −1 − 3 + 5 1 + 3 −5 
2x + z = 2(3) + 7 = 6 + 7 = 4 ⇒ 7 = 4 – 6 = –2  
=  −3 − 9 + 15 3 + 9 + 15 −3 − 9 + 15 
3y – w = 3(7) – w = 7 ⇒ w = 21 – 7 = 14  −5 − 15 + 25 5 + 15 − 25 −5 − 15 + 25
 
(x, y, z, w) = (3, 7, –2, 14)
1 6 . 5 2 | Matrices

 −1 1 −1 X X2 X3  1 −2 −5
  X=  1 =  
=  3 −3 3  =
A X 4 X5 X6  3 4 0 
 5 −5 5 
 

0 4 3 0 4 3 Exercise 2
   
B =  1 −3 −3  1 −3 −3
2

 −1 4 4   −1 4 4  Single Correct Choice Type


   

 4 −3 −12 + 12 −12 + 12  1 0 0  Sol 1: (A) Number of elements in a matrix = 60


   
B =  −3 + 3 4 + 9 − 12
2
3 + 9 − 12  = 0 1 0  60 = 22 51 31
 4 − 4 −4 − 12 + 16 −3 − 12 + 16  0 0 1 
    Number of order matrix can have = (2 + 1) (1 + 1) (1
+ 1)
A2B2 = A2,I = A. I = A
= 3 × 2 × 2 = 12
 2 −1  −1 8 −10 
    Sol 2: (C) A(x)×(x + 5) By×(11 – y)
Sol 43:  0 1  X = 3 4 0 
 −2 4  10 20 10  AB and BA both exist
   3×3
⇒ for AB x + 5 = y  …(i)
 2 −1
  ⇒ for BA 11 – y = x  …(ii)
X ‘s number of row = column of  0 1 
 −2 4  ⇒ y = 8; x = 3
 
order of X = 2 × 3
 X X 2 X3  Sol 3: (D) A is a square invertible matrix
assume X =  1 
 X 4 X5 X 6  A2 = A
 2 −1 Multiply A–1 both sides
   X1 X 2 X 3 
 0 1  X 
X5 X 6  A–1 A2 = A–1A = I
 −2 4   4
  A=I
 2x1 − x 4 2x2 − x5 2x3 − x6  So A2 = I
 
= x4 x5 x6  A2 – I = 0 (zero matrix)
 −2x + 4x −2x2 + 4x5 −2x3 + 4x6 
 1 4

Sol 4: (B) Total 143 elements all are different.


 −1 −8 −10 
  143 = 1 × 143 = 143 × 1 = 11 × 13 = 13 × 11
= 3 4 0 
10 20 10  Total Number of order that exist = 4
 
x4 = 3, x5 = 4, x6 = 0 Number of way to arrange 143 elements = 143!

–2x1 + 4x4 = 10 ⇒ –2x1 + 4(3) = 10 Total not of matrix = 4 × 1431

2x1 = 12 – 10 = 2 ⇒ x1 = 1
Sol 5: (C) A2 = A
–2x2 + 4x5 = – 2x2 + 4(4) = 20
(I + A)4 = (I2 + A2 + 2A)2
– 2x2 + 16 = 20
= [I + A + 2A]2 = [I + 3A]2 ( A2 = A)
2x2 = 16 – 20 = – 4 ⇒ x2 = – 4 = – 2 = I2 + 9A2 + 6A = I + 9A + 6A = I + 15A
2
–2x3 + 4x6 = – 2x3 + 4(0) = 10 = – 2x3
 α β
10 Sol 6: (C) A =  
x3 = =–5  −β α 
−2
Since, A is orthogonal matrix
M a them a ti cs | 16.53

So, AA’ = A’A = In 0 −2


det (I – A) = = 0 – (–2) (–2) = – 4
 α β  α −β   α + β −2 0
2
−αβ + αβ 
2
AA–1 =   = 
 −β α   β α   −αβ + αβ α + β 
2 2
(I + A) 2 1 1 0 2 
f(A) = =    
(I − A) −4 1 1 2 0 
α2 + β2 0  1 0 
  =   1 0 + 2 2 + 0  2 1 1  −1 −1
0 1 
2 2
 0 α + β  =–   =–   =  
2 0 + 2 2 + 0  2 1 1  −1 −1
a2 + b2 = 1 …(i)
x2 – px + q = 0 & x2 + px – q = 0 Sol 9: (D) A is skew symmetric matrix
Sum of both equation for common roots ⇒ ATA = I
x2 – px + q + x2 + px – q = 0 ⇒ AT = – A ⇒ (AT)2 = (A)2
2x = 0 ⇒ x = 0
2
⇒ ATA = – A2 = I
So if α is roots of x2 – px + q = 0 and β is roots of x2 + Taking square of both sides
px – q = 0
A4 = I
⇒α + 0 = p and α(0) = q = 0
⇒ A4n = I ( –A2 = I are A = – AT) \
β + 0 = – p and β(0) = – q = 0
A3A4n = A3I
⇒ α = p and β = – p
A4n+3 = – A2(–A) I
In (i) equation a2 + b2 = 1
A4(n + 1) – 1 = (–A)I = AT I = AI
⇒ p2 + (–p)2 – 1
4n - 1 ∈ N
⇒ 2p2 = 1
⇒A4n – 1 = AT
1
⇒p=± ,q=0
2
Sol 10: (D) |A| = |B|A2 × 2, B2 × 2

Sol 7: (A) (det A) = 3 Tr (A) = Tr(B)

(det λA) = 81 A2 – 3A + 14I = 0 and B2 – λB + µI= 0

if A’s order = n × n if A = B, |A| = |B|

then (det λA) = λn(det A) = λx 3 = 81 Tr |A| = Tr(B) satisfied so A2 – λA + µI = 0


81 µ = 14 ( A’s order = 2 × 2)
ln = = 27
3
ln = 27 = 3n  λ ∈ N Sol 11: (B) The adjoint of upper triangular matrix is
false.
So, n = 3
 That is equal to upper triangular not lower triangular
matrix.
1 2  1+x
Sol 8: (B) A =   , f(x) =
2 1  1−x
Sol 12: (B) A, B, (A + B) are non-singular
1 0  1 2  2 2  1 1 [A(A + B)–1B]–1
I + A =  + =   = 2 
0 1  2 1  2 2  1 1 = [A–1((A + B) –1)–1B–1] = (A–1(A + B) B–1)
= [(A–1A + A–1B)B–1) =[(I + A–1B)B–1)
1 0  1 2   0 −2  0 1 
I – A =  – =  = –2  
0 1  2 1   −2 0  1 0  = [B–1 + A–1BB–1] = A–1 + B–1

1 0 2  Sol 13: (C) A is an orthogonal matrix


(I – A)–1 =  
det(I − A) 2 0 
|A| = – 1
1 6 . 5 4 | Matrices

⇒ AAT = ATA = In
1 + tan2 x 1 0  1 0 
=   =  
⇒ |A| = |AT| = – 1 1 + tan x 0 1 
2
0 1 
⇒ AT = + A–1( A is an orthogonal matrix)
1 0
1 |AT A–1| = =1
⇒A = T
(adjA) = – (adjA) 0 1
det(A)

Sol 14: (C) A and B are square matrices of order 3 2 3 


Sol 17: (D) A =  
(A) adj(AB) = adj(A) + adj(B) is not necessary 5 −2
⇒ option (A) is wrong. ⇒ |A| = 2(–2) – 3(5) = – 15 – 4 = – 19
(B) (A + B)–1 ≠ A–1 + B–1 1  −2 −5
T
−1 2 3  1
A–1 =   =  = A
(C)AB = 0 |A|  −3 2  −19 5 −2  19
A, B are square matrix
So if AB = 0 Sol 18: (B) PT = P–1
⇒ |A| |B| = 0
P P  P1 P3 
⇒ |A| = 0 or |B| = 0 Assume P =  1 2  , PT =  
P3 P4  P2 P4 
a 0 0 ⇒ |P| = P1P4 – P2P3
 
Sol 15: (D) A = 0 a 0  ⇒ |A| = a3|I| = a3 T
0 0 a  1 1  P4 −P3  1  P4 −P2 
  P–1 = (adj P) =   =  
|P | | P |  −P2 P1  | P |  −P3 P1 
a2 0 0
  |P| = P1P4 – P2P3
adj(A) =  0 a2 0
 
 0 0 a2  P1 P3  1  P4 −P2 
   =  
P2 P4  | P |  −P3 P1 
|adj |A|| = (a2)3 = a6
|P| = 1
\|A| |adj (A)| = a3 a6 = a9
P2 = – P3
 1 tanx   cos θ sin θ 
Sol 16: (D) A =   Only option (B) 
 − tanx 1   is correct.
 − sin θ cos θ 
|A| = 1 + tan2x
Sol 19: (A) Br = I r > 1
 1 1 tanx  A
A = –1
  = A–1 Br–1 A – A–1B–1A = A–1Br B–1A – A–1B–1A
1 + tan x tanx
2 1  (1 + tan2 x)
= A–1B–1A – A–1B–1A = 0
 1 − tanx 
AT =  
 tanx 1  Sol 20: (C) A & B are orthogonal matrices

 1 1 tanx   1 − tanx  ⇒ AAT = ATA = In and BBT = BTB = In


AA =
T –1
   
(1 + tan x)  − tanx
2 1   − tanx 1  AB ⇒ (AB) (AB)T

 1 + tan2 x tanx − tanx  ⇒ (AB) (BTA)T


1  
= 2 ⇒ A(BBT)At = AIAT
(1 + tan2 x) tanx − tanx tan x + 1 
⇒ AAT = I
1 1 + tan x 0 2 
=   (AB)T (AB) = BTATAB = BTIB = BTB = In
2 2
(1 + tan x)  0 1 + tan x 
So (AB)T(AB)
M a them a ti cs | 16.55

gE = AB(AB)T= In Tr(A) = a1 + a2 + a3 = 12 and det |A| = a1 a2 a3 for


maximum of det (A) = a1 a2 a3
So, AB also satisfying property of orthogonal
a1 + a2 + a3 = 3a1 = 3a2 = 3a3 = 12
Sol 21: (C) C is an orthogonal matrix 12
a1 = =4
⇒ CC = C C = In
T T 3
Tr (CTAC) = Tr [(CTA)C] det |A| = 4 × 4 × 4 =64
= Tr [C(CTA)] = Tr (CCTA) = Tr (IA)= Tr (A)
Sol 27: (C) AB = B
Sol 22: (C) A and B are idempotent matrices BA = A
so, A = A and B = B
2 2
(A + B)2 = (AB + BA)2
|A|, |B| = or 1 A2 + B2 + AB + BA = (AB)2 + (BA)2 + (AB) (BA) + (BA) (AB)
AB = BA A2 + B2 + AB + BA = ABAB + BABA + AB + BA
A – B is non-singular
A2 + B2 = AAB + BBA = AB + BA
⇒ |A| = and |B| = 1 or |A| = 1 and |B| = 0
A 2 + B 2= A + B

Sol 23: (B) AAT = I


0 1 
Sol 28: (C) A =   , B is column matrix
(A BA)
T 10
3 0 
= (ATBA) (ATBA) (ATBA)………. . 0 
(A8 + A6 + A4 + A2 + I) B =  
⇒A BIBIBI………. BA =A B A
T T 10
1  2×1
0 1  0 1  3 0 
 0 c −b  A2 =     =   = 3I
 
Sol 24: (C) A =  −c 0 a  3 0  3 0  0 3 
 b −a 0  A4 = A2A2 = 32I
 
A6 = 33I, A8 = 34I
 a2 ab ca 
  A8 + A6 + A4 + A2 + I = I (1 + 3 + 32 + 33 + 34) = 121 I
B = ab b2 cb 
 2 0
 ac bc c  0   
2 I B =   ⇒B =1
 abc − abc b2 c − b2 c c2b − bc2  1  2
   
AB = −a2bc + ab2c −abc + abc −ac2 + ac2 
 2 2 ∞  Sol 29: (C) AB = BA and A2 = I
 a b − a b ab2 − ab2 abc − abc 
 ABA = A(AB) = A2B = IB = B
0 0 0 
 
AB = 0 0 0  = 0
0 0 0 
Previous Years’ Questions
 
Sol 1: (B)
Sol 25: (C) AB = BA, C2 = B
1 + a2 a a2
(A–1CA)2 = (A–1CA) (A–1CA) = A–1C ICA = A-1C2 A
⇒ ∆ = cos(p − d)x + cos(p + d)x cospx cos(p + d)x
= A–1BA = A–1(AB) = IB = B = C2 sin(p − d)x + sin(p + d)x sinpx sin(p + d)x

Sol 26: (D) Tr(A) = 12 1 + a2 a a2

a1 ⇒ ∆ = 2cospx cosdx cospx cos(p + d)x


0 0
  2sinpx cosdx sinpx sin(p + d)x
Assume A =  0 a2 0 ( A is diagonal matrix)
0 0 a3 

1 6 . 5 6 | Matrices

Applying C1 → C2 – 2cos dxC2 ⇒ 2cos x = – sin x or sin x = cos x


π π
⇒ cot x = –1/2 gives no solution in – ≤x≤
2
1 + a − 2acosdx a a 2 4 4
and sin x = cos x ⇒ tan x = 1
⇒∆= 0 cospx cos(p + d)x
0 sinpx sin(p + d)x ⇒ x = π/4

⇒ ∆ = (1 + a2 –2a cos dx) [sin (p+ d) x cos px Sol 4: (B) For infinitely many solutions, we must have

– sin px cos (p + d) x] k +1 8 4k
= = ⇒k=1
k k + 3 3k − 1
⇒ ∆ = (1 + a2 –2a cos dx) sin dx
Which is independent of p Sol 5: (B) Since, given system has no solution
∴ ∆ = 0 and any one amongst Dx, Dy, Dz is non-zero.
Sol 2: (A)
2 −1 2
1 x x +1 Let = 1 −2 1 = 0 ⇒ λ = 1
f(x) = 2x x(x − 1) (x + 1)x 1 1 λ
3x(x − 1) x(x − 1)(x − 2) (x + 1)x(x − 1)

Applying C3 → C3 – (C1 + C2) Sol 6: (A) Now,

 3 /2 −1 / 2   3 /2 1/2 
1 x 0 PTP =    
 1 / 2 3 / 2   −1 / 2 3 / 2 
= 2x x(x − 1) 0 =0
3x(x − 1) x(x − 1)(x − 2) 0 1 0 
⇒ PTP =  
0 1 
∴ f(x) = 0
⇒ PTP = I ⇒ PT = P–1
⇒ (100) = 0
Since, Q = PAPT
sinx cos x cos x
∴ PTQ2005P … (i)
Sol 3: (C) Given, cos x sinx cos x = 0
= PT(PAPT)(PAPT) ……. 2005 times]P
cos x cos x sinx
T
Applying C1 → C1 + C2 + C3 = (P P)A(P TP)A(P TP)........(P TP)A(P TP)

2005 times

sinx + 2cos x cos x cos x = IA2005 = A2005[from eq.(i)]


= sinx + 2cos x sinx cos x 1 1
sinx + 2cos x cos x sinx ∴A=  
0 1 
1 cos x cos x 1 1 1 1 1 2
A2 =    =  
= (2cos x+ sin x) 1 sinx cos x = 0 0 1  0 1  0 1 
1 cos x sinx 1 2 1 1 1 3
A3 =    =  
Applying R2 → R2 – R1, R3 → R3 – R1 0 1  0 1  0 1 
⇒ (2cos x + sin x) ………………………………………………..
………………………………………………...
1 cos x cos x
1 2005
0 sinx − cos x 0 =0 A2005 =  
0 0 sinx − cos x 0 1 
1 2005
⇒ (2cos x + sin x) (sin x – cos x)2 = 0 ∴ PTQ2005 P=  
0 1 
⇒ 2cos x + sin x = 0 or sin x – cos x = 0
M a them a ti cs | 16.57

Sol 7: (A) Every square matrix satisfied its characteristic 1 


equation = A + (p − 1)D 
a 
i.e. | A – λ I | = 0 1 
Sol 9: = A + (q − 1)D   … (i)
b 
1−λ 0 0 1 
⇒ 0 1−λ 1 =0 = A + (r − 1)D 
c 
0 −2 4 − λ
1 1 1
⇒ (1 – λ) {(1 – λ) (4 – λ) + 2}= 0
bc ca ab a b c
⇒ 13 –612 + 11λ – 6 = 0 Let ∆ = p q r = abc p q r ,
⇒ A3 –6 A2 + 11A – 6 I = O  … (i) 1 1 1 1 1 1

⇒ A2 6A + 11 I =
6A −1
[From equation (i)]
Sol 8: Since, a1, a2 are the roots of ax2 + bx + c = 0
A + (p − 1)D A + (q − 1)D A + (r − 1)D
b c
⇒ a1 + a2 = – and a1a2 = … (i) = abc p q r
a a
1 1 1
Also, b1, b2 are the roots of
px2 + qx + r = 0 Applying R1 → R1 – (A – D) R3 – DR2

q r 0 0 0
⇒ b1 + b2 = – and b1b2 = … (ii)
p p = abc p q r = 0

Given system of equations 1 1 1

a1y + a2z = 0 bc ca ab
And b1y + b2z = 0, has non-trivial solution ⇒ p q r =0
α1 1 1 1
α1 α2 β1
∴ = 0 ⇒ =
β1 β2 α2 β2
Sol 10: Given,
Applying componendo-dividendo
2ax 2ax − 1 2ax + b + 1
α1 + α2 β1 + β2 f’(x)= b b +1 −1
=
α1 – α2 β1 – β2 2(ax + b) 2ax + 2b + 1 2ax + b
⇒ (a1 + a2) (b1 – b2) = (a1 – a2) (b1 + b2)
Applying R3 → R3 – R1 – 2R2 , We get
⇒ (a1 + a2)2 {(b1 – b2)2 – 4b2b2}

= (b1 + b2)2 {(a1 + a2)2 – 4a1a2} 2ax 2ax − 1 2ax + b + 1


f’(x) = b b +1 −1
From equation (i) and (ii), we get 0 0 1
b2  q2 4r  q2  b2 4c 
 −  =  −  2ax 2ax − 1 2ax −1
a2  p2 p  p2  a2 a  =
b b +1
=
b 1
(C2 → C2 – C1)

b2q2 4b2r b2q2 4q2c ⇒ f’(x) = 2ax + b


⇒ 2 2 – = – 2
ap 2
ap 2 2
ap ap On integrating,

b2r q2c b2 ac we get f(x) = ax2 + bx + c


⇒ = ⇒ =
a p q2 pr Where c is an arbitrary constant
Since, f has maximum at x = 5/2
1 6 . 5 8 | Matrices

4π  π
⇒ f’(5/2) = 0 ⇒ 5a + b = 0 … (i) = 2sin2qcos = 2sin2qcos  π + 
3  3
Also, f(0) = 2 ⇒ c = 2 π
= – 2sin2qcos = – sin 2θ
and f(1) = 1 ⇒ a + b + c = 1 … (ii) 3
1
On solving equation (i) and (ii) for a, b, we get a = , sin θ cos θ sin2θ
4
5 ∴∆ = − sin θ − cos θ − sin2θ
b=–
4
 2π   2π   4π 
1 5 sin  θ −  cos  θ −  sin  2θ − 
Thus, f(x) = x2 – x + 2  3   3   3 
4 4
= 0 (since, R1 and R2 are proportional)
Sol 11:

sin θ cos θ sin2θ Sol 12: (B)

( )( ) ( )  A. ( A ) 
T T
 2π   2π   4π  =BBT = A −1 A T A −1 A − T A −1 A T −1

sin  θ +  cos  θ +  sin  2θ +  
 3   3   3  T
( ) ( ) ( )( )
T
−1
 2π   2π   = 4π  A= . A T A A T . A −1 A −1 AA T A −1
+ sin  θ –  + cos  θ –  + sin  2θ – 
 3   3   3 
(=
A A) A .(A ) ( A ) (=
A A)
T T
−1 T −1 −1 −1
 2π   2π   4=π A.= I
sin  θ –  cos  θ –  sin  2θ – 
 3   3   3 

 2π   2π  Sol 13: (D) AA T = 9T


Now, sin  θ +  + sin θ – 
 3   3  1 2 2  1 2 a 
  
A=
2 1 −2 2 1 2  = 9I
 2π 2π   2π 2π 
θ+ +θ−  θ+ –θ+   a 2 b  2 −2 b 
3 3  cos 3 3    
= 2sin  
 2   2  9 a + 4 + 2b 0
9 0 0
   
    ⇒ 0 9
2a + 2 − 2b ⇒ 0 9 0
2π  π a + 4 + 2b 2a + 2 − 2b a2 + 4 + b2 0 0 9
= 2sinθ cos = 2sinθ cos π − 
3  3
π Equation a + 4 + 2b =
0 ⇒ a + 2b =
−4 ....(1) ... (i)
= –2sinθ cos = – sinθ
3 2a + 2 − 2b =⇒
0 2a − 2b =
−2 ...(2) ... (ii)
 2π   2π  2 2 2 2
and cos  θ +  + cos  θ –  & a + 4 + b =0 ⇒ a + b =5 ...(3) ... (iii)
 3   3 
Solving a =
−2,b =
−1
 2π 2π   2π 2π 
θ+ +θ−  θ+ –θ+ 
= 2 cos  3 3  cos  3 3  5a −b 
 2   2  Sol 14: (B) A =  
    3 2 
   
5a −b  T
 2π   1 =A =  A adj A AA
= 2cosθ cos   = 2cosθ  −  = – cosθ  3 2 
 3   2
5a −b   2 b  5a −b   5a 3
 4π   4π    =  
and sin  2θ +  + sin  2θ −   3 2   −3 5a  3 2   −b 2
 3   3 
10a + 3b 0  25a2 + b2 15 − 2b 
 4π 4π   4π 4π   = 
 2θ + + 2θ −   2θ + – 2θ +   0 10a + 3b   15 − 2b 13 
3 3  cos  3 3 
= 2sin 
 2   2 
    Equate, 10a + 3b= 25a2 + b2
   
and 10a + 3b =
13
and 15a − 2b =
0
M a them a ti cs | 16.59

a b (ii) A is invertible ⇒ so |A| ≠ 0


= = k (let)
2 15 a11 a22 – a21a21 ≠ 0
2 a11a22 ≠ a21 a12
Solving=a = ,b 3
5
2 × (41 + 3) 2
2 total way → = . 27 = 18
So, 5a + b = 5 × + 3 = 5 3
C2 3
5
(iii) |A|max – |A|min
⇒|A|max = a11a22 – a21a12
JEE Advanced/Boards (a11 a22)max = 4 ( a11 + a22 + a12 + a12 = 4)
and + ( a21. a12)max = 3
Exercise 1
a21 a12 = 0 with a12 a22 = 4 = (2) (2)

Sol 1: (a) |A| = a, (B) = (adjA), |B| =b So |A|max = 2(2) – 0 = 4


|A|min = 0 – 2(2) = – 4
1 a a2 a3
S= + + + ....
2 b b3 b5 |A|max – |A|min = 4 – (–4) = 8
(iv) A is symmetric or skew symmetric or both |det A| is
a 
2
a  a  divisible by 2
= 1 + +   + .... 
b b 2 2
b  
  So |det A| can be 0, 2, 4
   4 0  0 0   2 0  2 1  0 2 
 A⇒  ,  ,  ,  ,  
a 1  a  b2  ab  0 0  0 4  0 2  1 0  2 0 
= =   =  
b a  b  b2 − a  b2 − a
1 − 2  Total no. → 5
 b 
2ab  4 −4 5 
S=
b2 − a  
Sol 3: A =  −2 3 −3
b > a, a = 3  3 −3 4 
 
|B| = |A|n–1 = |A|3–1 = (3)2 = 9 = b
A11 = 12 – 4 = 3 A13 =…………………. .
2(3)9 2(3)9 A12 = – 9 + 8 ⇒ – A21 = – 15 + 16 = 1
⇒ (ab2 + a2b + 1) = (3. (9)2 + 32. 9 + 1)
2
9 −3 92 − 3
A22 = 16 – 15 = 1, – A23 = – 12 + 12 = 0
6.9 6.9
= (1 + 81 + 243) = = 225 A31 = 12 – 15 = – 3, –A32 = – 10 + 12 = 2
78 78
(b) |A| = –2, |B| = 1 A33 = 12 – 8 = 4

(A–1) (adj B–1) adj(2A+1)  3 +1 −3


 
adjA =  −1 1 2 
⇒ |A–1| |adj B–1| 23–1 |adj A|  −3 0 4 
 
1 4×4 16
= × 1 × 22 × (–2)2 = = =–8
−2 −2 −2 |A| = 4[A11] – 4(A) + 5[A13]

= 4(3) – 4(–1) + 5(–3) = 12 + 4 – 15 = 1


Sol 2: A = [aij], aij ∈ {0, 1, 2, 3, 4}
a11 + a12 + a21 + a22 = 4  3 +1 −3
adjA  
A =
–1
=  −1 1 2 
(i) Tr (A) = a11 + a22 = 4 |A|  −3 0 4 
 
and a11, a22 ∈ {0, 1, 2, 3, 4}
total possibilities = 0 + 4 = 4 + 0 = 1 + 3 = 3 + 1 = 2 + 2  3 +1 −3  3 +1 −3  17 4 −19 
     
A–2 =  −1 1 2   −1 1 2  =  −10 0 13 
⇒5
 −3 0 4   −3 0 4   −21 −3 25 
     
1 6 . 6 0 | Matrices

 2 −1 –1  2 −1
1 1 1  adjB =  ,B =   adjC
  2 3   −3 2   −3 2 
Sol 4: (a) A = 2 4 1 , B =  
2 3 1  3 4  2×2  −3 −2  –1 1 3 2 
 3×3 =  ,C =  
 −5 3  19 5 −3
1 0 1  2 4 
BPA =   ⇒ BAC = 
0 1 0  2×3 
3 −1
P P2 P3   2 −1 2 4   4 −3 8 +1 
Assume P =  1  B–1BAC = AC =     =  
P4 P5 P6  2×3  −3 2   3 −1   −6 + 6 −12 − 2

1 0 1  1 9 
BPA =  AC =   ⇒ ACC
–1
  0 −14 
0 1 0 
1 0 1  1 9  3 2  1
B–1BPA = PA = B–1 0 1 0  =A=     –
   0 −14  5 −3  19

|B| = 8 – 9 = – 1 1 3 + 45 2 − 27  1  48 −25
A=   =  
19  −70 42  19  −70 42 
1  4 −3  −4 3 
adjB =   =  
−1  −3 2   3 −2
 2 − 3 P 0 
 
 −4 3  1 0 1   −4 3 −4  Sol 5: A =  0 m2 − 8 q 
PA =     =    
 3 −2   0 1 0   3 −2 3  r 0 n2 − 15
 
|A| = 1[4 –3]– 1 [2 – 2] + 1 [ 6 – 8] = 1 – 2 = – 1
A2 = A[ A is idempotent matrix]
 1 3 − 1 1 − 4  1 2 −3 A2 =
   
adjA =  0 −1 2 − 1  =  0 −1 1  ,
6 − 8 2 − 3 +2   −2 −1 +2  (2 − 3)2 + 0 p(2 − 3) + p[m2 − 8] pq 
     
2 2 2 2
 qr (m − 8) q(m − 8) + q(n − 15)
 −1 −2 3   2 2 
r( − 3) + r(n + 5) rp (n2 − 15)2 
 
A =  0 −1 −1
–1

 2 1 −2  2 − 3 p 0 
   
=  0 m2 − 8 q 
 −4 3 −4  –1  
PAA–1 =  A =P  r 0 n2 − 15
 3 −2 3  
 −1 −2 3  compare elements
 −4 3 −4   
P=   0 1 −1 ⇒ (12 – 3)2 =  2 – 3 ⇒ 12 – 3 = 0 or 1
 3 −2 3   2 1 −2
  =± 3 or ± 4= ±2
 −4 7 −7  p[l2 – 3 + m2 – 8] = p⇒ p = 0 or l2 + m2 – 11 = 1
P=  
 3 −5 5 
rp = 0 ⇒ r = 0 or p = 0
2 1  3 2  2 4 
(b)   ·A 5 −3 =   (n2 – 15)2 = n2 – 15 ⇒ n2 – 15 = 1 or 0
3 2    3 −1
q[(m2 – 8) + n2 – 15] = q ⇒ q = 0 or m2 + n2 – 23 = 0 + 1
2 1  3 2 
assume B =   C=   (m2 – 8)2 = m2 – 8 ⇒ m2 – 8 = or 1
3 2  5 −3
m = ± 8 or ± 9 = ± 3
|B| = 4 – 3 = 1 |C| = – 9 – 10 = – 19
if, l1m1n, q, q, r ∈ z
S = {0, ±2, ±3, ±4}
M a them a ti cs | 16.61

⇒ Sum of products of elements = 22+32+42=29  ∞  ∞ 1 1 1 


Tr  ∑ 3n An  = ∑ Tr(3 A3 ) = 18  + 2 + ..... ∞ 
n
  i=1 3 3 3 
cos x − sinx 0   n=1 
 
Sol 6: F(x) =  sinx cos x 0     
 0  1  1  18 8 18 
 0 1  = 18   = × = 
 3 1 − 1  3 3 − 1 2 
cos y − sin y 0    3  
 
F(y) =  sin y cos y 0 
3(1) − 1 + 3(2) − 2 + 3(3) − 3 2+4+6 12
 0 0 1  Tr (Bn) = = =
 22n
22n
22n
cos x − sinx 0  cos y − sin y 0  12
   Tr(2nBn) =
F(x). F(y) =  sinx cos x 0   sin y cos y 0  2n
 1   0 1 
 0 0 0 a 1 1 
m= ∑ [Tr(2n Bn )] = 12  + + ... 
cos x cos y − sinx sin y − cos x sin y − sinx cos y 0  n=1  2 22 
 
= sinx cos y + cos x sin y − sinx sin y + cos y cos x 0    
 0 0 1   1  1   12
 =12   =  = ·2 12
 2 1 − 1   2
cos(x + y) − sin(x + y) 0   
  2  
=  sin(x + y) cos(x + y) 0 
 0 0 1  l + m = 12 + 9 = 21

[F(x)]–1 = F(–x)
Sol 8: A is 3 × 3 matrix
L. H. S. ⇒ |F(x)| = cos2(x) + sin2x – 1
A11 = a33 = 2, all other aij = 1
 cos x sinx 0  2 1 1 
    R ⇒ R1 − R 2
adj[F(x)] =  − sinx cos x 0  A = 1 1 1  1
 0 0 1  R ⇒ R 3 − R1
 1 1 2  3
 
cos( −x) − sin( −x) 0  1 0 0 
adjF(x)    
[F(x)] =–1
=  sin( −x) cos( −x) 0  = F(–x) A = 1 1 1  R2 ⇒ R2 – R1 – R2
1
 0 0 1  0 0 1 
  
L. H. S. = R. H. S. 1 0 0 
 
Hence proved A = 1 1 0  = I
0 0 1 
 
Sol 7: An = [aij], Bn = [bij] A–1 = I = xA2 + yA + zI
2i + j 3i − i I = (x + y + z)I
aij = , bij =
32n 22n (x + y + z) = 1
l = nLim
→∞
Tr[3A1 + 32A2 + 33A3+…+ 3nAn+…. ]
1 2 2 
For An Tr(A) = a11 + a22 + a33  
Sol 9: A = 2 2 3
1 −1 3 
(2(1) + 1) + 2(2) + 2 + 2(3) + 3 9+6+3 18  
= = =
2n 2n
3 3 32n 2 1 1  10 
   
n
3 18 18 C=  2 2 1  , D = 13  , Cb = D
Tr(3nA n) = = 1 1 1  9
2n
3 3n    

|C| = 2[2 – 1] + 1 [1 – 2] + 1 [2 – 2] = 2 – 1 = 1
1 6 . 6 2 | Matrices

 1 0 −1  1 
   0 0
5x 0 0
adjC =  −1 1 0  = |C| C–1 = C–1  5x   
 0 −1 2  (AB)–1 =  0 1 0  = AB =  0 1 0
   −10x + 2 1  0 10x − 2 5x 
 
C–1Cb = C–1D 0 
 5x 5x 
1 0 −1  10  ⇒ x = 1/5
   
b =  −1 1 0  13 
 0 −1 2  9  −5 
 3×3  3×1  0 0 
 −1 0 0 
5   
 10 − 9  1  AB =  0 1 0  = 0 1 0
     −5   0 −4 −1
b =  10 + 13  =3 0 −4 
 −13 + 18  5   5 
   
(AB)2 = (AB) (AB) = (AB) (AB)–1 = I
|A| = 1[6 + 3] + 2[3 – 6] + 2[–2 –2] = 9 – 6 – 8 = – 5
Tr[AB+(AB)2+(AB)3+…+(AB)100]
6 + 3 −8 2   9 −8 2  = Tr[AB+I+AB+I+…+I]
   
adjA =  −3 3 − 2 1  =  −3 1 1 
= Tr[50AB+50I]=50 Tr(AB)+50Tr(I)
 −4 3 2 − 4   −4 3 −2
  
= 50[–1+1–1]+50 [1+1+1] =–50+3(50)=100
1 1
A–1 = adjA = – adjA
|A| 5 Sol 11: Mn = [mij]order = n
AX = b 1 ≤ i ≤ n, mij = 10;
 9 −8 2  1  1 ≤ i ≤ n – 1, mi + 1, I = mi, i + 1 = 3
1    
X = A b = –  −3 1 1 
–1
3
5 All other entries in Mn are zero
 −4 3 −2 5 
   
10 3 0 
9 − 24 + 10   −5   1   
M3 =  3 10 3  , |M3| = 10[100 – 9] + 3 [– 30]
1   1    
X=– −3 + 3 + 5 = – +5 = −1  0 3 10 
5   5      
 −4 + 9 − 10   −5   1 
      = 1000 – 90 – 90 = 820

10 3 
2 0 7   −x 14x 7x  M2 =   ⇒ |M2| = 100 – 9 = 91
  
Sol 10: A = 0 1 0  , B=  0 1 0 
  3 10 
1 −2 1   x −4x −2x  D3 – 9D2 = 820 – 9(91) = 820 – 819 = 1
   

 −2x + 7x 28x − 28x 14x − 14x   −5 1 3 1 1 2 


     
AB =  0 1 0 Sol 12: A =  7 1 −5 & B =
 3 2 1 
 −x + x 14x − 2 − 4x 7x − 2x   1 −1 1  2 1 3 
     
5x 0 0 x + y + 2z =
1 4 0 0
   
=0 1 0 3x + 2y + z =7 AB =  0 4 0  = 4I3
 0 10x − 2 5x  2x + y + 3z =2 0 0 4 
   

|AB| = 5x [5x] = 25x2  −5 1 3


 
4 4 4
5x 0 0 −1 A 7 1 −5 
1   1B = =
(AB)–1 = 0 25x 2
0 4 4 4 4
 
25x2  2  1 −1 1 
 0 −50x + 10x 5x   4 4 4 
M a them a ti cs | 16.63

|B| = 1[6 – 1] + 1[2 – 9] + 2[3 – 4] x  b 


3 −2 1 
=5–7–2=–4      
5 −8 9  y  =  3 
2 1 −3  z   −1 
1 1 2      
|X|
x= , |X| 7 2 1 = [6 – 1] + 7[2 – 3] R2 ⇒ R2 + 2R3
|B |
2 1 3
 3 −2 1  x  6  b
+ 2[1 – 4] = + 5 – 7 – 6 = – 8        
5 + 4 −8 + 2 9 − 6  y  = 3 − 2  =  1 
 2 1 −3  z   −1   −1 
1 1 2       
−8
x= =2,Y= 3 7 1
−4 3 −2 1  x b
2 2 3      
9 −6 3  y  =  1 
= 1[21 – 2] + 1 [2 – 9] + 2[6 – 14] = 19 – 7 – 16 = – 4 2 1 −3 z   −1 
     
1 1 1 Compare row 2nd and 1st
−4
y= =1, Z = 3 2 7 3x – 2y + z = b  … (i)
−4
2 1 2
9x – 6y + 3z = 1
= 1[4 – 7] + 1[14 – 6] + 1[3 – 4] = – 3 + 8 – 1 = 4 1
3x – 2y + z = … (ii)‘
or ⇒ Bx = C 3
x = B–1C From equation (i) and (ii)
−4 1
z= =–1 b=
−4 3
1
(x, y, z) = (2, 1, –1) For no. solution a = – 3 and b ≠
3
 −5 1 3 (iii) Has infinitely solution
 
4 4 4 1  1
so a = – 3 and b =
1
7 1 −5     
x =  7  =  +1 
3
4 4 4
  2   −1  so |D| = 0 and |D|x = 0
 1 −1 1   3×1  
 4 4 4 3×3
1 2  3 1  1 2 
Sol 14: A =   , B = ,C=  
3 4  1 0  2 4 
3 −2 1   x   6 
      X X2 
Sol 13: 5 −8 9   y  =  3  X = 1 
2 1 a   z   −1 
       X3 X4 

3 −2 1  (a) AX = B – I
 
|D| = 5 −8 9  = 3[–8a – 9] – 2 [18 – 5a] + 1[5 + 16] \
2 1 a  1 2   X1 X 2  3 1  1 0  2 1 
  X     =  –   =  
3 4   X3 X 4  1 0  0 1  1 −1
= – 24a – 27 – 36 + 10a + 21 = – 14a – 42
(i) System has a unique solution 101 ≠ 0 |A| = 4 – 6 = – 2

– 140 – 42 ≠ 0  4 −2 
adjA =  
42  −3 1 
a≠– =–3
14 1  4 −2  1
⇒ A–1 =−   = | A | adjA
a ≠ – 3 and b ∈ R −2  −3 1 
(ii) At a = – 3 has no solution ⇒ a = – 3
so A–1AX = X = A–1(B – I)
1 6 . 6 4 | Matrices

1  4 −2  2 1  1 8 − 2 4 + 2   a11 a12  1  −1 


X=     =–   so     =  
−2  −3 1  1 −1 2 6 + 1 −3 − 1 a21 a22   −1  2

 −3 −3 a11 – a12 = – 1;+a21 – a22 = 2 … (i)


16 6   
X=–   = 5
2  −5 −4   2
 2   a2 + a a a11a12 + a12a22 
M2 =  11 12 21

(b) (B – I)X = IC = C a11a21 + a21a22 a21a12 + a222 

2 1  1 1 
B–I=   M2 =   =  
1 −1  −1  0 
|B – I| Z[ –1] – 1 = – 3  a2 + a a a11a12 + a12a22  1 1 
⇒  11 12 21
   =  
 −1 −1 a11a21 + a21a22 a21a12 + a222  −1
  0 
adj(B – I) =   , (B – I)
–1

 −1 2 
2
a11 + a12a21 + a12a22 = 1 … (ii)
adj(B − I) 1  −1 −1 1 1 1 
= =   =  
|B−I | −3  −1 2  3 1 −2 ⇒ a11a21 + a21a22 – a21a12 + a222 = 0

1 1 1  1 2  ⇒ a11[a11 – a12] + a12[a21 + a22] = 1


X = (B – I)–1C =   
3 1 −2 2 4  ⇒ a11(–1) + a12(2) = 1

1 1 + 2 2 + 4  1 3 6  1 2 ⇒ 2a12 – a11 = 1
X=   =   =  
3 1 − 4 2 − 8  3  −3 −6   −1 −2  ⇒ a12 + 1 = 1 ⇒ a12 = 0

(c) CX = A ⇒ a11 = – 1
⇒ a21[a11 – a12] + a22[a21 – a22] = 0
1 2
|C| = =4–4=0
2 2 ⇒ a21[–1] + a22[2] = 0

So C–1 does not exit ⇒ Y has no solution ⇒ 2a22 – a21 = 0


–[a21 – a22 – a22] = 0
Sol 15: A is orthogonal matrix 2 – a22 = 0
⇒ AA’ = A’A = In ⇒ a22 = 2
and B = AP, P is non-singular ⇒ a21 = 4
if A is orthogonal, so A–1 is also orthogonal
a a12   −1 0 
B = AP M =  11 = 
a21 a22   4 2 
BB–1 = APB–1
|M – XI| = 0
I = APB–1
−1 − x 0
A–1 = A–1APB–1 =0
4 2−X
A–1 = PB–1
(1 + x) (x – 2) = 0 ⇒ x = – 1 or x = 2
A–1 is orthogonal, so PB–1 is also orthogonal
5x1 + 2x2 = 5(2) + 2(–1) 2 > – 1 = 10 – 2 = 8
1  −1  1 1   2 3
Sol 16: M   =   ; M2   =   Sol 17: A1 = 1, A2 =  ,
 −1  2  −1  0   4 5
a a12  6 7 8
Assume M =  11   
a21 a22  A3 =  9 10 11  …………….
12 13 14 
 
M a them a ti cs | 16.65

No. of element in An = n2 1 
 0 0
For An = 102 = 100, (10 in each row)
5 
 1 1 1 1 1
Sn = sum of all element of An A = 0 0  , |A|= . . =
6 5 6 7 210
So S1 = 1, S2 = 2 + 3 + 4 + 5  
0 1
0
S9 = 1 + 2 + 3+………m  7 
Where m = 1 + 22 + 32 + 42 +……+92 1 
 0 0
=
(2n + 1)n(n + 1)
=
9(18 + 1)(9 + 1)  42 
 1 1
6 6 Adj A =  0 0  , A–1 =
3 35 |A|
 
= × 10 × 19 = 285 0 1 
2 0
 30 
So in a10 ⇒ a11 = 285 + 1 = 286
a22 = 286 + 11 1 
 
ann = 286 + (n – 1)11 1  42 
 1 
10 adjA = 1  
tr(A) = ∑ aij = 286 × 10 + [11 + 11(2) 210

35

i=1
 1 
+ 3(11)+……+9(11)]  30 
= 286º + 11 [1 + 2 +…+ 9]
1 1 1
9 × 105 Tr(A–1) = 210  + +  = 5 + 6 + 7 = 18
= 286º + 11 × = 286º + 11 × 45 = 3355  42 35 30 
2
 J6,5 72 J11,5 
1 xn  
Sol 18: In, m = ∫ 0 xm − 1 dx ∀ n, m (b) A =  J7,5

63 J12,5 

1 xn  J8,5 56 J13,5 
In, m = ∫ 0 xm + 1 dx ∀ x > m, n, m ∈ N
 J6,5 72 J11,5 
 
(a)A = [aij]3×3 B =  J7,5 63 J12,5 
 
I −Ii+3,3 , i=
j  J8,5 56 J13,5 
aij =  6 +i,3
 0 ,i≠ j det(A) = – 72 [J7, 5 J13, 5 – J12, 5 J8, 5]+…………….
a11 = I6+1, 3 – I1 + 3, 3 = I7, 3 – I4, 3 Jn, α Jm, α – JN,α JM, α

1 x7 dx 1 x 4 dx 1x
7
− x4 1 xn + m xn + m 
= ∫ 0 x3 − 1 ∫ 0 x3 − 1
– =∫
0
x3 − 1
dx If n + m = N + N, the ∫ 0  x x +1 −  dx =
x α+1 
0

1
So det (A) = 0
1 4  x3 − 1   x5  1
=∫ x  3  dx =   = |B| = 72 [I12, 5I8, 5 – I7, 5 I13, 5] +……….
0  x −1  5  0 5
 
Sum as above 12 + 8 = 7 + 13
1
1  x6  1 So, |B| = 0
a22 = I8, 3 – I5, 3 = ∫ x5dx =   =
0
 6  0 6 det(A) – det (B) = 0
1
I 1 6  x3 − 1   x7  1
8+1 ,3 3 −4  a b
a33 = – I6, 3 = ∫ x  3 =
 x −1
 dx =  Sol 19: A =   and B =  
  0 7
7
0
  1 −1  0 1 
P is orthogonal matrix ⇒ Q = PAPT,
R = PTQKP, S = PBPT, T = PTSK P
1 6 . 6 6 | Matrices

Sol 20: A → p,q,t; B → s; C → p,r; D → r 1 2 3 


 
Sol 2: (A) A =  4 5 7 
A2×2 = [aij]
2 3 α
 
Elements are 0, 1, 2, 4
f(x) = x3 – 8x2 + bx + γ
a a12 
(A) A =  11  a satisfies f(x) = 0
a21 a22 

|A| = a11 a22 – a21a12 1 0 


Sol 3: (A) two rowed unit matrix I2 =  
If |A| > 0 ⇒ a11a12 > a21 a12 0 1 
2
F2 = I2
1 0  α β
  ⇒ |A| = 2 So square root of I2 = I2 =   (given)
 4 2  γ δ

2 1 1 2 1 0  α β 
⇒ 8, =4   = 
0 4 0 4 0 1   γ δ 

(B) If |A| = m, then is also a matrix ⇒ |A| = – m α=1= δ,γ=β=0

So for all matrix, have one –ve det (A) matrix so Σ det(A)
=0  4 2i
Sol 4: (A) A =   (A – 2I) (A – 3I) = ?
(C) Least value of det(A) = 2 or – 2  i 1

|adj(adj(adjA))| = ((±2)2–1)2–1)2–1 = ±2, 2 or –2  4 2i 1 0  4 − 2 2i  2 2i 


A – 2I =   –2  =   =  
(D) det(A) is algebraically least = – 8  i 1 0 1   i 1 − 2  i −1
 4 2i 1 0   4 − 3 2i  1 2i 
4adjA 4  1  A – 3J =   – 3  = = 
4A–1 = = adjA =   (adj A)  i 1 0 1   i 1 − 3  i −2
|A| −8  −2 
2 2i  1 2i 
|4A–1| = |–Z–1adjA| = (–Z)2 |A|2–1 (A – 2J) (A – 3J) =    
1  i −1  i −2
= ×–8=–2
4 2(1) + 2i(i) 4i − 4i  0 0 
=  =  = null matrix
 i−i 2i(i) − 1( −2) 0 0 

Exercise 2
 1 cos(α − β) cos(α − γ )
 
Single Correct Choice Type Sol 5: (D) A = cos(β − α ) 1 cos(β − γ ) 
cos( γ − α ) cos( γ − β) 1 
 
Sol 1: (B) Let [A, B] = AB – BA
|A= 1[1 – cos(β – γ)cos(γ – β)] + cos (α – β) [cos(β – r) cos
[[A, B], C] + [[B, C], A] + [[C, A], B]
(γ – α) – cos(β – α)]
⇒ [[A, B], C] = [AB – BA, C] = (AB – BA)C – C(AB – BA)
+cos (α – γ)[cos(β – α) cos(γ – β) – cos(γ – α)]
= ABC – BAC – CAB + CBA …(i)
( cos(A) = cos(–A))
[[B, C], A] = [BC, – CB, A] = (BC – CB)A – A(BC – CB)
= 1 – cos2 (β – γ) +2cos (α – β) cos (β – γ) cos (γ – α) –
= BCA – CBA – ABC + ACB …(ii) cos2 (β – α) = cos2(α – γ)
[[C, A], B] = [CA – AC, B] = (CA – AC) B – B(CA – AC)  ( α + β − γ − α + γ − β) 
2
= 1 – cos 
= CAB – A, B – BCA + BAC …(iii)  2 
sum of equation (i), (ii) & (iii) = 1 – cos20 = 1 – 1 = 0
[[A, B]C] + [[B, C], A] + [[C, A], B] = ABC – BAC + BAC –
ABB +……. = 0
M a them a ti cs | 16.67

x + a b c  −1
  1 + 1 0 
−1 2 0  1 2 0 
Sol 6: (C) A =  a x +b c  =   =   =  
 0 1 + 1 0 2  det(A) 0 2 
 a b x + c 

1 
matrix A is non singular  0
1 2 0  2
=   =  
|A| ≠ 0 4 0 2  0 1
 2 
x + a b c 
 
 a x +b c  ≠0 1 
 a b x + c   x  −2
 5 0
 Sol 9: (C)  25  =  
⇒ (x + a) [(x + b) (x + c) – bc] + b[ac – a(x + c)] + c[ab 0 1   −a 5 
 25 
– a(x + b)] ≠ 0
1
−1 −1
⇒ (x + a) [x2 + x(b + c)] + b [ac – ax – ac] + (c) (–ax) ≠ 0  5 0  5 0 1 5 a 25 5 a
=     =    
⇒ x + ax + x (b + c) + ax(b + c) – abx – acx ≠ 0
3 2 2
 −a 5   −a 5  25 0 5  0 5 
⇒ x3 + x2(a + b + 1) ≠ 0
1 5 a 5 a 1 / 25 x 
= −     
⇒ x2[x + (a + b + c)] ≠ 0 625 0 5  0 5   0 1 / 25
⇒ x ≠ 0 and x ≠ – (a + b + c)
1 25 5a + a5 1 25 10a
Sx x = R – {0, –(a + b + c)} =  =  
625  0 25  625  0 25 

Sol 7: (B) A is skew symmetric matrix 10a 2a


x= =
A = A and B B = B
2 T 625 125

BTB = B
Sol 10: (D) A2 = I
Multiply with B ⇒ (B B)B = BB = I
–1 T –1 –1
|A| =1, B = (adj A)–1
BI=B =I
T T
1
A–1 = adj(A) = adj(A)
BT = I. So B = I |A|
X = (A + B) (A – B) (A–1)–1 = (adjA)–1
X = A2 – AB + BA – B2( B = I) A =(adj A)–1 = B given
X=A–A+A–I=A–I A=B
X7 = (A – I)T = AT – I A2 = I
X7X = (AT – 1) (A – I) AA = AB = I
= AAT – AT – A + I AB = AA = BA = I
A = – A( A is skew symmetric)
T
⇒ B ≠ I we can’t say that B = I
X7X = – AA – A + A + I
Sol 11: (B) adj A = Border of both = 3 × 3
= – A2 + I = – A + I = I – A
Adj (3AB)= 33–1 adj (AB)
Sol 8: (C) Z, and Z2 are uni modular complex = 9(adj B) (adjA) = 9(adj B)B = 9|B| = I3

 z1 − z 2 
−1
 z1 z2 
−1 \adj(AB) = (adjB) (adj A)
    = A (assume)
 z2 z1   z2 z1 
Sol 12: (C) AT + B = 0
−1
 z z + z 2 z2 z1 z 2 − z 2 z1  A = adjB, tr(A) = 1, A2 = A
=  1 1 
 z2 z1 − z1 z2 z2 z 2 + z1 z1 
tr{adj (ATB)}
1 6 . 6 8 | Matrices

⇒ AT + B = 0 ⇒ M2 NI(–M–1)(–N–1)(–M)
⇒ AT = – B ⇒ – M2 NM–1N–1M
⇒ tr[(adj B) (adj AT)] ⇒ – M ⋅ (MN)M–1N–1 M
⇒ tr[A adj(–B)] ⇒ – M(NM)M–1N–1 M
⇒ tr(A(–1)n–1A] ⇒ – MN(NN–1)N–1 M
⇒ (–1)n–1 tr(A2) = (–1)n–1 tr(A) ⇒ – M(NN–1)M
⇒ (–1)n–1 (–1) = (–1)n ⇒ – M2
Note: Here, non-singular word should not be used,
Sol 13: (C) C = A + B since there is no non-singular 3 x 3 skew-symmetric
|C|2 = |A|2 |I – (A–1B)2| matrix.

AB = BAC = A + B x2 + x x +1 x−2
2
⇒ |C| = |A + B| = |A| [I + A–1B] Sol 3: Let ∆ = 2x + 3x − 1 3x 3x − 3
2
|C|2 = |A|2 |I – A–1B| |I + A–1B| … (ii) x + 2x + 3 2x − 1 2x − 1

(2) | C |2 Applying R2 → R2 – (R1 + R3), we get


Equation ⇒
(1) |C|
x2 + x x +1 x−2
2 −1 −1
| A | | I − A B || I + A B | ∆= −4 0 0
=
−1
| A || I + A B | x2 + 2x + 3 2x − 1 2x − 1
|C| = A–1 |I – A–1B| = |A – B| x2
Applying R1 → R1 + R
|C| = |A – B| 4 2
x2
and R3 → R3 + R,w
4 2
Previous Years’ Questions x x +1 x −2
∆ = −4 0 0
Sol 1: (A) | A | ≠ 0, as non-singular
2x + 3 2x − 1 2x − 1
1 a b
Applying R3 → R3 – 2R1
∴ ω 1 c ≠0
x + 0 x +1 x −2
ω2 ω 1
= −4 0 0
⇒ 1(1 – cω) – a (ω – cw2) + b (w2 – w2) ≠ 0
3 −3 3
⇒ 1 – cω – aω + acw2) ≠ 0
x x x 0 1 −2
⇒ (1 – cω) (1 – aω) ≠ 0
= −4 0 0 + −4 0 0
1 1 3 −3 3 3 −3 3
⇒a≠ ,c≠
ω ω
⇒ a = ω, c = ω and b ⇒{ ω, w2} 1 1 1 0 1 −2
⇒ 2 solutions = x −4 0 0 + −4 0 0
3 −3 3 3 −3 3
Sol 2: (C) Given, MT = – M, NT = – N ⇒ ∆ = Ax + B
and MN = NM … (i) 1 1 1 0 1 −2
∴ M N (M N) (MN )
2 2 T –1 –1 T Where A = −4 0 0 and B = −4 0 0
⇒ M2N2N–1(MT)–1(N–1)TMT 3 −3 3 3 −3 3

⇒ M2N(NN–1)(–M)–1(NT)–1(–M)
M a them a ti cs | 16.69

Sol 4: The given system of equation a−1 n 6


3x – y + 4z = 3 Sol 6: Given, Da = (a − 1)
2 2
2n 4n − 2
x + 2y – 3z = –2 (a − 1) 3 3
3n 3n2 − 3n
6x + 5y + lz = – 3 n

Has at least one solution, if ∆ ≠ 0 ∑ (a − 1) n 6


a=1
n
3 −1 4 ∴= ∑ (a − 1)2 2n2 4n − 2
∴ ∆ = 1 2 −3 ≠ 0 a=1
n
6 5 λ
∑ (a − 1)3 3n3 3n2 − 3n
a=1
⇒ 3(2λ + 15) + 1(λ + 18) + 4(5 – 12) ≠ 0
⇒ 7(λ + 5) ≠ 0 ⇒ λ ≠ – 5 = Applying C3 → C3 – 6C1
For λ = – 5 1 1 0
n3 (n − 1)
⇒∆=0 = 2n − 1 6n 0 = 0
12
3 −1 4 n − 1 6n 0
Then, D1 = −2 2 −3 = 0 n
−3 5 −5 ⇒ ∑ ∆a = c, ( c = 0 ie, constant)
a=1

3 3 4
p b c
D2 = 1 −2 −3 = 0
Sol 7: Let ∆ = a q c
6 −3 −5
a b r
3 −1 3
Applying R1 → R2 – R1 and R3 → R3 – R1, we get
D3 = 1 2 −2 = 0
6 5 −3 p b c
a−p q−b p b
∆ = a − p q − b 0 =c +(r–c)
D1 = D2 = D3 = 0 a−p 0 a−p q−b
a−p 0 r −c
Sol 5: The system of equations has non-trivial solution, = – c(a – p) (q – b) + (r – c) [p (q – b) – b(a – p)]
if ∆ = 0
= – c(a – p) (q – b) + p(r – c) (q – b) – b(r – c)(a – p)
sin3θ −1 1 Since, ∆ = 0
⇒ cos2θ 4 3 = 0
⇒ – c(a – p) (q – b) + p(r – c) (q – b) – b(r – c)(a – p) = 0
2 7 7
[On dividing both side by Radding 204 th side and –x
Expanding along C1, we get + ( sin α ) y − ( cos α ) z =
0 has non-
⇒ sin 3θ ⋅ (28 – 21) – cos 2θ (–7 –7) + 2(– 3 –4) = 0 b c b
(a – p) (q – b) (r – c)] + + +2 =2
⇒ 7sin3θ +14cos2θ – 14 = 0 b −a r −c q−b
⇒ sin 3θ + 2cos 2θ – 2 = 0 p q r
⇒ + +0+ +0=2
⇒ 3sinθ – 4sin3θ + 2(1 – 2sin2θ) – 2 = 0 p −a q−b r −c

⇒ sinθ (4sin2θ + 4sinθ – 3) = 0 p q r


⇒ + + =2
p −a q−b r −c
⇒ sinθ (2sinθ – 1) (2sinθ + 3) = 0
⇒ sinθ = 0, sinθ = 1/2 n! (n + 1)! (n + 2)!
(neglecting sinθ = – 3/2) Sol 8: Given, D = (n + 1)! (n + 2)! (n + 3)!
(n + 2)! (n + 3)! (n + 4)!
⇒ θ = nπ, nπ + (– 1)n π/6, n ∈ Z
Taking n!, (n + 1)! and (n + 2)! Common from R1 , R2 and
R3 respectively.
1 6 . 7 0 | Matrices

1 (n + 1) (n + 1)(n + 2) ∴ 2α – π/4 = 2n π ± π/4


∴ D = n!(n+1)!(n+2)! 1 (n + 2) (n + 2)(n + 3) ⇒ 2α = 2nπ – π/4 + π/4 or 2α = 2nπ + π/4 + π/4
1 (n + 3) (n + 3)(n + 4) ∴ α = nπ or nπ + π/4
Applying R2 → R2 – R1 and R3 → R3 – R2, we get
Sol 10: Given,
1 (n + 1) (n + 1)(n + 2)
ax − by − c bx + ay cx + a
D = n!(n+1)!(n+2)! 0 1 2n + 4
0 1 2n + 6 bx + ay −ax + by − c cy + b =0
cx + a cy + b −ax − by + c
Expanding along C1, we get
D = (n!)(n + 1)!(n + 2)![(2n + 6) – (2n +4)] a2 x − aby − ac bx + ay cx + a
1 2
D = (n!)(n + 1)!(n + 2)![2] ⇒ abx + a y −ax + by − c cy + b =0
2 2
acx + a cy + b −ax − by + c
On dividing both side by (n!)3

D (n!)(n!)(n + 1)(n!)(n + 1)(n + 2)2 Applying C1 → C1 + bC2 + cC3


⇒ =
3 3
(n!) (n!)
(a2 + b2 + c2 )x
ay + bx cx + a
D 1 2 2 2
⇒ = 2(n + 1) (n + 1) (n + 2) ⇒ (a + b + c )y by − c − ax b + cy = 0
(n!)3 a 2 2 2
a +b +c b + cy c − ax − by
D
⇒ = 2(n3 + 4n2 + 5n + 2) = 2n (n2 + 4n + 5) + 4 x ay + bx cx + a
(n!)3 1
⇒ y by − c − ax b + cy = 0
D a
⇒ – 4 = 2n(n2 + 4n + 5) 1 b + cy c − ax − by
(n!)3
( a2 + b2 + c2 = 1)
 D 
Which shows that  − 4  is divisible by n. Applying C2 → C2 – bC1
3
 (n!) 
and C3 → C3 – cC1
Sol 9: Given, lx + (sin α)y + (cos α)z = 0 x ay a
x + (cos α) y+(sin α) z = 0 1
⇒ y −c − ax b =0
a
and – x + (sin α) y – (cos α) z = 0 has non-trivial solution. 1 cy −ax − by

∴∆=0
x2 axy ax
λ sin α cos α 1
⇒ y −c − ax b =0
⇒ 1 cos α sin α = 0 ax
1 cy ax − by
−1 sin α − cos α
Applying R1 → R1 + yR2 + R3
⇒ λ (– cos2 α – sin2α) – sin α (– cos α + sin α) + cos α
(sin α + cos α) = 0 x2 + y 2 + 1 0 0
1
⇒ – λ + sin α cos α + sin α cos α – sin2 α + cos2 α = 0 ⇒ y −c − ax b =0
ax
⇒ λ = cos 2α + sin2a 1 cy −ax − by

 − a2 + b2 ≤ asin θ + bcos θ ≤ a2 + b2  1
  ⇒ [(x2 + y2 + 1) {(– c – ax) (– ax – by) – b(cy)}]= 0
  ax
∴– 2 ≤λ≤ 2  … (i) 1
⇒ [(x2 + y2 + 1) (acx + bcy + a2x2 + abxy – bcy)]= 0
Again, when λ =1, cos 2α + sin 2α = 1 ax
1 1 1 1
⇒ cos 2α + sin 2α = ⇒ [(x2 + y2 + 1) (acx + a2x2 + abxy)]= 0
2 2 2 ax
⇒ cos (2α – π/4) = cos π/4 1
⇒ [ax(x2 + y2 + 1) (c + ax + by)]= 0
ax
M a them a ti cs | 16.71

⇒ (x2 + y2 + 1) (ax + by + c) = 0 a α ι 
⇒ ax + by + c = 0  T 
P
= b β m
Which represents a straight line. c γ n 
 

( ( ) )
T
∆ 1 1 − cω − a ω − ω2c + b ( 0 )
Sol 11: (A) = P= 2P + I

∆= 1cω − aω + ω2ac  a α ι   2a 2b 2c  1 0 0 
     
b β m = 2α 2β 2γ  + 0 1 0 
∆ = 1 − ω ( c + a) + ω2ac  c γ n   2ι 2m 2n 0 0 1 
     
c=ω a=ω2 singular
a α ι  2a + 1 2b 2c 
ω2 a =
c= ω singular    
b β m=  2 α 2β + 1 2 γ 
c=
ω a=
ω non singular c γ n   2ι 2m 2n + 1 
  
ω2 a =
c= ω2 singular
=2b =α ,b =α
2 .It is possible when b = α , = 0
for every pair (a, c) there are two possible values of b
Similarly,, c = ι = 0
hence 2 matrices.
m=γ =0
 a1 a2 a3   −1 0 0 
   
Sol 12: Let M = b1 b2 b3  The matrix P is  0 −1 0 
c c  0 0 −1
 1 2 c3   
So , PX = −X
 a1 a2 a3  0   −1 
     
b1 b2 b3  . 1  =  2  Sol 14: (D) P = aij 
c c c3  0   3   
 1 2
Q = bij 
 
⇒ a2 =
−1,b2 =
2 ,c2 =
3
bij = 2i+ j.aij
 a1 a2 a3   1  1
      = 2
b11 2= 3
a11 b21 2= .a21 b31 24 a31
b1 b2 b3  .  −1=
  1  ⇒ a1 − a2= 1
c c2 c3   0   −1 
 1   = 3
b12 2= 4
a12 b22 2= .a22 b32 25 a32
⇒ a1=0, b1=3, c1=3 = 4
b13 2= a13 b23 25=
.a22 b33 26 a33

 a1 a2 a3  1 0 a11 a12 a13


     
b1 b2 b3  . 1=  0  ⇒ c1 + c2 + c3= 12 Given P a=
= 21 a22 a23 2
c c c3  1 12
 1 2   a31 a32 a33

⇒ c3 = 12 − 5 = 7 22 a11 23 a12 24 a13


∴ Sum of diagonal elements = a1 + b2 + c3 = 0 + 2 + 7 = 9 Q = 23 a21 24 a22 25 a23
24 a31 25 a32 26 a33
Sol 13: (D) There seems to be an ambiguity in the
question since 3 × 3 skew-symmetric matrices can’t be a11 a12 a13
non-singular. 2 3
Q = 2 .2 .2 a21 4
a22 a23
[Property: Determinant of an odd order skew- a31 a32 a33
symmetric matrix is always zero]
22 a11 23 a12 24 a13
P is a 3 × 3 matrix Q = 22.23.24 23 a21 24 a22 25 a23

 a b c 24 a31 25 a32 26 a33


 
Let P =α β γ 
 ι m n
 
1 6 . 7 2 | Matrices

a11 a12 a13 


( ) 
2
 ( −ω) + ω ω2s ( −ω) + ωr ω2s 
2r 2s r
2 3 4 2 1
Q = 2 .2 .2 .2 .2 a21 a22 a23 =  = −I (Given)
a31 a32 a33 ω2s ( −ω) + ωr ω2s

r
(
ω2r ωr + ω ) 

13
Q = 22.23.24 .22.21.21 = 2 r s r s
1 1 1 1
Sol 15: (B, C, D) P2 = 0 only when n is multiple of 3. 2 2 3 3
Total no. pairs = 1
ω2 1 ω  ω2 1 ω  0 0 0 
      −1
E.g:  1 ω ω2  .  1 ω ω2  =0 0 0  P  P 
    Sol 19: (B, C)   .Q = I ∴Q=  
 ω ω2 1  ω ω2 1  0 0 0  K
  K 
  
Comparing P23 we get, −K −K ( 3α + 4 )
∴ P2 ≠ 0 when n = 55,56,58 =
8 12α + 20
−K −K ( 3α + 4 ) α = −1
= ⇒
Sol 16: (A, B) MN = NM 8 12α + 20
α = −1
N2M = N (NM) = (NM) N = (MN) N = MN2 Also P Q = K 3

(M − N (M + N )) =M
2 2 2
+ MN2 − N2M − N=
4
M2 − N4
∴ (12a ++ 20)
∴(12a 20 )
K2
=K3
As M − N2 ≠ 0 ⇒ M + N2 =
0 ...2 2
KK == 6a
6α +
+ 10
10 = =44
M2 + MN2 = M M + _ N=
2
(
M M + N2
) Sol 20: (B)
2
= 0 ⇒ M+N =
0  1 0 0  1 0 0  1 0 0
  2  
= P =4 1 0  4 1 0  8 1 0
Sol 17: (D) When roots are purely imaginary. 16 4 1  16 4 1   48 8 1 
    
2
Then the form of equation is x + K =0
 1 0 0  1 0 0  1 0 0
where K is positive no. 2     
= P =8 1 0  4 1 0 12 1 0 
Let p ( x=
) x +K
2
 48 8 1  16 4 1  96 12 1 
    
( ) (p ( x ) ) + K
2
p(x)
p=  
 1 0 0
p (p ( x ) ) = ( x2 + K ) + K
n  4n
2
∴∴P = 1 0
 
 (
8 n2 + n ) 4n 1 

(
p p(x) = )
x 4 + 2Kx2 + K ⇒ p p ( x ) = 0 ( )
 
 1 0 0
x 4 + 2Kx2 + K =0
∴∴P50  200
= 1 0
 
All coefficients are positive and no odd degree of x are
present.  (
8 × n2 + n ) 4n 1 

−1 + i 3 P50 − Q =
I
Sol 18: (A) z = = ω
2 0 q21 =
Equation we get 200 − q21 =⇒ 200
 −ω r ω2s 
p=
( ) 400 × 51 − q31 =
0
 ω2s ωr  q= 400 × 51
 31

 −ω r 200 − q32 =0 ⇒ q32 =200


( ) ω2s  ( −ω) ω2s 
r
2
p =   q31 + q32 400 × 51 + 200
 ω2s

r 
ω   ω2s r 
ω 
q21
=
200
(51) +=1 130
= 103
2017-18 100 &
op kers
Class 12 T
By E ran culty
-JE Fa r
IIT enior emie .
S fP r es
o titut
Ins

MATHEMATICS
FOR JEE MAIN & ADVANCED
SECOND
EDITION

Exhaustive Theory
(Now Revised)

Formula Sheet
9000+ Problems
based on latest JEE pattern

2500 + 1000 (New) Problems


of previous 35 years of
AIEEE (JEE Main) and IIT-JEE (JEE Adv)

5000+Illustrations and Solved Examples


Detailed Solutions
of all problems available

Topic Covered Plancess Concepts


Tips & Tricks, Facts, Notes, Misconceptions,
Determinants Key Take Aways, Problem Solving Tactics

PlancEssential
Questions recommended for revision
17. DETERMINANTS

1. INTRODUCTION
Development of determinants took place when mathematicians were trying to solve a system of simultaneous
linear equations.

a1 x + b1 y =
c1  b2c1 − b1c2 a1c2 − a2c1
E.g.  ⇒ x= and y =
a2 x + b2 y =
c2  a1b2 − a2b1 a1b2 − a2b1

a1 b1
Mathematicians defined the symbol as a determinant of order 2 and the four numbers arranged in row
a2 b2
and column were called its elements. Its value was taken as (a1b2 − a2b1 ) which is the same as the denominator.
a b
If we write the coefficients of the equations in the following form then such an arrangement is called a
c d
determinant. In a determinant, horizontal lines are known as rows and vertical lines are known as columns. The
shape of every determinant is a square. If a determinant is of order n then it contains n rows and n columns.

a1 b1 c1
a1 b1
E.g. , a2 b2 c2 are determinants of second and third order respectively.
a2 b2
a3 b3 c3

Note: (i) No. of elements in a determinant of order n are n2. (ii) A determinant of order 1 is the number itself.

1.1 Evaluation of the Determinant using SARRUS Diagram

 a11 a12 a13 


 
If A = a21 a22 a23  is a square matrix of order 3, the below diagram is a Sarrus Diagram obtained by adjoining
a a32 a33 
 31
the first two columns on the right and draw dark and dotted lines as shown.

The value of the determinant is (a11a22a33 + a12a23a31 + a13a21a32 ) − (a13a22a31 + a11a23a32 + a12a21a33 ) .
1 7 . 2 | Determinants

a11 a12 a13 a11 a12

a21 a22 a23 a21 a22

a31 a32 a33 a31 a32

Figure 17.1

3 2 5
Illustration 1: Expand 9 −1 4 by Sarrus rules.  (JEE MAIN)
2 3 −5

Sol: By using Sarrus rule i.e. ∆ = (a11a22a33 + a12a23a31 + a13a21a32 ) − (a13a22a31 + a11a23a32 + a12a21a33 ) we can expand
the given determinant.

3 2 5 3 2

-1 4 9 -1

2 3 -5 2 3

Figure 17.2

3 2 5
Here, =
∆ 9 −1 4 ⇒ ∆= 15 − 36 + 90 + 16 + 135 + 10= 230
2 3 −5

x2 − x + 1 x − 1
Illustration 2: Evaluate the determinant :  (JEE MAIN)
x +1 x +1

Sol: By using determinant expansion formula we can get the result.

x2 − x + 1 x − 1
we have, = (x2 − x + 1)(x + 1) − (x + 1)(x − 1) = x3 + x2 − x2 − x + x + 1 − x2 + 1 = x3 − x2 + 2
x +1 x +1

2. COFACTOR AND MINOR OF AN ELEMENT


Minor: Minor of an element is defined as the determinant obtained by deleting the row and column in which that
a11 a12 a13
a a23
element lies. e.g. in the determinant D = a21 a22 a23 , minor of a12 is denoted as M12 = 21 and so on.
a31 a33
a31 a32 a33

Cofactor: Cofactor of an element ai j is related to its minor as Ci j = ( −1)i + j Mi j , where ‘i’ denotes the ith row and
‘j’ denotes the jth column to which the element ai j belongs.
Now we define the value of the determinant of order three in terms of ‘Minor’ and ‘Cofactor’ as

D = a11M11 − a12M12 + a13M13 or D = a11C11 + a12C 12 + a13C13


M a them a ti cs | 17.3

Note:
(a) A determinant of order 3 will have 9 minors and each minor will be a determinant of order 2 and a determinant
of order 4 will have 16 minors and each minor will be determinant of order 3.
(b) a11C21 + a12C22 + a13C23 =
0 , i.e. cofactor multiplied to different row/column elements results in zero value.

Row and Column Operations


(a) Ri ↔ Rj or Ci ↔ Cj , when i ≠ j; This notation is used when we interchange ith row (or column) and jth row (or
column).
(b) Ri ↔ Ci ; This converts the row into the corresponding column.
(c) Ri → Rki or Ci → kCi ; k ∈ R; This represents multiplication of ith row (or column) by k.
(d) Ri → Rik + Rj or Ci → Cik + Cj ; (i ≠ j); This symbol is used to multiply ith row (or column) by k and adding the
jth row (or column) to it.
2 −3 5
Illustration 3: Find the cofactor of a12 in the following 6 0 4  (JEE MAIN)
1 5 −7

Sol: In this problem we have to find the cofactor of a12, therefore eliminate all the elements of the first row and
the second column and by obtaining the determinant of remaining elements we can calculate the cofactor of a12.
Here a12 = Element of first row and second column = –3

2 −35

6 4
M12 = Minor of a12 ( −3) =6 0 4 = = 6(–7) − 4(1) = −42 − 4 = –46.
1 −7

1 5 −7

( −1)1+ 2 ( −46) =
Cofactor of ( −3) = −( −46) =
46

Illustration 4: Write the minors and cofactors of the elements of the following determinants:

2 −4 a c
(i) (ii)  (JEE MAIN)
0 3 b d

Sol: By eliminating row and column of an element, the remaining is the minor of the element.
2 −4
2 −4
(i) ; M11 = Minor of element (2) =  ( −1)1+1 M11 =
= 3 ; Cofactor of (2) = +3
0 3
0 3

2 −4
M12 = Minor of element (–4) =  = 0 ; ( 1)1+ 2 M12 =−
Cofactor of ( −4) =− ( 1)0 =0
0 3

2 −4
M21 = Minor of element (0) =  = −4 ; ( 1)2+1 M21 =−
Cofactor of (0) =− ( 1)( −4) =4
0 3

2 −4
M22 = Minor of element (3) =  = 2 ; ( −1)2+ 2 M22 =
Cofactor of (3) = +2
0 3
1 7 . 4 | Determinants

a c
a c
(ii) ; M11 = Minor of element (a) =  = d; ( −1)1+1 M11 =
Cofactor of (a) = ( −1)2 d =
d
b d
b d
a c
M12 = Minor of element (c) =  = b ; ( −1)1+ 2 M12 =
Cofactor of (c) = ( −1)3 b =
−b
b d

a c
M21 = Minor of element
= (b) = c ; ( −1)2+1 M21 =
Cofactor of (b) = ( −1)3 c =
−c
b d

a c
M22 = Minor of element
= (d) =  a ; ( −1)2+ 2 M22 =
Cofactor of (d) = ( −1)4 a =
a
b d
2 −2 3
Illustration 5: Find the minor and cofactor of each element of the determinant 1 4 5 .  (JEE ADVANCED)
2 1 −3

Sol: By eliminating the row and column of an element, the determinant of remaining elements is the minor of the
element. i.e. Mi× j and by using formula ( −1)i+ j Mi× j we will get the cofactor of the element.

4 5 1 5 1 4
The minors are M11 = = −17 , M12 = = −13 , M13 = = −7
1 −3 2 −3 2 1

−2 3 2 3 2 −2

= M21 = 3 , M22 = = −12 , M23 = = −6
1 −3 2 −3 2 1

−2 3 2 3 2 −2
M31 = = −22 , M32
= = 7 , =
M33 = 10
4 5 1 5 1 4

The cofactors are:

( −1)1+1 M11 =
A11 = M11 = ( −1)1+ 2 M12 =
−17 , A12 = −M12 = ( −1)1+3 M13 =
13 , A13 = M13 =
−7

( −1)2+1 M21 =
A21 = −M21 = ( −1)2+ 2 M22 =
−3 , A22 = M22 = ( −1)2+3 M23 =
−12 , A23 = −M23 =
6

( −1)3+1 M31 =
A31 = M31 = ( −1)3+ 2 M32 =
−22 , A32 = −M32 = ( −1)3+3 M33 =
−7 , A33 = M33 =
10

3. PROPERTIES OF DETERMINANTS
Determinants have some properties that are useful as they permit us to generate the same results with different
and simpler configurations of entries (elements).
(a) Reflection Property: The determinant remains unaltered if its rows are changed into columns and the
columns into rows.
(b) All-zero Property: If all the elements of a row (or column) are zero, then the determinant is zero.
(c) Proportionality (Repetition) Property: If the all elements of a row (or column) are proportional (identical)
to the elements of some other row (or column), then the determinant is zero.
(d) Switching Property: The interchange of any two rows (or columns) of the determinant changes its sign.
M a them a ti cs | 17.5

(e) Scalar Multiple Property: If all the elements of a row (or column) of a determinant are multiplied by a non-
zero constant, then the determinant gets multiplied by the same constant.

a1 + b1 c1 d1 a1 c1 d1 b1 c1 d1
(f) Sum Property: a2 + b2 c2 d2 = a2 c2 d2 + b2 c2 d2
a3 + b3 c3 d3 a3 c3 d3 b3 c3 d3

a1 b1 c1 a1 + αb1 + βc1 b1 c1
(g) Property of Invariance: a2 b2 c2= a2 + αb2 + βc2 b2 c2
a3 b3 c3 a3 + αb3 + βc3 b3 c3

That is, a determinant remains unaltered under an operation of the form Ci → Ci + αC j + βCk , where j, k ≠ i ,
or an operation of the form R i → R i + αR j + βRk , where j, k ≠ i

(h) Factor Property: If a determinant ∆ becomes zero when we put x = α , then (x − α ) is a factor of ∆ .

(i) Triangle Property: If all the elements of a determinant above or below the main diagonal consist of zeros,
then the determinant is equal to the product of diagonal elements. That is,

a1 a2 a3 a1 0 0
0 b2 b3
= a=
2 b2 0 a1b2c3
0 0 c3 a3 b3 c3

a11 a12 a13 C11 C12 C13
( j) Determinant of cofactor matrix: ∆ = a21 a22 a23 then ∆1 =C21 C22 C23 =∆2
a31 a32 a33 C31 C32 C33

where Cij denotes the cofactor of the element aij in ∆ .

PLANCESS CONCEPTS

By interchanging two rows (or columns), the value of the determinant differs by a –ve sign.
If ∆ ′ is the determinant formed by replacing the elements of a determinant ∆ by their corresponding
cofactors, then if ∆ =0 , then ∆1 =0 ,else ∆ ′ =∆n−1 , where n is the order of the determinant.
Vaibhav Gupta (JEE 2009 AIR 54)

a b c
Illustration 6: Using properties of determinants, prove that b c a = (a + b + c)(ab + bc + ca − a2 − b2 − c2 )
c a b (JEE MAIN)


Sol: By using invariance and scalar multiple property of determinant we can prove the given problem.

a b c a+b+c b c
∆= b c a = b + c + a c a [Operating C1 → C1 + C2 + C3 ]
c a b c+a+b a b

1 b c 1 b c
= (a + b + c) 1 c a = (a + b + c) 0 c − b a − c [Operating (R 2 → R 2 − R1 and R 3 → R 3 − R1 )]
1 a b 0 a−b b −c

= (a + b + c){(c − b)(b − c) − (a − b)(a − c)} = (a + b + c)(bc − b2 − c2 + bc − (a2 − ab − ac + bc)) = (a + b + c)(ab + bc + ca − a2 − b2 − c2 )


1 7 . 6 | Determinants

−α2 βα γα
2
Illustration 7: Prove the following identity αβ −β γβ = 4α2β2 γ 2  (JEE MAIN)
2
αγ βγ −γ

Sol: Take α , β, γ common from the L.H.S. and then by using scalar multiple property and invariance property of
determinant we can prove the given problem.
−α2 βα γα
2
∆= αβ −β γβ
αγ βγ −γ 2
−α α α
Taking α , β, γ common from C1 , C2 , C3 respectively ∆ = αβγ β −β β
γ γ −γ

−1 1 1
2 2 2
Now taking α , β, γ common from R1 ,R 2 ,R 3 respectively ∆ = α β γ 1 −1 1
1 1 −1

−1 1 1
2 2 2
Now applying R 2 → R 2 + R1 and R 3 → R 3 + R1 we have ∆ = α β γ 0 0 2
0 2 0
0 2
Now expanding along C1 , ∆ = α2 × β2 ( −1) × γ 2 ( −1) = α2β2 ( −1)γ 2 (0 − 4) = 4α2β2 γ 2
2 0
Hence proved.

α β γ β µ φ
Illustration 8: Show that θ φ ψ = α λ θ  (JEE ADVANCED)
λ µ ν γ ν ψ

Sol: Interchange the rows and columns across the diagonal using reflection property and then using the switching
property of determinant we can obtain the required result.

α β γ α θ λ
L.H.S. = θ φ ψ = β φ µ (Interchanging rows and columns across the diagonal)
λ µ ν γ ψ ν

α λ θ β µ φ β µ φ
2
( 1) α λ θ =α λ θ = R.H.S.
=( −1) β µ φ =−
γ ν ψ γ ν ψ γ ν ψ

a a2 1 + a3
Illustration 9: If a, b, c are all different and if b b2 1 + b3 =
0 , prove that abc = –1.  (JEE ADVANCED)
2 3
c c 1+c

Sol: Split the given determinant using sum property. Then by using scalar multiple, switching and invariance
properties of determinants, we can prove the given equation.

a a2 1 + a3 a a2 1 a a2 a3 a a2 1 1 a a2
D = b b2 1 + b3 = b b2 1 + b b2 b3 = b b2 1 + abc 1 b b2
c c2 1 + c3 c c2 1 c c2 c3 c c2 1 1 c c2
M a them a ti cs | 17.7

1 a2 a 1 a a2
[C1 ↔ C3 in 1st det.]
= ( −1)1 1 b2 b + abc 1 b b2
1 c2 c 1 c c2

1 a a2 1 a a2
= ( −1)2 1 b b2 + abc 1 b b2 [C2 ↔ C3 in 1st det.]
1 c c2 1 c c2

1 a a2 1 a a2 1 a a2
= 1 b b2 + abc 1 b b2 = (1 + abc) 1 b b2
1 c c2 1 c c2 1 c c2

1 a a2
= (1 + abc) 0 b − a b2 − a2 [R 2 → R 2 − R1 and R3 → R3 − R1 ]
0 c − a c2 − a2

b − a b2 − a2 1 b+a
= (1 + abc) (expanding along 1st row) = (1 + abc) (b – a) (c – a)
c−a c −a 2 2 1 c+a

= (1 + abc) (b – c) (c – a) (c + a – b – a) = (1 + abc) (b – a) (c – a) (c – b)

⇒ D = (1 + abc) (a – b) ( b – c) (c – a); But given D = 0

⇒ (1 + abc) (a – b) ( b – c) (c – a) = 0; ∴ (1 + abc) = 0

[since a, b, c are different a ≠ b, b ≠ c, c ≠ a ]; Hence, abc = – 1

a + b + 2c a b
Illustration 10: Prove that c b + c + 2a b = 2(a + b + c)3  (JEE ADVANCED)
c a c + a + 2b

Sol: Simply by using switching and scalar multiple property we can expand the L.H.S.

a + b + 2c a b
Given determinant = c b + c + 2a b
c a c + a + 2b

Applying C1 → C1 + (C2 + C3 ) , we obtain

2(a + b + c) a b 1 a b
2(a + b + c) b + c + 2a b = 2(a + b + c) 1 b + c + 2a b
2(a + b + c) a c + a + 2b 1 a c + a + 2b

R1 → R 2 − R1 and R3 → R3 − R1 given

1 a b
2(a + b + c) 0 b + c + a 0 = 2(a + b + c) ⋅ 1{(b + c + a)(c + a + b) − (0 × 0)} = 2(a + b + c)3
0 0 c+a+b

Hence proved.
1 7 . 8 | Determinants

a2 + 1 ab ac
2
Illustration 11: Prove that ab b +1 bc =1 + a2 + b2 + c2  (JEE ADVANCED)
2
ac bc c +1

a2 + 1 ab ac
2
Sol: Expand the determinant ab b +1 bc by using scalar multiple and invariance property.
2
ac bc c +1

a2 + 1 ab ac
2
L.H.S.= ab b +1 bc ; Multiplying C1 , C2 , C3 by a, b, c respectively
2
ac bc c +1

a(a2 + 1) ab2 ac2


1
= a2b b(b2 + 1) bc2 ; Now taking a, b, c common from R1 , R 2 , R3 respectively
abc 2 2 2
ac b c c(c + 1)

a2 + 1 b2 c2 1 + a2 + b2 + c2 b2 c2
abc
= a2 b2 + 1 c2 = 1 + a2 + b2 + c2 b2 + 1 c2 [C1 → C1 + C2 + C3 ]
abc 2 2 2 2 2 2 2 2
a b c +1 1+a +b +c b c +1

1 b2 c2 1 b2 c2
= (1 + a2 + b2 + c2 ) 1 b2 + 1 c2 = (1 + a2 + b2 + c2 ) 0 1 0 [R 2 → R 2 − R1 and R3 → R3 − R1 ]
1 b2 c2 + 1 0 0 1

= (1 + a2 + b2 + c2 ) (1.1.1) = 1 + a2 + b2 + c2 = R.HS.

Hence proved.

PLANCESS CONCEPTS

|AB| = |A||B|
The value of the determinant is the same when expanded by any row or any column. Using this property
it is easier to expand determinant using a row or column in which most zeroes are involved.
Vaibhav Gupta (JEE 2009 AIR 54)

4. SYMMETRIC AND SKEW SYMMETRIC DETERMINANTS


4.1 Symmetric Determinant
a h g
A determinant is called Symmetric Determinant if ai j = a ji , ∀ i, j e.g. h b f
g f c
M a them a ti cs | 17.9

4.2 Skew Symmetric Determinant


A determinant is called a skew symmetric determinant if ai j =−a ji ∀ i, j for every element.
0 3 −1
E.g. −3 0 5
1 −5 0

Note: (i) det |A| = 0 ⇒ A is singular matrix (ii) det | A | ≠ 0 ⇒ A is non-singular matrix

PLANCESS CONCEPTS

The value of a skew symmetric determinant of an even order is always a perfect square and that of an
odd order is always zero.
Vaibhav Krishnan (JEE 2009 AIR 22)

5. MULTIPLICATION OF TWO DETERMINANTS


(a) Multiplication of two second order determinants as follows: (as R to C method)

a1 b1 l1 m1 a1l1 + b1l2 a1m1 + b1m2


× =
a2 b2 l2 m2 a2l1 + b2l2 a2m1 + b2m2

(b) Multiplication of two third order determinants is defined.
a1 b1 c1 l1 m1 n1
a2 b2 c2 × l2 m2 n2 (as R to C method)
a3 b3 c3 l3 m3 n3

a1l1 + b1l2 + c1l3 a1m1 + b1m2 + c1m3 a1n1 + b1n2 + c1n3
= a2l1 + b2l2 + c2l3 a2m1 + b2m2 + c2m3 a2n1 + b2n2 + c2n3
a3l1 + b3l2 + c3l3 a3m1 + b3m2 + c3m3 a3n1 + b3n2 + c3n3

Note:
(i) The two determinants to be multiplied must be of the same order.
(ii) To get the Tmn (term in the mth row nth column) in the product, Take the mth row of the 1st determinant and
multiply it by the corresponding terms of the nth column of the 2nd determinant and add.
(iii) This method is the row by column multiplication rule for the product of 2 determinants of the nrd order
determinant.
(iv) If ∆ ′ is the determinant formed by replacing the elements of a ∆ of order n by their corresponding co-factors
then ∆ ′ =∆n−1 . ( ∆ ′ is called the reciprocal determinant).
2
0 c b
Illustration 12: Reduce the power of the determinant c 0 a to 1. (JEE MAIN)
b a 0
Sol: By multiplying the given determinant two times we get the determinant as required.
2
0 c b 0 c b 0 c b b2 + c2 ab ac
2 2
c 0 a = c 0 a c 0 a ⇒ ba c +a bc
b a 0 b a 0 b a 0 ca cb a + b2
2
1 7 . 1 0 | Determinants

2
a2 + x2 ab − cx ac + bx
x c −b
2 2
Illustration 13: Show that ab + cx b + x bc − ax = −c x a . (JEE ADVANCED)
2
ac − bx bc + ax c + x 2 b −a x

Sol: By replacing all elements of L.H.S. to their respective cofactors and using determinant property we will obtain
the required result.

x c −b
Let D = −c x a
b −a x

Co-factors of 1st row of D are x2 + a2 , ab + cx , ac − bx . Co-factors of 2nd row of D are ab − cx, x2 + b2, ax + bc and
co-factors of 3rd row of D are ac + bx , bc − ax , x2 + c2
∴ Determinant of cofactors of D is

x2 + a2 ab + cx ac − bx a2 + x2 ab − cx ac − bx
c 2 2
D = ab − cx x + b ax + bc = ab + cx b2 + x2 bc − ax = D2
ac + bx bc − ax x 2 + c2 ac − bx ax + bc x2 + c2
2
x c −b
(Row interchanging into columns) = −c x a ( Dc = D2 , D is third order determinant)
b −a x
2
a2 + x2 ab − cx ac + bx
x c −b
2 2
Hence ab + cx b + x bc − ax = −c x a
2
ac − bx bc + ax c + x 2 b −a x

6. SOME STANDARD DETERMINANTS

1 a a2 a b c 1 1 1
2 2 2 2 2
2
(i) 1 b b =(a − b)(b − c)(c − a) (ii) a b c = a b c2 = (a − b)(b − c)(c − a)(ab + bc + ca)
1 c c2 bc ca ab a3 b3 c3

2 3
a bc abc a a a 1 1 1
2 3
(iii) b ca abc= b b b = abc (a − b)(b − c)(c − a) ; (iv) a b c = (a − b)(b − c)(c − a) (a + b + c)
c ab abc c c2 c3 a3 b3 c3

a b c
(v) b c a =−a3 − b3 − c3 + 3abc
c a b

p+ q 2 r r
Illustration 14: Evaluate the determinant =
∆ qr + 2p r 2r , where p, q and r are positive real
numbers.  q + pr qr r (JEE MAIN)

Sol: Taking r common from C2 and C3 of the given determinant using scalar multiple property and then
expanding it using the invariance property we can evaluate the given problem.
M a them a ti cs | 17.11

p+ q 2 1
We get ∆ = r qr + 2p r 2
q + pr q r

Applying C1 → C1 − qC2 − pC3

− q 2 1
We get D = r 0 r 2 = −r q(r − 2q) = r q 2 − r q . ( )
0 q r

a b c
Illustration 15: Let a, b, c be positive and not equal. Show that the value of the determinant b c a is negative.
c a b
 (JEE ADVANCED)

Sol: By applying invariance and scalar multiple properties to the given determinant we can get the required result.

a b c a+b+c b c
D = b c a ; then D = a + b + c c a [C1 → C1 + C2 + C3 ]
c a b a+b+c a b

1 b c
= (a + b + c) 1 c a [Taking (a + b+ c) common from the first column]
1 a b

1 b c
= (a + b + c) 0 c − b a − c [ R 2 → R 2 − R1 and R 3 → R 3 − R1 ]
0 a−b b −c

= (a + b + c+ ) [(c − b)(b − c) − (a − b)(a − c)] = (a + b + c+ ) [bc + ca + ab − a2 − b2 − c2 ]


1
= −(a + b + c) (a2 + b2 + c2 − bc − ca − ab) = − (a + b + c)(2a2 + 2b2 + 2c2 − 2bc − 2ca − 2ab)
2
1
= − (a + b + c) [(a + b − 2ab) + (b + c − 2bc) + (c2 + a2 − 2ac)]
2 2 2 2
2
1 2 2 2
= − (a + b + c) [(a − b) + (b − c) + (c − a) ]  … … (i)
2
 a, b, c, are positive ⇒a+b+c>0
 a, b, c are unequal ⇒ (a − b)2 + (b − c)2 + (c − a)2 > 0  … …(ii)
∴ From (i) and (ii) , ∆ < 0 .

1 cos2 (α − β) cos2 (α − γ )
Illustration 16: Show that
= ∆ cos2 (β − α ) 1 cos2 (β −=
γ ) 2sin2 (β − γ )sin2 ( γ − α )sin2 (α − β) (JEE ADVANCED)
cos2 ( γ − α ) cos2 ( γ − β) 1
Sol:.By Putting β − γ = A , γ − α =B , α − β = C and then by using switching and invariance properties we can prove
the above problem.

1 cos2 C cos2 B
We can write ∆ as, ∆ = cos2 C 1 cos2 A (Note that A + B + C = 0.)
2 2
cos B cos A 1
1 7 . 1 2 | Determinants

Using C2 → C2 − C1 , C1 → C3 − C1 we get

1 − sin2 C − sin2 B 1 − sin2 C − sin2 B


∆ cos2 C sin2 C cos2 A − cos2 C cos2 C sin2 C sinBsin(C − A)
2 2 2 2 2
cos B cos A − cos B sin B cos B sinCsin(B − A) sin2 B

1 sin2 C sin2 B
= ( −1 ) cos2 C
2
− sin2 C sinB sin( −A)
2
cos B sinC sin(B − A) − sin2B

[ cos2 A − cos2 B =sin(A + B)sin(B − A), A + B =−C,C + A =−B ]; = sinC sinB [ ∆1 ]

1 sin2 C sinB
2 2
where =
∆1 cos C − sin C sin(C − A) Using R 2 → R 2 − R1 and R 3 → R 3 − R1 we get
2
cos B sin(B − A) − sinB

1 sinC sinB
2 2
∆1 = − sin C −2sin C sin(C − A) − sinB
2
− sin B sin(B − A) − sinC −2sin2 B

But sin (C – A) – sin B = sin (C – A) + sin (C + A) = 2 sin C cos A and sin (B – A) – sin C = 2 sin B cos A
1 sinC sinB
Therefore,
= ∆1 sinCsinB= ∆2 where ∆2 sinC 2 −2cos A
sinB −2cos A 2

Applying R 2 → R 2 − sinC R1 and R3 → –sinB R1 we get

1 sinC sinB
=∆2 0 2 − sin2 C −2cosA − sinBsinC = (2 − sin2 B)(2 − sinC) − (2cos A + sinBsinC)2
0 −2cos A − sinBsinC 2 − sin2 B

= 4 − 2sin2 B − 2sin2 C + sin2 Bsin2 C − [4 cos2 A + 4 cos A sinBsinC + sin2 Bsin2 C]

= 4 sin2 A − 2sin2 B − 2sin2 C − 4 cos A sinBsinC

= 2sin2 A − 2 [sin2 B + sin2 C − sin2 A + 2cos A sinBsinC]

But A + B + C = 0 implies; sin2 B + sin2 C − sin2 A =


−2cos A sinBsinC
∴ ∆2 =2sin2 A ; sinB∆1 sin2 C sin2 B∆2
Hence, D = sinC=
= 2sin2 A sin2 Bsin2 C = 2sin2 (α − β)sin2 (β − γ )sin2 ( γ − α ) .

Illustration 17: Prove that the following determinant vanishes if any two of x; y; z are equal
sinx sin y sinz  (JEE ADVANCED)
∆ = cos x cos y cos z
cos3 x cos3 y cos3 z

Sol: Taking cos x, cos y, and cos z common from first, second and third column using scalar multiple and then using
the invariance property we can prove the given statement.
M a them a ti cs | 17.13

tanx tan y tanz


Here, ∆ =cos x cos y cos z 1 1 1
cos2 x cos2 y cos2 z

tanx tan y − tanx tanz − tan y


= cos x cos y cos z 1 0 0 (C3 → C3 − C2 , C2 → C2 − C1 )
2 2 2 2 2
cos x cos y − cos x cos z − cos y

tan y − tanx tanz − tan y


Expanding along R 2 ; ∆ = − cos x cos y cos z
cos y − cos x cos2 z − cos2 y
2 2

sin(y − x) sin(z − y)
cos z.sin(x − y) cos x.sin(y − z)
= − cos x cos y cos z cos x cos y cos y.cos z =  … (i)
2 2 2 2
sin(x + y).sin(x − y) sin(y + z).sin(y − z)
sin x − sin y sin y − sin z

cos z cos x
=sin(x − y)sin(y − z) = sin(x − y)sin(y − z)[sin(y + z)cos z − sin(x + y)cos x)]
sin(x + y) sin(y + z)
1
= sin(x − y)sin(y − z)[{sin(y + 2z) + sin y} − {sin(y + 2x} + sin y}]
2
1 1
= sin(x − y)sin(y − z)[sin(y + 2z) − sin(y + 2x)]
= sin(x − y)sin(y − z)2cos(x + y + z)sin(z − x)
2 2
= sin(x − y)sin(y − z)sin(z − x)cos(x + y + z)
π
Clearly, ∆ is zero when any two of x, y, z are equal or x + y + z = .
2
Hence proved.

7. SYSTEM OF EQUATIONS
7.1 Involving Two Variables
a1x+b1y+c1 = 0
a2x+b2y+c2 = 0

Consistent Inconsistent
System of equation has solution System of equation has no solution
a1 b1 c1
= =
a2 b2 c2
Unique solution Infinite solution
a1 b1 a1 b1 c1 (Equation represents parallel disjoint lines)
= or a1b2 a2b1 0 = =
a2 b2 a2 b2 c2
(Equation represents (Equation represents
intersecting lines) coincident lines)

Figure 17.3

∆1 ∆2 b1c2 − b2c1 a2c1 − a1c2


Solution to this system of equations is given by x = ,y = or x = ; y=
∆ ∆ a1b2 − a2b1 a1b2 − a2b1

b c1 c a1 a b1
where ∆1 = 1 , ∆2 = 1 and ∆ = 1
b2 c2 c2 a2 a2 b2
1 7 . 1 4 | Determinants

7.2 Involving Three Variables

a1 x + b1 y + c1 z =
d1 a2 x + b2 y + c2 z =
d2 a3 x + b3 y + c3 z =
d3
To solve this system we first define the following determinants

a1 b1 c1 d1 b1 c1 a1 d1 c1 a1 b1 d1
∆ = a2 b2 c2 , ∆1 =d2 b2 c2 , ∆2 =a2 d2 c2 , ∆3 =a2 b2 d2
a3 b3 c3 d3 b3 c3 a3 d3 c3 a3 b3 d3

Now following algorithm is followed to solve the system (CRITERION FOR CONSISTENCY)

Check value of 

 0  0

Consistent system and Check the values of 1, 2 and 3


has unique solution
1 2 3
x= ;y= ;z=
   Atleast one of 1, 2 and 3 1= 2 = 3=0
is not zero
Put z = t and solve any two equations
Inconsistent system to get the values of x and y in terms of t

Figure 17.4

This method of finding solution to a system of equations is called Cramer’s rule.


Note:
(a) If ∆ =0 and ∆1 =∆2 =∆3 =0 , then system of equation may or may not be consistent:
(i) If the value of x, y and z in terms of t satisfy the third equation then system is said to be consistent and
will have infinite solutions.
(ii) If the values of x, y, z don’t satisfy the third equation, then system is said to be inconsistent and will have
no solution.
(b) If d=
1 d=
2 d=
3 0 , then system of linear equations is known as Homogeneous linear equations, which always
possess at least one solution i.e. (0, 0, 0). This is called trivial solution for homogeneous linear equations.
(c) If the system of homogeneous linear equations possess non-zero/nontrivial solutions, and ∆ = 0. In such case
given system has infinite solutions.
We can also solve these solutions using the matrix inversion method.
We can write the linear equations in the matrix form as AX = B where

 a1 b1 c1  x d1 
     
A = a2 b2 c2  , X =  y  and B = d2 
a b c3  z  d 
 3 3    3
Now, solution set is obtained by solving X = A-1B. Hence the solution set exists only if the inverse of A exists.

Illustration 18: Solve the following equations by Cramer’s rule x + y + z = 9, 2x + 5y + 7z = 52, 2x+ y – z = 0.
 (JEE MAIN)
M a them a ti cs | 17.15

Sol: Here in this problem define the determinants ∆ , ∆1 , ∆2 and ∆3 and find out their value by using the invariance
property and then by using Cramer’s rule, we can get the values of x, y and z.
1 1 1
Here ∆ = 2 5 7 (Applying C2 → C2 − C1 and C3 → C3 − C1 )
2 1 −1

1 0 0 9 1 1
∴ ∆ 2=
= 3 5 1 (–9 + 5) = – 4; ∆1 =52 5 7 (Applying C2 → C2 + C3 )
2 −1 −3 0 1 −1

9 2 1 1 9 1
∴ ∆1 =52 12 7 =−1 (108 – 104) = –4; ∆2 =2 52 7 (Applying C1 → C1 + 2C3 )
0 0 −1 2 0 −1

3 9 1 1 1 9
∴ ∆2 =16 52 7 =−1 (156 – 144) = –12 and ∆3 =2 5 52 (Applying C1 → C1 − 2C2 )
0 0 −1 2 1 0

−1 1 9
∴ ∆3 = −8 5 52 (Applying C1 → C1 − 2C2 ) =−1( −52 + 72) =−20
0 1 0

∆1 −4 ∆ 2 −12 ∆3 −20
∴ By Cramer’s rule=
x = = 1 ,=
y = = 3 and=
z = = 5
∆ −4 ∆ −4 ∆ −4
∴ x = 1, y = 3, z = 5

4 3 6 6
Illustration 19: Solve the following linear equations: + −1 and
= − −5 
= (JEE MAIN)
x +5 y + 7 x +5 y + 7

1 1
Sol: Here in this problem first put = a and = b and then define the determinants ∆, ∆1 and ∆2. Then by
x +5 y+7
using Cramer’s rule we can get the values of x and y.
1 1
Let us put = a and = b then the 2 linear equations become
x +5 y+7
4a + 3b = – 1  …(i)
and 6a – 6b = –5  …(ii);
Using Cramer’s Rule, we get,
x y 1 a b 1
= = ⇒= =
−1 3 4 −1 4 3 6 + 15 −20 + 6 −24 − 18
−5 −6 6 −5 6 −6

a b 1 −1 1
∴ = = ⇒ a = and b =
21 −14 −42 2 3
1 1 1
∴ a= − ⇒ = − ⇒ 2 = – x – 5 ⇒ x = –7
2 x +5 2
1 1 1
b= ⇒ = ⇒ 3 = y + 7 ⇒ y = –4
3 y+7 3
1 7 . 1 6 | Determinants

Illustration 20: For what value of k will the following system of equations possess nontrivial solutions.
Also find all the solutions of the system for that value of k.
x + y – kz = 0; 3x – y – 2z = 0; x – y + 2z = 0.  (JEE ADVANCED)

Sol: Here in this problem first define ∆. As we know that, for non-trivial solution ∆ =0 .
So by using the invariance property we can solve ∆ =0 and will get the value of k.
For non-trivial solution, ∆ =0
1 1 −k 2 0 −k + 2
⇒ 3 −1 −2 =0 ⇒ 2 0 −4 =0 [R1 → R1 + R3 , R 2 → R 2 − R3 ]
1 −1 2 1 −1 2

Expanding along C2 . ⇒ –(–1) [–8 – 2 (2 – k)] = 0 ⇒ 2k – 12 = 0 ⇒ k=6


Putting the value of k in the given equation, we get,
x + y − 6z = 
0 … (i)
3x − y − 2z = 
0 … (ii)
x − y + 2z =
0  ... (iii)
x
(i) + (ii) ⇒ 4x – 8z = 0 ∴ z=
2
Putting the value of z in (i), we get x + y – 3x = 0 ∴ y = 2x
t
Thus when k = 6, solution of the given system of equations will be x = t, y = 2t, z = , when t is an arbitrary number.
2

Illustration 21: Solve the following equations by matrix inversion.


2x + y + 2z = 0 2x – y + z = 10 x + 3y – z = 5  (JEE ADVANCED)

Sol: By writing the given equations into the form of AX = D and then multiplying both side by A −1 we will get the
required value of x, y and z.
2 1 2   x   0 
    
In the matrix form, the equations can be written as 2 −1 1  y  = 10 
1 3 −1  z   5 
    
2 1 2  x 0
     
∴ AX = D where A = 2 −1 1 , X = y  , D = 10 
1 3 −1  z  5
    
⇒ A −1 (AX) = A −1D ⇒ X = A −1D  … (i)

2 1 2
−1 adjA
Now A = ; =| 2 −1 1 = 2(1 – 3) – 1(–2 –1) + 2(6 + 1) = 13
|A
|A|
1 3 −1
 −2 3 7  −2 7 3  −2 7 3
    −1 1  
The matrix of cofactors of |A| is  7 −4 −5 . So, adj A=  3 −4 2=; A  3 −4 2 .
13
 3 2 4  7 −5 −4   7 −5 −4 
     
 −2 7 3  0   0 + 70 + 15   85 / 13  x  85 / 13 
1    1        
∴ from (1), X =  3 −4 2 10  = 0 − 40 + 10  =−  30 / 13 ; y  =  −30 / 13 .
13 13
 7 −5 −4   5   0 − 50 − 20   −70 / 13 z   −70 / 13
          
85 −30 −70
⇒ x= ,y= ,z=
13 13 13
M a them a ti cs | 17.17

PLANCESS CONCEPTS

In general if r rows (or columns) become identical when a is substituted for x, then (x − a)r −1 is a factor
of the given determinant.
Anvit Tawar (JEE 2009 AIR 9)

7.3 Some Important Results


The lines: a1 x + b1 y + c1 =
0  … (i)
a2 x + b2 y + c2 =
0 … (ii)
a3 x + b3 y + c3 =
0  ... (iii)
a1 b1 c1
are concurrent if, a2 b2 c2 = 0
a3 b3 c3

This is the condition for the consistency of three simultaneous linear equations in 2 variables.
2 2
(a) ax + 2hxy + by + 2gx + 2fy + c = 0 represents a pair of straight lines if
a h g
2 2 2
abc + 2fgh − af − bg − ch =0 =h b f
g f c
x1 y1 1
1
(b) Area of a triangle whose vertices are (xr , yr ) ; r = 1, 2, 3 is : D = x2 y 2 1 . If D = 0 then the three points
are collinear. 2
x3 y 3 1

x y 1
(c) Equation of a straight line passing through (x1 , y1 ) & (x2 , y 2 ) is x1 y1 1 = 0 .
x2 y2 1

(d) 
If each element of any row (or column) can be expressed as a sum of two terms, then the determinant can be
expressed as the sum of the determinants.
a1 + x b1 + y c1 + z a1 b1 c1 x y z
E.g., a2 b=2 c2 a2 b2 c2 + a2 b2 c2
a3 b3 c3 a3 b3 c3 a3 b3 c3

It should be noted that while applying operations on determinants at least one row (or column) must remain
unchanged i.e.
Maximum number of simultaneous operations = order of determinant – 1

PLANCESS CONCEPTS

Always expand a determinant along a row or a column with maximum zeros.


To find the value of the determinant, the following steps are taken.
Take any row (or column); the value of the determinant is the sum of products of the elements of the row
(or column) and the corresponding determinant obtained by omitting the row and the column of the
elements with a proper sign, given by ( −1)p + q where p and q are the no. of row and the no. of column
respectively.
Vaibhav Krishnan (JEE 2009 AIR 22)
1 7 . 1 8 | Determinants

8. DIFFERENTIATION AND INTEGRATION OF DETERMINANTS


f1 (x) g1 (x)
Let ∆(x) = , where f1 (x), f2 (x), g1 (x) and g2 (x) are functions of x. Then,
f2 (x) g2 (x)

f1′ (x) g1′ (x) f1 (x) g1 (x) f1′ (x) g1 (x) f1 (x) g1′ (x)
∆ ′(x)
= + Also,
= ∆ ′(x) +
f2 (x) g2 (x) f2′ (x) g2′ (x) f2′ (x) g2 (x) f2 (x) g2′ (x)
Thus, to differentiate a determinant, we differentiate one row (or column) at a time, keeping others unchanged. If we
write ∆(x) = [C1 C2 ] , where Ci denotes the ith column, then ∆= ′(x) [C1′ C2 ] + [C1 C2′ ] , where Ci′ denotes the
R1  R ′   R1 
column obtained by differentiating functions in the ith column Ci . Also, if ∆(x) =
  , then ∆ ′(x)=  1  +  
R 2  ′
 R 2  R 2 
Similarly, we can differentiate determinants of higher order.

Note: Differentiation can also be done column wise by taking one column at a time.

If f(x), g(x) and h(x) are functions of x and a, b, c, α, β and γ are constants such that

f(x) g(x) h(x) ∫ f(x)dx ∫ g(x)dx ∫ h(x)dx


∆(x) =a b c , then the integration of ∆(x) is given by ∫ ∆(x)dx =a b c
α β γ α β γ

sin2 x logcos x logtanx π /2


Illustration 22:=
If ∆(x) n2 2n − 1 2n + 1 , then evaluate ∫ ∆(x) dx .  (JEE MAIN)
1 −2log2 0 0

Sol: By applying integration on variable elements of determinant we will solve the given problem.
π/2 π/2 π/2

∫ sin2 x dx ∫ log cos x dx ∫ logtanx dx


sin2 x logcos x logtanx π /2 0 0 0
2 2
We have,
= ∆(x) n 2n − 1 ; ∫ ∆(x) dx
2n + 1 = n 2n − 1 2n + 1
1 −2log2 0 0 1 −2log2 0

π π
− log2 0
4 2 1 −2log2 0
π 2 π
= n2 2n − 1 2n + 1= n 2n − 1 2n + 1 = × 0 = 0
4 4
1 −2log2 0 1 −2log2 0

xn
sinx cos x
nπ nπ dn
Illustration 23: If f(x) = n! sin cos , then show that {f(x)} = 0 at x = 0. (JEE ADVANCED)
2 2 dxn
a a2 a3

Sol: By applying integration on variable elements of the determinant we will solve the given problem.

dn dn dn
n (xn ) (sinx) (cos x)
x sinx cos x dxn dxn dxn
nπ nπ dn nπ nπ
We have, f(x) = n! sin cos ; {f(x)} = n! sin cos
2 2 dx n 2 2
a a2 a3 a a2 a3
M a them a ti cs | 17.19

 nπ   nπ  nπ nπ
n! sin  x +  cos  x +  n! sin cos+
 2   2  2 2
nπ nπ  dn  nπ nπ
= n! sin cos  n {f(x)} 
; = n!
= sin cos 0
2 2  dx  2 2
  x =0
a a2 a3 a a2 a3

PROBLEM-SOLVING TACTICS

f1 (x) f2 (x) f3 (x) f1′ (x) f2′ (x) f3′ (x)


Let ∆(x) = b1 b2 b3 , then ∆ ′(x) = b1 b2 b3 and in general
c1 c2 c3 c1 c2 c3

f1n (x) f2n (x) f3n (x)


∆n (x) = b1 b2 b3 where n is any positive integer and f n (x) denotes the nth derivative of f(x).
c1 c2 c3

f(x) g(x) h(x)


Let ∆(x) =a b c , where a, b, c, l, m and n are constants.
l m n

b b b

∫ f(x) dx ∫ g(x)dx ∫ h(x) dx


b a a a
⇒ ∫ ∆(x) dx =a b c
a l m n

If the elements of more than one column or rows are functions of x then the integration can be done only
after evaluation/expansion of the determinant.

FORMULAE SHEET

a1 b1 c1
b c2 a c2 a b2
(a) Determinant of order 3 × 3 = a2 b2 c2 = a1 2 − b1 2 + c1 2
b3 c3 a3 c3 b3 b3
a3 b3 c3

a11 a12 a13


a a23
(b) In the determinant D = a21 a22 a23 , minor of a12 is denoted as M12 = 21 and so on.
a31 a33
a31 a32 a33

(c) Cofactor of an element ai j = Ci j = ( −1)i + j Mi j


1 7 . 2 0 | Determinants

(d) Properties of determinants:

(i) Reflection property: Ai× j = A j×i

(ii) All-zero property: If all the elements of a row (or column) are zero, then the determinant is zero.
(iii) Proportionality (Repetition) Property: If all the elements of a row (or column) are proportional
(identical) to the elements of some other row (or column), then the determinant is zero.
(iv) Switching Property: The interchange of any two rows (or columns) of the determinant changes its sign.
(v) Scalar Multiple Property: If all the elements of a row (or column) of a determinant are multiplied by a
non-zero constant, then the determinant gets multiplied by the same constant.
a1 + b1 c1 d1 a1 c1 d1 b1 c1 d1
(vi) Sum Property: a2 + b2 c2 d2 = a2 c2 d2 + b2 c2 d2
a3 + b3 c3 d3 a3 c3 d3 b3 c3 d3

a1 b1 c1 a1 + αb1 + βc1 b1 c1
(vii) Property of Invariance: a2 b2 c2= a2 + αb2 + βc2 b2 c2
a3 b3 c3 a3 + αb3 + βc3 b3 c3

That is, a determinant remains unaltered under an operation of the form Ci → Ci + αC j + βCk , where j, k ≠ i ,
or an operation of the form R i → R i + αR j + βRk , where j, k ≠ i .

a1 a2 a3 a1 0 0
(viii) Triangle Property: =
0 b2 b3 a=
2 b2 0 a1b2c3
0 0 c3 a3 b3 c3

∆ ∆ ∆
(e) Cramer’s rule : if a1 x + b1 y + c1 z =
d1 , a2 x + b2 y + c2 z = d3 then x = 1 , y = 2 , z = 3
d2 and a3 x + b3 y + c3 z =
where ∆ ∆ ∆

a1 b1 c1 d1 b1 c1 a1 d1 c1 a1 b1 d1
∆ = a2 b2 c2 , ∆1 =d2 b2 c2 , ∆2 =a2 d2 c2 and ∆3 =a2 b2 d2 .
a3 b3 c3 d3 b3 c3 a3 d3 c3 a3 b3 d3

∆1 ∆2
And if a1 x + b1 y + c1 =
0 and a2 x + b2 y + c2 =
0 then x = y= .
∆ ∆
b c1 c a1 a b1
Where ∆1 = 1 , ∆2 = 1 and ∆ = 1
b2 c2 c2 a2 a2 b2
a1 b1 c1
(f) (i) lines a1 x + b1 y + c1 =
0 , a2 x + b2 y + c2 =
0 and a3 x + b3 y + c3 =
0 are concurrent if, a2 b2 c2 = 0
a3 b3 c3
a h g
2 2
(ii) ax + 2hxy + by + 2gx + 2fy + c =0 represents a pair of straight lines if h b f =0
g f c

x1 y1 1
1
(iii) area of a triangle whose vertices are (xr , yr ) ; r = 1, 2, 3 is : D = x2 y2 1
2
x3 y3 1

x y 1
(iv) Equation of a straight line passing through (x1 , y1 ) & (x2 , y 2 ) is x1 y1 1 = 0
x2 y2 1
M a them a ti cs | 17.21

f1 (x) g1 (x) f ′ (x) g1′ (x) f1 (x) g1 (x) f1′ (x) g1 (x) f1 (x) g1′ (x)
(g) If ∆(x) = =
then ∆ ′(x) 1 + or +
f2 (x) g2 (x) f2 (x) g2 (x) f2′ (x) g2′ (x) f2′ (x) g2 (x) f2 (x) g2′ (x)

f(x) g(x) h(x) ∫ f(x)dx ∫ g(x)dx ∫ h(x)dx


(h) If ∆(x) =a b c then ∫ ∆(x)dx =a b c
α β γ α β γ

Solved Examples

JEE Main/Boards 3 3 3
⇒ R.H.S. = pqr(a + b + c − 3abc)  .… (ii)

Example 1: Prove that From eq. (i) and (ii), we get

pa qb rc a b c ∴ L.H.S. = R.H.S.
qc ra pb = pqr c a b . Use p + q + r = 0.
rb pc qa b c a Example 2: Prove that the determinant

x sin θ cos θ
Sol: By using the expansion formula of determinants we
− sin θ −x 1 is independent of θ.
can prove this.
cos θ 1 x
pa qb rc
L.H.S.= qc ra pb =
Sol: Simply by expanding the given determinant we can
rb pc qa prove it.
ra pb qc pb qc ra x sin θ cos θ
pa − qb + rc We have, − sin θ −x 1
pc qa rb qa rb pc
cos θ 1 x
= pa(a2qr − p2bc) − qb(q2ac − prb2 ) + rc(pqc2 − r 2ab)
−x 1 − sin θ 1 − sin θ −x
=x − sin θ + cos θ
3 3 3 3
= a pqr − p abc − q abc + b pqr − r abc3 1 x cos θ x cos θ 1

= pqr(a3 + b3 + c3 ) − abc(p3 + q3 + r 3 ) x( −x2 − 1) − sin θ ( −x sin θ − cosθ) + cos θ ( − sin θ + x cos θ)

 p+q+r =0 … (given) = −x3 − x + x sin2 θ + sin θ cosθ − sin θ cos θ + x cos2 θ

(p + q + r)3 = 0 = −x3 − x + x(sin2 θ + cos2 θ) = −x3 − x + x

⇒ p3 + q3 + r3 ‒ pqr = 0 ⇒ p3 + q3 + r3 = 3pqr Thus, the determinant is independent of θ .

⇒ L.H.S. = pqr(a3 + b3 + c3 ) − abc(3pqr) x+a x x


3 3 3
⇒ L.H.S. = pqr(a + b + c − 3abc)  .… (i) Example 3: Solve the equation x x+a x =0,
a≠0. x x x+a
 a b c b c a
R.H.S. = pqr a −b +c 
 c a b a b c  Sol: We can expand the above determinant by applying
the invariance and scalar multiple properties, and hence
= pqr[a(a2 − bc) − b(ca − b2 ) + c(c2 − ab)] we can easily solve this problem.
= pqr[a3 − abc − abc + b3 + c3 − abc]
1 7 . 2 2 | Determinants

x+a x x 2 −1 3
We have, x x+a x =0 Here, ∆ = 1 1 1
x x x+a 1 −1 1

Operation: C1 → C1 + C2 + C3 =2(1 + 1) + 1 (1 – 1) + 3 (–1 – 1) = – 2,


9 −1 3
3x + a x x 1 x x ∆ x =6 1 1 =9(1 + 1) + 1(6 – 2) + 3 (–6 – 2) = –2
3x + a x + a x =0 ⇒ (3x + a) 1 x + a x =0 2 −1 1
3x + a x x+a 1 x x+a
2 9 3
Operating R 2 → R 2 − R1 , R 3 → R 3 − R1 ∆ y =1 6 1 =2(6 – 2) – 9(1 – 1) + 3(2 – 6) = – 4
1 2 1
1 x x
We get (3x + a) 0 a 0 =
0 2 −1 9
0 0 a ∆ z =1 1 6 =2(2 + 6) + 1(2 – 6) + 9(–1 – 1) = –6
1 −1 2
a 0
⇒ (3x + a) 0 ⇒ a2 (3x + a) =
= 0 By Cramer’s Rule
0 a
∆x ∆y ∆z
a x =
= 1 ,=y = 2 ,=
z = 3
⇒ 3x + a = 0, [ a ≠ 0] ⇒ x = − ∆
3 ∆ ∆
Hence Proved.
Example 6: Show that
Example 4: Solve, using Cramer’s rule 3x ‒ 2y + 4z = 5;
a + b + 2c a b
x + y + 3z = 2; ‒x + 2y ‒ z = 1
c b + c + 2a b = 2(a + b + c)3
Sol: By defining D, D1, D2, D3 and by using Cramer’s c a c + a + 2b
Rule we will get required result.
Sol: By using invariance and scalar multiple property
3 −2 4 we can expand given determinant and can prove it.
D= 1 1 3 = −5
−1 2 −1 2(a + b + c) a b
∆ 2(a + b + c) b + c + 2a
= b
5 −2 4 3 5 4 2(a + b + c) a c + a + 2b
D1 = 2 1 3 = −33 , D2 = 1 2 3 = −13
[C1 → C1 + C2 + C3 ]
1 2 −1 −1 1 −1
1 a b
3 −2 5
= 2(a + b + c) 1 b + c + 2a b
=D3 =1 1 2 12
1 a c + a + 2b
−1 2 1
1 a b
D1 −33 33
x =
By Cramer’s Rule,= = , = 2(a + b + c) 0 b + c + a 0
D −5 5
0 a c+a+b
D2 −13 13 D3 12 −12
y
= = = ; =
z = =
D −5 5 D −5 5 [by R 2 → R 2 − R1 and R 3 → R 3 − R1 ]

= 2(a + b + c)[1{(b + c + a)2 − 0}]


Example 5: Solve the following system of equations by
Cramer’s Rule
= 2(a + b + c)(a + b + c)2 = 2(a + b + c)3
2x − y + 3z
= 9; x + y +=
z 6; x − y +=
z 2

Sol: By defining ∆ , ∆ x , ∆ y , ∆ z and by using Cramer’s Example 7: Using determinants, show that the points
Rule we will get the required result. (11, 7), (5, 5) and (–1, 3) are collinear.
M a them a ti cs | 17.23

Sol: If these points are collinear then the area of a JEE Advanced/Boards
triangle made by joining these points will be zero.
The area of the triangle formed by the given points Example 1: Without expanding, evaluate the determinant
11 7 1 sin α cos α sin(α + δ)
1
= 5 5 1 sin β cos β sin(β + δ)
2
−1 3 1 sin γ cos γ sin( γ + δ)
Operate: R1 → R1 − R 2 ; R 2 → R 2 − R 3
Sol: By using the formula sin(A+ B) = sin A cos B + cos
6 2 0 A sin B and invariance property of determinants we can
1 1
= 6 2 0 = ⋅0 = 0 expand the given determinant.
2 2
−1 3 1 sin α cos α sin(α + δ)
( R1 and R 2 are identical) Let =
∆ sin β cos β sin(β + δ)
sin γ cos γ sin( γ + δ)
Hence, the given points are collinear.
sin α cos α sin α cos δ + cos α sin δ
Example 8: If A and B are two matrices such that AB = ⇒=
∆ sin β cos β sin β cos δ + cos β sin δ
B and BA = A, then A2 + B2. sin γ cos γ sin γ cos δ + cos γ sin δ

Sol: By using the multiplication property of matrices we sin α cos α 0


can solve given problem.
⇒ =
∆ sin β cos β 0
A2 + B2 = AA + BB sin γ cos γ 0
= A(BA) + B(AB) [Given AB = B and BA = A]
[Applying C3 → C3 − cos δ . C1 − sin δ . C2 ]
= (AB)A + (BA)B
⇒ ∆ =0 [ C3 consists of all zeroes]
[Matrix multiplication is associatively]=BA + AB
[Given AB = B and BA = A] =A+B Example 2: By using properties of determinants prove
[Given AB = B and BA = A] that

1 x x2
12 22 32 42
22 32 42 52
x2 1 x = (1 − x3 )2
Example 9: Find the value of
32 42 52 62 x x2 1

42 52 62 72
Here in this problem by using invariance and
scalar multiple properties we will expand the given
Sol: By applying the invariance property we can find determinant and we will prove it.
the value of the given determinant.
1 x x2 1 + x + x2 x x2
12 22 32 42 1 4 9 16 1 4 9 16
Sol: L.H.S. = x2 1 x = 1 + x + x2 1 x
22 32 42 52
4 9 16 25 3 5 7 9
= = x x 2
1 1+x+x 2
x 2
1
32 42 52 6 2 9 16 25 36 5 7 9 11
42 52 62 7 2 16 25 36 49 7 9 11 13
[Applying C1 → C1 + C2 + C3 ]
[Applying R 4 → R 4 − R 3 , R3 → R3 − R 2 , R 2 → R 2 − R1 ]
1 x x2
1 4 9 16 2
= (1 + x + x ) 1 1 x
3 5 7 9 1 x2 1
=
2 2 2 2
2 2 2 2 1 x x2

[Applying R 4 → R 4 − R 3 , R 3 → R 3 − R 2 ] = (1 + x + x2 ) 0 1−x x − x2

=0 0 x2 − x 1 − x2
1 7 . 2 4 | Determinants

[Applying R 2 → R 2 − R1 and R 3 → R 3 − R1 ] a b 1
1
= (1 + x + x2 )(1){(1 − x)(1 − x2 ) − (x2 − x)(x − x2 )} c d 1 where (a, b) (c, d) (e, f) are the vertices of
2
e f 1
= (1 + x + x2 )(1 − x)2 {1 + x + x2 }
triangle. Therefore by substituting the value of vertices
= {(1 − x)(1 + x + x2 )}2 = (1 − x3 )2 = R.H.S. we will get required result.
Let the vertices of triangle be A(2, –6), B(5, 4) and C(k, 4).
Example 3: Show that x =−(a + b + c) is one root of
Since the area of the triangle ABC is 35 sq. units, we
x+a b c
2 −6 1 2 −6 1
the equation: b x+c a =0 and solve the 1 1
have, 5 4 1 = ±35 ⇒ 3 10 0 = ±35
c a x +b 2 2
k 4 1 k − 2 10 0
equation completely.
[Applying R 2 → R 2 − R1 and R 3 → R 3 − R1 ]
Sol: We can expand given determinant using the
invariance and scalar multiple properties and by solving 1 3 10
we will find out required result. ⇒ = ±35 [Expanding along C3]
2 k − 2 10
By C1 → C1 + C2 + C3 , we get
1
⇒ {30 − 10(k − 2)} =
±35
x +a+b+c b c 2
x +a+b+c x +c a =0 ⇒ 30 − 10k + 20 =
±70 ⇒ 10k = 50  70
x +a+b+c a x +b ⇒ k = +12 or k = –2
1 b c
Example 5: Solve the following system of equations by
⇒ (x + a + b + c) 1 x + c a =0
using determinants: x + y + z =,
1
1 a x +b
k ; a2 x + b2 y + c2 z =
ax + by + cz = k2
1 b c
Sol: Here in this problem first define D, D1, D2 and D3.
⇒ (x + a + b + c) 0 x − b + c a−c =0
then by using Cramer’s rule we can solve it.
0 a−b x +b−c
1 1 1
R 2 → R 2 − R1 ; R 3 → R 3 − R1 We have, D = a b c

On expanding by first column, we get a2 b2 c2

(x + a + b + c)[(x − b + c)(x + b − c) − (a − b)(a − c)] =0 1 0 0


= a b−a c−a
⇒ (x + a + b + c)[x2 − (b − c)2 − (a2 − ac − ab + bc)] =0
a2 b2 − a2 c2 − a2
⇒ (x + a + b + c)(x2 − b2 − c2 + 2bc − a2 + ac + ab − bc] =0
[Applying C2 → C2 − C1 and C3 → C3 − C1 ]
⇒ (x + a + b + c)(x2 − a2 − b2 − c2 + ab + bc + ca) =0
1 0 0
Either x + a + b + c = 0 ⇒ x = –(a + b + c)
= (b − a)(c − a) a 1 1
or x2 − a2 − b2 − c2 + ab + bc + ca =
0 a2 b + a c + a

⇒ x=± a2 + b2 + c2 − ab − bc − ca 1 1
= (b − a)(c − a).1.
b+a c+a
Example 4: If the area of a triangle is 35 sq. units with [Expanding along R1 ]
vertices (2, –6), (5, 4) and (k, 4), then find k.
= (b − a)(c − a)(c + a − b − a)
Sol: As we know that the area of the triangle =
= (b − a)(c − a)(a − b)  … (i)
M a them a ti cs | 17.25

1 1 1 [R1 → R1 − R 2 , R 2 → R 2 − R3 ]
D1 =k b c =(b − c)(c − k)(k − b)
2 −1 α
k 2 b2 c2
= −1 2 β −1
[Replacing a by k in (i)] u v − 1 1 + a3 + b3
1 1 1 [ C1 → C1 − C2 , C2 → C2 − C3 ]
D 2 =a k c =(k − c)(c − a)(a − k)
a2 k 2 c2 0 −1 α
= 3 2 β −1 [ C1 → C1 + 2C2 ]
[Replacing b by k in (i)]
u + 2v − 2 v − 1 1 + a3 + b3
1 1 1
and D3 =a b k =(a − b)(b − k)(k − a) = [3(1 + a3 + b3 ) − (u + 2v − 2)(β − 1)]
2 2 2
a b k
+ a 2 [3(v − 1) − 2(u + 2v − 2)]
[Replacing c by k in (i)]
= 3 + 3(a3 + β3 ) − uβ − 2vβ + 2β + u + 2 v
D1 (b − c)(c − k)(k − b) (c − k)(k − b)
x =
∴= = , −2 + α( − v + 1 − 2u)
D (b − c)(c − a)(a − b) (c − a)(a − b)
= 1 + 3(a3 + b3 ) + 2β + u + 2v + α − uβ −2vβ − αv − 2uα
D2 (k − c)(c − a)(a − k) (k − c)(a − k)
y =
= = = 1 + 3(a3 + β3 ) + 2(a2 − a3 ) + b1 − b2 + 2(β2 − b3 )
D (b − c)(c − a)(a − b) (b − c)(a − b)
+a − a − 2αu − 2β v − uβ − vα
D3 (a− b)(b− k)(k − a) (k − a)(b− k)
z =
= = = 1 + (a1 + b1 + a2 + b2 + a3 + b 3 ) − 2αu −2βv − uβ − vα
D (b − c)(c − a)(a − b) (c− a)(b− c)
= RHS [From (i)]
Example 6: Show that
Example 7: Find values of c for which the equations
1 + a1 + b1 a1 + b2 a1 + b3
2x + 3y =
3 ; (c + 2)x + (c + 4)y =c + 6
a2 + b1 1 + a2 + b2 a2 + b3
a3 + b1 a3 + b2 1 + a3 + b3 (c + 2)2 x + (c + 4)2 y = (c + 6)2 are consistent and hence
solve the equation.
3
=1 + ∑ (ai + bi ) + ∑ (ai − a j ) (b j − bi )
=i 1 1 ≤ i< j≤ 3 Sol: Here in this problem first define given equations as
∆ and solve it as ∆ =0 by using the invariance method.
Sol: By putting α = a1 ‒ a2, β = a2 ‒ a3 , then α + β = a1 ‒ a3 The equation will be consistent, if
u = b1 ‒ b2, v = b2 ‒ b3, then u + v = b1 ‒ b3. Using the
2 3 3
invariance property expand the given determinant, and
then comparing it to the R.H.S. of the given problem c+2 c+4 c+6 0
=
2 2 2
we can prove it. (c + 2) (c + 4) (c + 6)
Let Now R.H.S. Applying C3 → C3 − C2 , we get
3
=1 + ∑ (ai + bi ) + ∑ (ai − a j ) (b j − bi ) 2 3 0
=i 1 1 ≤ i< j≤ 3 c+2 c+4 2 0
=
=1 + (a1 + b1 + a2 + b2 + a3 + b3 ) + (a1 − a2 )(b2 − b1 ) (c + 2) 2
(c + 4)2
4(c + 5)
+ (a2 − a3 )(b3 + b2 ) + (a1 − a3 )(b3 − b1 )
Solving, we get c2 + 10c =
0
= 1 + (a1 + b1 + a2 + b2 + a3 + b3 ) − αu − βv − (α + β)(u + v)
or c = 0, –10  … (i)
= 1 + (a1 + b1 + a2 + b2 + a3 + b3 ) − 2αu − 2βv − βu − αv .
If c = 0, the system of equations becomes
 .... (i)
1+α α −1 α  2x + 3y =
3
 ⇒ x = – 3 , y = 3 .… (ii)
Now L.H.S.= β 1+β β −1 2x + 4y 6
=
a3 + b1 a3 + b2 1 + a3 + b3
1 7 . 2 6 | Determinants

If c = –10, then system of equations becomes Example 9: Let λ and α be real. Find the set of all
values of λ and α for which the system of linear
 2x + 3y = 3 1 4 equations λx + (sin α )y + (cos α )z =0
 ⇒ x = − , y =  .… (iii)
 −8x − 6y =− 4 2 3
x + (cos α )y + (sin α )z =
0 −x + (sin α )y − (cos α )z =0 has
16x + 9y = 4 a non-trivial solution. If λ =1 , find all values of α .

Hence the solutions are given by (ii) and (iii). Sol: Here in this problem first define the given equations
as ∆ and as we know that for non-trivial solution ∆ =0 .
Example 8: If (ar, br), r = 1, 2, 3 be the vertices of a For non-trivial solution, condition is ∆ =0 .
triangle, prove that
λ sin α cos α
a2 − a3 b2 − b3 a1 (a2 − a3 ) + b1 (b2 − b3 ) =∆ 1 cos α = sin α 0
∆= a3 − a1 b3 − b1 ) 0 … (i)
a2 (a3 − a1 ) + b2 (b3 − b1 =
−1 sin α − cos α
a1 − a2 b1 − b2 a3 (a1 − a2 ) + b3 (b1 − b2 )
or λ[ − cos2 α − sin2 α] − sin α [ − cos α + sin α]
and hence show that the altitudes of a triangle are
concurrent.
+ cos α [sin α + cos α] = 0
Sol: Using the invariance method we can expand the λ sin2α + cos2α ∴
or = α ∈R ; | λ | ≤ 2
given determinant and using the equations of altitude
If λ =1 , then=
1 sin2α + cos2α
we can prove it
 π 1 π
a2 − a3 b2 − b3 a1 (a2 − a3 ) + b1 (b2 − b3 ) cos  2α − = = cos
 4 2 4
∆= a3 − a1 b3 − b1 a2 (a3 − a1 ) + b2 (b3 − b1 =
) 0
π π π π
a1 − a2 b1 − b2 a3 (a1 − a2 ) + b3 (b1 − b2 ) ⇒ 2α − = 2nπ ± : n ∈ I ⇒ α = nπ ± + : n ∈ I
4 4 8 8
[Applying R1 → R1 + R 2 + R 3 ]

0 0 0 Example 10: For a fixed positive integer n, if


∆= a3 − a1 b3 − b1 a2 (a3 − a1 ) + b2 (b3 − b1 ) n! (n + 1)! (n + 2)!
a1 − a2 b1 − b2 a3 (a1 − a2 ) + b3 (b1 − b2 ) ∆= (n + 1)! (n + 2!) (n + 3)!
(n + 2)! (n + 3)! (n + 4)!
A(a1, b1)
 ∆ 
then show that  − 4  is divisible by n.
3
 (n!) 
F E
Sol: By using the scalar multiple property of
determinants we can take (n!)3 ,(n+1) and (n + 2)
B(a2, b2) D C(a3, b3) common and using the invariance property we can
solve the given problem.
∴ Equation of altitude AD is:
a − a3 1 n+1 (n + 2)(n + 1)
− 2
y − b1 = (x − a1 ) =∆ (n!)3 n+1 (n + 2)(n + 1) (n + 3)(n + 2)(n + 1)
b2 − b3
(n + 2)(n + 1) (n + 3)(n + 2)(n + 1) (n + 4)(n + 3)(n + 2)(n + 1)
or x(a2 − a3 ) + y(b2 − b3 )= a1 (a2 − a3 ) + b1 (b2 − b3 ) … (ii)
Taking (n+1) and (n + 2) (n + 1) common from C2 and
Similarly equation of altitudes BE and CF are C3 respectively, we get
x(a3 − a1 ) + y(b3 − b1 )= a2 (a3 − a1 ) + b2 (b3 − b1 ) .… (iii) =∆ (n!)3 (n + 2)(n + 1)

x(a1 − a2 ) + y(b1 − b2 )= a3 (a1 − a2 ) + b3 (b1 − b2 )  .… (iv) 1 1 1
n+1 n+2 n+3
Altitudes (ii), (iii), (iv) are concurrent, since the
determinant given by L.H.S. of (i) is zero. (n + 2)(n + 1) (n + 3)(n + 2) (n + 4)(n + 3)

[Apply C3 → C3 − C1 and C2 → C2 − C1 then


M a them a ti cs | 17.27

=∆ (n!)3 (n + 1)2 (n + 2) = (n!)3 (n + 1)2 (n + 2) ⋅ 2

1 0 0 =∆ (n!)3 (n2 + 2n + 1)(2n +=


4) (n!)3 (2n3 + 8n2 + 10n + 4)
n+1 1 2  ∆ 
∴  − 4  = 2n3 + 8n2 + 10n ,
(n + 2)(n + 1) 2(n + 2) 4n + 10 3
 (n!) 
= (n!)3 (n + 1)2 (n + 2)[4n + 10 − 4(n + 2)] 2n(n2 + 4n + 5) , which is divisible by n.

JEE Main/Boards

 3 1
Exercise 1 Q.11 Find the adjoint of matrix A =  .
 −5 4 
−1 2 2 x
Q.1 Find x, if = .  1 3
4 8 x −4 Q.12 Find the inverse of matrix   , if possible.
 −6 −18 
1 2 3 1 8
Q.2 It matrix A =   , find [A].
0 1  Q.13 Without expanding, find the value of −4 2 16 .
4 −1 0 −5 3 24
x 0 1
Q.3 Given 2 1 4 , find (i) M23 (ii) C32 .  
Q.14 If a =  2 −1 4  is a singular matrix, find x.
1 0 3 1 2 0
 

Q.4 Area of a triangle with vertices (k, 0), (1, 1) and Q.15 Find the area of the triangle whose vertices are
(0, 3) is 5 sq. units. Find the value(s) of k. (3, 1) (4, 3) and (–5, 4).

Q.5 Find the area of a triangle, whose vertices are (0, 3) Q.16 Find the value of x, if area of triangle
(–1, 4), (2, 6). is 35 square cms with vertices (x, 4), (2, –6) and (5, 4).

a11 a12 a13


Q.17 Show that the following determinant vanishes:
Q.6 Given determinant a21 a22 a23 .
a31 a32 a33 5 15 −25
7 21 30 .
Find the value of a11C21 + a12C22 + a13C33 . 8 24 42

 −1 24  Q.18 Using properties of determinants, prove that :


Q.7 Find the value of p, such that the matrix  
is singular.  4 p
1 a b+c
0.
1 b c+a =
Q.8 Given I2 . Find I2 . Also find 3I2 .
1 c a+b

Q.9 Find the value of x, such that the points (0,2), (1, x),
(3, 1) are collinear. Q.19 If points (2, 0), (0, 5) and (x, y) are collinear, then
x y
show that + = 1.
Q.10 For two given square matrices A and 2 5
B of the same order, such that |A| = 20 and |B| = –20,
find |AB|. Q.20 If for matrix A, |A| = 3 find |5A|, where matrix A is
of order 2 × 2.
1 7 . 2 8 | Determinants

 a11 a12   2 2 1 1 3 2 
Q.21 Given A = a 
a22  , such that |A| = –10. Find
 
Q.32 If A =  −2 1 2 and B =
 
 21 1 1 1  ,
a11C11 + a12C12 .  1 −2 2  2 −3 −1
   
verify that (AB)–1 = B−1 A –1 .
Q.22 Without expanding prove that, the value of
determinant (33-38) Using properties of determinant, prove that
1 a b+c a b c
1 b c + a is zero. Q.33 b c a =(a + b + c) (ab + bc + ca – a2 – b2 –c2)
1 c a+b c a b

= 3abc – a3 – b3 – c3.
Q.23 A is a non-singular matrix of order 3 and |A| = –4.
Find |adj.A|. y+z x y
Q.34 z + x y x =(x + y + z) (x – z)2.
Q.24 Is it possible to find the inverse of a matrix x+y z z

 2 1 5 −bc b2 + bc c2 + bc
  ? Given reasons.
 −1 0 3 Q.35 a2 + ac −ac c2 + ac = (ab+bc+ca)3.
a2 + ab b2 + ab −ab
Q.25 Given a square matrix A of order 3 × 3, such that
|A| = 12, find the value of |A.adj. A|.
(b + c)2 a2 bc
2 2
 2 3 Q.36 (c + a) b ac = (a – b) (b – c) (c – a) (a+ b +c)
Q.26 Compute A −1 for the matrix   and show 2 2
 5 −2  (a
(a + b) c ab 2
+ b2 + c2).
1
that A −1 = A.
19
a b−c c+b
Q.37 a + c b c − a = (a+b+c) (a2+b2+c2).
 1 −2 1
  a−b a+b c
Q.27 Let A=  −2 3 1
 1 1 5
 
a b ax + by
Verify that (i) (adj A)-1 (ii) (A−1)−1=A. Q.38 b c bx + cy =(b2– ac)(ax2+2bxy+cy2)
ax + by bx + cy 0
Q.28 Using matrix method, examine the system of
equations: 2x + 5y = 7, 6x + 15y = 13 for consistency. Q.39 Write the minors and cofactors of the elements of
second row of the following determinant:
a b  1 2 3
Q.29 Find the inverse of matrix A=  1 + bc  and
c  −4 3 6.
show that aA −1 = (a2 + bc + 1)I − aA .  a  2 −7 9

Q.40 Find the quadratic function defined by the equation


 1 tanx 
Q.30 If A =  , f(x) = ax2 + bx + c, if f(0) = 6(2) = 11 and f(–3) = 6, using
 − tanx 1  determinants.
cos2x − sin2x 
show that A′A −1 =  . Q.41 Examine whether the system of equations:
 sin2x cos2x  2x – y = 5, 4x – 2y = 10 is consistent or inconsistent.

 2 0 −1  Q.42 Verify, whether the system of equations: 3x – y – 2z


 
Q.31 If A =  5 1 0  , prove that A −1 = A2 − 6A + 11 I . = 2, 2y – z = –1, 3x – 5y = 3 is consistent or inconsistent.
 0 1 3
 
M a them a ti cs | 17.29

Without expanding the determinants, show that 2 −3


Q.52 Show that A =   satisfies the equation
2 3 4 
1 a bc 1 a a x2 – 6x + 17 = 0.
Q.43 1 b ca = 1 b b2 .
Hence find A–1.
1 c ab 1 c c2

a a2 bc 1 a2 a3 2 4  0 2  1 6 
2 2 3 Q.53 Find matrix A if,  A = .
Q.44 b b ca = 1 b b 1 3 1 3 3 −1
2 2
c c ab 1 c c3
5 0 4  1 3 3 
  −1  
0 p −q p −r Q.54 Given A 2 3 3  and B 1 4 3  . Compute
Q.45 q − p 0 q−r =0 (AB) .
–1
1 2 1  1 3 4 
   
r −p r −q 0
 2 −1 1
 
Q.55 For the matrix A =
 −1 2 −1 , verify that
x2 0 3
 1 −1 2
Q.46 Solve for x, x 1 −4 =
11 .  
1 2 0 A3 − 6A2 + 9A − 4I =
O , hence find A −1 .

3 2 2 1 0
Q.47 If   , verify that A − 4A − I =O =  
2 1 0 1 Exercise 2
0 0
and O =  −1
 and hence find A . Single Correct Choice Type
 0 0 
Q.1 If a, b, c are all different from zero and
1 / a a2 bc
1+a 1 1
Q.48 Evaluate : 1 / b b2 ca .
1 1+b 1 = 0, then the value of a−1 + b−1 + c−1
1/c c2 ab 1 1 1+c

is:
Q.49 Show that {–(a + b + c)} is root of the following
equation: (A) abc (B) a–1 b–1 c–1

x+a b c (C) –a – b – c (D) –1


b x+c a =0.
a a3 a4 − 1
c a x +b
Q.2 If a, b, c are all different and b b3 b 4 − 1 =
0,
then 3 4
c c c −1
Q.50 Using properties of determinants, prove
(A) abc (ab + bc + ca) = a + b + c
x+4 x x
that : x x+4 x =16(3x + 4) . (B) (a + b + c) (ab + bc + ca) = abc
x x x+4 (C) abc (a + b + c) = ab + bc + ca
(D) None of these
Q.51 Using properties of determinants, prove that:
b + c a−b a Q.3 If (sin–1 x + sin–1 w) (sin–1 y + sin–1 z)=p2, then
N1 N2
c + a b − c b= 3abc − a3 − b3 − c3 . x y
a+b c−a c N3 N4
; (N1 ,N2 ,N3 ,N4 ∈ N)
z w
1 7 . 3 0 | Determinants

(A) Has a maximum value 2. Q.9 The following system of equations 3x – 7y + 5z = 3;


3x + y + 5z = 7 and 2x + 3y + 5z = 5 are
(B) Has a minimum value 2.
(A) Consistent with trivial solution
(C) In independent of N1 ,N2 ,N3 ,N4
(B) Consistent with unique non-trivial solution
(D) None of these
(C) Consistent with infinite solution
2 n 2 2n
Q.4 If (1 + x + x ) = a0 + a1 x + a2 x + ...... + a2n x then (D) Inconsistent with no solution
an−3 an−1 an+1
an−6 an−3 an+3 is Q.10 The system of equations (sin θ)x + 2z =0,
an−14 an−7 an+ 7 (cos θ)x + (sin θ)y =,
0 (cos θ)y + 2z =a has

(A) Non unique solution


(A) 1 (B) 2 (C) 0 (D) –1
(B) A unique solution which is a function of a and θ
Q.5 The absolute value of the determinant (C) A unique solution which is independent of a and θ
−1 2 1 (D) A unique solution which is independent of θ only
3 + 2 2 2 + 2 2 1 is
3−2 2 2−2 2 1 Q.11 The equation

(A) 16 2 (B) 8 2 (C) 8 (D) None of these (1 + x)2 (1 − x)2 −(2 + x2 )


2x + 1 3x 1 − 5x +
a b c bc − a2 ac − b2 ab − c2 x +1 2x 2 − 3x
ac b2
Q.6 D1 = b c a , D2 =− ab − c2 bc − a2 ,
c a b (1 + x)2 2x + 1 x +1
ab − c2 bc − a2 ac − b2
3
(1 − x) 3x 2x = 0
a2 + b2 + c2 ab + bc + ca ab + bc + ca 1 − 2x 3x − 2 2x − 3
D3 = ab + bc + ca a2 + b2 + c2 ab + bc + ca then
ab + bc + ca ab + bc + ca a2 + b2 + c2 (A) Has no real solution
(B) Has 4 real solutions
(A) D1 ≤ 0 , if a+b+c>0 (B) D22 = D3
(C) Has two real and two non-real solutions
2
(C) D=
1 D=
2 D12
D3 (D) D2 ≠ D3 =
(D) Has infinite number of solutions, real or non-real

1 1 1 1 1 1 Q.12 The system of equation :


Q.7 a b c = a b c , where a, b, c are distinct
2x cos2 θ + y sin2θ − 2sin θ = 0 ;
bc ca ab a3 b3 c3
x sin2θ + 2y sin2 θ = −2cos θ ;
positive reals, then abc is always less than
x sin θ − y cos θ = 0 , for all values of θ , can
1 1 1 1
(A) (B) (C) (D)
243 729 27 81 (A) Have a unique nontrivial solution
(B) Not have a solution
Q.8 The value of ‘a’ for which the system of equations,
(C) Have infinite solutions
(a + 1)3 x + (a + 2)3 y = (a + 3)3 ,
(D) Have a trivial solution
logx xyz logx y logx z
logy xyz 1 logy z and x + y = 1 are consistent is Q.13 If x, y, z are not all simultaneously equal to zero,
satisfying the system of equations (sin3θ)x − y + z = 0;
logz xyz logz y 1
(cos2θ)x + 4y + 3z = 0; 2x + 7y + 7z = 0, then the
(A) –2 (B) 1 (C) 0 (D) None number of principal values of θ is
(A) 2 (B) 4 (C) 5 (D) 6
M a them a ti cs | 17.31

Q.14 For a non-zero, real a, b and c Q.6 The number of values of k for which the system of
2 2 equations (k + 1)x + 8y = 4k, kx+ (k + 3)y = 3k – 1 has
a +b
c c infinite solutions.
c
b2 + c2
a a = α abc
a Assertion Reasoning Type
2 2
c +a (A) Both assertion and reason are true and reason is the
b b
b
correct explanation of Assertion.
then the value of α is
(B) Both assertion and reason are true and reason is not
(A) –4 (B) 0 (C) 2 (D) 4
the correct explanation of assertion.
Q.15 Number of value of ‘a’ for which the system of (C) Assertion is true but reason is false
equations, a2 x + (2 − a)y = 4 + a2 ; ax + (2a − 1)y =a5 − 2
(D) Assertion is false but reason is true.
possess no solution is
(A) 0 (B) 1 (C) 2 (D) Infinite Q.7 Consider the system of equations x – 2y + 3z = –1,
x – 3y + 4z = 1 and –x + y – 2z = k

Previous Years’ Questions Statement-I: The system of equation has no solution


for k ≠ 3 . and
Q.1 The determinant
1 3 −1
xp + y x y Statement-II: The determinant −1 −2 k ≠ 0 , for
yp + z y z 0 
= (1997) 1 4 1
0 xp + y yp + z k ≠ 0.  (2008)
(A) x, y, z are in AP (B) x, y, z are in GP
Q.8 Given, x = cy + bz, y = az + cx, z = bx + ay, where
(C) x, y, z are in HP (D) xy, yz, zx are in AP
x, y, z are not all zero, prove that a2 + b2 + c2 + 2ab =1
 (1978)
1 x x +1
Q.2 If f(x)
= 2x x(x − 1) (x + 1)x , Q.9 If α be a repeated root of a quadratic equation
3x(x − 1) x(x − 1)(x − 2) (x + 1)x(x − 1) f(x) = 0 and A(x), B(x) and C(x) be polynomials of
degree 3, 4 and 5 respectively, then show that
then f(100) is equal to  (1999)
(A) 0 (B) 1 (C) 100 (D) –100 A(x) B(x) C(x)
A(α ) B(α ) C(α ) is divisible by f(x), where prime
Q.3 If the system of equations x – ky – z = 0, A ′(α ) B′(α ) C′(α )
kx – y – z = 0, x + y – z = 0 has a non-zero solution, then denotes the derivatives.  (1984)
possible values of k are  (2000)
(A) –1, 2 (B) 1, 2 (C) 0, 1 (D) –1, 1 a b c 
 
Q.10 If matrix A = b c a  , where a, b, c are real
Q.4 The number of distinct real roots of  (2001)  c a b
 
sinx cos x cos x positive number, abc = 1 and A T A = I , then find the
π π
cos x sinx cos x = 0 in the interval − ≤ x ≤ is value of a3 + b3 + c3 .  (2003)
4 4
cos x cos x sinx

(A) 0 (B)2 (C) 1 (D) 3 Q.11 The number of values of k , for which the system
of equations: (k + 1 ) x + 8y =
4k kx + (k + 3 ) = 3k − 1 Has
Q.5 If the system of equations x + ay = 0, az + y = 0 and no solution, is:  (2013)
ax + z = 0 has infinite solutions, then the value of a is
(A) Infinite (B) 1 (C) 2 (D) 3
 (2002)
(A) –1 (B) 1
Q.12 If α , β ≠ 0 , and f (n) = α + β
n n
(C) 0 (D) No real values
1 7 . 3 2 | Determinants

(B) Is a singleton
3 1 + f (1 ) 1 + f ( 2 )
1 + g (1 ) 1 + f ( 2 ) 1 + f ( 3= (C) Contains two elements
) K (1 − α ) (1 − β )( α − β ) ,
2 2

1 + f ( 2 ) 1 + g (3) 1 + f ( 4 ) (D) Contains more than two elements

then K is equal to:  (2014) Q.14 The system of linear equations  (2016)
α x + λy − z = 0
(A) αβ (B) (C) 1 (D) -1
β
λx − y − z =0
Q.13 The set of all values of λ for which the system of x + y − λz = 0
linear equations:
has a non-trivial solution for:
2x1 − 2x2 + x3 =
λx1
(A) Infinitely many values of λ
2x1 − 3x2 + 2x3 =
λx 2
(B) Exactly one value of λ
− x1 + 2x2 =λx3
(C) Exactly two values of λ
Has a non − trivial solution.  (2015)
(D) Exactly three values of λ
(A) Is an empty set

JEE Advanced/Boards

Exercise 1 Q.5 Given x = cy + bz; y = az + cx; z = bx + ay where x,


y, z are not all zero, prove that a2 + b2 + c2 + 2abc =
1.
Q.1 Solve the following using Cramer’s rule and state
whether consistent or not. x y z
Q.6 Given a = ; b= ; c= where x, y, z
(a) x + 2y + z =1 (b) x + y + z − 6 =0 y−z z−x x−y
3x + y + z =6 2x + y − z − 1 =0 are not all zero, prove that 1 + ab + bc + ca = 0.
x + 2y = 0 x + y − 2z + 3 =0
Q.7 If sinq ≠ cosq and x, y, z satisfy the equations
(c) 7x − 7y + 5z = 3 x cosp − y sinp +=
z cosq + 1
3x + y + 5z =7
x sinp + y cosp + z = 1 − sinq
2x + 3y + 3z =5
x cos(p + q) − y sin(p + q) + z =
2
Then find the value of x2 + y 2 + z 2 .

Q.2 For what value of K do the following system of Q.8 Investigate for what values of λ , µ the simultaneous
equations possess a non-trivial (i.e. not all zero) solution equations x + y + z = 6; x+ 2y + 3z = 10 and
over the set of rational Q? x + Ky + 3z = 0, 3x + Ky – 2z x + 2y + λz = µ have;
= 0, 2x + 3y – 4z = 0. For that value of K, find all the
(a) A unique solution
solutions of the system.
(b) An infinite number of solutions
Q.3 The system of equations αx + y + z = α − 1 (c) No solution
x + αy + z = α − 1 ; x + y + αz = α − 1 has no solution.
Find α . Q.9 For what values of p, the equations: x + y + z = 1;
x + 2y + 4z = p and x + 4y + 10z = p2 have a solution?
Q.4 If the equations a(y + z) = x, b(z + x) = y, c(x + y) Solve them completely in each case.
= z have non-trivial solutions, then find the value of
1 1 1 Q.10 Solve the equations : Kx + 2y – 2z = 1; 4x + 2Ky
+ + .
1+a 1+b 1+c – z = 2; 6x + 6y + Kz = 3 considering specially the case
M a them a ti cs | 17.33

when K = 2. x+y+z = 4
2x + y + 3z = 6
Q.11 (a) Let a, b, c, d are distinct numbers to be chosen x + 2y + az = b
from the set {1, 2, 3, 4, 5}. If the least possible positive
Let L: denotes number of ordered pairs (a, b) so that the
ax + by =
1
solution for x to the system of equation  system of equations has unique solution,
cx + dy =
2
p
can be expressed in the form where p and q are M: denotes number of ordered pairs (a, b) so that the
q
system of equations has no solution and
relatively prime, then find the value of (p + q).
N: denotes number of ordered pairs (a, b) so that the
(b) Find the sum of all positive integral values of a for system of equations has infinite solutions. Find (L + M – N).
which every solution to the system of equations x + ay
= 3 and ax + 4y = 6 satisfy the inequalities x > 1, y > 0. Q.16 (a) Prove that the value of the determinant
2
Q.12 If the following system of equations (a – t)x + by + −7 5 + 3i − 4i
3
cz = 0, bx + (c – t)y + az = 0 and cx + ay + (b – t)z = 5 − 3i 8 4 + 5i is real.
0 has non-trivial solutions for different values of t, then
2
show that we can express product of these values of + 4i 4 − 5i 9
3
t in the form of determinant.
(b) On which one of the parameter out of a, p, d or x value
Q.13 Show that the system of equations3x – y + 4z = 3,
x + 2y – 3z = –2 and 6x + 5y + λz = –3 has atleast one 1 a a2
solution for any real number λ . Find the set of solutions of the determinant cos(p − d)x cospx cos(p + d)x
of λ = −5 . sin(p − d)x sinpx sin(p + d)x
does not depend.
Q.14 Solve the system of equations:
x3 + 1 x 2 x
0
z + ay + a2 x + a3 =
 3
(c) If y + 1 y 2
y =
0 and x, y, z are all different then,
z + by + b2 x + b3 =
0 3 2
 z +1 z z
z + cy + c2 x + c3 =
0

prove that xyz = –1.
Q.15 (a) Consider the system of equations
αx − y + z =α a2 + 2a 2a + 1 1
x − αy + z =1 Q.17 Prove that (a) 2a + 1 a + 2 1 = (a − 1)3
x − y + αz =1 3 3 1

If L, M and N denotes the number of integral values 1 1 1


of α in interval [–10, 10] for which the system of the (b) x y z =[(x − y)(y − z)(z − x)(x + y + z)]
equations has unique solution, no solution and infinite
x3 y3 z3
solutions respectively, then find the value of (L – M + N).
(b) If the system of equations is −3
x 1
2x + 3y − z = 0 2
3x + 2y + kz = 0 Q.18 (a) Let f(x) = 2 2 1 Find the minimum
4x + y + z = 0
1 1
0
x −1 2
have a set of non-zero integral solutions then, find the
smallest positive value of z. value of f(x) (given x > 1).

(c) Given a, b ∈ {0, 1, 2, 3, 4,……, 9, 10}.


(b) If a2 + b2 + c2 + ab + bc + ca ≤ 0 ∀ a, b, c ∈ R, then
Consider the system of equations find the value of the determinant
1 7 . 3 4 | Determinants

(a + b + 2)2 a2 + b2 1 x − 2 2x − 3 3x − 4
1 (b + c + 2)2 b2 + c2 (b) x − 4 2x − 9 3x − 16 =0
x − 8 2x − 27 3x − 64
c2 + a2 1 (c + a + 2)2

a b c Q.25 If a + b + c = 0, solve for x :


Q.19 If D = c a b and a−x c b
b c a c b−x a =0
b+c c+a a+b b a c−x
D′ =+
a b b + c c + a then prove that D’ = 2D.
c+a a+b b+c Q.26 Let a, b, c are the solutions of the cubic
x3 − 5x2 + 3x − 1 =0 , then find the value of the
Q.20 Prove that a b c
determinant a − b b − c c − a
1 + a2 − b2 2ab −2b
2 2 b+c c+a a+b
2ab 1−a +b 2a = (1 + a2 + b2 )3
2b −2a 1 − a2 − b2
a2 + λ ab ac
Q.27 Show that, ab 2
b λ bc is divisible by λ2
Q.21 Let
ac bc c2 + λ
sinx sin(x + h) sin(x + 2h) and find the other factor.
f(x) =
sin(x + 2h) sinx sin(x + h) .
sin(x + h) sin(x + 2h) sinx Q.28 Prove that

f(x) a2 b2 c2 a2 b2 c2
If Lim has the value equation to k(sin3x + sin x) 3
h→0 h2 (a + 1)2 (b + 1)2 (c + 1)2 =
4 a b c
find k ∈ N. (a − 1)2 (b − 1)2 (c − 1)2 1 1 1

Q.22 Prove that Q.29 In a ∆ABC , determine condition under which


(β + γ − α − δ)4 (β + γ − α − δ)2 1 A B C
cos cot cot
2 2 2
( γ + α + β − δ )4 ( γ + α − β − δ)2 1
B C C A A B
( α + β − γ − δ )4 (α + β − γ − δ)2 1 tan + tan tan + tan tan + tan =0
2 2 2 2 2 2
1 1 1
= −64(α − β)(α − γ )(α − δ)(β − γ )(β − δ)( γ − δ)

Q.23 If a, b and c are the roots of the cubic x3 − 3x2 + 2 =0 Exercise 2


then find the value of the determinant.
Single Correct Choice Type
(b + c)2 a2 a2
b2 (c + a)2 b2 Q.1 Let m be a positive integer &
c2 c2 (a + b)2 m
2r − 1 Cr 1
2 m
Dr = m −1 2 m+1 (0 ≤ t ≤ m) ,
Q.24 Solve for x 2 2 2 2
sin (m ) sin (m) sin (m + 1)
x + 2 2x + 3 3x + 4 m

(a) 2x + 3 3x + 4 4x + 5 = 0
then the value of ∑ Dr is given by
r =0
3x + 5 5x + 8 10x + 17 (A) 0 2
(B) m − 1 (C) 2m (D) 2m sin2 (2m )
M a them a ti cs | 17.35

Q.2 If α,β and γ are real numbers, then then the maximum value of f(x) is equal to

1 cos(β − α ) cos( γ − α ) (A) 2 (B) 4 (C) 6 (D) 8


D cos(α − β) cos(β − γ ) cos( γ −=
= β)
cos(α − γ ) 1 1 Q.8 If px 4 + qx3 + rx 2 + sx + t

(A) –1 (B) cos α cos β cos γ x2 + 3x x − 1 x + 3


x + 1 2 − x x − 3 then t is equal to
(C) cos α + cos β + cos γ (D) Zero
x − 3 x + 4 3x

Q.3 If a, b and c are non-zero real numbers, then (A) 33 (B) 0 (C) 21 (D) None
2 2
b c bc b + c
=D c a 2 2
ca
= c+a 0 a2 + 1 ab ac
2
ab 2 2
ab a + b Q.9 If D = ba b +1 bc , then D is equal to
2
ca cb c +1
(A) abc (B) a2b2c2
(C) bc + ca + ab (D) Zero (A) 1 + a2 + b2 + c2 (B) a2 + b2 + c2

(C) (a + b + c)2 (D) None of these


mx mx − p mx + p
Q.4 If f ′(x)
= n n+p n−p , Q.10 If α + β + γ = π , then the value of
mx + 2n mx + 2n + p mx + 2n − p
sin(α + β + γ ) sin β cos γ
then y=f(x) represents − sin β 0 tan α is
cos(α + β) − tan α 0
(A) A straight line parallel to x-axis
(B) A straight line parallel to y-axis (A) 0 (B) 1
(C) Parabola (C) 2 (D) 2.sin β. cos γ. tan α
(D) A straight line with negative slope
Q.11 If the entries of 3 × 3 determinant are zero or one
then the value of the determinant
x − 1 (x − 1)2 x3
(A) Cannot be 3 (B) Cannot be 2
Q.5 If D(x) =
x −1 x2 (x + 1)3 ,
(C) Can be –2 (D) Is essentially zero
x (x + 1)2 (x + 1)3

then the coefficient of x in D(x) is Q.12 In a third order determinant, each element of
the first column consists of sum of two terms, each
(A) 5 (B) –2 (C) 6 (D) 0 element of the second column consists of sum of three
terms and each element of the third column consists of
Q.6 The number of integral solutions of |D|=8, where sum of four terms. Then it can be decomposed into n
determinants, where n has the value
y+z z y
=D z z+x x is (A) 1 (B) 9 (C) 16 (D) 24
y x x+y
Q.13 If the system of equations x + 2y + 3z = 4, x +
(A) 3 (B) 8 (C) 16 (D) 24 py + 2z =3, µx + 4 y + z = 3 has an infinite number of
solutions, then
1 + sin2 x cos2 x 4 sin2x (A) p = 2, µ =3 (B) p = 2, µ =4
2 2
Q.7
= Let f(x) sin x 1 + cos x 4 sin2x , (C) 3p = 2µ (D) None of these
2 2
sin x cos x 1 + 4 sin2x
1 7 . 3 6 | Determinants

Q.14 Number of triplets of a, b and c for which the Q.20 The set of equations λx − y + (cos θ)z =0 ;
system of equations, ax – by = 2a – b and (c + 1)x + cy 3x + y + 2z =;0 (cos θ)x + y + 2z =,
0 0 ≤ θ < 2π , has
= 10 – a + 3b has infinitely many solutions and x = 1, nontrivial solution(s)
y = 3 is one of the solutions, is
(A) For no value of λ and θ
(A) Exactly one (B) Exactly two
(B) For all values of λ and θ
(C) Exactly three (D) Infinitely many
(C) For all values of λ and only two values of θ

Q.15 If the system of equations ax + y + z = 0, (D) For only one value of λ and all values of θ
x + by + z = 0 & x + y + cz = 0 (a , b, c ≠ 1) has a non-
1 1 1 Multiple Correct Choice Type
trival solution , then the value of + + is
1−a 1−b 1−c
(A) –1 (B) 0 (C) 1 (D) None of these Q.21 The determinant
cos(x − y) cos(y − z) cos(z − x)
Q.16 The determinant cos(x + y) cos(y + z) cos(z + x) =
cos(θ + φ) − sin(θ + φ) cos2φ sin(x + y) sin(y + z) sin(z + x)
sin θ cos θ sin φ is (A) 2sin(x − y)sin(y − z)sin(z − x)
− cos θ sin θ cos φ
(B) −2sin(x − y)sin(y − z)sin(z − x)
(A) 0 (C) 2cos(x − y)cos(y − z)cos(z − x)
(B) Independent of θ (D) −2cos(x − y)cos(y − z)cos(z − x)
(C) Independent of φ
π π
(D) Independent of θ and φ both Q.22 The value of θ lying between − and and
4 2
π
−0 ≤ A ≤ and satisfying the equation
Q.17 The values θ , λ for which the following equations 2
x sin θ − y cos θ + (λ + 1)z = 0 ; x cos θ + y sin θ − λz = 0 ; 1 + sin2 A cos2 A 2sin 4θ
λx + (λ + 1)y + z cos θ =0 are consistent with infinite 2
sin A 1 + cos A 2
2sin 4θ are
solution, are 2 2
sin A cos A 1 + 2sin 4θ
(A) θ = nπ, λ ∈, λ ∈ R − {0}
π π 3π
(B) θ= 2nπ, λ is any rational number (A) A = , θ = − (B) A = = θ
4 8 8
θ (2n + 1)π, λ ∈ R + , n ∈ I
(C) = π π π 3π
(C) A = 9 θ = − (D) A = = θ =
π 5 8 6 8
(D) =
θ (2n + 1) , λ ∈ R, n ∈ I
2
1 a a2
Q.18 If the system of equations, a2 x − ay =1 − a and Q.23 If 1 x x2 = 0
bx + (3 − 2b)y =3 + a possess a unique solution x = 1, b2 ab a2
y = 1 then 1 a
(A) x = a (B) x = b (C) x = (D) x =
(A) a = 1; b = –1 (B) a = –1, b = 1 a b
(C) a = 0, b = 0 (D) None of these
a b aα + b
Q.24 The determinant b c bα + c
n+ 2 n+ 3 n+ 4
Cn Cn+1 Cn+ 2 aα + b bα + c 0
n+ 3 n+ 4 n+5
Q.19 Let D = Cn+1 Cn+ 2 Cn+3 and n ∈ N is equal to zero, if
n+ 4 n+5 n+ 6
Cn+ 2 Cn+3 Cn+ 6 (A) a, b, c are in AP
then the value of D is equal to (B) a, b, c are in GP
(A) –1 (B) 0 (C) 1 (C) α is a root of the equation ax2 + bx + c =0
(D) (n + 2) (n + 3) (n + 4) (n + 5) (n + 6) (D) (x − α ) is a factor of ax2 + 2bx + c
M a them a ti cs | 17.37

Q.25 The set of equations x − y + 3z =,


2 2x − y + z =,
4 (A) D is independent of x (B) D is independent of y
x − 2y + αz =3 has (C) D is independent of z (D) D is dependent of x, y, z
(A) Unique solution only for α =0
(B) Unique solution for α ≠ 8 Previous Years’ Questions
(C) Infinite number of solution of α =8
(D) No solution for α =8 Q.1 The parameter, on which the value of the
determinant
Q.26 Which of the following determinant(s) vanish(es)? 1 a a2
cos(p − d)x cospx cos(p + d)x does not depend
1 1
+ 1 ab sin(p − d)x sinpx sin(p + d)x
1 bc bc(b + c) a b
1 1 upon, is  (1997)
(A) 1 ca ca(c + a) (B) 1 bc +
b c (A) a (B) p (C) d (D) x
1 ab ab(a + b)
1 1
1 ca +
c a
Q.2 Let λ and α be real. Find the set of all values
0 a−b a−c logx xyz logx y logx z of λ for which the system of linear equations
λx + (sin α )y + (cos α )z =0
(C) b − a 0 b−c (D) logy xyz 1 logy z
c −a c −b 0 logz xyz logz y 1 x + (cos α )y + (sin α )z =and
0

−x + (sin α )y − (cos α )z =0 has a non-trivial solution.
Q.27 If the system of equation a2 x − by = a2 − b For λ =1 , find all value of α .  (1993)
and bx − b2 y =+2 4b possess an infinite number of
solutions then the possible values of ‘a’ and ‘b’ are Q.3 Let a, b, c be real numbers with a2 + b2 + c2 =
1.
(A) a = 1, b = –1 (B) a = 1, b = –2 Show that the equation
(C) a = –1, b = –1 (D) a = –1, b = –2 ax − by − c bx + ay cx + a
bx + ay −ax + by − c cy + b 0
=
Q.28 If p, q, r, s are in A.P, and cx + a cy + b −ax − by + c

p + sinx q + sinx p − r + sinx represents a straight line.


f(x) = q + sinx r + sinx −1 + sinx such that
r + sinx s + sinx s − q + sinx Q.4 For what value of k does the following system of
2
equations possess a non-trivial solution over the set of
rationals x + y – 2z = 0; 2x – 3y + z = 0 and x – 5y +
∫ f(x)dx = −4 , then the common difference of the A.P.
4z = k. Find all the solution.  (1979)
0
can be
Q.5 For what value of m does the system of equations
(A) –1 (B) ½ (C) 1 (D) None of these
3x + my = m and 2x – 5y = 20 has a solution satisfying
the conditions x > 0, y > 0. (1979)
Q.29 If the system of equations x+ y – 3 =0, (1 + K)x +
(2 + K)y – 8 = 0 and x – (1 + K) y + (2 + K) are consistent
Q.6 Prove that for all values of θ
then the value of K is
(A) 1 (B) 3/5 (C) –5/3 (D) 2
sin θ cos θ sin2θ

1 1 (x + y)
 2π   2π   4π 
− sin  θ +  cos  θ +  sin  2θ +  = 0  (2000)
z z  3   3   3 
z2
(y + z) 1 1  2π   2π   4π 
Q.30 If D = − then sin  θ −  cos  θ −  sin  2θ − 
x 2 x x  3   3   3 
y(y + z) x + 2y + z y(x + y)
− − −
2
x z xz xz 2
1 7 . 3 8 | Determinants

Q.7 The total number of ways in which 5 balls of Q.9 The total number of distinct x ∈ R for
different colours can be distributed among 5 persons
x x2 1 + x3
so that each person gets at least one ball is  (2012)
which 2x 4x2 1 + 8x3 =
10 is  (2016)
(A) 75 (B) 150 (C) 120 (D) 243 2 3
3x 9x 1 = 27x

Q.8 Which of the following values of α satisfy the


equation

(1 + α ) (1 + 2α ) (1 + 3α )
2 2 2

( 2 + α ) ( 2 + 2α ) ( 2 + 3α )
2 2 2
−648α ?
= (2015)

(3 + α ) (3 + 2α ) (3 + 3α )
2 2 2


(A) −4 (B) 9 (C) −9 (D) 4

PlancEssential Questions
JEE Main/Boards JEE Advanced/Boards
Exercise 1 Exercise 1
Q.15 Q.29 Q.38 Q.1 Q.3 Q.8
Q.48 Q.52 Q.54 Q.11 Q.15 Q.23
Q.55
Exercise 2
Exercise 2 Q.1 Q.11 Q.14
Q.1 Q.6 Q.13 Q.20 Q.25 Q.30

Previous Years’ Questions Previous Years’ Questions


Q.2 Q.6 Q.7
Q.2 Q.5 Q.7
Q.10
Q.10
M a them a ti cs | 17.39

Answer Key

JEE Main/Boards
Exercise 1

Single Correct Choice Type

7 13 5
Q.1 x = ±2 2 Q.2 1 Q.3 (i) 1 (ii) –16 Q.4 − , Q.5 sq. units
2 2 2
5
Q.6 0 Q.7 –96 Q.8 1; 9 Q.9 Q.10 –400
3
 4 −1 5 19
Q.11   Q.12 Not possible Q.13 0 Q.14 x = Q.15 sq. units
5 3  8 2

Q.16 -2 Q.20 75 Q.21 –10 Q.23 16 Q.24 No

−1 1 2 3 
Q.25 1728 Q.26 A =   Q.28 Inconsistent
19 5 −2 
 1 + bc 
−b 
Q.29  a Q.39 M21=39, M22=3, M23=−11, C21=−39, C22=3, C23=11
 
 −c a 

1 2 3
Q.40 f(x) = x + x+6 Q.41 Consistent Q.42 Inconsistent
2 2

7  −1 2 
Q.46 x = − or 1 Q.47 A −1 =   Q.48 0
4  2 −3

 3 −19 12 
1  4 3 1  49 −18  −1  
Q.52 A −1 Q.53  Q.54 4 −18 5 
=   
4  −23 10  11 
17  −3 2   4 −29 27 
 
 3 1 −1
1 
Q. 55  1 3 1 
4
 −1 1 3
 

Exercise 2

Single Correct Choice Type

Q.1 D Q.2 A Q.3 A Q.4 C Q.5 A Q.6 C

Q.7 C Q.8 A Q.9 B Q.10 B Q.11 D Q.12 B

Q.13 C Q.14 D Q.15 C


1 7 . 4 0 | Determinants

Previous Years’ Questions


Q.1 B Q.2 A Q.3 D Q.4 C Q.5 A Q.6 1

Q.7 A Q.10 4 Q.11 B Q.12 C Q.13 C Q.14 D

JEE Advanced/Boards

Exercise 1

Q.1 (a) x = 1, y = 2, z = 3; consistent (b) x = 2, y = –1, z = 1; consistent (c) Inconsistent

33 15
Q.2 K = ,x:y:z=
− : 1 : −3 Q.3 –2 Q.4 2 Q.7 2
2 2

Q.8 (a) λ ≠ 3 (b) λ =3 , µ =10 (c) λ= 3, µ ≠ 10

Q.9 x = 1 + 2K, y = –3K, z = K, when p = 1; x = 2K, y = 1 – 3K, z = K when p = 2; where K ∈ R

x y z 1 1 − 2λ
Q.10 If K ≠ 2, = = = , If K = 2, then x =λ, y = and z = 0 where λ ∈ R
2(K + 6) 2K + 3 6(K − 2) 2(K 2 + 2K + 15) 2

a b c
Q.11 (a) 19 (b) 4 Q.12 b c a
c a b

4 9 4 − 5K 13K − 9
Q.13 If λ ≠ −5 , then x = ; y = − and z = 0; If λ =5=
then x = ;y and z = K where K ∈ R .
7 7 7 7
Q.14 x = –(a + b + c), y = ab + bc + ca, z = – abc

Q.15 (a) 21 (b) 5 (c) 119 Q.16 (b) p Q.18 (a) 4 (b) 65

Q.21 3 Q.23 –108 Q.24 (a) x = –1 or x = –2; (b) x = 4

3 2
Q.25 X = 0 or x ± (a + b2 + c2 ) Q.26 80
2

Q.27 λ2 (a2 + b2 + c2 + λ ) Q.29 Triangle ABC is isosceles.

Exercise 2

Single Correct Choice Type

Q.1 A Q.2 D Q.3 A Q.4 A Q.5 A Q.6 D

Q.7 C Q.8 C Q.9 A Q.10 A Q.11 C Q.12 D

Q.13 D Q.14 B Q.15 C Q.16 B Q.17 D Q.18 A

Q.19 A Q.20 A
M a them a ti cs | 17.41

Multiple Correct Choice Type

Q.21 A,D Q.22 A, B, C, D Q.23 A, D Q.24 B, D Q.25 B, D Q.26 A, B, C, D

Q.27 A, B, C, D Q.28 A, C Q.29 A, C Q.30 A, B, C

Previous Years’ Questions


Q.1 B Q.2 Zero Q.4 k = 0, the given system has infinitely many solutions
15
Q.5 m < − or m > 30 Q.7 B Q.8 B C Q.9 2
2

Solutions

JEE Main/Boards ⇒ |–2k + 3| = 10

⇒ –2k + 3 = 10 or 2k – 3 = 10
Exercise 1 7 13
⇒k= – or k =
2 2
–1 2 2 x
Sol 1: =
4 8 x –4 Sol 5: Vertices of triangle (0, 3) (–1, 4) (2, 6)

–8 – 8 = 2(–4) –x2 = – 8 – x2 1 0 3
1 1
Area = 1 −1 4 =  –1(6) − 8  + 3[2 + 1]
⇒ x = ± 8 = ±2 2 2 2
1 2 6
1 2 1 1 5
Sol 2: A =   , |A| = 1[1] – 2(0) = 1 = –14 + 9 = 5 = Sq. Unit
0 1  2 2 2

4 −1 0 a11 a12 a13


Sol 3: 2 1 4 , Sol 6: D ⇒ a21 a22 a23
1 0 3 a31 a32 a33

4 −1 a11C21 + a12C22 + a13C23


(i) M23 = = 0 – (–1) = 1
1 0
a13 a12 a11 a13 a a
= a11 + a12 + a13 11 12 (–1)2+3
C23 = (–1) 2+3
= –1 a33 a32 a31 a33 a31 a32

4 0 = a11[a13a32 – a33a12] + a12[a11a33 – a31a13] – a13[a11a32 – a31a12]


(ii) C32 = (–1)2+3 = = –16
2 4
=0
It can be directly said as it is a property
Sol 4: Area of triangle, [(k, 0), (1, 1), (0, 3)] = 5 unit2

1 k 0 –1 24
1 1 Sol 7: = A (assume)
⇒ 1 1 1 = 1[3 – 0] + k(1 − 3) = 5 4 P
2 2
1 0 3
1 7 . 4 2 | Determinants

–1 24 So |a| = 0
|A| = = –P –4(24) = – (96 + P) = 0
4 P ⇒ x[–8] + 1{2(2) –(–1)(1)}] = –8x + 4 + 1 = 0

⇒ P = –96 5
⇒ 8x = 5 → x =
8

1 0  1 0  Sol 15: Vertices → (3, 1) (4, 3) and (–5, 4)


Sol 8: I2 =   , |I2| =   =1
0 1  0 1 
1 3 1
1
1 0  3 0  Area = 1 4 3
3I2 = 3   =   , 2
0 1  0 3  1 −5 4
1
3 0  = [16 + 15 + 3 [3 – 4] + 1 [–5 – 4]]
|3I2| =   =9 2
0 3  1 19
= [31 – 3 – 9] = sq. unit
2 2
Sol 9: (0, 2), (1, x) and (3, 1) points are collinear
Sol 16: Vertices (x, y) (2, –6), (5, 4)
1 0 2
So 1 1 x = 0 1 x 4
1 1
1 3 1 Area = 1 2 −6 = [8 + 30 + x[–6 – 4] + 4[5 – 2]]
2 2
1 5 4
⇒ [1 – 3x] + 2[3 – 1] = 0 ⇒ 1 – 3x + 4 = 0
5 38 – 10x + 12 = 70 ⇒ 10x = 50 – 70 = –20
⇒ 3x = 5 ⇒ x =
3 –20
⇒x= = –2
10
Sol 10: |A| = 20, |B| = – 20

|AB| = |A| |B| = 20(–20) = –400 5 15 −25 5 5 −25


Sol 17: 7 21 30 = (3) 7 7 30
 3 1 8 24 42 8 8 42
Sol 11: A =  
 –5 4  Two column are same so Determinants is 0
C11 = 4, C12 = 5, C21 = –1, C22 = 3 1 a b+c
T
C C12   4 –1 Sol18: 1 b c + a C2 → C2 + C3
adjA =  11  =   1 c a+b
C21 C22  5 3 
1 a+b+c b+c
1 3  ⇒ 1 a+b+c c+a
Sol 12: A =   1 a+b+c a+b
 –6 –18 
1 1 b+c
|A| = – 18 – [3] [–6] = –18 + 18 = 0
= (a+b+c) 1 1 c + a C1 → C2 – C1
So, A–1 does not exist
1 1 a+b
0 1 b+c
3 1 8 3 1 1
= (a + b + c) 0 1 a + c =0
Sol 13: −4 2 16 = (8) −4 2 2 0 1 a+b
−5 3 24 −5 3 3

Two columns are same, so determinant is 0 Sol 19: (2, 0), (0, 5) and (x, y) are collinear

1 2 0
x 0 1
  ⇒ 1 0 5 =0
Sol 14: a =  2 −1 4  is singular 1 x y
1 2 0 
 
M a them a ti cs | 17.43

1[–5x] + 2[5 – y] = 0 C11 = 15 – 1 = 14


–5x + 10 –2y = 0 C12 = 1 + 10 = 11
x C13 = –2 – 3 = –5,
5x + 2y = 10 → + y - 5= 1
2
C21 = 1 + 10 = 11
Sol 20: |A| = 3, A’s order → 2 × 2 C22 = 5 –1 = 4
|5A| = (5)2 |A| = 25 × 3 = 75 C23 = –2 – 1 = –3
C31 = –2 – 3 = –5,
a a12  C32 = –2 – 1 = –3
Sol 21: A =  11  , |A| = –10
a21 a22 
C33 = 3 – 4 = –1
|A| = a11C11 + a12C12 (along first row) = |A| = –10
|A| = 1[14] – 2[11] + 1[–5]
= 14 – 5 – 22 = –13
1 a b+c 1 a b+c
R 2 −R1 , R3 −R1
Sol 22: 1 b a + c  → 0 b −a a−b 14 11 –5 
 
1 c a+b 0 c−a a−c AdjA = 11 4 –3 ,
 –5 –3 –1 
 
1 a b+c 1
A–1 = adjA
=( a − b )( a − c ) 0 −1 1 |A|
0 −1 1
14 11 −5
1  
A =
–1
 11 4 −3 ,
Since the columns are linearly dependent, hence the –13
 −5 −3 −1 
value of determinant is zero.  
1 1
|A–1| = =
Sol 23: |A| = –4 |A| –13

Order of A = 3 For adjA,


|adjA| = |A| 3–1
= (–4) = 16
2
4 −3
C11 = = –4 – 9 = –13,
−3 −1
 2 1 5
Sol 24:   C12 = 15 + 11 = 26
 −1 0 3
C13 = –33 + 20 = –13,
It is not a square matrix, so inverse not exist
C21 = 15 + 11 = 26,
Sol 25: |A| = 12, A’s order 3 × 3 C22 = –14 – 25 = –39
|A. adjA| = |A| |A| 3–1
= |12| = 1728
3
C21 = C12, C31 = C13,
C23 = –55 + 42 = –13,
2 3  –1 1
Sol 26: A =  ,A = A C38 = 56 – 121 = –65
5 –2 19
 −13 26 −13
C11 = –2, C12 = –5, C21 = –3, C22 = 2 1  
So |A | =
–1 –1
 26 −39 −13
adjA 1  −2 −3 1 2 3  1 | A –1 | 
A–1 = =   =   = A  −13 −13 −65 
|A| (–4 – 15)  −5 2  19 5 –2 19
 −13 26 −13  1 –2 1
1    
 1 −2 1 =  26 −39 −13 =  –2 3 1 = A
  –13
Sol 27: A =  −2 3 1   −13 −13 −65  1 1 5
   
 1 1 5
 
1 7 . 4 4 | Determinants

Sol 28: 2x + 5y = 7, 6x + 15y = 13


 1 – tanx   cos2 x – sinx cos x 
AA = 
1 –1
 
2 5 tanx 1  sinx cos x cos2 x 
D= = 30 – 30 = 0
6 15
 cos2 x − sin2 x – sinx cos x − sinx cos x 
D = 0. So system is inconsistent =  
sinx cos x + sinx cos x – sin2 x + cos2 x 

a b cos2x – sin2x 
Sol 29: A = =   = R.H.S.
c (1 + bc) / a  sin2x cos2x 

 1 + bc 
|A| = a   – bc = 1 + bc – bc = 1 2 0 –1
 a   
Sol 31: A = 5 1 0  ,
(1 + bc) / a –b  0 1 3 
adjA =    
 –c a
Assume A – xI = 0  ….. (i)
adjA (1 + bc) / a –b  |A1 – xI| = 0
A–1 = = 
|A|  –c a
2 − x 0 –1 
(a2 + bc + 1)I – aA  
So  5 1–x 0  =0
a2 + bc + 1 0  a b   0 1 3 − x 

=  – a 
c (1 + bc) / a
2
 0 a + bc + 1 ⇒ (2 – x) [x2 + 3 – 4x] –1[5] = 0

a2 + bc + 1 − a2 –ab  ⇒ –x3 – 3x + 4x2 + 2x2 + 6 – 8x = 5


=  
 –ac 2
a + bc + 1 – (1 + bc) ⇒ x3 – 6x2 + 11x = 1
1
⇒ x2 – 6x + 11 = = x–1
(1 + bc) / a –b  x
= a  = aA
–1
(A – xI) = 0
 –c a 
⇒ A2 – 6A + 11I = A–1
R.H.L. = L.H.S.
 2 2 1 1 3 2 
 1 tanx     
Sol 30: A =  Sol 32: A =  −2 1 2  = 1 1 1 
 ,
 – tanx 1   1 −2 2 
 
2 −3 −1
 
 1 – tanx   2 + 2 + 2 6 + 2 −3 4 + 2 −1 
A1 =    
tanx 1  AB =  −2 + 1 + 4 –6 + 1 − 6 –4 + 1 – 2
 1−2+ 4 3 − 2 − 6 2 − 2 − 2 
1 
|A| = 1 + tan2x = 6 5 5
cos2 x  
= 3 −11 −5
 1 – tanx  3 −5 −2 
adjA =    
 tanx 1 
For A
 sinx  C11 = 2 + 4 = 6,
adjA  1 – 
A–1 = cos2 x
= cos x 
C12 = 2 + 4 = 6,
|A|  sinx
 cos x 1  C13 = 4 – 1 = 3,

 cos2 x – sinx cos x  C21 = –2 – 4 = –6,
=  
sinx cos x cos2 x  C22 = 4 – 1 = 3,
C23 = 2 + 4 = 6,
M a them a ti cs | 17.45

C31 = 4 – 1 = 3, C22 = –12 – 15 = –27


C32 = –2 – 4 = –6, C23 = 30 + 15 = 45
C33 = 2 + 4 = 6 C31 = –25 + 55 = 30
|A| = 2(6) + 2(6) + 1(3) = 27 C32 = –30 + 15 = 15,

6 −6 3  C33 = –66 – 15 = –81


adjA 1  
A
= –1
= 6 3 −6   –3 –15 30 
| A | 27   
3 6 6 
  So (AB) =  –9 −27 15 
–1

2 −2 1  18 45 –81
 
1 
= 2 1 −2  −1 −5 10 
9 1 
1 2 2 
  =  −3 −9 15  = B–1A–1
9
For B  −30 3 9 
 
C11 = –1 + 3 = 2 ,
a b c
C12 = 2 + 1 = 3, Sol 33: b c a
C13 = –3 –2 = –5, c a b
C21 = –6 + 3 = –3, = (a+b+c) (ab + bc + ca – a2 – b2 – c2)
C22 = –1 – 4 = –5, = 3abc – a3 – b3 – c3
C23 = –3 – 6 = –9, C1 → C1 + C2 + C3
C31 = 3 – 2 = 1,
a+b+c b c 1 b c
C32 = 2 – 1 = 1, = a + b + c c a = (a+b+c) 1 c a
C33 = 1 – 3 = –2 a+b+c a b 1 a b
|B| = 1[2] + 3[3] + 2[–5] = 11 – 10 = 1 R1 → R1 – R3, R2 → R2 – R3
 2 −3 1  0 b −c c −b
adjB  
B = =  3 −5 1 
–1
(a+b+c) 0 c − a a − b
|B |
 −5 −9 −2
  1 a b
 4 −6 +1 –4 − 3 + 2 2 + 6 + 2  = (a+b+c)[(b–a) (a–b) – (c–a) (c–b)]
1  
B A =
–1 –1
 6 − 10 + 11 –6 − 5 + 2 3 + 10 + 2  = (a+b+c) (ab + bc + ca – a2 – b2 – c2)
9
 –10 − 18 − 2 16 − 3 − 4 –5 + 18 − 4 
  = a2b – a2b + ....... + 3abc – a3 – b3 – c3
 −1 −5 10  = 3abc – a3 – b3 – c3
1 
=  −3 −9 15 
9
 −30 3 9  y+z x y
 
6 5 5 Sol 34: z + x z x = (x + y + z) (x – z)2
  x+y y z
AB = 3 −11 −5 ,
3 −5 −2  R → R1 + R2 + R3
 
|AB| = 6(–3) + 5(–9) + 5(18) = –18 – 45 + 90 = +27 2(x + y + z) x + y + z x + y + z
C11 = 22 – 25 = –3 z+x z x
x+y y z
C12 = –9
C13 = –15 + 33 = 18 2 1 1
= (x+y+z) z + x z x
C21 = –25 + 10 = –15
x+y y z
1 7 . 4 6 | Determinants

C1 → C1 – C2 – C3 R1 → R1 + R2

2 −1 −1 1 1 = (c – a)(b – c)
= (x+y+z) z + x − z − x z x a + 2b + ( −2a − b + c) −a − c + b + c b − a
x+y−y−z y z −(2a + b + c) b+c −a
0 1 1 (a + b)2 c2 ab
= (x+y+z) 0 z x
b–a b−a b−a
x−z y z
= (c–a)(b–c) −2a − b − c b + c −a
= (x + y + z) (x – z) (x – z)
(a + b)2 c2 ab

–bc b2 + bc c2 + bc 0 1 1
Sol 35: a2 + ac –ac c2 + ac = (ab + b + ca)3 0 b + c −a
a2 + ab b2 + ab −ab a2 + b2 + c2 c2 ab

C1 → C1 + C3, C2 → C2 + C3 C1 → C1 + C2 – 2C3
= (c–a)(b–c)(b–c)
c2 (b + c)2 c2 + bc
(a + c)2 c2 c2 + ac z−z 1 1
a2
b 2
−ab –2a − b − c + b + c + 2a b + c –a
a2 + b2 + 2ab + c2 − 2ab c2 ab
= c2[–abc2 – b2(c2 + ac)] +
= –(a–b)(b–c)(c–a)
(b+c)2 [a2(c2+ac) + ab(a+c)2]
z−z 1 1
(c2 + b) [b2(a + c)2 – c2a2]
–2a − b − c + b + c + 2a b + c –a
= [–abc4 – c4b2 – c3b2a + (b+c)2 (a2c2 + a3c + a3b + abc2 a2 + b2 + 2ab + c2 − 2ab c2 ab
+ 2a2bc] + (c2 + bc)[b2a2 + b2c2 + 2acb2 – c2a2]
This on simplification comes out to be equal to = –(a–b)(b–c) (c–a) (a2+b2+c2)(–a–b–c)

(ab + bc + ca)3 = (a–b)(b–c)(c–a)(a2+b2+c2)(a+b+c)

(b + c)2 a2 bc a b−c c+b


2 2
Sol 36: (c + a) b ca = (a–b) (b–c) (c–a) Sol 37: a + c b c−a
2 2
(a + b) c ab a−b a+b c

R1 → R1 – R3, R2 → R2 – R3 = (a b + c) (a2 + b2 + c2)

(b − c)2 − (a + b)2 a2 − c2 bc − ab a b−c c+b 0 b−c c+b


= a b c−a + c b c−a
= (c + a)2 − (a + b)2 b2 − c2 ac − ab
2 2 a a+b c –b a + b c
(a + b) c ab
1 b−c c+b 0 1 c+b
(c − a)[c + a + 2b] (c − a)(–a − c) b(c − a)
=a 1 b c−a + c 1 c−a
= (b − c)[–c − b − 2a] (b − c)(b + c) –a(b + c)
1 a+b c –b b c
(a + b)2 c2 ab
0 –c c + b
a + 2b + c −a − c b + c 0 c−a
= (c – a) (b – c) −(2a + b + c) b + c −a –b a c
(a + b)2 c2 ab
M a them a ti cs | 17.47

1 b−c c+b 0 1 c+b 1 3


M22 = =9–6=3
=a 1 b c − a +b c 1 c − a 2 9
1 a+b c –b 1 c
1 2
M23 = = –7 – 4 = –11
0 –c b 0 –c c 2 –7
+ c 0 –a + c 0 c C21 = –39, C22 = 3, C23 = 11
–b a 0 –b a c
Sol 40: f(x) = ax2 + bx + c, f(0) = 6
Using, C2 → C2 – C1 and C3 → C3-C1
f(2) = 11, f(–3) = 6
in (i), (ii), (iii)
 02 0 1  a  6
    
1 b−c c+b 0 1 c+b 4 2 1 b  = 11 
a 0 c −a − b + c 0 −a − b 9 –3 1  c  6
   
0 a+c −b –b 0 −b
D = –12 – 18 = –30
0 –c 1  6 0 1
+c c c 0  
Da= 11 2 1
–b a + c 0  6 −3 1
 
= a2[a + b + c] + b2[a + b + c] + c2[a + b + c] = 6[2+3]+1[–33 – 12] = 30 – 33 – 12 = –15
= (a2 + b2 + c2) (a + b + c) Da –15 1
a= = =
D –30 2
a b ax + by 0 6 1
Sol 38: b c bx + cy Db= 4 11 1
ax + by bx + cy 0 9 6 1
R3 → R3 – xR1 – yR2 = 6[9 – 4] + 1[24 – 99] = 30 + 24 – 99 = –45

a b ax + by –45 3
b= =
0= b c bx + cy –30 2

0 0 (ax2 + byx + byx + cy 2 ) 0 0 6


Dc= 4 2 17 = 6[–12 – 18] = –30 × 6
0 = –(ax2 + 2bxy + cy2) (ac – b2)
9 −3 6
0 = (b2 – ac) (ax2 + 2bxy + cy2)
DC –30 × 6
1 b−c c+b 0 1 c+b C=
= = 6
D –30
a 0 c −a − b + b c 0 −a − b x2 3
Equation → ax2+bx+c = + x +6
0 a+c −b −b 0 −b 2 2

0 –c 1 Sol 41: 2x – y = 5
+c c c 0
4x – 2y = 10
−b a + c 0
2 −1
D= =0
1 2 3 4 −2
Sol 39: –4 3 6 5 −1 2 5
2 –7 9 Dx = = 0, Dy = =0
10 −2 4 10
2 3
⇒M21 = = 18 + 21 = 39 So system has infinite solution (consistent).
−7 9
1 7 . 4 8 | Determinants

Sol 42: 3x – y – 2z = 2
1 a2 a3
2y – z = – 1 = 1 b2 b3 = R.H.S.
3x – 5y = 3 1 c 2
c 3

3 −1 −2 C2 ↔ C3 and then C1 ↔ C2
D = 0 2 −1
3 −5 0 0 p −q p −r
= 3[–5] –1[–3] –2[–6] = –15 + 3 + 12 = 0 Sol 45: q − p 0 q−r
r −p r −q 0
2 −1 −2
Dx = –1 2 −1 q−p q−r q−p 0
= –(p – q) + (p – r)
3 −5 0 r −p 0 r −p r −q

= 3[1 + 4] – 5[2 + 2] = 15 – 20 = –5 ≠ 0 =+(p – q (q – r) (r – p) – (p – q) (q – r) (r – p)=0


So system is inconsistent.
x2 0 3
1 a bc 1 a a 2 Sol 46: x 1 −4 = 11
Sol 43: 1 b ca = 1 b b2 1 2 0
1 c ab 1 c c2 ⇒ x2[8] + 3[2x – 1] = 11

1 a bc a a2 abc ⇒ 4x2 + 3x – 7 = 0
1 ⇒ (x – 1) (4x + 7) = 0
L.H.S. = 1 b ca = b b2 abc
abc
1 c ab c c2 abc ⇒ (x – 1)(4x + 7) = 0
7
a a2 1 ∴ x = 1 or –
abc 4
= b b2 1 C2 ↔ C3
abc
c c2 1 3 2  2
Sol 47: A =   , A – 4A – I = 0
a 1 a 2 2 1 
= – b 1 b2 C1 ↔ C2 Assume A – xI = 0 →
2
c 1 c 3 − x 2 
  =0
 2 1 − x
2
1 a a
2
= (–1)2 1 b b = R.H.S. (3 – x)(1 – x) – 4 = 0
1 c c2 3 + x2 – x – 3x – 4 = 0
⇒ x2 – 4x – 1 = 0
2 2 3
a a bc 1 a a A – xI = 0
2 2
Sol 44: b b ca = 1 b b3
⇒ A2 – 4A – I = 0 Hence proved.
c c2 ab 1 c2 c3
⇒ A–1[A2 – 4A – I] = 0
a a2 bc A – 4I – A–1 = 0
2 abc
L.H.S. = b b ca ×
2
abc 3 2  1 0 
c c ab ⇒ A – 4I =   –4  
2 1  0 1 
a2 a3 abc a2 1 a3
1 abc 2 3 − 4 2   –1 2 
= b2 b3 abc =
3
b b 1 A–1 = 
abc 2 abc 2  =  
c c3 abc c c3 1  2 1 − 4  2 –3
M a them a ti cs | 17.49

1 / a a2 bc a/a a3 abc 0 a + c − 2b a − b
2 1 = (a + b + c) 0 b − 2c + a b − c
Sol 48: 1 / b b ac = b / b b3 abc
(abc) 1 c−a c
1 / c c2 ab c / c c3 abc
= (a + b + c) [(b – c)(a + c – 2b) – (a – b)(b + a – 2c)]
1 a3 1
abc = a2b + b2a – b2a – a2b + ….. + 3abc – a3 – b3 – c3
= 1 b3 1 = 0
(abc) = 3abc – a3 – b3 – c3
1 c3 1

2 −3
x+a b c Sol 52: A =  
Sol 49: b x+c a =0 3 4 
c a x +b Assume |A – xI| = 0
Have to show that x = –(a + b + c) 2 −3 1 0 
⇒   –x   =0
R1 → R1 + R2 + R3 3 4  0 1 
x +a+b+c x +a+b+c x +a+b+c 2 − x − −3 
b x+c a =0 ⇒   = |0| = 0
 3 4 − x
c a x +b
⇒ (2 – x) (4 – x) + 9 = 0
1 1 1
⇒ 8 + x2 – 4x – 2x + 9 = 0
(x+a+b+c) b x + c a =0
⇒ x2 – 6x + 17 = 0 and |A – xI| = 0
c a x +b
So, A satisfied this equation
x + a + b + c = 0 ⇒ x = –(a + b + c)
⇒ A2 – 6A + 17 I = 0
A–1[A2 – 6A + 17I] = 0
x+4 x 2
Sol 50 : 2 x+4 x ⇒ A – 6I + 17A–1 = 0
x x x+4 –17A–1 = (A – 6I)

C1 → C1 − C3 ,C2 → C2 − C3 –1 1  2 −3 1 0  
A–1 = (A – 6I) =   −6 
17 17  3 4  0 1  
4 0 x
1 2 − 6 −3  1  −4 −3
0 4 x A–1 = –   =–  
17  3 4 − 6 17  3 –2 
−4 −4 x + 4

= ( 4x + 16 + 4x ) + 16x = 48x+64 1  4 3
A–1 =  
17  −3 2

b + c a−b a
2 4  0 2  1 6 
Sol 51: c + a b − c b = 3abc – a3 – b3 – c3 Sol 53:   A 1 3 = 3 –1
1 3     
a+b c−a c
Assume BAC = D
C1 → C1 + C3
2 4
a+b + c a−b a 1 a−b a |B| = =6–4=2
1 3
= a + b + c b − c b = (a+b+c) 1 b − c b
a+b+c c−a c 1 c−a c 3 –4
Adj B =
R1 → R1 – R3, R2 → R2 – R3 –1 2

1  3 −4 
B–1 =  
2  −1 2 
1 7 . 5 0 | Determinants

B–1BAC = B–1D  −3 + 3 + 3 8 + 3 − 30 −12 − 21 + 45   3 −19 12 


–1   –1  
1 3 −4 1 6 −3 + 4 + 3 8 + 4 − 30 −12 − 28 + 45 =  4 −18 5 
AC = B–1D = 11  11
 −3 + 3 + 4 8 + 3 − 40 −12 − 21 + 60   4 −29 27 
2 −1 2 3 –1    

1  3 − 12 18 + 4  1  −9 22   2 −1 1 
AC =   =  
2  –1 + 6 –6 − 2  2  5 –8  Sol 55: A =  −1 2 −1 ,
 1 −1 2 
0 2   
|C| =   = –2 Assume (A –XI) = 0
1 3
2 − x −1 1 
 3 –2 1  –3 2   
adjC =   ,C = 
–1
 ⇒  −1 2 − x −1  = 0
 –1 0  2  1 0  1
 −1 2 − x 
ACC–1 = B–1DC–1 (2 – x) [(2 – x)2 – 1[–1 + 2 – x] + 1[1 – 2 + x] = 0
1 1  –9 22   −3 2  ⇒ (2 – x)[4 + x2 – 4x –1] –1 + x –1 + x = 0
A= ×   
2 2  5 −8   1 0  ⇒ 6 – x3 + 2x2 – 8x – 3x + 4x2 – 2 + 2x =0

1 27 + 22 −18  1  49 −18  ⇒ –x3 + 6x2 – 9x + 4 = 0


A=   =+  
4  −15 − 8 10  4  −23 10  ⇒ x3 – 6x2 + 9x – 4 = 0
|A – xI| = 0, so this equation satisfied A

5 0 4  1 3 3  ⇒ A3 – 6A2 + 9A – 4I = 0 
⇒ A–1[A3 – 6A2 + 9A – 4I] =


    A–10 = 0
Sol 54: A = 2 3 3  , B–1 = 1 4 3 
1 2 1  1 3 4  ⇒ A2 – 6A + 9I – 4A–1 = 0
   
(AB)–1 = B–1A–1  2 −1 1   2 −1 1 
   
C11 = 3 – 6 = –3, A =  −1 2 –1  −1 2 –1
2

 1 −1 2   1 −1 2 
C12 = 3 – 2 = 1,    

C13 = 4 – 3 = 1,  4 + 1 + 1 −2 – 2 – 1 2 + 1 + 2 
 
C21 = 8, =  −2 − 2 − 1 +1 + 4 + 1 –1 − 2 − 2 
 2 + 1 + 2 −1 – 2 – 2 1 + 1 + 4 
C22 = 5 – 4 = 1,  

C23 = –10,  6 −5 5 
 
C31 = –12, A =  −5 6 −5 ,
2

 5 −5 6 
C32 = 8 – 15 = –7,  

C33 = 15 4A–1 = A2 – 6A + 9I

|A| = 5(–3) + 4(1) = –15 + 4 = –11  6 −5 5   2 −1 1  1 0 0 


     
4A =  −5 6 −5 – 6  −1 2 −1 + 9 0 1 0 
–1

 −3 8 −12  5 −5 6   1 −1 2  0 0 1 
1 –1        
A =
–1
adjA =  1 1 −7 
|A| 11 6 − 12 + 9
 1 −10 15  −5 + 6 5−6 
   
4A =  −5 + 6
–1
6 − 12 + 9 −5 + 6 
1 3 3   –3 8 12   5−6 −5 + 6 6 − 12 + 9 

1    
B A =–
–1 –1
1 4 3  1 1 –7 
11   3 1 −1
1 3 4   1 –10 15 
    1 
A–1 = 1 3 1
4
 −1 1 3 
 
M a them a ti cs | 17.51

Exercise 2 1 a2 a3 a a3 1
2 3 3
⇒ abc 1 b b = b b 1
Single Correct Choice Type
2 3 3
1 c c c c 1

1+a 1 1 R1 → R1 – R3, R2 → R2 – R3
Sol 1: (D) 1 1+b 1 =0
1 1 1+c 0 a2 – c2 a3 – c3 a – c a3 – c3 0
2 2 3 3 3 3
⇒ abc 0 b – c b –c = b–c b –c 0
1 1 1 2 3 3
+1 1 c c c c 1
a a a
1 1 1 ⇒ abc [(a2–c2) (b3 – c3) – (b2 – c2) (a3 – c3)
abc 1+ ` =0
b b b
1 1 1 = [(a – c) (b3 – c3) – (b – c) (a3 – c3)
+1
c c c ⇒ abc (a – c) (b – c)[(a + c) (b2 + c2 + bc)

R1 → R1 + R2 + R3 – (b + c) (a2 + c2 + ac)
= (a – c) (b – c) [b2 + c2 + bc – (a2 + c2 + ac)]
1 1 1 1 1 1 1 1 1
1+ + + 1+ + + 1+ + +
a b c a b c a b c abc [ab2 + ac2 + abc + cb2 + c3 + bc2
1 1 1 – ba2 – bc2 – abc – ca2 – c3 – ac2]
abc 1+ =0
b b b
1 1 1 = b2 + c2 + bc – a2 – c2 – ac
1+
c c c = (b – a) (b + a + c)
⇒ abc (b – a) [ab + c(b + a)]
1 1 1 = (b – a) (a + b + c)
 1 1 1 1 1 1
1 + + +  1+ =0 ⇒ abc [ab + bc + ca] = [a + b + c]
 a b c b b b
1 1 1
1+ Sol 3: (A) (sin–1x + sin–1w) (sin–1y + sin–1z) = p2
c c c
N1 N2
C2 → c2 – c1, C3 → c3 – c1 x y
D→
N3 N4
z w

1 0 0 –1 ≤ (x, y, w, z) ≤ 1
 1 1 1 1 N N4 N N4
1 + + +  1 0 =0 x 1w – z 3y
 a b c b
1 If x = y = 7 = w = – 1
0 1
c
N1 N2
x y N2 +N4 N2 +N3
 1 1 1 → (–1) – (–1)
 1 + + +  = 0 ⇒a + b + c = – 1 N N
–1 –1 –1
a b c z 3 w 4
 
For max value
3 4 N1 + N4 = 2n. N2 + N3 = 2m + 1
a a a –1
Sol 2: (A) b b3 b 4 – 1 = 0 ⇒ n,m ∈ N
3 4
c c c –1 Value (–1)2n – (–1)2n – (–1)2m+1
⇒ 1 – (–1) = 2
a a3 a4 a a3 –1
Min value → – 1 – 1 = – 2
⇒ b b3 b 4 + b b3 –1 = 0
c c3 c4 c c3 –1 Dependent of N1, N2, N3, N4
1 7 . 5 2 | Determinants

Sol 4: (C) (1+ x + x2)n = a0 + a1x + a2x2 + …+… a2n x2n = D3 (given)

an–3 an–1 an+1 in D3 → C1 → C1 – C3, C2 → C2 – C3


an–6 an–3 an+3 and assume T = a2+b2 + c2 – ab – bc – ca
an–14 an–7 an+ 7
T 0 ab + bc + ca
(1 + x + x2)n = (x2 + x + 1)n +0 T ab + bc + ca
an–1 = an+1 –T –T a2 + b2 + c2
a0 = an 1 0 ab + bc + ca
an – r = an + rO ≤ r ≤ n = T2 0 1 ab + bc + ca
an–3 an–1 an–1 –1 –1 a2 + b2 + c2
So determinate → an–6 an–3 an–3 T2[a2+b2+c2+ab+bc+ca+ab+bc+ca(1)]
an–14 an–7 an–7
= T2[a2 + b2 + c2 + 2(ab + bc + ca)]
C2 → C2 – C3
bc – a2 ac – b2 ab – c2
an–3 0 an–1
D2 = ac – b2 ab – c2 bc – a2
an–6 0 an–3 = 0
ab – c2 bc – a2 ac – b2
an–14 0 an–7
= C1 → C1 + C2 + C3
–1 2 1
Sol 5: (A) 3 + 2 2 2 + 2 2 1 C1 → C1 – C2 – C3 –T ac – b2 ab – c2
3–2 2 2–2 2 1 D2 = –T ab – c2 bc – a2
–T bc – a2 ac – b2
–1 – 2 – 1 2 1
–1 ac – b2 ab – c2
(– 1) 3 + 2 2 – 2 – 2 2 – 1 2 + 2 2 1
3

D2 = T –1 ab – c2 bc – a2
3–2 2 –2+2 2 –1 2–2 2 1
–1 bc – a2 ac – b2
–4 2 1
R1 → R1 – R3, R2 → R2 – R3
=– 0 2 + 2 2 1 = + 4 [2 + 2 2 –(2 – 2 2 )]
0 2–2 2 1 0 ac – b2 + a2 – bc ab – c2 + b2 – ac
2 2 2 2
D2 = T 0 ab – c + a – bc bc – a + b – ac
= + 4 [ 4 2 ] = 16 2
–1 bc – a2 ac – b2
a b c D2 = – T [(ac – b2 + a2 – bc) (bc – a2 + b2
Sol 6: (C) D1 = b c a ,
– ac) – (ab – c2 + b2 – ac) (ab – c2 + a2 – bc)]
c a b
D2 = (– T) (– T) [T + 3 (ab + bc + ca)]
2 2 2
bc – a ac – b ab – c
D2 = T2[a2 + b2 + c2 + 2(ab + bc + ca)]
2 2 2
D2 = ac – b ab – c bc – a
2 2 ∴ D12 = D2 = D3
ab – c bc – a ac – b2

a b c a b c
1 1 1 1 1 1
D12 = b c a b c a
Sol 7: (C) a b c = a b c
c a b c a b
bc ca ab a3 b3 c3
2 2 2
a +b +c ab + bc + ca ab + bc + ca
L.H. S. = (a – c) (b – c) (b – a)
= ab + bc + ca a2 + b2 + c2 ab + bc + ca
in L.H.S. C1 → C1 – C3, C2 → C2 – C3
ab + bc + ca ab + bc + ca a2 + b2 + c2
M a them a ti cs | 17.53

= (a + 1) (a + 3) [a2 + 1 + 2a – a2 – 9 – 6a]
0 0 1
a–c b–c c = (a + 1) (a + 3) ( – 4a – 8)
a3 – c3 b3 – c3 c3 = – 4 (a + 2) (a + 1) (a + 3)
D = D x = Dy = 0
0 0 1
⇒ a = – 2 (common solution in all)
= (a – c) (b – c) 1 1 c
a2 + c2 + ac b2 + c2 + bc c3
Sol 9: (B) 3x – 7y + 5z = 3, 3x + y + 5z = 7
⇒ (a – c) (b – c)[b2 + c2 + bc – a2 – c2 – ac]
2x + 3y + 5z = 5
⇒ (a – c) (b – c) (b – a) (b + a + c)
3 –7 5
→a+b+c=1
D= 3 1 5
= (a – c) (b – c) (b – a) 2 3 5
abc = a + b +c = 3 [5 – 15] – 7 [10 – 15] + 5 [9 – 2]
A.M. ≥ G.M = – 30 + 35 + 35 = 40 ≠ 0
a+b+c 1
≥ (abc)1/3 ; ≥ (abc)1/3 So system is consistent with unique non trivial solution.
3 3
1 Sol 10: (B) (sin θ)x + 27 = 0
≥ abc → abc is always less than 1/27
27
(cos θ)x + sin θ y = 0
Sol 8: (A) (a + 1)3x + (a + 2)3y = (a + 3)3 (cos θ)y + 2z = 0
(4 + 1)x + (a + 2)y = (a + 3) sin θ 0 2
x+y=1 D = cos θ sin θ 0
Here for two variable thus equation 0 cos θ 2

So D = Dx = Dy=0 for consistent D = sin θ (sin θ.2) + 2(cos2θ)

(a + 1)3 (a + 2)3 = 2 (sin2θ + cos2θ) = 2 Constant


D=
(a + 1) (a + 2) 0 0 0 2
C = 0 , Dx = 0 sin θ 0
(a + 1)2 (a + 2)2 0 a cos θ 2
= (a + 1) (a + 2)
1 1
So system has a unique solution which is a function of
= (a + 1) (a + 2) [a2 + 1 + 2a – a2 – 4 – 4a] = (a + 1) a and θ
(a + 2) (–2a – 3) … (i)
(1 + x)2 (1 – x)2 –(2 + x2 )
3 2
(a + 3) (a + 2) Sol 11: (D) 2x + 1 3x 1 – 5x
Dx =
(a + 3) (a + 2) x +1 2x 2 – 3x
(a + 3)2 (a + 2)2 (1 + x)2 2x + 1 x +1
= (a + 2) (a + 3)
1 1 2
+ (1 – x) 3x 2x = 0
= (a + 2) (a + 3) (a + 9 + 6a – a – 4 – 4a)
2 2
1 – 2x 3x – 2 2x – 3
= (a + 2) (a + 3) (5 + 2a) … (ii) In 2nd determinate R3 → + R3 + R1
(a + 1)3 (a + 2)3 (1 + x)2 2x + 1 x +1
Dy =
(a + 1) (a + 3) (1 – x)2
3x 2x =- B  1 – 2x + (1 + x)2
(a + 1)2 (a + 3)2 +(2 + x2 ) –(1 – 5x) 3x – 2
= (a + 1) (a + 3)
1 1 = 2 + x2 + 2x – 2x = 2 + x2
1 7 . 5 4 | Determinants

R3 → – R3 So Dx = Dy = Dz = 0

(1 + x)2 (2x + 1) x +1 sin3θ –1 1


2
⇒ |A| – (1 – x) 3x 2x = B D = cos2θ 4 3
–(2 + x2 ) 1 – 5x 2 – 3x 2 7 7

x, y, z are not all simultaneously equal to zero so for


Now all rows of A is equal to columns of B
solution (not-trivial), D = 0
⇒ |B| = – |A|
sin 3θ [28 – 21] – 1[6 – 7cos 2q]
|A| – |A| = 0 (always)
+ 1 [7 cos2θ – 8] = 0
For every valued of x
⇒ 7 sin 3θ – 6 + 7 cos2θ + 7 cos2θ – 8 = 0
|A| + |B| is zero
⇒ 7 sin 3θ + 14 cos 2θ = 14
Therefore infinite solutions
⇒ Sin 3θ + 2 cos 2θ = 2

Sol 12: (B) 2x cos2 θ + y sin 2θ – 2sin θ = 0  … (i) ⇒ Sin 3θ + 2 cos 2θ = 2

x sin 2θ + 2y sin2 θ = – 2 cos θ …(ii) ⇒ 3 sin θ – 4 sin3θ + 2[1 – 2 sin2q] = 2

x sin θ – y cos θ = 0 …(iii) ⇒ 3 sin θ – 4 sin3θ + 2 – 4 sin2θ = 2

for (i) & (ii) assume sin θ = x


⇒ 4x3 + 4x2 – 3x = 0
2cos2 θ sin2θ
D= ⇒ x[4x2 + 4x – 3] = 0
sin2θ 2sin2 θ
⇒ x[4x2 + 6x – 2x – 3] = 0
= 4 sin2θ cos2θ – 4 sin2 θ cos2 θ = 0
x = 0 or 2x (2x + 3) – 1 (2x + 3) = 0
= (sin2 θ = 2 sin θ cos θ)
(2x + 3) (2x – 1) = 0 ⇒ x = 1/2 or –3/2
2sin θ sin2θ
Dx = x = 0, ½, –3/2 but – 1 ≤ sin θ ≤ 1
–2cos θ 2sin2 θ
sin θ ≠ –3/2
= 4 sin θ + 4 sin θ cos θ
3 2
sin θ ∈ {0, ½}, x = 0, π/6, 5π/6, π, 12π
= 4 sin θ(sin2 θ + cos2 θ) = 4 sin x
between [0, 2p]
for consistent Dx = 0 → 4 sin θ = 0
No. of principle value = 5
θ ∈ n π, n ∈ I
a2 + b2
2sin2 θ 2sin θ c c
Dy = = 4 cos3θ – 4 sin2θ cos θ c
sin2θ – sin θ
b2 + c2
Sol 14: (D) a a = α abc
= – 4 cos θ [sin θ + cos θ) = – 4 cos θ
2 2
a
Dy = 0 θ = (2n + 1)π/2, n ∈ I a2 + c2
b b
b
Sin θ and cos θ both are not zero for same θ, so for
every value of θ system has not a solution
a2 + b2
1 1
c2
Sol 13: (C) (sin 3θ)x – y + z = 0
b2 + c2
abc 1 1
(cos 2θ)x + 4y + 3z = 0 a2
2x + 7y + 7z = 0 a2 + c2
1 1
0 b2
( c = 0  a2 + b2   (b2 + c2 )(a2 + c2 ) 
0 ⇒ abc    – 1
 c2   a2b2 

M a them a ti cs | 17.55

 a2 + c2    b2 + c2    0 x y
– – 1 + 1 1 –   
 b
2
   a2    ⇒ 0 y z =0
2
−(xp + yp + yp + z) xp + y yp + z
 (a b + a c + b + b c )(a + c ) – b a – a b
2 2 2 2 4 2 2 2 2 4 2 4 2
⇒ abc  ⇒ −(xp2 + 2yp + z)(xz − y 2 ) =
0
 a2b2c2
∴ Either xp2 + 2yp + z =0 or y 2 = xz
a +c –b
2 2 2  a –b –c
2 2 2 
– +  ⇒ x, y, z are in GP.
 b2  a2 
 
a4b2 + a4 c2 + a2b 4 + a2b2c2 + a2b2c2 + a2c 4  Sol 2: (A) Given
abc  4 2 
1 x x +1
⇒  +b c + b2c 4 – a2b2c2 – a4 c2 – a2c 4 + a2c2b2 
a2b2c2  2 2 2 4 2 2 4 4 2 2 4  f(x)
= 2x x(x − 1) (x + 1)x
 +b a c – b c – b c – a b – a b 
3x(x − 1) x(x − 1)(x − 2) (x + 1)x(x − 1)
1  2 2 2
= 4a b c + a4b2 + a2b 4  = 4abc =2abc
abc   Applying C3 → C3 − (C1 + C2 )

⇒α=4 1 x 0
= 2x x(x − 1) = 0 0
Sol 15: (C) a2x + (2 – a)y = 4 + a2 3x(x − 1) x(x − 1)(x − 2) 0
ax + (2a – 1)y = a5 – 2
∴ f(x) = 0 ⇒ f(100) = 0
2
a 2–a
D= = a2(2a – 1) + (a – 2) a
a 2a – 1 Sol 3: (D) Since, the given system has non-zero solution.

= 2a3 – a2 + a2 – 2a 1 −k −1
∴ k −1 −1 =0
For D = 0 = 2a (a2 – 1) → +1, – 1, 0
1 1 −1
4 + a2 2–a
Dx = Applying C1 → C1 − C2 , C2 → C2 + C3
5
a – 2 2a – 1
1 + k −k − 1 −1
= (4 + a2) (2a – 1) + (a – 2) (a5 – 2)
⇒ 1+k −2 −1 =0
= 8a – 4 + 2a3 – a2 + a6 – 2a5 – 2a + 4 0 0 −1
at a = 0 Dx = 0 ⇒ 2(k + 1) − (k + 1)2 =
0
a2 4 + a2 0 4+0 ⇒ (k + 1)(2 − k − 1) =0
So Dy = =
a a3 – 2 0 0–2 ⇒k= ± 1
So at a = 0, system has infinite solution Note: There is a golden rule in determinant that n one’s
⇒ (n – 1) zero’s or n(constant) ⇒ (n – 1) zero’s for all
At a = – 1, + 1, D = 0, and Dx, Dy ≠ 0
constant should be in a single row or a single column.
⇒ No solution, no. of values = 2
sinx cos x cos x
Sol 4: (C) Given cos x sinx cos x = 0
Previous Years Questions
cos x cos x sinx

Applying C1 → C1 + C2 + C3
xp + y x y
Sol 1: (B) Given yp + z y z 0
= sinx + 2cos x cos x cos x
0 xp + y yp + z = sinx + 2cos x sinx cos x
Applying C1 → C1 − (pC2 + C3 ) sinx + 2cos x cos x sinx
1 7 . 5 6 | Determinants

1 cos x cos x When k ≠ 3 , the given system of equation has no


solution.
(2cos x + sinx) 1 sinx cos x =
= 0
1 cos x sinx ⇒ Statement I is true. Clearly, Statement II is also true
as it is rearrangement of rows and columns of
Applying R 2 → C2 − R1 , R 3 → C3 − R1  1 −2 3
 
1 cos x cos x  1 −3 4 
 −1 1 −2
⇒ (2cos x + sinx) 0 sinx − cos x 0 0
=  
0 0 sinx − cos x
Sol 8: Given systems of equation can be rewritten as
⇒ (2cos x + sinx)(sinx − cos x)2 =
0
–x + cy + bz = 0
0 or sinx − cos x =
⇒ 2cos x + sinx = 0
cx – y + az = 0 and bx + ay – z = 0
⇒ 2cos x = − sinx or sinx = cos x
Above system of equations are homogeneous equation.
1 π π
⇒ cot x = − gives no solution in − ≤ x ≤ and sin Since, x, y and z are not all zero, so it has non-trivial
2 4 4 solution.
x = cos x ⇒ tan x = 1
Therefore, the coefficient of determinant must be zero
π
⇒ x= −1 c b
4
∴ c −1 a =
0
Sol 5: (A) Given equations b a −1

x + ay = 0, az + y = 0 and ax + z = 0 ⇒ −1(1 − a2 ) − c( −c − ab) + b(ca + b) =0


has infinite solutions. ⇒ a2 + b2 + c2 + 2abc − 1 =0
⇒ a2 + b2 + c2 + 2abc =
1
1 a 0
∴ 0 1 a = 0 ⇒ 1 + a3 =
0 or a = –1
Sol 9: Since α is repeated root of f(x) = 0.
a 0 1
= a(x − α )2 , a ∈ constant ( ≠ 0)
∴ f(x)
Sol 6: (1) For infinitely many solution, we must have A(x) B(x) C(x)
k +1 8 4k Let φ(x) = A(α ) B(α ) C(α )
= = ⇒k=1
k k + 3 3k − 1 A ′(α ) B′(α ) C′(α )

[To show φ(x) is divisible by (x − α )2 , it is sufficient to


Sol 7: (A) The given system of equation can be show that φ(α ) and φ′(α ) =0 ].
expressed as
A(α ) B(α ) C(α )
 1 −2 3  x   −1 ∴ φ(α=
) A(α ) B(α ) C(α ) = 0
     
 1 −3 4  y  = 1 A′(α ) B′(α ) C′(α )
 −1 1 −2 z   k
     
[ R1 and R 2 are identical]
Applying R 2 → R 2 − R1 , R 3 → R 3 + R1
A ′(x) B′(x) C′(x)
1 −2 3  x   −1 Again, φ′(x) = A(α ) B(α ) C(α )
     A ′(α ) B′(α ) C′(α )
⇒ ~ 0 −1 1   y  =
 2
0 −1 1   z  k − 1 A ′(α ) B′(α ) C′(α )
    
φ′(α )= A(α ) B(α ) C(α ) =0
1 −2 3   x   −1
     A ′(α ) B′(α ) C′(α )
⇒ ~ 0 −1 1   y  =
 2
0 0 0   z  k − 3 [ R1 and R3 are identical]
    
Thus, α is repeated root of φ(x) =
0
⇒ R3 → R3 − R 2
Hence, φ(x) is divisible by f(x).
M a them a ti cs | 17.57

a b c  1+1+1 1+α+β 1 + α2 + β21 1 1 1 1


  Sol 12: (C) 1+α+β 2
1+α +β 2 3 3
1 + α + β= 1 α β ×1 α
Sol 10: (4) Given A = b c a  , abc = 1
 c a b 1 + α2 + β2 1 + α3 + β3 1 + α 4 + β3 1 α2 β2 1 β
 
T
and A A = 1  … (i)
1 1 1 1 1 1 1 1 1
Now, A T A = 1
= 1 α α × 1 α α = 1 α α2
2 2

 a b c   a b c  1 0 0  1 α2 β 1 β β2 1 β β2
    
⇒ b c a  b c a  = 0 1 0 
= (1 − α ) ( α − β ) ( β − 1 )
2 2 2
 c a b   c a b  0 0 1 
    
K =1
 a2 + b2 + c2 ab + bc + ca ab + bc + ca
 
⇒ ab + bc + ca a2 + b2 + c2 ab + bc + ca
 Sol 13: (C) ( 2 − λ ) x1 − 2x1 + x3 = 0
2 2 2
ab + bc + ca ab + bc + ca a + b + c 
2x1 − ( 3 + λ ) x2 + 2x3 = 0
1 0 0  −x1 + 2x2 − λx3 = 0
 
= 0 1 0 
Non-trivial solution
0 0 1 
  ∆ =0
2 2 2
1 and ab + bc + ca = 0 
⇒ a +b +c = … (ii) 2−λ −2 1
2 −3 − λ 2 = 0
We know, a3 + b3 + c3 − 3abc
−1 2 −λ
= (a + b + c)(a2 + b2 + c2 − ab − bc − ca)

⇒ a3 + b3 + c3 = (a + b + c)(1 − 0) + 3
(1 − λ ) {3λ + λ2 − 4} + 2. {−2λ + 2} + ( 4 − 3 − λ ) = 0
[from equation (i) and (ii)] (
⇒ 6λ + 2λ2 − 8 − 3λ2 − λ3 + 4λ − 4λ + 4 + 1 − λ = 0 )
3 3 3
∴ a + b + c = (a + b + c) + 3  … (iii) ⇒ −λ2 − λ2 − 5λ + 3 =0

Now, (a + b + c)2 =a2 + b2 + c2 + 2(ab + bc + ca) =1… (iv) x3 − λ 2 + 2λ 2 − 2λ − 3λ + 3 =0


∴ From equation (iii), a3 + b3 + c3 =1 + 3
3 3 3
( )
⇒ λ2 λ2 − 1 + 2λ ( λ − 1 ) − 3 ( λ − 1 ) =0
⇒ a +b +c =
4
(
⇒ ( λ − 1 ) λ2 + 2λ − 3 =0 )
k +1 8
Sol 11: (B) ∆= = k 2 + 4k + 3 − 8k ⇒ ( λ − 1 )( λ + 3)( λ − 1 ) =0
k k +3
λ 1,1, −3
=
= k 2 − 4k + 3
=(k − 3)(k − 1) Sol 14: (D) x + λy − z = 0
4k 8 λx − y − z =0
∆1= = 4k 2 + 12k − 24k + 8
3k − 1 k + 3 x + y − λz = 0
2
( 2
)
= 4k − 12k + 8 = 4 k − 3k + 2 = 4 (k − 2 )(k − 1 ) For non-trivial solution ⇒ ∆ =0
1 λ −1
k +1 4k
∆2= = 3k 2 + 2k − 1 − 4k 2 ⇒ λ −1 −1 =0
k 3k − 1
1 1 −λ
− (k − 1 )
2
=−k 2 + 2k − 1 =
As given no solution ⇒ ∆1 & ∆2 ≠ 0
{ }
⇒ λ + 1 − λ −λ2 + 1 − ( λ + 1 ) =0

⇒ λ ( λ − 1 ) =0
2
∆ =0
⇒k =
3 λ= 0, ±1
1 7 . 5 8 | Determinants

JEE Advanced/Boards D z 10
y= = = 2
D 5
Exercise 1 1 1 1
Dy = 3 6 1 = 1[1 – 6] = –5,
Sol 1: (a) x + y + z = 6 1 0 0
2x + y – z = 1 D y –5
y = = = –1
x + y – 2z = –3 D 5

1 1 +1 1 2 1
D = 2 1 –1 C1 → C1 – C2 Dz = 3 1 6 = 1[6 – 1] + 6[0] = 5m,
1 1 −2 1 2 0
Dz 5
0 1 +1 Z= = =1
D 5
⇒ 1 1 –1 = 1[+2 + 1] = 3
(c) 7x – 7y + 5z = 3
0 1 −2
3x + y + 5z = 7
6 1 +1
2x + 3y + 5z = 5
Dx = 1 1 –1 C3 → C3 + C2
–3 1 −2 7 −7 5
D= 3 1 5 R1 → R1 – R3; R2 → R2 – R3
6 1 2
2 3 5
⇒ 1 1 0 = 1[6 – 1] + 2(1 + 3) = –5 + 8 = 3
–3 1 −1 5 −10 0
1 –2 0 = 5[–10 + 10] = 0
1 6 1
2 3 5
Dy = 2 1 –1
1 –3 –2 3 −7 5
Dx = 7 1 5 R1 → R1 – R3; R2 → R2 – R3
= 1[–2 – 3] + 6[–1 + 4] + 1[–6 – 1] = –5 + 18 – 7 = 6
5 3 5
1 1 6
−2 −10 0
Dz = 2 1 1
2 –2 0 = 5[4 + 20] = 120 ≠ 0
1 1 –3
5 3 5
= 1[–3 – 1] + 1[1 + 6] + 6[2 – 1] = –4 + 7 + 6 = 9
D = 0 but Dx ≠ 0, so, system is inconsistent
Dx 3 Dy 6 Dz 9
x= = = 1 ,y= = = 2 , z= = = 3
D 3 D 3 D 3
Sol 2: x + ky + 3z = 0  … (i)
Here, it is consistent 3x + ky – 2z = 0 … (ii)

(b) x + 2y + z = 1 2x + 3y – 4z = 0  … (iii)

3x + y + z = 6 Equation has non-trivial solution.

x + 2y = 0 So, D = Dx = Dy = Dz = 0

1 2 1 1 k 3
D = 3 1 1 = 1[6 – 1] + 1[0] = 5 D = 3 k −2
1 2 0 2 3 –4

1 2 1 = 1 [–4k + 6] + k[–4 + 12] + 3[9 – 2k]


Dx = 6 1 1 = 2[6 – 1] = 10, = –4k + 6 + 8k + 27 – 6k = 33 – 2k = 0
0 2 0
M a them a ti cs | 17.59

33 Sol 4: a(y + z) = x → x – ay – az = 0
K= , assuming x = t
2 b(z + x) = y → bx – y + bz = 0
From equation (ii) – (i) c(x + y) = z → cx + cy – z = 0
2x – 5z = 0 0 
2x 2t  
z= = , (x = t) c = 0  , so Dx = Dy = Dz = 0
5 5 0 
 
In (iii)
So for non-trivial solution, D = 0
⇒ 2t + 3y – 4z = 0 → 3z = 4z – 2t
1 −a –a
 2t  8t − 10t –2t
⇒ 3y = 4   – 2t = = D= b –1 b C1 → C1 – C3; C2 → C2 – C3
5 5 5
c c –1
–2t
⇒y=
15 1+a 0 –a
 –2t 2t  D= 0 –(1 + b) b
(x, y , z) ⇒  t, ,  t∈R
 15 5  1+c 1+c –1

 1 1 1 
Sol 3: ax + y + z = α –1 →  + +  =0
 1 + a 1 + b 1 +c
x + ay + z = α – 1
or from equation
x + y + az = α – 1
x y z
a= ,b= ,c=
α 1 1 y+z x+z x+y
D= 1 α 1 x+y+z x+y+z
1+a= ; 1 + b = ;
1 1 α y+z x+z
D = a[a2 –1] + 1[1 – a] + 1[1 – a] x+y+z
1+c=
= a – α + 2 – 2α = a – 3α + 2
3 3 x+y

a3 – 3α + 2, 1 1 1 x+y+y+z+z+x
+ + =
1+a a+b 1+c x+y+z
At α = 1 ⇒ 1 – 3 + 2 = 0
2(x + y + z)
So (α – 1) is a factor of a3 – 3α + 2 = =2
(x + y + z)
Now, a3 – 3α + 2 can be witten as
⇒ a3 – a2 + a2 – α – 2α + 2 Sol 5: x = cy + bz → x – cy – bz = 0
⇒ a (α – 1) + α(α – 1) – 2(α – 1)
2
y = az + cx → cx – y + az = 0
⇒ (α – 1) (a + α – 2)
2
z = bx + ay → bx + ay – z = 0
D = (α – 1) (a + α – 2)
2
0 
D = (α – 1) (a2 + 2α – α – 2) (α – 1)  
c = 0  →
D = (α – 1)[α(α + 2) – 1(α + 2)] 0 
 
D = (α – 1) (α + 2) (α – 1) Dx = Dy = Dz = 0,
For D = 0, α = 1 or –2 But system has solution. So D = 0
For α = 1,
1 −c −b
0 1 1 D = c −1 a = 1[1–a2] + c[–c – ab) – b[ac +b] = 0
Dx = 0 1 1 = 0, so consistent b a −1
0 1 1
1 – a2 – c2 – abc – abc – b2 = 0
So on Dy and Dt = 0 \α ≠ 1 ⇒ α = –2 a2 + b2 + c2 + 2ab = 1
1 7 . 6 0 | Determinants

x Sol 8: x + y + z = 6
Sol 6: a = → x – ay + az = 0
y−z x + 2y + 3z = 10
y
b= → bx + y – bz = 0 x + 2y + lz = µ
z−x
z (a) A unique solution, D ≠ 0
c= → cx – cy – z = 0
x−y
1 1 1
0  D= 1 2 3
 
c = 0  , so Dx = Dy = Dz = 0, 1 2 λ
0 
  = 1[2λ – 6] + 1[–λ + 3] + 0
For solution → D = 0
= 2λ – 6 + 3 – λ = λ – 3 ≠ 0
1 −a a
λ≠3
D = b 1 −b
(b) Infinite solution
c −c −1
So D = 0, Dx = Dy = Dz = 0
= 1[–1 – bc] – a[–bc + b] + a[–bc–c]
D=0→λ=3
= –1 – bc + abc – ab – abc – ac
6 1 1 6 1 1
= –1(ab + bc + ca + 1) = 0
Dx = 10 2 3 = 10 2 3
= ab + bc + ca + 1 = 0 µ 2 λ µ 2 3

Sol 7: sinq ≠ cosq ⇒ 6[0] + 1[3µ – 30] + [20 – 2µ]

Xcosp – ysinp + z = cosq + 1  ... (i) ⇒ (µ−10)5 = 0

xsinp + ycosp + z = 1 – sinq ... (ii) µ = 10

xcos(p + q) – ysin(p + q) + z = 2  ... (iii) (c) No solution → D = 0, Dx ≠ 0

cos(A + B) = cosA cosB – sinAsinB λ = 3, µ ≠ 10

sin(p + q) = sinpcosq + cosp sinq


Sol 9: x + y + z = 1
equation (i)2 + equation (ii)2
x + 2y + 4z = p
⇒ x2(sin2p+cos2p) + y2(cos2p+sin2p)
x + 4y + 10z = p2
–2xy cospsinp + 2xcospz – 2yzsinp + 2xysinpcosp
1 1 1
+ 2xzsinp + 2z2 + 2yzcosp
D= 1 2 4
= z + 1 – 2sinq + 2cosq
1 4 10
⇒ x2 + y2 + z2 + 2xycosp – 2yzsinp
D = 1[20 – 16] + 1[4 – 10] + 1[4 – 2] = 4 – 6 + 2 = 0
+ 2xzsinp + z2 + 2yzcosp
So for solution, Dx = Dy = Dz = 0
= 2 + 1 – 2sinq + 2cosq
1 1 1
From equation (iii) and (i)
Dx = p 2 4
= x2 + y2 + z2 + 2z (1 + cosq – z) + 2q(1 – sinq – z) z2
p2 4 10
= 3 – 2(sinq – cosq)
= 1[20 – 16] + 1[4p2 – 10p] + 1[4p – 2p2] = 0
= x2+y2+z2 + 2z(2 + cosq – sinq – 2z)
= 4 + 4p2 – 10p + 4p – 2p2 = 0
= 3 + 2(cosq – sinq)
2p2 – 6p + 4 = 0
For equation (iii)
p2 – 3p + 2 = 0
⇒ 2z(2 + cosq – sinq – 2z) = 1 + 2 |cosq – sinq|
p2 – 2p – p + 2 = 0
\x + y + z = 2
2 2 2
M a them a ti cs | 17.61

(p – 2) (p – 1) = 0 ⇒ p = 1 or 2 4x + 4y – z = 2 … (ii)
For p = 1 6x + 6y + 2z = 3 … (iii)
⇒ x + y + z = 1 ... (i) Assume x = l
x + 2y + 4z = 1  … (ii) Equation (iii), (ii) – (iii).(ii)
x + 4y + 10z = 1 … (iii) 7z = 0 → z = 0
Assume that x = k 2y = 1 + 2z – 2x = 1 – 2λ
Equation (ii) – ii(i) (x, y, z) = (λ, 1–2λ, λ)
–x + 2z = –1 If K ≠ 2
k −1
⇒ 2z = x – 1 ⇒ z = K 2 −2 
2  
D =  4 2K −1
(k − 1)
So y = 1 – z – x = 1 – k – 6 6 K 
2  
2 − 2k − k + 1 3 − 3k = K[2K2 + 6] + 2[–6 – 4K] – 2[24 – 12K]
y= =
2 2 = 2K3 + 6K – 12 – 8K – 48 + 24K
 3 − 3k k − 1  = 2K3 + 22K – 60 = 2(K3 + 11K – 30)
(x,y,z) =  k, , 
 2 2 
At K = 2
At p = 2 ⇒ 2(8 + 11(2) – 30) = 0
x + y + z =1  … (1) So (K – 2) is a factor
x + 2y + 4z = 2 … (2)
k 3 + 11k – 30
= K2 + 2K + 15
x + 4y + 10z = 4 … (3) k −2
Assume x = k D = 2(K – 2) (K2 + 2K + 15)
Equation (2) – 2(1)
1 2 −2
–x + 2z = 0  
Dx = 2 2K −1
k 3 6 K 
⇒ x = 2z = k ⇒ z =  
2
= 2K2 + 6 + 2[–3 – 2K] – 2[12 – 6K]
k
y=1–x–z=1–k–
2 = 2K2 + 6 – 6 – 4K – 24 + 12K
3k 2 – 3k = 2K2 + 8K – 24 = 2[K2 + 4K – 12]
=1– =
2 2 = 2[K2 + 6K – 2K – 12] = 2[K(K+6) – 2(K + 6)]

Sol 10: Kx + 2y – 2z = 1 = 2(K – 2) (K + 6)

4x + 2Ky – z = 2 Similarly, Dy = (K – 2) (2K + 3) and Dz = 6(K – 2)2


6x + 6y + Kz = 3 if K ≠ 2,

K 2 −2  x y z 1
= = =
  2(K + 6) 2K + 3 6(K − 2) 2(K + 2K + 15)
2
D =  4 2K −1 at K = 2 (given)
6 6 K 
 
Sol 11: (a) a, b, c, d are distinct no.
2 2 −2
a, b, c, d ∈ {1, 2, 3, 4, 5}
= 4 4 −1 = 0
ax + by = 1
6 6 2
cx + dy = 2
⇒ 2x + 2y – 2z = 1 … (i)
1 7 . 6 2 | Determinants

a b Sol 12: (a – t)x + by + cz = 0


D= = ad – bc … (i)
c d bx + (c – t)y + az = 0

1 b cx + ay + (b – t)z = 0
Dx = = d – 2b,
2 d Has non-trivial solution,
Dx d − 2b So D = 0
x= =
D ad − bc a−t b c
for least possible +ve value of x D= b c−t a =0
d – 2b = 1 (least natural number) c a b−t

(d, b) → (3, 1) or (5, 2) Assume D = a0t3 + b0t2 + c0t + d0 = 0

ad – bc should be maximum for least x –d0


So t1t2t3 =
a0
(a, b) → (3, 1) (ad – bc) → (3a – c)
At t = 0, D = d0
a,c ∈ {7, 4, 5}
a b c
Max. → 3(5) – 2 = 15 – 2
So d0 = b c a
1
x= c a b
13
If a, b → (5, 2), And a0 is coefficient of t3 = (–1)(–1)(–1) = –1

ad – bc → 5a – 2c, a b c
–d0
t1t2t3 = = d0 = b c a
a, c ∈ {1, 3, 4} –1
c a b
Max. 5a – 2c → 5(4) – 2(1) = 18
1 p
→x= = (min.) Sol 13: 3x – y + 4z = 3
18 q
X + 2y – 3z = –2
p + q = 1 + 18 = 19
6x + 5y + lz = –3,
(b) x + ay = 3 and ax + 4y = 6 → x > 1, y > 0
3 −1 4
1 a
D= =4–a , 2 D = 1 2 –3
a 4
6 5 λ
3 a
Dx = = 12 – 6a ⇒ 3(2λ + 15) –1[–18 – λ) + 4[5 – 12]
6 4
⇒ 6λ + 45 + 18 + λ – 28 = 7λ + 35 = 7(λ + 5)
1 3
Dy = = 6 – 3a, D = 7(λ + 5)
a 6
3 −1 4
Dx 6(2 − a)
x > 0, > 101→ > 101 Dx = –2 2 –3
D (2 − a)(2 + a)
–3 5 λ
6
> 1,
01 = 3[2λ + 15] + 1[–2λ – 9] + 4[–10 + 6]
(2 − a)
= 6λ + 45 – 2λ – 9 – 16
2 + a < 6 → a = 1, 3
= 4λ + 20 = 4(λ + 5);
Dy3(2 − a) 3 1
y= = = = + ve D 4(λ + 5) 4
D 6(2 − a) 6 2 = x
x= =
D 7(λ + 5) 7
So a is 1 and 3 3 3 4
1+3=4 Dy = 1 −2 −3
6 −3 λ
M a them a ti cs | 17.63

= 3[–2λ – 9] + 3[–18 – l] + 4[–3 + 12] 1 a –a3


= –6λ – 27 – 54 – 3λ + 36 Dx = 1 b –b3
= –9λ – 45 = –(λ + 5) 1 c −c3
Dy –9(λ + 5) –9 = –(a + b + c) (a – b) (b – c) (c – a);
⇒y= = =
D 7(λ + 5) 7
\x = –(a + b + c)
3 −1 3
1 –a3 a2
Dz = 1 2 −2
Dy = 1 –b3 b2
6 5 −3
1 –c3 c2
= 3[–6 + 10] + 1[–3 + 12] + 3[5 – 12]
= (ab + bc + ca) (a – b)(b – c) (c – a)
= 12 + 9 – 21 = 0,
Dz \y = [ab + bc + ca]
z= =0
D –a3 a a2
So x, y, z is not dependent on λ Dz = −b3 b b2
(if λ ≠ –5) −c3 c c2
At λ = –5 = –abc(a – b) (b – c) (c – a)
3x – y + 4z = 3  … (i) \ z = –abc
x + 2y – 3z = –2 … (ii)
6x + 5y – 5z = –3  … (iii) Sol 15: (a) ax – y + z = α

Assume z = k, (iii) – (ii)(i) x – ay + z = 1

7y – 13z = –9 x – y + az = 1

13k − 9 D = a[–a2 + 1] –1 [1 – a] + [–1+a]


⇒y=
7 = –a3 + α – 2 + 2a
2 4 − 5k = (–a3 + 3α – 2) = –(a3 – 3α + 2)
So, x = 3z – 2y – z = 3k – (13k – 9) – z =
7 7
At α = 1
 4 − 5k 13k − 9 
(x, y, z)  , ,k  D = –(1 – 3+ 2) = 0
 7 7 
So (α – 1) is a factor
Sol 14: z + ay + a2x + a3 = 0
α3 − 3α + 2
= a2 + α – 2
z + by + b x + b = 0
2 3
α −1
z + cy + c2x + c3 = 0 So D = – (α – 1) (a2 + α – 2)
 a3  = –(α –1)(a2 + 2α – α – 2)
 
Now, c = b3  = –(α – 1)[α(α + 2) – 1(α + 2)]
 3
 c  = –(α – 1) (α – 1) (α + 2)
α ∈ [–10, 10]
1 a a2
So, α has an integral value
D = 1 b b2 = (a – b) (b – c) (c – a)
1 c c2 α −1 1
Dx = 1 −α 1
Dx Dy Dz 1 −1 α
x= ,y= ,z=
D D D
So x = 1,
Dx = –(α – 1)2(α + 2)
1 7 . 6 4 | Determinants

α α 1 (c) a, b ∈ {0, 1, 2, ……., 10}


Dy = 1 1 1 = 0, x+y+z=4
1 1 α 2x + y + 3z = 6
α –1 α x + 2y + az = b
Dz = 1 – α 1 =0
1 1 1
1 –1 1
D= 2 1 3
(a) Unique solution, 1 2 a
So D≠0 →α ≠1, –2 = 1(a – 6) + 1(3 – 2a) + 1(4 – 1)
Number of values for α in = a – 6 + 3 – 2a + 3 = – 9
[–10, 10] = 21 – 2 = 19 = L
4 1 1
(b) Number solution is not possible for every value of α,
Dx= 6 1 3
system has atleast one solution. So M = 0
b 2 a
(c) Infinite solution → D = 0
= 4(a–6) + 1(3b–6a) + 1(12–b)
α = 1, –2 → N = 2
= 4a – 24 + 3b – 6a + 12 – b
L – M + N = 19 + 2 = 21
= –2a + 2b – 12
(b) 2x + 3y – z = 0
1 4 1
3x + 2y + kz = 0
Dy= 2 6 3
4x + y + z = 0
1 b a
Has non-trivial solution
= 6a – 3b + 4[3 – 2a] + b – 6
2 3 −1 = 6a – 3b + 12 – 8a + b – 6
So, D = 0 ⇒ 3 2 k
= –2a – 2b + 6
4 1 1
1 1 4
= 2[2 – k] + 3[4k – 3] – 1[3 – 8] = 0
Dz= 2 1 6
4 – 2k + 12k – 9 + 5 = 10k = 0
1 2 b
⇒k=0
= b – 12 + 6 – 2b + 4[4 – 1]
⇒ 2x + 3y – z = 0 … (i)
= b – 12 + 6 – 2b + 12
3x + 7y = 0  … (ii)
= –b + 6
4x + y + z = 0  … (iii)
(i) Unique solution so D ≠ 0
(iii) – (ii) (i) →
→a≠0
– 5y + 32 = 0 → 3z = 5y
∴ a ∈ {1, 2, ………. , 10},
–2y
3x = – 2y → x = , b ∈ {0, 1, ………. , 10}
3
5 L = 10 × 11 = 110
z= y,y=y
3 (ii) Number solution D = 0, a = 0
x, y, z are integer , so at for x and z to be integer x = n Dx ≠ 0 → 2b ≠ 12 → b ≠ 6,
2
= – y and Dy ≠ 0 → b ≠ 3
3
3n M = 1(11 – 2) = 9
→y= (also an integer)
–2
(iii) Infinite solution D=0 → a=0,
So at n = –2, –7y = 3, z = 5 (minimum +ve value)
D x = Dy = Dz = 0
M a them a ti cs | 17.65

But Dx and Dz can’t be zero at same times, so no possible = [sin x (p+d – p) + a [sinx (p – d – p – d)]
common solution N = 0
+ a2[sin x (p – p + d)]
L + M – N = 110 + 9 – 0 = 119
= sin x d + a sin (– 2d) + a2 sin dx
2 It dose not depend upon p
–7 5 + 3i – 4i
3
Sol 16: 5 – 3i 8 4 + 5i x3 + 1 x 2 x x3 x2 x 1 x2 x
2 3
(c) y + 1 y 2
y = y 3
y 2
y + 1 y 2
y
+ 4i 4 – 5i 9
3 3
z +1 z 2
z z 3
z 2
z 1 z 2
z
2
(a) Assume z1 = 5 + 3i, z2 = + 4i
3
z3 = 4 + 5i x2 x 1 1 x2 x
2 2
= xyz y y 1 + 1 y y
(z3)2 = 42 + 52 = 41
2 2
z z 1 1 z z
–7 z1 z2
= (xyz + 1) (x – y) (y – z) (z – x) = 0
⇒ z1 8 z3
z2 z3 9 (given) x, y, z are all different
So (xyz + 1) = 0 ⇒ xyz = – 1
= –7[72–z3 z ]+z1[z2z3 – 9 z 2 ]+ z 2 [ z 1 z 3 – 8z2]
3

= –7[72 – 41] + (5 + 3i) a2 + 2a 2a + 1 1


Sol 17: (a) 2a + 1 a + 2 1 = (a – 1)3
 2  2 
 + 4i  ( 4 + 5i) – 9  – 4i   3 3 1
 3  3 
R1 → R1 – R3, R2 → R2 – R3
2   2 
+  – 4i  (5 – 3i) (4 – 5i) – 8  + 4i  
3   3  a2 + 2a – 3 2a + 1 0
= – 7 (31) + (5 + 3i) 2a – 2 a–2 0
3 3 1
8 10 
 + 16i + i – 20 – 6 + 36i = (a – 1) (a2 + 2a – 3) – 4(a – 1)2
3 3 
= (a – 1) [(a2 + 3a – a – 3) – 4 (a – 1)]
2   16 
+  – 4i  20 – 15 – 12i – 25i – – 32i
3   3  (a – 1) [(a – 1) (a + 3) – 4(a – 1)]

 –70 160i  2   1  = (a – 1)2 [a + 3 – 4] = (a – 1)3


= –217 + (5 + 3i)  +  +  – 4i   – – 69i
 3 3  3   3 
1 1 1 0 0 1
Coefficient of i (b) x y z ⇒ x–2 y–2 2
800 4 804 x3 y3 z2 x3 – z 3 y 3 – z3 z3
= – 70 + – 46 + = – 106 +
3 3 3
0 0 1
1 a a2 = (x – z) (y – z) 1 1 2
(b) cos(p – d)x cospx cos(p + d)x x2 + y 2 + xz y 2 + z 2 + yz z3
sin(p – d)x sinpx sin(p + d)x
∴ a2 – b3 = (a – b) (a2 + b2 + ab)
= 1[cos px sin (p+d)x – cos(p+d)x sin px] = (x – z) (y – z) (y2 + z2 + yz – x2 – z2 – xz)
+ a[cos(p+d)x sin (p – d)x – ws(p – d)x sin (p + d)x] = (x – z) (y – z) [ z (y – x) + (y2 – x2)
+ a2[cos (p –d)x sin px – cos px sin (p –d)x] = (x – y) (y – z) (z – x) (x + y + z)
1 7 . 6 6 | Determinants

22 0 1
x 1 –3 / 2 2
1 2 0 = 4[16] + [1] = 65
Sol 18: (a) f(x) = 2 2 1 x>1
1 0 1 22
0 1/2
x –1
a b c b+c c+a a+b
 1  3 2  Sol 19: D = c a b , D’ = a + b b + c c + a
f(x) = x [1 – 0] + 1  – 1 +  
x – 1  2 x – 1 b c a c+a a+b b+c
1 3
=x+ –1+ b c+a a+b c c+a a+b
x –1 x –1
D’= a b + c c + a + b b + c c + a
3 (x – 1)2 + 3 c a+b b+c a a+b b+c
= (x – 1) + =
x –1 x –1
C2 → C2 + C1 – C3, C2 → C2 – C1,
2 2
x + 1 + 3 – 2x x – 2x + 4 C3 → C3 – C1C3 → C3 – C2
= =
x –1 x –1
b c a c a b
3 3
F’(x) = 1 – ⇒ 01 = D’ = a b c + b c a
(x – 1)2 (x – 1)2
c a b a b c
⇒ (x – 1)2 = 3 ⇒ x = 1 ± 3 After swapping rows according to D

But x . so x = 1 ± 3 a b c a b c
D’ = c a b + c a b = 2D
–6
f’’(x) = , at x = 1 + 3 b c a b c a
(x – z)3
–6
f”(x) = > 0 so minima 1 + a2 – b2 2ab –2b
3 3 2 2
Sol 20: 2ab 1–a +b 2a
3
f(1 + 3)= 3 + = 2 3 2b –2a 1 – a2 – b2
3
But if x is integer for min. volue of f(x) C1 → C1 – BC3, C2 → C1 + AC3

⇒ x = [1 + 3]=2 1 + a2 + b2 0 –2b
3 0 2
1–a +b 2
2a
F(x) = f(2) = 1 + =4
1 2 3 3 2
b+b+a b+b –a + a – ab 1 – a2 – b2
(b) a2 + b2 + c2 + ab + bc + ca ≤ O ∀ a,b,c ∈ R
R3 → R3 + aR2 – bR1
(a + b + 2)2 a2 + b2 1
1 (b + c + 2)2 b2 + c2 1 + a2 + b2 0 –2b
c2 + b2 1 (c + a + 2)2 = 0 1 + a2 + b2 2a
0 0 1 + a2 + b2
(a + b) + (b + 1) + (c + a) ≥ 0
2 2 2

= (1 + a2 + b2)3
(always & square is +ve)
= 2 (a2 + b2 + c2 + bc + ca + ab)
sinx sin(x + h) sain(x + 2h)
Its given that a2+b2+c2 + bc + ca + ab ≤ 0
Sol 21: f(x) = sin(x + 2h) sinx sin(x + h)
So 0 ≤ a2 + b2 + c2 + ab + ca + ab ≤ 0 sin(x + h) sin(x + 2h) sinx
⇒ (a + b)2 + (b + 1)2 + (c + a)2 = 0
sin (A + B) = sin A cos B + cos A sin B
⇒a=b=c=0
M a them a ti cs | 17.67

sin (x + nh) = sin x cos (nh) + sin nh cos x ((γ + α – β – δ)2 – (α + β – γ – δ)2)

⇒ lim sin (x + nh) = (sin x ) 1+ (nh) cos x [(β + γ – α – δ)2 + (d2 + b2 – γ – δ)2
h→0
(γ + α – β – δ)2 – (α + β – ρ – δ)2]
sinx sinx + ncos x sinx + 2hcos x
= – 2 (α – β)2 (α – γ)2 (α – γ)2 (β – γ)2
⇒f(x) sinx + 2hcos x sinx sinx + hcos x
sinx + hcos x sinx + 2hcos x sinx (β – δ)2 (γ – δ) (– 1)6

C1 → C1 → C3, C2 → C2 – C3 = –64(α – β)(α – γ)(α – δ)(β – γ)(β – γ)(γ – δ)

–2hcos x –hcos2 x sinx + 2hcos x Sol 23: x3 – 3x2 + 2 = 0


f(x) = hcos x –hcos x sinx + hcos x
At x = 1 ⇒1 – 3 + 2 = 0.
cos x 2hcos x sinx
So (x – 1) is a factor of x3 – 3x2 + 2 = 0
–2cos x – cos x sinx + 2hcos x
(b + c)2 a2 a2
f(x) = h cos x
2 – cos x sinx + hcos x
b2 (c + a)2 b2
cos x 2cos x sinx
c2 c2 (a + b)2
–2cos x – cos x sinx
f(x) ⇒ x3 – 3x2 + 2 = (x – 1) (x2 – 2x – 2)
lim = cos x – cos x sinx
h→0 h2 a = 1, and bc = – 2, b + c = 2
cos x 2cos x sinx

R1 → R1, R3, R2 → R2 – R3 ⇒ bc = 1 ± 3 , c2b2 = 4 ± 2 3

–3cos x –3cos x 0 22 1 1
= 0 –3cos x 0 = 4 + 2 3 (2 – 3) 2
4+2 3
cos x 2cos x sinx 4–2 3 4–2 3 (2 + 3)2
= sin x (9 cos2x) = sin x (9 – 9sin2x) 4 1 1
= 9 sin x – 9sin3x = 3 (3sin x – 3 sin3x) = 4+2 3 7– 4 3 4+2 3

= 3 [sin 3x + sin3x] = k(sin 3x + sin3x) 4–2 3 4–2 3 7+4 3

⇒K=3 = 4[49 – 48] – [16 – 12] – 1 [16 + 12) + 28

(β + γ – α – δ)4 (β + γ – α – δ)2 1 + 30 3 + 24]+[16 – 12– (28 + 24 – 3 (30) = – 108

Sol 22: ( γ + α – β – δ)4 ( γ + α – β – δ)2 1


x + 2 2x + 3 3x + 4
( α + β – γ – δ )4 (α + β – γ – δ)2 1
Sol 24: (a) 2x + 3 3x + 4 4x + 5 = 0
R1 → R1 – R3, R2 → R2 – R3 3x + 5 5x + 8 10x + 17

x+2 4x + 6 3x + 4
1
4
(β + γ – α – δ) – (α + β – γ – δ) 4 4
(β + γ – α – δ) – (α + β – γ – δ) 2
0 = 2x + 3 6x + 8 4x + 5
2
( γ + α – β – δ )4 – ( α + β – γ – δ )4 ( γ + α – β – δ)2 – (α + β – γ – δ)2 0 3x + 5 10x + 16 10x + 17
( α + β – γ – δ )4 (α + β – γ – δ)2 1
c2→ c2 – c1 – c3
= ((β + γ – α – δ)2 – (α + β – γ – δ)2
x+2 0 3x + 4
– ((γ + α – β – δ)2 – (α + β – γ – δ)2) 1
= 2x + 3 0 4x + 5
2
3x + 5 –3x – 6 10x + 17
(β + γ – α – δ)2 + (α2 + β – γ – δ)2 1 0
2 2
( γ + α – β – δ) + (α + β – γ – δ) 1 0 [3x + 6]
4 2 ⇒ [(3x + 4) (2x + 3)–(x + 2) (4x + 5)]= 0
(α + β – γ – δ) (α + β – γ – δ) 1 2
= ((β – γ – δ – δ)2 – (α + β – γ – δ)2)
1 7 . 6 8 | Determinants

⇒ (3x + 6)[6x2 + 17x + 12 – 4x2 – 13x – 10]=0 x2 = a2 + b2 + c2 – (ab + bc + ca)


⇒ (3x + 6) [2x2 + 4x + 2] = 0 \ a + b + c = 0 ⇒ (a + b + c)2 = 0
⇒ (x + 2) (x2 + 2x + 1) = 0 a2 + b2 + c2 + 2 (ab + bc + ca)
⇒ (x + 2) (x + 1)2 = 0 –(a2 + b2 + c2 )
⇒ ab + bc + ca =
⇒ x = – 2, – 1 2

x – 2 2x – 3 3x – 4 a2 + b2 + c2 3
x2 = a2 + b2 + c2 + = (a2 + b2 + c2)
(b) x – 4 2x – 9 3x – 16 = 0 2 2
x – 8 2x – 27 3x – 64 3 2
x=± (a + b2 + c2 )
2
R1 → R2 – R3, R2 → R2 – R3
a c b
6 24 60 x = 0 c b a = a3 + b3 + c3 − 3abc = 0
⇒ 4 18 48 =0 b c a
x – 8 2x – 27 3x – 64
∴a+b +c =0
1 4 10
⇒ 2 9 24 =0 a
x – 8 2x – 27 3x – 64 Sol 26: xX33−–5x
5x2++3x
2
3x−–11==00 b

= 9(3x – 64) – 24(2x – 27) + 4 [24 (x – 8) c


– 2 (3x – 64)] + 10 [2(2x – 27) – 9(x – 8)] a b c
= (6 – 48 + 96 – 24 + 40 – 90) (x – 4) = 0 a – b b – c c – a C1 → C1 + C2 + C3
b+c c+a a+b
⇒x=4
a+b+c b c
Sol 25: a + b + c = 0 = 0 b–c c–a
a–x c b 2(a + b + c) c + a a + b
c b–x a =0
1 b c 1 b c
b a c–x
=5 0 b–c c–a =5 0 b–c c–a
c1 → c1 + c2 + c3 2 c+a a+b 2 a+c a+b
a+b+c – x c b
= 5[(b – c) (a + b) + (a – c) (a + c) + 2 (bc – ab – bc + c2)]
a+b+c – x b – x a
a+b+c – x a c–x = 5[ab – ac + b2 – bc + a2 – c2 – 2ab + 2c2]
= 5[a2 + b2 + c2 – (ab + bc + ca)]
1 c b
a2 + b2 + c2 = (a + b + c)2 – 2(ab + bc +ca)
= (a + b + c – x) 1 b.x a =0
1 a c–x = 25 – 2 (3) = 19
= 5[19 – 3] = 5. 16 = 80
a + b + c = 0 (a + b + c – x) = – x = 0

1 c b a2 + λ ab ac
1 b–x a Sol 27: ab 2
b +λ bc
1 a c–x 2 2
ac b c +λ
= (b – x) (c – x) – a + c (a – c + x) + b (c – b + x] = 0
2

(a2 + λ ) ab2 ac2


bc – x (b + 1) + x2 – a2 + a (– c2 + cx + ba – b2 + bx = 0 1
= a2b b(b2 + λ ) bc2
x + x(b + c – b – c) = a + b + c – (ab + bc + ca)
2 2 2 2 abc
a2c b2 c c(c2 + λ )
M a them a ti cs | 17.69

a2 + λ b2 c2 C A B A B C
= tan cot – 1 – tan cot + tan cot
abc 2 2 2 2 2 2 2 2 2
= a b +λ c
abc 2
a b 2 2
c +λ B C A B A C
– cot tan + tan cot +1 – tan cot
2 2 2 2 2 2
R1 → R1 – R3, R2 → R2 – R3 A 1
We know that tan =
2 A
λ 0 –λ 1 0 –1 cot
2
2
= 0 λ –λ = λ 0 1 –1
C B C A
a2 b2 c2 + λ a2 b2 (c2 + λ ) tantan tan tan
= 2 – 2 – 2 + 2
A A B B
= l2 (c2 + λ + b2 – 1[– a2]) tan tan tan tan
2 2 2 2
= l2 (a2 + b2 + c2 + λ)
A B
tan tan
– 2 +1–1+ 2 =0
a2 b2 c2 C C
tan tan
Sol 28: = 4 a b c 2 2
1 1 1
1  C B 1  A C
⇒  tan – tan  +  tan – tan 
C2 → C2 − C1 ,C3 → C3 − C1 A 2 2 B 2 2
tan  tan 
2 2
a2 b2 − a2 c2 − a2
1  B A
4 a b−a c−a = =
 ( ) ( )
4  b2 − a2 ( c − a) − c2 − a2 (b − a+) 
 C tan – tan  = 0
 2 2
1 1 1 tan
2
a2 c2 − a2
a c−a = =
=
 ( ) (
4  b2 − a2 ( c − a) − c2 − a2 (b − a) 
 ) It can only happen when two angles are equal.

1 ⇒ ∆ABC is isosceles
=-4 ( c − a)(b − a)(b − c )
= 4 ( c − b )(b − c )( c − a)

Exercise 2
Sol 29:
Single Correct Choice Type
A B C
cot cot cot
2 2 2 m
B C C A A B 2r – 1 Cr 1
tan + tan tan + tan tan + tan =0 2 m
2 2 2 2 2 2 Sol 1: (A) Dr = m –1 2 1+m
1 1 1 sin (m ) sin m sin2 (m + 1)
2 2 2

m
C1 → C1 – C3, C2 → C2 – C3 ∑ Dr =
r =0
A C B C C
cot – cot cot – cot cot m m
2 2 2 2 2
C A C B A B
∑ (2r – 1) ∑ m Cr m+1
=r 0=r 0
tan – tan tan – tan tan + tan
2 2 2 2 2 2 2
(m + 1)(m – 1) (m + 1)2m (m + 1)2
0 0 1
(m + 1)sin2 (m2 ) (m + 1)sin2 m (m + 1)sin2 (m + 1)

 C B  A C (m + 1)(m – 1) 2m (m + 1)
=  tan – tan  cot – cot  2 m
 2 2  2 2 = (m + 1)(m – 1) (m + 1)2 (m + 1)2
(m + 1)sin2 m2 (m + 1)sin2 m (m + 1)sin2 (m + 1)
 B C  C A
–  cot – cot  tan – tan 
 2 2  2 2
1 7 . 7 0 | Determinants

Common (m + 1) from C1, C3 and R2 C3 → C3 + C1

m–1 2m 1 bc 1 ab + bc + ca
= abc ac 1 ab + bc + ca
= (m+1)3 m – 1 2m 1 =0
ab 1 ab + bc + ca
sin2 m2 2 2
(m + 1)sin m sin (m + 1)
bc 1 1
= (abc) (ab + bc + ca) ac 1 1 = 0
1 cos(β – α ) cos( γ – α )
ab 1 1
Sol 2: (D) D = cos(α – β) 1 cos( γ – β)
cos(α – γ ) cos(β – γ ) 1
Sol 4: (A)
D = 1 – cos (β – γ) cos (γ – β) + cos(β – α)
mx mx – p mx + p
[cos (γ – β) cos (α – γ) – cos (α – β)] f’(x) = n n+p n–p
+ cos (γ – α) [cos (α – β) cos (β – γ) mx + 2n mx + 2n + p mx + 2n – p
– cos (α – γ)] C2 → C2 + C3
D = 1 – cos2(β – γ) + cos (β – α) cos (γ – β)
mx 2mx mx + p
cos (α – γ) – cos2 (β – α) + cos (γ – α)
f’(x) = n 2n n–p
cos (α – β) cos (β – γ) – cos2 (γ – α) mx + 2n 2(mx + 2n) mx + 2n – p
cos(β – γ )
D=1+2 [cos (γ – β) + cos C2 → C2 – 2C1
2
(γ – β – 2α)] – cos2 (γ – α) – cos2(β – α) – cos2(β – γ) mx 0 mx + p
f’(x) = n 0 n–p =0
 cos2(β – α ) + cos2( γ – α ) 
D = 1+cos2(β–γ)+   mx + 2n 0 mx + 2n – p
 2 
y = f(x)
– cos2(β – γ) – cos2(γ – α) – cos2 (β – α)
y’ = 0
=1+
1
2
(
2cos2 (β – α ) – 1 + 2cos2 ( γ – α ) – 1 ) y=K
It is a straight line parallel to x–axis.
– cos2 (β – α) – cos2(γ – α)
2
= 1 –   = cos2(β – α) + cos2 (γ – α) x − 1 (x − 1)2 x3
2
Sol 5: (A) D(x) = x − 1 x2 (x + 1)3
– cos2 (β – α) – cos2 (γ – α) = 1 – 1 = 0
x (x + 1)2 (x + 1)3
Assume D(x) a0 + a1x + …
b2 c2 bc b + c
Sol 3: (A) D = c2a2 ca c+a D’(x) = a1 + 2a2x

a2b2 ab a + b At x = 0 D’(0) = a1

+1 (x – 1)2 x3 x – 1 2(x – 1) x3
ab2c2 abc a(b + c)
1 D’(x) = +1 x2 (x + 1)3 + x – 1 2x (x + 1)3
= bc2a2 abc b(a + c)
abc 2 2 1 (x + 1)2 (x + 1)3 x 2(x + 1) (x + 1)3
ca b abc c(a + b)
x – 1 (x – 1)2 3x2
bc 1 a(b + c)
abc.abc + x –1 x2 3(x + 1)2 at x = 0
= ac 1 b(a + c)
abc x (x + 1)2 3(x + 1)2
ab 1 c(a + b)
M a them a ti cs | 17.71

1 1 0 –1 –2 0 –1 1 0 = 1 + (sin2x + cos2x) + 4 sin2x = 2 + 4 sin2x

D’(0) = 1 0 1 + –1 0 1 + –1 0 3 For max value


1 1 1 0 2 1 0 1 3 Sin 2x = 1
⇒ 2 + 4 sin 2x = 2 + 4 = 6
= 1[–1] + 1 [1 – 1] – 1[–2] –2 (1) –1 (–3) + 1[3]
=–1+0+2–2+3+3=–1+6=5 x2 + 3x x – 1 x + 3
Sol 8: (C) x + 1 2 – x x – 3 = px4+qx3+rx2 +5x+t
y+z z y
x – 3 x + 4 3x
Sol 6: (D) D = z z+x x , |D| = 8
y x x+y At x = 0

R1 → R1 – R2 – R3 0 –1 3
R2 → R2 – R3 1 2 –3 = t
–3 4 0
0 –2x –2x
D= z–y z –y t = 1 [– 9] + 3 [4 + 6] = 30 – 9 = 21
y x x+y

0 1 1 a2 + 1 ab ac
D = (–2x) z – y z –y Sol 9: (A) D = ba 1+b 2
bc
y x x+y 2
ca cb c +1
C2 → c2 – c3
a3 + a a2b a2c
0 0 1 1
= b2 a b + b3 b2 c
D = (–2x) z – y z + y –y =(–2x) [– y(z – y) – y (z+y) abc
y –y x + y c2 a c2b c + c3

= – 2x [–yz + y2 – y2 – yz] = }4xyz| = 8 given |xyz| = 2 R1 → aR1, R2 → bR2, R3 → cR3

For → 2 → (2, 1, 1) (–2, 1, –1) (2, – 1, – 1) 1 + a2 a2 a2


abc
3! 3! D= b2 1 + b2 b2
⇒ + 3! + = 12 abc
2! 2! c2 c2 1 + c2
For → –2→ (2, 1,–1), (–2,–1,–1), (–2,1,1) = 12
C1 → C1 – C3, C2 → C2 – C3
3 + 6 + 3 = 12
1 0 a2
Total solution = 12 + 12 = 24
0 1 b2 = 1[1 + c2 + b2] + a2 [+1]
1 + sin2 x cos2 x 4 sin2x –1 –1 1 + c2
Sol 7: (C) f(x) = sin2 x 1 + cos2 x 4 sin2x
2 2 = 1 + a2 + b2 + c2
sin x cos x 1 + 4 sin2x

R1 → R1 – R3, R2 → R2 – R3 Sol 10: (A) α + β + γ = π

sin(α + β + γ ) sin β cos γ


1 0 –1
sin β 0 tan α
f(x) = 0 1 –1
cos(α + β) tan α 0
sin2 x cos2 x 1 + 4 sin2x
sin π = 0
= 1 + 4 sin 2x + cos2x – 1(– sin2x)
α + β = π – γ, cos (π – γ) = – cos γ
1 7 . 7 2 | Determinants

0 sin β cos γ For infinite solution D= 0, Dx = Dy = Dz = 0


– sin β 0 tan α P + 4µ – 3pµ + 2 = 0
– cos α – tan α 0 4 3 3
  sin α     sin α   Dx = 3 p 2 = 4[p–8] + 2[6–3] + 3 [12 – 3p]
= sin β  –   cos γ  + cos γ sin β  
  cos α     cos α   3 4 1

sin β sin α cos γ = 4p – 32 + 6 + 36 – 9p = 0


=
cos γ → p = 10/5 = 2

a11 a12 a13 1 4 3


Sol 11: (C) a21 a22 a23 , aij ∈ {0, 1} Dy = 1 3 2 = 1[3–6] + 4[2µ–1] +3[3–3m]=0
a31 a32 a33 µ 3 1

– 3 + 8µ – 4 + 9 – 9µ = 0
1 0 1
2=µ
0 1 1 ⇒ 1 [– 1] + 1 [– 1] = – 2
1 1 0 For equation (i) p + 4(2) – 3p (2) + 2 = 0
P + 8 – 6p + 2 = 0
0 1 1
1 0 1 = – 1 [– 1] + 1 [1] = 2 → p = 10/5 = 2
1 1 0 1 2 4
1 1 1 1 1 1 Dz = 1 p 3
1 1 1 = 0, 0 1 1 µ 4 3
1 1 1 1 1 0 = 3p – 12 + 2(3µ – 3) + 4 (4 – pµ)
= 1 [–1] – 1[–1] + 1 [– 1] = 3p – 12 + 6µ – 6 + 16 – 4pµ
Cannot be → 3 = 3p + 6µ – 4pµ – 2
D = a11 a22 a33 + a12 a23 a31 + a21 a32 a13 – a31 a22 a13 – a32 At p = 2, µ = 2
a23 a11 – a21 a12 a33 < 3 ⇒ 3(2) + 6(2) – 4(2) (2) – 2
As, for it to be 3, atleast one terms must be 0 but there ⇒ 6 + 12 – 16 – 2 = 0
sum would not be 3
At p = 2, µ = 2, system has infinite solutions.

Sol 12: (D) Order 3 × 3


Sol 14: (B) ax – by = 2a – b
First column consists of sum of 2 terms
(c + 1)x + cy = 10 – a + 3b
2nd ‘’3’’
For infinitely many solution
3rd”4”
Total no. of determinats = 2.3.4 = 24 a –b
D= = 0 ac + b (c + 1) = 0
c +1 c
Sol 13: (D) x + 2y + 3z = 4 ac + bc + b = 0 …(i)
x + py + 2z = 3
2a – b –b
mx + 4y + z = 3 Dx = 0
10 – a + 3b c
1 2 3 4
  = c(2a – b) + b (10 – a + 3b) = 0
D= 1 p 2 ,C= 3 
µ 4 1 3  2ac – bc + 10b – ba + 3b2 = 0 … (ii)
 
a 2a – b
D = p – 8 + 2 [2µ 4 – 1] + 3 [4 – pµ) Dy = 0 =
c + 1 10 – a + 3b
D = p – 8 + 4µ – 2 + 12 – 3pµ = p + 4µ – 3pµ + 2
M a them a ti cs | 17.73

⇒ a(10 – a + 3b) + (2a – b) (–1 – c) = 0 And a, b, c ≠ 1 , So


⇒ 10a – a2 + 3ba – 2a – 2ac + b + bc = 0 1
–1 0
At x = 1, y = 3 1–a
1
a – 3b = 2a – b 0 –1 =0
1–b
o = a + 2b ⇒ a = – 2b c
1 1
c + 1 + 3c = 10 – a + 3b( – a = 2b) 1–c
4c = 9 + 2b + 3b = 9 + 5b  –c 1  1
⇒ – 1 –  + 1 = 0
4c = 9 + 5b  1 – c 1 – b  1 –a 
In equation (i) ac + bc + b = 0
1 c +1 –1 1
(9 + 5b) (9 + 5b) ⇒ + + =0
(– 2b) +b +b=0 1–a 1–c 1–b
4 4
– 18b – 10b2 + 9b + 5b2 + 4b = 0 1 1 1 (1 – c)
⇒ + + – =0
1–a 1–b 1–c (1 – c)
– 5b2 – 5b = 0
1 1 1
b2 + b = 0 ⇒ + + =1
1–a 1–b 1–c
b = – 1 or o
a = 2 or 0 cos(θ + φ) – sin(θ + φ) cos2 φ
c = 1 or 9/4 Sol 16: (B) sin θ cos θ sin φ
(a, b, c) → exactly ⇒ (–1, 2, 1) or (0, 0, 9/4) – cos θ sin θ cos φ

⇒ cos (θ + φ) [cos θ cos φ – sin θ sin f] +


Sol 15: (C) ax + y + z = 0
sin (θ + φ) [ sin θ – cos f] + sin φ cos q]
x + by + z = 0
+ cos2φ (sin2θ + cos2θ)
x + y + cz = 0 a, b, c ≠ 1
⇒ cos2 (θ + θ) + sin2 (θ + φ) + cos 2φ
a 1 1 0
= 1 + cos 2φ
D= 1 b 1,C= 0
1 1 c 0 So determinant is only dependent of φ
So, Dx = Dy = Dz = 0
Sol 17: (D) x sin θ – y cos θ + (λ + 1)z = 0
But system has nontrival solution
x cos θ + y sin θ – λ . z = 0
So, D = 0 and a, b, c ≠ 1
λ x + (λ + 1)y + z cos θ = 0
a 1 1
D= 1 b 1 sin θ – cos θ λ + 1
1 1 c D = cos θ sin θ –λ
λ λ + 1 cos θ
C1 → C1 – C3, C2 → C2 – C3
D = sin θ [sin θ cos θ + l2 + l]
a–1 0 1
D= 0 b –1 1 + cos (cos2θ + l2] + (λ + 1) (λ cos θ + cos θ – λ sin θ)
1–c 1–c c D = (sin2θ + cos2θ) cos θ + sin θ (l2 + λ
1 – l2 – λ) + cosθ (l2 + l2 + λ + λ + 1)
–1 0
1–a D = cos θ (2l2 + 2λ + 2) = cos θ [l2 + 1 + (λ + 1)2]
1
= (1 – a) (1 – b) (1 – c) 0 –1 =0 So for D = 0 ( System has infinite solution)
1–b
c cos θ = 0, θ ∈ (2n + 1) π/2, λ ∈ R, n ∈ I
1 1
1–c
1 7 . 7 4 | Determinants

Sol 18: (A) a2x – ay = 1 – a


(n + 3)(n + 2)
–(n + 2) n+3
1 – a 2
bx + (3 – 2b)y = 3 + a C =  
3 + a –1 n+3 1 =–1
Unique solution x = 1, y = 1 0 1 0

a2 –a
D= , at (x, y) ⇒ (1, 1)
b 3 – 2b Sol 20: (A) lx – y + cos θ z = 0
a2 – a = 1 – a 3x + y + 2z = 0
a2 = 1 ⇒ a = ± 1 cos x + y + 2z = 0
And b + 3 – 2b = 3 + a 0 ≤ θ ≤ 2π
3–b=3+a 0 
 
a=–b C = 0 
0 
So (a, b) ⇒ (1, – 1) or (–1, 1)  

At (–1, 1) ⇒ x + y = 1 – (–1) = 2 So Dx = Dy = Dz = 0

x+y=2 λ –1 cos θ
And x + (3 – 2)y = 3 – 1 = 2 D= 3 1 2
cos θ 1 2
x+y=2
Both equations are same so, D =0 at (–1, 1) For non-trivial solution
So it is not unique solution D = 0 ∴Dx = Dy = Dz = 0
(a, b) ≠ (–1, 1) (a, b) = (1, – 1) l[2–2]+1[6 – 2 cos q]+cos θ [3 – cos q] = 0
⇒ 6 – 2 cos θ + 3 cos θ – cos2θ = 0
n+ 2 n+ 3 n+ 4
Cn Cn+1 Cn+ 2 ⇒ cos2θ – cos2θ – 6 = 0
n+ 3 n+ 4 n+5
Sol 19: (A) D = Cn+1 Cn+ 2 Cn+3
⇒ cos2θ – 3cos θ + 2 cos θ – 6 = 0
(n+ 4) n+5 n+ 6
Cn+ 2 Cn+3 Cn+ 6
⇒ cos θ (cos θ – 3) + 2 (cos θ – 3) = 0
⇒ (cos θ – 3) (cos θ + 2) = 0
(1 + n)(n + 2) (n + 3)(n + 2) (n + 4)(n + 3)
2 2 2 ⇒ cos θ = 3 or cos θ = – 2
(n + 3)(n + 2) (n + 4)(n + 3) (n + 5)(n + 4) But – 1 ≤ cos θ ≤ 1
D=
2 2 3
(n + 4)(n + 3) (n + 5)(n + 4) (n + 6)(n + 5) So cos θ ≠ 3, – 2
2 2 2 There is no solution for non-trivial solution

At n = 1
Multiple Correct Choice Type
3 6 10
D = 6 10 15 cos(x – y) cos(y – z) cos(z – x)
10 15 21 Sol 21: (A, D) cos(x + y) cos(y + z) cos(z + x)
sin(x + y) sin(y + z) sin(z + x)
= 3 [210 – 225) + 6 (150 – 126) +10(90–100)
= – 45 + 144 – 100 = – 1 cos (A – B) = cos A cos B + sin A sin B

There is only one option (A) sin (A + B) = cos B sin A + sin B cos A

Which satisfied the ans. cos (A + B) = cos A cos B – sin A sin B

Using, C1 → C1 – C2, C3 → C3 – C2, = cos (x – y) [cos (y + z) sin (z + x) – cos (z + x)


sin (y + z)] + cos (y – z) [cos (z + x) sin (x + y) – sin (x + z)
R3 → R3 – R2, we get
M a them a ti cs | 17.75

cos (x + y)] + cos (z – x) [cos (x + y) sin (y + z) – cos (y ⇒ x2ab (b – 1) + a2x (1 – b2) + a3 (b – 1) = 0


+ z) cos (x + y)]
⇒ x2ab – a2x (1 + b) + a3 (+1) = 0
= cos (x – y) [sin (z + x – y – z)]+ cos (y – z) [ sin (x +
⇒ x2ab – x (a2 + a2b) + a3 = 0
y – z – x)]+ cos (z – x) [sin (y + z – x – y)]
⇒ x2ab – a2x – a2 (bx – a) = 0
= cos (x – y) sin (x – y) + cos (y – z) sin (y – z) + cos
(z – x) sin (z – x) ⇒ (bx – a) (ax – a2) = 0

1 ⇒ bx – a = 0 or ax – a2 = 0
= [sin 2 (x –y) + sin (2(y – z) + sin 2(z – x)]
2 a a2
⇒x= or x = =a
1 b a
= [2 cos (x + z – zy) sin (x + z) – 2 sin (x – z) cos (x – z)]
2
a b aα + b
1
= sin (x – z) [cos (x + z – 2y) – cos (x – z)] Sol 24: (B, D) b c bα + c
2
aα + b bα + c 0
 (x – y) (y – z) 
= sin (x – z) 2sin sin 
 2 2  R3 → R3 – aR1 – R2
a b aα + b
= 2 sin (x – y) sin (y – z) sin (z – x)
b c bα + c
–π π 0 0 –(aα2 + bα + bα + c)
Sol 22: (A, B, C, D) < θ < , 0 ≤ A ≤ π/2
4 2
= (aa2 + 2bα + c) (b2 – ac) = 0
1 + sin2 A cos2 A 2sin 4θ
sin2 A 1 + cos2 A 2sin 4θ =0 So (b2 – ac) = 0

sin2 A cos2 A 1 + 2sin 4θ b2 = ac → b is GM of a, c → ab, c are in GP


or (aa2 + 2b α + c) = 0
R1 → R1 – R3 , R2 → R2 – R3
⇒ x = α → (x – α)
1 0 –1 ⇒ ax2 + 2bx + c, (x – α) is a factor of this)
0 1 –1 =0
2 2 2
sin A cos A 1 + sin θ Sol 25: (B, D) x – y + 3z = 2
2x – y + z = 4
⇒ 1 + 2 sin4θ + cos2A – 1 [– sin2 A]
x – 2y + az = 3
⇒ 1 + 2 sin 4θ + sin2θ + cos2 θ
1 –1 3
⇒ 2 + 2 sin 4θ = 0 (only depend on θ)
D = 2 –1 1
⇒ sin 4θ = – 1 ⇒ 4q∈ – π/2 + 2nπ, n ∈ I 1 –2 α
θ ∈ – π/8 + nπ/2, n ∈ I = 1 [– α + 2 ] – 1 [1 – 2a] + 3 [– 4 + 1]
(A) θ → – π/8 = – α + 2 – 1 + 2α – 9= α – 8
(B) θ → 3π/8 sin 4θ = – 1 D≠0→α≠8
(C) θ = – π/8 sin 4θ = – 1 If α = 8, D = 0
(D) θ = 3π/8 2 –1 3
Dx = 4 –1 1
1 a a2
3 –2 α
Sol 23: (A, D) 1 x x2 = 0
= 2 [– α + 2] – 1 [3 – 4a] + 3 [– 8 + 3]
b2 ab a2
= – 2α + 4 – 3 + 4α – 15 = 2α – 14 = 2(α – 7)
⇒ xa2 – x2ab + a [x2b2 – a2] + a2 [ab – xb2]=0
= At α = 8 (0 = 0), Dx ≠ 0
⇒ x [a – a b ] + x [ab – ab] – a + a b = 0
2 2 2 2 2 3 3
So, at α = 8, system has no solution.
1 7 . 7 6 | Determinants

1 bc bc(b + c) logx xyz logx y logx z


Sol 26: (A,B,C,D) 1 ca ca(c + a) (D) logy xyz 1 logx z
1 ab ab(a + b) logx xyz logz y 1

a abc abc(b + c) a 1 b+c logxyz log y logx


1 (abc)2
= b abc abc(c + a) = b 1 c+a logx logx logx
abc abc
c abc abc(a + b) c 1 a+b logxyz log y logz
log y log y log y
C3 → C3 + C1 logxyz log y logz
a 1 a+b+c a 1 1 logz logz logz
= (abc) b 1 a + b + c = (abc) (a + b + c) b 1 1
1 1 1
c 1 a+b+c c 1 1
=
(logxyz )(log y )(logz ) 1 1 1 =0
C2 → c2 – c3
(logx )(log y )(logz ) 1 1 1
a 0 1 ( C1 = c2 = c3)
= (a b c) (a + b + c) b 0 1 = 0
c 0 1 Sol 27: (A, B, C, D) a2x – by = a2 – b2posses an infinite
no. of solution
1 1 1 1 
1 ab + c abc c  +  bx – b2y = 2 + 4b
a b a b
1 1 1 1 1 a2 –b
(B) 1 bc + = a abc a  +  So D = 0 ⇒ = – b2a2 + b2 = 0 Þ
b c abc b c b –b2
1 1 1 1
1 ca + b abc b  +  ⇒ b2(1 – a2) = 0
c a
c a
→ b = 0 or a = ± 1 … (i)
c c c
c +
c a b a2 – b –b
Dx = 0 ⇒
abc a a a 2 + 4b –b2
a + C → c3 + c2
abc a b c 3
b b b = – b2 (a2 – b) + b (2 + 4b) = 0
b +
b a c – a2b2 + b3 + 2b + 4b2 = 0

1 1 1 b(b2 + 4b + 2 – a2b) = 0
c 1 c + + 
a b c c 1 C
⇒ b = 0 or b2 + 4b + 2 = a2b … (ii)
1 1 1 1 1 1
a 1 a + +  =  + +  a 1 a a2 a2 – b
a b c a b c b 1 b Dx = 0 ⇒ = a2(2 + 4b) + b(b – a2) = 0
b 2 + 4b
1 1 1
b 1 b + +  2a2 + 4a2b + b2 – ba2 = 0 …(iii)
a b c
All option are satisfied equation (i, ii, iii)
C1 → c1 – c3

0 1 c Sol 28: (A, C) p, q, r, s are in AP


1 1 1
=  + +  0 1 a =0 P = p q = p + d r = p + 2d, s = p + 3d
a b c 0 1 b
D is common difference of the A.P.
0 a–b a–c 0 (a – b) (a – c) p + sinx q + sinx p – r + sinx
(C) b – a 0 b – c = –(a – b) 0 (b – c) f(x) = q + sinx r + sinx –1 + sinx
c–a c–b 0 –(a – c) –(b – c) 0 r + sinx s + sinx s – q + sinx
This is stew symmetric matrix so value of determinate C2 → c2 – c1, C3 → c3 – c1
is zero.
M a them a ti cs | 17.77

p + sinx q – p –p + q – r Dx 3k – 2
x= =
D 1
f(x) = q + sinx r – q –1 – q
r + sinx s – r s–q–r 1 3
Dy = = 8 – 3 (1 + k) = 5 – 3k
1+k 8
p + sinx d –p – 2d
= q + d + sinx d –1 – p – d y = 5 – 3k
p + 2d + sinx d –p Which will satisfied other equation of (x, y)

R3 → R3 – R1, R2 → R2 – R1 ⇒ (3k – 2) – (1 + k) (5 – 3k) + 2 + k = 0


⇒ 3k – 2 – (5 – 3k2 + 2k) + 2 + k = 0
p + sinx d –p – 2d
f(x) = d 0 –1 – p – d + p + 2d ⇒ 3k – 2 – 5 + 3k2 – 2k + 2 + k = 0
2d 0 –p + p + 2d ⇒ 3k2 + 2k – 5 = 0
⇒ 3k2 + 5k – 3k – 5 = 0
p + sinx d –p – 2d
= k (3k + 5) – 1(3k + 5) = 0
d 0 d–1
2d 0 2d ⇒ (k – 1) (3k + 5) = 0
⇒ k – 1 = 0 or 3k + 5 = 0
= d[2d(d – 1) – 2d2]
k =1 or k = –5/3
= d(2d2 – 2d – 2d2) = – 2d2
2 2 2
2
 –2d2  Sol 30: (A, B, C)
∫ f(x)dx = ∫ –2d dx = [ 
 d
x  = – 2d2 (2) = – 4
 0
0 0
1 1 (x + y)
⇒ d2 = 1 ⇒ d = ± 1 z z z2
–(y + z) 1 1
D=
Sol 29: (A, C) x + y = 3 … (i) x2 x x
(1 + k) x + (2 + k)y = 8 … (ii) –y(y + 2) –(x + 2y + z) –y(x + y)
x2 z xz xz 2
x – (1 + k)y = – (2 + k) … (iii)
1 1 1
1 1 D= . .
2 2 2 2
D12 = = 2 + k – (1 + k) = 1 z x x z
1+K 2+K
z z –(x + y)
D12 ≠ 0 , So
–(y + z) x x
1 1 –zy(y + z) –(x + 2y + z)(xz) –yx(x + y)
D13 = = – 1 – k – 1 = – 2 – k = – (2 + k)
1 –(1 + k) C1 → C2 – C3 and

3 1 –zy (y + z) + yx (x + y) = – zy2 – z2y + yx2 + y2x


So Dx = = – 3 (1 + k) + 2 + k
–(2 + k) –(1 + k) = y [y(x –z) + x2 – z2] = y(x – z) [y + x + z]

= – 3 – 3k + 2 + k = –2k – 1 1
D=
4 4
x z
1+k (2 + k)
∴ D23 = 0
1 –(1 + k) (x + y + z) (x + y + z) –(x + y)
= – 1(1 + k)2 – (2 + k) = – (k2 + 3k + 3) –(x + y + z) 0 x
y(x – z)(x + y + z) x[(y – z)(x + y + z) – 2yz] –xy(x + y)
From (ii) & (iii)
(x + y + z)
3 1 D=
Dx = = 3(2 + k) – 8 = 6 + 3k – 8 = 3k – 2 x4 z4
8 2+k
1 7 . 7 8 | Determinants

1 x+y+z –(x + y) variables and that could have only unique, no solution
or infinitely many solution.
–1 0 x
y(x – z) x[(y – z)(x + y + z) – 2yz] –xy(x + y) ∴ It is not possible to have two solutions.
Hence, number of matrices A is zero.
R1 → R1 + R2

(x + y + z) ax − by − c bx + ay cx + a
D=
4 4
x z Sol 3: Given bx + ay −ax + by − c cy + b 0
=
cx + a cy + b −ax − by + c
0 x+y+z –y
–1 0 x a2 x − aby − ac bx + ay cx + a
y(x – z) x[(y – z)(x + y + z)(–xyz) –xy(x + y) 1 2
⇒ abx + a y −ax + by − c cy + b 0
=
a 2
x+y+2 acx + a cy + b −ax − by + c
x +4 y + 2
x +4
x4 + 4 Applying C1 → C1 + bC2 + cC3
( x + y + z )( xy + y ) − xy ( x − z ) + yx ( −xyz )( x + y + z )( y − z ) 
( x + y + z )( xy + y ) − xy ( x − z ) + yx ( −xyz )( x + y + z )( y − z )  
 
(a2 + b2 + c2 )x ay + bx cx + a
1 2 2 2
⇒ (a + b + c )y by − c − ax b + cy = 0
Previous Years Questions a 2 2
a +b +c 2
b + cy c − ax − by

1 a a2 x ay + bx cx + a
1
Sol 1: (B) Let
= ∆ cos(p − d)x cospx cos(p + d)x ⇒ y by − c − ax b + cy = 0
a
sin(p − d)x sinpx sin(p + d)x 1 b + cy c − ax − by

Applying C1 → C1 + C3 ( a2 + b2 + c2 =
1)

Applying C2 → C2 − bC1 and C3 → C3 − cC1


1 + a2 a a2
=
⇒ ∆ cos(p − d)x + cos(p + d)x cospx cos(p + d)x x ay a
1
sin(p − d)x + sin(p + d)x sinpx sin(p + d)x ⇒ y −c − ax b =0
a
1 cy −ax − by
1 + a2 a a2
⇒ ∆ 2cospx cosdx cospx cos(p + d)x x2 axy ax
1
2sinpx cosdx sinpx sin(p + d)x ⇒ y −c − ax b =0
ax
1 cy −ax − by
Applying C1 → C1 − 2cosdx C2
Applying R1 → R1 + yR 2 + R 3
1 + a2 − 2acosdx a a2
⇒ ∆ 0 cospx cos(p + d)x x2 + y 2 + 1 0 0
1
0 sinpx sin(p + d)x ⇒ y −c − ax b =0
ax
1 cy −ax − by
⇒ ∆= (1 + a2 − 2acosdx)
1 2
[sin(p + d)x cospx − sinpx cos(p + d)x] ⇒ [(x + y 2 + 1){( −c − ax)( −ax − by) − b(cy)}] =0
ax
⇒ ∆= (1 + a2 − 2acos dx)sindx
1 2
Which is independent of p. ⇒ [(x + y 2 + 1)(acx + bcy + a2 x2 + abxy − bcy)] =
0
ax
1 2
 x  1  ⇒ [(x + y 2 + 1)(acx + a2 x2 + abxy)] =
0
    ax
Sol 2: Since,  y  = 0  is linear equation in three
A 1
 z  0  ⇒ [ax(x2 + y 2 + 1)(c + ax + by)] =
0
    ax
M a them a ti cs | 17.79

Applying R 2 → R 2 + R 3
⇒ (x2 + y 2 + 1)(ax + by + c) =
0 ⇒ ax + by + c =0
Which represents a straight line.
sin θ cos θ sin2θ
Sol 4: Since, the given system of equations posses non-  2π   2π   4π 
sin  θ +  cos  θ +  sin  2θ + 
trivial solution, if  3   3   3 
=
0 1 −2  2π   2π   4π 
+ sin  θ −  + cos  θ −  + sin  2θ − 
0 k=0
0 −3 1 =  3   3   3 
k −5 4  2π   2π   4π 
sin  θ −  cos  θ −  sin  2θ − 
 3   3   3 
On solving the equations x = y = z = λ (say)
∴ For k = 0, the system has infinite solutions for λ ∈ R .  2π   2π 
Now, sin  θ +  + sin  θ − 
 3   3 
Sol 5: Given system of equations are
 2π 2π   2π 2π 
3x + my = m and 2x – 5y = 20 θ+ +θ−  θ+ −θ+ 
= 2sin  3 3  cos  3 3 
3 m  2   2 
Here, ∆ = = −15 − 2m    
2 −5    
2π  π
m m 3 m = 2sin θ cos = 2sin θ cos  π − 
and ∆ x = =−25m ; ∆y = = 60 − 2m 3  3
20 −5 2 20
π =
− sin θ
=−2sin θ cos
If ∆ =0 , then system inconsistent i.e. it has no solution. 3
15  2π   2π 
If ∆ ≠ 0 i.e. m ≠ , then system has a unique solution and cos  θ +
2  + cos  θ − 
 3   3 
for any fixed value of m.
 2π 2π   2π 2π 
∆x −25m 25m θ+ +θ−  θ+ −θ+ 
x
We have,= = = 3 3 3 3 
∆ −15 − 2m 15 + 2m = 2cos   cos 
 2   2 
   
∆y 60 − 2m 2m − 60    
y
and= = =
∆ −15 − 2m 15 + 2m  2π   1
= 2cos θ cos  =  2cos θ  −  =
− cos θ
25m  3   2
For x > 0, >0
15 + 2m
 4π   4π 
15 and sin  2θ +  + sin  2θ − 
⇒ m > 0 or m < − … (i)  3   3 
2 
2m − 60  4π 4π   4π 4π 
and y > 0, >0  2θ + + 2θ −   2θ + − 2θ + 
2m + 15 = 2sin  3 3  cos  3 3 
 2   2 
15    
⇒ m > 30 or m < − … (ii)    
2 
4π  π
From equation (i) and (ii) we get = 2sin2θ cos= 2sin2θ cos  π + 
3  3
15 π
m<− or m > 30 =−2sin2θ cos = − sin2θ
2 3

sin θ cos θ sin2θ


sin θ cos θ sin2θ ∴∆
= − sin θ − cos θ − sin2θ
 2π   2π   4π   2π   2π   4π 
Sol 6: Let sin  θ +  cos  θ +  sin  2θ +  sin  θ −
 3   3   3   cos  θ −  sin  2θ − 
 3   3   3 
 2π   2π   4π 
sin  θ −  cos  θ −  sin  2θ − 
 3   3   3  = 0 (Since, R1 and R 2 are proportional).
1 7 . 8 0 | Determinants

Sol 7: (B) Method I


α 2 − 4 4 α 2 − 4 9α 2 − 4
5 3 5 3 5
Total no. of ways = 3 − C1 3 − 1 + C2 3 − 1 −2 −2 −2 = −648α

= 243 − 3 × 32 + 3= 246 − 96= 150 5 + 2α 5 + 4α 5 + 6α

Alternative Method α 2 − 4 4 α 2 − 4 9α 2 − 4
System I −2 1 1 1 648α
=
5 + 2α 5 + 4α 5 + 6α
Boxes I II III
Balls I 2 2 α2 − 4 3α2 8α 2
−2 1 0 0 = −648α
For this system no. of ways 5 + 2α 2α 4α
 5!
= 
 2!2!1!
1  5× 4 ×3× 2 
× ×
2!   2×2×2 
= × 6 90 (
−2 12α3 − 16α3 =−648α)
System II ( ) 3
⇒ 2 −4α =−648α

⇒ α ( α − 81 ) =0
2
Boxes I II III
⇒=
α 0,9, −9
Balls I 3 1

For this system no. of ways 1 1 1 + x3


Sol 9: (2) 2 4 1 + 8x3 =
10
 5! 1
=  ×  × 3! = 10 × 6 = 60 3 9 1 + 27x3
 3!1!1! 2! 
1 1 1 1 1 1
Total no. of ways = 90 + 60 = 150 3 6
x 2 4 1 +x 2 4 8 = 10
3 9 1 3 9 27
Sol 8: (B, C)

(1 + α ) (1 + 2α ) (1 + 3α )
2 2 2
1 0 0 1 0 0
3 6
x 2 2 −1 + x 2 2 6 = 10
( 2 + α ) ( 2 + 2α ) ( 2 + 3α )
2 2 2
−648α
=
3 6 −2 3 6 24
(3 + α ) (3 + 2α ) (3 + 3α )
2 2 2

6x3 + x3 − 5 = 0 ⇒ 6x6 + 6x3 − 5x3 − 5 = 0


1 + α2 + 2α 1 + 4α2 + 4α 1 + 9α 2 + 6α
4 + α2 + 4α 4 + 4 α 2 + 8α 4 + 9α2 + 12 R 2 → R 2 − R1
(6x − 5)( x
3 3
)
+1 =
0

9 + α2 + 6α 9 + 4α2 + 12α 9 + 9α2 + 18α 5


x3 = or x3 = −1 Two real distinct values of x.
6
R3 → R3 − R 2 =
−648α

1 + α2 + 2α 1 + 4α2 + 4α 1 + 9α2 + 6α
3 + 2α 3 + 4α 3 + 6α C2 → C2 − C3
5 + 2α 5 + 4α 5 + 6α
C2 → C2 − C2 =
−648α
2017-18 100 &
op kers
Class 12 T
By E ran culty
-JE Fa r
IIT enior emie .
S fP r es
o titut
Ins

MATHEMATICS
FOR JEE MAIN & ADVANCED
SECOND
EDITION

Exhaustive Theory
(Now Revised)

Formula Sheet
9000+ Problems
based on latest JEE pattern

2500 + 1000 (New) Problems


of previous 35 years of
AIEEE (JEE Main) and IIT-JEE (JEE Adv)

5000+Illustrations and Solved Examples


Detailed Solutions
of all problems available

Topic Covered Plancess Concepts


Tips & Tricks, Facts, Notes, Misconceptions,
Matrices Key Take Aways, Problem Solving Tactics

PlancEssential
Questions recommended for revision
16. M AT R I C E S

1. INTRODUCTION
A rectangular array of m x n numbers (real or complex) in the form of m horizontal lines (called rows) and n vertical
lines (called columns), is called a matrix of order m by n, written as m × n matrix. Such an array is enclosed by [ ] or
( ) or || ||. An m × n matrix is usually written as
 a11 a12 ...... a1n 
 
a a22 ...... a2n 
A =  21
     
 
am1 am2 ...... amn 

In brief, the above matrix is represented by A = [aij]m×n. The number a11, a12, …… etc., are known as the elements of
the matrix A, where aij belongs to the ith row and jth column and is called the (i, j)th element of the matrix A = [aij].

2. ORDER OF A MATRIX
3 −1 5 
A matrix which has m rows and n columns is called a matrix of order m × n E.g. the order of   matrix
is 2 × 3.  6 2 −7 

Note: (a) The difference between a determinant and a matrix is that a determinant has a certain value, while the
matrix has none. The matrix is just an arrangement of certain quantities.
(a) The elements of a matrix may be real or complex numbers. If all the elements of a matrix are real, then the matrix
is called a real matrix.
(a) An m × n matrix has m.n elements.

Illustration 1: Construct a 3×4 matrix A = [aij], whose elements are given by aij = 2i + 3j.  (JEE MAIN)

Sol: In this problem, i and j are the number of rows and columns respectively. By substituting the respective values of
rows and columns in aij = 2i + 3j we can construct the required matrix.
 a11 a12 a13 a14 
 
We have A = a21 a22 a23 a24  ; ∴ a11 = 2 × 1 + 3 × 1 = 5; a12 = 2 × 1 + 3 × 2 = 8
a a32 a33 a34 
 31
Similarly, a13 = 11, a14 = 14, a21 = 7, a22 = 10, a23 = 13, a24 = 16, a31 = 9, a32 = 12, a33 = 15, a34 = 18

5 8 11 14 
∴ A = 7 10 13 16 
9 12 15 18 
 
1 6 . 2 | Matrices

1
Illustration 2: Construct a 3 × 4 matrix, whose elements are given by: aij = | –3i + j |  (JEE MAIN)
2
Sol: Method for solving this problem is the same as in the above problem.
1
Since aij = | –3i + j | we have
2
1 1 1 2
a11 = | –3(1) + 1 | = | – 3 + 1 | = | – 2 | = = 1
2 2 2 2
1 1 1 1
a12 = | –3(1) + 2 | = | – 3 + 2 | = | – 1 | =
2 2 2 2
1 1 1
a13 = | –3(1) + 3 | = | – 3 + 3 | = (0) = 0
2 2 2
1 1 1 1 1 5
a14 = | –3(1) + 4 | = | – 3 + 4 | = ; a21 = | –3(2) + 1 | = | – 6 + 1 | =
2 2 2 2 2 2
1 1 4 1 1 3
a22 = | –3(2) + 2 | = | – 6 + 2 | = = 2; a23 = | –3(2) + 3 | = | – 6 + 3 | =
2 2 2 2 2 2
1 1 2 7 5
a24 = | –3(2) + 4 | = | – 6 + 4 | = = 1; Similarly a31 = 4, a32 = , a33 = 3, a34 =
2 2 2 2 2
 1 1
1 0 
 2 2
5 3
Hence, the required matrix is given by A =  2 1 
2 2
 
4 7 3 5
 2 2 

3. TYPES OF MATRICES

3.1 Row Matrix


A matrix having only one row is called a row matrix. Thus A = [aij]m×n is a row matrix if m = 1;
E.g. A = [1 2 4 5] is row matrix of order 1 × 4.

3.2 Column Matrix


A matrix having only one column is called a column matrix. Thus A = [aij]m×n is a column matrix if
 −1 
 
2
n = 1; E.g. A =   is column matrix of order 4 × 1.
 −4 
 
 5 

3.3 Zero or Null Matrix


If in a matrix all the elements are zero then it is called a zero matrix and it is generally denoted by 0.Thus, A = [aij]
0 0 0 
m×n
is a zero matrix if aij = 0 for all i and j; E.g.   is a zero matrix of order 2 × 3.
0 0 0 
0 0  0 0 0 
   
A = 0 0  is a 3 × 2 null matrix & B = 0 0 0  is 3 × 3 null matrix.
0 0  0 0 0 
   
M a them a ti cs | 16.3

3.4 Singleton Matrix


If in a matrix there is only one element then it is called singleton matrix. Thus, A = [aij]m×n is a singleton matrix if m
= n = 1. E.g. [2], [3], [a], [–3] are singleton matrices.

3.5 Horizontal Matrix


1 2 3 4 
A matrix of order m × n is a horizontal matrix if n > m; E.g.  
2 5 1 1 

3.6 Vertical Matrix 2 5


 
1 1
A matrix of order m × n is a vertical matrix if m > n; E.g. 
3 6
 
2 4 
3.7 Square Matrix
If the number of rows and the number of columns in a matrix are equal, then it is called a square matrix.
 a11 a12 a13 
 
Thus, A = [aij]m×n is a square matrix if m = n; E.g. a21 a22 a23  is a square matrix of order 3 × 3.
a a32 a33 
 31
The sum of the diagonal elements in a square matrix A is called the trace of matrix A, and which is denoted by
n
tr(A); tr(A) = ∑ aii = a 11
+ a22 + ….. ann.
i=1

3.8 Diagonal Matrix


If all the elements, except the principal diagonal, in a square matrix are zero, it is called a diagonal matrix.
2 0 0 
 
Thus, a square matrix A = [aij] is a diagonal matrix if aij = 0, when i ≠ j; E.g. 0 3 0  is a diagonal matrix of order
3 × 3, which can also be denoted by diagonal [2 3 4]. 0 0 4 
 

3.9 Scalar Matrix


If all the elements in the diagonal of a diagonal matrix are equal, it is called a scalar matrix. Thus, a square matrix A
0, i ≠ j
= [aij]n×n is a scalar matrix if aij =  where k is a constant.
k, i = j
 −7 0 0 
 
E.g.  0 −7 0  is a scalar Matrix.
 0 0 −7 
 

3.10 Unit Matrix


If all the elements of a principal diagonal in a diagonal matrix are 1, then it is called a unit matrix. A unit matrix of
order n is denoted by In. Thus, a square matrix A = [aij]n×n is a unit matrix if
1 0 0 
1, i = j  
aij =  E.g. I3 = 0 1 0 
0, i ≠ j 0 0 1 
 
Note: Every unit matrix is a scalar matrix.
1 6 . 4 | Matrices

3.11 Triangular Matrix


A square matrix is said to be a triangular matrix if the elements above or below the principal diagonal are zero.
There are two types:

3.11.1 Upper Triangular Matrix


A square matrix [aij] is called an upper triangular matrix, If aij = 0, when i > j.

3 1 2 
 
E.g. 0 4 3  is an upper uriangular matrix of order 3 × 3.
0 0 6 
 

3.11.2 Lower Triangular Matrix


A square matrix is called a lower triangular matrix, if aij = 0 when i < j.

1 0 0 
 
E.g.  2 3 0  is a lower triangular matrix of order 3 × 3.
 4 5 2
 

3.12 Singular Matrix


Matrix A is said to be a singular matrix if its determinant | A | = 0, otherwise a non-singular matrix, i.e.
If det | A | = 0 ⇒ Singular and det | A | ≠ 0 ⇒ non-singular

PLANCESS CONCEPTS

• A triangular matrix A = [aij]n×n is called strictly triangular if aij = 0 ∀ i=j


• The multiplication of two triangular matrices is a triangular matrix.
 very row matrix is also a horizontal matrix but not the converse. Similarly every column matrix is also
• E
a vertical matrix but not the converse.
Vaibhav Gupta (JEE 2009 AIR 54)

3.13 Symmetric and Skew Symmetric Matrices


Symmetric Matrix: A square matrix A = [aij] is called a symmetric matrix if aij = aji, for all i,j values;

1 2 3
 
E.g. A =  2 4 5  is symmetric, because a12 = 2 = a21, a31 = 3 = a13 etc.
3 5 2
 
Note: A is symmetric ⇔ A = A ’(where A’ is the transpose of matrix)
Skew-Symmetric Matrix: A square matrix A = [aij] is a skew-symmetric matrix if aij = –aji, for all values of i,j.
 aij = –aji, for all i,j. ⇒ aii = –aii, [putting j = i] ⇒ 2aii = 0 ⇒ aii = 0
0 2 1
   0 2
Thus, in a skew-symmetric matrix all diagonal elements are zero; E.g. A =  −2 0 −3 , B =   are skew-
symmetric matrices.  −2 0 
 −1 3 0 
 
Note: A square matrix A is a skew-symmetric matrix ⇔ A’ = –A.
M a them a ti cs | 16.5

Few results:
(a) If A is any square matrix, then A + A’ is a symmetric matrix and A – A’ is a skew-symmetric matrix.
(b) Every square matrix can be uniquely expressed as the sum of a symmetric matrix and a skew-symmetric
1 1 1
matrix. A = (A + A’) + (A – A’) = (B + C), where B is symmetric and C is a skew symmetric matrix.
2 2 2
(c) If A and B are symmetric matrices, then AB is symmetric ⇔ AB = BA, i.e. A & B commute.
(d) The matrix B’AB is symmetric or skew-symmetric in correspondence if A is symmetric or skew-symmetric.
(e) All positive integral powers of a symmetric matrix are symmetric.
(f) Positive odd integral powers of a skew-symmetric matrix are skew-symmetric and positive even integral
powers of a skew-symmetric matrix are symmetric.

PLANCESS CONCEPTS

Elements of the main diagonal of a skew-symmetric matrix are zero because by definition aii = – aii ⇒ 2aii =
0 or aii = 0 for all values of i.
Trace of a skew symmetric matrix is always 0. The sum of symmetric matrices is symmetric.
Every square matrix can be uniquely expressed as the sum of a symmetric matrix and a skew-symmetric
1 1 1
matrix A = ( A + A’) + ( A – A’) = (B + C), where B is symmetric and C is a skew symmetric matrix.
2 2 2
If A and B are symmetric matrices, then AB is symmetric ⇔ AB = BA, i.e. A & B commute. The matrix
B’AB is symmetric or skew-symmetric accordingly when A is symmetric or skew symmetric. All positive
integral powers of a symmetric matrix are symmetric. Positive odd integral powers of a skew-symmetric
matrix are skew-symmetric and positive even integral powers of a skew-symmetric matrix are symmetric.
Chen Reddy Sandeep Reddy (JEE 2012 AIR 62)

3.14 Hermitian and Skew-Hermitian Matrices


A square matrix A = [aij] is said to be a Hermitian matrix if aij= aji ∀ i, j; i.e. A = Aq

 3 3 − 4i 5 + 2i 
 a b + ic   
E.g.   . 3 + 4i 5 −2 + i are Hermitian matrices
b − ic d 
5 − 2i −2 − i 2 

Note: (a) If A is a Hermitian matrix then aii = aii ⇒ aii is real ∀ i, thus every diagonal element of a Hermitian matrix
must be real.
(b) If a Hermitian matrix over the set of real numbers is actually a real symmetric matrix; and A a square matrix,
A = [aij] is said to be a skew-Hermitian if aij = – aji , ∀ i, j;

 3i −3 + 2i −1 − i 
 0 −2 + i  
i.e. A = – A; E.g. 
θ
.  3 − 2i −2i −2 − 4i  are skew-Hermitian matrices.
2 − i 0   1 + i 2 + 4i 0 

(c) If A is a skew-Hermitian matrix then aii =– aii ⇒ aii + a = 0


ii

i.e. aii must be purely imaginary or zero.

(d) A skew-Hermitian matrix over the set of real numbers is actually is real skew-symmetric matrix.
1 6 . 6 | Matrices

4. TRACE OF A MATRIX
Let A = [aij]n×n and B = [bij]n×n and λ be a scalar,
(i) tr(λA) = λ tr(A) (ii) tr(A + B) = tr(A) + tr(B) (iii) tr(AB) = tr(BA)

Square Matrix

Triangular Matrix Diagonal Matrix


Atleast one, aii ≠ 0 and aij = 0 if i ≠ j
d1 0 0 
Upper triangular Lower triangular 0 d
2 0 
Matrix Matrix 
If aij = 0 ∀ i > j If aij = 0 ∀ i < j  0 0 d3 
x x x x 0 0 |
, Abbreviated as dia (d1, d2, d3, …….dn)
A = 0 x x  A = x x 0 |
0 0 x  x x x 
Scalar Matrix Unit Matrix
If d1=d2=d3 ….= a≠0 If d1=d2=d3 ….= 1
a 0 0 1 0 0
0 a 0 0 1 0 =I
    3
0 0 a 0 0 1 

5. TRANSPOSE OF A MATRIX
The matrix obtained from a given matrix A by changing its rows into columns or columns into rows is called the
transpose of matrix A and is denoted by AT or A’. From the definition it is obvious that if the order of A is m × n,
then the order of AT becomes n × m; E.g. transpose of matrix
 a1 b1 
 a1 a2 a3   

b b b
 is a2 b2 
 1 2 3  2×3 a b 
 3 3  3×2

5.1 Properties of Transpose of Matrix


(i) (AT)T = A (ii) (A ± B)T = AT ± BT (iii) (AB)T = BTAT (iv) (kA)T = k(A)T
(v) (A1 A2 A3 ..........An−1 An )T = AnT AnT−1 …….. A3T A2T A1T (vi) IT = I (vii) tr(A) = tr(AT)

 1 3
 1 −2 3  
Illustration 3: If A =   and B =  −1 0  then prove that (AB) = B A . 
T T T
(JEE MAIN)
 −4 2 5   2 4
 
Sol: By obtaining the transpose of AB i.e. (AB)T and multiplying BT and AT we can easily get the result.

 1 3
 1 −2 3    1(1) − 2( −1) + 3(2) 1(3) − 2(0) + 3(4)   9 15
Here, AB =    −1 0  =  = 
 −4 2 5   2 4   −4(1) + 2( −1) + 5(2) −4(3) + 2(0) + 5(4)  4 8 
 
 1 −4 
 9 4 1 −1 2     1(1) − 1( −2) + 2(3) 1( −4) − 1(2) + 2(5)   9 4 
∴ (AB)T =   ; B A
T T
=    −2 2  =   =  = (AB)
T

15 8  3 0 4   3 5  3(1) + 0( −2) + 4(3) 3( −4) + 0(2) + 4(5) 15 8 


 
M a them a ti cs | 16.7

5 −1 3 0 2 3 
Illustration 4: If A = 0 1 2 and B =   then what is (AB’)’ is equal to?  (JEE MAIN)
  1 −1 4 

Sol: In this problem, we use the properties of the transpose of matrix to get the required result.

 5 0
0 2 3     7 8
We have (AB′)’ = (B’)’ A’ = BA’ =    −1 1  =  
1 −1 4   3 2  18 7 
 

3 − x 2 2 
 
Illustration 5: If the matrix A =  2 4−x 1  is a singular matrix then find x. Verify whether AAT = I for that
value of x.   −2 −4 −1 − x  (JEE ADVANCED)

Sol: Using the condition of singular matrix, i.e. | A | = 0, we get the value of x and then substituting the value of x in
matrix A and multiplying it to its transpose we will obtain the required result.

3−x 2 2
Here, A is a singular matrix if | A | = 0, i.e., 2 4−x 1 =0
−2 −4 −1 − x

3−x 2 2 3−x 0 2
or 2 4 − x 1 = 0, using R3 → R3 + R2 or 2 3 − x 1 = 0, using C2 → C2 – C3
0 −x −x 0 0 −x

or – x(3 – x)2 = 0, ∴ x = 0, 3.

3 2 2 3 2 2 3 2 −2 
     
When x = 0, A =  2 4 1  ; ∴ AA =  2 4 1 
T
2 4 −4 
 −2 −4 −1  −2 −4 −1 2 1 −1 
     

 17 16 −16 
 
=  16 21 −21  ≠ I
 −16 −21 21 
 

0 2 2 0 2 2 0 2 −2   8 4 −16 
       
When x = 3,A =  2 1 1  ∴ AAT = 2 1 1 2 1 −4  =  4 6 −12  ≠ I
 −2 −4 −4   −2 −4 −4  2 1 −4   −16 −12 36 
       

Note: A simple way to solve is that if A is a singular matrix then |A| = 0 and |AT| = 0. But |I| is 1. Hence, AAT ≠ I if
|A| = 0.

a b c 
 
Illustration 6: If the matrix A = b c a  where a, b, c, are positive real numbers such that abc = 1 and ATA = I
 c a b
 
then find the value of a + b + c3.
3 3
(JEE ADVANCED)

a b c  a b c 
   
Sol: Here, A = b c a  . So, AT = b c a  , interchanging rows and columns.
 c a b  c a b
   
1 6 . 8 | Matrices

2
a b c 
 
∴ A A = b c a  = A2 ∴ | ATA | = | A2 |; But ATA = I (given). ∴ | I | = | A |2 ⇒ 1 = | A |2
T

 c a b
 

a b c 1 1 1
Now, | A | = b c a = (a + b + c) b c a , R1 → R1 + R2 + R3
c a b c a b

1 0 0
C → C2 − C1
= (a + b + c) b c − b a − b , 2
C3 → C3 − C1
c a−c b−c

= (a + b + c) {(c – b) (b – c) – (a – b) (a – c)} = (a + b + c) (– b2 – c2 + 2bc – a2 + ac + ab – bc)


= – (a + b + c) (a2 + b2 + c2 – bc – ca – ab) = – (a3 + b3 + c3 – 3 abc)
= – (a3 + b3 + c3 – 3) ( abc = 1) ∴ | A |2 = 1 ⇒ (a3 + b3 + c3 – 3)2 = 1  … (i)

a3 + b3 + c3 3 3 3 3
As a, b, c, are positive, > abc (∵ abc = 1); ∴ a3 + b3 + c3 > 3
3
∴ (i) ⇒ a3 + b3 + c3 – 3 = 1 ∴ a3 + b3 + c3 = 4

6. MATRIX OPERATIONS

6.1 Equality of Matrices

Two matrices A and B are said to be equal if they are of the same order and their corresponding elements are equal,
i.e. Two matrices A = [aij]m×n and B=[bij]r×s are equal if
(a) m = r i.e. the number of rows in A = the number of rows in B.
(b) n = s, i.e. the number of columns in A = the number of columns in B
(c) aij = bij, for i = 1, 2, …., m and j = 1, 2, ….., n, i.e. the corresponding elements are equal;

0 0  0 0 0 
E.g. Matrices   and   are not equal because their orders are not the same.
0 0  0 0 0 

1 6 3  a1 a2 a3 
E.g. If A =   and B=   are equal matrices then,
5 2 1  b1 b2 b3 

a1 = 1, a2 = 6, a3 = 3, b1 = 5, b2 = 2, b3 = 1.

6.2 Addition of Matrices


If A[aij]m×n and B[bij]m×n are two matrices of the same order then their sum A + B is a matrix, and each element of
that matrix is the sum of the corresponding elements. i.e. A + B = [aij + bij]m×n

Properties of Matrix Addition: If A, B and C are matrices of same order, then


(a) A + B = B + A (Commutative law),
(b) (A + B) + C = A + (B + C) (Associative law),
(c) A + O = O + A = A, where O is zero matrix which is additive identity of the matrix,
M a them a ti cs | 16.9

(d) A + (–A) = 0 = (–A) + A, where (–A) is obtained by changing the sign of every element of A which is additive
inverse of the matrix,

A + B = A + C
(e)   ⇒ B = C
B + A = C + A

(f) tr (A ± B) = tr (A) ± tr (B)


(g) Additive Inverse: If A + B = 0 = B + A, then B is called additive inverse of A and also A is called the additive
inverse of A.
(h) Existence of Additive Identity: Let A = [aij] be an m × n matrix and O be an m × n zero matrix, then
A + O = O + A = A. In other words, O is the additive identity for matrix addition.

6.3 Subtraction of Matrices


If A and B are two matrices of the same order, then we define A – B = A + (– B).

6.4 Scalar Multiplication of Matrices


If A=[aij]m×n is a matrix and k any number, then the matrix which is obtained by multiplying the elements of A by k
is called the scalar multiplication of A by k and it is denoted by k A thus if A = [aij]m×n
Then kAm×n = Am×nk = [kai×j]

Properties of Scalar Multiplication: If A, B are matrices of the same order and λ, µ are any two scalars then

(a) λ (A + B) = λA + λB (b) (λ + µ)A = λA + µA (c) λ(µA) = (λ µA) = µ(λA)


(d) (–λ A) = – (λ A) = λ(–A) (e) tr (kA) = k tr (A)

6.5 Multiplication of Matrices


If A and B be any two matrices, then their product AB will be defined only when the number of columns in A is
equal to the number of rows in B. If A[aij]m×n and B[bij]n×p then their product AB = C = [cij], will be a matrix of order
n
m × p, where (AB)ij = Cij = ∑ airbrj
r =1

Proof: Let A=[aij] be an m × n matrix and B = [bij] be an n × p matrix. Then the m × p matrix C = [cij] is called
the product if Cij=AiBj Where Ai is the ith row of A and Bj is the jth column of B. Thus the product AB is obtained as
following:

A=m×n B=n×p
C1 C2 Cj Cp

↓ ↓ ↓ ↓
a11 a12 a13 ……… a1j ……… a1n b11 b12 ……… b1j ……… b1p
R1 →
a21 a22 a23 ……… a2j ……… a2n b21 b22 ……… b2j ……… b2p
R2 →
……………………………………………….……… × ………………………………………………
Ri →
ai1 ai2 ai3 ……… aij ……… ain bi1 bi2 ……… bij ……… bip
Rm →
am1 am2 am3 ……… amj ……… amn bn1 bn2 ……… bnj ……… bnpn
1 6 . 1 0 | Matrices

R1C1 R1C2 ………. R1Cj R1Cp


R2C1 R2C2 ………. R2Cj R2Cp
………. ………. ………. ……….……….
= Thus (AB)ij = AiBj
RiC1 RiC2 ………. RiCj RiCp
………. ………. ………. ……….……….
RmC1 RmC2 ………. RmCj RmCp

b1 j 
 
b2 j 
n
 
(AB)ij = [ai1 ai2 …….. aij……. ain]  ...  = [ai1 b1j + ai2 b2j + …….. + ainbnj] ; (AB)ij = ∑ (air .brj )
b  r =1
 ij 
 .... 
 
 bnj 

Properties of matrix multiplication:


(a) Matrix multiplication is not commutative in general, i.e. in general AB ≠ BA.
(b) Matrix multiplication is associative, i.e. (AB)C = A(BC).
(c) Matrix multiplication is distributive over matrix addition, i.e. A.(B + C) = A.B + A.C and (A + B)C = AC + BC.
(d) If A is an m × n matrix, then ImA = A = AIn .
(e) The product of two matrices can be a null matrix while neither of them is null, i.e. if AB = 0, it is not necessary
that either A = 0 or B = 0.
(f) If A is an m × n matrix and O is a null matrix then Am×n. On×p = Om × p, i.e. the product of the matrix with a null
matrix is always a null matrix.
(g) If AB = 0 (It does not mean that A = 0 or B = 0, again the product of two non-zero matrices may be a zero
matrix).
(h) If AB = AC ⇒ B ≠ C (Cancellation Law is not applicable).
(i) tr(AB) = tr(BA).

 2 1 3 1 −2
   
Illustration 7: If A =  3 −2 1  and B =  2 1  find AB and BA if possible  (JEE MAIN)
 −1 0 1   4 −3
   

Sol: Using matrix multiplication. Here, A is a 3 × 3 matrix and B is a 3 × 2 matrix, therefore, A and B are conformable
for the product AB and it is of the order 3 × 2 such that

1 
 
(AB)11 = (First row of A) (First column of B) = [2 1 3]  2  = 2 × 1 + 1 × 2 + 3 × 4 = 16
4
 
 −2
 
(AB)12 = (First row of A) (Second column of B) = [2 1 3]  1  = 2 × (– 2) + 1 × 1 + 3 × (– 3) = –12
 −3
 
M a them a ti cs | 16.11

1 
 
(AB)21 = (Second row of A) (First column of B)= [3 –2 1]  2  = 3 × 1 + (–2) × 2 + 1 × 4 = 3
4
 
16 −12
 
Similarly (AB)22 = – 11, (AB)31 = 3 and (AB)32 = –1; ∴ AB =  3 −11
 3 −1 
 
BA is not possible since number of columns of B ≠ number of rows of A.

1 3  y 0  5 6 
Illustration 8: Find the value of x and y if 2  +  =    (JEE MAIN)
0 x  1 2  1 8 

Sol: Using the method of multiplication and addition of matrices, then equating the corresponding elements of
L.H.S. and R.H.S., we can easily get the required values of x and y.

1 3  y 0  5 6  2 6   y 0  5 6  2 + y 6 + 0  5 6 
We have, 2  +  =   ⇒  + =  ⇒  =  
0 x  1 2  1 8  0 2x  1 2  1 8   0 + 1 2x + 2 1 8 
Equating the corresponding elements, a11 and a22, we get
2 + y = 5 ⇒ y = 3; 2x + 2 = 8 ⇒ 2x = 6 ⇒ x = 3;
Hence x = 3 and y = 3.

 a − b 2a + c   −1 5 
Illustration 9: Find the value of a, b, c and d, if  =   (JEE MAIN)
2a − b 3c + d  0 13

Sol: As the two matrices are equal, their corresponding elements are equal. Therefore, by equating the corresponding
elements of given matrices we will obtain the value of a, b, c and d.

 a − b 2a + c   −1 5 
 =   (given)
2a − b 3c + d  0 13
a – b =−1  … (i)
2a + c = 5  … (ii)
2a – b = 0  … (iii)
3c + d = 13  … (iv)
Subtracting equation (i) from (iii), we have a = 1;
Putting the value of a in equation (i), we have 1 – b = – 1 ⇒ b = 2;
Putting the value of a in equation (ii), we have 2 + c = 5 ⇒ c = 3;
Putting the value of c in equation (iv), we find 9 + d = 13 ⇒ d =
Hence a = 1, b = 2, c = 3, d = 4.

2 3  2 −2 
Illustration 10: Find x and y, if 2x + 3y =   and 3x + 2y =    (JEE MAIN)
4 0  −1 5 
Sol: Solving the given equations simultaneously, we will obtain the values of x and y.
2 3
We have 2x + 3y =   … (i)
4 0
1 6 . 1 2 | Matrices

 2 −2 
3x + 2y =    … (ii)
 −1 5 
 6 9
Multiplying (i) by 3 and (ii) by 2, we get 6x + 9y =    … (iii)
12 0 
 4 −4 
6x + 4y =    … (iv)
 −2 10 
 6 − 4 9 + 4   2 13 
Subtracting (iv) from (iii), we get 5y =  = 
12 + 2 0 − 10  14 −10 
2 13  2 13 
5 5  5 5
⇒y=   ⇒y=  
 14 −10   14 −2 
 5 5   5 
2 13 
5 5 2 3
Putting the value of y in (iii), we get 2x + 3  =  
 14 −2  4 0
 5 
6 39   6 39   4 24   2 12 
2 3  5 5   2− 5 3− 5   5 −
5   5 −
5
⇒ 2x =   –   =  =   ⇒ x =  
 4 0   42 −6   4 − 42 0 + 6   − 22 6   − 11 3 
 5   5   5   5 

 2 12  2 13 
 5 −
5 5 5
Hence x=   and y =  
 − 11 3   14 −2 
 5   5 

 x + 3 z + 4 2y − 7   0 6 3y − 2
   
Illustration 11: If  −6 a − 1 0  =  −6 −3 2c + z  then find the values of a, b, c, x, y and z.
b − 3 −21 0  2b + 4 −21 0 
   (JEE ADVANCED)
Sol: As the two matrices are equal, their corresponding elements are also equal. Therefore, by equating the
corresponding elements of the given matrices, we will obtain the values of a, b, c, x, y and z.

 x + 3 z + 4 2y − 7   0 6 3y − 2
   
 − 6 a − 1 0  =  −6 −3 2c + z 
b − 3 −21 0  2b + 4 −21 0 
 

Comparing both sides, we get x + 3 = 0 ⇒ x = – 3  … (i)


and z + 4 = 6 ⇒ z = 6 – 4 ⇒ z = 2  … (ii)
and 2y – 7 = 3y – 2 ⇒ 2y – 3y = – 2 + 7 ⇒ –y = 5 ⇒ y = – 5  … (iii)
and a – 1 = –3 ⇒ a = –3 + 1 ⇒ a = –2  … (iv)
and b – 3 = 2b + 4 ⇒ b – 2b = 4 + 3 ⇒ – b = 7 ⇒ b = – 7  … (v)
−2
and 2c + 2 = 0 ⇒ 2c + 2 = 0 ⇒ 2c = – 2 ⇒c= ⇒c=–1 … (vi)
2
[from(2)] Thus; a = –2, b = –7, c = –1, x =–3, y = –5 and z = 2
M a them a ti cs | 16.13

7. RANK OF A MATRIX
If A = (aij)m×n is a matrix, and B is its sub-matrix of order r, then | B |, the determinant is called r-rowed minor of A.

Definition: Let A = (aij)m×n be a matrix. A positive integer r is said to be a rank of A if

(a) A possesses at least one r-rowed minor which is different from zero; and
(b) Every (r + 1) rowed minor of A is zero.

From (ii), it automatically follows that all minors of higher order are zeros. We denote rank of A by ρ(A)

Note: The rank of a matrix does not change when the following elementary row operations are applied to the matrix:
(a) Two rows are interchanged (Ri ↔ Rj);
(b) A row is multiplied by a non-zero constant, (Ri → kRi , with k ≠ 0);
(c) A constant multiple of another row is added to a given row (Ri → Ri, + kRj) where i ≠ j.

Note: The arrow → means “replaced by”.


Note that the application of these elementary row operations does not change a singular matrix to a non-singular
matrix nor does a non-singular matrix change to a singular matrix. Therefore, the order of the largest non-singular
square sub-matrix is not affected by the application of any of the elementary row operations. Thus, the rank of a
matrix does not change by the application of any of the elementary row operations. A matrix obtained from a given
matrix by applying any of the elementary row operations is said to be equivalent to it. If A and B are two equivalent
matrices, we write A ~ B. Note that if A ~ B, then ρ(A) = ρ(B)
By using the elementary row operations, we shall try to transform the given matrix in the following

1 * * *
 
0 1 * *
0 0 1 *
 
form  . . . .
. . . .
 
. . . .
 
0 0 0... *

Where * stands for zero or non-zero element. That is, we shall try to make aii as 1 and all the elements below aij as zero.

PLANCESS CONCEPTS

A non zero matrix A is said to have rank r, if


• Every square sub-matrix of order (r + 1) or more is singular;
• There exists at least one square sub-matrix or order r which is non singular.
B Rajiv Reddy (JEE 2012 AIR 11)

3 + x 5 2 
 
Illustration 12: For what values of x does the matrix  1 7+x 6  have the rank 2?  (JEE ADVANCED)
 2 5 3 + x 

Sol: The given matrix has only one 3rd-order minor. In order that the rank arrive at 2, we must bring about its
determinant to zero. Hence, by applying the invariance method we can obtain values of x.
1 6 . 1 4 | Matrices

3+x 5 2
1 7+x 6 = 0  … (i)
2 5 3+x

Now, using R1 → R1 – R3

3+x 5 2 1+x 0 −1 − x 1+x 0 0


1 7+x 6 = 1 7+x 6 ; using C3 → C3 + C1 = 1 7+x 7
2 5 3+x 2 5 3+ x 2 5 5+x

7+x 7
= (1 + x) = (1+ x) [(7 + x ( 5 + x) – 35] = (1 + x) (x2 + 12x) = x(1+ x) (x + 12)
5 5+x

\ (i) holds for x = 0, –1, –12

3 5 2 
  3 5 
When x = 0, the matrix = 1 7 6  Clearly, a minor   ≠ 0, So, the rank = 2
2 5 3  1 7 
 

2 5 2 
  2 5 
When x = –1, the matrix = 1 6 6  Clearly, a minor   ≠ 0, So, the rank = 2
2 5 2  1 6 
 

 −9 5 2
   −9 5 
When x = –12, the matrix =  1 −5 6  Clearly, a minor   ≠ 0, So, the rank = 2
 2 5 −9   1 −5
 
∴ The matrix has the rank 2 if x = 0, –1, –12.

8. POSITIVE INTEGRAL POWERS OF A SQUARE MATRIX


The positive integral powers of a matrix A are defined only when A is a square matrix.
Also then, A2 = A.A; A3 = A.A.A = A2A. Also for any positive integers m, n:

(a) AmAn = Am+n (b) (Am)n = Amn = (An)m


(c) In = I, Im= I (d) A0 = In

Matrix polynomial: If f(x) = a0xn + a1xn–1 + a2xn–2 +………. + anx0, then we define a matrix polynomial a,b
f(A) = a0An + a1An–1 + a2An–2 +………. + anIn where A is the given square matrix. If f(A) is a null matrix, then A is called
the zero or root of the matrix polynomial f(A)

9. SPECIAL MATRICES
(a) Idempotent Matrix: A square matrix is idempotent, provided A2 = A. For an idempotent matrix A, An = A ∀
n>2, n ∈ N ⇒ An = A, n ≥ 2.
For an idempotent matrix A, det A = 0 or 1 A2, ‫ ׀‬A ‫ ׀‬2 = ‫ ׀‬A ‫)׀‬.
(b) Nilpotent Matrix: A nilpotent matrix is said to be nilpotent of index p, (p ∈ N), if Ap = O, Ap-1 ≠ O, i.e. if p is
the least positive integer for which Ap = O, then A is said to be nilpotent of index p.
M a them a ti cs | 16.15

(c) Periodic Matrix: A square matrix which satisfies the relation AK+1 = A, for some positive integer K, then A is
periodic with period K, i.e. if K is the least positive integer for which AK+1 = A, and A is said to be periodic with
period K. If K =1 then A is called idempotent.

 2 −3 −5 
 
E.g. the matrix  −1 4 5  has the period 1.
 1 −3 −4 
 
Note: (i) Period of a square null matrix is not defined. (ii) Period of an idempotent matrix is 1.

(d) Involutary Matrix: If A2 = I, the matrix is said to be an involutary matrix. An involutary matrix its own inverse

0 1  0 1  1 0 
E.g. (i) A =     = 
1 0  1 0  0 1 

PLANCESS CONCEPTS

Two matrices cannot be added if they are of different order


1 1
If A is an involutary matrix, then (I + A) and (I – A) are idempotent and (I + A)(I – A) = 0
2 2
Chinmay S Purandare (JEE 2012 AIR 698)

 2 0 1
 
Illustration 13: Let A =  2 1 3  and f(x) = x2 – 5x + 6I3. Find f(A). (JEE MAIN)
 −1 −1 0 
 

Sol: By using methods of multiplication and addition of matrices we will obtain the required result. Here f(A) =
A2 – 5A + 6I3
2
 2 0 1  2 0 1 1 0 0 
     
=  2 1 3 – 5  2 1 3 + 6 0 1 0 
 −1 −1 0   −1 −1 0  0 0 1 
     

 2 0 1  2 0 1  10 0 5  6 0 0 
       
=  2 1 3  ×  2 1 3  – 10 5 15 + 0 6 0 
 −1 −1 0   −1 −1 0   −5 −5 0  0 0 6 
       

 2 × 2 + 0 × 2 + 1(–1) 2 × 0 + 0 × 1 + 1(–1) 2 × 1 + 0 × 3 + 1 × 0   6 − 10 0−0 0 −5 


   
=  2 × 2 + 1 × 2 + 3(–1) 2 × 0 + 1 × 1 + 3(–1) 2 × 1 + 1 × 3 + 3 × 0  +  0 − 10 6 −5 0 − 15 
( −1)2 + ( −1)2 + 0(–1) ( −1)0 + ( −1)1 + 0(–1) ( −1)1 + ( −1)3 + 0 × 0  0 − ( −5) 0 − ( −15) 6 − 0 
   

 3 −1 2   −4 0 −5   3 − 4 −1 + 0 2 − 5   −1 −1 −3 
       
=  3 −2 5  +  −10 1 −15 =  3 − 10 −2 + 1 5 − 15  =  −7 −1 −10 
 −4 −1 −4   5 5 6   −4 + 5 −1 + 5 −4 + 6  1 4 2 
      

a b
Illustration 14: Let A =   be such that A3 = 0, but A ≠ 0, then (JEE MAIN)
 c d 

Sol: (a) As A3 = 0, we get | A3 | = 0; | A3 | = 0 ⇒ | A | = 0 ⇒ ad – bc = 0


1 6 . 1 6 | Matrices

In this problem, A3 = 0 means | A | also is equal to 0; therefore, by calculating A2 we can obtain the result.
(a) A2 = 0 (b) A2 = A (c) A2 = I – A (d) None of these

 a2 + bc (a + d)b   a2 + ad (a + d)b 
Also, A2 =   =   = (a + d) A
 (a + d)c bc + d2   (a + d)c ad + d2 
   

If a + d = 0, we get A2 = 0. But, if a + d ≠ 0, then A3 = A2A = (a + d) A2 ⇒ 0 = (a + d) A2 ⇒ A2 = 0

1 0  1 0 
Illustration 15: If A =   and I =   then which one of the following holds for all n > 1, by the principle
1 1  0 1 
of mathematical induction. (JEE MAIN)
(a) An = nA + (n – 1) I (b) An = 2n–1 A + (n – 1) I
(c) An = nA – (n – 1) I (d) An = 2n–1 A – (n – 1) I

Sol: By substituting n = 2 we can determine the correct answer.

1 0  3 0 
For n = 2, A2 =   For n = 2, RHS of (a) = 2A + I = 3  ≠ A
2

0 1   2 3 
For n = 2, RHS of (b) = 2A + I ≠ A2 So possible answer is (c) or (d)

1 0 
In fact An =   which equals nA – (n – 1) I;
n 1 
0 0 
Alternatively. Write A = I + B Where B =  
1 0 
As B2 = 0, we get Br = 0 ∀r > 2

By the binomial theorem, An = I + nB = I + n(A – I) = nA – (n –1)I

10. ADJOINT OF A MATRIX


Let the determinant of a square matrix A be | A |

 a11 a12 a13  a11 a12 a13


 
If A = a21 a22 a23  Then | A | = a21 a22 a23
a a33 
 31 a32 a31 a32 a33
 A11 A12 A13 
 
The matrix formed by the cofactors of the elements in | A | is  A21 A22 A23 
A A32 A33 
 31
a22 a23
Where A11 = (–1)1+1 = a22 a33 – a23. a32
a32 a33

a21 a23 a a22


A12 = (–1)1+2 = –a21. a33 + a23. a31; A13 = (–1)1+3 21 = a21 a32 – a22 a31;
a31 a33 a31 a32

a12 a13 a a13


A21 = (–1)2+1 = –a12 a33 + a13. a32; A22 = (–1)2+2 11 = a11 a33 – a13. a31;
a32 a33 a31 a33
M a them a ti cs | 16.17

a11 a12 a12 a13


A23 = (–1)2+3 = –a11 a32 + a12. a31; A31 = (–1)3+1 = a12 a23 – a13. a22;
a31 a32 a22 a23

a11 a13 a a12


A32 = (–1)3+2 = –a11 a23 + a13 a21; A33 = (–1)3+3 11 = a11 a22 – a12. a21;
a21 a23 a21 a22

Then the transpose of the matrix of co-factors is called the adjoint of the matrix A and is written as

 A11 A21 A31 


 
adj A. adj A=  A12 A22 A32 
A A23 A33 
 13
The product of a matrix A and its adjoint is equal to unit matrix multiplied by the determinant A.
Let A be a square matrix, then (Adjoint A). A = A. (Adjoint A) = | A |. I

 a11 a12 a13   A11 A21 A31 


   
Let A = a21 a22 a23  and adj A =  A12 A22 A32 
a a32 a33  A A23 A33 
 31  13
 a11 a12 a13   A11 A21 A31 
   
A. (adj. A) = a21 a22 a23  ×  A12 A22 A32 
a a32 a33  A A23 A33 
 31  13
 a11 A11 + a12 A12 + a13 A13 a11 A21 + a12 A22 + a13 A23 a11 A31 + a12 A32 + a13 A33 
 
= a21 A11 + a22 A12 + a23 A13 a21 A21 + a22 A22 + a23 A 23 a21 A31 + a22 A32 + a23 A33 
a A + a A + a A a31 A21 + a32 A22 + a33 A 23 a31 A31 + a32 A32 + a33 A33 
 31 11 32 12 33 13

| A | 0 0  1 0 0 
   
=  0 | A | 0  = | A | 0 1 0  = |A| I.
 0 0 | A | 0 0 1 
  

x 3 2
 
Illustration 16: If A =  −3 y −7  and A = – A’, then x + y is equal to
 −2 7 0 
 
(a) 2 (b) –1 (c) 0 (d) 12 (JEE MAIN)

Sol: (c) A = – A’; ⇔ A is skew-symmetric matrix; ⇒ diagonal elements of A are zeros


⇒ x = 0, y = 0; ∴ x + y = 0

Illustration 17: If A and B are two skew-symmetric matrices of order n, then,  (JEE MAIN)
(a) AB is a skew-symmetric matrix (b) AB is a symmetric matrix
(c) AB is a symmetric matrix if A and B commute (d) None of these

Sol: (c) We are given A’ = – A and B’ = – B; Now, (AB)’ = B’A’ = (–B) (–A) = BA = AB if A and B commute.

Illustration 18: Let A and B be two matrices such that AB’ + BA’ = O. If A is skew symmetric ,then BA  (JEE MAIN)
(a) Symmetric (b) Skew symmetric (c) Invertible (d) None of these

Sol: (c) we have, (BA)’ = A’B’ = – AB’ [ A is skew symmetric]; = BA’ = B(–A) = – BA ⇒ BA is skew symmetric.
1 6 . 1 8 | Matrices

1 2 3 
 
Illustration 19: Let A = 1 3 4  , then the co-factors of elements of A are given by - (JEE MAIN)
1 4 3 
 
Sol: Co-factors of the elements of any matrix are obtain by eliminating all the elements of the same row and
column and calculating the determinant of the remaining elements.
3 4
A11 = =3×3–4×4=–7
4 3

1 4 1 3 2 3 1 3
A12 = – = 1, A13 = = 1; A21 = – = 6, A22 = =0
1 3 1 4 4 3 1 3

1 2 2 3 1 3 1 2
A23 = – = – 2, A31 = = – 1; A32 = – = – 1, A33 = =1
1 4 3 4 1 4 1 3

−7 6 −1
∴ Adj A = 1 0 −1
1 −2 1

Illustration 20: Which of the following statements are false – (JEE MAIN)
(a) If | A | = 0, then | adj A | = 0;
(b) Adjoint of a diagonal matrix of order 3 × 3 is a diagonal matrix;
(c) Product of two upper triangular matrices is a upper triangular matrix;
(d) adj (AB) = adj (A) adj (B);

Sol: (d) We have, adj (AB) = adj (B) adj (A) and not adj (AB) = adj (A) adj (B)

11. INVERSE OF A MATRIX


If A and B are two square matrices of the same order, such that AB = BA = I (I = unit matrix)
Then B is called the inverse of A, i.e. B = A–1 and A is the inverse of B. Condition for a square matrix A to possess an inverse
is that the matrix A is non-singular, i.e., | A | ≠ 0. If A is a square matrix and B is its inverse then AB = I. Taking determinant
of both sides | AB | = | I | or | A | | B | = I. From this relation it is clear that | A | ≠ 0, i.e. the matrix A is non-singular.
To find the inverse of matrix by using adjoint matrix:
A.(AdjA)
We know that, A. (Adj A) = | A | I or = I (Provided | A | ≠ 0)
|A|
1
and A. A–1 = I; \ A–1 = (Adj. A)
|A|

1 0 −1 
 
Illustration 21: Let A = 3 4 5  . What is inverse of A ? (JEE MAIN)
0 −6 −7 
 
adj A
Sol: By using the formula A–1 = we can obtain the value of A–1.
|A|
M a them a ti cs | 16.19

4 5 3 5 
We have A11 =   = 2 A12 = –   = 21
 −6 −7  0 −7 

And similarly A13 = –18, A31 = 4, A32 = –8, A33 = 4, A21 = +6, A22 = - 7, A23 = 6

 2 6 4 1 0 −1
 
∴ adj A =  21 −7 −8  Also | A | = 3 4 5 = {4×(–7) – (–6)×5 – 3×(–6)}
 −18 6 4  0 −6 −7

 2 6 4
adj A 1  
= – 28 + 30 + 18 = 20 \ A = –1
=  21 −7 −8 
|A| 20
 −18 6 4 

1 1  3 2
Illustration 22: If the product of a matrix A and   is the matrix   , then A is given by: (JEE MAIN)
–1

 2 0  1 1 
0 −1   0 −1  0 1 
(a)   (b)   (c)   (d) None of these
2 −4   −2 −4  2 −4 

Sol: (a) We know if AB = C, then B–1 A–1 = C–1 ⇒ A–1 = BC–1 by using this formula we will get value of A–1 in the above
problem.
−1
1 1  3 2 1 1  3 2 1 1   1 −2  0 1 
Here, A   =   ⇒ A = 
–1
  =   = 
2 0  1 1  2 0  1 1  2 0   −1 3  2 −4 

2 1 −1  1 2 5
   
Illustration 23: Let A = 0 1 0  and B =  2 3 1 . Prove that (AB)–1 = B–1A–1.  (JEE ADVANCED)
1 3 −1  −1 1 1 
   
adj AB
Sol: By obtaining | AB | and adj AB we can obtain (AB)–1 by using the formula (AB)–1 = . Similarly we can also
| AB |
obtain the values of B–1 and A–1.Then by multiplying B–1 and A–1 we can prove the given problem.

2 1 −1  1 2 5  2 + 2 + 1 4 + 3 − 1 10 + 1 − 1 5 6 10 
       
Here, AB = 0 1 0   2 3 1  = 0 + 2 + 0 0 + 3 + 0 0 + 1 + 0  =  2 3 1 
1 3 −1  −1 1 1  1 + 6 + 1 2 + 9 − 1 5 + 3 − 1  8 10 7 
       

5 6 10
Now, | AB | = 2 3 1 = 5(21 – 10) – 6(14 – 8) + 10(20 – 24) = 55 – 36 – 40 = – 21.
8 10 7

 3(7) − 1(10) −{2(7) − 8(1)} 2(10) − 3(8)   11 −6 −4 


   
The matrix of cofactors of | AB | is =  −{6(7) − 10(10)} 5(7) − 8(10) −{5(10 − 6(8)}=   58 −45 −2 
 5(3) − 6(2)   3 
 6(1) − 10(3) −{5(1) − 2(10)}  −24 15

11 58 −24  11 58 −24 


adj AB −1  
∴ adj AB =  −6 −45 
15  ; ∴ (AB)–1 = = −6 −45 15 
| AB | 21 
 −4
 −2 3   −4 −2 3 
1 6 . 2 0 | Matrices

1 2 5
Next, | B | = 2 3 1 = 1(3 – 1) – 2(2 + 1) + 5(2 + 3) = 21
−1 1 1

 2 3 −13 2 1 −1
adj B 1    
∴B = –1
=  −3 6 9  ; | A | = 0 1 0  = 1 (–2 + 1) = – 1
|B | 21
 5 −3 −1  1 3 −1
   

 −1 −2 1   2 3 −13  −1 −2 1 
adj A 1   1    
∴ A–1 = = 0 −1 0 ; ∴ B A
–1 –1
= – −3 6 9   0 −1 0 
|A| −1   21 
 −1 −5 2   5 −3 −1   −1 −5 2 
     

11 58 −24 
1  
=– −6 −45 15  Thus, (AB)–1 = B–1 A–1
21 
 −4 −2 3 

0 2y z 
 
Illustration 24: If A =  x y −z  satisfies A’ = A–1, then (JEE ADVANCED)
 x −y z 
 
(a) x = ± 1/ 6 , y = ± 1/ 6 , z = ± 1/ 3 (b) x = ± 1/ 2 , y = ± 1/ 6 , z = ± 1/ 3

(c) x = ± 1/ 6 , y = ± 1/ 2 , z = ± 1/ 3 (d) x = ± 1/ 2 , y = ± 1/3, z = ± 1/ 2

Sol: (b) Given that A’ = A–1 and we know that AA–1= I and therefore AA’ = I. Using the multiplication method we can
obtain values of x, y and z.
A’ = A–1 ⇔ AA’ = I
0 2y z   0 x x  4y 2 + z 2 2y 2 − z 2 −2y 2 + z 2 
     
Now, AA’ =  x y −z  2y y − y  =  2y 2 − z 2 x2 + y 2 + z2 x2 − y 2 − z2 
 x − y z   z −z z   2 2 
     −2y + z x2 − y 2 − z2 x2 + y 2 + z2 

Thus, AA’ = I ⇒ 4y2 + z2=1, 2y2 – z2 = 0, x2 + y2 + z2 =1, x2 – y2 – z2 = 0

\ x = ± 1/ 2 , y = ± 1/ 6,z=±1/ 3

0 1 2  1 / 2 −1 / 2 1 / 2
   
Illustration 25: If A = 1 2 3  and A–1 =  −4 3 y  , then (JEE ADVANCED)
3 x 1  5 / 2 −3 / 2 1 / 2
   
(a) x = 1, y = –1 (b) x = –1, y = 1
1
(c) x = 2, y = –1/2 (d) x = 1/2, y =
2
Sol: (a) We know AA–1= I, hence by solving it we can obtain the values of x and y.
We have

1 0 0  0 1 2  1 / 2 −1 / 2 1 / 2  1 0 y +1 
       
0 1 0  = AA = 1 2 3   −4 3 y  =  0 1 2(y + 1)
–1

0 0 1  3 x 1  5 / 2 −3 / 2 1 / 2  4(1 − x) 3(x − 1) 2 + xy 
       

⇒ 1 – x = 0, x – 1 = 0; y + 1 = 0, y + 1 = 0, 2 + xy = 1; \ x = 1, y = – 1
M a them a ti cs | 16.21

12. SYSTEM OF LINEAR EQUATIONS


Let the equations be
a1x + a2y + a3z = d1
b1x + b2y + b3z = d2
c1x + c2y + c3z = d3
We write the above equations in the matrix form as follows

 a1 x + a2 y + a3 z  d1   a1 a2 a3  x d1 


         
b1 x + b2 y + b3 z  = d2  ⇒ b1 b2 b3  y  = d2  ⇒ AX = B  … (i)
c x +c y +c z  d  c c2 c3  z  d 
 1 2 3   3  1    3
 a1 a2 a3  x d1 
     
Where, A = b1 b2 b3  , X =  y  and B = d2 
c c2 c3  z  d 
 1    3

Multiplying (i) by A–1, we get A–1AX = A–1B ⇒ I. X = A–1B ⇒ X = A–1B

12.1 Solution to a System of Equations


A set of values of x, y, z which simultaneously satisfy all the equations is called a solution to the system of equations.
Consider, x + y + z = 9 2x – y + z = 5 4x + y – z = 7
Here, the set of values – x = 2, y =3, z = 4, is a solution to the system of linear equations.
Because, 2 + 3 + 4 = 9 4 – 3 + 4 = 5 8+3–4=7

12.2 Consistent Equations


If the system of equations has one or more solution, then it is said to be a consistent system of equations, otherwise
it is an inconsistent system of equations. For example, the system of linear equations x + 3y = 5 x – y = 1 is
consistent, because x = 2, y = 1 is a solution to it. However, the system of linear equations x + 3y = 5 2x + 6y = 8
is inconsistent, because there is no set of values of x and y which may satisfy the two equations simultaneously.
Condition for consistency of a system of linear equation AX = B
(a) If |A| ≠ 0, then the system is consistent and has a unique solution, given by X = A–1B
(b) If |A| = 0, and (Adj A) B ≠ 0 then the system is inconsistent.
(c) If |A| = 0, and (Adj A) B = 0, then the system is consistent and has infinitely many solutions.
Note, AX = 0 is known as homogeneous system of linear equations, here B = 0. A system of homogeneous
equations is always consistent.
The system has non-trivial solution (non-zero solution), if | A | = 0

Theorem 1: Let AX = B be a system of linear equations, where A is the coefficient matrix. If A is invertible then the
system has a unique solution, given by X = A–1 B

Proof: AX = B; Multiplying both sides by A–1. Since A–1 exists ⇒ | A | ≠ 0


⇒ A–1 AX = A–1B ⇒ IX = A–1B ⇒ X = A–1B
Thus, the system of equations AX = B has a solution given by X = A–1B

Uniqueness: If AX = B has two sets of solutions X1 and X2, then


AX1 = B and AX2 = B (Each equal to B) ⇒ AX1 = AX2
1 6 . 2 2 | Matrices

By cancellation law, A being invertible ⇒ X1 = X2


Hence, the given system AX = B has a unique solution. Proved
Note: A homogeneous system of equations is always consistent.

x + y y  2 3 
Illustration 26: Let A =   , B =   and C =   . If AB = C. Then find the matrix A (JEE MAIN)
2

 2x x − y   −1 2 
Sol: By solving AB = C we get the values of x and y. Then by substituting these values in A we obtain A2.

x + y y  2 3   2(x + y) – y  3 
Here     =   ⇒   =   ⇒ 2(x + y) – y = 3 and 4x – (x – y) = 2
 2x x − y −1
  2
  2x .2 – (x − y) 2 
⇒ 2x + y = 3 and 3x + y= 2 Subtracting the two equations, we get, x = –1, So, y = 5.

 −1 + 5 5  4 5 4 5 4 5
∴A=   =   ; ∴ A2 =    
 2( −1) −1 − 5  −2 −6   −2 −6   −2 −6 

 4 × 4 + 5(–2) 4 × 5 + 5( −6)   6 −10 


=   =  
 −2 × 4 + ( −6)( −2) −2 × 5 + ( −6)( −6)  4 26 

Illustration 27: Solve the following equations by matrix inversion


2x + y + 2z = 0 2x – y + z = 10 x + 3y – z = 5 (JEE ADVANCED)

Sol: The given equation can be written in a matrix form as AX = D and then by obtaining A–1 and multiplying it on
both sides we can solve the given problem.

2 1 2  x 0 2 1 2  x 0


           
2 −1 1   y  = 10  ∴ AX = D where A = 2 −1 1  , X =  y  , D= 10 
1 3 −1 z  5 1 3 −1 z  5
           
⇒ A–1(AX) = A–1D ⇒ (A–1A)X = A–1D ⇒ IX = A–1D ⇒ X = A–1D …(i)

2 1 2
adj A
Now A = –1
; |A| = 2 −1 1 = 2(1 – 3) – 1 (– 2 –1) + 2(6 + 1) = 13
|A|
1 3 −1

 −2 3 7  −2 7 3
   
The matrix of co-factors of | A | is  7 −4 −5  . So, adj A =  3 −4 2 
 3 2 −4   7 −5 −4 
   

 −2 7 3  −2 7 3 0
1   1    
∴A = –1
3 −4 2  . ∴ from (i), X = 3 −4 2  10 
13  13 
 7 −5 −4   7 −5 −4  5
     

 0 + 70 + 15   85 / 13  x  85 / 13 
1         85 −30 −70
= 0 − 40 + 10 = −30 / 13 ; ∴ y  =  −30 / 13 ⇒ x = 13 , y = 13 , z = 13
13    
 0 − 50 − 20   −70 / 13 z   −70 / 13
       
M a them a ti cs | 16.23

2 1   −3 2  1 0 
Illustration 28: If 7 4  A   =   , then matrix A equals:
   5 −3 0 1 

 7 5 2 1   7 1  5 3
(a)   (b)   (c)   (d)    (JEE ADVANCED)
 −11 −8  5 3 34 5 13 8 

Sol: (a) We know that if XAY = I, then A = X–1 Y–1 = (YX)–1.


−1
 −3 2  2 1   8 5  8 5  7 5
In this case YX =     =   ; ∴ A =   =  
 5 −3 7 4   −11 −7   −11 −7  −11 −8 

 3 −2 1   x  b
     
Illustration 29: The system of equations  5 −8 9   y  =  3  has no solution if a and b are
2 1 a  z   −1 
     
(a) a = –3, b ≠ 1/3 (b) a = 2/3, b ≠ 1/3
(c) a ≠ 1/4, b = 1/3 (d) a ≠ –3, b ≠ 1/3 (JEE ADVANCED)

Sol: By applying row operation in the given matrices and comparing them we can obtain the required result.

 3 −2 1 b 
 
(a) The augmented matrix is given by (A|B) =  5 −8 9 3 
 2 1 a −1 
 
 1 4 −7 2b − 3 
 
Applying R1 →2R1 – R2, we get (A|B) ~  5 −8 9 3 
2 1 a −1 

1 4 −7 2b − 3 
 
Applying R2 →R2 – 5R1, R3 → R3 – 2R1, we get (A|B) ~  0 −28 44 18 − 10b 
 0 −7 a + 14 5 − 4b 
 
−28 44 18 − 10b
The system of equations will have no solution if = ≠
−7 a + 14 5 − 4b
⇒ a + 14 = 11 and 20 – 16b ≠ 18 – 10b
⇒ a = –3 and b ≠ – 1/3.

1 0 0 1 
   
Illustration 30: Let A =  2 1 0  . If u1 and u2 are column matrices such that Au1 =  0  and
3 2 1 0
   
0
 
Au2 =  1  , then u1 + u2 equals:
0
 

 −1   −1  1   −1 
       
(a)  1  (b)  −1  (c)  −1  (d)  1  (JEE ADVANCED)
 −1  0  −1  0
       
1 6 . 2 4 | Matrices

Sol: (c) Adding Au1 and Au2 we get A(u1 + u2). Then using the invariance method we obtain u1 + u2.

1  0 1 
     
By adding, we have A(u1 + u2) = Au1 + Au2 =  0  + 1  = 1 
0 0 0
     
1 0 0 1 
 
We then solve the above equation for u1 + u2, if we consider the augmented matrix (A|B) =  2 1 0 1 
3 2 1 0
 
1 0 0 1  1 
   
Applying R3 →R3 – 2R2 + R1 and R2 →R2 – 2R1, we get (A|B) ~  0 1 0 −1  ⇒ u1 + u2 =  −1 
 0 0 1 −1   −1 
   

PROBLEM-SOLVING TACTICS

If A, B are square matrices of order n, and In is a corresponding unit matrix, then

(a) A(adj.A) = | A | In = (adj A) A

(b) | adj A | = | A |n-1 (Thus A (adj A) is always a scalar matrix)

(c) adj (adj.A) = | A |n-2 A


2
(d) | adj (adj.A) | = | A |(n-1)

(e) adj (AT) = (adj A)T

(f) adj (AB) = (adj B) (adj A)

(g) adj (Am) = (adj A)m, m ∈ N

(h) adj (kA) = kn–1 (adj. A) , k ∈ R

(i) adj (In) = In

( j) adj 0 = 0

(k) A is symmetric ⇒ adj A is also symmetric

(l) A is diagonal ⇒ adj A is also diagonal

(m) A is triangular ⇒ adj A is also triangular

(n) A is singular ⇒ | adj A | = 0


M a them a ti cs | 16.25

FORMULAE SHEET

(a) Types of matrix:


(i) Symmetric Matrix: A square matrix A = [aij] is called a symmetric matrix if aij = aji, for all i,j.
(ii) Skew-Symmetric Matrix: when aij = –aji
(iii) Hermitian and skew – Hermitian Matrix: A = Aθ (Hermitian matrix)
Aθ = – A (skew-Hermitian matrix )
(iv) Orthogonal matrix: if AAT = In = ATA
(v) Idempotent matrix: if A2 = A
(vi) Involuntary matrix: if A2 = I or A–1 = A
(vii) Nilpotent matrix: if ∃ p ∈ N such that AP = 0

(b) Trace of matrix:


(i) tr(λA) = λ tr(A)
(ii) tr(A + B) = tr(A) + tr(B)
(iii) tr(AB) = tr(BA)
(c) Transpose of matrix:
(i) (AT)T = A (ii) (A ± B)T = AT ± BT (iii) (AB)T = BTAT (iv) (kA)T = k(A)T

(v) (A1 A2 A3 ..........An−1 An )T = AnT AnT−1 …….. A3T A2T A1T (vi) IT = I (vii) tr(A) = tr(AT)

(d) Properties of multiplication:


(i) AB ≠ BA (ii) (AB)C = A(BC) (iii) A.(B + C) = A.B + A.C

(e) Adjoint of a Matrix:


(i) A(adj A) = (adj A)A = | A | In (ii) | adj A | = | A |n–1
(iii) (adj AB) = (adj B) (adj A) (iv) adj (adj A) = | A |n–2
(v) (adj KA) = Kn–1(adj A)

(e) Inverse of a matrix: A–1 exists if A is non singular i.e. | A | ≠ 0


1
(i) A–1 = (Adj. A) (ii) A–1A = In = AA–1
|A|

(iii) (AT)–1 = (A–1)T (iv) ( A–1 )–1 = A

(v) | A–1 | = | A |–1 = 1


|A|
1 6 . 2 6 | Matrices

Solved Examples

JEE Main/Boards Similarly,

0 0 0 1 1 3
 x−y 2x + z   −1 5     
Example 1: If   =  , ∴ A = A . A = 3 3 9 × 5 2 6  =
3 2

3x + y 3z + 4w   5 25  −1 −1 −3  −2 −1 −3


   
find x, y, z, w.
0 0 0 
 
Sol. We know that in equal matrices the corresponding 0 0 0 
elements are equal. Therefore, by equating the elements 0 0 0 
 
of these two matrices which have the same number of
rows and columns, we get the value of x, y, z and w. ⇒ A3 = 0.

 x−y 2x + z   −1 5  i.e. Ak = 0
Given  =  
3x + y 3z + 4w   5 25 Here k = 3
x – y = –1, Hence, A is nilpotent matrix of index 3
2x + z = 5;
3x + y = 5, Example 3: Solve the following system of homogeneous
equations:
3z + 4w = 25
2x + 3y – z = 0, x – y – 2z = 0 and
By solving these equations, we get
3x + y + 3z = 0
x = 1, y = 2, z = 3, w = 4
Sol: In this problem we can write the given homogeneous
Example 2: Show that the matrix equations in a matrix form, i.e. [A][X] = [O] and then by
calculating the determinant of matrix A we can find if
1 1 3 that given system has a trivial solution or not.
 
 5 2 6  is a nilpotent matrix of index 3 The given system can be written as
 −2 −1 −3
 
2 3 −1  x  0 
     
Sol: Value of the index at which all elements of the 1 −1 −2  y  = 0  or AX = O
matrix become 0, i.e. null matrix, is called the nilpotent 3 1 3   z  0 
     
matrix of that index. Here we calculate the nth power of
the matrix, where n =1, 2, 3, …. The value of n at which 2 3 −1 x 0 
the matrix becomes null matrix is the index value.      
Where, A = 1 −1 −2 X =  y  and O = 0 
1 1 3 3 1 3  z  0 
     
 
Given A =  5 2 6 
 −2 −1 −3 2 3 −1
  Now, | A | = 1 −1 −2
1 1 3 1 1 3 3 1 3
   
⇒ A = A×A=  5 2 6  ×  5 2 6  =
2
= 2(– 3 + 2) – 3 (3 + 6) – 1(1 + 3)
 −2 −1 −3  −2 −1 −3
   
= –2 – 27 – 4 = –33 ≠ 0
0 0 0
  Thus | A |≠ 0.
3 3 9
 −1 −1 −3 So the given system has only the trivial solution given
  by x = y = z = 0
M a them a ti cs | 16.27

Example 4: Find x, y, z and a for which 4 5


C31 = = ( 24 − 10 ) =
14 ,
 x + 3 2y + x  0 −7  2 6
 =  
 z − 1 4a − 6  3 2a  1 5
C32 = – = − ( 6 − 15 ) =9,
Sol: We know that, in equal matrices the corresponding 3 6
elements are equal. Therefore by equating the elements 1 4
of these two matrices which have the same number of C33 = = ( 2 − 12 ) =
–10,
3 2
rows and columns we get the values of x, y, z and w.
T
 x + 3 2y + x  0 −7   −6 0 3   −6 5 14 
Given,  =      
 z − 1 4a − 6  3 2a  ∴ adj A =  5 0 −1  =  0 0 9 
14 9 −10   3 −1 −10 
We know, that for equal matrices the corresponding    
elements are equal, therefore
x+3 = 0; cos θ sin θ  1 n
Example 6: If A =   , Then find nlim A.
2y + x =−7;  sin θ cos θ  →∞ n

z −1 = 3;
Sol: For this problem, we first have to calculate the nth
4a − 6 = 2a; power of matrix A, i.e. An, and multiply the matrix An
By solving these equations, we get 1
by .
n
∴ x = – 3, z = 4, y = – 2, a = 3.
Then, by with the given limit we can find the solution
of this problem.
Example 5: Compute the adjoint of the matrix
cos θ sin θ 
1 4 5  Given A =  
   sin θ cos θ 
A = 3 2 6 
0 1 0  cos θ sin θ 
n
 cosnθ sinnθ 
 
A =
n
 =  
Sol: For this problem, we use the formula to get the co-  sin θ cos θ   − sinnθ cosnθ 
factors of all the elements of matrix A. Then by taking  cosnθ sinnθ 
the transpose of the co-factor matrix we can get the 1 n  n 
⇒ A =  n 
adjoint of matrix A.
n  − sinnθ cosnθ 
Consider Cij be a co-factor of aij in matrix A.  n n 
Then the co-factors of the elements of A are given by But – 1 ≤ cos nθ, sin nθ ≤ 1;
2 6 cos nθ
C11 = =0 – 6 = − 6 , ∴ lim = 0,
1 0 n→∞ n

3 6 sinnθ
lim = 0,
C12 = – = 0, n→∞ n
0 0
3 2 1 n 0 0 
∴ lim A =  
C13 = = 3 – 0 = 3, n→∞ n 0 0 
0 1
4 5
C21 = – = − (0 − 5) =
5 Example 7: A trust fund has Rs. 50,000 that is to be
1 0 invested into two types of bonds. The first bond pays 5%
1 5 interest per year and the second bond pays 6% interest
C22 = = 0, per year. Using matrix multiplication determine how to
0 0
divide by Rs, 50,000 among the two types of bonds so as
1 4 to obtain an annual total interest of Rs. 2,780.
C23 = – = –(1 – 0) = −1 ,
0 1
Sol: In this problem, investment amounts can be written
in the form of a row matrix and interest amounts can
1 6 . 2 8 | Matrices

be written in the form of column matrix. By multiplying  cos γ sin γ 


these two matrix we will get the equation for annual f ( γ ) = 
interest rates. By equating this to the given annual  − sin γ cos γ 
interest value we will get the required answer.  cos α sin α   cos β sin β 
f(α)f(β) =    
Consider investment of first type of bond = Rs. x  − sin α cos α   − sin β cos β 
And second type of bond = Rs. 50,000 – x  cos α cos β − sin α sin β cos α sin β + sin α cos β 
= 
These amounts can be written in the form of a row  − sin α cos β − cos α sin β − sin α sin β + cos α cos β 
matrix A which is given by  cos(α + β) sin(α + β) 
=  
A =  x 50000 − x   − sin(α + β) cos(α + β)
1×2
Similarly f(α) f(β) f(γ)
The interest amounts per rupee, per year from the two
5 6  cos(α + β + γ ) sin(α + β + γ ) 
bonds are Rs. and which can be written in the =  
100 100  − sin(α + β + γ ) cos(α + β + γ )
form of a column matrix B which is given by
 cos π sin π 
=   and as α + β + γ = p
 5   − sin π cos π 
 
B =  100   −1 0 
= 
1 0 
 6   =–   = – I2
 0 −1  0 1 
 100  2×1
∴ The total interest per year is given by cos α − sin α 0 
 
 5  Example 9: If M(α) =  sin α cos α 0 
   0 0 1 
A.B = [x 50,000 – x] ×  100  
 6 
 100   cos β 0 sin β 
 
M(β) =  0 1 0 
= [x. 5/100 + (50,000 – x). 6/100]
 − sin β 0 cos β 
 
= [3000 – x/100]
then prove that [M(α) M (β)]–1 = M ( −β ) M ( −α )
Since the required total annual interest is
= Rs.2,780. ∴ [3000 – x/100] = [2780] Sol: In this problem, by finding the inverse of the matrix
we can easily get the required answer.
⇒ 3000 – x/100 = 2780
[M(α) M (β)]–1 = M(β)–1 M(α)–1
⇒ x = 100(3000 – 2780) = 22,000
cos α − sin α 0 
Hence the required amounts to be invested in the two  
bonds are Rs. 22,000 and Rs. (50,000 – 22,000), i.e. Rs. Given M(α) =  sin α cos α 0 
22,000 and Rs. 28,000 respectively.  0 0 1 

T
cos α − sin α 0 
 cos α sin α   
Example 8: If f(α) =   and if α, β, γ are M(α) =  sin α cos α 0 
–1

 − sin α cos α   0
 0 1 
the angles of a triangle, then prove that f(α). f(β) ,
f(γ) = –I2  cos α sin α 0 
 
=  − sin α cos α 0 
Sol: In this problem, by the methods of substitution  0 0 1 
and multiplication of matrices we can easily prove the 
given equation. We can also write this in the form
 cos α sin α 
Given that f(α) =   cos( −α ) − sin( −α ) 0 
 − sin α cos α   
 sin( −α ) cos( −α ) 0  = M(– α)
 cos β sin β   0 0 1 
∴ f(β)=  
 and
 − sin β cos β 
M a them a ti cs | 16.29

Similarly, 1 −2 x


where A =  ,X=  
 cos β 0 − sin β  1 1  y 
 
M(β) =  0
–1
1 0 
 −k  1 −2
 sin β 0 cos β  and B =   Now, | A | = = 3 ≠ 0.
  k 1 1
 
 cos( −β) 0 sin( −β)   1 2
  So A–1 exists; We have, adj A =  
=  0 1 0  = M(–β)
 −1 1 
 − sin(–β) 0 cos( −β)
  1  1 2
1
So, A–1 = adj A ⇒ A–1 =  
∴ [M(α) M (β)] = M(–β) M(–α)
–1
|A| 3  −1 1 

Now X = A–1B
Example 10: Show that the homogeneous system of
equations x – 2y + z = 0, x + y – z = 0, 3x + 6y – 5z = 0  x  1  1 2  −k   k / 3 
has a non-trivial solution, Also, find the solution. ⇒  =    =  
 y  3  −1 1   k  2k / 3
Sol: In this problem we can write the given homogeneous ⇒ x = k/3, y = 2k/3
equations in a matrix form, i.e. [A][X] = [O] and then by
These values of x, y and z also satisfy the third equation.
calculating the determinant of matrix A we can find if
Hence x = k/3, y = 2k/3 and z = k, where k is any real
that given system has a non- trivial solution or not.
number and which satisfy the given system of equations.
The given equations are
x – 2y + z = 0,
JEE Advanced/Boards
x + y – z = 0,
3x + 6y – 5z = 0, Example 1: Let A and B be symmetric matrices of the
same order. Then show that
We can write these equations in the form of matrices
as shown below (i) A + B is symmetric

1 −2 1   x  0  (ii) AB – BA is skew-symmetric
     
1 1 −1   y  = 0  or AX = O, where (iii) AB + BA is symmetric
3 6 −5   z  0 
     
Sol: In this problem, by using the conditions for
1 −2 1  x 0  symmetric and skew-symmetric matrices we can get
      the required result.
A = 1 1 −1  , X =  y  and O = 0 
3 6 −5  z  0  As given, A and B are symmetric.
     
1 −2 1 ∴ A’ = A and B’ = B
Now, | A | = 1 1 −1 (i) (A + B)’ = A’ + B’ = A + B
3 6 −5 ∴ A + B is symmetric
= 1(– 5 + 6) + 2 (– 5 + 3) + 1(6 – 3) = 0 (ii) (AB – BA)’ = (A’B)’ – (BA)’
Thus, | A | = 0 = B’A’ – A’B’ [by reversal law]
Hence, the given system of equations has a non-trivial = BA – AB [A’ = A, B’ = B]
solution.
∴ AB – BA is skew-symmetric
To find the solution, we take z = k in the first two
(iii) (AB + BA)’ = (AB)’ + (BA)’
equations and write them as follows:
= B’A’ + A’B’= BA + AB = AB + BA
x – 2y = – k and x + y = k
∴ AB + BA is symmetric.
1 −2  x   −k 
or     =   or AX = B,
1 1   y   k 
1 6 . 3 0 | Matrices

Example 2: Solve the following equations: 1 2 2 


 
2x – 3y + z = 9 x + y + z = 6 x – y + z = 2 Example 3: Let A = 2 1 2 ,
2 2 1 
 
Sol: In this problem, we can write the given
homogeneous equations in a matrix form, i.e. [A][X] = prove that A – 4A – 5I = 0, hence obtain A–1:
2

[O]. Then, by calculating the determinant of matrix A


Sol: In this problem, by using a simple multiplication
and adjoint of A, we get an inverse of matrix A, i.e. A −1 .
method we can get the matrix A2, then by substituting
By multiplying this into [A][X] = [O] we get the required
these in the given equation we will easily obtain the
values of x, y, and z.
required result.
We can also find if that given system has a trivial
1 2 2  1 2 2 
solution or not.    
A = A. A = 2 1 2 2 1 2
2

As given, 2x – 3y + z = 9 2 2 1  2 2 1 
   
x+y+z=6
1 + 4 + 4 2 + 2 + 4 2 + 4 + 2  9 8 8
x–y+z=2    
= 2 + 2 + 4 4 + 1 + 4 4 + 2 + 2  =  8 9 8 
This system can be written as AX = B, 2 + 4 + 2 4 + 2 + 2 4 + 4 + 1 8 8 9
   
2 −3 1  x Now A2 – 4A – 5I
   
Where, A = 1 1 1 X = y  9 8 8   4 8 8  5 0 0 
1 −1 1 z       
    = 8 9 8  −  8 4 8  − 0 5 0 
9  8 8 9   8 8 4  0 0 5 
     
 
and B = 6  9 − 4 − 5 8 − 8 − 0 8 − 8 − 0  0 0 0 
2     
  = 8 − 8 − 0 9 − 4 − 5 8 − 8 − 0  =0 0 0 
   
|A|=2(2)+3(0)+1(-2) = 2 8 − 8 − 0 8 − 8 − 0 9 − 4 − 5  0 0 0 

 1 −1 1 1 1 1 
T = 0 [Here 0 is the zero matrix]
 − 
 −1 1 1 1 1 −1  Thus A2 – 4A – 5I = O
 −3 1 2 1 2 −3  ∴ A–1 A2 – 4A–1 A – 5A–1I = A–1O = O
−
Adj A = − 
 −1 1 2 1 1 −1  or (A–1A)A – 4(A–1A) – 5A–1I = O;
 
 −3 1 − 2 1 2 −3 
or IA – 4I – 5A–1 = O; ∴ 5A–1 = A – 4I
 1 1 1 1 1 1 

 1 2 2   4 0 0   −3 2 2 
 2 2 −4       
  =  2 1 2  − 0 4 0  =
 2 −3 2 
=  0 1 −1   2 2 1   0 0 4   2 2 −3
 −2 −1 5       
 
 −3 2 2   −3 / 5 2 / 5 2 / 5 
1 1    
∴ A −1 = Adj A ∴ A =  2 −3 2  =  2 / 5 −3 / 5 2 / 5 
–1

|A| 5
 2 2 −3  2 / 5 2 / 5 −3 / 5
   
Now, X = A −1B
Example 4: Find the product of two matrices
 2 2 −4  9   22 
1   1   −5 1 3
=  0 1 −1  6  =  4   
2 2 A and B where A =  7 1 −5 
 −2 −1 5  2   −14 
      1 −1 1 
1 1 2  
∴ x =11, y=2, z=-7 is the solution.  
B =  3 2 1  and use it for solving the equations
2 1 3
 
x + y + 2z =1, 3x + 2y + z = 7 and 2x + y + 3z = 2
M a them a ti cs | 16.31

Sol: As the given system of equations is in the form Sol: Pre-multiplying both sides by B–1 and Post-
BX = C, multiplying it by B–1, which is obtained by the multiplying both sides by A–1 in
multiplication of AB, we can get the required result.
1 0 1 
 −5 1 3  1 1 2  BPA =   we can find P.
    0 1 0 
AB =  7 1 −5 3 2 1 
 1 −1 1  2 1 3  1 0 1 
    Given BPA =  
 −5 + 3 + 6 −5 + 2 + 3 −10 + 1 + 9  4 0 0 0 1 0 
   
=  7 + 3 − 10 7 + 2 − 5 14 + 1 − 15  =  0 4 0  1 0 1  –1
 1−3+2 B–1BPA A–1= B–1   A
 1−2+1 2 − 1 + 3  0 0 4 
  0 1 0 

Also the given system of equations in matrix form is 1 0 1  –1


⇒ IPI = B–1   A
BX = C  … (ii) 0 1 0 
x 1 
    1 0 1  –1
Where X =  y  and C = 7  ⇒ P = B–1   A  …(i)
z  2  0 1 0 
   
From (ii), X = B–1C 2 3 
To find B–1, B=  
[Multiplying both sides of (ii) by B–1 3 4 

∴ B–1 B = I 2 3
|B|= = 8 – 9 = –1 ≠ 0
A 3 4
From (1), AB = 4I3 ∴ . B = I3
4 Let C be the matrix of co-factors of elements in | B |;
 −5 / 4 1 / 4 3/4  C
A   C12 
∴ B–1 = =  7 / 4 1 / 4 −5 / 4  C =  11 
4
 1 / 4 −1 / 4 1 / 4  C21 C22 
 
∴ C11 = 4 C12 = –3 C21 = – 3 C22 = 2
x
 
∴  y  = X = B–1C  4 −3
∴C=  
z   −3 2 
 

 −5 / 4 1 / 4 3/4  1  adjB C'


∴ B–1 = = = – C’
    |B | −1
=  7 / 4 1 / 4 −5 / 4  7 
 1 / 4 −1 / 4 1 / 4  2   4 −3  −4 3 
   
=–  =    … (ii)
 5 7 6  −3 2   3 −2
− + + 
 4 4 4 2 1 1 1 
 7 7 10     
= + − =  1  ∴ x = 2, y = 1, z = –1 To Find A–1, Since A = 2 4 1
4 4 4 
   −1 2 3 1 
 1–7+2     
 4 4 4 
 
∴ | A | = 1(4 – 3) – 1(2 – 2) + 1(6 – 8)
1 1 1 
  2 3  =1–0–2=–1≠0
Example 5: Given A = 2 4 1 , B =  
2 3 1  3 4  Let C be the matrix of co-factors of elements in | A |;
 
C11 C12 C13 
1 0 1   
Find P such that BPA =   C = C21 C22 C23 
0 1 0  C C32 C33 
 31
1 6 . 3 2 | Matrices

 4 1 2 1 2 4  cos x − sinx 0  cos y − sin y 0 


 −     
 3 1 2 1 2 3  F(x) F(y) =  sinx cos x 0   sin y cos y 0 
 1  0 0 1   0 0 1 
1 1 1 1 1 
= − − 
 3 1 2 1 2 3
  cos x cos y − sinx sin y − cos x sin y − sinx cos y 0 
 1 1

1 1 1 1  
= sinx cos y + cos x sin y − sinx sin y + cos x cos y 0 

 4 1 2 1 2 4 
  0 0 1 

 1 0 −2  1 2 −3 cos(x + y) − sin(x + y) 0 


     
=  2 −1 −1 ; ∴ C’ = =  sin(x + y) cos(x + y) 0  = F(x +y)
 0 −1 1 
 −3 1 2   −2 −1 2   0 0 1 
    
i.e. F(x).F(y) = F(x +y)  … (i)
 1 2 −3
  2nd part.
Adj A =  0 −1 1 
 −2 −1 2  As we know that F(x) [F(x)]–1= I  … (ii)
 
Adj A Replacing y by –x in (i),
A–1 = = – Adj, A
|A| we get F(x). F(–x) = F(x – x) = F(0)

 −1 −2 3  cos0 − sin0 0  1 0 0 
     
=  0 1 −1 … (iii) =  sin0 cos0 0  = 0 1 0 
 2 1 −2  0 0 1  0 0 1 
    
Substituting eq. (ii) and (iii) in eq. (i), we get i.e. F(x) F(–x) = I  … (iii)
therefore from (ii) and (iii)
 −1 −2 3 
 −4 3  1 0 1    ⇒ [F(x)]–1 = F(–x).
P=   ×   ×  0 1 −1
 3 −2   0 1 0   2 1 −2
 
Example 7: Show that every square matrix A can
 −1 −2 3  be uniquely expressed as P + iQ where P and Q are
 −4 3 −4   
P=   ×  0 1 −1 Hermitian matrices.
 3 −2 3   2 1 −2
  1
Sol: By considering P = (A + Aθ)
2
 4 + 0 − 8 8 + 3 − 4 −12 − 3 + 8  1
P=   And Q = (A –Aθ) we get A = P + iQ
 −3 − 0 + 6 −6 − 2 + 3 9 + 2 − 6  2i
Then, using the property of a Hermitian matrix we can
 −4 7 −7  prove the above problem.
P = 
 3 −5 5 
θ
1  1
Now P =  (A + A θ ) = (A + Aθ)q
θ

2  2
cos α − sin α 0 
  1 θ 1 1
Example 6: If F(α) =  sin α cos α 0  then = {A + (Aθ)q} = (Aθ + A) = (A + Aθ) = P
 0 0 1  2 2 2

show that F(x). F(y) = F( x+ y). ∴ P = Pθ , hence P is a Hermitian matrix.
Hence, prove that [F(x)]–1 = F(– x). Similarly
θ
1   1 
Sol: By substituting x and y in place of α in given Qθ =  (A − A θ ) =   (A – Aθ)q
matrices we will get F(x) and F(y) respectively and then  2i   2i 
by multiplying them we will get the required result. 1 θ 1 1
=– {A – (Aθ)q} = – (Aθ – A) = (A – Aθ) = Q
2i 2i 2i
∴ Q is also Hermitian matrix,
M a them a ti cs | 16.33

Therefore A can be expressed as P + iQ ,where P and Q Example 9: If the non-singular matrix A is symmetric,
are Hermitian matrices. then prove that A–1 is also symmetric.
Let A = R + iS where R and S are both Hermitian matrices
Sol: By using the conditions of non-singular and
We have Aθ = (R + iS)θ = Rθ + (iS)q symmetric matrix we can easily find the required result.
= Rθ + iSθ = Rθ – iSθ= R – iS As given matrix A is a non-singular symmetric matrix.
(since R and S are both Hermitian) ∴ | A | ≠ 0 and AT = A,

∴ A + Aθ = (R + iS) + (R – iS) = 2R So, A–1 exists

1 Now, AA–1 = I = A–1 A


⇒R= (A + Aθ) = P
2 ⇒ (AA–1)T = (I)T = (A–1A)T
Also A – Aθ = (R + iS) – (R – iS) = 2iS ⇒ (A–1)T AT= I = AT(A–1)T
1 ⇒ (A–1)T A = I = A(A–1)T [ AT = A]
⇒S= (A – Aθ) = Q
2i ⇒ A–1 = (A–1)T
Hence expression (1) for A is unique
⇒ A–1 is symmetric.

Example 8: If A is Hermitian such that A2 = 0, show that


Example 10: Find the rank of the matrix
A = 0,
2 3 −1 −1 
Sol: As A is a Hermitian matrix therefore Aθ = A. By  
1 −1 −2 −4 
considering A = [aij]n×n to be a Hermitian matrix of order A= 
3 1 3 −2 
n and as given A2 = 0, we can solve given problem as  
follows. 6 3 0 −7 
 a11 a12 ...... a1n 
  Sol: given
a a22 ...... a2n 
A =  21 and 1 −1 −2 −4 
 ...... ...... ...... ...... 
   
2 3 −1 −1 
 an1 an2 ...... ann  A= 
3 1 3 −2 
 a11 a21 ...... an1   
  6 3 0 −7 
a a22 ...... an2 
A =  12
θ
R 2 → R 2 – 2R1 ,R 3 → R 3 − 3R1 
...... ...... ...... ...... 
   
 and R 4 → R 4 – 6R1 
 a1n a2n ...... ann 

Since A2 = 0; 1 −1 −2 −4 
 
0 5 3 7
Let AAθ = [bij]n×n ⇒ AAθ = 0 A= 
0 4 9 10 
Then each element of AAθ is zero and so all the principal  
0 9 12 17 
diagonal elements of AAθ are zero
∴ bii = 0 for all i = 1, 2, ……, n [Applying R4 → R4 - R2 - R3]
Now, bii = ai1 a i1+ ai2 a i2 + … + ain a in 1 −1 −2 −4 
 
= |ai1| + |ai2| + …… |ain| ∴ bii = 0
2 2 2 0 5 3 7
= 
0 4 9 10 
⇒ |ai1|2 + |ai2|2 + …… + |ain|2 = 0  
⇒ |ai1| = |ai2| = …… = |ain| = 0
0 0 0 0 

⇒ ai1 = ai2 = …… = ain = 0 [Applying R2 → R2 – R3]

⇒ each element of the ith row of A is zero, but bii = 0 ∀ 1 −1 −2 −4 


i = 1, ……. n  
0 1 −6 −3 
= 
∴ Each element of each row of A is zero. Hence, A = O 0 4 9 10 
 
0 0 0 0 
1 6 . 3 4 | Matrices

[Applying R3 → R3 – 4R2] [Applying R3 → 1/11 R3]

1 −1 −2 −4  1 −1 −2 −4 
   
0 1 −6 −3  0 1 −6 −3 
=  ~ 
0 0 33 22  0 0 3 2 
   
0 0 0 0  0 0 0 0 
Since the equivalent matrix is in echelon form having
three non-zero rows. Hence, r(A) = 3

JEE Main/Boards

Exercise 1 Q.11 If f(x) = 3x2 – 9x + 7, then for a square matrix A,


write f(A).
 2x − 1   3 
Q.1 Find x and y, if   =  
 5  x + y Q.12 If A, B and AB are symmetric matrices, then what
is the relation between AB and BA?
Q.2 A matrix has 2 rows and 3 columns. How many 1 
elements a matrix has? Find the number of elements of  
Q.13 If A = 2  B, [2 –2 4], find AB.
a matrix if it has 3 rows and 2 columns.
3 
 
Q.3 Order of matrix A is 2 × 2 and order of matrix B is 2 1 2 0
1 2   
× 3. Find the order of AB and BA, if defined. Q.14 Are the matrix   and  3 4 0  equal?
Give reasons. 3 4  0 0 0
 
Q.4 Given a matrix A = [aij], 1 ≤ i ≤3 and 1 ≤ j ≤ 3, where
aij = i + 2j. Write the element  2 −1
Q.15 Given a matrix A =  .
(i) a11 (ii) a32 (iii) a23 (iv) a34 4 2 
1
Find matrix kA, where k = –
Q.5 A matrix has 18 elements. Write the possible orders 2
of matrix.
sec θ tan θ 
Q.16 Simplify: tan θ  
Q.6 Give an example of a diagonal matrix, which is not tan θ − sec θ 
a scalar matrix. Also give an example of a scalar matrix.
 − tan θ − sec θ 
+ sec θ  .
Q.7 For the matrix A, show that A + AT is a symmetric  − sec θ tan θ 
matrix.
Q.17 If Xm×3YP×4 = Z2×b, for three matrices X, Y, Z, find the
values of m, p and b.
Q.8 For the matrix A, Show that A – AT is a skew-
symmetric matrix.  0 −1 2 
 
Q.18 Is matrix A =  1 0 −3 symmetric or skew-
Q.9 The total number of elements in a matrix represents  −2 3 0 
 
a prime number. How many possible orders a matrix
can have ? symmetric? Give reasons.

 x   −1  0 cos θ sin θ 


Q.10 Find x and y, if   +   =   π
Q.19 If R (θ) =   , write (i) R   , (ii) R(x+y)
 2y   4  3  sin θ − cos θ  2
M a them a ti cs | 16.35

Q.20 For a skew-symmetric matrix A = [aij], what is the  cos α sin α 


nature of elements aij, if i = j. Q.34 If A =   , then show that
 − sin α cos α 
a 0  cos2α sin2α 
Q.21 If A =   , find A16. A2 =  
0 0  − sin2α cos2α 
 1 0  x 
Q.22 Find x, if [x 1]     =0 0 1  1 0 
 −2 −3 3 Q.35 If A =   and I =  ,
0 0  0 1 
 2 −1  prove that (aI + bA)3 = a3I + 3a2bA.
Q.23 Find the sum of matrix A =   and its
additive inverse. 4 6 
 0 1
Q.36 If A =   , find the values of
2 −1 2 4   −1 1
Q.24 Find X, if X +  =  
3 −1 5 0  p and q such that (pI + qA)2 = A.

 sin2 θ 1   cos2 θ  2 3 −4  5 1 2
0   0 −1    
Q.25 Evaluate,  +  +   Q.37 If A =  1 0 6  and B = 6 −1 4
cot2 θ 0   − cosec2 θ 1   −1 0  
 −2 1 5  5 3 −4 
   
Q.26 If A and B are symmetric matrices, show that AB find 2A – 3B.
is symmetric.
Q.38 Construct a 3 × 3 matrix [aij], whose elements are
Q.27 If a matrix has 8 elements, what the possible given by aij = 2i – 3j.
orders it can have ? What if it has 5 elements?
 x 3x − y  3 2
Q.39 If  =   , find x, y, z, w.
Q.28 Evaluate the following: 2x + z 3y − w  4 7
a
  Q.40 Find matrices X and Y, if
c  b
[a, b]   + [a b c d]   5 2  3 6 
d c  X+Y=   and X – Y =  
  0 9  0 −1
d

0 0 1   cos2 θ cos θ sin θ 


  Q.41 If A =  ;
Q.29 If A = 0 1 0  , find A2. Hence find A6  cos θ sin θ sin2 θ 

1 0 0 
 
 cos2 φ cos φ sin φ 
B =  , then show that AB is zero
 cos φ sin φ sin2
φ 
Q.30 Show that the element of the main diagonal of a  
skew-symmetric matrix are all zeros. matrix, provided (θ – φ) is an odd multiple of π/2.

0 −4  5 −7   −1 1 −1  0 4 3
Q.31 Find AB, if A =   and B =      
0 −3  0 0  Q.42 If A =  1 −3 3  and B =  1 −3 −3  ,
 5 −5 5   −1 4 4 
   
 4 3
Q.32 If A =   , find values of x and y such that compute A2B2.
 2 5
A2 – xA + yI = O where I is a 2×2 unit matrix and O is a
Q.43 Find the matrix X such that,
2 × 2 zero matrix.
 2 −1   −1 −8 −10 
 1 3 5  4 5 −9     
Q.33 If A =   and A – 3B =  , 0 1 X+ 3 4 0 
find B.  −2 5 7  1 2 3   −2 4  10 20 10 
   
1 6 . 3 6 | Matrices

Exercise 2 Q.9 If A is a skew symmetric matrix such that ATA = I,


then A4n–1(n∈N) is equal to
Single Correct Choice Type (A) –AT (B) I (C) – I (D) AT

Q.1 If number of elements in a matrix is 60 then how Q.10 A and B are 2 × 2 matrices satisfying det
many dimensions of matrix are possible A = det B and tr(A) = tr(B), further A2 – 3A + 14I = 0 and
(A) 12 (B) 6 (C) 24 (D) None of these B2 – λB + µI = 0, then µ is equal to
(A) 3 (B) 11 (C) –11 (D) 14
Q.2 Matrix A has x rows and x + 5 columns. Matrix B has
y rows and 11 – y columns. Both AB and BA exist, then Q.11 The false statement is -
(A) x = 3, y = 4 (B) x = 4, y = 3 (A) The adjoint of a scalar matrix is scalar matrix.
(C) x = 3, y = 8 (D) x = 8, y = 3 (B) The adjoint of upper triangular matrix is lower
triangular matrix.
Q.3 If A is square invertible matrix such that A2 = A, (C) The adjoint of upper triangular matrix is upper
then det.(A2 – I) is triangular matrix.
(A) 1 (B) 2 (C) 3 (D) None of these (D) adj(adj A) = A, A is a square matrix of order 2.

Q.4 Number of distinct matrices that can be formed Q.12 If the matrices A, B, (A + B) are non-singular, then
using all the 143 distinct elements is [A(A + B)–1B]–1 is equal to
(A) 4! (B) 4(143)! (C) 2(143)! (D) (143)! (A) A + B (B) A–1 + B–1
(C) (A + B)–1 (D) None of these
Q.5 If A = A, then (I + A) is equal to
2 4

(A) I + A (B) I + 4A Q.13 If A is an orthogonal matrix | A | = – 1, then AT is


equal to
(C) I + 15A (D) None of these
(A) – A (B) A
 α β (C) – (adj A) (D) (adj A)
Q.6 If A =   is an orthogonal matrix, where α,
 −β α 
β and the roots other than the common root of the Q.14 If A and B are square matrices of order 3, then
equations x2 – px + q = 0 & x2 + px – q = 0, then (A) adj(AB) = adj A+adj B
1 1 1 (B) (A + B)–1 = A–1 + B–1
(A) p=± , q=± (B) p = 0, q = ±
2 2 2
(C) AB = 0 ⇒ | A | = 0 or | B | = 0
1
(C) p = ± , q = 0 (D) None of these (D) AB = 0 ⇒ | A | = 0 and B = 0
2
a 0 0
Q.7 A is a square matrix of order n and (det A) = 3. If  
Q.15 If A = 0 a 0  , then | A | | adj A | is equal to
det (λA) = 81; where λ ∈ N, then possible value of n is
0 0 a 
 
(A) 3 (B) 5 (C) 2 (D) 7
(A) a25 (B) a27
(C) a81 (D) None of these
1 2 1+x
Q.8 If A =   and f(x) = , then f(A)is
2 1  1−x
 1 tanx 
Q.16 If A =   , then the value of | A A | is
T –1
1 1  −1 −1 − tanx 1
(A)   (B)    
1 1   −1 −1
(A) cos 4x (B) sec2x
2 2  (C) – cos 4x (D) 1
(C)   (D) None of these
2 2 
M a them a ti cs | 16.37

2 3   a2 ab ac 
Q.17 If A =   , then 19A–1 is equal to  
5 − 2  B = ab b2 bc  then AB is equal to
1  
(A) AT (B) 2A (C) A (D) A ac bc c2 
2  
(A) A3 (B) B2 (C) O (D) I
Q.18 If P is a two-rowed matrix satisfying PT= P–1, then
P is Q.25 If A, B, C are square matrices of same order & AB
 cos θ − sin θ   cos θ sin θ  = BA, C2 = B, then (A–1CA)2 is equal to
(A)   (B)  
 − sin θ cos θ   − sin θ cos θ  (A) B2 (B) A2 (C) C2 (D) C

 − cos θ sin θ  Q.26 A is a diagonal matrix of order 3, and tr(A) = 12.


(C)   (D) None of these
 sin θ − cos θ  If all diagonal entries are positive then maximum value
of det (A) is
Q.19 If A and B are two non-singular matrices of the (A) 8 (B) 16 (C) 32 (D) 64
same order such that Br = I, for some positive integer
r > 1, then A–1Brr–1 A A–1B–1 A is equal to. Q.27 If A and B are two matrix such that AB = B and
(A) 0 (B) I BA = A, then A2 + B2 is equal to

(C) A–1 (D) None of these (A) 2AB (B) 2BA (C) A + B (D) AB

Q.20 If A and B are orthogonal matrices of same order, 0 1 


Q.28 A =   and B is column matrix such
then: 3 0 
(A) A + B is also orthogonal. 0 
that (A8 + A6 + A4 + A2 + I), B =   where I
(B) A – B is also orthogonal. 1 
is a unit matrix of order 2 ×2, then B is equal to
(C) AB is also orthogonal.
    1
(D) AB + BA is also orthogonal. 0    
0    0  11 
(A)   (B)   (C)   (D)  
Q.21 If C is an orthogonal matrix and A is a square 1  2 1 1
   
matrix of same order then, trace of CTAC is equal to  11  2
 
 
   11 
(A) Trace of C (B) Trace of AC
(C) Trace of A (D) None of these matrix
Q.29 If A and B are square matrices of same order such
that AB = BA and A2 = I, then ABA is equal to
Q.22 Let A and B are idempotent matrices such that
A.B = BA and A – B is non singular then | A + B | is equal (A) (AB)2 (B) I (C) B (D) B2
to
(A) 0 (B) –1 (C) 1 (D) ±1
Previous Years’ Questions
Q.23 If A and B are square matrices of same order and
AAT = I, then (ATBA)10 is equal to Q.1 The parameter, on which the value of the

(A) AB10AT (B) ATB10A 1 a a2


(C) A10B10(AT)10 (D) 10ATBA determinant cos(p − d)x cospx cos(p + d)x
sin(p − d)x sinpx sin(p + d)x
 0 c −b 
  does not depend upon, is  (1997)
Q.24 If A =  −c 0 a  and
 b −a 0  (A) a (B) p (C) d (D) x
 
1 6 . 3 8 | Matrices

1 x x +1 bc ca ab
Q.2 If f(x) = 2x x(x − 1) (x + 1)x Q.9 Find the value of the determinant p q r
3x(x − 1) x(x − 1)(x − 2) (x + 1)x(x − 1) 1 1 1
then f(100) is equal to  (1999) where a, b, and c are respectively the pth, qth and rth
(A) 0 (B) 1 (C) 100 (D) – 100 terms of a harmonic progression (1987)

Q.10 Suppose, f(x) is a function satisfying the following


Q.3 The number of distinct real roots of
conditions:  (1998)
sinx cos x cos x (a) f(0) = 2, f(1) =1
cos x sinx cos x = 0 in the interval 5
(b) f has a minimum value at x = , and
cos x cos x sinx 2
π π (c) For all x,
– ≤x≤ is  (2001)
4 4 2ax 2ax − 1 2ax + b + 1
f’(x)= b b +1 −1
(A) 0 (B) 2 (C) 1 (D) 3
2(ax + b) 2ax + 2b + 1 2ax + b
Q.4 The number of values of k for which the system of where a, b are some constants. Determine the constants
equations  (2004) a, b and the function f(x)
(k + 1)x + 8y = 4k, kx + (k + 3)y = 3k – 1
(A) 0 (B) 1 (C) 2 (D) –1 Q.11 Prove that for all values of θ, (2000)
sin θ cos θ sin2θ
Q.5 Given, 2x – y + 2z = 2, x – 2y + z = –4, x +y + lz =
4, then the value of λ such that the given system of  2π   2π   4π 
sin  θ +  cos  θ +  sin  2θ + 
equations has no solution is  (2004)  3   3   3  =0

(A) 3 (B) 1 (C) 0 (D) –3


 2π   2π   4π 
sin  θ −  cos  θ −  sin  2θ − 
 3   3   3 
 3 /2 1/2  1 1
Q.6 If P =  ,A=   and
 −1 / 2 3 / 2  0 1  Q.12 If A is an 3 × 3 non – singular matrix such that
AA' = A' A and B = A −1 A then BB’ equals:  (2014)
Q= PAPT, then PTQ2005P is  (2005)
(A) I + B (B) I (C) B−1 (D) (B−1 )
1 2005  1 2005
(A)   (B)  
0 1  2005 1 
1 2 2 
 
 1 0 1 0  =
Q.13 If A 2 1 −2 is a matrix satisfying the
(C)   (D)   a 2 b 
2005 1  0 1   
equation AA T = I where I is 3 3 × 3 identity matrix, then
1 0 0  the ordered pair (a, b) is equal to:  (2015)
 
Q.7 If A = 0 1 1  , 6A–1 = A2 + cA + dI, then
(A) ( 2, −1 ) (B) ( −2,1 )
0 −2 4 
 
(C) ( 2,1 ) (D) ( −2, −1 )
(c, d) is (2005)

(A) (–6, 11) (B) (–11, 6) (C) (11, 6) (D) (6, 11)
5a −b  T
Q.14 If A =   and A adjA = AA , then 5a + b is
 3 2 
Q.8 Let α1, α2, β1, β2 be the roots of ax2 + bx + c = 0 and
px2 + qx + r = 0 respectively. If the system of equations equal to  (2016)
α1y + α2z = 0 and β1y + β2z = 0 has a non-trivial (A) -1 (B) 5 (C) 4 (D) 13
2
b ac
solution. Then prove that =  (1987)
2 pr
q
M a them a ti cs | 16.39

JEE Advanced/Boards

Exercise 1 idempotent matrix. Find the absolute value of sum of


the products of elements of the set S taken two at a
time.
Q.1 (a) A3×3 is a matrix such that | A | = a, B = (adj A)
such that | B | = b. Find the value of cos x − sinx 0 
 
1 a a2 a3 Q.6 If F(x) =  sinx cos x 0  then show that F(x).
(ab2 + a2b + I) S Where S= + + +  0 0 1 
2 b b3 b5 
F(y) = F(x + y).
…… up to ∞, and a = 3
Hence, Prove that [F(x)]–1 = F(–x).
(b) If A and B are square matrices of order 3, where | A |
= –2 and | B | = I, then find (A–1) adj (B–1) adj(2A–1)
Q.7 Let An and Bn be square matrices of order 3, which
are defind as An = [aij] and Bn = [bij]
Q.2 Let A be the 2 × 2 matrices given by A = [aij] where
aij ∈ {0, 1, 2, 3, 4} such that a11 + a12 + a21 + a22 = 4 2i + j 3i – j
where aij = and bij = for all i and j, 1 ≤ i.,j ≤ 3.
2n
(i) Find the number of matrices A Such that the trace of 3 22n
A is equal to 4. If l = lim Tr (3A1 + 32A2 + 33A3 + …. 3nAn) and
n→∞
(ii) Find the number of matrices A such that A is
invertible. m = lim Tr(2B1 + 22B2 + 23B3 + …. 2nBn)
n→∞
(iii) Find the absolute value of the difference between
maximum value and minimum value of det (A). Then find the value of ( l + m).

(iv) Find the number of matrices A such that A is either [Note: Tr (P) denotes the trace of matrix P]
symmetric or skew-symmetric or both and det (A) is
divisible by 2. Q.8 Let A be a 3 × 3 matrix such that a11 = a33 = 2 and
all the other aij = 1.
 4 −4 5 
  Let A–1 = xA2 + yA + zI then find the value of (x + y + z)
Q.3 For the matrix A =  −2 3 −3 find A2.
where I is a unit matrix of order 3.
 3 −3 4 
 
1 2 2
1 1 1   
2 3  Q.9 Given that A = 2 2 3 ,
 
Q.4 (a) Given A = 2 4 1 ,B =  , 1 −1 3 
2 3 1  3 4   
 
2 1 1  10 
1 0 1     
C = 2 2 1  , D = 13 and that Cb = D.
Find P such that BPA =  
0 1 0  1 1 1 
 
9
 
(b) Find the matrix A satisfying the matrix Solve the matrix equation Ax = b.
2 1  3 2  2 4 
equation   . A.   =   2 0 7   −x 14x 7x 
3 2  5 −3 3 −1    
Q.10 Let A = 0 1 0  and B = 0 1 0  are
1 −2 1   x −4x −2x 
Q.5 Let S be the set which contains all possible values    
of I, m, n, p, q, r for which two matrices such that AB = (AB)–1 and AB ≠ 1 (where I
is an identity matrix of order 3 × 3).
l2 − 3 p 0 
  Find the value of Tr (AB + (AB)2 + (AB)3 + …… + (AB)100)
A =  0 m2 − 8 q  Be a non singular
  where Tr.(A) denotes the trace of matrix A.
 r 0 n2 − 15 

1 6 . 4 0 | Matrices

Q.11 Let Mn = [mij] denotes a square matrix of order n Q.17 The set of natural numbers is divided into arrays
with entries as follows. of rows and columns in the form of matrices as A1 = (1),
For 1 ≤ i ≤ n , mii = 10; For 1 ≤ i ≤ n – 1, mi+1, i = mi, i + 1 = 3; 6 7 8
 2 3  
And all other entries in Mn are zero. Let Dn be the A2 =   , A3 =  9 10 11  …………… So on.
 4 5  12 13 14 
determinant of matrix Mn, then find the value of  
(D3 – 9D2).
Find the value of Tr(A10).
[Note: Tr(A) denotes trace of A]
Q.12 Find the product of two matrices A & B,

 −5 1 3  1 1 2  I
    xn
where A =  7 1 −5 & B= 3 2 1  and use it to Q.18 Consider In.m = ∫ xm − 1 dx and J n.m
0
 1 −1 1  2 1 3  I
    xn
solve the following system of linear equations ∫ xm + 1 dx∀ n > m and n, m ∈ N.
0

x + y + 2z = 1; 3x + 2y + z =7; 2x + y + 3z = 2 (a) Consider a matrix A = [aij]3×3,

I –I , i= j
Q.13 Determine the values of a and b for which the where aij =  6 +i,3 i+3,3 . Then find trace (A–1).
 0, i≠ j
3 −2 1   x   b 
      [Note: Trace of a square matrix is sum of the diagonal
system 5 −8 9   y  =  3  elements.]
2 1 a   z   −1
     
 J6,5 72 J11,5  I6,5 72 I11,5 
(i) Has a unique solution;    
(b) Let A =  J7,5 63 J12,5  and B = I7,5 63 I12,5 
(ii) Has no solution and    
 J8,5 56 J13,5  I8,5 56 I13,5 
(iii) Has infinitely many solutions.
then find the value of det (A) – det (B)

1 2  3 1  1 2  3 −4 
Q.14 If A =  ;B=  ;C=   Q.19 Consider the matrices A = 
3 4  1 0  2 4   and
1 −1 
x x2  a b
and X =  1 B=   and let P be any orthogonal matrix and
 then solve the following
 x3 x4  0 1 
Q = PAPT and R = PTQKP also S = PBPT and T = PTSKP
matrix equations.
(a) AX = B – 1 (b) (B – 1) X = IC Column I Column II
(c) CX = A (A) If we vary K from 1 to n (p) G.P. with common
then the first row first column ratio a
Q.15 If A is an orthogonal matrix and B = AP where P elements of R will form
is a non singular matrix, then show that the matrix PB–1 (B) If we vary K from 1 to n then (q) A.P. with volume
is also orthogonal. the 2nd row 2nd column elements difference 2
of R will form
Q.16 Let M be a 2 × 2 matrix such that M (C) If we vary K from 1 to n (r) G.P. with common
1 then the first row first column ratio b
1  −1 1  elements of T will form
  =   and M  −1 =   . If x1 and
2
−1
  2
    0  (D) If we vary K from 3 to n (s) A.P. with volume
x2 (x1 > x2) are the two values x for which det (M – xI) = 0, then the first row 2nd column difference –2
where I is an identity matrix of order 2, then find the value elements of T will represent the
of (5x1 + 2x2). sum of
M a them a ti cs | 16.41

Q.20 Consider a square matrix A of order 2 which has Q.5 If α, β, γ are the real numbers and
its elements as 0, 1, 2 and 4. Let N denote the number
of such matrices, all elements of which are distinct.  1 cos(α − β) cos(α − γ ) 
 
A =  cos(β − α ) 1 cos(β − γ )  then
Column I Column II  cos( γ − α ) cos( γ − β) 1 
 
(A) Possible non-negative value of det (A) is (p) 2
(A) A is skew symmetric
(B) Sum of values of determinants (q) 4
corresponding to N matrices is (B) A is invertible

(C) If absolute value of (det(A)) is least, then (r) – 2 (C) A is non singular
possible value of |adj(adj(adj A))| (D) | A | = 0
(D) If det (A) is algebraically least, then (s) –2
possible value of det (4A–1) is Q.6 The values of x for which the matrix
(t) 8
x + a b c 
 
 a x +b c  is non-singular are
 a b x + c 

Exercise 2
(A) R – {0}
Single Correct Choice Type (B) R – { – (a + b + c)}

Q.1 Let A, B be two square matrices of the same (C) R – { 0, – (a + b + c)}


dimension and let [A, B] = AB – BA, then for three 2 × (D) None of these
2 matrices
A, B, C, [[A, B],C] + [[B, C],A] + [[C, A],B] = Q.7 Let A is a skew symmetric matrix such A2 = A, and B
is a square matrix such that BTB = B; | B | ≠0. If X = (A +
(A) 1 (B) 0
B) (A – B), then XTX is
(C) ABC – CBA (D) None of these
(A) A – I (B) I – A
1 2 3 (C) A (D) None of these
Q.2 A = 4 5 7 & f(x) = x3 – 8x2 + bx + γ. If A
2 3 α Q.8 For two uni-modular complex numbers z1 and z2,
satisfies f(x) = 0, then ordered pair (α, γ) is  z1 −z2 
−1
 z1 z2 
−1

    equal to
(A) (2, –7) (B) (–2, 7)  z2 z1   − z2 z1 
(C) (2, 7) (D) (–2, –7)
z z2  1 0 
(A)  1  (B)  
α β  z1 z2  0 1 
Q.3 If   is a square root of the two rowed unit
 γ δ 1 / 2 0 
(C)   (D) None of these
matrix, then δ is equal to  0 1 / 2
(A) a (B) β
−2
(C) g (D) None of these 1 / 25 0   5 0
Q.9 If   =  ,
 x 1 / 25  −a 5 
4 2i then the value of x is
Q.4 For A = , (A – 2I) (A – 3I) is a
i 1 a 2a
(A) (B)
125 25
(A) Null-matrix (B) Hermitian matrix
2a
(C) Unit matrix (D) None of these (C) (D) None of these
125
1 6 . 4 2 | Matrices

Q.10 If A is square matrix such that A2 = I, |A| = 1 and = Ax + B Where A and B are determinants of order 3
B = (adj A)–1 then incorrect statement is not involving x.  (1982)
(A) AB = BA (B) AB = I
Q.4 Show that the system of equations 3x – y + 4z = 3, x +
(C) A = B (D) B = I
2y – 3z = – 2, 6x + 5y + lz = – 3 has at least one solution
for any real number λ ≠ –5. Find the set of solutions, if
Q.11 If A and B are square matrices of order 3 and adj λ = – 5.  (1983)
A = B, then adj (3AB) is equal to
(A) 3 | B |2 I3 (B) 9 | B | I3 Q.5 Consider the system of linear equations in x, y, z
(sin 3θ) x – y + z = 0, (cos 2θ) x + 4y + 3z = 0 and 2x +
(C) 3 | A |2 I3 (D) 9 | A | I3
7y + 7z = 0. Find the values of θ for which this system
has non-trivial solution.  (1986)
Q.12 Let A and B are square matrices of order n such
that AT + B = O, O is a null matrix, A = adj B, tr (A) = –1 a−1 n 6
and A2 = A then tr {adj(ATB)} is equal to
Q.6 Let Da = (a − 1)2 2n2 4n − 2
(A) (– 1)n–1 (B) 1 3 8
(a − 1) 3n 3n2 − 3n
(C) (– 1) n
(D) None of these n
Show that ∑ ∆a = c ∈ constant.  (1989)
a=1
Q.13 If A is a non-singular matrix such that C = A + B,
| C |2 = | A |2 | I – (A–1 B)2| and AB =BA, then
p b c
(A) B is null matrix (B) A is null matrix Q.7 If a ≠ p, b ≠ q, c ≠ r and a q c = 0
(C) | C | = | A – B | (D) | A | = | B | a b r
p q r
Then, find the value of + +  (1991)
p−a q−b r −c
Previous Years’ Questions
Q.8 For a fixed positive integer n, if
Q.1 Let ω ≠ 1 be a cube root of unity and S be the set of
n! (n + 1)! (n + 2)!
all non-singular matrices of the
D = (n + 1)! (n + 2)! (n + 3)! , then show that
1 a b (n + 2)! (n + 3)! (n + 4)!
 
form  ω 1 c  , where each of a, b or c is
ω2  D 
 ω 1   3 − 4  is divisible by n.  (1992)
 (n!) 
either ω and w . Then, the number of distinct matrices
2

in the set S is  (2011)


Q.9 Let λ and α be real. Find the set of all values of λ
(A) 2 (B) 6 (C) 4 (D) 8
for which the system of linear equations lx + (sin α)y +
(cos α)z = 0 x + (cos α)y + (sin α)z = 0 and –x + (sin α)
Q.2 Let M and N be two 3 × 3 non-singular skew- y – (cos α)z = 0 has a non-trivial solution for λ = 1, find
symmetric matrices such that MN = NM. If PT denotes all values of α.  (1993)
the transpose of P, then M2N2(MTN)–1(MN–1)T is equal to
 (2011)
Q.10 Let a, b, c be real numbers with a2 + b2 + c2 = 1.
(A) M 2
(B) –N 2
(C) – M 2
(D) MN Show that the equation

ax − by − c bx + ay cx + a
Q.3 Without expanding a determinant at any
bx + ay −ax + by − c cy + b =0
x2 + x x +1 x−2 cx + a cy + b −ax – by + c
2
stage, show that 2x + 3x − 1 3x 3x − 3 represents a straight line.  (2001)
2
x + 2x + 3 2x − 1 2x − 1
M a them a ti cs | 16.43

U is the zero matrix.


Q.11 Let ω ≠ 1 be a cube root of unity and S be the set
of all non−singular matrices of the form
(C) determinant of (M2+MN2) ≥ 1 .
1 a b
  (D) for a 3 × 3 matrix U, if (M2+MN2) U equals the zero
ω 1 c  where each of a, b and c is either ω or ω2 .
ω2 ω 1  matrix the U is the zero matrix.

Then the number of distinct matrices in the set S is
Q.17 The quadratic equation p(x)=0 with real coefficients
 (2011)
(A) 2 (B) 6 (C) 8 (D) 4 has purely imaginary roots. Then the equation p(p(x))=0
has
Q.12 Let M be a 3 × 3 matrix satisfying
(A) Only purely imaginary roots.
0   −1   1  1 1   0  (B) All real roots.
           
M=
1   2  ,M =
 −1   1  and M 1  =  0  (C) Two real and two purely imaginary roots.
0  3 0  −1  1  12 
            (D) Neither real nor purely imaginary roots.
Then the sum of the diagonal entries of M is  (2011)
−1 + 3i
(A) 5 (B) 6 (C) 9 (D) 8 Q.18 Let z = , where −1 ,and r,s ∈ {1,2,3} .
2
 −z r z 2s 
Q.13 If P is 3 × 3 matrix such that PT = 2P + I where PT is Let P = ( ) and I be the identity matrix of
he transpose of P and I is the 3 × 3 identity matrix, then  z 2s zr 

there exists a column matrix
order 2. Then the total number of or dered pairs (r, s)
 x  0 
   
X  y  ≠ 0  such that 
= (2012)
 z  0  for which P2 = − I is
   
3 −1 −2
0   
  Q.19
= Let P 2 0 α  where α ∈ R .
(A) PX 0  (B) PX = X (C) PX = 2X (D) PX = − X
3 5 0 
0   
 
Suppose Q = qij  is a matrix such that PQ = kI
 
Q.14 Let P = aij  is 3 × 3 matrix and let Q = bij  , where R,k ≠ 0 and I is the identity matrix of order 3. If
   
k k2
where bij = 2i+ j aij 1 ≤ i, j ≤ 3 .If the determinant of P is 2, q23 = − and det ( Q ) = , then
8 2
then the determinant of the matrix Q is  (2012)
α 0,k
(A) = = 8 (B) 4α − k + 8 = 0
(A) 210 (B) 211 (C) 212 (D) 213
( )
(C) det Padj ( Q ) = 29

( )
(D) det Padj (P ) = 213
Q.15 Let ω be a complex cube root of unity with ω ≠ 1
 1 0 0
and P = pij  be a n × n matrix with Pij = ωi+ j Then  
  Q.20 Let P =  4 1 0  and I be the identity matrix
P2 ≠ 0 , when n =  (2013) 16 4 1 
 
(A) 57 (B) 55 (C) 58 (D) 56
of order 3. If Q = qij  is a matrix such that p50 − Q =I
 
Q.16 Let M and N be two 3 × 3 matrices such that MN = q + q32
then 31 equals  (2016)
NM. Further, if M ≠ N2 and M2 = N4 then  (2014) q21

(A) 52 (B) 103 (C) 201 (D) 205


(A) determinant of (M2+MN2) is 0.
(B) there is a 3×3 non−zero matrix U such that (M2+MN2)
1 6 . 4 4 | Matrices

PlancEssential Questions
JEE Main/Boards JEE Advanced/Boards
Exercise 1 Exercise 1
Q.7 Q.8 Q.17 Q.7 Q.10 Q.13
Q.23 Q.32 Q.35 Q.18 Q.19 Q.20
Q.38 Q.41 Q.44 Q.17

Exercise 2 Exercise 2
Q.4 Q.11 Q.14 Q.2 Q.5 Q.8
Q.19 Q.22 Q.26 Q.12

Previous Years’ Questions Previous Years’ Questions


Q.1 Q.2 Q.6 Q.2 Q.4 Q.11
Q.10 Q.13 Q.12

Answer Key

JEE Main/Boards
Exercise 1

Q.1 x = 2, y = 3 Q.2 6; 6
Q.3 Order of AB is 2 × 3; order of BA is not defined Q.4 (i) 3 (ii) 7 (iii) 8 (iv) 11

2 0   −1 0 
Q.5 1 × 18, 2 × 9, 3 × 6, 6 × 3, 9 × 2, 18× 1 Q.6    
0 3   0 −1 
1
Q.9 Two Q.10 x = 1, y = –
2
Q.11 f(A) = 3A2 – 9S + 7I Q.12 AB = BA

 2 −3 4 
 
Q.13  4 −6 8  Q.14 No
 6 −9 12 
 

 −1 1 / 2  0 −1 
Q.15   Q.16  
 −2 −1   −1 0 
M a them a ti cs | 16.45

Q.17 m = 2, p = 3, b = 4 Q.18 Skew-symmetric

0 1  cos(x + y) sin(x + y) 
Q.19 (i)   (ii)   Q.20 Each element is zero
1 0   sin(x + y) − cos(x + y)

0 0
Q.21   Q.22 1 ± 10
0 0

0 0 0 5 
Q.23   Q.24  
0 0 2 1 

 1 0
Q.25   Q.27 1 × 8, 2 × 4, 4 × 2, 8 × 1; 1 × 1, 5 × 1
 −2 1 
1 0 0 
  6
Q.28 [ac + bd + a + b + c + d ]
2 2 2 2
Q.29 0 1 0  ; A = A
2

0 0 1 
 
0 0 
Q.31   Q.32 x = 9m, y = 14
0 0 

1  −2 1 19  1 1
Q.33 B =   Q.36 P ± ,q±
3  −5 8 11  3 3

 −11 3 −14   −1 −4 −7 
   
Q.37  −16 3 0  Q.38  1 −2 −5 
 −19 −7 22   3 0 −3
   
4 4 1 −2
Q.39 x = 3, y = 7, z = –2, w = 14 Q.40 X =  ,Y=  
0 4  0 5 

1 −1 −5
Q.42A2 = A, B2 = I; A2B2 = AI =A Q.43 X =  
3 4 0 

Exercise 2
Single Correct Choice Type

Q.1 A Q.2 C Q.3 D Q.4 B Q.5 C Q.6 C


Q.7 A Q.8 B Q.9 D Q.10 D Q.11 B Q.12 B
Q.13 C Q.14 C Q.15 D Q.16 D Q.17 D Q.18 B
Q.19 A Q.20 C Q.21 C Q.22 C Q.23 B Q.24 C
Q.25 C Q.26 D Q.27 C Q.28 C Q.29 C

Previous Years’ Questions


Q.1 B Q.2 A Q.3 C Q.4 B Q.5 B

1 5 1 2 5
Q.6 A Q.7 A Q.9 0 Q.10 a = ,b= f (x) = x − x+2
4 4 4 4
Q.12 B Q.13 D Q.14 B
1 6 . 4 6 | Matrices

JEE Advanced/Boards
Exercise 1
Q.1 (a) 225(b) –8 Q.2 (i) 5 (ii) 18 (iii) 8 (iv) 5

 17 4 −19 
   −4 7 −7  1  48 −25
Q.3  −10 0 13  Q.4 (a)   (b)  
 3 −5 5  19  −70 42 
 −21 −3 25 
 
Q.5 29 Q.7 21
Q.8 1 Q.9 x1 = 1, x2 = –1, x3 = 1
Q.10 100 Q.11 1
Q.12 x = 2, y = 1, z = – 1
Q.13 (i) a ≠ –3, b ∈ R    (ii) a = –3 and b ≠ 1/3   (iii) a = –3, b = 1/3
 −3 −3
1 2
Q.14 (a) X =  5     (b) X =
     (c) No solution
 2  −1 −2 
 2 
Q.16 8` Q.17 3355
Q.18 (a) 18   (b) 0 Q.19 A→q; B → s; C→ p; D→ p
Q.20 A → p, q, t; B → s; C→ p, r; D → r

Exercise 2
Single Correct Choice Type

Q.1 B Q.2 A Q.3 A Q.4 A Q.5 D Q.6 C


Q.7 B Q.8 C Q.9 C Q.10 D Q.11 B Q.12 C
Q.13 C

Previous Years' Questions


π
Q.1 A Q.2 C Q.4 ∆ = ∆1 = ∆2 = ∆3 = 0 Q.5 θ = nπ, nπ + (–1)n ,n∈Z
6
n
Q6. ∑ ∆a = c Q.7 2 Q.8 2n(n2 + 4n + 5) Q.9 α = nπ or nπ + π/4
a=1
Q.10 0 Q.11 A Q.12 9 Q.13 D Q.14 D Q.15 B, C, D

Q.16 A, B Q.17 D Q.18 A Q.19 B, C Q.20 B


M a them a ti cs | 16.47

Solutions

JEE Main/Boards Sol 7: Matrix A = [aij] assume


AT = [aji]
Exercise 1 So A + AT = [aij + aji] = [bij] assume
2x − 1  3  Here bij = aij = aji
Sol 1:   =  
 5  x + y  bji = aij + aji
2x – 1 = 3 Here bij = bji
2x = 4 ⇒ x = 4/2 = 2 Matrix is symmetric.
5=x+y=2+y
Sol 8: A – AT
y=5–2=3
⇒ [aij] – [aji] = [bij] assume
(x, y) = (2, 3)
bij = aij – aji
Sol.2 row = n bji = aji – aij
Column = m ⇒ bij = – bij
Then total elements = mn This matrix is known as symmetric matrix.
if (n, m) = (2, 3) ⇒ nm = 2.3 = 6
Sol 9: A matrix → row n, column = m
if (n, m) = (3, 2) ⇒ mn = 3.2 = 6
Total element = mn
Sol 3: A2×2 and B2×3 mn is prime no.
so mn could be → 2, 5, 7, 11
For AB ⇒ order will be ⇒ 2 × 3 factor of 2 = 1 × 2 or 2 × 1
For BA ⇒ row of B ≠ column of A
So for any prime no. of only 2 order
So, BA does not exist
⇒ 1 × n and n × 1(n ∈ prime no. )

Sol 4: A = [aij], 1 ≤ i ≤ 3, i ≤ j ≤ 3
 x   −1 0 
Sol 10:   +   =  
aij = i + 2j 2y   4  3 
(i) a11 = 1 + 2 = 3 (ii) a32 = 3 + 2(2) = 3 + 4 = 7
 x − 1  0 
(iii) a23 = 2 + 3(2) = 8 (iv) a34 ⇒ not a element = i ≤ j ≤ 3 ⇒  = 
2y + 4  3 
but here 4 > 3
x–1=0⇒x=1
Sol 5: Total element = 18 2y + 4 = 3 ⇒ 2y = 3 – 4 = – 1
Assume no of row = n 1
Y=–
And no. of column = m 2
so n × m = 18 = 1 × 18 = 2 × 9 = 6 × 3 = 3 × 6 = 9 ×
2 × 18 × 1 Sol 11: f(x) = 3x2 – 9x + 7
f(A) ⇒ if A is a matrix
2 0 
Sol 6: Diagonal matrix =  
0 3  f(A) = 3A2 – 9A + 7I
3 0  A is a square matrix so A2 is possible.
scalar matrix =  
0 3 
1 6 . 4 8 | Matrices

Sol 12: A, B and AB are symmetric matrices Sol 17: Xm×3 Yp×4 = Z2×b
A = aij
Column of x = row of y ⇒ 3 = p and 2 × b = (m × 4)
B = bij
So m = 2; b = 4
AB = Aij Bji= Cij
BA = Bij . Aji = dij  0 −1 2 
 
Sol18: A =  1 0 −3
but Bij = B ji 
 −2 3 0 
 symmetric matrix’s property  
and Aij = A ji 

a12 = – a21, a13 = – a31
∴ AB = Aij Bij = Aij . Bij = BA
a23 = – a32,
AB = BA
so A is skew symmetric.

1 
 
Sol 13: A = 2  = B 2 −2 4  cos θ sin θ 
1×3 Sol 19: R(θ) =  
3   sin θ − cos θ 
 3×1
1   2 −2 4   π π 
     cos sin 
AB = 2  2 −2 4 1×3 =  4 −4 8 
  π
R  =  2 2  = 0 1 
3   6 −6 12  
 3×1   2 sin π π
− cos  
1 0
 12 2

1 2 0 
cos(x + y) sin(x + y) 
R(x + y)=  
1 2   
Sol 14:   and 3 4 0   sin(x + y) − cos(x + y)
3 4  2×2 0 0 0 
 3×3
Sol 20: Skew symmetric A = [aij]
Both have different orders. So they are not same.
For all skew symmetric Matrix dia. l element (aij) are
zero so aij = 0 & when i = j
 2 −1  1
Sol 15: A =  ,K=–
4 2  2
a 0
Sol 21: A =  
 2 −1  1  2 −1 0 0 
KA = K   =–  
 4 2  2 4 2 
a 0 a 0 a2 0
A2 =     ⇒A =
2
 
 2 1 
 1 0 0  0 0   0 0 
 − − ( −1)
2 2  −1 
=  = 2 a2
 − 1 (4) − 1 (2)   0 a2 0  a4 0
 −2 −1 A2 × A2 =     ⇒ A4 =  
 2 2   0 0   0 0   0 0 

a16 0
sec θ tan θ   − tan θ − sec θ  ⇒ A16=  
Sol 16: tanθ   + secθ    0 0 
tan θ − sec θ   − sec θ tamθ 

tan θ sec θ − tan θ sec θ tan2 θ − sec2 θ  1 0 X 


=  Sol 22: [X 1]1×2     =0
2 2
 tan θ − sec θ − tan θ sec θ + cos θ sec θ   −2 −3 2×2  3  2×1

 sin2 θ − 1  X  X 
 0  ⇒ [ x − 2 0 − 3]1×2   [x – 2–3]   = 0
=  cos2 θ  =  0 −1   3  2×1 3
 sin2 θ − 1   
 −1 0 
 0 
 cos2 θ  [(x – 2)x –3(3)] = 0 ⇒ x2 – 2x – 9 = 0
M a them a ti cs | 16.49

2 ± 22 − 4( −9) 0 0 1 
x= =1± 10  
2 Sol 29: A = 0 1 0
1 0 0
 2 −1
Sol 23: A =   0 0 1  0 0 1 
 4 6  2×2    
A2 =  0 1 0   0 1 0 
Additive inverse B which is – A 1 0 0  1 0 0 
   
So, A + B =A – A = 0
1 0 0 
 
A2 =  0 1 0  = I
2 −1 2 4 
Sol 24: x +  =  0 0 1 
 
3 −1 5 0 
A6 = [A2]3 = [I]3 = I
x x2 
Assume x =  1  A6 = I = A2
 x3 x4 

 x + 2 x2 − 1  2 4  Sol 30: Properties of skew – symmetric matrix [a, j]


⇒  1  =  
 x3 + 3 x 4 − 1  5 0  ⇒ All diagonal element are zero

⇒ x1 + 2 = 2 ⇒ x1 = 0 ⇒ aij = – aji

⇒ x2 – 1 = 4 ⇒ x2 = 1 + 4 = 5
0 −4  5 −7 
⇒ x3 + 3 = 5 ⇒ x3 = 5 – 3 = 2 Sol 31: A =  . B= 
0 −3  0 0 
⇒ x4 – 1 = 0 ⇒ x4 = 1
0 −4  5 −7 
x x2  0 5  AB =    
x=  1  =   0 −3  0 0 
 x3 x4  2 1 
0.5 − 4.0 0( −7) 0 0 
=   =  
Sol 25:  0(5) 0( −3)  0 0 
 sin2 θ 1   cos2 θ 0  0 −1 
  +   +    4 3
 −1 0 
2 2
cot θ 0   − cosec θ 1  Sol 32: A =  
 2 5
 sin2 θ + cos2 θ + 0 1 + 0 − 1 
=   A2 – XA +YI = 0
cot2 θ − cosec2 θ − 1 0 + 1 + 0 
 4 3  4 3  42 + 3.2 4.3 + 3.5
 1 0  1 0 A2 =     =  
=  =    2 5  2 5 2.4 + 5.2 2.3 + 5.2 
 −1 − 1 1   −2 1 
22 27 
A2 =  
Sol 27: Matrix has 8 element 18 31 
m×n=8=1×8=8×1=2×4=4×2 A2 – XA + YI = 0
if m × n = 5 = 1 × 5 = 5 × 1 (only 2 possible order) 22 27   4 3 0 0 
1
⇒   ×   + =  
a 18 31   2 5 1 0  0 0 
 
c 
 
b 0 1 
Sol 28: a b    × a b c   
   
22 − 4x + y
 c 27 − 3x  0 0 
d ⇒   =  
 
d  18 − 2x + x 31 − 5x + y  0 0 
[ac + bd] × [a2 + b2 + c2 + d2] ⇒ Compare elements
⇒ [a2 + b2 + c2 + d2 + ac + bd] 27 – 3x = 0
1 6 . 5 0 | Matrices

27 0 1  1 0 
3x = 27 ⇒ x = = 9 ⇒ y = 45 – 31 = 14 Sol 35: A =  , F = 
3 0 0 
  0 1 
(x, y) = (9, 14)
1 0  a 0
For (aI + bA)3; aI = a   =  
 1 3 5  0 1  0 a 
Sol 33: A =  
 −2 5 7  0 1  0 b 
bA = b   =  
 4 5 −9  0 0  0 0 
2A – 3B =  
1 2 3  a 0 0 b  a b
aI + bA =   +   =  
b b2 b3  0 a  0 0  0 a 
Assume B =  1 
b 4 b5 b6 
a b a b a b
(aI + bA)3 =      
2.1 − 3b1 2.3 − 3b2 2 × 5.3b3  0 a  0 a  0 a 
⇒ 2A – 3B =  
 −4 − 3b 4 2.5 − 3b5 2.7 − 3b6 
a2 ab + ba a b a3 a2b + 2a2b 
=     =  
 4 5 −9   0 a2  0 a   0 a3 
=  
1 2 3  a3 3a2b 
=  
4−2 2  0 a3 
2 – 3b1 = 4 → b1 = =–
−3 3
and R. H. S. = a3I + 3a2bA
⇒ 6 – 3b2 = 1
1 0  0 1 
⇒ 3b2 = 6 – 5 = 1 = a3   + 3a b 0 0 
2

0 1   
1
⇒ b2 =
3 a3 0 0 3a2b  a3 3a2b 
19 =  +   =  
Same as b3 =  0 a3  0 0   0 a3 
3
5 8 11 L. H. S. = R. H. S
b4 = – , b5 = , b6 =
3 3 3
 0 1
1  −2 1 19  Sol 36: A =  
So B =    −1 1
3  −5 8 11 
(pI + qA)2 = A
 cos α sin α  p 0   0 1  0 q
Sol 34: A =   pI = 
 − sin α cos α   , qA = q   =  
 0 p   −1 1   −q q
 cos α sin α   cos α sin α  p 0   0 q p q 
A2 =     pI + qA = 
 − sin α cos α   − sin α cos α  +  =  
0 p   −q q  −q p + q

 cos2 α − sin2 α cos α sin α + sin α cos α  p q  p q 


A2 =   (pI + qA)2 =    
 − sin α cos α − sin α cos α − sin2 α + cos2 α   −q p + q  −q p + q

we know → cos2α – sin2α = cos2α  p2 − q2 pq + q(p + q) 


=   = A (given)
2 2
and 2cosα sinα = sin2α  −pq − q(p + q) −q + (q + q) 

 0 1
2  cos2α sin2α  A=  
so A =    −1 1
 − sin2α cos2a
So p2 – q2 = 0 ⇒ p2 = q2 ⇒ p = ± q
M a them a ti cs | 16.51

pq + qp + q2 = 1 5 2  3 6 
Sol 40: X + Y =  ,X–Y=  
q2 + 2qp = q2 ± 2q2 = 1 0 9  0 −1
–ve → q2 – 2q2 = 1 ⇒ q2 = 1 not possible sum of X + Y, X – Y
+ve → q + 2q = 3q = 1 ⇒ q = 1/3
2 2 2 2
⇒ X + Y + X – Y = 2X
1
So p = q = ± 5 2  3 6  5 + 3 2 + 6 
3 =   +   =  
0 9  0 −1  0 9 − 1
 2 3 −4 
  8 8  4 4
Sol 37: A =  1 0 6  2X =   = 2 
 −2 1 5  0 8  0 4 
 
4 4
5 1 2 X=  
  0 4 
B = 6 −1 4 
5 3 −4 
  5 2  5 2  4 4
Y=   –X=   –  
 2 3 −4  5 1 2 0 9  0 9  0 4 
   
2A – 3B = 2  1 0 6  – 3 6 −1 4 
 −2 1 5  5 3 −4  5 − 4 2 − 4  1 −2
    Y=   = 
 0 9 − 4  0 5 
 −11 3 −14 
 
=  −16 3 0   cos2 θ cos θ sin θ 
 −19 −7 22 
  Sol 41: A =  
cos θ sin θ sin2 θ 

Sol 38: A3×3 = [aij]  cos2 φ cos φ sin φ 


B=  
aij = 2i – 3j cos φ sin φ sin2 θ 
\a11 = 2(1) – 3(1) = – 1, a12 = 2(1) – 3(2) = – 4
AB = (Cij)
a13 = 2(1) – 3(3) = – 7, a21 = 2(2) – 3(1) = 1
C11 = cos2θ cos2φ + cosθ sinθ cosφ sinφ
a22 = 2(3) – 3(2) = – 2, a23 = 2(2) – 3(3) = – 5
C11 = cosθ cosφ (cosθ cosφ + sinθ sinφ)
a31 = 2(3) – 3(1) = 3, a32 = 2(3) – 3(2) = 0
C11 = cosθ cosφ cos(θ – φ) = 0
a33 = 2(3) – 3(3) = – 3
Similarly C12, C21 and C22 will also be zero
 a11 a12 a13   −1 −4 −7 
    0 0 
So A = a21 a22 a23  =  1 −2 −5  So AB =  
a
 31 a32 a33   3 0 −3
  0 0 

 x 3x − y  3 2   −1 1 −1 0 4 3
Sol 39:  =      
Sol 42: A =  3 −3 3  , B=  1 −3 −3
2x + z 3y − w  1 7 
 5 −5 3   −1 4 4 
   
Compare elements
 −1 1 −1  −1 1 −1
x=3    
A =  3 −3 3   3 −3 3 
2

3x – y = 3(3) – y = 9 – y = 2  5 −5 3   5 −5 3 
   
y=9–2=7
 1 + 3 −5 −1 − 3 + 5 1 + 3 −5 
2x + z = 2(3) + 7 = 6 + 7 = 4 ⇒ 7 = 4 – 6 = –2  
=  −3 − 9 + 15 3 + 9 + 15 −3 − 9 + 15 
3y – w = 3(7) – w = 7 ⇒ w = 21 – 7 = 14  −5 − 15 + 25 5 + 15 − 25 −5 − 15 + 25
 
(x, y, z, w) = (3, 7, –2, 14)
1 6 . 5 2 | Matrices

 −1 1 −1 X X2 X3  1 −2 −5
  X=  1 =  
=  3 −3 3  =
A X 4 X5 X6  3 4 0 
 5 −5 5 
 

0 4 3 0 4 3 Exercise 2
   
B =  1 −3 −3  1 −3 −3
2

 −1 4 4   −1 4 4  Single Correct Choice Type


   

 4 −3 −12 + 12 −12 + 12  1 0 0  Sol 1: (A) Number of elements in a matrix = 60


   
B =  −3 + 3 4 + 9 − 12
2
3 + 9 − 12  = 0 1 0  60 = 22 51 31
 4 − 4 −4 − 12 + 16 −3 − 12 + 16  0 0 1 
    Number of order matrix can have = (2 + 1) (1 + 1) (1
+ 1)
A2B2 = A2,I = A. I = A
= 3 × 2 × 2 = 12
 2 −1  −1 8 −10 
    Sol 2: (C) A(x)×(x + 5) By×(11 – y)
Sol 43:  0 1  X = 3 4 0 
 −2 4  10 20 10  AB and BA both exist
   3×3
⇒ for AB x + 5 = y  …(i)
 2 −1
  ⇒ for BA 11 – y = x  …(ii)
X ‘s number of row = column of  0 1 
 −2 4  ⇒ y = 8; x = 3
 
order of X = 2 × 3
 X X 2 X3  Sol 3: (D) A is a square invertible matrix
assume X =  1 
 X 4 X5 X 6  A2 = A
 2 −1 Multiply A–1 both sides
   X1 X 2 X 3 
 0 1  X 
X5 X 6  A–1 A2 = A–1A = I
 −2 4   4
  A=I
 2x1 − x 4 2x2 − x5 2x3 − x6  So A2 = I
 
= x4 x5 x6  A2 – I = 0 (zero matrix)
 −2x + 4x −2x2 + 4x5 −2x3 + 4x6 
 1 4

Sol 4: (B) Total 143 elements all are different.


 −1 −8 −10 
  143 = 1 × 143 = 143 × 1 = 11 × 13 = 13 × 11
= 3 4 0 
10 20 10  Total Number of order that exist = 4
 
x4 = 3, x5 = 4, x6 = 0 Number of way to arrange 143 elements = 143!

–2x1 + 4x4 = 10 ⇒ –2x1 + 4(3) = 10 Total not of matrix = 4 × 1431

2x1 = 12 – 10 = 2 ⇒ x1 = 1
Sol 5: (C) A2 = A
–2x2 + 4x5 = – 2x2 + 4(4) = 20
(I + A)4 = (I2 + A2 + 2A)2
– 2x2 + 16 = 20
= [I + A + 2A]2 = [I + 3A]2 ( A2 = A)
2x2 = 16 – 20 = – 4 ⇒ x2 = – 4 = – 2 = I2 + 9A2 + 6A = I + 9A + 6A = I + 15A
2
–2x3 + 4x6 = – 2x3 + 4(0) = 10 = – 2x3
 α β
10 Sol 6: (C) A =  
x3 = =–5  −β α 
−2
Since, A is orthogonal matrix
M a them a ti cs | 16.53

So, AA’ = A’A = In 0 −2


det (I – A) = = 0 – (–2) (–2) = – 4
 α β  α −β   α + β −2 0
2
−αβ + αβ 
2
AA–1 =   = 
 −β α   β α   −αβ + αβ α + β 
2 2
(I + A) 2 1 1 0 2 
f(A) = =    
(I − A) −4 1 1 2 0 
α2 + β2 0  1 0 
  =   1 0 + 2 2 + 0  2 1 1  −1 −1
0 1 
2 2
 0 α + β  =–   =–   =  
2 0 + 2 2 + 0  2 1 1  −1 −1
a2 + b2 = 1 …(i)
x2 – px + q = 0 & x2 + px – q = 0 Sol 9: (D) A is skew symmetric matrix
Sum of both equation for common roots ⇒ ATA = I
x2 – px + q + x2 + px – q = 0 ⇒ AT = – A ⇒ (AT)2 = (A)2
2x = 0 ⇒ x = 0
2
⇒ ATA = – A2 = I
So if α is roots of x2 – px + q = 0 and β is roots of x2 + Taking square of both sides
px – q = 0
A4 = I
⇒α + 0 = p and α(0) = q = 0
⇒ A4n = I ( –A2 = I are A = – AT) \
β + 0 = – p and β(0) = – q = 0
A3A4n = A3I
⇒ α = p and β = – p
A4n+3 = – A2(–A) I
In (i) equation a2 + b2 = 1
A4(n + 1) – 1 = (–A)I = AT I = AI
⇒ p2 + (–p)2 – 1
4n - 1 ∈ N
⇒ 2p2 = 1
⇒A4n – 1 = AT
1
⇒p=± ,q=0
2
Sol 10: (D) |A| = |B|A2 × 2, B2 × 2

Sol 7: (A) (det A) = 3 Tr (A) = Tr(B)

(det λA) = 81 A2 – 3A + 14I = 0 and B2 – λB + µI= 0

if A’s order = n × n if A = B, |A| = |B|

then (det λA) = λn(det A) = λx 3 = 81 Tr |A| = Tr(B) satisfied so A2 – λA + µI = 0


81 µ = 14 ( A’s order = 2 × 2)
ln = = 27
3
ln = 27 = 3n  λ ∈ N Sol 11: (B) The adjoint of upper triangular matrix is
false.
So, n = 3
 That is equal to upper triangular not lower triangular
matrix.
1 2  1+x
Sol 8: (B) A =   , f(x) =
2 1  1−x
Sol 12: (B) A, B, (A + B) are non-singular
1 0  1 2  2 2  1 1 [A(A + B)–1B]–1
I + A =  + =   = 2 
0 1  2 1  2 2  1 1 = [A–1((A + B) –1)–1B–1] = (A–1(A + B) B–1)
= [(A–1A + A–1B)B–1) =[(I + A–1B)B–1)
1 0  1 2   0 −2  0 1 
I – A =  – =  = –2  
0 1  2 1   −2 0  1 0  = [B–1 + A–1BB–1] = A–1 + B–1

1 0 2  Sol 13: (C) A is an orthogonal matrix


(I – A)–1 =  
det(I − A) 2 0 
|A| = – 1
1 6 . 5 4 | Matrices

⇒ AAT = ATA = In
1 + tan2 x 1 0  1 0 
=   =  
⇒ |A| = |AT| = – 1 1 + tan x 0 1 
2
0 1 
⇒ AT = + A–1( A is an orthogonal matrix)
1 0
1 |AT A–1| = =1
⇒A = T
(adjA) = – (adjA) 0 1
det(A)

Sol 14: (C) A and B are square matrices of order 3 2 3 


Sol 17: (D) A =  
(A) adj(AB) = adj(A) + adj(B) is not necessary 5 −2
⇒ option (A) is wrong. ⇒ |A| = 2(–2) – 3(5) = – 15 – 4 = – 19
(B) (A + B)–1 ≠ A–1 + B–1 1  −2 −5
T
−1 2 3  1
A–1 =   =  = A
(C)AB = 0 |A|  −3 2  −19 5 −2  19
A, B are square matrix
So if AB = 0 Sol 18: (B) PT = P–1
⇒ |A| |B| = 0
P P  P1 P3 
⇒ |A| = 0 or |B| = 0 Assume P =  1 2  , PT =  
P3 P4  P2 P4 
a 0 0 ⇒ |P| = P1P4 – P2P3
 
Sol 15: (D) A = 0 a 0  ⇒ |A| = a3|I| = a3 T
0 0 a  1 1  P4 −P3  1  P4 −P2 
  P–1 = (adj P) =   =  
|P | | P |  −P2 P1  | P |  −P3 P1 
a2 0 0
  |P| = P1P4 – P2P3
adj(A) =  0 a2 0
 
 0 0 a2  P1 P3  1  P4 −P2 
   =  
P2 P4  | P |  −P3 P1 
|adj |A|| = (a2)3 = a6
|P| = 1
\|A| |adj (A)| = a3 a6 = a9
P2 = – P3
 1 tanx   cos θ sin θ 
Sol 16: (D) A =   Only option (B) 
 − tanx 1   is correct.
 − sin θ cos θ 
|A| = 1 + tan2x
Sol 19: (A) Br = I r > 1
 1 1 tanx  A
A = –1
  = A–1 Br–1 A – A–1B–1A = A–1Br B–1A – A–1B–1A
1 + tan x tanx
2 1  (1 + tan2 x)
= A–1B–1A – A–1B–1A = 0
 1 − tanx 
AT =  
 tanx 1  Sol 20: (C) A & B are orthogonal matrices

 1 1 tanx   1 − tanx  ⇒ AAT = ATA = In and BBT = BTB = In


AA =
T –1
   
(1 + tan x)  − tanx
2 1   − tanx 1  AB ⇒ (AB) (AB)T

 1 + tan2 x tanx − tanx  ⇒ (AB) (BTA)T


1  
= 2 ⇒ A(BBT)At = AIAT
(1 + tan2 x) tanx − tanx tan x + 1 
⇒ AAT = I
1 1 + tan x 0 2 
=   (AB)T (AB) = BTATAB = BTIB = BTB = In
2 2
(1 + tan x)  0 1 + tan x 
So (AB)T(AB)
M a them a ti cs | 16.55

gE = AB(AB)T= In Tr(A) = a1 + a2 + a3 = 12 and det |A| = a1 a2 a3 for


maximum of det (A) = a1 a2 a3
So, AB also satisfying property of orthogonal
a1 + a2 + a3 = 3a1 = 3a2 = 3a3 = 12
Sol 21: (C) C is an orthogonal matrix 12
a1 = =4
⇒ CC = C C = In
T T 3
Tr (CTAC) = Tr [(CTA)C] det |A| = 4 × 4 × 4 =64
= Tr [C(CTA)] = Tr (CCTA) = Tr (IA)= Tr (A)
Sol 27: (C) AB = B
Sol 22: (C) A and B are idempotent matrices BA = A
so, A = A and B = B
2 2
(A + B)2 = (AB + BA)2
|A|, |B| = or 1 A2 + B2 + AB + BA = (AB)2 + (BA)2 + (AB) (BA) + (BA) (AB)
AB = BA A2 + B2 + AB + BA = ABAB + BABA + AB + BA
A – B is non-singular
A2 + B2 = AAB + BBA = AB + BA
⇒ |A| = and |B| = 1 or |A| = 1 and |B| = 0
A 2 + B 2= A + B

Sol 23: (B) AAT = I


0 1 
Sol 28: (C) A =   , B is column matrix
(A BA)
T 10
3 0 
= (ATBA) (ATBA) (ATBA)………. . 0 
(A8 + A6 + A4 + A2 + I) B =  
⇒A BIBIBI………. BA =A B A
T T 10
1  2×1
0 1  0 1  3 0 
 0 c −b  A2 =     =   = 3I
 
Sol 24: (C) A =  −c 0 a  3 0  3 0  0 3 
 b −a 0  A4 = A2A2 = 32I
 
A6 = 33I, A8 = 34I
 a2 ab ca 
  A8 + A6 + A4 + A2 + I = I (1 + 3 + 32 + 33 + 34) = 121 I
B = ab b2 cb 
 2 0
 ac bc c  0   
2 I B =   ⇒B =1
 abc − abc b2 c − b2 c c2b − bc2  1  2
   
AB = −a2bc + ab2c −abc + abc −ac2 + ac2 
 2 2 ∞  Sol 29: (C) AB = BA and A2 = I
 a b − a b ab2 − ab2 abc − abc 
 ABA = A(AB) = A2B = IB = B
0 0 0 
 
AB = 0 0 0  = 0
0 0 0 
Previous Years’ Questions
 
Sol 1: (B)
Sol 25: (C) AB = BA, C2 = B
1 + a2 a a2
(A–1CA)2 = (A–1CA) (A–1CA) = A–1C ICA = A-1C2 A
⇒ ∆ = cos(p − d)x + cos(p + d)x cospx cos(p + d)x
= A–1BA = A–1(AB) = IB = B = C2 sin(p − d)x + sin(p + d)x sinpx sin(p + d)x

Sol 26: (D) Tr(A) = 12 1 + a2 a a2

a1 ⇒ ∆ = 2cospx cosdx cospx cos(p + d)x


0 0
  2sinpx cosdx sinpx sin(p + d)x
Assume A =  0 a2 0 ( A is diagonal matrix)
0 0 a3 

1 6 . 5 6 | Matrices

Applying C1 → C2 – 2cos dxC2 ⇒ 2cos x = – sin x or sin x = cos x


π π
⇒ cot x = –1/2 gives no solution in – ≤x≤
2
1 + a − 2acosdx a a 2 4 4
and sin x = cos x ⇒ tan x = 1
⇒∆= 0 cospx cos(p + d)x
0 sinpx sin(p + d)x ⇒ x = π/4

⇒ ∆ = (1 + a2 –2a cos dx) [sin (p+ d) x cos px Sol 4: (B) For infinitely many solutions, we must have

– sin px cos (p + d) x] k +1 8 4k
= = ⇒k=1
k k + 3 3k − 1
⇒ ∆ = (1 + a2 –2a cos dx) sin dx
Which is independent of p Sol 5: (B) Since, given system has no solution
∴ ∆ = 0 and any one amongst Dx, Dy, Dz is non-zero.
Sol 2: (A)
2 −1 2
1 x x +1 Let = 1 −2 1 = 0 ⇒ λ = 1
f(x) = 2x x(x − 1) (x + 1)x 1 1 λ
3x(x − 1) x(x − 1)(x − 2) (x + 1)x(x − 1)

Applying C3 → C3 – (C1 + C2) Sol 6: (A) Now,

 3 /2 −1 / 2   3 /2 1/2 
1 x 0 PTP =    
 1 / 2 3 / 2   −1 / 2 3 / 2 
= 2x x(x − 1) 0 =0
3x(x − 1) x(x − 1)(x − 2) 0 1 0 
⇒ PTP =  
0 1 
∴ f(x) = 0
⇒ PTP = I ⇒ PT = P–1
⇒ (100) = 0
Since, Q = PAPT
sinx cos x cos x
∴ PTQ2005P … (i)
Sol 3: (C) Given, cos x sinx cos x = 0
= PT(PAPT)(PAPT) ……. 2005 times]P
cos x cos x sinx
T
Applying C1 → C1 + C2 + C3 = (P P)A(P TP)A(P TP)........(P TP)A(P TP)

2005 times

sinx + 2cos x cos x cos x = IA2005 = A2005[from eq.(i)]


= sinx + 2cos x sinx cos x 1 1
sinx + 2cos x cos x sinx ∴A=  
0 1 
1 cos x cos x 1 1 1 1 1 2
A2 =    =  
= (2cos x+ sin x) 1 sinx cos x = 0 0 1  0 1  0 1 
1 cos x sinx 1 2 1 1 1 3
A3 =    =  
Applying R2 → R2 – R1, R3 → R3 – R1 0 1  0 1  0 1 
⇒ (2cos x + sin x) ………………………………………………..
………………………………………………...
1 cos x cos x
1 2005
0 sinx − cos x 0 =0 A2005 =  
0 0 sinx − cos x 0 1 
1 2005
⇒ (2cos x + sin x) (sin x – cos x)2 = 0 ∴ PTQ2005 P=  
0 1 
⇒ 2cos x + sin x = 0 or sin x – cos x = 0
M a them a ti cs | 16.57

Sol 7: (A) Every square matrix satisfied its characteristic 1 


equation = A + (p − 1)D 
a 
i.e. | A – λ I | = 0 1 
Sol 9: = A + (q − 1)D   … (i)
b 
1−λ 0 0 1 
⇒ 0 1−λ 1 =0 = A + (r − 1)D 
c 
0 −2 4 − λ
1 1 1
⇒ (1 – λ) {(1 – λ) (4 – λ) + 2}= 0
bc ca ab a b c
⇒ 13 –612 + 11λ – 6 = 0 Let ∆ = p q r = abc p q r ,
⇒ A3 –6 A2 + 11A – 6 I = O  … (i) 1 1 1 1 1 1

⇒ A2 6A + 11 I =
6A −1
[From equation (i)]
Sol 8: Since, a1, a2 are the roots of ax2 + bx + c = 0
A + (p − 1)D A + (q − 1)D A + (r − 1)D
b c
⇒ a1 + a2 = – and a1a2 = … (i) = abc p q r
a a
1 1 1
Also, b1, b2 are the roots of
px2 + qx + r = 0 Applying R1 → R1 – (A – D) R3 – DR2

q r 0 0 0
⇒ b1 + b2 = – and b1b2 = … (ii)
p p = abc p q r = 0

Given system of equations 1 1 1

a1y + a2z = 0 bc ca ab
And b1y + b2z = 0, has non-trivial solution ⇒ p q r =0
α1 1 1 1
α1 α2 β1
∴ = 0 ⇒ =
β1 β2 α2 β2
Sol 10: Given,
Applying componendo-dividendo
2ax 2ax − 1 2ax + b + 1
α1 + α2 β1 + β2 f’(x)= b b +1 −1
=
α1 – α2 β1 – β2 2(ax + b) 2ax + 2b + 1 2ax + b
⇒ (a1 + a2) (b1 – b2) = (a1 – a2) (b1 + b2)
Applying R3 → R3 – R1 – 2R2 , We get
⇒ (a1 + a2)2 {(b1 – b2)2 – 4b2b2}

= (b1 + b2)2 {(a1 + a2)2 – 4a1a2} 2ax 2ax − 1 2ax + b + 1


f’(x) = b b +1 −1
From equation (i) and (ii), we get 0 0 1
b2  q2 4r  q2  b2 4c 
 −  =  −  2ax 2ax − 1 2ax −1
a2  p2 p  p2  a2 a  =
b b +1
=
b 1
(C2 → C2 – C1)

b2q2 4b2r b2q2 4q2c ⇒ f’(x) = 2ax + b


⇒ 2 2 – = – 2
ap 2
ap 2 2
ap ap On integrating,

b2r q2c b2 ac we get f(x) = ax2 + bx + c


⇒ = ⇒ =
a p q2 pr Where c is an arbitrary constant
Since, f has maximum at x = 5/2
1 6 . 5 8 | Matrices

4π  π
⇒ f’(5/2) = 0 ⇒ 5a + b = 0 … (i) = 2sin2qcos = 2sin2qcos  π + 
3  3
Also, f(0) = 2 ⇒ c = 2 π
= – 2sin2qcos = – sin 2θ
and f(1) = 1 ⇒ a + b + c = 1 … (ii) 3
1
On solving equation (i) and (ii) for a, b, we get a = , sin θ cos θ sin2θ
4
5 ∴∆ = − sin θ − cos θ − sin2θ
b=–
4
 2π   2π   4π 
1 5 sin  θ −  cos  θ −  sin  2θ − 
Thus, f(x) = x2 – x + 2  3   3   3 
4 4
= 0 (since, R1 and R2 are proportional)
Sol 11:

sin θ cos θ sin2θ Sol 12: (B)

( )( ) ( )  A. ( A ) 
T T
 2π   2π   4π  =BBT = A −1 A T A −1 A − T A −1 A T −1

sin  θ +  cos  θ +  sin  2θ +  
 3   3   3  T
( ) ( ) ( )( )
T
−1
 2π   2π   = 4π  A= . A T A A T . A −1 A −1 AA T A −1
+ sin  θ –  + cos  θ –  + sin  2θ – 
 3   3   3 
(=
A A) A .(A ) ( A ) (=
A A)
T T
−1 T −1 −1 −1
 2π   2π   4=π A.= I
sin  θ –  cos  θ –  sin  2θ – 
 3   3   3 

 2π   2π  Sol 13: (D) AA T = 9T


Now, sin  θ +  + sin θ – 
 3   3  1 2 2  1 2 a 
  
A=
2 1 −2 2 1 2  = 9I
 2π 2π   2π 2π 
θ+ +θ−  θ+ –θ+   a 2 b  2 −2 b 
3 3  cos 3 3    
= 2sin  
 2   2  9 a + 4 + 2b 0
9 0 0
   
    ⇒ 0 9
2a + 2 − 2b ⇒ 0 9 0
2π  π a + 4 + 2b 2a + 2 − 2b a2 + 4 + b2 0 0 9
= 2sinθ cos = 2sinθ cos π − 
3  3
π Equation a + 4 + 2b =
0 ⇒ a + 2b =
−4 ....(1) ... (i)
= –2sinθ cos = – sinθ
3 2a + 2 − 2b =⇒
0 2a − 2b =
−2 ...(2) ... (ii)
 2π   2π  2 2 2 2
and cos  θ +  + cos  θ –  & a + 4 + b =0 ⇒ a + b =5 ...(3) ... (iii)
 3   3 
Solving a =
−2,b =
−1
 2π 2π   2π 2π 
θ+ +θ−  θ+ –θ+ 
= 2 cos  3 3  cos  3 3  5a −b 
 2   2  Sol 14: (B) A =  
    3 2 
   
5a −b  T
 2π   1 =A =  A adj A AA
= 2cosθ cos   = 2cosθ  −  = – cosθ  3 2 
 3   2
5a −b   2 b  5a −b   5a 3
 4π   4π    =  
and sin  2θ +  + sin  2θ −   3 2   −3 5a  3 2   −b 2
 3   3 
10a + 3b 0  25a2 + b2 15 − 2b 
 4π 4π   4π 4π   = 
 2θ + + 2θ −   2θ + – 2θ +   0 10a + 3b   15 − 2b 13 
3 3  cos  3 3 
= 2sin 
 2   2 
    Equate, 10a + 3b= 25a2 + b2
   
and 10a + 3b =
13
and 15a − 2b =
0
M a them a ti cs | 16.59

a b (ii) A is invertible ⇒ so |A| ≠ 0


= = k (let)
2 15 a11 a22 – a21a21 ≠ 0
2 a11a22 ≠ a21 a12
Solving=a = ,b 3
5
2 × (41 + 3) 2
2 total way → = . 27 = 18
So, 5a + b = 5 × + 3 = 5 3
C2 3
5
(iii) |A|max – |A|min
⇒|A|max = a11a22 – a21a12
JEE Advanced/Boards (a11 a22)max = 4 ( a11 + a22 + a12 + a12 = 4)
and + ( a21. a12)max = 3
Exercise 1
a21 a12 = 0 with a12 a22 = 4 = (2) (2)

Sol 1: (a) |A| = a, (B) = (adjA), |B| =b So |A|max = 2(2) – 0 = 4


|A|min = 0 – 2(2) = – 4
1 a a2 a3
S= + + + ....
2 b b3 b5 |A|max – |A|min = 4 – (–4) = 8
(iv) A is symmetric or skew symmetric or both |det A| is
a 
2
a  a  divisible by 2
= 1 + +   + .... 
b b 2 2
b  
  So |det A| can be 0, 2, 4
   4 0  0 0   2 0  2 1  0 2 
 A⇒  ,  ,  ,  ,  
a 1  a  b2  ab  0 0  0 4  0 2  1 0  2 0 
= =   =  
b a  b  b2 − a  b2 − a
1 − 2  Total no. → 5
 b 
2ab  4 −4 5 
S=
b2 − a  
Sol 3: A =  −2 3 −3
b > a, a = 3  3 −3 4 
 
|B| = |A|n–1 = |A|3–1 = (3)2 = 9 = b
A11 = 12 – 4 = 3 A13 =…………………. .
2(3)9 2(3)9 A12 = – 9 + 8 ⇒ – A21 = – 15 + 16 = 1
⇒ (ab2 + a2b + 1) = (3. (9)2 + 32. 9 + 1)
2
9 −3 92 − 3
A22 = 16 – 15 = 1, – A23 = – 12 + 12 = 0
6.9 6.9
= (1 + 81 + 243) = = 225 A31 = 12 – 15 = – 3, –A32 = – 10 + 12 = 2
78 78
(b) |A| = –2, |B| = 1 A33 = 12 – 8 = 4

(A–1) (adj B–1) adj(2A+1)  3 +1 −3


 
adjA =  −1 1 2 
⇒ |A–1| |adj B–1| 23–1 |adj A|  −3 0 4 
 
1 4×4 16
= × 1 × 22 × (–2)2 = = =–8
−2 −2 −2 |A| = 4[A11] – 4(A) + 5[A13]

= 4(3) – 4(–1) + 5(–3) = 12 + 4 – 15 = 1


Sol 2: A = [aij], aij ∈ {0, 1, 2, 3, 4}
a11 + a12 + a21 + a22 = 4  3 +1 −3
adjA  
A =
–1
=  −1 1 2 
(i) Tr (A) = a11 + a22 = 4 |A|  −3 0 4 
 
and a11, a22 ∈ {0, 1, 2, 3, 4}
total possibilities = 0 + 4 = 4 + 0 = 1 + 3 = 3 + 1 = 2 + 2  3 +1 −3  3 +1 −3  17 4 −19 
     
A–2 =  −1 1 2   −1 1 2  =  −10 0 13 
⇒5
 −3 0 4   −3 0 4   −21 −3 25 
     
1 6 . 6 0 | Matrices

 2 −1 –1  2 −1
1 1 1  adjB =  ,B =   adjC
  2 3   −3 2   −3 2 
Sol 4: (a) A = 2 4 1 , B =  
2 3 1  3 4  2×2  −3 −2  –1 1 3 2 
 3×3 =  ,C =  
 −5 3  19 5 −3
1 0 1  2 4 
BPA =   ⇒ BAC = 
0 1 0  2×3 
3 −1
P P2 P3   2 −1 2 4   4 −3 8 +1 
Assume P =  1  B–1BAC = AC =     =  
P4 P5 P6  2×3  −3 2   3 −1   −6 + 6 −12 − 2

1 0 1  1 9 
BPA =  AC =   ⇒ ACC
–1
  0 −14 
0 1 0 
1 0 1  1 9  3 2  1
B–1BPA = PA = B–1 0 1 0  =A=     –
   0 −14  5 −3  19

|B| = 8 – 9 = – 1 1 3 + 45 2 − 27  1  48 −25
A=   =  
19  −70 42  19  −70 42 
1  4 −3  −4 3 
adjB =   =  
−1  −3 2   3 −2
 2 − 3 P 0 
 
 −4 3  1 0 1   −4 3 −4  Sol 5: A =  0 m2 − 8 q 
PA =     =    
 3 −2   0 1 0   3 −2 3  r 0 n2 − 15
 
|A| = 1[4 –3]– 1 [2 – 2] + 1 [ 6 – 8] = 1 – 2 = – 1
A2 = A[ A is idempotent matrix]
 1 3 − 1 1 − 4  1 2 −3 A2 =
   
adjA =  0 −1 2 − 1  =  0 −1 1  ,
6 − 8 2 − 3 +2   −2 −1 +2  (2 − 3)2 + 0 p(2 − 3) + p[m2 − 8] pq 
     
2 2 2 2
 qr (m − 8) q(m − 8) + q(n − 15)
 −1 −2 3   2 2 
r( − 3) + r(n + 5) rp (n2 − 15)2 
 
A =  0 −1 −1
–1

 2 1 −2  2 − 3 p 0 
   
=  0 m2 − 8 q 
 −4 3 −4  –1  
PAA–1 =  A =P  r 0 n2 − 15
 3 −2 3  
 −1 −2 3  compare elements
 −4 3 −4   
P=   0 1 −1 ⇒ (12 – 3)2 =  2 – 3 ⇒ 12 – 3 = 0 or 1
 3 −2 3   2 1 −2
  =± 3 or ± 4= ±2
 −4 7 −7  p[l2 – 3 + m2 – 8] = p⇒ p = 0 or l2 + m2 – 11 = 1
P=  
 3 −5 5 
rp = 0 ⇒ r = 0 or p = 0
2 1  3 2  2 4 
(b)   ·A 5 −3 =   (n2 – 15)2 = n2 – 15 ⇒ n2 – 15 = 1 or 0
3 2    3 −1
q[(m2 – 8) + n2 – 15] = q ⇒ q = 0 or m2 + n2 – 23 = 0 + 1
2 1  3 2 
assume B =   C=   (m2 – 8)2 = m2 – 8 ⇒ m2 – 8 = or 1
3 2  5 −3
m = ± 8 or ± 9 = ± 3
|B| = 4 – 3 = 1 |C| = – 9 – 10 = – 19
if, l1m1n, q, q, r ∈ z
S = {0, ±2, ±3, ±4}
M a them a ti cs | 16.61

⇒ Sum of products of elements = 22+32+42=29  ∞  ∞ 1 1 1 


Tr  ∑ 3n An  = ∑ Tr(3 A3 ) = 18  + 2 + ..... ∞ 
n
  i=1 3 3 3 
cos x − sinx 0   n=1 
 
Sol 6: F(x) =  sinx cos x 0     
 0  1  1  18 8 18 
 0 1  = 18   = × = 
 3 1 − 1  3 3 − 1 2 
cos y − sin y 0    3  
 
F(y) =  sin y cos y 0 
3(1) − 1 + 3(2) − 2 + 3(3) − 3 2+4+6 12
 0 0 1  Tr (Bn) = = =
 22n
22n
22n
cos x − sinx 0  cos y − sin y 0  12
   Tr(2nBn) =
F(x). F(y) =  sinx cos x 0   sin y cos y 0  2n
 1   0 1 
 0 0 0 a 1 1 
m= ∑ [Tr(2n Bn )] = 12  + + ... 
cos x cos y − sinx sin y − cos x sin y − sinx cos y 0  n=1  2 22 
 
= sinx cos y + cos x sin y − sinx sin y + cos y cos x 0    
 0 0 1   1  1   12
 =12   =  = ·2 12
 2 1 − 1   2
cos(x + y) − sin(x + y) 0   
  2  
=  sin(x + y) cos(x + y) 0 
 0 0 1  l + m = 12 + 9 = 21

[F(x)]–1 = F(–x)
Sol 8: A is 3 × 3 matrix
L. H. S. ⇒ |F(x)| = cos2(x) + sin2x – 1
A11 = a33 = 2, all other aij = 1
 cos x sinx 0  2 1 1 
    R ⇒ R1 − R 2
adj[F(x)] =  − sinx cos x 0  A = 1 1 1  1
 0 0 1  R ⇒ R 3 − R1
 1 1 2  3
 
cos( −x) − sin( −x) 0  1 0 0 
adjF(x)    
[F(x)] =–1
=  sin( −x) cos( −x) 0  = F(–x) A = 1 1 1  R2 ⇒ R2 – R1 – R2
1
 0 0 1  0 0 1 
  
L. H. S. = R. H. S. 1 0 0 
 
Hence proved A = 1 1 0  = I
0 0 1 
 
Sol 7: An = [aij], Bn = [bij] A–1 = I = xA2 + yA + zI
2i + j 3i − i I = (x + y + z)I
aij = , bij =
32n 22n (x + y + z) = 1
l = nLim
→∞
Tr[3A1 + 32A2 + 33A3+…+ 3nAn+…. ]
1 2 2 
For An Tr(A) = a11 + a22 + a33  
Sol 9: A = 2 2 3
1 −1 3 
(2(1) + 1) + 2(2) + 2 + 2(3) + 3 9+6+3 18  
= = =
2n 2n
3 3 32n 2 1 1  10 
   
n
3 18 18 C=  2 2 1  , D = 13  , Cb = D
Tr(3nA n) = = 1 1 1  9
2n
3 3n    

|C| = 2[2 – 1] + 1 [1 – 2] + 1 [2 – 2] = 2 – 1 = 1
1 6 . 6 2 | Matrices

 1 0 −1  1 
   0 0
5x 0 0
adjC =  −1 1 0  = |C| C–1 = C–1  5x   
 0 −1 2  (AB)–1 =  0 1 0  = AB =  0 1 0
   −10x + 2 1  0 10x − 2 5x 
 
C–1Cb = C–1D 0 
 5x 5x 
1 0 −1  10  ⇒ x = 1/5
   
b =  −1 1 0  13 
 0 −1 2  9  −5 
 3×3  3×1  0 0 
 −1 0 0 
5   
 10 − 9  1  AB =  0 1 0  = 0 1 0
     −5   0 −4 −1
b =  10 + 13  =3 0 −4 
 −13 + 18  5   5 
   
(AB)2 = (AB) (AB) = (AB) (AB)–1 = I
|A| = 1[6 + 3] + 2[3 – 6] + 2[–2 –2] = 9 – 6 – 8 = – 5
Tr[AB+(AB)2+(AB)3+…+(AB)100]
6 + 3 −8 2   9 −8 2  = Tr[AB+I+AB+I+…+I]
   
adjA =  −3 3 − 2 1  =  −3 1 1 
= Tr[50AB+50I]=50 Tr(AB)+50Tr(I)
 −4 3 2 − 4   −4 3 −2
  
= 50[–1+1–1]+50 [1+1+1] =–50+3(50)=100
1 1
A–1 = adjA = – adjA
|A| 5 Sol 11: Mn = [mij]order = n
AX = b 1 ≤ i ≤ n, mij = 10;
 9 −8 2  1  1 ≤ i ≤ n – 1, mi + 1, I = mi, i + 1 = 3
1    
X = A b = –  −3 1 1 
–1
3
5 All other entries in Mn are zero
 −4 3 −2 5 
   
10 3 0 
9 − 24 + 10   −5   1   
M3 =  3 10 3  , |M3| = 10[100 – 9] + 3 [– 30]
1   1    
X=– −3 + 3 + 5 = – +5 = −1  0 3 10 
5   5      
 −4 + 9 − 10   −5   1 
      = 1000 – 90 – 90 = 820

10 3 
2 0 7   −x 14x 7x  M2 =   ⇒ |M2| = 100 – 9 = 91
  
Sol 10: A = 0 1 0  , B=  0 1 0 
  3 10 
1 −2 1   x −4x −2x  D3 – 9D2 = 820 – 9(91) = 820 – 819 = 1
   

 −2x + 7x 28x − 28x 14x − 14x   −5 1 3 1 1 2 


     
AB =  0 1 0 Sol 12: A =  7 1 −5 & B =
 3 2 1 
 −x + x 14x − 2 − 4x 7x − 2x   1 −1 1  2 1 3 
     
5x 0 0 x + y + 2z =
1 4 0 0
   
=0 1 0 3x + 2y + z =7 AB =  0 4 0  = 4I3
 0 10x − 2 5x  2x + y + 3z =2 0 0 4 
   

|AB| = 5x [5x] = 25x2  −5 1 3


 
4 4 4
5x 0 0 −1 A 7 1 −5 
1   1B = =
(AB)–1 = 0 25x 2
0 4 4 4 4
 
25x2  2  1 −1 1 
 0 −50x + 10x 5x   4 4 4 
M a them a ti cs | 16.63

|B| = 1[6 – 1] + 1[2 – 9] + 2[3 – 4] x  b 


3 −2 1 
=5–7–2=–4      
5 −8 9  y  =  3 
2 1 −3  z   −1 
1 1 2      
|X|
x= , |X| 7 2 1 = [6 – 1] + 7[2 – 3] R2 ⇒ R2 + 2R3
|B |
2 1 3
 3 −2 1  x  6  b
+ 2[1 – 4] = + 5 – 7 – 6 = – 8        
5 + 4 −8 + 2 9 − 6  y  = 3 − 2  =  1 
 2 1 −3  z   −1   −1 
1 1 2       
−8
x= =2,Y= 3 7 1
−4 3 −2 1  x b
2 2 3      
9 −6 3  y  =  1 
= 1[21 – 2] + 1 [2 – 9] + 2[6 – 14] = 19 – 7 – 16 = – 4 2 1 −3 z   −1 
     
1 1 1 Compare row 2nd and 1st
−4
y= =1, Z = 3 2 7 3x – 2y + z = b  … (i)
−4
2 1 2
9x – 6y + 3z = 1
= 1[4 – 7] + 1[14 – 6] + 1[3 – 4] = – 3 + 8 – 1 = 4 1
3x – 2y + z = … (ii)‘
or ⇒ Bx = C 3
x = B–1C From equation (i) and (ii)
−4 1
z= =–1 b=
−4 3
1
(x, y, z) = (2, 1, –1) For no. solution a = – 3 and b ≠
3
 −5 1 3 (iii) Has infinitely solution
 
4 4 4 1  1
so a = – 3 and b =
1
7 1 −5     
x =  7  =  +1 
3
4 4 4
  2   −1  so |D| = 0 and |D|x = 0
 1 −1 1   3×1  
 4 4 4 3×3
1 2  3 1  1 2 
Sol 14: A =   , B = ,C=  
3 4  1 0  2 4 
3 −2 1   x   6 
      X X2 
Sol 13: 5 −8 9   y  =  3  X = 1 
2 1 a   z   −1 
       X3 X4 

3 −2 1  (a) AX = B – I
 
|D| = 5 −8 9  = 3[–8a – 9] – 2 [18 – 5a] + 1[5 + 16] \
2 1 a  1 2   X1 X 2  3 1  1 0  2 1 
  X     =  –   =  
3 4   X3 X 4  1 0  0 1  1 −1
= – 24a – 27 – 36 + 10a + 21 = – 14a – 42
(i) System has a unique solution 101 ≠ 0 |A| = 4 – 6 = – 2

– 140 – 42 ≠ 0  4 −2 
adjA =  
42  −3 1 
a≠– =–3
14 1  4 −2  1
⇒ A–1 =−   = | A | adjA
a ≠ – 3 and b ∈ R −2  −3 1 
(ii) At a = – 3 has no solution ⇒ a = – 3
so A–1AX = X = A–1(B – I)
1 6 . 6 4 | Matrices

1  4 −2  2 1  1 8 − 2 4 + 2   a11 a12  1  −1 


X=     =–   so     =  
−2  −3 1  1 −1 2 6 + 1 −3 − 1 a21 a22   −1  2

 −3 −3 a11 – a12 = – 1;+a21 – a22 = 2 … (i)


16 6   
X=–   = 5
2  −5 −4   2
 2   a2 + a a a11a12 + a12a22 
M2 =  11 12 21

(b) (B – I)X = IC = C a11a21 + a21a22 a21a12 + a222 

2 1  1 1 
B–I=   M2 =   =  
1 −1  −1  0 
|B – I| Z[ –1] – 1 = – 3  a2 + a a a11a12 + a12a22  1 1 
⇒  11 12 21
   =  
 −1 −1 a11a21 + a21a22 a21a12 + a222  −1
  0 
adj(B – I) =   , (B – I)
–1

 −1 2 
2
a11 + a12a21 + a12a22 = 1 … (ii)
adj(B − I) 1  −1 −1 1 1 1 
= =   =  
|B−I | −3  −1 2  3 1 −2 ⇒ a11a21 + a21a22 – a21a12 + a222 = 0

1 1 1  1 2  ⇒ a11[a11 – a12] + a12[a21 + a22] = 1


X = (B – I)–1C =   
3 1 −2 2 4  ⇒ a11(–1) + a12(2) = 1

1 1 + 2 2 + 4  1 3 6  1 2 ⇒ 2a12 – a11 = 1
X=   =   =  
3 1 − 4 2 − 8  3  −3 −6   −1 −2  ⇒ a12 + 1 = 1 ⇒ a12 = 0

(c) CX = A ⇒ a11 = – 1
⇒ a21[a11 – a12] + a22[a21 – a22] = 0
1 2
|C| = =4–4=0
2 2 ⇒ a21[–1] + a22[2] = 0

So C–1 does not exit ⇒ Y has no solution ⇒ 2a22 – a21 = 0


–[a21 – a22 – a22] = 0
Sol 15: A is orthogonal matrix 2 – a22 = 0
⇒ AA’ = A’A = In ⇒ a22 = 2
and B = AP, P is non-singular ⇒ a21 = 4
if A is orthogonal, so A–1 is also orthogonal
a a12   −1 0 
B = AP M =  11 = 
a21 a22   4 2 
BB–1 = APB–1
|M – XI| = 0
I = APB–1
−1 − x 0
A–1 = A–1APB–1 =0
4 2−X
A–1 = PB–1
(1 + x) (x – 2) = 0 ⇒ x = – 1 or x = 2
A–1 is orthogonal, so PB–1 is also orthogonal
5x1 + 2x2 = 5(2) + 2(–1) 2 > – 1 = 10 – 2 = 8
1  −1  1 1   2 3
Sol 16: M   =   ; M2   =   Sol 17: A1 = 1, A2 =  ,
 −1  2  −1  0   4 5
a a12  6 7 8
Assume M =  11   
a21 a22  A3 =  9 10 11  …………….
12 13 14 
 
M a them a ti cs | 16.65

No. of element in An = n2 1 
 0 0
For An = 102 = 100, (10 in each row)
5 
 1 1 1 1 1
Sn = sum of all element of An A = 0 0  , |A|= . . =
6 5 6 7 210
So S1 = 1, S2 = 2 + 3 + 4 + 5  
0 1
0
S9 = 1 + 2 + 3+………m  7 
Where m = 1 + 22 + 32 + 42 +……+92 1 
 0 0
=
(2n + 1)n(n + 1)
=
9(18 + 1)(9 + 1)  42 
 1 1
6 6 Adj A =  0 0  , A–1 =
3 35 |A|
 
= × 10 × 19 = 285 0 1 
2 0
 30 
So in a10 ⇒ a11 = 285 + 1 = 286
a22 = 286 + 11 1 
 
ann = 286 + (n – 1)11 1  42 
 1 
10 adjA = 1  
tr(A) = ∑ aij = 286 × 10 + [11 + 11(2) 210

35

i=1
 1 
+ 3(11)+……+9(11)]  30 
= 286º + 11 [1 + 2 +…+ 9]
1 1 1
9 × 105 Tr(A–1) = 210  + +  = 5 + 6 + 7 = 18
= 286º + 11 × = 286º + 11 × 45 = 3355  42 35 30 
2
 J6,5 72 J11,5 
1 xn  
Sol 18: In, m = ∫ 0 xm − 1 dx ∀ n, m (b) A =  J7,5

63 J12,5 

1 xn  J8,5 56 J13,5 
In, m = ∫ 0 xm + 1 dx ∀ x > m, n, m ∈ N
 J6,5 72 J11,5 
 
(a)A = [aij]3×3 B =  J7,5 63 J12,5 
 
I −Ii+3,3 , i=
j  J8,5 56 J13,5 
aij =  6 +i,3
 0 ,i≠ j det(A) = – 72 [J7, 5 J13, 5 – J12, 5 J8, 5]+…………….
a11 = I6+1, 3 – I1 + 3, 3 = I7, 3 – I4, 3 Jn, α Jm, α – JN,α JM, α

1 x7 dx 1 x 4 dx 1x
7
− x4 1 xn + m xn + m 
= ∫ 0 x3 − 1 ∫ 0 x3 − 1
– =∫
0
x3 − 1
dx If n + m = N + N, the ∫ 0  x x +1 −  dx =
x α+1 
0

1
So det (A) = 0
1 4  x3 − 1   x5  1
=∫ x  3  dx =   = |B| = 72 [I12, 5I8, 5 – I7, 5 I13, 5] +……….
0  x −1  5  0 5
 
Sum as above 12 + 8 = 7 + 13
1
1  x6  1 So, |B| = 0
a22 = I8, 3 – I5, 3 = ∫ x5dx =   =
0
 6  0 6 det(A) – det (B) = 0
1
I 1 6  x3 − 1   x7  1
8+1 ,3 3 −4  a b
a33 = – I6, 3 = ∫ x  3 =
 x −1
 dx =  Sol 19: A =   and B =  
  0 7
7
0
  1 −1  0 1 
P is orthogonal matrix ⇒ Q = PAPT,
R = PTQKP, S = PBPT, T = PTSK P
1 6 . 6 6 | Matrices

Sol 20: A → p,q,t; B → s; C → p,r; D → r 1 2 3 


 
Sol 2: (A) A =  4 5 7 
A2×2 = [aij]
2 3 α
 
Elements are 0, 1, 2, 4
f(x) = x3 – 8x2 + bx + γ
a a12 
(A) A =  11  a satisfies f(x) = 0
a21 a22 

|A| = a11 a22 – a21a12 1 0 


Sol 3: (A) two rowed unit matrix I2 =  
If |A| > 0 ⇒ a11a12 > a21 a12 0 1 
2
F2 = I2
1 0  α β
  ⇒ |A| = 2 So square root of I2 = I2 =   (given)
 4 2  γ δ

2 1 1 2 1 0  α β 
⇒ 8, =4   = 
0 4 0 4 0 1   γ δ 

(B) If |A| = m, then is also a matrix ⇒ |A| = – m α=1= δ,γ=β=0

So for all matrix, have one –ve det (A) matrix so Σ det(A)
=0  4 2i
Sol 4: (A) A =   (A – 2I) (A – 3I) = ?
(C) Least value of det(A) = 2 or – 2  i 1

|adj(adj(adjA))| = ((±2)2–1)2–1)2–1 = ±2, 2 or –2  4 2i 1 0  4 − 2 2i  2 2i 


A – 2I =   –2  =   =  
(D) det(A) is algebraically least = – 8  i 1 0 1   i 1 − 2  i −1
 4 2i 1 0   4 − 3 2i  1 2i 
4adjA 4  1  A – 3J =   – 3  = = 
4A–1 = = adjA =   (adj A)  i 1 0 1   i 1 − 3  i −2
|A| −8  −2 
2 2i  1 2i 
|4A–1| = |–Z–1adjA| = (–Z)2 |A|2–1 (A – 2J) (A – 3J) =    
1  i −1  i −2
= ×–8=–2
4 2(1) + 2i(i) 4i − 4i  0 0 
=  =  = null matrix
 i−i 2i(i) − 1( −2) 0 0 

Exercise 2
 1 cos(α − β) cos(α − γ )
 
Single Correct Choice Type Sol 5: (D) A = cos(β − α ) 1 cos(β − γ ) 
cos( γ − α ) cos( γ − β) 1 
 
Sol 1: (B) Let [A, B] = AB – BA
|A= 1[1 – cos(β – γ)cos(γ – β)] + cos (α – β) [cos(β – r) cos
[[A, B], C] + [[B, C], A] + [[C, A], B]
(γ – α) – cos(β – α)]
⇒ [[A, B], C] = [AB – BA, C] = (AB – BA)C – C(AB – BA)
+cos (α – γ)[cos(β – α) cos(γ – β) – cos(γ – α)]
= ABC – BAC – CAB + CBA …(i)
( cos(A) = cos(–A))
[[B, C], A] = [BC, – CB, A] = (BC – CB)A – A(BC – CB)
= 1 – cos2 (β – γ) +2cos (α – β) cos (β – γ) cos (γ – α) –
= BCA – CBA – ABC + ACB …(ii) cos2 (β – α) = cos2(α – γ)
[[C, A], B] = [CA – AC, B] = (CA – AC) B – B(CA – AC)  ( α + β − γ − α + γ − β) 
2
= 1 – cos 
= CAB – A, B – BCA + BAC …(iii)  2 
sum of equation (i), (ii) & (iii) = 1 – cos20 = 1 – 1 = 0
[[A, B]C] + [[B, C], A] + [[C, A], B] = ABC – BAC + BAC –
ABB +……. = 0
M a them a ti cs | 16.67

x + a b c  −1
  1 + 1 0 
−1 2 0  1 2 0 
Sol 6: (C) A =  a x +b c  =   =   =  
 0 1 + 1 0 2  det(A) 0 2 
 a b x + c 

1 
matrix A is non singular  0
1 2 0  2
=   =  
|A| ≠ 0 4 0 2  0 1
 2 
x + a b c 
 
 a x +b c  ≠0 1 
 a b x + c   x  −2
 5 0
 Sol 9: (C)  25  =  
⇒ (x + a) [(x + b) (x + c) – bc] + b[ac – a(x + c)] + c[ab 0 1   −a 5 
 25 
– a(x + b)] ≠ 0
1
−1 −1
⇒ (x + a) [x2 + x(b + c)] + b [ac – ax – ac] + (c) (–ax) ≠ 0  5 0  5 0 1 5 a 25 5 a
=     =    
⇒ x + ax + x (b + c) + ax(b + c) – abx – acx ≠ 0
3 2 2
 −a 5   −a 5  25 0 5  0 5 
⇒ x3 + x2(a + b + 1) ≠ 0
1 5 a 5 a 1 / 25 x 
= −     
⇒ x2[x + (a + b + c)] ≠ 0 625 0 5  0 5   0 1 / 25
⇒ x ≠ 0 and x ≠ – (a + b + c)
1 25 5a + a5 1 25 10a
Sx x = R – {0, –(a + b + c)} =  =  
625  0 25  625  0 25 

Sol 7: (B) A is skew symmetric matrix 10a 2a


x= =
A = A and B B = B
2 T 625 125

BTB = B
Sol 10: (D) A2 = I
Multiply with B ⇒ (B B)B = BB = I
–1 T –1 –1
|A| =1, B = (adj A)–1
BI=B =I
T T
1
A–1 = adj(A) = adj(A)
BT = I. So B = I |A|
X = (A + B) (A – B) (A–1)–1 = (adjA)–1
X = A2 – AB + BA – B2( B = I) A =(adj A)–1 = B given
X=A–A+A–I=A–I A=B
X7 = (A – I)T = AT – I A2 = I
X7X = (AT – 1) (A – I) AA = AB = I
= AAT – AT – A + I AB = AA = BA = I
A = – A( A is skew symmetric)
T
⇒ B ≠ I we can’t say that B = I
X7X = – AA – A + A + I
Sol 11: (B) adj A = Border of both = 3 × 3
= – A2 + I = – A + I = I – A
Adj (3AB)= 33–1 adj (AB)
Sol 8: (C) Z, and Z2 are uni modular complex = 9(adj B) (adjA) = 9(adj B)B = 9|B| = I3

 z1 − z 2 
−1
 z1 z2 
−1 \adj(AB) = (adjB) (adj A)
    = A (assume)
 z2 z1   z2 z1 
Sol 12: (C) AT + B = 0
−1
 z z + z 2 z2 z1 z 2 − z 2 z1  A = adjB, tr(A) = 1, A2 = A
=  1 1 
 z2 z1 − z1 z2 z2 z 2 + z1 z1 
tr{adj (ATB)}
1 6 . 6 8 | Matrices

⇒ AT + B = 0 ⇒ M2 NI(–M–1)(–N–1)(–M)
⇒ AT = – B ⇒ – M2 NM–1N–1M
⇒ tr[(adj B) (adj AT)] ⇒ – M ⋅ (MN)M–1N–1 M
⇒ tr[A adj(–B)] ⇒ – M(NM)M–1N–1 M
⇒ tr(A(–1)n–1A] ⇒ – MN(NN–1)N–1 M
⇒ (–1)n–1 tr(A2) = (–1)n–1 tr(A) ⇒ – M(NN–1)M
⇒ (–1)n–1 (–1) = (–1)n ⇒ – M2
Note: Here, non-singular word should not be used,
Sol 13: (C) C = A + B since there is no non-singular 3 x 3 skew-symmetric
|C|2 = |A|2 |I – (A–1B)2| matrix.

AB = BAC = A + B x2 + x x +1 x−2
2
⇒ |C| = |A + B| = |A| [I + A–1B] Sol 3: Let ∆ = 2x + 3x − 1 3x 3x − 3
2
|C|2 = |A|2 |I – A–1B| |I + A–1B| … (ii) x + 2x + 3 2x − 1 2x − 1

(2) | C |2 Applying R2 → R2 – (R1 + R3), we get


Equation ⇒
(1) |C|
x2 + x x +1 x−2
2 −1 −1
| A | | I − A B || I + A B | ∆= −4 0 0
=
−1
| A || I + A B | x2 + 2x + 3 2x − 1 2x − 1
|C| = A–1 |I – A–1B| = |A – B| x2
Applying R1 → R1 + R
|C| = |A – B| 4 2
x2
and R3 → R3 + R,w
4 2
Previous Years’ Questions x x +1 x −2
∆ = −4 0 0
Sol 1: (A) | A | ≠ 0, as non-singular
2x + 3 2x − 1 2x − 1
1 a b
Applying R3 → R3 – 2R1
∴ ω 1 c ≠0
x + 0 x +1 x −2
ω2 ω 1
= −4 0 0
⇒ 1(1 – cω) – a (ω – cw2) + b (w2 – w2) ≠ 0
3 −3 3
⇒ 1 – cω – aω + acw2) ≠ 0
x x x 0 1 −2
⇒ (1 – cω) (1 – aω) ≠ 0
= −4 0 0 + −4 0 0
1 1 3 −3 3 3 −3 3
⇒a≠ ,c≠
ω ω
⇒ a = ω, c = ω and b ⇒{ ω, w2} 1 1 1 0 1 −2
⇒ 2 solutions = x −4 0 0 + −4 0 0
3 −3 3 3 −3 3
Sol 2: (C) Given, MT = – M, NT = – N ⇒ ∆ = Ax + B
and MN = NM … (i) 1 1 1 0 1 −2
∴ M N (M N) (MN )
2 2 T –1 –1 T Where A = −4 0 0 and B = −4 0 0
⇒ M2N2N–1(MT)–1(N–1)TMT 3 −3 3 3 −3 3

⇒ M2N(NN–1)(–M)–1(NT)–1(–M)
M a them a ti cs | 16.69

Sol 4: The given system of equation a−1 n 6


3x – y + 4z = 3 Sol 6: Given, Da = (a − 1)
2 2
2n 4n − 2
x + 2y – 3z = –2 (a − 1) 3 3
3n 3n2 − 3n
6x + 5y + lz = – 3 n

Has at least one solution, if ∆ ≠ 0 ∑ (a − 1) n 6


a=1
n
3 −1 4 ∴= ∑ (a − 1)2 2n2 4n − 2
∴ ∆ = 1 2 −3 ≠ 0 a=1
n
6 5 λ
∑ (a − 1)3 3n3 3n2 − 3n
a=1
⇒ 3(2λ + 15) + 1(λ + 18) + 4(5 – 12) ≠ 0
⇒ 7(λ + 5) ≠ 0 ⇒ λ ≠ – 5 = Applying C3 → C3 – 6C1
For λ = – 5 1 1 0
n3 (n − 1)
⇒∆=0 = 2n − 1 6n 0 = 0
12
3 −1 4 n − 1 6n 0
Then, D1 = −2 2 −3 = 0 n
−3 5 −5 ⇒ ∑ ∆a = c, ( c = 0 ie, constant)
a=1

3 3 4
p b c
D2 = 1 −2 −3 = 0
Sol 7: Let ∆ = a q c
6 −3 −5
a b r
3 −1 3
Applying R1 → R2 – R1 and R3 → R3 – R1, we get
D3 = 1 2 −2 = 0
6 5 −3 p b c
a−p q−b p b
∆ = a − p q − b 0 =c +(r–c)
D1 = D2 = D3 = 0 a−p 0 a−p q−b
a−p 0 r −c
Sol 5: The system of equations has non-trivial solution, = – c(a – p) (q – b) + (r – c) [p (q – b) – b(a – p)]
if ∆ = 0
= – c(a – p) (q – b) + p(r – c) (q – b) – b(r – c)(a – p)
sin3θ −1 1 Since, ∆ = 0
⇒ cos2θ 4 3 = 0
⇒ – c(a – p) (q – b) + p(r – c) (q – b) – b(r – c)(a – p) = 0
2 7 7
[On dividing both side by Radding 204 th side and –x
Expanding along C1, we get + ( sin α ) y − ( cos α ) z =
0 has non-
⇒ sin 3θ ⋅ (28 – 21) – cos 2θ (–7 –7) + 2(– 3 –4) = 0 b c b
(a – p) (q – b) (r – c)] + + +2 =2
⇒ 7sin3θ +14cos2θ – 14 = 0 b −a r −c q−b
⇒ sin 3θ + 2cos 2θ – 2 = 0 p q r
⇒ + +0+ +0=2
⇒ 3sinθ – 4sin3θ + 2(1 – 2sin2θ) – 2 = 0 p −a q−b r −c

⇒ sinθ (4sin2θ + 4sinθ – 3) = 0 p q r


⇒ + + =2
p −a q−b r −c
⇒ sinθ (2sinθ – 1) (2sinθ + 3) = 0
⇒ sinθ = 0, sinθ = 1/2 n! (n + 1)! (n + 2)!
(neglecting sinθ = – 3/2) Sol 8: Given, D = (n + 1)! (n + 2)! (n + 3)!
(n + 2)! (n + 3)! (n + 4)!
⇒ θ = nπ, nπ + (– 1)n π/6, n ∈ Z
Taking n!, (n + 1)! and (n + 2)! Common from R1 , R2 and
R3 respectively.
1 6 . 7 0 | Matrices

1 (n + 1) (n + 1)(n + 2) ∴ 2α – π/4 = 2n π ± π/4


∴ D = n!(n+1)!(n+2)! 1 (n + 2) (n + 2)(n + 3) ⇒ 2α = 2nπ – π/4 + π/4 or 2α = 2nπ + π/4 + π/4
1 (n + 3) (n + 3)(n + 4) ∴ α = nπ or nπ + π/4
Applying R2 → R2 – R1 and R3 → R3 – R2, we get
Sol 10: Given,
1 (n + 1) (n + 1)(n + 2)
ax − by − c bx + ay cx + a
D = n!(n+1)!(n+2)! 0 1 2n + 4
0 1 2n + 6 bx + ay −ax + by − c cy + b =0
cx + a cy + b −ax − by + c
Expanding along C1, we get
D = (n!)(n + 1)!(n + 2)![(2n + 6) – (2n +4)] a2 x − aby − ac bx + ay cx + a
1 2
D = (n!)(n + 1)!(n + 2)![2] ⇒ abx + a y −ax + by − c cy + b =0
2 2
acx + a cy + b −ax − by + c
On dividing both side by (n!)3

D (n!)(n!)(n + 1)(n!)(n + 1)(n + 2)2 Applying C1 → C1 + bC2 + cC3


⇒ =
3 3
(n!) (n!)
(a2 + b2 + c2 )x
ay + bx cx + a
D 1 2 2 2
⇒ = 2(n + 1) (n + 1) (n + 2) ⇒ (a + b + c )y by − c − ax b + cy = 0
(n!)3 a 2 2 2
a +b +c b + cy c − ax − by
D
⇒ = 2(n3 + 4n2 + 5n + 2) = 2n (n2 + 4n + 5) + 4 x ay + bx cx + a
(n!)3 1
⇒ y by − c − ax b + cy = 0
D a
⇒ – 4 = 2n(n2 + 4n + 5) 1 b + cy c − ax − by
(n!)3
( a2 + b2 + c2 = 1)
 D 
Which shows that  − 4  is divisible by n. Applying C2 → C2 – bC1
3
 (n!) 
and C3 → C3 – cC1
Sol 9: Given, lx + (sin α)y + (cos α)z = 0 x ay a
x + (cos α) y+(sin α) z = 0 1
⇒ y −c − ax b =0
a
and – x + (sin α) y – (cos α) z = 0 has non-trivial solution. 1 cy −ax − by

∴∆=0
x2 axy ax
λ sin α cos α 1
⇒ y −c − ax b =0
⇒ 1 cos α sin α = 0 ax
1 cy ax − by
−1 sin α − cos α
Applying R1 → R1 + yR2 + R3
⇒ λ (– cos2 α – sin2α) – sin α (– cos α + sin α) + cos α
(sin α + cos α) = 0 x2 + y 2 + 1 0 0
1
⇒ – λ + sin α cos α + sin α cos α – sin2 α + cos2 α = 0 ⇒ y −c − ax b =0
ax
⇒ λ = cos 2α + sin2a 1 cy −ax − by

 − a2 + b2 ≤ asin θ + bcos θ ≤ a2 + b2  1
  ⇒ [(x2 + y2 + 1) {(– c – ax) (– ax – by) – b(cy)}]= 0
  ax
∴– 2 ≤λ≤ 2  … (i) 1
⇒ [(x2 + y2 + 1) (acx + bcy + a2x2 + abxy – bcy)]= 0
Again, when λ =1, cos 2α + sin 2α = 1 ax
1 1 1 1
⇒ cos 2α + sin 2α = ⇒ [(x2 + y2 + 1) (acx + a2x2 + abxy)]= 0
2 2 2 ax
⇒ cos (2α – π/4) = cos π/4 1
⇒ [ax(x2 + y2 + 1) (c + ax + by)]= 0
ax
M a them a ti cs | 16.71

⇒ (x2 + y2 + 1) (ax + by + c) = 0 a α ι 
⇒ ax + by + c = 0  T 
P
= b β m
Which represents a straight line. c γ n 
 

( ( ) )
T
∆ 1 1 − cω − a ω − ω2c + b ( 0 )
Sol 11: (A) = P= 2P + I

∆= 1cω − aω + ω2ac  a α ι   2a 2b 2c  1 0 0 
     
b β m = 2α 2β 2γ  + 0 1 0 
∆ = 1 − ω ( c + a) + ω2ac  c γ n   2ι 2m 2n 0 0 1 
     
c=ω a=ω2 singular
a α ι  2a + 1 2b 2c 
ω2 a =
c= ω singular    
b β m=  2 α 2β + 1 2 γ 
c=
ω a=
ω non singular c γ n   2ι 2m 2n + 1 
  
ω2 a =
c= ω2 singular
=2b =α ,b =α
2 .It is possible when b = α , = 0
for every pair (a, c) there are two possible values of b
Similarly,, c = ι = 0
hence 2 matrices.
m=γ =0
 a1 a2 a3   −1 0 0 
   
Sol 12: Let M = b1 b2 b3  The matrix P is  0 −1 0 
c c  0 0 −1
 1 2 c3   
So , PX = −X
 a1 a2 a3  0   −1 
     
b1 b2 b3  . 1  =  2  Sol 14: (D) P = aij 
c c c3  0   3   
 1 2
Q = bij 
 
⇒ a2 =
−1,b2 =
2 ,c2 =
3
bij = 2i+ j.aij
 a1 a2 a3   1  1
      = 2
b11 2= 3
a11 b21 2= .a21 b31 24 a31
b1 b2 b3  .  −1=
  1  ⇒ a1 − a2= 1
c c2 c3   0   −1 
 1   = 3
b12 2= 4
a12 b22 2= .a22 b32 25 a32
⇒ a1=0, b1=3, c1=3 = 4
b13 2= a13 b23 25=
.a22 b33 26 a33

 a1 a2 a3  1 0 a11 a12 a13


     
b1 b2 b3  . 1=  0  ⇒ c1 + c2 + c3= 12 Given P a=
= 21 a22 a23 2
c c c3  1 12
 1 2   a31 a32 a33

⇒ c3 = 12 − 5 = 7 22 a11 23 a12 24 a13


∴ Sum of diagonal elements = a1 + b2 + c3 = 0 + 2 + 7 = 9 Q = 23 a21 24 a22 25 a23
24 a31 25 a32 26 a33
Sol 13: (D) There seems to be an ambiguity in the
question since 3 × 3 skew-symmetric matrices can’t be a11 a12 a13
non-singular. 2 3
Q = 2 .2 .2 a21 4
a22 a23
[Property: Determinant of an odd order skew- a31 a32 a33
symmetric matrix is always zero]
22 a11 23 a12 24 a13
P is a 3 × 3 matrix Q = 22.23.24 23 a21 24 a22 25 a23

 a b c 24 a31 25 a32 26 a33


 
Let P =α β γ 
 ι m n
 
1 6 . 7 2 | Matrices

a11 a12 a13 


( ) 
2
 ( −ω) + ω ω2s ( −ω) + ωr ω2s 
2r 2s r
2 3 4 2 1
Q = 2 .2 .2 .2 .2 a21 a22 a23 =  = −I (Given)
a31 a32 a33 ω2s ( −ω) + ωr ω2s

r
(
ω2r ωr + ω ) 

13
Q = 22.23.24 .22.21.21 = 2 r s r s
1 1 1 1
Sol 15: (B, C, D) P2 = 0 only when n is multiple of 3. 2 2 3 3
Total no. pairs = 1
ω2 1 ω  ω2 1 ω  0 0 0 
      −1
E.g:  1 ω ω2  .  1 ω ω2  =0 0 0  P  P 
    Sol 19: (B, C)   .Q = I ∴Q=  
 ω ω2 1  ω ω2 1  0 0 0  K
  K 
  
Comparing P23 we get, −K −K ( 3α + 4 )
∴ P2 ≠ 0 when n = 55,56,58 =
8 12α + 20
−K −K ( 3α + 4 ) α = −1
= ⇒
Sol 16: (A, B) MN = NM 8 12α + 20
α = −1
N2M = N (NM) = (NM) N = (MN) N = MN2 Also P Q = K 3

(M − N (M + N )) =M
2 2 2
+ MN2 − N2M − N=
4
M2 − N4
∴ (12a ++ 20)
∴(12a 20 )
K2
=K3
As M − N2 ≠ 0 ⇒ M + N2 =
0 ...2 2
KK == 6a
6α +
+ 10
10 = =44
M2 + MN2 = M M + _ N=
2
(
M M + N2
) Sol 20: (B)
2
= 0 ⇒ M+N =
0  1 0 0  1 0 0  1 0 0
  2  
= P =4 1 0  4 1 0  8 1 0
Sol 17: (D) When roots are purely imaginary. 16 4 1  16 4 1   48 8 1 
    
2
Then the form of equation is x + K =0
 1 0 0  1 0 0  1 0 0
where K is positive no. 2     
= P =8 1 0  4 1 0 12 1 0 
Let p ( x=
) x +K
2
 48 8 1  16 4 1  96 12 1 
    
( ) (p ( x ) ) + K
2
p(x)
p=  
 1 0 0
p (p ( x ) ) = ( x2 + K ) + K
n  4n
2
∴∴P = 1 0
 
 (
8 n2 + n ) 4n 1 

(
p p(x) = )
x 4 + 2Kx2 + K ⇒ p p ( x ) = 0 ( )
 
 1 0 0
x 4 + 2Kx2 + K =0
∴∴P50  200
= 1 0
 
All coefficients are positive and no odd degree of x are
present.  (
8 × n2 + n ) 4n 1 

−1 + i 3 P50 − Q =
I
Sol 18: (A) z = = ω
2 0 q21 =
Equation we get 200 − q21 =⇒ 200
 −ω r ω2s 
p=
( ) 400 × 51 − q31 =
0
 ω2s ωr  q= 400 × 51
 31

 −ω r 200 − q32 =0 ⇒ q32 =200


( ) ω2s  ( −ω) ω2s 
r
2
p =   q31 + q32 400 × 51 + 200
 ω2s

r 
ω   ω2s r 
ω 
q21
=
200
(51) +=1 130
= 103
2017-18 100 &
op kers
Class 12 T
By E ran culty
-JE Fa r
IIT enior emie .
S fP r es
o titut
Ins

MATHEMATICS
FOR JEE MAIN & ADVANCED
SECOND
EDITION

Exhaustive Theory
(Now Revised)

Formula Sheet
9000+ Problems
based on latest JEE pattern

2500 + 1000 (New) Problems


of previous 35 years of
AIEEE (JEE Main) and IIT-JEE (JEE Adv)

5000+Illustrations and Solved Examples


Detailed Solutions
of all problems available

Topic Covered Plancess Concepts


Tips & Tricks, Facts, Notes, Misconceptions,
Probability Key Take Aways, Problem Solving Tactics

PlancEssential
Questions recommended for revision
18. PROBABILITY

1. INTRODUCTION
There are various phenomena in nature leading to an outcome, which cannot be predicted beforehand. For example,
tossing a coin may result into two outcomes- a head or a tail. Probability theory aims to measure the uncertainties
of such outcomes. Consequently, probability is the measure of uncertainty of random experiments.

2. RANDOM EXPERIMENT
An experiment is said to be random if it has more than one possible known outcomes which cannot be predicted
in advance. For example - Throwing of a die is a random experiment.
Sample Space: The set of all possible outcomes of a trial (random experiment) is called its sample space. It is
generally denoted by S and each outcome of the trial is said to be a sample point.
For example - In throwing of a die, the sample space for the number that shows up on the top face would be:
S = {1, 2, 3, 4, 5, 6}
EVENT: Every subset of a sample space is called an event.
For example, in throwing a dice, the sample space
S = {1, 2, 3, 4, 5, 6} and n(S) = 6
E1 = {1, 3, 5} ⊂ S. So E1 is an event and n (E1) = 3.
The event E1 = {1, 3, 5} can also be expressed as the event of getting an odd number in throwing a dice.
(a) Simple event:A simple event or an elementary event is an event containing only a single sample point.
(b) Compound events: Compound events or decomposable events are those events that are obtained by
combining together two or more elementary events.
For instance, the event of drawing a heart from a deck of cards is the subset A = {heart} of the sample space
S = {heart, spade, club, diamond}. Therefore, A is a simple event. The event B of drawing a red card is a
compound event since B = {heart U diamond} = {heart and diamond}.
(c) Mutually exclusive or disjoint events: Events are said to be mutually exclusive or disjoint or incompatible if
the occurrence of any one of them prevents the occurrence of all the other events.
(d) Mutually non-exclusive events: The events which are not mutually exclusive are known as compatible events
or mutually nonexclusive events.
(e) Independent events: Events are said to be independent, if the happening(or non-happening) of one event is
not affected by the happening (or non-happening) of other events.
1 8 . 2 | Probability

(f) Dependent events: Two or more events are said to be dependent, if the happening of one event affects
(partially or totally) the other event.

The relationship between mutually exclusive and independent events


Mutually exclusive events can’t happen at the same time. Mathematically put,
Independent events: P(A and B) = P(A) × P(B)
Mutually exclusive: P(A and B) = 0, where A and B are two events.
→ On comparing two definitions, we see that the events can’t be independent and Mutually exclusive at the
same time.

Equally likely events: Events which have the same chance of occurring are said to be equally likely events.
For example, in the experiment of tossing a coin,
where,
A: The event of getting a “HEAD” and
B: The event of getting a “TAIL”
Events “A” and “B” are said to be equally likely events.
[Both the events have the same chance of occurrence]
In the experiment of throwing a die,
where,
A: The event of getting 1
B: The event of getting 2


F: The event of getting 6
Events “A”, “B”, “C”, “D”, “E” and “F” are said to be equally likely events.
[All these events have the same chance of occurrence.]

3.COMPLEMENT OF EVENTS
The complement of an event ‘A’ with respect to a sample space S is the set of all elements of ‘S’ which are not in A.
It is usually denoted by A’, A or Ac

(a) The union E1∪ E2 of events E1 and E2 is the event of at least one of the events E1, E2 happening.

(b) The intersection E1∩ E2 of events E1 and E2 is the event of both the events E1, E2 happening.
e.g. Tossing of coin sample space s = {H, T}. Event of getting head in tossing of coin A = {H} ⇒ AC =
{T}

PLANCESS CONCEPTS

Terminology: Being closely familiar with the terminology of probability helps a lot in thinking with clearly.
In particular, always think of outcomes as the most elementary results of an experiment, events as a set of
outcomes, the sample space as the set of all possible outcomes and events as subsets of the sample space.

Ravi Vooda (JEE 2009, AIR 71)


M a them a ti cs | 18.3

4. ALGEBRA OF EVENTS
Verbal description of the event Equivalent set theoretic notation
Not A A
A or B (at least one of A or B) A∪B
A and B A∩B
A but not B A∩B
Neither A nor B A∩B
At least one of A, B or C A∪B∪C
Exactly one of A and B (A ∩ B) ∪ (A ∩ B)
All three of A, B and C A∩B∩C
Exactly two of A, B and C (A ∩ B ∩ C) ∪ (A ∩ B ∩ C) ∪ (A ∩ B ∩ C)

5.PROBABILITY
If a random experiment results in n mutually exclusive, equally likely and exhaustive outcomes out of which m are
favorable to the occurrence of an event A, then the probability of occurrence of A is given by
m Number of outcomes favourable to A
P (A) = =
n Number of total outcomes
It is obvious that 0 ≤ m ≤ n. If an event A is certain to happen, then m = n, thus P (A) = 1.
If A is impossible to happen, then m = 0 and so P (A) = 0.
Hence we conclude that 0 £P (A) ≤ 1

PLANCESS CONCEPTS

Working Rule to find probability


Step 1. For the given experiment, find out all possible outcomes n(S) of the sample space.
Step 2. Identify the outcomes n (A), which are favorable to the event A, whose probability is required.
Step 3. Apply the formula to find P (A),
n(A)
P (A) =
n(S)
Equal Likelihood, the formula that we apply to calculate probability,
Number of favorable outcome
, is valid only when all the cases have equal likelihood of occurrence.
Number of total outcome
This important point is overlooked a lot of times. For example, if a rolling die is not fair, then you cannot
1
assign a probability of for each face showing up. Sometimes, the way you count the total cases and
6
favorable cases can lead to a mistake. Consider a random experiment involving the rolling of two dice
simultaneously. Suppose you have to evaluate the probability of getting a total of less than 6. The
following argument has a mistake: “There are a total of 11 possible cases, namely {2,3,4,5,6,7,8,9,10,11,12},
4
out of which 4 are favorable, namely {2,3,4,5}, and thus the required probability is .”
11
1 8 . 4 | Probability

PLANCESS CONCEPTS

The mistake is that, the different cases do not have equal likelihood of occurrence. For example, the
sum 6 is more likely to occur than the sum 2 (why?). The correct way to solve this problem would be to
consider the 36 equally likely outcomes (x, y) where x and y can take integer values from 1 to 6, and then
consider those outcomes from this set of 36 outcomes, which leads to a sum of less than 6. You can
verify that there will be 10 such favorable outcomes. And now, it would be correct to apply the formula
Number of favorable outcome 10
to obtain the required probability as .
Number of total outcome 36
Shrikant Nagori (JEE 2009, AIR 30)

Illustration 1: In a single case with two fair dice, find the chance of getting
(A) Two 4’s (B) A doublet (C) Five-six (D) A sum of 7 (JEE MAIN)

Sol: Write all the possible outcomes and the favorable events in each case.
(A) There are 6 × 6 equally likely cases (as any face of any die may turn up)
⇒ 36 possible outcomes. For this event, only one outcome (4 – 4) is favourable
\ Probability = 1/36.
(B) A doublet can occur in six ways {(1, 1), (2, 2), (3, 3), (4, 4) (5, 5), (6, 6)}.
Therefore, probability of doublet = 6/36 = 1/6.
(C) Two favorable outcomes {(5, 6), (6, 5)}.Therefore, probability = 2/36 = 1/18.
Sample space = (1, 1), (1, 2), (1, 3), (1, 4), (1, 5), (1, 6)
(2, 1), (2, 2), (2, 3), (2, 4), (2, 5), (2, 6)
(3, 1), (3, 2), (3, 3), (3, 4), (3, 5), (3, 6)
(4, 1), (4, 2), (4, 3), (4, 4), (4, 5), (4, 6)
(5, 1), (5, 2), (5, 3), (5, 4), (5, 5), (5, 6)
(6, 1), (6, 2), (6, 3), (6, 4), (6, 5), (6, 6)
(D) A sum of 7 can occur in the following cases {(1, 6), (2, 5), (3, 4), (4, 3), (5, 2), (6, 1)} which are 6 in number.
Therefore, probability = 6/36 = 1/6.

Illustration 2: Seven accidents occur in a week. What is the probability that they take place on the same day?
 (JEE MAIN)
Sol: Find the total number of ways accidents can happen. And clearly, all the accidents can take place on the same
day in 7 ways.
Total no. of cases = Total no. of ways in which 7 accidents can happen in a week (or be distributed= 77
Favorable No. of cases out of these = number of those in which all 7 happen on one day (any the week)= 7
7 1
\ Required probability = =
7
7 76
M a them a ti cs | 18.5

6. MUTUAL INDEPENDENCE AND PAIRWISE INDEPENDENCE


Three events A, B, C are said to be mutually independent if, P(A ∩ B) = P(A).P(B), P(A ∩ C) = P(A).P(C), P(B ∩ C) =
P(B).P(C) and P(A ∩ B ∩ C) = P(A).P(B).P(C)
These events would be said to be pairwise independent if, P(A ∩ B ∩ C) =⇒ P(A ∩ B) = P(A).P(B), P(B ∩ C) =
P(B).P(C) and P(A ∩ C) = P(A).P(C)
Thus, mutually independent events are pairwise independent but the converse may not be true.

Illustration 3: From a bag containing 5 white, 7 red and 4 black balls, a man draws 3 balls at random. Find the
probability of them being all white. (JEE MAIN)
Sol: Use the principle of restricted combination.
Total number of balls in the bag = 5 + 7 + 4 = 16
16 × 15 × 14
Total number of ways in which 3 balls can be drawn is 16C3 = = 560
3× 2×1
Thus, the sample space S for this experiment has 560 outcomes i.e. n(S) = 560
Let E be the event of all the three balls being white. Total number of white balls is 5. So, the number of ways in which
5× 4×3
3 white balls can be drawn = 5C3 = = 10
3× 2×1
Thus, E has 10 element of S, ∴ n(E) = 10

n(E) 10 1
\ Probability of E, P(E) = = =
n(S) 560 56

PLANCESS CONCEPTS

Exhaustive Event: A Set of events is said to be exhaustive if the performance of random experiments
always results in the occurrence of at least one of them. For instance, consider an ordinary pack of cards.
The events ‘drawn card is heart’, drawn card is diamond’, ‘drawn card is club’ and ‘drawn card is spade’ is
a set of events that is exhaustive. In other words all sample points put together (i.e. sample space itself)
would give us an exhaustive event.
If ‘E’ is an exhaustive event then P (E) = 1.
Vaibhav Gupta (JEE 2009, AIR 54)

7. ODDS IN FAVOUR, ODDS AGAINST


P(E)
(a) The odds in favor of the event E = .
P(E')
P(E')
(b) The odds against the event E =
P(E)
a
(c) If odds in favor of the event E = a: b then P(E) =
a+b
b
(d) If odds against the event E = a: b then P(E) =
a+b
1 8 . 6 | Probability

8. A FEW THEOREMS ON PROBABILITY


(a) If A and B are two mutually exclusive events, then P(A ∪ B) = P(A) + P(B).

(b) If A is any event, then P(A’) = 1 – P(A)

(c) If A and B are two events, then P(A ∩ B’) = P(A) – P(A ∩ B) A B
(d) If A and B are two events, then P(A ∪ B) = P(A) + P(B) – P(A ∩ B) 
(e) If A and B are two events, then
P(exactly one of A, B occurs)
= P[(A ∩ B’) ∪ (A’ ∩ B)]= P(A) – P(A ∩ B) + P(B) – P(A ∩ B)
= P(A) + P(B) – 2P(A ∩ B) = P(A ∪ B) – P(A ∩ B)
Also, P(exactly one of A, B occurs)
= P(A ∩ B’) + P(A’ ∩ B) = P(B’) – P(A’ ∩ B’) + P(A’) – P(A’ ∩ B’)= P(A’) + P(B’) – 2P(A’ ∩ B’)
= P(A’ ∪ B’) – P(A’ ∩B’)

(f) If A and B are two events, P(A’ ∪ B’) = 1 – P(A ∩ B) and P(A’ ∩ B’) = 1 – P(A ∪ B)
(g) If A1, A2, …, An are n events, then P(A1∪ A2∪ … ∪ An)
n
= ∑ P(Ai ) - 1 ≤ i∑
< j≤ n
p(Ai ∩ A j ) ++ ∑ P(Ai ∩ A j ∩ Ak) − ... + (–1)n-1 P(A1∩ A2) … ∩ An)
i=1 1≤i< j<k ≤n

(h) If A, B and C are three events, then


P(A ∪ B ∪ C) = P(A) + P(B) + P(C) – P(B ∩ C) – P(C ∩ A) – P(A ∩ B) + (A∩ B ∩ C)
(i) P(at least two of A, B, C occur) = P(B ∩ C) + P(C ∩ A) + P (A ∩ B) –2P(A ∩ B ∩ C)
(ii) P(exactly two of A, B, C occur)=P(B ∩ C) + P(C ∩ A) + P(A ∩ B) – 3P(A ∩ B ∩ C)
(iii) P(exactly one of A, B, C occurs)
= P(A) + P(B) + P(C) – 2P(B ∩ C)–2P (C ∩ A) –2P(A ∩ B) + 3P (A ∩ B ∩ C)
(i) If A1, A2 ,…, An are n events, then
(i) P(A1∪ A2∪ … ∪ An)≤ P(A1) + P(A2) + … + P(An)
(ii) P(A1∩ A2∩ … ∩ An)≥ 1 – P(A’1) – P(A’2) – … –P(A’n)

( j) If A1, A2, …, An are n events, then P(A1∩ A2∩ … ∩ An)³P(A1) + P(A2) + … + P(An) – (n – 1)

(k) If A and B are two events, such that A ⊆ B, then P(A) ≤ P(B)

9. BOOLE’S INEQUALITY
(a) For any two events A and B

(i) P(A ∪ B) = P(A) + P(B) – P(A∩ B)


(ii) \ P(A ∪ B) ≤ P(A) + P(B) { P(A ∩ B) ≥ 0}

(b) In general for any n events A1 A2 … An

(i) P(A1∪ A2∪ … ∪ An)≤ P(A1) + P(A2) + … + P(An)


M a them a ti cs | 18.7

Illustration 4: Let A, B, C be three events. If the probability of the occurrence of one event out of A and B is 1 –a ,
out of B and C is 1 – 2a, out of C and A is 1 – a and that of occurrence ofthree events simultaneously is a2, then prove
that the probability that at least one event out of A, B, C will occur is greater than or equal to 0.5. (JEE ADVANCED)

Sol: Apply Boole’s Inequality.


Probability that exactly one event out of A and B occur is P(A) + P(B)=2P(A ∩ B) and probability that exactly one
event out of B and C occur is P(B) + P(C) – 2P(B ∩ C) and so on.
Now, P(A ∪ B ∪ C) = P(A) + P(B) + P(C) – P(A ∩ B) –P(A ∩ C) –P(B ∩ C) + P(A ∩ B ∩ C)
1
= [P(A)+P(B) –2P(A∩B) + P(B) + P(C)–2P (B∩C) + P(C) + P(A) –2P(A∩C)] + P(A∩B∩C)
2
1 − a + 1 − 2a + 1 − a 3 3 3
⇒ + a2 = a2 – 2a + . Let,a2 – 2a + = y ⇒ a2 – 2a + – y = 0
2 2 2 2
3  1
Since a is real, so 4 – 4  − y  ≥ 0 ⇒ y ≥
 2  2

Illustration 5: From a pack of 52 cards, two cards are drawn at random. Find the probability of the following events:
(A) Both cards are of spade.
(B) One card is of spade and one card is of diamond. (JEE MAIN)

Sol: Use combination to calculate the number of favorable ways and the total number of ways in both cases.
52 × 51
The total number of ways in which 2 cards can be drawn =52C2= = 26 × 51 = 1326
1×2
\ Number of elements in the space S are n(S) = 1326
(A) Let the event that both cards are of spade be denoted by E1.Then,n(E1) = Number of elements in E1 = Number
13 × 12
of ways in which 2 cards can be selected out of 13 cards of spade = 13C2 = = 78.
1×2
n(E1 ) 78 1
\ Probability of E1 = P(E1) = = = .
n(S) 1326 17

(B) Let E2 be the event that one card is of spade and one is of diamond. Then, n(E2) = number of elements in E2 =
number of ways in which one card of spade can be selected out of 13 spade cards and one card of diamond can
be selected out of 13 diamond cards. = 13C1 × 13C1 = 13 × 13 = 169
n(E2 ) 169 13
\P(E2) = = = .
n(S) 1326 102

Illustration 6: Two numbers x and y are chosen at random from the set {1, 2, 3,…, 3n}. Find the probability that
x2 – y2 is divisible by 3. (JEE ADVANCED)

Sol: Divide the above given set in three subsets such that the difference of any two elements in any of these three
sets is divisible by 3. Use this partition of set to find the answer.
x2 – y2 = (x + y) (x – y) and 3 is a prime number.
\ x2 – y2 is divisible by 3 if x + y or x – y is divisible by 3.
Now, {1, 2, 3,…, 3n} = {3, 6, 9,…, 3n}∪ {1, 4, 7,…, 3n – 2}∪ {2, 5, 8,…, 3n – 1}= A ∪ B ∪ C (say).
Clearly, if x, y are selected from A or B or C then x + y or x – y are divisible by 3; and, if x, y are selected one from
B and the other from C then x + y is divisible by 3.
\ The probability of x2 – y2 is divisible by 3
1 8 . 8 | Probability

= Probability of selecting both x, y from A or B or C + probability of selecting x, y one from B and the other from C
n n
C2 C1 × nC1 3n(n − 1) 2n2 3n − 3 2n 5n − 3
= ×3+ = + = + = .
3n
C2 3n
C2 3n(3n − 1) 3n(3n − 1) 3(3n − 1) 3(3n − 1) 3(3n − 1)

10. CONDITIONAL PROBABILITY


The probability of occurrence of an event A, given that B has already occurred is called the conditional probability
of occurrence of A. It is denoted by P(A | B).If the event B has already occurred, then the sample space reduces to B.
Not the outcome favorable to the occurrence of A (given that B has already occurred) are those that are common
to both A and B, that is, those which belong to A ∩ B.
NA ∩B
Thus, P(A | B) =
NB
A
where NA ∩ B is the number of elements in A ∩ B and NB≠ 0 is the number of 

elements in B and N the total number of elements in


B
NA ∩ B / N P(A ∩ B) AB
S. ⇒ P(A | B) = =
NB / N P(B)

 P(B) P(A | B) if P(B) ≠ 0


Hence, P(A ∩ B) = 
P(A) P(B | A) if P(A) ≠ 0

PLANCESS CONCEPTS

Trick to solve conditional probability


The trick is to identify when a probability is a conditional probability (in a word problem)
When dealing with conditional probability, the difference is that, we know for certain that something else
has already happened.
This means that in our definition of probability that says
Number of ways for something to Happen
P(E) =
Total Number of Ways
P(A ∩ B)
P(A/(B) =
P(B)
Consider events A and B.
If A and B are independent events, then P(A/B) = P(A) and P(B/A) = P(B). Therefore, P(A ∩ B) = P(A) P(B)
Also, P(A ∪ B) = P(A) + P(B) – P(A ∩ B)
= P(A) + P(B) – P(A)P(B) = 1 – (1 – P(A) – P(B) + P(A) P(B)]= 1 – [(1 – P( A )) (1 – P( B ))] = 1 – P(A) P(B)

Vaibhav Krishnan (JEE 2009, AIR 22)

Illustration 7: If m is a natural such that m ≤ 5, then the probability that the quadratic equation x2 + mx +
1 m
+ = 0 has real roots is (JEE MAIN)
2 2
(A) 1/5 (B) 2/3 (C) 3/5 (D) 2/5
M a them a ti cs | 18.9

Sol: Apply Discriminant ≥ 0.


1 m
Discriminant D of the quadratic equationx2 + mx + + 0
=
2 2
D = m2 – 4  1 + m  = m2 – 2m – 2 = (m – 1)2– 3
 
2 2
Now, D ≥ 0 Û (m – 1)2≥ 3
This is possible for m = 3, 4 and 5. Also, the total number of ways of choosing m is 5.
\ Probability of the required event = 3/5.

11. PROBABILITY OF AT LEAST ONE OF THE N INDEPENDENT EVENTS


If P1, P2,, … Pn are the probabilities of n independent events A1, A2, A3 … An then the probability that at least one of
these events will happen is 1 – [(1 – P1) (1 –P2) …. (1 – Pn)]
P(A1 ∪ A2∪ A3∪ … ∪ An) = 1 − P(A1 ) P(A2 ) P(A3 ...P(An )

Illustration 8: A mathematics problem is given to three students A, B and C whose chances of solving it are 1/2,
1/3, 1/4 respectively. Then the probability that the problem is solved is (JEE MAIN)

Sol: Apply the principle of probability for independent events.


Obviously, the events of solving the problem by A, B and C are independent. Therefore required probability

 1  1  1  1 2 3 3
= 1 −  1 −   1 −   1 −  =1− . . =
 2  3  4  2 3 4 4

(1) Multiplication theorems on probability


(i) If A and B are two events associated with a random experiment:
then P(A ∩ B) = P(A) .P(B/A), If P(A) ≠ 0 or P(A ∩ B) = P(B). P(A/B), if P(B) ≠ 0.
(ii) Extension of multiplication theorem: If A1, A2,…, An are n events related to a random experiment, then
P(A1∩ A2∩ A3∩ … ∩ An) = P(A1) P(A2 / A1) P(A3 / A1∩ A2)
… P(An /A1∩ A2∩ … ∩ An – 1)where P(Ai/A1∩ A2∩ … ∩ Ai -1) represents the conditional probability of the event Ai, given
that the events A1, A2, …, Ai – 1 have already happened.
(iii) Multiplication theorem for independent events: If A and B are independent events associated with a random
experiment, then P(A ∩ B) = P(A) . P(B) i.e., the probability of simultaneous occurrence of two independent events is
equal to the product of their probabilities. By multiplication theorem, we have P(A ∩ B) = P(A). P(B/A). Since A and
B are independent events, therefore P(B/A) = P(B). Hence, P(A ∩ B) = P(A).P(B).
(iv) Extension of multiplication theorem for independent events: If A1, A2, … An are independent events
associated with a random experiment, then P(A1∩ A2∩ A3∩ … ∩ An) = P(A1) P(A2) … P(An).
By multiplication theorem, we have
P(A1∩ A2∩ A3∩ … ∩ An) = P(A1) P(A2 / A1) P(A3 / A1∩ A2)… P(An / A1∩ A2∩… ∩ An– 1)
Since A1, A2,…,An – 1, An are independent events, therefore

P(A
= 2 / A1 ) P(A 2 ),P(A3 /=
A1 ∩ A2 ) P(A3 ),........,

P(An / A1 ∩ A2 ∩ ....... ∩ An−1 ) =


P(An )

Hence, P(A1 ∩ A2 ∩ ....... ∩ An ) =


P(A1 )P(A2 )......P(An )
1 8 . 1 0 | Probability

Illustration 9: The probability that a married man watches a certain T.V. show is 0.4 and the probability that a
married woman watches the show is 0.5. The probability that a man watches the show, given that his wife does, is
0.7. Find (JEE MAIN)
(A) the probability that married couples watch the show
(B) the probability that a wife watches the show given that her husband does.
(C) the probability that at least one person of a married couple will watch the show.

Sol: Refer to Multiplication theorems on probability.


Let ‘H’ be the event that a married man watches the show and ‘W’ be the probability that a married woman watches
the show,
⇒ P(H) = 0.4, P(W) = 0.5, P(H/W) = 0.7

(A) P(H ∩ W) = P(W).P(H/W) = 0.5 × 0.7 = 0.35

P(H ∩ W) 0.35 7
(B) P(W/H) = = =
P(H) 0.4 8

(C) P(H ∪ W) = P(H) + P(W) – P(H ∩ W) = 0.4 + 0.5 – 0.35 = 0.55

Illustration 10: Consider the sample space ‘S’ representing the adults in a small town who have completed the
requirements for a college degree. They have been categorized according to sex and employment as under: 
 (JEE MAIN)
Employed Unemployed
Male 460 40
Female 140 260
An employed person is selected at random. Find the probability that the chosen person is male.

Sol: Same as previous illustration.


Let M be the event that a man is chosen and E be the event that the chosen one is employed.
460 23
From the concept of reduced sample space we immediately get, P(M/E) = =
600 30
600 2 460 23 23 / 45 23
Also, P(E) = = ; P(E ∩ M) = = ⇒ P(M/E) = =
900 3 900 45 2/3 30

Illustration 11: A bag contains 3 white balls and 2 black balls, another contains 5 white and 3 blackballs. If a bag
is chosen at random and a ball is drawn from it, what is the probability that it is white? (JEE MAIN)

Sol: Consider two cases. Case I – When the ball is chosen from the first bag and Case II – When the ball is chosen
from the second bag.
The probability that the first bag is chosen is 1/2 and the chance of drawing a white ball from it is 3/5.
∴ Chance of choosing the first bag and drawing a white ball is 1/2, 3/5 respectively
Similarly the chance that the second bag is chosen and a white ball is drawn is 1/2, 5/8 respectively
\ The chance of randomly choosing a bag and drawing a white ball is
1 3 1 5
= . + . (Mutually exclusive cases) = 49/80.
2 5 2 8
M a them a ti cs | 18.11

Illustration 12: Find the probability that a year chosen at random has 53 Sundays. (JEE MAIN)

Sol: Divide the solution in two parts, when the year is a leap year and otherwise.
Let P(L) → be the probability that a year chosen at random is a leap year P(L) = 1/4.
∴ P( L ) = 3/4
Let P(S) → be the probability that a year chosen at random has 53 Sundays.
\ P(S) = P(L) . P(S/L) + P( L ). P(S/ L )
Now, P(S/L) is the probability that a leap year has 53 Sundays.
A leap year has 366 days, 52 weeks + the remaining 2 days may be Sunday-Monday, M-T, T-W, W-Th, Th-F, F-Sat
or Sat –Sunday,
Out of the 7 possibilities, 2 are favorable
2 1
\ P(S/L) = . Similarly P(S/ L ) =
7 7
1 2 3 1 5
\ P(S) = . + . =
4 7 4 7 28

Theorem of total probability: If E1, E2,…En are mutually exclusive and exhaustive events such that P(Ei)≠ 0 for each
i and A is an event, then P(A) = P(E1) P(A|E1) + P(E2) P(A|E2) +…+ P(En) P(A|En)

Bayes’ Theorem: If E1, E2,…,En are n mutually exclusive and exhaustive events such that
P(Ei)> 0 (1 ≤ i ≤ n) and A is an event, then for 1 ≤ k ≤ n,
P(Ek )P(A | Ek )
P(Ek|A)=
P(Ei )P(A | Ei ) + P(E2 )P(A | E2 ) + ... + P(En )P(A | En )

The probabilities P(Ej) (1 ≤ j ≤ n) are called ‘a priori probabilities’ and conditional probabilities
P(Ej|A) are known as ‘posteriori probabilities’. Two events are said to be independent if occurrence (non-occurrence)
of one does not affect the probability of occurrence (non occurrence) of the other i.e. P(B|A) = P(B)

Bayes’ Theorem: The probability of event A, given that event B has subsequently occurred, is

P(A).P(B | A)
P(A|B) =
[P(A).P(B | A)] + [P(A).P(B | A)]

This is a direct result from condition probability and theorem of total probability. In general we

P(B | Ai ) × P(Ai )
can write Bayes’ theorem as P(Ai | B) =
P(B | A1 )P(A1 ) + P(B | A2 )P(A2 ) + ... + P(B | An )P(An )

PLANCESS CONCEPTS

Generally, Bayes’ theorem is remembered as a formula, and whenever students encounter an inverse
probability problem, they try to apply that formula without in-depth analysis of that problem. In our
opinion, in any such problem, you should always draw a probability tree corresponding to the situation
described. This will always give you more insight into the problem than a direct application of the
formula and it may even prevent you from obtaining wrong results.
The method of probability tree diagrams has been discussed later.
Nitish Jhawar (JEE 2009, AIR 7)
1 8 . 1 2 | Probability

Illustration 13: One bag contains four white balls and three black balls. The second bag contains three white
balls and five black balls. One ball is drawn from the first bag and placed unseen in the second bag. What is the
probability that a ball now drawn from the second bag is black?  (JEE ADVANCED)

Sol: Use Total Probability theorem. Consider two events


A1 – When white ball is transferred from bag I to II, and
A2 – When black ball is transferred from bag I to II and proceed.
Bag-I Bag-II
4W 3W
3B 5B
Let A1 be the event that a white ball is transferred from bag-I to bag-II and A2 be the event that a black ball is
transferred from bag-I to bag-II.
4 3
P(A1) = , P(A2) =
7 7
Let ‘A’ be the probability that finally a black ball is drawn from the second bag
5 6
P(A/A1) = , P(A/A2) =
9 9
4 5 3 6 38
Now from total probability theorem we get, P(A) = P(A1).P(A/A1) + P(A2).P(A/A2) = . + . =
7 9 7 9 63

Illustration 14: A real estate man has eight master keys to open several new homes. Only one master key will open
any given house. If 40% of these homes are usually left unlocked, what is the probability that the real estate man
can get into a specific home if he selects three master keys at random before leaving the office?
 (JEE ADVANCED)

Sol: Use Total Probability theorem. Let A1 and A2 be the events that the specific home is left unlocked and is left
locked respectively
⇒ P(A1) = 0.4, P(A2) = 0.6
Let ‘A’ be the event that the real estate man get into the specific home P(A/A1) = 1,
7
C2 3 4 18 5
P(A/A2) = = ⇒ P(A) = P(A1) P(A/A1) + P(A2) P(A/A2) = (0.4)(1) + (0.6) (3/8) = + = .
8
C3 8 10 80 8

Illustration 15: A bag ‘A’ contains 2 white balls and 3 red balls, a bag ‘B’ contains 4 white and 5 blackballs. A bag
is selected at random and a ball is drawn from it. Drawn ball is observed to be white. Find the probability that bag
‘B’ was selected.  (JEE ADVANCED)

Sol: Take two cases, when bag A is selected and another when bag B is selected.
Bag A Bag B
2W, 3R 4W, 5B
Let A1 be the event that bag ‘A’ is selected and A be the event that bag B is selected
P(A1) = P(A2) = 1/2
Let ‘A’ be the event that a white ball is drawn from the selected bag.
4
⇒ P(A/A1) = 2/5, P(A/A2) =
9
1 2 1 4 1  2 4  38
P(A) = P(A1) . P(A/A1) + P(A2) .P(A/A2)= . + . =  + =
2 5 2 9 2  5 9  90
M a them a ti cs | 18.13

P(A2 ).P(A / A2 ) (1 / 2).(4 / 9) 90 × 4 10


= Finally, P(A2/A) = = = =
P(A) (38 / 90) 18 × 38 19

Illustration 16: A card from a pack of 52 cards is lost. From the remaining cards, two cards are drawn and are found
to be spades. Find the probability that the missing card is also a spade. (JEE ADVANCED)

Sol: Take two cases, when the missing card is a spade or a non-spade. Let A1 be the event that missing card is spade
and A2 be event that missing card is non-spade.
1 3
⇒ P(A1) = .P(A2) =
4 4
Let ‘A’ be the event that 2 spade cards are drawn from the remaining cards,

 A  12 C2  A  13
C2  A   A 
P   = and P   = ; P(A) = P(A ) . P   + P(A2 ).P  
 A1   A2   A1  A2 
51 51 1
C2 C2 
12 13
1 C2 3 C2 1  12
= + = C2 + 3. 13C2 
4 51
C2 4 51 51
C2 4. C2  

12
 A  1 C2
P(A1 )P  
 A1   A1  4 51
C2 11
Now,
= P   = =
 A  P(A) 1  12 C + 3.13 C  50
4.51 C2 
2 2

12. BINOMIAL DISTRIBUTIONS


Binomial distributions occur in relation to those experiments that are binary in nature, i.e. whose outcomes can be
grouped into two classes, say, success and failure, or, say 1 and 0. For example, when you toss a coin, there are only
two outcomes possible: Heads (which you may call success) and Tails (which then becomes Failure).

PLANCESS CONCEPTS

Note that an experiment need not have only two outcomes for it to be called binary. For example,
consider the experiment of rolling a die.If youmake the following definitions-
Success: Numbers 1, 2 or 3
Failure: Numbers 4, 5 and 6
Then, with respect to this definition, the experiment is binary. Thus, an experiment needs to have two
classes of outcomes for it to be called binary.
Let us consider a binomial experiment which has been repeated ‘n’ times. Let the probability of success
and failure in any trial be p and q respectively. We are interested in the probability of occurrence of
exactly ‘r’ successes in these n trials. Now, number of ways of choosing ‘r’ success in ‘n’ trials = nCr.
Probability of ‘r’ successes and (n-r) failures is pr.qn-r. Thus probability of having exactly r successes = nCr .
pr.qn–r
Let ‘X’ be a random variable representing the number of successes, then
P(X = r) = nCr . pr.qn–r (r = 0, 1, 2, …, n)
1 = (p + q)n = nC0 p0qn + nC1 p1 qn–1 + nC2 p2qn – 2 + … + nCr pr qn-r + … + nCr pnq0
X → Number of successes 0, 1, 2, ………. r, ……. N
1 8 . 1 4 | Probability

PLANCESS CONCEPTS

r
• Probability of at most ‘r’ successes in n trials = ∑ n Crpr qn−r
r =0
n
• Probability of at least ‘r’ successes in n trials = ∑ n Crpr .qn−r
r =r

• Probability of having 1st success at the rth trial = p.qr–1


Shivam Aggarwal (JEE 2009, AIR 27)

13. BINOMIAL PROBABILITY DISTRIBUTION


(a) A probability distribution spells out how a total probability is distributed over several values of a random
variable.
∑ pixi = p x
(b) Mean of any probability distribution of a random variable is given by µ = ∑ ii (Since Spi = 1)
∑ pi
(c) Variance of a random variable is given by, s = Σ(xi – µ) pi
2 2

∴ s2 = Spixi2 – µ2 (Note that SD = + σ2

(d) The probability distribution for a binomial variable ‘X’ is given by P(X = r) = nCr pr qn – r where p(X = r) is the
probability of r successes.
P(r + 1) n − r p
The recurrence formula = . is very helpful for computing P(1). P(2) . P(3) etc. quickly, if P(0) is
known. P(r) r +1 q

(e) Mean of BPD = np ; Variance of BPD = npq.


(f) If P represents a person’s chance of success in any venture and ‘M’ represents the sum of only what he will
receive in case of success, then his expectations of probable value = PM.

14. MODE AND MEDIAN


Usually the mode of a binomial B(n, p) distribution is equal to (n + 1)p  , where[.] is the greater integer function.
However when (n + 1)p is an integer and p is neither 0 nor 1, then the distribution has two modes: (n + 1)p and
(n + 1)p − 1. When p is equal to 0 or 1, the mode will be 0 and n correspondingly. These cases can be summarized
as follows:
 (n + 1)p  if (n + 1)p is 0 or a non integer,

mode = (n + 1)p and (n + 1)p − 1 if (n + 1)p ∈ {1,...,n},
 n if (n + 1)p = n +1

In general, there is no single formula to find the median for a binomial distribution and it may even be non-unique.
However several special results have been established:
(a) If np is an integer, then the mean, median, and mode coincide and equal np.
1
(b) When p = 1/2 and n is odd, any number m in the interval (n − 1) ≤ m ≤ 1/2(n + 1) is a median of the binomial
2
distribution. If p = 1/2 and n is even, then m = n/2 is the unique median.
M a them a ti cs | 18.15

PROBLEM SOLVING TACTICS

Following are some extra methods which may be useful to solve probability questions:

Venn Diagrams: It is a diagram in which the sample space is represented by a rectangle and the element of the
sample space by points within it. Subsets (or events) of the sample space are represented by the region within the
rectangle, usually using circles.
For example, consider the following events when a die is thrown,
A = {odd numbers} = {1, 3, 5}
A C B
B = {even numbers} = {2, 4, 6}
C = {prime numbers} = {2, 3, 5} 3 4
1 2
Let us see how Venn diagrams are to be applied by using them to prove 5
6
some results as follows:

Theorem 1: For any two events A and B, A ⊆ B ⇒ P(A) ≤ P(B).


Proof. From the adjoining diagram, we have S

A ∪ (B – A) = B and A ∩ (B – A) = f (B - A)
\P(B) = P[A ∪ (B – A)] [ A ∩ (B – A) = f]
⇒ P(B) = P(A) + P(B – A) [ P(B – A) ≥ 0] A
⇒ P(A) ≤ P(B) Proved

B
Theorem 2: For any two events A and B, P(A – B) = P(A) – P (A ∩ B)
Proof: Let A and B be two compatible events. Then A ∩ B ≠ φ. From the adjoining
Venn diagram. it is clear that:
(A – B) ∩ (A ∩ B) = fand (A – B) ∪ (A ∩ B) = A A B S
⇒ P ( A − B ) + P(A ∩ B) =
P(A)
⇒ P(A − B)= P(A) − P(A ∩ B) Proved
A-B
Remarks: This result may be expressed as
P(A ∩ B)= P(A) − P(A ∩ B)

Also P(A ∩ B)= P(B) − P(A ∩ B)

Theorem 3: For any three events A, B, C A


P(A ∪B ∪ C) = P(A) + P(B) + P(C) – P(A ∩ B) –P (B ∩ C) –P(C ∩ A) + P(A ∩ B ∩ C) AB AC
Proof: We have ,P(A ∪ B ∪ C) = P[(A ∪ Β) ∪ C] ABC
= P(A ∪ B) + P(C) –P [(A ∪ B) ∩ C] B
BC C
= P(A ∪ B) + P(C) –P[(A ∩ C) ∪ (B ∩ C)] (Distributive Law)
= [P(A) + P(B) – P(A ∩ B)] + P(C)– P[(A ∩ B) ∪ (C ∩ C)] [Addition law]
= P(A) + P(B) –P(A ∩ B) + P(C) –P[(A ∩ C)∪ (B ∩ C)]
= P(A) + P(B) + P(C) –P(A ∩ B) – [P(A ∩ C) + P(B ∩ C) –P[(A ∩ C) ∩ (B ∩ C)]
= P(A) + P(B) + P(C) –P(A ∩ B) –P(A ∩ C) + P(B ∩ C) + P[A ∩B∩ C] Proved
1 8 . 1 6 | Probability

Probability Tree Diagrams: Calculating probabilities can be hard. Sometimes you add them, sometimes you
multiply them and often, it is hard to figure out what to do. That’s when tree diagrams come to the rescue!
Here is a tree diagram for two tosses of a coin:

0.5 Head Head, Head


0.5 Head
0.5 Tail Head, Tail
O
0.5 0.5 Head Tail, Head
Tail
0.5 Tail
Tail, Tail

How do you calculate the overall probabilities?

0.5 Head Head, Head 0.5x0.5=0.25


0.5 Head
0.5 Tail Head, Tail 0.5x0.5=0.25
0.5 Head Tail, Head Add
0.5 Tail 0.5x0.5=0.25
0.5 Tail
Tail, Tail 0.5x0.5=0.25
1.00

So, there you go. When in doubt, draw a tree diagram, multiply along the branches and add the columns. Make
sure all probabilities add to 1 and you are good to go!

FORMULAE SHEET

(a) Mathematical definition of probability:


Number of favorable cases to event A
Probability of an event =
Total Number of cases
Note: (i) 0 ≤ P(A) ≤ 1
(ii) Probability of an impossible event is zero
(iii) Probability of a sure event is one.
(iv) P(A) + P(Not A) = 1 i.e. P(A) + P(A) = 1

m n−m
(b) Odd for an event: If P(A) = and P(A) =
n n
P(A) m p(A) n − m
Then odds in favor of A = = and odd in against of A = =
P(A) n − m P(A) m

(c) Set theoretical notation of probability and some important results:

(i) P(A ∪B) = 1 – P(A ∩ B)

P(A ∩ B)
(ii) P(A/B) =
P(B)
M a them a ti cs | 18.17

(iii) P(A ∪ B) = P(A∩ B) + P(A ∩ B) + P(A ∩ B)

(iv) A ⊆ B ⇒ P(A) ≤ P(B)

(v) P(A ∩ B)= P(B) − P(A ∩ B)

(vi) P(A∩ B) ≤ P(A) P(B) ≤ P(A ∪ B) ≤ P(A) + P(B)

(vii) P(Exactly one event) = P(A ∩ B) + P(A ∩ B)

(viii) P(A ∪ B) = 1 – P(A∩ B) = P(A) + P(B) – 2P (A∩B) = P(A + B) – P(A∩ B)

(ix) P(neither A nor B) = P(A ∩ B) = 1 – P(A ∪ B)


1
(x) When a coin is tossed n times or n coins are tossed once, the probability of each simple event is
2n
1
(xi) When a dice is rolled n times or n dice are rolled once, the probability of each simple event is
6n
(xii) When n cards are drawn (1 ≤ n ≤ 52) from well shuffled deck of 52 cards, the probability of each simple
1
event is 52
.
Cn
(xiii) If n cards are drawn one after the other with replacement, the probability of each simple event is 1
(52)n
(xiv) P(none) = 1 – P (at least one)

(xv) Playing cards


•• Total cards: 52 (26 red, 26 black)
•• Four suits: Heart, diamond, spade, club (13 cards each)
•• Court (face) cards: 12 (4 kings, 4 queens, 4 jacks)
•• Honor cards: 16 (4 Aces, 4 kings, 4 queens, 4 Jacks)

(xvi) Probability regarding n letters and their envelopes:


If n letters corresponding to n envelopes are placed in the envelopes at random, then
1
•• Probability that all letters are in the right envelopes =
n!
1
•• Probability that all letters are not in the right envelopes = 1 -
n!
1 1 1 1
•• Probability that no letter is in the right envelope = – + + ..... + (–1)n
2! 3! 4! n!

Probability that r letters are in the right envelope = 1  1 − 1 + 1 + ... + ( −1)n−r 1 


 
••
r!  2! 3! 4! (n − r)! 

(d) Addition Theorem of Probability:


(i) When events are mutually exclusive
i.e. n(A ∩ B) = 0 ⇒ P(A ∩ B) = 0
\ P(A ∪ B) = P(A) + P(B)
(ii) When events are not mutually exclusive i.e. P(A ∩ B) ≠ 0
\ P(A ∪ B) = P(A) + P(B) – P(A ∩ B) or P(A + B) = P(A) + P(B) – P(AB)
(iii) When events are independent i.e. P(A ∩ B) = P(A) P(B)
\ P(A + B) = P(A) + P(B) – P(A) P(B)
1 8 . 1 8 | Probability

(e) Conditional probability:


P(A ∩ B)
P(A/B) = Probability of occurrence of A, given that B has already happened =
P(B)

P(A ∩ B)
P(B/A) = Probability of occurrence of B, given that A has already happened =
P(A)
Note: If the outcomes of the experiment are equally likely, then

Number of sample points in A ∩ B


P(A/B) =
Number of points in B
(i) If A and B are independent events, then P(A/B) = P(A) and P(B/A) = P(B)
(ii) Multiplication Theorem:
P(A ∩ B) = P(A/B). P(B), P(B) ≠ 0 or P(A ∩ B) = P(B/A) P(A), P(A) ≠ 0
Generalized: P(E1∩ E2∩ E3∩ … ∩ En)
= P(E1) P(E2/E1) P(E3/E1∩ E2) P(E4/E1∩ E2∩ E3) … If events are independent, then
P(E1∩ E2∩ E3 … ∩ En) = P(E1) P(E2) … P(En)

(f) Probability of at least one of the n Independent events: If P1, P2, … Pn are the probabilities of n independent
events A1, A2, … An then the probability that at least one of these events will happen is 1 – [(1 – P1) (1 – P2) …
(1 – Pn)]
or P(A1 + A2 + … + An) = 1 – P (A1 ) P(A2 ) ... P(An )

(g) Total probability: Let A1, A2, … An be n mutually exclusive & set of exhaustive events. If event A can occur
through any one of these events, then the probability of occurrence of A
n

P(A) = P(A ∩ A1) + P(A ∩ A2) + … + P(A ∩ An) = ∑ P(Ar )P(A / Ar )


r =1

(h) Bayes’ Rule: Let A1, A2, A3 be any three mutually exclusive & exhaustive events (i.e. A1∪ A2∪ A3 = sample space
& A1∩ A2∩ A3 = φ) of a sample space S and B is any other event on sample space then,

P(B / Ai )(P(Ai )
P(Ai/B) = , i = 1, 2, 3
P(B / A1 ) P(A1 ) + P(B / A2 )P(A2 ) + P(B / A3 )P(A3 )

(i) Probability distribution:


(i) If a random variable x assumes values x1, x2, … xn with probabilities P1, P2, … Pn respectively then
•• P1 + P2 + P3 + … + Pn = 1
•• Mean E(x) = SPixi
•• Variance = ∑x2Pi – (mean)2 = ∑ (x2) – (E(x))2

(ii) Binomial distribution: If an experiment is repeated n times, the successive trials being independent of one
n
another, then the probability of r success is nCr Pr qn-rat least r success is ∑ n CkPk qn–k where p is probability
of success in a single trial, q = 1 – p k =r

•• Mean E(x) = np
•• E(x2) = npq + n2p2
•• Variance E(x2) – (E(x))2 = npq
•• Standard deviation = npq
M a them a ti cs | 18.19

(j) Truth of the statement:


(i) If two persons A and B speak the truth with probabilities P1& P2 respectively and if they agree on a
PP
1 2
statement, then the probability that they are speaking the truth will be given by .
PP
1 2 + (1 − P1 )(1 − P2 )

(ii) If A and B both assert that an event has occurred, the probability of occurrence of which is α, then the
αPP
1 2
probability that the event has occurred given that the probability of A & B
αPP
1 2 + (1 − α )(1 − P1 )(1 − P2 )
speaking truth is p1,p2 respectively.

(iii) If in the second part, the probability that their lies coincide is β, then from the above case, the required
αPP
1 2
probability will be
αPP
1 2 + (1 − α )(1 − P1 )(1 − P2 )β

Solved Examples

JEE Main/Boards not together is equal to the number of ways of choosing


3 places marked with X out of eight places
Example 1: If there are two events A and B such that XWXWXWXWXWXWXWX
P(A’) = 0.3 P(B) = 0.5 and P(A ∩ B) = 0.3, then P(B|A ∪
This can be done in 8C3 ways. Thus, probability of the
B’) is: 8
C3 8×7×6 7
(A) 3/8 (B) 2/3 (C) 5/6 (D) 1/4 required event=
is =
10
C3 10 × 9 × 8 15
Sol: Use set theory and probability of complimentary Example 3: A group of 2n boys and 2n girls is randomly
events to calculate P(A ∪ B’) divided into two equal groups. The probability that
We have P(A ∪ B’) catch contains the same number of boys and girls is:

= P(A) + P(B’) – P(A ∩ B’) (A) 1/2 (B) 1/n

= [1 – P(A’)] + [1 – P(B)] – [P(A) – P(A ∩ B)] (C) 1/2n (D) None of these

= (1 – 0.3) + (1 – 0.5) – (0.7 – 0.3) = 0.8 Sol: If one group is selected the second group
P[B ∩ (A ∪ B')] automatically gets created. Hence, select n boys and n
Now, P(B|A ∪ B’) = girls from the given group.
P(A ∪ B')
P[(B ∩ A) ∪ (B ∩ B')] P(A ∩ B) 0.3 3 Total number of ways of choosing a group is 4nC2n The
= = = = number of ways in which each group contains equal
P(A ∪ B') P(A ∪ B') 0.8 8
number of boys and girls is (2nCn) (2nCn)
Example 2: Seven white balls and three black balls are (2n Cn )2
∴ Required probability = .
randomly placed in a row. The probability that no two 4n
C2n
black balls are placed adjacently equals:
(A) 1/2 (B) 7/15 (C) 2/15 (D) 1/3 Example 4: Let A and B be two events such that
P(A) = 0.3 and P(A ∪ B) = 0.8. If A and B are independent
Sol: Each black balls can be arranged in between any events, then P(B) is:
two white balls. Us e this idea to find the number of (A) 3/7 (B) 4/7 (C) 5/7 (D) 6/7
ways in which no two black balls are together.
The number of ways of placing 3 black balls at 10 places Sol: If say A and B are two independent events then
is 10C3. The number of ways in which two black balls are P(A ∩ B) = P(A) × P(B)
1 8 . 2 0 | Probability

We have 0.8 = P(A ∪ B) 1


⇒(1 – P(E) (1 – P(F)) =
= P(A) + P(B) – P(A ∩ B) = P(A) + P(B) – P(A) P(B) 2
1
[ A and B are independent] ⇒ 1 – (P(E) + P(F) - P(E) P(F)) =
2
= 0.3 + P(B) – (0.3) P(B) 1 1 7
⇒ P(E) + P(F) = 1 + − =
5 12 2 12
⇒ 0.5 = (0.7) P(B) ⇒ P(B) = ∴Equation whose roots are P(E) and P(F) is
7
x2 – (P(E) + P(F))x + P(E) P(F) = 0
Example 5: A natural number x is chosen at random
from the first one hundred natural numbers. 1 1
or x2 – x+
12 12
(1 − 20)(x − 40)
The probability that < 0 is:
x − 30 ⇒ 12x2 – 7x + 1 = 0

(A) 1/50 (B) 3/50 (C) 3/25 (D) 7/25 ⇒(3x – 1)(4x – 1) = 0

Sol: Find the range of values the variable x can take and 1 1
⇒x= ,
then find the required probability. 3 4
1 1
(x – 20)(x – 40) (x – 20)(x – 30)(x – 40) As P(E) < P(F), we take P(E) = and P(F) =
Let E = = 4 3
x – 30 (x – 30)2
Sign of E is same as that of sign of1 Example 7: Fifteen coupons are numbered 1, 2, ……. ,
(x – 20) (x – 30) (x – 40) = F(say) 15 respectively. Seven coupons are selected at random
one at a time with replacement. The probability that the
Note that F < 0 if and only if
largest number on a selected coupon as 9 is:
0 < x < 20 or 30 < x < 40 6 7
 9   8 
∴ E < 0 in (0, 20) ∪ (30, 40) (A)   (B)  
 15   15 
Thus E is negative for x = 1, 2, ……. , 19, 31, 32, ……, 39 7
that is E, < 0 for 28 natural numbers 3
(C)   (D) None of these
5
28 7
\ Required probability = =
100 25
Sol: Calculate the probability for getting highest
number as 9 and 8. Subtract the two to get the desired
Example 6: Let E and F be two independent events
probability.
such that P(E) < P(F). The probability that both E and F
Let p = the probability that a selected
1
happen is and the probability that neither E nor F
12 coupon bears number ≤ 9.
1
happen is . Then, 9 3
2 p
⇒= = and
15 5
1 2
(A) P(E) = 1/3, P(F) = (B) P(E) = 1/2, P(F) = n = Number of coupons drawn with replacement
2 3
X = The number of coupons bearing number ≤ 9
3 1
(C) P(E) = 2/3, P(F) = (D) P(E) = 1/4, P(F) = Note that X – B (n, p)
4 3
Probability that the largest number on the
Sol: Use the concept of Probability for independent selected coupons does not exceed 9
events.
1 1 = probability that all the coupons bear
We are given P(E ∩ F) = and P(E’ ∩F’) =
12 2 number ≤ 9 7
As E and F are independent, we get P(E) P(F) 3
= P(X = 7) = 7C7 p7 =  
5
1 1
= and P(E’) P(F’) =
12 2
M a them a ti cs | 18.21

Similarly, probability that largest number on We have, P(A) = P(choosing 3 and two other numbers
7
 8  7
C2 7×6 3× 2 7
the selected coupon is ≤ 8 is   . from 4 to 10)= = × =
 15  10
C3 2 10 × 9 × 8 40
7 7
3  8 
Hence, probability of the required event =   –   . P(B) = P(Choosing 7 and two other numbers
 5   15 
6
C2 6 ×5 3× 2 1
from 1 to 6)= = × =
Example 8: A four digit number (numbered from 10
C3 2 10 × 9 × 8 8
0000 to 9999) is said to be lucky if the sum of its first
two digits is equal to the sum of its last two digits. If P(A ∩ B) = P (choosing 3 and 7 and one other
a four digit number is picked up at random, then the
3 3×3× 2 1
probability that it is lucky is: =
number from 4 to 6)= =
10
C3 10 × 9 × 8 40
(A) 0.065 (B) 0.064 (C) 0.066 (D) 0.067
7 1 1 11
Now, P(A ∪ B) = P(A) + P(B) – P(A ∩ B) = + – =
40 8 40 40
Sol: The sum of the first two digits can be any number
from 0 to 18. Use the formula for the number of non- Example 10: A signal which can be green or red with
negative integral solutions of x+y=m to proceed further.
4 1
probability and respectively, is received by station
The total number of ways of choosing the ticket is 5 5
10000. A and then transmitted to station B. The probability of
Let the four digits number on the ticket be x1 3
each station receiving the signal correctly is . If the
x2 x3 x4. Note that 0 ≤ x1 + x2≤ 18 and 0 ≤ x3 + x4≤ 18. 4
signal received at station B is green, then the probability
Also, the number of non-negative integral
that the original signal was green is:
solutions of x + y = m (with 0 ≤ x, y ≤ 9) is
3 6 20 9
m + 1 if 0 ≤ m ≤ 9 and is 19 – m if 10 ≤ m £18. (A) (B) (C) (D)
5 7 23 20
Thus, the number of favorable ways
Sol: Draw a tree diagram for all the possibilities and
= 1 × 1 + 2 × 2 + ………. + 10 × 10 + 9 × 9 + calculate the probability for all the different cases.
8 × 8 + ……… + 1 × 1
S
 9 × 10 × 19 
= 2  + 100 = 670
 6  4/5 1/5
670
\ Probability of required event = = 0.067
10000
G G
Example 9: Three numbers are chosen at random 3/4 1/4 1/4
3/4
without replacement from {1, 2, 3, …… ..10).The
probability that minimum of the chosen number is 3 or
their maximum is 7, is: AG AR AR AG
1/4 1/4
11 11 1 1 3/4 3/4 3/4
(A) (B) (C) (D) 3/4 1/4
30 40 7 8 1/4
BG BR BG BR B
Sol: Find the probability for getting 3 as the minimum
and 7 as the maximum number among the three Let G, E1, E2 and E denote the following events:
numbers selected. Then use the formula P(A ∪ B) =
G: Original signal is green
P(A) + P(B) – P(A ∩ B)
E1: A receives the signal correctly
Let A and B denote the following events
E2:B receives the signal correctly
A: minimum of the chosen number is 3
E = B receives the green signal
B: maximum of the chosen number is 7
We have,
1 8 . 2 2 | Probability

E = GE1E2∩ GE’1E’2∩ G’E1E’2∩ G’E’1E2 Example 2: A bag contains some white and some black
balls, all combinations of balls being equally likely. The
⇒ P(E) = P(GE1E2) + P(G’E’1E’2) + P(G’E1E’2) + P(G’E’1E2)
total number of balls in the bag is 10. If three balls are
drawn at random without replacement and all of them
=  4  3  3  +  4  1  1  +
       
  4  4 
5   4  4 
5 are found to be black, the probability that the bag
contains 1 white and 9 black balls is:
 1  3  1   1  1  3 
    +     14 12 2 8
 5  4  4   5  4  4  (A) (B) (C) (D)
55 55 11 55
36 + 4 + 3 + 3 23
= =
80 40 Sol: In this case, the number of black balls can be
40 1 anything between 3 and 10. Apply Baye’s theorem to
Also, P(G ∩ E) = P(GE1E2) + P(GE’1E’2) = = find the required probability.
80 2
P(G ∩ E) 1/2 20 Let Ei denote the event that the bag contains i black
\P(G/E) = = = and (10 – i) white balls (i =0, 1, 2, ..., 10). Let A denote
P(E) 23 / 40 23
the event that the three balls drawn at random from the
bag are black. We have,
1
JEE Advanced/Boards P(Ei) = (i = 0, 1, 2, … , 10)
11
Example 1:Let A, B, C, be three mutually independent P(A|Ei) = 0 for i = 0, 1, 2
events. Consider the two statement S1 and S2 i
C3
and P(A|Ei) = for i ≥ 3
10
S1: A and B ∪ C are independent C3
S2: A and B ∩ C are independent Now, by the total probability rule,
10
Then, P(A) = ∑ P(Ei )P(A | Ei )
i=0
(A) Both S1 and S2 are true (B) Only S1 is true
1 1
= × [3C3 + 4C3 + … + 10C3]
(C) Only S2 is true (D) Neither S1 nor S2 is true 11 10
C3
Sol: Use the basic understanding of sets and probability But 3C3 + 4C3 + 5C3 + … +10C3
of union and intersection of two sets to find the answer. = 4C4 + 4C3 + 5C3 + … + 10C3
We are given that = 5C4 + 5C3 + 6C3 + …+ 10C3
P(A ∩ B) = P(A) P(B) = 6C4 + 6C3 + … + 10C3 = … = 11C4
11
P(B ∩ C) = P(B) P(C), P(C ∩ A) = P(C) P(A), C4 1
Thus, P(A) = =
and P(A ∩ B ∩ C) = P(A) P(B) P(C) 11 × C3 10 4
By the Bayes’ rule
We have 9
1 ( C3 )
P(A ∩ (B ∩ C)) = P(A ∩ B ∩ C) P(E9 )P(A | E9 ) 11 10 C3 14
P(E9|A) = = =
= P(A) P(B) P(C) = P(A) P(B ∩ C) P(A) 1 55
4
⇒A and B ∩ C are independent. Therefore, S2 is true.
Example 3:A pair of biased dice is rolled together till a
Also P[(A ∩ (B ∪ C)] = P[(A ∩ B) ∪ (A ∩ C)] sum of either 5 or 7 is obtained. The probability that 5
= P(A ∩ B) + P(A ∩ C) – P[(A ∩ B) ∩ (A ∩ C)] comes before 7 is:
= P(A ∩ B) + P(A ∩ C) – P(A ∩ B ∩ C) (A) 2/5 (B) 3/5
= P(A) P(B) + P(A) P(C) – P(A) P(B) P(C) (C) 4/5 (D) None of these
= P(A) [P(B) + P(C) – P(B) P(C)] Sol: The possible outcomes could be 5, X5, XX5, XXX5,
∴A and B ∪ C are independent. XXXX5 and so on, where X denotes a sum of neither
5 nor 7. Also it can be easily understood that this
sequence goes on till infinity.
M a them a ti cs | 18.23

Let A denote the event that a sum of 5 occurs, B denote Example 5: A ship is fitted with three engines E1, E2 and
the event that a sum of 7 occurs and C the event that E3. The engines function independently of each other
neither a sum of 5 nor a sum of 7 occurs. We have, 1 1 1
with respective probabilities , and . For the ship
4 1 2 4 4
P(A) = =
36 9 to be operative at least two of its engine must function.
Let X denote the event that ship is operational and let
6 1
P(B) = = X1, X2 and X3 respectively the events that the engines E1,
36 6
E2 and E3 are functioning. Let,
26 13
and P(C) = = 3
36 18 (A) P(X'1 / X) =
8
Thus,
7
P(A occurs before B) = P[A or (C ∩ A) or (C ∩ C ∩ A) (B) P(X/X2) =
8
or …….]
7
(C) P(Exactly two engines are functioning) =
= P(A) + P(C ∩ A) + P(C ∩ C ∩ A) + ….. 8
= P(A) + P(C) P(A) + P(C)2 P(A) + ….. (D) P(X/X1) = 7/16
2
1  13  1  13  1 Sol: The ship can be operational in four possible cases.
= +   +   + ........
9  18  9  18  9 Calculate the probability of the ship being operational
and then proceed accordingly.
1/9 2 ' ' '
= = [sum of an infinite G.P.] We have, X= (X1 X 2 X3 ) ∪ (X1 X 2 X3 ) ∪ (X1 X 2 X3 ) ∪ (X1 X 2 X3 )
13 5
1 – and X '1 ∩ X =X '1 X 2 X3
18
P(X '1 ∩ X) P(X '1 X 2 X3 )
Now,=
P(X '1 / X) =
Example 4: If A, B and C are three events such that P(B) P(X) P(X)
3 1 1 We have P(X '1 X 2 X3 ) = P(X '1 )P(X 2 )P(X3 )
= , P(A ∩ B ∩ C’) = and P(A’ ∩ B ∩C’) = , then
4 3 3
 1  1  1  1
P(B ∩ C) is equal to:     =
 2   4   4  32
1 1 1 1
(A) (B) (C) (D)
and P(X) =  1   1   3  +  1   3   1  +
       
12 6 15 9
 2  4  4   2  4  4 
Sol: Apply the knowledge of set theory to write B ∩ C’
 1  1  1   1  1  1  1
in terms of A ∩ B ∩ C’ and A’ ∩ B ∩C’.     +     =
 2  4  4   2  4  4  4
We have, P(B ∩ C’) = P[(A ∪ A’) ∩ (B ∩ C’)]
1
1 1 2 ∴ P(X'1 / X) =
= P(A ∩ B ∩ C’) + P(A’ ∩ B ∩ C’)= + = 8
3 3 3
'
Next, X ∩ X2 = X − X1 X 2 X3
B∩C’ 5
P(X1 x'2 X3 )
P(X ∩ X 2 ) P(X) –=
=
32
P(X ∩ X 2 ) 5 / 32 5
∴ P(X =
/ X2 ) = =
P(X 2 ) 1/4 8

Example 6: A fair coin is tossed 100 times. The


probability of getting tails 1, 3 … …49 times is:

C 1 1 1 1
B (A) (B) (C) (D)
2 4 8 16
B∩C
3 2 1 Sol: Let p = probability of getting a tail in a single trial
Now, P(B ∩ C) = P(B) – P(B ∩ C’) = – =
4 3 12 1
= and
2
1 8 . 2 4 | Probability

n = number of trials = 100 – 2P(C ∩ A) + 3P(A ∩ B ∩ C)


X = Number of trials in 100 trials = P(A) + P(B) + P(C) – P(A ∩ B) – P(C ∩ A)
We have, P(X = r) = 100
Cr p qr n–r
– [P(A ∩ B) + P(B ∩ C) + P(C ∩ A) – 3P(A ∩ B ∩ C)]
r 100–r 100 = P(A) + P(B) + P(C) – P(A ∩ B) – P(B ∩ C)
1 1 1
= 100
Cr     = 100
Cr   P(C ∩ A) – P (exactly two of A, B, C occur)
2 2 2
≤ P(A) + P(B) + P(C) – P(A ∩ B) – P(B ∩ C)
Now,
100 P(C ∩ A)]
1
P(X = 1) + P(X = 3) + ….. + P(X = 49) = 100
C1   +
2 Lastly P(A and at least one of B, C occur)
100 100 = P[A ∩ (B ∪ C)] = P[(A ∩ B) ∪ (A ∩ C)]
1 1
100
C3   + ……… + 100C49  
2 2 = P(A ∩ B) + P(A ∩ C) – P[(A ∩ B) ∩ (A ∩ C)]

100 = P(A ∩ B) + P(A ∩ C) – P(A ∩ B ∩ C)


1
= ( C1 +
100 100
C3 + …… + 100
C49)   ≤ P(A ∩ B) + P(A ∩ C)
2
But100C1 + 100C3 + ……. + 100C99 = 299 Example 8: For two events A and B, if P(A) =
Also, 100
C99 = 100
C 1, 1 1
P(A | B) = and P(B | A) = , then
100
C97 = 100C3, …….. 100C51 4 2
(A) A and B are independent
100
C49
(B) A and B are mutually exclusive
Thus, 2(100C1 + 100C3 + …… + 100C49)= 299
3
⇒ 100
C1 + 100C3 + …… + 100C49 = 298 (C) P (A’ | B) =
4
298 1 1
∴ Probability of required even = (D) P (B’ | A’) =
2100 4 2

Sol: Use basic formulae.


Example 7: If A, B and C are three events, then P(A ∩ B)
We have, P(A) = P(A/B) = ⇒ P(A ∩ B) = P(A)
(A) P (exactly two of A, B, C occur) ≤ P(A ∩ B) + P(B ∩ P(B)
P(B)
C) + P(C ∩ A)
Therefore, A and B are independent. Also
(B) P(A ∪ B ∪ C) ≤ P(A) + P(B) + P(C)
(C) P(exactly one of A, B, C occur ≤ P(A) + P(B) + P(C) – 1
 1
  1   
P(A ∩ B) = P(A) P(B | A) =     =   ¹0
P(B ∩ C) – P(C ∩ A) – P(A ∩ B) 4 2 8
(D) P (A and at least one of B, C occurs) ∴A and B are mutually exclusive.
≤ P(A ∩ B)+ P(A ∩ C) As A and B are independent
Sol: Apply Boole’s Inequality. 1
  3
P(A’ | B) = P(A’) = 1 – P(A) = 1 –   =  
 

We have, P(exactly two of A, B, C occur) 4 4


Since A and B are independent.
= P(A ∪ B) + P(B ∩ C) + P(C ∩ A) – 3P(A ∩ B ∩
1
C) ≤ P(A ∩ B) + P(B ∩ C) + P(C ∩ A) P(B) = P(B | A) =
2
Also, P(A ∪ B ∪ C) ≤ P(A ∪ B) + P(C) ≤ P(A) + 1
⇒ P(B’ | A’) = P(B’) = 1 – P(B) =
P(B) + P(C) 2

Next P(exactly one of A, B, C occurs) Example 9: Let X be a set containing n elements. If


= P(A) + P(B) + P(C) – 2P(A ∩ B) – 2P(B ∩ C) two subsets,A and B, of X are picked at random, the
probability that A and B have the same number of
M a them a ti cs | 18.25

elements is: Example 10: Two numbers are selected at random from
2n 1 the number 1, 2, ……., n. Let p denote the probability
Cn
(A) (B) 2n that the difference between the first and second is not
2 2n
Cn less than m (where 0 < m < n). If n = 25 and m = 10,
3n find 5p.
(C) 1.3.5 ........ (2n − 1) (D)
2n (n!) 4n
Sol: Apply Total probability theorem.

Sol: The number of ways of choosing a set of k Let the first number be x and the second be y. Let A
elements is nCk. The total number of subsets from a set denote the event that the difference between the first
of n elements is 2n. and second numbers is at least m. Let Ex denote the
event that the first number chosen is x. We must have
We know that the number of subsets of a set containing
x – y ≥ m or y ≤ x – m. Therefore x > m and y < n – m.
n elements is 2n. Therefore, the number of ways of
choosing A and B is 2n. 2n = 22n. We also know that Thus, P(Ex) = 0 for 0 < x ≤ m and P(Ex) = 1/n for m < x ≤ n.
the number of subsets (of X) which contain exactly Also, P(A | Ex) = (x – m)/(n – 1).
r elements is nCr. Therefore, the number of ways of
choosing A and B so that they have the same number n

of elements is Therefore, P(A) = ∑ P(Ex ) P(A | Ex )


x =1
(nC0)2 + (nC1)2 + (nC2)2 + …….. + (nCn)2
n n
1 x −m
1.2.3 ....... (2n − 1)(2n)
= ∑ P(Ex ) P(A | Ex ) = ∑ .
= Cn =
2n x= m+1 x= m+1 n n − 1
n!n!
1 (n − m)(n − m + 1)
[1.3.5.......(2n – 1)][2.4.6.....(2n)] [1 + 2 + ...... + (n – m)] =
= n(n − 1) 2n(n − 1)
n!n!
Put n = 25 and m = 10
Thus, the probability of the required event is
(25 − 10)(25 − 10 + 1)
2n
Cn 1.3.5.....(2n − 1) ⇒ 5P = 5 =1
= 2.25(25 − 1)
22n 2n (n!)

JEE Main/Boards

Exercise 1 Q.5 If P(A)=0.4, P(B)=p and P(A∪B)=0.7. Find the value


of p, if A and B are independent set of events.
1 2 3
Q.1 Given P(A) = , P(B) = and P(A ∪ B) = . Are
4 3 4 Q.6 Does the following table represents a probability
the events independent? distribution? Give reasons.

1 1 1 X –2 –1 0 1 2
Q.2 Given P(A) = , P(B) = and P(A ∩ B) = . Are the
2 3 6 P(X) 0.1 0.2 –0.2 0.4 0.5
events A and B independent?
Q.7 Given P(A) = 0.2, P(B) = 0.3 and P(A∩B) = 0.1 Find
Q.3 A die is thrown twice. Find the probability of getting P(A/B).
a number 6 on the first throw and number > 4 on the
second.
Q.8 The parameters n and p of a binomial distribution
are 12 and 1/3 respectively, Find the standard deviation.
Q.4 Given P(A) = 0.3, P(B)= 0.2. Find P(B/A) if A and B
are mutually exclusive events.
Q.9 Given P(A) = 0.4, P(B) = 0.7 and P(B/A) = 0.6. Find
P(A∪ B).
1 8 . 2 6 | Probability

Q.10 A coin is tossed three times and the Random which five tickets x1, x2,……
variable X represents “number of heads”. What values
x5 are drawn at random and arranged in
X can take?
ascending order of magnitude x1, < x2<x3<
Q.11 Does the following table represents a probability x4< x5. What is the probability that x3 =30 ?
distribution? Give reasons.
X 0 1 2 Q.23 A random variable X has the following probability
1 1 1 distribution:
P(X)
3 3 6 X –2 –1 0 1 2 3
P(X) 0.1 k 0.2 2k 0.3 k
Q.12 Find the value of k, such that the following
distribution represents a probability distribution Find the value k(i) P(X ≤ 1)
(ii) P(X ≥ 0)
X 0 1 2 3 4
P(X) k 0 3k 2k 4k
Q.24 Two cards are drawn successfully with replacement
from a well shuffled pack of 52cards.Find the probability
Q.13 Two cards are drawn successively, with replacement, distribution of number of aces.
from a deck of 52 cards. Find the probability of getting
both spades. Q.25 In a lottery, a person choose six different numbers
at random from 1 to 20 and if these six numbers match
Q.14 Find the mean of the distribution. with the six numbers already fixed by the lottery
X 1 2 3 committee, he wins the prize. What is the probability
of winning the prize in the game, (order of numbers is
P(X) 0.4 0.3 0.3 not important)?

Q.15 A coin is tossed 7 times, write the Probability Q.26 The probability of students A passing an
distribution of getting r heads. examination is 3/5 and of student B is 4/5. Assuming that
the two events “A passes”, “B passes” as independent.
Q.16 In two successive throws of a pair of dice, find the Find the probability of:
probability of getting a total of 8 each time. (i) Both the students passing the examination (ii) only A
passing the examination
Q.17 Events E and F are given to be independent. Find
(iii) Only one of them passing the examination (iv) none
P(F) if it is given that: P(E) = 0.60 and P(E∩F) = 0.35.
of them passing the examination.

Q.18 If A and B are two independent events such that


P(A∪ B)=0.7 P(A)=0.4. Find P(B). Q.27 A box contains 12 items of which 3 are defective.
A sample of 3 items is selected from the box. Let x
denote the number of defective items in a sample, find
Q.19 Two cards are drawn from a pack of 52 cards at the probability distribution of X.
random and kept out. Then one card is drawn from the
remaining 50 cards. Find the probability that it is an ace.
Q.28 Two dice are thrown. Find the probability that the
numbers appeared have a sum 8 if it is known that the
Q.20 Three cards are drawn with replacement from a
second dice always exhibits 4.
well shuffled pack of cards. Find the probability that the
cards are a king, a queen and a jack.
Q.29 In an examination, an examinee either guesses
or copies or knows the answer of multiple choice
Q.21 A policeman fires four bullets on a dacoit. The
questions with four choices. The probability that he
probability that the dacoit will be killed by one bullet
makes a guess is 1/3 and probability that he copies the
is 0.6. What is the probability that dacoit is still alive?
answer is 1/6.The probability that his answer is correct,
given that he copied it, is 1/8. Find the probability that
Q.22 A bag contains tickets numbered 1,2,3,..... , 50 of he known the answer to the question, given that he
M a them a ti cs | 18.27

correctly answered it. Q.39 A survey of200 families each having 4 children
was conducted. In how many families do you expect
Q.30 There are three bags which contains 2 white, 3black; 3 boys and 1 girl if boys and girls are equal probable?
4 white, 1 black; 3 white, 7 black balls respectively A ball
is drawn at random from one of the bags and is found Q.40 Past experience shows that 80% of Operations
to be black. Find the probability that is was drawn from performed by a doctor are successful. If he performs 4
the bag containing operations in a day, what is the probability that at least
three operations will be successful?
(i) Maximum number of black balls.
(ii) Maximum number of white balls.
Q.41 The probability that a student entering a collage
will graduate is 0.6. Find the probability that out of a
Q.31 Two cards are drawn successively with replacement group of 6 students,
from a pack of 52 cards. Find the probability distribution
(i) None (ii) Atleast one
of the number of aces. Find its mean and standard
deviation. (iii) At most 3 will graduate.

1
Q.32 The probability that a bulb produced by a factory Q.42 The probability of a bomb hitting a target is .
will fuse after 150 days of use is 0.05.Find the probability 3
that out of 5 such bulbs. Two bombs are enough to destroy a bridge. If five
bombs are dropped at the bridge, find the probability
(i) None that the bridge is destroyed.
(ii) Not more than one
(iii) More than one will fuse after 150 days of use. Q.43 In a binomial distribution, the sum of mean
and variance is 42 and their product is 360.Findthe
(iv) At least one distribution.

Q.33 In a hurdle race, a player has to cross 10 hurdles. Q.44 A bag contains 3 red and 7 black balls. Two balls are
The probability that he will clear each hurdle is 5/6. selected are selected at random without replacement.
What is the probability that he will knock down fewer If the second selected is given to be red, what is the
than 2 hurdles? probability that the first selected is also red?

Q.34 If on an average 1 ship in every 10 sinks, then find Q.45 Five dice are thrown simultaneously. If the
the chance that out of 5 ships at least 4 will arrive safely. occurrence of an odd number in a single die is
considered a success, find the probability that there are
Q.35 About 70% of certain kind of seeds sold in the odd number of successes.
retail market germinate when planted under normal
conditions. Suppose one packed contains 10 seeds. If Q.46 A die is thrown 10 times. If getting an even
these are planted, then what is the probability of 2 of number is a success, find the probability of getting at
these germinating? least 9 successes.

Q.36 A man takes a step forward with probability0.4 Q.47 There are three urns A, B and C. Urn A contain4
and backwards with probability 0.6. Find the probability red balls and 4 green balls. Urn B contains red ball and
that at the end of eleven steps, he is just one step away 5 green balls. Urn c contains 5 red balls and 2 green
from the starting point. balls. One ball is drawn from each of the three urns.
What is the probability out of these three drawn, two
Q.37 A bag contains 10 balls, each marked with one of are green ball one is a red ball?
the digits 0 to 9. If four balls are drawn successively with
replacement from the bag, what is the probability that Q.48 A bag contains 4 red, 3 black and 3 white ball two
none is marked with digit 6? balls are drawn from the bag. What is the probability
that none of the balls drawn is white ball?
Q.38 Six dice are thrown 729 times. How many times
do you expect at least three dice to show five or six?
1 8 . 2 8 | Probability

Q.49 A and B appear for an interview for two post, the Q.6 For any 2 events A&B, the probabilities P(A), P(A+B),
probability of A’s selection is 1/3 and the of B’s selection P(AB) & P(A) + P(B) when arranged in the increasing
is 2/5. Find the probability the only of them will be order of their magnitudes is:
selected.
(A) P(AB)≤ P(A) ≤ P(A+B)≤ P(A)+P(B)
(B) P(A)+P(B)≤ P(A+B) ≤ P(AB)≤P(A)
Q.50 A coin is tossed thrice and all eight out come are
assumed equally likely Let the event E”the first throw (C) P(A+B) ≤ P(AB) ≤ P(A)+P(B)≤ P(A)
results in head” and event F”the last throw results in
(D)P(AB) ≤ P(A) ≤ P(A)+P(B) ≤ P(A+B)
tail”. Find whether events E and F are independent.

Q.7 An integer x is chosen from the first 50 positive


100
integers. The probability that, x + > 50, is:
Exercise 2 x
1 2 1
(A) (B) (C) (D) None of these
Single Correct Choice Type 25 25 10

Q.1 If cards are drawn at random from a pack of 52 Q.8 The probability of India winning a test match
playing cards without replacement then the probability against West Indies is 1/2. Assuming independent from
that a particular card is drawn at the nth draw is: match to match the probability that in a 5 match series,
(A) 1/(53-n) (B) 1/52 India’s second win occurs at the 3rd test is:

(C) n/52 (D) n/(53 – n) (A) 1/4 (B) 1/8 (C) 1/2 (D) 2/3

Q.2 4 persons are asked the same question by an


interviewer. If each has independent probability 1/6 of Previous Years’ Questions
answering the question correctly. The probability that
at least one answers correctly is:
Q.1 The probability that an event A happens in one trial
(A) 2/3 (B) (1/6)4 of an experiment is 0.4. Three independent trials of the
experiments are performed. The probability that the
(C) 1 – (5/6)4 (D)1 – (1/6)4
event A happens at least ones is (1980)

Q.3 A person draws a card from a pack of 52 cards, (A) 0.936 (B) 0.784
replaces it & shuffles the pack. He continues doing this (C) 0.904 (D) None
till he draws a spade. The probability that he will fail
exactly the first two times is:
Q.2 If A and B are two independent events such that
(A) 1/64 (B) 9/64 (C) 36/64 (D) 60/64 P(A )> 0,and P(B) ≠ 1, then P(A / B) is equal to (1982)

(A) 1 – P(A/B) (B) 1 –P(A/ B )


Q.4 A committee of 5 is to be chosen from a group of
9 people. The probability that a certain married couple 1 − P(A ∪ B) P(A)
will either serve together or not at all is: (C) (D)
P(B) P(B)
(A) 1/2 (B) 5/9 (C) 4/9 (D)2/3
Q.3 The probability that at least one of the events A
Q.5 For a biased die the probabilities for the different and B occurs is 0.6. If A and B occur simultaneously with
faces to turn up are given below: probability 0.2, then P(A) + P(B) is: (1987)

Faces: 1 2 3 4 5 6 (A) 0.4 (B) 0.8 (C) 1.2 (D) 1.4

Prob.: 0.10 0.32 0.21 0.15 0.05 0.17 (Here A and B are complements of A and B,
respectively).
The die is tossed & you are told that either face one or
face two has turned up. Then the probability that it is
face one is: Q.4 One hundred identical coins, each with probability
p, of showing up heads are tossed once. If 0< p< 1 and
(A) 1/6 (B)1/10 (C) 5/49 (D) 5/21
M a them a ti cs | 18.29

the probability of heads showing on 50 coins is equal Q.11 Let (0 be a complex cube root of unity with
to that of heads showing on 51 coins, then the value of ω ≠ 1. A fair die is thrown three times. If r1, r2 and r3are
p is (1988) the numbers obtained on the die, then the probability
1 that ωr1 + ωr2 + ωr3 = 0 is: (2010)
(A) (B) 49/101 (C) 50/101 (D) 51/101
2 (A) 1/18 (B) 1/9 (C) 2/9 (D) 1/36

Q.5 Seven white balls and three black balls are randomly Q.12 A signal which can be green or red with probability
placed in a row. The probability that no two black balls 4 1
are placed adjacently, equals  (1998) and respectively, is received by station A and
5 5
1 7 2 1 then transmitted to station B. The probability of each
(A) (B) (C) (D)
2 15 15 3 3
station receiving the signal correctly is . If the signal
4
Q.6 If E and F are events with P(E)≤ P(F) and P(E∩ F)>0, received at station B is green, then the probability that
then: (1998)
the original signal is green is (2010)
(A) Occurrence of E ⇒ occurrence of F
3 6 20 9
(B) Occurrence of F ⇒ occurrence of E (A) (B) (C) (D)
5 7 23 20
(C) Non-occurrence of E ⇒ non-occurrence of F
(D) None of the above implication holds Q.13 It is given that the event A and B are such that
1 A 1
and P  B  = 2 . Then P(B) is(2008)
 
P(A)
= =, P 
3 1 4 B 2 A 3
Q.7 If P(B) = , P(A ∩ B ∩ C) = and
4 3
1 1 1 2 1
P(A ∩ B ∩ C) = then P(B ∩ C) is: (2002) (A) (B) (C) (D)
3 6 3 3 2
1 1 1 1
(A) (B) (C) (D)
12 6 15 9 Q.14 A die thrown. Let A be the event that the number
obtained is greater than 3. Let B be the event that the
Q.8 If three distinct numbers are chosen randomly number obtained is less than 5. Then P(A ∪ B) is(2008)
from the first100 natural numbers, then the probability 3 2
that all three of them are divisible by both 2 and 3, is (A) (B) 0 (C) 1 (D)
5 5
 (2004)
4 4 4 4
(A) (B) (C) (D) Q.15 The mean of the number a, b, 8, 5, 10 is 6 and the
55 35 33 1155 variance is 6.80. Then which one of the following gives
possible values of a and b? (2008)
Q.9 One Indian and four American men and their wives
are to be seated randomly around a circular table. (A) a = 0, b = 7 (B) a = 5, b = 2
Then, the conditional probability that the Indian man (C) a = 1, b = 6 (D) a = 3, b = 4
is seated adjacent to his wife given that each American
man is seated adjacent to his wife, is  (2007)
Q.16 In a stop there are five types of ice-creams
(A) 1/2 (B) 1/3 (C)2/5 (D)1/5 available. A child buys six ice-creams
Statement-I: The number of different ways the child
Q.10 An experiment has 10 equally likely outcomes. Let can buy the six ice-creams is 10 C
5
A and B be two non-empty events of the experiment.
Statement-II: The number of different ways the child
If A consists of 4 outcomes, the number of outcomes
can buy the six ice-creams is equal to the number of
that B must have so that A and B are independent, is
different ways of arranging 6 A/s and 4 B’s in a row.
 (2008)
 (2008)
(A) 2,4 or 8 (B) 3, 6 or 9
(A) Statement-I is false, statement-II is true.
(C) 4 or 8 (D)5 or 10
(B) Statement-I is true, statement-II is true; statement-II
1 8 . 3 0 | Probability

is a correct explanation for statement-I (C) Statement-I is true, statement-II is false


(C) Statement-I is true, statement-II is true ; statement-II (D) Statement-I is false, statement-II is true
is not a correct explanation for statement-I.
(D) Statement-I is true, statement-II is false Q.22 Four numbers are chosen at random (without
replacement) from the set {1, 2, 3, ….., 20} (2010)
Q.17 How many different words can be formed by Statement-I: The probability that the chosen numbers
jumbling the letters in the word MISSISSIPPI in which 1
no two S are adjacent?  (2008) when arranged in some order will form an AP is .
85
(A) 8 . 6 C 4 . 7 C 4 (B) 6 . 7 . 8 C 4 Statement-II: If the four chosen numbers from an AP,

(C) 6 . 8 . 7 C 4 (D) 7 . 6 C 4 . 8 C 4 then the set of all possible values of common difference

{
is ±1, ±2, ±3, ±4, ±5 . }
Q.18 If the mean deviation of number 1, 1 + d, 1 + 2d,
…., 1 + 100d from their mean is 255, then the d is equal (A) Statement-I is true, statement-II is true; statement-II
to (2009) is not the correct explanation for statement-I

(A) 10.0 (B) 20.0 (C) 10.1 (D) 20.2 (B) Statement-I is true, statement-II is false
(C) Statement-I is false, statement-II is true
Q.19 From 6 different novels and 3 different dictionaries, (D) Statement-I is true, statement-II is true; statement-II
4 movies and 1 dictionary are to be selected and is the correct explanation for statement-I
arranged in a row on the shelf so that the dictionary
is always in the middle. Then the number of such 10 10
Q.23 Let S1 =∑ j( j − 1) 10
Cj, S2 ⇒ ∑ j 10
Cj and
arrangements is (2009) j=1 j=1
10
(A) Less than 500 S3 = ∑ j2 20
C j . (2010)
j=1
(B) At least 500 but less than 750
(C) At least 750 but less than 1000 9
Statement-I: S3 = 55 × 2
(D) At least 1000 8 8
Statement-II: S=
1 55 × 2 and S=
2 10 × 2

(A) Statement-I is true, statement-II is true; statement-II


Q.20 One ticket is selected at random from 50 tickets
is not the correct explanation for statement-I
numbered 00, 01, 02, …., 49. Then the probability that
the sum of the digits on the selected ticket is 8, given (B) Statement-I is true, statement-II is false
that the product of these digits is zero, equals. (2009)
(C) Statement-I is false, statement-II is true
1 1 5 1
(A) (B) (C) (D) (D) Statement-I is true, statement-II is true, Statement-II
14 7 14 50 is the correct explanation for statement-I

Q.21 Statement-I: The variance of first n even natural


Q.24 There are two urns. urn A has 3 distinct red balls
n2 − 1 and urn B has 9 distinct blue balls. From each urn two
numbers is
4 balls are taken out at random and then transferred to
the other. The number of ways in which this can be
Statement-II: The sum of first n natural number is
done is (2010)
n(n + 1)
and the sum of squares of first n natural (A) 36 (B) 66 (C) 108 (D) 3
2
n(n + 1)(2n + 1)
numbers is  (2009)
6 Q.25 An urn contains nine balls of which three are red,
(A) Statement-I: is true, statement is true; statement-II is
four are blue and two are green. Three balls are drawn
a correct explanation for statement-I
at random without replacement from the urn. The
(B) Statement-I is true, statement-II is true; statement-II probability that the three balls have different colour is
is not a correct explanation for Statement-I  (2010)
2 1 2 1
(A) (B) (C) (D)
7 21 23 3
M a them a ti cs | 18.31

Q.26 For two data sets, each of size 5, the variances are Q.32 The number of integers greater than 6,000 that
given to be 4 and 5 and the corresponding means are can be formed, using the digits 3, 5, 6, 7 and 8 without
given to be 2 and 4, respectively. The variance of the repetition is: (2015)
combined data set is (2010)
(A) 216 (B) 192 (C) 120 (D) 72
11 13 5
(A) (B) 6 (C) (D)
2 2 2 Q.33 If 12 identical balls are to be placed in 3 identical
boxes, then the probability that one of the boxes
Q.27 Assuming the balls to be identical except for contains exactly 3 balls is (2015)
difference in colours, the number of ways in which one 11 10
or more balls can be selected from 10 white, 9 green 55  2  2
and 7 black balls is (2012)
(A)   (B) 55  
3 3 3
(A) 880 (B) 629 (C) 630 (D) 879 12 11
1 1
(C) 220   (D) 22  
Q.28 Three numbers are chosen at random without 3 3
replacement {1, 2, 3, ….., 8}. The probability that their
minimum is 3, given that their maximum is 6, is(2012) Q.34 The mean of the data set comprising of 16
3 1 1 2 observations is 16. If one of the observation valued 16
(A) (B) (C) (D) is deleted and three new observation valued 3, 4 and 5
8 5 4 5
are added to the data, then the mean of the resultant
data, is (2015)
Q.29 All the students of a class performed poorly in
Mathematics. The teacher decided to give grace marks (A) 16.8 (B) 16.0 (C) 15.8 (D) 14.0
of 10 to each of the students. Which of the following
statistical measures will not change even after the Q.35 If the standard deviation of the numbers 2, 3, a
grace marks were given? (2013) and 11 is 3.5, then which of the following is true(2016)
(A) Mean (B) Median (A) 3a2 – 32a + 84 = 0 (B) 3a2 – 34a + 91 = 0
(C) Mode (D) Variance (C) 3a2 – 23a + 44 = 0 (D) 3a2 – 26a + 55 = 0

Q.30 Let A and B be two events such that Q.36 Let two fair six-faced dice A and B be thrown
1 1 1 simultaneously. If E1 is the event that die A shown up
P(A ∪=
B) , P(A ∩=
B) and P(A) = , where A
6 4 4 four, E2 is the event that die B shows up two and E3 is
the event that the sum of numbers on both dice is odd,
stands for the complement of the event A. Then the
then which of the following statements is NOT true?
events A and B are (2014)
 (2016)
(A) Independent but not equally likely (A) E2 and E3 are independent
(B) Independent and equally likely (B) E1 and E3 are independent
(C) Mutually exclusive and independent (C) E1, E2 and E3 are independent
(D) Equally likely but not independent (D) E1 and E2 are independent

Q.31 The variance of first 50 even natural numbers is Q.37 If all the words (with or without meaning) having
 (2014) five letters, formed using the letter of the word SMALL
437 833
(A) 437 (B) (C) (D) 833 and arranged as in dictionary, then the position of the
4 4 work SMALL is (2016)
(A) 59 (B) 52 (C) 58 (D) 46
1 8 . 3 2 | Probability

JEE Advanced/Boards

Exercise 1 Q.8 Let an ordinary fair dice is thrown for five times.
a
If P = expressed in lowest form be the probability
b
Q.1 There are 2 groups of subjects one of which consists
that the outcome of the fifth throw was already thrown,
of 5 science subjects and 3 engineering subjects and
then find the value of (a + b).
other consists of 3 science and 5 engineering subjects.
An unbiased die is cast. If the number 3 or 5 turns
up a subject is selected at random from first group, Q.9 A bomber wants to destroy a bridge. Two bombs
otherwise the subject is selected from 2ndgroup. Find are sufficient to destroy it. If four bombs are dropped,
the probability that an engineering subject is selected. what is the probability that it is destroyed, if the chance
of a bomb hitting the target is 0.4
Q.2 A pair of fair dice is tossed. Find the probability that
the maximum of the two numbers is greater than 4. Q.10 The chance of one event happening is the square
of the chance of a 2nd event, but odds against the first
are the cubes of the odds against the 2nd. Find the
Q.3 In a given race, the odds in favor of four horses A, B,
chances of each, (assume that both events are neither
C & D are 1: 3, 1: 4, 1: 5 and 1: 6 respectively. Assuming
sure nor impossible).
that a dead heat is impossible, find the chance that one
of them wins the race.
Q.11 A box contains 5 radio tubes of which 2 are
defective. The tubes are tested one after the other until
Q.4 A covered basket of flowers has some lilies and
the 2 defective tubes are discovered. Find the probability
roses. In search of rose, sweety and shweta alternately
that the process stopped on the (i) Second test;
pick up a flower from the basket but puts it back if it is
(ii) Third test. If the process stopped on the third test, (iii)
not a rose. Sweety is 3 times more likely to be the first
find the probability that the first tube is non-defective.
one to pick a rose. If Sweety begin this ‘rose hunt’ and
if there are 60 lilies in the basket, find the number of
roses in the basket. Q.12 An aircraft gun can take a maximum of four
shots at an enemy’s plane moving away from it. The
probability of hitting the plane at first, second, third &
Q.5 A hotel packed breakfast for each of the three
fourth shots are 0.4, 0.3, 0.2 & 0.1 respectively. What is
guests. Each breakfast should have consisted of three
the probability that the gun hits the plane?
types of rolls, one each of nut, cheese and fruit rolls.
The preparer wrapped each of the nine rolls and once
warped, the rolls were indistinguishable from one Q.13 In a batch of 10 articles, 4 articles are defective. 6
another. She then randomly put three rolls in a bag for articles are taken from the batch for inspection. If more
each of the guests. If the probability that each guest got than 2 articles in this batch are defective, the whole
one roll of each type is m where m and n are relatively batch is rejected. Find the probability that the batch will
prime integers, find the value of(m + n). be rejected.

Q.6 Players A and B alternately toss a biased coin, with A Q.14 A game is played with a special fair cubic die
going first. A wins if A tosses a Tail before B tosses a Head; which has one red side, two blue sides, and three green
otherwise B wins. If the probability of a head is p, find the sides. The result is the colour of the top side after the
value of p for which the game is fair to both players. die has been rolled. If the die is rolled repeatedly, the
probability that the second blue result occurs on or
a b before the tenth roll, can be expressed in the form
Q.7 The entries in a two-by-two determinant are 3p – 2q
c d where p, q, r are positive integers, find the
integers that are chosen randomly and independently, 3r
and, for each entry, the probability that the entry is odd value of p2 + q2 + r2.
is p. If the probability that the value of the determinant
is even is 1/2, then find the value of p.
M a them a ti cs | 18.33

Q.15 An author writes a good book with a probability (ii) If n = 4 then find the probability that each of the 4
of 1/2. If it is good it is published with a probability of persons draw balls of the same colour.
2/3. If it is not, it is published with a probability of 1/4.
(iii) If n = 7 then the find the probability that each of the
Find the probability that he will get atleast one book
7 persons draw balls of same colour.
published if he writes two.

Q.22 Eight players P1, P2, P3, ……….. P8 play a knock-out


Q.16 Consider 4 independent trials in which an event
tournament. It is known that whenever the players Pi
1 and Pj play, the player Pj will win if i<j. Assuming that
A occurs with probability . The event B will occur
3 the players are paired at random in each round, what is
with probability if the event A occurs atleast twice, the probability that the player P4 reaches the final.
it can not occur if the event A does not occur and it
1 Q.23 Let A & B be two events defined on a sample
occurs with a probability if the event A occurs once.
2 space. Given P (A) = 0.4 ; P(B) = 0.80and P(A / B) = 0.10.
If the probability P of the occurrence of event B can be Then find;
m
expressed as
n
, find the least value of(m + n), where ( ) (
(i) P(A ∪ B) and P  A ∩ B ∪ A ∩ B )
m, n ∈ N.‘
Q.24 Mr. A randomly picks 3 distinct numbers from the
set {1, 2, 3, 4, 5, 6, 7, 8, 9} and arranges them in the
Q.17 A uniform unbiased die is constructed in the shape
descending order to form a three digit number. Mr. B
of a regular tetrahedron with faces numbered 1,2,3 and
randomly picks 3 distinct numbers from the set {1, 2, 3,
4 and the score is taken from the face on which the die
4, 5, 6, 7, 8} and also arranges them in descending order
lands. If two such dice are thrown together, find the
to form a 3 digit number.
probability of scoring.
(i) Find the probability that A and B has the
(i) Exactly 6 on each of 3 successive throws.
same three digit number.
(ii) More than 4 on at least one of the three successive
throws. (ii) Find the probability that Mr. A’s number is larger
than Mr. B’s number.
Q.18 Two cards are drawn from a well shuffled pack of
52 cards. Find the probability that one of them is a red Q.25 A pair of students is selected at random from a
card & the other is a queen. probability class. The probability that the pair selected
10
will consist of one male and one female student is .
Q.19 A person flips 4 fair coins and discards those 19
which turn up tails. He again flips the remaining coin Find the maximum number of students the class can
and then discards those which turn up tails. Find the contain.
probability that he discards atleast 3 coins.

Q.20 Each of the ‘n’ passengers sitting in a bus may


get down from it at the next stop with probability p.
Exercise 2
Moreover, at the next stop either no passenger or
Single Correct Choice Type
exactly one passenger boards the bus. The probability
of no passenger boarding the bus at the next stop being
Q.1 Suppose, that it is 9 to 7 against a person A who is
po. Find the probability that when the bus continues on
now 35 years of age living till he is 65 and 3 to 2 against
its way after the stop, there will again be ‘n’ passengers
a person B now 45 living till he is 75, then the chance
in the bus.
that one at least of these persons will be alive 30 years
hence is:
Q.21 A jar contains 2n thoroughly mixed balls, n white
(A) 14/27 (B) 53/80
and n black balls. n persons each of whom draw 2 balls
simultaneously from the bag without replacement. (C) l/2 (D) None of these
(i) If the probability that each of the n person draw both
balls of different colours is 8 35, then find the value of n.
1 8 . 3 4 | Probability

Q.2 An experiment results in four possible outcomes S1, Q.9 A gambler has one rupee in his pocket. He tosses
S2, S3 and S4 with probabilities p1, p2, p3&p4 respectively. an unbiased normal coin unless either he is ruined or
Which one of the following probability assignment is unless the coin has been tossed for a maximum of five
possible? [Assume S1, S2, S3, S4 are mutually exclusive] times. If for each head he wins a rupee and for each tail
he loses a rupee, then the probability that the gambler
(A) p1=0.25, p2=0.35, p3=0.10,p4=0.05
is ruined is:
(B) p1=0.40, p2 =-0.20, p3=0.60,p4=0.20
(A) 1/2 (B) 5/8 (C) 3/8 (D) 22/32
(C) p1= 0.30, p2 = 0.60,p3 = 0.10, p4= 0.10
(D) p1 = 0.20, p2 = 0.30,p3= 0.40,p4 = 0.10 Q.10 If x be chosen randomly from the set of first 50
natural numbers, then the probability that xx is perfect
Q.3 Let P denotes the probability that in a group of 4 square of a natural number is-
persons all are born on different days of the week, then (A) 12/25 (B)1/2 (C) 29/50 (D) 31/50
P must lie in the interval:
1 1 1 1 Q.11 A and B independently solve a problem. The
(A) < P < (B) < P <
3 2 4 5 chance that A and B will solve the problem correctly
are P & 1/2 respectively. The chance that they will make
1 1
(C) <P< (D) None of these 1
6 3 the same mistake is . If the probability that their
100
answer is correct and they get the same answer which
Q.4 The probability that 4th power of a positive integer
ends in the digit 6 is: 300
is , then P is:
301
(A) 10% (B) 20% (C) 25% (D) 40%
(A) 1/2 (B) 3/4 (C) 1/4 (D) None of these

Q.5 India plays 2 matches each with West-Indies


& Australia. In any match the probabilities of India Q.12 Two dice are thrown until a 6 appears on atleast
getting points 0,1 & 2 are 0.45, 0.05 & 0.50 respectively one of them. Then the probability that for the first time,
Assuming that the outcomes are independent, the a 6 appears in the second throw is:
probability of India getting atleast 7 points is: (A) 175/1296 (B) 275/1296
(A) 0.8750 (B) 0.0875 (C) 375/1296 (D) None of these
(C) 0.0625 (D) 0.0250
Q.13 Box A has 3 white & 2 red balls, box B has 2
Q.6 A women has ‘n’ keys, of which one will open her white & 4 red balls. If two balls are selected at random
door. If she tries the keys randomly, discarding those (without replacement) from A & 2 more are selected at
that do not work (with out using the discarded key random from B, the probability that all the four balls
again), the probability that she will open the door with are white is:
the last key is: (A) 10% (B) 2% (C) 12% (D) 4%
1 1 1 1
(A) (B) (C) (D) n
n−1 n (n − 1)! 2 Q.14 A & B are two independent events such that
P(A) = 0.7, P(B) = a & P(A ∪B) = 0.8, then, a =
Q.7 If A & B are two independent events, each with (A) 5/7 (B) 2/7 (C) 1 (D) None
probability P, ( P ≠ 0) then P(A/A∪ B) is:
(A) 1/P (B) 1/2 Q.15 A writes a letter to his friend B and gives it to his
(C)2/P (D) 1/(2 – P) son to post it. The chance that his son will post the letter
is 1/2 and the chance that a letter posted will reach it’s
destination is 5/6. If the letter was not received by B,
Q.8 The probability of obtaining more tails than heads
the chance A’sson did not post the letter is-
in 6 tosses of a fair coins is:
5 6 2 6
(A) 2/64 (B) 22/64 (A) (B) (C) (D)
11 11 3 7
(C) 21/64 (D) None of these
M a them a ti cs | 18.35

Q.16 Two numbers are randomly selected from the set Q.7 A person goes to office either by car, scooter, bus
of first 20 natural numbers. Find the chance that their 1 3 2 1
product is even given that their sum is odd- or train probability of which being , , and .
7 7 7 7
9 10 29 respectively. Probability that he reaches offices late, if
(A) (B) (C) (D) None of these
19 19 38 2 1 4
he takes car, scooter, bus or train is , , and
9 9 9
1
respectively. Given that he reached office in time,
9
Previous Years’ Questions then what is the probability that he travelled by a car?

Q.1 A box contains 2 black, 4 white and 3 red balls. One  (2005)
ball is drawn at random from the box and kept aside.
From the remaining balls in the box, another ball is Paragraph 1 (Q.8 to Q.9): Read the following Paragraph
drawn at random and kept beside the first. This process and answer the questions.
is repeated till all the balls are drawn from the box. Find There are n urns each containing (n + 1) balls such that
the probability that the balls drawn are in the sequence the ith urn contains 7 white halls and (n + 1–i) red halls.
of 2 black, 4 white and 3 red. (1979) Let u, be the event of selecting ith urn, i = 1,2,3, ........,
n and W denotes the event of getting a white balls.
Q.2 A and B are two independent events. The probability  (2006)
1
that both A and B occur is and the probability that Q.8 If (ui) ∝ I, where i = 1, 2,3, ....... , then lim P(W) is
6 n→∞
1 equal to
neither of them occurs is . Find the probability of the
3 2 1 3
occurrence of A. (1984) (A) 1 (B) (C) (D)
3 4 4

Q.3 In a multiple-choice question there are four Q.9 If P(ui)=c, where c is a constant, then P(un/W) is
alternative answers, of which one or more are correct. A equal to
candidate will get marks in the question only if he ticks
2 1 n 1
the correct answers. The candidates decide to tick the (A) (B) (C) (D)
answers at random, if he is allowed up to three chances n+1 n+1 n+1 2
to answer the questions, find the probability that he will
get marks in the question.  (1985) Q.10 If E and F are independent events such that
0< P(E)<1 and0<P(F)<1, then (1989)
Q.4 Three players. A, B and C. toss a coin cyclically in (A) E and F are mutually exclusive
that order(that is A, B, C, A, B, C, A, B......) till a head (B) E and Fc (the complement of the event F) are
shows. Let p be the probability that the coin shows a independent
head. Let α, β and γ be, respectively, the probabilities
that A, B and C gets the first head. Prove that β = (1 – p) (C) EC and FC are independent
α. Determine α, β and γ(in terms of P)  (1998) (D) P(E/F)+P(EC/F)= 1

Q.5 An unbiased die, with faces numbered 1, 2, 3, 4, 5, Q.11 Let E and F be two independent events. The
6, is thrown n times and the list of n numbers showing probability that both E and F happen is 1/12 and the
up is noted. What is the probability that among the probability that neither E nor F happen is 1/2. Then,
numbers 1, 2, 3, 4, 5, 6 only three numbers appear in  (1993)
this list?  (2001) (A) P(E) = 1/3, P(F) = 1/4
(B) P(E)=1/2,P(F)= 1/6
Q.6 A bag contains 12 red balls and 6 white balls. Six
balls are drawn one by one without replacement of (C) P(E)= 1/6, P(F) = 1/2
which at least 4 balls are white. Find the probability that (D) P(E) = 1/4, P(F) = 1/3
in the next two drawn exactly one white ball is drawn.
(Leave the answer in nCr). (2004)
1 8 . 3 6 | Probability

Q.12 If E and F are the complementary events of E and Q.17 Let Ec denote the complement of an event E. Let E,
F respectively and if0 <P(F)< 1, then (1998) F, g be pairwise independent events with P(G) > 0 and

(A) P(E/F)+P( E /F)= 1 P(E ∩ F ∩ G) =0 . Then P(Ec ∩ Fc |G) equals (2007)

(B) P(E/F) + P(E/ F )= 1 (A) P(Ec ) + P(Fc ) (B) P(Ec ) − P(Fc )


(C) P( E /F) + P(E/ F )= 1 (C) P(Ec ) − P(F) (D) P(E) − P(Fc )
(D) P(E/ F )+P( E / F )= 1
Q.18 An experiment has 10 equally likely outcomes. Let
Q.13 Let E and F be two independent events. The A and B be two non-empty events of the experiment.
11 If A consists of 4 outcomes, the number of outcomes
probability that exactly one of them occurs is and that B must have so that A and B are independent, is
25
2  (2008)
the probability of none of them occurring is . If P(T) (A) 2, 4 or 8 (B) 3, 6 or 9
25
denotes the probability of occurrence of the event T, (C) 4 or 8 (D) 5 or 10
then  (2011)
4 3 1 2 Q.19 Consider all possible permutations of the letters
(A) P(E) = , P(F) = (B) P(E) = , P(F) = of the word ENDEANOEL.
5 5 5 5
Match the statement/expressions in column I with the
2 1 3 4
(C) P(E) = , P(F) = (D) P(E) = , P(F) = statement/expressions in column II. (2008)
5 5 5 5

Q.14 One Indian and four American men and their Column I Column II
wives are to be seated randomly around a circular table. (A) The number of permutations containing (p) 5!
Then the conditional probability that the Indian man is the word ENDEA is
seated adjacent to his wife given that each American
(B) The number of permutations in which (q) 2 × 5!
man is seated adjacent to his wife is  (2007)
the letter E occurs in the first and the last
(A) 1/2 (B) 1/3 (C) 2/5 (D) 1/5 positions is
(C) The number of permutations in which (r) 7 × 5!
Q.15 Let H1, H2, …..Hn be mutually exclusive and none of the letters D, L, N occurs in the last
exhaustive event with P(H1) > 0, I = 1, 2, …., n Let E be five position is
any other event with 0 < P (E) < 1. (2007)
(D) The number of permutations in which the (s) 21 × 5!
Statement-I: P (Hi|E) > P (E | Hi). P (Hi) for I = 1, 2, …..,n letter A, E, O occur only in odd positions is
Beause
n
Statement-II: ∑ P(Hi ) = 1 Q.20 The number of seven digit integers, with sum of
i=1 the digit equal to 10 and formed by using the digits 1,
(A) Statement-I is True, statement-II is true, statement-II 2 and 3 only, is  (2009)
is a correct explanation for statement-I (A) 55 (B) 66 (C) 77 (D) 88
(B) Statement-I is True, statement-II is True, statement-II
is NOT a correct explanation for statement-I Paragraph 2 (Q.21 to Q.23): A fair die is tossed
repeatedly until a six is obtained. Let X denote the
(C) Statement-I is True, statement-II is False number of tosses required.  (2009)
(D) Statement-I is False, statement-II is True.
Q.21 The probability that X = 3 equals
25 25 5 125
Q.16 The letters of the word COCHIN are permuted and (A) (B) (C) (D)
all the permutations are arranged in an alphabetical 216 36 36 216
order as in an English dictionary. The number of words
that appear before the work COCHIN is (2007) Q.22 The probability that X ≥ 3 equals
(A) 360 (B) 192 (C) 96 (D) 48 125 25 5 25
(A) (B) (C) (D)
216 36 36 216
M a them a ti cs | 18.37

Q.23 The conditional probability that X ≥ 6 given X > (B) P [Exactly two engines of the ship are functioning |
3 equals 7
X] =
125 25 5 25 8
(A) (B) (C) (D) 5
216 216 36 36 (C) P[X | X 2 ] =
16
Q.24 A signal which can be green or red with probability 7
(D) P[X | X1 ] =
4 1 16
and respectively, is received by station A and
5 5
then transmitted to station B. The probability of each Q.29 Four fair dice D1, D2, D3 and D4 each having six faces
numbered 1, 2, 3, 4, 5 and 6 are rolled simultaneously.
3
station receiving the signal correctly is . If the signal The probability that D4 shows a number appearing on
4
one of D1, D2 and D3 is  (2012)
received at station B is green, then the probability that
the original signal was green is  (2010) 91 108 125 127
(A) (B) (C) (D)
216 216 216 216
3 6 20 9
(A) (B) (C) (D)
5 7 23 20
Q.30 Let X and Y be two events such that
1 1 1
Paragraph 3 (Q.25 to Q.26): Let U1 and U2 be two urns = P(X | Y) = , P(Y | X) and P(X ∩ Y) =. Which of
such that U1 contains 3 white and 2 red balls, and U2 2 3 6
contains only 1 white ball. A fair coin is tossed. If head the following is (are) correct ?  (2012)
appears then 1 balls is drawn at random from U1 and 2
put into U2. However, if tail appears then 2 balls are (A) P(X ∪ Y) =
3
drawn at random from U1 and put into U2. Now 1 ball is
drawn at random from U2. (2011) (B) X and Y are independent
(C) X and Y are not independent
Q.25 The probability of the drawn ball from U2 being
white is 1
(D) P(X c ∩ Y) =
3
13 23 19 11
(A) (B) (C) (D)
30 30 30 30
Q.31 Four persons independently solve a certain
1 3 1 1
Q.26 Given that the drawn ball from U2 is white, the problem correctly with probabilities , , , . Then
2 4 4 8
probability that head appeared on the coin is
the probability that the problem is solved correctly by
17 11 15 12 the at least one of them is  (2013)
(A) (B) (C) (D)
23 23 23 23
235 21 3 253
(A) (B) (C) (D)
256 256 256 256
Q.27 The total number of ways in which 5 balls of
different colours can be distributed among 3 persons
so that each person gets at least one ball is  (2012) Q.32 Of the three independent events E1, E2 and E3 the
probability that only E1 occurs is α only E2 occurs is
(A) 75 (B) 150 (C) 210 (D) 243 β and only E3 occurs is γ . Let the probability p that
none of events E1, E2 or E3 occurs satisfy the equations
Q.28 A ship is fitted with three engines E1, E2 and E3. (α − 2β)p = αβ and (β − 3γ )p = 2βγ . All the given
The engines function independently of each other with probabilities are assumed to lie in the interval (0, 1).
1 1 1
respective probabilities , and . For the ship to be Probability of occurence of E1
2 4 4 Then, = _________
operational at least two of its engines must function. Probability of occurrence of E3
Let X denote the event that ship is operational and let
X1, X2 and X3 denote respectively the events that the Q.33 A pack contains n cards numbered from 1 to n.
engines E1, E2 and E3 are functioning. Which of the Two consecutive numbered card are removed from the
following is(are) true ?  (2012) pack and the sum of the numbers on the remaining
cards is 1224. If the smaller of the numbers on the
3
(A) P  X1c | X  = removed cards is k, then k – 20 =  (2013)
  16
1 8 . 3 8 | Probability

Q.34 If 1 ball is drawn from each of the boxes B1, B2 and Q.41 Let n be the number of ways in which 5 boys and 5
B3, the probability that all 3 drawn balls are of the same girls can stand in a queue in such a way that all the girls
colour is  (2013) stand consecutively in the queue. Let m be the number
of ways in which 5 boys and 5 girls can stand in a queue
82 90 558 556
(A) (B) (C) (D) in such a way that exactly four girls stand consecutively
648 648 648 648
m
in the queue. Then the value of is  (2015)
n
Q.35 If 2 balls are drawn (without replacement) from a
randomly selected box and one of the balls is white and Paragraph 5 (Q.42 to Q.45): Let n1 and n2 be the
the other ball is red, the probability that these 2 balls number of red and black balls, respectively, in box I.
are drawn from box B2 is  (2013) Let n3 and n4 be the number of red and black balls,
116 126 65 55 respectively, in box II  (2015)
(A) (B) (C) (D)
181 181 181 181
Q.42 One of the two boxes, box I and box II, was selected
at random and a ball was drawn randomly out of this
Q.36 Three boys and two girls stand in a queue. The box. The ball was found to be red. If the probability that
probability, that the number of boys ahead of every girl
1
is at least one more than the number of girls ahead of this red ball was drawn from box II is , then the correct
3
her, is  (2014)
option(s) with the possible of n1, n2, n3 and n4 is (are)
1 1 2 3
(A) (B) (C) (D) (A)=
n1 3,
= n2 3,
= n3 5,=
n4 15
2 3 3 4
(B)=
n1 3,
= n2 6,
= n3 10,=
n4 50
Q.37 Six cards and six envelopes are numbered 1, 2, 3, (C)=
n1 8,
= n2 6,
= n3 5,=
n4 20
4, 5, 6 and cards are to be placed in envelopes so that
each envelope contains exactly one card and no card (D)=
n1 6,
= n2 12,
= n3 5,=
n4 20

is placed in the envelope bearing the same number


and moreover the card numbered 1 is always placed in Q.43 A ball is drawn at random from box II. If the
envelope numbered 2. Then the number of ways it can probability of drawing a red ball from box I, after this
be done is  (2014) transfer, is 1/3, then the correct option(s) with the
(A) 264 (B) 265 (C) 53 (D) 67 possible values of n1 and n2 is (are)
(A)=
n1 4,
= n2 6

Paragraph 4 (Q.38 to Q.39): Box 1 contains three (B)=
n1 2,
= n2 3
cards bearing number 1, 2, 3 ; box 2 contains five cards
bearing numbers 1, 2, 3, 4, 5; and box 3 contains seven (C)
= n1 10,
= n2 20

cards bearing numbers 1, 2, 3, 4, 5, 6, 7. A card is drawn (D)=
n1 3,=
n2 6
from each of the boxes. Let xi be the number on the
card drawn from the ith box, I = 1, 2, 3.  (2014)
Q.44 A computer producing factory has only two plants
T1 and T2. Plant T1 produces 20% and plant T2 produces
Q.38 The probability that x1 + x2 + x3 is odd, is 80% of the total computers produced. 7% of computers
produced in the factory turn out to be defective. It is
29 53 57 1
(A) (B) (C) (D) known that
105 105 105 2
P (computer turns out to be defective given that it is
Q.39 The probability that x1, x2, x3 are in an arithmetic produced in plant T1)
progression, is = 10 P (computer turns out to be defective given that it
9 10 11 7 is produced in plant T2)
(A) (B) (C) (D)
105 105 105 105 Where P(E) denotes the probability of an event E. A
computer produced in the factory is randomly selected
Q.40 The minimum number of times a fair coin needs and it does not turn out to be defective.
to be tossed, so that the probability of getting at least
Then the probability that it is produced in plant T2 is
two heads is at least 0.96 is  (2015)
 (2016)
36 47 78 75
(A) (B) (C) (D)
73 79 93 83
M a them a ti cs | 18.39

Q.45 A debate club consists of 6 girls and 4 boys. A 1 1 1


team of 4 members is to be selected from this club game against t2 are , and , respectively. Each
2 6 3
including the selection of a captain (from among these
team gets 3 points for a win, 1 point for a draw and 0
4 members) for the team. If the team has to include at
most one boy, then the number of ways of selecting the point for a loss in a game. Let X and Y denote the total
team is  (2016) points scored teams T1 and T2 respectively, after two
games  (2016)
(A) 380 (B) 320 (C) 260 (D) 95 Q.46 P (X > Y) is
1 5 1 7
Paragraph 6 (Q.46 to Q.47): Football teams T1 and T2 (A) (B) (C) (D)
4 12 2 12
have to play two games against each other. It is assumed
that the outcomes of the two games are independent.
The probabilities of T1 winning, drawing and lo sin a Q.47 P (X = Y) is
11 1 13 1
(A) (B) (C) (D)
36 3 36 2

PlancEssential Questions
JEE Main/Boards JEE Advanced/Boards
Exercise 1 Exercise 1
Q.6 Q.9 Q.14 Q.5 Q.12 Q.14
Q.22 Q.25 Q.31 Q.20 Q.22 Q.25
Q.36 Q.39 Q.42
Q.44

Exercise 2 Exercise 2
Q.1 Q.3 Q.7 Q.3 Q.6 Q.9
Q.9 Q.15 Q.16

Previous Years’ Questions Previous Years’ Questions


Q.6 Q.7 Q.9 Q.3 Q.4 Q.7
Q.11 Q.12 Q.11 Q.13
1 8 . 4 0 | Probability

Answer Key

JEE Main/Boards
Exercise 1
Q.1 Yes Q.2 Yes Q.3 1/18 Q.4 0 Q.5 0.5 Q.6 No
Q.7 1/3 Q.8 1.63 Q.9 0.86 Q.10 0, 1, 2, 3 Q.11 No Q.12 0.1
7
1 25 7
Q.13 1/16 Q.14 1.9 Q.15 P(r) = 7Cr   , r = 0, 1, 2, …. 7 Q.16 Q.17
2 1296 12
551
Q.18 0.5 Q.19 1/13 Q.20 6/2197 Q.21(0.4)4 Q.22
15134

X0 1 2
1
Q.23 k=0.1 (i) 0.6 (ii) 0.8 Q.24 144 24 1 Q.25 20
P(X) C6
169 169 169

X 0 1 2 3
12 3 11 2
Q.26. (i) (ii) (iii) (iv) Q.27 21 27 27 1 Q.28 1/6
25 25 25 25 P(X)
55 55 220 220

24 7 2 2
Q.29 Q.30 (i) (ii) Q.31 µ = , σ = 0.38
29 15 15 13
5 4 4 5
 19   19  6  19   19 
Q.32 (i)   (ii)   (iii) 1 –   (iv) 1 –  
 20   20  5  20   20 
4
 9 
Q.33 0.4845 Q.34 0.9185 Q.35 0.00145 Q.36 462(0.24)5 Q.37  
 10 
6 6
2 2 1424
Q.38 233 Q.39 50 Q.40 0.8192 Q.41 (i)   (ii) 1 −   (iii)
5
  5
  3125
50
131 2 3 11
Q.42 Q.43  +  Q.44 2/9 Q.45 1/2 Q.46
243 5 5 1024

41 7
Q.47 Q.48 Q.49 7/15 Q.50 Yes independent
112 15

Exercise 2
Single Correct Choice Type

Q.1 B Q.2 C Q.3 B Q.4 D Q.5 D Q.6 A


Q.7 C Q.8 A
M a them a ti cs | 18.41

Previous Years’ Questions


Q.1 B Q.2 B Q.3 C Q.4 D Q.5 B Q.6 D

Q.7 A Q.8 D Q.9 C Q.10 D Q.11 C Q.12. C

Q.13 B Q.14 C Q.15 D Q.16 A Q.17 D Q.18 C

Q.19 D Q.20 A Q.21 D Q.22 B Q.23 B Q.24 C

Q.25 A Q.26 A Q.27 D Q.28 B Q. 29 D Q.30 A

Q.31 D Q.32 D Q.33 A Q.34 D Q.35 A Q.36 C

Q.37 C

JEE Advanced/Boards
Exercise 1

13 5 −1
Q.1 Q.2 5/9 Q.3 319/420 Q.4 120 Q.5 79 Q.6
24 2
328 1 1
Q.7 1 Q.8 1967 Q.9 Q.10 , Q.11 (i) 1/10 (ii) 3/10 (iii) 2/3
2 625 9 3

Q.12 0.6976 Q.13 19/42 Q.14 283 Q.15 407/576 Q.16 130

125 63 189
Q.17 (i) (ii) Q.18 101/1326 Q.19 Q.20 (1 – p)n–1.[p0(1–p) + np(1– p0)]
163 64 256

3
Q.21 (i) 4 (ii) (iii) 0 Q.22 4/35 Q.23 (i) 0.82 (ii) 0.76
35
1 37
Q.24 (i) (ii) Q.25 20
84 56

Exercise 2

Single Correct Choice Type

Q.1 B Q.2 D Q.3 A Q.4 D Q.5 B Q.6 B

Q.7 D Q.8 B Q.9 D Q.10 C Q.11 D Q.12 B

Q.13 B Q.14 B Q.15 D Q.16 D

Previous Years’ Questions

1 11 11 p p(1 − p) p – 2p2 + p3
Q.1 Q.2 or
or
or Q.3 1/5 Q.4 α = , β= ,γ=
1260 33 22 1 − (1 − p)3 1 − (1 − p)3 1 − (1 – p)3

(3n − 3.2n + 3)× 6


C3 12
C 2 .6 C 4 10
C1 . 2
C1 12
C2 .6 C5 11
C1 . 1C1 1
Q.5 n
Q.6 . + . Q.7
6 18
C6 12
C2 18
C6 12
C2 7
1 8 . 4 2 | Probability

Q.8 B Q.9 A Q.10 B, C, D Q.11 A, D Q.12 A, D Q.13 A, D

Q.14 C Q.15 D Q.16 C Q.17 C Q.18 D

Q.19 A → p; B → s; C → q; D → q Q.20 C Q.21 A Q.22 B Q.23 D

Q.24 C Q.25 B Q.26 D Q.27 B Q.28 B, D Q.29 A

Q.30 A, B Q.31 A Q.32 6 Q.33 5 Q.34 A Q.35 D

Q.36 A Q.37 C Q.38 B Q.39 C Q.40 8 Q.41 5

Q.42 A, B Q.43 C, D Q.44 C Q.45 A Q.46 B Q.47 C

Solutions

JEE Main/Boards ⇒
1 1 1
+ =
6 6 3
Exercise 1 1 1 1
P(A ∩ B) = P(A) P(B) = · =
6 3 18
1 2 3
Sol 1: P (A) = , P(B) = , P(A ∪ B) = Sol 4: P(A) = 0. 3P(B) = 0. 2
4 3 4
P(A ∪ B) = P (A) + P(B) – P (A ∩ B) P(B ∩ A)
P(B / A) =
P(A)
3 1 2
= + – P(A ∩ B) P(A ∩ B) = 0
4 4 3
1 2 3 3+8 – 9 +2 +1 Because it’s given that A and B both are exclusive events
P(A ∩ B) = + – = = =
4 3 4 12 12 6 ∴ P(B / A) = 0
1 2 1
P(A) · P(B) = · = = P(A ∩ B)
4 3 6 Sol 5: P(A) = 0. 4,P(B) = P
So the events are independent. P(A ∩ B) = 0

1 1 A and B are independents so


Sol 2: P (A) = ,P(B) =
2 3 P(A ∩ B) = P(A) · P(B) = 0. 4P
1 P(A ∪ B) = P(A) + P(B) – P(A ∩ B)
P(A ∩ B) =
6
0. 7 = 0. 4 + P – 0. 4P ⇒ 0. 4 + 0. 6P
1 1 1
P(A) · P(B) = · = = P (A ∩ B) 0. 7 – 0. 4 = 0. 3 = 0. 6P
2 3 6
So events A and B are independent. 0.3 1
⇒P= = = 0. 5
0.6 2
Sol 3: A dice has 6 number on it 1, 2, 3, 4, 5, 6
1 x −2 −1 0 1 2
In a thrown probability of getting a member 6 P (A)= Sol 6:
Total number of possibility 6 P(x) 0.1 0.2 −0.2 0.4 0.5

4 has two possibility = 5 or 6 both are greater than 4. The table does not represents probability distribution
as probability never can be negative.
So P(B) = Probability of getting 5 or 6
M a them a ti cs | 18.43

Sol 7: P(A) = 0. 2,P(B) = 0. 3 x 1 2 3


Sol 14:
P(A ∩ B) = 0. 1 P(x) 0.4 0.3 0.3
P(A ∩ B) 0.1 1
P(A / B) = = = Mean = Σ P(x); x
P(B) 0.3 3
= 1(0. 4) +2 (0. 3) +3 (0. 3)
Sol 8: n = 12,P = 1/3 = 0. 4 + 0. 6 + 0. 9 = 1. 9
Standard deviation = npq
Sol 15: A coin is tossed 7 times
1 2 12 1 2
q = 1 – P = 1− = = × × = 1. 63 1
3 3 1 3 3 P (getting heads in one time) =
2
If total number of heads = r
Sol 9: P (A) = 0. 4,P(B) = 0. 7
P(B ∩ A) There is 7Cr way to get r heads in 7 times probability of
P(A / B) = = 0. 6 getting r heads
P(A)
r
⇒ P(B ∩ A) = P(A) 0. 6 = (0. 4) (0. 6) = 0. 24 7 1
Cr   , r = 0 1, 2, 3, 4, 5, 6, 7
P(A ∪ B) P(A) + P(B) – P (A ∩ B) 2
r could be any integer between 0 and 7
= 0. 4 + 0. 7 – 0. 24 = 1. 1 – 0. 24 = 0. 86
5
Sol 16: P(getting 8 in one throw of a pair dice) =
Sol 10: A coin is tossed 3 times 36
X = number of head P (in two successive throws of a pair of dice getting 8
each time)
X = 0 (T, T, T), 1(T, H, T), 2(H, H, T), 3(H, H, H)
5 5 25
X can be 0, 1, 2, 3 = × =
36 36 1296

x 0 1 2 Sol 17: E and F are independent


Sol 11: 1 1 1
P(x) P(E) = 0. 60 and P(E ∩ F) = 0. 35
3 3 6
 P(E ∩ F) = P (E) · P(F)
1 1 1 2 1 5
Σ P(x) = + + = + = ≠1
3 3 6 3 6 6 P | E ∩F | 0.35 7
P(F) = = =
So it’s not probability distribution P(E) 0.60 12

Sol 18: P(A ∪ B) = 0. 7P(A) = 0. 4


x 0 1 2 3 4
Sol 12:
P(x) k 0 3k 2k 4k A and B are independent 50
P(A ∩ B) = P(A) · P(B)
Σ P(x) = 1
 P(A ∪ B) = P(A) + P(B) – P(A ∩ B)
K + 0 + 3k + 2k + 4k = 1
⇒ 0. 7 = 0. 4 + P(B) – P(A) P(B)P(B) (1 – 0. 4) = 0. 7 – 0.
⇒ 10 k = 1
4 = 0. 3
1
k= = 0.1 0.3 1
10 P(B) = =
0.6 2

Sol 13: Total spades in a deck of 52 cards = 13


Sol 19: There is 3 possibility –
13 1
In a time probability of getting one spades = = = P 48
52 4 C2
First two cards are not ace =
P(In two times and both time gets spades) 52
C2
1 1 1 48× 4
= P. P = × = One of them is ace =
4 4 16 51×52
1 8 . 4 4 | Probability

4 ×3 (ii) P(x ≥ 0)
Both are ace =
51×52 = P(x = 0) + P(x = 1) + P(x = 2) + P(x = 3)
Respectively probability of getting ace in third draw
= 0. 2 + 2k + 0. 3 + k = 0. 5 + 3k
4 3 2
⇒ , , = 0. 5 + 3(0. 1) = 0. 5 + 0. 3 = 0. 8
50 50 50
48
C2 4 48 × 4 3x2 4×3 2 Sol 24: Total aces in a pack of 52 cards = 4
P= × + + ×
52
C2 50 52 × 51 50 51 × 52 50
Two cards are drawn with replacement assume
425 1 x = number of getting aces in two drawn
= =
5525 13 So (i) x1 = 0,no aces

Sol 20: There is 4 King, queen and 4 jack is a deck of 52  52 – 4  52 – 4  48 48 122 144
P(x1) =    = · = =
cards. For three drawn  52  52  52 52 132 169
4 4 4
P(One king, one queen, one jack) = · · · 3! (ii) x2 = 1 (one ace)
52 52 52
4 48 24
6 6 P(x2) = 2 × × =
There is 3! types of way to get them = = 52 52 169
169 × 13 2197
(iii) x3 = 2 (both are ace)
Sol 21: Total fires = 4
P = Probability of killed by one bullet = 0. 6 after 4 fire. x 0 1 2
4 4 1
P(x3) = · = 144 24 1
P(still alive) = (1 – 0. 6)4 = (0. 4)4 52 52 169 P(x)
169 169 169

Sol 22 : Drawn randomly 5 cards


Sol 25: Number of way to get 6 numbers from 1 to
x1 , x2 , x3 , x4 , x5 and x1< x2< x3< x4< x5 20 = 20C6

1,2,3… Number of way to get the fixed 6 numbers = 1


… 49,50 1
P=
20
C6
P = Probability of x3 = 30. So, x1 and x2 should be less
than 30 for this number of total way to get x1 and
Sol 26: P(A) = probability of student A passing exam
x2 = 29C2
P (A) = 3/5P(B) = 4/5
And x4, x5>30 ⇒ Total way ⇒20C2
⇒ P(A) and P(B) are independent
Total way for x1 x2 x3 x4 x5 = 50C5
3 4 12
29 20 So, P(A ∩ B) = P(A) · P(B) = · =
C2 C2 551 5 5 25
P= =
50
C5 15134 (i) Both the student passing exam
12
x –2 –1 0 1 2 3 P(A ∩ B) =
Sol 23 : 25
P(x) 0.1 k 0.2 2k 0.3 k
(ii) Only A pass the exam
Σ P(x); = 1(always true) 3 12 15 − 12 3
= P (A) – P(A ∩ B) = − = =
0. 1 + K + 0. 2 + 2k + 0. 3 + k = 1 5 25 25 25

4k = 1 – 0. 1 – 0. 2 – 0. 34k = 0. 4 (iii) Only one of them passing exam


0.4
⇒k= = 0. 1 = P(A) + P(B) – 2P(A ∩ B)
4
(i) P(x ≤ 1) 3 4 2.12 15 + 20 − 24 11
= + − = =
= P(x = –2) + P(x = –1) + P (x = 0) + P(x = 1) 5 5 25 25 25
= 0. 1 + k + 0. 2 + 2k = 3k + 0. 3 = 0. 6 (iv) None of them passing exam
M a them a ti cs | 18.45

= 1 – P(A ∪ B) = 1 + P(A ∩ B) – P(A) – P(B) P(C) = 1/6P(G) = 1/3


12 3 4 25 + 12 − 15 − 20 2 1 1 1
= 1+ − − = = ⇒ P(K) = 1 − − =
25 5 5 25 25 6 3 2
P(R ∩ C) 1
P(R
= / C) =
P(C) 8
A B
P(R) =P(CR) + P(KR) + P(GR) there is 4 choices for each
3 12 question
1 1 1 1
25 25
2 So, P(GR) = ·P(G) = · =
4 3 4 12
25 1
P(KR) 2 24 24
= = =
Sol 27: 3 are defective out of 12 items P(R) 1 1 1 1 1 + 4 + 24 29
× + +
8 6 12 2
A sample of 3 items is selected from the box
Where x = number of defective items
2 white 4 white 3 white
(i) x0 = 0, all are in will condition Sol 30 : 3 black 1 black 7 black
3 I II III
C19 C3 21 12 = 9 + 3
⇒ P(x0) = =
12
C3 55 def B = Ball is black
P(B) = P(BI) + P(BII) + P(BIII)
(ii) x1 = 1, one defective
1 3 1 1 1 7
3
C19 C2 27 = × + × + ×
P(x1) = = 3 5 3 5 3 10
12
C3 55
3 1 7 6+2+7 15 1
(iii) x2 = 2, two defective = + + = = =
15 15 30 30 30 2
3
C29 C2 27 (i) Bag II has max. number of black balls
P(x2) = =
12
C3 220
7
(iv) x3 = 3, all are defective  III  P(IIIB) 30 7
P  = = =
3
C3 B P(B) 1 15
1
P(x3) = = 2
12
C3 220
(ii) Bag II has max. white balls
x 0 1 2 3 1
 II  P(IIB) 15 2
21 27 27 1 So P   = = =
P(x) B P(B) 1 15
55 55 220 220  
2

Sol 28: Two dice are thrown, second dice always Sol 31: Two cards are drawn with replacement
exhibits.
X = number of getting aces from question number 29
S = All possibility: (1, 4) (2, 4) (3, 4) (5, 4) (6, 4) (4, 4)
x 0 1 2
Sum of 8 → (4, 4)
144 24 1
P(x)
x(4, 4) 1 169 169 169
⇒P= =
n(S) 6
 144   24   1 
mean = Σ P(x)x1= 0   + 1  + 2 
Sol 29 : C → Copies ans.  169   169   169 
K → Known ans. 24 + 2 2
= =
169 13
G → Guess ans.
R → Ans. is right σ = SD = Σ P(x)(x1 )2 − (Σ P(x)·(x1 ))2
1 8 . 4 6 | Probability

2 4 5
 144  2  24  2  1   2  5
C  9  1  9 
= (0)2   + 1 .  + 2 . −  = 1  +   = 0. 9185
 169   169   169   13   10  10  10 

1 Sol 35 : n = 10,P = 0. 7
= 24 + 22 − 22
169
x = number of seed germinate
24 P(x = 2) = 10C2 (0. 7)2 (1 – 0. 7)8
= = 0. 377 = 0. 38
13
10 × 9
2 = × (0.49)(0.3)8 = 0. 00145
SD = 0. 38, µ = 2
13
Sol 36 : P(F) = P(step forward) = 0. 4
Sol 32 : Total choose bulb = 5
P(B) = P(step backward) = 0. 6
P = P (bulb will fuse after 150 days) = 0. 05
N = 11 (number of step forward)
19
q = 1 – P = 0. 95 = , n = 5, xF + xB = 11| xF – xB| = 1
20
x = number of fuse ⇒ | 11 – 2xB | = 1
5
 19 
(i) P(none of 5 will fuse) = (q)5 =   11 − 1
 20  ⇒ xB = = 5 or 2xB–11 = 1
2
(ii) P(not more than ore) = P(x ≤ 1)
⇒ 2xB = 1 + 11 = 12
= P(x = 0) + P(x = 1) = (q) + C1 q P
5 5 4 1
xB = 6 xF = 11 – 5
5 4 5
 19   19   1   19   20 1  or 11 – 6 = 6 or 5
=   +    =   1 + · 
 20   20   20   20   19 20 
P(xF = 6, 5)= P (x = 6) + P (x = 5)
5 4
20  19   19  = 11C6 (0. 4)6 (0. 6)5 + 11C5 (0. 4)5 (0. 6)6
= ×  =  
19  20   20  = 11C5 [(0. 4 + 0. 6) (0. 24)5]
4
6  19  = 11C5 (0. 24)5 = 462 (0. 24)5
(iii) P(x > 1) = 1 – P(x = 0) – P(x = 1)= 1 −  
5  20 

 19 
5 Sol 37 : 4 balls are drawn with replacement from the
(iv) P(x ≥ 1) = 1 – P(x = 0) = 1 −   bag. Assume x = number ball marked with 6
 20  4 4
 10 − 1   9 
P(x = 0) =   = 
Sol 33 : x = number of hurdles; he down  10   10 

Total hurdles = 10 = n ( There is only one ball which is marked 6 out of 10)
P = 5/6 ⇒ q = 1 – P = 1/6
Sol 38 : 6 dice are thrown assume x5 = number of dice
P(x < 2) = P(x = 0) + P(x = 1) which show 5 or 6
10 9
5 5 1 2 1 4 2
=   + 10 ×   × =
0.4845 P= = ,q= =
6 6 6 6 3 6 3
P (x5≥ 3) = P(x = 3) + P(x = 4) + P(x = 5) + P(x = 6)
Sol 34 : Total ship = 5 3 3 4 2
2 4 2 4
X = n (arrive safe ships) = C3     +6 C 4    
6

6 6 6 6


1 9
q= ⇒P=1–q= 5 6
10 10 2 4 2 233
+6 C5     +6 C6   =
6 6
    6
  729
P(x ≥ 4) = P(x = 4) + P(x = 5)
M a them a ti cs | 18.47

233 112 243 – 112 131


Expected number → 729 × = 233 = 1=
– =
729 35 243 243

1
Sol 39 : 200 families, P(B) = = P (G) Total children =
2 Sol 43: (p + q)n
4 for each family
4 variance s2 = npq mean µ = np
1 4 1
P(3B, 1G) = C1 ×   =
4
= Given → np + npq = 42
2 4×4 4
⇒ np (1 + q) = 42  …(i)
1
expectation → 200 × = 50 families
4 and (np) (npq) = 360
⇒ (np)2 q = 360 …(ii)
Sol 40 : P = probability of success of a operation
2 2
(np) (1 + q) 422 49
P = 0. 8, n = 4x = number of successful operation (1)2 (2) → = =
2
(np) q 360 10
P(x ≥ 3) = P(x = 3) + P(x = 4)
⇒ 10(1 + q2 + 2q) = 49 q
= [4C3 (0. 8)3 (1–0. 8) + 4C4(0. 8)7]
⇒ 10q2 + 202 – 49q + 10 = 0
= [4 × 0. 83 · 0. 2 + (0. 8)4]
⇒ 10q2 – 29q + 10 = 0
= 0. 84 × 2 = 0. 8192
⇒ 10q2 – 25q – 4q + 10 = 0
Sol 41 : P = Probability of graduate student ⇒ 5q (2q – 5) – 2 (2q – 5) = 0
P = 0. 6 ⇒ (2q – 5) (5q – 2) = 0
⇒ q = 1 – 0. 6 = 0. 4 5 2
q= or , q < 1
x = number of students will graduate 2 5
2 3
6 So, q = ⇒P=1–q=
2 5 5
(i) P(x = 0) = 6C0 (0. 4)6 =  
5 and (np)2q = 360
6
2
(ii) P(x ≥ 1) = 1 – P (x = 0) = 1 −   360 360 × 53
5 n2 = =
3
2 2×9
(iii) P(x ≤ 3) = P (x = 0) + P(x = 1) + P(x = 2) + P(x = 3) 215 ×  
5
50
2
6
2 3
5
2 3
4
2 3
2 3 3 2 3
=   +6 C1   +6 C2     +6 C3     n = 10 × 5 n = 10 × 5 = 50⇒  + 
2 2 2

5 5 5 5 5 5  5 5 5

1 Sol 44 : R1 = Ist ball is red


= [26 + 6 · 3 · 25 + 15 · 24 · 32 + 20 · 23 · 33]
56
3 red
7120 1424 R2 = 2nd ball is red
= = 7 black
56 3125 R 
Total 2 balls are related without replacement P  1  > ?
Sol 42 : P = P(bomb hitting a target)  R2 
P(R1 ∩ R 2 )

1 1 2 P(R 2 )
P= ⇒ q = 1− =
3 3 3
7 3 3 2 21 + 6 22 3
Two bombs are enough to destroy target P(R2)2 = × + × = = =
10 9 10 9 90 90 10
n = 5x = number of bombs that hit the target
3 2 6 1
so for destroy bridge P(R1∩ R2) = × = =
10 9 90 15
P(x ⇒ 2) = 1 – P(x = 0) – P(x = 1)
 R  1 / 15 10 2
 2 5 P  1  = = =
 2  1
4
 5 1
= 1 −   + C1   ×  = 1 − [25 + 5.24 ]  R 2  3 / 10 3 × 15 9
 3   3  3  35

1 8 . 4 8 | Probability

Sol 45: Five dice are thrown Since both events are independent
Success = odd number 1 2 2
So, P(A ∩ B) = P(A) P(B)= · =
There is number of odd number and even number are 3 5 15
same which is 3 (1, 3,5 and 2, 4, 6) 1 2 2 5+6– 4 7
⇒P= + –2· = =
So, P(success) = P(no success) 3 5 15 15 15

but P(success) + P (no success) = 1


Sol 50 : E = 1st throw is head
 There is either odd or even number
F = Last throw is tail
1
P (success) = 1 2 2 1 2 2 1 1
2 P(E) = × − = P(F)= · − =
2 2 2 2 2 2 2 2

Sol 46: n = 10x = number of getting even number P(E∩F) = P(1st is head and last is tail)

1 1 1 1 2 1 1
P= ⇒ q = 1− = = × × =
2 2 2 2 2 2 4
1
P(x ≥ 9) = P(x = 9) + P(x = 10) P(E) · P(F) = = P(E ∩ F)
4
10 10 10
10 1 1 1 11 11 So, E and F are independent.
= C9   + 1  =   (1 + 10) = =
2 2 2 210 1024

Exercise 2
4 red 3 red 5 red
Single Correct Choice Type
4 green 5 green 2 green
Sol 47: A B C
Sol 1: (B) P(special card at nth drawn)
↓ ↓ ↓
one ball one ball one ball 51 1 1
⇒ · =
52 51 52
= 2 green balls and one is red
P(2G, 1R) = P[(GRG) + (GGR) + (RGG)] Sol 2: (C) n = 4 (r person)

4 5 5 3 2  4 5 2 1
P= (correct ans. by one)
P(2G, 1R)=  · + · + × · 6
8 8 7 8 7  8 8 7
x = number of correct ones
1  25 6  5 1 1 41 4 4
=  + + = × [25 + 6 + 10] =  1 5
2  56 56  56 56 2 112 P(x ≥ 1) = 1 – P(x = 0) = 1 −  1 −  = 1 −  
 6 6
4 red Sol 3: (B) n = 2
Sol 48: Two balls are drawn from 3 black
x = get spade
3 white
Total spades in 52 cards = 13
7
C2 7
P(none of the ball is white)= = P = P(he fails exactly first two times)
10
C2 15
 52 – 13  52 – 13  13 39 39 13 9
1 P=   · = · · =
Sol 49: P(A) = probability of A selection =  52  52  52 52 52 52 64
3
2
P(B) = Sol 4: (D) 5 is to be chosen from 9 people there is a
5
couple in group of 9
Total post = 2
P = P(couple chosen)
P(only one of them will selected) = P(A) + P(B) – 2P (A ∩ B)
q = P(couple don’t chose)
M a them a ti cs | 18.49

1 7 7 2 1
P+q=  C5 + C3 .  Sol 8: (A) P = events
9
C5  2 2
P = probability of win test match
1.2.3. 4  7.6 7.6.5  7.8. 4 4
=  + = = L = Lose match; W = win match
9.8.7.6  1.2 2.3  9.8.7 9
1 1
P(L) = 1 − =
Sol 5: (D) P(n) = probability of shown in fall 2 2
There is 5 match series.
 1  P(1 ∩ (1 ∪ 2) P(1)
P = = Total possibility = 25
 1 ∪ 2  P(1 ∪ 2) P(1) + P(2)
P(India’s second win occurs at the 3rd test)
\P(1 ∩ 2) = 0
P(LWW) + P(WLW)
0.1 0.10 5
= = = 1 1 1 1 1 1 1 1 1 1
0.1 + 0.32 0.42 21 = × × + × × = × × ×2 =
2 2 2 2 2 2 2 2 2 4
Sol 6: (A) P(A + B) = P(A) + P(B) – P(AB)
and P(A), P(B) > P(AB) Previous Years’ Questions
∴ P(A + B) > P(A), (AB)
Sol 1: (B) Given that , P(A) = 0.4, P(A) = 0.6
and P(A), P(B), P(AB), P(A + B) ≥ 0
P(the event A happens at least one
∴ option (A) is correct
= 1 – P(none of the event happens)
P(AB) ≤ P(A) ≤ P(A + B) ≤ P(A) + P(B)
= 1 – (0.6) (0.6) (0.6) = 1 – 0.216 = 0.784
Sol 7: (C) x ∈ {1, 2,……,50}
Sol 2: (B) Since, P(A / B) + P(A / B) =
1
∴ P(A / B) = 1 – P(A / B)

Sol 3: (C) Given, P(A ∪ B) = 0.6, P(A ∩ B) = 0.2


x1 x2 ∴ P(A) + P(B) = [1 – P(A)] + [1 – P(B)]
region = 2 – [P(A) + P(B)] = 2 – [P(A ∪ B) + P(A ∩ B)]

100 = 2 – [0.6 + 0.2] = 1.2


x+ > 50; ⇒ x2 – 50 x + 100 > 0
x
Sol 4: (D) Let X be the number of coins shoeing heads.
50 ± 502 − 4(100) Let X be a binomial variate with parameter n = 100 and p.
⇒x =
2
Since, p(X = 50) = P(X = 51)
1 10 21
x = 25 ± 2500 − 400 ; x = 25 ±
2 2 ⇒ 100
C50P50 (1=
− p)50 100
C51(p)51(1 − p)49
x = 25 ± 5 2 (100)! (51!) × (49!) p
⇒ =
x1 = 2. 0871 and x2 = 47. 912 (50!)(50!) 100! 1− p
In region integer y ∈ {3, 4,……,47} p 51 51
⇒ = ⇒=p
So only for x = 1, 2, 48, 49, 50; 1 − p 50 101
100
y is greater than 0 or x + > 50
x Sol 5: (B) The number of ways of placing 3 black balls
n(1,2, 48, 49,50) 5 1 without any restriction is 10C3. Since, we have total 10
⇒P= = = places of putting 10 balls in a row. Now the number of
50 50 10
ways in which no two black balls put together is equal
1 8 . 5 0 | Probability

to the number of ways of choosing 3 places marked Sol 9: (C) Let E = event when each American man is
‘……….’ Out of eight places seated adjacent to his wife and
–W–W–W–W–W–W–W– A = Event when Indian man is seated adjacent to his wife.
The can be done in C3 ways
8
Now, n(A ∩ E) = (4!); × (2!)5
8
C3 8×7×6 7 Even when each American man is seated adjacent to
\ Required probability =
= =
10
C3 10 × 9 × 8 15 his wife
Again, n(E) = (5!) × (2!)4
Sol 6: (D) It is given that P(E) ≤ P(F) ⇒ E ⊆ F  ….. (i)
n(A ∩ E) (4!) × (2!)5 2
∴ P(A / E) = = =
and P(E ∩ F) > 0 ⇒ E ⊂ F  ….. (ii) n(E) (5!) × (2!)4 5
(a) Occurrence of E ⇒ occurrence of
Alternate Solution: Fixing four American couples and
F [from Eq.; (i)] one Indian man in between any two couples; we have 5
(b) Occurrence of F ⇒ occurrence of E different ways in which his wife can be seated, of which
2 cases are favourable.
[from Eq. (ii)]
2
(c) Non-occurrence of E ⇒ non-occurrence of F ∴ Required probability =
5
[from Eq. (i)]
2
Sol 10: (D) Since, P(A) =
3 1 5
Sol 7: (A) Given, P(B) = (A ∩ B ∩ C ) = and
4 3 For independent events,
2
P(A ∩ B) = P(A)P(B) ⇒ P(A ∩ B) ≤
A B (ABC) 5
1 2 3 4
(ABC) ⇒ P(A ∩ B) = , , ,
10 10 10 10
(BC) (Maximum 4 outcomes may be in A∩B)
1
C 1. Now, P(A ∩ B) =
10
1 1
P = (A ∩ B ∩ C) = Which can be shown in Venn ⇒ P(A).P(B) =
diagram. 3 10
1 5 1
∴ P(B ∩ C) = P(B) – {P(A ∩ B ∩ C ) + P( A ∩ B ∩ C ) ⇒ P(B) = × = , not possible
10 2 4
3 1 1 3 2 1
= – +  = − = 2
4  3 3  4 3 12 2. Now, P(A ∩ B) =
10
2 2
Sol 8: (D) Since, three distinct number are to be ⇒ × P(B) =
5 10
selected from first 100 natural numbers.
5
⇒ n(S) = 100C3 ⇒ P(B) = ,
10
E(fuvourable events) = All three of them are divisible by both Outcomes of B = 5
2 and 3.
3
⇒ divisible by 6 i.e., {6, 12, 18, …… , 96} 3. Now, P(A ∩ B) =
10
Thus, out of 16 we have to select 3. 3 2 3
⇒ P(A) P(B)= ⇒ × P(B) =
∴ n(E) = 16C3 10 5 10
16
C3 3
4 P(B) = , not possible
∴ Required probability = = 4
100
C3 1155
4
4. Now, P(A ∩ B) =
10
M a them a ti cs | 18.51

Sol 14: (C) A = {4, 5, 6}, B = {1, 2, 3, 4}


4 Obviously P(A ∪ B) = 1
⇒ P(A).P(B) =
10
⇒ P(B) = 1, outcomes of B = 10 Sol 15: (D) Mean of a, b, 8, 5, 10 is 6
a + b + 8 + 5 + 10
Sol 11 : (C) Sample space A dice is thrown thrice, ⇒ 5
=
5
n(s) = 6 × 6 × 6.
⇒ a+b =7  …(i)
r r r 2
Favourable events ω1 + ω2 + ω3 =0 ∑(x1 − A)
∴ Variance =
i.e. (r1, r2, r3) are ordered 3-triple which can take values. n
(a − 6)2 + (b − 6)2 + 4 + 1 + 16
(1,2,3), (1,5,3), (4,2,3), (4,5,3)
=  = 6.8
 5
(1,2,6), (1,5,6), (4,2,6), (4,5,6) 
⇒ a2 + b2 = 25
i.e. 8 orc and each can be arranged in 3! Ways = 6
a2 + (7 − a)2 = 25 (Using (i))
∴ n(E) = 8 × 6
8×6 2 ⇒ a2 − 7a + 12 = 0
⇒ P(E) = =
6×6×6 9 ∴= a 3,3 and = b 3, 4.

Sol 12: (C) From the tree-diagram it follows that Sol 16: (A) x1 + x2 + x3 + x5 =
6
S
5 + 6 −1
C5−1 =10 C 4
4/5 1/5
Sol 17: (D) Other than S, seven letters M, I, I, I, P, P, I can
G G 7!
be arranged in = 7 . 5 . 3.
3/4 1/4 1/4 2! 4!
3/4
Now four S can be placed in 8 spaces in 8 C 4 ways.
AG AR AR AG
Desired number of ways = 7 . 5 3 . 8 C 4 7 . 6 C 4 . 8 C 4
1/4 1/4
3/4 3/4 1/4 3/4 3/4
n
1/4 (a + 1)
sum of quantities 2
BG BR BG BR B 18: (C) Mean (x) =
Sol =
n n
46 1
P(BG) = = [1 + 1 + 100d] = 1 + 50d
80 2
10 5
P(BG|G) = = 1
16 8 M.D. = ∑ | x1 − x |⇒ 255
n
5 4 1
∴ P(BG∩ G) = × =
8 5 2 1
= [50d + 49d + 48d + ..... + d + 0 + d + ..... + 50d]
101
1 / 2 1 80 20
P(G/BG) = = × = .
P(BG ) 2 46 23 2d  50 × 51 
=  
101  2 
P(A ∩ B) 1 P(A ∩ B) 2
Sol 13: (B)
= = , 255 × 101
P(B) 2 P(A) 3 ⇒ =d = 10.1
50 × 51
P(A) 3
Hence = . (But P (A) = 1/4)
P(B) 4 Sol 19: (D) 4 novels can be selected from 6 novels
1 in 6 C 4 ways. 1 dictionary can be selected and 3
⇒ P(B) = dictionaries in 3 C1 ways. As the dictionary selected
3
1 8 . 5 2 | Probability

is fixed in the middle, the remaining 4 novels can be Common difference is 6 ; total number of cases = 2
arranged in 4! Ways.
17 + 14 + 11 + 8 + 5 + 2 1
=
∴ The required number of ways of arrangement Prob. =
6 3
20
C4 85
= C 4 × C1 × 4!= 1080

10
Sol 20: (A) S = {00, 01, 02, …., 49} 10!
Sol 23: (B)=
S1 ∑ j( j − 1) j( j − 1)( j − 2)!(10 − j)!
Let A be the even that sum of the digits on the selected j=1

ticket is 8 then 10
8!
= 90∑ = 90 ⋅ 28
A = {08, 17, 26, 35, 44} ( j − 2)!(8 − ( j − 2))!
j=2
Let B be the event that the product of the digits is zero
10
10!
B = {00, 01, 02, 03, ….., 09, 10, 20, 30, 40} S2 = ∑
j=1 j( j − 1)!(9 − ( j − 1))!
A ∩B ={8}
1 10
9!
P(A ∩ B) 50 1 = 10∑ = 10 ⋅ 29
Required probability P(A /=
B) = = ( j − 1)!(9 − ( j − 1))!
P(B) 14 14 j=1

50
10
10!
Sol 21: (D) Statement-II is true S3 = ∑ [ j( j − 1) + j]
j=1 j!(10 − j)!
Statement-I: Sum of n even natural numbers = n (n + 1)
10 10
n(n + 1)
Mean (x)= = n+1 = ∑ j( j − 1) 10C j = ∑ j 10C j = 90 ⋅ 28 + 10 ⋅ 29
n j=1 j=1

Variance
= 90 ⋅ 28 + 20 ⋅ 28 = 110 ⋅ 28 = 55 ⋅ 29
1  1
=  ∑(x1 )2  − (x) = [22 + 42 + ..... + (2n)2 ] − (4 + 1)2
n  2 Sol 24: (C) The number of ways
= 3
C2 ×9 C2

1 2 2 9×9
= 2 [1 + 22 + ...... + n2 ] − (n + 1)2 =3 × =3 × 36 =108
2 2
4 n(n + 1)(2n + 1)
= − (n + 1)2
n 6 Sol 25: (A) n(S) =9 C3

(n + 1)[2(2n + 1) − 3(n + 1)] n(E) = 3 C1 ×4 C1 ×2 C1


=
3
3 × 4 × 2 24 × ! 24 × 6 2
(n + 1)[4n + 2 − 3n − 3] = = = × 6! =
=
9
C3 9! 9×8×7 7
3

(n + 1)(n − 1) n2 − 1 Sol 26: (A) σ2x =4 σ2y = 5 x =2, y = 4


= = ' '
3 3
∑ xi
∴ Statement-I is false. = 2 ∑=xi 10; y=
i 20
5

Sol 22: (B) N(S) = 20


C4 1  1
σ2x =  ∑ xi2  − (x)2 = ( ∑ y i2 ) − 16
 2  5
Statement-I:
Common difference is 1 ; total number of cases = 17 ∑ xi2 =
40 ∑ y i2 =
105
,
Common difference is 2 ; total number of cases = 14 2

Common difference is 3 ; total number of cases = 11


2
σ=
2
1
10
( x+y
∑ xi2 + ∑ y i2 −  
 2 
)
Common difference is 4 ; total number of cases = 8 1 145 − 90 55 11
= (40 + 105) − 9 = = =
Common difference is 5 ; total number of cases = 5 10 10 10 2
M a them a ti cs | 18.53

Sol 27: (D) Number of ways of selecting one or more


∑ xi2
balls from 10 white, 9 green, and 7 black balls Sol 31: (D) Variance
= − (x)2
N
=(10 + 1)(9 + 1)(7 + 1) − 1 2
22 + 42 + ..... + 1002  2 + 4 + .... + 100 
= 11 × 10 × 8 − 1 = 879 . =⇒ σ2 − 
50  50 

Sol 28: (B) Let A be the event that maximum is 6. 4(12 + 22 + 32 + ... + 502 )
= − (51)2
B be event that minimum is 3 50

5  50 × 51 × 101  2
C2 = 4  − (51) = 3434 – 2601
P(A) = (the numbers < 6 are 5)  50 × 6 
8
C3 2
⇒ σ = 833
5
C2
P(B) = (the numbers > 3 are 5)
8
C Sol 32: (D) Number of integer greater than
3

2 6000 may be 4 digit or 5 digit


C
P(A ∩ B) = 1
8
C3 C-1 when number is of 4 digit
2
 B  P(A ∩ B) C1 2 1 C-2 when number is of 5 digit = 5! = 120
Required probability is P  =  = = =
A P(A) 5
C2 10 5 total = 120 + 72 = 192 digit
{6, 7, 8}

Sol 29: (D) If initially all marks were x1 then

∑(x − x)2 3 4 3 2 = 72
σ12 = 1
N
Sol 33: (A) There seems to be ambiguity in the question.
Now each is increased by 10 It should be mentained that boxes are different and
one particular box has 3 balls:
∑[(x1 + 10) − (x + 10)]2
σ22 = =σ12
N Then,
12 11
C3 × 29 55  2 
So, variance will not change whereas mean, median Number=
of ways =  
and mode will increase by 10. 312 3 3

1 1 5 x1 + x2 ........x16
Sol 30: (A) P(A ∪ B) = ⇒ P(A ∪ B) =1 − = Sol 34: (D) = 16
2 6 6 16
If x1 = 16
1 1 3
P(A) = ⇒ P(A) =1 − =
4 4 4 x1 + x2 .........x10 − 16 + 3 + 4 + 5

 P(A ∪ B)= P(A) + P(B) − P(A ∩ B) 18


16 × 10 − 16 + 12 240 + 12 252
5 3 1 = = = = 14
= + P(B) − 18 18 29
6 4 4
1 Sol 35: (A) Standard deviation of numbers 2, 3, a and
P(B) =
3 11 is 3.5

 P(A) ≠ P(B) so they are not equally likely. ∑ x12


∴ (3.5)2 = − (x)2
3 1 1 4
Also P(A) × P(B) = × = = P(A ∩ B) 2
4 3 4 2 4 + 9 + a2 + 121  2 + 3 + a + 11 
⇒ (3.5)
= − 
4  4 
 P(A ∩ B)= P(A) ⋅ P(B) so A and B are independent.
On solving, we get 3a2 – 32a + 84 = 0
1 8 . 5 4 | Probability

Sol 36: (C) E1 : {(4, 1), ……..(4, 6)} 6 cases Sol 2: two dice throw possibility 10
E2 : {(1, 2),…….. (6, 2)} 6 cases ⇒ (4, 5) (1, 5) (2, 5) (3, 5) (6, 5)……
E3 : 18 cases (sum of both are odd)}
6 1
= At least one should be greater than 4
P(E1=
) = = P(E2 )
36 6 6 ×1 + 1× 6 − 1 6 ×1 + 1× 6 − 1 2
⇒P= + –
36 36 36
18 1
P(E=
3) = 20 5
36 2 P= =
36 9
1
P(E1 ∩ E2 ) =
36
Sol 3: Odds for A, B, C, D
1
P(E2 ∩ E3 ) = ⇒ 1: 3, 1: 4, 1: 5, 1: 6
12
1 1 1
1 P(A) = = , P(B) = ,
P(E3 ∩ E1 ) = 1+3 4 5
12
1 1
P(C) = , P(D) =
P(E1 ∩ E2 ∩ E3 ) =
0 6 7

∴ E1 ,E2 ,E3 are not independent P(one of them will wins)


= P(A) + P(B) + P(C) + P(D)
Sol 37: (C) SMALL 1 1 1 1 319
= + + + =
4! 4 5 6 7 420
A _ _ _ _# = 12
2!
Sol 4: Assume number of roses = x
L _ _ _ _# 4! = 24
P(sweety win when she start first) = P(S)
4!
M_ _ _ _# = 12 x 602 x
2! = + +…
(60 + x) (60 + x)3
3!
SA _ _ _ _# =3 2
2! x   60  
= 1 +   + …
SL _ _ _# 3! = 6 60 + x   60 + x  

S M A L L #1  
 
x  1  = 60 + x
58th position = 2
60 + x   60   720 + x
1 −   
  60 + x  

JEE Advanced/Boards 60 + x
So P(sweety wins) = P(w) = 1 −
120 + x
Its given that(s) = 3P (w)
Exercise 1
60 + x 3.60
5 science 3 science =
120 + x 120 + x
Sol 1: 3 engg. 5 engg.
x = 180 – 160 = 120
I II
E = engg. subject select Sol 5: Total roll = 9(3, 3, 3)
if, 3 or 5 came after thrown a dice then, select a subject
3 3 2 2 1 1
from I otherwise from II m 3 C1 C1 C1 2 C1 C1 C1 1 C1 C1 C1 9 m
= . . = =
2 3  4 5 1 5 3 + 10 13 n 9
C3 6
C3 3
C3 70 n
⇒ P(E) =   × +  · = + = =
 6  8  6  8 8 12 24 24 m + n = 9 + 70 = 79
M a them a ti cs | 18.55

1
⇒ P4 + P 4 – 2P+21 =
2
4P4 + 4P2 + 1 = 0
(2P2 – 1)2 = 0
⇒ 2P2 – 1 = ⇒ P2 = 1/2 or
1
P=  0≤P≤1
Sol 6: Probability to head shown = P’ 2
P’(A wins, when first A start)
Sol 8: P = P(out cons of the 5th throw was already thrown)
= P’ + (1 – P’) P’ + (1 – P’) P(1 – P’) P’ +…
2 2
P = 1 – P (out com of 5th throw was first line throw)
= P[1 + (1 − P')P'((1 – P')P')2 +((1 – P')P')3 + …]
5 5 5 5 1 
P = 1−6 × × × ×
 1  P'  6 6 6 6 6 
= P'   = 1 − P'+ P2
 1 − P'(1 – P')  625 671 a
P = 1− = =
P 1296 1296 b
P(B win) = 1 −
1 – P'+ P'2
a + b = 671 + 1296 = 1967
1 − P'+ P'2 − P' 1 − 2P'+ P'2
= =
1 − P'+ P'2 1 − P'+ P'2 Sol 9: x = number of bomb hitting on target

For first to both n=4

P(A) = P(B) P = 0. 4(probability of hitting the target for and bomb)


x ≥ 2 for destroy bridge
1 − 2P'+ P2 P'
⇒ =
1 − P'+ P'2 1 − P'+ P'2 P(x ≥ 2) = P(x = 2) + P(x = 3)+ P(x = 4)

⇒ P’2 – 3P’ + 1 = 0 = 4C2 (0. 4)2 (0. 6)2+4C3(0. 4)3 (0. 6)1+4C4(0. 4)4

⇒ P’ + P = 1⇒ P’ (1 – P) 4×3 328
= × (0. 24)2 + 4 (0. 064) (0. 6) + (0. 0256) =
⇒ (1 – P) – 3 (1 – P) + 1 = 0
2 2 625

⇒ P2 – 2P + 1 – 3 + 3P + 1 = 0
Sol 10: assume P = Ist event’s probability
⇒ P2 + P – 1 = 0
q= IIndevent’s probability
−1 ± 1 + 4 5 −1 P = q2
P= = =P
2 2 1 −P
0 < P < 1 Odds against the first =
P

a b 1−q
odds against the first =
Sol 7: a, b, c, d → integer q
c d
3
| D | = ad – bc 1 −P 1 − q 1 − q2
= = 
q  q  q2
if (D) is even than
a or d is even (1 − q)(1 + q) (1 − q)(1 − q)2
⇒ =
ad and bc are even and q2 q3
ad and bc are odd b or c is even q + q2 = 1 + q2 – 2q
3q = 1⇒ q = 1/3
a, d are odd and b, c are odd 2
1 1 1 1
So P = (q)2 =   = ; (P, q) →  , 
1 3 9 9 3
P(|D| is even) = (P)4 + (1 – P2) (1 – P2) =
2
1 8 . 5 6 | Probability

Sol 11: Total tubes = 5(3G, 2D) Sol 14: Total side = 6
defective = 2(D) B ⇒ one red , 2 blue, 3 green
(i) Test stopped on the 2nd test P(second blue result occurs on or before the tenth)
C2 1 × 2 1
2
= 1 – P(second blue result occurs after 10th)
P= = =
5
C2 5 × 4 10 10 9
4  4   2  10 610 – 410 – 410 × 5
=1–   −     C1 =
(ii) Test stopped on 3rd test 6 6 6 610
(DGD) + (GDD) + (GGG) 610 – 6.410 69 − 410 39 − 211
= = =
2
C1 ×3 C1 × 1 3
C1 × 2 × 3 3 × 2 × 1 610 69 39
⇒ + +
5
C3 5× 4×3 5× 4×3 3P − 2q
= , P = r, q = 11, r = 9
1 1 1 3 3r
= + + =
10 10 10 10 P2 + q2 + r2 = 92 + 92 + 112 = 283
1 1
+ 1
 first tube is non − defective 
(iii) P  10 10 = 2 Sol 15: Probability of good book (G) =
 = 3 2
 test stopped on 3 test 
rd
3
10 1
So P(bod book) (B) =
2
Sol 12: Hitting the plane at first = 0. 4
Hitting the plane at 2nd = 0. 3 P P  1
P   = 2/3 and P   =
G B 4
Hitting the plane at 3rd = 0. 2
n=2
Hitting the plane at 4rd = 0. 1
P(at lead one book published)
P = P(gun hits the plane)
= 1 – P(no-book published)
= 1 – (1 – 0. 4) (1 – 0. 3) (1 – 0. 2) (1 – 0. 1)
= 1 – P(GP’ BP’) – P(GP’ GP’) – P(B’P BP’)
= 0. 6976
2 2
1 1 1 3 1 1 1 1 1 3 
= 1 − 2× × × × − × ×  − ×  
Sol 13: Total articles = 10 2 2 3 4 2 2 3 2 2 4 
Defective (D) = 4
169 407
= 1− =
Non-defective (R) = 6 576 576
Number of chosen articles = 6
1
⇒ 10C6, x = number of defective articles Sol 16: P(A) =
3
P (batch will be rejected)
 B   B  1  B 
P   = 1, P   = , P  =0
= 1−
P(x =0) + P(x =1) + P(x =2) A
 2  A1  2  A 0 
1 Assume x = number of trial when A occur
4 4
 C 6
C C 6
C C  6
n=4
= 1 −  10 0 + 105 1 + 104 2 
 C6 C6 C6  P(B) = 1. P(x ≥ 2) + 0: P(x = 0) + 1/2 P(x = 1)
 
 1 + 6 × 4 + 15 × 6  P(x = 1)
= P(x = 2) +P(x = 3) + P(x = 4) + 0+
=  10 × 9 × 8 × 7  2
 
 1·2·3· 4  2 2 3
1 2 1 2
= 4 C2 ×   ×   + 4 C3  
115 210 − 115 95 19
3 3 3 3
= 1−= = = 4 3
210 210 210 42 1  1  2  1
+ 4 C 4   + 4 C1    ×
3  3  3  2
M a them a ti cs | 18.57

1  4 49 m 1  4! 22.4! 
=  6× 4 + 4×2 +1 + 4×  = = − 8 
+ + 4 × 2 × 3
81  2 81 n 2  2!2! 2!2! 
⇒ m + n = 49 + 81 = 130 1 256 – 67 189
= 1− [1 + 12 + 30 + 24] = =
28 256 256
Sol 17: faces → 1, 2, 3, 4
Sol 20: Total passengers before stop = n
When two dice thrown together
P(get down) = P
(i) Exactly 6 on each of successive throws
P(boarding the bus at next stop) = 1 – P0
1 2 2
(4,2) × = P(n passenger are in bus after stop)
4 4 10
2 1 2 = (1 – P) nP0 + nC1 P(–P)n–1 (1 – P0)
6 (2, 4) × =
4 4 16 = (1 – P)n–1 [P0(1 – P) + nP(1 – P0)]
1 1 1
(3,3) × =
4 4 16 Sol 21: Total balls = 2n (n while, n black)
⇒ For 3 throws n person each draw 2 balls
3 3
 2 2 1   5  125 (i) P(each of n person drawn both balls)
⇒  + +  ⇒  = 3
 16 16 16   16  16 2n (n × n)(n – 1)2 (n – 2)2
= − 32.22
(ii) More than 4 on at least are of the three throws. 2n!
⇒ 1 – P(less than 4 or 4 on all thrown) n | n | 2n 2n 8
= = =
⇒ Less than 4 or 4 → (2, 2) 2n 2n
Cn 35
3 24 16 × 4 × 3 × 2 8
2 2 1 1 64 – 1 63 n=4→ = =
⇒ × =⇒ 1 −   = = C4
8
1× 7 ×1×5 35
4 4 4 4 64 64
So n = 4
Sol 18: Total red card out of 52 = 26 (R) (ii) n = 4
Total green = 4 (Q) Each of 4 draw the balls of same colour
q card are red and green both (RQ)
Two cards drawn
⇒ P(one is red & one is green)
4
C1 ×3 C12C11C1 × 4! 4
⇒ [26 × 2 + 25 × 1 + 25 × 1 – (1) (1)] P= × C2 .22
8! 2!2!
101
⇒ 4! 4 × 3 × 2 × 1 3
1326 P= 4×6=
4 × 8 × 7 × 6 ×5× 4 35
Sol 19: Total coin = 4 (iii) n = 7 there is 7 white balls and 7 black ball there is
Discard those which turn up tails no way to each of the person draw balls of same colour.
P = (at least 3 coins discard after 2nd flip)  Number of same color ball is 7 (odd)
= P(3) + P(4) = 1 – P(0) – P(1) – P(1) 7 = 2 + 2 + 2 + 1 – ? (1)⇒not possible
= 1 – [P(HHHH, HHHH)] – P[(HHHT, HHH) + (HHHH, HHHT)] P=0
– P [(HHHH, HHTT) + (HHTT, HH) + (HHHT, HHT)]
8 Sol 22: Between Pi and Pj
1 1 4
= 1 −   − 8  C1 × 2 × 1 + C1 
4
If i < j → Pi win
2 2
P(player P4, reaches the Final)
1 8 . 5 8 | Probability

P4 P( A ∩ B) = 0. 58
P(A ∩ B ) = P(only A)

P4 = P(A) – P(A ∩ B) = 0. 4 – 0. 22 = 0. 18
P[( A ∩ B) ∩ (A ∩ B )] = 0 from dia.
So P[( A ∩ B) ∪ (A ∩ B )]
= P( A ∩ B) + P(A ∩ B ) = 0. 18 + 0. 58 = 0. 76
P4
Sol 24: (i) Number of ways = 8C3
to get in final for P4 exactly 3 match will be there and
choose 3 digits from set B.
P4 all win.
Total ways = 8C3 × 9C3
So other 3 are P5, P6, P7 or P8 8
C3 1
P(A and B have same 3-digit number) = =
4
C1 C1 C1 .4! 2
3 2 3 8
C3 × C3 84
9

P=
8! (ii) Case-I: Mr A’s number contains 9

4 × 3 × 2 × 4!23 4 8
C2 ×8 C3 8 C2
P= = P1 = =
8 × 7 × 6 × 5 4! 35 9
C3 ×8 C3 9 C3

Case-II: Mr A’s number do not contain 9


Sol 23: P(A) = 0. 4 → P( A ) = 1 – 0. 4 = 0. 6
(1 − P(A & B have same number))
P(B) = 0. 8 ⇒P( B ) = 1 – 0. 8 = 0 – 2 P2 =
2
P(A ∩ B) × P(Mr A’s number don’t contains 9)
P( A / B ) = = 0.1
P(B)
 1  1 8C
= 1 − 8  × × 9 3
(i) P( A ∪ B) = ?, P( A ∩ B ) = P( B ) 10.1  C3  2 C3
= 0. 2 × 0. 1 = 0. 02
P(Mr A’s number > Mr. B’s number)
1 – P( A ∩ B ) = P(A ∪ B) = 1 – 0. 02 = 0. 98
= P1 + P2
P( A ∪ B) = P( A ) + P(B) – P( A ∩ B) 8
C2 1 / 2(8 C3 − 1) 111 37
P( A ∩ B) = P(only B) = + = =
9
C3 9
C3 168 56
A B
22 Sol 25: One pair is selected
10
P(one is mole and one female) =
= P(B) – P(A ∩ B) 19
assume total student is = 2m
P(A ∩ B) = P(A) + P(B) – P(A ∪ B)
= n boy + n girl
= 0. 4 + 0. 8 – 0. 98 = 0. 22
10 n
C1 .n C1 10
P( A ∩ B) = 0. 8 – 0. 22 = 0. 58 P= = =
19 2n
C2 19
P( A ∪ B) = 0. 6 + 0. 8 – (0. 58) ⇒ 19 n = 10(2n – 1) = 20 n – 10
= 1. 4 – 0. 58 = 0. 82 ⇒ 20n – 19 n = 10
(ii) P [( A ∩ B) ∪ (A ∩ B )] ⇒ n = 10
Total student is 2n = 20

A B
M a them a ti cs | 18.59

Exercise 2 max point = 2 × 4 = 8


P(x ≥ 7) = P(x = 7) + P(x = 8)
Single Correct Choice Type
= 4C1 P(2, 2, 2, 1) + P(2, 2, 2, 2)
Sol 1: (B) P(A) = P(person A lives above 35 years) = 4 × (0. 5)3 (0. 05) + (0. 5)4
9 9 = (0. 5)3 [0. 2 + 0. 5] = (0. 5)3 0. 7 = 0. 0875
So, P (A’) = =
9 + 7 16
3 3 Sol 6: (B) Total key = n
And P(B‘) = =
3+2 5 There is only one key to open door
So, P(at least one lives)
P(last key is the right key)
= 1 – P(no lives) = 1 – P(A’) . P(B’) n − 1 (n − 1) 2 1 1 1
= · …… · · =
9 3 80 − 27 53 n (n − 1) 3 2 2 n
= 1− · = =
16 5 80 80
Sol 7: (D) A and B are independent
Sol 2: (D) Σ P = 1for a close event
P(A) = P; P(B) = P ⇒ P(A ∩ B) = P(A) . P(B) = P2
0≤P≤1
 A  A ∩ (A ∪ B) P(A ∩ A ∪ (A ∩ B)
P1 + P2 + P3 + P4 = 0. 2 + 0. 3 + 0. 4 + 0. 1 = 1 ∴P = =
 A ∪B  (A ∪ B) P(A ∪ B)
0 ≤ P1, P2, P3, P4≤ 1
P(A ∪ (A ∩ B) P(A)
= =
P(A ∪ B) P(A ∪ B)
Sol 3: (A) P = Probability in a group of 4 person
P(A ∪ B) = P(A) + P(B) – P(A ∩ B)
all are born on different days of the week.
P(A ∪ B) = P + P – P2 = 2P – P2
7 × 6 × 5 × 4 120
P= = P(A) P 1
7 × 7 × 7 × 73 343 = =
P(A ∪ B) P(2 – P) 2 − P
1 1
<P <
3 2
Sol 8: (B) x = number of getting tails
Sol 4: (D) x is a integer 1
n = bp = q =
2
P(x4 ends in the digits 6)
More tail than heads so x > 3 ⇒ P(x > 3)
14 = 1, 24 = 16, 34 = 81, 44 = 256,
P(x = 4) + 0 (x = 5) + P(x = 6)
54 = 625, 64 = ……6, 74 = ……1
6 6
84 =……6, 94 = ……1, 04 = 0 1 1
⇒ C 4   + 6 C5   +6 C6
6

2 2
There are 4 out of 6 digits which has 6 in the last for 4th
power 1 6 ×5  1 22 22
= + 6 + 1 = [15 + 7] = =
4 6  1.2 6 6 64
So, =
P = 40% 2   2 2
10
Sol 9: (D) Success = head = H
Sol 5: (B) 2 match each with two other teams.
Success = one rupee win = 0
x = 0, 1, 2(Points)
Lose = Tail → lose one rupee = T
For one match
P (he loses)
P(x = 0) = 0. 45
= P(T) + P(HT) + P(HHT) + P(HHHT) + P(HHHHT) +
P(x = 1) = 0. 05 P(HHHHHT) + P(HHHHHH)
P(x = 2) = 0. 50 1 1 16 + 6 22
= + (1 + 1 + 1 + 1 + 1 + 1)= =
For all matches 2 26 32 32
1 8 . 6 0 | Probability

Sol 10: (C) Let {1, 2, 3,……,50} 3 2


C2 C2 3×1
P(all balls are white) = · =
xx should be perfect square 5
C2 6
C2 5× 4 6 ×5
·
So x → seven 2 2
1
= = 2%
Or x → perfect 50
x = all even → 1, 22, 32, 42, 52, 62, 72
Sol 14: (B) P( A ) = 0. 7
82> 50
⇒ P(A) = 1 – P( A ) = 1 – 0. 7 = 0. 3
So x ≠ 82
P( B ) = a
Total number of x = 25 + 7 – (3)
⇒ P(B) = 1 – a
because 22, 42, 62 all are even and perfect square both.
P(A ∪ B) = P(A) + P(B) – P(A ∩ B)
25 + 7 − 3 29
P= =
50 50 P(A ∩ B) = P(A) · P(B)
 A and B are independent
Sol 11: (D) P(A) = P(a solve problem)
⇒ 0. 8 = 0. 3 + 1 – a + (0. 3) (1 – a) (–1)
P(A) = P
⇒ a (1 – 0. 3) = 1 + 0. 3 – 0. 3 – 0. 8 = 0. 2
P(B) = 1/2
0.2 2
1 ⇒a= =
P(they will make same mistakes)= 0.7 7
100
 their ans. is correct  300 1
P = Sol 15: (D) P(son will post the letter) = = P(5p)
 they get same ans.  301 2
5
P(letter react its destination) = = P(D)
⇒ P(they get same ans. ) 6
P(R) = probability of letter was received
1 1 1 P
= P× + = +
2 100 100 2  S1  P(S1 ∩ R ')
P P  = P
1 P  R'  P(R ')
P(their ans. is correct) = P × =  
2 2
1 1 1 6 +1 7
300 P/2 100 × P P(R’) = P(SP∩ D’) + P (S1P ) = + + = =
⇒ = = 2 2 6 12 12
301 1 P 2 + 100P
+  S1  1 / 2 6
100 2  P =
P=
 R '  7 / 12 7
⇒ 3(2 + 100P) = 301P  

⇒ 6 + 300P = 301P
Sol 16: (D) Set = {1, 2, 3…,20}
⇒ P = 6, P ≠ 6  = P < 1
x1x2∈ set

Sol 12: (B) Two dice are thrown until a 6 appears x1x2 is even

P(For the Ist time, 46 appear in second throw) when x1 + x2 = odd

5 5 11 275 possibility for this condition = (one odd, one even)


= × × =
6 6 36 1296 (not of possibility /or this condition)
Therefore there is 1 case to show at least one 6 = 20
C1 · 10 C1 = 200
(6, 1) (1, 6) (2, 6) (6, 2) (3, 6) (6, 3) (4, 6) (6, 4) (5, 6) (6, Total possibility = 20 × 20 = 400
5) (6, 6)
200 1
P= =
3 white 2 white 400 2
2 red 4 red 1, 2, 4, 5, 7, 8, 10, 11, 12, 14, 15, 16, 20, 21, 24, 25
Sol 13: (B) A B
3, 6, 7, 13, 17, 18, 19, 22, 23.
↓ ↓
2 balls 2 balls
M a them a ti cs | 18.61

Previous Years’ Questions Sol 4: Let Q = 1 – P = probability of getting the tail. We


have α = probability of A getting the head on tossing
Sol 1: Since, the drawn balls are in the sequence black, firstly
black, white, white, white, red, red and red. = P (H1 or T1T2T3H4 or T1T2T3T4T5T6H7 or …..
Let the corresponding probabilities be = P(H)P(T) + P(H)P(T)3 + P(H)P(T)6 …..
P1, P2, ……… . P9 P(H) P
= =
3
2 1 4 1 − P(T) 1 − Q3
Then, P1 = ; P2 = , P3 = ;
9 8 7 Also β = probability of B getting the heat on tossing
3 2 secondly
P4 = , P5 =
6 5
= P(T1H2 or T1T2T3T4H5 or T1T2T3T4T5T6T7H8 or …….)
1 3 2
P6 = , P7 = , P8 = , P6 = 1
4 3 2 = P(H) P(T) + P(H) P(T)4 + P(H) P(T)7 + …
∴ Required probabilities = P(T) [P(H) + P(H) P(T)3 + P(H)P(T)6 +…]
P1.P2.P3……..P9 P(1 − P)
= Qα = (1 – P)α =
 2  1  4  3  2  1  3  2  1 − Q3
1
=          (1) = Again, we have α + β + γ = 1
9 8 7 6 5
         4 3 2 1260
P + P(1 − P) P + P(1 − P)
1 1 1−
⇒ γ = 1 – (α + β) = 1−
=
Sol 2: Given, P(A).P(B) = , P(A).P(B) = 1−Q 3
1 − (1 − P)3
6 3
1 1 − (1 − P)3 − 2P + P2 P − P2 + P3
\[1 – P(A)] [1 – P(B) = =γ =
3 1 − (1 − P)3 1 − (1 − P)3
Let P(A) = x and P(B) = y
P P(1 − P)
1 1 =
Also, α = ,β
3
⇒ (1 – x) (1 – y) = and xy = 1 − (1 − P) 1 − (1 − P)3
3 6
1 1
⇒ 1 – x – y + xy = and xy = Sol 5: The total no. of outcomes = 6n
3 6
We can choose three numbers out of 6 in 6 C3 ways. By
5 1
⇒x+y= and xy = using three numbers out of 6 we can get 3n sequences
6 6
of length n. But these sequences of length n which use
5  1 exactly two umbers and exactly one number.
⇒ x − x =
6  6 The number of n – sequences which use exactly two
numbers
⇒ 6x2 – 5x + 1 = 0 ⇒ (3x – 1) (2x – 1) = 0
1 1
=
  ( )
C2 2n − 1n − 1n = 3 2n − 2 and the number of n
3

⇒x= and sequence which are exactly one number.


3 2

∴ P(A) =
1
3
, P(B) =
1
2
= (=
C )(1 )
3
1
n
3

Thus, the number of sequences, which use exactly three


Sol 3: The total number of ways to answer the question numbers
= 4C1 + 4C2 + 4C3 + 4C4 = 24 – 1 = 15
= 6
 ( )
C3 3n − 3 2n − 2 −=
3

6
 ( )
C3 3n − 3 2n + 3

P(getting marks) = P(correct answer in I chance) +
P(correct answer in II change) + P(Correct answer in III ∴ Probability of the required event.
chance)

1  14 1   14 13 1  3 1
= 6
 ( )
C3 3n − 3 2n + 3 / 6n

= + . + . . = =
15  15 14   15 14 13  15 5
1 8 . 6 2 | Probability

Sol 6: Let A1 be the event exactly 4 white balls have n


2i2
been drawn. A2. Be the event exactly 5 white balls have ∴ lim P(W) = lim ∑ 2
i = t n(n + 1)
n→∞ n→∞
been drawn. A3 be the event exactly 6 white balls have
been drawn. B be the event exactly 1 white ball is drawn
from two draws. 2n(n + 1)(2n + 1) 2
= lim =
n→∞ 6n(n + 1) 2 3
Then,

 B   B   B  n
P(B) = P   P(A1 ) + P   P(A2 ) + P   P(A3 )  un  n + 1 2
 A1   A2   A3  Sol 9: (A) P =  =
W Σi n+1
 B  n+1
But P   = 0 ( there are only 6 white balls in the
 A3 
bag) Sol 10: (B, C, D) Since, E and F are independent events.
Therefore P(E ∩ F) = P(E).P(F) ≠ 0, so E and F are not
 B   B  mutually exclusive events.
∴ P(B) = P   P(A1 ) + P   P(A2 )
 A1   A2  Now, P(E ∩ F ) = P(E) – P(E ∩ F)
12
C 2 .6 C 4 10
C1 .2 C1 12
C1 .6 C5 11
C1 .1 C1 = P(E) – P(E).P(F) = P(E)[1 – P(F)] = P(E).P (F)
= . + .
18 10 18 12
C6 C2 C6 C2 and P(E ∩ F) = P(E ∪ F) = 1 – P(E ∪ F)

= 1 – [1 – P(E).P(F) ]
Sol 7: As, the statement shows problem is to be related
to Baye’s law. ( E and F are independent)
Law C, S, B, T be the events when when person is going = P(E).P(F)
by car, scooter, bus or train respectively.
1 3 2 1 So, E and F as well and E and F are independent
∴ P(C) = , P(S) = , P(B) = , P(T) = events.
7 7 7 7
Again, L be the event of the person reaching office late. Now,
P(E ∩ F) + P(E ∩ F) P(F)
∴ L be the event of the person reaching office in time. P(E/F) + P(E / F) = = =1
P(F) P(F)
L 7 L  8 L  5
Then, P   = , P   = , P   = 1
C 9 S 9 B 9 Sol 11: (A, D) Both E and F happen ⇒ P(E ∩ F) =
and neither E nor F happens 12
L  8
And P   = 1
T 9 ⇒ P(E ∩ F) =
2
L
P   .P(C) But for independent events, we have
C
∴P  = C
1
 L  P  L  .P(C) + P  L  .P(S) P(E ∩ F) = P(E) P(F) = ….. (i)
    12 
C S
L  L  and P(E ∩ F) =
P(E)P(F)
+P   .P(B) + P   .P(T)
B
  T = {1 – P(E)} {(1 – P(F)}

7 1 = 1 – P(E) – P(F) + P(E)P(F)


×
9 7 1
= = 1 1
7 1 8 3 5 2 8 1 7 ⇒ = 1 – {P(E) + P(F)} +
× + × + × + × 2 12
9 7 9 7 9 7 9 7
1 1 7
P(E) + P(F) = 1 – + =  ….. (ii)
2 12 12
Sol 8 : (B) Here, P(ui) = Ki, SP(ui) = 1
2 On solving Equation (i) and (ii), we get
⇒k=
n(n + 1) 1 1 1 1
either P(E) = and P(F) = or P(E) = and P(F) =
3 4 4 3
M a them a ti cs | 18.63

P(E ∩ F) P(E ∩ F) 7 12
Sol 12: (A, D) P(E/F) + P(E / F) = + ⇒ (P(E))2 – P(E) + =0
P(F) P(F) 5 25
P(E ∩ F) + P(E ∩ F) P(F)  3  4
= = =1
P(F) P(F) ⇒  P(E) −  P(E) −  =0
 5  5 
(b) P(E/F) + P(E/ F ) 3 4 4 3
∴ P(E) = or ⇒ P(F) = or
P(E ∩ F) P(E ∩ F) 5 5 5 5
= +
P(F) P(F) Sol 14: (C) Let E = event when each American man is
seated adjacent to his wife
P(E ∩ F) P(E ∩ F)
= + ≠1
P(F) 1 − P(F) A = event when Indian man is seated adjacent to his wife

(c) P(E / F) + P(E / F) =


P(E ∪ F) P(E ∩ F)
+ Now n(A ∩ E) = (4!) × (2!)5
P(F) P(F)
Even when each American man is seated adjacent to
P(E ∩ F) P(E ∩ F) his wife
= + ≠1
P(F) 1 − P(F) = (5!) × (2!)4
Again n(E)
P(E ∩ F) P(E ∩ F)
(d) P(E / F) + P(E / F) = +  A  n(A ∩ E) (4!) × (2!)5 2
P(F) P(F) ⇒ P  = = =
E n(E) (5!) × (2!)4 5
P(E ∩ F) + P(E ∩ F) P(F)
= = =1
P(F) P(F) Sol 15 : (D) Statement-I:
If P(Hi ∩ E) =
0 for some I, then
11
Sol 13: (A, D) P(E ∪ F) – P(E ∩ F) = ….. (i)  Hi  E
25  P=
  P=
  0
(i.e. only E or only F) Neither of them occurs =
2 E  Hi 
25
If P(Hi ∩ E) ≠ 0 for ∀ i =1,2.....n then
2
⇒ P(E ∩ F) = ..... (ii)
25   H  P(Hi ∩ E) P(Hi )
P  i 
= ×
E P(Hi ) P(E)

E
P   × P(Hi )
H E
=  i > P   ⋅ P(Hi ) [as 0 < P (E) < 1]
P(E)  Hi 
Hence, Statement-I may not always be true.
Statement-II: Clearly H1 ∪ H2 ..... ∪ Hn =
S (sample space)

11 ⇒ P(H1 ) + P(H2 ) + .... + P(Hn ) =


1
From Eq. (i) P(E) + P(F) – 2P(E ∩ F) = ….. (iii)
25 
2
From eq. (ii), (1 – P(E)) (1 – P(F)) = Sol 16 : (C) COCHIN
25
2 The second place can be filled in 4 C1 ways and the
⇒ 1 – P(E) – P(F) + P(E).P(F) = ….. (iv) remaining four alphabets can be arranged in 4! Ways in
25 
four different places. The next 97th word will be COCHIN
From Eq. (iii) and (iv), we get
Hence, there are 96 word before COCHIN.
7 12
P(E) + P(F) = and P(E).P(F) =
5 25
 Ec ∩ Fc  P(Ec ∩ Fc ∩ G)
7  12 Sol 17: (C) P  =
∴ P(E).  – P(E) =  G  P(G)
 
 5  25
1 8 . 6 4 | Probability

P(G) − P(E ∩ G) − P(G ∩ F) Sol 23 : (D) For X ≥ 6 , the probability is


=
P(G) 55 56 55  1  5
5
= + + ....∞ =
6 1 −5 / 6   
P(G)(1 − P(E) − P(F)) 6 6
6 7
6   6
[ P(G) ≠ 0]
P(G)
For X ≥ 3
1 P(E) − P(F)
=−
3
53 54 55 5
c
= P(E ) − P(F) + + + .....∞ = 
6 4
65
6 6
6
6
5
4 p 2p / 5  
Sol 18: (D) P(A ∩ B) = 6 25
×
10 10
=
10 Hence the conditional probability   = .
5
3 36
2p  
⇒ is an integer 6
5
⇒ 5 or 10
=
Sol 24: (C) Event G = original signal is green
E1 = A receives the signal correct
Sol 19: A → p; B → s; C → q; D → q
E2 = B receives the signal correct
(A) ENDEA, N, O, E, L are five different letter, then
permutation = 5! E = signal received by B is green

(B) If E is in the first and last position then P(signal received by B is green)
(9 − 2)! = P(GE1E2 ) + P(GE1E2 ) + P(GE1E2 ) + P(GE1 E2 ) + P(GE1E2 )
= 7 × 3 × 5! = 2!× 5!
2!
4! 46
(C) For first four letters = P(E) =
2! 5 × 16
For last five letters = 5!/3! 40 / 5 × 16 20
P(G / E)
= =
46 / 5 × 16 23
4! 5!
Hence × =2 × 5!
2! 3!
Sol 25: (B) H → ball from U1 to U2
(D) For A, E and O 5!/3! And for others 4!/2!
T → 2 ball from U1 to U2
5! 4!
Hence × =2 × 5! E : 1 ball drawn from U2
3! 2!
1 3  1 2 1 1
Sol 20 : (C) Coefficient of x10 in (x + x2 + x3 )7 P/W from U 2 = ×  ×1 + ×  ×  +
2 5  2 5 2 2
Coefficient of x3 in (1 + x + x2 )7
 3C  1  2C 1  1
Coefficient of x3 in (1 − x3 )7 (1 − x)−7 × 2 × 1 + ×  2 ×  +
 5C  2  5C 3 2
7 +3−1  2   2 
= C3 − 7

= 9
C3 − 7  3 C ⋅2 C 2  23
× 1 1
× =
 5C 3  30
9×8×7  2 
= = − 7 77
6
H P(W / H) × P(H)
 5  5  1 25 Sol 26: (D) P   =
Sol 21 : (A) P(X= 3)
=    = W
  P(W / T) ⋅ P(T) + (W / H) ⋅ P(H)
 6  6  6 216
13 2 1
25  ×1 + × 
Sol 22: (B) 25 5 2  12
216 = =
23 / 30 23
11 25
Required probability =1− =
36 36
M a them a ti cs | 18.65

Sol 27: (B) Number of ways 2


P(X ∪ Y)= P(X) + P(Y) − P(X ∩ Y)= (A is correct)
= 35 − 3C1 ⋅ 25 + 3C2 15 3
= 243 − 96 + 3= 150 P(X ∩ Y)= P(X) ⋅ P(X) ⇒ X and Y are
independent (B is correct)
1 1 1
Sol 28: (B, D)
= P(X1 ) = , P(X 2 ) = , P(X3 )
2 4 4 P(X c ∩ Y)= P(Y) − P(X ∩ Y)
1 1 1
P(X)= P(X1 ∩ X 2 ∩ X3C ) + P(X1 ∩ X C2 ∩ X3 ) = − = (D is not correct)
3 6 6
1
+P(X1C ∩ X 2 ∩ X3 ) + P(X1 ∩ X 2 X3 ) =
4 Sol 31: (A) P (at least one of them solves correctly) =
1 1 – P (none of them solves correctly)
P(X ∩ X1C ) 32 1  1 1 3 7  235
(A) P(X1C /=
X) = = =1 −  × × ×  =
P(X) 1 8  2 4 4 8  256
4
(B) P [exactly two engines of the ship are
Sol 32: (6) Let P (E1) = x, P (E2) = y and P (E3) = z
7
Then (1 – x) (1 - y) (1 - z) = p
32 7
functioning | X] = =
1 8 (1 − x)y(1 − z) =α
4 (1 − x)y(1 − z) =β
5 (1 − x)(1 − y)(1 − z) =γ
 X  32 5
(C) P  =  = 1−x p α
 X2  1 8 so
= = x
4 x α α +p
γ
7 Similarly z =
 X  32 γ +p
7
(D) P  =  =
X
 1 1 16 α γ +p p
1+
2 P(E1 ) α + p γ γ
So, = = =
P(E3 ) γ α +p p
Sol 29: (A) Favourable : D4 shows a number and only 1 1+
γ +p α α
of D1 D2 D3 shows same number
Or only 2 of D1 D2 D3 shows same number αβ 2βγ 5αγ
Also given = p= ⇒ β=
α − 2β β − 3γ α + 4γ
Or all 3 of D1 D2 D3 shows same number
Required probability   5αγ   α ⋅ 5αγ
Substituting back  α − 2   p =
6
C1 (3 C1 × 5 × 5 + 3C2 × 5 + 3C3 )   α + 4γ   α + 4γ
=
216 × 6 ⇒ αp − 6pγ = 5αγ
6 × (75 + 15 + 1) 6 × 91 91 p
= = = +1
216 × 6 216 × 6 216 p  p  γ
⇒  + 1 = 6  + 1  ⇒ = 6
γ  α  p
1 +1
Sol 30: (A, B) P(X / Y) = α
2
P(X ∩ Y) 1 1 Sol 33: (5)
= ⇒ P(Y) =
P(Y) 2 3 Clearly, 1 + 2 + 3 + .... + n − 2 ≤ 1224 ≤ 3 + 4 + ....n
1 (n − 2)(n − 1) (n − 2)
P(Y / X) = ⇒ ≤ 1224 ≤ (3 + n)
3 2 2
P(X ∩ Y) 1 1
=⇒ P(X) = ⇒ n2 − 3n − 2446 ≤ 0 and n2 + n − 2454 ≥ 0
P(X) 3 2
1 8 . 6 6 | Probability

⇒ 49 < n < 51 ⇒ n =
50 Case-II: All 3 odd
Number of ways = 2 × 3 × 4 = 24
n(n + 1)
∴ − (2k + 1) = 1224 ⇒ k = 25 ⇒ k − 20 = 5 Favourable ways = 53
2
53 53
Required probability
= =
3 × 5 × 7 105
Sol 34: (A) P (required) = P (all are white) + P (all are
red) + P (all are black) Sol 39: (C) Here 2x= x1 + x3
2
1 2 3 3 3 4 2 4 5
= × + + × × + × × ⇒ x1 + x3 =
even
6 9 12 6 9 12 6 9 12
6 36 40 82 Hence number of favorable ways
= + + =
648 648 648 648
= 2C1 ⋅ 4 C2 + 1C1 ⋅ 3C1 = 11

Sol 35: (D) Let A : one ball is white and other is red
Sol 40: (8) Let coin was tossed ‘n’ times
E1 : both balls are from box B1 1 n
1  + 
Probability of getting atleast two heads =− n
E2 : both balls are from box B2 2 2n 
E3 : both balls are from box B3 n + 1 
⇒ 1−  ≥ 0.96
 2n 
E 
Here, P (required) = P  2  2n
A ⇒ ≥ 25
  n+1 ⇒ n≥8

A
P   ⋅ P(E2 )
 E2  Sol 41: (5) n = 6! . 5! (5 girls together arranged along
= with 5 boys)
A A A
P   ⋅ P(E1 ) + P   ⋅ P(E2 ) + P   ⋅ P(E3 ) 5
m= C 4 ⋅ (7!− 2.6!) ⋅ 4!
 E1   E2   E3 
(4 out of 5 girls together arranged with others – number
2
C1 × 3C1 1 of cases all 5 girls are together)
×
9
C2 3 m 5 ⋅ 5 ⋅ 6!⋅ 4!
= = = 5
1 3
C1 × C1 1
2
C × C1 1 3
C1 × C1 1 3 4 n 6!⋅ 5!
× + 19 × + 12 ×
6
C2 3 C2 3 C2 3
Sol 42: (A, B) P (Red ball) =P(I) ⋅ P(R | I) + P(II) ⋅ P(R | II)
1
55 1 P(II) ⋅ P(R | II)
= = 6 P(II | R)= =
1 1 2 181 3 P(I) ⋅ P(R | I) + P(II) ⋅ P(R | II)
+ +
5 6 11 n3
1 n3 + n4
=
Sol 36: (A) Either a girl will start the sequence or will be 3 n1 n3
+
at second position and will not acquire the last position n1 + n2 n3 + n4
as well.
(3 C1 + 3C1 ) 1 Of the given options, A and B satisfy above condition
Required probability
= =
3
C2 2
Sol 43: (C, D) P (Red after Transfer) = P(Red Transfer) .
Sol 37: (C) Number of required ways P(Red Transfer in II Case) + P (Black Transfer) . P(Red
Transfer in II Case)
= 5!− {4 ⋅ 4!− 4 C2 ⋅ 3!+ 4 C3 ⋅ 2!− 1} = 53
n1 (n1 − 1) n2 n1 1
=P(R) + ⋅=
n1 + n2 (n1 + n2 − 1) n1 + n2 n1 + n2 − 1 3
Sol 38: (B) Case-I : One odd, 2 even
Total number of ways = 2 × 2 × 3 + 1 × 3 × 3 + 1 × 2 × 4 = 29 Of the given options, option C and D satisfy above
condition.
M a them a ti cs | 18.67

1 4 7 Sol 45: (A)


= 6
C3 × 4 C1 × 4 + 6 C 4=
× 380
Sol 44: (C) P(T1 ) = , P(T2 ) = , P(D) =
5 5 100

D D D 1 1 1 1 1 1 5


P   10.
= P   . Let P   x Sol 46: (B) P(X > Y) =  ×  +  ×  +  ×  =
 2 2   2 6   6 2  12
 T1   T2   T2 

D D 7 1 1   1 1  13
Now, P(T1 ) × P   + P(T2 ) ⋅ P   = Sol 47: (C) P(X = Y) =  × × 2  +  ×  =
T   T  100 2 3   6 6  36
 1  2
1 4 7 1
= × 10x + × x = ⇒x=
5 5 100 40

4 39
 T2  5 × 40 78
∴ P   = =
D 93 93
100
2017-18 100 &
op kers
Class 12 T
By E ran culty
-JE Fa r
IIT enior emie .
S fP r es
o titut
Ins

MATHEMATICS
FOR JEE MAIN & ADVANCED
SECOND
EDITION

Exhaustive Theory
(Now Revised)

Formula Sheet
9000+ Problems
based on latest JEE pattern

2500 + 1000 (New) Problems


of previous 35 years of
AIEEE (JEE Main) and IIT-JEE (JEE Adv)

5000+Illustrations and Solved Examples


Detailed Solutions
of all problems available

Topic Covered Plancess Concepts


Tips & Tricks, Facts, Notes, Misconceptions,
Solutions of Triangle Key Take Aways, Problem Solving Tactics

PlancEssential
Questions recommended for revision
19. SOLUTIONS OF
TRIANGLE

1. INTRODUCTION
A
In any triangle ABC, the side BC, opposite to the angle A is denoted by a; the side CA and
AB, opposite to the angles B and C respectively are denoted by b and c respectively. The c b
semi-perimeter of the triangle is denoted by s and its area by ∆ or S. In this chapter, we shall
discuss various relations between the sides a, b, c and the angles A, B, C of ∆ ABC.
B a C

2. SINE RULE Figure 19.1

The sides of a triangle (any type of triangle) are proportional to the sines of the angle opposite to them in triangle
a b c
ABC, = =
sinA sinB sinC
sinA sinB sinC
Note: (i) The above rule can also be written as = =
a b c
(ii) The sine rule is a very useful tool to express the sides of a triangle in terms of sines of the angle and vice-versa
a b c
in the following manner: = = = k (Let ) ; ⇒ a = k sinA, b = k sinB, c = k sinC
sinA sinB sinC
sinA sinB sinC
Similarly, = = = λ (Let ) ; ⇒ sinA =
λa , sinB = λb , sinC = λc
a b c

3. COSINE RULE
b2 + c2 − a2 c2 + a2 − b2 a2 + b2 − c2
In any ∆ABC , cos A = ; cosB = ; cosC =
2bc 2ac 2ab
Note: In particular
∠A =60 ⇒ b2 + c2 − a2 =
bc
∠B =60 ⇒ c2 + a2 − b2 =
ca
∠C =60 ⇒ a2 + b2 − c2 =
ab

Figure 19.2
4. PROJECTION FORMULAE

If any ∆ABC=
:(i) a bcosC + ccosB =
(ii) b ccos A + acosC =
(iii) c acosB + bcos A
i.e. any side of a triangle is equal to the sum of the projection of the other two sides on it.
1 9 . 2 | Solutions of Triangle

Case I: When ∆ABC is an acute angled triangle,


BD
cosB = ⇒ BD = AB.cosB ⇒ BD = c.cosB and
AB
CD
cosC = ⇒ CD = AC.cosC ⇒ CD = bcosC
AC

then, BD+DC=BC
Figure 19.3
∴ a = c cos B + b cos C
Case II: When ∆ABC is an obtuse angled triangle,
CD
cosC = ⇒ CD = AC.cosC
AC
BD
b.cosC and cos (180 − B ) = ⇒ BD =
CD = −c.cosB then,
AB
⇒ a bcosC + ccosB
a = BC and CD – BD=
Figure 19.4

Illustration 1: If A = 75ο , B = 45ο , then what is the value of b + c 2 ?  (JEE MAIN)

Sol: Here, c = 180ο − 120ο = 60ο . Therefore by using sine rule, we can solve the above problem.

a = k sin 75o
a b c
Use sine rule = = = K ⇒ b = k sin 45o
sin75ο sin 45ο sin60ο
c = k sin 60o

( ) (
= k sin 45ο + 2 sin60
consider, b + c 2 = 0 3 +1
)
3 +1
sin75ο 2k
k= 2k = 2k =
2 2 2
= sinA 2a

Illustration 2: In a ∆ ABC, if B = 30ο and c = 3b , then find the value of A. (JEE MAIN)
2 2 2
c + a −b
Sol: Here, by using cosine rule cosB = we can easily solve the above problem.
2ca

c2 + a2 − b2 3 3b2 + a2 − b2
cosB
We have= ⇒
= ; ⇒ a2 − 3ab + 2b2 = 0 ⇒ (a− 2b)(a− b) = 0
2ca 2 2× 3b×a
⇒ a−b =0 OR a – 2b = 0

⇒ Either a = b ⇒ A = 30ο or aa== 2b ⇒aa22 ⇒


2b⇒ 4b22⇒
⇒4b == bb22++cc22 ⇒ 90οο .
⇒AA== 90

Illustration 3: Prove that asin (B − C ) + bsin(C − A) + csin ( A − B ) =


0 (JEE MAIN)

a b c
Sol: By sine rule i.e. = = = k we can simply prove the above equation.
sinA sinB sinC
In equation asin (B − C ) + bsin(C − A) + csin ( A − B ) = 0, putting a = k sin A, b = k sin B, c = k sin C

( )
= k sinA sin (B − C ) + sinBsin ( C − A ) + sinCsin ( A − B ) =0 (expanding all terms gets cancelled)

(Using sin (α − β=
) sin α. cos β − sin β cos α )

b2 − c2
Illustration 4: Prove that sin (B − C ) = sinA  (JEE MAIN)
a2
M a them a ti cs | 19.3

b2 − c2 2
Sol: Given, sin(B − C) = 2 sinA ⇒ a sin(B − C) =
a
(b 2
)
− c2 sinA

Takin L.H.S., a2sin(B − C) = a2(sinBcosC − cosBsinC)


sinA sinB sinC a2 + c2 − b2 a2 + b2 − c2
Now using sine rule, = = and cosine rule, cosB =
= k (say) = , and cosC
a b c 2 ac 2 ab
 a2 + b2 − c2 a2 + c2 − b2   a2 + b2 − c2 − a2 + c2 − b2 
= a2  kb

 2 ab

2 ac
× kc  = ka 



 2
 = sinA × b2 − c2 = RHS.


( )

Illustration 5: The angles of a triangle are in 4:1:1 ratio. Find the ratio between its greater side and perimeter?
 (JEE ADVANCED)

Sol: Here, the angles are 120ο , 30ο , 30ο . Therefore, by using sine rule, we will get the required ratio.
Angles are 120ο , 30ο , 30ο .
If the sides opposite to these angles are a, b and c respectively, a will be the greatest side.

a b c a b c a b c
Now from sine formula, = = ; ⇒ = = ; ⇒ = == k (say)
sin120 ο
sin30 ο
sin30 ο
3 /2 1/2 1/2 3 1 1

(
then a = 3k , perimeter= 2 + 3 k ; ) ∴ Required ratio =
3k
=
3
.
( 2+ 3 k ) 2+ 3

C B
Illustration 6: Solve bcos2 + c cos2 in term of k where k is perimeter of the ∆ABC .  (JEE ADVANCED)
2 2
1 + cos 2θ
Sol: We can solve the given problem simply by cos2 θ =
2
C B b c
Here, bcos2 + c cos2 = (1 + cosC ) + (1 + cosB) [using projection formula]
2 2 2 2
b+ c 1 a+b+c C B Kk
= + a= ; ∴ bcos2 + c cos2 = [where k=a+b+c, given]
2 2 2 2 2 22
 A −B 
tan  
Illustration 7: In any triangle ABC, show that
a−b
=  2  (JEE ADVANCED)
a+b  A +B
tan  
 2 

Sol: We can derive the values of a, b and c using sine rule and putting it to L.H.S. we can prove the above problem.
a b c
We know that, = = = k
sinA sinB sinC
⇒a=
k sinA , b = ksinB , c = k sinC  …(i)
On putting the values of a and b from (1) on L.H.S., we get
A +B A −B
2cos sin
a − b k sinA − k sinB sinA − sinB 2 2
L.H.S.
= = = =
a + b k sinA + k sinB sinA + sinB A +B A −B
2sin cos
2 2
 A −B 
tan  
 A +B
= cot  tan
=
 A− B 
=  2  R.H.S.
  
 2   2  tan  A + B 
 
 2 
1 9 . 4 | Solutions of Triangle

5. NAPIER’S ANALOGY (LAW OF TANGENTS)

B − C  b − c  A  A −B   a −b  C C− A c−a B 


In any ∆ABC, (i) tan  =  cot   (ii) tan  =  cot   (iii) tan  =  cot  
 2  b +c 2  2  a+b 2  2  c +a 2

b−c  A  k sinB − k sinc A  sinB − sinc  A


Proof:(i) R.H.S. = cot   = cot   [By the sine rule] =   cot  
b+c 2
  k sinB + k sinc 2
   sinB + sinc  2
  B− C  B + C  
 2sin   cos  
=  2   2   .cot  A  [By C & D formulae]
  
 B+ C  B − C   2
 2sin   cos   
  2   2 
B − C  B + C  A B − C  A A
tan   .cot   ⇒ cot   = tan   . tan   .cot   [By condition identities]
 2   2  2  2  2 2

B − C 
= tan   = LHS
 2 
Similarly, (ii) and (iii) can be proved.

Illustration 8: In any triangle ABC, if A = 30ο , b=3 and c = 3 3 , then find ∠B and ∠C .  (JEE MAIN)
C −B c −b A
Sol: By using formula, tan = cot , we can easily obtain the values of ∠B and ∠C .
2 c+b 2
B+C A
Here ∠A =30ο ∴ = 90ο − = 90 − 15ο = 75ο  … (i)
2 2

Since c > b ⇒ ∠C > ∠B and B + C =150ο  … (ii)

C −B c −b A c −b B + C  C −B 3 3 −3
tan
= = cot tan   ; ⇒ tan = tan75ο
2 c+b 2 c+b  2  2 3 3 +1 ( )
 C −B  c −b π  c −b A
⇒ tan =  cot = − A tan  
 2  c+b 2  c+b 2

C −B 3 3 −1( ) ( )
3 − 1  tan 45ο + tan30ο 
[Using (1) ] =⇒ tan (
tan= 45ο + 30ο )  
2
(
3 3 +1 ) ( )
3 + 1  1 − tan 45 tan30
ο ο 

 1 
(
=
)
3 −1 1 +


3  =

3 − 1  3 + 1 
  1 ⇒
C −B
45ο  tan 45ο = 1
=
( )
3 +1 1−

1 
 
   
3 + 1  3 − 1  ; 2 ;  
 3

⇒ C −B =90ο  … (iii)

Solving (ii) and (iii), we get ∠B =30ο and ∠C =120ο

6. TRIGONOMETRIC RATIOS OF HALF ANGLES


Sine, cosine and tangent of half the angles of any triangle are related to their sides as given below. Note that the
perimeter of ∆ABC will be denoted by 2s i.e. 2s=a+b+c and the area denoted by ∆ .
M a them a ti cs | 19.5

A B  C
Formulae for sin   , sin   , sin   for any ∆ABC
2 2 2

A
(i) sin   =
( s − b )( s − c ) (ii) sin  B  = ( s − c )( s − a) (iii) sin  C  = ( s − a)( s − b )
   
2 bc 2 ac 2 ab

A B  C
Formulae for cos   , cos   , cos   for any ∆ABC
2
  2
  2

A s ( s − a) B  s (s − b) C s (s − c)
(i) cos   = (ii) cos   = (iii) cos   =
2 bc 2 ac 2 ab

A B  C
Formulae for tan   , tan   , tan   for any ∆ABC
2 2 2

A
(i) tan   =
( s − b )( s − c ) (ii) tan  B  = ( s − c )( s − a) (iii) tan  C  = ( s − a)( s − b )
   
2 s ( s − a) 2 s (s − b) 2 s (s − c)

s −a s −b s −c
Illustration 9: In a triangle ABC, if = = , then find the value of tan2 (A/ 2) . (JEE MAIN)
11 12 13

A
Sol: As we know, tan   =
( s − b )( s − c ) . Therefore, by using this formula, we can solve the above problem.
2 s ( s − a)

s − a s − b s − c 3s − ( a + b + c )
= = = =
s 2A
; Now tan=
( s − b )(=s − c) 12 × 13 13
=
 
11 12 13 11 + 12 + 13 36 2 s ( s − a) 36 × 11 33

 A B C
Illustration 10: In a triangle ABC, prove that (a+ b + c)  tan + tan  =2ccot . (JEE MAIN)
 2 2  2

A
Sol: Here by using tan   =
( s − b )( s − c ) B 
and tan   =
( s − c )( s − a) we can prove the above problem.
2 s ( s − a) 2 s (s − b)

 A B 
L.H.S. = (a+ b + c)  tan + tan  = 2s 
( s − b )( s − c ) + ( s − c )( s − a)  = 2s
s −c  s −b
+
s−a
 
 2 2  s ( s − a) s (s − b)  s  s − a s − b 
 

( s − c )  s=
−b + s −a  2 s s−c s (s − c) 2c C
= 2s   ( a + b + c − b=
− a) 2c = = 2c cot = R.H.S.
s  s −a s −b  s −a s −b ( s − a)( s − b )
tan
C 2
2
 ∆ ∆   1 1  2∆ ( 2s − a − b ) 2c∆2 2cs ( s − c )
Alternate: L.H.S. = 2s  + 2∆ 
= +  = = =
 (
s s − a ) ( ) 
s s − b  s − a s − b ( s − a)( s − b ) ( s − a)( s − b ) ∆ ∆

∆   1 1  2∆ ( 2s − a − b ) 2c∆2 2cs ( s − c ) c
+ = 2∆  + = = = = 2c.cot = R.H.S.
a) s ( s − b )   s − a s − b  ( s − a)( s − b ) ( s − a)( s − b ) ∆ ∆ 2

A B C
Illustration 11: In a ∆ABC , if cot , cot , cot are in AP, then prove that the sides of ∆ABC are in A.P.
 2 2 2 (JEE MAIN)

Sol: Here by using trigonometric ratios of half angles formula, we can prove the above illustration.
A B C
Given cot , cot , cot are in A.P.
2 2 2
1 9 . 6 | Solutions of Triangle

s ( s − a) s ( s − b ) s ( s − c )
⇒ , , are in A.P.
∆ ∆ ∆

⇒ ( s − a) , ( s − b ) , ( s − c ) are in A.P.

⇒ a, b, c are in A.P. Proved

 A C B
Illustration 12: In a ∆ ABC , the sides a, b and c are in A.P. Then what is the value of  tan + tan  : cot ?
 2 2 2
 (JEE ADVANCED)
A C B
Sol: Simply by using formula of tan , tan and cot we can easily get the required result.
2 2 2


tan
A
+ tan
C
: cot
B


 ( s − b )( s − c ) + ( s − a)( s − b )  : s ( s − b ) ⇒
( s − c ) + ( s − a) : s
 
 2 2  2  s ( s − a) s ( s − c )  ( s − c )( s − a) s
 
a+b+c
= 2s − ( a + c ) : s ; ⇒ b : ; ⇒ 2b : a + b + c ⇒ a.b.c are in A.P.
2

∴ 2b:a+b+c = 2 : 3

A B C a+b+c C
Illustration 13: In any triangle ABC, show that cot + cot + cot = cot .  (JEE ADVANCED)
2 2 2 a+b−c 2
A B C
Sol: Similar to the above problem, by putting the values of cot , cot and cot we can prove the above
problem. 2 2 2

A B C s ( s − a) s (s − b) s (s − c)
cot + cot =+ cot + +
2 2 2 ( s − b )( s − c ) ( s − c )( s − a) ( s − a)( s − b )
1  2 2  1
 s ( s − a) + s ( s − b ) + s ( s − c )  = s ( s − a + s − b + s − c ) 
2 2 2 2
L.H.S.  
s ( s − a)( s − b )( s − c )   s ( s − a)( s − b )( s − c )

1 1
=
 {
s 3s − ( a + b + c )  =
 } s ( 3s − 2s ) 
 
s ( s − a)( s − b )( s − c ) s ( s − a)( s − b )( s − c )

A B C s2
⇒ cot + cot + cot = … (i)
2 2 2 s ( s − a)( s − b )( s − c )

a+b+c C 2s C 2s C s C
Now, R.H.S. = = cot = cot = cot cot
a+b−c 2 a + b + c − 2c 2 2s − 2c 2 s−c 2
L.H.S = R.H.S

s s (s − c) s2 a+b+c C s2
= ;⇒ cot = … (ii)
s − c ( s − a)( s − b ) s ( s − a)( s − b )( s − c ) a+b−c 2 s ( s − a)( s − b )( s − c )

A B C a+b+c C
From (i) and (ii), we have cot + cot + cot = cot . Proved
2 2 2 a+b−c 2
M a them a ti cs | 19.7

7. AREA OF TRIANGLE
1 1 1
If ∆ be the area of a triangle ABC,=
then ∆ bcsinA
= = casinB absinC
2 2 2

Proof: Let ABC be a triangle. Then the following cases arise.

AD
Case I: When ∆ABC is an acute angled triangle, sinB = 
AB
1
B; ∆ Areaof ∆ABC ; ∆ =
AD=AB sin B ; AD=c sin=
2
(BC )( AD ) ; ∆ =12 ac sinB
AD
Case-II: When ∆ ABC is an obtuse angled triangle, sin (180 − B ) = ;
AB Figure 19.5
AD=AB sin B ⇒ AD=c sin B
1 1 1
∆ =Area of ∆ABC ; ∆ = (BC) ( AD ) ; ∆ = ac sinB ; So in each case, ∆ = ac sinB
2 2 2

(ii) Heron’s formula=


∆ s ( s − a)( s − b )( s − c )

1  A A A A
Proof: ∆ = bc  2sin cos  = bcsin   .cos  
2  2 2 2 2

bc
( s − b )( s − c ) × s ( s − a) [By half angle formula] = s ( s − a)( s − b )( s − c )
Figure 19.6
bc bc

1 a2 sinBsinC 1 b2 sinCsinA 1 c2 sinA sinB


=(iii) ∆ = =
2 sin (B + C ) 2 sin ( C+ A ) 2 sin ( A + B )

From the above results, we obtain the following values of sinA, sinB and sinC

2∆ 2 2∆ 2
(iv) sinA = = s ( s − a)( s − b )( s − c ) (v) sinB = = s ( s − a)( s − b )( s − c )
bc bc ca ca

2∆ 2
(vi) sinC = = s ( s − a)( s − b )( s − c )
ab ab
sinA sinB sinC 2∆
Further with the help of (iv), (v), (vi) we obtain = = =
a b c abc

Illustration 14: In any triangle ABC, prove that 4 ∆ cotA = b2 + c2 − a2 .  (JEE MAIN)

1 b2 + c2 − a2
Sol: We can prove the above problem by using formula of area of triangle i.e. ∆ = bc sinA and cos A = .
2 2bc
1 cos A b2 + c2 − a2
L.H.S.= 4 ∆ cotA =
4. bcsinA.
= 2bc cos A 2bc.
= = b2 + c2 − a2 = R.H.S.
2 sinA 2bc

a2 − b2 sinA sinB
Illustration 15: In any triangle ABC, prove that . = ∆ (JEE MAIN)
2 sin ( A − B )

=
Sol: By putting a k=sinA and b k sinB we can prove the above illustration.

L.H.S. =
a2 − b2 sinA sinB
. =
( ) =
(
k 2 sin2 A − k 2 sin2 B sinA sinB k 2 sin2 A − sin2 B sinA sinB )
2 sin ( A − B ) 2sin(A − B) 2sin(A − B)
1 9 . 8 | Solutions of Triangle

k 2 sin ( A + B ) sin ( A − B ) sinA sinB k 2


[using sine formula a=k sinA etc.]= = .sin ( A + B ) sinA sinB
2sin(A − B) 2

1
=
2
(k sinA )(k sinB ) sin ( π − c ) ;  A + B = π − C ; 12 absinC = ∆ = R.H.S.
Illustration 16: A tree stands vertically on a hill side which make an angle of 15o with the horizontal. From a point
on the ground 35m down the hill from the base of the tree, the angle of elevation of the top of the tree is 60o. Find
the height of the tree. (JEE MAIN)

Sol: We can simply obtain the height of the tree from the given figure.  P

AQR, QR AQ
In ∆= = sin15ο 35sin15
= ο
; AR AQ
= cos15ο 35cos15ο
PR
In ∆APR , tan60ο = AR. 3 ; ⇒ PQ + QR =
; ⇒ PR = 3AR
AR h

⇒ h + 35sin15ο =3.35cos15ο
 3 −1
=⇒ h 35 (
3 cos15ο=− sin15ο


)
35  3.
3 +1
2 2
− 
2 2  m Q
35
 3 + 3 − 3 + 1  35
o
60
 2 2( )
= 35  =  = 4 35 2m 15
o

 2 2
A R
 
Figure 19.7
Hence, the height of the tree= 35 2m

8∆2
Illustration 17: In any triangle ABC, prove that acos A + bcosB + c cosC
= 2asinBsinC
= . (JEE ADVANCED)
abc

Sol: we can solve this illustration by substituting a = k sinA, b = k sinB, and c = k sinC .
As a = k sinA, b = k sinB, c = k sinC  …(i)

L.H.S.= acos A + bcosB + ccosC = k sinA cos A + k sinBcosB + k sinCcosC [using sine formula]

k k
= {sin2A + sin2B} + sin2C
= 

2sin ( A + B ) cos ( A − B ) + 2sinCcosC 
2 2 

=
k
2
2sinCcos ( A − B ) =
+ 2sinCcosC 
k
2 
{ (
2sinC cos ( A − B ) + cos π − A + B 
 )}
= k sinC cos ( A − B ) − cos ( A + B )  =
k sinC 2sinA sinB 

 2∆  2∆   1 2∆ 2∆ 
= 2 (k sinA ) .sinBsinC = 2asinBsinC = 2a   =; ∆ acsinB ⇒
= sinB andsinC
= 
 ac  ab   2 ac ab 
8∆2
= =R.H.S.
abc

Illustration 18: The angle of elevation of the top point P of the vertical tower PQ of height h from a point A is 45o
and from a point B, the angle of elevation is 60o, where B is a point at a distance d from the point A measured along
the line AB which makes an angle 30ο with AQ. Prove that
= h d ( 3 −1 ) (JEE ADVANCED)

=
Sol: By using sine rule in ∆ABP , we can prove that h d ( 3 − 1)
In the figure, PQ represents a tower of height h. The angle of elevation of the point P from the point A on the
ground is
M a them a ti cs | 19.9

ο ο ο
⇒ ∠PAQ = 45ο ; ⇒ ∠PAB + ∠BAQ = 45 ; ⇒ ∠PAB + 30 = 45
30ο ]
∠PAB = 45ο − 30ο [Given ∠BAQ =
15ο 
∠PAB = ... (i)
45ο ; ⇒ ∠APB + ∠BPH = 45ο (given) ⇒ ∠APB + 30 =
(Given) ∠APH = 45ο ⇒ ∠APB =15ο  … (ii)
∠APB So BP=AB=d; ⇒ BP =
From (i) and (ii), we have ∠PAB = d  P

[Given AB=d]
45ο , ∠Q =90ο ⇒ ∠APQ = 45ο

o
Again ∠PAQ =

15
In ∆APQ , ∠PAQ =
∠APQ ⇒ AQ
= PQ
= h h
2 2 2 2 2 2 2
AP =PQ + AQ =h + h ⇒ AP =
2h ⇒ AP =2h
AB AP
Applying sine formula in ∆ABP , we get = H
ο
sin15 sin150ο d
B
2h  3 − 1 
d 2h
( )
o
45
⇒ = ⇒ d=   ⇒ d= 3 −1 h
o
30
sin15ο sin150ο 1  2 2  A Q
2 Figure 19.8

Illustration 19: A lamp post is situated at the middle point M of the side AC of a triangular plot ABC with BC=7m,
CA=8m and AB=9m. This lamp post subtends an angle tan−1 ( 3) at the point B. Determine the height of the lamp
post. (JEE ADVANCED)
PM
Sol: Here in ∆BMP ⇒ tan ∠PBM = , therefore by obtaining the value of BM we can find out the height of lamp
post. BM

Here, ABC is a triangular plot. A lamp post PM is situated at the mid-point M of the side AC. Here PM subtends an
angle tan−1 ( 3) at the point B. a=7m, b=8m and c=9m
a2 + b2 − c2 BC2 + CA2 − AB2
In ∆ABC , cos C = or cosC =
2ab 2BC.CA
72 + 82 − 92 49 + 64 − 81 32 2  … (i)
cosC
= = = =
2×7×8 112 112 7
A P
BC2 + CM2 − BM2 72 + 42 − BM2 65 − BM2
In ∆BCM, cosC = = ; cosC =
2BC.CM 2×7× 4 56 
2 65 − BM2 2
⇒ = ⇒ 65 − BM2 = × 56 = 16 [Using(i)] c=9 ) M
7 56 7 -1 (3
n b=8
ta
⇒ BM2 = 65 − 16 = 49 ⇒ BM =
7m =
PM PM
( ) PM
−1 C
In ∆BMP ⇒ tan ∠PBM = ⇒ tan tan = 3 3
⇒= ⇒ PM =
21m B c=7
BM BM 7
Figure 19.9
Hence, the height of the lamp post =21m.

8. PROPERTIES OF TRIANGLE

8.1 Circumcircle
A circle passing through the vertices of a triangle is called a circumcircle of the triangle. The centre
of the circumcircle is called the circumcentre of the triangle and it is the point of intersection of
the perpendicular bisectors of the sides of the triangle. The radius of the circumcircle is called Figure 19.10
the circumradius of the triangle and is usually denoted by R and is given by the following
1 9 . 1 0 | Solutions of Triangle

a b c abc a+b+c
=
formulae: R = = = Where ∆ is area of triangle and s = .
2sinA 2sinB 2sinC 4 ∆S 2

8.2 Incircle
The circle which can be inscribed within the triangle so as to touch all the three sides of the triangle is called
the incircle of the triangle. The centre of the incircle is called the incentre of the triangle and it is the point of
intersection of the internal bisectors of the angles of the triangle. The radius of the circle is called the inradius of
the triangle and is usually denoted by rin-Radius: The radius r of the inscribed circle of a triangle ABC is given by

( s − a) tan  A2  , r =
( s − b ) tan  B2  and =r ( s − c ) tan  C2 
∆      
(a) r = (ii) r =
s      
B   C  A C B   A 
asin   sin   bsin   sin   c sin   sin  
=(b) r = 2  2, r 2=
  2  and r 2  2 
A B  C
cos   cos   cos   Figure 19.11
2
  2
  2
A B  C
(c) r = 4R sin   .sin   .sin  
2 2 2

8.3 Centroid
In ∆ ABC , the mid-points of the sides BC, CA and AB are D,E and F respectively. The lines AD, BE and CF are called
medians of the triangle ABC. The points of concurrency of three medians is called the centroid. Generally it is
represented by G.
2 2 2 A
Also,
= AG = AD, BG BE and
= CG CF.
3 3 3 
Length of medians from Figure 9.12
F E
a2  b2 + a2 − c2 
⇒ AD2 = b2 + − ab  
4  2ab  G
 
2b2 + 2c2 − a2 1
⇒ AD2 = ⇒ AD
= 2b2 + 2c2 − a2
4 2 B C
D
1 1
Similarly, BE
= 2a2 + 2c2 − b2 and CF
= 2a2 + 2b2 − c2
2 2 Figure 19.12

8.4 Apollonius Theorem

(
AB2 + AC2= 2 AD2 + BD2 ) A
1 a  2
( ) ( )
Proof: 2 AD2 + BD2 = 2  2b2 + 2c2 − a2 +  = b2 + c2 = AB2 + AC2
 2 4  F
E

8.5 Orthocentre O

The point of intersection of perpendiculars drawn from the vertices on the


opposite sides of a triangle is called its orthocentre.Let the perpendicular
B D C
AD,BE and CF from the vertices A,B and C on the opposite sides BC,CA and
AB of ABC, respectively meet at O. Figure 19.13
Then O is the orthocentre of the ∆ ABC. The triangle DEF is called the pedal Triangle of the ∆ ABC.
M a them a ti cs | 19.11

Centroid (G) of a triangle is situated on the line joining its circumcentre (O) and orthocenter (H) show that the line
divides joining its circumcentre (O) and orthocenter (H) in the ratio 1:2. A
Proof: Let AL be a perpendicular from A on BC, then H lies on AL. If OD is
perpendicular from O on BC, then D is mid-point of BC.
∴ AD is a median of ∆ ABC. Let the line HO meet the median AD at G. Now, we H G
shall prove that G is the centroid of the ∆ ABC. Obviously, ∆ OGD and ∆ HGA O
are similar triangles. 
B C
∴ OG GD OD R cos A 1
= = = =
L D
HG GA HA 2R cos A 2 Figure 19.14
1
∴ GD= GA ⇒ G ⇒ is centroid of ∆ ABC and OG : HG = 1 : 2
2
The distances of the orthocenter from the vertices and the sides: If O is the orthocenter and DEF the pedal triangle
of the ∆ ABC, where AD, BE, CF are the perpendiculars drawn from A,B,C on the opposite sides BC,CA,AB respectively,
then
(i) OA = 2R cosA, OB = 2R cosB and OC = 2R cosC
(ii) OD = 2R cosBcosC, OE=2R cosCcosA and OF =2R cosAcosB, where R is circumradius.
R
(iii) The circumradius of the pedal triangle =
2
(iv) The area of pedal triangle= 2∆ cos A cosBcosC .

(v) The sides of the pedal triangle are acos A, bcosB and ccosC and its angles are π − 2A, π − 2B and π − 2C.
(vi) Circumradii of the triangles OBC, OCA, OAB and ABC are equal.

PLANCESS CONCEPTS

•• The circumcentre, centroid and orthocentre are collinear.


•• In any right angled triangle, the orthocentre coincides with the vertex containing the right angle.
•• The mid-point of the hypotenuse of a right angled triangle is equidistant from the three vertices of
a triangle.
•• The mid-point of the hypotenuse of a right angled triangle is the circumcentreof the triangle.
•• The centroid of the triangle lies on the line joining the circumcentre to the orthocentre and divides
it in the ratio 1:2
Vaibhav Krishnan (JEE 2009,AIR 22)

9. PEDAL TRIANGLE
E
The triangle formed by the feet of the altitudes on the side of
F
a triangle is called a pedal triangle.
H
In an acute angled triangle, orthocentre of ∆ ABC is the
in-centre of the pedal triangle DEF.
Proof: Points F,H,D and B are concyclic
π
⇒ ∠FDH = ∠FBH = ∠ABE = −A
2 B D
C
Similarly, points D, H, E and C are concyclic Figure 19.15
1 9 . 1 2 | Solutions of Triangle

π
⇒ ∠HDE = ∠HCE = ∠ACF = −A
2
Thus, ∠FDH = ∠HDE ⇒ AD is the angle bisector of ∠FDE . Hence, altitudes of ∆ABC are internal angle bisectors of
the pedal triangle. Thus, the orthocentre of ∆ ABC is the incentre of the pedal triangle DEF.

Sides of pedal triangle in acute angled triangle


In ∆AEF, AF
= b cos A, AE
= c cos A
By cosine rule, EF2 = AE2 + AF2 − 2AE × AFcos ( ∠EAF )

EF2 b2 cos2 A + c2 cos2 A − 2bccos3 A


⇒=


= (
EF2 cos2 A b2 + c2 − 2bccos
= )
A cos2 A a2 ( ) ⇒ EF =
acos A

Circumradius of pedal triangle


EF acos A acos A a
Let circumradius be R ' ⇒ 2R=' = = = = R ⇒ R' =
R/2
sin ( ∠ EDF ) sin ( π − 2A ) 2sinA cos A 2sinA

PLANCESS CONCEPTS

•• The circle circumscribing the pedal triangle of a given triangle bisects the sides of the given triangle
and also the lines joining the vertices of the given triangle to the orthocenter of the given triangle.
This circle is known as “Nine point circle”.
•• The circumcentre of the pedal triangle of a given triangle bisects the line joining the circumcentre of
the triangle to the orthocentre.
•• It also passes through midpoint of the line segment from each vertex to the orthocenter.
•• Orthocenter of triangle is in centre of pedal triangle.
Shrikant Nagori (JEE 2009, AIR 30)

10. ESCRIBED CIRCLES OF THE TRIANGLE


The circle which touches the sides BC and two sides AB and AC produced of a triangle ABC is called the escribed
circle opposite to the angle A. Its radius is denoted by r1 . Similarly, r2 andr3 denote the radii of the escribed circles
opposite to the angles B and C respectively. The centres of the escribed circles are called the ex-centres. The centre
of the escribed circle opposite to the angle A is the point of intersection of the external bisector of angles B and C.
The internal bisector also passes through the same point. This centre is generally denoted by I1 .

Formulae for r1 , r2 , r3
∆ ∆ ∆
In any ∆ABC , we have
= (i) r1
= ,r = ,r A
s −a 2 s −b 3 s −c 
A B C
(ii) r1 s =
= tan , r2 s=
tan , r3 s tan
2 2 2 B C
B C C A A B
cos cos cos cos cos cos
=(iii) r1 a= 2 2 , r b= 2 2, r c 2 2
2 3
A B C
cos cos cos
2 2 2
A B C A B C A B C
(iv) r1 4R
= sin co s cos , r2 4R
= cos sin cos , r3 4R cos cos sin
2 2 2 2 2 2 2 2 2 Figure 19.16
M a them a ti cs | 19.13

PLANCESS CONCEPTS

If I1 is the centre of the escribed circle opposite to the angle B, then

A B C A B C A B C
OI
= 1 R 1 + 8 sin .cos .cos ; OI
= 2 R 1 + 8 cos .sin .cos ; OI
= 3 R 1 + 8 cos .cos .sin
2 2 2 2 2 2 2 2 2
Where R is circum radius
•• The Sum of the opposite angles of a cyclic quadrilateral is 180o.
•• In a cyclic quadrilateral, the sum of the products of the opposites is equal to the product of diagonals.
This is known as Ptolemy’s theorem.
•• If the sum of the opposite sides of a quadrilateral is equal, then and only then a circle can be inscribed
in the quadrilateral.

•• If l1 , l2 and l3 are the centres of escribed circles which are opposite to A,B and C respectively and l
is the centre of the incircle, then triangle ABC is the pedal triangle of the triangle l1l2l3 and l is the
orthocenter of triangle l1l2l3 .
•• The circle circumscribing the pedal triangle of a given triangle bisects the sides of the given triangle
and also the lines joining the vertices of the given triangle to the orthocenter of the given triangle.
This circle is also known as nine point circle.

•• The circumradius of a cyclic quadrilateral, R =


1 ( ac + bd)( ad + bc )( ab + cd)
4 ( s − a)( s − b )( s − c )( s − d)
Nitish Jhawar (JEE 2009, AIR 7)

11. LENGTH OF ANGLE BISECTOR AND MEDIANS


If ma and βa are the lengths of a median and an angle bisector from the angle A then,
A
2bc cos
ma
=
1
2
2b2 + 2c2 − a2 and=
βa
b+c a b c
4
(
2 . Note that m2 + m2 + m2= 3 a2 + b2 + c2
)
Illustration 20: The ratio of the circumradius and in radius of an equilateral triangle is________ (JEE MAIN)

Sol: Here, as we know, all angles of an equilateral triangle are 600 , therefore by using formula of Circumradius and
In radius we can obtain the required ratio.
r acos A + bcosB + ccosC
= 600= A= B= C
. In an equilateral triangle,=
(=
a + b + c ) cos600 1
R a+b+c (a + b + c) 2

Illustration 21: In a ∆ABC, a=18 and b=24cm and c=30cm then find the value of r1 , r2 and r3 . (JEE MAIN)
∆ ∆ ∆
Sol: As we know, r1 = , r2 = and r3 = . Hence, we can solve the above problem by using this formula.
s−a s −b s−c

a=18cm, b=24cm,c=30cm; ∴ 2s = a + b + c = 72cm ; s=36cm But,=


∆ s ( s − a)( s − b )( s − c )

∆ 216 ∆ 216 ∆ 216


∆ =216 sq. units Then,
= r1 = = 12cm =
; r2 = = 18cm =
; r3 = = 36cm
s − a 18 s − b 12 s−c 6
So, r1 , r2 , r3 are 12cm, 18cm, and 36cm respectively.
1 9 . 1 4 | Solutions of Triangle

Illustration 22: If the exradii of a triangle are in HP, the corresponding sides are in ____ (JEE MAIN)

Sol: Here, in this problem, r1, r2 and r3 are in H.P.


1 1 1 s −a s −b s −c
⇒ , , are in A.P. ⇒ , , are in A.P. ⇒ s − a, s − b, s − c are in A.P.
r1 r2 r3 ∆ ∆ ∆

⇒ −a, − b, − c are in A.P. ⇒ a, b, c are in A.P.

b −c c −a a−b
Illustration 23: Find the value of + + . (JEE ADVANCED)
r1 r2 r3
∆ ∆ ∆
Sol: By using r1 = , r2 = and r3 = , we can solve the above problem.
s−a s −b s−c
(b − c ) + ( c − a ) + ( a − b ) =
(b − c )  s ∆− a  + ( c − a)  s −∆ b  + ( a − b )  s ∆− c 
     
r1 r2 r3      

=
( s − a)(b − c ) + ( s − b )( c − a) + ( s − c )( a − b )

s (b − c + c − a + a − b ) − ab − ac + bc − ba + ac − bc  0
= = = 0
∆ ∆

B C
Illustration 24: Find the value of the r cot cot . (JEE ADVANCED)
2 2 

Sol: Here, in this problem, r=4RsinA/2.sinB/2.sinC/2. By putting this value, we can solve the above problem.

cosB / 2 cosC / 2
rcotB/2.cotC/2 = 4R sinA / 2sinB / 2.sinC / 2. . [as r=4RsinA/2.sinB/2.sinC/2]
sinB / 2 sinC / 2

= 4R.sinA/2.cosB/2.cosC/2= r1 {as,r1 = 4R sinA/ 2.cosB/ 2.cosC/ 2}

∴ rcotB / 2.cotC / 2 = r1

12. EXCENTRAL TRIANGLE I3 A I2

The triangle formed by joining the three excentres I1 ,I2 and I3 of ∆ABC is
called the excentral or excentric triangle. Note that: I
B
B C
(i) The incentre I of ∆ABC is the orthocentre of the excentral ∆I1 I2 I3 .
2
 B o
-
2 2 90
(ii) ∆ABC is the pedal triangle of the ∆I1 I2 I3 .

(iii) The sides of the excentral triangle are


I1
A B C
4R cos , 4R cos and 4R cos and
2 2 2 Figure 19.17
π A π B π C
Its angles are − , − and − . A
2 2 2 2 2 2

(iv) Distance between the incentre and excentre   

A B C
=I I1 4R
= sin ; I I2 4R
= sin ; I I3 4R sin . 
2 2 2 B C
m D n
Figure 19.18
M a them a ti cs | 19.15

13. M-N THEOREM (RATIO FORMULA)


If D be a point on the side BC of a ∆ABC such that BD:DC = m:n and ∠ADC = α and ∠DAC =
θ , ∠BAC = β.
(a) (m + n) cot
= θ mcot α − ncot β (b) (m + n) cot
= θ ncotB − mcot C

BD m
Proof: (a) Given that, = and ∠ADC =
θ
DC n

( )
 ∠ADB = 180ο − θ ; ∠BAD = α and ∠DAC = β

( )
= 180ο − ( θ + β )
∴ ∠ABD = 180ο − α + 180ο − θ = θ − α and ∠ACD
BD AD
From ∆ABD, =  ... (i)
sin α sin ( θ − α )

DC AD DC AD
From ∆ADC, = or =  ... (ii)
sin β sin 180 − ( θ + β ) 
ο sin β sin ( θ + β )
 
BD sin β sin ( θ + β ) m sin β sin θ.cos β + cos θ.sin β
dividing (i) by (ii), then = or =
DCsin α sin ( θ − α ) n sin α sin θ.cos α − cos θ sin α

or msin θ sin β cos α − mcos θ.sin α.sin β= nsin α sin θ cos β + nsin α cos θ sin β
mcot α − mcot
= θ ncot β + ncot θ [dividing both sides by sin α sin β sin θ ] or m + n cot
= ( )
θ mcot α − ncot β
BD m
(b) Given = and ∠ADC = = 180ο − θ ; ∠ABD = Band ∠ACD = C
θ ; ∴ ∠ADB
DC n
( )
= 180ο − ( θ + C )
and ∠BAD = 180ο − 180ο − θ + B = θ − B ; ∴ ∠DAC
BD AD
and now from ∆ABD. =  … (i)
sin ( θ − B ) sinB

DC AD DC AD
and from ∆ADC, = or =  ... (ii)
sin 180 − ( θ + C )  sinC
ο sin ( θ + C ) sinC
 
BD sin ( θ + C ) sinC m sin θ cosC + cos θ sinC sinC
dividing (i) by (ii), then . = or, =
DC sin ( θ − B ) sinB n sin θ cosB − cos θ sinB sinB

or, msin θ cosCsinB + mcos θ sinCsinB =nsin θ sinCcosB − ncos θ sinBsinC


or, mcot C + mcot
= θ ncotB − ncot θ [dividing both sides by sinBsinCsinθ ]
or, (m + n) cot
= θ ncotB − mcot C

A B B C C A
Illustration 25: In a triangle ABC, if=
cot cot c,=
cot cot a and=
cot cot b,
2 2 2 2 2 2
1 1 1
then find the value of + + .  (JEE MAIN)
s −a s −b s −c

Sol: Here, by using trigonometric ratios of half angle, we can solve above problem.

A B s ( s − a) s (s − b) s 1 c 1 a 1 b
cot cot = × = c; c⇒
= =similarly =and =
2 2 ( s − b )( s − c ) ( s − c )( s − a) s−c s−c s s−a s s −b s

1 c 1 a 1 b
c⇒
= similarly
=Similarly =and =
s−c s s−a s s −b s
1 1 1 a + b + c 2s
So that + + = = = 2
s −a s −b s −c s s
1 9 . 1 6 | Solutions of Triangle

14. SOLUTION OF DIFFERENT TYPES OF TRIANGLE


In a triangle, there are six elements- three sides and three angles. In plane geometry, we have done that if three
of the elements are given, at least one of which must be side, then the other three elements can be uniquely
determined. The procedure of determining unknown elements from the known elements is called solving a
triangle.

Solution of a right angled triangle


Case I: When two sides are given: Let the triangle be right angled at C. Then we can determine the remaining
elements as given in the table.

Given Required
a a
(i) a, b tanA = , B = 90ο − A, c =
b sinA

a
(ii) a, c sinA
= = B 90ο − A
, b ccosA,=
c

Case II: When a side and an acute angle are given: In this case, we can determine the remaining elements as given
in the table.

Given Required
a
(i) a, A B = 90ο − A, b = acot A, c =
sinA

(ii) c, A B = 90ο − A, a = csinA, b = ccos A

Solution of a triangle in general

Case I: When three sides a, b, c are given: In this case, the remaining elements are determined by using the

following formulae.=
∆ s ( s − a)( s − b )( s − c ) , where 2s = a + b + c

2∆ 2∆ 2∆ A ∆ B  ∆ c ∆
sinA
= = , sinB = , sinC = . OR tan   = , tan   = , tan  
bc ac ab 2
  s ( )   ( )   ( − c)
s − a 2 s s − b 2 s s

Case II: When two sides a, b and the included angle C are given: In this case, we use the following formulae:

1  A −B  a −b  2C  A + B C asinC
∆ absinC, tan 
=  cot   ; 90ο − and c =
=
2  2  a + b  2 2 2 sinA

Case III: When one side a and two angle A and B are given: In this case, we use the following formulae to determine
the remaining elements.
asinC 1
A +B + C =180ο ; C= 180ο − ( A + B ) and c= ; ∆ = casinB
sinA 2
Case IV: When two sides a, b and the A opposite to one side is given: In this case, we use the following formulae.
b
sinB = sinA  ... (i)
a
asinC
C= 180ο − ( A + B ) , c =
sinA
From (i), the following possibilities will arise:
When A is an acute angle and a < bsinA .
M a them a ti cs | 19.17

b
In this case, the relation sinB = sinA gives that sinB > 1 , which is impossible. Hence no triangle is possible.
a
When A is an acute angle and a = bsinA .
In this case, only one triangle is possible which is right angled at B.
When A is an acute angle and a > bsinA
bsinA
In this case, there are two values of B given by sinB = 180ο and side c can
say B1 and B2 such that B1 + B2 =
a
asinC
be obtained by using c =
sinA

Some useful results:


Solution of oblique triangles:
The triangle which are not right angled are known as oblique triangles. The problems on solving an oblique triangle
lie in the following categories:
(a) When three sides are given
(b) When two side and included angle are given
(c) When one side and two angles are given
(d) When all the three angles are given
(e) Ambiguous case in solution of triangle

When the three sides are given: When three sides a, b, c of a triangle are given, then to solve it, we have to find
its three angles A,B,C. For this cosine rule can be used.

When two sides and included angle are given: Problem based on finding the angles when any two sides and
the angles between them or any two sides and the difference of the opposite angles to them are given, Napier’s
analogy can be used.

When one side and two angles are given: Problems based on finding the sides and angles when any two and
side opposite to one of them are given, then sine rule can be used.

When all the three angles are given: In this case unique solution of triangle is not possible. In this case only the
ratio of the sides can be determined.
a b c
For this the formula, = = can be used
sinA sinB sinC
Ambiguous case in solution of triangles: When any two sides and one of the corresponding angles are given,
under certain additional conditions, two triangles are possible. The case when two triangles are possible is called
the ambiguous case.
In fact, when any two sides and the angle opposite to one of them are given either no triangle is possible or only
one triangle is possible or two triangles are possible.
Now, we will discuss the case when two triangles are possible.

=
Illustration 26: Solve the triangle, if b 72.95,
= c 82.31,
= B 42ο 47'  (JEE MAIN)
sinC sinB
Sol: By using sine rule i.e. = , we can solve the given triangle.
c b
sinC sinB c sinB 82.31 × sin 42ο 47'
(i) To find C = ⇒ sinC = = = 0.7663
c b b 72.95

C = sin−1 ( 0.7663) C1 = 50ο1'12'' and C2 = 129ο58' 48''


1 9 . 1 8 | Solutions of Triangle

I solution II solution

C = 50ο1'12" C = 129ο58' 48"

A= 180ο − (B + C ) A= 180ο − (B + C )

( )
180ο − 42ο 47'+ 50ο1'12" = 87ο11' 48"
A= =180ο − (42ο 47'+ 129ο58' 48") = 7ο14'12"
To find a To find a
a b a b
= =
sinA sinB sinA sinB

bsinA 72.95 × sin87ο11' 48" ο


bsinA 72.95 × sin7 14'12"
a= = a= =
sinB sin 42ο 27' sinB sin 42ο 27'

a = 107.95 a = 13.62

∴ Two solutions are


A
C1 = 50ο1'12" A1 = 87ο11' 48" a1 = 107.95 C2 = 129ο58' 48" A2 = 7ο14'12" a2 = 13.62
Geometrically, we draw the triangle with given data c,b and angle B. 
c b
AN ( csinB ) b (exactly). The triangle is a right angled triangle.
b
(a) If= =
AN ( csinB ) > b , the triangle cannot be drawn.
(b) If=
=( csinB ) < b < c, two triangles are possible.
(c) If AN B C1 N C2

(d) b>c, only one triangle is possible. Figure 19.19

Illustration 27: In a triangle ABC, b=16cm, c=25cm, and B = 33ο15' . Find the angle C. (JEE MAIN)

Sol: Simply by using sine rule, we can find out the angle C.
sinC sinB
We know that, = [Here, b=16cm, c=25cm, B= 33ο15' ]
c b
c 25sin33ο15'
sinC
= =
b
sinB
16
= = 0.8567 ; C sin
= −1
( 0.8567 ) 58ο57' ; C1 = 58ο57' ; C2 =180ο − 58ο57' =121ο3'

PROBLEM SOLVING TACTICS


In the application of sine rule, the following points are to be noted. We are given one side a and some other side x
is to be found. Both these are in different triangles. We choose a common side y of these triangles. Then apply sine
rule for a and y in one triangle and for x and y for the other triangle and eliminate y. Thus, we will get the unknown
side x in terms of a.
A
In the adjoining figure, a is the known side of ∆ABC and x is the unknown side of a 
triangle ACD. The common side of these triangles is AC=y(say). Now, apply sine rule. B  y  D
a y x y
∴ = …(i) and = . … (ii) 
sin α sin β sin θ sin γ x
C
x sin α sin β asin β sin θ Figure 19.20
Dividing (ii) by (i) we get, = ; ∴x =
asin θ sin γ sin α sin γ
In case of generalized triangle problems, option verification is very useful using equilateral, isosceles or right angle
triangle properties. So, it is advised to remember properties of these triangles.
M a them a ti cs | 19.19

FORMULAE SHEET

(a) In ∆ABC, ∠A + ∠B + ∠C = π

(B + C
a) sin
(a) = ) sin ( π − A=) sinA
( C + A ) = cos ( π − B ) = − cos B
b) cos
(b)

A+ B π C C
c) sin
(c) = sin  − =
 cos
2 2 2 2
B+C π A A
d) cos
(d) = cos  − =  sin
2  2 2  2
a b c
(b) Sine rule: In, = = = 2R Where R = Circumradius and a, b, c are sides of triangle.
sinA sinB sinC

b2 + c2 − a2 a2 + c2 − b2 a2 + b2 − c2
(c) Cosine rule: cos A = , cosB = , cosC =
2bc 2ac 2ab
(d) Trigonometric ratios of half – angles:

(a) sin
A
=
( s − b )( s − c ) where 2s = a + b + c; (b) cos
A
=
s ( s − a)
; (c) tan
A
=
( s − b )( s − c )
2 bc 2 bc 2 s ( s − a)

(e) Area of a triangle


= : ∆ 1 bcsinA
= =
1
casinB
1
absinC
2 2 2
a+b+c
(f) Heron’s formula : = ∆ s ( s − a)( s − b )( s − c ) , where s = .
2
a b c abc
R
(g) Circumcircle Radius : = = = =
2sinA 2sinB 2sinC 4 ∆

( s − a) tan  A2  , r =
( s − b ) tan  B2  and =r ( s − c ) tan  C2 
∆      
(h) Incircle Radius : (a) r = ; (b) r =
s      
(i) Radius of the Escribed Circle :

∆ ∆ ∆
(a)
= r1 = , r = ,r
s −a 2 s −b 3 s −c
A B C
(b) r1 s =
= tan , r2 s=
tan , r3 s tan
2 2 2
B C C A A B
cos cos cos cos cos cos
(c) r1 a=
= 2 2 , r b= 2 2, r c 2 2
2 3
A B C
cos cos cos
2 2 2
A B C A B C A B C
=(d) r1 4R
= sin co s cos , r2 4R
= cos sin cos , r3 4R cos cos sin
2 2 2 2 2 2 2 2 2

(j) Length of Angle bisector and Median:


A
2bc cos
1 2 ⇒ m - length of median, β - length of bisector.
ma
= 2b2 + 2c2 − a2 and=
βa a a
2 b+c
1 9 . 2 0 | Solutions of Triangle

Solved Examples

JEE Main/Boards a c 1
= ⇒ sinC = 135ο or45ο
; ∴C =
sinA sinC 2
Example 1: In any triangle PQR, prove that,
(b + c ) cosP+ ( c + a) cosQ+ ( a + b ) cosR =a+b+c. C = 45ο ⇒ B = 105=
ο
;b
c sinB 100
=
sinC 2
( 3 +1 )
Sol: Simply, by using projection rule, we can solve the
above problem.
C = 135ο ⇒ B = 15
= ο
;b
c sinB 100
=
sinC 2
( 3 −1 )
given L.H.S. (b + c ) cosP + ( c + a) cosQ + ( a + b ) cosR
3
Example 4: In a triangle ABC, if a=3, b=4 and sinA = ,
= bcosP + ccosP + ccosQ + acosQ + acosR + bcosR 4
then find the value of ∠B .
= (bcosP + acosQ ) + ( ccosP + acosR ) + ( ccosQ + bcosR )
Sol: By using sine rule, we can obtain ∠B .
= c + b + a = R.H.S. [By using projection Rule]
sinA sinB b
We =
have, = or sinB sinA
Example 2: In any ∆ABC , a b a
Since, a=3, b=4, sinA=3/4,
a 2,b
If= = 3 + 1 and C = 60ο , solve the triangle.
4 3
B− A b − a C We get, sinB = × = 1
Sol: Here, by using tan = cot , 3 4
2 b+a 2 ∴ ∠B = 90ο
we can obtain the value of B – A.
Two sides and the included angle is given. Example 5: Find the smallest angle of the triangle
B− A b − a C 3 +1−2 3 −1 whose sides are 6 + 12, 48, 24 .
∴ tan = = cot = cot30ο 3
2 b+a 2 3 +1+ 2 3 +3
Sol: The smallest angles of a triangle are those angles
3 −1 tan60ο − tan 45ο whose opposite sides are small.
= =
3 + 1 1 + tan60ο tan 45ο Let a =
6 + 12, b =48, c =
24

(
tan 60ο − 45ο = )
tan15ο Here, c is the smallest side.
∠C is the smallest angle of the triangle.
B−A
∴ = 30ο ...(i)
15ο or B − A =
2 a2 + b2 − c2
Now cosC =
We know, A + B + C =180ο 2ab

⇒ A +B = 120ο ...(ii)
=
(=
48 + 24 3 ) + 48 − 24 3
Solving (i) and (ii), we get B = 75ο & A = 45ο 4 ( 3 + 3 ) .4 3 2

To find side c, we use sine rule So, ∠C =π / 6


a c 3
= = ; or c 2=
2 6
sinA sin60ο 2 Example 6: In a ∆ABC , tanA tanB tanC = 9 . For such
triangles, if tan2 A + tan2 B + tan2 C =
λ then find the
ο
=
Thus A 45
= , B 75ο and c = 6 . value of λ .

If A 30
Example 3: = = ο
, a 100,
= c 100 2, solve the Sol: Here, by solving
( tanA − tanB ) + ( tanB − tanC ) + ( tanC − tanA )
2 2 2
triangle >0,

Sol: Here, simply by using sine rule, we can obtain the we can obtain the value of λ .
required values.  tan2 A + tan2 B + tan2C− tanA tanB 
⇒ 2  > 0
 − tanB tanC − tanC tanA
 
M a them a ti cs | 19.21

( )
⇒ 3 tan2 A + tan2 B + tan2C − ( tanA + tanB + tanC ) > 0
2 Example 9: In any ∆ , prove that,
1 A 1 B 1 C s2
⇒ 3λ − ( tanA ⋅ tanB ⋅ tanC ) > 0 ;
2
cos2 + cos2 + cos2 =
a 2 b 2 c 2 abc
⇒ 3λ − 81 > 0 ∴ λ > 27
Sol: Here, simply by using trigonometric ratios of half
angle formula we can prove the above example.
Example 7: In a triangle, a, b and A are given and c1 , c2 1 A 1 B 1 C
are two values of the third side c. Find the sum of the Given L.H.S.= cos2 + cos2 + cos2
a 2 b 2 c 2
areas of two triangles with side a, b, c1 and a, b, c2 .
b2 + c2 − a2 1  s ( s − a)  1  s ( s − b )  1  s ( s − c ) 
Sol: Here, as we know cos A = , therefore, = .  + .  + . 
2bc a  bc  b  ac  c  ab 
     
by solving this equation we can obtain c1 and c2 .
s
=
abc
(s − a + s − b + s − c)
cos A =
b2 + c2 − a2
(
or c2 − 2bcos A c + b2 − a2 =
0 ) ( ) s s
2bc =
abc
{
3s − ( a + b=
+ c) } abc
(3s − 2s )
Which is a quadratic in c, whose roots are c1 andc2 ;
∴ c1 + c2 = 2bcos A and c1 c2 = b2 − a2 s2
== =R.H.S
∴ Sum of areas of two ∆ ’s with sides a, b, c1 & a, b, c2 ; abc
Hence proved.
1 1
= bc1 sinA + bc2 sinA
2 2
Example 10: In any ∆ABC , prove that
1 1 1
= b ( c1 + c2 ) sinA = b.2bcos A.sinA = b2 sin2A 8∆2
2 2 2 acos A + bcosB + c cosC = .
abc
2 ab C Sol: By using sine rule, we can obtain values of a, b and
Example 8: In any triangle ABC, if tan θ = sin
a−b 2 c and then by substituting these values in L.H.S. we can
( a b ) sec θ
Prove that c =− prove this.
a b c
= = = k (let)[by sine rule]
2 ab C sinA sinB sinC
Sol: As given, tan θ = sin . Hence, by solving this
a−b 2 Then, a k=
= sinA, b k sinB=
and c k sinC
a2 + b2 − c2
and using formula cosC = , we can solve Now, acos A + bcosB + ccosC
2ab
the above problem. = k sinA cos A + k sinBcosB + k sinCcosC
k
2 ab C = sin2A + sin2B + sin2C 
 tan θ = sin 2
a−b 2
k
C =  4 sinA sinBsinC  = 2k sinA sinBsinC
∴ ( a − b ) tan2 θ = 4absin2
2
2
2
2∆ 2∆
( )
or ( a − b ) sec2 θ − 1 = 4absin2
2 C
2
= 2asinBsinC
= 2a. .
ac ab
1 1 2∆ 2∆
C [∴ ∆
= =absinC acsinB ∴ sinB
= , sinC
= ]
or ( a − b ) sec θ= ( a − b ) + 4absin
2 2 2 2
2 2 bc ab
2
8∆2
 C = = R.H.S
or ( a − b ) sec2 θ= a2 + b2 − 2ab.  1 − 2sin2 
2
abc
 2

or ( a − b ) sec2 θ= a2 + b2 − 2abcosC
2
Example 11: In a ∆ABC , ∠=c 90ο , =
a 3, =b 4 and D
30ο . Find the length
is a point on AB so that ∠BCD =
 a2 + b2 − c2  of CD.
 cosC = 
 2ab 
Sol: Here, by using Pythagoras theorem and sine rule,
or ( a − b ) sec2 θ =c2 ; ∴ c =
2
( a − b ) sec θ we can obtain the length of CD.
1 9 . 2 2 | Solutions of Triangle

o Sol: Here, simply by using sine rule i.e.


In 
in ABC, ∠
∆ABC, ∠CC =
= 90
90°
( cosP ) a = 2R sinP cosP and so on, we can prove the
∴ AB2 = 32 + 42 = 25 above problem.
AB = 32 + 42 = 5 In ∆PQR, consider, PX = a1 , QY = a2 , RZ = a3 .
and Area of triangle PQR
3 4
∴ sinA= ,sinB= 1 1 1 P
5 5 =∆= = aa1 = ba2 ca
in ∆BCD, by sine rule 2 2 2 3
Z Y
BD CD 2∆ 2∆ 2∆
= =a1 = , a2 = , a3
sin30 ο sinB a b c Q R
sin30ο 1 5 5 X
∴ BD = .CD = × CD = CD ... (i) cosP cosQ cosR
sinB 2 4 8  + +
a1 a2 a3
And in ∆ACD, by sine rule
AD CD =
( cos P ) a + ( cosQ ) b + ( cosR ) c
= 2∆ 2∆ 2∆
sin60ο sinA
sin60ο 3 5 5 3 1
⇒ AD = CD = × CD =CD ... (ii) = 2R sinP cosP + 2R sinQ cosQ + 2R sinR cosR 
sinA 2 3 6  2∆ 
5 5 3 [using sine rule]
But BD+AD=AB ∴ CD + CD =
5
8 6 R
sin2P + sin2Q + sin2R 
=
15 + 20 3 2∆ 
Or CD = 5
24 R
= 2sin (P + Q ) cos (P − Q ) + 2sinR cosR 

Or CD =
=
24 24 4 3 − 3 ( ) 2∆  

48 − 9 2R sinR
3+ 4 3 = cos (P − Q ) − cos (P + Q ) 
2∆  
CD
Hence=
24
39
(
4 3=
−3
8
13
)
4 3 −3 ( ) [using P+Q+R= π ]
R sinR 2R
= 2sinP sinQ = sinP sinQ sinR
∆ ∆
JEE Advanced/Boards 2R a b c
= . . [using sine rule]
B−C ∆ 2R 2R 2R
Example 1: Prove that, acos
2
= (b + C ) sin A2 . abc 1 R 1 abc
= × = = [by using R= ]
Sol: By using sine rule i.e. 4∆ R 2
R 2 R 4∆

b + c sinB + sinC
= , we can prove the above example. Example 3: If the sides of a triangle PQR are in A.P and
a sinA
if its greatest angle exceeds the least angle by α ,
b + c sinB + sinC
= , [using sine Rule] show that the sides are in the ratio 1 − x : 1 : 1 + x
a sinA
1 − cos α
B+C B−C A B−C B−C where x =
2sin cos 2cos cos cos 7 − cos α
= 2 2 = 2 2 ,= 2
A A A A A Sol: As the sides of a given P
2sin cos 2sin cos sin
2 2 2 2 2 triangle are in A.P., by
A B−C considering the sides to be
∴ (b + c ) sin = acos .
2 2 a, a + d, a+2d and using
a+2d
sine rule, we can obtain the a+d

Example 2: If a1 , a2 , a3 are the altitudes of the triangle required result.


PQR, Consider the sides to be a,
a + d, a + 2 d : d > 0 Q R
cosP cosQ cosR 1 a
prove that, + + = .
a1 a2 a3 R Let P be the least angle and R be the greatest angle.
M a them a ti cs | 19.23

Example 4: Let O be a point inside a triangle PQR


Q 180ο − ( 2φ + α )
Let P = φ ; then R = φ + α and=
such that ∠OPQ = ∠OQR =∠ORP = θ . Show that
a a + 2d a+d cot
= θ cotP + cot Q + cotR .
= =
sin φ sin(φ + α ) sin( π − (2φ + α ))
2 (a+ d) Sol: Simply, by applying sine rule in ∆POQ and ∆QOR,
a a + 2d a+d
⇒ = = = …(i) we can prove the above problem.
sin φ sin(φ + α ) sin(2φ + α ) sin φ + sin (φ + α )
 P
From the first and the second term,

a a + 2d a sin φ P-
= = =
sin φ sin(φ + α ) a + 2d sin(φ + α )
O
By componendo and dividendo, we get,

-
a + a + 2d sin φ + sin(φ + α ) 

Q
= 
a + 2d − a − sin φ + sin(φ + α ) Q R-
R
 α α  α ∠POQ = π − Q and ∠QOR = π − R
2sin  φ +  .cos tan  φ + 
2(a+ d)  2 2 a+d  2
⇒ = ⇒ =
2d  α α d α Applying the sine rule in ∠POQ,
2cos  φ +  .sin tan
 2 2 2 c OQ csin θ
we have = ⇒ OQ =  … (i)
tan
α sin ( π − Q ) sin θ sinQ
d
⇒ = 2  ... (ii) Applying the sine rule in ∆QOR,
a+d  α
tan  φ +  OQ a asin (R − θ )
 2 we have = ⇒ OQ = ... (ii)
sin (R − θ ) sin ( π − R ) sinR
From the third and the fourth term of equation (i) we
get c sin θ asin (R − θ )
From (i) and (ii), we have =
a+d 2(a+ d) sin φ + sin (φ + α ) sinR sinR
= ⇒2=
sin(2 φ + α ) sin φ + sin (φ + α ) sin(2 φ + α ) Using Sine Rule we have
α α
cos
2  α
cos
2 2R sinR sin θ 2R sinP sin (R − θ )
⇒2= ⇒ cos  φ +  = =
 α  2  2 sinQ sinR
cos  φ + 
 2 sinR sin ( R − θ ) sin (P + Q )
= ; = cot θ − cotR
α sinP sinQ sinR sin θ sinP sinQ
4 − cos2
 α 2 
∴ tan  φ +  = … (iii) cot Q + cotP
= cot θ − cotR or cot
= θ cotP + cot Q + cotR .
 2  α
cos
2
From (ii) and (iii) we get, Example 5: In a triangle XYZ, the median XQ and the
perpendicular XP from the vertex X to the side QR
α α divide angle X into three equal parts. Show that
sin sin2
d 2 2
= =
a+d α α
4 − cos2 4 − cos2
2 2

1 − cos α
d 2 1 − cos α
=
⇒ = =x
a+d 1 + cos α 7 − cos α
4−
2
Required ratio is a : a + d : a + 2d
d d
1−
= :1:1+ 1 − x : 1 : 1+ x
=
a+d a+d
X X 3a2
cos sin2 = .
3 3 32bc
1 9 . 2 4 | Solutions of Triangle

Sol: By using the cosine rule in ∆ XYQ and ∆PQR and Area of triangle XYZ = s ( s − a)( s − b )( s − c )
then subtracting them, we’ll get the result.
X a
As given, ∠YXQ =
∠QXP =
∠PXZ = ; YQ
= ZQ
= = ( α + β + γ ) αβγ
3 2
a
QP
= PZ
=
4
[Since ∆XQP and ∆XPZ are congruent] ∆ ( α + β + γ ) αβγ
∴r = =
s α+β+ γ
XQ = XZ =b ∴ In ∆ XYQ
a2 αβγ
c2 + b2 − 2 2 2 ⇒ r2 = .
cos
X
= 4 = 4c + 4b − a ...(i)
α+β+ γ
3 2bc 8bc 
Example 7: In any triangle ABC, if
c2 + b2 − a2
In∆PQR, we have cosP = a b c
2bc cos θ
= =, cos φ =, cos ϕ
b+c a+c a+b
X X c2 + b2 − a2
4 cos3 − 3cos = …(ii) where θ, φ, ϕ lie between 0 and π ,
3 3 2bc
 θ φ ϕ A B C
prove that tan tan tan = tan tan tan .
Subtracting (ii) from (i) we get 2 2 2 2 2 2
X X X Sol: Here, by using formula
cos − 4 cos3 + 3cos
3 3 3 θ
1 − tan2
2 a
4c2 + 4b2 − a2 c2 + b2 − a2 cos θ = and = cos θ ,we can
= − 2 θ b +c
8bc 2bc 1 + tan
2
X 3a2 solve the above problem.
2 X
4 cos  1 − cos = ; θ
3 3  8bc 1 − tan2
a 2
= cos= θ
X 2X 3a2 b+c
1 + tan2 θ
cos sin = .
3 3 32bc 2
By componendo and dividendo,
Example 6: If α , β and γ are the distances of the 2 a+b+c θ 2s − 2a s − a
= ; ⇒ tan2 = =
vertices of a triangle XYZ from nearest points of contact 2 θ b + c − a 2 2s s
2 tan
of the incircle with sides of ∆ XYZ , 2
φ s −b ϕ s−c
αβγ Similarly, tan2 = and tan2 =
prove that r 2 = . 2 s 2 s
α+β+ γ
X ∴ tan
= 2 θ φ
tan2 tan2
ϕ (=
s − a)( s − b )( s − c ) ∆2
2 2 2 s3 s4
 
θ φ ϕ ∆
∴ tan tan tan = ...(i)
2 2 2 s2
L K
A B C
O Now tan tan tan
2 2 2
 

=
( s − b )( s − c )( s − c )( s − a) ( s − a)( s − b )
Y  J  Z s ( s − a) s(s − b) s (s − c)

Sol: Here, as we know, ∆ = s ( s − a)( s − b )( s − c ) and


= =
( s − a)( s − b )( s − c ) ∆ ...(ii)
r= .
∆ s3 s2
s
From (i) and (ii)
Given XL = XK = α ,YL = YJ = β ,ZJ = ZK = γ
θ φ ϕ A B C
2s =XY + YZ + ZX =c + a + b = 2 α + 2 β + 2 γ tan tan tan = tan tan tan .
2 2 2 2 2 2
M a them a ti cs | 19.25

Example 8: The bisector of angle X of triangle XYZ Sol: Here, area of cyclic quadrilateral = area of triangle
meets YZ at A. If XA=  then, prove that ABC + area of triangle BCD. Therefore by using cosine
X
rule in triangle ABD and BCD, we will be solving the
above example.
x x
2 2
Given AB=1, BD= 3
OA=OB=OD=OC=1=R (O being center of the circle),
in triangle ABD,
BD 3
=2R ⇒ =sinA ( Given circle is circumcircle
sinA 2
Z
Y A π 2π
of ∆ABD ) ⇒ A =;Hence C =
3 3
( ABCD is a cyclic quadrilateral)
2bc X a 12
(i)  = cos (ii) = 1−
b+c 2 b+c bc Using cosine rule in triangle ABD,

( AB ) + ( AD ) − (BD )
2 2 2
Sol: (i) Simply by using area of triangle formula, we can
cos A =
prove the above equation. 2AB.AD

1 1 + ( AD ) − 3
2
(ii) Here, by using cosine rule, we can prove it.
= or AD2 − AD − 2 =0
(i) Area of ∆XYZ =Area of ∆XYA+ Area of ∆XAZ 2 2AD
1 1 X 1 X
= bcsinX c sin + b sin or ( AD − 2 )( AD + 1 ) =
0 ; ∴ AD =
2
2 2 2 2 2
X 2bc X Using cosine rule in triangle BCD, we have
2bc cos=  (b + c ) ; ⇒  = cos
2 b+c 2
(BC ) + ( CD ) − 3
2 2
(BC ) + ( CD ) − (BD )
2 2 2
1
cosC = ⇒ − =
YA XY c YA ZA YA + ZA a 2 (BC ) . ( CD ) 2 2 (BC ) . ( CD )
(ii) = = or = = =
AZ XZ b c b c+b c+b
or (BC ) + ( CD ) + (BC )( CD ) − 3 =
2 2
0 ... (i)
YA a 
⇒ =  … (i)
c c+b Area of cyclic quadrilateral = Area of triangle ABC +
X c +  − YA 2 2 2 Area of triangle BCD
In triangle XYA, cos =
2 2c 3 3 1 π 1 2π
= .1.2sin + .BC.CD sin
X 4 2 3 2 3
⇒ −2c  cos + c2 + 2 =YA2
2 3=2+BC.CD or BC.CD=1  … (ii)
X
Substituting value of cos from (i) we get Solving (i) and (ii), we get BC=CD=1
2
Hence length of sides of cyclic quadrilateral are
2
 (b + c ) 2 c2b − 2c 2
QA =−2c +c + = AD=2, BC=CD=1.
2bc b
( QA )
2 2
 a  Example 10: The sides of a triangle are in A.P. and
Equation (A) gives = 
c2 c+b 3
its area is th of an equilateral triangle of the same
2 5
c2b − 2c  a  2 a perimeter. Prove that the sides are in the ratio 3:5:7.
or =  or 1− = .
bc2 c+b bc c + b
Sol: Here, sides of triangle are in A.P. Hence, by
3 3 considering the sides to be a – d, a and a + d and
Example 9: A cyclic quadrilateral ABCD of area
4 then by using area of triangle formula and the given
is inscribed in a unit circle. If one of its sides AB=1 and conditions, we can prove the given ratios.

∠A is acute and the diagonal BD = 3 ,find the lengths Let the sides be a – d, a and a + d.
of the other sides.
1 9 . 2 6 | Solutions of Triangle

2s=sum of the sides = 3a Now, ∆2 = Area of the equilateral triangle


3a 3 3
= × ( side ) = a2 
2
∴s = … (ii)
2 4 4
Now, ∆1 =Area of the triangle whose sides are in A.P. ∆1 3 a2 − 4d2 3
From question,
= = = or =
3a  3a   3a   3a  ∆2 5 a 5
=  − a + d  − a  − a − d
2 2  2  2  or 25a2 − 100d2 =
9a2 or 16a2 = 100d2
a 5
3a 3a 2 ⇒ =
=
4
( a + 2d)( a − 2d) or
= ∆1
4
a − 4d2  ... (i) d 2
Ratio of the sides= a – d : a + d
Perimeter of equilateral triangle = Perimeter of the
a a a 5 5 5
given triangle = −1 : : +1 = −1 : : +1
d d d 2 2 2
∴ 3 × One side of equilateral triangle = 3a 3 5 7
= : : =3:5:7
⇒ Side of the equilateral triangle = a 2 2 2

JEE Main/Boards

Exercise 1 Q.10. The angles of a triangle are in the ratio 2:3:7.


Find the ratio of its sides.
Q.1. If a=13, b=14, c=15, find r and R.
Q.11. In a triangle ABC, if 3a = b + c, prove that:
Q.2. In an equilateral triangle, find the relation between B C
the in radius and the circum radius. cot cot = 2.
2 2
a2 − b2 sin ( A − B ) Q.12. D is the mid point of BC in a triangle ABC. If AD is
Q.3. If in a triangle = , prove that it is
sin ( A + B )
2
a +b 2

perpendicular to AC, prove that cos A cosC =


(
2 c2 − a2 ).
either a right angled or an isosceles triangle.
3ac

4 sin2 C / 2
Q.4. If ∆ABC is scalene and cos A + cosB = Q.13. In ∆ABC , prove that:
then prove that A, B, C are in A.P. cos A cosB cosC
+ +
bcosC + ccosB ccos A + acosC acosB + bcos A
Q.5. Solve the triangle, if a = 2, b = 6,=
c 3 −1.
a2 + b2 + c2
= .
3 2abc
Q.6. If a=5, b=7 and sinA = , solve the triangle, if
possible. 4
Q. 14. Prove that
Q.7. Two sides of the triangle are of length 6 and (b + c − a){cot (B / 2) + cot ( C/ 2)} =2acot ( A / 2) .
4 and the angle opposite to smaller side is 30ο . How
many such triangles are possible? Find the length of Q.15. If p1 , p2 , p3 be the altitudes of a triangle ABC from
their third side and area. the vertices A, B, C respectively and ∆ be the area of
C
π
Q.8. If in a triangle ABC, ∠A = and AD is a median 2abcos2
the triangle ABC, prove that 1 1 1 2
3 + − =
p1 p2 p3 ∆ ( a + b + c )
then prove that 4AD2 = b2 + bc + c2 .

Q.16. The sides of a triangle are three consecutive


Q.9. If A=30o, b=8 and a=6, find c.
natural numbers and its largest angle is twice the
smallest one. Determine the sides of the triangle.
M a them a ti cs | 19.27

Q.17. Prove that Q.28. Let ABC be a triangle having O and I as its
circumcentre and incentre respectively if R and r be the
asin (B − C ) + bsin ( C − A ) + csin ( A − B ) =
0. circumradius and the inradius respectively, then prove
that ( IO )= R 2 − 2Rr. Further show that the triangle
2

Q.18. With usual notations, if in a triangle, BIO is a right angled if and only if b is the A.M. of a
and c.
b+c c+a a+b
ABC = = then prove that
11 12 13 Q.29. If α , β and γ are the altitudes of the ∆ABC from
cos A cosB cosC the vertices A, B and C respectively then
= = .
7 19 25 1 1 1 1
show that
2
+
2
+=
2 ∆
( cot A + cotB + cot C )
α β γ
Q.19. In a triangle ABC, prove that
a b c
+ +
cosBcosC cosCcos A cos A cosB
Exercise 2
=2a tan B.tan C.sec A Single Correct Choice Type

Q.20. Prove that the radius of the circle passing through Q.1. If A is the area and 2s the sum of the 3 sides of a
the centre of the inscribed circle of the triangle ABC triangle, then
a A s2 s2
and through the end points of the base BC is sec . (A) A ≤ (B) A =
2 2 3 3 2
Q.21. In a triangle ABC, Prove that s2
(C) A > (D) None
A B C 3
(b + c − a) tan 2
= ( c + a − b ) tan 2
= ( a + b − c ) tan 2
Q.22. 3 circles of radius 3, 4, 5 touches externally. Find Q.2. In a triangle ABC, CH and CM are the lengths of
the distance from point of contact to intersection point the altitude and median to the base AB. If a=10, b=26,
of tangents. c=32 then length (HM) is
(A) 5 (B) 7 (C) 9 (D) None
Q.23. Perpendiculars are drawn from the vertices A, B,
C of an acute angled triangle on the opposite sides, Q.3.In a triangle ABC, CD is the bisector of the angle C.
and produced to meet the circumcircle of the triangle.
C 1 1 1 
If these produced parts be α , β, γ respectively. If cos has the value and l ( CD ) = 6 , then  + 
2 3 a b
a a a has the value equal to
Show that + =
+ 2 ( tanA + tanB + tanC ) .
α β γ 1 1 1
(A) (B) (C) (D) None
2 2 2 2 9 12 6
Q.24. If in a triangle 8R = a + b + c , prove that the
triangle is right angled.
Q.4. With usual notations in a triangle ABC,
Q.25. If ∆ is the area of a triangle with side length a, b, ( I I1 ) . ( I I2 ) . ( I I3 ) has the value equal to
1
c then show that ∆ ≤
4
( a + b + c ) abc. (A) R 2 r (B) 2R 2 r (C) 4R 2 r (D) 16R 2 r

Q.26. In any triangle ABC, prove that


Q.5.With usual notation in a ∆ABC
A B C A B C
cot + cot + cot =cot cot cot 1 1 1 1 1 1 KR 3
2 2 2 2 2 2  +   +   +  =
 r1 r2   r2 r3   r3 r1  a b c
2 2 2

Q.27. In a triangle of base a, the ratio of the other two


Where K has the value equal to
sides is r(<1). Show that the altitude of the triangle is
a (A)1 (B)16 (C) 64 (D) 128
less than or equal to .
1 − r2
1 9 . 2 8 | Solutions of Triangle

Q.6. If the incircle of the ∆ABC touches its sides Q.2. If in a triangle PQR, sinP,sinQ,sinR are in AP, then
respectively at L, M and N and if x,y,z be the circumradii  (1998)
of the triangles MIN, NIL and LIM where I is the incentre
(A) The altitudes are in AP.
then the product xyz is equal to
1 1 (B) The altitudes are in HP.
(A) Rr 2 (B) rR 2 (C) Rr 2 (D) rR 2
2 2 (C) The medians are in GP.
Q.7. The product of the distances of the incentre from (D) The medians are in AP.
the angular points of a ∆ABC is
P  Q
(A) 4R 2r (B) 4Rr 2 (C)
( abc ) R (D)
( abc ) s π
Q.3. In a triangle PQR, ∠R = , if tan   and tan  
2 2
  2
s R
are the roots of the equation ax + bx + c= 0 ( a ≠ 0 ) ,
2

Q.8. If x,y and z are the distances of incentre from the then  (1999)
abc
vertices of the triangle ABC respectively then is
equal to xyz (A) a + b = c (B) b + c = a

A A (C) a + c = b (D) b = c
(A) ∏ tan (B) ∑ cot
2 2
1
A
(C) ∑ tan (D) ∑ sin
A Q.4. In a triangle ABC, 2acsin
2
( A − B + C) = (2000)

2 2
(A) a2 + b2 − c2 (B) c2 + a2 − b2
Q.9. For each natural number k, let Ck denotes the (C) b2 − c2 − a2 (D) c2 − a2 − b2
circle with radius k centimeters and centre at the
origin. On the circle Ck , a particle moves k centimeters Q.5. In a triangle ABC, let ∠C =π / 2 , if r is the inradius
in the counter-clockwise direction. After completing and R is the circumradius of the triangle, then 2(r + R)
its motion on Ck , the particle moves to Ck +1 in the is equal to (2000)
radial direction. The motion of the particle continues
in this manner. The particle starts at (1,0). If the particle (A) a + b (B) b + c
crosses the positive direction of the x-axis for the first (C) c + a (D) a + b + c
time on the circle Cn then n equal to
(A)6 (B)7 (C)8 (D)9 Q.6. The number of integral points (integral point
means both the coordinates should be integers) exactly
2cos A cosB 2cosC a b in the interior of the triangle with vertices (0, 0), (0, 21)
Q.10. If in a triangle ABC + + = +
a b c bc ca and (21, 0) is (2003)
then the value of the angle is
(A) 133 (B) 190 (C) 233 (D) 105
π π π π
(A) (B) (C) (D)
8 4 3 2 Q.7. The sides of a triangle are in the ratio 1 : 3 : 2 ,
then the angles of the triangle are in the ratio  (2004)
(A) 1:3:5 (B) 2:3:2 (C) 3:2:1 (D) 1:2:3
Previous Years’ Questions
π π Q.8. There exists a triangle ABC satisfying the conditions
Q.1. In a triangle ABC, =
∠B and= ∠C . Let D
3 4  (1986)
sin ∠BAD π π
divides BC internally in the ratio 1:3, then (A) bsinA
= a, A < (B) bsinA > a, A >
sin ∠CAD 2 2
is equal to (1995) π π
(C) bsinA > a, A < (D) bsinA < a, A < , b>a
1 1 2 2
1 2
(A) (B) (C) (D)
6 3 3 3
M a them a ti cs | 19.29

Q.9. A polygon of nine sides, each of length 2, is Q.18. Prove that a triangle ABC is equilateral if and only
inscribed in a circle. The radius of the circle is ….. if tanA + tanB + tanC =
3 3. (1998)
 (1987)
Q.19. In is the area of n sided regular polygon inscribed
Q.10. The sides of a triangle inscribed in a given in a circle of unit radius and On be the area of the
circle subtend angles α , β and γ at the centre. polygon circumscribing the given circle,
The minimum value of the arithmetic mean of  2 
 π  π  π On   21n  
cos  α +  , cos  β +  and cos  γ +  is equal to …… prove that I= 1 + 1 −    (2003)
 2  2  2
n
2   n  
 
 (1987)
Q.20. Circle with radii 3,4 and 5 touch each other
Q.11. If the angles of a triangle are 30ο and 45ο and externally, if P is the point of intersection of tangents to

the included side is ( )


3 + 1 cm, then the area of the
these circles at their points of contact. Find the distance
of P from the point of contact.  (2005)
triangle is…….  (1988)
Q.21 Consider a ∆ABC and let a, b and c denote
Q.12. If in a triangle ABC the lengths of the sides opposite to vertices A,B,C
respectively. Suppose a = 6, b = 10 and the area of the
2cos A cosB 2cosC a b
+ + = + then the value of the triangles is 15 3 . If ∠ACB is obtuse and if r denotes
a b c bc ca
angle A is …….. degree.  (1993) the radius of the incircle of the triangle, then r 2 is equal
to……… (2010)

Q.13. If p1 , p2 , p3 are the perpendiculars from the Q.22 In a ∆ PQR , if 3 sin P + 4 cos Q = 6 and 4 sin Q +
vertices of a triangle to the opposite sides, 3 cos P = 1, then the angle R is equal to  (2012)
a2b2c2 6π π π 3π
prove that p1p2p3 = (1978) (A) (B) (C) (D)
8R 3  6 6 4 4

Q.14. If p1 , p2 , p3 are the altitudes of a triangle from Q.23 ABCD is a trapezium such that AB and CD are
the vertices A, B, C and ∆ the area of the triangle, then parallel and BC ⊥ CD. If ∠ ABD = θ , BC = p and CD =
q then AB is equal to :  (2013)
1 1 1 2ab C
prove that + − = cos2 (1978)
p1 p2 p3 ( a + b + c ) ∆ 2 (p2 + q2 )sin θ p2 + q2 cos θ
 (A) (B)
p cos θ + qsin θ p cos θ + qsin θ

Q.15. If in a triangle ABC, a= 1 + 3 cm, b=2cm and 2
p +q 2
(p2 + q2 )sin θ
(C) (D)
∠C = 60ο , then find the other two angles and the third p2 cos θ + q2 sin θ (pcos θ + q sin θ)2
side.  (1978)

Q.24 If the angles of elevation of the top of lower from


Q.16. The ex-radii r1 , r2 , r3 of ∆ ABC are in HP, show three collinear points A, B and C, on a line leading to
that its sides a,b,c are in AP. (1983) the foot of the lower, are 30o, 45o and 60o respectively,
then the ratio, AB : BC, is  (2015)
Q.17. The sides of a triangle are three consecutive (A) 3 : 1 (B) 3: 2
natural numbers and its largest angle is twice the
smallest one. Determine the sides of the triangle. (C) 1 : 3 (D) 2 : 3
 (1991)
1 9 . 3 0 | Solutions of Triangle

JEE Advanced/Boards

Exercise 1 2: ( )
3 + 2 . Find the acute angles B & C. Also find

Q.1 Given a triangle ABC with AB=2 and AC=1. Internal the ratio of the two sides of the triangle other than the
Bisector of ∠BAC intersects BC at D. If AD=BD and ∆ hypotenuse.
is the area of triangle ABC, then find the value of 12 ∆2 .
Q.12 If a, b, c are the sides of triangle ABC satisfying
Q.2 In a triangle ABC, let angles A, B, C are in G.P. with  c
log  1 +  + loga − logb − log2 .
common ratio 2. If circumradius of triangle ABC is 2,  a

(
then find the value of b−1 + c−1 − a−1 ) ( ) (
Also a 1 − x2 + 2bx + c 1 + x2 =)
0 has two equal roots.

Find the value of sinA + sinB + sinC .


Q.3 In a triangle ABC, BD is a median.

3 π
If l (BD )
= .l ( AB ) and=
∠ DBC . Q.13 Given a triangle ABC with sides a= 7, b = 8 and c =
4 2
( ∑ sinA ) /  ∑ cot A2 
Determine the ∠ ABC .  
5. If the value of the expression
 
P p
Q.4 In an isosceles ∆ ABC, if the altitudes intersect on can be expressed in the form where p, q ∈ N and
the inscribed circle then find the secant of the vertical q q
angle ‘A’. is in its lowest form find the value of (p + q).

Q.5 ABCD is a rhombus. The circumradii of ∆ ABD and Q.14 If r1 =r + r2 + r3 , then prove that the triangle is a
∆ ACD are 12.5 and 25 respectively. Find the area of right angled triangle.
rhombus.
Q.15 If two times the square of the diameter of the
Q.6 In a triangle ABC if a2 + b2 =
101c2 then find the circumcircle of a triangle is equal to the sum of the
cotC squares of its sides then prove that the triangle is right
value of . angled.
cot A + cotB
Q.7 The two adjacent sides of a cyclic quadrilateral are Q.16 In acute angled triangle ABC, a semicircle with
2 & 5 and the angle between them is 60ο . If the area of radius ra is constructed with its base on BC and tangent
the quadrilateral is 4 3 , find the remaining two sides. to the other two sides rb and rc are defined similarly. If
r is the radius of the incircle of triangle ABC, then prove
Q.8 If in a ∆ABC , a = 6, b = 3 and cos(A-B) = 4/5 then 2 1 1 1
find its area. that, = + +
r ra rb rc

Q.9 The triangle ABC (with side lengths a,b,c as usual) Q.17 In a right angled triangle ABC, ∠C = 90ο and sides
satisfies loga2 = logb2 + logc2 − log ( 2bc cos A ) . What AC, AB are roots of the equation 2 + y 2 = 3y. If the
internal angle bisector of angle A intersects BC at D
can you say about this triangle? such that BD : CD= x2 + 1 : 2x , then find the sum of all
A
Q.10 The sides of a triangle are consecutive integers n, possible values of tan .
2
n+1 and n+2 and the largest angle is twice the smallest
angle. Find n. Q.18 Given a right triangle with ∠ A = 90ο . Let M be
the mid-point of BC. If the inradii of the triangle ABM
Q.11 The triangle ABC is a right angled triangle, r
and ACM are r1 and r2 then find the range of 1 .
right angle at A. The ratio of the radius of the circle r2
circumscribed to the radius of the circle escribed to
the radius of the circle escribed to the hypotenuse is, Q.19 If the length of the perpendiculars from the
vertices of a triangle A,B,C on the opposite sides are
M a them a ti cs | 19.31

p1, p2, p3, then prove that


1
+
1
+
1 1 1 1 1
= = + + . abc R3 4∆ ∆
(A) (B) (C) (D)
p1 p2 p3 r r1 r2 r3 2R 2 abc R 2
4R 2

Q.20 Tangents parallel to the three sides of ∆ABC are Q.7 In a ∆ ABC, a semicircle is inscribed, whose diameter
drawn to its incircle. If x, y, z be the lengths of the parts lies on the side C. Then the radius of the semicircle is
of the tangents within the triangle (with respect to the
2∆ 2∆ 2∆ c
x y z (A) (B) (C) (D)
sides a, b, c) then find the value of + + . a+b a+b−c s 2
a b c
Where ∆ is the area of the triangle ABC.
Exercise 2
cos A cosB cosC
Q.8 If in a ∆ ABC, = = then the
Single Correct Choice Type triangle is a b c

(A) Right angled (B) Isosceles


Q.1 If the median of a triangle ABC through A is
tanA (C) Equilateral (D) Obtuse
perpendicular to AB then has the value equal to
tanB
Q.9 If cosA + cosB + 2cosC = 2 then the sides of the ∆
1 1
(A) (B) 2 (C) −2 (D) − ABC are in
2 2
(A) A.P. (B)G.P. (C) H.P. (D)None
acosA + bcosB + ccosC
Q.2 In a ∆ABC , the value of is
a+b+c Q.10 The product of the arithmetic mean of the lengths
equal to
of the sides of a triangle and harmonic mean of the
r R R 2r lengths of the altitudes of the triangle is equal to:
(A) (B) (C) (D)
R 2r r R
(A) ∆ (B) 2 ∆ (C) 3 ∆ (D) 4 ∆
Q.3 With usual notation in a ∆ABC , if R=K [where ∆ is the area of the triangle ABC]
(r1 + r2 )(r2 + r3 )(r3 + r1 ) where k has the value equal to
r1r2 + r2r3 + r3r1 Q.11 If in a triangle sinA : sinC = sin(A−B) : sin(B−C) then

(A) 1 (B) 2 (C)1/4 (D)4 a2 : b2 : c2


(A) Are in A.P. (B) Are in G.P.
Q.4 In a right angled triangle the hypotenuse is 2 2 (C) Are in H.P. (D) None of these
times the perpendicular drawn from the opposite
vertex. Then the other acute angles of the triangle are Q.12 The sines of two angles of a triangle are equal to
π π π 3π π π π 3π 5 99
(A) & (B) & (C) & (D) & & . The cosine of the third angle is
3 6 8 8 4 4 5 10 13 101
(A) 245/1313 (B) 255/1313
Q.5 Let f, g, h be the lengths of the perpendiculars from
the circumcentre of the ∆ABC on the sides a, b and c (C) 735/1313 (D) 765/1313
respectively.
a b c abc Multiple correct choice type
If + + =λ then the value of λ is
f g h fgh
Q.13 If the side of a right angled triangle are
(A)1/4 (B)1/2 (C)1 (D)2
{cos2α + cos2β + 2cos ( α + β)} and
Q.6 If ‘O’ is the circumcentre of the ∆ABC and
{sin2α + sin2β + 2sin ( α + β)} , then the length of the
R1 ,R 2 andR3 are the radii of the circumcircles of triangle hypotenuse is
a b c
OBC, OCA and OAB respectively then + + has (A) 2 1 + cos ( α − β )  (B) 2 1 + cos ( α + β ) 
R1 R 2 R 3
the value equal to α −β α+β
(C) 4 cos2 (D) 4 sin2
2 2
1 9 . 3 2 | Solutions of Triangle

Match the Columns Q.3 Orthocentre of triangle with vertices (0,0), (3,4) and
(4,0) is  (2003)
Q.14 Let P be an interior point of ∆ABC . Match the
 5
correct entries for the ratios of the Area of ∆PBC : Area (A)  3,  (B) ( 3,12 )
of ∆ PCA: Area of ∆ PAB depending on the position of  4
the point P w.r.t. ∆ ABC.  3
(C)  3,  (D) ( 3,9 )
Column I Column II  4
(A) If P is centroid (G) (p) tanA:tanB:tanC
(B) If P is incentre (I) (q) sin2A:sin2B:sin2C Q.4 In a ∆ ABC, among the following which one is true?
(C) If P is orthocenter (r) sinA:sinB:sinC
 (2005)
(H)
A B + C 
(D) If P is circumcentre (s) 1:1:1 (A) (b + c ) cos asin 
= 
2  2 
(S)
(t) cosA:cosB:cosC B + C  A
(B) (b + c ) cos  =asin
 2  2
Q.15 In a ∆ ABC, BC=2, CA= 1 + 3 and ∠ C = 60ο . Feet
of the perpendicular from A, B and C on the opposite B − C  A
sides BC, CA and AB are D, E and F respectively and are (C) (b − c ) cos  =acos  
 2  2
concurrent at P. Now match the entries of Column I with
respective entries of Column II.
A B − C 
(D) (b − c ) cos   =
asin  
Column I Column II 2
   2 
(A) Radius of the circle (p)
6− 2 Q.5 In radius of a circle which is inscribed in a isosceles
circumscribing the
∆DEF , is 4 triangle one of whose angle is 2π / 3, is 3 , then area
of triangle is  (2006)
(B) (q)
Area of the ∆ 1
DEF, is 2 (A) 4 3 (B) 12 − 7 3

(C) Radius of the circle (r) (C) 12 + 7 3 (D)None of these


3
inscribed in the ∆
DEF, is 4 Q.6 If the angles A,B and C of a triangle are in an
arithmetic progression and if a,b and c denote the
(s)
6+ 2 lengths of the sides opposite to A,B and C respectively,
4 a c
then the value of the expression sin2C + sin2A is
c a
 (2010)
Previous Years’ Questions 1 3
(A) (B) (C)1 (D) 3
2 2
Q.1 Which of the following pieces of data does not
uniquely determine an acute-angled triangle ABC (R Paragraph (Q.7 to Q.9):
being the radius of the circumcircle)?  (2002)
Read the following Paragraph and answer the questions.
(A) a, sinA, sinB (B) a, b, c
Consider the circle x2 + y 2 = 9 and the parabola
(C) a, sinB, R (D) a, sinA, R y 2 = 8x . They intersect at P and Q in the first and the
fourth quadrants, respectively. Tangents to the circle at
Q.2 If the angles of a triangle are in the ratio 4:1:1, then P and Q intersect the x-axis at R and tangents to the
the ratio of the longest side to the perimeter is (2003) parabola at P and Q intersect the x-axis at S. (2007)

(
(a) 3 : 2 + 3 ) (b)1:3: Q.7 The ratio of the areas of the triangle PQS and PQR
is
(c) 1 : 2 + 3 (d)2:3
(A) 1 : 2 (B)1:2 (C)1:4 (D)1:8
M a them a ti cs | 19.33

Q.8 The radius of the circumcircle of the triangle PRS is Q.16 For atriangle ABC it is given that
3
(A)5 (B) 3 3 (C) 3 2 (D) 2 3 cos A + cosB + cosC = . Prove that the triangle is
2
equilateral.  (1984)
Q.9 The radius of the incircle of the triangle PQR is
4 8 Q.17 With usual notation, if in a triangle ABC
(A) (B)3 (C) (D)2
8 3 b+c c+a a+b
= = , then
11 12 13
Q.10 Internal bisector of ∠A of triangle ABC meets cosA cosB cosC
side BC at D. A line drawn through D perpendicular to prove that = = . (1984)
7 19 25
AD intersects the side AC at E and side AB at F. If a,b,c
represent sides of ∆ ABC, then (2006) Q.18 In a triangle ABC, the median to the side BC is
2bc A
(A) AE is HM of b and c (B) AD = cos 1
b+c 2 of length and it divides the angle A into
4bc A 11 − 6 3
(C) EF = sin (D)The ∆ AEF is isosceles
b+c 2 angles 30ο and 45ο . Find the length of the side BC.
 (1985)
Q.11 A straight line through the vertex P of a triangle
PQR intersects the side QR at the point S and the Q.19 If in a triangle ABC cos A cosB + sinA sinB sinC =
l,
circumcircle of the triangle PQR at the point T. If S is
show that a: b : c = 1 : 1 : 2 . (1986)
not the centre of the circumcircle, then  (2008)

1 1 2 1 1 2 Q.20 In a triangle of base a, the ratio of the other two


(A) + < (B) + >
PS ST QS × SR PS ST QS × SR sides is r ( < 1 ) . Show that the altitude of the triangle is

ar
1 1 4 1 1 4 less than or equal to . (1991)
(C) + < (D) + > 1 − r2
PS ST QR PS ST QR
Q.21 Let A1 , A2 , ……….. An be the vertices of an n-sided
π
Q.12 Let ABC be a triangle such that ∠ACB = and 1 1 1
6 regular polygon such that = + . Find
A1 A2 A1 A3 A1 A 4
let a,b and c denote the lengths of the sides opposite
the value of n. (1994).
to A,B and C respectively. The value(s) of x for which
a = x2 + x + l, b = x2 − 1 and =
c 2x + 1 is (are)  (2010)
Q.22 Consider the following statements concerning a
(
(A) − 2 + 3 ) (B) 1 + 3 triangle ABC
(i) The sides a, b, c and area of ∆ are rational
(C) 2 + 3 (D) 4 3
B C
(ii) a, tan , tan are rational
2 2
Q.13 In a triangle ABC, AD is the altitude from A. Given
(iii) a, sinA, sinB, sinC are rational.
abc
b>c, ∠C = 23ο and AD = , then ∠B = ……… Prove that (i) ⇒ (ii) ⇒ (iii) ⇒ (i) (1994)
b − c2
2
π
 (1994) Q.23 Let A, B, C be three angles such that A = and
4
Q.14 In a triangle ABC, a:b:c = 4:5:6. The ration of the tanB tanC = p . Find all positive values of P such that A,
radius of the circumcircle to that of the incircle is ………… B, C are the angles of triangle. (1997)
 (1996)
Q.24 Let ABC be a triangle having O and I as its
Q.15 ABC is a triangle, D is the middle point of BC. If AD circumcentre and incentre, respectively. If R and r are
is perpendicular to AC, then prove that the circumradius and the inradius, respectively, then
prove that ( IO )= R 2 − 2Rr. Further show that the
2

cos A cosC =
(
2 c2 − a2 ) (1980)
triangle BIO is a right angled triangle if and only if b is
3ac the arithmetic mean of a and c. (1999)
1 9 . 3 4 | Solutions of Triangle

Q.25 Let ABC be a triangle with incentre I and radius r. Q.30 Equation of the sides QR, RP are
Let D, E, F be the feet of the perpendiculars from I to the 2 2
sides BC, CA and AB respectively. If r1 , r2 and r3 are (A) y =x + 1, y =− x −1
the radii of circles inscribed in the quadrilaterals AFIE, 3 3
BDIF and CEID respectively, 1
(B) y =
= x, y 0
r r r rrr 3
prove that: 1 + 2 + 3 = 1 2 3 .
r − r1 r − r2 r − r3 (r − r1 )(r − r2 )(r − r3 ) 3
(C) y = x + 1, y =

3
x −1
(2000) 2 2

(D) y
= 3x, y 0
=
Q.26 If ∆ is the area of a triangle with side lengths a, b,
1 Q.31 z cos θ + i sin θ .
Let = Then the value of
c then show that ∆ ≤
4
( a + b + c ) abc . 15
Also show that the equality occurs in the above ∑ Im (z2m−1 ) at θ =2o is  (2009)
inequality if and only if a=b=c.  (2001) m=1
1 1
(A) (B)
o
Q.27 Let ABCD be a quadrilateral with area 18, with side sin 2 3sin 2o
AB parallel to the side CD and AB = 2CD. Let AD be
1 1
perpendicular to AB and CD. If a circle is drawn inside (C) (D)
o
the quadrilateral ABCD touching all the sides, then its 2sin 2 4 sin 2o
radius is  (2007)
(A) 3 (B) 2 (C) 3/2 (D) 1 Q.32 In a triangle ABC with fixed base BC, the vertex A
A
moves such that cosB + cosC = 4 sin2 . If a, b and c
2
Paragraph (Q.28 to 30) denote the length of the sides of the triangle opposite
A circle C of radius 1 is inscribed in an equilateral triangle to the angles A, B and C, respectively, then  (2009)
PQR. The points of contact of C with the sides PQ, QR, (A) b + c = 4a
RP are D, E, F. respectively. The line PQ is given by the
(B) b + c = 2a
3 3 3
equation 3x + y − 6 = 0 and the point D is  , . (C) Locus of point A is an ellipse
 2 2
 
Further, it is given that the origin and the centre of C are (D) Locus of point A is a pair of straight lines
on the same side of the line PQ.  (2008)
π
Q.33 For 0 < θ < , the solution(s) of
Q.28 The equation of circle C is 2
6
 (m − 1)π   mx 
2
(A) (x − 2 3) + (y − 1) =
1 2 ∑ cosec  θ +
4
 cosec  θ +  =4 2
4 
m=1   
2
 1 is (are)  (2009)
(B) (x − 2 3)2 +  y +  =
1
 2
π π π 5π
(A) (B) (C) (D)
2
(C) (x − 3) + (y + 1) =
1 2 4 6 12 12

(D) (x − 3)2 + (y − 1)2 =


1 Q.34 The centres of two circles C1 and C2 each of unit
radius are at a distance of 6 units from each other. Let P
be the mid point of the line segment joining the centres
Q.29 Points E and F are given by of C1 and C2 and C be a circle touching circles C1 and
C2 externally. If a common tangent to C1 and C passing
 3 3  3 1
(A)  , ,
 2 2
 
( 3,0 ) (B)  , ,
 2 2
 
( 3,0 ) through P is also a common tangent to C2 and C, then
the radius of the circle C is  (2009)

 3 3  3 1 3 3  3 1 Q.35 Let ABC and ABC’ be two non-congruent triangles
(C)  , ,  ,  (D)  , ,  , 
 2 2  2 2 2 2   2 2 with sides AB = 4, AC = AC’ = 2 2 and angle B = 30o.
       
The absolute value of the difference between the areas
M a them a ti cs | 19.35

of these triangles is  (2009) Q.41 Let P and Q be distinct points on the parabola
y2 = 2x such that a circle with PQ as diameter passes
π through the vertex O of the parabola. If P lies in the first
Q.36 Let ABC be a rectangle such that ∠ACB = and
6 quadrant and the area of the triangles ∆OPQ is 3 2 ,
let a, b and c denote the lengths of the sides opposite then which of the following is (are) the coordinates of
to A, B and C respectively. The value(s) of x for which P?  (2015)
a =x2 + 1, b =x2 − 1 and c = 2x + 1 is (are)  (2010)
(A) (4, 2 2) (B) (9, 3 2)
(A) −(2 + 3) (B) 1 + 3
1 1 
(C)  ,  (D) (1, 2)
(C) 2 + 3 (D) 4 3 4 2

Q.37 Let θ, ϕ ∈ [0, 2π] be such that Q.42 The circle C1 : x2 + y2 = 3, with centre at O, intersects
the parabola x2 = 2y at the point P in the first quadrant.
 θ θ
ϕ) sin2 θ  tan + cot  cos ϕ − 1 ,
2cos θ(1 − sin = Let the tangent to the circle C1 at P touches other two
 2 2 circles C2 and C3 at R2 and R3, respectively. Suppose C2
3
tan(2π − θ) > 0 and −1 < sin θ < − . and C3 have equal radii 2 3 and centres Q2 and Q3,
2
respectively. If Q2 and Q3 lie on the y-axis, then (2016)
Then ϕ cannot satisfy  (2012)
π π 4π (A) Q2Q3 = 12
(A) 0 < ϕ < (B) <ϕ<
2 2 3 (B) R 2R3 = 44 46
4π 3π 3π (C) Area of the triangle OR 2 R 3 is 6 2
(C) <ϕ < (D) < ϕ < 2π
3 2 2
(D) Area of the triangle PQ 2Q 3 is 4 2
7
Q.38 Let PQR be a triangle of area ∆ with
= a 2,=b Q.43 In a triangle XYZ, let x, y, z be the length of sides
2
5
and c = , where a, b and c are lengths of the sides opposite to the angle X, Y, Z, respectively, and 2s = x
2
of the triangle opposite to the angle at P, Q and R s−x s−y s−z
+ y + z. If = = and area of incircle of
4 3 2
2sin P − sin2 P 8π
respectively. Then equals  (2012) triangle XYZ is , then  (2016)
2sinP + sin 2 P 3
2 2
3 45  3   45  (A) Area of the triangle XYZ is 6 6
(A) (B) (C)   (D)  
4∆ 4∆  4∆   4∆ 
(B) The radius of circumcircle of the triangle XYZ is
Q.39 In a triangle PQR, P is the largest angle and 35
6
1 6
cos P = . Further the incircle of the triangle touches
3 X Y Z 4
the sides PQ, QR and RP at N, L and M respectively, (C) sin sin sin =
2 2 2 35
such that the length of PN, QL and RM are consecutive
 X + Y  3
even integer. Then possible length(s) of the side(s) of (D) sin2  =
the triangle is (are)  (2013)  2  5

(A) 16 (B) 18 (C) 24 (D) 22 Q.44 The orthocenter of the triangle F1MN is  (2016)

Q.40 In a triangle the sum of two sides is x and the  9  2 


(A)  − , 0  (B)  , 0 
product of the same two sides is y. If x2 – c2 = y, where  10  3 
c is the third side of the triangle, then the ratio of the
in-radius to the circumradius of the triangle is  (2014)  9  2 
(C)  , 0  (D)  , 6 
3y 3y 3y 3y  10  3 
(A) (B) (C) (D)
2x(x + c) 2c(x + c) 4x(x + c) 4c(x + c)
1 9 . 3 6 | Solutions of Triangle

PlancEssential Questions
JEE Main/Boards JEE Advanced/Boards

Exercise 1 Exercise 1

Q.10 Q.13 Q.15 Q.22 Q.8 Q.13 Q.17 Q.20

Q.24 Q.29 Q.27

Exercise 2 Exercise 2

Q.3 Q.4 Q.6 Q.9 Q.3 Q.6 Q.13 Q.14

Previous Years’ Questions Previous Years’ Questions

Q.5 Q.12 Q.14 Q.19 Q.4 Q.6 Q.11 Q.12

Q.22 Q.13 Q.22 Q.25

Answer Key

JEE Main/Boards
Exercise 1
65 π π 5π
Q.1
= r 4,R
= Q.2 2r=R A
Q.5= = ,B = ,C
8 4 3 12
Q.6 No triangle can be formed Q.7 sides: 2 3 ± 2 ; Area: 2 3 − 2 , 2 3 ± 2

Q.9
= c 4 3 ± 2 5 Q.10 2 : 2 : 3 +1 Q.16 4,5 and 6 Q.22. 5

Exercise 2

Single Correct Choice Type

Q.1 A Q.2 C Q.3 A Q.4 D Q.5 C Q.6 C

Q.7 B Q.8 B Q.9 B Q.10 D

Previous Years’ Questions


Q.1 A Q.2 B Q.3 A Q.4 B Q.5 A Q.6 B
3 1+ 3
Q.7 D Q.8. (A, D) Q.9. cosec20ο Q.10. − Q.11. sq. unit
2 2
ο
Q.12. 90 Q.15 c= 6 , ∠B= 45ο and ∠A= 75ο Q.17 4, 5, 6 unit
M a them a ti cs | 19.37

Q.20 5 Q.21 3 Q.22 B Q.23 A Q.24 A

JEE Advanced/Boards
Exercise 1
1
Q.1 9 Q.2 Q.3 120ο
4 sinA
A B C
Q.4 1/(1 – 8 sin sin sin ) Q.5 400 Q.6 50
2 2 2
Q.6 3 cms & 2 cms ( ) ( )(
Q.7 two triangles: 2 3 − 2 & 2 3 + 2 ; 2 3 − 2 & 2 3 + 2 sq. units ) ( )
5π π b
Q.9 Isosceles Q.10 4, 5, 6 Q.11 B= ; C= ; = 2+ 3
12 12 c
12
Q.12 Q.13 10 Q.14 90o
5
2
Q.15 right angled triangle Q.16 Q.17 8
r
 sinC sinB  1 1 1
Q.18 bc   Q.19 + + Q.20 1
 (1 + sinC)(1 + sinB)  r1 r2 r3

Exercise 2

Single Correct Choice Type


Q.1 C Q.2 A Q.3 C Q.4 B Q.5 A Q.6 C

Q.7 A Q.8 C Q.9 A Q.10 B Q.11 A Q.12 B

Multiple correct choice type

Q.13 A,C

Match the Columns

Q.14 A → s; B → r; C → p; D → q Q.15 A → q; B → r;C → p

Previous Years’ Questions


Q.1 D Q.2 A Q.3 C Q.4 D Q.5 C Q.6 D

Q.7 C Q.8 B Q.9 D Q.10 A,B,C,D Q.11 B,D Q.12 B


2 2
16 2(c − a )
Q.13. 113ο Q.14. Q.15 Q.16 Equilateral Q.17 7 : 19 : 25 Q.18. 2
7 3ac
1
2
:
1
2
:Q.19
1 = 1 : 1 : 2 Q.20 p ≤
ar
1 − r2
Q.21. 7 Q.22 (iii) ⇒ (i). ( ) (
Q.23. p ∈ −∞ ,0 ∪ 3 + 2 2, ∞ )
r1r2r3
Q.24 2b = a + c Q.25 Q.26 Equilateral Q.27 B Q.28 D
(r − r1 ) (r − r2 ) (r − r3 )

Q.29 A Q.30 D Q.31 D Q.32 B, C Q.33 C, D Q.34 8

Q.35 4 Q.36 A, B Q.37 A, C, D Q.38 C Q.39 B, D Q.40 B

Q.41 A, D Q.42 A, B, C Q.43 A, C, D Q.44 A


1 9 . 3 8 | Solutions of Triangle

Solutions

JEE Main/Boards sinA + sinB a+b


⇒ =2 ⇒ =2
sinC c
Exercise 1
b2 + c2 − a2
Sol 5: cosA =
∆ abc 2bc
Sol 1: r = ; R =
S 4∆ Using cosine formula, we get
π π 5π
∆= s(s − a)(s − b)(s − c) = 21(8)(7)(6) = 84 A= ,B= ,C=
4 3 12
abc (13)(14)(15) 65
R= = = 4
4∆ 4 × 84 8 Sol 6: a = 5, b = 7, sin A =
3
84
r= =4 a b
21 =
sinA sinB
3 2 28
a sin B = >1
∆ 4 a 15
Sol 2: r = = × a2 =
S 3a 2 3 no triangle can be formed.
abc a3 a
a b
R= = = Sol 7: =
4∆ 3 2 3 sinA sinB 
4 a
4
2r = R 6 4
=
sin30º sinB 4
2 2
a −b sinA cosB − cos A sinB 2 2
Sol 3: = sinB = =
2
a +b 2 sinA cosB + cos A sinB 6 3

a2 sinA cosB sinA cosB 2 1


= = . sinB = cosB = ±
b 2 cos A sinB sinB cos A 3 3
a cosB 1 3
⇒ = ⇒ a cosA = b cosB sinA = cosA =
b cos A 2 2
⇒ either cosA = 0 or cosB or a = b a2 + b2 − c2
cosC = = cos(180 – (A + B))
2ab
Sol 4: ∆ABC = –cosA cosB + sinA sinB
C
cosA + cosB = 4 sin 2
22 − c2 2± 3
2 ⇒ = ; we get c = 2 3 ± 2
8 6 2 3
A +B A −B C
2cos cos = 4sin2 1
2 2 2 Area = ab sinC= 2 3 – 2 or 2 3 + 2
2
C A −B C
2 sin cos = 4 sin2
2 2 2 π
Sol 8: ∠ A = A
A −B 3 
cos
A −B C 2 =2 Length of median
cos = 2 sin ⇒ 30
o

2 2 C
sin AD2 = AC2+CD2 – 2AC×CD×C
2
A +B A −B 1
C
2cos cos
A −B
2sin cos AD2 = 2b2 + 2c2 − a2
2 2 = 2 2 =2 2 B C
⇒ D
C C sinC
2cos sin
2 2
M a them a ti cs | 19.39

4AD2 = 2b2 + 2c2 – a2


a4 6b2 + 2c2 − a2
=
4AD2 = (b2 + c2 + bc)+(b2 + c2 – a2 – ac) 4 4
b2 + c2 − a2 1 a2– c2 = 3b2
cos A = =
2bc 2
(b2 + c2 − a2 )(a2 + b2 − c2 )
cos A cos C =
b2 + c2 – a2 – bc = 0 4b2ac
6 8 c ( −2b2 )(4b2 ) −2b2 2 ( −3b2 ) 2(c2 − a2 )
Sol 9: = = = = = =
sin30º sinB sinC 4b2ac ac 3 ac 3ac
2 ± 5
sin B = ⇒ cos B = Sol 13: From projection formula
3 3
1 a cos B + b cos A = C
sin 30º =
2 b2 + c2 − a2
⇒ sin C = sin(180 – 30 – B) = sin(150 – B) cos A =
2bc
= sin 150 cos B – cos 150 sin B
b2 + c2 − a2 c2 + a2 − b2 a2 + b2 − c2
So L.H.S. = + +
5 3 2 ± 5 +2 3 abc abc abc
=± + × =
6 2 3 6
6 (a2 + b2 + c2 )
C= .sin C= 4 3 ± 2 5 =
sin30 abc

Sol 10: ∠A = 2x; ∠B = 3x; ∠C = 7x B (s − c)(s − a)


Sol 14: tan =
2 s(s − b)
∠A + ∠B + ∠C = 180⇒12x = 180⇒x = 15º
⇒ ∠A = 30°; ∠B = 45º; ∠C = 105º B C s(s − b) s(s − c)
cot + cot = +
2 2 (s − a)(s − c) (s − a)(s − b)
a b c
= =
sinA sinB sinC
(s − b) s + (s − c) s a s
=
a : b : c = sin A : sin B : sin C (s − a)(s − b)(s − c) (s − a)(s − b)(s − c)
1 1 3 +1
= : : = 2 :2: 3 +1 a s
2 2 2 2 L.H.S. = (b+c−a)
(s − a)(s − b)(s − c)
b+c
Sol 11: a = , S = 2a 2a s(s − a) s(s − a)
3 = = 2a
(s − a)(s − b)(s − c) (s − b)(s − c)
B C S 2a
cot cot = = =2 A
2 2 S−a 2a − a = 2a cot = R.H.S.
2

1
Sol 12: AD = 2b2 + 2c2 − a2 ; AC = b Sol 15: Length of CD = (a sin B)
2
sin C = sin(180 – (A + B) = sinA cosB
A C

B C B A
D a cos B D

a 2b2 + 2c2 − a2 1 1 1
CD = = b2 + L.H.S. = + –
2 4 asinB c sinB c sinA
1 9 . 4 0 | Solutions of Triangle

2R 2R 2R (a + b − c) a2 + b2 − c2 49 + 36 − 25 5
= + – = 2R cosC = = =
bc ca ab abc 2ab 2(6)(7) 7
2(s − c) 4R(s − c) s−c 35cosB 75cosB
= 2R = = 5 cosA = =
abc abc ∆ 19 75
C C cos A cosB cosC
abcos2 2ab cos2 = =
2 = 2 7 19 25
=
4 ∆S (a + b + c)∆
acos A + b cosB + c cosC
Sol 19:
a a+1 a+2 cos A cosBcosC
Sol 16: = =
sinA sin(180 − A − 2A) sin2A = 2a tanB tanC secA
A < 180 – A – 2A < 2A ⇒ a cosA + b cosB + c cosC = 2a sinB sinC
A < 180 – 3A < 2A ⇒ a cosA + b cosB + cosC = 2(2R sinA) sinB sinC
4A < 180 < 5A ⇒ a cosA + b cosB + c cosC = 4R sinA sinB sinC
180 L.H.S. = a cosA + b cosB + c cosC
A < 45º; A > ⇒ A > 36º
5 = (2R sinA) cosA + 2RsinBcosB + 2RsinCcosC
a+1 a+2
a= = = R(sin 2A + sin 2B + sin 2C)
2
3 − 4 sin A 2cos A
from property of a triangle
a+1 a+2
a= = sin2A + sin2B + sin2C = 4 sinA sinB sinC
2
4 cos A − 1 2cos A
(we can prove this property using basic trigonometric
a+1 (a + 1)a2
a= = formulas)
a+2
2 4 + 4a
  −1 = 4R sinA sinB sinC
 a 
⇒ 4 + 4a = (a + 1)a Sol 20:
⇒a=4 A

Sol 17: L.H.S. = a sin(B – C) + b sin(C – A) + c sin(A – B)


D
= a sinB cosC – a cosB sinC + b sinC cosA – b cosC sinA
+ C sinA cosB – C cosA sinB B E C

= cosC (a sin B – b sin A) + cos B(C sin A – a sinC) + cos


C(a sin B – b sinA) O
= 0 = R.H.S..(as a sin B = b cosA)
Hence Proved.
F
b+c c+a a+b BE = EC = a/2
Sol 18: = =
11 12 13
∠BOE = ∠EOC
b=
+ c 11x=
 a 7x
 ∠DBE = ∠B/2
c=
+ a 12x=
 b 6x
 c 5x ∠DCE = ∠C/2
a=
+ b 13x=

B + C   180 − A  A
b2 + c2 − a2 36 + 25 − 49 1 ∠BDC = 180° –   = 180° –   = 90° +
cosA = = =  2   2  2
2bc 2(6)(5) 5
 A A
a2 + c2 − b2 49 + 25 − 36 38 19 ∠BFC = 180° –  90 +  = 90° –
cosB = = = =  2  2
2ac 2(7)(5) 70 35
M a them a ti cs | 19.41

 A Sol 24: 8R2 = a2 + b2 + c2


∠BOC = 2  90 −  = 180° – A
 2 a2 = 4R2 sin2A

A So, 2 = sin2A + sin2B + sin2C


∠BOE = 90° –
2 1 − cos2A 1 − cos2B 1 − cos2C
2= + +
 A A 2 2 2
∠EBO = 90° –  90 −  =
 2 2 4 = 3 – cos2A – cos2B – cos2C
a a A cos2A + cos2B + cos2C = 1
⇒ BE = ⇒ BO = sec
2 2 2
2cos(A + B) cos(A – B) + 2 cos2C – 1= 1

A B 2cosC[cosC – cos(A – B)] = 0


Sol 21: (b + c – a) tan = (c + a – b)tan
2 2 cosC [cos(A + B) + cos(A – B)] = 0
C cosA cosB cosC = 0
= (a + b – c)tan
2
So one angle should be 90º.
b + c – a = 2(s – a)
c + a – b = 2(s – b) 1
Sol 25: ∆≤ (a + b + c)abc
a + b – c = 2(s – c) 4

A 1 (abc)
r = (s – a) tan   R.H.S. = (a + b + c)abc = S
4 8
2
Hence proved. S  abc   S   abc  ∆2 R
=  ∆ =    = ∆
2  4∆   ∆   4∆  2 2r
Sol 22: C1C3 = 3 + 5 = 8  C3

C 3C 2 = 5 + 4 = 9 1
=∆
C 1C 2 = 3 + 4 = 7 B A B C
A 8 sin sin sin
2 2 2
OA = OB = OC = k(let’s say)
C1 C2
since C3C is angle bisector of A B C A B C
angle ∠C1C3C2. C Sol 26: cot + cot + cot = cot cot cot
2 2 2 2 2 2
So OA is radius of incircle of triangle ∆C1C2C3.
B C C A A B
tan tan +tan tan + tan tan
∆ 12(12 − 7)(12 − 8)(12 − 9) 2 2 2 2 2 2
OA = r = =
S 12
A (s − b)(s − c)
= 5 tan =
2 s(s − a)

Sol 23: DO1 = a, EO2 = b, FO3 = g s − a s − b s − c 3s − (a + b + c)


L.H.S. = + + = =1
s s s s
A
O2 a
Sol 27: b cos B ≤
O3  1 − r2

F E A
O
D
B  C
br b
O1

∠BOA = 2C
∠A B a
C
1 9 . 4 2 | Solutions of Triangle

 a2 + b2r 2 − b2  a s
3/2
b  ≤   ≥ [(s – a)(s – b)(s – c)]1/2
 2abr  1 − r2
  3
a2 + b2 (r 2 − 1) 1 5(s)3/2
≤ ≥ [s(s – a)(s – b)(s – c)]1/2
2a2r 1 − r2 3 3
s2
A≤
Sol 28: 3 3
A

Sol 2: (C) CH =b sin C = 2R sin B sin C


C

B C
a D

let’s say B lie on center and BC lies on x-axis B(0, 0),


A B
C(a, 0) 16
H M
32
A(c cosB, csinB )

 c  AM = 16, AC = b = 26, CB = 10 = a
BD =  a
b +c  b2 + c2 − a2 
AH = b cos A= b  
 2bc 
 
Sol 29: AD = α = C sinB = b sinC = (2R) sinB sinC
(26)2 + (32)2 − (10)2
= = 25
A 2(32)
HM = 25 – 16 = 9

 b 
F E Sol 3: (A) AB = C; AD =  C
b +a
C
B C
D
1 1
R.H.S. = ∑ ∆ ( cot A ) = ∑ ∆ 2 ∆ cot A
1 1 cos A 1 1
=
∆ 2 ∑ 2 bc sinA =
sinA ∆2
∑ 2 bc cos A
A B
D
2 2 2
1 b +c −a 1 length of angle bisector = 6
=
2∆ 2 ∑ 2
=
4 ∆2
∑ a2
c
2ab cos
1
2
2 ⇒ 1 + 1 = 1
= ∑ a  = ∑
  =
 2∆  α2 a+b a b 9

A
Exercise 2 Sol 4: (D) II1 = 4R sin
2
1 A B C
Sol 1: (A) Area = s(s − a)(s − b)(s − c) = ab sinC (II1)(II2)(II3) = 64R3 sin sin sin
2 2 2 2

(s − a) + (s − b) + (s − c)  A B C
≥ [(s – a)(s – b)(s – c)]1/3 = 16R2  4R sin sin sin  = 16 R2r
3  2 2 2
M a them a ti cs | 19.43

∆ A
Sol 5: (C) r1 =
s−a

1 1 s −a+ s −b c
+ = = O
r1 r2 ∆ ∆
B C
abc a3b3c3 64 64R 2
L.H.S. = = = a
∆3 64∆3 a2b2c2 a2b2c2 R=
2sinA
K = 64
 2A B C
 sin sin sin 
xyz = abc  2 2 2
Sol 6: (C) ON = r = OL = OM  sinA sinB sinC 
 
∠NOL = 180 – B  
 A B C
A xyz = abc  tan tan tan 
 2 2 2

abc A B C A
= cot cot cot = Scot
N M xyz 2 2 2 2
O
Sol 9: (B) arc length = k

B C radius of circle = k
L
C
180 − (180 − B) B
∠ONL =
2
=
2 
14 ,0
r k(θ) = k ⇒ θ = 1 radius
R of DNOL =
2sin ∠ONL
Particle moves to next circle after completing arc length
r 3 = k.
xyz =
A B C To cross positive x-axis
8 sin sin sin
2 2 2
n(θ) > 2p
A B C
r = 4R sin sin sin n(θ) > 6.28
2 2 2
θ = 1 radian
r 2R
xyz =
2 n > 6.28

A n=7
Sol 7: (B) ∠ABO = ∠B/2 
r 2cos A 2cosB 2cosC
OF = r; OB = E Sol 10: (D) L.H.S. = + +
B F a b c
sin O
2 by cosine rule
4r3R B D
C
(OB)(OC)(OA) = 2b2 + 2c2 − 2a2 + 2a2 + 2c2 − 2b2 + 2a2 + 2b2 − 2c2
A B C =
4R sin sin sin 2abc
2 2 2
4r3R a2 + b2 + c2 a b c a b
= = 4r2R = = + + = +
r abc bc ca ab bc ca
c
then we can conclude that there should nor term in
r ab
Sol 8: (B) OB = x = L.H.S.. It is possible only if cosA = 0
B
sin
2 π
A=
A B C 2
r = 4R sin sin sin
2 2 2
1 9 . 4 4 | Solutions of Triangle

Previous Years’ Questions P + Q 


⇒ tan   = tan 45º
 2 
Sol 1: (A) In ∆ ABD, applying sine rule, we get A

AD x
=  
sin π / 3 sin α

3
⇒ AD = x sin α ... (i) P
2 3 4

B x D 3x C
And in ∆ ACD, applying sine rule, we get

AD x tan(P / 2) + tan(Q / 2)
= ⇒ =1
sin π / 4 sin β 1 − tan(P / 2)tan(Q / 2)
3 −b / a
⇒ AD = x sin β ... (ii) −b / a −b
⇒ =1 ⇒ =1 ⇒ =1
2  1−c /a a−c a−c
From Eqs. (i) and (ii) a

3x 3x sin α 1 ⇒ –b = a – c ⇒ a + b = c
= ⇒ =
2sin α 2 sin β sin β 6
Sol 4: (B) We know than, A + B + C = 180º
Sol 2: (B) By the law of sine rule ⇒ A + C – B = 180 – 2B.

P 1 
Now, 2ac sin  (A − B + C) = 2ac sin (90º – B)
2 
2ac·(a2 + c2 − b2 )
c b =2ac cos B = = a2 + c2 − b2
2ac
p2 p3
p1
Sol 5: (A) y
Q a R

a b c A
= = = k (say)
sinP sinQ sinR M(a/2, b/2)
b
1 2∆ 2∆
Also, ap1 = ∆ ⇒ = p1 ⇒ p1 =
2 a k sinP x
C a B
2∆ 2∆
Similarly, p2
= = and p3 1 2
k sinQ k sinR Here, R2 = MC2 = (a + b2 ) (by distance from origin)
4
Since, sin P, sin Q, sin R are in AP, we get that p1, p2, p3
1 2
are in HP. c (Pythagorus theorem)
4
c
Sol 3: (A) It is given that tan (P/2) and tan (Q/2) are the ⇒R=
roots of the quadratic equation 2
Next, r = (s – c) tan (C/2) = (s – c) tan π /4 = s – c
π
ax2 + bx + c = 0 and ∠R =
2 ∴ 2(r+R) = 2r + 2R = 2s – 2c + c = a + b + c – c = a + b
∴ tan (P/2) + tan (Q/2) = –b/a
Sol 6: (B)
and tan (P/2) tan (Q/2) = c/a B(0,21)

Since, P + Q + R = 180º
b
P+Q
⇒ P + Q = 90º ⇒ = 45º
2 x’ x
O (0,0) A(21,0)
y’
M a them a ti cs | 19.45

The given vertices of triangle are (0, 0) (0, 21) and (21, ∴ Radius of circle = cosec 20º
0). To find number of integral points inside the ∆ AOB,
A
then Sol 10:
x > 0, y > 0 and x + y < 21
∴Number of points exactly in the interior of the triangle
 
20 × 20 − 20 
= = 190
2
B C

Sol 7: (D) B
Since, sides of a triangle subtends α , β, γ at the center.

c ∴ α +β+ γ = 2π ... (i)


a
∴ Arithmetic mean
π  π  π 
cos  + α  + cos  + β  + cos  + γ 
C
b A
= 2  2  2 
3
Let a : b : c = 1 : 3 : 2 ⇒ c2 = a2 + b2 The value is minimum when the three angles are equal.
∴ Triangle is right angled at C 3
The minimum value of the AM is − .
2
a 1
or ∠ C =
90º and =
b 3 Sol 11: A
a 1
In ∆ BAC, tan A = = 105
o

b 3
c b
⇒ A = 30º and B = 60º (A + B = 90º)
∴ Ratio of angles, A : B : C = 30º : 60º : 90º 30
o
45
o

B C
⇒A:B:C=1:2:3 a=( 3+1)

a b c
Sol 8: (A, D) The sine formula is By sine rule, = =
sinA sinB sinC
a b
= ⇒ a sin B = b sin A 3 +1 b ( 3 + 1)sin30º
sinA sinB ⇒ = ⇒ b=
sin(105º ) sin30º sin105º
π
(a) b sin A = a ⇒ a sin B = a ⇒ B = ∴ Area of triangle
2
π 1 1 ( 3 + 1)sin30º sin 45º
Since, ∠ A < therefore, the triangle is possible = ab sin=
45º ( 3 + 1)
2 2 2 sin105º
(b) and (c) : b sin A > a ⇒ a sin B > a ⇒ sin B > 1
1 ( 3 + 1)2 1 1
= · · ·
∴ ∆ ABC is not possible 2 (sin 45º cos60º + cos 45º sin60º ) 2 2
(d): b sin A < a ⇒ a sin B < a
1 (3 + 1 + 2 3) (4 + 2 3)
⇒ sinB < 1 ⇒ ∠B exists = = ·2 2
4 2  1 1 1 3 4 2(1 + 3)
π  · + · 
Now, b > a ⇒ B > A since A <  2 2 2 2 
2 
∴ The triangle is possible. (1 + 3)2 1+ 3
= = sq unit
2(1 + 3) 2
360º
Sol 9: Here, central angle = = 40º
9
1 2cos A cosB 2cosC a b
In ∆ ACM, = sin20º ⇒ r = cosec 20º Sol 12: Given, + + = +  ... (i)
r a b c bc ca
1 9 . 4 6 | Solutions of Triangle

b2 + c2 − a2 ⇒ c2 = 6 ⇒ c = 6
We know that, cos A =
2bc Using sine rule,
c2 + a2 − b2 a b c
cos B = = =
2ac sinA sinB sinC
a2 + b2 − c2 1+ 3 2 6
and cos C = ⇒ = =
2ab sinA sinB sin60º
On putting these values in Eq. (i), we get 3

2 2 2 2 2 2 2 2 2 2sin60º 2 1
2(b + c − a ) c + a − b 2(a + b − c ) ∴sin B = = =
+ + 6 6 2
2abc 2abc 2abc
a b ∴ ∠B = 45º
= +
bc ca
⇒ ∠A = 180º – (60º + 45º) = 75º
2(b2 + c2 − a2 ) + c2 + a2 − b2 + 2(a2 + b2 − c2 )

2abc
Sol 16: Since, r1, r2, r3 are ex-radii of ∆ ABC are in HP.
a2 + b2
= ⇒ 3b2 + c2 + a2 = 2a2 + 2b2 1 1 1
abc ∴ , , are in AP
r1 r2 r3
⇒ b2 + c2 = a2
s −a s −b s −c
Hence, the angle A is 90º ⇒ , , are in AP
∆ ∆ ∆
8 ∆3
Sol 13: p1p2p3 = ⇒ s – a, s – b, s – c are in AP
abc
⇒ – a, – b, – c are in AP
abc
Since, ∆ = ⇒ a, b, c are in AP.
4R
8 (abc)3 (abc)2
∴ p1p2p3 = · = Sol 17: Let ABC be the triangle such that the lengths of
abc 64R 3 8R 3 its sides CA, AB and BC are x – 1, x and x + 1 respectively
where x ∈ N and x > 1. Let ∠ B = α be the smallest
1 1 a angle and ∠A =2 α be the largest angle.
Sol 14: Since, ∆ = ap1 ⇒ =
2 p1 2∆
A
1 b 1 c
Similarly,
= = , 2
p2 2∆ p3 2∆
x x-1
1 1 1 1
∴ + − = (a + b − c)
p1 p2 p3 2∆

2(s − c) s − c s(s − c) ab B C
= = = · x+1
2∆ ∆ ab s ∆
Then, by sine rule, we have
ab C 2ab C
= · cos2 = cos2 sin α sin2α
a+b+c 2 (a + b + c)∆ 2 =
 ∆ x −1 x +1
 2 
sin2α x + 1 x +1
⇒ = ⇒ 2cos α =
Sol 15: Given that, sin α x −1 x −1
a = 1 + 3 , b = 2and ∠C =60º
x +1
∴cos α = ... (i)
We have, c2 = a2 + b2 – 2 ab cos C 2(x − 1) 
⇒ c2 = (1+ 3 )2 + 4 – 2(1+ 3 ) · 2 cos60º x2 + (x + 1)2 − (x − 1)2
Also, cos α = , using cosine law
⇒ c2 = 1+2 3 + 3 + 4 – 2 – 2 3 2x(x + 1)
M a them a ti cs | 19.47

x+4 On π
⇒ cos α =  ... (ii) = tan ,  … (ii)
2(x − 1) n n
From Eqs. (i) and (ii), we get 2π 2π
2In I sin 1 + cos
n ∴ n 2 π n
∴ = = cos
=
x +1 x+4 On π On n 2
= tan
2(x − 1) 2(x + 1) n
⇒ (x+1)2 = (x+4) (x–1) 1 + 1 − (2In / n)2
In
= [from Eq. (i)]
⇒ x2 + 2x + 1 = x2+ 3x – 4 ⇒ x = 5 On 2
Hence, the lengths of the sides of the triangle are 4, 5 On  2 
and 6 unit. I=
n  1 + 1 − (2In / n) 
2  

Sol 18: If the triangle is equilateral, then


Sol 20: Since, the circles with radii 3, 4 and 5 touch
A = B = C = 60º each other externally and P is the point of intersection
⇒ tanA + tanB + tanC = 3tan60º = 3 3 of tangents

Conversely assume that


tanA + tanB + tanC = 3 3
But in triangle ABC, A + B = 180º – C C2 4 5 C1
Taking tan on both sides, we get 4 P 5
tan(A+B) = tan (180º–C) 3 3
tanA + tanB
⇒ = − tanC C3
1 − tanA tanB
⇒ tanA + tanB = – tanC + tanA tanB tanC
⇒ P is incentre of ∆ C1C2C3
⇒ tanA + tanB + tanC + tanA tanB tanC = 3 3
Thus, distance of point P from the points of contact
⇒ none of the tanA, tanB, tanC can be negative
= In radius (r) of ∆ C1C2C3
⇒ ∆ ABC cannot be obtuse angle triangle
Also, AM ≥ GM ∆ s(s − a)(s − b)(s − c)
ie, r = =
1 s s
[tanA + tanB + tanC ] ≥ [tanA tanB tanC ]1/3
3 where 2s = 7 + 8 + 9
1 ∴s = 12
⇒ ( 3 3 ) ≥ ( 3 3 )1/3 ⇒ 3 ≥ 3
3
(12 − 7)(12 − 8)(12 − 9) 5· 4 ·3
So, the equality can hold if and only if Hence, r = = = 5
12 12
tanA = tanB = tanC or A = B = C or when the triangle
is equilateral.
1 2∆ 2 × 15 3 3
Sol 21: =
∆ absinCsinC
= = = =C 120o
n 2π 2 ab 6 × 10 2
Sol 19: We know, In = r 2 sin (I is area of regular
polygon) 2 n n
⇒ c= a2 + b2 − 2abcosC
2In 2π = 62 + 102 − 2 × 6 × 10 × cos120o= 14
⇒ = sin (r=1)...(i)
n n
∆ 225 × 3
π r= ⇒ r2 = = 3
and On = nr tan {On is area of
2
s  6 + 10 + 14 
2  
 2 
circumscribing polygon}
1 9 . 4 8 | Solutions of Triangle

Sol 22 : (B) C

3 sinP + 4 cosQ = 6  … (i)


4 sinQ + 3 cosP = 1 … (ii) D
From (1) and (2) ∠ P is obtuse,
(3sinP + 4 cosQ)2 + (4 sinQ + 3cosP)2 =
37 A B
⇒ 9 + 16 + 24(sinP cosQ + cosP sinQ) =
37
A
⇒ 24 sin(P + Q) =
12 2(2)cos
⇒ 2 = 2a ⇒ cos A = a
1 5π π 3 3 2 2
⇒ sin(P + Q) = ⇒ P + Q= ⇒ R=
2 6 6 a2 b2 + c2 − a2 5 − a2
⇒ cosA = - 1= =
2 2bc 4
Sol 23: (A) Let AB = x
⇒ a2 = 3
p p
tan( π − θ −=
α) ⇒ tan(θ +=
α) 1
x−q q−x ∴ 12∆2 = 12 × bcsinA = 9
2
⇒ q=
− x p cot(θ + α )
 cot θ cot θ − 1  Sol 2: A = x, B = 2x, C = 4x
⇒ x= q − p cot(θ + α ) = q − p  
 cot α + cot θ  ⇒ 7x = 180
q  180
 cot θ − 1   qcot θ − p  x= ; C is obtuse angle
p 7
q − p
= =q − p 
 q   q + pcot θ  a b c
 p + cot θ  2R = 4 = = = ;
  sinA sinB sinC
 q cos θ − p sin θ  sinC = sin3A
= q − p 
 q sin θ + p cos θ  1 1 1 1 1 1 1 
– + + =  + − 
a b c 4  sinB sinC sinA 
h
Sol 24: (A) tan 30o = ⇒ AD = h 3 1 1 1 1  1
AD =  + − =
4  3A − 4 sin A sin A sinA 
3 2 4 sinA
h
=BD h=
; CD
3
3
Sol 3: Length of median BD = (AB)
AB AD − BD 3 −1 3− 3 4
= = = = 3
BC BD − CD 1 3 −1
1− B
3

JEE Advanced/Boards D

Exercise 1 A
D b/2
C

Sol 1: AB = 2, AC = 1, BC = a
3 1
BD = c= 2a2 + 2c2 − b2
1 a 2a 2a 4 2
CD = (a) = , DB = , AD =
1+2 3 3 3 3
A c= 2a2 + 2c2 − b2
2bccos 2
Length of angle bisector = 2 = 2a
b+c 3 3 2 5
c = 2a2 + 2c2 – b2 ⇒ b2 = 2a2 + c2
4 4
M a them a ti cs | 19.49

⇒ 4b2 = 8a2 + 5c2 ... (i)


A
3 b2 AC = 2a cos
BD = c= − a2 2
4 4 A
2asin
3 2 b 2
2 = 2a a
⇒ c = – a2 ⇒ 3c2 = 4b2 – 16a2 R1(of ∆ABD) =
16 4 2sinA A
A a 2cos
cos = 2
4b2 = 3c2 + 16a2 ... (ii) 2 25
From (1) & (2)
R2 (of ∆ACB) = 25
2c2 = 8a2; c = 2a A
2acos
⇒ 4b2 = 12a2 + 16a2 ⇒ b = 7a 2= a
=
2sinA A
a2 + c2 − b2 a2 + 4a2 − 7a2 −3 2sin
cos∠ABC = = = 2
2ac 2a(2a) 4 A A
sin =
2 50
⇒ ∠ABC = 120º
 1 1 
a2  +  =1
Sol 4: OD = 2r  A  (25)
2
(50)2 
secA = ? = we need to find 4
F E a2 = (25)2 = 500
OA = 2R cosA 5
O Area of rhombus = 2(Area ∆ABD)
a A
OA + OD = AD = cot
2 2  A 
1 2  2 tan 
a A
B D C
= 2 a sinA= a sinA= 500 
2 2  = 400
2r + 2R cosA = cot a/2
2 2
2  1 + tan2 A 
 
 2
 A B C 2R sinA A
2  R sin sin sin  +2RcosA = cot cosC
 2 2 2 2 2
cot C sinC
A Sol 6: =
cos cot A + cotB cos A sinB + sinA cosB
A B C A A 2
8 sin sin sin +2cosA = 2sin cos sinA sinB
2 2 2 2 2 A
sin cosC sinA sinB cosC sinA sinB
2 = . = .
sin(A + B) sinC sinC sinC
A B C A
8 sin sin sin +2cosA = 2cos2 -1+1
2 2 2 2
Applying sine & cosine rule
A B C
8 sin sin sin + 2 cos A = cos A + 1  a2 + b2 − c2  1 a.b 100c2  ab 
2 2 2 =   =   = 50
 2ab c c 2ab  c2 
 
A B C
cosA = 1 – 8 sin sin sin
2 2 2 Sol 7: We know that in cyclic quadrilateral, sum of
opposite angle is 180º
A B C
∴ sec A = 1/(1 – 8 sin sin sin )
2 2 2 A a D
Sol 5: ∠DAB = A A o
2 120
A b
∠DAC = a a
2 60
o

A B C
DB = 2a sin D B 5
2
∠BAC = 180 – A a a ∠ABC = 60º, AD = a, CD = b
C
1 9 . 5 0 | Solutions of Triangle

∠ADC = 120º A
1 5 3
Area (∠ABC) = (2)(5) sin60º = 
2 2
n+2 n+1
Total area = 4 3

5 3
Area (∆ADC) = 4 3 – 2
2 B C
n
3 3 1 Let the smallest, angle ∠A = θ then the greatest ∠C =
= = ab sin120º
2 2 2θ. In ∆ABC by applying Sine Law we get,
⇒ ab = 6 ... (i) Sin θ/n = sin 2 θ/n + 2
2 2 2
(5) + (2) − (AC) ⇒ sin θ/n = 2 sin θ cos θ/ n + 2
cos ∠ABC =
2(5)(2)
⇒ 1/n = 2 cos θ/n + 2 (as sin θ ≠ 0)
(AC)2 = 19
⇒ cos θ = n + 2/2n  ... (i)
2 2
a + b − 19
cos ∠ADC = In ∆ ABC by Cosine Law, we get
2(a)(b)
Cos θ = (n + 1)2 + (n + 2)2 – n2/2(n + 1) (n + 2) ... (ii)
1 a2 + b2 − 19
– = Comparing the values of cos θ from (i) and (ii), we get
2 2(6)
(n + 1)2 + (n + 2)2 – n2/2(n + 1) (n + 2) = n + 2/2n
a2 + b2 = 13  ... (ii)
⇒ (n + 2)2 (n + 1) = n(n + 2)2 + n(n + 1)2 – n2
From (1) & (2)
⇒ n (n + 2)2 (n + 2)2 = n(n + 2)2 + n(n + 1)2 – n3
a = 3cm, b = 2cm
⇒ n2 + 4n + 4 = n3 + 2n2 + n – n3
2 2
(4) + c − 6 3
Sol 8: cos30º = = ⇒ n2 – 3n – 4 = 0 ⇒ (n + 1) (n – 4) = 0
2(4)(c) 2
⇒ n = 4 (as n ≠ - 1)
10 + c2
= 3 ∴ Sides of ∆ are 4, 4 + 1, 4 + 2, i.e. 4, 5, 6.
4c
48 − 40
c2 – 4 3 c + 10 = 0 ⇒ c = 2 3 ±
2 A B C
Sol 11: r1 = 4R sin cos cos
⇒c= 2 3± 2 2 2 2
Two triangles are possible. R 2 B
=
r1 3+ 2
b2 c2
Sol 9: a2 =
2b cos A

bc b2 + c2 − a2
cosA = =
2a2 2bc A C

⇒bc =ab +ac –a
2 2 2 2 2 2 4

⇒ b2(c2 – a2) – a2(c2 – a2) = 0 A B C 3+ 2


4 sin sin sin =
2 2 2 2
⇒ (c2 – a2)(b2 – a2) = 0
⇒ either c = a or a = b 1 B  B 3+ 2
⇒ 4 sin sin  45 −  =
Hence triangle is isosceles. 2 2  2 2

1 B B B 3+ 2
Sol 10: Let the sides of ∆be n, n + 1, n + 2 where n ∈ N. ⇒ sin cos − sin  =
2 2 2 2 4
Let a = n, b = n + 1, c = n + 2
M a them a ti cs | 19.51

Sol 14: r1 – r = r2 + r3
B B B 6 +2
⇒ sin cos – sin2 =
2 2 2 4 ∆ ∆ ∆ ∆
– = +
s−a s s −b s −c
6 +2
⇒ sinB – 1 + cosB = ∆a ∆a
2 ⇒ =
s(s − a) (s − b)(s − c)
6+4
⇒ sinB + cosB = A (s − b)(s − c)
2 ⇒ s(s – a) = (s – b)(s – c) ⇒ tan = =1
2 s(s − a)
6+4
⇒ sin(B + 45º) = ⇒ A = 90º
2 2
5π π Sol 15: 2(2R)2 = a2 + b2 + c2
⇒B= ,C=
12 12 2 2 2
 a   b   c 
b sinB 2=   +   +  
⇒ = =2+ 3  2R   2R   2R 
c sinC
⇒ sin2A + sin2B + sin2C = 2
a+c a
Sol 12:     =2 ⇒ 3 – cos2A – cos2B – cos2C = 4
 a  b
a + c = 2b ... (i) ⇒ cos2A + cos2B + cos2C = –1

⇒ a(1 – x2)+ 2bx + c(1 + x2) = 0 ⇒ 2cos(A + B)[cos(A – B) + 1] = –1

x2(c – a) + 2bx + c + a = 0 ⇒ cosC cosA cosB = 0

for equal roots b2 – c2 + a2 = 0 Hence it is right angled triangle.

a2 + b2 = c2 ... (ii)


3 1 1 1 1
Sol 16: – = + +
Hence it is right angle at ∠C r r ra rb rc
Putting value in equation (2) from equation (1)
A
⇒ a + b = (2b – a) ⇒ a + b = 4b + a – 4ab
2 2 2 2 2 2 2

4 5 5 P
⇒ 3b2 = 4ab ⇒ b = a; c = b ⇒c = a
3 4 3 E
Hence a = 3k, b = 4k, c = 5k
3 4 12
sinA + sinB + sinC = + +1= B C
5 5 5 x D
Sol 13: a = 7, b = 8, c = 5
A
2∆ 2∆ 2∆ 2bccos
+ + r 2
sinA + sinB + sinC OD = ; AD =
= = bc ac ab A b+c
A B C s c c sin
cot + cot + cot cot + cot 2
2 2 2 s−c 2 2 A A
 AD  2bc cos sin r
ra = DE =  2 2 = ∆
2∆ (2)s(s − c)  OP=
=  OD  (b + c)r b+c
abc s(s − c)
(a + b) 2 1 1 1
(s − a)(s − b) = + +
r ra rb rc

4 ∆ s(s − a)(s − b)(s − c) 1 1 1  1 1  1 1  s


= =  −  + − + −  =
abc (a + b)2 r r r  r r  r r  ∆
 a   b   c 

4 ∆2 1 b+c
= =
(a + b)abc ra ∆

Calculating value p + q = 10 b + c + c + a + a + b 2s 2
R.H.S. = = =
∆ ∆ r
1 9 . 5 2 | Solutions of Triangle

Sol 17 : y2 + 2= 3y  A bc bc
r1r2 =
y = 1, 2 a+c a+b
2
AC = 1, AB = 2 1  
r1  1 1   sinC sinB 
BD x2 + 1 = bc  .  = bc  
= B
r2 a a
 + 1 + 1  (1 + sinC)(1 + sinB) 
CD 2x C
D  c b 
3
BD + CD x2 + 1 + 2x
=
CD 2x Sol 19: AD = (AB) sinB = C sinB

3 (x + 1)2 P1 = AD = 2R sinC sinB


=
CD 2x A
2 3x b
CD = = ( 3)
(x + 1) 2 c+b

3(x2 + 1)
Similarly BD =
(x + 1)2 B C
D
2 3x 1
=
2 1 (sinA + sinB + sinC)
(x + 1) 3 ∑A =
2R sinA sinBsinC
6x = (x + 1)2 ⇒ x2 + 1 – 4x = 0
A B C
⇒ x1 + x2 = 4 4 cos
cos cos
= 2 2 2
A1 A2 2x1 2x2  A A   B B 
 C C
tan +tan = + =8 2R  2sin cos   2sin cos   2sin cos 
2 2 1 1  2 2  2 2  2 2

Sol 18: Area (∆AMC) = Area(∆AMB) 1 1


= = = R.H.S.
A B C r
(BC) a 4R sin sin sin
= AM = MB = 2 2 2
2 2
a 1 ∆ ∆ ∆ ∆ 1 1 1
AC = 2(AE) = b, AD = DB = = = = + + = + +
2 r s s − a s − b s − c r1 r2 r3

In radii of ∆AMB =
s
Sol 20: Area of ∆ABC = Area of ABGF + Area of ∆GFC
1 Area ( ∆ABC) Area ( ∆ABC) bc
= = =
2 a+c a+c a+c A
 
 2 
C E
D
F
O P
E M
B C
D G
1 z+c 1
cP3= (2r)+ z(P3 – 2r)
A B 2 2 2
D
z 2r
1 Area ( ∆ABC) ⇒ zP3 + 2cr = cP3 ⇒ = 1−
In radius of ∆AMC = c P3
2 a+b
  x y z 1 1 1
 2  ∴ + + = 3 – 2r  + + 
a b c  P1 P2 P3 
Area ( ∆ABC) bc
= =
a+b a+b = 3 – 2=1
M a them a ti cs | 19.53

Exercise 2 R.H.S. =
C  A +B  B A + C  A B + C
64R 3 cos  sin   cos sin   cos sin
Single Correct Choice Type 2 2  2 2  2 2 
A B C A B C
Sol 1: (C) BD =
1
BC ∑ 16R2 cos 2 cos 2 cos 2 sin 2 sin 2 cos 2 
2  
A B C
A 4R cos cos cos
C
= 2 2 2
3
A B C
D
∑ sin 2 sin 2 cos 2
i=1

B s s(s − a)(s − b)(s − c)


4R
A s(s − a) abc
Pythagoras theorem in ∆BAD cos = =
2 bc s(s − a)(s − b)(s − c)
c2 + (AD)2 = (BD)2 ∑ abc
(s − c)

1 a2
c2 + (2b2 + 2c2 – a2) =
4 4  4RS 
 ∆
⇒ a2 = b2 + 3c2 =  abc  = 4R ⇒ k =
1
∆ 4
tanA sinA cosB (s − c) + (s − b) + (s − a)
= abc
tanB sinB cos A

a  a2 + c2 − b2   2bc  a2 + c2 − b2 x x
=   = = –2 Sol 4: (B) AC = =
 2 2  sin ∠ACD sinC
b  2ac 

2
b +c −a  b2 + c2 − a2
x
AB =
Sol 2: (A) Put a = 2R sin A sinB

2(sinA cos A + cosBsinB + sinCcosC  1 1 


= x2  +  = 8x
2

2(sinA + sinB + sinC)  sin C sin2 B 


2

sin2A + sin2B + sin2C sin2B + sin2C = 8 sin2C sin2B


=
2(sinA + sinB + sinC)
C
2sin(A + B)cos(A − B) + 2sinCcosC
=
2(sinA + sinB + sinC) D
2 2x
sinC[cos(A − B) − cos(A + B)]
= x
(sinA + sinB + sinC)
A B
2sinA sinBsinC
=
sinA + sinB + sinC 1 = 8 sin2C cos2C

1 π 3π
2sinA sinBsinC (sin2C) = ⇒C = ⇒B =
= 8 8
 A +B  A −B  C C 2
2sin   cos   + 2sin cos
 2   2  2 2
Sol 5: (A) OB = R
2sinA sinB sinC
= ∠AOC = 2A, ∠BOD = A
C A −B A +B
2cos cos + cos f = OD = R cos A
2 2 2 
A
A B C r
= 4 sin sin sin =
2 2 2 R F E
O
A B C C
Sol 3: (C) r1 = 4R sin cos cos B D
2 2 2
1 9 . 5 4 | Solutions of Triangle

a b c a b c a b c
L.H.S. = + + = + + Sol 8: (C) From sine rule = = =2R
f g h R cos A R cosB R cosC sinA sinB sinC
a Given situation
Put R = = 2(tanA+tanB+tanC) = 2tanA tanB tanC
2sinA a b c
= = =k
abc cos A cosB cosB
R.H.S. = λ
3
R cos A cosBcosB 2R sinA = k cosA
a tanA = tanB = tanC
Put R = = 8λ tanA tanB tanC
2sinA
Hence it is equilateral triangle.
1
⇒λ=
4
Sol 9: (A) cosA + cosB = 2(1 – cosC)
Sol 6: (C) ∠AOB = 2∠C = 2C A  A +B  A −B  C
cos   cos   = 2sin
2

OA = OB = R  2   2  2
AB  A −B  C  A +B
= 2R1 ⇒ cos 
sin ∠AOB
O  = 2sin = 2cos  
 2  2  2 
C
R1 = B C  A −B 
2sin2C cos  
 2 
L.H.S. = 2[sin2A + sin2B + sin2C] ⇒ =2
 A +B
= 2[2 sin(A + B) cos(A – B) + 2 sin C cos C] cos  
 2 
= 4 sin C[cos(A – B) ± cos(A + B)] = 8 sinA sinB sinC
 A −B   A +B
cos   + cos  
1  1  1 
= 8  absinC   bc sinA   casinB   2   2  2+1
⇒ =
2  2  2   A −B 
cos 
 A +B 2 −1
 − cos  
 2   2 
8a 64 ∆3 4∆
= = =
2 2 2
ab c abc2 2 2
R2 A B 5
⇒ cot cot = 3 =
2 2 5−c
r
Sol 7: (A) DF = ?; OE = r; OD =
A a + b + c = 3a + 3b – 3c⇒2c = a + b
sin
2
a+b+c
Sol 10: (B) Arithmetic mean =
A 3
2R
= (sinA + sinB + sinC)
E 3
O
F O Length of altitude = b sinC = 2R sinBsinC
r
3
B
PD
C Harmonic mean =
1
∑ 2R sinB sinC
A
2bc cos
AD = 2 6R sinA sinB sinC
b+c =
sinA + sinB + sinC
 AD   2R 
FD =   Product = (6R)   (sinA sinBsinC)
 OD   3 
A = 4R2 sinA sin B sinC
2bc cos
FD = 2 r sin A = bc sinA = 2∆
r(b + c) 2 b+c b+c  a  b 
=    sinA sinBsinC = 2∆
2∆  sinA   sinB 
If C is the base =
a+b
M a them a ti cs | 19.55

sinA sin(B − C) Match the columns type


Sol 11: (A) =1
sinC sin(A − B) Sol 14: A → s; B → r; C → p; D → q
sin(B + C)sin(B − C) cos2B − cos2C (A) Centroid divides the
L.H.S.= = =1 A
sin(A + B)sin(A − B) cos2A − cos2B triangle into three equal
⇒ 2sin2B = sin2A + sin2C⇒2b2 = a2 + c2 part so 1 : 1 : 1
P
(B) P is incentre
5
Sol 12: (B) sinA = A = acute angle PD = r
13 B
D
C
99 BC = a
sinB = ~1
101 Area (DPBC) = ar
B ~ 90 hence ratio = a : b : c = sinA : sinB : sinC
C will be acute angle. (C) P is orthocentre
cos C > 0 1 a
Area of (DPBC) = × PD × BC = × (AD + AP)
2 2
cos C = 1 − sin2 C = 1 − sin2 (A + B)
a
= (2R sinB sinC – 2R cosA)
2
= 1 − [sinA cosB + cos A sinB]2
= aR(sinB sinC – cosA) = aR[sinB sinC + cos(B + C)
2
 99  20 = 2R cosB cos C = 2R sinA cosB cosC
⇒ cosB = 1−  =
 101  101
Hence ratio tanA : tanB: tanC
12 (D) If P is circumcentre 
⇒ cosA = A
13 PB = R
2
 5  20  12  99   255 a
⇒ cosC = 1−  +   = BD = P
 12  101  13  101   1313 2
a2
PD = R2 − B
D
C
Multiple correct choice type 4
1
Sol 13: (A, C) Hypotenuse Area DPBC = ×PD × BC
2

[(cos2α + cos2β + 2cos(α + β)]2 a 2 a2 a a2 a2


= = R − = −
2 4 2 4 sin2A 4
+ [sin2α + sin2β + 2sin(α + β)]2
cos2α + cos2β + 2cos(α + β) a4 cos A
= = (2R)2 sin2A
4 sinB
= cos2α – sin2α + cos2β – sin2b + 2cosacosβ – 2sinasinb
Ratio = sin2A : sin2B : sin2C
= (cosα + cosβ)2 – (sinα + sinβ)2
sin2α + sin2β + 2sin(α + β)
Sol 15: A → q; B → r; C → p; D → q
= 2sinacosα + 2 sinbcosβ + 2sinacosb + 2cosasinb
CA = b = 1 + 3
= 2sina[cosα + cosb] + 2sinb[cosβ + cosa]
1
= 2(cosα + cosβ)(sinα + sinβ) BC = a = 2, cos C =
2
Hypotenuse B
= (cosα + cosβ) + (sinα + sinβ)
2 2
D F
= 2 + 2cos2acosβ + 2sinbcosb
α −β P
= 2 + 2 cos(α – β)= 4cos2
2 C A
E
1 9 . 5 6 | Solutions of Triangle

(c)If a, sin B, R are given, then we can determine b = 2R


(AB) = 6 =C
a sinB
sinB, sinA = . So, sin C can be determined.
b
a2 + c2 − b2 (2)2 + (6) − ( 3 + 1)2
⇒ cosB = =
2ac 2(2)( 6) Hence, side c can also be uniquely determined.

6−2 3 3− 3 3 −1 (d) If a, sin A, R are given, then


⇒ cos B = = =
4 6 2 3 2 b c
= = 2R
B = 75º sinB sinC

A = 45º But this could not determine the exact values of b and
c.
 3
⇒ (BE) = a sinC = 2  = 3
 2 
  Sol 2: (A)
⇒ DA = C sinB = 6 (sin 75º) C
o
1 30
⇒ CF = b sinA = ( 3 + 1) a
2 b
∆DEF is pedal triangle of ∆ABC
120o o
30
π
∠DEB = – A ⇒ ∠DEF = π – 2A A c B
2
EF Given, ratio of angles are 4 : 1 : 1
(A) Circum scribing circle =
sin(EDF) ⇒ 4x + x + x = 180º
acos A R a 1 ⇒ x = 30º
= = = =
2sin( π − 2a) 2 ∆ sinA 2
∴ ∠A =120º, ∠B = ∠C =30º
1 a
(B) Area = × (DE)(DF) × sin(EDF) Thus, the ratio of longest side to perimeter =
2 a+b+c
1 Let b = c = x
= (c cosC × b cosB) × sin(π – 2A)
2 ⇒ a2 = b2 + c2 – 2bc cosA
= bc sinA cosA cosB cosC ⇒ a2 = 2x2 – 2x2 cosA = 2x2(1–cos)

1  3 − 1  1  2 6 ⇒ a2 = 4x2 sin2 A/2 ⇒ a = 2xsin A/2


= ( 3 +1 ) 6 

2 2
  =
 2
  8
=
4
3

y
∆ 3/4 6− 2
(C) r = = =
s Σacos A 4 B(3,4)

Previous Years’ Questions

Sol 1: (D) Here, for


(a) If a, sinA, sinB are given, then we can determine b
a a
= sinB, c = sinC. So, all the three sides are ⇒ a = 2xsin 60º = 3x
sinA sinA
unique. Thus, required ratio is
So, option (A) is incorrect. a 3 3
= =
(b)The three sides can uniquely make an acute angled a + b + c x + x + 3x 2 + 3
triangle. So, option (B) is incorrect.
Sol 3: (C) To find orthocentre of the triangle formed by
M a them a ti cs | 19.57

(0, 0), (3, 4) and (4, 0). = 2k (b) = 2sinB [using, b = a cosC + c cosA] = 3
Let H be the orthocentre of ∆OAB
Sol 7: (C) y
∴ (Slope of OP ie, OH) · (slope of BA) = –1

y −0 4−0 4 3
⇒  · −1 ⇒ y =
 = −1 ⇒ − y =
P
(1,2 2)
3−0 3−4 3 4
 3 S (1,0) R
∴Required orthocentre = (3, y) =  3,  x’
O
x
 4
(-3,0) (-1,0)
(9,0)

Sol 4: (D) Let a, b, c are the sides of triangle ABC. Q


(1,-2 2)
B + C  B − C 
2sin   cos   y’
b + c k(sinB + sinC)  2   2 
Now, = =
a k sinA A A Coordinates of P and Q are (1, 2 2 ) and (1, –2 2 )
2sin cos
2 2
Now, PQ = (4 2)2 + 02 = 4 2
B − C  B − C 
cos   sin  

b+c
=  2  Also,
b−c
=  2  1
Area of ∆ PQR = · 4 2 ·8 = 16 2
a A a A 2
sin cos
2 2 1
Area of ∆ PQS = · 4 2 ·2 = 4 2
2
Sol 5: (C) A Ratio of area of triangle PQS and PQR is 1 : 4
D 60o o
a 60 a
Sol 8: (B) Equation of circumcircle of ∆ PRS is
3
O (x + 1) (x + 9) + y2 + λy = 0
15o
15o It will pass through (1, 2 2 ), then
B C
–16 + 8 + λ · 2 2 = 0
Let AB = AC = a and ∠A =120º
8
⇒ λ = =2 2
1 2 2 2
∴ Area of triangle = a sin120º
2
∴Equation of circumcircle is
where, a = AD = BD
x2 + y2 – 8x + 2 2 y – 9 = 0
= 3 tan30º + 3 cot15º Hence, its radius is 3 3 .
3  1 + tan 45º tan30º 
=1+ =1+ 3 
tan(45º −15º )  tan 45º − tan30º  Alternate Solution
2 2 PR
 3 +1 Let ∠PSR =
θ ⇒ sin θ = ∴ sin θ =
= 1 + 3  ∴a=4+2 3 2 3 2R
 3 −1
 
 3 ⇒ PR = 6 2 = 2R · sin θ ⇒ R = 3 3
1
⇒ Area of triangle = (4 + 2 3)2  =12 + 7 3
2  2 
  ∆
Sol 9: (D) Radius of incircle is, r =
Sol 6: (D) Since, A, B, C are in AP s
⇒ 2B = A + C ie, ∠B = 60º Since, ∆ = 16 2
a a 6 2 +6 2 +4 2
∴ (2sinC cosC)+ (2sinA cosA) = 2k (a cosC + c=
cosA) Now, s = 8 2
c c 2
using
 a b c 1  16 2
Using,  = = =  ∴r = 2
=
 sinA sinB sinC k  8 2
1 9 . 5 8 | Solutions of Triangle

Sol 10: (A, B, C, D) Since, ∆ ABC = ∆ ABD + ∆ ACD a2 + b2 − c2


Using, cosC =
1 1 A 1 A 2ab
⇒ bc sin A = c AD sin + b AD sin
2 2 2 2 2 3 (x2 + x + 1)2 + (x2 − 1)2 − (2x + 1)2
⇒ =
2bc A 2 2(x2 + x + 1)(x2 − 1)
⇒ AD = cos
b+c 2 3 (x2+x+1)(x2–1)
⇒ (x+2)(x+1)(x–1)x + (x2 – 1)2 =
A 2bc
Again, AE = AD sec = ⇒ AE is HM of b and c. ⇒ x2 +2x +(x2 –1) = 3 (x2+x+1)
2 b+c
A ⇒ (2 – 3 )x2 +(2 – 3 )x– ( 3 +1) = 0
EF = ED + DF = 2DE = 2AD tan
2
⇒ x = –(2 + 3 ) and x = 1 + 3
2bc A A 4bc A
= 2 cos tan = sin
b+c 2 2 b+c 2 But,x = –(2 + 3 ) ⇒ c is negative

Since, AD ⊥ EF and DE = DF and AD is bisector. ∴x = 1+ 3 is the only solution.


⇒ ∆ AEF is isosceles.
AD
Sol 13: In ∆ ADC, = sin23º
P b
Sol 11: (B, D)
⇒ AD = b sin 23º
abc
But AD = (given)
b − c2
2

abc a sin23º
Q
S R ⇒ = bsin23º ⇒ = ...(i)
2
b −c 2 2
b −c 2 c 
T
Again, in ∆ ABC,
Let a straight line through the vertex P of a given ∆
sinA sin23º
PQR intersects the side QR at the point S and the =
circumcircle of ∆ PQR at the point T. a c
sinA a
Points P, Q, R, T are concyclic, then PS · ST = QS · SR ⇒ = [from Eq. (i)]
a b − c2
2
PS + ST
Now, > PS·ST (AM > GM) a2 k 2 sin2 A
2 ⇒ sin A = ⇒ sin A =
b2 − c2 k 2 sin2 B − k 2 sin2 C
1 1 2 2
and + >
PS ST PS·ST QS·SR sin2 A
⇒ sin A =
sin2 B − sin2 C
SQ + QR
Also, > SQ ·SR
2 sin2 A
⇒ sin A =
QR 1 2 2 4 sin(B + C)sin(B − C)
⇒ > SQ ·SR ⇒ > ⇒ >
2 SQ ·SR QR SQ ·SR QR sin2 A
⇒ sin A =
1 1 2 4 sinA ·sin(B − C)
Hence, + > >
PS ST QS·SR QR ⇒ sin (B – C) = 1 (sin A ≠ 0)
⇒ sin (B – 23º) = sin90º ⇒ B – 23º = 90º
Sol 12: (B) A
⇒ B = 113º

abc ∆
Sol 14: We have, R = and r =
2
b=x -1 c=2x+1 4∆ s
R abc s abc·s abc
= = · =
r 4∆ ∆ 4∆ 2 4(s − a)(s − b)(s − c)
C B
2 But a : b : c = 4 : 5 : 6(given)
a=x +x+1
M a them a ti cs | 19.59

a b c (c – a)2 = 0
⇒ = = = k(let)
4 5 6
(as we know, a+b–c > 0, b+c–a > 0, c+a–b > 0)
⇒ a = 4k, b = 5k, c = 6k
∴ Each term on the left of equation has positive
1 1 15k coefficient multiplied by perfect square, each term
Now, s = (a+ b + c) = (4k + 5k + 6x) =
2 2 2 must be separately zero.
R (4k) (5k) (6k) ⇒a=b=c
∴ =
r  15k   15k   15k  ∴Triangle is an equilateral.
4 − 4k   − 5k   − 6k 
 2  2  2 
b+c c+a a+b
30k 3 30·8 16 Sol 17: Let = = = λ
= = = 11 12 13
 15 − 8  15 − 10  15 − 12  7·5·3 7
k3     ⇒ (b+ c ) = 11 λ , c + a = 12 λ , a + b = 13 λ  ... (i)
 2  2  2 
⇒ 2(a+ b + c) = 36 λ
AC
Sol 15: In ∆ ADC, we have cos C = ⇒ a + b +c = 18 λ  ... (ii)
CD
2b On solving the Eqs. (i) and (ii), we get
cos C = ... (i)
a 
a=7 λ , b=6 λ and c=5 λ
Applying cosine formula in ∆ ABC, we have
b2 + c2 − a2 b2 + c2 − a2 36λ2 + 25λ2 − 49λ2 1
∴ cos A
= = =
cos A = 2bc 2(30)λ 2 5
2bc
a2 + b2 − c2 a2 + c2 − b2
and cos C = ... (ii) cosB =
2ab 2ac

From Eqs. (i) and (ii), we get 49λ2 + 25λ2 − 36λ2 19
= =
70λ 2 35
a2 + b2 − c2 2b
=
2ab a a2 + b2 − c2 49λ2 + 36λ2 − 25λ2 5
=cosC = =
⇒ a2 + b2 – c2 = 4b2 2ab 84λ2 7

⇒ a2 – c2 = 3b2 ... (iii) 1 19 5


∴ cosA : cosB : cosC = : : = 7 : 19 : 25
5 35 7
b2 + c2 − a2 2b b2 + c2 − a2
Now, cos A cos C = · =
2bc a ac A
Sol 18:
2 2 2
3b + 3(c − a )
= o
30 45
o

3ac
(a2 − c2 ) + 3(c2 − a2 ) 2(c2 − a2 )
= =
3ac 3ac

Sol 16: Let a, b, c are the sides of a ∆ ABC. 


B C
D
3 a/2 a/2
Given, cos A + cos B + cos C =
2
Let AD be the median to the base BC = a of ∆ ABC, and
b2 + c2 − a2 a2 + c2 − b2 a2 + b2 − c2 3
⇒ + + = let ∠ADC = θ , then
2bc 2ac 2ab 2
a a a a 3 −1
⇒ ab2 + ac2 – a3 + ba2+ bc2 – b3+ ca2+ cb2 – c3 = 3abc  +=  cot θ cot30º − cot 45º ⇒ cot θ =
2 2 2 2 2
⇒ a(b – c)2 + b(c – a)2 + c(a – b)2
Applying sine rule in ∆ ADC, we get
(a + b + c)
= [(a − b)2 + (b − c)2 + (c − a)2 ] AD DC
2 =
sin( π − θ − 45º ) sin 45º
⇒ (a + b – c) (a – b)2 + (b + c – a) (b – c)2+ (c + a – b)
1 9 . 6 0 | Solutions of Triangle

A
a Sol 20:
AD 2
⇒ =
sin(θ + 45º ) 1
c b
2
p
a
⇒ AD = ( sin 45º cos θ + cos 45º sin θ )
2
a  cos θ + sin θ  a B
D C
⇒ AD =  =  (cos θ + sin θ) a
2  2)  2 Let ABC be a triangle with base BC=a and altitude AD =P

1 a  3 − 1 2 
 then, Area of ∆ABC =
1
bc sinA
⇒= + 2
11 − 6 3 2 8−2 3 8−2 3 
 
1
Also, area of ∆ABC = ap
2 8−2 3 8−2 3 2
⇒a= =2 =2
1 1
44 − 24 3 + 22 3 − 36 8−2 3 ∴ ap = bc sinA
2 2
Sol 19: Given, bc sinA abc sinA
⇒p= ⇒p=
a a2
cosA cosB + sinA sinB sinC = 1
abc sinA · (sin2 B − sin2 C)
1 − cos A cosB ⇒p=
⇒ sinC = ... (i) a2 (sin2 B − sin2 C)
sinA sinB 
abc sinA · sin(B + C) sin(B − C)
1 − cos A cosB ⇒p=
⇒ ≤1 (sin C ≤ 1) (b2 sin2 A − c2 sin2 A)
sinA sinB 
 a b c 
⇒ 1–cosA cosB ≤ sinA sinB  sin rule : = = 
 sinA sinB sinC 
⇒ 1 ≤ cos(A – B)
⇒ cos(A – B) ≥ 1 abc sin2 A·sin(B − C) abc sin(B − C)
= =
2 2 2
(b − c )·sin A b2 − c2
⇒ cos(A – B) = 1 [as cos( θ )≤1)
⇒ A–B=0 ab2r sin(B − C) ar sin(B − C)
= =
2
b −b r 2 2
1 − r2
On putting A = B in Eq. (i), we get
1 − cos2 A ar
sinC= ⇒p≤ [sin(B–C) ≤1]
sin2 A 1 − r2 
π
⇒ sinC=1 ⇒ C= Sol 21:
2
O
Now,A + B + C = π
2
π n
⇒A+B=
2 r r
π  π
 A B=
⇒ A == and C 
4  2
A1 A2
∴ sinA : sinB : sinC
Let O be the centre and r be the radius of the circle
π π π passing
= sin : sin : sin
4 4 2 through the vertices A1, A2, ..., An
1 1 2π
⇒a:b:c= : :1 = 1:1: 2 Then, ∠A1 OA2 = , also OA1 = OA2 = r
2 2 n
M a them a ti cs | 19.61

Again by cos formula we know that,  4π   3π 


⇒ sin   = sin  
2 2 2  n   n 
 2π  OA1 + OA2 − A1 A2
cos   = 4π 3π 7π
 n  2(OA1 )(OA2 ) ⇒ =π – ⇒ =π ⇒ n =7
n n n
 2π  r 2 + r 2 − A1 A22
⇒ cos   =
 n  2 (r) (r) Sol 22 : It is given that a, b, c and area of ∆ are rational.
 2π  2 B (s − c)(s − a)
⇒ 2r2 cos   = 2r – A1 A2
2
We have, tan =
n
  2 s(s − b)
 2π 
⇒ A1 A22 = 2r2 – 2r2 cos   s(s − a)(s − b)(s − c) ∆
 n  = =
s(s − b) s(s − b)
  2π   a+b+c
⇒ A1 A22 = 2r2  1 − cos   Again, a, b, c are rational given, s = are rational,
  n  2
π Also, (s – b) is rational since, ∆ is rational, therefore, we
⇒ A1 A22 = 2r2·2sin2   get
n
π B  ∆
π tan   = is rational.
⇒ A1 A22 = 4r2sin2   ⇒ A1 A2 = 2rsin   2 s(s − b)
n  
n
 2π  C ∆
Similarly, A1 A3 = 2rsin   Similarly, tan   = is rational,
2
  s(s − c)
 n 
 3π  B C
and A1 A 4 = 2rsin   Therefore a, tan , tan are rational.
2 2
 n 
1 1 1 Which shows that (i) ⇒ (ii)
Since, = + (given)
A1 A2 A1 A3 A1 A 4 Again, it is given that

1 1 1 B C
⇒= + a, tan , tan are rational, then
2r sin( π / n) 2r sin(2π / n) 2r sin(3π / n) 2 2

1 1 1 A π B+C
⇒= + tan = tan  − 
sin( π / n) sin(2π / n) sin(3π / n) 2 2 2 
B  C
1 − tan   · tan  
 3π   2π  B + C  1 2 2
sin   + sin   ⇒ cot  =
1  n   n  =
⇒ =  2  tan  B + C  B  C
tan   + tan  
sin( π / n) sin(2π / n) sin(3π / n)  
2 2 2
  2
 2π   3π   π   3π  π  2π  Since, tan (B/2) and tan(C/2) are rational numbers, we
⇒ sin =  · sin   sin   sin   + sin   · sin   get tan (A/2) is a rational number.
n
  n
  n
    n n
   n 

 2π    3π   π  π  3π  2 tanA/2
⇒ sin  
Now, sin A = as tan (A/2) is a rational
sin   − sin    =sin   · sin   1 + tan2 A/2
 n     n n
  n
   n 
number, sinA is a rational number. Similarly, sinB and
 2π     3π + π   3π − π   sinC are rational numbers. Thus, a, sinA, sinB, sinC are
⇒ sin    2cos   sin    rational numbers therefore (ii) ⇒ (iii)
 n     2n   2n  
Now again, a, sinA, sinB, sinC are rational.
π  3π 
= sin   · sin  
n  n  a b c
By the sine rule = =
sinA sinB sinC
 2π   2π  π  π   3π 
⇒ 2sin   · cos   · sin   =
sin   sin   a sinB a sinC
⇒ b = and c
 n   n  n  n   n = sinA sinA
 2π   2π   3π  since a, sin A, sin B and sin C are rational numbers,
⇒ 2sin   cos   = sin  
 n   n   n  ⇒ b and c are also rational.
1 9 . 6 2 | Solutions of Triangle

1 Combing above expressions;


Also, ∆ = bcsinA
2
p< 0 or p ≥ ( 2 +1)2
As b, c and sinA are rational numbers, ∆ is a rational
numbers. ie, P ∈ (– ∞ , 0) ∪ [( 2 +1)2, ∞ )

Therefore, a, b, c and ∆ are rational numbers. or P ∈ (– ∞ , 0) ∪ [3+2 2 , ∞ )


Therefore (iii) ⇒ (i).
Sol 24 : A
o
90 -B
Sol 23: Since, A + B + C = π
A/2
⇒ B + C = π – π /4 = 3 π /4 ...(i) (A= π /4, given) F G
R
∴0 < B, C< 3 π /4.
o
90
I
Also, given tanB · tanC = p r O
sinB·sinC p
⇒ =  ... (i)
cosB·cosC 1 B C
sinB·sinC + cosBcosC p + 1 It is clear from figure that OA = R.
⇒ =
sinB·sinC − cosB·cosC p − 1 IF
AI =
cos(B − C) 1 + p sin(A / 2)
⇒ =
cos(B + C) 1 − p r
∴ ∆ AIF is right angled triangle =
sin(A / 2)
(1 + p)
⇒ cos(B – C) = − ... (ii) (B+C = 3 π /4) Butr = 4R sin (A/2) sin (B/2) sin(C/2)
2(1 − p)

Since, B or C can vary from 0 to 3 π /4 ∴ AI = 4R sin (B/2) sin(C/2)

0 ≤ B – C < 3 π /4 Again, ∠ GOA = B ⇒ OAG = 90º – B


1 Therefore, ∠ IAO = ∠ IAC – ∠ OAC
− < cos(B − C) ≤ 1 ... (iii)
2  1
= A/2 – (90º – B) = (A+2B–180º)
From Eqs. (ii) and (iii), we get 2
1 1
1 1+p = (A + 2B – A – B – C) = (B – C)
− < ≤1 2 2
2 2(p − 1)
In ∆OAI, OI2 = OA2 + AI2 – 2(OA)(AI)cos ( ∠ IAO)
1 1+p 1+p
⇒ − < and ≤1
2 2(p − 1) 2(p − 1) = R2 + [4R sin (B/2)sin (C/2)]2 – 2R· [4Rsin(B/2) sin(C/2)]
B − C 
cos  
1+p 1 + p − 2p + 2  2 
⇒ + 1 ≥ 0 and ≤0
p −1 2(p − 1)
= [R2+16R2 sin2(B/2) sin2(C/2) – 8R2 sin(B/2) sin(C/2)] cos
 1+ 2  B − C 
(1 − 2)  p −   
2p  1 − 2   2 
≥ 0 and  ≤0

p −1 2(p − 1)
= R2 [1+16 sin2(B/2) sin2(C/2) – 8sin(B/2) sin(C/2)] cos
2p (p − ( 2 + 1)2 )  B − C 
⇒ > 0 and ≥0  
p −1 (p − 1)  2 
+ – + + – +
= R2 [1+8 sin(B/2)
0 1 1 ( 2 + 1)
2
  B − C  
sin(C/2) 2sin (B / 2 ) sin ( C / 2 ) cos   
⇒ (p< 0 or p > 1   2  

and( p < 1 or p>( 2 + 1)2 ) = R2 [1+8 sin(B/2)


M a them a ti cs | 19.63

 B − C  B + C   B − C  
sin(C/2) cos   +cos   – cos    Sol 25: A
  2   2   2  
  B + C 
= R 2 1 − 8 sin(B/2)sin(C/2)cos 
A/2 A/2

  2 
r1 K

2  π A  o
= R 1 − 8 sin(B/2)sin(C/2)cos  −   J 90
  2 2  F o E
r1 90 r1
 A B C π
 + + = H
 2 2 2 2

= R2 [1–8 sin(A/2) sin(B/2) sin(C/2)]


  r 
= R2 1 − 8    = R – 2Rr
2
B C
  4R  D o
90
Now, in right ∆BIO
The quadrilateral HEKJ is a square because all four
⇒ OB2 = BI2 + IO2 angles are right angle and JK = JH.
⇒ R2 = BI2 + R2 – 2Rr Therefore, HE = JK = r1 and IE = r(given)
⇒ 2Rr = BI2 ⇒ IH = r – r1
⇒ 2Rr = r /sin (B/2)
2 2
Now, in right angled triangle IHJ,
⇒ 2R = r/sin (B/2) 2
∠JIH = π/2 − A/2
⇒ 2R sin B/2 = r
2
[ ∠ IEA = 90º, ∠ IAE = A/2 and ∠ JIH = ∠ AIE ] in triangle
JIH
⇒ R (1–cosB) = r
π A r A r
tan  −  = 1 ⇒ cot = 1
 2 2  r − r1 2 r − r1
abc ∆
⇒ (1–cosB) =
4∆ s B r C r
Similarly, cot = 2 and cot = 3
2 r − r2 2 r − r3
4 ∆2
⇒ abc (1–cosB) =
s On adding above results, we get

 a2 + c2 − b2  4 ∆2 cot A/2 + cot B/2 + cot C/2 = cot A/2 cot B/2 cot C/2
⇒ abc 1 − =
 2ac  s r1 r2 r3 r1r2r3
⇒ + + =
 2ac − a2 − c2 + b2  4 ∆2 r − r1 r − r2 r − r3 (r − r1 ) (r − r2 ) (r − r3 )
⇒ abc  =
 2ac  s
1
4 ∆2 Sol 26: Given ∆ ≤ (a + b + c)abc
⇒ b[b2 – (a – c)2] = 4
s
1
⇒ b[b2 – (a – c)2] = 8(s – a)(s – b)(s – c) ⇒ (a + b + c)abc ≥ 1
4∆
⇒ b[{b – (a – c)} {b + (a – c)}] = 8(s – a)(s – b)(s – c)
(a + b + c)abc 2s abc
⇒ b[(b + c – a)(b + a – c)] = 8(s – a)(s – b)(s – c) ⇒ ≥1 ⇒ ≥1
2
16∆ 16∆2
⇒ b[(2s – 2a)(2s – 2c)] = 8(s – a)(s – b)(s – c)
s abc
⇒ ≥1
⇒ b[(2·2 (s – a)(s – c)] = 8(s – a)(s – b)(s – c) 8·s(s − a)(s − b)(s − c)
⇒ b = 2s – 2b
abc
⇒ ≥1
⇒ 2b = a + c 8(s − a)(s − b)(s − c)
Which shows that b is arithmetic mean between a abc
and c. ⇒ ≥ (s – a)(s – b)(s – c)
8
1 9 . 6 4 | Solutions of Triangle

Now, puts – a = x ≥ 0, s – b = y ≥ 0, s – c = z ≥ 0
3 3 3
x− y−
s–a+s–b=x+y 2 = 2 = −1 ⇒ C ≡ ( 3, 1)
Equation of CD is
3 1
2s – a – b = x + y
2 2
c=x+y Equation of the circle is (x − 3)2 + (y − 1)2 =
1
Similarly,a = y + z, b = x + z

(x + y) (y + z) (x + z) Sol 29: (A) Since the radius of the circle is 1 and C


⇒ · · ≥ xyz
2 2 2 ( 3,1) , coordinates of F ≡ ( 3,0)

which it true x − 3 y −1  3 3
Equation of CE is = = 1 ⇒ E≡ , 
3 1  2 2
Now quality will hold if −  
2 2
x = y = z ⇒ a = b = c ⇒ triangle is equilateral.
Sol 30: (D) Equation of QR is y −=
3 3(x − 3)
Sol 27: (B)
C ( ,2r) ⇒ y=3x
(0, 2r) D
Equation of RP is y = 0.

(r, r) Sol 31: (D) X


= sin θ + sin 3θ + .... + sin 29 θ
(0, 0) A B (2 ,0) 2(sin θ)X = 1 − cos 2θ + cos 2θ − cos 4θ + ... +
cos 28θ − cos 30θ
1
18
= (3α )(2r) ⇒ α=
r 6
2 1 − cos 30 θ 1
=X =
2r 2 sin θ 4 sin 2o
Line y =− (x − 2α ) is tangent to
α
Sol 32: (B, C)
(x − r)2 + (y − r)2 =
r2
2α 3r and
= = αr 6 B + C  B − C  2 A
2cos   cos   = 4 sin
 2   2  2
r = 2.
B − C 
cos   = 2sin (A / 2)
Sol 28: (D)  2 
2 2 B − C 
(x - 2 3) + (y - 1) = 1 cos  
 2 = sinB + sinC
⇒ 2 ⇒ 2
=
sin A / 2 sin A
 3, 3
Q
⇒ b+c =2a (constant)

 3 3
,
2 2  3 3 3
,
2 2  Sol 33: (C, D) Given solutions
E D
1
sin( π / 4)
C
 sin(θ + π / 4 − θ) sin(θ + π / 2 − (θ + π / 4)) 
 + 
R 60o 60
o
P  sin θ ⋅ sin(θ + π / 4) sin(θ + π / 4) ⋅ sin(θ + π / 2) 
 sin((θ + 3π / 2) − (θ + 5π / 4)) 
F  3, 0 3x + y - 6 -0
 +.... +
 sin( θ + 3ο / 2) ⋅ sin( θ + 5 π / 4)


x- 3
x- 3 =4 2

⇒ 2[cos θ − cot(θ + π / 4) + cot(θ + π / 4)


− cot(θ + π / 2) + .... + cot(θ + 5π / 4) − cot(θ + 3π / 2)] = 4 2
M a them a ti cs | 19.65

2 2 1 2 2 ⇒ 3(x2 + x + 1)= 2x2 + 2x − 1


Sol 34: cos α = , sin α = , tan α =
3 3 R
⇒ ( 3 − 2)x2 + ( 3 − 2)x + ( 3 + 1) =
0
2 2
⇒ R
= = 8 units
tan θ On solving
C1 ⇒ x2 + x − (3 3 + 5) =0

2 2 C2 x= 3 + 1, − (2 + 3)
A 1
P 

3 B
Sol 37: (A, C, D)

2sin2 θ
2cos θ(1 − sin=
ϕ) cos ϕ −
= 1 2sin θ ϕ − 1
R  sin θ
2cos θ − 2cos θ sin
= ϕ 2 sin θ cos ϕ − 1
A
C 2 cos θ=
+ 1 2 sin (θ + ϕ)
3
tan(2π − θ) > 0 ⇒ tan θ < 0 and −1 < sin θ < −
2
 3π 5π 
30
o ⇒ θ∈ , 
B C  2 3 
Sol 35: 1
< sin (θ + ϕ) < 1
2 π 5π
a2 + 16 − 8 3 a2 + 8 ⇒ 2π + < θ + ϕ < + 2π
cos β = ⇒ = 6 6
2× a× 4 2 8a
π 5π
⇒ a2 + 4 3 a + 8 =0 2π + − θmax < ϕ < 2π + − θmin
6 6
⇒ a= π 4π
1 + a2 4 3,=
a1a2 8 <ϕ<
2 3
⇒ | a1 − a2 |= 4
1
⇒ | ∆1 − ∆2 =| × 4 sin 30o × 4= 4 Sol 38: (C)
2

Sol 36: (A, B) A a = 2 = QR, b= 7= PR , c= 5= PQ


2 2
a+b+c 8
s
= = = 4
2 4
2
2x+1 x -1
2 sin P − 2sinP cos P 2 sin P(1 − cosP)
=
2 sin P + 2sin P cos P 2sin P(1 + cosP)
/6 P
B C 2 sin2
1 − cosP 2 tan2 P
= = =
2
x +x+1
1 + cosP P 2
2cos2
π (x2 − 1)2 + (x2 + x + 1)2 − (2x + 1)2 2
cos =
6 2(x2 + x + 1)(x2 − 1) (s − b)(s − c) (s − b)2 (s − c)2
= =
3 (x2 − 1)2 + (x2 + 3x + 2)(x2 − x) s(s − a) ∆2
=
2 2(x2 + x + 1)(x2 − 1) 2 2
 7  5
4 −  4 −  2
3 (x2 − 1)2 + (x + 1)(x + 2)x(x − 1)  2  2  3 
= = =  
2 2(x2 + x + 1)(x2 − 1) ∆2  4∆ 
x2 − 1 + x(x + 2)
⇒ 3=
x2 + x + 1
1 9 . 6 6 | Solutions of Triangle

Sol 39: (B, D) P P(at2, 2at)


s-a
M  16a 8a 
Q ,− 
N
 t2 t 
s-c
1
∆OPQ = OP ⋅ OQ
2
Q s-b L R
1 a( −4) 16
⇒ at t2 + 4 ⋅ +4 =
3 2
Let s – a = 2k – 2, s – b = 2k, s – c = 2k + 2, k ∈ I, k > 1 2 t t2
Adding we get,
⇒ t2 − 3 2t + 4 =
0 ⇒ t = 2, 2 2
s = 6k
 t2 
So, a = 4k + 2, b = 4k, c = 4k – 2 P(at2 , 2at) = P  ,t 
2 
 
1
Now, cosP =
3 =t 2 ⇒ P(1, 2)

b2 + c2 − a2 1 =t 2 2 ⇒ P(4, 2 2)
⇒ =
2bc 3
⇒ 3[(4k)2 + (4k − 2)2 − (4k + 2)2 ] =2 × 4k(4k − 2)
Sol 42: (A, B, C)
2
⇒ 3[16k − 4 (4k) ×=
2] 8k (4k − 2)
Q2
⇒ 48k 2 − 96k = 32k 2 − 16k ⇒ 16k 2= 80k ⇒ k= 5
So, sides are 22, 20, 18
R2

Sol 40: (B) x = a + b, y = ab


x2 – c2= y T

2 2 2
a +b −c 1 ( 2, 1 (
⇒ =− =cos(120o )
2ab 2 0
2π 0
⇒ ∠C =
3
abc ∆
R
⇒ = ,=
r
4∆ s R3
2
1  2π  
4  absin    Q3
r 4∆  22
 3 
⇒= = x2 + y 2 =
3
R s(abc) x+c
⋅ y ⋅c
2 x2 = 2y
r 3y
=
R 2c(x + c) Intersection point is P ≡ ( 2,1)

Equation of tangent is 2x + y =
3
Sol 41: (A, D) y 2
P(at , 2at)
tan(θ) =− 2
1
tan(α ) = tan(θ − 90) = − cot θ =
0
x 2
1 2 3
sin(α=
) =
3 Q3 T

 
Q
16a 8a ⇒ Q3 T =
6
,-
t
2 t
M a them a ti cs | 19.67

∴ Q 2 Q3 =
2Q3 T =
12 5s 2s 7s
⋅ ⋅
xyz 9 3 = 9 35
R =
= 6
1 2 3 4∆ 4×6 6 24
tan(α=
) = ⇒ R3 =
T 2 6
2 R3 T 8
x y z r 3 = 4
∴ R 2R 3 =2R3 T =4 6 sin   sin   sin  =  =
2 2
      2 4R 35 6 35

3 24
⊥ distance of o from R2 R3 is = 3
( 2)2 + 12 x+y 2z 1 + cos(z) 3
sin2=
  cos
=   =
 z  2
  2 5
1
∴ Area (OR 2R 3 ) = × 3 × 4 6 =6 2 square units
2
1 Sol 44: (A)
Similarly, Area (PQ2Q3 ) = × 2 × 12 =6 2 square
units 2
M

Sol 43: (A, C, D)


s − X s − y s − z 3s − (x + y + z) s
= = = = F1 F2
4 3 2 9 9
(1, 0)
5s 2s 7s (-1,0)
∴ x= , y= , z=
9 3 9

A=πr 2 = N
3

∆ 8 2 8s2 a=3
⇒ = ⇒ ∆
=
s 3 3 1
e=
3
4s s 2s 8 2
⇒ s⋅ ⋅ ⋅ =s ∴ F1 ≡ ( −1, 0)
9 3 9 3
F2 ≡ (1,0)
⇒ s=
9
So, equation of parabola is y2 = 4x
8
∴ =
∆ 9 6 6 square units
×= 3 
3 Solving simultaneously, we get  , ± 6 
 2 
 −9 
∴ Orthocentre is  , 0
 10 
2017-18 100 &
op kers
Class 12 T
By E ran culty
-JE Fa r
IIT enior emie .
S fP r es
o titut
Ins

MATHEMATICS
FOR JEE MAIN & ADVANCED
SECOND
EDITION

Exhaustive Theory
(Now Revised)

Formula Sheet
9000+ Problems
based on latest JEE pattern

2500 + 1000 (New) Problems


of previous 35 years of
AIEEE (JEE Main) and IIT-JEE (JEE Adv)

5000+Illustrations and Solved Examples


Detailed Solutions
of all problems available

Topic Covered Plancess Concepts


Tips & Tricks, Facts, Notes, Misconceptions,
Inverse Trigonometric Key Take Aways, Problem Solving Tactics
Functions
PlancEssential
Questions recommended for revision
20. INVERSE
TRIGONOMETRIC
FUNCTIONS
1. INTRODUCTION TO INVERSE TRIGONOMETRY
The inverse trigonometric functions are the inverse functions of the trigonometric functions. They are sometimes
referred to as cyclometric functions.

2. IMPORTANT DEFINITIONS
Given two non-empty sets X and Y, let f:X → Y be a function, such that y = f(x). The set X is called as the domain
of f while the set Y is called as the co-domain of f. The set {f(x): x ∈ X} is called as range of f.A map f: A → B is
said to be one-one or injective, if and only if, distinct elements of A have distinct images in B, i.e. if, and only if,
x1 ≠ x2 ⇒ f(x1 ) ≠ f(x2 ) , for all x1 , x2 ∈ A
Onto map or Surjective map: A map f: A → B is said to be an onto map or Surjective map if, and only if, each
element of B is the image of some element of A, i.e. if, and only if, Range of f = co-domain of f.
Objective map: A map f : A → B is an objective map if, and only if, it is both one – one and onto.

3. INVERSE FUNCTIONS
If f : X → Y is one-to-one and onto (i.e. f is objective), then, we can define a unique function g : Y → X, such that
g(y) = x, where x ∈ X is such that y = f (x). Thus, the domain of g = range of f and range of g = domain of f. The
function is called the inverse of f and is denoted by f-1.
(a) Trigonometric functions are many-one functions but these become one-one, onto, if we restrict the domain of
trigonometric functions. Similarly, co-domain is equated to range to make it an onto function. We can say that
the inverse of trigonometric functions are defined within restricted domains of corresponding trigonometric
functions.
(b) Inverse of sin (sine functions) is denoted by sin-1 (arc sine function). We also write it as sin-1 x. Similarly, other
inverse trigonometric functions are given by cos-1 x, tan-1 x, sec-1 x, cot-1x and cosec-1 x.
1
(c) Note that sin−1 x ≠ and ( sin−1 x)² ≠ sin−2 x, Also sin−1 x ≠ (sinx)−1
sinx
(d) Domain and Range of Inverse Trigonometric Functions:

Function Domain Range (Principal value branch)


(i)
y = sin−1 x −1 ≤ x ≤ 1 −
π
≤y ≤
π
2 2
2 0 . 2 | Inverse Trigonometric Functions

Function Domain Range (Principal value branch)


(ii)
y = cos−1 x −1 ≤ x ≤ 1 0≤y≤π

(iii)
−∞ < x < ∞ π π
y = tan−1 x − ≤y <
2 2

(iv)
x ≥ 1 or x ≤ − 1 π π
y = cosec−1 x − ≤y ≤ ,y ≠ 0
2 2

(v)
x ≥ 1 or x ≤ − 1 π
y = sec−1 x 0 ≤ y ≤ π, y ≠
2

(vi)
y = cot −1 x −∞ < x < ∞ 0<y<π

(e) The principal value of an inverse trigonometric function is the value of that inverse trigonometric function
which lies in the range of principal branch.

PLANCESS CONCEPTS

If no branch of an inverse trigonometric function is mentioned, then it can be implied that the principal
value branch of that function.
You can remember range as set of angles that have the smallest absolute values satisfying for all the
values of domain.
Vaibhav Gupta (JEE 2009 AIR 54)

4. TRANSFORMATION OF TRIGONOMETRIC FUNCTIONS TO INVERSE


TRIGONOMETRIC FUNCTIONS

4.1 sin x to sin-1 x

The graph of an inverse trigonometric function can be obtained from the graph of the original by interchanging x
and y axes.
Note: It can be shown that the graph of an inverse function can be obtained from the corresponding graph of
original function as the mirror image in the line y = x.
Y
-5 - 1 3
X’ 2 2
 
2 X
-2  O 2 5
-1 2 2
Y’
Figure 20.1
−1
≤ 1 ; y sin x, x ≤ 1, y ∈[ −π / 2, π / 2]
(a) y = sin x, x ∈ R and y=
M a them a ti cs | 20.3

4.2 cos x to cos-1 x

(b) y =cos x, x ∈R and =


y ≤ 1 y cos−1 x, x ∈[ −1,1]and y ∈[0, π]

Y Y
5 5
2 2
2
3 3

x
2 2

y=
 

2
-
-2 2  -1 1
X’ X X’ X
- O  2 O 
- 2
-x 2
2
2
 -

-3 -3
2 2
-2
-5 -5
2 2
Y’ Y’
-1 -1
y-sin x and y-sin (x) y-sin (x)

Figure 20.2

5
2
2
3
2


Y 2
1 X’ -1 1X
-5  5 O
- 2 
X’ 2 2 X -
-2 -3 - O 3 2
2 2 -1 2
2 -
Y’
-3
y=cos x 2
-2

-5
2
Y’
-1
Figure 20.3 y=cos (x)
2 0 . 4 | Inverse Trigonometric Functions

4.3 tan x to tan-1 x

 π   π π
∈R y tan−1 x, x ∈R and y ∈ − , 
(c) y= tanx, x ∈R − x : x= (2π + 1) ,n∈ Z  and y=
 2   2 2
Y

Y 2 3
2

2 
 3 2
- 1 2 2 2
X’ X O
- O  X’
-2 -1
-1 X
2 1 2
-2
-
2 -

Y’ Y’
y=tan x

Figure 20.4

4.4 cot x to cot-1 x


(d) y =cot x, x ∈R − {x : x =nπ, n∈ Z} and y ∈
=R y cot −1 x, x ∈R and y ∈(0, π)

Y 2 3
2

2 
 3 2
- 1 2 2 2
X’ X O
- O  X’
-2 -1
-1 X
2 1 2
-2
-
2 -

Y’ Y’
y=tan x -1
y=tan x

Figure 20.5

4.5 sec x to sec-1 x


 π  π
(e) (v) y= sec x, x ∈R − x : x= (2n+ 1) ,n ∈ z  and y ∈R − ( −1,1)
= y sec−1 x, x ∈R − ( −1,1)and y ∈[0, π]  
 2  2 
M a them a ti cs | 20.5

2
Y 3
2


2

- 1  2 2
X’ X
- O  3
-1 X’ X
2 -2 2 2
-2 -1 1 2

-
2
-
Y’
y = sec x
Y’ -1
y = sec x

Figure 20.6

4.6 cosec x to cosec-1 x


 π π
(f) ( −1,1) y co sec−1 x, x ∈R − ( −1,1)and y ∈  − ,  − {0}
y = cosec x, x ∈{x : x = n π,n∈ Z} and y ∈R −=
 2 2

2
Y
3
2

2 

- 3
1 2
- 2  2 2
X’ X X
O
-1  X’ -2 -1
2
O 1 2
-2

-
2
-
Y’

y = cosec x Y’
-1
y = cosec x

Figure 20.7
2 0 . 6 | Inverse Trigonometric Functions

Illustration 1: Find the domain of definition of the function f(x) = 3cos−1 (4 x) − π . (JEE MAIN)

Sol: Use the condition that the expression inside the square root is ≥ zero.

For domain of f(x) = 3cos−1 (4 x) − π , we must have

π  1 1
4x ≥ cos   ⇒ 4x ≥ ⇒x ≥  …..(i)
3  2 8
−1 1
Also −1 ≤ 4x ≤ 1 ⇒ ≤ x ≤  .….(ii)
4 4
 −1 1 
∴ From (i) and (ii), we get x ∈  ' 
 4 8

PLANCESS CONCEPTS

In case of confusion, try solving problems by replacing inverse functions with angles and applying
trigonometric identities.
Shrikant Nagori (JEE 2009 AIR 30)

Illustration 2: If 0 < cos−1 x < 1and 1 + sin(cos−1 x) + sin2 (cos−1 x) + sin3 (cos−1 x) + ......∞ =2, then find the value of x.
 (JEE MAIN)
Sol: Use summation of infinite GP series.
We have 1+sin(cos-1 x) + sin²(cos-1 x) + ……… ∞ =2

1 1 1 π 3
⇒ =2 ⇒ =1 − sin(cos−1 x) ⇒ sin(cos−1 x) = ⇒ cos=
−1
x ⇒
= x
−1
1 − sin(cos x) 2 2 6 2

2
Illustration 3: Let f(x)= (sin−1 [x] + tan−1 [x] + cot −1 [x]) , where [x] denotes the greatest integer less than or equal to
π
x. If A and B denote the domain and range of f(x) respectively, find the number of integers in A∪B.
 (JEE ADVANCED)
π
Sol: Use tan−1 [x] + cot −1 [x] = and proceed.
2
For domain of f(x), we must have −1 ≤ [x] ≤ 1 ⇒ − 1 ≤ x < 2, so set A = [-1,2)

2  −1 π  π 
=f(x)  sin [x] +   As tan−1 [x] + cot −1 [x]= , ∀ x∈A 
π 2  2 

So, set B = {0, 1, 2} = Range of f(x). Now, A ∪ B =−


[ 1,2) ∪ {0,1,2} =−
[ 1,2]
Hence, number of integers in ( A ∪ B )=4

5.PROPERTIES/IDENTITIES OF INVERSE TRIGONOMETRIC FUNCTIONS

5.1 Complementary Angles

π
(a) sin−1 x + cos−1 x= , ∀x ∈ [ −1,1]
2
M a them a ti cs | 20.7

π
(b) tan−1 x + cot −1 x = , ∀x ∈R
2
π
(c) sec−1 x + cosec−1=
x , ∀ x ∈( −∞ , −1] ∪ [1, ∞ )
2

5.2 Negative Arguments

(a) sin−1 ( −x) = − sin−1 x, ∀ x ∈[ −1,1]

(b) cos−1 ( −x) = π − cos−1 x, ∀ x ∈[ −1,1]

(c) tan−1 ( −x) =− tan−1 x, ∀ x ∈R

(d) cot −1 ( − x) = π − cot −1 x, ∀ x ∈R

(e) sec−1 ( − x) = π − sec−1 x, ∀ x ∈( −∞ , − 1] ∪ [1, ∞ )

(f) cosec−1 ( − x) = − cosec−1 x∀x ∈ ( −∞ , −1) ∪ [1, ∞ )

5.3 Reciprocal Arguments


−1 1
(a) cosec
= x sin−1 ; x ≥ 1 (Both the functions are identical)
x
1
sin−1 x cosec−1
and= ; x ≤ 1, x ≠ 0 (Both the functions are not identical)
x

1
sec−1 x cos−1
(b) = ; x ≥ 1 (Both the functions are identical)
x
1
and
= cos−1 x sec−1 ; x ≤ 1 (Both the functions are not identical)
x

1 1
(c)
= tan−1 x cot −1   , x ∈(0, ∞ ) = − π + cot −1   , x ∈( −∞ ,0) ,
x x
1 1
and
= cot −1 x tan−1   , x ∈(0, ∞ ) = π + tan−1   , x ∈( −∞ ,0)
x
  x

5.4 Forward Inverse Identities

y= sin(sin−1 x)= x, x ∈ [ −1,1], y ∈ [ −1,1], y is


(a)  y= cos(cos−1 x)= x, x ∈ [ −1,1], y ∈ [ −1,1], y is
(b) 
aperiodic aperiodic
y y
1 1
x

x
y=

y=

o o
-1 45 x -1 45 x
O 1 O 1

-1 -1

Figure 20.8
2 0 . 8 | Inverse Trigonometric Functions

y = tan ( tan−1 x )=x, x ∈ R, y ∈ R, y is aperiodic


(c)  (d) y = cot (cot −1 x) = x, x ∈ R, y ∈ R, y is aperiodic

y y

x
y=

y=
o o
45 45
x x
O O

Figure 20.9

y = cosec ( cosec−1 x )=x, x ≥ 1, y ≥ 1 ,y is aperiodic (f) y = sec (sec−1 x) = x, x ≥ 1, y ≥ 1 , y is aperiodic


(e) 

y y
x

x
y=

y=
o
1 1 45

-1 -1
x x
O 1 O 1

-1 -1
x

x
y=

y=

Figure 20.10

Also,

cos(sin−1=
x) 1 − x2 sin(cos−1=
x) 1 − x2

(
cos tan−1 x = ) 1
1 + x2
tan(cos−1 x) =
1 − x2
x

(
sin tan−1 x =) x
1+x 2
(
tan sin−1 x = ) x
1 − x2

5.5 Inverse Forward Identities

 π π
y sin−1 (sinx)
(a) = = x,  x ∈ R  , y ∈  − ,  , Periodic with period 2 π
 2 2
y
/2
+x

x
y=

2
y=

y=
2

x-
+

-

-/2 3/2
y=

y=

o
x

45
x )

 x
-2 -3/2 - O /2 2

-/2

Figure 20.11
M a them a ti cs | 20.9

(b) y = cos−1 (cosx) = x, x ∈R, y ∈[0, π], Periodic with period 2 π

y

x y=

2
y=

y
2

=
/2

x+
-x

-x
y=
x
-2 - -/2 O /2  2

Figure 20.12

 π   π π
(c) y= tan−1 (tanx)= x, x ∈R − (2n− 1) n ∈ I  , y ∈ − ,  , Periodic with period π
 2   2 2

y
/2



y=
x+

x-
- 3

y=
y=

-2 2 2
 x
-3 - O  2
2 2

/2

Figure 20.13

(d) y = cot −1 (cotx)= x, x ∈R − {n π}, y ∈ (0, π), periodic with π


y

2

x


y=

-
x+

x+

x
y=


y=

y=

O  2
2

x
-2 - O  2
Figure 20.14

 π   π
(e) y = cosec−1 (cosec x) = x ∈R − {n π,n ∈I}, y ∈  − ,0  ∪  0,  y is periodic with period 2π
 2   2

y
/2
x
2
y=

y=

y=
x-
(

-
+

y=

-/2 3/2
x

o
45
x)

 x
-3/2 - O /2 2

-/2
Figure 20.15
2 0 . 1 0 | Inverse Trigonometric Functions

−1
(f) y sec
= = (sec x) x, y is periodic,

y

y
y= =


2

x
-x

x+

y=
-x

y= x
-2 -3 - - O   3 2
2 2
2 2
Figure 20.16

 π   π π 
x ∈ R − (2n− 1) n ∈ I  , y ∈ 0,  ∪  , π  with period 2π
 2   2 2 
1
(i) tan−1 (cotx)= π − x for x ∈[0, π]
2
1
sin−1 (cosecx)=
(ii) π − x for x ∈[0, π]
2
1  1 
sec−1 (cosx)=
(iii) π − x for x ∈ 0, π  .
2  2 

5.6 Sum of Angles


 sin−1  x 1 − y 2 + y 1 − x2  if x ≥ 0; y ≥ 0 and x2 + y 2 ≤ 1
  
(a) sin−1 x + sin−1 y =

π − sin−1  x 1 − y 2 + y 1 − x2  if x ≥ 0; y ≥ 0 and x2 + y 2 > 1
  

(b) sin−1 x − sin


= −1
y sin−1  x 1 − y 2 − y 1 − x2  x > 0; y > 0
 

(c) cos−1 x ± cos


= −1
y cos−1 [xy  1 − x2 1 − y 2 ] if x, y >0 and x² + y² ≤ 1

(d) cos−1 x ± cos−1 y = π − cos−1 [xy  1 − x2 1 − y 2 ] if x, y > 0 and x²+y² ≤ 1

 −1 x + y π
 tan x > 0 y > 0 and xy < 1 ⇒ 0 < tan−1 x + tan−1 y <
 1 − xy 2
(e) tan−1 x + tan−1 y =

π − tan−1 x + y x > 0 y > 0 andxy > 1 ⇒ π < tan−1 x + tan−1 y < π
 1 − xy 2

x−y
(f) x > 0 & y > 0 then tan−1 x − tan−1 y =
tan−1 (with no other restriction)
1 + xy

 x + y + z − xyz   x−y 
(g) tan−1 x + tan−1 y + tan−1 z =
tan−1  −1 −1
tan−1 
 ; tan − tan y = 
 1 − xy − yz − zx   1 + xy 
M a them a ti cs | 20.11

PLANCESS CONCEPTS

The above results can be generalized as follows:

 S − S3 + S5 − .............. 
tan−1 x1 + tan−1 x2 + ... + tan−1 xn =tan−1  1 
 1 − S2 + S 4 − S6 + ....... 
where Sk denotes the sum of products of x1 , x2 , , xn taken k at a time
Rohit Kumar (JEE 2012 AIR 78)

 15 
Illustration 4: Evaluate: sin  tan−1   (JEE MAIN)
 8 

15
Sol: Convert tan−1 to sin−1 .
8
We know that sin (sin−1 x) = x , for all x ∈ [-1, 1], So, will convert each expression in the
form sin (sin−1 x) by using h=17 p=15
b −1 p p −1 p p b
cos−1
= sin= , tan−1 sin
= ,cot −1 sin−1 etc.
h h b h b h b=8
Where b, p and h denote the base, perpendicular and hypotenuse of a right triangle. Figure 20.17

 −1 15   −1 15  15
sin
=  tan  sin
=  sin 
 8   17  17

 13 
Illustration 5: Evaluate: cos  cosec−1  (JEE MAIN)
 12 
h=13 p=12
Sol: Write cosec-1 in terms of cos-1.
b=5
 −1 13   −1 5  5  13 5
cos=
 cosec  cos
=  cos   cosec−1
= cos−1 
 12   13  13  12 13  Figure 20.18

Illustration 6: Find the principal value of cot-1 (- 3 ) (JEE MAIN)

Sol: The principal value of cot-1 x lies in between 0 to π .

Let cot −1 ( − 3) =
θ
π
Then cot θ = − 3 = − cot
6
Since principal value branch of cot-1 x is 0 < θ < π . Therefore, we want to find the value of θ such that 0 < θ < π .

π  π 5π
Now, cot θ = − cot = cot  π −  = cot
6  6  6

Therefore, principal value of cot −1 ( − 3) =.
6
2 0 . 1 2 | Inverse Trigonometric Functions

−1  10π  10π
Illustration 7: sin  sin =  (JEE MAIN)
 7  7

10π 3π
Sol: Write as π + and expand.
7 7

 10π    3π     3π   3π
sin−1  sin
= =sin−1  − sin    = sin−1  sin  −   =

 7    7    7  7

  π 
Illustration 8: cos−1  sin  −    (JEE MAIN)
  9 

  π π    11π   11π
Sol: cos−1  cos  =
= +   cos−1  cos = 
  2 9    18   18

−1  13π 
Illustration 9: sin  cos   (JEE MAIN)
 10 

Sol: Similar to previous example.

13π  3π    5π 2π    π   π  π
=sin−1 cos =sin−1  − cos  =sin−1  − sin  −  =sin−1  − sin  =sin−1  sin  −   =

10  10    10 10   5    5  5

 1 
Illustration 10: Find the principal value of sin−1   . (JEE MAIN)
 2
 1  1  1 
Sol: Let sin−1   = y. Then sin y = ⇒ sin−1 
y= 
 2 2  2
 π π π 1
We know that, the range of the principal value branch of sin−1 is  − ,  and sin   = .
 2 2 4 2
 1  π
Therefore, principal value of sin−1   is .
 2 4

Illustration 11: Find the integral solution of the inequality 3x2 + 8x < 2sin−1 (sin 4) − cos−1 (cos 4) .
 (JEE ADVANCED)

Sol: Use inverse forward identities to simplify the equation.


3x2 + 8x < − 4 ⇒ 3x2 + 8x + 4 < 0
⇒ 3x2 + 6x + 2x + 4 < 0 ⇒ 3x(x + 2) + 2(x + 2) < 0
(x + 2)(3 x + 2) < 0 x=-1 -2 -1 -2/3
Figure 20.19

Illustration 12: Find the largest integral value of k, for which (k-2)x² + 8x + k + 4 > sin-1(sin12) +cos-1(cos 12), for
all x ∈ R. (JEE ADVANCED)

Sol: Use inverse forward identities.


sin−1 (sin 12) = sin−1 (sin(12 − 4 π)) = 12 − 4 π
M a them a ti cs | 20.13

cos−1 (cos 12) = cos−1 (cos(4 π − 12)) = 4 π − 12



∴ (k − 2) x2 + 8x + k + 4 > 0, ∀ x ∈ R -6 0 4
If k = 2, then 8x+4>0, (not possible)
and if k ≠ 2 , then k-2>0 ⇒ k>2 2
and 64 − 4(k − 2)(k + 4) < 0 ⇒ 16 < k 2 + 2k − 8 Figure 20.20

⇒ k 2 + 2k − 24 > 0 ⇒ (k + 6)(k − 4) > 0


K=5

1
Illustration 13: Find domain of f(x) = .  (JEE MAIN)
(
ln cot −1 x )
Sol: Find the range of x for which ln(cot −1 x) > 0 ⇒ cot −1 x > 1 ⇒ x < cot 1 ⇒ x ∈ ( −∞ ,cot1)

Illustration 14: Evaluate the following: (JEE MAIN)

 π  π  7π    3  π 
(i) sin−1  sin  (ii) tan−1  tan  (iii) cos−1  cos  (iv) cos cos−1  +
 3  4  6   2  6
   
π π
Sol: Recall that, sin−1 (sin θ) =θ, if − ≤ θ ≤ ,cos−1 (cos θ) = θ, if 0 ≤ θ ≤ π and
2 2
π π
tan−1 (tan θ) = θ, if − < θ < . Therefore,
2 2

 π π  π π
(i) sin−1  sin  = (ii) tan−1  tan  =
 3 3  4 4

 7π  7π 7π
(iii) cos−1  cos  ≠ , because does not lie between 0 and π.
 6  6 6

−1  7π  −1   5π    7 π 5π   5π  5π
Now, cos=  cos  cos  cos  2π −    = 2π −  = cos−1  cos  [cos(2π − =
θ) cos θ] =
 6    6   6 6   6  6


  3  π  5π π    3  5π 
(iv) cos cos−1   + = cos  +   cos−1  = 
 2  6  6 6   2  6

   
   

Illustration 15: Evaluate the following:

 3 π  1 
(i) sin  cos−1  (ii) sin  − sin−1  −   (iii) sin(cot −1 x)  (JEE MAIN)
 5   2  2 

3 3 4
Sol: (i) Let cos−1 = θ. Then, cos
= θ ⇒ sin
= θ
5 5 5

 3 4
∴ sin  cos−1 =
 sin=
θ
 5 5
2 0 . 1 4 | Inverse Trigonometric Functions

π  1   π  π  2π 3
(ii) sin  − sin−1  − =
  sin  −  − =
  sin=
2  2   2  6  3 2

(iii) Let cot −1 x =


θ, Then, x =
cot θ

1 1
Now, cot θ= x ⇒ sin θ= ∴ sin(cot −1 x)= sin θ=
1 + x2 1 + x2

Illustration 16: Evaluate the following:


(i) sin−1 (sin5) (ii) cos−1 (cos10)  (JEE MAIN)

Sol: Notice that the angle is in radians.


π π
(i) Here, θ =5 radians. Clearly, it does not lie between - and . But
2 2
π π
2π − 5 and 5 − 2π both lie between - and such that
2 2
sin (5 − 2π) = sin( −(2π − 5)) = − sin(2π − 5) =− ( − sin5) = sin5

⇒ sin−1 (sin 5) = sin−1 (sin(5 − 2π)) = 5 − 2π.

(ii) We know that cos−1 (cos θ) =θ, if 0 ≤ θ ≤ π. Here, θ =10 radians. Clearly, it does not lie between 0 and π such
that, (4 π − 10) =cos10 ⇒ cos−1 (cos 10) = cos−1 (cos(4 π − 10)) = 4 π − 10

Illustration 17: Evaluate the following:


 1 π
(i) sin−1 (2 sin−1 0.8) (ii) tan  2 tan−1 −   (JEE MAIN)
 5 4

Sol: Write the term inside the brackets in (i) and (ii) as sin-1 and tan-1 respectively.

(i) We know that:


= 2sin−1 x sin−1 (2 x 1 − x2 )

∴ 2 sin−1 0.8
= sin−1 (2 x 0.8 x 1 − 0.64 )

⇒ sin−1 (2sin−1 0.8) =sin{sin−1 (0.96)} =0.96

 1 π
(ii) tan  2 tan−1 − 
 5 4

 5 π  −1 1 5
= tan  tan−1 −  From (ii) we have, 2 tan = tan−1 
 12 4   5 12 

 5   −1 −1 −1  x − y    −1  −7   7
= tan  tan−1 − tan−1 1  =
 tan x − tan y tan   if xy > −1 = tan tan    = −
 12    1 + xy    17
  17

Illustration 18: Write the following in their simplest forms:

1 − cos x
(i) tan−1 (ii) sin [cot −1 {cos(tan−1 x)}]  (JEE ADVANCED)
1 + cos x

Sol: (i) Use the formula 1 − cos x = 2sin2 x / 2 and 1 + cos x = 2cos2 x / 2
M a them a ti cs | 20.15

(ii) Write the term inside the square bracket in terms of sin-1.

1 − cos x 2sin2 x / 2 x x
(i) tan−1 = tan−1 = tan −1
= tan
1 + cos x 2
2cos x / 2 2 2

(ii) sin[cot −1 {cos(tan−1 x)}]

   1    1   1 + x2   1 
= sin cot −1 cos  cos−1   = sin  cot −1  = sin sin−1   cot −1 x = sin−1 
       
   1 + x2    1 + x2   2 + x2  1 + x2

1 + x2
=
2 + x2

 cos x  π π
Illustration 19: Express tan−1   , − < x < in the simplest form. (JEE ADVANCED)
 1 − sinx  2 2

x
Sol: Convert the term inside the bracket in terms of tan and proceed.
2

 x x 
 cos x   cos2 − sin2 
We write, ⇒ tan  −1 −1
tan  2 2
= 
 1 − sinx   cos2 x + sin2 x − 2sin x cos x 
 2 2 2 2 

 x x  x x 
  cos + sin   cos − sin    x x  x
 2 2  2 2   cos 2 + sin 2   1 + tan 2    π x  π x
 =  tan
= −1
 
−1
tan   =tan−1 tan  +   = +
 cos x − sin x   1 − tan x    4 2  4 2
2
  x x 
  cos − sin   
 2 2 
 2
  2 2 

Alternatively,
 π     π − 2x  
 sin  − x    sin   
 cos x 
tan−1  −1  2   tan−1   2  
=  tan
=

 1 − sinx  π    π − 2x  
 1 − cos  − x    1 − cos  
 2    2 

  π − 2x   π − 2x  
 2sin   cos     π x  π x
= tan−1   4  =  4   tan−1 cot  π − 2x   −1   π π − 2x  
tan−1 tan  +   = +
   =    tan tan  −  =
 π − 2x 
2sin2    4    2 4    4 2  4 2
  
  4  

 1 
Illustration 20: If sin  sin−1 + cos−1 x  =1, find the value of x. (JEE MAIN)
 5 
 1  π
Sol: From the question, we have  sin−1 + cos−1 x  = and proceed.
 5  2
 1 
We have sin  sin−1 + cos−1 x  =1
 5 
2 0 . 1 6 | Inverse Trigonometric Functions

1 1 π
⇒ sin−1 + cos−1 x =
sin−1 1 ⇒ sin−1 + cos−1 x =
5 5 2
π 1 1 1
⇒ cos−1 x = − sin−1 ⇒ cos−1 x = cos−1 ⇒ x=
2 5 5 5

Illustration 21: Find the value of cos (sec−1 x + cosec−1 x), x ≥ 1. (JEE MAIN)
−1 −1 π
Sol: Use sec x + cosec x =
2

(
We have cos sec−1 x + cosec−1 x = )
π
cos   =
2
0

( ) + (cosec x ) .
2 2
Illustration 22: Find maximum & minimum values of sec−1 x −1
(JEE ADVANCED)
π
Sol: Apply the identity sec−1 x + cosec−1 x = and then use suitable substitution to form a quadratic.
2
=y (sec−1 x)2 + (cosec−1 x)2

( sec−1 x + cosec−1 x )2 − 2sec−1 x cosec−1 x


=
π
put t = sec−1 x ; sec−1 x + cosec−1 x =
2
π2 π  2 π2
y
= − 2t  − t=
 2t − πt +
4 2  4

 π π2   π
2
π2  π2  π
2
π2 5π2
y = 2 t2 − t +  = 2  t −  +  = + 2  t −  ∴ ymin = ; ymax = at t =
π
 2 8   4  16  8  4 8 4
 

Illustration 23: Find the range of f(x) = sin−1 x + tan−1 x + sec−1 x . (JEE MAIN)

Sol: Find the domain of the given function and then find the range.
f(x) =sin−1 x + tan−1 x + sec−1 x
Here domain is only x = 1 or −1;
So range will contain only 2 elements {3π / 4, π / 4}

π
Illustration 24: Find the number of solutions of the equation tan−1 x3 + cot −1 (ex ) = . (JEE ADVANCED)
2 
π
Sol: Use tan−1 A + cot −1 A = to simplify the given equation and then take
2
the help of graph to find the number of solution. y=1
π x 3 3 −x
cot −1 (ex ) = − tan−1 (x3 ) =
cot −1 (x3 ) ⇒ e =x ⇒ x e =1 O 3
2
3 −x
Plotting the graph of y = l and y = x e we can see that the line intersects Figure 20.21
the curve at two points. Hence there are 2 solutions for the above equation.
M a them a ti cs | 20.17

Illustration 25: Find the number of values of x satisfying the equation

 x3 x5   x 2 x3  π
tan−1  x − + −   + cot −1  x + + − 
=  for 0 < x < 2.  (JEE ADVANCED)
 4 16   2 4  2
   
π
Sol: Use tan−1 A + cot −1 A = .
2
x3 x5 x 2 x3
We must have x − + −  =x + + + 
4 16 2 4

x x 4x 2x
⇒ = ⇒ = ⇒ 2x2 (x + =
2) 0
x 2 x 4 + x 2 2−x
1+ 1−
4 2

∴ x = 0, − 2 (As 0 < x < 2)

Clearly no value of x satisfies given equation.

2 7 1
Illustration 26: Prove that tan−1 + tan−1 tan−1 
= (JEE MAIN)
11 24 2

 x+y 
Sol: Use the formula tan−1 x + tan−1 y =
tan−1  
 1 − xy 

 2 7   tan−1 x + tan−1 y 
 + 
−1 2 −1 7 −1  11 24   
We have, = tan + tan = tan    −1  x + y  
11 24 = 1 − 2 x 7   tan  1 − xy  If xy < 1

 
11 24     

 48 + 77  −1  125  1
= tan−1 =
  tan
=   tan−1  
 264 − 14   250  2

Illustration 27: If tan−1 4 + tan−1 5 =cot −1 (λ ) then find λ.  (JEE MAIN)

Sol: Write the L.H.S. in terms of cot-1 and compare.

4 +5 9 19
We have tan−1 4 + tan−1 5 = tan−1 = π − tan−1 = π − cot −1
1 − 20 19 9
 19  19
= cot −1  −  ⇒λ=−
 9  9

 
1−x 1−y y−x
Illustration 28: Prove that: tan−1 − tan−1 sin−1 
=   (JEE MAIN)
1+x 1+y  1 + x2 1 + y 2 
 

 x+y 
Sol: Use the formula tan−1  = −1 −1
 tan x + tan y .
 1 − xy 

1−x 1−y
We have,
= LHS tan−1 − tan−1 = (tan−1 1 − tan−1 x) − (tan−1 1 − tan−1 y) = tan−1 y − tan−1 x
1+x 1+y
2 0 . 1 8 | Inverse Trigonometric Functions

   
−1  y − x  −1  y−x  sin−1  y−x 
tan
=   sin = =   RHS
 1 + yx   2 2 
 (1 + yx) + (y − x)   (1 + x2 )(1 + y 2 ) 

1 1 2 3 3 8 π
Illustration 29: Prove that: (i) tan−1 + tan−1 tan−1 (ii) tan−1 + tan−1 − tan−1
= =
7 13 9 4 5 19 4
1 1 1 1 π
(iii) tan−1 + tan−1 + tan−1 + tan−1 =  (JEE ADVANCED)
5 7 3 8 4

Sol: Same as above.

1 1
(i)
= LHS tan−1 + tan−1
7 13

 1 1 
−1
 7 + 13   −1  x+y   1  20  −1 2
= tan    tan x=+ tan−1 y tan−1   if xy
= < 1  tan−=   tan= R.H.S.
1 − 1 x 1    1 − xy    90  9
 7 13 
3 3 8
(ii) L.H.S. = tan−1 + tan−1 − tan−1
4 5 19
 3 3 
 + 
 3 3 8 8
=  tan−1 + tan−1  − tan−1 = tan−1  4 5  − tan−1
 4 5 19 1 − 3 x 3  19
 
 4 5
 27 8 
 −  425 π
 27  −1 8
−1
= tan   − tan = tan  11 19 = tan−1
−1
= tan−1 1= = R.H.S.
 11  19  1 + 27 x 8  425 4
 
 11 19 

1 1 1 1
(iii) L.H.S. = tan−1 + tan−1 + tan−1 + tan−1
5 7 3 8
 1 1   1 1 
 +   + 
 1 1  1 1 5 7
=  tan−1 + tan−1  +  tan−1 + tan−1  = tan −1
 + tan  3 8 
−1

 5 7   3 8 1 − 1 x 1  1 − 1 x 1 
   
 5 7  3 8
 6 11 
 + 
6 11  325  π
tan−1
= + tan−1 tan  17 23  =
= −1
tan−1  −1
 = tan = = R.H.S.
17 23 1 − 6 11   325  4
 x 
 17 23 

1 2 3
Illustration 30: Show that tan−1 + tan−1 tan−1 .
= (JEE MAIN)
2 11 4

1 2
+
−1 1 2 2 11 =tan−1 15 =tan−1 3
Sol: We have, L.H.S. =tan + tan−1 =tan−1 =R.H.S.
2 11 1 2 20 4
1− x
2 11

 acos x − bsinx  a
Illustration 31: Simplify tan−1   if tanx > − 1.  (JEE MAIN)
 bcos x + asinx  b
M a them a ti cs | 20.19

Sol: Divide the numerator and denominator inside the bracket by bcos x and expand.

 acos x − b sinx   a 

−1 acos x − b sinx   b cos x   b − tanx  a a
We have, tan
=   tan
= −1   tan 
−1
 =tan−1 − tan−1 (tanx) =
tan−1 −x
 b cos x + asinx   b cos x − asinx  a
 1 + tanx  b b
   b 
 b cos x 

Illustration 32: Solve the following equations:

x −1 x +1 π
(i) tan−1 + tan−1 = (ii) 2tan−1 (cos x) = tan−1 (2cosec x)  (JEE ADVANCED)
x−2 x + 2 4

π
Sol: Write as tan-1 1 and simplify.
4

x −1 x +1 π
(i) tan−1 + tan−1 =
x−2 x+2 4

x −1 x +1 x −1 x +1
⇒ tan−1 + tan−1 tan−1 1 ⇒ tan−1
= tan−1 1 − tan−1
=
x−2 x+2 x−2 x+2
 x +1 
x −1 1 −  1 x −1 x + 2 − x −1
−1
⇒ tan= tan  −1 x + 2  ⇒ tan−= tan−1
x−2 1 + x + 1  x − 2 x + 2+ x +1
 
 x+2
−1 x − 1 −1 1 x −1 1
⇒ tan =tan ⇒ = ⇒ (2 x + 3)(x − 1) =x − 2
x−2 2x + 3 x − 2 2x + 3
1
⇒ 2x2 + x − 3 =x − 2 ⇒ 2x2 − 1 =0 ⇒ x =±
2

(ii) 2tan−1 (cos x) = tan−1 (2cosec x)

 2cos x 
⇒ tan−1  tan−1 (2cosec x)
=
2 
 1 − cos x 
2cos x π
⇒ = 2cosec x ⇒ cos x = sinx ⇒ tanx = 1 ⇒ x =
2
sin x 4

3 8 84
Illustration 33: Prove that: sin−1 − sin−1 cos−1
=  (JEE MAIN)
5 17 85

Sol: Covert the L.H.S. in terms of cos-1.

3 −1 8 4 15  3 −1 4 8 15 
We have sin−1 − sin= cos−1 − cos−1 =  sin−1 cos
= & sin−1 cos−1 
5 17 5 17  5 5 17 17 

 2 2
 4 15 4  15   −1  4 15 3 8 −1  60 24  −1 84
= cos−1  x + 1−  x 1− =   cos  5 x 17 + 5 x 17
=  cos  + =  cos
 5 17 5  17      85 85  85
 

4 5 16 π
Illustration 34: Prove that: sin−1 + sin−1 + sin−1 =  (JEE MAIN)
5 13 65 2
2 0 . 2 0 | Inverse Trigonometric Functions

−1−1 4 5 16  −1 4 5 16
Sol: We havesin
sin − sin−1 + sin−1 =sin + sin−1  + sin−1
5 13 65  5 13  65

 2 2
4  5  5 4  16
= sin−1  1 −   + 1 −    + sin−1
 5 13
  13 5
   25
 

 4 12 5 3  −1 16 63 16
= sin−1  x + =
x  + sin sin−1 + sin−1
 5 13 13 5  25 65 25

 2 
16 −1 16  −1 63  63  16
= cos −1
+ sin  sin = cos −1
1 −   = cos−1 
65 25  65  65  65 
 

π  −1 −1 π
= sin x + cos = x 
2  2

6. SIMPLIFICATION OF INVERSE FUNCTIONS BY ELEMENTARY


SUBSTITUTION

2sin−1 x
(a) = sin−1 (2 x 1 − x2 ) if − 1 ≤ x ≤ 1

−1
(b) 2cos
= x cos−1 (2 x2 − 1) if − 1 ≤ x ≤ 1


 tan−1  2x  − 1 ≤ x ≤ 1
  1 − x2 

  2x 
(c) 2 tan−1 x  sin−1 
= 0≤ x ≤1
2 
 1 + x 
  2 
cos−1  1 − x  0≤x<∞
  2 
 1 + x 

 2 tan−1 x − 1 ≤ x ≤ 1

2x
(d) sin−1 =  π − 2 tan−1 x x ≥ 1
2
1+x  −1
−π − 2 tan x x ≤ −1

1 − x2  2 tan−1 x x≥0
(e) cos−1 =
1 + x2 − −1
 2 tan x x<0

 π + 2 tan−1 x x < −1

−1 2x −1
(f) tan= 2 tan x − 1 < x < 1
2
1−x  −1
 2 tan x − π x >1

 x   1 − x2   1  1
=  1 − x2  tan−1 =
sin−1 x cos−1=   cot −1 =
  sec−1 =
  cosec−1  
(g)  
   2   x   2  x
 1−x     1−x 
M a them a ti cs | 20.21

 1 − x2   x  1  1 
1
(h) cos−= x sin−1  1 − x=
2  −1
 tan  =  cot −1  =  sec−1 = cosec−1  
   x   2  x  2 
   1−x   1−x 

 x   1  1  1 + x2 
(i) −1
tan= x sin−1  =  cos−1  =  cot −1 = sec−1  1 + =
x2  cosec−1  
 2   2  x    x 
 1+x   1+x   

2x
(j) sin−1
f(x) = + 2 tan−1 x =
π, if x ≥ 1
1 + x2

2x
(k) f(x) = sin−1 + 2 tan−1 x = − π if x ≤ − 1
2
1+x

−( π + 3sin−1 x) − 1 ≤ x ≤ 1 / 2

(l) sin−1=
(3 x − 4 x3 )  3 sin−1 x − 1 / 2 ≤ x ≤ 1 / 2
 −1
 π − 3sin x 1/2≤ x ≤1

 3cos−1 x − 2π − 1 ≤ x ≤ −1 / 2

(m) cos−1 (4 x3 − 3 x)= 2π − 3cos−1 x − 1 / 2 ≤ x ≤ 1 / 2
 −1
 3cos x 1/2≤ x ≤1

 −1 1 1
 3tan x − <x
 3 3
3 
3x − x  1
(n) tan−1 = −π + 3tan−1 x x>
2
1 − 3x  3
 −1 1
 π + 3tan x x<−

 3

PLANCESS CONCEPTS

While writing inverse trigonometric functions in their simplest forms, we use the following substitutions.

• For a2 − x2 , we substitute x =a sin θ or x = a cos θ

• For a2 + x2 , we substitute x =a tan θ or x = a cot θ

• For x2 − a2 , we substitute x = a sec θ or=


x a cosec θ

• For a + x and a − x occurring together or separately, we substitute x = a cos θ

Rohit Kumar (JEE 2012 AIR 78)

( (
Illustration 35: Solve for x: sin 2cos−1 cot(2 tan−1 x )) = 0  (JEE ADVANCED)

(
Sol: The R.H.S. is equal to zero implies cos−1 cot(2 tan−1 x) = ) nπ
2
and proceed accordingly to find the value of x.
2 0 . 2 2 | Inverse Trigonometric Functions

 0 if n = 0
 1
cos−1 ( −1
) nπ
cot(2 tan x) =
2
π
=

2
if n = −1 
1 ⇒ cot 2 tan x =
0
−1
( )
π if n = 2 

 π  nπ π  π 3π
nπ +  +  ,−
 4 2 8 8 8
−1  π  nπ π π π
⇒ 2 tan x= nπ + ⇒ tan−1 x=  + ⇒ tan−1 x=  ,−
 2  2 4 4 4
 π  nπ π  π 3π
nπ − 4  2 −8 − 8 , 8
  

⇒ x=
±1, ± ( 2 − 1), ± ( 2 + 1)

Illustration 36: Solve the system of inequalities involving inverse circular functions arc tan2 x − 3 arc tanx + 2 > 0
and [sin−1 x] > [cos−1 x] where [ ] denotes the greatest integer function. (JEE ADVANCED)

Sol: Substitute tan-1 x equal to t.

⇒ ( t − 2) ( t − 1) > 0

⇒ t > 2 or t > 1

⇒ tan−1 x > 2 or tan−1 x > 1

x ∈ ( −∞ , tan 1 ) x > tan1

Again [sin−1 x] > [cos−1 x]

[sin−1 x] can take the values { −2, −1,0,1}

And [cos−1 x] can take the values {0,1,2,3}

Hence [sin−1 x] can be greater than [cos−1 x] only

If [sin−1 x] = 1 and [cos−1 x] = 1

Now, [sin−1 x] = 1 ⇒ 1 ≤ sin−1 x ≤ π / 2 (1 ≤ sin−1 x < 2)

sin1 ≤ x ≤ 1

And [cos−1 x] = 0 ⇒ 0 ≤ cos−1 x < 1

cos1 < x ≤ 1

Now, x must satisfy

From this x ∈ [sin1,1]


M a them a ti cs | 20.23

PROBLEM-SOLVING TACTICS

•• Making a habit of writing angle values in radians rather in degrees makes the calculation of inverse trigonometric
functions easier.

•• Try to remember graphs of inverse trigonometric functions. Sometimes it is easier to approximate answers
using graphical methods.

•• Always verify whether the results are in the range or domain of the respective function.

•• In some cases, constructing a right angled triangle for the given inverse function and then solving using
properties of triangle is much helpful.

•• In case of identities in inverse circular functions, principal values should be taken. As such signs of x, y, etc.,
will determine the quadrant in which the angles will fall. In order to bring the angles of both sides in the same
quadrant, one should make an adjustment by π .

FORMULAE SHEET

1. If y = sin x, then x = sin‒1 y, similarly for other inverse T-functions.

2. Domain and Range of Inverse T-functions:


Function Domain(D) Range (R)

sin−1 x −1 ≤ x ≤ 1 π π
− ≤ θ ≤
2 2

cos−1 x −1 ≤ x ≤ 1 0 ≤ θ ≤ π

tan−1 x −∞ < x < ∞ π π


− < θ <
2 2

cot −1 x −∞ < x < ∞ 0 < θ < π

sec−1 x x ≤ − 1, x ≥ 1 π
0 ≤ θ ≤ π, θ ≠
2

cosec−1 x x ≤ − 1, x ≥ 1 π π
− ≤ θ ≤ , θ ≠ 0
2 2

3. Properties of Inverse T-functions:

π π
(i) sin−1 (sin θ) = θ provided − ≤ θ ≤
2 2

cos−1 (cos θ) = θ provided 0 ≤ θ ≤ π


π π
tan−1 (tan θ) = θ provided − < θ <
2 2

cot −1 (cot θ) = θ provided 0 < θ < π


2 0 . 2 4 | Inverse Trigonometric Functions

π π
sec−1 (sec θ) = θ provided 0 ≤ θ < or <θ≤π
2 2
π π
co sec−1 (co sec θ) = θ provided − ≤ θ < 0 or 0 < θ ≤
2 2

(ii) (sin−1 x)
sin = x provided − 1 ≤ x ≤ 1
(cos−1 x)
cos = x provided − 1 ≤ x ≤ 1
tan
= (tan−1 x) x provided − ∞ < x < ∞
cot
= (cot −1 x) x provided − ∞ < x < ∞
sec
= (sec−1 x) x provided − ∞ < x ≤ −1 or 1 ≤ x < ∞
−1
cosec (cosec
= x) x provided − ∞ < x ≤ −1 or 1 ≤ x < ∞

−1
(iii) sin − sin−1 x,
( − x) = π
(iv) sin−1 x + cos−=
1
x , ∀ x ∈ [ −1, 1]
2
cos−1 ( − x) =π − cos−1 x
π
tan−1 ( − x) =
− tan−1 x tan−1 x + cot −1=
x , ∀ x∈ R
2
cot −1 ( − x) = π − cot −1 x π
sec−1 x + cosec−=
1
x , ∀ x ∈ ( −∞ , −1] ∪ [1, ∞ )
cosec −1
( − x) =− cosec −1
x 2
−1 −1
sec ( − x) = π − sec x

4. Value of one inverse function in terms of another inverse function:

−1 x 1 − x2
(i) sin= x cos−1 1 −=
x2 tan−1 = cot −1
1 − x2 x

1 1
=
sec−1= cosec−1 , 0 ≤ x ≤ 1
1 − x2 x

1 1 − x2 x
(ii) cos−= x sin−1 1 −=
x2 tan−1 = cot −1
x 1 − x2
−1 1 1
= sec = cosec−1 , 0 ≤ x ≤ 1
x 1 − x2
x −1 1 1 1 + x2
=
(iii) tan−1 x sin
= −1
cos
= −1
cot −1= sec x2
1 += cosec−1 , x ≥ 0
1 + x2 1 + x2 x x

1
sin−1  
(iv) = cosec−1 x, ∀ x ∈ ( −∞ ,1] ∪ [1, ∞ )
x

−1  1 
(v) cos
=   sec−1 x, ∀ x ∈ ( −∞ ,1] ∪ [1, ∞ )
x

1  cot −1 x for x > 0


(vi) tan−1   = 
x −π + cot
−1
x for x < 0
M a them a ti cs | 20.25

5. Formulae for sum and difference of inverse trigonometric function:

−1  x+y 
(i) tan= x + tan−1 y tan−1   ; if x > 0, y > 0, xy < 1
 1 − xy 
 x+y 
(ii) tan−1 x + tan−1 y = π + tan−1   ; if x > 0, y > 0, xy > 1
 1 − xy 

−1 −1 −1  x+y 
=
(iii) tan x − tan y tan   ; if xy > − 1
 1 + xy 

−1  x−y 
(iv) tan x − tan−1 y = π + tan−1   ; if x > 0, y < 0, xy < − 1
 1 + xy 
 x + y + z − xyz 
(v) tan−1 x + tan−1 y + tan−1 z =
tan−1  
 1 − xy − yz − zx 

(vi) sin
−1
x ± sin
= −1
y sin−1  x 1 − y 2 ± y 1 − x2  ;
 
2 2
If x, y, ≥ 0 & x + y ≤ 1

(vii) sin
−1
x ± sin−1 y = π − sin−1  x 1 − y 2 ± y 1 − x2  ;
 
2 2
If x, y, ≥ 0 & x + y > 1

(viii) cos
−1
x ± cos
= −1
y cos−1  xy  1 − x2 1 − y2  ;
 
2 2
If x, y, > 0 & x + y ≤ 1

(ix) cos
−1
x ± cos−1 y = π − cos−1  xy  1 − x2 1 − y2  ;
 
2 2
If x, y, > 0 & x + y > 1

6. Inverse trigonometric ratios of multiple angles

(i) 2sin−1 x sin−1 (2 x 1 − x2 ), if − 1 ≤ x ≤ 1


=

−1
(ii) 2cos
= x cos−1 (2 x2 − 1), if − 1 ≤ x ≤ 1

−1 −1  2x  −1  2x  −1
 1 − x2 
=
(iii) 2 tan x tan
=   sin
=   cos  
 1 − x2   1 + x2   1 + x2 
 
−1 −1
(iv) 3sin
= x sin (3x − 4x3 )

−1 −1
3cos x cos
(v) = (4 x3 − 3x)

−1
 3x − x3 
(vi) 3tan x = tan−1  
 1 − 3x2 
 
2 0 . 2 6 | Inverse Trigonometric Functions

Solved Examples

JEE Main/Boards Example 4: Prove that sin(2sin


=−1
x) 2 x 1 − x2

Example 1: Evaluate the following Sol: Use substitution.


 3
(a) tan−1 ( −1) (b) cot −1 ( −1) (c) sin−1  −  Let 2 sin−1 x = θ, where θ ∈ [ −π, π] ;
 2 
  θ
then x = sin
Sol: Do it yourself. 2
θ θ
−π  π ∴ sin(2sin−1 x)= sin θ= 2 sin cos
(a) tan−1 ( −1) = as tan  −  =−1 2 2
4  4
θ θ
3π  3π  2sin
= 1 − sin2 2x 1 − x2
=
(b) cot −1 ( −1) = as cot   = −1 2 2
4  4 

 3 π  π 3 Example 5: Find the angle


(c) sin−1  − =− as sin  −  =

 2  3  3 2  3π 
  (a) tan−1  tan  ; (b) sin−1 (sin5)
 4 
  2π  
Example 2: Find the angle sin−1  sin    . Sol: (a) Write

as π −
π
  3  4 4
2π π (b) Write 5 as 5 - 2 π
Sol: Write the angle as π − and proceed.
3 3
 3π  −1   π 
(a) tan−1=
 tan  tan  tan  π −  
  2π   2π  2π  π π    4    4 
sin−1  sin    ≠  as ∉ − ,  
  3  3  3  2 2
 π  π π
= tan−1  − tan  =
− tan−1  tan  =

  2π     π   4  4 4
−1 −1
⇒ sin  sin
=    sin  sin  π − 
  3    3  (b) We know sin−1 (sin θ) =θ

  π  π  π π
⇒ sin−1  sin    =
. θ ∈  − ,  =  −1.57,1.57 
  3  3  2 2
 π π  π π
5 ∉  − ,  , while 5 − 2π ∈  − , 
Example 3: Find the value of  2 2  2 2
1
cos [2sin−1 x + cos−1 x] at x = . sin5
= sin(5 − 2π + 2=
π) sin (5 − 2π)
5
∴ sin−1 sin 5 = 5 − 2π
π
Sol: Use cos−1 x + sin−1 x =
2
−1 −1
Example 6: If cos−1 x + cos−1 y + cos−1 z =
π prove that
cos[2sin x + cos x]
x2 + y 2 + z 2 + 2xyz =
1.
= cos[cos−1 x + sin−1 x + sin−1 x]
Sol: Take one of the term to the R.H.S. and take cosine
π  on both sides.
cos  + sin−1 x  =
= − sin(sin−1 (x))
2  cos−1 x + cos−1 y + cos−1 z =
π
1
=− x =− cos−1 x + cos−1 y = π − cos−1 z
5
M a them a ti cs | 20.27

Taking cosine on both sides we get Example 9: Solve cos−1 x + cos−1 y = and tan−1 x − tan−1 y =
π / 2 and 0

xy − 1 − x2 1 − y 2 =
−z cos−1 x + cos−1 y =
π / 2 and tan−1 x − tan−1 y =
0

⇒ xy + z = 1 − x2 1 − y 2 Sol: From the second of the given equations, we have

Squaring we get x = y. ⇒ tan−1 − tan−1 y = 0 ⇒ x = y

x2 y 2 + z 2 + 2xyz =1 − x2 − y 2 + x2 y 2 Substituting x = y in the first, we have


⇒ x2 + y 2 + z 2 + 2xyz =
1. 2cos−1 x =
π / 2 or cos−1 x =
π/4
2x or x = cos π / 4 = 1 / 2 = y
Example 7: Prove that
= f(x) 2 tan−1 x + sin−1 is a
2
1+x It is clearly evident that these values satisfy the given
constant for all x ≥ 1. Find this constant. equations. Hence the solution set of the given equations

Sol: Convert the sin-1 function on the R.H.S. to tan-1 and is ( x = 1 / 2 , y = 1 / 2 )


proceed.
x y
2x Example 10: If cos−1 + cos−1 =θ prove that
If 0 ≤ x ≤ 1 then sin −1 −1
= 2 tan x. 2 3
1 + x2
9x22 − 12xy cos θ +=
4y 22 36 sin22 θ.
2x
Hence If x ≥ 1(0 < 1 / x ≤ 1) then sin−1
1 + x2
Sol: Do it yourself.
 2 
  x y
sin
= −1  x  2 tan−1 1 For x ≥ 1 Let cos−1 α and cos−1
= =
β
 1  x 2 3
1 + 2  x y
 x  ∴ cos α
= and
= cos β
2 3
2x 1
=f(x) 2 tan−1 x + sin−1= 2 tan−1 x + 2 tan−1 Given α + β =θ ∴ cos(α + β) = cos θ
1+x 2 x
or cos α cos β − sin α sin β
= cos θ
= 2[tan−1 x + cot −1 x] = 2.π / 2 = π = cons tant.
x y x2 y2
or . − 1− . 1− = cos θ
2 3 4 9
Example 8: Solve the equation: 2 tan−1 (cos x)
= tan−1 (2cosec x).
xy 4 − x2 9 − y 2
or, − cos θ
=
Sol: Substitute a variable in place of tan−1 (cos x) and 6 6
take tan on both sides.
or (xy − 6cos θ)2 = (4 − x2 )(9 − y 2 )
−1 −1
If 2tan (cos x) tan (2cosec x):sinx ≠ 0
=
−1
or, x2 y 2 + 36cos2 θ − 12xy cos=
θ 36 − 9x2 − 4y 2 + x2 y 2
tan[2tan (cos x)] = 2cosec x ….(i)
 π π or, 9x2 − 12xy cos θ + 4=
y 2 36(1 − cos θ2 )
Assume tan−1 (cos x) = θ : θ ∈  − , 
 4 4
tan[2tan−1 (cos x)]
∴ L.H.S. = JEE Advanced/Boards
2 tan θ 2cos x 2cos x x y
= tan2
=θ = =
2 2
Example 1: If cos−1 + cos−1 =α
1 − tan θ 1 − cos x sin2 x a b
x2 2xy y2
Substituting this value (i) we get prove that − cos α + = sin2 α.
a2 ab b2
2cos x
= 2cosec
= x; cos x sinx;
= tanx 1;
sin2 x Sol: In the given equation take cosine on both sides
and proceed.
π
∴ x = nπ + ; n∈I
4
2 0 . 2 8 | Inverse Trigonometric Functions

x y
cos−1 + cos−1 =
α  sec θ − 1  θ
a b = tan−1 
R.H.S. 2= −1
 2 tan tan
 tan θ  2
 x  y  x  −1 y 
cos  cos−1  cos  cos−1  − sin  cos−1  sin  cos  θ
 a  b  a  b  = 2.= 1
tan−= x L.H.S.
2
= cos α

xy x2 y2 Example 3: Prove that


⇒ − 1− 1− = cos α
ab a2 b2 1 
tan−1  tan2A  + tan−1 (cot A) + tan−1 (cot3 A)
 2 
xy x2 y2
⇒ − cos=
α 1− 1−
ab a2 b2  0 if
π
<A<
π
= 4 2
π
Squaring both sides  π if 0 < A <
4
2
 xy   x2  y2  π π
 − cos α  =  1 −  1 − 2  Sol: Divide the solution in two cases when <A<
 ab   a2  b  4 2
   π
and 0 < A < and use the definition accordingly.
2 2
x y 2xy x y 2
x y 2 2 2 4
or + cos2 α − cos α = 1 − − +
2 2
ab ab a 2
b 2
a2b2 π π
Case I: <A<
4 2
x2 y2 2xy
or + − cos α = 1 − cos2 α
a2
b 2 ab 0 < cot A < 1 and 0 < cot3 A < 1

x2 2xy y2 ∴ tan−1 (cotA) + tan−1 (cot3 A)


or − cos α + = sin2 α
a2 ab b2  cot A + cot3 A  −1  sin2A 
= tan−1  =  tan  − 
 1 − cot 4
A   2cos2A 
Example 2: Prove

tan−1 x = 2tan−1 (cosec tan−1 x − tancot −1 x).  sin2A  −1  1 


= tan−1  −  = − tan  tan2A 
 2cos2A   2 
Sol: Use substitution. 1  −1 3
−1 −1 −1
⇒ tan−1  tan2A  + tan−1 (cotA) + tan (cot A) =
0
=R.H.S. 2 tan (cosec tan x − tancot x) 2 
  2  π
cosec cosec−1  1 + x   or Case II: 0 < A <
4
  x 
= 2 tan−1   
  cot A > 1 and cot3 A > 1 ⇒ cot A.cot3 A > 1
−1  1 
 − tan tan   
 x  Hence, tan−1 (cot A) + tan−1 (cot3 A) =

 cot A + cot3 A 
  1 + x2   π + tan−1  
2 tan−1 cosec cosec−1   − tan π + tan−1 1   1 − cot 4 A 
 
  x   x 
     x+y 
[As tan−1 x + tan−1 y = π + tan−1   If
depending on x > 0 or x < 0  1 − xy 
x > 0. y > 0 and xy > 1]
 1 + x2 1   1 + x2 − 1  1 
= 2 tan−1  = −  2 tan−1   = π − tan−1  tan2A  [From case 1]
 x

x



x 
 2 
1  −1 3
 π π tan−1  tan2A  + tan−1 (cotA) + tan (cot A) =
π
Let tan−1 x = θ : θ∈ − ,  ; thenx = tan θ 2 
 2 2
M a them a ti cs | 20.29

Example 4: Find the sum ∴ tan−1 X1 + tan−1 X 2 + tan−1 X3 + tan−1 X 4 =


−1 −1 −1
cot 2 + cot 8 + cot 18 + …………….. to Infinity.
π
nπ + − β for some n ∈ I.
Sol: Write the general term of the series and express it 2
as a difference of two terms (telescopic series).
Example 6: Find the value of
Let Tn denote the nth term of the series
 
−1 2 −1
 4r 2   π x 
∴=Tr cot (2r
= ) cot   sin  sin 
-1

 2 
  

3  ( 2 2
x + k − kx 
 )
 1 + 4r − 1  2
 1 + (2r + 1)(2r − 1) 
= cot −1   = cot −1   
    π x 
 2   (2r + 1) − (2r − 1)  −1
− cos  cos  
 (2r + 1) − (2r − 1) 


6  ( 2 2
x + k − kx 
 )
= tan−1  
 1 + (2r + 1) (2r − 1)  k 
Where  < x < 2k,k > 0 
 2 
= tan−1 (2r + 1) − tan−1 (2r − 1)

∴ cot −1 2 + cot −1 8 + cot −1 18 +  + Sol: We have

cot −1=
2n2 tan−1 (2n+ 1) − tan−1 (1) As  
 π x 
n → ∞ tan −1
(2n+ 1) → π / 2 Sin  sin 
-1

π π π


3
( x2 + k 2 − kx 
 )
Hencerequired
Hence, requiredsum
sum== − = .
2 4 4  
−1  π x 
− cos  cos  
Example 5: If X1, X2, X3 , X 4 are the roots of the equation 

6  ( 2 2
x + k − kx 
 )
4 3 2
X − X sin2β + X cos2β − X cos β − sin β = 0
 3x   3x 
1 = sin−1   − cos
−1
where sin β ≠ prove that tan−1 X1 + tan−1 2 2

2 2

2  2 x + k − kx   2 x + k − kx 
π
X 2 + tan−1 X3 + tan−1 X 4 = nπ + − β for some n ∈ I.  
2
π −1  3x 
= -2 cos  
Sol: Use theory of equations. 2 

( 4x 2
− 4kx + 4k 2 ) 

X1, X2, X3, X4 are the roots of the given equation
π  6x2 
∴ ∑ X1 =sin2β, ∑ X1 X 2 =cos2β = − cos−1  2 − 1
2  4x − 4kx + 4k
2

∑ X1, X2, X3 = cosβ , X1, X2, X3 , X 4 = − sin β
 2X 2 + 4kx − 4k 2   X 2 + 2xk − 2k 2 
tan tan−1 X1 + tan−1 X 2 + tan−1 X3 + tan−1 X 4  = sin−1   = sin−1  
   4X 2 − 4kx + 4k 2   2X 2 − 2xk + 2k 2 
   
∑ X 1 − ∑ X1 X 2 X 3 sin 2 β − cos β
= =
1 − ∑ X1 X 2 + X1 X 2 X3 X 4 1 − cos2 β − sin β Example 7: Find the number of real solutions of the

cos β ( 2sin β − 1 ) 1 + cos2x


equation= 2 sin−1 ( sinx ) − π ≤ x ≤ π
= cos β
sin β ( 2sin β − 1 )
Sol: Divide the solution into three cases when
tan tan−1 X1 + tan−1 X 2 + tan−1 X3 + tan−1 X 4  π π π π
  − ≤ X ≤ , < X ≤ π and −π ≤ X < − and proceed.
2 2 2 2
π 
==cos β tan  − β 
2  Here cos x = sin−1 ( sinx ) .
2 0 . 3 0 | Inverse Trigonometric Functions

π π
If − ≤ X ≤ , then cos-1cos x= x
2 2
( ) ≤ π / 4 sin y ≠ 0 [From ii]
2
So 0 < sin−1 y 2 −1
In the case there is one solution obtained graphically.
x + ( sin y ) ≤ π + π / 4
2
π ∴ 0 < cos−1 −1 2
{
If < X ≤ π then − cos x = sin−1 sin ( π − x ) = π − x
2
}
pπ π2
∴ cos x = x − π i.e. 0 < ≤ π+ .…(iii)
4 4 
In the case there is one solution obtained graphically. From (i) and (ii) we get p ≤ 0
π π4 pπ2
If −π ≤ X < − then cos−1 x + =
2 4
16 cos−1 x
{
− cos x = sin−1 sin ( −π − x ) = −x − π }
( )
2
Or 16 cos−1 x − 4pπ2 cos−1 x + π4 =0 ….(iv)

i.e. cos x = x + π
As cos−1 x is real 16 p2 π2 ≥ 0
This gives no solution as can be seen from their graphs.
Or p2 ≥ 4 i.e. p ≤ −2  ….(v)
Example 8: Find the integral values of p at which the From (iii) and (v)

( )
2
−1 −1 2
system of equations cos x + sin y pπ / 4; and
= p2 ≤ ( π / 4 ) + 1, p ≥ 20
(cos x )(sin y )
2
−1 −1
= π2 / 16 possess solutions. Also p is integer so p = 2 for p = 2 (4) gives
find these solutions.
( )
2
16 cos−1 x − 8π2 cos−1 x + π4 =0 or
Sol: Start with the range of cos-1 x and sin-1 y and use it 2
 4 cos−1 x − π2  = 0 or
in the two given equations.  
The given system of the equation is cos−1 x − π2 / 4 i.e.=
x cos π2 / 4 ( ) ….(vii)

( )
2
cos−1 x + sin−1 y pπ2 / 4
= ….(i)
( )
2
 Then (ii) gives sin−1 y = π2 / 4

(cos x )( sin y )
2
−1 −1
= π4 / 16 ….(ii) Or sin−1 y = ±π2 / 2 i.e. ± 1 ….(viii)

It is clear that 0 < cos−1 x ≤ π Hence p=2 and (x,y)= cos π2 / 4 , ±1
  ( )
0 < cos−1 x ≤ π ; −π / 2 ≤ sin−1 y ≤ π / 2.
−π / 2 ≤ sin−1 y ≤ π / 2.

JEE Main/Boards

Exercise 1 
−1
 3  π 
Q.5 Evaluate: cos cos  − +
 2  6
   
Q.1 Evaluate: sin−1 ( sin π / 4 )
Q.6 Evaluate: sin cos−1 3 / 5( )
Q.2 Evaluate: tan −1
( tan ( −6 ) )
π  1  1 1 2
Q.3 Evaluate: sin  − sin−1    Q.7 Prove that: tan−1 + tan−1 tan−1
=
7 13 9
2  2 
1 1 1 π

Q.4 Prove that: tan−1 2 + tan−1 3 = Q.8 Prove that: 4 tan−1 + tan−1 + tan−1 =
4 5 70 99 4
M a them a ti cs | 20.31

5π2 12 4 63
( ) ( )
2 2
Q.9 Solve for x: tan−1 x + cot −1 x = Q.22 Prove that: sin−1 + cos−1 + tan−1 =
π
8 13 5 16

π
Q.23 Solve tan−1 2x + tan−1 3x =
Q.10 Solve for x: tan−1 ( x + 1 ) + tan−1 x + tan−1 4
( x − 1) =
tan−1 3
1 1 31
Q24 Prove that: 2 tan−1 + tan−1 =tan−1
2 7 17
Q.11 Find the value of
x y
Q25 If cos−1   + cos−1   =
θ prove that
1 2x 1 − y2  a b
tan−1 sin−1 + cos−1 
2  1 + x2 1 + y 2 
x2 2xy y2
− cos=
θ = sin2 θ
a2 ab b2
2 7 1
Q.12 Prove that: tan−1 + tan−1 tan−1
=
11 24 2
Q.26 Find the value of the following:
 1 − cos x   1  1
Q.13 Differentiate tan−1   w.r.t.x. tan−1 (1) + cos−1  −  + sin−1  − 
2  2
 sin x   

 1 − sinx  π π 9π 9  1  9 −1  2
Q.14 Express tan−1   < x < in the simplest Q.27 Prove that: − sin−1   = sin  2 
8 4 3 4  
form.  cos x  2 2  3 

Q.28 The value of


−1  1
Q.15 Find the principle value cos − 
 2  1  1  10 − 1 
cos−1   − cos−1   + cos−1   Is___________.
 3  6  3 2 
     
Q.16 Write the following functions in the simplest
Q.29 The number of roots of the equation
form: cot −1  1 + x2 − x  .
  sinx = cos−1 ( cos x ) is

Q.17 Find the principle value of cot −1 − 3 . ( )


Exercise 2
Q.18 Prove that 3cos
= x cos−1 −1
( 4 × 3 − 3x ) ( − 3 )
1  Single Correct Choice Type
x ∈  ,1 
2 
Q.1 tan cos−1 x is equal to

Q.19 Write the following function in the simplest form:


1 − x2 x
(A) (B)
 cos x − sinx  x 1 + x2
tan−1   ,x < π
 cos x + sinx 
1 + x2
(C) (D) x 1 + x2
x
x −1 x +1 π
Q.20 If tan−1 + tan−1 = then find the value
x−2 x+2 4 2 2
of x. Q.2 sin−1 x + sin−1 y + 2 sin−1 x sin−1 y =
π2

Q.21 Write the following function in the simplest form: then x2 + y 2 is equal to
 3a2 x − x3  (A) 1 (B) 3/2 (C) 2 (D) ½
tan−1  .
 a3 − 3ax2 
 
2 0 . 3 2 | Inverse Trigonometric Functions

Q.3 Number of solution(s) of the equation Q. 9 The function

cot −1 x2 + 3x + 2 + cos−1
π
x2 − 3x − 3 = is f ( x ) cot −1
= ( x + 3) x + cos−1 x2 + 3x + 1
2
is defined on the set S where S=
(A) 2 (B) 1
(A) {0,3} (B) ( 0,3 ) (C) {0, −3} (D) ( −3,0 )
(C) 4 (D) Infinite

( )
Q.4 If cos tan−1 x = x, then x2 is equal to Q.10 α =sin−1 cos sin−1 x ( ( )) and
(A)
5 +1
(B)
5 −1 ( (
β =cos−1 sin cos−1 x )) then
4 2
(A) tan α
= cot β (B) tan α = −cot β
5 +1 5 −1 (C) tan α
= tan β (D) tan α = − tan β
(C) (D)
2 4

Q.5 The value of a for which x + ax + sin 2 −1


1
2
1
Q.11 If x= 2cos−1   + sin−1   + tan−1 3
2
( )
(x 2
) (
− 4x + 5 + cos−1 x2 − 4x + 5 =
0, is ) 1  x 
and y = cos  sin−1  sin   then which of
2  2 
π π the following statements hold good?
(A) +2 (B) +1
4 4
3π 5π
π π (A) y = cos (B) y = cos
(C) − + 1 (D) −( + 2) 16 16
4 4
(C) x = 4 cos−1 y (D) None of these
Q.6 Domain of the function
Q.12 The set values of x satisfying the equation
=f (x) sin−1 ( sin x ) + cos−1 ( cos x ) is
(
tan2 sin−1 x > 1is )
π  2 2
(A) 2nπ,2nπ + ,n ∈ I
2 (A)  −1,1 (B)  − , 
 2 2 
(B) ( 2n + 1 ) π, ( 2n + 2 ) π  ,n ∈ I
 2 2  2 2
(C) ( −1,1) −  − ,  (D  −1,1 −  − , 
 2 2 
(C) 2nπ, ( 2n + 1 ) π  ,n ∈ I  2 2   

π 3π −1 −1
(D) 2nπ + ,2nπ + ,n ∈ I Q.13 The equation sin x = 2 sin a has a solution for
2 2
(A) All real values of a (B) a<-1
−1 −1 −1
Q.7
= If θ sin x + cos x − tan x, x ∈ 0,1 −1 1
(C) a > 1 (D) ≤a≤
Then the interval in which θ lies is given by 2 2

 π
(A) 0, 
 3
π π
(B)  , 
4 2
( )(
Q.14 If sin−1 x + sin−1 w sin−1 y + sin−1 z =
π2 )
N1 N2
x y
 π  π 3π  then ; (N1 ,N2 ,N3 ,N4 E N) −
N3 N4
(C) 0,  (D)  ,  z w
 4 2 4 
(A) Has a maximum value 2.

Q.8 tan−1 2 + tan−1 3 =


cosec−1 x then x is equal to (B) Has a minimum value 2.
(C) Is independent of N1 ,N2 ,N3 ,N4
(A) 4 (B) 2 (C) − 2 (D) None of these
(D) None of these
M a them a ti cs | 20.33

Q.15 =If θ cot −1 7 + cot −1 8 + cot −1 18 then cot θ is Q.22 Number of point(s) where f(x) =sin-1 (3x-4x3) is not
equal to differentiable is

(A) 1 (B) 2 (C) 3 (D) 4 (A) 1 (B) 2 (C) 3 (D) 4

Q.16 Which of the following function(s) is/are periodic? Q.23 Solution of the equation

(A) f(x)=x-[x], [x] denotes integral part of x  1  5 


sec−1=
x cos−1  −  + sin−1   is
(B) g(x)= sin(1/x) x ≠ 0 and g(0) =0  2 3 3 

(C) h(x)=x cosx 18 18


(A) (B)
(D) sin (sin-1x) 3− 6 6−3

6+3
  1  (C) (D) None of these
Q.17 cos  2 tan −1    equals 8
  7 
(A) sin(4cot-13) (B) sin(3 cot-14) Q.24 The value of
−1
(C) cos (3cot-14) (D) cos (4cot-14)   π  π 1 −1  a  
−1  a  
tan  + sin    + tan  − sin   
  4  b    4 2  b  
Q.18 sin−1  2 × 1 − x2  = 2sin 2
sin–1 xx is true if: x ∈ Where (0<a<b) is
 
 1 1  b a
(A) [0,1] (B)  − ,  (A) (B)
 2 2 2a 2b

 1 1  3 3 b2 − a2 b2 − a2
(C)  − ,  (D)  − ,  (C) (D)
2b 2a
 2 2  2 2 

 1 1
k =n
2k 6 x tan−1 1 − cos −1  −  + sin −1 ;
Q.25 If=
Q.19 If the sum ∑ tan −1

2 + k2 + k 4
−1
= tan
7
 2 2
k =1
1  1 
then the value of n is equal to : y= cos  cos −1    then:
2  8 
(A) 2 (B) 3 (C) 4 (D) 5
(A) x = πy (B) y = πx
Q.20 The domain of definition of the function (C) tanx = − ( 4 / 3) y (D) tanx = ( 4 / 3) y
 3x − 7x + 8 
2
f ( x ) = arc cos   where [x]
2 Q.26 Which of the following satisfy the equation?
 1 + x 
denotes the greatest integer function is:  2x2 − 1 
2cos-1 x=cot-1  
 2 2 
(A) (1, 6) (B) [1, 6]  4x − 4x 
(C) [0, 1] (D) (-2, 5) (A) (-1, 0) (B) (0, 1)
 1 1 
Q.21 Consider two geometric progressions (C)  − ,  (D) [-1, 1]
1  2 2
a1 ,a2 ,a3 .......an & b1 ,b2 ,b3 .......bn with a=
r = 2r −1
br
Q.27 Find values of x if sin x =cos x + sin ( 3x − 2 ) ?
−1 −1 −1
and another sequence t1 ,t2 ,t3 ....... tn such that
n
1  1 
=tr cot −1 ( 2ar + br ). Then lim ∑ tr is : (A)  ,1 (B)  ,1
n→∝
r =1 2  2 
(A) 0 (B) π / 4 (C) tan −1 2 (D) π / 2 1  1 
(C)  ,1 (D)  ,1 
3  3 
2 0 . 3 4 | Inverse Trigonometric Functions

1 − x2 Previous Years’ Questions


Q.28 f ( x ) = sin−1 and
2
1+x
 4  2 
g ( x ) cot −1 x − tan−1 x are identical for :
= Q.1 The value of tan cos−1   + tan−1    is  (1983)
 5  3 
(A) x ∈ [0,1] (B) xX∈
Ε ( −∞ ,0]
6 17
(A) (B)
(C) x ∈ [-1, 1] (D) xX∈
Ε ( −∞ , −1]U[1, ∞ ) 17 6
16
(C) (D) None of above
 { (
Q.29 tan cos−1 sin 2 tan−1 2  is equal to

)} 7

4 4 3 3  2π 
(A) (B) (C) (D) Q.2 The principle value of sin−1  sin  is  (1986)
3 5 5 4  3 

2π 2π π 5π
−1 −1 n (A) − (B) (C) (D)

sin x + cos x 3 3 3 3
Q.30 ∑ πr
is finite.
n=1
Q.3 The number of real solutions of
x ∈  −1,1  and r>0.then the possible values of ‘r’ is. π
tan−1 x ( x + 1 ) + sin−1 x2 + x + 1 = is  (1999)
2
1
(A) ,∞ (B) ( 2,∞ ) (A) Zero (B) One
2
(C) Two (D) Infinite
(C) (1,∞ ) (D) ( 0,∞ )
 x 2 x3   x 4 x6 
Q.4 If sin−1  x − + − ...  + cos−1  x 2 − + −
Q.31 y sin ( sinx ) ,xxΕis0the
= −1
π  element of [0 p] divides  2 4   2 4 
  
the region bounded by coordinate axes
π
= , for 0 < x < 2, then x equals  (2001)
π 2
x=
π and y = into 3 region whose areas are
2
(A) 1/2 (B) 1 (C) -1/2 (D) -1
A1 , A2 , A3 with A1 ≤ A2 ≤ A3 then

Q.5 The value of x for which


π2
(A) A1 + A2 + 2A3 =

π2
(B) A1 + A3 − A2 =
sin cos−1 (1 + x )  =
  (
cos tan−1 x is

) (2004)
2
1 1
(A) (B) 1 (C) 0 (D) −
(C) A1 + A2 − A3 =
0 2 2

(D) 2 ( A1 + A2 ) − A3 =
0 Q.6 If 0<x<1, then
1/2

{ ( ) ( )} 
2

4n 1 + x2  x cos cot −1 x + sin cot −1 x − 1 is equal
Q.32 The sum ∑ tan−1 n4 − 2n2 + 2 is equal to:  
n=1 to  (2008)
x
(A) tan−1 2 + tan−1 3 (B) 4 tan−1 1 (A) (B) x
1 + x2
(C)
π
2
(D) sec−1 1 − 2 ( )
(C) x 1 + x2 (D) 1 + x2
M a them a ti cs | 20.35

Q.7 Let a, b, c be positive real number Q.13 For a regular polygon, let r and R be the radii
of the inscribed and the circumscribed circles. A false
a (a + b + c + ) b (a + b + c + ) statement among the following is  (2010)
θ tan −1 + tan−1
bc ca r 1
(A) There is a regular polygon with =
c (a + b + c + ) R 2
+ tan−1 .
ab r 2
(B) There is a regular polygon with =
Then tan θ equal ……… (1981) R 3
r 3
(C) There is a regular polygon with =
Q.8 The numerical value of R 2
 1 π r 1
tan 2 tan−1   −  is equal to....  (1984) (D) There is a regular polygon with =
R 2
 5 4

Q.14 A line AB in three-dimensional space makes angle


Q.9 The greater of the two angle A = 2 tan−1 2 2 − 1 ( ) 45o and 120o with the positive x-axis and the positive
y-axis respectively. If AB makes an acute angle θ with
1 3 the positive z-axis, then θ equals  (2010)
B= 3 sin−1   + sin−1   is …….. (1989)
3 5
(A) 45° (B) 60° (C) 75° (D) 30°

Q.10 AB is a vertical pole with B at the ground level and


A at the top. A man finds that the angle of elevation −1  2x 
Q.15 Let tan= y tan−1 x + tan−1   , where
of the point A from a certain point c on the ground is  1 − x2 
60o. He moves away from the pole along the line BC 1
| x |< . Then the value of y is  (2015)
to a point D such that CD = 7m. From D the angle of 3
elevation of the point A is 45o. Then the height of the
pole is  (2008) 3x − x3 3x + x3
(A) (B)
1 − 3x2 1 − 3x2
7 3 1 7 3
(A) ⋅ m (B) ⋅ ( 3 + 1)m
2 3 −1 2 3x − x3 3x + x3
(C) (D)
1 + 3x2 1 + 3x2
7 3 7 3 1
(C) ⋅ ( 3 − 1)m (D) ⋅
2 2 3 −1 Q.16 If 0 ≤ x < 2π, then the number of real values of
x, which satisfy the equation cosx + cos2x + cos3x +
 5 2 cos4x = 0, is  (2016)
Q.11 The value of cot  cosec−1 + tan−1  is  (2008)
 3 3 (A) 5 (B) 7 (C) 9 (D) 3
6 3 4 5
(A) (B) (C) (D)
17 17 17 17  1 + sinx   π
Q.17 Consider f(x) tan−1 
=  , x ∈  0,  .
 1 − sinx   2
4 5  
Q.12 Let cos(α + β) = and let sin(α − β) = , where π
5 13 y f(x)
A normal to= = also passes through the point:
π  6 (2016)
0 ≤ α , β ≤ , then tan 2α = (2010)
4
 2π  π 
(A)  0,  (B)  , 0 
56 19 20 25  3  6 
(A) (B) (C) (D)
33 12 7 16
π 
(C)  , 0  (D) (0, 0)
4 
2 0 . 3 6 | Inverse Trigonometric Functions

JEE Advanced/Boards

Exercise 1 Q.7 If α and β are the roots of the equation


x2 − 4x + 1= 0 ( α > β ) then find the value of
−1  1 + x 

−1 1 + x
2 
Q.1 If α 2 tan
= =   & β sin   β3 1 β  α3 1 α
1 − x 
2  f ( α , β ) = cosec2  tan2  + sec2  tan2 
1 − x 
2  2 α  2  2 β
For 0<x<1 then prove that α + β = π what is the value
of α + β will be if x>1?
 2x 
Q.8 Consider the functions f ( x ) = sin−1  
 1 + x2 − 1 − x2   1 + x2 
−1
Q.2 If y = tan   prove that  1 − x2   2x 
 1 + x2 + 1 − x2


 g ( x ) = cos−1   and h ( x ) = tan−1  
 1 + x2   1 − x2 
 
x2= sin 2 y.
(i) If x ∈ (-1, 1) then the find the solution of the

Q.3 Find the sum of following series upto n terms Equation f ( x ) + g ( x ) + h ( x ) =


π/2
where x > 0. (ii) Find the value of f(2) +g(2)=h(2).
1 2 2n−1
(i) tan−1 + tan−1 + ..... + tan−1 ...0
3 9 1 + 2n−1+ Q.9 Solve the following inequalities
1 1 (i) arc cot2 x − 5arc cot x + 6 > 0
(ii) tan−1 + tan−1
x2 + x + 1 x2 + 3x + 3 (ii) arc sin x>arc cos x
1 1 (iii) tan2 (arc sin x)>1
+ tan−1 + tan−1
x2 + 5x + 7 x2 + 7x + 13
Q.10 Show that roots r, s and t of the cubic
 1
Q.4 If x ∈  −1, −  then express the function x(x-2)(3x-7)=2 are real and positive.
 2
Also compute the value of tan−1 (r ) + tan−1 (s)tan−1 (t).
f ( x= ( ) ( )
) sin−1 3x − 4x3 + cos−1 4x3 − 3x in the form of a
cos−1 x + bπ where a and b are rational numbers. π  x 
Q.11 Let f ( x ) = + cos−1   − tan−1 x
4  2 
 1+x 
Q.5 Solve the following equations:
and ai ( ai < ai+1∀i =1,2,3,....n) be the
π
(i) sin−1 x + sin−1 2x = positive integral values of x for which
3

(ii) tan−1 ( x − 1 ) + tan−1 ( x ) + tan−1 ( x + 1 ) = tan−1 ( 3x )


( )
sgn f ( x ) = 1, where sgn (y) denotes signum
n

1− x 2x − x 23
function of y. Find ∑ ai2 .
(iii) tan−1 + tan−1 = tan−1 i=1
1+ x 2x + x 36
  2x2 + 4  
−1 x2 − 1 −1 2x 2π Q.12 Solve for x: sin−1  sin    < π − 3.
(iv) cos + tan =   1 + x2  
x2 + 1 2
x −1 3    

Q.6 Find all the positive integral solution of Q.13


= Let f ( x ) tan−1 ( cot x − 2cot 2x ) and
y 3 5
tan−1 x + cos−1 sin−1
1 + y2
=
10 ∑ f (r ) = a − bπ where a, b E N. Find the value of (a+b).
r =1
M a them a ti cs | 20.37

Q.14 Let f ( x ) = ( 2a + b ) cos−1 x+ ( a + 2b ) sin−1 x Q.20 Prove that:

 5   7  36
Where a, b ∈ R and a>b. (i) cos−1   + cos−1  −  + sin−1 =
π
 13   25  325
If domain of and range of f are the same
set then find the value of π ( a − b ) . 2 6 +1 π
(ii) arc cos − arc cos =
3 2 3 6
Q.15 Identify the pairs(s) of functions which are
identical. Also plot the graphs in each case. Q.21 If a>b>c>0 then find the value of:
 ab + 1  −1  bc + 1  −1  ca + 1 
1 − x2 cot −1   + cot   + cot  
(i) y = tan(cos−1 x); y =  a−b   b−c   c−a 
x
1
(ii) y = tan(cot −1 x); y = Q.22 If α and β are the roots of the equation
x
x x2 + 5x − 49 =
0 then find the value of
(iii) y = sin (arc tan x); y=
1 − x2 ( )
cot cot −1 α + cot −1 β .

(iv) y = cos (arc tan x); y= sin (arc cot x)


Q.23 Find all value of k for which there is a triangle

Q.16 Find the domain and the following functions. 1


Whose angles have measure tan−1  
2
(
(i) f(x) cot −1 2x − x2
= )  1   1 
tan−1  + k  andtan−1  + 2k 
2  2 
=(ii) f(x) sec−1 (log tanx + log
3 tan x 3 )
Q.24 In a ∆ABC if ∠A =∠B
 2x2 + 1  
−1   
(iii) f(x) = cos 1  −1  6 + 1   1 
 x2 + 1 
  =  sin   + sin−1    and
2  2 3   3  
     

( (
(iv) f(x) tan−1 log4 5x2 − 8x + 4
= )) 1
C = 6.3 4     then find the area of ∆ABC .

Q.17 Let y = sin−1 ( sin8 ) − tan−1 ( tan 1 ) + cos−1


5
Q.25 Find the simplest value of
(cos 12) ‒ sec ( sec9 ) + cot ( cot 6 ) − cosec
−1 −1
‒ (cosec 7). (i) f (x) =arc cos x +arc
If y simplifies to aπ + b then find (a-b).
x 1  1 
 36  cos  + 3 − 3x2  , x ∈ ,1 
−1  4 
Q.18
= −1
  β cos   and
Let α sin = 2 2  2 
85
  5
 1 + x2 − 1 
 8   , x ∈ R − {0}
γ =tan−1   find ( α + β + γ ) and hence (ii) f(x) = tan−1 
 15   x 
 
Prove that (i) ΣΣcot
cotαα == ∏
∏cot (ii) ΣΣtan
cotαα (ii) tanαα..tan
tanββ ==11
Q.26 Let
= (
f ( x ) cot −1 x2 + 4x + α2 − α ) be a function
Q.19 Show that: defined R → (0, π / 2] then find the complete set of real
 33π  −1  46π  values of α for which f ( x ) is onto.
sin−1  sin  + cos  cos +
 7   7 
1 2 1 3
 13π  −1   19π   Q.27 Prove that: tan−1 + tan−1 = cos−1
tan−1  − tan  + cot  cot  −  4 9 2 5
 8    8 
1 −1 4
= sin .
13π 2 5
=
7
2 0 . 3 8 | Inverse Trigonometric Functions

Q.28 Prove that Q.2 If x1, x2, x3, x4 are roots of the equation x4 – x3 sin
4

cot −1
 1 + sin x + 1 − sin x  x π 2β + x2 cos 2β ‒ x cos β ‒ sin β = 0 then ∑ tan-1 xi
 = <x< i=1
 1 + sin x + 1 − sin x  2 2 is equal to

(A) x ‒ β (B) π ‒ 2β
Q.29 Find the domain of definition the following
functions. π π
(C)   − β (D)   − 2β
2 2
(Real the symbols [*] and {*} as greatest integers and
fractional part functions respectively.)
Q.3 Range of the function,
2x
(i) f(x) = arc cos
1+x
1+x 2
= (
f ( x ) cot −1 log4/5 5x2 − 8x + 4 ( )) is .:
(ii) f(x)
= cos ( sinx ) + sin−1
2x π
(A) ( 0, π ) (B)  , π )
 x −3 4
= sin−1 
(iii) f(x)  − log10 ( 4 − x )
 2   π  π
(C)  0,  (D)  0, 
 4  2
(iv) f(x) sin−1 2x + x2
= ( )
Q.4 Domain of the explicit form of the function y
1 − sin−1 x
f(x)
(v)= + cos −1
(1 − {x}) represented implicitly by the equation.
(
log5 1 − 4x2 ) (1+x) cos y –x2 =0

where {x} is the fractional part of x.  1− 5


(A) (-1, 1] (B)  −1, 
 2 
 3 − 2x  
(vi) f(x)= 3 − x + cos−1  
 5  1 − 5 1 + 5   1+ 5
(C)  ,  (D) 0, 
( )
+ log6 2 x − 3 + sin−1 (log2 x )  2 2   2 

(
f(x) log10 1 − log7 x2 − 5x + 13
(vii) = ( )) Q.5 Number of integral value(s) of x satisfying

( ) − ( tan x ) − 3 ≤ 0is −
2
  4 tan−1 x −1
 3 
+ cos−1  
 2 + sin 9πx  (A) 1 (B) 2 (C) 3 (D) 4
 
 2 
xx
sin−−11 
Q.6 The area of the region bounded by the curves y=x2
(( ))
sin
−−11xx
 and sec-1[-sin-2 x] (where [.] denotes greatest integer
(viii)=
=f(x) e
f(x) e 22 tan  −−11++ln
++tan nn xx−−[x]
[x]
2
2  function) is
4
(A) π π (B) π π
3
Exercise 2 2 1
(C) π π (D) π π
3 3
Single Correct Choice Type

2
Q.1 Solution set of the inequality x − 4x + 5 a b c d π
Q.7 If tan−1 + tan−1 + tan−1 + tan−1 =
x x x x 2
> sin−1 ( sin3) + cos−1 ( cos2 ) − π is.. Then x4 – x2 ∑ab + abcd is equal to
(A) R (B) R-{1}
(A) ‒ 1 (B) 0 (C) 1 (D) 2
(C) R-{2} (D) R-{-2}
M a them a ti cs | 20.39

Q.8 The solution set of the equation Q.13 Which of the following equation represents a
circle
 1 − x2 
sin−1 1 − x2 + cos−1 x =
cot −1   − sin−1 x


x 

(
(A) y 2 = sin cos−1=
x

)
(B) y sin cos−1 (1 − x ) ( )
(A) [-1, 1] –{0} (B) (0, 1] U {-1} (
(C) y 2 = sin2 cos−1 x ) (D) y = sin (cos x )]
−1 2

(C) [-1, 0) U {1} (D) [-1, 1]


Q.14 If cos−1 x + cos−1 y + cos−1 z =
π then
Q.9 The domain and range of the function
(A) x2 + y 2 + z 2 + 2xyz =
1
3 − 4 sec x2
f ( x ) = cosec−1 log are respectively
1 − 2 sec x
(
(B) 2 sin−1 x + sin−1 y + sin−1 z )
 π π
(A) R;  − ,  = cos−1 x + cos−1 y + cos−1 z
 2 2
(C) xy + yz + zx = x + y + z − 1
 π
(B) R + ;  0, 
 2  1  1  1
(D)  x +  +  x +  +  x +  ≥ 6
 π π  π  x  y  z
(C)  2π − ,2nπ +  − {2nπ} ;  0, 
 2 2   2
 π π  π π
(D)  2π − ,2nπ +  − {2nπ} ;  − ,  − {0} Match the Columns
 2 2   2 2
Q.15 Column I contains functions and column II
Q.10 Solution set of equation [sin-1 x] = [cos-1 x] where contains their range. Match the entries of column I with
[*] represents integral part function is the entries of column II.

(A) (cos 1sin1) (B) [cos 1sin1] Column I Column II

(C) (sin 1 cos 1) (D) [sin 1 cos 1] (A) (p) (0, π )


 x 
f(x)=sin-1  
1 + x 
 
Multiple Correct Choice Type
(B)  x  (q)  π 3π 
g(x)=cos-1    , 
Q.11 Which of the following statement (s) is/ are 1 + x  4 4 
 
meaningless?
(C)  x  (r)  π π
h(x)=tan-1   − , 
  2e + 4   −1  π  1 + x   4 4
(A) cos−1  ln    (B) cos ec  4   
  3   

−1  π 
(D)  x  (s)  π π
(D) sec ( π )
−1
(C) cot   k(x)=cot-1   − , 
2 1 + x   2 2
 

Q.12 If the numerical value of tan

 −1 4 −1  2   a
 cos   + tan    is b then
 5  2 

(A) a + b = 23 (B) a – b = 11
(C) 3b = a + 1 (D) 2a = 3b
2 0 . 4 0 | Inverse Trigonometric Functions

Previous Years’ Questions ( −1 −1


Q.3 Find the value of cos 2cos x + sin x at x =) 1
5
Q.1 Match the conditions/expressions in column I with where 0 ≤ cos−1 x ≤ π and − π / 2 ≤ sin−1 x ≤ π / 2.
statement in column II.  (1981)
π
Let (x, y) be such that sin-1(ax)+cos-1(y) +cos-1 (bxy)=
2 x2 + 1
Q.4 Prove that cos tan−1 [sin(cot −1 x)] = (2002)
Column I Column II x2 + 2
(A) If a=1 and b=0, then (P) lies on the circle
Q.5 If the angle A, B and C of a triangle are in arithmetic
(x, y) x2+y2=1
progression and if a, b and c denote the length of the
(B) If a=1 and b=1, then (q) lies on (x2-1) (y2-1)=0 sides opposite to A, B and C respectively, then the value
(x, y) a c
of the expression sin 2C + sin 2A is  (2010)
(C) If a=1 and b=2, then (r) lies on y=x c a
(x, y)
(D) If a=2 and b=2, then (s) lies on(4x2-1)(y2-1)=0  6  4
Q.6 If α =3 sin−1   and β =3 cos−1   , where the
(x, y)  11  9
inverse trigonometric functions take only the principal
Q.2 Solve the following equation for x values, then the correct option(s) is(are)  (2015)

π (A) cos β > 0 (B) sin β > 0


tan−1 2x + tan−1 3x = (1978, 3M)
4 (C) cos(α + β) > 0 (D) cos α < 0

PlancEssential Questions
JEE Advanced/Boards
JEE Main/Boards
Exercise 1
Exercise 1 Q.3 Q.11 Q.15
Q.5 Q.10 Q.13 Q.17 Q.21 Q.24
Q.16 Q.22 Q.29 Q.29 Q.30

Exercise 2 Exercise 2
Q.3 Q.10 Q.18 Q.3 Q.4 Q.6
Q.21 Q.24 Q.29 Q.9 Q.10 Q.12
Q.30 Q. 32
Previous Years’ Questions
Previous Years’ Questions
Q.1 Q.3
Q.3 Q.4 Q.5
Q.7
M a them a ti cs | 20.41

Answer Key

JEE Main/Boards
Exercise 1
3
Q.1 π / 4 Q.2 2π − 6 Q.3
2
4
Q.5 −1 Q.6 Q.9 −1
5
x+y
Q.10 −1; 5 ± 19 Q.11 z = Q.13 1/2
1 − xy
π x 2π π 1
Q.14 y= − Q.15 Q.16 y= + tan−1 x
4 2 3 4 2
5π π 1
Q.17 Q.19 − x Q.20 ±
6 4 2
x 3π
Q.21 3tan−1 Q.23 x = 1/6 Q.26
a 4
1
Q.28 = cos–1   Q.29 ∞
 3 

Exercise 2

Single Correct Choice Type

Q.1 A Q.2 C Q.3 A Q.4 D Q.5 D Q.6 C

Q.7 B Q.8 D Q.9 C Q.10 A Q.11 A Q.12 C

Q.13 D Q.14 C Q.15 C Q.16 A Q.17 A Q.18 B

Q.19 B Q.20 A Q.21 B Q.22 B Q.23 D Q.24 C

Q.25 C Q.26 B Q.27 A Q.28 A Q.29 D Q.30 A

Q.31 C Q.32 A

Previous Years’ Questions

Q.1 B Q.2 C Q.3 C Q.4 B Q.5 D Q.6 C


7
Q.7 0 Q.8 − Q.9 A Q.10 B Q.11 A Q.12 A
17

Q.13 B Q.14 B Q.15 A Q.16 B Q.17 A


2 0 . 4 2 | Inverse Trigonometric Functions

JEE Advanced/Boards
Exercise 1
2 2 tan y
Q.1 0 Q.2 x
= = sin2y
1 + tan2 y

π 9π 9
Q.3 (i) (ii
(b)) arc tan (x + n) − arc tanx; Q.4 6 cos−1 x − so a =
6 b=

4 2 2

1 3 1 1 4
Q.5 (i) x =; (ii) x =
0, , − ; (iii) x = ; (iv) x =
2 − 3 or 3 Q.6 x=1; y=2 & x=2 ; y=7
2 7 2 2 3

−1  −3 
Q.7 56 Q.8 (i) 2 − 3; (ii) cot  
 4 

2  2   2 3π
Q.9 (i) ( cot 2, ∞ ) ∪ ( −∞ ,cot3) (ii) ,1 (iii)  ,1  ∪  −1,  Q.10
2    2 
 2   4

Q.11 5 Q.12 x ∈ ( −1,1 ) Q.13 20


Q.14 -2 Q.15 (i), (ii), (iii) and (iv) all are identical

 π   π   π 2π   π 
ε R R :  π / 4, π ) (ii) D: ∈  nπ.nπ +  − x x +  n ∈ I : R :  ,  −  
Q.16 (i) D:x∈
 2   2  3 3  2 

 π  π π
(iii) D:x ∈ R R : 0,  (iv) D:x ∈ R R :  − , 
 2  2 4
Q.17 53 Q.21 0 Q.22 10
11 π tan−1 x
Q.23 k= Q.24 27 Q.25 (i) ; (ii)
4 3 2

1 ± 17 1 −1 4 x
Q.26 Q.27 sin = RHS Q.28
2 2 5 2

Q.29 (i) -1/3 ≤ x ≤ 1 ; (ii) {1, − 1} ; (iii) 1 ≤ x < 4 ; (iv) [ −(1 + 2),( 2, − 1)] ; (v) x ∈ ( −1 / 2,1 / 2 ) , x ≠ 0 ; (vi) ( 3 / 2,2 ;
(vii) {7 / 3,25 / 9} ;(viii) (-2, 2)-{-1, 0,1}

Exercise 2

Single Correct Choice Type

Q.1 C Q.2 C Q.3 B Q.4 C Q.5 B Q.6 B


Q.7 B Q.8 C Q.9 C Q.10 A

Multiple Correct Choice Type

Q.11 A, B Q.12 A, B, C Q.13 B, C Q.14 A, B


M a them a ti cs | 20.43

Match the Columns

Q.15 A → s; B → p; C → r; D → q

Previous Years’ Questions


1 2 6
Q.1 A → p; B → q; C → p; D → s Q.2 x = Q.3 − Q.5 3 Q.6 B, C, D
6 5

Solutions

JEE Main/Boards 3 4


= sin cos–1   =
5
  5
Exercise 1
1  1 
Sol 7: tan–1   + tan–1  
 π  1  π 7  13 
Sol 1: sin–1  sin  = sin–1   =
 4   2 4
 1 1 
 +  2
= tan  7 13  = tan–1  
−1
Sol 2: tan–1(tan (–6)) = tan–1tan(2π – 6) = 2π – 6 1 − 1  9
 
 13 × 7 

π 1
Sol 3: sin  − sin−1  1  1   1 
2 2 Sol 8: 4 tan–1   –tan–1   +tan–1  
5
   70   99 
1 π
sin−1 =  5   −1 1 1 
2 6 =2tan–1   – tan − tan−1 
 12   70 99 
π 3  1 1 
sin = −
3 2  
2×5 / 2 70 99 
=tan–1 – tan–1 
2+3 5
2 1 1
Sol 4: tan–12 + tan–13 = π + tan–1 xy> 1 1−  1+ ×
1−6 70 99
2
5 π 3π
= π + tan–1 =π– =
−5 4 4  120   29 
= tan–1   – tan 
–1

 119   6931 
  − 3  π 
Sol 5: cos cos−1  +
 2  6  120 
= tan–1 
 1 
 – tan 
–1
    
 119   239 
cos {p} = –1
 120 1 
 119 − 239 

Sol 6: = tan–1   ≈ tan (1)
–1
5 1 + 120 1 
4 ×
 119 239 
3
2 0 . 4 4 | Inverse Trigonometric Functions

5π2  2 7 
Sol 9: (tan–1x) 2 + (cot–1x) 2 = 2  + 
7
8 Sol 12: tan–1 + tan–1 = tan–1  11 24 
π 11 24  1 − 14 
tan–1x + cot–1x =  
2  264 
π2 π 5π2  48 + 77  1
(tan–1x) 2 + + (tan–1x) 2 – 2 (tan–1x) = = tan–1   = tan
–1

4 2 8  250  2
3π2
2(tan–1x) 2 – p(tan–1x) – =0
8  1 − cos x   x
Sol 13: f(x) = tan–1   = tan  tan 
–1

π ± π2 + 3π2 π ± 2π  sinx   2
tan–1x = = 1
4 4 f′(x) =
2
3π −π
tan–1x = or =  2 
4 4   cos x − sin x  
 1 − sinx  
–1 
2 2 
x = –1 Sol 14: tan–1   =tan  
 cos x   cos2 x − sin2 x 
 2 2 
Sol 10: tan–1(x + 1) + tan–1x + tan–1(x – 1)  
= tan–1(3)  x x  x
 cos − sin   1 − tan 
= tan 
–1 2 2  = tan 
–1 2 
 (x + 1) + x + (x − 1) − x(x2 − 1) 
tan–1   = tan–1(3)  cos x + sin x   1 + tan x 
2    
 1 − x(x + 1) − (x − 1) − x(x − 1)   2 2  2

3x − x3 + x  π x 
=3  tan − tan  π x
2 2
1− x − x − x +1− x + x 2
= tan 
–1 4 2  = tan–1 tan  − 
 1 + tan π . tan x   4 2
 
4x − x3  4 2
⇒ =3
2 − 3x2 π x
= −
⇒ 4x – x3 = 6 – 9x2 4 2

⇒ x3 – 9x2 – 4x + 6 = 0  1 π π 4π 2π
Sol 15: cos–1  −  = + = =
 2  2 6 6 3
⇒ (x + 1) (x2 – 10x + 6) = 0

⇒ x = –1, 5± 19
Sol 16: cot–1 ( 1 + x2 − x)
Put x = tan y
1  -1 2x 1-y 2 
Sol 11: tan sin +cos-1  = cot–1(sec y – tan y)
2  1+x2 1+y 2 
y y
 cos − sin
1 − sin y  2 2
1  -1 2x 2y  = cot–1   = cot
–1
=tan sin +sin-1 
2  1+x 2
1+y 2   cos y  y
cos + sin
y
2 2

1 2x  1  2y   π y π y π 1
tan  sin−1  + tan  sin−1   = cot–1 tan  −  = + = + tan–1x
2 1 + x2  2  1 + y2    4 2  4 2 4 2
  
=
1 2x   1 −1  2y  
1 − tan  sin−1  tan  sin   π 5π
2  1 + y2   Sol 17: cot–1(– 3 ) = π – =
2 1 + x2     6 6
 2x 
= sin–1   = 2 tan x
–1
Sol 18: 3 cos–1x = cos–1(4x3 – 3x)
 1 + x2 
put x = cos x
x+y
= L. H. S. = 3x
1 − xy
R. H. S. = cos–1(4cos3x – 3cosx)=cos–1cos3x = 3x
M a them a ti cs | 20.45

 cos x − sinx   1 − tanx  (–1) does not satisfy,


Sol 19: tan–1   = tan–1  
 cos x + sinx   1 + tanx  1
so answer is x =
6
π  π
= tan–1tan  − x  = − x
4  4 1 1
Sol 24: 2tan–1   + tan–1  
2
  7
x −1 x +1 π
Sol 20: tan–1 + tan–1 = 1 1
x−2 x+2 4 +
1
= tan–1 2 2 + tan–1  
 x −1 x +1  1 7
 +  1−
–1  x − 2 x + 2  = π 4
tan
 x −1 
2 4
4 1
 1 − 2  +
 x −4  31
= tan–1 3 7 = tan–1
4 17
 −2 + x − 2 − x + 2x2  π 1−
⇒ tan–1  = 21
 −4 + 1  4
 
2x2 − 4 π xy x2 y2
⇒ tan–1 = Sol 25: − 1− 1− = cosθ
−3 4 ab a2 b2
2x2 − 4 1
⇒ = 1⇒2x2 = 1⇒x = ± xy − a2 − x2 b2 − y 2
−3 2 = cosθ
ab

 3x  x 3  1 – sin2θ =
 −  
 3a2 x − x3  a a 
–1  x2 y 2 + (a2 − x2 )(b2 − y 2 ) − 2xy a2 − x2 b2 − y 2
Sol 21: tan 
–1
 = tan 
 a2 − 3ax2  2
   x  a2b2
x 1 − 3 a  
= tan y     1 – sin2θ
a
 3tan y − tan3 y  x
⇒ tan–1   = tan–1tan3y = 3tan–1 2x2 y 2 − a2 y 2 − b2 x2 − 2xy(xy − abcos θ)
 1 − 3tan2 y  a = +1
  a2b2

12 4 63 a2 y 2 + b2 x2 − 2xyabcos θ
Sol 22: sin–1 + cos–1 + tan–1 ⇒ sin2θ =
13 5 16 a2b2
12 3 63
= tan–1 +tan–1 +tan–1 y2 x2 2xy
5 4 16 ⇒ sin2θ = + − cos θ
b 2
a2 ab
 12 3 
 +  63
= π + tan–1  5 4  + tan–1  1  1
 1 − 36  16 Sol 26: tan–1(1) + cos–1  −  + sin–1  − 
  2
 20     2
63 63 π 2π π π π 3π
= π + tan–1 + tan–1 =π = + − = + =
−16 16 4 3 6 4 2 4

π 9π 9 −1  1 
Sol 23: tan–12x + tan–13x = Sol 27: L. H. S. ⇒ − sin  
4 8 4 3
 2x + 3x  π
tan–1  =
2  4 9  −1 −1  1  
 1 − 6x  = sin (1) − sin   
4  3 
5x
= 1⇒6x2 – 1 + 5x = 0
1 − 6x2 9 π −1 1 
So L. H. S. =  − sin 
4 2 3
(6x – 1) (x + 1) = 0⇒x = 1/6, –1
2 0 . 4 6 | Inverse Trigonometric Functions

9 Sol 3: (A)
9 1 −1 2 2
= .cos−1 = .sin
4 3 4 3 2
 3 3 π
cot–1 (x − 1)(x − 2) + cos–1  x −  + =
 4  4 2
 1  1 10 − 1
Sol 28: cos−1   − cos−1   + cos−1
 3  6 3 2
    Domain for cot–1 (x − 1)(x − 2) is
 1 2 5 10 − 1
= cos−1  +  + cos−1 x∈ (–∞, 1] ∪ [2, ∞)
 3 6 
3 6 18

2
 3 3
1 + 10 10 − 1 while cos–1  x −  + is defined for x = [1,2]
= cos−1 + cos−1  2 4
18 18
At x = 1
π
 ( 10 − 1)( 10 + 1) (7 − 40 )(7 + 40 )  ⇒ cot–1(0) + cos–1(1) =
cos−1  − 2
 18 18 
 At x = 2
1 π
= cos–1   ⇒ cot–1(0) + cot–1(1) =
2
3
Hence two solutions.

Sol 29: sinx = cos–1cosx⇒0 ≤ sinx ≤ 1 1


Sol 4: (D) cos(tan–1x) = =x
1 + x2
1
= x2 ⇒ x4 + x2 – 1 = 0
1 + x2

x2 = t ⇒ t2 + t – 1 = 0
/2  2
2
 1 5
⇒t +  =
 2  4
x = 2nπ always satisfy
so infinite roots. 1 5
x2 = − ± = positive
2 2

5 −1
Exercise 2 x2 =
2
Single Correct Choice Type
Sol 5: (D) x2 – 4x + 5 = (x – 2)2 + 1

1 − x2 x = 2, to define sin–1(x2–4x+5)
Sol 1: (A) = tan cos x =–1
x π π
So 4 + 2a + +0=0⇒a= − –2
2 4
Sol 2: (C) (|sin–1x| + |sin–1y|2) = p2
⇒ (|sin–1x| + |sin–1y|) = π Sol 6: (C) f(x) = sin−1 sinx + cos−1 cos x
π sin x must not be negative to define f(x). So the domain is
⇒ |sin–1x| = = |sin–1y|
2
x∈ [2nπ, (2n + 1) p], n ∈ I
⇒ x = ±y = ±1
⇒ x2 + y2 = 2 π
Sol 7: (B) θ = sin–1x + cos–1x – tan–1x = – tan–1x
2
π π
x∈ [0, 1]⇒ ⇒θ≤
4 2
M a them a ti cs | 20.47

Sol 8: (D) tan–1(2) + tan–1(3)  x 


2

Sol 12: (C) [tan(sin x) ] = 


–1 2
 >1
 3 + 2  3π  1 − x 
2
= π - tan–1  = = cosec–1(x)
 1 − 6  4 x2
⇒ >1
−π π 1 − x2
≤ cosec–1(x) ≤
2 2
−1 + 2x2
So none of these. ⇒ >0
1 − x2

Sol 9: (C) f(x) = cot–1 (x + 3)x + cos–1 x2 + 3x + 1 ( 2x − 1)( 2x + 1)


⇒ >0
(x − 1)(x + 1)
2
 3 5
= cot–1
x(x + 3) + cos –1
x +  −  2 2
 2 4 x∈ (–1, 1) –  − , 
 2 2 
x(x + 3) ≥ 0 ⇒ x ∈ (–∞, –3] ∪ [0, ∞)
2 Sol 13: (D) sin–1x = 2 sin–1a
 3 5
 x +  − ⇒0
 2  4 π π
⇒ − ≤ 2sin–1a ≤
2 2
 −3 5   −3 5 
⇒ x ∈  −∞ , − ∪ + ,∞ π π
 2 2   2 2  ⇒ − ≤ sin–1a ≤
 
4 4
2
 3 5 1 1
 x +  − ≤ 1 ⇒ x ∈ [–3, 0] ⇒ − ⇒ a≤
 2 4 2 2

So the answer x ∈ {0, –3} Sol 14: (C) (sin–1x + sin–1 w) (sin–1y + sin–1z) = pz for this
to satisfy
Sol 10: (A) α = sin–1 cos sin–1x
x=w=y=z=1
β = cos–1sin cos–1x
or x = w = y = z = –1
tanα = tan sin cos sin x
–1 –1

1 1 −1 −1
= 0 or =0
1 − x2 1 1 −1 −1
= tan sin–1 1 − x2 =
x
x independent of N1, N2, N3, N4
tanβ = tan cos–1sin cos–1x = tan cos–1 1 − x2 =
2
1−x
cotβ = tan α Sol 15: (C) θ = cot–17 + cot–18 + cot–118

1  −1  1 1 1
Sol 11: (A) x = 2cos–1   +sin–1   +tan 3
–1
= tan−1 + tan−1 + tan−1
2
   2  7 8 18
2π π π 5π  1 1 
x= + + ; x=  +  1
3 4 3 4 = tan  7 8  + tan−1
−1

1 − 1 × 1  18
1  x  1 5π  
y = cos  sin−1  sin   = cos sin−1 sin  7 8
2  2  2 8
 3  1
⇒ θ = tan−1   + tan−1
1 5π  3π  11  18
= cos  π −  = cos
2 8 16
3 1
+
65 1
⇒ θ = tan−1 11 18 = tan−1 = tan−1
3 195 3
1−
11 × 8
⇒ cotθ = 3
2 0 . 4 8 | Inverse Trigonometric Functions

Sol 16 : (A) (A) f(x) = {x} (Periodic)  3


2
1 13
1 ⇒ tan−1  n +  +  = tan−1
(B) g(x) = x sin (not periodic)  2 4 1
x  
2
(C) h(x) = x cos x (not periodic)  1 49
n +  = ⇒n = 3
(D) sin(sin–1x) (not periodic)  2  4

 3x2 − 7x + 8  
1 1 Sol 20: (A) f(x) = cos−1  
+
−1 1 −1 7 7 = tan−1 7  1 + x2 

Sol 17: (A) 2 tan = 2 tan
7 1 24 1 + x2≥ 1
1−
1 49
4cot–13 = 4tan–1 3x2 – 7x + 8 = 3(x2 + 1) – 7x + 5
3
 7
2
49   7
2
23 

= 3 x −  + 8 −  = 3 x −  + 

3 6/4 24  6 36   6  36 
= 2tan–1 = tan–1 = tan–1    
4 9 7
1−
16 3x2 − 7x + 8 7x − 5
1 ⇒ =3–
3cot–14 = 3tan–1 x +1 x2 + 1
4
8 1 47 7x − 5
= tan–1 +tan–1 = tan–1 ⇒ –1 ≤ 3 – <2
15 4 52 x2 + 1
47 1 240 −(7x − 5)
4cot–14 = tan–1 + tan–1 = tan–1 ⇒ –4 ≤ < –1
52 4 173
(x2 + 1)
Checking all options one by one
(7x − 5)
⇒4≥ >1
x2 + 1
Sol 18: (B) sin–1 (2x 1 − x2 ) = 2 sin–1x
4x2 – 7x + 9 ≥ 0 & x2 – 7x + 6 < 0
Put x = sin y
always true & (x – 6) (x – 1) < 0⇒x ∈ (1, 6)
sin–1 sin2y = 2sin–1siny cos y
⇒ –1 ≤ sin2y ≤ 1 1
Sol 21: (B) ar = 2r–1 =
π π br
⇒ − ≤ 2y ≤
2 4 1 2
2ar + = 2r + 21–r = 22 +
1 1 br 2r
⇒ − ≤x≤
2 2 2r
tr = cot–1(2ar + br) = tan–1
 1 1  22r + 2
So it is true if x ∈  − , 
 2 2
2r −1 2r − 2r −1
= tan–1 = tan−1
k =n
1 + 22r −1 1 + 2r 22r −1
2k 6
Sol 19: (B) ∑ tan−1 2 + k 2 + k 4 = tan−1
7 = tan–1(2r) – tan–1(2r–1)
k =1

π π π
L. H. S. = ∑ tan −1 2k ∑ tr = tan–1(2∞) – tan–1(21–1) = − =
2 4 4
r =1
1 + (k 4 + k 2 + 1)
Sol 22: (B) f(x) = sin–1(3x – 4x3)
k 2 + k + 1 − (k 2 − k + 1)
= ∑ tan −1

1 + (k 2 − k + 1)(k 2 + k + 1)
Let’s put x = sin y = sin–1sin3y
π π
∞ – ≤ 3y ≤ f(x) = 3sin–1x
= ∑ (tan−1 (k 2 + k + 1) − tan−1 (k 2 − k + 1) 2 2
k =1
3π π
−1 6 ≥ 3y ≥ f(x) = π – 3sin–1x
= tan (n + n + 1) – tan (1) = tan
–1 2 –1
2 2
7
M a them a ti cs | 20.49

3π π 2x2 − 1
– ⇒3y≤ – f(x) = –π – 3sin–1x Sol 26: (B) RHS = cot–1
2 2
4x2 − 4x 4
It’s not differentiable 2 times.
4x2 − 4x 4
= cos−1 = cos–1 4x2 − 4x 4
4 2 2 4
 1 5 4x + 1 − 4x + 4x − 4x
Sol 23: (D) sec–1x = cos–1  −  + sin–1
 2  3 3 Put x = cos y
1 2 f(x) = cos–1| sin2y |
sec–1(x) = π – cos–1   +cos–1
2 3 3 π
RHS = – sin–1|sin 2y|
 2 1 2
5 3
= π + cos–1  +  Since |sin2y| ≥ 0, so RHS will always be greater than
3 3 2 3 3 2 
  zero.
5 3 + 2  Then x can be (0, 1)
sec–1x = π + cos–1  
 6 3 
 
Sol 27: (A) sin–1x = cos–1x + sin–1(3x – 2)
 15 + 6  18
x = sec  π + cos−1  = − x∈ [–1, 1]
 18   15 + 6
 (3x – 2) ∈ [–1, 1]
1 
−1 ⇒ x ∈  ,1
 x 1 a π 1 a  3 
Sol 24: (C) tan  + sin−1  + tan  − sin−1  
 4 2 b 4 2 b  ⇒ sin–1x – cos–1x = sin–1(3x – 2)
−1 π
 1 −1 a 1 −1 a  ⇒ 2sin–1x = + sin–1(3x – 2)
 1 + tan 2 sin b 1 − tan 2 sin b  2
=  + 
 1 − tan 1 sin−1 a 1 + tan 1 sin−1 a  Taking cosine of both sides
 2 b 2 b  ⇒ cos(2sin–1x) = –(3x – 2)
−1
  21 −1 a  
⇒ 1 – 2x2 = –3x + 2
 2  1 + tan sin 
2 b  ⇒ 2x2 – 3x + 1 = 0
=  
  21 −1 a 

  1 − tan sin    1
2 b  ⇒ (x – 1)  x −  = 0
   2
1 a 1
1 − tan2 sin−1 x = 1,
2 b 1 a 1 b2 − a2 2
= = cos sin–1 =
 1 a 2 b 2 b
2  1 + tan2 sin−1  1 − x2
 2 b Sol 28: (A) f(x) = sin−1
1 + x2
 1 1 π
Sol 25: (C) x = tan–1(1) – cos–1  −  + sin–1   g(x) = cot–1x – tan–1x = – 2tan–1x
 2 2 2
π 2π π π Put x = tan y in f(x)
= − + = −
4 3 6 4 f(x) = sin–1 |cos2y|
1
1 + coscos−1   π
– cos–1|cos2y|
1  1  8
y = cos  cos−1    = 2
2  8  2
f(x) = g(x) when x ∈ [0, 1]
9 3
= =
16 4 Sol 29: (D) tan[cos–1{sin(2tan–12) }]
3 = tan[cos–1{2sin(tan–12) cos(tan–12) }]
y=– tan x.
4
2 0 . 5 0 | Inverse Trigonometric Functions

  2 × 2 1  4 3 Previous Years’ Questions


= tan cos−1  ×   = tan cos–1 =
  5 5   5 4
 4  2 
Sol 1: (B) tan cos−1   + tan−1   

sin−1 x − cos−1 x  5  3 
Sol 30: (A) ∑ πr
is finite
n=1  3  2 
= tan tan−1   + tan−1   
π
n
 4  3 
∞ − 2cos−1 x
2
= ∑ πr  −1  4  −1  3  
n=1  cos   = tan   
 5  4 
π π 3π
≥ − 2cos−1 x ≥ −   3 2 
2 2 2  +
    17   17
π = tan tan  4 3   = tan tan−1    =
−1

pr >  3 2
 1 − ·    6  6
2   
 4 3 
1
⇒r>
2
 2π  −1   π 
Sol 2: (C) sin−1 =
 sin  sin sin  π −  
Sol 31: (C) y = sin–1(sin x) , x ∈ [0, p]  3    3 

π  π π
0<x≤ y = x = sin−1  sin  =
2  3  3
π /2
< x ⇒ py = π – x
2 Sol 3: (C) Given function is tan–1 x(x + 1) + sin–1
A1 ≤ A2 ≤ A3 π
O /2 x2 + x + 1 =
A3 2
A1 = A2 =
2 Function is defined, if
2
1 π π π (i) x (x + 1) ≥ 0 ⇒ Domain of square root function.
A1 = × × =
2 2 2 8
(ii) x2 + x + 1≥ 0 ⇒ Domain of square root function.
π2
A3 =
4 (iii) x2 + x + 1 ≤ 1 ⇒ Domain of sin–1 function.

4n From (ii) and (iii)
Sol 32: (A) ∑ tan−1 n4 − 2n2 + 2
n=1 0 ≤ x2 + x + 1 ≤ 1 ∩ x2 + x ≥ 0

4n ⇒ 0 ≤ x2 + x + 1 ≤ 1 ∩ x2 + x + 1 ≥ 1
= ∑ tan−1 1 + (n2 − 1)(n2 − 1)
n=1 ⇒ x2 + x + 1 = 1

(n + 1)2 − (n − 1)2 ⇒ x2 + x = 0
= ∑ tan−1 1 + (n − 1)2 (n + 1)2 ⇒ x(x + 1) = 0
n=1

∞ ⇒ x = 0, x = –1
= ∑ tan−1 (n + 1)2 − tan−1 (n − 1)2
n=1
π
π 3π Sol 4: (B) We know that, sin–1( α ) + cos–1( α ) =
= 2[tan–1(∞)] – tan–1(1) – tan–10= π – = 2
4 4
Therefore, α should be equal in both functions.

tan (2) +tan (3) = π+tan (–1) =
–1 –1 –1
x 2 x3 x 4 x6
4 ∴ x− + − ... = x2 − + − ...
2 4 2 4
4tan–1(1) – π
 1  x x2
⇒ =
sec (1 –
–1
2 ) =cos  –1
 =–cos–1( 2 + 1) x x2
1 − 2  1+ 1+
2 2
M a them a ti cs | 20.51

x
x x2 and cos θ = = cos (cot–1x)
⇒ =
2+x 2 + x2 1 + x2
2 Now
2
1
2x 2x2 1 + x2 [{x cos(cot −1 x) + sin(cot −1 x)}2 − 1]2
⇒ =
2+x 2 + x2 1

⇒ 2x(2 + x2) = 2x2(2 + x)  


2 2
 x x 1
= 1 + x2 +  − 1
⇒ 4x + 2x = 4x + 2x  2  
1 + x2
3 2 3
 1 + x  
⇒ x(4 + 2x2 – 4x – 2x2) = 0 1

⇒ Either x = 0 or 4 – 4x = 0  2 
2 2
= 1 + x2  1 + x  − 1
⇒ x = 0 or x = 1  2  
 1 + x  

0 < |x| < 2 , 1

∴ x = 1 and x ≠ 0 = 1 + x2 [1 + x2 − 1]2 = x 1 + x2

Sol 5: (D) Given, sin [cot–1(1+x)] = cos(tan–1x)...(i) Sol 7: Given,


and we know,
a(a + b + c) b(a + b + c)
  =θ tan−1 + tan−1 +
1 bc ac
cot–1 θ = sin–1  ,
 2 
 1+θ 
c(a + b + c)
tan−1
 1  ab
and tan–1 θ = cos–1  
 2    x + y + z − xyz  
 1+θ  −1 −1 −1
tan−1 
 tan x + tan y + tan z = 
∴ From Eq. (i),   1 − xy − yz − zx  

    = tan−1
1 1
sin  sin−1  = cos  cos−1    a 
    b c  a+b+c
 1 + (1 + x)2   1 + x2   a+b+c + +  − (a + b + c) 
  bc ca ab  abc 
 
1 1  
⇒ =  1 1 1 
1 − (a + b + c)  + + 
1 + (1 + x)2 1 + x2  a b c 
 
⇒ 1 + x2 + 2x +1 = x2 + 1  a+b+c a+b+c 
 (a + b + c) − (a + b + c) 
1 = tan−1  abc abc 
⇒x=–  (a + b + c) (ab + bc + ca) 
2 1−
 
 abc 
Sol 6: (C) We have, 0 < x < 1
⇒ θ = tan–1 0 ⇒ tan θ = 0
Let cot–1 x = θ
  1  
C  2. 
 −1  1  π  −1

5 − π
Sol 8 : tan 2 tan
=   −  tan tan  
  5  4    1  4
1 − 
1+x
2
  25  
1  −1  5  π 
= tan tan   − 
  12  4 

B X A   5  π
tan tan−1    − tan   5
12 4 −1
⇒ cot θ = x       7
= = 12 = −
1   5  π 5 17
⇒ sin θ = = sin (cot–1x) 1 + tan tan−1    tan 1 + ·1
  12   4 12
1 + x2
2 0 . 5 2 | Inverse Trigonometric Functions

Sol 9: (A) Given, A = 2 tan–1 (2 2 –1) and  5 2


Sol 11: (A) Let cot  cosec−1 + tan−1 
 3 3 
1 3
B = 3sin–1   + sin–1  
3 5  3  2 
⇒ E cot  tan−1   + tan−1   
=
Here, A = 2tan–1 (2 2 –1)  4  3 

  3 2 
= 2tan–1(2 × 1.414 – 1) = 2tan–1(1.828)   + 
⇒ cot  tan  4 3  
E= −1
π 2π  1 − 3 ⋅ 2 
∴ A > 2 tan–1( 3 ) = 2. =   
3 3   4 3 
To find the value of B, we first say  17  6
⇒ E cot =
= tan−1 
1 1 π  6  17
sin–1 < sin–1· =
3 2 6
1 π 4 3
So that, 0< 3sin–1 < Sol 12: (A) cos(α=
+ β) ⇒ tan(α=
+ β)
3 2 5 4
1  1 1   23  5 5
Now, 3 sin–1 = sin–1  3· – 4 ·  = sin–1   sin(α=
− β) ⇒ tan(α=
− β)
3 3 27 13 12
   27 
3 5
 3 π +
4 12 56
= sin (0.851) < sin 
–1
 = –1
tan 2=
α tan(α + β + α − =
β) =
 2  3 3 5 33
  1−
4 12
3  3 π
sin–1   = sin–1(0.6) < sin–1  = a π
5  5  3 Sol 13: (B) r = cot
  2 n
π π 2π ‘a’ is side of polygon.
∴ B< + =
3 3 3
a π
2π 2π R= cosec
Thus, A> and B< 2 n
3 3
π
Hence, greater angle is A. cot
r n π
= = cos
R π n
cosec
Sol 10: (B) n
A
π 2
cos ≠ for any n ∈ N
n 3

1
Sol 14:=
(B)  cos
= 45o
o o 2
45 60
1
D 7 C x B m = cos 120o = −
2
BD = AB = 7 + x n cos θ
=
Also AB = x tan 60 = x 3 o
Where θ is the angle which line makes with positive
∴ x 3=
7+x z-axis.

7 1 ⇒ 1 + 1 + cos2 θ =1
Now  + m2 + n2 =
x= 2 4
3 −1 1 1
cos2 θ = ⇒ cos θ = ( θ being acute)
7 3 4 2
=AB ( 3 + 1) π
2 ⇒ θ= .
3
M a them a ti cs | 20.53

−1 1 JEE Advanced/Boards
Sol 15: (A) <x<
3 3
x tan θ
= Exercise 1
−π π 1 + x   1 − x2 
<θ< Sol 1: α = 2 tan–1  ; β = sin–1
 
6 6   1 + x2 
1 − x   ,
tan−1 y = θ + tan−1 tan 2θ = θ + 2θ = 3θ Put x = tan y
3  π
3tan θ − tan θ α = 2tan–1tan  y + 
=y tan=

2
1 − 3 tan θ  4
π
3x − x3 β = sin–1cos2y = – cos–1cos2y
y= 2
1 − 3x2
π
If 0 < tan x < 1; 0 < y <
4
Sol 16: (B) 0 ≤ x < 2π
π π π π
0 < 2y < ⇒ <y+ <
cos x + cos2x + cos3x + cos 4x =
0 2 4 4 2
 π π
(cos x + cos 4x) + (cos2x + cos3x) =
0 α = 2  y +  = 2y +
 4  2
5x 3x 5x x π
2 cos cos + 2cos cos =0 β = – 2y
2 2 2 2 2
5x  x ⇒α+β=π
2 cos2cos x cos  = 0
2  2
If x > 1⇒tan y > 1
5x x
cos
= 0 or
= cos x 0= or cos 0 π π π
2 2 ⇒ > y > ⇒π> 2y >
2 4 2
(2n + 1)π π 3π π π
x= or x = (2n + 1) or x = (2n + 1)π ⇒ >y+ >
5 2 4 4 2
 π 3π 7π 9π π 3π 
=x  , , π, , , ,   π  −3π
5 5 5 5 2 2 α = 2  −π + + tan−1 x  = + 2 tan–1x
 4  2
Number of solution is 7 π 3π
β= – [2 + 2tan–1] = + – 2 tan–1x
2 2
π π
Sol 17: (A) At x= ⇒ y= ⇒α+β=0
6 3
 x x   1 + x2 − 1 − x2 
 cos + sin   π Sol 2: y = tan–1  
f(x) tan−1  2 2   x ∈  0,   1 + x2 + 1 − x2 
 cos x − sin x   2  
 
 2 2 
 1 + x2 − 1 − x2 
  π x 
−1
tan y =  
= tan  tan  +    1 + x2 + 1 − x2 
 4 2   

π x 1 1 − tan y 1 − x2
f(x) =+ f '(x) = ⇒ =
4 2 2 1 + tan y 1 + x2
Slope of normal = -2 1 + tan2 y − 2 tan y 1 − x2
⇒ =
π  π 1 + tan2 y + 2 tan y 1 + x2
Equation of normal y − =−2  x − 
3  6
2π 2 tan y
y=−2x + ⇒ x2
= = sin2y
3 1 + tan2 y
2 0 . 5 4 | Inverse Trigonometric Functions

2n−1 3 1
Sol 3: (i) nth term = tan–1 ⇒x= cos sin–12x – (2x)
2n−1
1+2 2 2
n n−1
2 −2 3 16x2
= tan–1 n 2x −1 ⇒ 2x = 1 − 4x2 ⇒ = 1 – 4x2
1 + 2 (2 ) 2 3
nth term = tan–1(2n) – tan–1(2n–1) 28x2 3 1 3
= 1⇒x = =
Sum of infinite series 3 28 2 7
π (ii) tan–1(x – 1) + tan–1(x) + tan–1(x+1) = tan–13x
= tan–1(∞) – tan–1(1) =
4
(x + 1) − x (x + 2) − (x + 1) ⇒ tan–1(x – 1) + tan–1(x+1) = tan–1(3x) – tan–1(x)
(ii) tan–1 + tan–1 +…. .
1 + x(x + 1) 1 + (x + 2)(x + 1) (x − 1) + (x + 1) 3x − x
⇒ =
2
=tan–1(x+1)–tan–1(x)+tan–1(x+2)–tan–1(x+1) +… 1 − (x − 1) 1 + 3x2

= tan–1(x + 2) – tan–1(x) + …. 2x 2x
⇒ = ⇒4x2 = 1 ; x = 0
2
= tan (x + n) – tan (x)
–1 –1 2−x 1 + 3x2

–1 2x − x
 x − 1  
(iii) tan–1   + tan  
 −1   x + 1  2x + x 
Sol 4: x ∈  −1, 
 2 
x − 1 2x − 1
+
f(x) = sin (3x – 4x ) + cos (4x – 3x) = tan–1 x + 1 2x + 1
–1 3 –1 3

(x − 1)(2x − 1)
f(x) = g(x) + h(x) 1−
(x + 1)(2x + 1)
g(x) = sin–1sin3y where y = sin–1x
2x2 − 1 − x + 2x2 − 1 + x
h(x) = cos–1cos3z where z = cos–1x = tan–1
6x
 −1 
x∈  −1,   4x2 − 2   2x2 − 1 
 2  = tan–1   = tan–1  
 6x   3x 
   
 π π  3π π 
y = sin–1x ∈  − , −  ⇒ 3y ∈  − , − 
 2 6   2 2 2x2 − 1 23
⇒ =
x 12
 2π 
z = cos–1x ∈  , π  ⇒ 3z ∈ [2π, 3p] ⇒ 24x2 – 12 = 23x
3 
⇒ 24x2 – 23x – 12 = 0
g(x) = –π – 3sin–1x
π  5π 23 ± 529 + 1152 4
= –π – 3  − cos−1 x  = − + 3 cos–1x x= ⇒x=
2  2 48 3
h(x) = –2π + 3 cos–1x  x2 − 1  2x 2π
9π (iv) cos–1   + tan–1 2 =
f(x) = 6 cos–1x –  x2 + 1  x −1 3
 
2
9 (x2 + 1)2 − (x2 − 1)2 2x
∴ a = 6, b = − LHS = tan−1 + tan−1
2 2
(x − 1) 2
x −1
π | 2x | 2x
Sol 5: (i) sin–1x + sin–12x = = tan−1 + tan−1
3 2 2
x −1 x −1
π
⇒ sin–1x = – sin–12x
3 2x
If x < 0 LHS = = 2 tan−1
2
π  x −1
⇒ x = sin  − sin−1 2x  2x
3  If x > 0= 2tan–1
2
x −1
M a them a ti cs | 20.55

2x π β3 α3
= tan ⇒ 2x = 3 (x2 – 1) = +
x2 − 1 3 α β
1− 1−
4x = 3(x + 1 – 2x ) ⇒3x – 10x + 3 = 0
2 4 2 4 2 2
α +β 2
α2 + β2
1
⇒ (x2 – 3) (3x2 – 1) = 0 ⇒x = ± 3 , ±  β3 α3 
3 = α2 + β2  + 
1  α2 + β2 − α α 2
+ β2
− β 
x=± does not satisfy.  
3
By putting all value, we get = 56
y 3
Sol 6: tan x + cos –1 –1
= sin –1

1 + y2 10 2x 1 − x2
Sol 8: f(x) = sin–1 ; g(x) = cos–1
1 + x2 1 + x2
 1
x +  2x
1  y h(x) = tan–1
LHS = tan–1x + tan–1 = tan–1 = tan−1 (3) 1 − x2
y x
1− put x = tan y
y
1
x+ f(x) = sin–1sin2y; g(x) = cos–1cos2y
y 1 3x
= 3⇒ x + = 3 – h(x) = tan–1tan2y
x y y
1−
y (i) x∈ (–1, 1)
1 1 + 3x π π π π
(1 + 3x) = 3 – x⇒y = − ≤ y ≤ ⇒ − ≤ 2y ≤
y 3−x 4 4 2 2

At x = 1 ; y = 2 f(x) = 2y = 2tan–1x

At x = 2 ; y = 7 2 tan−1 x ; x ≥ 0
g(x) = 
−1
−2 tan x ; x ≤ 0
Sol 7: x2 – 4x + 1 = 0
h(x) = 2 tan–1x
(x – 2) 2 = 3
2 tan x
−1
;x≤0
α=2+ 3 ;β=2– 3 ; α + β = 4 ; aβ = 1 f(x) + g(x) + h(x) =  −1
6 tan x ;x≥0
(2 − 3) 1 3
2− 3
f(α, β) = cosec2  tan−1  π
2 2 2 + 3  x = tan =2– 3
 12

(2 + 3)3 1 2+ 3 (ii) f(2) + g(2) + h(2)


+ sec2  tan−1 
2  2 − 3 
2 4
f(2) = sin–1  
5
(2 − 3)3 1 (2 + 3)3 1
= +
2 [sin θ1 / 2]2 2 [cos θ2 / 2]2  −3 
g(2) = cos–1  
 5 

(2 − 3)3 (2 + 3)3  4   −3 
+ h(2) = tan–1   = cot–1  
 (2 − 3)  
−1 (2 + 3)
  −3   4 
1 − cos tan−1   1 − cos tan  
 (2 − 3)   (2 − 3)  f(2) = –g(2)

 −3 
f(2) + g(2) + h(2) = cot–1  
(2 − 3)3 (2 + 3)3  4 
= +
2+ 3 (2 − 3)
1− 1+
14 2 7 Sol 9: (i) (cot–1x) 2 – 5(cot–1x) + 6 > 0
(cot–1x – 3) (cot–1x – 2) > 0
2 0 . 5 6 | Inverse Trigonometric Functions

cot–1x∈ (–∞, 2) ∪ (3, ∞) π


since r, s, t are always positive so value will be π –
cot–1x∈ (0, 2) ∪ (3, π) 4

x∈ (cot 2, ∞) ∪ (–∞, cot 3) = .
4
(ii) sin–1x > cos–1x π  x 
Sol 11: f(x) = + cos−1   − tan–1 x
π π 4  2 
− ≤ sin−1 x ≤  1+x 
2 2 /2
sgn(f(x) ) = 1 when f(x) > 0
0 ≤ cos x ⇒π –1
/4 1
1 for any x > 0
-1
 1  2 π 1
x∈  ,1  f(x) = + tan–1 – tan–1x
 2  4 x

(iii) tan2(sin–1x) > 1 f(x) = – 2 tan–1x
4
π π
− ≤ sin−1 x ≤ 3π
2 2 – 2tan–1x > 0
4
x
tan(sin–1x) = 3π
0 < tan–1x <
1 − x2 4
x2
⇒ >1  3π / 4 
1 − x2 0 ≤ x < tan  
 2 
2x2 − 1
⇒ <0 ⇒0<x< 2 +1
x2 − 1
0≤x< 2+1
 −1   1 
x∈  −1, ∪ ,1  x = 0, 1, 2
 2  2 
  2x2 + 4  
Sol 10: x(x – 2) (3x – 7) = 2 are real and positive at x = −1
Sol 12: sin  sin    <π – 3
 2 
7   1 + x 
0, +2, it has (–2) value.
3
2x 4 + 4 2
At x = 4f(4) = 38 =2+
1+x 2
1 + x2
1 1 17
At x = f  = 2x2 + 4
2 2 8 4≥ >2
1 1 1 + x2
One root between 0 to , one between to 2, one
2 2  2x2 + 4 
7 −  + π<π – 3  4 3π/2
between to 4.  1 + x2 
3   π/2 2
tan–1(r) + tan–1(s) + tan–1(t) 2x2 + 4
>3
 r + s + t − rst  1 + x2
= tan  –1
  ….. (i)
 1 − (rs + st + tr) 
1 − x2
equation is >0
1 + x2
⇒ 3x2 – 13x2 + 14x – 2 =0
x∈ (–1, 1)
13
r+s+t=
3
Sol 13: f(x) = tan–1(cot x – 2 cot 2x)
−( −2) 2 5
rst = =
3 3 ∑ f(r) = a – bπ
r =1
+14 −π
rs + st + tr = = tan–1(–1) =  1 2(1 − tan2 x) 
3 4 f(x) = tan−1  −  = tan–1(tan x)
 tanx 2 tanx 
M a them a ti cs | 20.57

23 5 2

1 4
-1.25 -1 -0.75 -0.5 -0.25 0.25 0.5 0.75 1 1.25

Sf(r) = 1 + (–π + 2) + (π + 3) + (π + 4) + (2π + 5) -2

= 15 – 5π -4

a = 15, b = 5, a + b = 20
x
(iii) y = sin(tan–1x) = identical
Sol 14: f(x) = (2a + b) cos–1x + (a + 2b) sin–1x 1 + x2
Domain–1 ≤ x ≤ 1
Then range should be –1 ≤ f(x) ≤ 1
f(x) = a[2cos–1x + sin–1x] + b[cos–1x + 2sin–1x] 1.0

π −1  π 
= a  + cos x  + b  + sin−1 x 
2  2  - -/2 /2  3/2
π
= (a + b) + (acos–1x + bsin–1x)
2 -1.0

π
= (a + b) + a(cos–1x + sin–1x) + (b – a) sin–1x
2 -2.0
π
= (2a + b) + (b – a) sin–1x (iv) y = cos(tan–1x)
2
π π π 
(3a) < f(x) < (a + 2b) = cos  − cot −1 x  = –sin(–cot–1x)
2 2 2 
π −2 4 = sin(cot–1x) identical
(3a) = –1 ⇒ a = ;b=
2 3π 3π
π(a – b) = –2

1 − x2 3.0
Sol 15: (i) y = tan(cos–1x) = except x = 0
x

1 − x2 2.0
y= identical
x
6
1.0
4

-2 -1.5 -1 -.5 0.5 1 1.5 2 -/2 -/2

-4

-6 Sol 16: (i) f(x) = cot–1(2x – x2)


2x – x2 = x(2 – x)
-8

-10
Domain x ∈ R
1
(ii) y = tan(cot–1x) = except x = 0 identical π 
x Range 2x – x2< 1 ⇒ x ∈  , π 
4 
2 0 . 5 8 | Inverse Trigonometric Functions

(ii) f(x) = sec–1(log3tan x + logtanx3)  8 


γ = tan−1  
Domain tan x > 0, tan x ≠ 1  15 
 36  4  8 
1 α + β + γ = sin−1   + cos−1   + tan−1  
log3 tan x + >2
log3 tanx  85  5  15 
 36  3  8 
1 = tan−1   + tan−1   + tan−1  
or log3 tan x + < –2  77  4  15 
log3 tanx
3 8
 π  π  +
 36 
x∈  nπ,nπ +  − π +  = tan−1   + tan−1 4 15
 2  4   77  24
1−
60
 π 2π   π 
Range ∈  , −   36   77  π
3 3  2  = tan–1   + tan–1   =
 77   36  2
2x2 + 1 (i) Scot α = cot α + cotβ + cot γ
(iii) f(x) = cos–1
x2 + 1
π
sinceα + β + γ =
2
2x2 + 1
Domain ≤1 1 = tan α. tanβ + tan btanγ + tan gtanα
x2 + 1
cotα cotβ cotγ = cotα + cotβ + cotγ
2x2 + 1 ≤ x4 + 2x2 + 1
Hence prove.
x 4≥ 0
(ii) Since α + β + γ = π/2
Always true x∈ R
Hence, Σ(tan atanβ) = 1
(iv) f(x) = tan–1(log4/5(5x2 – 8x + 4) )
5x2 – 8x + 4 > 0  33π  46π
Sol 19: sin−1  sin −1
 + cos cos
 8 
2
4 64  7  7
x −  + − >0
 10  5 100  13π   −19π 
+ tan−1  − tan −1
 + cot cot  
 8 
2
16  7   8 
x −  + >0
 10  100 33π 46π
LHS = – 5π + 7π –
7 7
Domain x ∈ R
 13π   19π 
 −π π  –  − 2π  + π +  − 2π 
Range x ∈  ,  8 8
   
 2 4
−13π 13π 13π
Sol 17: y = sin–1sin8 – tan–1tan 1 = + 2π + π – =
7 4 4
+ cos–1cos12 – sec–1sec9 + cot–1cot6 – cosec–1cosec7
5π 5  −7  −1  36 
8~ + h ; 12 ~ 4π – h ; 6 ~ 2π – h Sol 20: (i) cos−1 + cos−1   + sin  
2 13  25   325 
π 5π
1~ – h ; 9 ~ 3π – h ; 7 ~ –h  12  24  36 
2 2 = tan−1   − tan−1 + sin−1  
 5 7  325 
y = (3π – 8) – 1 + 4π – 12 – (9 – 2π)
+ (6 – π) – (7 – 2π) = –31 + 10π  12 24 
 − 
−1  5 7  36
= tan + sin–1
−1  36  −1 4 12 24 325
Sol 18: α = sin   β = cos 1+ ×
85
  5 5 7
M a them a ti cs | 20.59

−1  −36  −1  36  (2k + 1)(4k + 1)


= tan   + tan  = π ⇒ 1 + 2k + 1 + 4k + 1 =
 323   323  4
⇒ 24k + 12 = 8k2 + 1 + 6k
2 6 +1 ⇒ 8k2 – 18k – 11 = 0
(ii) LHS = cos–1 – cos–1
3 2 3 11
⇒k=
 2  6 +1 4
1 7 + 2 6 
= cos−1   + 1−
 3  2 3  3 12 
  Sol 24 :

 6 + 1  5 − 2 6  c
= cos–1  +
 3 2  3 2 2 
 
b a
 12 + 2 + 5 − 2 6 
= cos–1  
A B
 6  C
 
 12 + 3  1
3 π Area (∆ABC) = × c × (b sinA)
= cos–1   = cos–1 = 2
 6  2 6
 
∠A = ∠B =

Sol 21: 1  −1  2 3 1  −1    1 
sin  −    + sin−1  
2  3 2 3  2    3  
 ab + 1  −1  bc + 1  −1  ca + 1   
cot −1   + cot   + cot  
 a−b   b−c   c−a  1  −1  2π 1  
−1  1 
= sin sin  − sin−1  + sin  
 a−b  −1  b − c  −1  c − a  2   3 3  3  
= tan−1   + tan   + tan  
 1 + ab   1 + bc   1 + ca 
π π
∠A = ∠B = , ∠C =
= tan–1(a) – tan–1(b) + tan–1(b) 3 3
– tan–1(c) + tan–1(c) – tan–1(a) = 0 C C
Area ∆ABC = × sinBsinA
2 sinC

Sol 22: x2 + 5x – 49 = 0 ⇒α, β C2 sin2 A C2 sin2 A C2


= × = × = tanA
cot(cot–1α + cot–1β) 2 sinC 2 sin(180º −2A) 4

(cot cot −1 α )(cot cot −1 β) − 1 (6)2 (3)1/2


= = 3 = 27
(cot cot −1 α ) + (cot cot −1 β) 4

αβ − 1 −1 − 49 x 1  1 
= = = 10 Sol 25: (i) f(x) = cos–1x+cos–1 + 3 − 3x2 , x∈ , 1 
α+β −5 2 2  2 
1 3 
Sol 23: q1 + q2 + q3 = π f(x) = cos–1x + cos–1  (x) + (1 − x2 ) 
1 1  1   2 2 
tan–1   + tan–1  + k  + tan–1  + 2k  = π
2 2  2  1
= cos–1x + cos−1 x − cos−1
2
⇒ Use the formula
1  1
 x+y+z  x∈  , 1  cos–1x < cos–1
tan−1 x + tan−1 y + tanz =
tan−1   2  2
 1 − xy − yz − zx 
1 1 π
= cos–1x + cos–1 – cos–1x = cos–1 =
1 1  1  11 1
1  1 1  2 2 6
+  + k  +  + 2k  =   ++k k ++  ++2k2k 
2 2  2  22 22    22 
2 0 . 6 0 | Inverse Trigonometric Functions

 1 + x2 − 1  2x
(ii) f(x) = tan–1   Sol 29 : (i) f(x) = cos–1
 x  1+x
 
2x
Put x = tan y We know that –1 ≤ ≤1
y 1+x
2sin2
1 − cos y 2 2x 2x
f(x) = tan –1
= tan−1 + 1 ≥ 0 and –1≤0
sin y y y 1+x 1+x
2sin cos
2 2
3x + 1 x −1
y 1 ≥ 0 and ≤0
= tan = tan−1 x x +1 x +1
2 2
 −1 
x∈ (–∞,–1) ∪  , ∞  and x∈ (–1, 1]
Sol 26 : f(x) = cot–1(x2 + 4x + a2 – α)  3 
⇒ f(x) is onto function so  −1 
So x∈  ,1
⇒ x2 + 4x + a2 – a ≥ 0 3 

⇒ (x + 2) 2 + a2 – α – 4 ≥ 0 1 + x2
(ii) f(x) = cos(sinx) + sin–1
2x
⇒ (a2 – α – 4) should be zero
cos(1) ≤ cos(sinx) ≤ 1
⇒ a2 – α – 4 = 0
1 + x2 11 
1 ± 17 and =  + x ≥ 1
⇒α= 2x 2 x 
2
for x > 0
1 2
Sol 27 : LHS = tan–1 + tan–1
4 9 11 
or =  + x  ≤ 1 for x < 0
 1 2 2 x 
 +  1 y
= tan–1  4 9  = tan–1   = tan–1    x −3
1 − 2  2
  2 (iii) f(x) = sin–1   – log10(4 – x)

 36 
  2 
1 1 then x = 1, –1.
+
1 −1 1  1 −1 2 2 x −3
=  2 tan  = tan –1 ≤ ≤ 1 ⇒ 1 ≤ x ≤ 5; 4 – x > 0
2 2 2 1 2
1−
4
⇒ x< 4 so x ∈ [1, 4]
1 4 1 3
= tan−1 = cos−1
2 3 2 5 (iv) f(x) = sin–1[x(x + 2)]
1 −1 4 We can write here that –1 ≤ x2 + 2x ≤ 1
= sin = RHS
2 5 x∈ [–(1 + 2 ) , ( 2 – 1) ]

Sol 28 : LHS = 1 − sinx


(v) f(x) = + cos–1[1 – {x}]
log5 (1 − 4x2 )
x x x x x x x x
sin2 + cos2 + 2sin cos + sin2 + cos2 − 2sin cos
cot –1
−1 2 2 2 2 2 2 2 2 1 – sin x ≥ 0 ⇒ sin x ≤ 1
cot
x x x x x x x x
sin2 + cos2 + 2sin cos − sin2 + cos2 − 2sin cos and 1 – 4x2≠ 1. Also x≠ 0 and1 – 4x2> 0
2 2 2 2 2 2 2 2

x x x x  1  1  1 1
sin + cos + cos − sin ⇒  x −   x +  < 0 ⇒ x∈  − , 
−1 2 2 2 2  2  2  2 2
= cot
x x x x
sin + cos − cos + sin  1 1
2 2 2 2 So domain x ∈  − ,  – {0}
 2 2
x x
= cot–1cot =
2 2  3 − 2x 
(vi) f(x) = 3 − x + cos–1  
 5 
M a them a ti cs | 20.61

+ log6(2|x| – 3) + sin–1(log2x) Sol 2: (C) We have


1 S1 = ∑ x1 = sin 2β
⇒ –1 ≤ log2x ≤ 1 ⇒ ≤x≤2
2 S2 = ∑ x1x2 = cos 2β
3
and 2 | x | – 3 > 0 ⇒ | x | > S3 = ∑ x1x2x3 = cos β
2
S4 = x1x2x3x4 = ‒ sin β
 −3   3  3 − 2x
So now x∈  −∞ ,  ∪  , ∞  and –1≤ ≤1 4 S1 − S3
2 2 5
    So that ∑ tan-1 xi = tan‒1
1 − S2 + S 4
i=1
⇒ –8 ≤ – 2x ≤ 2 ⇒ 4 ≥ x ≥ 1
sin2β − cos β cos β (2sin β − 1)
Now we have x∈ [1, 4] and 3 – x ≥ 0 = tan−1 = tan−1
1 − cos2β − sin β sin β (2 sin β − 1)
⇒ x≤ 3
3  1  π 
So domain will be x ∈  , 2 = tan−= (cot β) tan−1  tan  − β  
2     2 
 3 
(vii) f(x) = log10(1 – log7(x2 – 5x + 13) + cos–1   π
 2 + sin 9π x  ⇒ −β
  2
 2 
3
We can write here –1 ≤ ≤ +1 Sol 3: (B) f(x) = cot–1 log4/5(5x2 – 8x + 4)

2 + sin x
2 4
9π 5x2 – 8x + 4 ≥
⇒ sin x = +1 5
2
log4/5(5x2 – 8x + 4) ≤ 1
9π π 4n + 1
⇒ x = 2nπ + ⇒x= π 
2 2 9 f(x) ∈  , π 
2 2 4 
 5 25  5  27
⇒ x – 5x + 13 =  x −  + 13 –
2
= x −  + <7
 2 4  2 4
Sol 4: (C) (1 + x) cosy – x2 = 0
This gives x∈ (2, 3)
x2
21 25 y = cos –1

So the domain would be x = , 1+x


9 9
x2
x
sin−1   ⇒ –1 ≤ ≤1
(viii) f(x) = e  2  + tan–1  x − 1  + ln x − [x] 1+x
 
2  x2 x2 + x + 1
≤ ‒1 ⇒ ≤0
Now x – [x] ≠ 0⇒x ∉ I 1+x 1+x
x x2 − x − 1
and –1≤ ≤ 1 ⇒ –2 ≤ x ≤ 2
2 ≤0
1+x
So the Domain will be (–2, 2) – {–1, 0, 1}
2
 1 5
x −  −
 2 4
≤0
Exercise 2 x +1

Single Correct Choice Type 1 − 5 1 + 5 


x∈  , 
 2 2 
Sol 1: (C) x2 – 4x + 5 > sin–1(sin 3) + 2cos–1(cos2) – π
(x – 2) 2 + 1 >π – 3 + 4 – π Sol 5: (B) 4(tan–1x) 2 – (tan–1x) – 3 ≤ 0
(x – 2) 2 + 1 ≥ 1 1  1 1 3
(tan–1x) 2 – 2  tan−1 x  + – – ⇒0
Always true except {2} 8  64 64 4
2 0 . 6 2 | Inverse Trigonometric Functions

 −1 1 49
2 π −1  1 − x2
cos  + sin 1 − x2  = cos cot–1
tan x −  − ≤0  2  x
 8 64
 3 1 − x2
(tan–1x – 1)  tan−1 x +  ≤ 0 –sin sin–1 1 − x2 = cos cot–1
 4 x
3 1 − x2
− ≤ tan–1x ≤ 1 – 1 − x2 = coscot–1
4 x
3 π If x > 0 then it won’t satisfy except 1.
– tan–1   ≤ x ≤
4 4
If x < 0 then it will satisfy.

Sol 6: (B) sec–1[–sin2x] is defined only if x∈ [–1, 0) ∪ {1}

[–sin2x] = 1, –1
Sol 9: (C) f(x) = cosec−1 log3− 4 sec x 2
[–sin x] = –1 when x ∉ n π
2
1 −2 sec x

sec–1(–1) = π 3 − 4 sec x
2≥ >1
1 − 2sec x
So area bounded
 π π
⇒x∈  2nπ − ,2nπ +  – {2np}
 2 2 

 π
Range ∈  0, 
 2
- 
Sol 10: (A) [sin–1x] = [cos–1x]
π
π  x3  π π
2 − ≤ sin−1 x ≤
= ∫ ( π − x )dx = 

πx − 
3  2 2
− π  − π
π π
1~ ; ~1. 6
π π π π 4 3 2
= π π– + π π – = π π
3 3 3 [sin–1x] = –2, –1, 0, 1
0 ≤ cos–1x ⇒ π ⇒ [cos–1x] = 0,1,2
Sol 7: (B) We have from the given equation
so[sin–1x] = [cos–1x] = 0 or 1
(a + b) x π (c + d) x x∈ [cos 1, sin 1]
tan −1 = − tan−1
x − ab 2
2
x2 − cd

(a + b) x (c + d) x Multiple Correct Choice Type


⇒ tan −1 cot −1
=
2
x − ab x2 − cd  2e + 4 
Sol 11: (A, B) (A) cos−1  ln 
 3 
x2 − cd
= tan−1
(c + d) x 2e + 4  2e + 4 
~ 3 > e ⇒ ln  > 1
3  3 
⇒ (x2 − ab) (x2 − cd) =(a + b) (c + d) x2
meaning less because
⇒ x 4 − x2 ∑ ab + abcd =
0  2e + 4 
cos−1  ln  is not defined.
 3 
 1 − x2 
Sol 8: (C) sin–1 1 − x2 + cos–1x = cot–1   – sin–1x π π
(B) In cosec–1   , < 1
 x 
  4 4
2 π 1 − x2 π
sin–1 1 − x + = cot–1 cosec–1   not defined
2 x 4
x∈ [–1, 1] – {0}
M a them a ti cs | 20.63

π L. H. S.
(C) cot–1   defined
2 π π π
= –cos–1x+ –cos–1y+ –cos–1z
(D) sec–1(π) defined 2 2 2
3π π
= – (cos–1x + cos–1y + cos–1z) =
2 2
−1 4 4
Sol 12: (A, B, C) Let cos   = α, that is, cos α = ,
5
  5 Match the Columns
2
5 3  Sol 15: A → s; B → p; C → r; D → q
so that tan
= α  =−1 ( 0 < α < π and
cos α > 0) 4 4
 x 
(A) f(x) = sin−1  
2  1+ | x | 
tan α +
 4 2 3
And tan  cos−1 + tan−1  =  π π
 5 3 2 Range f(x) ∈  − , 
1− tan α ⋅
3  2 2
3 2
+ x
17 a (B) g(x) = cos–1.
= 4 3= = (given) 1+ | x |
2 3 6 b
1− ⋅
3 4 x x
if x ≥ 0 ⇒ 0 ≤ <1
1+ | x | 1+x
So, a = 17, b = 6, a +b = 23, a - b = 11 and 3b = a + 1
x
x≤ 0 ⇒ 0 ≥ > –1
Sol 13: (B, C) (A) y = 2
1−x 2 1−x

⇒y +x =1
4 2 the Range f(x) ∈ (0, x)
Not circle x
(C) h(x) = tan–1
(B) y = sin(cos (1 – x) )
–1 1+ | x |

y= 1 − (1 − x)2  π π
Range f(x) ∈  − , 
 4 4
Half circle for y > 0
 x 
(C) y2 = (sin cos–1x) 2 (D) k(x) = cot–1  
 1+ | x | 
y2 = (1 – x2) ⇒ y2 + x2 = 1
 π 3π 
Which is a circle Range f(x) ∈  , 
4 4 
(D) y = sin–1cos2y
Not a circle
Previous Years’ Questions
Sol 14: (A, B) cos x + cos y + cos z = π
–1 –1 –1

Sol 1: A → p; B → q; C → p; D → s
cos–1(xy – 1 − x2 1 − y 2 ) = π – cos–1z
(A) If a = 1, b = 0, then sin–1x + cos–1y = 0
Taking cosine of both sides ⇒ sin–1x = –cos–1y

xy – 1 − x2 1 − y 2 = –z ⇒ x2 + y2 = 1

(B) If a = 1 and b = 1, then


(xy + z) 2 = (1 – x2) (1 – y2)
π
x2y2 + z2 + 2xyz= 1 – x2 – y2 + x2y2 sin–1x + cos–1y + cos–1xy =
2
x2 + y2 + z2 = 1 – 2xyz ⇒ cos–1x – cos–1y = cos–1xy
π
(B) sin–1x + sin–1y + sin–1z = ⇒ xy + 1 − x2 1 − y2 =
xy
2
2 0 . 6 4 | Inverse Trigonometric Functions

⇒ (x2 – 1)(y2 – 1) = 0 Sol 3: Let f(x) cos(2cos–1x + sin–1x)

(C) If a = 1, b = 2, then  π  π
= cos  cos−1 x +   cos−1 x + sin−1 x =
π  2  2
sin–1x + cos–1y + cos–1(2xy) =
2 = –sin(cos–1x)
⇒ cos x – cos y = cos (2xy)
–1 –1 –1

⇒ f(x) = –sin  sin−1 1 − x2 


 
⇒ xy + 1 − x2 1 − y2 =
2xy
1  1 
⇒ x2 + y2 = 1 ⇒ f   = –sin  sin−1 1 − 
5  52 

(D) If a = 2, b = 2, then
 2 6 2 6
π = –sin  sin−1 = −
sin (2x) + cos (y) + cos (2xy) =
–1 –1 –1
 
5  5
2 
⇒ cos–1(2x) – cos–1(y) = cos–1(2xy)
Sol 4: LHS = cos tan–1[sin(cot–1x)]
2 2
⇒ 2xy + 1 − 4x 1−y =
2xy
  1 
⇒ (4x2 – 1) (y2 – 1) = 0 = cos tan–1 sin  sin−1 
  
1 + x2  
π   x2 + 1
Sol 2: Given than, tan–1 2x + tan–1 3x = −1 1
4 = cos  tan = = RHS
  x2 + 2
 2x + 3x  π  1 + x2 
⇒ tan−1  =
 1 − 6x2  4 a c 2
Sol 5: sin 2C + sin 2A = (a cos C + c cos A)
5x c a 2R
⇒ = 1 ⇒ 6x2 + 5x – 1 = 0 b
1 − 6x2 = = 2sin B= 2 sin 60o= 3
1 R
⇒ (x + 1) (6x – 1) = 0 ⇒ x = –1 or
6
π 3π 3π 5π
But x = –1 does not satisfy the given equation. Sol 6: (B, C, D) < α < π, π < β < ⇒ < α+β <
2 2 2 2
1
∴ We take x= ⇒ sin β > 0; cos α < 0
6
⇒ cos(α + β) > 0
2017-18 100 &
op kers
Class 12 T
By E ran culty
-JE Fa r
IIT enior emie .
S fP r es
o titut
Ins

MATHEMATICS
FOR JEE MAIN & ADVANCED
SECOND
EDITION

Exhaustive Theory
(Now Revised)

Formula Sheet
9000+ Problems
based on latest JEE pattern

2500 + 1000 (New) Problems


of previous 35 years of
AIEEE (JEE Main) and IIT-JEE (JEE Adv)

5000+Illustrations and Solved Examples


Detailed Solutions
of all problems available

Topic Covered Plancess Concepts


Tips & Tricks, Facts, Notes, Misconceptions,
Methods of Differentiation Key Take Aways, Problem Solving Tactics
and Applications of
PlancEssential
Derivatives
Questions recommended for revision
21. METHODS OF
D I F F E R E N T I AT I O N
A N D A P P L I C AT I O N S
O F D E R I VAT I V E S

1. INTRODUCTION
The rate of change of one dependent quantity with respect to another dependent quantity has great importance.
E.g. the rate of change of displacement of a particle with respect to time is called its velocity and the rate of change
of velocity is called its acceleration. The rate of change of a quantity ‘y’ with respect to another quantity ‘x’ is known
as the derivative or differentiable coefficient of ‘y’ with respect to ‘x.’
According to the first principle of calculus, if y = f(x) is the derivative function, then the derivative of f(x) with respect
to x is given by:
dy f(x+h)-f(x)
f’(x) = = lim
dx h→0 h
Note: y’, y1, Dy can also be used to denote the derivative of y with respect to x. Differentiation is the process of
finding the derivative of a function. ⇒ sin β > 0; cos α < 0

2. DERIVATIVES OF SOME STANDARD FUNCTIONS


Different types of differentiation formulae

d d n
(a) (constant) = 0 (f) (x ) = nxn–1
dx dx

d x d x
(b) (e ) = ex (g) (a ) = ax loge a
dx dx

d 1 d 1
(c) (loge x) = (h) (logax) =
dx x dx xloge a

d d
(d) (sin x) = cos x (i) (cos x) = – sin x
dx dx

d d
(e) (tan x) = sec2x (j) (cot x) = – cosec2x
dx dx
2 1 . 2 | Methods of Differentiation and Applications of Derivatives

d d
(k) (sec x) = sec x tan x (z) (cosec x) = – cosec x cot x
dx dx

d 1 d 1
(l) (sin–1x) = , – 1 < x < 1 (aa) (cos–1x) = – ,– 1 < x < 1
dx 1−x 2 dx 1 − x2
d 1 d 1
(m) (tan–1x) = , x ∈ R (ab) (cot–1x) = – ,∀x∈R
dx 1 + x2 dx 1 + x2
d 1 d −1
(n) (sec–1x) = | x | > 1 (ac) (cosec–1x) = | x | >1
dx | x | x2 − 1 dx | x | x2 − 1
d d
(o) (sinh x) = coshx (ad) (coshx) = sinhx
dx dx

d d
(p) (tanh x) = sech2x (ae) (cothx) = –cosech2x
dx dx

d d
(q) (sechx) = – sechx tanhx (af) (cosechx) = –cosechx cothx
dx dx

d 1 d 1
(r) (sinh–1x) = ,∀x∈R (ag) (cosh–1x) = ,|x|>1
dx 1+x 2 dx 2
x −1
d 1 d 1
(s) (tanh–1x) = ,x±1 (ah) (coth–1x) = ,x≠±1
dx 1 − x2 dx 2
x −1
d 1 d −1
(t) (sech–1x) = – ,|x|<1 (ai) (cosech–1x) = ,∀ x ∈ R
dx | x | 1 − x2 dx | x | x2 + 1
d ax
(u) (e sin bx) = eax (a sin bx + b cos bx) = a2 + b2 eaxsin (bx + tan–1 b/a)
dx

d ax
(v) (e cos bx) = eax (a cos bx – b sin bx) = a2 + b2 eaxcos (bx + tan–1 b/a)
dx

d x d 1
(w) |x|= (x ≠ 0) (aj) log | x | = , (x ≠ 0)
dx |x| dx x

d
(x) [x] = 0, ∀ x ∈ –I (where [ . ] denotes greatest integer function)
dx

d
(y) {x} = 1, ∀ x ∈ R (where { . } denotes fractional part function)
dx

PLANCESS CONCEPTS

If the function is continuous, you do not have to apply the first principle method to check differentiability.
You can go directly for dy/dx and check whether dy/dx exists on both the left and right sides and are
equal. If dy/dx does not exist for either one side or both the sides or if both the derivatives exist, but are
not equal or finite, then the function is not differentiable.
E.g. Let y = sin(x) be a continuous function. Check differentiability at x = π/2.On checking for dy/dx =
cos(x) on both the right and left sides, it is found to be equal and finite. Hence, y = sin(x) is differentiable
at x = π/2.
M a them a ti cs | 21.3

PLANCESS CONCEPTS

Misconception:
(i) In dy/dx, dy or dx does not exist individually.
dy 1 dy dx
(ii) = only if both and exist.
dx dx dx dy
dy
Rohit Kumar (JEE 2012, AIR 79)

3. PRODUCT RULE
d dv du d d(w) d(v) d(u)
(a) (uv) = u +v (b) (uvw) = uv + uw + vw
dx dx dx dx dx dx dx

4. DIVISION RULE
 du   dv 
v   – u 
d u  dx   dx  , where v ≠ 0 (known as the quotient rule)
 =
dx  v  v 2

5. CHAIN RULE
d dy dy dt
(f(g(x))) = f’(g(x)) . g’(x) or = .
dx dx dt dx
d d d
Note: (a) (f(x) ± g(x)) = (f(x)) ± (g(x)), on condition that both f’(x) and g’(x)exist
dx dx dx
d d
(b) (k f(x)) = k (f(x)), where k is any constant
dx dx

x 4 + x2 + 1 dy
Illustration 1:If y = ; = px + q, find p and q. (JEE MAIN)
2
x + x +1 dx

Sol: Differentiate and compare.

y=
(x2 + 1)2 − x2
y=
(( x + 1) + x ) ( x
2 2
+1− x ) = x + 1 – x ⇒ dy = 2x – 1 ⇒ p = 2 and q = –1
2

x2 + x + 1 x2 + x + 1 dx

((x1 + 1) + x)(x2 + 1 − x)
y=
x2 + x + 1

x3 + 2x dy
Illustration 2: If y = , then find . (JEE MAIN)
e x dx

Sol: Differentiate

ex (3x2 + 2x ln2) − (x3 + 2x )ex (3x2 + 2x ln2) − (x3 + 2x )


y’ = =
e2x ex
2 1 . 4 | Methods of Differentiation and Applications of Derivatives

tan–1 x − cot −1 x  dy 
Illustration 3: If y = , find   . (JEE MAIN)
−1
tan x + cot −1
x  dx x =1

Sol: Differentiate and put x = 1.

y=
2
π
(
tan–1 x − cot −1 x )  π
......  tan−1 + cot −1 x =


2
dy 2 2 4  dy  4 2
⇒ = + = ⇒   = =
dx 2 2
π(1 + x ) π(1 + x ) 2
π(1 + x )  dx x =1 2 × π π

Illustration 4: Differentiate the following functions with respect to x: (JEE MAIN)

1 2x
(i) 3x + 2 + (ii) esec + 3cos–1x (iii) log7(log x)
2
2x + 4
1
Sol: (i) Let y = 3x + 2 + = (3x + 2)1/2 + (2x2 + 4)–1/2
2
2x + 4
1 −1
dy 1 −1 d  1 2
−1 d
= (3x + 2) 2 (3x + 2) +  −  (2x + 4) 2 (2x2 + 4)
dx 2 dx  2 dx

1 3
1 − 1 − 3 2x
= (3x + 2) 2 . (3) –   (2x2 + 4) 2 . 4x = –
2 2 2 3x + 2 (2x + 4)3/2
2

2x
(ii) Let y = esec + 3 cos–1x

dy 2 d  1  2  d   1 
= esec x . (sec2x) + 3  −  = esec x .  2sec x (sec x)  + 3  − 
dx dx    dx   
 1 − x2   1 − x2 

2x  1  2  1 
= 2 sec x (sec x tan x) esec + 3−  = 2 sec2 x tan x esec x – 3  + 
   
 1 − x2   1 − x2 

log(logx)
(iii) Let y = log7(log x) = (using change of base formula)
log7

dy 1 d 1 1 d 1
= (log(logx)) = . (log x) =
dx log7 dx log7 logx dx xlog7logx

dy 1
Illustration 5: Find , if y =3 tan x + 5 logax + x – 3ex + . (JEE MAIN)
dx x

Sol: Chain rule.


1
We have y = 3 tan x + 5 logax + x – 3ex +
x
dy d  x 1
Thus, =  3tanx + 5loga x + x − 3e + 
dx dx  x
1
d d d d d 1 5 1 −
= (3 tan x) + (5 loga x) + ( x)– (3ex) +   = 3 sec x + (logea) + x 2 – 3e – x .
2 -1 x –2
dx dx dx dx dx  x  x 2
M a them a ti cs | 21.5

Illustration 6: Let f, g and h be differentiable functions. If f(0) = 1, g(0) = 2, h(0) = 3 and the derivative pairwise
products at x = 0 are (fg)’ (0) = 6, (gh)’(0) =4 and (hf)’ (0) = 5, then compute the value of (fgh)’ (0). 
 (JEE ADVANCED)
Sol: Product rule
(fgh)’ = f’gh + fhg’ + fgh’  …….(i)
(fg)’ (0) = 6 ⇒ (fg’ + gf’) (0) = 6
(gh)’ (0) = 4 ⇒ (gh’ + hg’) (0) = 4
(hf)’ (0) = 5 ⇒ (hf’ + fh’) (0) = 5
1 1
(fgh)’ = (2f’gh + 2fg’h + 2fgh’) = (f’gh + f’gh + fg’h + fg’h + fgh’ + fgh’)
2 2
1 1
= [h(f’g + fg’) + g(f’h + fh’) + f(g’h + gh’)] = [h(fg)’ + g(fh)’ + f(gh)’]
2 2
1 1
⇒ (fgh)’ (0) = [(3)(6) + (2)(5) + (1)(4)]= [18 + 10 + 4] = 16
2 2

6. TRIGONOMETRIC TRANSFORMATIONS
In case of inverse trigonometric functions, it becomes very easy to differentiate a function by using trigonometric
transformations.Given below are some important results on trigonometric and inverse trigonometric functions.

2 tanx
(a) sin2x = 2 sin x cos x =
1 + tan2 x
1 − tan2 x
(b) cos 2x = 2 cos2 x –1 = 1 – 2 sin2 x = cos2 x – sin2 x =
1 + tan2 x
2 tanx
(c) tan 2x = (n) sin 3x = 3 sin x – 4 sin3x
2
1 – tan x
3tanx − tan3 x
(d) cos 3x = 4 cos3x – 3 cos x (o) tan 3x =
1 – 3tan2 x
π
(e) sin–1 x + cos–1 x = tan–1 x + cot–1 x =sec–1 x + cosec–1 x =
2

(f) sin–1 (–x) = – sin–1 x (p) cos–1 (–x) = π – cos–1 x

(d) tan–1 (–x) = – tan–1 x (q) cosec–1 (–x) = – cosec–1 x

(h) cot–1 (–x) = π – cot–1 x (r) sec–1 (–x) = π – sec–1 x

2 2 2 2
(i) sin–1 x ± sin–1 y = sin–1 (x 1 − y ± y 1 − x ) (s) cos–1 x ± cos–1 y = cos–1 (xy  1 − x 1 − y )

x±y
(j) tan–1 x ± tan–1 y = tan–1 1  xy (t) 2sin–1 x = sin–1 (2x 1 − x2 ) (Be aware of ranges for ‘x’)

 2x   2x   1 − x2 
  –1   –1  
(k) 2cos x = cos (2x – 1)
–1 –1 2
(u) 2 tan x = tan
–1 –1
= sin =cos  2 
 1 − x2   1 + x2  1 + x 
π 1 − x 
(l) – tan–1x = tan–1  1 + x  (v) 3 sin–1x = sin–1 (3x – 4x3)
4  
3x − x3
(m) 3 cos–1x = cos–1 (4x3 – 3x) (w) 3 tan–1x = tan–1
1 − 3x2
2 1 . 6 | Methods of Differentiation and Applications of Derivatives

PLANCESS CONCEPTS

Some useful substitutions in finding derivatives are given below.

Sl. No. Function Substitution

(i) x = a sinθ or a cosθ


a2 – x2
(ii) x = a tan θ or a cotθ
x2 + a2
(iii) x = a sec θ or a cosecθ
x2 – a2
(iv) x = a cos 2θ
a–x
a+ x

(v) x2 = a2 cos 2θ
a2 – x2
a2 + x2

(vi) x = a sin2 θ
ax – x2

(vii) x = a tan2 θ
x
a+ x

(viii) x = a sin2 θ
x
a–x

(ix) x = a sec2 θ– b tan2 θ


(x – a)(x – b)

(x) x = a cos2 θ+ b sin2 θ


(x – a)(b – x)

Rohit Kumar (JEE 2012, AIR 79)

 1 + x2 + 1 
  dy
Illustration 7: If y = cot 
–1
x  , find dx .  (JEE MAIN)
 

Sol: Substitute a suitable trigonometric function in place of x and simplify.


Putting x = tan θ, we have
 sec θ + 1   1 + cos θ  θ 1
y= cot–1  tan θ  = cot–1  sin θ  = cot–1(cot θ/2) = = tan–1x
    2 2
dy 1 1
\ = .
dx 2 1 + x2
M a them a ti cs | 21.7

PLANCESS CONCEPTS

To differentiate a complex function, put x in some trigonometric form so that the function can be easily
differentiated and then put back x in the form of an inverse trigonometric function.
E.g. Find the derivatives of sec–1 [1/(2x2 – 1)] with respect to 1 − x2 at x = 1/2.
Sol. Putting x = cosθ, we get
1
u = sec–1 = sec–1(sec2θ) = 2θ and y = 1 − x2 = sinθ
2cos2 θ − 1

du 2 2 du
∴ u = 2sin–1y ⇒ = = Thus, =4
dy 1−y 2
x 2 dy x =1/2

Ravi Vooda (JEE 2009, AIR 71)

a−x  dy
Illustration 8: If y = (a − x)(x − b) – (a – b) tan–1  x − b  , find . (JEE ADVANCED)
  dx

Sol: Use Substitution to simplify the given expression and then differentiate.
Let x = a cos2θ + b sin2θ
∴ a – x = a – a cos2θ – b sin2 θ = (a– b) sin2θ … (i)
x – b = a cos θ + b sin θ – b = (a – b) cos θ
2 2 2
… (ii)
∴ y = (a – b) sinθ cosθ – (a – b) tan–1 (tanθ)
(a − b)
y= sin2θ – (a – b) θ
2
 dy 
 
dy dθ (a − b)cos2θ − (a − b) 1 − cos2θ a−x 
Then, =   = = = tan θ=   [From (i) and (ii)]
dx  dx  (b − a)sin2θ sin2θ  x −b
 
 dθ 

7. LOGARITHMIC DIFFERENTIATION
If differentiation of an expression is done after taking log on both the sides, then it is known as logarithmic
differentiation. This method is used when a given expression is in one of the following forms:
(a) (f(x))g(x)
Let y = (f(x))g(x)
Taking logarithm of both the sides, we get log y = g(x) log f(x)
Differentiating with respect to x, we get

1 dy 1 dy  g(x) 
. = g(x) . . f’(x) + log f(x) . g’(x) ⇒ = y f '(x) + logf(x).g'(x) 
y dx f(x) dx  f(x) 

dy  g(x) 
⇒ = (f(x))g(x)  f(x) f '(x) + logf(x).g'(x) 
dx  
2 1 . 8 | Methods of Differentiation and Applications of Derivatives

Short method: The derivative of [f(x)]g(x)can be directly written as:

d  d 
(f(x))g(x) = f(x)g(x)  {g(x)logf(x)} 
dx  dx 
(b) Product of three or more functions
 f '(x) g'(x) h'(x) 
If y = f(x).g(x).h(x), then y’ = f(x).g(x).h(x).  + + 
 f(x) g(x) h(x) 

100
Illustration 9: If f(x) = ∏ (x− n)n(101−n) , find f’(101)/f(101). (JEE ADVANCED)
n=1

Sol: Use logarithms followed by differentiation.


100
f(x) = ∏ (x− n)n(101−n)
n=1

100
f '(x) 100 n(101 − n)
ln f(x) = ∑ n(101 − n)ln(x− n) ⇒ =∑
f(x) n=1 x − n
n=1

100
f '(101) n(101 − n) 100x101

f(101)
= ∑ = = 5050
n=1 101 − n 2

Illustration 10: Find the derivative of (sin x)cosx. (JEE MAIN)

Sol: Take logarithms on both sides and differentiate.

d d 
(sin x)cos x = (sin x)cos x  dx {cos xlog(sinx)}  = (sin x)cos x [cos x . cot x – sin x . log sin x]
dx  

Illustration 11: Find the derivative of xx with respect to x.  (JEE MAIN)

Sol: Use logarithms to find the derivative.


Let y = xx or y = xx
log y = x log x = ex ln x

1 dy 1 dy d xlnx d
= x   + log x (or) = e = ex ln x dx (x ln x)
y dx x dx dx

dy  1 
= xx(1 + log x) (or) = ex ln x x. + lnx1
dx  x 
d x
∴ = x x x (1 + loge x ) (or) = xx(1 + ln x)
dx
d x
Hence (x ) = xx(1 + ln x)
dx

Illustration 12: Differentiate xsin x with respect to x. (JEE MAIN)

Sol: Similar to the previous illustration.


First method: Let y = xsin x
M a them a ti cs | 21.9

∴ log y = log xsinx = sinx log x


1 dy sinx
Differentiating we get, = + cos x log x
y dx x

dy  sinx 
∴ =  + cos xlogx  xsinx
dx  x 
Second Method: y = xsinx = esinx log x

dy  sinx   sinx 
Therefore, = esin x log x  x + cos xlogx  =  + cos xlogx  xsinx
dx    x 

cos−1 ( x +1 )
Illustration 13: Differentiate e with respect to x.  (JEE ADVANCED)

Sol: Similar to the previous illustration.


−1 (x +1) dy −1 dy
Let y = ecos Then, = ecos (x +1) . [cos–1(x+1)]
dx dx

−1 (x +1) −1 d −1 −1
= ecos . . (x + 1) = ecos (x +1)
1 − (x + 1)2 dx 1 − (x + 1)2

Illustration 14: Differentiate xsinx, x > 0, with respect to x. (JEE ADVANCED)

Sol: Let y = xsin x


Taking logarithm on both the sides, we get log y = sin xlog x

1 dy d d
Therefore, . = sin x (log x) + logx (sin x)
y dx dx dx

1 dy 1 dy  sinx   sinx 
. = (sin x) + log x cos x = y + cos xlogx  = xsin x  x + cos xlogx 
y dx x dx  x   

Illustration 15: Find f’(x), if f(x) = (sin x)sin x, for all 0 < x <π. (JEE ADVANCED)

Sol: Similar to the previous illustration.


log y = log(sin x)sin x = sin x log (sin x)

1 dy d 1 d
Then, = (sin x log (sin x)) = cos x log (sin x) + sin x . . (sin x)
y dx dx sinx dx

= cos x log (sin x) + cos x = (1 + log (sin x)) cos x


dy
⇒ = y((1 + log (sinx)) cos x) = (1 + log (sinx)) sin xsinx cosx
dx

−1 x
Illustration 16: Differentiate xcos with respect to x. (JEE MAIN)
−1 x
Sol: (i) Let y = xcos
−1 x.ln x
Then, y = ecos
Differentiating both the sides with respect to x, we get
2 1 . 1 0 | Methods of Differentiation and Applications of Derivatives

dy −1 d dy −1  d d 
= ecos x.log x (cos–1x . log x) ⇒ = xcos x logx. (cos−1 x) + cos−1 x. (logx)
dx dx dx  dx dx 
dy −1  − logx cos−1 x 
⇒ = xcos x  + 
dx  2 x 
 1−x
−1 x
(ii) Let (sinx)cos
−1 x.log sin x
Then, y = ecos
Differentiating both the sides with respect to x, we get
dy −1 d
= ecos x.logsin x (cos–1x . log sin x)
dx dx

dy −1  d d 
⇒ = (sinx)cos x cos−1 x. (log sin x) + log sinx (cos–1 x)
dx  dx dx 

dy −1 
 1  −1  
⇒ = (sinx)cos x cos−1 . cos x + log sin x  
dx  sinx  2 
 1−x 

dy −1 
 logsinx 
⇒ = (sinx)cos x cos−1 x . cot x − 
dx  1 − x2 

dy sin x..... ∞
Illustration 17: Find , if y = (sinx)sin x (JEE ADVANCED)
dx 

Sol: Write the given expression as y = (sin x)y and proceed.


We have y = (sin x)y , Therefore, log y = y log sin x
Differentiating both the sides with respect to x, we get

1 dy d dy cos x dy
=y (log sin x) + (log sin x) =y + log sin x
y dx dx dx sinx dx

1  dy dy y 2 cot x y 2 cot x
⇒  − logsinx  = y cot x or = =
y  dx dx 1 − y logsinx 1 − log y

8. DIFFERENTIATION OF IMPLICIT FUNCTION


If in an equation, both x and y occur together, i.e. f(x, y) = 0, and the equation cannot be solved for either x or y,
then x (or y) is called the implicit function of y (or x).
E.g. x3 + y3 + 3axy + c = 0, xy + yx = ab, etc.
Working rule for finding the derivative
First method:
(a) Every term of f(x, y) = 0 should be differentiated with respect to x.
(b) The value of dy/dx should be obtained by rearranging the terms.
Second method:
dy −∂f / ∂x ∂f ∂f
If f(x, y) = constant, then = , where and are the partial differential coefficients of f(x, y) with
dx ∂f / ∂y ∂x ∂y
respect to x and y, respectively.
M a them a ti cs | 21.11

Note: Partial differential coefficient of f(x, y) with respect to x can be defined as the ordinary differential coefficient
of f(x, y) with respect to x keeping y constant.
∂z ∂x
E.g. z = x2 y ⇒ = x2 , = 2xy
∂y ∂x

sinx dy (1 + y)cos x + y sinx


Illustration 18: If y = , then prove that = . (JEE ADVANCED)
cos x dx 1 + 2y + cos x − sinx
1+
sinx
1+
cos x
1+
1 + ....to ∞

Sol: Write the R.H.S. in terms of x and y. Then differentiate the equation on both sides.
sinx (1 + y)sinx
We have, y = = ⇒ y + y2 + y cos x = (1 + y) sin x
1 + ( (cos x) / (1 + y) ) 1 + y + cos x
On differentiating both the sides with respect to x, we get
dy dy dy dy
+ 2y + cos x – y sin x = sin x + (1 + y) cos x
dx dx dx dx
dy dy (1 + y)cos x + y sinx
⇒ {1 + 2y + cos x – sin x} = (1 + y) cos x + y sin x ⇒ =
dx dx 1 + 2y + cos x − sinx

1
Illustration 19: If f(x) = x + , then compute the value of f (100) . f’(100). (JEE MAIN)
1
2x +
1
2x +
2x + ......∞
1
Sol: Same as above y – x =
1
2x +
1
2x +
2x + ......∞
1
⇒ y–x= ⇒ (y – x) (x + y ) = 1 ⇒ y2 – x2 = 1
2x + y − x

⇒ (f(x))2 = 1 + x2 ⇒ 2(f(x)) × f’(x) = 2x


⇒ f(100) . f’(100) = 100

(lnx )(lnx )∞ dy
(
Illustration 20: If y = (lnx)lnx ) , then find
dx
.(JEE MAIN)

Sol: Same as above


ln y = y ln(lnx)

1 y 1  y  1 − yln(lnx)  y
× y’ = + ln(lnx).y ' ⇒ y’  − ln(lnx)  = ⇒ y’   =
y xlnx  y  xlnx  y  xlnx
y2
⇒ y’ =
(xlnx)(1 − ln(lnx)y)

y dy x+y
Illustration 21: If log (x2 + y2) = 2 tan–1  x  , then prove that = .(JEE ADVANCED)
  dx x−y

Sol: Differentiating both the sides of the given relation with respect to x,
2 1 . 1 2 | Methods of Differentiation and Applications of Derivatives

d d  −1  y  
We get, [log(x2 + y2)] = 2 tan   
dx dx   x  
 dy 
1 d 1 d y 1  dy  x2  x dx − y.1 
⇒ . (x2 + y2) = 2 . .   ⇒ 2x + 2y  = 2.  
x2 + y 2 dx 1 + (y / x)2 dx  x  x2 + y 2  dx  x2 + y 2  x2 
 
 dy   dy  dy dy dy dy x+y
⇒ 2 . x + y  = 2 x − y ⇒ x + y =x –y ⇒ (y – x) = – (x + y) ⇒ =
 dx   dx  dx dx dx dx x−y

9. DIFFERENTIATION OF PARAMETRIC FORM


x and y are sometimes given as functions of a single variable, E.g. x = φ(t) and y = ψ(t) are two functions, where t
is a variable. Then in such cases, x and y are called parametric functions or parametric equations and t is called the
dy
parameter. To find in parametric functions, the relationship between x and y should be obtained by eliminating
dx
the parameter t and then it should be differentiated with respect to x. However, it is not convenient to eliminate the
dy dy dy / dt
parameter every time. Therefore, can also be found by using the formula = .
dx dx dx / dt
dy
E.g. If x = a(θ + sinθ) and y = a(1 – cosθ), then find .
dx
dx dy
Sol: Given that = a(1 + cosθ), = a(sinθ)
dθ dθ
Therefore, x= 0(θ + sin θ). y= a(1 − cos θ)
dy
dy asin θ θ
= dθ = = tan
dx dx a(1 + cos θ) 2

dy
Note: It may be noted here that can be expressed in terms of the parameter only without directly involving the
main variables x and y. dx

dy
Illustration 22: Find , if x = a cosθ and y = a sinθ. (JEE MAIN)
dx
Sol: Differentiate the two equations w.r.t. θ and eliminate θ.
Given that x = a cos θ and y = a sinθ
dx dy
Therefore, = – a sinθ, = a cosθ.
dθ dθ
dy
dy acos θ
Hence, = dθ = = – cotθ.
dx dx −asin θ

dy π
Illustration 23: If x = a sec2θ and y = a tan3θ, where q∈ R, find at θ = . (JEE MAIN)
dx 8
Sol: Differentiation of Parametric form.
dy
3 a tan2 θ × sec2 θ
dy
dx
= dθ =
dx
3 π dy
= tanθ; At θ = ,
2asec θ × sec θ tan θ 2 8 dx
=
3
2
( 2 −1 )

M a them a ti cs | 21.13

dy
Illustration 24: If x = cosecθ – sinθ and y = cosecnθ – sinnθ, then find . (JEE ADVANCED)
dx

Sol: Differentiation of Parametric form.


x = cosec θ – sinθ
⇒ x2 + 4 = (cosec θ – sin θ)2 + 4 = (cosec θ + sin θ)2 … (i)
and y2 + 4 = (cosecnθ – sinnθ)2+ 4 = (cosecnθ + sinnθ)2 … (ii)
 dy 
 
dy dθ n(cosecn−1 θ)( − cosecθ cot θ) – nsinn−1 θ cos θ
Now, =   =
dx  dx  − cosecθ cot θ − cos θ
 
 dθ 

n(cosecnθ cot θ + sinn−1 θ cos θ ncot θ(cosecnθ + sinn θ) n(cosecnθ + sinn θ)


= = =
(cosecθ cot θ + cos θ) cot θ(cosecθ + sin θ) (cosecθ + sin θ)

dy
Illustration 25: Find if x = at2and y = 2at.  (JEE MAIN)
dx

Sol: Given that x = at2, y = 2at

dx dy
Therefore, = 2at and = 2a.
dt dt
dy
dy 2a 1
Hence, = dt = = .
dx dx 2at t
dt

dy   π 
Illustration 26: If x = cos3t and y = sin3t, then find , for  t ∈  0,   . (JEE MAIN)
dx   2 
dy
dx dy dy 3 sin2 t cos t
Sol: = – 3 cos2t sint (≠ 0) ⇒ = 3 sin2 t cos t ⇒ = dt = = – tan t
dt dt dx dx -3cos2 t sint
dt
dy 16t(1 − t 4 )
Illustration 27: If y = sec 4x and t = tan x, then prove that = . (JEE ADVANCED)
dt (1 − 6t2 + t 4 )2
Sol: Write y in terms of t and differentiate.

( )
2
2
1 1 + tan2 2x 1 + 2t / (1 − t )
y= = =
cos 4x 1 − tan2 2x
( )
2
1 − 2t / (1 − t2 )

(1 − t2 )2 + 4t2 1 + t 4 – 2t2 + 4t2 1 + t 4 + 2t2 dy 16t(1 − t 4 )


y= = = ; =
(1 − t2 )2 − 4t2 1 + t 4 − 2t2 − 4t2 1 + t 4 − 6t2 dt (1 − 6t2 + t 4 )2

2
1 + lnt 3 + 2lnt ydy  dy 
Illustration 28: If x = and y = , then show that = 2x   + 1 (JEE MAIN).
t2 t dx  dx 
Sol: Differentiation of Parametric form.
dy dy / dt
=
dx dx / dt
2 1 . 1 4 | Methods of Differentiation and Applications of Derivatives

dy t ( 0 + 2(1 / t) ) − (3 + 2lnt)1 2 − 3 − 2lnt  1 + 2lnt 


= = =–  
dt t2 t2  t
2

dx t2 ( 0 + (1 / t) ) − (1 + lnt)2t t − 2t − 2tlnt 1 − 2 − 2lnt  1 + 2lnt 


= = = =–  
dt t 4
t 4 t 3
 t
3

2
 dy 
⇒ dy = t ⇒ 2x   + 1 = 2.
1 + lnt 2
.t + 1 = 3 + 2lnt = yt = y
dy
dx dx
  t 2 dx

10. DIFFERENTIATING WITH RESPECT TOANOTHER FUNCTION


du
Suppose u = f(x) and v = g(x) are two functions of x. To find the derivative of f(x) with respect to g(x), i.e. to find , the
dv
du du / dx
formula = is used. Thus, to find the derivative of f(x) with respect tog(x), both are differentiated with
dv dv / dx
respect to x and then the derivative of f(x) with respect to x is divided by the derivative of g(x) with respect to x. The
procedure is demonstrated in illustration 29.

Illustration 29: Differentiate sin2x with respect to ecosx. (JEE MAIN)

Sol: Differentiate both the functions with respect to the common variable and use parametric form.
du du / dx
Let u(x) = sin2x and v(x) = ecosx. We want to find the value of =
dv dv / dx
du dv
Clearly, = 2 sin x cos x and = ecosx (– sin x) = – (sin x) ecosx
dx dx
du 2sinx cos x 2cos x
Hence, = = − .
dv ( − sinx)ecos x
ecos x

 1 + x2 + 1 − x2 
Illustration 30: Differentiate   with respect to 1 − x 4 . (JEE ADVANCED)
 1 + x2 − 1 − x2 
 

Sol: Similar to the previous illustration.


2
 
 1 + x2 + 1 − x2   1 + x2 + 1 − x2  1 + x2 + 1 − x2 + 2 1 − x 4
Let u =  =   =
 1 + x2 − 1 − x2  (1 + x 2
) – (1 − x 2
) 2x2
 
 
x2  0 +  ( −4x3 ) / (2 1 − x 4 )   −  1 + 1 − x 4  2x
1 + 1 − x4 du     
⇒ u= ⇒ =
x2 dx x4

 ( −2x5 ) / ( 1 − x 4 )  − 2x  1 − x 4 + 1   4  4  
     x +  1 − x  1 + 1 − x4 
du du −2x  
⇒ =     ⇒ =  
dx x4 dx 1 − x4  x4 
 
−2x  x4 + 1 − x4 + 1 − x4  = −2x  1 − x4 + 1  
=     ….(i)
x 4
1−x  4  x 4
1−x  4 

Let v = 1 − x4
M a them a ti cs | 21.15

dv 1 dv −2x3
= (–4x3) ⇒ =  ….(ii)
dx 2 1−x 4 dx 1 − x4

du du dv du −2x  1 − x4 + 1  1 − x4 du (1 + 1 − x 4 )
= / ⇒ =   ⇒ =
dv dx dx dv 4
x 1−x 4   −2x3 dv x6

11. DIFFERENTIATION OF DETERMINANTS


To differentiate a determinant, one row (or column) at a time should be differentiated, keeping others unchanged,
which is illustrated by the examples given below.

f(x) g(x) d f '(x) g'(x) f(x) g(x)


(i) If F(x) = , then {F(x)} = +
u(x) v(x) dx u(x) v(x) u'(x) v '(x)

d f '(x) g(x) f(x) g'(x)


Also {F(x)} = +
dx u'(x) v(x) u(x) v '(x)
f g h
(ii) If F(x) =  m n Where f, g, h,  , m, n, u, v, w are functions of x and differentiable, then
u v w

f ' g' h' f g h f g h f' g h f g' h f g h'


F’(x) =  m n +  ' m' n' +  m n ⇒ F(x) =  ' m n +  m' n +  m n'
u v w u v w u' v ' w' u' v w u v' w u v w'

sec θ tan2 θ 1
Illustration 31: If f(x) = sec θ tanx x , then find f’(θ). (JEE MAIN)
1 tanx − tan θ 0

Sol: Above discussed method.

0 0 0 sec θ tan2 θ 1 sec θ tan2 θ 1


f’(x) = sec θ tanx x + 0 sec2 x 1 + sec θ tanx x
1 tanx − tan θ 0 1 tanx − tan θ 0 0 sec2 x 0

sec θ tan2 θ 1 sec θ tan2 θ 1


⇒ f’(θ) = 0 sec2 θ 1 + sec θ tan θ θ = (tan2θ – sec2θ) – sec2θ (qsecθ –secθ)= – 1 – (sec3θ) (θ – 1)
1 0 0 0 sec2 θ 0

12. SUCCESSIVE DIFFERENTIATION


dy
If the first derivative of a function y = f(x) is also a differentiable function, then it can be further differentiated
dx
with respect to x. The derivative thus obtained is called the second derivative of y with respect to x and is denoted
d2 y d2 y d3 y
by . If is also differentiable, then its derivative is called the third derivative of y and is denoted by .
dx2 dx2 dx3
dn y
Similarly, denotes the nth derivative of y. This process is known as successive differentiation and all these
dxn
2 1 . 1 6 | Methods of Differentiation and Applications of Derivatives

derivatives are called as successive derivatives of y.


The following symbols are also used to denote the successive derivatives of y = f(x):

y1, y2, y3, ………. Yn, ………..

dy d2 y d3 y dn y d
y’, y’’, y’’’, ………. yn, ……….. ⇒, , , …….. …….., ⇒ Dy, D2y, D3y, ……….Dny ………… (where D ≡ )
dx dx2 dx3 dx n dx
The following symbols are used to denote the value of the nth derivative at x = a.

 dn y 
yn(a), yn(a),   , Dny(a) & fn(a)
 dxn 
  x =a

PLANCESS CONCEPTS

n
dn y  dy 
Misconception: ≠ 
 dx  
n
dx Rohit Kumar (JEE 2012 AIR 79)

13. Nth DERIVAVES OF SOME STANDARD FUNCTIONS


(a) Dn(ax + b)m = m(m – 1) (m –2) ……… (m – n + 1) an(ax + b)m-n
m! m!
(b) If m ∈ N and m > n, thenDn(ax + b)m = an(ax + b)m–n; Dn(xm) = xm–n
(m − n)! (m − n)!
(c) Dn(ax + b)n = n! an; Dn(xn) = n!
 1  ( −1)n n!an 1 ( −1)n n!
(d) Dn  ax + b  = ; Dn  x  =
  (ax + b)n+1   xn+1

( −1)n−1 (n − 1)! ( −1)n−1 (n − 1)!


(e) Dn{log(ax + b)} = an; Dn(log x) =
(ax + b)n xn
(f) Dn(eax) = aneax
(g) Dn(amx) = mn (log a)n amx
 π  π
(h) Dn{sin (ax + b)} = an sin  ax + b + n  ; Dn(sin x) = sin  x + n 
 2   2 
 π  π
(i) Dn {cos (ax + b)} = ancos  ax + b + n  ; Dn (cos x) = cos x +n 
 2  2

 b
(j) Dn {eax sin (bx + c)} = (a2 + b2)n/2 eax sin  bx + c + ntan−1 
 a

 b
(k) Dn {eax cos (bx + c)} = (a2 + b2)n/2 eax cos  bx + c + ntan−1 
 a
 −1 x ( −1)n−1 (n − 1)!sinn θ sinnθ a
(l) Dn  tan  =
a ,where θ = tan–1  x 
 an  
1
(m) Dn (tan–1 x) = (–1)n–1(n – 1)! sinnθ sin nθ, where θ = tan–1  x 
 
M a them a ti cs | 21.17

14. LEIBNITZ THEOREM


If u and v are two functions such that their nth derivative exists, then the nth derivative of their product can be found
by the following formula:
Dn(uv) = (Dnu)v + nC1Dn–1u. Dv + nC2Dn–2u. D2v +……+……+nCn-1Du. Dn-1v + ……..+ u.Dnv
The nth derivative of a product of two functions can be found out by using this theorem. While using this theorem,
the second function in the product is the function whose successive derivative starts to vanish (if it is possible) after
some stepsand the first function is a function whose nth derivative is easily known.

Illustration 32: If y = x3 cos x, find Dny. (JEE MAIN)

Sol: Leibnitz theorem


Choose cos x as the first function and x3 as the second function
Dn(cos x, x3) = Dn(cos x) (x3) + nC1Dn–1(cos x) (Dx3) +nC2Dn–2(cos x) . (D2x3) + nC3Dn–3(cos x) . (D3x3)

 nπ   (n − 1)π  (n − 1)  (n − 2)π  n(n − 1)(n − 2)  x + (n − 3)π 


= x3cos  x +  + n.3x cos
2
x + + 6x.cos x + + .6.cos  
 2   2  1.2  2  1.2.3  2 
 nπ   nπ   nπ   nπ 
= x3cos  x +  + 3nx sin  x +
2
 – 3n(n – 1)x cos  x +  – n(n – 1) (n – 2) sin  x + 
 2   2   2   2 

APPLICATION OF DERIVATIVES
1. THE INTERPRETATION OF THE DERIVATIVE
dy
If y = f(x) be a given function, then the derivative/differential coefficient f’(x) or
at the point P(x1, y1) is called the
dx
trigonometric tangent of the angle ψ (say), which the positive direction of the tangent to the curve at P makes with
dy
the positive direction of the x-axis. Therefore, represents the slope of the tangent.
dx
dy Y
Thus, f’(x) = =Y 
dx (x1 ,y1 ) nt
ge
y=f(x)
Then,
Tan

o
90
dy
(a) The inclination of the tangent with x-axis = tan–1 dx P(x1, y1)
No
rm
dy al
(b) Slope of the tangent =
dx 
dx V
(c) Slope of the normal = –
dy
Figure 21.1

2. EQUATION OF TANGENT
 dy 
(a) Equation of tangent to the curve y = f(x) at A(x1, y1) is given by y – y1 =   (x – x1)
 dx (x1 ,y1 )
If the tangent at P (x1, y1) of the curve y = f(x) is parallel to the x-axis (or perpendicular to the y-axis), then
Ψ = 0, i.e its slop will be equal to zero.
2 1 . 1 8 | Methods of Differentiation and Applications of Derivatives

 dy 
⇒ m=   =0
 dx (x1 ,y1 )
The converse also holds true. Thus, the tangent at (x1, y1) is parallel to the x-axis.
 dy 
⇒   =0
 dx (x1 ,y1 )
(b) If the tangent at P (x1, y1) of the curve y = f(x) is parallel to the y-axis (or perpendicular to the x-axis), then Ψ
= π / 2 and its slope will be infinity, i.e.
 dy 
m=   =∞
 dx (x1 ,y1 )
The converse also holds true. Thus, the tangent at (x1, y1) is parallel to the y-axis.
 dy 
⇒   =∞
 dx (x1 ,y1 )
(c) If at any point P (x1, y1) of the curve y = f(x) the tangent makes equal angles with both the axes, then at the
point P, ψ = x / 4 or 3π / 4.Therefore at P, tan Ψ = dy / dx = ± 1.
The converse of the result also holds true. Thus, at (x1, y1), the tangent line makes equal angles with both the axes.
 dy 
⇒   =±1
 dx (x1 ,y1 )
(d) Concept of vertical tangent: y = f(x) has a vertical tangent at the point x = x0 if Y

f(x0 + h) − f(x0 )
Lim = ∞ or – ∞, but not both.
h→0 h

E.g. The functions f(x) = x1/3 and f(x) = sin x both have a vertical tangent at x = 0 X
0 if x < 0
But f(x) = x2/3, f(x) =x and f(x) =  have no vertical tangents at x = 0.
1 if x ≥ 0
Figure 21.2
(e) If a curve passes through the origin, then the equation of the tangent at the
origin can be directly written by equating the lowest degree terms present in the
equation of the curve to zero.
E.g. Y

(i) x2 + y2 + 2gx + 2fy = 0


Equation of tangent is gx + fy = 0
(ii) x3 + y3– 3x2y + 3xy2 + x2 – y2 = 0 X
O
Equation of tangent at the origin is x2 – y2 = 0
(iii) x3 + y3– 3xy = 0
Figure 21.3
Equation of tangent is xy = 0
Note: This concept is valid only if the powers of x and y are natural numbers.
(f) Same line could be the tangent and normal to a given curve at a given point.
E.g. In x3 + y3 – 3xy = 0 (folium of Descartes), the line pair xy = 0 is both the tangent and normal at x = 0.

Some common parametric coordinates on a curve that are useful for differentiation
(a) For x2/3 + y2/3 = a2/3, take parametric coordinates x = a cos3qand y = a sin3θ.
(b) For x + y = a , take x = a cos4θ and y = a sin4θ.
M a them a ti cs | 21.19

xn yn
(c) + = 1,where x = a (sin θ)2/nand y = b(cos θ)2/n.
an
bn

(d) For c2 (x2 + y2) = x2y2, take x = c sec θ and y = c cosec θ.

(e) For y2 = x3 , take x = t2 and y = t3.

Illustration 33: If the tangent to the curve 2y3 = ax2 + x3 at the point (a, a) cuts off intercepts α and β on the
coordinate axes, where a2 + b2 = 61, the value of | a | is ____. (JEE MAIN)
(A) 16 (B) 28 (C) 30 (D) 31

Sol: (C) Write the equation of the tangent and find the value of α and β in terms of a. Then use a2 + b2 = 61 to find
the value of a.
dy 2ax + 3x2
The slope of the tangent is given by = The value of this slope at (a, a) is 5/6.
dx 6y 2
5 x y
Hence, the equation of tangent is y – a = (x – a) ⇒ + =1
6 −a / 5 a / 6
a a
Thus, the x-intercept α is – , and the y-intercept β is .
5 6
a2 a2
From a2 + b2 = 61, we get + = 61 ⇒ a2 = 25 × 36 ⇒ | a | = 30
25 36

3. EQUATION OF NORMAL
Equation of normal at (x1, y1) to the curve y = f(x) isgiven by the following formula:

−1  dy 
(y – y1) = (x – x1) ⇒ (y – y1)   + (x – x1) = 0
 dy   dx (x1 ,y1 )
 
 dx (x1 ,y1 )

Some facts regarding the normal


 dx 
(a) Slope of the normal drawn at point P (x1,y1) to the curve y = f(x) = –  
 dy (x1 ,y1 )

dx dy
(b) If the normal makes an angle of θ with the positive direction of the x-axis, then – = tanθ or = – cotq
dy dx

dx dy
(c) If the normal is parallel to the x-axis, then = 0 or =∞
dy dx

 dx  dy
(d) If the normal is parallel to the y-axis, then   = ∞ or =0
 dy  dx

 dx   dy 
(e) If the normal is equally inclined from both the axes or cuts equal intercept, then –   = ± 1 or   = ± 1
 dy   dx 

 dy 
x1 + y1  
(f) The length of the perpendicular from the origin to the normal is P’ =  dx 
2
 dy 
1+ 
 dx 
2 1 . 2 0 | Methods of Differentiation and Applications of Derivatives

 dy 
(g) The length of the intercept made by the normal on the x-axis is x1 + y1   and the length of the intercept
 dx 
on the y-axis is y1+ x1  dx  .
 
 dy 

Illustration 34: Find out the distance between the origin and the normal to the curve y = e2x + x2 at the point
whose abscissa is 0. (JEE MAIN)
1 2 3 2
(A) (B) (C) (D)
5 5 5 3
Sol: (B) Write the equation of the normal and find the distance of origin from the normal.
The point on the curve corresponding to x = 0 is (0, 1)
dy dy
= 2e2x + 2x ⇒ =2
dx dx x =0
Therefore, the equation of the normal at the point (0, 1) is
y – 1 = (– 1/2) (x – 0) ⇒ 2y + x – 2 = 0
2
Hence, the distance of the point (0, 0) from this line is .
5

4. LENGTH OF TANGENT, NORMAL, SUBTANGENT AND SUBNORMAL

4.1 Tangent
2
 dy 
y 1+ 
 dx 
PT = MP cosec Ψ = y 1 + cot2 ψ = P
dy
dx 
y
4.2 Subtangent 
T O M G
y
TM = MP cot Ψ = Figure 21.4
(dy / dx)

4.3 Normal
2
 dy 
GP = MP sec Ψ = y 1 + tan2 ψ = y 1 +  
 dx 

4.4 Subnormal

 dy 
MG = MP tan ψ = y  
 dx 

Illustration 35: For the parabola y2 = 16x, the ratio of the length of the subtangent to the abscissa is _____.
(A) 2 : 1 (B) 1 : 1 (C) X : Y (D) X2 : Y  (JEE MAIN)
M a them a ti cs | 21.21

y
Sol: (A) The length of subtangent is
(dy / dx)

dy dy 8
Differentiating, 2y = 16 Hence, =
dx dx y
dx y 2 16x
Thus, the length of the subtangent is y = = = 2x
dy 8 8
Therefore, the ratio of the length of the subtangent to the abscissa = 2x : x = 2 : 1.

Illustration 36: Find out the length of the normal to the curve x = a(θ + sinθ), y = a (1 – cos θ) at θ = π/2.
 (JEE MAIN)
2
 dy 
Sol: Use differentiation of the Parametric form. Length of the normal = 1+ 
 dx 
 dy 
 
dy  dθ  asin θ θ  dy  π
= = = tan ⇒   = tan   = 1
dx  dx  a(1 + cos θ) 2  dx  4
 
 dθ 
π  π
Moreover, at θ = , y = a  1 − cos  =a
2  2
2
 dy 
Therefore, the required length of the normal = y 1 +   = a 1 + 1 = 2a
 dx 

Illustration 37: The length of the subtangent to the ellipse x = a cos t, y = b sin t at t = π/4 is _____.

(A) A (B) B (C) B/ 2 (D) A/ 2  (JEE MAIN)

Sol: (D) Similar to the previous illustration.


dx dy dy b b
= – a sin t and = b cos t; Therefore, =– cot (π / 4) = –
dt dt dx t= π/ 4
a a

dx π a a
Therefore, the length of the subtangent = y = bsin × – =
dy r = π / 4 4 b 2

5. ANGLE OF INTERSECTION OF TWO CURVES


The angle of intersection between two intersecting curves C1 and C2 is defined C1 T1
as the acute angle between their tangents (T1 and T2 or the normals) at the
point of intersection of the two curves.
m1 − m2 
tan ψ = , where m1 and m2 are the slopes of the tangents T1 and T2 T T2
1 + m1m2 (x1,x2)
at the intersection point (x1, y1)
C2
π
Note: If the two curves intersect orthogonally, i.e at right angle, then φ= .
Hence, the condition will be 2 Figure 21.5

 dy   dy 
  .  =–1
 dx 1  dx 2
2 1 . 2 2 | Methods of Differentiation and Applications of Derivatives

Illustration 38: Which of the following options represents the tangent of the angle at which the curves y = ax and
y = bx(a ≠ b > 0)intersect? (JEE ADVANCED)
logab loga / b logab
(A) (B) (C) (D) None of these
1 + logab 1 + (loga)(logb) 1 + (loga)(logb)

Sol: (B) Differentiate the two curves and use the formula for angle between two lines.
Intersection of the two curves is given by ax = bx, which implies that x = 0. If α is the angle at which the two curves
intersect, then
m1 − m2 ax loga − bx logb (loga / b)
tan α = = = (Putting x = 0)
1 + m1m2 x x
1 + a b (loga)(logb) 1 + (loga)(logb)

6. RATE MEASURE
dy
Whenever a quantity y varies with another quantity x, satisfying the rule y = f(x), then (or f’(x)) represents the
dx
dy
rate of change of y with respect to x and (or f’(a)) represents the rate of change of y with respect to x at x = a.
dx x =a

Illustration 39: The volume of a cube increases at the rate of 9 cm3. How fast does the surface area increase when
the length of an E.g. is 10 cm? (JEE MAIN)

Sol: Rate measurer.


Let x be the length of the side, V be the volume and S be the surface area of the cube.
dV dx dx 3 dS d  3  36
Thus, = 9 cm3/s ⇒ 3x2 dt = 9 cm3/s ⇒ = cm/s ⇒ = (6x2) = 12x   = cm2/s
dt dt x 2 dt dt 2
x  x

dS 
⇒  = 3.6 cm2/s
dt  x =10cms

Illustration 40: A man of height 2 meters walks away from a 5-meter lamppost at a uniform speed of 6 meters per
minute. Find the rate at which the length of his shadow increases. (JEE MAIN)

Sol: Use similarity to establish the relation between the rate at which length of shadow increases and speed of the
man.
B
Let AB be the lamp-post. Let at any time t, the man CD be at a distance x metres from the
lamp-post and y metres be the length of his shadow CE.
D
dx
Then, = 6 meters / minute [given] … (i)
dt
Clearly, the triangles ABE and CDE are similar X y
AB AE 5 x+y A C E
⇒ = ⇒ = ⇒3y = 2x
CD CE 2 y Figure 21.6
dy dx dy dy
⇒ 3 =2 ⇒3 = 2(6) [Using (i)] ⇒ = 4 meters / minute
dt dt dt dt

Illustration 41: An object has been moving in the clockwise direction along the unit circle x2 + y2 = 1. As it passes
through the point (1/2, 3 /2), its y-coordinate decreases at the rate of 3 units per second. The rate at which the
x-coordinate changes at this point is ______ units per second.

(A) 2 (B) 3 3 (C) 3 (D) 2 3  (JEE MAIN)


M a them a ti cs | 21.23

Sol: (B) Differentiate and proceed.


dx 1 3 dy
We find that when x = and y = given that = – 3 units/s and x2 + y2 = 1.
dt 2 2 dt
dx dy
Differentiating x2 + y2 =1, we get 2x + 2y =0
dt dt
1 dy 1 dx 3 dx
Putting x = ,y= 3 /2 and = – 3, we get + (– 3) = 0 ⇒ = 3 3 (increasing)
2 dt 2 dt 2 dt

Illustration 42: A given right circular cone has a volume p.The largest right circular cylinder that can be inscribed
in the cone has a volume q. The ratio of p to q is _____. (JEE MAIN)
(A) 9 : 4 (B) 8 : 3 (C) 7 : 2 (D) None of these

Sol: (A) Let H be the height of the cone and α is its semi-vertical angle.
Let x be the radius of the inscribed cylinder and h be its height.
h = QL = OL – OQ = H – x cot a
1
p= π(H tan α)2 H … (i)
3
V = volume of the cylinder = px2 (H – x cot α)  O
dV
= π(2Hx – 3x2 cot α) 
dx x
dV
Hence, =0 ⇒x=0 Q
dx
2 d2 V
x= H tan α, = – 2πH < 0, so
3 dx=2
2  M P
x Htan α
3
2 4 1 4 Figure 21.7
V is maximum when x = H tan α and q = Vmax = π H2 tan2α H = p [using (i)]
3 9 3 9
Therefore, p : q = 9 : 4

7. APPROXIMATION USING DIFFERENTIALS


To calculate the approximate value of a function, differentials may be used, wherein the differential of a function is
equal to its derivative multiplied by the differential of the independent variable.
dy = f’(x)dx or df(x) = f’(x) dx

PLANCESS CONCEPTS

For the independent variable ‘x’, increment Dx and differential dx can be made equal, but the same
cannot be applied in case of the dependent variable ‘y,’ i.e. Dy ≠ dy.
Therefore, the approximate value of y when the increment ∆ x is given to the independent variable x in
y = f(x) is
dy
y + Dy = f(x + Dx) = f(x) + . Dx
dx
⇒ f(x + Dx) = f(x) + f’(x) Dx
Vaibhav Gupta (JEE 2009 AIR 54)
2 1 . 2 4 | Methods of Differentiation and Applications of Derivatives

Illustration 43: Find the approximate value of the square root of 25.2. (JEE MAIN)

Sol: Consider a function f(x) = x and differentiate to get the derivative. Then replace x by x+Dx and proceed.
1
Let f(x) = x , so f’(x) =We can write 25.2 as 25 + 0.2
2 x
By taking x = 25 and Dx = 0.2, now f(x + Dx) = f(x) + f’(x). Dx
1 1
⇒ x + .Dx = 25 + . 0.2
2 x 2 25
0.2
= 5+ = 5 + 0.02 = 5.02
10

Illustration 44: What is the approximate change in the volume V of a cube of side x meters caused by increasing
the side by 2%? (JEE MAIN)
dV
Sol: Differentiate the equation V = x3 and use the relation DV = Dx.
dx
Let ∆(x) be the change in x and DV be the corresponding change in V
∆x
Given that × 100 = 2
x
dV
We know that V = x3 ∴ = 3x2
dx
dV 2x
Therefore, DV = Dx ⇒ DV= 3x2Dx = 3x2 × = 0.06 x3m3
dx 100
The approximate change in volume is 0.06 x3m3.

Illustration 45: What is the approximate value of cos 40º? (JEE ANDANCED)
(A) 0.7688 (B) 0.7071 (C) 0.7117 (D) 0.7

Sol: (A) Take a function f(x) = cos x and proceed.


π π π π
Let f(x) = cos x. 40º = 45º – 5º = – ×5= – radians
4 180 4 36
A differential is used to estimate the change in cos x
π π  π 
When x decreases from to –  
4 4  36 

f’(x) = – sin x and df (x)= f’(x) h = – h sin x


π π
With x = and h = – , df is given by
4 36

 π  π π 1 π 2
df = – f’(x)h = –  −  sin   = . = = 0.0617
 36  4 36 2 72

cos 40 ≡ cos 45 + 0.0617 ≡ 0.7071 + 0.0617 = 0.7688.

8. SHORTEST DISTANCE BETWEEN TWO CURVES


It has been found that the shortest distance between two non-intersecting curves is always along the common
normal (wherever defined).
M a them a ti cs | 21.25

Illustration 46: Find out the shortest distance between the line y = x – 2 and the parabola y = x2 + 3x + 2.
 (JEE MAIN)

y = x + 3x + 3y y
2
Sol: The distance would be minimum at the point on the parabola where
the slope of the tangent is equal to the slope of the given line. 
y=x-2
Let P(x1, y1) is the point closest to the line y = x – 2 P=(x1, y1)
x’ x
dy O
Then, = slope of the line
dx (x
1 ,y1 )

⇒ 2x1+ 3 = 1 ⇒ x1 = –1 and y1 = 0
y’
Therefore, point (–1, 0) is the closest and its perpendicular distance from the
line y = x – 2 gives the shortest distance. Figure 21.8

3
⇒ Shortest distance = units
2

Illustration 47: Which of the following points of the curve y = x2 is closest to (4, – ½)? (JEE MAIN)
(A) (1, 1) (B) (2, 4) (C) (2/3, 4/9) (D) (4/3, 16/9)

Sol:(A) Using distance formula find the distance of the given point from the curve and find the minima.
Let the required point be (x, y) on the curve.

Hence, d = (x − 4)2 + (y + 1 / 2)2 should be minimum, which is enough to consider.


D = (x – 4)2 + (y + 1/2)2 = (x – 4)2 + (x2 + 1/2)2
D’ = 4x3 + 4x – 8
Now for critical points
D’ = 0 so x3 + x – 2 = 0 ⇒ x = 1
Clearly D” at x = 1 is 16 > 0.
Thus, D is minimum when x = 1. Hence the required point is (1, 1).

PROBLEM-SOLVING TACTICS

•• Reduce any fractions to be as basic as possible.


•• Recognise when we can use the chain rule. it enables us to differentiate functions that often seem impossible
to differentiate. Whenever you see a nested function, try to assess if the chain rule is needed (it usually is).
•• We always want to start a long chain of differentiation by differentiating the last part of the function to touch
the input - in short, the outermost part of the function.
2 1 . 2 6 | Methods of Differentiation and Applications of Derivatives

FORMULAE SHEET

dc d du
=0 (cu) = c
dx dx dx

d du dv d dv du
(u ± v) = ± (uv) = u +v
dx dx dx dx dx dx

du dv
v −u dy dy du
d u dx dx =
  = dx du dx
dx  v  v2

d n d n du
x = nxn–1 u = nun–1 dx
dx dx

d x d u du
a = (ln a) ax a = (ln a) au dx
dx dx

d x d u du
e = ex e = eu dx
dx dx

d 1 d 1 du
logax = logau =
dx (lna)x dx (lna)u dx

d 1 d 1 du
ln x = ln u =
dx x dx u dx

d d du
sin x = cos x sin u = cos u
dx dx dx

d d du
cos x = – sin x cos u = – sin u
dx dx dx

d d du
tan x = sec2 x tan u = sec2 u
dx dx dx

d d du
cot x = – cosec2 x cot u = – cosec 2 u
dx dx dx

d d du
sec x = sec x tan x sec u = sec u tan u
dx dx dx

d d du
cosec x = – cosec x cot x cosec u = – cosec u cot u
dx dx dx

d 1 d 1 du
sin–1 x = sin–1 u =
dx 1−x 2 dx 1−u 2 dx

d 1 d 1 du
tan–1 x = tan–1 u =
dx 1 + x2 dx 1 + u2 dx
M a them a ti cs | 21.27

 dy 
* Equation of tangent to the curve y = f(x) at A(x1, y1) is y – y1 =   (x – x1)
 dx (x1 ,y1 )

−1
* Equation of normal at (x1, y1) to the curve y = f(x) is (y – y1) = (x – x1)
 dy 
 
 dx (x1 ,y1 )

* Length of Tangent, Normal, Subtangent and Subnormal

2
 dy 
y 1+ 
 dx 
Tangent: PT = MP cosec Ψ = y 1 + cot2 ψ = P
dy
dx 
y
y
Subtangent: TM = MP cot Ψ = 
(dy / dx)
T O M R G
2
 dy  Figure 21.9
Normal: GP = MP sec Ψ = y 1 + tan2 ψ = y 1 +  
 dx 

 dy 
Subnormal: MG = MP tan Ψ = y   C1 T1
 dx 
* Angle of Intersection of Two Curves

m1 − m2 T T2
tan Ψ = ,  (x1,x2)
1 + m1m2
C2
where m1 and m2 are the slopes of the tangents T1 and T2 at the intersection
point (x1, y1). Figure 21.10

Solved Examples

JEE Main/Boards f’(0–) =


lim f(x) − f(0) lim −( −x) − 0
= x →0 = –1
x →0 – x −x
Example 1: Show that the function f(x) = | x | is Since, f’(0+)≠f’(0–), the function is not differentiable at
continuous at x = 0, but not differentiable at x = 0. x=0

Sol: Evaluate f’(0+) and f’(0-).


Example 2: Find the derivative of the function f(x),
 x, x ≥ 0 defined by f(x) = sin x by 1stprinciple.
We have f(x) = 
 –x, x < 0
Sol: Use the first principle to find the derivative of the
Since lim f(x) = lim f(x) = 0 = f(0) given function.
x →0 + x →0 –
The function is continuous at x = 0 Let dy be the increment in y corresponding to an
increment dx in x. We have
We also have
y = sin x
f(x) − f(0) x−0
f’(0+) = lim = lim =1
x →0 + x x →0 x y +dy = sin (x + dx)
2 1 . 2 8 | Methods of Differentiation and Applications of Derivatives

Subtracting, we get x
y log x = x – y ⇒ y =
dy = sin(x + dx) – sin x = 2 cos (x + dx/2) sin (dx / 2) 1 + logx

Dividing by dx, we obtain dy (1 + logx).1 − x(1 / x) log x


∴ = =
dx (1 + logx)2
(1 + log x)2
δy cos(x + δx / 2)sin(δx / 2)
=
δx (δx / 2)
Taking limits on both side, we get Example 6: If y = cot–1 x2 − 1 + sec–1x, then dy/dx
equals
dy δy sin(δx / 2)
= lim = lim cos(x + dx/2) lim
dx δx →0 δx δx →0 δx →0 ( δx / 2) Sol: Use substitution to simplify the terms and then
differentiate.
= cos x . 1 = cos x
2
Hence, we have d/dx(sin x) = cos x Put x = secθ ; cot-1 x − 1 = cot sec2 θ − 1

π π
Example 3: The derivative of log | x | is = cot–1(tanθ) = – θ = – sec–1 x
2 2
Sol: Use the definition of the modulus to expand the π  π dy
∴ y =  − sec –1 x  + sec–1 x = ⇒ =0
given function. Then evaluate L.H.D. and R.H.D. at the  2  2 dx
critical point.
dy
Let y = log | x | then Example 7: If x2ey + 2xyex + 13 = 0, then equals -
dx
 logx, whenx > 0 Sol: Use the formula for derivative of implicit function.
y= 
log( −x), whenx < 0 Using partial derivatives, we have

dy 1 ∂f
∴ = when x > 0 dy ∂x
dx x ⇒ =
dx ∂f
dy 1 1
and = (– 1) = when x < 0 ∂y
dx −x x
dy 1 dy 2xey + 2yex + 2xyex
⇒ = when x ≠ 0 =–
dx x dx x2ey + 2xex

1+x dy 2xey − x + 2y + 2xy 2xey − x + 2y (x + 1)


Example 4: If y = , then equals =– =–
1−x dx
x2ey − x + 2x x(xey − x + 2)

Sol: Differentiate using u/v rule.


d  −1  cos x  
Differentiating w.r.t., we get Example 8: tan    equals -
dx   1 + sinx  
dy 1 1 − x (1 − x)1 − (1 + x)( −1) cos x
= Sol: Convert in terms of tan and proceed.
dx 2 1+x (1 − x)2 1 + sinx


1−x 1

1 1 cos x sin ( π / 2 − x )
1 + x (1 − x)2  =
1 + x (1 − x)3/2 1 + sinx 1 + cos( π / 2 − x)

1  1  π x  cos x 
⇒   = tan  −  ∴ tan–1  1 + sinx 
1−x 2 1 − x   4 2  

  π x  π x 1
Example 5: If xy = ex-y, then dy/dx equals - = tan–1  tan  4 − 2   = − ⇒ Derivative = –
   4 2 2
Sol: Take logarithms on both sides and differentiate.
Taking log on both sides, we get sec x − tanx dy
Example 9: If y = , then equals
sec x + tanx dt
M a them a ti cs | 21.29

Sol: Simplify the R.H.S. and differentiate. dy


sec x − tanx sec x − tanx dy dx 1
y= . ∴ = =(2x2 + 1)
dz dz
(1 / (2x )
2
sec x + tanx sec x − tanx 2
+ 1) −1
y =(sec x – tan x)2 dx

dy  −4x  2  dy 
∴ = 2(sec x – tan x)(sec x tan x – sec2 x)  2  1 + 3x ∴   =0
dx  2x + 1  3  dz x = −1
3
⇒ – 2 sec x (sec x – tan x)2
  
d 2 −1 1 + x 
1 dy Example 14: Find sin  cot  =
Example 10: If y = , then equals dx 1 − x  
  
(t + 2)(t + 1) dx

Sol: Use the partial fraction method to find the Sol: Use Substitution to simplify the inside the square
derivative of given fn root and then differentiate.

1 1 1  −1 1 + x

y= = – Let y = sin2  cot 
1 − x  .
(t + 2)(t + 1) t + 1 t + 2 
dy 1 1
⇒ =– + Put x = cos 2θ.
dx (t + 1)2
(t + 2)2
 1 + cos2θ  
∴ y = sin2cot–1  1 − cos2θ   = sin2cot–1 (cot θ)
1 1   
Example 11: If x = θ – and y = θ + ,
θ θ
dy 1 − cos2θ 1 − x 1 x
then =? ∴ y = sin2θ= = = –
dx 2 2 2 2
1 dx 1 dy 1
Sol: x = θ – ⇒ =1+ ∴ =–
θ dθ θ2 dx 2
1 dy 1
y=θ+ ⇒ =1– Example 15: Find the equation of the normal to the
θ dθ θ2
π
dy curve y = x + sin x cos x at x = .
dy 1 − (1 / θ2 ) θ − (1 / θ) x 2
∴ = dθ = = = Sol: Find a point on the curve slope of the normal at
dx dx 1 + (1 / θ ) θ + (1 / θ)
2 y
dθ that point.
2 π π π π π
Example 12: Derivative of sin–1 x w.r.t. cos–1 1 − x is - x= ⇒ y = +0 = , so the given point =  ,  .
2 2 2 2 2
Sol: Substitute sin θ in place of x. dy
Now from the given equation = 1 + cos2x – sin2 x
2 dx
Let y = sin–1x and z = cos–1 1 − x
 dy 
⇒  =1+0–1=0
Put x = sin θ⇒ z = cos (cos θ)=θ  dx  π , π 
–1

2 2
dy
∴ y = z and =1 ∴ required equation of the normal is
dz
 1  π −1  π π
y– = x −  ⇒ x – = 0 ⇒ 2x = p
Example 13: Derivative of sec  2
–1  w.r.t. 1 + 3x 2 0  2  2
 2x + 1 
−1
at x = is
3 Example 16: Find the point on the curve y = x2 – 3x at
which tangent is parallel to x-axis.
Sol: Differentiate the two functions and divide.
Sol: Differentiate the given equation and put it equal to
 1  zero and proceed.
Let y= sec–1  2  and z = 1 + 3x
 2x + 1  Let the point at which tangent is parallel to x-axis be
P(x1, y1)
2 1 . 3 0 | Methods of Differentiation and Applications of Derivatives

3 dy 9x
Then it must be on curve i.e.,y1 = x1 – 3x1 We have =–
dx by
dy For curves to intersect at right angles, we must have at
Also differentiating w.r.t. x, we get, = 3x2 – 3
dx the point of intersection.
 dy  3  −9x 
⇒  = 3 x12 – 3 … (i)
 = – 1 ⇒ 27 x = by .
2
dx
 (x1 ,y1 ) 
y  by 
since, the tangent is parallel to x-axis Thus we must have
 dy 
∴  = 0⇒ 3 x12 – 3 = 0 9x2 + by2 = 16 ⇒ 9x2 + 27x – 16 = 0 … (ii)
dx
 (x1 ,y1 )
(i) and (ii) must be identical so 27 = 6b ⇒ b = 9/2.
⇒ x1 = ±1  … (ii)
From (1) and (2); y1 = x12 – 3x1 Example 19: If the tangent at (1, 1) on y2 = x(2 – x)2
When x1 = 1 when x1 = – 1 meets the curve again at P, then P is

y1 = 1 – 3 = – 2, y1 = –1 + 3 = 2 Sol: Solve the equation of the tangent with the equation


∴ points at which tangent is parallel to x-axis are of the curve.
(1, – 2) and (–1, 2). dy
2y = (2 – x)2 – 2x (2 – x)
dx
Example 17: Find the equation of normal to the curve dy 1
= 3x2 – 8x + 4. So = −
x + y = xy, where it cuts x-axis. dx (1,1) 2

Sol: Given curve is x + y = xy  … (i) An equation of tangent at (1, 1) is Y – 1 = (–1/2) (X – 1).


at x-axis y = 0, i.e. Y = (–1/2)x + 3/2. The intersection of this line with
the given curve is given by ((– x/2) + 3/2)2 = x(2 – x)2
∴x+0=x ⇒x=1 0

⇒ x2 – 6x + 9 = 16x + 4x3 – 16x2. So,


∴ Point is A(1, 0)
4x3 – 17x2 + 22x – 9 = 0
Now to differentiation x + y = xy
⇒ (x – 1) (4x – 9) (x + 1) = 0
take log on both sides
Thus x = 1, 9/4, –1. But x = – 1 cannot lie on the given
⇒ log (x + y) = y log x
curve so required point is (9/4, 3/8).
1  dy  1 dy
∴ 1 +  = y . +(log x)
x+y  dx  x dx
JEE Advanced/Boards
 dy 
Putting x = 1, y = 0, 1 +  =0
 dx  Example 1: Examine differentiability of f(x) at
 dy 
⇒  = – 1 ∴ slope of normal = 1 1 − cos x
 dx (1,0)  , x≠0
y −0 x = 0 for f(x)=  x sinx
Equation of normal is, = 1 ⇒ y=x–1  1
x −1 , x=0
 2
Example 18: If the curve y2 = 6x, 9x2 + by2 = 16, cut Sol: Find the left and right hand derivative of the
each other at right angles then the value of b is function f(x) about the point x = 0.
dy First we obtain L.f ‘(0)
Sol: Equate the product of from the two equations
to -1. dx
 f( −h) − f(0)   1  1 − cosh 1 
= lim   = lim  − h   hsinh − 2 
The intersection of the two curves is given by 9x + 6bx 2
h→0  −h  h→0   
= 16  … (i)
dy 3  hsinh + 2(1 − cosh)   0 
Differentiating y2 = 6x, We have =
dx y = lim   ;  in from 
h→0  2
2h sinh   0 
Differentiating 9x + 6y = 16
2 2 
M a them a ti cs | 21.31

  h3 h5   h2 h4 h6   ( −1)n h   π
hh − + − .....  − 2  − + − .....   = lim   = (–1)n–1Rf’  nπ + 
  3! 5! 

 2! 4! 6!



h→0
 −h   2
= lim  
h→0  h3 
 2h2  h − + .....    f (nx + ( π / 2) + h) − f (nx + ( π / 2) ) 
  3!   = lim  
    h→0 h
 
  1 1   1 2   
 h4  −  +  −  h2   { } { } 
 sin−1  cos (nx + ( π / 2) + h) − sin−1 cos (nπ + ( π / 2))
 
  12 3!   5! 6!    = lim  
= lim    h→0
 h 
h→0
 3
 h2    
2h  h − + ..... 



 3! 


 { } {
 sin−1 ( −1)n cos (( π / 2) + h) − sin−1 ( −1)n cos( π / 2)

= lim 
} 

  1 1   1 2  
h→0
 h 
 h  −  +  −  h + .....    
  12 3!   5! 6!    = 0 and
= lim     sin−1 {( −1)n+1 sinh}   sin−1 {sin( −1)n+1 h} 
h→0
  h2   = lim   = lim  
2 1 − + .....  h→0 h h→0 h
  3!      
   
( −1)n+1 sin−1 sin h ( −1)n+1 h
 f(0 + h) − f(0)  = lim = lim = (– 1)n+1
Rf’(0) = lim   h→0 h h→0 h
h→0
 h 
(Which is equal to (– 1)n–1)
 1  1 − cosh 1  
= lim   −   = 0,
h→0
 h  hsinh 2    π  π
Thus we find Lf’  nx +  = Rf’  nx + 
 2  2
similarly as above i.e. L.f’(0) = Rf’(0)
⇒ f(x) is differentiable at x = 0 ∴ f(x) is differentiable at (nx + π/2)

dy
Example 2: Examine differentiability of the function f(x) Example 3:If x (1 + y) + y (1 + x) = 0, then equals
dx
π Sol: Simplify the equation given and then differentiate
= sin–1 (cos x) at x = np + . where n ∈ I.
2 it.
Sol: Similar to the previous example. We have
 π
first, we obtain Lf’  nx +  x (1 + y) + y (1 + x) = 0  … (i)
 2 
 f (nx + ( π / 2) − h) − f (nx + ( π / 2) )  ⇒ x (1 + y) = –y (1 + x)
= lim  
h→0  −h 
  On squaring both sides x2(1+y) = y2(1+x)
⇒ x2 – y2 + x2y – xy2 = 0
{ } {
 sin−1 cos (nx + ( π / 2) − h) − sin−1 cos (nx + ( π / 2))} 
= lim   ⇒ (x – y) (x + y + xy) = 0
h→0  −h 
  x – y ≠ 0 [For y = x does not satisfy (1)]


= lim 
{ } {
 sin−1 ( −1)n cos ( ( π / 2) − h) − sin−1 ( −1)n cos( π / 2)
}  ∴ x + y + xy = 0⇒ y = –
x
(1 + x)
h→0 −h 
  dy 
 (1 + x).1 − x.1 
 1
∴ =–   =– .
{ } (1 + x)2
2
 sin−1 sin( −1)n h − sin−1 0  dx  (1 + x)
 

= lim  
h→0  −h 
  dy
Example 4: If xyyx = 1, then equals –
dx
( −1)n sin−1 sin h − sin−1 0
= lim Sol: Use logarithms on both sides and then differentiate
h→0 −h
Taking log on both sides, we have y log x + x log y = 0
2 1 . 3 2 | Methods of Differentiation and Applications of Derivatives

Now using partial derivatives, we have Sol: Differentiation w.r.t another function.
dy y / x + log y y(y + xlog y) Putting x = tanθ
=– ⇒–
dx logx + x / y x(x + y logx)  x   tan θ 
y = sin–1   = sin–1 
 =q
 1+x
2
  sec θ 
dy
Example 5: If 1 − x2 + 1 − y 2 = a(x – y) then
equals – dx  1 − x2   1 − tan2 θ 
& z = cos–1  2
 = cos–1 


 1 + tan2 θ  = 2q
1 + x   
Sol: Use substitution for x and y.
Putting x = a sin A, y = a sin B, then given relation ∴ Required derivative = ½
becomes dy
dy dθ θ 1
cos A + cos B = a(sin A – sin B) ⇒ = ==
dz dz 2θ 2
 A +B  A −B  dθ
⇒ 2a cos   cos  
 2   2 

 A +B  A −B 
Example 8: If y = sin–1 ( )
sinx then
dy
dx
equals–
= 2a cos   sin  
 2   2  Sol: Differentiation of function
 A +B
.Divided
Divide
… andmultiply
multiplybby
y cos   dy 1 1
 2  = . .cosx
dx 1 − sinx 2 sinx
 A −B    A +B 
⇒ cot   = a  cos   ≠ 0 1 + sinx 1
 2    2   = = 1 + cosecx
2 sinx 2
⇒ A – B = 2 cot–1a ⇒ sin–1 x – sin–1y = 2 cot–1a

1 1 dy dy 1 − y2 v µ dv
– =0 ⇒ = Example 9: If + = 6 then =?
1−x 2
1−y 2 dx dx 1−x 2 µ v dµ

Sol: Square the given equation and proceed.


1 1
Example 6: If x2 + y2 = t – , x4 + y4 = t2 + then
t t 2
v µ v µ
dy + = 6 ⇒ + +2 = 36
equals µ v µ v
dx
Sol: Eliminate t from the first and the second equation ⇒m2 + v2 = 34 mv
and then find the derivative.
Differentiating both sides w.r.t. µ we have
Squaring the first equation, we have
dv dv
1 2µ + 2ν = 34 ν + 34 µ
x + y + 2x y = t +
4 4 2 2 2
–2 dµ dµ
t2
1 1 dv dv µ − 17ν
⇒ t2 + + 2x2y2 = t2 + – 2 (from second equation) ⇒ 2[17µ – n] = 2 [µ –17 n] ∴ =
dµ dµ 17µ − v
t2 t2
1
⇒ x2y2 = – 1⇒ y2 = –
 2 
dy
x2 Example 10: If x = exp. tan–1  y − x  , then equals
dy 2 dy 1  x2  dx
∴ 2y = ⇒ = 3  
dx x 3 dx x y
Sol: Simplify the given equation and differentiate.
Example 7: The derivation of Taking log on both sides, we get
 x  1 − x 2   y − x2 
sin 
–1
2
 w.r.t. cos–1 
  1 + x2
 is
 log x = tan  2 
–1
 1+x     x 
M a them a ti cs | 21.33

⇒ tan(log x) = (y – x2) / x2 ⇒ y = x2 + x2 tan (log x) p’(x) = 2yy’ ⇒ p’’(x) = 2yy’’ + 2y’2 ⇒ p’’’(x) = 2yy’’’ + 4y’ y’’
dy  3 d2 y 
∴ = 2x + 2x tan (log x) + x sec2(log x) d d 3
dx Also 2 y  =2 (y y’’)
dx  dx2  dx
 
⇒ 2x [1 + tan (log x)] + x sec2(log x)
= 2[y3y’’’ + 3y’2y’y’’ ] = y2 [2yy’’’ + 6 y’y’’] = p(x) p’’’(x)
 4x3 
d
Example 11: Find cos–1  27 − x  Example 14: If the tangent at the point P(at2, at3) on
dx  
the curve ay2 = x3 intersects the curve again at the point
 4x3    x 3  x   Q, find the point Q.
 − x  –1  4   − 3  
Sol: Let y = cos  27 –1
 = cos   3 
    3   Sol: Solve the equation of the tangent and the equation
of the curve.
x x
= cosθ⇒θ = cos–1  3  ay2 = x3⇒ 2 ay dy/dx = 3x2
3  
 3x2  3a2 t 4 3
∴ y = cos (4 cos θ – 3 cos θ) = cos (cos 3θ) = 3q
–1 3 –1 Slope of tangent at P is  =  = t
 2ay  2 3 2
 p 2a t
x
∴ y = 3cos–1  3  Let Q be (at12, at13). Slope of line
 
at13 − at3 t12 + tt1 + t2
dy −1 1 −3 PQ = =
∴ =3 = . at13 − at2 t1 + t
dx 1 − (x / 9) 3
2
9−x 2
which must be the slope of tangent at P. Hence,
 x2 − y 2  t12 + tt1 + t2 3t
dy
Example 12: If cos  2 
2  = log a then
–1
= = ⇒ 2 t12 – tt1 – t2 = 0
 x + y  dx t1 + t 2
t
Sol: Take cosine on both sides and then apply ⇒ (t1 – t) (2t1 + t) = 0 ⇒ t1 = –
2
componendo and dividendo.
 at2 at3 
 x2 − y 2  Thus, Q has coordinates  ,− 
 4 8 
cos  2
–1 
2  = log a 
x +y 
x2 − y 2 Example 15: Show that the curves ax2 + by2 = 1 and cx2
⇒ = cos (log a) = k (say)
x2 + y 2 1 1 1 1
+ dy2 = 1 cut orthogonally if, – = –
a b c d
by componendo and dividends,
dy
Sol: Equate the product of from the two equations
2 2
(x − y ) + (x + y ) 2 2
k +1 to -1. dx
∴ =
2 2
(x − y ) − (x + y ) 2 2 k −1
Let the two curves cut each other at the point (x1, y1);
then
2x2 k +1 x k +1
⇒ 2
= ∴ = ax12 + by12 = 1 … (i)
−2y k −1 y k −1
& cx12 + dy12 = 1 … (ii)
Differentiating both sides w.r.t. ‘x’ we get
From (i) and (ii), we get
1 x dy dy y
– 2 =0 ⇒ =
y y dx dx x =(a – c) x12 + (b – d) y12 = 0 … (iii)
Slope of the tangent to the curve
Example 13: If y2 = p(x) is a polynomial of degree 3,  dy  ax
ax2 + by2 = 1, at (x1, y1) is given by,   =– 1
d  3d y 2  dx (x1 ,y1 ) by1
then 2 y  is equal to Slope of the tangent to the curve
dx  dx2 
 
cx2 + dy2 = 1, at (x1, y1) is given by,
Sol: Find first order, second order and third order
derivative of p(x).
2 1 . 3 4 | Methods of Differentiation and Applications of Derivatives

 dy  cx dy
=– 1 Differentiating both sides w.r.t. x, then = 2x2 + x
  dx
 dx (x1 ,y1 ) dy1
 Tangents make equal angles with coordinate axes.
If the two curves cut orthogonally, we must have,
 ax1   cx1  dy
∴ = ± 1 or 2x2 + x = ± 1 or
 −   −  dx
 by1   dy1 
2x2 + x + 1 ≠ 0 and 2x2 + x – 1 = 0
⇒ acx12 + bdy12 = 0  … (iv)
or 2x2 + 2x – x – 1 = 0
From (iii) and (iv), we have
(If 2x2 + x + 1 = 0 then x is imaginary)
a−c b−d 1 1 1 1
= ⇒ – = – 1
ac bd a b c d or (2x – 1) (x + 1) ∴ x = ,–1
2
1 2 1 1 1 5
Example 16: Find the acute angle between the curves y From (1), x= , y= . + . =
= | x2 – 1 | and y = | x2 – 3 | at their points of intersection 2 3 8 2 4 24
when x > 0. 2 1 1
and for x = –1, y = – + =–
3 2 6
Sol: Solve the two curves and find the slope for the 1 5   1
two tangents. Proceed to find the angle between the hence point are  ,  and  −1, − 
two lines.  2 24   6

For the intersection of the given curves Example 18: The side of the rectangle of the greatest
| x2 – 1 | – | x2 – 3 | ⇒ (x2 – 1)2 = (x2 – 3)2 area, that can be inscribed in the ellipse x2 + 2y2 = 8,
are given by
⇒ (x2 – 1)2 – (x2 – 3)2 = 0
(A) 4 2 , 4 (B) 4, 2 2
⇒ [(x2 – 1) – (x2 – 3)] [(x2 – 1) + (x2 – 3)] = 0
(C) 2, 2 (D) 2 2 , 2
⇒ [2x2 – 4] = 0 ⇒ 2x2 = 4 ⇒ x = ± 2
Sol: (B) Consider a point on the ellipse and write the
neglecting x = – 2 as x > 0 expression for the area of the rectangle. Then find the
We have point of intersection as x = – 2 maximum area using first and second order derivative.

Here y = | x – 1 | = (x – 1) in the neighbouring of


2 2 x2 y 2
Any point on the ellipse + = 1 is
8 4
x= 2 and y = – (x2 – 3) in the neighbouring of x = 2
(2 2 cos θ, 2 sin θ) [see figure]
 dy   dy 
⇒   = 2x = 2 2 and   = – 2x = – 2 2 y
dx
 c1  dx c1
(22 cos, 2 sin)
Hence, if θ is angle between them,

4 2 0 x
2 2 − ( −2 2) 4 2
⇒ tan θ = = =  
1 + 2 2( −2 2) −7  7 
 
4 2
∴ θ = tan–1  7  A = area of the inscribed rectangle
 
= 4(2 2 cos θ) (2 sin θ) = 8 2 sin2q
dA π
Example 17: At what points on the curve = 16 2 cos 2θ = 0⇒θ =
dθ 4
2 1
y = x3 + x2, then tangent make equals angles with d2 A π
3 2 Also = – 32 2 sin2θ< 0 for θ =
coordinate axes. dθ 2 2
Hence, the inscribed rectangle is of largest area if the
Sol.: Find dy/dx and equate it to ± 1. π π
sides are 4 2 cos and 4 sin   i.e. 4 and 2 2 .
2 3 1 2 4 4
Given curve is y = x + x … (i)
3 2
M a them a ti cs | 21.35

JEE Main/Boards

Exercise 1  1+x + 1−x


Q.18 y = sin–1  
 2 
Methods of Differentiation
 2 2
Q.19 y = sin–1 2ax 1 − a x 
x +3
Q.1 Find the derivative of e , with respect to x.

Q.2 Differentiate, sin(log x), with the respect to x. Q.20 y = a+ a+ x

 ax − b 
Q.3 If x = sinθ, y = – tanθ, find dy/dx. Q.21 y = tan–1  a + bx 
 

Q.4 Differentiate, cos–1 ( x ) , with the respect to x. Q.22 y = log  x + x2 + a2 


 
−1 x
Q.5 Differentiate, em tan , with the respect to x.
Q.23 y = log  sin 1 + x2 
 
Q.6 Differentiate, sin {log(x3 – 1)}, with the respect to x.

Q.7 Differentiate, cos x, with the respect to ex. Application of Derivatives

Q.8 Differentiate the following w.r.t., x : log2(sin x). Q.1 Find the point on the curve y = x2 – 4x + 5, where
tangent to the curve is parallel to x-axis.

Q.9 Differentiate the following w.r.t., x : y = 5log(sin x).


Q.2 If two curves cut orthogonally, then what can we
dy say about the angle between tangents at the point of
Q.10 Find ,when x + y =5 at (4, 9). intersection of the curves.
dx

1 + x  Q.3 Find the slopes of tangent and normal to the curve


Q.11 y = cot–1  1 − x 
  1
f(x)=3x2 –5 at x = .
2
1 + tanx
Q.12 y = Q.4 If the tangent of the curve y = f(x)
1 − tanx
at point (x, y) on the curve is parallel to y– axis, then
dy
what is the value of .
Q.13 y = sin  cos x  dx
 
Q.5 Find a point on the curve y = 2x2 – 6x – 4 at which
 x + a the tangent is parallel to x-axis.
Q.14 y = tan–1  

 1 − ax 
Q.6 Find a point on the curve y = x2 – 4x – 32 at which
−1 x the tangent is parallel to x-axis.
Q.15 y = (sinx)cos

Q.7 Find the equations of tangent and normal to the


Q.16 y cos−1 ( 2 cos x + 3 sinx ) 13 
= curve y = 3
5 − x at (–3, 2).
 

 1 + x2  Q.8 Find equations of tangent to the curve y = 4x − 3 ,


Q.17 y = sec–1   if parallel to x-axis.
 1 − x2 
 
2 1 . 3 6 | Methods of Differentiation and Applications of Derivatives

Q.9 Verify that the point (1, 1) is a point of intersection Q.23 Find the points on the curve y = x3 – 2x2 – 2x at
of the curves x2 =y and x3 +6y= 7 and show that these which the tangent lines are parallel to the line y = 2x – 3.
curves cut orthogonally at this point.
Q.24 Find the angle between the parabolas y2 = 4ax
Q.10 Find the equation of tangent to the parabola y2 = and x2 = 4by at their point of intersection other than
8x which is parallel to line 4x – y + 3 = 0. the origin.

Q.11 Find the equation of tangent to the curve y = – 5x2 Exercise 2


 1 35 
+ 6x + 7 at the point  ,  .
2 4  Methods of Differentiation
Q.12 Find the equation of tangent to the curve xy = c2 Single Correct Choice Type
c 
at the point  ,ck  on it.
k   3x + 4  dy
Q.1 If y = f   & f’(x) = tanx then
2
=
 5x + 6  dx
Q.13 Prove that the tangents to the curve y = x3 + 6 at
the points (–1, 5) and (1, 7) are parallel. (A) tan x3
2
 3x + 4  1
Q.14 At what point on the curve y = x2 does the tangent (B) – 2 tan   .
make an angle of 45º with x-axis?  5x + 6  (5x + 6 )2

x2 y2  3tanx2 + 4 
Q.15 Find the point (s) on the curve + =1 (C) f   tan x2
9 4  5 tanx2 + 6 
parallel to y-axis.  
(D) None
1
Q.16 Find the slope of the normal to the curve x =
y= 2t at t = 2. t x10
Q.2 Let g is the inverse function of f & f’(x) = .
2
If g(2) = a then g’(2) is equal to (1 + x )
Q.17 Show that equation of the tangent to the curve 5 1 + a2 a10 1 + a10
x2 y 2 x x y y (A) (B) (C) (D)
+ =1 at (x0, y0) is 0 + 0 =1. 210 a10 1 + a2 a2
2
a b 2
a2 b2
1 1
Q.3 If y = +
n−m p −m m−n
Q.18 Find the equation of the normal lines to the curve y 1+x +x 1+x + xp −n
= 4x3– 3x + 5 which are parallel to the line 9y + x + 3 = 0.
1
+
m−p
1+x + xn−p
Q.19 Find the equation of normal line to the curve y(x –
2) (x – 3) – x = 7 = 0 at the point where it meets x-axis. dy np
Then at em is equal to :
dx
Q.20 Find the equation of tangent to the curve x +
(A) emnp (B) emn/p (C) enp/m (D) None
y = a at the point (x1, y1) and show that the sum of
its intercepts on axis is constant. Q.4 Let f is differentiable in (0, 6) & f’(4) = 5 then lim
x →2
f(4) − f(x2 )
=
Q.21 Find the equation of the normals to the curve 2−x
3x2 + y2= 8 parallel to the line x +3y= 4. (A) 5 (B) 5/4 (C) 10 (D) 20

Q.22 Find the equation of the tangents to the curve


Q.5 Let  = Lim xm(lnx)n where m, n∈N then
 a2 a2  x →0
x + y = a at the point  ,  .
 4 4 (A)  is independent of m and n
(B)  is independent of m and depend on m
M a them a ti cs | 21.37

(C)  is independent of n and depend on m 7 


(C) (2, ∞) (D)  , ∞ 
(D)  is dependent on both m and n 2 

Q.13 Let f(x) = sin x; g(x) = x2& h(x) = loge x & f(x) =
cos x x 1
f '(x) df(x)
Q.6 Let f(x) = 2sinx x2 2x then alim = h[g(f(x))] then is equal to :
→0 x dx2
tanx x 1
(A) 2 cosec3x (B) 2 cost (x2) – 4x2 cosec2 (x2)
(A) 2 (B) – 2 (C) – 1 (D) 1 (C) 2x cot x2 (D) – 2cosec2 x

cos x sinx cos x Q.14 Let f(x) = xn, n being a non-negative integer. The
π
Q.7 Let f(x) cos2x sin2x 2cos2x then f’   = b
cos3x sin3x 3cos3x 2 number of value of n for which f’ (p + q) = f’
ab + 2ay
(p) + f’(q) is valid for all p, q > 0 is :
(A) 0 (B) – 12 (C) 4 (D) 1
(A) 0 (B) 1 (C) 2 (D) None of these
 2x 
Q.8 If y = sin–1  2  , then dy/dx at x = π/2 is 100
1 + x  f(101)
Q.15 If f(x) = ∏ (x − n)n(101−n) ; then f '(101)
=
−8 4 n=1
(A) (B)
2 2
π +4 π +4 1 1
(A) 5050 (B) (C) 10010 (D)
8 5050 10010
(C) (D) Does not exists
π2 + 4
 3x2 + 2x − 1 1
Q.9 If f(4) = g(4) = 2; f’(4) = 9 ; g’(4) = 6 then  2 for x ≠
Q.16 Let f(x)=  6x − 5x + 1 3 then f’  1 
 
f(x) − g(x)  1 3
lim is equal to :  −4 for x =
a→ 4  3
x −2
3 (A) is equal to – 9 (B) is equal to – 27
(A) 3 2 (B) (C) 0 (D) None
2 (C) is equal to 27 (D) does not exist
d2 x
Q.10 If y = x + ex then is : Q.17 Let f(x) be a quadratic expression which is positive
dy 2
for all real x. If g(x) = f(x) + f’(x) + f”(x), then for any real
ex
(A) ex (B) – x, which one is correct.
(1 + ex )3
(A) g(x) < 0 (B) g(x) > 0 (C) g(x) = 0 (D) g(x) ≥ 0
ex −1
(C) – (D)
(1 + ex )2 (1 + ex )3 x4 + 4 dy 
Q.18 If y = then  is :
2
x − 2x + 2 dx  x =1/2
Q.11 If f is twice differentiable such that f”(x) = – f(x),
f’(x) = g(x) h’(x) = [f(x)]2 + [g(x)]2 and h(0) = 2, h(1) =4,
(A) 3 (B) – 1 (C) 4 (D) None
then the equation y = h(x) represents :
(A) A curve of degree 2
Q.19 A function f, defined for all positive real numbers,
(B) A curve passing through the origin satisfies the equation f(x2) = x3 for every x > 0. Then the
(C) A straight line with slope 2 value of f’(4)

(D) A straight line with y intercept equal to– 2 (A) 12 (B) 3 (C) 3/2 (D) Cannot be determined

Q.12 Let f(x) = x + 3 ln(x – 2) & g(x) = x + 5 n(x – 1), Q.20 If x = sin t and y = sin 3t, then the value of ‘K’ for
then the set of x satisfying the inequality f’(x) < g’(x) is d2 y dy
which (1 – x2) –x + Ky = 0 is
 7  dx 2 dx
 7
(A)  2,  (B) (1, 2) ∪  − , ∞  (A) 3 (B) 6 (C) 12 (D) 9
 2  2 
2 1 . 3 8 | Methods of Differentiation and Applications of Derivatives

Q.21 If x = lnt & y = t2 – 1 then y”(1) at t = 1 is Q.7 The subnormal at any point on the curve xyn = an+1
is constant for :
(A) 2 (B) 4 (C) 3 (D) None
(A) n = 0 (B) n =1
(C) n = –2 (D) No value of n
Application of Derivatives
Q.8 Equation of the line through the point (1/2, 2) and
Single Correct Choice Type −x2
tangent to the parabola y = +2 and secant to the
2
Q.1 The angle at which the curve y = kekx intersects the curve y= 4 − x2 is
y-axis is (A) 2x + 2y – 5 = 0 (B) 2x + 2y – 3 = 0
(A) tan–1k2 (B) cot–1(k2) (C) y – 2 = 0 (D) None of these
 4 
(C) sec–1  1 + k  (D) None
Q.9 Two curves C1: y = x2 – 3 and C2 : y = kx2, k ∈ R
intersect each other at two different point. The tangent
Q.2 The angle between the tangent lines to the graph
x drawn to C2 at one of the point of intersection A ≡ (a,
y1), (a > 0) meets C1 again at B(1, y2) (y1≠ y2). The value
of the function f(x)= ∫ (2t − 5) dt at the point where the of ‘a’ is
2
graph cuts the x-axis is -
(A) 4 (B) 3 (C) 2 (D) 1
(A) π/6 (B) π/4 (C) π/3 (D) π/2
Q.10 Number of roots of the equation x2.e2–|x| = 1 is:
Q.3 If a variable tangent to the curve x y = c makes 2 3
(A) 2 (B) 4 (C) 6 (D) Zero
intercepts a, b on x and y axis respectively then the
value of a2b is
Q.11 The x-intercept of the tangent at any arbitrary
4 3
27 3 4
(A) 27c3 (B) c c (C) (D) c3 a b
27 4 9 point of the curve + = 1 is proportional to
2
 π x y2
x sin for x > 0
Q.4 Consider the function f(x) =  x (A) Square of the abscissa of the point of tangency
 0 for x = 0
 (B) Square root of the abscissa of the point of tangency
then the number of points in (0, 1) where the derivative
(C) Cube of the abscissa of the point of tangency
(D) Cube root of the abscissa of the point of tangency
f’(x) vanishes, is
(A) 0 (B) 1 (C) 2 (D) infinite
Q.12 The line which is parallel to x-axis and crosses the
π
curve y= x at an angle of is
Q.5 The tangent to the graph of the function y = f(x) 4
at the point with abscissa x = a forms with the x-axis (A) y = –1/2 (B) x = ½ (C) y = 1/4 (D) y = ½
an angle of π/3 and at the point with abscissa x = b
at an angle of π/4, then the value of the integral, Q.13 The lines tangent to the curves y3 – x2y + 5y – 2x
h
= 0 and x4 – x3y2 + 5x + 2y = 0 at the origin intersect at
∫ f '(x).f "(x)dx is equal to
an angle θ equal to
a
(A) 1 (B) 0 (C) – 3 (D) – 1
(A) π/6 (B) π/4 (C) π/3 (D) π/2
[assume f”(x) to be continuous]
x
 1
Q.6 Let C be the curve y = x (where x takes all real
3
Q.14 Consider f(x) = ∫  t + t  dt and g(x) = 'f‘ for
1  0
values). The tangent at A meets the curve again at B. If x ∈  ,3
the gradient at B is K times the gradient at A then K is 2 
equal to
(A) 4 (B) 2 (C) – 2 (D) ¼
M a them a ti cs | 21.39

If P is a point on the curve y = g(x) such that the Q.5 The equation of the common tangent to the curves
tangent to this curve at P is parallel to a chord joining y2= 8x and xy = – 1 is (2002)
 1  1  (A) 3y = 9x + 2 (B) y = 2x + 1
the points  , g    and (3, g(3)) of the curve, then
 2  2 
(C) 2y = x + 8 (D) y = x + 2
the coordinates of the point P
 7 65 
(A) can’t be found out (B)  ,  Q.6 Tangents are drawn to the ellipse x2 + 2y2 = 2, then
 4 28  the locus of the mid point of the intercept made by the
 3 5  tangents between the coordinate axes is  (2004)
(C) (1, 2) (D)  , 
 2 6 1 1 1 1
  (A) + = 1 (B) + =1
2 2 2
2x 4y 4x 2y 2
Q.15 The co-ordinates of the point on the curve 9y2 = x3
where the normal to the curve makes equal intercepts x2 y 2 x2 y 2
(C) + =1 (D) + =1
with the axes is 2 4 4 2
6 2 6 
 1
(A)  1, 
 3
(
(B) 3, 3 )  8
(C)  4, 
 3
(D)  ,


5 5 5 

Q.7 The angle between the tangent drawn from the
point (1, 4) to the parabola y2 = 4x is  (2004)
π π π π
(A) (B) (C) (D)
Previous Years’ Questions 6 4 3 2

Q.1 The normal to the curve x = a (cos θ + θ sin θ), y = Q.8 The tangent at (1, 7) to the curve x2 – y – 6 touches
a (sin θ – θ cos θ) at any / point "θ” is such that (1983) the circle x2 + y2 + 16 x + 12y + c = 0 at (2005)
(A) It makes a constant angle with the x-axis (A) (6, 7) (B) (– 6, 7) (C) (6, –7) (D) (–6, – 7)
(B) It passes through the origin
Q.9 The tangent to the curve y = ex drawn at the point
(C) It is at a constant distance from the origin
(c, ec) intersects the line joining the point (c – 1, ec–1) and
(D) None of the above (c + 1, ec+1)  (2007)
(A) On the left of x = c (B) On the right of x = c
Q.2 The slope of tangent to a curve y = f(x) at [x, f(x)] is
2x + 1. If the curve passes through the point (1, 2),then (C) At no point (D) At all points
the area bounded by the curve, the x-axis and the line
x = 1 is  (1995)   1 
(x − 1)sin   , if x ≠ 1
(A) 5/6 (B) 6/5 (C) 1/6 (D) 6 Q.10 Let f(x) =   x −1 .
 0, if x = 1

Q.3If the normal to the curve y = f(x) at the point (3, 4)
Then which one of the following is true ? (2008)

makes an angle with the positive x-axis, then f’(3) is
4 (A) f is neither differentiable at x = 0 not at x = 1
equal to (2000) (B) f is differentiable at x = 0 and at x = 1
(A) – 1 (B) –3/4 (C) 4/3 (D) 1 (C) f is differentiable at x = 0 but not at x = 1
(D) f is differentiable at x = 1 but not at x = 0
Q.4 The point(s) on the curve y3 + 3x2 = 12y where the
tangent is vertical, is (are)  (2002)
Q.11 The area of the plane region bounded by the
 4   11 
(A)  ± , −2  (B)  ± ,0  curves x + 2y 2 =
0 and x + 3y 2 =
1 is equal to  (2008)
 3   3 
 5 1 2 4
(A) (B) (C) (D)
 4  3 3 3 3
(C) (0, 0) (D)  ± ,2 
 3 
2 1 . 4 0 | Methods of Differentiation and Applications of Derivatives

JEE Advanced/Boards

Exercise 1 [1 + (dy / dx)2 ]3/2


Q.10 Show that R = can be reduced
to the form d2 y / dx2
Methods of Differentiation
1 1
R2/3 = 2 +
Q.1 Let y = x sin kx. Find the possible value of k for (d y dx2 )2/3 (d2 x dy 2 )2/3
d2 y
which the differential equation +y= 2k cos kx holds Q.11 Suppose f and g are two functions such that f, g:
dx2
true for all x ∈ R. R → R, f(x) = ln(x+ 1 + x2 ) then find the value of xeg(x)
'
  1 
Q.2 Find a polynomial function f(x) such that f(2x) =  f    + g’(x) at x = 1.
f(x) f”(x).   x 
x+y
Q.12 Let f(x) be a derivative function at x = 0 & f  
Q.3 Let f and g be two real-valued differentiable  k 
function on R If f’(x) = g(x) and g’(x) = f(x) “x ∈ R and f(x) + f(y)
= (k ∈ R, k ≠ 0, 2). Show that f(x) is either a
k
( )
f(3) = 5, f’(3) = 4 then find the value of f 2 ( π) − g2 ( π) . zero or an odd linear function.
2
d y
Q.4 Find the value of the expression y3 2 on the (x − a)4 (x − a)3 1
dx
ellipse 3x2 + 4y2 = 12. Q.13 If f(x) = (x − b) 4
(x − b) 3
1 then
4 3
(x − c) (x − c) 1
Q.5 The function f : R → R satisfies f(x2) . f”(x) = f’(x) .
f’(x2) for all real x. Given that f(1) = 1 and f”’(1) = 8, (x − a)4 (x − a)2 1
compute the value of f’(1) + f”(1). 4 2
f’(x) = λ . (x − b) (x − b) 1 .
1 1
− 4 2
Q.6 If 2x = y5 + y 5 then (x2 – 1) (x − c) (x − c) 1
d2 y dy
+x = ky, then find the value of ‘k’. Find the value of λ.
dx 2 dx
Q.14 Let P(x) be a polynomial of degree 4 such that
Q.7 If the dependent variable y is changed to ‘z’ by the P(1) = P(3) = P(5) = P’(7) = 0. If the real number x
substitution y = tan z then the differential equation ≠ 1, 3, 5 is such that P (x) = 0 can be expressed as
2 x = p/q where ‘p’ and ‘q’ are relatively prime, then find
d2 y 2(1 + y)  dy  d2 z (p + q).
=1+ is changed to = cos2z + k
2  dx  2
dx2 1+y   dx
2
 dz 
  , then find the value of k. Q.15 If f : R → R is a function such that
 dx  f(x)=x3+x2f’(1)+xf”(2)+f”’(3) for all x ∈ R, then prove
 x that f(2) = f(1) – f(0).
Q.8 Show that the substitution z = ln  tan  changes
the equation  2 cos(x + x2 ) sin(x + x2 ) − cos(x + x2 )

d2 y dy sin(x − x2 ) cos(x − x2 ) sin(x − x2 )


+ cot x + 4 y cosec2x = 0 Q.16 If f(x) =
dx 2 dx sin2x 0 sin2x2
 d2 y  then find f’(x)
to   + 4y = 0
 dz 2 
 
a+ x b+ x c+ x
sinx Q.17 Let f(x) =  + x m + x n + x . Show that f”(x) = 0
Q.9 Let f(x) = if x ≠ 0 and f(0) = 1. Define the
x
p+ x q+ x r + x
function f’(x) for all x and find f”(0) if it exist.
M a them a ti cs | 21.41

and that f(x) = f(0) + kx where k denotes the sum of all The function g(x) is defined by g(x) = eax + f(x) ∀ x ∈ R,
the co-factors of the elements in f(0) where ‘a’ is any constant. If g’(0) + g”(0) = 0. Find the
value(s) of ‘a’.
1 1
Q.18 If y = tan-1 2
+ tan–1 2
+ tan–1 Application of derivatives
x + x +1 x + 3x + 3
1 1
2 + tan–1 2 = + ….. to n terms. Q.1 Find the equations of the tangents drawn to the
x + 5x + 7 x + 7x + 13
curves y2 – 2x3 – 4y + 8 = 0 from the point (1, 2).
Find dy/dx, expressing your answer in 2 terms.
7
Q.2 The tangent to y = ax2 + bx + at (1,2) is parallel
2
Q.19 If Y = sX and Z = tX, where all the letter denotes the to the normal at the point (– 2, 2) on the curvey = x2 +
functions of x and suffixes denotes the differentiation 6x +10. Find the value of a and b.
X Y Z s1 t1 Q.3 Find the point of intersection of the tangents
w.r.t. then prove that X1 Y1 Z1 = X3 s t2
2 drawn to the curve x2y = 1 – y at the points where it is
X2 Y2 Z2 intersected by the curve xy = 1 – y .

u
Q.20 If y = tan–1 & Q.4 Find the equation of the normal to the curve y =
1 − u2 (1+ x)y + sin–1 (sin2x) at x = 0.
1  1  1 
x = sec–1 2 ,u ∈  0,  ∪  ,1  Q.5 A function is defined parametrically by the equation
2u − 1  2  2 
dy  2 1
prove that 2 +1=0 2t + t sin if t ≠ 0
dx f(t) = x =  t and
 0 if t = 0

x  1−x 
Q.21 If y = tan–1 + sin  2 tan−1 , 1
1+ 1−x  2
1 + x   sint
2
if t ≠ 0
 g(t) = y =  t
dy  0
The find for x ∈ (–1, 1)  if t = 0
dx
Find the equation of the tangent and normal at the
1 + sinx + 1 − sinx point for t = 0 is exist.
Q.22 If y = cot–1 ,
1 + sinx − 1 − sinx
41x3
dy  π π  Q.6 A line is tangent to the curve f(x)= at the point
find if x ∈  0,  ∪  , π  . 3
dx  2 2  P in the first quadrant, and has a slope of 2009. This line
intersects the y-axis at(0, b). Find the value of ‘b’.
Q.23 Prove that the second order derivative of a
single valued function parametrically represented by x Q.7 Find all the tangents to the curve y = cos (x + y), –
= φ(t) and y = Ψ(t), α< t <β where φ(t) and Ψ(t) are 2π≤ x ≤ 2π, that are parallel to the line x + 2y = 0
differentiable functions and φ’(t) ≠ 0 is given
 dx   d2 y   d2 x   dy  Q.8 There is a point (p, q) on the graph of f(x) = x2 and
   2  −  2   
d2 y  dt   dt   dt   dt  −8
by = a point (r, s) on the graph of g(x) = where p > 0 and
2 3
dx  dx  x
  r > 0. If the line through (p,q) and (r, s) is also tangent to
 dt 
both the curves at these points respectively, then find
Q.24 (a) If y = y(x) and it follows the relation exy + y cos the value of (p + r)
x = 2, then find (i) y’(0) and (ii) y”(0).
(b) A twice differentiable function f(x) is defined for all Q.9 (i) Use differentials to approximate the values of; (a)
real numbers and satisfies the following conditions f(0) 3
36.6 and (b) 26 .
= 2; f’(0) = –5 and f”(0) = 3
2 1 . 4 2 | Methods of Differentiation and Applications of Derivatives

(ii) If the radius of a sphere is measured as 9 cm with Q.18 Let the function f : [– 4, 4] → [–1, 1] be defined
an error of 0.03 cm, then find the approximate error in implicitly by the equation x=5y – y5=0.
calculating its volume.
Find the area of triangle formed by tangent and normal
to f(x) at x = 0 and the line y = 5.
Q.10 The chord of the parabola y=– a2x2 +5ax – 4
 1
1 Q.19 The normal at the point P  2,  on the curve
touches the curve y = at the point x = 2 and is  2
1−x xy = 1, meets the curve again at Q. If m is the slope of
bisected by that point. Find ‘a’.
the curve at Q, then find | m |.
Q.11 Tangent at a point P1 [other than (0, 0)] on the
curve y = x3 meets the curve again at P2. The tangent Q.20 Let C be the curve f(x) = ln2x + 2lnx and A(a, f(a),
at P2 meets the curve at P3& so on. Show that the B(b, (f(b)) where (a < b) are the points of tangency of
abscissae of P1, P2, P3 ……. Pn, form a GP. Also find the two tangents drawn from origin to the curve C.
area(P1P2P3 ) (i) Find the value of the product ab.
ratio .
area(P2P3P4 ) (ii) Find the number of values of x satisfying the equation
5x f’(x) – x ln 10 – 10 = 0.
Q.12 Determine a differentiable function y = f(x) which
1
satisfies f’(x) = [f(x)]2 and f(0) = – . Find also the Q.21 A particle moves along the curve 6y = x3 + 2. Find
2
the points on the curve at which the y coordinate is
equation of the tangent at the point where the curve
changing 8 times as fast as the x coordinate
crosses the y-axis.

Q.22 A man 1.5 m tall walks away from a lamp post 4.5
Q.13 The curve y = ax3 + bx2 + cx + 5, touches the
m high at the rate of 4 km/hr.
x-axis at P(–2, 0) & cuts the y-axis at a point Q where its
gradient is 3. Find a, b, c (i) How fast is the father end of the shadow moving on
the pavement?
Q.14 Find the gradient of the line passing through the (ii) How fast is his shadow lengthening?
point (2, 8) and touching the curve y = x2.
Q.23 A water tank has the shape of a right circular cone
Q.15 Let f : {0, ∞) → R be a continuous, strictly increasing with its vertex down. Its altitude is 10 cm and the radius
x
2 of the base is 15 cm. Water leaks out of the bottom at a
function such thatf3(x) = ∫ tf
(t)dt . If a normal is drawn
constant rate of 1 cu. cm/sec. Water is poured into the
0 −1 tank at a constant rate of C cu. cm/sec. Compute C so
to the curve y=f(x) with gradient , then find the
2 that the water level will be rising at the rate of 4 cm/sec
intercept made by it on the y-axis. at the instant when the water is 2 cm deep.

Q.16 The graph of a certain function f contains the point Q.24 Water is dripping out from a conical funnel of
(0, 2) and has the property that for each number’p the π
line tangent to y = f(x) at (p, f(p)) intersect the x-axis at semi vertical angle , at the uniform rate of 2 cm3/sec
4
p + 2. Find f(x)
through a tiny hole at the vertex at the bottom. When
the slant height of the water is 4 cm, find the rate of
Q.17 (a) Find the value of n so that the subnormal at decrease of the slant height of the water.
any point on the curve xyn = an+1 may be constant(b)
Show that in the curve y = a.ln (x2 – a2)sum of the length
Q.25 Sand is pouring from a pipe at the rate of 12 cc/
of tangent & subtangent varies as the product of the
sec. The falling sand forms a cone on the ground in
coordinates of the point of contact
such a way that the height of the cone is always 1/6th
(c) If the two curve C1 : x = y2 and C2 : xy = k cut at right of the radius of the base. How fast is the height of the
angles find the value of k. sand cone increasing when the height is 4 cm?
M a them a ti cs | 21.43

Q.26 A circular ink blot grows at the rate of 2 cm2 per Q.2 The function f(x) = ex + x, being differentiable and
second. Find the rate at which the radius is increasing one to one to one, has a differentiable inverse f–1(x). The
6 22 d –1
after 2 seconds. Use π = value of (f ) at the point f(ln2) is
11 7 dx
1 1 1
Q.27 A variable ∆ABC in the xy plane has its orthocentre (A) (B) (C) (D) None
at vertex ‘B’, a fixed vertex ‘A’ at the origin and the third ln2 3 4
7x2
vertex ‘C’ restricted to lie on the parabola y = 1 + . Q.3 f ’(x) = g(x) and g’(x) = – f(x) for all real x and f(5) = 2 = f
36
‘(5) then f2(10) + g2(10) is-
The point B starts at the point (0, 1) at time t = 0 and
moves upward along the y axis at a constant velocity of (A) 2 (B) 4 (C) 8 (D) None
2 cm/sec. How fast is the area of the triangle increasing
7
when t = sec. Q.4 Differential coefficient of
2 1 1 1
  +m  n−   m+n   −m  n+   m−n
Q.28 At time t > 0, the volume of a sphere is increasing  x m−n  .  x n−   .  x  −m  w.r.t. is
at a rate proportional to the reciprocal of its radius. At      
     
t = 0, the radius of the sphere is 1 unit and at t = 15 the
radius is 2 units. (A) 1 (B) 0 (C) –1 (D) xlmn

(a) Find the radius of the sphere as a function of time t x)


Q.5 Let f(x)=(xx)x and g(x)= x(x then:
(b) At what time t will the volume of the sphere be 27
times its volume at t = 0 (A) f’(1) = 1 and g’(1) = 2 (B) f’(1) = 2 and g’(1) = 1
(C) f’(1) = 1 and g’(1) = 0 (D) f’(1) = 1 and g’(1) = 1
Q.29 Water is flowing out at the rate of 6 m3/min from 1 1
a reservoir shaped like a hemispherical bowl of radius R
− (x2 − 1)y "+ xy '
Q.6 If y m + y m =2x, then the value of
= 13 m. The volume of water in the hemispherical bowl is equal to value equal to y
π 2
is given by V = . y (3R – y) when the water is y meter
deep . Find 3 (A) 4 m2 (B) 2 m2 (C) m2 (D) – m2
(a) At what rate is the water level changing when the  d 
water is 8 m deep? Q.7 If y2 = P(x), is a polynomial of degree 3, then 2  
 dx 
(b) At what rate is the radius of the water surface  3 d2 y 
 y . 2  equals:
changing when the water is 8 m deep?  dx 

(A) p”’(x) + P’(x) (B) P”(x).P”’(x)
Exercise 2 (C) P(x).P”’(x) (D) a constant

Methods of Differentiation − x3
Q.8 Given f(x) = + x2 sin 1.5 a – x sin a . sin 2a – 5
3
Single Correct Choice Type are sin (a2– 8a + 17) then :

x x x x x x dy (A) f(x) is not defined at x = sin 8


Q.1 If y = ….∞, then
a+ b+ a+ b+ a+ b+ dx (B) f ‘ (sin 8) > 0
a b (C) f ‘ (x) is not defined at x = sin 8
(A) (B)
ab + 2ay ab + 2by
(D) f ‘ (sin 8) < 0
a b
(C) (D)
ab + 2by ab + 2ay cos6x + 6 cos 4x + 15cos2x + 10 dy
Q.9 If y = , then =
cos5x + 5cos3x + 10 cos x dx

(A) 2 ain x + cos x (B) – 2 sin x


(C) cos 2x (D) sin 2x
2 1 . 4 4 | Methods of Differentiation and Applications of Derivatives

Q.10 A curve is parametrically represented by y = R(1 – Multiple Correct Choice Type


d2 y
cos θ) & x = R(θ – sinθ), then at θ = π is -
dx2  1 + x2 − 1 
 
Q.18 Let f(x) = tan–1  x  , then
1 1 1 1  
(A) – (B) (C) (D) –
2R 4R 2R 4R
(A) f(x) = 112 tan–1x ∆ x∈R – {0}
1
Q.11 If f(x) = (1 + x)n then the value of f(0) + f ‘ (0) + …. (B) f ‘ (x) = ζ x∈R – {0}
2(1 + x2 )
f n (0)
+ is – (C) f(x) is an odd function
n!
(A) n (B) 2n (C) 2n–1 (D) None (D) f(x) + f(– x) = p

dy
Q.19 If y = tan x tan 2x tan 3x then has the value to:
Q.12 If the function y = e + 2e is a solution of the
4x –x dx
d3 y dy (A) 3 sec2 3x tan x tan 2x + sec2 x tan 2x tan 3x + 2 sec2
− 13
3 dx
differential equation dx = K. then the value of K 2x tan 3x tan x
y
(B) 2y (cosec 2x + 2 cosec 4x + 3 cosec 6x)
(A) 4 (B) 6 (C) 9 (D) 12 (C) 3 sec2 3x – 2 sec22x – sec2 x
(D) sec2 x + 2 sec2 2x + 3 sec2 3x
d2 y
Q.13 x4 + 3x2y2 + 7xy3 + 4x3y – 15y4 = 0, then at
(1, 1) is - dx2 dy
Q.20 Let y = √x + Öx + x + ......∞ then
dx
(A) 2 (B) 1 (C) 7 (D) 0 1 x 1 y
(A) (B) (C) (D)
2y − 1 x − 2y 1 + 4x 2x + y
x
Q.14 If f(x) = ee . Let g(x) be it’s inverse then g’ (x) at
x = 2 is - dy
Q.21If 2x + 2y = 2x+y then has the value equal to
dx
n2 1 2y 1 2x (1 − 2y )
(A) (B) (C) 2n2 (D) e2 (A) – (B) (C) 1 − 2y (D)
2 2n2 x
2 1 − 2x 2y (2x − 1)
 1 − 2n | x |   3 + 2n | x | 
Q.15 y=tan–1  1 + 2n | x |  +tan–1  1 − 6n | x |  , then dy
    Q.22 If y + x + y – x = c, then is equal to
dx
d2 y
equals 2x x
dx2 (A)
2
(B)
c y + y 2 − x2
(A) 2 (B) 1 (C) 0 (D) –1
y − y 2 − x2 c2
xx
(C) (D)
Q.16 lim (x − x x ) equals - x 2y
x →0 +
(A) 1 (B) –1 (C) 0 (D) None of these
Application of Derivatives

Q.17 lim {(cot x)x + (1 – cos x)cosecx} is equal to - Single Correct Choice Type
x →0
(A) 2 (B) +1
3 2
(C) 0 (D) None of these Q.1 The line y = – x and y = – x intersect the curve
2 5
3x2 + 4xy + 5y2 – 4 = 0 at the  P(x1, y1) 2
), )
point P and O respectively. 5 2
Q (x2, y2)
The tangent drawn to the 3
curve at P and Q 2
3
(A) Intersect each other at angle 2
), )
of 45º 5
M a them a ti cs | 21.45

(B) Are parallel to each other x


n
Q.8 The equation of the tangent to the curve   +
(C) Are perpendicular to each other n a
y
(D) None of these   = 2 (n ∈ N) at the point with abscissa equal to
b
‘a’ can be
Q.2 A curve is represented by the equations x = sec2 t
and y = cot t where t is a parameter. If the tangent at x y x y
(A)   +   = 2 (B)   –   = 2
the point P on the curve where t = π/4 meets the curve  a b  a b
again at the point Q then | PQ | is equal to
x y x y
5 3 5 5 (C)   –   = 0 (D)   +   = 0
(A) (B)  a b  a b
2 2
2 5 3 5
(C) (D) Multiple Correct Choice Type
3 2
 x35 x x 1
if x ≤ 1 Q.9 If + = 1 is a tangent to the curve x = Kt, y = ,
Q.3 Let f(x) =  then the number of a b t
3
−(x – 2) if x > 1 K > 0 then
critical points on the graph of the function is
(A) a > 0, b > 0 (B) a > 0, b < 0
(A) 1 (B) 2 (C) 3 (D) 4
(C) a < 0, b > 0 (D) a < 0, b < 0

Q.4 At any two points of the curve represented dy


parametrically by x = a(2 cos t –cos 2t), y = a(2 sin t – sin 2t) Q.10 The abscissa of the point on the curve = a + x,
dx
the tangents are parallel to the axis of x corresponding the tangent at which cuts off equal intersects from the
to the values of the parameter t differeing from each co-ordinate axes is (a > 0)
other by
a a
(A) 2π/3 (B) 3π/4 (C) π/2 (D) π/3 (A) (B) – (C) a 2 (D) – a 2
2 2

Q.5 At the point P (a, a”) on the graph of y = xn(n ∈ Q.11 The parabola y = x2 + px + q cuts the straight line y
N) in the first quadrant a normal is drawn. The normal = 2x – 3 at a point with adscissa 1. If the distance between
1 the vertex of the parabola and the x-axis is least then
intersects the y-axis at the point (0, b). If lim b = ,
then n equal
a→ 0 2
(A) p = 0 & q = – 2
(A) 1 (B) 3 (C) 2 (D) 4 (B) p = – 2 & q = 0
(C) least distance between the parabola and x–axis is 2
 −x2 for x > 0
Q.6 Let f (x) =  . Then x intercept of (D) least distance between the parabola and x–axis is 1
2
 x + 8 for x ≥ 0
the line that is tangent to the graph of f(x) is Q.12 The co-ordinates of the point(s) on the graph ……
(A) zero (B) – 1 (C) – 2 (D) – 4 x3 5x2
function, f(x) = – + 7x – 4 where the tangent
3 2
Q.7 The ordinate of all points on the curve drawn cut off intercepts from the co-ordinate axes
which are equal in magnitude but opposite in sign, is
1 (A) (2, 8/3) (B) (3, 7/2)
y= where the tangent is horizontal, is -
2sin x + 3cos2 x
2
(C) (1, 5/6) (D) None of these
(A) Always equal to 1/2
(B) Always equal to 1/3 Q.13 Equation of a tangent to the curve y cot x = y3 tan
π
(C) 1/2 or 1/3 according as n is an even or an odd x at the point where the abscissa is is
4
integer
(A) 4x + 2y = π + 2 (B) 4x – 2y = π + 2
(D) 1/2 or 1/3 according as n is an odd or an odd integer
(C) x = 0 (D) y = 0
2 1 . 4 6 | Methods of Differentiation and Applications of Derivatives

Q.14 The angle made by the tangent of the curve x = (v > u). At what point on the shore should be land so that
a(t + sin t cos t) ;y = a(1 + sin t)2 with the x-axis at any he reaches his house in the shortest possible time?(1983)
point on it is -
1 1 − sint 1 1 + sint Q.5 Find the coordinates of the point on the curve y =
(A) (π + 2t) (B) (C) (2t – π) (D)
4 cos t 4 cos2t x
, where the tangent to the curve has the greatest
1 + x2
Q.15 Consider the curve represented parametrically by the slope. (1997)
equation x = t3 – 4t2 – 3t and y = 2t2+ 3t – 5 where t ∈ R
If H denotes the number of point on the curve where Q.6 Find all the tangents to the curve y = cos (x + y), –
the tangent is horizontal and V the number of point 2π≤ x 2π, that are parallel to the line x + 2y = 0 (1997)
where the tangent is vertical then
(A) H = 2 and V = 1 (B) H = 1 and V = 2 Q.7 Find the point on the curve 4x2 + a2y2 = 4a2, 4 < a2<
(C) H = 2 and V = 2 (D) H = 1 and V = 1 8 that is farthest from the point (0, –2). (1987)

Q.8 Three normals are drawn from the point (c, 0) to the
Previous Years’ Questions 1
curve y2 = x. Show that c must be greater than . One
normal is always the x-axis. Find c for which the2 other
Q.1 If the line ax + bx + c = 0 is a normal to the curve two normals are perpendicular to each other  (1991)
xy = 1, then (1986)
(A) a > 0 , b > 0 (B) a > 0, b < 0 Q.9 What normal to the curve y = x2 forms the shortest
chord? (1992)
(C) a < 0, b > 0 (D) a < 0, b < 0

Q.10 Find the equation of the normal to the curve y =


Q.2 On the ellipse 4x2 + 9y2 = 1, the point at which the
(1 + x)y + sin–1(sin2x) at x = 0 (1993)
tangents are parallel to the line 8x = 9y, are  (1999)

2 1  2 1  2 1 2 1 Q.11 Tangent at a point P1{other than (0, 0)} on the


(A)  ,  (B)  − ,  (C)  − , −  (D)  ,− 
5 5  5 5  5 5 5 5 curve y = x3 meets the curve again at P2. The tangent
at P2 meets the curve at P3, and so on. Show that the
abscissae of P1, P2, P3, …. < Pn, form a GP. Also find the
Q.3 Let C be the curve y3 – 3xy + 2 = 0. If H is the set of
ratio. [Area (dP1P2P3)] / [area (dP2P3P4)]  (1993)
points on the curve C where the tangent is horizontal
and V is the set of points on the curve C where the
tangent is vertical, then H = ………. and V = ……..[1997] Q.12 A rectangular sheet of fixed perimeter with sides
having their lengths in the ratio 8 : 15 is converted
an open rectangular box by folding after removing
Q.4 A swimmer S is in the sea at a distance d km from
square of equal area from all four corners. If the total
the closest point A on a straight shore. The house of the
area of removed squares is 100, the resulting box has
swimmer is on the shore at a distance L km from A. He can
maximum volume. Then the lengths of the sides of the
swim at a speed of u km/h and walk at a speed of v km/h
rectangular sheet are  (2013)
(A) 24 (B) 32 (C) 45 (D) 60

Q.13 Match List I with List II and select the correct answer using the code given below the lists : (2013)

List - I List – II
P 1/2 1.
 1  cos(tan−1 y) + y sin(tan−1 y)   1 5
   + y4  2 3
 2 −1 −1  
 y  cot(sin y) + tan(sin y)  

Q x−y 2.
If cos x + cos y + cos z = 0 = sin x + sin y + sin z then possible value of cos is 2
2
M a them a ti cs | 21.47

R 3. 1
π  π 
If cos  − x  cos2 x + sin x sin2x
= sec x cos x sin 2x sec x + cos  + x  cos2x cos 2 x 2
4  4 
then possible value of sec x is

S 4. 1
If cot  sin−1 1 − x2  =
 
( )
sin tan−1 (x 6 ) , x ≠ 0 , then possible value of x is

(A) P → 4 ; Q → 3 ; R → 1 ; S → 2 (B) P → 4 ; Q → 3 ; R → 2 ; S → 1

(C) P → 3 ; Q → 4 ; R → 2 ; S → 1 (D) P → 3 ; Q → 4 ; R → 1 ; S → 2

PlancEssential Questions
JEE Main/Boards JEE Advanced/Boards
Exercise 1 Exercise 1

Methods of Differentiation Methods of Differentiation


Q.9 Q.12 Q.15 Q.23 Q.5 Q.8 Q.11 Q.12
Q.14 Q.18 Q.23
Application of Derivatives
Application of Derivatives
Q.7 Q.11 Q.15 Q.21
Q.8 Q.11 Q.15 Q.17
Q.20 Q.23 Q.24 Q.27
Exercise 2
Exercise 2
Methods of Differentiation
Q.2 Q.11 Q.12 Q.15 Methods of Differentiation

Q.17 Q.20 Q.1 Q.3 Q.5 Q.9


Q.11 Q.15 Q.20
Application of Derivatives
Q.5 Q.7 Q.8 Q.10 Application of Derivatives
Q.2 Q.4 Q.7 Q.11
Previous Years’ Questions
Q.2 Q.5 Q.7 Q.9 Previous Years’ Questions
Q.4 Q.9 Q.11
2 1 . 4 8 | Methods of Differentiation and Applications of Derivatives

Answer Key

JEE Main/Boards
Exercise 1
Methods of Differentiation

1 x +3 1
Q.1 e Q.2 cos (log x) Q.3 – sec3θ
2 x+3 x

−1 x
−1 mem tan 3x2 cos{log(x3 − 1)}
Q.4 Q.5 Q.6
2 x 1 − x 1 + x2 x3 − 1

Q.7 – e–x sin x Q.8 cot x . log2e Q.9 5ln sinx (cot x) (ln 5)

3 −1 sec2 x 1 − tanx
Q.10 − Q.11 Q.12
2 2
1 + x (1 − tanx) 2 1 + tanx

− cos cos x sin x 1  1 


Q.13 Q.24  
4 x cos x 2 x  1 + x2 
cos−1 x
 −1 log(sinx)  2
Q.15 (sinx) cos x.cot x −  Q.16 1 Q.17
 1 + x2  1 + x2

dy 1 –2 1 1 1
Q.18 =– Q.19 Q.20 4 × ×
dx 2 1 − x2 1 − a2 x2 a+ a+ x a+ x

x cot  1 + x2 
1 1  
Q.21 Q.22 Q.23
1 + x2 2 2
a +x 1+x 2

Application of Derivatives

1
Q.1 (2, 1) Q.2 90º Q.3 –
3
dy  3 −17 
Q.4 is not defined Q.5  ,  Q.6 (2 – 36)
dx 2 2 
Q.7 x + 12y – 21 = 0; 12x – y + 38 = 0 Q.8 2 bx + 2 ay – ab Q.9 M1M2= –1

Q.10 8x – 2y + 1 = 0 Q.11 4x – 4y + 33 = 0 Q.12 k2x + y – 2ck = 0

1 1 
Q.13 3 Q.14  ,  Q.15 (±3, 0)
2 4
Q.161/818.x + 9y – 55 = 0 ; x + 9y – 35 = 0 Q.17 -1 Q.18 55

x x
Q.19 20 x + y – 140 = 0 Q.20 + = a Q.21 x + 3y = 8; x + 3y = – 8
x1 y1

 −2 4  3a1/3b1/3
Q.22 2x + 2y = a 2
Q.23 (2, –4);  ,  Q.24 tan –1

 3 27  2(a2/3 + b2/3 )

M a them a ti cs | 21.49

Exercise 2

Methods of Differentiation

Single Correct Choice Type


Q.1 B Q.2 B Q.3 D Q.4 D Q.5 A Q.6 B
Q.7 C Q.8 A Q.9 A Q.10 B Q.11 C Q.12 D
Q.13 D Q.14 D Q.15 B Q.16 B Q.17 C Q.18 A
Q.19 B Q.20 D Q.21 B

Application of Derivatives

Single Correct Choice Type


Q.1 B Q.2 D Q.3 C Q.4 D Q.5 D Q.6 A
Q.7 C Q.8 A Q.9 D Q.10 B Q.11 C Q.12 D
Q.13 D Q.14 D Q.15 C

Previous Years’ Questions


Q.1 C Q.2 A Q.3 D Q.4 D Q.5 D Q.6 A
Q.7 C Q.8 D Q.9 A Q.10 A Q.11 D

JEE Advanced/Boards
Exercise 1
Methods of Differentiation
4x3
Q.1 k =1, –1 or 0 Q.2 Q.3 9
−9 9
Q.4 Q.56 Q.6 25
4
 x cos x − sinx
if x ≠ 0 1
Q.7 k = 2 Q.9 f ‘ (x) =  x2 ; f “(0) = –
 3
 0 if x = 0
Q.11 Zero Q.13 3 Q.14 100

Q.15 6 Q.16 2(1 + 2x) . cos2(x + x2) Q.17 f(0) + kx

1 1
Q.18 – Q.19 = X[S1 t2 X 2 − S2 t1 X 2 ] + X3 S1 t1
2
1 + (x + n) 1 + x2 S2 t2
1 − 2x 1 1
Q.20 0 Q.21 Q.22 or –
2 1−x 2 2 2
Q.23 L.H.S = R.H.S Q.24 (a) (i) y’(0) = –1 ; (ii) y”(0) = 2; (b) a = 1, – 2

Application of Derivatives
−5
Q.1 2 3 x – y = 2 ( 3 − 1) or 2 3 x + y = 2 ( 3 + 1) Q.2 a = 1, b =
2
Q.3 (0, 1) Q.4 x = y – 1 = 0 Q.5 T : x – 2y = 0; N : 2x + y = 0
2 1 . 5 0 | Methods of Differentiation and Applications of Derivatives

82.73
Q.6 – Q.7 x + 2y = π/2 & x + 2y = –3π/2 Q.8 5
3
80
Q.9 (i) (a) 6.05, (b) ; (ii) 9.72 pcm2 Q10 a = 1 Q.11 1/16
27
1
Q.12– ; x – 4y = 2 Q.13 a = –1/2; b = – ¾; c = 3 Q.14 3, 12
x+2
−x
1
Q.15 9 Q.16 2e 2 Q.17 (a) n = –2, (c) ±
2 2
Q.18 65 Q.19 64 Q.20 (i) 1, (ii) 2

Q.21 (4, 11) & (– 4, – 31/3) Q.22 (i) 6 km/h; (ii) 2 km/hr Q.23 1 + 36 π cu. cm / sec

2 1
Q.24 cm/sec Q.25 1/48 π cm/s Q.26 cm/sec
4π 4
66
Q.27 Q.28 (a) r = (1 + t)1/4, (b) t = 80
7
1 5
Q.29 (a) – m/min, (b) – m/min
24π 288π

Exercise 2

Methods of Differentiation

Single Correct Choice Type


Q.1 D Q.2 B Q.3 C Q.4 B Q.5 D Q.6 C
Q.7 C Q.8 D Q.9 B Q.10 D Q.11 B Q.12 D
Q.13 C Q.14 C Q.15 C Q.16 B Q.17 B

Multiple Correct Choice Type


Q.18 B, C Q.19 A, B, C Q.20 ACD Q.21 A, B, C, D Q.22 A, B, C

Application of Derivatives

Single Correct Choice Type


Q.1 C Q.2 D Q.3 C Q.4 A Q.5 C Q.6 B
Q.7 D Q.8 A

Multiple Correct Choice Type


Q.9 A, D Q.10 A, B Q.11 B, D Q.12 A, B Q.13 A, B, D Q.14 A, B
Q.15 B, D

Previous Years’ Questions


ud
Q.1 B, C Q.2 B, D Q.3 H = ∅, V = {1.1} Q.4 Q.5 (0, 0)
v − u2
2

π 3π
Q.6 x + 2y = and x + 2y = – Q.7 (0, 2) Q.8 3/4 Q.9 2 x – 2y + 2 = 0
2 2
Q.10 dy/dx = 1 Q.11 1/16 Q.12 A C Q.13 B
M a them a ti cs | 21.51

Solutions

JEE Main/Boards
= cos{log(x – 1)}.
3
1
.3x =
2
{ }
3x2 cos log(x3 − 1)

Exercise 1 3
x −1 3
x −1

Methods of Differentiation dcos x dcos x 1 – sinx


Sol 7: = . = = –(sinx)e–x
de x dx dex ex
Sol 1: y = e x +3 dx
Sol 8: y = log2(sinx)
dy de x +3 d x + 3 d(x + 3)
= . . [Chain Rule]
dx x+3 dx dx dy 1 dlog(sinx) d(sinx) cos x
 = . = log2 e = (cot x)loge2
dx log2 d(sinx) dx sinx
x +3 1 1 x +3
= e . .1 = e
2 x+3 2 x+3
Sol 9: y = 5log(sinx)

Sol 2: y = sin(logx) logy = (log5) log(sinx)

dy dsin(logx) dlogx 1 dy d.log(sinx) d(sinx) cos x


=  [Chain rule] = (log5) . =(log5)
dx d(logx) dx y dx d(sinx) dx sinx

1 1 dy
= cos(logx)   = cos(logx) ∴ = 5log(sin x) ( (cot x)log5 )
dx
x x

Sol 10: x+ y =
5
Sol 3: y = –tanθ, x = sinq
Differentiate w.r.t x
d(– tan θ) 1 1 dy dy y
dy dy / dθ dθ – sec2 θ + 0 ∴
= =–
= = = = – sec3q 2 x 2 y dx dx x
dx dx / dθ d(sin θ) cos θ
dθ For (4, 9)
Sol 4: y = cos–1 x
dy 9 3
= –= –
dy dcos x d x −1
–1 1 –1 dx 4 2
= = =
dx d x dx 1−x 2 x 2 x − x2
1 + x  1 − x 
Sol 11: y = cot–1   = tan 
–1

−1 x 1 − x  1 + x 
Sol 5: y = em tan
−1 Take x = tanq
dy dem tan x d(mtan−1 x)
= .
dx d(mtan−1 x) dx  π 
 tan − tan θ 
∴ y = tan–1  4 
−1 x
m tan −1 x  1  mem tan  1 + tan π tan θ 
= e .  m. = 
 4


 1 + x2  1 + x2
 π  π π
= tan−1  tan  − θ   = – θ = – tan–1x
Sol 6: y = sin{log(x – 1)} 3
  4  4 4

=
3
dy dsin log(x − 1)
×
{
d log(x3 − 1)
×
d(x3 − 1) } { } ∴
dy dtan−1 x
= =–
1
dx {
d log(x3 − 1) d{x3 − 1} dx } dx dx 1 + x2
2 1 . 5 2 | Methods of Differentiation and Applications of Derivatives

1 + tanx  π
Sol 12: y = = tan  x +  sin θ sin x  cos−1 cos ( x − θ ) 
= cos−1 cos θ cos x + =
1 − tanx  4
 cos−1 ( cos x ) = x ∴ y= x − θ
 π  π
d tan  x +  dtan  x + 
dy  4  4 dy
∴ = . ⇒ =1
dx  π dx dx
dtan  x + 
 4   1 + x2   1 – x2 
Sol 17: y = sec–1   = cos–1  
1  π  1 − x2   1 + x2 
= .sec2  x +     
 π  4
2 tan  x +  Take x = tanq
 4
 1 − tan2 θ 
1  1 − tanx    π  ∴ y = cos–1   = cos–1(cos2θ)
=    1 + tan2  x +    1 + tan2 θ 
2  1 + tanx    4   
⇒ y = 2q \ y = 2tan–1x
2
1  1 − tanx  1 + tan x  2
dy 2
=    ⇒ =
2  1 + tanx   (1 − tanx)2  dx 1 + x2

sec2 x 1 − tanx  1+x + 1−x


= Sol 18: y = sin−1  
(1 − tanx) 2 1 + tanx  2 
Put x = cos2q
− cos cos x sin x
Sol 13:
4 x cos x  1 + cos2θ + 1 − cos2θ 
∴ y = sin–1  
 2 
2
Sol 14:  2cos2 θ + 2sin2 θ 
x(logx )log(logx2 )
2
= sin–1  
 2 
−1 x
 
Sol 15: y = (sinx)cos
 1 1 
Taking log both sides = sin–1  cos θ + sin θ 
 2 2 
Log y = cos–1xlog(sinx)
  π  π 1 π
Differentiate both side w.r.t. x = sin–1 sin  θ +   = θ + \y = cos–1x +
  4  4 2 4
1 dy dcos−1 xlog(sinx)
= dy 1
y dx dx =–
dx 2 1 − x2
dlog(sinx) dcos−1 x
= cos–1x + log(sinx)
dx dx
Sol 19: y = sin–1 2ax 1 − a2 x2 
log(sinx)  
= cotx cos–1x –
1 − x2 1   cos2 θ  
Put x = cosq ∴ y = sin–1 2cos θ 1 − a2  
 a   a2  
dy −1 x log(sinx)    
∴ (sinx)cos cot x cos−1 x − 
dx  1 − x2 
= sin–1 2cos θ 1 − cos2 θ 
 
Sol 16: y cos−1 ( 2 cos x + 3 sinx ) 13 
=
  = sin–1[2cosqsinq]= sin–1[sin2q] = 2θ = 2cos–1ax
 2 3 
cos−1  cos x + sin x  dy 1 –2
 13 13  ∴ = –2 × =
dx 1 − a2 x2 1 − a2 x2
2 3
where = cos θ and = sin θ
13 13
M a them a ti cs | 21.53

Sol 20: y = a + a + x 1  2  ×
1 d(1 + x2 )
=  cos 1 + x  ×
 2 1 + x2 dx
sin  1 + x2  
dy d a + a + x
= ×
d a+ a+ x ( )  
dx d(a + a + x ) dx x cot  1 + x2 

=  
1 1  1  1 + x2
= × ×  ×1
2 a+ a+ x 2 a+ x 
 x+ a 
1 1 1 Sol 24: y = tan–1  
= × × 1 − a x 
4 a+ x  
a+ a+ x
 b  Put x = tant
 ax − b   x− a 
Sol 21: y = tan–1   = tan 
–1
 a = tana
 a + bx  1 + b x 
 a   tant + tan α 
b \y = tan–1   = tan tan(t + α) = t + a
–1
Let tanα = , x = tant  1 − tant tan α 
a

 tant − tan α  y = tan–1 x + a


∴ y = tan–1   or y = tan tan(t – α)
–1

 1 + tan α tant 
= t – α , α is constant =∴
dy d tan
−1

.
(
x d x dα
+
)
dx d x dx dx
dy dy / dt
∴ = or y = t – α = tan–1x – a
dx dx / dt 1 1 1  1 
= . =  
( x) 2 x  1 + x2 
2
dy 1 1+ 2 x
∴ =
dx 1 + x2

Sol 22: y = log  x + x2 + a2  Application of Derivatives


 
Sol 1: y = x2 – 4x + 5
dlog  x + x2 + a2  d  x + x2 + a2 
dy  .   dy
= Tangent is parallel to x axis ⇒ = 0= 2x − 4
dx d  x + x2 + a2  dx dx
 
⇒ x = 2; y = 4 – 8 + 5 =1
 
 d  x2 + a2 
1   d(x2 + a2 )  Point A (2, 1)
= . 1 + . 
 2  d(x2 + a2 ) dx
x + x + a   
2
    Sol 2: If two curves cut orthogonally, then the tangents
at point of intersection are perpendicular.
1  1 
= 1 + × 2x 
 2 
 x + x + a   2 x2 + a2 
2
Sol 3: f(x) = 3x2 – 5
 
 1 3 
 x2 + a2 + x  Tangent at =x , y  − 5 
=
1 1  2 4 
= . =
 x + x2 + a2   a2 + x2  a2 + x2
     dy 
    = 6x= 3= (m)tangent
 dx x =1/2
Sol 23: y = log  sin 1 + x2   dx  1
  −
dy
 =− =
3
(m)normal
 
dlog  sin 1 + x2  d  sin 1 + x2  2 dy
dy  ×  × d 1+x Sol 4: y = f (x), = ∞ i.e. not defined
= dx
dx d  sin 1 + x2  d  1 + x2  dx
   
2 1 . 5 4 | Methods of Differentiation and Applications of Derivatives

Sol 5: y = 2x2 – 6x – 4 2
Sol 10: y = 8x
dy 3
= 4x − 6 = 0 ⇒ x = dy dy 8
dx 2 ⇒ 2y = 8 ⇒ = 4=
dx dx 2y
9 3 9 17
y = 2× − 6 × − 4 ⇒ − 9 − 4 =− 1
4 2 2 4 This give y = 1; x =
8
 3 −17 
 ,  y −1
2 2  y −1 1
⇒ =4 ⇒ = ⇒ 2y − 2 = 8x − 1
1 8x − 1 2
x−
Sol 6: y = x2 – 4x – 32 8
⇒ 8x – 2y + 1 = 0
dy
= 2x − 4 = 0
dx
Sol 11: y = – 5x2 + 6x + 7
x = 2; y = 4 – 8 – 32 = – 36
Point (2, – 36) y’ = – 10x + 6⇒ ( y ′ ) 1 =−5 + 6 =1
x=
2
35 1
y− =x −
Sol 7: y = (5 – x)1/3 4 2
At P (–3, 2) 4y – 35 = 4x – 2⇒4x – 4y + 33 = 0
y = (8)1/3
=2
c2
 dy  1 −1 −1 Sol 12: y =
= (5 − x )
−2/3
  = = x
 dx x = −3 3 3 (8) 12
2/3
dy −c2  dy 
= ⇒  = −k 2
y − 2 −1
dx x 2 dx
  c
= ⇒ 12y − 24 =− ( x + 3 ) ⇒ x + 12y =21 k
x + 3 12
Equation: – y – ck
y −2  c
Equation of normal is = +12
x+3 = – k2  x −  ⇒ y + k2x =2x
 k 
⇒ y – 12x = 38
dy
Sol 13: y = x3 + 6⇒ = 3x2
dy 4 2 dx
Sol 8: y = (4x – 3)
= 1/2
= ≠ 0 , it
dx 2 4x − 3 4x − 3
can never be parallel to x axis. ( y ′)x =−1y =
5
= 3 ; ( y′)
x=
−1y =
7
=3

So the tangents are parallel


Sol 9: x2 = y & x3 + 6y = 7
x3 + 6x2 = 7  dy 
Sol 14: y = x2 ⇒  =  2x
= 1
(x2 +7x + 7) (x – 1) = 0  dx 
⇒ x=1 1 1 1 1 
x= ; y= ; P , 
Point of intersection (1, 1) 2 4 2 4

P1 (x2 = y)P2 (x3 + 6y = 7) x2 y 2


Sol 15: + 1
=
dy 6dy 9 4
= 2x 3x2 + 0
=
dx dx 2x 2y 4x
+ 0 ⇒ y' = −
y′ =
 dy   dy  −1 9 4 9y
  =2   =
 dx x =1  dx x =1 2 This will be parallel to y axis if y = 0
M1M2= –1 i.e., tangents are orthogonal at (1, 1) x=±3
P (+ 3, 0), (– 3, 0)
M a them a ti cs | 21.55

1 Sol 20: x+ y =a
Sol 16: x = ; y = 2t ⇒ xy =2
t
1 1 y y
2 −2 + 0 ⇒ y′ =
y' = − − 1
=
y= ⇒ y′ = = −8 x x1
x x2 2 x 2 y
1
At t = 2 i.e. x = y − y1 y1
2 ⇒ = − ⇒ y x1 − y1 x1 =
−x y1 + x1 y1
x − x1 x1
y’ = – 8
1 ⇒ x y1 + y x1 = y1 x1 + x1 y1
Slope of normal =
8
x y
+ a
=
x2 y2 x1 y1
Sol 17: + 1
=
a2 b2
2x 2yy ′ b2 x −3x
+ 0 ⇒ y′ = −
= Sol 21: 3x2 + y 2 =
8 ⇒ 6x + 2yy ′ =
0 ⇒ y’ =
a 2
b 2 2
a y y
y −1
At (x0, y0); y ′ = −
b2 x 0 (m)normal
= =
3x 3
a2 y 0
y=–x
y − y0 b2 x 0
= −
x − x0 a2 y 0 x=± 2,y=  2

y− 2 1
a2 yy 0 − a2 y 20 = b2 x20 − b2 xy 0 = − ⇒ 3y + x =2 2
x+ 2 3
2 2
xx0b + yy 0 a = a2 y 20 + b2 x20 2 2
=a b y+ 2 −1
= ⇒ 3y + x =−2 2
xx0 yy 0 x− 2 3
⇒ + 1
=
2 2
a b
Sol 22 : x+ y =a
Sol 18: y = 4x – 3x + 5 3 1 y'
+ 0
=
9y + x + 3 = 0 [Given] 2 x 2 y
−1
Mnormal =
9 y
y′ =
− − 1=
= −1
x
y’ = 12x2 – 3 = 9⇒x = ± 1; y = 6, 4
a2
y − 4 −1 y − 6 1 y− 2
= , = − 4 =1 ⇒ y + x = a
x +1 9 x −1 9
a2 2
x + 9y = 35,x + 9y = 55 x−
4
Sol 23: y = x3 – 2x2 – 2x
Sol 19: y (x – 2) (x – 3) = x – 7
y’ = 3x2 – 4x – 2 = 2
x−7
y= =0⇒x=7
( x − 2)( x − 3) 3x2 – 6x + 2x – 4 = 0
(3x + 2) (x – 2) = 0
dy ( x − 2 )( x − 3) − ( x − 7 )( 2x − 5 )
= −2
dx
)( ) (
2 2
x −2 x −3 x = 2, ; y = 8 – 8 – 4 = – 4,
3
( x − 2 ) ( x − 3)
2 2
 −dx  52 (4)2 −8 2x4 4 8 8 4
  = = = −20 y= − + =− − +
 dy x =7 ( x − 7 )( 2x − 5 ) − ( x − 2 )( x − 3) 0 − (5)(4) 27 9 3 27 9 3

y 36 − 24 − 8 4
=−20 ⇒ 20x + y =140 ⇒ =
x−7 27 27
2 1 . 5 6 | Methods of Differentiation and Applications of Derivatives

Therefore, the points are  f(g(x)) = x


\f’(g(x))g’(x) = 1
 −2 4 
(2, –4)  ,  1
 3 27  \g’(x) =
f '(g(x))

1 1 1 1 + a2
Sol 24: x2 = 4 by \g’(2) = = = =
f '(g(2)) f '(a) a10 a10
y2 = 4axx2 = 16b (a2 b)1/3 = 16b4/3a2/3 2
1+a
y4 = 16a2 4byx = 4 (b2a)1/3
x 1 1
Sol 3: (D) y = +
n−m p–m m−n
1+x +x 1+x + xp −n
y3 = 64a2by’ = 2b
2a
1
y +
m−p
1/3 1+x + xn−p
a
  1/3
xm xn xp
y = 4(a2b)1/3y’= 2 b =
1a = + +
  xp + xm + xn xn + xm + xp xp + xm + xn
2a 2b

( )
1/3
4 a2b xm + xn + xp
= =1
1/3 1/3 1/3
xm + xn + xp
a 1a 3 a dy
2  −     ∴ =0
=tan θ b
=
2b 2b dx
a
1+ 
2/3
a
1− 
2/3

2
(
d f(4) − f(x)2 )
b b f(4) − f(x ) dx
Sol 4: (D) lim = lim
x →2 2−x x →2 d(2 − x)
3a1/3 b1/3 dx
=
(
2 b2/3 + a2/3 ) = lim
x →2
f '(x2 )[2x]
+1
= f’(22).2.2 = 4f’(4) = 20

Exercise 2 Sol 5: (A)  = lim xm (lnx)n m, n ∈ N


x →0

1
Methods of Differentiation (lnx)n n(lnx)n−1
 = lim = lim x
x →0 m x →0 m+1
Single Correct Choice Type 1 1
  –m  
x x
 3x + 4 
Sol 1: (B) y = f   f’(x) = tanx
2
(using L-Hospital rule)
 5x + 6 
 
 3x + 4   3x + 4   
df   d 
dy 5x + 6   5x + 6   n (lnx)n−1  –n(n − 1)(n − 2)......1lnx
=  . = lim  –  = xlim
 m 1
m m
dx  3x + 4  dx x →0

→0
n−1  1 
d      m  
 5x + 6   x  x
  3x + 4   (5x + 6)3 − (3x + 4)5
2
1
= tan    –(n)!
5x + 6   x –(n)!

   (5x + 6)2 = lim = lim (x)m = 0
m+1
x →0
1 x →0 mn
2 mn  
 3x + 4  1 x
= –2tan   .
 5x + 6  (5x + 6)2
∴Independent of n and m

cos x x 1
Sol 2: (B) f’(x) = g(x) = f–1(x) 2
Sol 6: (B) f(x) = 2sinx x 2x
x10 tanx x 1

(1 + x2 )
M a them a ti cs | 21.57

– sinx 1 0 π π
cos x x 1 \2θ = 2tan–1 >
2 2
f’(x)= 2sinx x2 2x + 2cos x 2x 2
tanx x 1 \y = sin–1sin(π – 2θ)
tanx x 1
y = π – 2q
cos x x 1
y = π – 2tan–1x
+ 2sinx x2 2x
dy –2
sec2 x 1 0 ∴ =
dx 1 + x2
f’(x) = [x2sinx (2xtanx – 2sinx)] dy –8
∴ =
+ x[2tanx – 2cosx] + [2xcosx – 2xtanx] + (–2xcosx) + dx x=
π π + 42
2
2
2x2sec2x + 2sinx – x2sec2x
= x2sinx + x2sec2x + 2xtanx – 2xcosx
f(x) − g(x)
f '(x) x2 (sinx + sec2 x)(2x(tanx − cos x) Sol 9: (A) lim
lim = lim x→4 x −2
x →0 x x →0 x

= lim x(sinx + sec2x) + 2(tanx – cosx)= –2


x →0
d  f(x) − g(x) 
  ( )
= lim dx
– sinx + cos x – sinx x→4
d x −2 ( )
Sol 7: (C) f’(x) = cos2x sin2x 2cos2x dx
cos3x sin3x 3cos3x 1 1
f '(x) − g'(x)
cos x sinx cos x 2 f(x) 2 g(x)
= lim
+ –2sin2x 2cos2x –4 sin2x x→4 1
cos3x sin3x 3cos3x 2 x

cos x sinx cos x 1 1 9 6


f '(4) − g'(4) −
+ cos2x sin2x 2cos2x 2 f(4) 2 g(4) 2 2
= =
–3sin3x +3cos3x –9 sin3x 1 1
2 4 2
–1 0 –1 0 1 0
π = 3 2
f '   = –1 0 –2 + 0 –2 0
2 0 –1 0 0 –1 0 dy
Sol 10: (B) y = x + ex ⇒ = 1 + ex
dx
0 1 0 dx 1
⇒ =
+ –1 0 –2 = (2 – 1) + (0) + (3) = 4 dy 1 + ex
3 0 9
d2 x d(1 + ex )−1 1
= ×
 2x  dy 2 dx (1 + ex )
Sol 8: (A) y = sin–1  
 1 + x2  1 d(1 + ex )−1 d(1 + ex )
=
Put x = tanq (1 + ex ) d(1 + ex ) dx

2 tan θ 1  1  x –ex
\y = sin–1 = –  e =
x 
1 + tan2 θ (1 + e )  (1 + ex )2  (1 + ex )3

y = sin–1sin2q
Sol 11: (C) h’(x) = [f(x)]2 + [g(x)]2
π π π π
– ≤ 2θ≤ ⇒ – ⇒θ≤ h”(x) = 2f(x)f’(x) + 2g(x)g’(x)
2 2 4 4
π Also f’(x) = g(x)
 as x =
2 ∴ f”(x) = g’(x) = –f(x)
π
\θ = tan–1 ∴ h”(x) = 2f(x)g(x) + 2g(x)(–f(x)) = 0
2
2 1 . 5 8 | Methods of Differentiation and Applications of Derivatives

∴ h(x) = ax + b \f(x) = 1
h(0) = b = 2; h(1) = a + b = 4 ∴ f’(p + q) = 0 = f’(p) + f’(q)
\a = 2, b = 2 \n = 0, 2 (two values)
∴ h(x) is a straight line with slope 2 and y intercept 2 100
Sol 15: (B) f(x) = ∏ (x − n)n(101–n)
n=1
Sol 12: (D) f(x) = x + 3n(x – 2) 100

F’(x) = 1 +
3 lnf(x) = ∑ n(101 – n)ln(x
n(x ‒− n)
n=1
(x − 2)
100
1 n(101 − x)
g(x) = x + 5n(x – 1) ⇒ f '(x) = ∑
f(x) n=1 (x − n)
5
g(x) = 1 + f '(101) 100 n(101 – n) 100
100 × 101
(x − 1) =∑

f(101) n=1 (101 – n)
= ∑n = 2
= 5050
\f’(x) < g’(x) n=1

3 5 3(x − 1) − 5(x − 2) f(101) 1


⇒ < ⇒ <0 ∴ =
(x − 2) (x − 1) (x − 2)(x − 1) f '(101) 5050

7 − 2x 7 
∴ < 0⇒x ∈ (1, 2) ∪  , ∞  Sol 16: (B) f(x) is continuous and differentiableat
(x − 2)(x − 1) 2 
1
x=
Also x – 2 > 0 and x – 1 > 0 3
3x2 + 2x − 1 x +1
7  f(x) = =
\x > 2 ⇒x ∈  , ∞  2
6x − 5x + 1 2x − 1
2 
( ) − 2 ( x + 1) −3
∴ f’(x) = 2x − 1 =
Sol 13: (D) g(x) = x2, f(x) = sinx, h(x) = logex ( 2x − 1 ) 2
( 2x − 1 )2
\g(f(x)) = (sinx)2 1 −3
f’   = = 27
h(g(f(x)) = loge(sinx) = F(x)
2 3
   1 
2

2  − 1
∴F(x) = 2logsinx  3 
dF 2

= = .cos x 2cot x Sol 17: (C) f(x) = ax2 + bx + c
dx sinx
d2 t For x ∈ R f(x) is always positive
∴ = –2cosec2x
2
dx \a > 0 and b2 – 4ac < 0
\g(x) = f(x) + f’(x) + f”(x)
Sol 14: (D) f(x) = xn
= ax2 + bx + c + 2ax + b + 2a
f’(p+q) = n(p+q)n–1
= ax2 + (2a + b)x + (2a + b + c)
f’(p) = n(p)n–1
D = (2a + b)2 – 4(2a + b + c )a
f’(q) = n(q)n–1
= 4a2 + b2 + 4ab – 8a2 – 4ab – 4ac
for f’(p+q) = f’(p) + f’(q)
= b2 – 4a2 – 4ac = (b2 – 4ac) – 4a2
(p + q)n–1 = [(p)n–1 + (q)n–1] n ≠ 0
n−1 n−1
Qb2 – 4ac < 0
 q q
⇒ 1 +  –1=   ∴D < 0 ∴ g(x) = 0
 p p
x4 + 4
This condition satisfies of n – 1 =1 Sol 18: (A) y =
x2 − 2x + 2
⇒n=2
Also if n = 0 dy (x2 − 2x + 2)(4x3 ) − (x 4 + 4)(2x − 2)
=
dx (x2 − 2x + 2)2
M a them a ti cs | 21.59

dy 4x5 − 8x 4 + 8x3 − 2x5 + 2x 4 − 8x + 8 Application of Derivatives


=
dx (x2 − 2x + 2)2
Single Correct Choice Type
5 3 4
dy 2x + 8x − 8x + 8 − 6x
=
dx (x2 – 2x + 2)2 Sol 1: (B) y = kekx ⇒ y’ = k2ekx ⇒ y = k, x = 0

5 3 y’ = k2
1 1 1 1 θ
2×   + 8  − 8 × + 8 – 6 × 1
dy 2 2 2 16 ⇒ tan θ = ⇒ cot θ =k 2 
= k2
dx 1/2 1 
2

 − 1 + 2 ⇒ θ =cot −1 k 2
4 
1 6 81 6 x x
+1− 4 + 8 − −
5 ) == ∫t2( 2t − 5 ) == xt22 − 5x
x
( 2t −f(x)
x
16 16 16 16 = 75 / 16 = 3 Sol
f(x) 2:
= ∫(D) − 5t 5t − =4 x+210
− 5x − 4 + 10
= = 2 2
5
2 25 25 / 16 2 2

− 2+)(6x =− (3x) − 2 )( x − 3 )
= x2 − 5x + =6 x= (−x 5x
2
  16
4

Sol 19: (B) f(x2) = x3 f1 ( x=


) 2x − 5
\ f(x) = x3/2  x = 2,f ( x ) = 0  f ′ ( x ) = −1
 
3 1/2
\ f’(x) = x
2  x 3,f(x)
= f′(x) 1
= 0  =
3 1/2
f’(4) = ×4 = 3 Angle between the 2 tangents is 90°
2
(as m1m2 = –1)
Sol 20: (D) x = sint, y = sin3t
Sol 3: (C) x2 y = c3
dy 3cos3t 3(4 cos3 t − 3cos t)
= =
dx cos t cos t c3 3
y=
2
⇒ y ′ = −2c
= 12cos2t – 9 x x3
d2 y 24 cos t(– sint) c3 2c3  c3 
= –24sint
dx 2
=
cos t y– =
− (x − t) x =
ty= 
t2 t3  t2 
∴ (1 – sin2t)(–24sint) – (sint)(12cos2t – 9) + k(sin3t) = 0
3t
= –24sint + 24sin3t – 12sint(1 – sin2t) + 9sint + k(3sint x intercept = = a y intercept
2
– 4sin3t) = 0
3c3 9t2 3c3 27c3
⇒ (3k – 36)sint + (36 – 4k)sin3t = 0 = = b ⇒ a2b = × = [C]
t2 4 t2 4
∴ 3k – 27 = 0 and 36 = 4k
⇒k=9  π
x sin   x > 0
Sol 4: (D) f ( x ) =  x
 0 x=0
Sol 21: (B) x = lnt, y = t2 – 1 
dy dy / dt 2t
= = = 2t2
dx dx / dt 1 / t

 dy 
d 
π
 dx 
d2 y 4t 36
= dt = = 4t2
dx 2 dx 1 /t
dt  π  −π  π π π
f ′(x) x cos   =
=  + sin 0 = tan  
 x  x 
2 x x x
\y”(1) = 4
x ∈ 0,1  infinite solution
2 1 . 6 0 | Methods of Differentiation and Applications of Derivatives

b ⇒ x2 + 2mx – m = 0
Sol 5: (D) y = f (x) ⇒ ∫ f ′(x) f ′′(x)
b b a I II
−2m ± 4m2 + 4m
⇒x= =0
=⇒ I f ′(x)∫ f ′′(x) − ∫ f ′′(x) f ′(x) 2
a a
For D = 0
2
⇒ 2I = f ′ ( x ) 
⇒ 4m2 + 4m = 0 ⇒ m = 0, – 1
2 2 1
⇒ 2I = f ′b  f ′a ⇒y–2=–x+
2
5
⇒ 2I = – 2 ⇒ I = –1 ⇒x+y=
2
3
Sol 6: (A) y = x  y x2 − 3 and c2 y = kx2
Sol 9: (D) c1=
B
y ′ = 3x2 A 2
⇒ kx= x2 − 3
3 ( xB ) = k ( xA )
2 3 2
3 3 3k
⇒ x= ± =a ⇒= y −3=
1−k 1−k 1−k
xB y − y1
xA
= ± k ⇒ = 2ka ⇒ y − y=
1 2ka ( x − a)
x−a

( )
⇒ y − t3 = 3t2 ( x − t ) at x= t  ⇒ x2 – 3 – y1 = 2 ka (x – a)
⇒ – 2 – y1 = 2ka (1 – a)
⇒ x3 – t3 = 3t2 (x – t)
y2 = – 2 
⇒ x2 + t2 + xt =
3t3
y1 = ka2
⇒ ( xB ) + ( xB x A ) =
2 ( xA )
2 2
⇒ – 2 – ka2 = 2ka – 2ka2
⇒ kx2A ± kx2A =
2x2A ⇒ ka2 – 2ka – 2 = 0

⇒ 2 − k =± k  3  3 5k − 2
⇒ k −2 =2k ⇒ = 2k 3
 1 − k  1 − k 1−k
⇒k=4

dy ⇒ 5k −=
2 2k 3 − 3k
Sol 7: (C) Subnormal = y
dx 2
k= ,a=1
xnyn−1 y ′ + yn =
0 3

−y x −2
y′ = Sol 10: (B) x2 = e
nx
No. of roots are 4
dy y2 y 2 +n
y = it is constant for ⇒
dx nx an+1n a b
Sol 11: (C) − − 0 at any general point
y′ =
3
Constant for n = – 2 x y3
 b 
 t,t 2 
x2  t − a 
Sol 8: (A) y =
− +2 
2  ay 3
y’= –

y = −x bx3
3/2
y −2 b −a t3  b 
⇒ =m y −=
t   (x − t)
1 t 2 − a b t3  t 2 − a 
x−
2
 1 −x2 2mx m ⇒ x intercept
(
t − b t2 − a + t ) =
bt3 t3
= = B 
⇒ y −=
2 m x −  ⇒ = − ab ab a
 2 2 2 2
M a them a ti cs | 21.61

Sol 12: (D) m = 0 Previous Years’ Questions


y=c 
y= x Sol 1: (C) Given, x = a (cos θ + θ sin θ) and

1 1 1 y = a (sin θ – θ cos θ)
y1 = = 1 ; x= y= = c
2 x 4 2 dx
∴ = a (– sin θ + sin θ + θ cos θ)

Sol 13: (D) y3 – x2y + 5y – 2x = 0 dy
= a θ cos θ and = a (cos θ – cos θ + θ sin θ)

⇒ x4 – x3y2 + 5x + 2y = 0 dy dy
= a θ sinθ ⇒ = tan q
⇒ 3y y’ – x y’ – 2xy + 5y’ – 2 = 0
2 2 dθ dx

2 + 2xy 2 Thus, equation of normal is


⇒ y’ = , y’ (0, 0) =
3y 2 − x2 + 5 5 y − a(sin θ − θ cos θ) − cos θ
=
x − a(cos θ + θ sin θ) sin θ
⇒ 4x3 – 3x2y2 – 2x3 yy’ + 5 + 2y’ = 0
⇒ – x cos θ + a θ sin θ cos θ + a cos2q
3 2 2
4x − 3x y + 5 5
⇒ y’ = , y ′ ( 0,0 ) = − = y sin θ + θ a sin θ cos θ – a sin2q
−2 + 2x y 3 2
⇒ x cos θ + y sin θ = a
2 5
+ = ∞ Whose distance from origin is,
5 2 π
⇒ tan θ = ∴ θ=
1 −1 2 |0+0−a|
=a
x
cos2 θ + sin2 θ
 1
Sol 14: (D) f (=
x) ∫  t + t  dt dy
0 Sol 2: (A) Given, = 2x + 1
dx
1
g ( x )= f ′ ( x )= x + on integrating both sides
x
11
gg((33))−−gg  33++11−−11−−22 55−−55 y
22
=
= 33 22= 22 11
m= = 22= = 33 = = y = x (x + 1)
11 55 55 66××11 33
33−−
22 22 22
1 1
y’ = 1 – = x’ -1 O x=1 x
x 2 3

3 5
x=
= x = y y’
2 6

∫ dy = ∫ (2x + 1)dx
3/2
x
Sol 15: (C) y =
3 ⇒ y = x2 + x + c which passes through (1, 2)

x2 ∴2=1+1+c⇒c=0
18yy’ = 3x2 ⇒ y’ =
6y ∴ y = x2 + x
 −1  −6y Thus, the required area bounded by x axis, the curve
mnormal =   = = ±1
 y1  x2 and x = 1
 
1
2  x3 x 2 
6y = ± x ⇒ y = x
2 1 2
= ∫ (x + x)dx =  + 
6 0
 3 2 
0
x4 8 1 1 5
⇒9× = x3 ⇒ x = 4 and y = = + = sq unit
36 3 3 2 6
2 1 . 6 2 | Methods of Differentiation and Applications of Derivatives

Sol 3: (D) Slope of tangent y = f(x) is y


dy B(0, cosec )
= f’(x)(3,4) (mid point of AB)
dx M
P (2 cos , sin)
Therefore, slope of normal
x’ x
A
1 1
=– =– (2 sec , 0)
f '(x)(3,4) f '(3)
y’
1  3π 
But – = tan   (given) Equation of tangent is,
f '(3)  4 
x 2
1 π π cos θ + y sin θ = 1
⇒– = tan  +  = – 1 2
f '(3) 2 4
whose intercept on coordinate axes are A ( 2 sec θ, 0)
f’(3) = 1 and B (0, cosec θ)
∴ Mid point of its intercept between axes
Sol 4: (D) Given y3 + 3x2 = 12y  …(i)  2 
1
 sec θ, cosecθ  = (h, k)
dy dy  2 2 
⇒ 3y2 + 6x = 12  
dx dx 1 1
dx 12 − 3y 2 ⇒ cos θ = and sin θ =
dy 6x 2h 2k
⇒ = ⇒ =
dx 12 − 3y 2 dy 6x
dx Thus, locus of mid point M is
For vertical tangent, =0
dy 1 1
(cos2θ + sin2 θ) = +
2
⇒ 12 – 3y = 0 ⇒ y = ± 2
2 2h 4k 2
4 1 1
On putting y = 2 in Eq. (i), we get x = ± and again ⇒ + = 1, is required locus
2
3 2x 4y 2
putting y = – 2 in Eq. (i), we get 3x2 = – 16, no real
solution Sol 7: (C) We know, tangent to parabola y2 = 4ax is y
 4  a
∴ The required point  ± ,2  = mx +
 3  m
1
∴ Tangent to y2 = 4x is y = mx +
Sol 5: (D) Tangent to the curve y2 = 8x is, m
Since, tangent passes through (1, 4)
2
y = mx + . 1
m ∴4=m+
So it must satisfy xy = –1 m
⇒ m2 – 4m + 1 = 0
 2
⇒ x  mx +  = – 1
 m (Whose roots are m1 and m2)
2 ∴ m1 + m2 = 4 and m1m2 = 1
⇒ mx2 + x + 1 = 0,
m
and | m1–m2|= (m1 + m2 )2 − 4m1m2
Since it has equal roots, therefore D = 0
4
⇒ – 4m = 0 ⇒ m3 = 1 ⇒ m = 1 = 12 = 2 3
2
m
So equation of common tangent is y = x + 2 Thus, angle between tangents

m2 − m1 2 3 π
Sol 6: (A) Let the point be P( 2 cos θ, sin θ) tan θ = = = 3 ⇒θ =
1 + m1m2 1+1 3

x2 y2
on + =1 Sol 8: (D) The tangent at (1, 7) to the parabola x2 =
2 1
y – 6 is
M a them a ti cs | 21.63

1 Sol 10: (A)


x (1) = (y + 7) – 6
2 f(1 + h) − f(1)
f '(1) = lim
(replacing x → x x1 and 2y → y + y1)
2 n→0 h
⇒ 2x = y + 7 – 12  1 
(1 + h − 1)sin  −0
 1 +h−1 h 1
⇒ y = 2x + 5 = ⇒ f '(1) lim = lim sin  
n→0 h n→ 0 h h
Which is also tangent to the circle  1 
(1 + h − 1)sin  −0
x2 + = y2 + 16x + 12y + c = 0  1 +h−1 h 1
⇒ f '(1) lim = lim sin  
n→0 h n→0 h h
i.e., x2 + (2x + 5)2 + 16x
+ 12(2x + 5) + c = 0 1
⇒ f '(1) =
lim sin  
must have equal roots i.e., α = b n→0 h
⇒ 5x2 + 60x + 85 + c = 0
∴ f is not differentiable at x = 1.
60
⇒α + β = –
5 f(h) − f(0)
Similarly, f '(0) = lim
⇒ a = – 6⇒ x = – 6 h→0 h

and y = 2x + 5 = –7  1 
(h − 1)sin   − sin(1)
 h−1
∴ Point of contact is (– 6, – 7) ⇒ f '(0) =
lim
h→0 h

Sol 9: (A) Slope of the line joining the point


ec +1 − ec −1
(c – 1, ec–1) and (c + 1, ec+1) is equal to > ec
2

y B

A c
(c, e )
x’ x
O

y’

⇒ Tangent to the curve y = ex will intersect the given ⇒ f is also not differentiable at x = 0.
line to the left of the line x = c.
Alternate Solution Sol 11: (D)

The equation of the tangent to the curve y = ex at Solving the equation we get the points of intersection
(c, ec) is (-2, 1) and (-2, -1)

y – ec = ec(x – c)  ….(i) The bounded region is shown as shaded region.


1
Equation of the line joining the given points is
= 2∫ (1 − 3y 2 ) − ( −2y 2 )
The required area
ec (e − e−1 ) 0
y – ec–1 = [x – (c – 1)] … (ii)
2 1
1 
2 y3  2 4
Eliminating y from equation (i) and (ii), we get = 2∫ (1 − y )dy = 2  y −  = 2 × =
0  3  3 3
0
[x – (c – 1)] [2 – ( e – e–1)] = 2e–1
e + e–1 − 2
⇒x–c= <0⇒x<c
2 − (e − e−1 )
2 1 . 6 4 | Methods of Differentiation and Applications of Derivatives

JEE Advanced/Boards = 2f(x) g(x) – 2g(x)f(x) = 0


∴ h(x) is a constant function whose value is constant
for every value of x
Exercise 1 2
∴ h(3)=f2(3) – g2(3)=(5)2 – f '(3) = 52 – 42 = 9
Methods of Differentiation
\ f2(π) – g2(π) = 9

Sol 1: y = x sin kx 3 2
Sol 4: 3x2 + 4y2 = 12 ⇒ y2 = 3 – x
dy 4
= sin kx + xk cos kx
dx Differentiating both sides, we get
2
d y
= k cos kx + k cos kx – xk2sinkx dy 3 dy –3 x
dx2 2y =– x ⇒ =
dx 2 dx 4 y
d2 y
∴ +y=2k cos kx–xk2 sin kx+x sin kx = 2k cos kx  dy 
2
d2 y
dx2 3
2   + 2y =–
 dx  2
2
⇒ x sin kx(1 – k2) = 0 dx
2
\k = 1, –1 when 1 – k2 = 0  –3 x  d2 y 3
or 2   + 2y 2 = –
 4 y  dx 2
and k = 0 when sinkx = 0
18 x2 d2 y 3
+ 2y = –
Sol 2: Let f(x) = ax3 + bx2 + cx + d 16 y 2 dx 2 2
f(2x) = f’(x)f”(x) d2 y
18x2 + 2y3× 16 = –24y2
2
a(2x)3 + b(2x)2 + c(2x) + d dx
2
= (3ax + 2bx + c) (6ax + 2b)
2
d y –24y − 18x2
2
y3 =
8ax3 + 4bx2 + 2cx + d dx2 32

= 18a2x3 + (6ab + 12ab)x2 6 –12 × 6 9


= –(3x2+ 4y2) × = =–
+ 4b2x + 6acx + 2bc 32 32 4

4
∴8a = 18a2⇒ a = 0, Sol 5: f(x2)f”(x) = f’(x) f’(x2) … (i)
9
\f(x) should be a cubic equation f(1) = 1, f”’(1) = 8

4 Find f’(1) + f”(1)


\a ≠ 0 and a =
9 Differentiate the given equation
also 18ab = 4b f(x2)f”’(x) + f’(x2)f”(x)2x = f”(x)f’(x2) + f’(x)f”(x2)2x … (ii)
 4  Put x = 1 in equation (1)
b(18a – 4) = 0 ⇒ b  18 × – 4  = 8b = 0⇒b= 0
 9 
⇒ f(1) f”(1) = f’(1)f’(1) ⇒ f”(1) = [f’(1)]2 … (iii)
and 4b + bac = 2c
2

Put x = 1 in equation (2)


⇒c(6a – 2) = 0 ⇒ c = 0
f(1) f”’(1) + f’(1)f”(1)×2 = f”(1)f’(1) + f’(1)f”(1)2
4 3
∴ f(x) = x and d = 2bc = 0 ⇒ f”(1)f’(1) = 8 … (iv)
9
From equation (3) and (4)
Sol 3: f’(x) = g(x) ; g’(x) = f(x)
[f’(1)]3 = 8
Let h(x) = f (x) – g (x)
2 2
\f’(1) = 2 and f”(1) = (2)2 = 4
h’(x) = 2f(x)f’(x) – 2g(x)g’(x)
\f’(1) + f”(1) = 2 + 4 = 6
M a them a ti cs | 21.65

1  x
Sol 6: 2x = y 1/5
+ y 1/5 Sol 8: z = ln  tan 
 2
Take y1/5 = t
dz 1 x 1 dx dx dy dy
t2 – 2xt + 1 = 0 = sec2 × = cosecx ⇒ sinx =
dy x 2 2 dy dy dx dz
tan
2x + 4x2 − 4 2
∴t= =x+ x2 − 1
α
d2 y  dy d2 y  dx
= cos x + sinx 
\y1/5 = x + x2 − 1 dz 2  dx dx2  dz

1 –4/5 dy x x + x2 − 1 y1/5 d2 y dy d2 y dz d2 y
y = 1+ = = ∴ + cot x = cosecx × = cosec2x 2
5 dx x2 − 1 x2 − 1 x2 − 1 dx2 dx dz 2 dx dz
dy 5y d2 y
∴ = ∴ cosec2x + 4ycosec2x = 0
dx x −1 2
dx 2

2
dy 5yx d y
5 x2 − 1 − ⇒ + 4y = 0
d y 2 dx x2 − 1 dx2
=
dx2 (x2 − 1) sinx
Sol 9: f(x) = , x ≠ 0 , f(0) = 1
x
 dy 
5 (x2 − 1) − xy  2 d2 y dy
 dx  (x − 1) +x  x cos x − sinx 
= dx 2 dx  , x≠0 
(x2 − 1)3/2 f’(x) =  x 2

 0 , x = 0 
 
dy 5yx 5xy
= 5 x2 − 1 − + = 25y hcosh− sinh
dx 2
x −1 x2 − 1 −0
h2
\ k = 25 f”(0) = lim
h→0 h

2  hcosh− sinh 
d2 y
2(1 + y)  dy  = lim   (L-Hospital’s rule)
Sol 7: = 1+   h→0  h3 
dx 2
1 + y 2  dx 
 cosh− hsinh− cosh  1  sinh  1
y = tanz = lim   = lim–   =–
h→0  3h2
 h→0 3  h  3
dy dz dz dy
= sec2 z ⇒ = cos2 z 1
dx dx dx dx ∴ f "(0) = –
3
d2 z d2 y dy dz
∴ = cos2 z – 2coszsinz . 3/2
dx 2
dx 2 dx dx   dy 2 
1 +   
  dz      dx  
2(1 + tanz) 
2
1 1
= cos2 z 1 + sec 4 z     Sol 10: R =   ⇒ R2/3 = +
 sec2 z   dx    2
d y 2/3 2/3
  d2 y   d2 x 
dx2  2  2
2  dx   dy 
 dz     
– 2coszsinzsec2z  
 dx  dy dx 1
2
Let =t ⇒ =
 dz  dx dy t
= cos2z + [2(1 + tanz) – 2tanz]  
 dx  1 1
2
∴ R2/3 = + 2/3
2/3
 dz   dt   d (1 / t ) 
= cos2z + 2   ⇒ k = 2    
 dx   dx   dy 
 
2 1 . 6 6 | Methods of Differentiation and Applications of Derivatives

1 1 1 1 f(x) + f(–x)
= + = + f(0) = =0
 dt 
2/3
  dt  
2/3
 dt 
2/3
 1 dt 
2/3 k
        – 3 dx  \f(x) = –f(–x) or f(–x) = –f(x)
 dx   –1  dx    dx   t 
 t2  dy   ∴ If above function is satisfying the given condition
   then the function should be odd or f(x) = 0
  dx  

1  1  1 (x − a)3 (x − a)3 1
dy 1 + t2 
∴ =t= 2/3 1 + 2
= 2/3   Sol 13: f’(x)=4 (x − b) 3
(x − b) 3
1
dx  dt   1/t   dt 
    (x − c) 3 3
(x − c) 1
 dx   dx 
3/2 (x − a)4 (x − a)2 1 (x − a)4 (x − a)3 0
  dy 2   2
1 +     1 +  dy  
4 2 4 3
+ 3 (x − b) (x − b) 1 + (x − b) (x − b) 0
  dx     dx  
∴ R2/3 =  ⇒R= 2 4 3
  (x − c) (x − c) 1 (x − c) (x − c) 0
2/3
 d y 
2
d y
2
 2
 dx    dx2   (x − a)4 (x − a)2 1
      4 2
\f’(x) = 3 (x − b) (x − b) 1
4 2
(x − c) (x − c) 1
Sol 11: f(x) = n  1 + 1 + x2 
 
\λ = 3
g(x) = n  x + 1 + x2  ⇒ eg(x) =  x + 1 + x2 
 
   
Sol 14: P(x) = ax4 + bx3 + cx2 + dx + e
1  1  P(1) = a + b + c + d + e = 0  … (i)
f   = n  1 + 1 +  = n  x + 1 + x2  – n x
x
   2 
x   
 P(3) = 81a + 27b + 9c + 3d + e = 0 … (ii)

1 1 1 P(5) = 625a + 125b + 25c + 5d + c = 0 … (iii)


f '  = –
X 1+x 2 x P’(7) = 4a(7)3 + 3b(7)2 + 2c(7) + d = 0
= 1372a + 147b + 14c + d = 0 … (iv)
 1 
∴ xeg(x)  f   '+ g'(x) 
(2) – (1)⇒80a + 26b + 8c + 2d = 0
 x 
⇒ 40a + 13b + 4c + d = 0  … (v)
 x − 1 + x2  1
= x  x + 1 + x2   + (3) – (1)⇒624a + 124b + 24c + 4d = 0
   x 1 + x2  1 + x2
  ⇒ 156a + 31b + 6c + d = 0  … (vi)
2 2
x − (1 + x ) 1 (6) – (5)⇒116a + 18b + 2c = 0  … (vii)
= + =0
2 2
1+x 1+x (4) – (6)⇒1216a + 116b + 8c = 0

\For every x, given function is zero ⇒ 304a + 29b + 2c = 0  … (viii)


(8) – (7)⇒188a + 11b = 0
 x + y  f(x) + f(y)
Sol 12: f  = b 188
k ∴– =
 k  a 11
Put x, y = 0 –b 188
also =1+3+5+x= (sum of roots)
a 11
2f(0)
f(0) = 188 89
k \x = –9=
k – 2  11 11
⇒ f(0)  =0  k ≠ 2, 0  89 
 k  ∴ x −  is a root of 4 degree polynomial
 11 
\f(0) = 0
\p = 89 q = 11
Put y = –x
\p + q = 100
M a them a ti cs | 21.67

Alternate: \f”(x) = 0
Take P(x) = (x – 1) (x – 3) (x – 5) (qx – p) a+ x b+ x c+ x
f(x) =  − a x − b n − c
Now apply condition that P’(7) = 0
p −a q−b r −c
Sol 15: f(x) = x3 + x2 f(1) + xf”(2) + f”’(3) (a+x) [(m–b)(r–c) – (q–b) (n–c)]
T.P. f(2) = f(1) – f(0) + (b+x) [(  –a) (r–c) – (n–c) (p–a)
f’(x) = 3x + 2x f’(1) + f”(2)
2
+ (c+x) [(  –a) (q–b) – (m–b) (p–a)]
f’(1) = 3 + 2f’(1) + f”(2)
= a[(m–b)(r–c) – (q–b)(n–c)]
\f’(1) + f”(2) + 3 = 0  … (i)
+ b[(  –a)(r – c) – (n–c) (p–a)]
f”(x) = 6x + 2f’(1)
+ c[(  –a) (q–b) – (m–b) (p–a)]
f”(2) = 12 + 2f’(1) … (ii)
+ x[{(m – b(r–c) – (q–b)(n–c)]
f”’(x) = 6
+ {(  –a) (r–c) – (n–c) (p–a)}
\ f”’(3) = 6  … (iii)
+ {(  –a) (a, 0) – (m – b) (p–a)}]
f(2) = 8 + 4f’(1) + 2f”(2) + f”’(3)
= f(0) + kx
From (1), (2) and (3)
k= sum of all the co-factor of elements of f(0)
f’(1) = –5, f”(2) = 2
f”’(3) = 6 1
Sol 18: y = ∑ tan−1
2
f(1) = 1 + f’(1) + f”(2) + f”’(3) = 1 – 5 + 2 + 6 = 4 x + (2n − 1)x + {(n)(n − 1) + 1}
1
f(2) = 8 – 20 + 4 + 6 = –2 = ∑ tan−1
(x + n)(x + n − 1) + 1
f(0) = f”’(3) = 6
(x + n) − (x + n − 1)
\f(2) = f(1) – f(0) Hence proved = ∑ tan−1 (x + n)(x + n − 1) + 1
Let tanα = x + n
Sol 16: f(x) = sin2x[sin(x + x2)sin(x– x2) + cos(x + x2)
cos(x – x2)] + sin2x2[cos(x + x2) cos(x – x2) – sin(x + x2) tanβ = x + n – 1
sin(x – x2)]
= sin2xcos(x+x + x – x) + sin2x [cos(x+x + x – x )]
2 2 2 2 2
\y = ∑ tan−1 tan(α − β) = Sα – Sb
n n
= sin2xcos2x2 + sin2x2cos2x = sin(2x + 2x2) y= ∑ tan−1 (x + n) – ∑ tan−1 (x + n − 1)
n=1 n=1
\f’(x) = 2(2x+1)cos2(x2 + x)
\y = tan (x + n) – tan–1x
–1

a b c
1 1
Sol 17: f(0) =  m n y’ = −
2 2
1 + (x + n) x +1
p q r
1 + x2 − 1 − (x + n)2 x2 − (x + n)2
1 b+x c+x a+ x 1 c+ x = =
(1 + x2 )(1 + (x + n)2 (1 + x2 )(1 + (x + n)2 )
F’(x) = 1 m + x n + x +  + x 1 n+ x
1 q+ x r + x p+x 1 r+x
Sol 19: Y = SX
a+ x b+x 1 Z = tX
+  + x m+ x 1
Y1 = SX1 + S1X
p+ x q+ x 1
Z1 = tX1 + t1X
f’(x) = (m – b)(r – c) – (n – c) (q – b)
Y2 = SX2 + S1X1 + S2X + S1X1
+ (–1) [(  – a) (r – c) – (n – c) (p – a)]
2 1 . 6 8 | Methods of Differentiation and Applications of Derivatives

= SX2 + 2S1X1 + S2X ∴ Put x = sinq


Z2= tX2 + 2t1X1 + t2X = 5X2 + 2S1X1 + S2X x sin θ θ
∴ = = tan
 1 − x2  + 1 1 + cos θ 2
Z2 = tX2 + 2t1X1 + t2X  
 
X Y Z  θ θ 1
X1 Y1 Z1 \y1 = tan–1  tan  = = sin−1 x
 2 2 2
X2 Y2 Z2
1 −1
\y = sin x + 1 − x2
X SX tX 2
= X1 SX1 + S1 X tX1 + t1 X 1 1(–2x)
y’ = +
X2 SX 2 + 2S1 X1 + S2 x tX 2 + 2t1 X1 + t2 X 2
2 1−x 2 1 − x2

R2→ R2 – SX1, R3→ R3 – tX1 1 x 1 − 2x


= – =
2 1 − x2 1 − x2 2 1 − x2
X 0 0
X1 S1 X t1 X
1 + sinx + 1 − sinx
X2 2S1 X1 + S2 X 2t1 X1 + t2 X Sol 22: y = cot–1
1 + sinx − 1 − sinx
S1 t1
= X[S1 t2 X 2 − S2 t1 X 2 ] + X3
( )
2
S2 t2 1 + sinx + 1 − sinx
= cot −1
1 + sinx − (1 − sinx) )
x 1  2 + 2 1 − sin2 x 
Sol 20: y = tan–1 , x = sec–1  
2
1−u 2 2u − 1
= cot −1   = cot–1  1 + cos x 
 
2sinx  sinx 
 1   1 
µ =  0, ∪ ,1 
2  2  x  π x  π
 = cos–1cot ,x∈  0,  = , x ∈  0, 
2  2  2  2 
dy
To prove 2 + 1 = 0, take u = cosq
dx dy 1 π x π x π 
= or cot–1cot  −  = − x∈  , π 
\y = tan tanθ = θ = cos u
–1 –1 dx 2  2 2  2 2 2 
1 dy 1
x = sec–1 = sec–1sec2q = π – 2θ = π – 2cos–1u ∴ =–
2cos2 θ − 1 dx 2

dy –dcos−1 u –1 Sol 23: x = φ(t), y = ψ(t)


∴ = =
dx 2dcos−1 u 2
dx dy
= φ '(t) = ψ '(t)
2dy dy dt
⇒ +1=0
dx
d2 x d2 y
= φ”(t) = ψ”(t)
  1 − x  dt2 dt2
Sol 21: sin  2 tan−1  
  1 + x   dy 
   d 
 dx 
1−x 1−x dy ψ '(t) d2 y dt
2 tantan−1 2 = = –
1+x 1 +x = dx φ '(t) dx2 dx
= = 1 − x2

2 2 dt
1−x 
1 +  tantan−1  (1 + x) ψ '(t)
 1 + x  d
 φ '(t)
 x  d2 y dt φ '(t)ψ "(t) = ψ '(t)φ "(t)
∴ y = tan  –1
 + 1 − x2 = =
 2  dx dx 2
[φ '(t) φ '(t)]
 1− x +1
dt
M a them a ti cs | 21.69

 dx   d2 y   d2 x  dy dy
tangent 2 ( y − 2 ) 6x2
=
   2  −  2  dx
 dt   dt   dt  dt
= 3 dy 3h2
 dx  =
  dx h,k k − 2
 dt 
k − 2 3h2
=
Sol 24: (a) exy + ycosx = 2 h−1 k − 2
Differentiate the equation w.r.t. x
(k − 2 ) = 3h2 (h − 1 )
2

 dy  dy
exy  y + x  + cos x – ysinx = 0
 dx  dx ⇒ 2h3 – 4 = 3h3 – 3h2
dy
dx
(
xexy + cos x = ysinx – yexy ) ⇒ 3h2 – 4 = h3 ⇒ h3 – 3h2 +4 = 0
⇒ (h + 1) (h2 – 4h + 4) = 0
dy y sinx − yexy
∴ = h = – 1, 2
dx xexy + cos x
dy h = 2, ⇒ k = 2 ± 12
=–y
dx x =0 Eq. of tangent
12
at x = 0; y = 1 y–2=±
1
( x − 1)
dy
(i) ∴ = –1 7
dx x =0
Sol 2: y = ax2 + bx + at (1, 2)
2
also Now at (1, 2), we will get

d2 y 7
dx 2 ( xe xy
+ cos x + ) dy  xy xy 
 e + xe  y + x
dx  
dy  
 – sinx 
dx  
2=a+b+
2
 ........(i)

The tangent will be


dy  dy  dy xy y’ = 2ax + b
= ycosx + sinx – yexy  y + x – e
dx  dx  dx
y = x2 + 6x + 10
dy dy
y – y2 – − (1) y1( −2,2) = 2 ( −2 ) + 6 = 2
d2 y dx dx
(ii) =
dx (0 + 1)
x =0 −1
mnormal =
1 – 1 – (–1) – (–1) 2
= =2
1 −1
2a (1) + b = ⇒ 4a + 2b = – 1
2
(b)g(x) = eax + f(x)
−3
⇒a+b=
g’(x) = aeax + f’(x) 2
g”(x) = a2eax + f”(x) ⇒ 2a = 2; a = 1
g’(x) + g”(x) = (a + a2)eax + f’(x) + f”(x) 5
b=–
2
\g’(0) + g”(0) = a + a2 + f’(0) + f”(0) = 0
⇒a + a2 – 5 + 3 = 0 ⇒a2 + a – 2 = 0⇒a = 1, –2
Sol 3: xy = 1 – y

Application of Derivatives x2y = xy ⇒ y(x2 – x) = 0


y=0|x=0|x=1
Sol 1: y – 2 = m (x – 1)
curve (y – 2)2 = 2x3 – 4  1
A(0, 1),B  1, 
 2
2 1 . 7 0 | Methods of Differentiation and Applications of Derivatives

2xy Sol 6: y’ = 41x2 = 2009


2
−y′ ⇒ y′ = −
x y ′ + 2xy =
x2 + 1 2009 2
x2 = = t= 49
−1 41
y (′0,1) = 0 , y1 1
=
 1,  2  41t3 
 2
y −  = 2009 ( x − t )
Equation of tangents  3 

1
y− 41t3
2 =−1 ⇒ y =− x + 1 b– = −2009t
x −1 2 2 3
41t3
This gives y = 1, x = 0 or (0,1) b=
3
=
= −2009t
-2009t
t
3
(
41t2 − 2009.3 )
Sol 4:  41 × 49 
=7× − 2009  =-9375.33
   3 
if y = ( + x )
y

 
y’ = y (1 + x)y–1 =ln y y ln1 + y  − sin ( x + y ) 1 + y ′
Sol 7: y 2 =
 2 
= y ′ yy ′ ln (1 + x ) + y  sin ( x + y ) 1
 1 + x  y1 = − =

1 + sin ( x + y ) 2
 P2
(1 + x )
2y

y’= +
2sinx cos x sin ( x +=y ) 1,cos ( x += y) 0
 
(1 + x ) 1 − (1 + x ) ln (1 + x ) 1 + sin4 x
y
i.e. y = 0
π 3π P1
at x = 0, y’ = 1 + 0 = 1 x
= ,−
2 2
mnormal = –1
y 1 y 1
y −1 = − & = −
Equation of normal ⇒ =−1 ⇒ x + y =1 x−π/2 2 3π 2
x−0 x+
2
1 π 3π
Sol 5: x = 2t + t2 sin   , t≠0 2y + x = 2y + x = –
2 2
t
= 0 t=0
Sol 8: q = p2 P > 0
2
sint
y= t≠0 8
t s=– r > 0, s < 0
r
= 0 t=0
y – t2 = 2t (x – t) – tangent to curve (1) at x = t
 1   −1  1 8 8
x’ = 2 + t2 cos     + 2t sin  
2
 t  t  t
y+
=
z z2
( x − z ) – tangent to curve (2) at x – z
1 1  Both pass through (p, q) (r, s)
=2 – cos + 2t sin  
t t y = 2t x – t2 t = p
2t2 cos t2 − sint2 sint2 Same tangent
y’ = = 2cos t2 −
t2 t2 8x 16
y= − z=r
y′ 2t2 cos t2 − sint2 z 2 z
=
x′  1 1 tz = 4t2z = 16
2
t2  2 − cos + 2t sin 
 t t t2z4 = 16
2t2 cos t2 − sint2 z = 1, t = 4
Slope at (t=0) =
 1 1 z+t=p+r=5
t2 1 + 2t sin + 2sin2 
 t 2t 
Does not exist
M a them a ti cs | 21.71

x1 = x2 = α
Sol 9: (a) y = 36.6

y = x ⇒ y′ =
1 (
x3 = 2α ⇒ P2 −2α , −8α3 )
2 x
x = 36, ∆x =0.6
( α, −2α, 4α......... ) forms a G.P.
Tangent at P2
⇒ f ( x + ∆=
x ) f ( x ) + f ′x ∆x
y + 8α 3
1 0.6 = 12α2
⇒ f ( 36.6 ) = f ( 36 ) + × 0.6 = 6 + = 6.05 x + 2α
2 36 120
y = 12α2 x + 16α3
(b) ( 26 )
1/3
α1 + α2 + α3 = 0
α1 + α2 = −2α
y = (x)
1/3

y′ =
1 −2/3
x x = 27 ∆x =−1
α3 = 4α P3 = 4α ,64α3( )
3
1 α α3
1 x ( −1 ) 1 80
( 26 ) f ( 27 ) + 2/3 = 3 − 3 ×1 9 = 3 − 27 = 27
f= 1 3
Area of P1P2P3 = −2α −8α 1
3x 2
4α 64α3 1
(ii) r= 9 ± 0.03
−2α
1 −8α3
4 3 1
v= πr Area of P2P3P1 = +4α 64α 3
1
3 2 3
−8α −512α 1
dv 4 πr 2dr 3dr 3 × 0.03 1
= = = =
v v R 9 100 P1P2P3
=
( )
α −72α3 − α3 ( −6α ) + 1 ( −128 + 32 ) α 4
4 3
πr P2P3P4 −2α (576 ) + 8α3 (12α ) + 1 ( −2048 + 512 ) α 4
3 4π
dv= = × 9 × 9 × 9 = 9.72 π
100 300 −72 + 6 − 96 162 1
= = =
96 − 1536 − 1152 2592 16
Sol 10: Mid point was (2, –1)
Sol 12: f1 x = ( fx )
2
y +1
= 1 ⇒ x – y = 3 (equation of tangent)
x−2 dy 1
x – 3 = – a2x2 + 5ax – 4 ∫ y 2 = ∫ dx –
y
= x+c

⇒ a2x2 + x (1 – 5a) + 1 = 0 −1
For f (0) = , We have c = 2
 1 − 5a  5a − 1 2
α + β = −1
 −a2  a2 ⇒ y=
x+2
α+β 5a − 1
= 2= ⇒ 4a2 – 5a + 1 = 0 1
2 2a2 y+
2 = 1
1 x 4
a = 1, +
4 4y + 2 =x

Sol 11: P1 α , α3( ) Sol 13: y = ax3 + bx3 + cx + 5 


y − α3 = 3α2 ( x − α ) y − 3α2 x − 2α3 y’ = 3ax2 + 2bx + c (-2, 0)

y’ (–2) = 12 a – 4b + c = 0
x3 − 3α2 x + 2α3 =0 
– 8a + 4b – 2c + 5 = 0
x1 + x2 + x3 =
0
2 1 . 7 2 | Methods of Differentiation and Applications of Derivatives

c = 3 [y1 (x = 0) = 3)] Passes through (0, 2)


12a – 4b + 3 = 0 f(o) = c = 2
– 8a + 4b = 1 f(p) = 2e–x/2
1 −3
4a = – 2 ⇒ a = – and b =
2 4 Sol 17: (a) Similar to exercise (3) q.7
Sol 14: y − t= 3t 3
(x − t)
2
(b) y = a ln (x2 – a2) 
3
⇒ 8 − t= 3t ( 2 − t ) ⇒ 8 − t3 =
2
6t2 − 3t3
y 1 + ( y′)
2
y
⇒ 2t – 6t + 8 = 0
3 2
+
y′ y′
t = – 1, 2, 2
2ax
m = 3t2 , (m)–1 = 3, (m)2 = 12 y′ =
x − a2
2

x
Sol 15: f 3 ( x ) = ∫ t f 2 ( t ) dt x2 + a2
1 + ( y′) =
2

0 x2 − a2
3f 2
( x=) ′x x f 2 x
f= 2 2
( 2 2
)( )
y ln x − a x − a
=
f x ( 3f ′x − x ) =
2
0 y′ ( 2x )
Either f(x) = 0 (not possible)  x2 + a2 
x2 − a2
x
or f ′x = Mnormal =
−3 1
− ⇒x=6

2x
ln x2 − a2 ( 2
 x − a
2
+ 1)

3 x 2

7x =
x2
+c ( xy
⇒ xln x2 − a2 = i.e. αxy
a
)
6
Equation of normal x = y2,xy = k

−1 1 −k
y–6–c= (x – 6) y′ = y′ =
2 2y x2
Intercept on y axis y3 = k intersection is k 2/3 ,k1/3 ( )
y=9+c
y1′ × y 2′ =
−1
3 x 2 2
 x2  x   x4 x2  −k
 + c = ∫  6 + c x =  + c2 +  x = −1
 6    36 3  2x2 y
  0  
x k = 2x2y = 2y5
x5 x3
= ∫ + c2 x + k = 2k5/3
36 3
0
k–2/3 = 2 ⇒ k–2 = 8
6 2 2 4 6 2 2 4
x 3c x 3x c x c x x 1
+ c3 + + = + + =c=0 k=±
6.36 6 36 6.36 2 12
2 2
Intercept is 9
Sol 18: x + 5y – y5 = 0(0, 0) (0, 51/4)

Sol 16:  y − f (p )  = f ′ (p )( x − p ) 1 + 5y’ – 5y4y’ = 0 


1
y’ = ,
– f (p ) = f ′ (p ) 2 5y − 54

1 f ′p 1
⇒ f ′p = − f (p ) ⇒ = − Equation of tangent
2 fp 2
−1x
−x =y=
ln fp
= + c ⇒ fp = c e–x/2 5
2
M a them a ti cs | 21.73

Equation of normal y = 5x
Coordinates are (0, 0) (–25, 5), (1, 5) 64 + 2 −62
y=± = or11
6 6
1
Area = × 5 × 26 =65  −31 
2 Hence (4, 11)  −4, 
 3 
1
y−
Sol 19: 2 =4
x−2 Sol 22:
4.5
15 1 15
y = 4x – ⇒ = 4x − 1.5
2 x 2 x y 4 km/hr
⇒ 8x2 – 15x – 2 = 0 ⇒ 8x2 – 16x + x – 2 = 0
1 1.5 4.5
⇒ (8x + 1) (x – 2) = 0 ⇒ x = – =
8 y y + x ⇒ 3y = 1.5x
−1
y’ = = −64 x dy 1 dx dy 1 dx dy
x2 y= = ⇒ = ⇒ 2
=
2 dx 2 dt dt 2 dt dt
y ′ = 64
dx dy 12
+ = =6
dt dt 2
Sol 20: f(x) = ln2x + 2lnx  dy
hcc shadow is lightening at rate i.e. 2 km/ hr
y = m 1x dt
2lnx + 2 dv
y1 = Sol 23: = −1cm3 / sec
x dt
y – (ln2 t + 2lnt) 15 cm

2 2I
= (ln t + 1) [x – t] 10
t
It passes through (0, 0) 
10 cm
–ln2t – 2lnt = – 2 (1 + ln t)
h
ln2t = 2

lnt = ± 2

t= e 2
, e− 2
ab = 1 dv 3 r
= c cm3 / sec c, tan θ= =
dt 2 h
 2lnx + 2 
(ii) 5x   − xln10 − 10 =
0 dh dr 3 dh
x = 4= = 6
  dt dt 2 dt
10 lnx + 10 – x ln 10 – 10 = 0 1 2
v= πr h
3
10lnx = xln10
dv π r 2dh πh dr
=
dt 3 dt
+
3
( 2r )
dt
2 solution from graph
πr 2 4 2π dr
1 is x = 10 c–1 = + hr
3 3 dt
h = 2r = 3
Sol 21: Given that 6y = x3 + 2 and also dy = 8dx
π 2π
6dy = 3x2dx (Differentiating the given equation) c − 1= ×9× 4 + × 6.6= 12π + 24 π= 36π
3 3
x2
⇒ =8 ⇒x=±4 c = 1 + 36 π
2
2 1 . 7 4 | Methods of Differentiation and Applications of Derivatives

dv r Sol 27: A (0, 0)


Sol 24: = −2 
dt h  7t2 
1 2 C  t,1 + 
/4  36 
v= πr h 
3 4
l B (0, 1) initially
r= h=
2 Co-ordinate of B at time (0, 1 + 2t)
π 3 Co-ordinate of C at time
v=
32 2 7x2 B(0, 1)
1+ = 1 + 2t
2
C (x, 1 + 7x )
dv π d 36 36
= 32
dt 6 2 dt 2t
x=6
π × 16 d 7
–2= A(0, 0)
2 2 dt 2t
BC = x = 6
d 1 − 2 7
=
− =
dt π2 2 4π AB = 1 + 2t

1 2t 2t 2t
Sol 25: h =
1
r
Area =
2
( AB )(BC ) =
3 (1 + 2t )
7
3
=
7
+ 6t
7
6
dA 3 3 2t
1 2 1 = + ×6
⇒ ∫ dv= πr h ⇒ v= π 36h3 dt 7t 2 7
3 3
dA
∫ dv= 12πh3 = 3 2 +9
dt t = 7
2
dv dh
= 36πh2
dt dt
Sol 28:  R
dh 2 r
⇒ 12 = 36πh ×
dt dv k h
=
1 dh 1 dt r y
⇒ = = cm / sec
3π × 16 dt 48π 4 3
v= πr
3
dA dv dr
Sol 26: = 2cm2 / sec = 4 πr 2
dt dt dt
A = πr 2 dr k ⇒ dr k
4 πr 2 = =
dt r dt 4 πr 3
dA dr dr
= 2πr ⇒ 2 = 2πr
dt dt dt 2 15
⇒ πr 4 kt ⇒ π15 =
= 15k
1 0
dr 1 dt
⇒ = ⇒ rdr = ⇒k= π
dt πr π
t t
r 2 1 28 7.28 ⇒ πr 4 =
π
⇒ = ⇒ =r 2 1 0
2 π π 11.11

⇒r=
14 (
⇒ π r 4 − 1 πt )
π
(1 + t )
1/ 4
dr 7 × 11 1 r
⇒=
= = cm/sec
dt 22 × 14 4 dv k
⇒ =
dt
(1 + t )
1/ 4
M a them a ti cs | 21.75

Also f(f–1(x)) = x
π (1 + t )
−1/ 4
⇒ dv = dt
f’(f–1(x)) (f–1(x))’ = 1
27v t 1
4π ∴(f–1(x))’ =
( 1 + t)
3/ 4
∫v
⇒=
3 0
f ' f (x)

–1

v
\f(ln2) = y
4π 
(1 + t ) − 1
3/ 4
⇒ 26v = \f–1(y) = ln2
3 
Qf’(ln2)= 1 + eln2= 1 + 2 = 3

⇒ 27 =(1 + t )
3/ 4
⇒v= 1 1 1
3 \[f–1(y)] = = =
 –1
f ' f (y)  f '(  n2) 3
 
⇒ (1 + t )
1/ 4
3 ⇒ t = 80 sec
=
Sol 3: (C) y(x) = f2(x) + g2(x)
dv
Sol 29: = 6m3 / min y’(x) = 2f(x) g(x) + 2g(x) g’(x)
dt
= 2f(x) g(x) – 2f(x)g(x)= 0
π 2
=v y ( 3R − y ) \y(x) = a = f2(x) + g2(x)
3
dv π  dy dy  y(5) = a = f2(5) + g2(5)= (2)2 + (2)2
= 6Ry − 3y 2 π 2Ry − y 2  y ′
–6 =
dt 3  dt dt    \a = 8
−6 −6 1 \y(10) = f2(10) + g2(10) = 8
For y = 8 , y ′ = = = − m / min
8π ( 2R − y ) 8π (18 ) 24 π
r   +m  1  m +n  1  n+   1 
tan θ =   +  +  
 m−n  n−   n−    –m   –m  m−n 
y Sol 4: (B) y(x) = x

dr dy −1 × 5 5
= tan θ = =−  1   +m m+n   n+   1
+
dt dt 24 π × 12 288π    + 
= x n−  m–n  −m    –m  m−n

 1   2 −m2 +m2 −n2  (n+  ) 1


+ ×
Exercise 2 = x
 
 n−    (m−n)(  −m)  (  −m) (m−n)

Methods of Differentiation –
–(  +n)
+
(n+  )
(m−n)(  −m) (  −m)(m−n)
= x = x0 = 1
Single Correct Choice Type
dy
x ∴ =0
Sol 1: (D) y = dx
x
a+ x)
b+y Sol 5: (D) f(x) = (xx)x g(x) = x(x
xy
⇒ay + =x logf(x) = xlogxx= x2logx
b+y
1
aby + ay2 + xy = xb + xy f '(x) = 2xlogx + x
f(x)
\aby + ay2 = xb f’(x) = (xx)x[2xclogx + x]
dy dy log(x) = xxlogx
⇒ ab + 2ay b
=
dx dx
1 xx dx x
dy b g'(x)
= + logx
∴ = g(x) x dx
dx 2ay + ab
= xx–1 + (logx) (xx(logx+1))
Sol 2: (B) f(x) = ex + x  xx 
 
∴ g’(x) = x   x x −1 + x x (logx + 1)logx 
f’(x) = 1 + ex  
2 1 . 7 6 | Methods of Differentiation and Applications of Derivatives

f’(1) = [2.1.log1 + 1] (11)1 = 1 – 5sin–1(a2 – 8a + 17)


1 f’(x) = –x2 + 2xsin1.5a – sinasin2a
g’(1) = (1)(1 ) [11–1 + 11(log1+1)log1] = 1
f’(sin8) = – sin28 + 2sin8sin1.5a – sinasin2a
Sol 6: (C) y1/m + y–1/m = 2 x Also –1 ≤ a2 – 8a + 17 ≤ 1

Let y1/m = a –1 ≤ (a – 4)2 + 1 ≤ 1

1 –2 ≤ (a – 4)2 ≤ 0
\a + = 2x ⇒a2 – 2ax + 1 = 0
a ⇒a = 4

⇒a = x + x2 − 1 ∴ f’(sin8) = – sin28 + 2sin8sin6


– sin4sin8
y1/m = x + x2 − 1 f’(sin8) = –sin28 + sin8(2sin6 – sin4)
m
\y =  x + x2 − 1   2sin6 < sin 4 π< 4 < 6 < 2p
  m

 m  x + x2 − 1  and sin8 < 0


m−1
2x 
⇒ y’= m  x + x2 − 1  1 + =  
   2  2 \f’(sin8) < 0
 2 x −1  x −1
cos6x + 6 cos 4x + 15cos2x + 10
Sol 9: (B) y =
⇒ y ' x2 − 1 = my cos5x + 5cos3x + 10 cos x
cos6x + cos4x + 5cos4x + 5cos2x
y '2x
⇒y” y ' x2 − 1 + = my’
+ 10(cos2x + 1)
2 x2 − 1
10x 2x
⇒y”(x2 – 1) + xy’ = my’ x2 − 1 ⇒ 2cos cos
2 2
(4 + 2)x x
y "(x2 − 1) + xy ' y' +2×5cos cos + 10 × 2cos2x
∴ = m x2 − 1 2 2
y y
= 2cosx[cos5x + 5cos3x + 10cosx]
2 m
= m x −1 × =m 2
\y = 2cosx
x2 − 1
dy
∴ = –2sinx
Sol 7: (C) y = P(x)
2 dx
dy
2.y = P'(x) Sol 10: (D) y = R(1 – cosθ)
dx
x = R(θ – sinθ)
⇒2.yy” + 2(y’)2 = P”(x)
dy dy / dθ sin θ
Multiply this equation by y2 = =
dx dx / dθ (1 − cos θ)
⇒ 2y3y” + 2(yy’)2 = P”(x)y2 = P”(x) P(x) 1 + cos θ
= = cosecθ + cotq
sin θ
d  3 d2 y 
∴2 y  = [P”(x) P(x) – 2(yy’)2]’  dy 
dx  dx2  d 
 dx 
 [P'(x)]2 ′ d2 y
= dθ
= [P”(x) P(x)]’ – 2   dx
dx2
 4 

= P’’’(x) P(x) + P”(x)P’(x) – P’(x) P”(x)= P”’(x)P(x) – cosecθ cot θ − cosec2 θ
=
R(1 – cos θ)
x3 (1 + cos θ) 1 (1 + cos θ)
Sol 8: (D) f(x) = – + x2sin1.5a – x.sina.sin2a = –cosecθ ×
3 sin θ R sin2 θ
M a them a ti cs | 21.77

x
1 1 Sol 14: (C) f(x) = ee g(x) = f–1(x), f(g(x)) = x
=– (1 + cos θ)2 ×
sin4 θ R
f’(g(x) g’(x) = 1
2 2
1  1 + cos θ  –1  1  1
=–   =   g’(x) =
R  sin θ 
2 R  1 − cos θ  f '(g(x))
x
2 2 g(2) ⇒ 2 = ee ⇒ x = ln (ln2)
d y 1 1  1
∴ =–   = –
dx 2 R  1 − (–1)  4R \g(2) = ln ln2
θ=π

1 1
Sol 11: (B) f(x) = (1 + x)n \g’(2) = =
f '(g(2) f '(lnln2)
f’(x) = n(1 + x)n–1 x
f’(x) = ex .ee
f”(x) = n (n –1)(1 + x) n–2

lnln2
fn(x) = n(n – 1) ……. 2.1 (1+x)0 \f’(ln ln2) = eln ln . ee = 2ln2

f(0) = 1, f’(0) = n, f”(0) = n(n – 1)


Sol 15: (C)
f n 0)
f(0) + f’(0) + ….+  1 − 2ln | x |   3 − 2ln | x | 
n!
y = tan–1   + tan–1  
n(n − 1) n(n − 1)...(2.(1)  1 + 2ln | x |   1 – 6ln | x | 
=1+n+ …..
2! n!
Let 3 = tanα, 2ln|x| = tanb
n
= C0 + nC1 + nC2 +  + nCn = 2n
 π 
∴ y = tan–1  tan  − β   + tan–1(tan(α + β)
Sol 12: (D) y = e4x + 2e–x  4 
π
dy = –β+α+b
= 4e4x – 2e–x 4
dx
π π
y= + α = + tan–13
d2 y 4 4
= 16e4x + 2e–x
dx2 dy
∴ =0
dx
d3 y
= 64e4x – 2e–x
dx3 x
Sol 16: (B) lim (x x − x x )
d3 y dy x →0 +
− 13 = 64e4x– 2e–x– 13(4e4x– 2e–x) xlnx
dx3 dx xx = e lnx
1
= 12e + 24e = 12y
4x –x  
x
∴ lim x x = lim exlnx = xlim
→0
e
∴K = 12 x →0 x →0
1
 
(lnx)' x
Sol 13: (C) x + 3x y + 7xy + 4x y – 15y = 0
4 2 2 3 3 4
 1 
1
 ' – 
 2
⇒ (x – y)(x + 5x y + 8xy + 5y ) = 0
3 2 2 3
= lim e x = lim e  x  = lim e–x = 1
x →0 x →0 x →0
This is in the form f(x, y) g(x, y) = 0 where f(x, y) = 0 and
g(x, y) ≠ 0 at P(x1, y1)
lim x x = lim ( x )( x ) = lim ( x ) = 0
x x 1

⇒ f’(x, y) g(x, y)+f(x, y) g’(x, y) = 0⇒f’(x1, y1)=0 x →0 + x →0 + x →0 +

Also  xx x
∴ lim  x − x  = 0 – 1 = –1
x →0 
–1 
f’’(x, y) g(x, y)+2 f’(x, y) g’(x, y)+f(x, y)g’’(x, y)=0

⇒ f’’(x1, y1) = 0 ⇒
d2 y
dx 2
= 0 at (1, 1) x →0
{
Sol 17: (B) lim (cot x)x + (1 − cos x)cos ecx }
2 1 . 7 8 | Methods of Differentiation and Applications of Derivatives

ln(1 −cos x) 2 tanx tan2x tan3x


lim exlncot x + lim e sin x +
x →0 x →0 sin2x cos2x
lncot x
1 = 2tanxtan2xtan3x ×
 
lim e x + lim e cos ecxln(1–cos x)
x →0 x →0  3 1 
 + 
 2sin3x cos3x 2sinx cos x 
lim ecos ecxln(1–cos x) doesn’t exist as LHL ≠ RHL  2 
x →0  +
2sin2x cos2x 
= 2y [2cosec6x + 2cosec4x + cosec2x]
Multiple Correct Choice Type

Sol 20: (A, C, D)


Sol 18: (B, C)

y= x x + x + .....∞
 1 + x2 − 1 
f(x) = tan–1  
 tan θ  ⇒y = x+y
 

Put x = tanq x
⇒y2 – y = x ⇒ (y – 1) =
y
 1 + tan2 θ − 1  dy dy
∴ f(x) = tan−1  , ∴ 2y – =1
 tan θ  dx dx
 
tanq≠ 0 dy 1 1 1 y
∴ = = = =
dx 2y − 1 2(y – 1) + 1 x 2x + y
2. + 1
 sec θ − 1  −1  1 − cos θ  y
= tan−1   = tan   1 ± 1 + 4x
 tan θ   sin θ  also y =
2
 θ  θ tan−1 x dy 1 1
= tan–1tan   = = ∴ = =
2 2 2 dx
2
(1 ± 1 + 4x ) −1 ± 1 + 4x

tan−1 (x) tan−1 x 2


∴ f(–x) = =– = – f(x)
2 2 Sol 21: (A, B, C, D)
∴ f(x) is a odd function 2x + 2y = 2x+y
1  1 dy  dy 
also f’(x) =  tan−1 x  = , x ∈ R – {0} 2xln2 + 2yln2 = 2x+yln2  1 +
2  2(1 + x2 ) 
dx  dx 
dy
Sol 19: (A, B, C) (2y– 2x+y) = 2x+y – 2x
dx
y = tanx tan2xtan3x
dy 2x (2y − 1) 2x (1 − 2y )
= =
x+2x–3x = 0 dx 2y (1 − 2x ) 2y (2x − 1)
⇒ tanx+tan2x–tan3x = tanxtan2xtan3x = y Also 2x+y – 2y = 2y(2x – 1) = 2x
⇒y’ = 3sec23x – sec2x – 2sec22x or 2x+y – 2x = 2x(2y – 1) = 2y
or y’ = (tan3x)’tanxtan2x + (tanx)’tan2xtan3x dy 2x 2y (2x – 1)
∴ =– or =1
+ (tan2x)’tanxtan3x dx 2y 2x
= 3sec23x tanx tan2x + sec2x tan2xtan3x dy 2x (2y − 1)
∴ = (1 − 2y ) or =1
+ 2sec22xtanxtan3x dx 2y
3tanx tan2x tan3x tanx tan2x tan3x dy 1 1
= + ∴
= –=
sin3x cos3x sinx cos x dx (2 − 1) 1 – 2x
x
M a them a ti cs | 21.79

Sol 22: (A, B, C)  3 −3/5


 x x ≤1 
y+x + y–x=c f ′(x) =  5
 −3 ( x − 2 )2 x >1

1 dy  1  dy  1
 + 1 +  − 1 = 0
2 y+x dx  2 y−x dx  3
x=1 , −3
5
  y+x − y−x
dy  y − x + y + x 
= x = 1 is critical point
dx  y 2 − x2  y 2 – x2
   −9 −8/5 
 x x ≤ 1
f ' ( x ) =  25 
Application of Derivatives −6 ( x − 2 ) x > 1
 
x = 0
Single Correct Choice Type  critical point
x = 2

Sol 1: (C) 3x2 + 4xy + 5y2 – 4 = 0 at x = 2, f’’ (2) changes its sign

y′ = −
(3x + 2y )
Sol 4: (A) x = a (2 cos t – cos 2t)
( 2x + 5y )
y = a (2 sin t – sin 2t)
3
sin θ y ′ =
0 when y =− x
2 dy
= a ( 2cos t − 2cos2t )
2 dt
and y ′ =
∞ when y =
− x
5
dx
So angle is 90º. a ( 2sint + 2sin2t )
=−
dt

Sol 2: (D) x = sec2t, y = cot t dy cos t − cos2t cos t − 2cos2 t + 1



= = = 0
dx sin2t − sint 2sint cos t − sint
1
⇒ x = 1+
y2 ⇒ 2 cos2t – cos t – 1 = 0
π ⇒ 2 cos2t –2 cost + cost – 1 = 0
At t = , x = 2, y = 1
4 ⇒ (2 cos t + 1) (cos t – 1)
P (2, 1) −1
cos t = 1, cos t =
y −1 −1 2
⇒ ′
= y= ⇒ 2y − 2 = 2 − x ⇒ x + 2y = 4
x−2 2 2π 4 π
t = 0, 2t, t = ,
y +12 3 3
⇒ 4 – 2y = putting the value of x in first equation dy
y2 t = 0 is not possible as is not defined
dx
⇒ 4y 2 − 2y 3 =y 2 + 1 ⇒ 2y 3 − 3y 2 + 1 =0 4 π 2π 2π
t= − =
⇒ (y – 1) (2y – y – 1) = 0 ⇒ (y –1) (2y + 1) (y – 1)
2
3 3 3
1  1
y= − ; x = 5;  5, −  Sol 5: (C)
2  2

9 45 3 5
PQ = 9+ = =
4 4 2

 x3/5 x ≤1
Sol 3: (C) f(x) = 
 − ( x − 2 )3 x >1

y − an −1 −1
= m
= =
x−a y 1
nxn−1
2 1 . 8 0 | Methods of Differentiation and Applications of Derivatives

y1 = nxn−1 −b
⇒ y’ =
a
a
y = an + y − b −b
nan−1 ⇒ =
x−a a
2 −n
aa1−n n a na2n + a2 ⇒ ay + bx = 2ab
= an +
b a + =
n n nan
x y
⇒ + =2
2 2n−1
2n a + 2a 2n ( 2n − 1 ) a
2 2n−2
+2 a b
lim b = = ,
a→ 0 na 2 n−1
n (n − 1 ) a
2 n− 2
Multiple Correct Choice Type
1
value of b exist & equal to
2 Sol 9: (A, D) We can write
only if (n = 2) xy = k  … (i)
x y
 −x2 x < 0 + =1  ... (ii)
Sol 6: (B) f(x) =   a b
 x2 + 8 x ≥ 0  Solving these two equations, we get
For points (a, –a ) 2
x2b + abx + ka = 0
2
y+a For D = 0, (ab)2 - 4abk = 0
= −2a
x−a
ab (ab - 4k) = 0
⇒ y + a2 = 2a2 – 2ax
⇒ ab + 4k
⇒ y = a2 – 2ax = x2 + 8
⇒ ab > 0
⇒ x2 + 2ax – a2 + 8
⇒ 4a2 = –4 (8 – a2)
Hence a > 0, b > 0 or a < 0, b < 0
At a = – 2
y+4 Sol 10: (A, B) xy= a + x
⇒ = +4
x+2
Therefore x intercept = – 1 ( xy ′ + y ) = 1
2 xy
1
Sol 7: (D) y =
2 + cos2 x 2 xy − y
y′ =
x
2cos x sinx 
 a + t  (a + t )
2
⇒ y’ = –  (a + t )
2
2 t
( ) −
2
2 + cos2 x y− t
t  t 
=
x−t t
π π
=x 0 π For x=0 or x=
(a + t ) ( a + t ) t − ( a + t )( a + t )
2
2 2
y− 2
t = t t
11 11 ππ
⇒ y ==
=
33
( ( 0)=
ifif xx 0= )
, ifx
22
ifx 
22
x−t t
a+t  a
= 1 − 
t  t
Sol 8: (A) At P (a, b), the equation will be
2 2
a+t t −a
y −b = ( t − a) =
= M where M is the slope t2 2
t
x−a
n−1 n−1 x intercept will be
n x  n y 
⇒   +   y′ =
0
( t + a ) t2 t − ( t + a) t −2at
2
a a bb
⇒t− = =

n n
+ y′ =
0 ( 2
t t −a 2
) ( t − a) t − a
a a
M a them a ti cs | 21.81

y intercept will be Equating (i) and (ii) and keeping them opposite in sign

=
(a + t )
2


(t 2
− a2 ) =2a
2
+ 2at 2at
=
 t3 5t2 
− − + 7t − 4 
t t t a−t 3 2   t3 5t2 
= a2 – t2 = t2 ⇒ a2 = 2t2

t2 − 5t + 7
 +=
( )
t t t2 − 5t + 7 −  +
3
 2
− 7t + 4 


a a Solving above, we get t = 2,3
t= ,− [A, B]
2 2  8  7 
Therefore, co-ordinates are  2,   3, 
Sol 11: (B, D) 2x – 3 = x2 + px + q  3  2 

p + q + 1 = –1  Sol 13: (A, B, D)

p + q = –2 y cot x = y3 tan x

 y2 = cot2x
p2 
y = q −  is minimum
4  y cot x, − cot x
⇒=

p2  π  π 
=–2–p–  − , −1  − , +1 
4  4  4 
2p
⇒–p– =0 y + 1 −1 π
4 = ⇒ y + 1 = −2x − ⇒ 4x + 2y = 2 + π
π 1 2
x+
p = −2 4 2
Therefore
a=0 y −1 1 π
= ⇒ y − 1 = 2x + = 4x − 2y = 2 + π
π 1 2
4 x+
Least distance is 0 − 1 D is correct
= 4 2
4

Sol 14: (A, B)


x3 5x2 x = a ( t + sin t cos t)
Sol 12: (A, B) Given that f(x) = − + 7x − 4
3 2 y = a (1 + sin t)2
f ′ ( x ) = x2 − 5x + 7
2a (1 + sint ) cos t
y’ =
Equation of triangle will be (
a 1 − sin2 t + cos2 t )
 t3 5t2 
y − − + 7t + 4  2 (1 + sint ) cos t
3 2  =
  = t2 − 5t + 7
2cos2 t
x−t
1 + sint
x intercept will be tan θ =
cos t
 t3 5t2 
− − + 7t − 4  −1  1 + sint  π + 2t
3 2  =θ tan
=  
x−t = 
 cos t  4
t2 − 5t + 7

 t3 5t2  Sol 15: (B, D) y =t3 − 4t2 − 3t


 − + 7t − 4 
3 2  y = 2t2 + 3 – 5
⇒x=−  +t ... (i)
2
t − 5t + 7 dy
= 4t + 3
y intercept will be dt

 t3 3t2  dx
= 3t2 − 8t − 3
−t(t2 − 5 t + 7) +  −
y= + 7t − 4   ... (ii) dt
3 2 
 
2 1 . 8 2 | Methods of Differentiation and Applications of Derivatives

dy 4t + 3 Thus, the point where the tangents are parallel to 8x =


=  2 1 2 1
dt 3t − 8t − 3
2
9y are  − ,  and  , −  .
 5 5  5 5
dy −3
= 0 ,t = , H=1 Therefore, (b) and (d) are true answers.
dx 4
dy
= not defined at 3t2 – 8t – 3 = 0 Sol 3: Given, y3 – 3xy + 2 = 0
dx
dy dy
3t2 – 9t + t – 3 = 0 ⇒3y2 – 3x – 3y = 0
dx dx
(3t + 1) (t – 3) = 0
dy
⇒ (3y2 – 3x) = 3y
−1 dx
t = 3,
3 dy 3y
⇒ =
v=2 dx 3y 2 – 3x

Thus, the point where tangent is horizontal. The slope


Previous Years’ Questions of tangent is 0
dy 3y
∴ = 0 ⇒ =0
Sol 1: (B, C) dx 2
3y – 3x
⇒ y = 0 but y = 0 does not satisfy the given equation of
1 dy 1
Given, xy = 1 ⇒ y = ⇒ =– the curve therefore y cannot lie on the curve.
x dx x2
So, H = φ(null set)
Thus, slope of normal = x2 (Which is always positive)
dy
and it is given ax + by + c = 0 is normal whose slope For the point where tangent is vertical, then =∞
a dx
=–
b y
⇒ =∞
2
a a y −x
⇒ – > 0 or < 0,
b b ⇒y2 – x = 0 ⇒ y2 = x
∴ a and b are of opposite sign.
On putting this value in the given equation of the curve,
we have
Sol 2: (B, D) Given, 4x2 + 9y2 = 1
On differentiating w.r.t. x, we get Sol 4: Let the house of the swimmer be at B
dy ∴AB = L km
8x + 18 y =0
dx
Let the swimmer land at C, on the shore and let
dy 8x 4x
⇒ =– =– AC = x km
dx 18y 9y
The tangent at point (h, k) will be parallel to 8x = 9y,
4h 8 S
then – =
9k 9
d 2 2
⇒h = – 2k x +d

Point (h, k) also lies on the ellipse


A x C (L - x ) B
∴ 4h2 + 9k2 = 1 … (ii) L
On putting value of h in Eq. (ii), we get
4(– 2k)2 + 9k2 = 1 ∴ SC = x2 + d2 and CB = (L – x)
⇒16k2 + 9k2 = 1
Dis tance
⇒25k2 = 1 ∴ Time =
Speed
1 1
⇒ k2 = ;k=± Time from S to B = time from S to C + time from C to B.
25 5
M a them a ti cs | 21.83

Atx = 0, g’’(x) = – 6 < 0


x2 + d2 L−x
\T= + ∴ g’(x) has maximum value at x = 0
u v
⇒ (x = 0, y = 0) is the required point at which tangent
1 L x
Let f(x) = T = x2 + d2 + – to the curve has the greatest slope.
u v v

1 1.2x 1 Sol 6: Given, y = cos (x + y)


⇒ f ’(x) = . +0–
u 2 x2 + d2 v  dy   dy 
⇒   = – sin(x + y) .  1 +  …(i)
For maximum or minimum,  dx   dx 

Put f ’ (x) = 0 Since, tangent is parallel to x + 2y = 0,

⇒ v2x2 = u2(x2 + d2) dy 1


Then slope =–
dx 2
u2d2
⇒ x2 = ∴ From Eq.(i),
v 2 − u2
\ f ‘(x) = 0 1  1
– = – sin(x + y) .  1 − 
2  2
ud
at x = ± , (v > u) ⇒sin(x + y) = 1
v 2 − u2
Which shows cos ( x + y) = 0
−ud
But x ≠ \y = 0
v 2 − u2
π 3π
ud ⇒x + y = or –
∴ We consider x = 2 2
v 2 − u2 π 3π
\x = or –
1 d2 2 2
Now, f”(x)= > 0 for all x
u x + d2 (x2 + d2 )
2 Thus, required points are

ud π   3π 
∴ f has minimum at x =  ,0  and  − ,0 
2   2 
v 2 − u2
∴ Equation of tangents are
x
Sol 5: Given y = y −0 1 y −0 1
1 + x2 = – and =–
x−π/2 2 x + 3π / 2 2
dy (1 + x2 ).1 − x(2x) 1 − x2 π 3π
⇒ = = ⇒2y = – x + and 2y = – x –
dx (1 + x2 )2 (1 + x2 )2 2 2
dy π 3π
Let = g(x) (i.e. slope of tangent) ⇒x + 2y = and x + 2y = –
dx 2 2
are the required equation of tangents
1 − x2
\g(x) =
(1 + x2 )2
Sol 7: Let P(a cos θ, 2 sin θ) be a point on the ellipse
(1 + x2 )2 .(–2x) – (1 – x2 ).2(1 + x2 ).2x x2 y2
⇒ g’(x)= 4x2 + a2 y2 = 4a2 ie, + =1
2 4
(1 + x ) a2 4
2 2 2 2
−2x(1 + x )[(1 – x ) + 2(1 − x )] −2x(3 − x ) Let A(0, – 2) be the given point. Then,
= =
2 4
(1 + x ) (1 + x2 )3 (AP)2 = a2cos2θ + 4(1 + sin θ)2
For greatest or least values of m we should have d
⇒ (AP)2 = – a2 sin22q + 8 (1 + sin θ) . cos q
g’(x) = 0 ⇒x = 0, x = ± 3 dθ

Now, d
⇒ (AP)2 = [(8 – 2a2) sin θ + 8] cos q

(1 + x2 )3 (6x2 − 6) − (2x3 − 6x).3(1 + x2 )2 .2x
g”(x) = d
(1 + x2 )6 For maximum or minimum, we put (AP)2 = 0

2 1 . 8 4 | Methods of Differentiation and Applications of Derivatives

⇒ [(8 – 2a2) sin θ + 8] cos θ = 0 Any point on the parabola y = x2 is of the form (t, t2).
4 dy  dy 
⇒ cos θ = 0 or sin θ = Now, = 2x ⇒   = 2t
2
a −4 dx  dx  x =t
4 Which is the slope of the tangent. So, the slope of the
( 4 < a2< 8 ⇒ > 1 ⇒ sin θ> 1, which is impossible)
2
a −4 normal to y = x2 at A(t, t2) is – 1/2t.
d2
Now, (AP)2 =–{(8–2a2) sinq+8} sinθ+(8–2a2) . cos2q Therefore, the equation of the normal to y = x2 at A(t,
2
dθ t2) is
π
For θ = , we have  1
2 Y – t2 =  −  (x – t)  …. (i)
 2t 
d2
(AP)2 = – (16 – 2a2) < 0
dθ2 Suppose eq. (1) meets the curve again at B(t1, t12 )
1
π Then, t12 – t2 = – (t – t)
Thus, AP2 ie, AP is maximum when θ = . The point on 2t 1
2
the curve 4x2 + a2y2 = 4a2 that is farthest from the point 1
⇒ (t1 – t) (t1 + t) = – (t – t)
A (0, – 2) is 2t 1
1
⇒ (t1 + t) = –
 π π 2t
 acos ,2sin  = (0, 2)
 2 2  1
⇒ t1 = – t –
2t
Sol 8: Since, equation of normal to y2 = 4ax is Therefore, length of chord,
y = mx – 2 am – am 3
L = AB2 = (t – t1)2 + (t2– t12 )2
Equation of normal for y2 = x is = (t – t1)2 + (t – t1)2(t + t1)2
m 1 3 = (t – t1)2 [1 + (t + t1)2]
y = mx – – m which passes through (c, 0)
2 4
2   1 
2
 1
 1 m2  = t + t +  1 +  t − t −  
\0 = m  c − −  ⇒m=0  2t    2t  
 2 4   

2 3
m2 1 1  1  1   1 
c− ⇒L =  2t +   1 + = 4t2  1 + 
and =c– ⇒m=±2
2t 2 
4 2 2    4t   4t2 
Which gives a normal as x-axis and for other two ∴ On differentiating w.r.t., we get
normals
3 2
1 1 dL  1   1   2 
c – > 0⇒c > = 8t  1 +  + 12t  1 + 2   − 3 
2

2 2 dt  2
4t   4t   4t 
Now, if normals are perpendicular 2
 1    1  3
= 2 1 + 4t  1 + − 
2  
 1  1  4t    4t2  t 
⇒ 2 c −  .  −2 c −  = – 1
 2   2 
  2 2
 1   2  1   1
= 2 1 + 4t −  = 4  1 +   2t − 
1 1 3 2   t 2 t
⇒c – = ⇒c=  4t    4t  
2 4 4
dL
For maxima or minima, we must have =0
2
y y=x
Sol 9: dt
1 1 1
⇒ 2t – = 0 ⇒ t2 = ⇒t = ±
t 2 2
Next,
x’ x 2
d2L  1  1  1  1   1
=8  1 + − 2t −  +4  1 + 2+ 
2 2   3  t 2 
y’ dt  4t   2t     4t   t2 
M a them a ti cs | 21.85

 d2L   1
2 du
⇒ = 0 + 4  1 +  (2+2) > 0 ⇒ = (1 + x)y
 dx
2
 dt  t = ±1/  2
2

1  y dy 
Therefore, L is minimum, when t = ± , point A is  + loge (1 + x) … (ii)
 1 + x dx 
2
 1 1 −1
 ,  and point B is (– 2 , 2) when t = , A is Again, v = sin–1 (sin2 x)
 2 2 2
⇒sin v = sin2 x
 1 1
− ,  , B is (+ 2 , 2) dv
 2 2 ⇒cos v = 2 sin x cos x
dx
1 dv 1
Again, when t = , the equation of AB is ⇒ = (2 sin x cos x)
2 dx cos v

y −2 x+ 2 dv 2sinx cos x 2sinx cos x


= ⇒ = =  … (iii)
1 1 dx 2
1 − sin v 1 − sin4 x
−2 + 2
2 2
∴ From Eq. (i)
 1   1 
⇒ (y – 2)  + 2  = (x + 2 )  − 2 dy  y dy  2sinx cos x
 2   2  = (1 + x)y  + loge (1 + x) +
dx  1 + x dx  1 − sin4 x
⇒– 2y + 4 = 2x+2
dy y(1 + x)y −1 + 2sinx cos x / 1 − sin4 x
⇒ =
⇒ 2 x + 2y – 2 = 0 dx 1 − (1 + x)y log (1 + x)
e

1 y −2 At x = 0,
And when t = – , the equation of AB is
2 1 y = (1 + 0)y + sin–1sin(0) = 1
−2
2
x− 2
= dy 1(1 + 0)1−1 + 2sin0.cos0 / (1 − sin4 0)
 1  ∴ =
− − 2 dx 1 − (1 + 0)1 log (1 + 0)
e
 2
dy
 1  1  ⇒ =1
⇒(y – 2)  − − 2  = (x – 2 )  − 2 dx
 2   2  Again, the slope of the normal is
1
⇒2y – 4 = 2 (x – 2) m=– =–1
dy / dx
⇒ 2 x – 2y + 2 = 0
Hence, the required equation of the normal is

Sol 10: Given, y = (1 + x)y + sin–1(sin2)x) y – 1 = (–1)(x – 0)

Let y = u + v, ie, y + x – 1 = 0

where u = (1 + x)y, v = sin–1(sin2 x)


Sol 11: Let any point P1 on y = x3 be (h, h3)
On differentiating, we get
Then tangent at P1 is
dy du dv
= + … (i) y – h3 = 3h2 (x – h)….(i)
dx dx dx 
Now, u = (1 + x)y It meets y = x3 at P2.

Take logarithm on both sides, we get On putting the value of y in Eq. (i)

Loge u = y loge(1 + x) x3 – h3 = 3h2(x – h)

1 du y dy ⇒ (x – h)(x2 + xh + h2) = 3h2(x – h)


⇒ = + {loge(1 + x)}
u dx 1 + x dx ⇒ x2 + xh + h2 = 3h2
2 1 . 8 6 | Methods of Differentiation and Applications of Derivatives

or x = h dv
=0
⇒ x2 + xh + 2h2 = 0 ⇒ (x – h) (x + 2h) = 0 dx x =5
⇒ x = h or x = – 2h
6x2 − 46kx + 60k2 |x =5 =
0
Therefore, x = – 2h is the point P2,
6k 2 − 23k + 15 =
0
Which implies y = – 8h3
Hence, point P2≡ (– 2h, –8h3) 5
=k 3,=k . Only k = 3 is permissible
6
Again, tangent at P2 is
So, the sides are 45 and 24.
y + 8 h3 = 3(– 2h)2 (x + 2h)
It meets y = x3 at P3 Sol 13: (B)
⇒x + 8h = 12h (x + 2h)
3 3 2
cos(tan−1 y) + y sin(tan−1 y)
P→
⇒x3 – 2hx – 8h2 = 0 cot(sin−1 y) + tan(sin−1 y)
⇒(x + 2h) (x –4h) = 0
1 y2
+
⇒x = 4h ⇒y = 64h 3
1 + y2 1 + y2 1 + y2
Therefore, P3≡ (4h, 64 h ) 3 = = = y 1 − y4
1 − y2 y 1
Similarly, we get P4≡ (– 8h, –8 h ) 3 3 +
y 1 − y2 y 1 − y2
Hence, the abscissae are 2
h, –2h, 4h, –8h, …. which form a GP 1  cos(tan−1 y) + y sin(tan−1 y) 
⇒   + y4
y  cot(sin y) + tan(sin y) 
2  −1 −1
Let D’ = ∆P1P2P3 and D” = ∆P2P3P4

h h3 1
1
y2
( ( ))
= y 2 1 − y 4 + y 4 =1 − y 4 + y 4 =1
1
−2h −8h3 1 Q → cos x + cos y + cos z = 0
2
D' ∆P1P2P3 4h 64h3 1 sin x + sin y + sin z = 0
= =
D" ∆P2P3P4 −2h −8h 1 3
cos x + cos y = – cos z  ... (i)
1 3
4h 64h 1 sin x + sin y = – sin z  ... (ii)
2 3
−8h −512h 1
(1)2+ (2)2

h h3 1 1 + 1 + 2(cos x cos y + sin x sin y) = 1


1 2 + 2 cos (x – y) = 1
−2h −8h3 1
2
4h 64h3 1 2 cos (x – y) = –1
=
h 1h3 1
cos(x − y) =−
1 2
× ( −2) × ( −8) −2h −8h3 1
2 x−y
4h 64h3 1 1 2x−y 1
⇒ 2 cos2   − 1 =− ⇒ 2 cos  =
 2  2  2  2
1
= 2x−y 1 x−y 1
16 ⇒ cos   = ⇒ cos  =
 2  4  2  2
which is the required ratio.
π 
R → cos  − x  cos2x + sinx sin 2x sec x
Sol 12: (A, C) 4 
Let the sides of rectangle be 15k and 8k and side of π 
square be x then (15k – 2x)(8k – 2x)x is volume.
= cos x sin 2x sec x + cos  + x  cos 2x
4 
3 2 2
v = 2(2x − 23kx + 60k x)  π  π 
cos  − x  − cos  + x   cos 2x
 4  4 
M a them a ti cs | 21.87

= (cos x sin 2x − sin x sin 2x)sec x


tan−1 (x 6 ) = φ
2
sin x cos 2x = (cos x – sin x) sin 2x sec x
2 x 6
sin φ =
2 sin x cos 2x = (cos x – sin x) 2 sin x 6x2 + 1
1 1 π x x 6
= = ⇒x ⇒ =
2 cos x + sinx 4 2
1−x 6x2 + 1

π 6x2 + 1 = 6 − 6x2
sec
= x sec
= 2
4
12x2 = 5
S → cot  sin−1 1 − x2 
  5 1 5
=x =
x 12 2 3
cot α =
1 − x2
2017-18 100 &
op kers
Class 12 T
By E ran culty
-JE Fa r
IIT enior emie .
S fP r es
o titut
Ins

MATHEMATICS
FOR JEE MAIN & ADVANCED
SECOND
EDITION

Exhaustive Theory
(Now Revised)

Formula Sheet
9000+ Problems
based on latest JEE pattern

2500 + 1000 (New) Problems


of previous 35 years of
AIEEE (JEE Main) and IIT-JEE (JEE Adv)

5000+Illustrations and Solved Examples


Detailed Solutions
of all problems available

Topic Covered Plancess Concepts


Tips & Tricks, Facts, Notes, Misconceptions,
Indefinite Integration Key Take Aways, Problem Solving Tactics

PlancEssential
Questions recommended for revision
22. INDEFINITE
I N T E G R AT I O N

1. INTRODUCTION

Integration is a reverse process of differentiation. The integral or primitive of a function f(x) with respect to x is a
differential function φ(x) such that the derivative of φ(x) with respect to x is the given function f(x). It is expressed
symbolically as ∫ f(x)dx = φ(x)
d
Thus. ∫ f(x)dx =φ(x) ⇔ φ(x) =f(x) .
dx 
The process of finding the integral of a function is called Integration and the given function is called Integrand.
Now, it is obvious that the operation of integration is the inverse operation of differentiation. Hence the integral of
a function is also named as the anti-derivative of that function.
Further we observe that

d 2
dx
x ( ) 
= 2x 

d 2
dx
x +2( ) 
= 2x  ⇒ ∫ 2xdx =x2 + constant

d 2
dx
x +k( ) =

2x 

So we always add a constant to the integral of function, which is called the constant of Integration. It is generally
denoted by c. Due to the presence of this arbitrary constant such an integral is called an Indefinite Integral.

2. ELEMENTERY INTEGRATION
The following integrals are directly obtained from the derivatives of standard functions.

(a) ∫ 0.dx = c
(b) ∫ 1.dx= x + c
(c) ∫ k.dx =kx + c(k ∈ R)
n xn+1
(d) ∫ x=
dx
n+1
+ c(n ≠ −1)
1
(e) ∫=
x
dx loge x + c
x
(f) ∫ e dx= ex + c
2 2 . 2 | Indefinite Integration

x ax
(g) ∫a=
dx + c ax loga e + c
=
loge a
(h) ∫ sinx dx =
− cos x + c

(i) ∫ cos x=
dx sinx + c

3. BASIC THEOREMS OF INTEGRATION


If f(x), g(x) are two functions of a variable x and k is a constant, then

(a) ∫ k f(x)dx = k ∫ f(x)dx


(b) ∫ f(x) ± g(x) dx = ∫ f(x)dx ± ∫ g(x)dx
d
(c)
dx
( ∫ f(x)dx ) = f(x)
 d 
(d) ∫  dx f(x) dx = f(x) +c

PLANCESS CONCEPTS

The results of integration are very different from differentiation. There is no standard formula for
integration.
Always make sure to write the constant of integration. NEVER assume it as zero from your side.

Vaibhav Gupta (JEE 2009, AIR54)

1 − sinx
Illustration 1: Evaluate: ∫ cos2 x
dx  (JEE MAIN)

Sol: As we know, ∫ f(x) ± g(x) dx = ∫ f(x)dx ± ∫ g(x)dx therefore we can split


1 − sinx 1 sinx
∫ cos x2
dx as ∫ cos2 xdx − ∫ cos2 xdx and then by solving we can get result.

1 − sinx 1 sinx
∫ dx = ∫ dx − ∫ dx = ∫ sec2 x dx − ∫ tanx sec x dx = tanx − sec x + c
2 2
cos x cos x cos2 x

Illustration 2: Evaluate: ∫ 1 + sin2x dx  (JEE MAIN)

1 and sin2x = 2sinx cos x , therefore by using these formulae and solving we will get the
Sol: Here sin2 x + cos2 x =
result.

∫ 1 + sin2x dx = ∫ sin2 x + cos2 x + 2sinx cos x dx= ∫ (sinx + cos x)2 dx

= ∫ (sinx + cosx)dx = ∫ sinx dx + ∫ cos x dx


= −cos x + sin x + c
M a them a ti cs | 22.3

Illustration 3: Evaluate: ∫ sin4 x dx  (JEE MAIN)

Sol: Here as we know, sin2 x = 1 − cos2x , Now by putting this in the above integration and solving we will get the
2
1 2
term ∫ (1 − 2cos2x + cos 2x)dx , After that by using the formula
4
1 + cos 4x
cos2 2x = we can solve the problem given above.
2
2
4  1 − cos2x  1 2
∫ sin x dx = ∫  2  dx =4 ∫ (1 − 2cos2x + cos 2x)dx
1  1 + cos 4x  1 1 sin 4x 
= ∫
4 
 1 − 2cos2x +
2
dx = ∫ (3 − 4 cos2x + cos 4x)dx = 3x − 2sin2x +
 8 8 4 
+C

3
= 4x3 −
Illustration 4: If f '(x) such that f(2)=0,then, find f(x)  (JEE ADVANCED)
x4
3  3 
= 4x3 −
Sol: Here f '(x) f(x)
therefore= ∫  4x
3
− dx hence by splitting this integration and solving we will get
the result. x4 x4 

d 3  3 3  3 3
We have, = 4x3 −
f(x) ⇒ f(x)= ∫  4x − dx= ∫ 4x dx − ∫ dx= 4 ∫ x3dx − 3∫ x −4 dx
dx x4 x4  x4

x3+1 x −4 +1 1
= 4 −3 + C = x4 + + C  …(i)
3+1 −4 + 1 x3
1 1 129
Given f(2) = 24 + +C = 0 ⇒ 0 =16 + + C ⇒ C =−
23 8 8
1 129
Putting the value of C in (i), we get f(x) = x 4 + −
x 3 8

4. METHODS OF INTEGRATION
When the integrand can’t be reduced into some standard form then integration is performed using following
methods

4.1 Integration by Substitution

4.1.1 Integrand is a Function of Another Function

If the integral is of the form ∫ f φ(x) φ' (x)dx , then we put φ(x) =
t so that φ ' (x) dx=dt. Now integral is reduced ∫ f(t) dt.

PLANCESS CONCEPTS

In this method the function is broken into two factors so that one factor can be expressed in terms of
the function whose differential coefficient is the second factor.
In case of objective questions in which direct indefinite integration is asked, function being very
complicated to integrate, then try differentiating the options.
2 2 . 4 | Indefinite Integration

dx
If I = ∫ sin(x − a)cos(x − b) , then I is Equal to
1 sin(x − a)
(a) log +C
sin(a − b) cos(x − b)

1 sin(x − a)
(b) log +C
cos(a − b) cos(x − b)

1 sin(x − a)
(c) log +C
sin(a + b) cos(x − b)

1 sin(x − a)
(d) log +C
cos(a + b) cos(x − b)
Vaibhav KrishnanJEE 2009, AIR 22

Illustration 5: Evaluate: ∫ x tanx2 sec x2dx  (JEE MAIN)

Sol:This problem is based on integration using substitution method. In this we can put x2 =t and therefore 2x
dx=dt and then solving we will get the result.
Let x2 =t
1 1 1 1
⇒ 2x dx=dt ⇒ x dx = dt ∴ ∫ x tanx
2
sec x2dx = ∫ tant sec t=
dt sec t =
+c sec x2 + c
2 2 2 2

4.1.2 Integrand is the Product of Function and its Derivative


If the integral is of the form I= ∫ f ' (x) f(x) dx we put f(x) = t and convert it into a standard integral.
Illustration 6: Evaluate: ∫ tanx sec2 x dx  (JEE MAIN)

Sol: Here sec2 x is a derivatives of tanx hence we can put tan x = t and sec2 x.dx = dt thereafter we can solve the
given problem.
Let tan x =t ⇒ sec2 x.dx = dt
2 t2 tan2 x
=∴ I ∫=
tanx sec x dx ∫ t dt =
2
+c
=
2
+c

4.1.3 Integrand is a Function of the Form f(ax+b)

Here we put ax+b =t and convert it into a standard integral. Now if,
1
∫ f(x)dx = φ(x), then ∫ f(ax + b)dx =aφ(ax + b)

Illustration 7: Evaluate: ∫ cos3x cos5x dx  (JEE MAIN)

Sol: By multiplying and dividing by 2 in the given integration and using the formula
= cos ( A + B ) + cos ( A − B ) we can solve it.
2cos A.cosB
1 1 1 1 
I= ∫ cos3x cos5x dx = 2 ∫ ( cos8x + cos2x ) dx=  sin8x + sin2x  + c
2 8 2 
M a them a ti cs | 22.5

x dx
Illustration 8: Evaluate: I = ∫ x 4 + x2 + 1  (JEE ADVANCED)

1 dt 1 dt
Sol: Here by putting x2 = t ⇒ dt = 2xdx we will get the term ∫ = ∫ and then by
2 t + t +1 2
( )
2 2
( t + (1 / 2)) +
2
1 3 3 /2
putting t += tan θ , we can solve it.
2 2
1 dt 1 dt
Let x2 = t ⇒ dt = 2xdx ∴ I=
2 ∫ t2 + t + 1 2 ∫
=
( )
2
( t + (1 / 2)) +
2
3 /2

  1 
1 1 −1
 f (x)  1

1  1 
−1 
t + 
2  + c
∫ 2
= dx
a
tan  =  ×
 a  f ' ( x )
+ c ∴ I
2 3
tan
 3 
f ( x )  + a2
    
 2  2 

1  −1  2t + 1   1  −1  2x2 + 1  
=tan   + c tan 

 + c
3   3   3   3  

1 3 3
Now put t += tan θ ⇒
= dt sec2 θ dθ
2 2 2

1 ( 3 / 2)sec2 θdθ 1 1 1  2t + 1  1  2x2 + 1 


tan−1  tan−1 
2 ∫ (3 / 4) tan2 θ + 1 ∫ dθ=
∴ = θ + c=  + c= +c
( ) 3 3 3  3  3 
 3 

Standard integration results

f ' (x)
∫ f(x) dx loge f(x) + c
(a) =
n+1
f(x)
f (x)dx 
n '
(b) ∫ f(x)= n+1
+ c (provided n≠-1)

f ' (x)
(c) ∫ =
f(x)
dx 2 f(x) + c

sec2 x
Illustration 9: Evaluate: ∫ dx  (JEE MAIN)
tanx
Sol: Here simply substituting t=tan x ⇒ dt = sec2 x dx we can solve it.
Let t=tan x ⇒ dt = sec2 x dx

1
dt
∴ I= ∫ =
t
2 t2 c 2 tanx + c
+=

4.1.4 Integral of the Form

dx b b
∫ asinx + b cosx then substitute a=r cos θ and b=r sin θ , tan θ= ⇒ θ= tan−1   ,we get
a a

I= ∫ =
dx 1 1 x +θ (
logtan (x / 2) + (1 / 2)tan−1 (b / a) )
r sin(x + θ) r ∫
cosec(x
= + θ)dx logtan  +c= +c
r  2  a2 + b2
2 2 . 6 | Indefinite Integration

PLANCESS CONCEPTS
m
∫ sin x cosn x dx , where m, n ϵ N
⇒ If m is odd put cos x=t
If n is odd put sin x = t
If both m and n are odd, put sin x=t if m ≥ n and cos x=t otherwise.
If both m and n are even, use power reducing formulae
1 − cos 2x 1 + cos 2x
= sin2 x = or cos2 x
2 2
If m+n is a negative even integer, put tan x=t
Shrikant Nagori (JEE 2009, AIR 30)

1
Illustration 10: Evaluate: ∫ dx  (JEE ADVANCED)
sinx + cos x
dx
Sol: As we know, if integration is in the form of ∫ asinx + b cosx then we can put

1 x+θ
a=r cos θ and b=r sin θ hence the integration will be log tan  +c .
r  2 
Here a=1 & b=1
1 1 x 1 −1  1 x π
So ∫ sinx + cos xdx =
1+1
logtan  + tan
2 2
= 1 + c
 2
logtan  +  + c
2 8

4.1.5 Standard Substitutions


The following standard substitutions will be useful

Integrand form Substitutions


1
a2 − x2 or x = a sin θ or x = a cos θ
a2 − x2

1
x2 + a2 or x = a tan θ or x = a cot θ or x=a sinh θ
   x2 + a2   

1
x2 − a2 or x = a sec θ or x = acosec θ
x2 − a2

x a+x 1 x=a tan2 θ


or or x(a + x) or
a+ x x x(a + x)

x a−x 1 x = a sin2 θ or x= a cos2 θ


or or x(a − x) or
a−x x x(a − x)
M a them a ti cs | 22.7

x x−a 1 x=a sec2 θ or x=a cosec2 θ


or or x(x − a) or
x−a x x(x − a)

a−x a+ x x = a cos2 θ
or
a+x a−x

x−α x = α cos2 θ + β sin2 θ


or (x − α )(β − x) (β > α )
β−x

Some Standard Integrals

(a) ∫ tanxdx =logsec x + c =− logcos x + c


(b) ∫ cot xdx =logsinx + c =− logcosec x + c
π x
(c) ∫ sec xdx=
log(sec x + tanx) + c =− log(sec x − tanx) + c =logtan  +  + c
 4 2
(d) ∫ cosecxdx = x
− log(cosecx + cot= x) + c log(cosecx − cotx)
= + c log tan   + c
2
(e) ∫ sec x tanxdx
= sec x + c

(f) ∫ cosec x cot x dx =


− cosec x + c

2
(g) ∫ sec xdx tanx + c
=

2
(h) ∫ cosec xdx =
− cot x + c
x
(i) ∫ logxdx
= xlog  =+ c x (logx + 1 ) + c
e

PLANCESS CONCEPTS

 1 1 1 
If the integral is of the form ∫ R  x p , x q , x r ...... dx , where R is a rational function then,
 
 
Let a = lcm of (p,q,r,…….) and put x = ta
Nitish Jhawar (JEE 2009, AIR 7)

dx
dx 2 2
Illustration 11: Prove that: ∫∫ 22 22
n(x
 n(x++ x x2−−aa2) +
= = log ) +CC  (JEE ADVANCED)
x x −−aa
Sol: By putting x = a sec θ ⇒ dx = a sec θ tan θdθ , we can solve the problem given above.

dx asec θ tan θdθ


Let x = a sec θ ⇒ dx = a sec θ tan θdθ ⇒ ∫ 2 2
= ∫ atan θ
= ∫ sec θdθ = log
 n(sec θ + tan θ) + C
x −a
x2 + a2 x
= log
 n(  n(x + x2 + a2 ) + C'
+ ) + C = log
a a
2 2 . 8 | Indefinite Integration

Illustration 12: Evaluate: ∫ cos2x.cos 4x.cos6xdx  (JEE MAIN)

= cos ( A + B ) + cos ( A − B )
Sol: By multiplying and dividing by 2 in the given integration and then by using 2cos A.cosB
we can solve it.
1 1
Let I = ∫ cos2x.cos 4x.cos6xdx = ∫ (2cos2x.cos 4x)cos6 xdx = ∫ (cos6x + cos2x)cos6 xdx
2 2
B cos ( A + B ) + cos ( A − B ) 
∴ 2 cos A cos=

=
1
2 ∫ (
cos2 6x + cos 6x − cos2x dx
=
1
4∫
) (
2cos2 6x + 2cos 6x − cos2x dx )
1 1 1 1 1
=  ∫ (1 + cos 12 x ) + ( cos8x + cos 4a) dx  = ∫ dx + ∫ cos12xdx + ∫ cos8xdx + ∫ cos 4xdx
4  4 4 4 4

 sin  f ( x )  
 ∫ cos
= f ( x ) dx +C
 f '(x) 
 

1 sin 12 x sin8x sin 4x 


I = x + + + +C
4 12 8 4 

cos2x − cos2α
Illustration 13: Evaluate: ∫ cos x − cos α
dx  (JEE ADVANCED)

Sol: Here in this problem by using the formulae

C +D C − D , sin 2A = 2 sin A cos A and 2 cos C cos D = cos (C+D)+cos (C-D)


cosC − cosD =
−2sin .sin
2 2

We can solve the problem above step by step.


We have,
cos2x − cos2α −2sin(x + α )sin(x − α ) sin(x + α )sin(x − α )
∫ cos x − cos α
dx = ∫
−2sin ( (x + α ) / 2 ) sin ( (x − α ) / 2 )
dx = ∫ sin ((x + α) / 2 ) sin ((x − α) / 2)dx

2sin ( (x + α ) / 2 ) cos ( (x + α ) / 2 ) .2sin ( (x − α ) / 2 ) cos ( (x − α ) / 2 )


= ∫ sin ( (x + α ) / 2 ) sin ( (x − α ) / 2 )
dx (sin 2A = 2 sin A cos A)

x+α x−α
= 4 ∫ cos   cos  dx  [2 cos C cos D = cos (C+D)+cos (C-D)]
 2   2 
= 2∫ (cos x + cos α=
)dx 2∫ cos xdx + 2cos α=
∫ dx 2sinx + 2x cos α + C
sin8 x − cos8 x
Illustration 14: Evaluate: ∫ 1 − 2sin2 x.cos2 x dx  (JEE ADVANCED)

Sol: Here by using the formula a2 − b2 = ( a + b )( a − b ) and putting (sin2 x + cos2 x)2 in place of 1 in the denominator,
we can reduce the above integration and then using cos2x = cos2 x − sin2 x we can solve it.

sin8 x − cos8 x (sin4 x + cos4 x)(sin4 x − cos4 x)


We have, ∫ 1 − 2sin2 x.cos2 x dx = ∫
(sin2 x + cos2 x)2 − 2sin2 x.cos2 x
dx
M a them a ti cs | 22.9

(sin4 x + cos4 x)(sin2 x + cos2 x)(sin2 x − cos2 x)


= ∫ − ∫ cos2xdx  cos2x =
dx = ∫ 1.(sin2 x − cos2 x)dx = cos2 x − sin2 x 
(sin4 x + cos4 x)  

sin 2x
=
− +C
2

4.2 Integration by Parts


If u and v are two functions of x, then

∫ (u.v)dx u
= ( ∫ v dx ) − ∫  du 
 ( v dx ) dx
dx  ∫
This is also known as uv rule of integration. This method of integrating is called integration by parts.

PLANCESS CONCEPTS

•• From the first letter of the words inverse circular, logarithmic, Algebraic, Trigonometric, Exponential
functions, we get a word ILATE. Therefore the preference of selecting the u function will be according
to the order ILATE.
•• In some problems we have to give preference to logarithmic function over inverse trigonometric
functions. Hence sometimes the word LIATE is used for reference.
•• For the integration of Logarithmic or Inverse trigonometric functions alone, take unity (1) as the v
function.

Shivam Agarwal (JEE 2009, AIR 27)

Illustration 15: Evaluate: ∫ (1 + x)logxdx  (JEE MAIN)

Sol: Here we can integrate the given problem by using Integration by parts i.e.

∫ (u.v)dx u
= ( ∫ v dx ) − ∫  du 
 ( v dx ) dx
dx  ∫

Here u = logx and v = (1 + x) .

Let I= ∫ (1 + x)logxdx

Integrating by parts, taking log x as 1st function, (by LIATE rule) we get

d   x2  1 x2   x2   x
I= logx ∫ (1 + x )dx − ∫  (logx). ∫ (1 + x)dx dx = logx  x +  − ∫ .  x +  dx =  x +  logx − ∫  1 + dx
 dx   2  x  2   2   2 
     

 x2   x2 
=  x +  logx −  x +  + C
 2   4 
 
3
Illustration 16: Evaluate: ∫ sec xdx  (JEE ADVANCED)

Sol: Here we can solve by integrating by parts, taking sec x as the first function.
3
I= ∫ sec xdx = ∫ sec x.sec2 xdx Let
= u sec
= x & v sec2 x

=I sec x tan x − ∫ ( sec x tan x ). tan x dx = sec x tanx − ∫ sec x tan2 xdx = sec x tanx − ∫ sec x(sec2 x − 1)dx
2 2 . 1 0 | Indefinite Integration

I = sec x tanx − ∫ sec3 xdx + ∫ sec xdx ⇒=I sec x + tan x − I + ∫ sec x dx
1
⇒ 2 I sec x. tan x + log ( sec x + tan x=
= )+C ⇒ I sec x tanx + log(sec x tanx) + C
2

Illustration 17: Evaluate : ∫ (sin−1 x)2 dx  (JEE ADVANCED)

(=
sin x ) & v
2
−1
Sol: We can write the given integration as ∫ (sin x)2 .1dx and then
= taking u −1
1
solving by integration by parts.
 
d 
I= (sin−1 x)2 .x − ∫  (sin−1 x)2 .x  dx = (sin−1 x)2 .x − 2 sin−1 x .
 dx  
1
.x  dx
1 − x2 
( )
dx
Now, putting sin−1 x = t ⇒ x = sin t so that = dt
1 − x2
{
⇒ I = x(sin−1 x)2 − 2∫ t.sintdt = x(sin−1 x)2 − 2 −t cos t + ∫ cos tdt (again Integrating by parts) }
= x(sin−1 x)2 − 2 {−t cos t + sint} + C = x(sin−1 x)2 + 2t cos t − 2sint + C = x(sin−1 x)2 + 2sin−1 x. 1 − x2 − 2x + C

sin−1 x − cos−1 x
Illustration 18: Evaluate: ∫ sin−1 x + cos−1 x
dx  (JEE ADVANCED)

π
Sol: By using the formula sin−1 x + cos−1 x = , we can solve the above problem.
2
sin−1 x − cos−1 x (
sin−1 x − ( π / 2) − sin−1 x )dx  −1 π
x + cos−1 x =
Let I= ∫ sin−1 x + cos −1
x
dx = ∫ ( π / 2)
 sin


2

2  −1 π 4
∫ sin−1 xdx − ∫ 1dx
π∫
=  2sin x −=dx
π  2
4
= ∫ sin−1 xdx − x  … (i)
π
Putting sin−1 x = θ ⇒ x= sin2 θ so that dx = 2 sin θ . cos θ d θ = sin 2 θ d θ .
−1
∴ ∫ sin xdx = ∫ θ.sin2θdθ Let u = sin 2 θ , then integing by parts we get
θ& V=

=
−θ
2
( 1
)
1 − 2sin2 θ + 2 sin θ.cos θ
4
cos2θ 1 θ 1 1 1
= −θ. + ∫ cos2θdθ = − cos2θ + sin2θ = − θ(1 − 2sin2 θ) + sin θ 1 − sin2 θ
2 2 2 4 2 2
1 1
= − (sin−1 x )(1 − 2 x) + x. 1 − x + C  … (ii)
2 2

From (i) and (ii), we get

I=
4 1
− (1 − 2x)sin
π 2
−1
x+
1
2

x − x2  − x + C =

2
π { x−x 2
}
− (1 − 2x)sin−1 x − x + C

4.2.1 Integration by Cancellation

 2 1 1
Illustration 19: Evaluate : ∫  3x tan − x sec2 dx  (JEE MAIN)
x x
M a them a ti cs | 22.11

 1 1 1 1
Sol: Let ∫  3x2 tan − x sec2 dx = ∫ 3x2 tan dx − ∫ x sec2 dx and then by using the integration by parts formula
 x x x x

∫ (u.v)dx u
i.e. = ( ∫ v dx ) − ∫  du 
 ( v dx ) dx we can solve the problem above.
dx  ∫
 1 1 1 1 1  1  1  1 1
∫  3x
2
tan − x sec2 dx = ∫ 3x2 tan dx − ∫ x sec2 dx = tan x3 − ∫  sec2  − x3dx − ∫ x sec2 dx = x3 tan + c
x x x x x  x  x 
2 x x

4.2.2 Integration of the Form:


If the integral is of the form ∫ ex f(x) + f ' (x) dx , then use the formula;
 
x
∫e ex f(x) + c
f(x) + f '(x) dx =

x
Illustration 20: Evaluate: ∫ e (logx + 1 / x)dx  (JEE MAIN)

Sol: Solution of this problem is based on the method mentioned above, here f(x) = log x and f’(x)

x  1
==
1 / x . ∫e log x + dx
 x

 x ex  Here, f(x) = logx 


I= ∫
 e logx +  dx = ex logx + c  ' 
x  &f (x) = 1/ x 
 

∫ xf (x) + f(x)dx ∫ xf (x) + f(x)dx =xf(x) + c


' '
If the integral is of the form then use the formula;

Illustration 21: Evaluate : ∫ (x sec2 x + tanx)dx  (JEE MAIN)

Sol: Similar to the problem above.


Here I = ∫ (x sec2 x + tanx)dx = ∫ xf '(x) + f(x)dx [where f(x) = tanx ] = x . tan x + c

4.2.3 Special Integrals

ax eax
∫e sinbxdx
=
a2 + b2
(asinbx − bcosbx) + Cc

ax eax
∫ e cosbxdx
=
a2 + b2
(bsinbx + acosbx) + c

ax eax
∫ e cosbxdx = a2 + b2
(b sin bx+a cos bx)+c

sin−1 x
Illustration 22: Evaluate : ∫e dx  (JEE MAIN)

Sol: By putting sin−1 x = t ⇒ x=sin t ⇒ dx = cos t dt and then integrating by parts we can solve the given problem.

sin−1 x
I= ∫e dx

Let sin−1 x = t ⇒ x=sin t ⇒ dx = cos t dt


−1 x
et
t esin
⇒ I = ∫ e cos=
tdt (sint + cos t)=
+c (x + 1 − x2 ) + c
2 2
2 2 . 1 2 | Indefinite Integration

4.3 Integration of Rational Functions

4.3.1 When the Denominator can be Factorized (Using Partial Fraction)


f(x)
Let the integrand be of the form , where both f(x) and g(x) are polynomials. If degree of f(x) is greater than
g(x)
degree of g(x), then first divide f(x) by g(x) till the degree of the remainder becomes less than the degree of g(x).
Let Q(x) be the quotient and R(x), the remainder then
f(x) R(x)
= Q(x) +
g(x) g(x)

Now in R(x)/g(x), factorize g(x) and then write partial fractions in the following manner:

(a) For every non-repeated linear factor in the denominator. Write


1 A B
= +
(x − a)(x − b) x − a x − b

(b) For every repeated linear factor in the denominator. Write


1 A B C D
= + + +
(x − a) (x − b) (x − a) (x − a)
3 2
(x − a)3 (x − b)

(c) For every non-repeated quadratic factor in the denominator. Write


1 Ax + B C
= +
(a x + bx + c)(x − d) a x + bx + c x − d
2 2

(d) For every repeated quadratic factor in the denominator. Write


1 Ax + B Cx + D E
= + +
2 2 2
(a x + bx + c) (x − d) (a x + bx + c)2 2
a x + bx + c x − d

PLANCESS CONCEPTS

Consider f(x) as the function we need to factorize


1. For non- repeated linear factor in the denominator.
1 A B
Let f(x)= = +
(x − a)(x − b) (x − a) (x − b)

To obtain the value of A remove (x−a) from f(x) and find f(a).
Similarly, to obtain value of B, remove (x-b) from f(x) and find f(b).
2. For repeated linear factor in the denominator.

1 A B C D
Let f(x) = = + + +
(x − a)3 (x − b) (x − a) (x − a)2 (x − a)3 (x − b)

To obtain value of D remove (x−b) from f(x) and find f(b).


To obtain value of c remove (x−a)3 from f(x) and find f(a).
Now that we have reduced the number of unknowns from 4 to 2, we can find A and B easily by equating.
M a them a ti cs | 22.13

x 4 + x3 + 2x2 − x + 4
Now let’s try this method for
x(x2 + 2)(x2 + 1)3
Partial fraction will be of the form

x 4 + x3 + 2x2 − x + 4 A Bx + C Dx + E Fx + G Hx + I
=+ + + +
2 2
x(x + 2)(x + 1) 3 x (x2 + 2) (x2 + 1) (x2 + 1)2 (x2 + 1)3

Now remove x and put x=0, we get A=2


Now remove (x2 + 1)3 and put x2 =-1 i.e. x = i (you can also substitute x = −i ).
We get Hi+I = –3i –2. Hence H = –3 and I = –2.

Now remove (x2 + 2) and put x= 2i . We get B ( 2i ) +C=2 2i +3. Hence B = 2 and C = 3
Now the number of unknowns have reduced from 9 to 4 and the remaining unknowns can be solved
easily.
This method very useful instead of solving for all the unknowns at the same time.
Also remember that substituting an imaginary number for x is not discussed anywhere in NCERT. So, use
this method only for competitive exams.
Ravi Vooda (JEE 2009, AIR 71)

x
Illustration 23: Evaluate : ∫ dx  (JEE MAIN)
2
x −x−2

1
Sol: Here the given integration is in the form of , hence by using partial fractions we can split it as
(x − a)(x − b)
A B
+ and then by solving we will get the required result.
(x − a) (x − b)
x 1 2 1  1 1
Here I = ∫ (x − 2)(x + 1)dx = ∫ 3  x − 2 + x + 1  dx = 2log(x − 2) + log(x + 1)=
+c log (x − 2)2 (x + 1) + c
3 3  

xdx
Illustration 24: Evaluate : ∫  (JEE ADVANCED)
3x − 18x2 + 11
4

Sol: Here simply by putting t= x2 ⇒ dt = 2x dx and then by using partial fractions we can solve the given problem.
1
dt
xdx 2
I= ∫ dx = ∫ (Put t= x2 ⇒ dt = 2x dx)
3x 4 − 18x2 + 11 3t2 − 18t + 11

1 dt 1 dt 1 dt
6 ∫ t2 − 6t + (11 / 3) 6 ∫ (t − 3)2 − (16 / 3) 6 ∫
= =
( )
2
(t − 3)2 − 4 / 3

1 1 (t − 3) − (4 / 3) 3 3t − 3 3 − 4 3 3x2 − 3 3 − 4
log = +C log
= +C log +C
6 2 × (4 / 3) (t − 3) + (4 / 3) 48 3t − 3 3 + 4 48 3x2 − 3 3 + 4
2 2 . 1 4 | Indefinite Integration

4.3.2 When the Denominator cannot be Factorized


In this case the integral may be in the form
dx (px + q)
(i) ∫ ax2 + bx + c (ii) ∫ ax2 + bx + cdx

Method:
(i) Here taking the coefficient of x2 common from the denominator , write
b2 − 4ac
x2 + (b / a) x + c/ a =+
(x b/ 2a)2 −
4a2
Now the integrand obtained can be evaluated easily by using standard formulae.
d 
(ii) Here suppose that px + q = A  (ax2 + bx + c) + B = A(2ax+b)+B  …..(i)
 dx 
Now comparing coefficient of x and constant terms.

We get A=p/2a, B=q-(pb/2a)

p 2ax + b  pb  dx
=∴I ∫
2a ax bx + c
2
dx +  q −

∫ 2
2a  ax + bx + c

Now we can integrate it easily.

4.3.3 Integrand Containing Only Even Powers of x


To find integral of such functions, first we divide numerator and denominator by x2 , then express the numerator
as d(x±1/x) and the denominator as a function of (x±1/x). The following examples illustrate it.

PLANCESS CONCEPTS

∫ R(sinx,cos x)dx where R is a rational function (universal substitution tan(x/2)=t)


Special cases:
(a) If R (−sin x, cos x) = −R(sinx, cos x)
Put cos x=t
(b) If R (sin x, cos x) = −R(sinx, cos x)
Put sin x=t
(c) If R (−sin x, −cos x) = R(sinx, cos x)
Put tan x=t
Akshat Kharaya (JEE 2009,AIR 235)

x2 + 1
Illustration 25: Evaluate: ∫ x 4 + 1dx  (JEE ADVANCED)
1 + (1 / x2 ) 1 + (1 / x2 )
Sol: Here dividing the numerator and denominator by x2 , we get ∫ x2 + (1 / x2 )dx = ∫  dx and
{ } 
2

then by putting x-1/x=t ⇒[1+1/ x2 ]dx = dt, we can solve it.  x − (1 / x ) + 2



M a them a ti cs | 22.15

1 + (1 / x2 ) 1 + (1 / x2 )
I= ∫ x2 + (1 / x2 )dx = ∫  dx
{ } 
2
 x − (1 / x ) + 2

Now taking x-1/x=t ⇒[1+1/ x2 ]dx = dt, we get

dt 1  t  1  2
−1 x − 1

I= ∫
= tan−1   + c = tan  +c
2
t +2 2 2  2x 
 2  

4.4 Integration of Irrational Functions


If any one term in numerator or denominator is irrational then it is made rational by a suitable substitution. Also
if the integral is of the form
dx
∫ or ∫ ax2 + bx + c dx
2
ax + bx + c

Then we integrate it by expressing ax2 + bx + c= (x + α )2 + β2


px + q
Also for integrals of the form ∫ dx or ∫ (px + q) ax2bx + c dx
2
ax + bx + c
First we express px+q in the form
d
px+q = A [ (ax2bx + c) ]+B and then proceed as usual with standard form.
dx
ex
Illustration 26: Evaluate : ∫ dx  (JEE MAIN)
5 − 4ex − e2x
x
Sol: Simply by putting e = t , then ex dx = dt, we can solve the given problem.
Put ex = t , then ex dx = dt
ex dt dt dt dt
∴ ∫ dx = ∫ = ∫ = ∫ = ∫
9 − ( t + 2)
x 2x 2 2 2 2
5 − 4e − e 5 − 4t − t 5 − (t + 4t) 5 − (t + 4t + 4) + 4

dt t+2  ex + 2 
= ∫ = sin−1   + C = sin−1  +C
 3   3 
(3)2 − (t + 2)2  

1
Illustration 27: Evaluate : ∫ (x − a)(x − b)
dx  (JEE ADVANCED)

2
a+b
Sol: Here first expand (x − a)(x − b) and then adding and subtracting by   , we can reduce the above
 2 
a+b
integration. After that by putting x −  = u , we can solve the given problem.
 2 
1 1 dx
Let, I = ∫ (x − a)(x − b)
dx = ∫ dx = ∫
x2 − (a + b) x + ab x2 − (a + b) x + ( (a + b) / 2 ) − ( (a + b) / 2 ) + ab
2 2

dx dx
= ∫ = ∫
( x− ((a + b) / 2) ) ( ) ( x− ((a + b) / 2) )
2 2
− ( a − b ) / 4 
2
−  (a2 + b2 + 2ab) / 4 − ab 
   
dx
= ∫  … (i)
( x− ((a + b) / 2) ) − ((a − b) / 2)
2 2
2 2 . 1 6 | Indefinite Integration

a+b
On putting x −  = u so that dx = du in (i), we get
 2 
du  1  2
I= ∫ ∫ dx = log x + x2 − a2  = log u + u2 −  a − b  + Cc
 2
 x −a
2   2 
u2 − ( (a − b) / 2 )
2

  a + b 
Putting u=  x −    , we get
  2 

2 2
  a + b    a + b   a − b  a+b
I = log  x − 
2
 + x − 
2
 − 
2
 + Cc = log x −
2
+ ( x − a)( x − b ) + Cc
       

5. STANDARD INTEGRALS

1 1 x
(a) ∫= dx tan−1   + c
x2 + a2 a a

1 1 x−a
∫ x2 − a2 dx 2a log x + a + c
(b) =

1 1 a+ x
∫ a2 − x2 dx 2a log a − x + c
(c) =

1 x x
(d) ∫ dx =sin−1   + c =− cos−1   + c
a −x2 2 a
  a

1 x
(e) ∫ dx sinh−1   +
= = c log  x + x2 + a2  + c
2
x +a2 a
   

1 x
(f) ∫ dx cosh−1   +
= = c log  x + x2 − a2  + c
x −a2 2 a  

x 2 a2 x
(g) ∫ a2 − x2 dx
= a − x2 + sin−1 + c (Substitute
= x acos θ or=x asin θ and proceed)
2 2 a

2 2 2 2 x x 2 2 2 2 a2a2
(h) ∫ ∫ x x+ a+ adx=dx= n nx +x + x2x+2 a+2a2+ c+ c (Substitute
x x+ a+ a+ + log = x atan θ or
= x acot θ and proceed)
22 22

22 22 xx 22 22 a2a2 22 22
(i) ∫ ∫ x x − −a adx=dx= 2 2 x x − −a a − −2 2log
n nx x+ + x x − −a a + +c c (Substitute
= x asec θ=
or x acosec θ and proceed)

1 1 x
( j) ∫ = dx sec−1 + c (Valid for x > a > 0)
x x2 − a2 a a

ax eax eax   b  
(k) ∫e sinbx
= dx
2 2
(asinbx − bcosbx) + c = sin bx − tan−1    + c
a +b a2 + b2   a  
M a them a ti cs | 22.17

ax eax eax  b
(l) ∫e cosbx
= dx (acosbx + bsinbx) + c = cos bx − tan−1  + c
a +b2 2 2
a +b 2
 a

Integration of irrational algebraic functions:

dx
Type 1: (a) ∫ (x − α) (Put
= : x acos2 q + b sin2 q )
(x − α )(β − x)

dx
(b) ∫ (x − α) (Put
= : x asec2 q − b tan2 q )
(x − β)

dx
Type 2: ∫ (a x + b) (Put: px + q =t2 )
px + q

dx 1
Type 3: ∫ (Put: ax + b = )
t
(ax + b) px2 + qx + r

dx
Type 4: ∫ (ax2bx + c) (Put: px + q =t2. )
px + q

dx
Type 5: ∫
(ax + bx + c) px2 + qx + r
2

Case I: When (ax2bx + c) breaks up into two linear factors, e.g.

dx  A B  1 dx dx
I= ∫ dx then = ∫  +  dx = A ∫ + B∫
( 2
x − x −2 x + x +1 ) 2  x − 2 x + 1 2
 x + x +1 2
x −2 x + x +1

( 
(
x + 1 ) x2 + x + 1

)
1 1
Put x − 2 = Put x + 1 =
t t
1
Case II: If ax2 + bx + c is a perfect square say (lx + m)2 , then put lx + m =
t
Case III: If b = 2, q = 0

dx 1
e.g. ∫ then, put x =
t
or trigonometric substitutions are also helpful.
(ax2 + b) px2 + r
dx
Integral of the form ∫P Q
, where P, Q are linear or quadratic functions of x.

Integral Substitutions

1
∫ dx cx + d =z2
( ax + b ) cx + d

dx
∫ px + q =z2
( 2
ax + bx + c ) px + q

dx 1
∫ px + q =
z
(px + q) ax2 + bx + c

dx 1
∫ x=
(ax 2
+b ) cx2 + d z
2 2 . 1 8 | Indefinite Integration

dx
∫ ax + b =
1/t
( ax + b )
m
ax2 + bx + c

dx
Illustration 28: Evaluate : ∫ (x + 1)  (JEE MAIN)
( x − 2)
Sol: Simply by putting x − 2 =t2 , ∴dx =
2t dt we can solve the given problem by using the appropriate formula.
dx
∫ (x + 1)
( x − 2)
Put x − 2 =t2
∴dx =
2t dt

2t dt dt 2  t  2  x −2
∴I= ∫ (t2 + 3)t = 2∫ = tan−1  +c = tan−1    + c (∵=t (x − 2) )
t2 + ( 3)2 3  3 3   3  
 

dx
Illustration 29: Evaluate : ∫ (x2 − 4) x
 (JEE MAIN)

Sol: Here first put x = t2 therefore dx = 2t dt and then using partial fractions we reduce the given integration in
standard form. After that by solving we will get the result.

dx
Let I = ∫ (x2 − 4) x
2t dt
Put x = t2 ∴ dx = 2t dt then I = ∫ (t 4 − 4)t dt =2 ∫
(t + 2)(t2 − 2)
2

1 1 A B
Put t2 = z ∴ = = +
2
(t + 2)(t − 2) 2 (z + 2)(z − 2) z+2 z−2

1 1 1 1 1
A= − and B = ⇒ = +
4 4 2 2 2 2
(t + 2)(t − 2) 4(t + 2) 4(t − 2)

1 1 dt 1 dt 1  t  1 t− 2
∴ I = 2∫ − ∫
= + ∫ = − tan−1  + log +c
2
(t + 2)(t − 2) 22 t +2 2 t −2
2 2
2 2  2 4 2 t+ 2

1 x 1 x− 2
= − tan−1   + log + Cc (∵ t = x )
2 2 2 4 2 x+ 2

6. SPECIAL TRIGONOMETRIC FUNCTIONS


Here we shall study the methods for evaluation of the following types of integrals.

Type 1
dx dx dx dx
(i) ∫ a + bsin2 x (ii) ∫ a + b cos2 x (iii) ∫ acos2 x + bsinx cos x + c sin2 x (iv)
2
∫ (asinx + bcos x)
Method: Divide the numerator and denominator by cos2x in all such types of integrals and then put tan x=t
M a them a ti cs | 22.19

dx
Illustration 30: Evaluate : ∫ 1 + 3sin2 x  (JEE MAIN)

Sol: Here dividing the numerator and denominator by cos2x we can solve it.

sec2 xdx sec2 x dx 1 −1


I= ∫ sec2 x + 3tan2 x ∫ 1 + 4 tan2 x 2 tan (2 tanx) + c
==

Type 2
dx dx dx dx
(i) ∫ a + b cos x (ii) ∫ a + b sinx (iii) ∫ acos x + b sinx (iv) ∫ asinx + b cos x + c
Method: In such types of integrals we use the following substitutions

2 tan ( x / 2 ) 2t 1 − tan2 ( x / 2 ) 1 − t2 2dt


=sinx = = , cos x = = ; dx
1 + tan ( x / 2 ) 1 + t
2 2
1 + tan ( x / 2 ) 1 + t
2 2
1 + t2
and integrate another method for the evaluation of the integral.

dx
Illustration 31: Evaluate: ∫ 5 + 4 cos xdx  (JEE MAIN)

1 − tan2 ( x / 2 )
Sol: Here by putting cos x = and then by taking tan (x/2) = t we can solve the given problem
1 + tan2 ( x / 2 )
dx sec2 (x / 2) dt
I= ∫ 5 + 4 (1 − tan2 (x / 2)) / (1 + tan2 (x / 2)) = ∫ 9 + tan2 (x / 2)dx = 2∫ where tan (x/2) = t
3 + t2
2
 

1  t  1  tan x / 2  
2  tan−1    + C 2  tan−1 
=  + C
3  3  3  3 

Type 3
p sinx + qcos x p sinx qcos x
(i) ∫ asinx + b cos x dx (ii) ∫ asinx + b cos xdx (iii) ∫ asinx + b cos xdx
For their integration, we first express numerator as follows-
Numerator = A (denominator) + B (derivative of denominator)
Then integral = Ax + B log (denominator) + C

6 + 3sinx + 14 cos x
Illustration 32: Evaluate : ∫ 3 + 4 sinx + 5cos x
dx  (JEE ADVANCED)

Sol: By using partial fractions, we can reduce the given integration to the standard form.
6 + 3sinx + 14 cos x
∫ 3 + 4 sinx + 5cos x
dx

⇒ 6 + 3sinx + 14 cos x = A ( 3 + 4 sinx + 5cos x ) + B ( 4 cos x − 5sinx ) + c

Solving R.H.S. & comparing both sides, we get 4A – 5B = 3 5A + 4B = 14

A(3 + 4 sinx + 5cos x) + B(4 cosx − 5sinx) + c


Also, 3A+C=6 ∴ ∫ 3 + 4 sinx + 5cos x
2 2 . 2 0 | Indefinite Integration

C dx
 n(3 + 4 sinx + 5cos x) + ∫
⇒ AxAx++log
3
+ 4 sinx + 5cosx

this is of type 2

2sin2φ − cos φ
Illustration 33: Evaluate : ∫ 6 − cos2 φ − 4sin φdφ  (JEE ADVANCED)

2(sin2φ − 4 cos φ) + 7 cos φ


Sol: Here we can write the given integration as ∫ 6 − cos2 φ − 4 sin φ
dφ and as we know 2(sin2φ − 4 cos φ) is

the derivative of 6 − cos2 φ − 4 sin φ hence by putting 6 − cos2 φ − 4 sin φ = t , we can solve the given problem.

2sin2φ − cos φ d
I= ∫ 6 − cos2 φ − 4sin φdφ = (6 − cos2 φ − 4sin φ)

= 2cos φ sin φ − 4 cos φ = sin2φ − 4cosφ = 2sin2φ − cos


= φ 2(sin2φ − 4cosφ) + 7cosφ

2(sin2φ − 4 cos φ) + 7 cos φ 2(sin2φ − 4 cos φ)dφ 7 cos φdφ dt 7 cos φdφ
I= ∫ ∫ +∫ = 2∫
t ∫ 6 − (1 − sin2 φ) − 4 sin φ
dφ = +
6 − cos2 φ − 4 sin φ 2
6 − cos φ − 4 sin φ 2
6 − cos φ − 4 sin φ

7 cos φdφ 7dx


= 2logt + C1 + ∫ = 2log(6 − cos2 φ − 4 sin φ) + C1 + ∫ ( sin φ =x )
2 2
5 + sin φ − 4 sin φ x − 4x + 5

7dx x−2
= 2log(6 − cos2 φ − 4 sin φ) + C1 + ∫ = 2log(6 − cos2 φ − 4 sin φ) + C1 + 7 tan−1 + C2
2
(x − 2) + 1 1

= 2log(6 − cos2 φ − 4 sin φ) + 7 tan−1 (sin φ − 2) + C

7. SPECIAL EXPONENTIAL FUNCTIONS


aex x
(a) ∫ b + cex dx [put e =t]

1
(b) ∫ 1 + ex dx [Multiply and divide by e− x and e− x = t]

1
(c) ∫ 1 − ex dx [Multiply and divide by e− x and e− x = t]

1
(d) ∫ ex − e−x dx [Multiply and divide by ex ]

ex − e− x  f ' (x) 
(e) ∫ ex + e− x dx   form
 f(x) 
ex + 1
(f) ∫ ex − 1 dx [Multiply and divide by e− x/2 ]

1
(g) ∫ (1 + ex )(1 − e−x ) dx [Multiply and divide by ex and put ex =t]

1
(h) ∫ dx [Multiply and divide by e − x/2 ]
x
1−e
M a them a ti cs | 22.21

1
(i) ∫ dx [Multiply and divide by e − x/2 ]
x
1+e
1 − x/2
( j) ∫ x
dx [Multiply and divide by e ]
e −1

1
(k) ∫ dx [Multiply and divide by 2e− x/2 ]
x
2e − 1

(l) ∫ 1 − ex dx [Integrand = (1 − ex ) / 1 − ex ]

(m) ∫ 1 + ex dx [Integrand = (1 + ex ) / 1 + ex ]

(n) ∫ ex − 1dx [Integrand = (ex − 1) / ex − 1 ]

ex + a
(o) ∫ dx [Integrand = (ex + a) / e2x − a2 ]
x
e −a

Illustration 34: Evaluate : ∫ ex − 1dx  (JEE MAIN)

Sol: Here by multiplying and dividing by ex − 1 in the given integration and then by putting ex − 1 = t2 we can
evaluate the given integration.

ex − 1 ex 1
Here I = ∫ ex − 1dx = ∫ dx = ∫ dx − ∫ dx
x x x
e −1 e −1 e −1
Let ex − 1 = t2 , then e=
x
dx
= 2t dt

2

= I 2∫ dt − ∫ dt = 2t − 2 tan−1 (t) + c = 2  ex − 1 − tan−1 ex − 1  + c
t +12  

ex
Illustration 35: Evaluate : ∫ dx  (JEE MAIN)
5 − 4ex − e2x

ex
Sol: We have, ∫ dx
5 − 4ex − e2x

Put ex = t , then ex dx = dt

ex dt
∴∫ dx = ∫ dx
x 2x
5 − 4e − e 5 − 4t − t2

dt dt
= ∫ = ∫
5 − (t2 + 4t) 5 − (t2 + 4t + 4) + 4

dt dt t+2  x
−1 e + 2

= ∫ = ∫ = sin−1   + C = sin  +C
 
9 − (t + 2)2 (3)2 − (t + 2)2  3   3 
2 2 . 2 2 | Indefinite Integration

PROBLEM-SOLVING TACTICS

Integration by Parts
(a) Integration by parts is useful for dealing with integrals of the products of the following functions
uu << tan−−11 x,
<< tan sin−−11 x,cos
x, sin x,cos−−11 xx (logx)
nx)kkk sin
((nx) sin x,
x, cos x       eexx >>
cos x        dv
>> dv
Priority for choosing u and dv: ILATE
(b) Integration by parts is sometimes useful for finding integrals of functions involving inverse functions such as
n x and sin−1 x .
(c) Sometimes when dealing with integrals, the integrand involves inverse functions (like sin−1 x ), it is useful to
substitute x = the inverse of that inverse function (like x = sin u), then do integration by parts.
2 x 3
(d) Sometimes you will have to do integration by parts more than once (for example, ∫ x e dx and ∫ x sinxdx .
Sometimes you need to do it twice by parts, then manipulate the equation (for example, ∫ ex sinxdx ).
(e) Try u – substitution first before integration by parts.

Trigonometric Integral
m
(a) Integral Type : ∫ sin x cosn xdx
Case 1: One of m or n are even, and the other odd
Use u – substitution by setting u = sin or cos that with an even power. Use the identity sin2 x + cos2 x =
1.
Case 2: Both m and n are odd
Use u – substitution by setting u = sin or cos that with a higher power. Use the identity sin2 x + cos2 x =
1.
Case 3: Both m and n are even (hard case)
Do not use u – substitution. Use the half double angle formula to reduce the integrand into case 1 o r2:
1 2 1 2 1
sinx cos x = sin2x ; sin= x (1 − cos2x) ; cos= x (1 − cos2x)
2 2 2
(Note: 0 is also an even number. For example, sin3 x = sin3 x coso x , so it is in case 1)
Just remember that when both are even, you can’t use u-substitution, but you can use the half – double angle
formula. When it is not that case, let u = sin x or cos x, and one will work (at the end there is no square root
term after substitution).
m
(b) Integral type : ∫ tan x secn xdx
Case 1: sec is odd power, tan is even power.
Hard to do, we omit (most likely won’t pop out in the exam).
Case 2: Else
Set u = sec x or tan x, and use 1 + tan2 x =
sec2 x . One will work at the end (there is no square root term after
substitution).
(c) Integral type : ∫ sin(Ax)cos(Bx)dx , ∫ cos(Ax)cos(Bx)dx , ∫ sin(Ax)sin(Bx)dx
Use the product to sum formula:
1 1
cos θ cos
= φ (cos θ − φ) + (cos θ + φ)) ; sin θ cos
= φ (cos θ − φ) − (cos θ + φ))
2 2
1
sin θ cos
= φ (sin θ − φ) + (sin θ + φ))
2
Reduce product into sum and then integrate.
M a them a ti cs | 22.23

Trigonometric Substitution
(a) Trigonometric substitution is useful for quadratic form with square root:
a2 − x2 : Let=x asin θ

x2 + a2 : Let
= x atan θ

x2 − a2 : Let
= x asec θ
(b) General procedure for doing trig sub:
Step 1: Draw the right triangle, and decide what trigonometric function to substitute for x.
Step 2: Find dx, then substitute the integrand using triangle, convert integral into trigonometric integral.
Step 3: Solve the trigonometric integral.
Step 4: Substitute back using triangle.
(i) If the quadratic form is not in the Pythagoras form (for example, 2 + 2x + x2 , then use the perfecting
the square method to transform it into Pythagoras form).
(ii) Try u – substitution before trigonometric substitution.
(iii) Integrals involving (1 − x2 ) and (x2 − 1) without square roots can be solved easily with partial fractions.
So don’t use trigonometric substitution.

Rational Integral and Partial Fraction


(a) General step for solving rational integral:
Step 1: Do long division for the rational function if the degree of the numerator is higher than the denominator.
Step 2: Do partial fraction decomposition.
Step 3: Evaluate the integral of each simple fraction.
(b) General step for partial fraction:
Step 1: Factorize the denominator.
Step 2: Set the partial fraction according to “rule”.
Step 3: Solve the unknown of the numerator of the partial fraction.

Improper Integral
(a) General steps for evaluating improper integral:
Step 1: Change the improper integral into the appropriate limit. [Change ±∞ or singular point (where) to
appropriate limit.]
Step 2: Evaluate the integral.
Step 3: Find the limit.
(b) The very first step to test improper integral involving ∞ is to check its limit. If its limit is not zero, then the
integral diverges.
(c) Whenever you see improper integrals involving the quotient of a rational or irrational function, such as

x3 + 3x
∫a 3
3
dx
(8x + 7x) 2

Use limit comparison test. The appropriate comparing function can be found by looking at the Integrand
(quotient of rational irrational). “Discard” the lower degree terms.
2 2 . 2 4 | Indefinite Integration

(d) Sometimes, using u – substitution before using any test will be easier.
(e) Sometimes, to determine if an improper integral converges or diverges, directly evaluating the improper
integral is easier.
(f) When doing a comparison test, beware of the comparing function that you choose. It might not give an
appropriate conclusion if the comparing function is not correct.
(g) Try the limit comparison test before the comparison test.
(h) Useful comparing function, which is good to know their convergence or divergence
∞ k −βx

∫a x e dx < ∞ For k ≥ 0, β > 0

∞ 1 < ∞ if p > 1;
∫a xp
dx 
= ∞ if p ≤ 1;

∞ 1 < ∞ if p < 1;
∫a xp
dx 
= ∞ if p ≥ 1;

FORMULAE SHEET

Basic theorems of Integration:

1. ∫ k f(x)dx = k ∫ f(x)dx 2. ∫ f(x) ± g(x) dx = ∫ f(x)dx ± ∫ g(x)dx

d
3.
dx
( ∫ f(x)dx ) = f(x) 4.
 d 
∫  dx f(x) dx = f(x)

Elementary Integration:

1. ∫ 0.dx = c 2. ∫ 1.dx= x + c

n xn+1
3. ∫ k.dx =kx + c(k ∈ R) 4. ∫ x= dx
n+1
+ c(n ≠ −1)

1
5. ∫= dx loge x + c x
ex + c
x
6. ∫ e dx=

ax
7. x
∫a= dx + c ax loga e + c
= 8. ∫ sinx dx =
− cos x + c
loge a

n (ax + b)n+1
9. ∫ cos x=
dx sinx + c 10. ∫ (ax + b) =dx
a(n + 1)
+C
c

cc cc
11. ∫ ax dx
∫ ax+ +b=
=
b
n naxax+ +b b+ +CC
dx log
aa
c 12. '
∫ f (x)e
f(x)
= ef(x) + C
dx c

13. ∫ logx=
dx xlogx − x + c x
14. ∫ log a x dx= xloga x − +c
loga
M a them a ti cs | 22.25

Standard substitution:

1 x = a sin θ or x = a cos θ
1. a2 − x2 or
a2 − x2

1 x = a tan θ or x = a cot θ
2. x2 + a2 or
   x2 + a2   

1 x = a sec θ or x = acosec θ
3. x2 − a2 or
x2 − a2
x a+x 1
4. , , x(a + x) and x=a tan2 θ
a+ x x x(a + x)

x a−x 1 x = a sin2 θ or x= a cos2


5. or x(a − x) and
a−x x x(a − x)

1
x x−a x=a sec2 θ or x=a cosec2 θ
6. or or x(x − a) or x(x − a)
x−a x

a−x a+ x x = a cos2 θ
7. and
a+x a−x

x−α x = α cos2 θ + β sin2 θ


8. or (x − α )(β − x) (β > α )
β−x

Some standard Integrals:

1. ∫ tanxdx =logsec x + c =− logcos x + c 2. ∫ cot


= xdx logsinx + c

3. ∫ sec xdx= log(sec x + tanx) + c 4. ∫ cosecxdx =


− log(cosecx + cot x) + c
π x x
=− log(sec x − tanx) + c =logtan  +  + c = log(cosecx − cotx)
= + c log tan   + c
 4 2 2

5. ∫ sec x tanxdx
= sec x + c 6. ∫ cosec x cot x dx =
− cosec x + c

2 2
7. ∫ sec xdx tanx + c
= 8. ∫ cosec xdx =
− cot x + c

x
9. ∫ logxdx
= xlog  =+ c x (logx + 1 ) + c 2 1 sin2x 
e
10. ∫ sin xdx = x −
2 2 
+C

1
=−(x sinxcosx) + C
2
2 2 . 2 6 | Indefinite Integration

1 sin2x  12. 3xdx == 1


33 11
11. 2
∫ cos xdx= x +  + C= ∫∫sec
secxdx
∫sec xdx = sec
sec
222
xxtanx
sec xtanx
tanx
2 2 
11
1 +++ 1
ln
lnlnsec
log secxxx+++tanx
sec tanx+++CCcC
tanx
(x + sinxcosx) + C 222
2

sinn−1 x cos x n cosn−1 x sinx


13. ∫ sin xdx =
n

n
+ 14. ∫ cos xdx = −
n
+

n−1 n−1
cosn−2 xdx
n ∫
sinn−2 xdx
n ∫

Integration by Parts:

∫ (u.v)dx u
1. = ( ∫ v dx ) − ∫  du 
 ( v dx ) dx
dx  ∫
2. ∫e
x
ex f(x) + c
f(x) + f '(x) dx =

Standard Integrals:

1 1 x
1. ∫= dx tan−1   + c
2
x +a 2 a a

1 1 x−a
∫ x2 − a2 dx 2a log x + a + c
2. =

1 1 a+ x
∫ a2 − x2 dx 2a log a − x + c
3. =

1 x x
4. ∫ dx =sin−1   + c =− cos−1   + c
2
a −x 2 a
  a

1 x
5. ∫ dx sinh−1   +
= = c log  x + x2 + a2 +c

2
x +a 2 a
   

1 x
6. ∫ dx cosh−1   +
= = c log  x + x2 − a2 +c

2
x −a 2 a
   

x 2 a2 x
7. ∫ a2 − x2 dx
= a − x2 + sin−1 + c
2 2 a

xx 2 2 2 2 aa2 2
8. ∫∫ xx2 2++aa2 2dx
=
dx
= xx ++aa ++ log 2 2
nnxx++ xx2++aa2 ++cc
22 22

22 22 x x 2 2 2 2 a2a2
9. ∫∫ x x − a
− adx
=
dx
= n nx x+ + x2x2− −a2a2+ +c c
x x − −a a − − log
22 22
M a them a ti cs | 22.27

1 1 x
10. ∫ = dx sec−1 + c (Valid for x > a > 0)
2
x x −a2 a a

eax eax   b  
ax
(asinbx − bcosbx) + c = sin bx − tan−1    + c
11. ∫ e sinbx=
dx
a2 + b2 a2 + b2   a  

ax eax eax  b
12. ∫ e cosbx=dx
2
a +b 2
(acosbx + bsinbx) + c =
2
cos bx − tan−1  + c
2 a
a +b 

Solved Examples

JEE Main/Boards ∫
dx
 n ( sec θ + tan θ )
x2 + a2
= ∫ sec θ dθ = log
x + sinx
Example 1: Evaluate : ∫ 1 + cos xdx
 x2 + a2 x 
 n
= log + 
Sol: Here by using the formula  a a
 
x x x
=sinx 2sin cos and
= 1 + cos x 2cos2    x2 + 
2 2 2 11 2 2 2 2 a2a2 x x 
x2 +
=∴I I  x xx x+
∴= + nn 
a a + +log + + + + cc
we can solve the given problem. 
22   aa a a 
    
x + sinx x + 2sinx/ 2cosx/ 2
∫ 1 + cos xdx = ∫ dx
1 − sinx
2cos2 x / 2 −1
Example 3: Evaluate : ∫ tan 1 + sinx
dx
x 2 x
= ∫ 2 sec x / 2 + tan dx = x tanx / 2 + c
2 Sol: Here first write cos ( ( π / 2) − x ) at the place of sin x
x
2sin2
then by using the formula 1 − cos x =
Example 2: Evaluate : ∫ x2 + a2 dx 2
x
2cos2
And 1 + cos x = we can solve it.
Sol: By applying integration by parts and taking 2

x2 + a2 as the first function we can solve the given 1 − sinx 1 − cos ( ( π / 2) − x )


I = ∫ tan−1 dx = ∫ tan−1 dx
problem. 1 + sinx 1 + cos ( ( π / 2) − x )

x2 + a2 dx = 2sin2 ( ( π / 4) − (x / 2) )
∫ = ∫ tan
−1
dx
2cos2 ( ( π / 4) − (x / 2) )
(x2 + a2 ) − a2 x2
= x2 + a2 x − ∫ dx x2 + a2 x − ∫ dx
−1 π x π x π x2
x2 + a2 x2 + a2 = ∫ tan tan  4 − 2  dx = ∫  4 2 
− dx = x − +C
4 4
dx
x x2 + a2 + a2 ∫
= x2 + a2 Example 4: Evaluate : ∫ log(2 + x2 )dx
2
Put x = atan θ Sol: Here integrating by parts by taking log(2 + x2 ) as
the first function we can solve the given problem.
2 2 . 2 8 | Indefinite Integration

I = ∫ log(2 + x2 )dx = ∫ log(2 + x2 ).1dx dx


Example 7: Evaluate: ∫
2
(1 + x ) 1 − x2
2
Taking log(2 + x ) as first function and integrating by
parts, we get Sol: By putting x=sin θ ⇒ dx = cos θ d θ , we will
2x reduce the given integration as
I = log(2 + x2 ) x − ∫ x. dx
  2 + x2 sec2 θ ⇒ dt = sec2 θ dθ
2
(x + 2) − 2
∫ 1 + 2 tan2 θdθ and then put t = tan θ
= xlog(2 + x2 ) − 2∫ dx
2 + x2
and solve it.
 2  Put x=sin θ ⇒ dx = cos θ d θ
= xlog(2 + x2 ) − 2∫ 1 − dx
2
 x + 2
dx 1
  x 
⇒ I= ∫ 2 2
= ∫ 1 + sin2 θdθ
= xlog(x2 + 2) − 2  x − 2 tan−1    + Cc (1 + x ) 1 − x
  2  
sec2 θ
= ∫ 1 + 2 tan2 θdθ
e2x − e−2x
Example 5: Evaluate : ∫ e2x + e−2x dx Again put t = tan θ ⇒ dt = sec2 θ dθ

dt 1 1 1
Sol: Simply put e2x + e−2x = t ⇒ (e2x − e−2x )dx = I= ∫ 1 + 2t2 dt = ∫ dt
( )
2
and then solving we will get the result. 2 2 t2 + 1 / 2
2x −2x
e −e
I= ∫ e2x + e−2x dx =
1 1 

−1 
 tan 
t 
 + c
21 / 2  1 / 2 
dt
t ⇒ (e2x − e−2x )dx =
Put e2x + e−2x =
2  x 2 
1 1
1 dt 1 1 = tan−1 ( 2 tan θ) + c = tan−1  +c
2x −2x 2 2  
∴I = =∫
2 t 2
log t + C = log e + e
2
+ Cc  1−x
2

x3 − 1 xdx
Example 6: Evaluate : ∫ x3 + xdx Example 8: Evaluate : ∫ (x− 1)(x2 + 4)
Sol: By splitting the given integration as Sol: By partial fractions, we can reduce the given
3 fraction as a sum of two fractions which will be easier
x 1
∫ x(x2 + 1)dx − ∫ x(x2 + 1)dx to integrate.
x A Bx + C
= +
We can solve the given problem. 2
(x − 1)(x + 4) x − 1 x2 + 4

x3 − 1 x3 1 x = 1 ⇒ A = 1/5
∫ x3 + xdx = ∫ x(x2 + 1)dx − ∫ x(x2 + 1)dx x = 2i ⇒ B = –1/5, C = 4/5

x2 1 x  
1 4 − 2x
= ∫ x2 + 1dx − ∫ x − (x2 + 1)dx =∴ I ∫ +  dx
 5 ( x − 1)


5 x2 + 4 ( ) 


 1  1 x
= ∫ 1 − dx − ∫ dx + ∫ 2 dx 11 11   2x
2x 88  
 2
x +1 x x +1 =
= n(x
logn(x−−1)1)−−   2 −− 
55 10
10 xx2++44 xx2 2++44 
= x − tan−1 x − logx + log x2 + 1 + c
=
1
n(x(x− −
log 1)1)− −
(
x2 + 24 + 4)2 2 x x
ln log(x
+ −tan
+ tan
)
1 −1
5 10 10 5 5 2 2
M a them a ti cs | 22.29

sinx x2 −1 1 x2 x2 1 (1 − x2 ) − 1
Example 9: Evaluate: ∫ sin 4xdx =
2
sin x − ∫ . dx = sin−1 x + ∫ dx
1 − x2 2 2 2 1 − x2
Sol: By using the formula sin2x = 2sinx.cos x , we can
reduce the given fraction and then by putting sin x = t x2 1 1 1
= sin−1 x + ∫ 1 − x2 dx − ∫ dx
we can solve it. 2 2 2 1 − x2
sinx sinx dx dx
∫ sin 4xdx = ∫ 2cos2x sin2x = ∫ 4 cos x cos2x x2 x 1 − x2 1 −1
= sin−1 x + − sin x + Cc
2 4 4
cos xdx
= ∫ 4(1 − sin2 x)(1 − 2sin2 x) 2x2 − 1 −1 x
= sin x + 1 − x2 + C
c
4 4
Put sin x = t
1 dt ex (2 − x2 )dx
⇒I= ∫
4 (1 − t )(1 − 2t2 )
2 Example 2: Evaluate : ∫
(1 − x) 1 − x2
1  1 2 
= ∫  2 −  dt Sol: We can split the given fraction as
4  (t − 1) (2t − 1) 
 2
 1 + x 1 
x
11 t −1 1 2t − 1  ∫e 
2
+ dx and this integration is
= log − log +c  1 − x (1 − x) 1 − x2 
42 t +1 2 2t + 1 
 in the form of ex (f(x) + f ' (x))
1 sinx − 1 1 2 sinx + 1
= log + log +c ex (2 − x2 )dx x (1 − x2 ) + 1
8 sinx + 1 4 2 2 sinx − 1 I= ∫ = ∫e dx
(1 − x) 1 − x2 (1 − x) 1 − x2
dx  1 + x 1 
Example 10: Evaluate : ∫ (x)(x 4 − 1) = ∫e
x
 + dx
2
 1 − x (1 − x) 1 − x2 
Sol: Here we can write the given integration as
d  1+x  1 x
x −5 −4 But   = + (1 + x)
∫ (1 − x −4 ) dx and then by putting 1 − x t
= dx  1 − x2 
 1−x 2
(1 − x2 )
3
2

⇒ 4x −5 = dt we can solve it. 1 (1 + x)x


= +
dx x −5 1 − x2 (1 − x)(1 + x) 1 − x2
∫ (x)(x 4 − 1) = ∫ (1 − x −4 ) dx
1 x 1−x+x
= + =
Put 1 − x −4 =
t ⇒ 4x −5 = dt 1−x 2
(1 − x) 1 − x 2
(1 − x) 1 − x2
dt 1 1 1
log | 1 − x −4 | + c =
=
⇒ ∫=
4t 4
log| t | +=
c
4 (1 − x) 1 − x2
Hence, the integrand is of type ex (f(x) + f ' (x))
1+x
JEE Advanced/Boards ∴ I = ex +C
1 − x2
−1
Example 1: Evaluate : ∫ x sin xdx cos3 x + cos5 x
Example 3: Evaluate : ∫ sin2 x + sin4 x dx
Sol: Integrating by parts taking sin−1 x as the first term
we can solve the given integration. Sol: Here by taking cos3 x and sin2 x common from the
−1 numerator and denominator respectively and then by
∫ x sin xdx
putting sin x =t we can solve the given problem.
−1
Let u sin
= = x, v x cos3 x(1 + cos2 x)
I= ∫ sin2 x(1 + sin2 x)
dx
2 2 . 3 0 | Indefinite Integration

Since power of cos x is odd, put sin x =t; 1 1 z −z −5 5


dt =
−= n n + C+ cC
⇒ I = −∫ − log
Then, cos x dx = dt 2
z −5 2 25 5 z +z +5 5

(1 − t2 )(1 + 1 − t2 ) (1 − t2 )(2 − t2 )
I= ∫ dt = ∫ t2 (1 + t2 ) dt
t2 (1 + t2 )
=
−=
11
n n
x x+ (1 (( 2 2 2 2
))
/ x/ x) +)3+ 3− −5 5
+ (1
+ C+ C
c
− log
=
 2 6 
∫  1 + t2 − 1 + t2  dt = t−
2
t
− 6 tan−1 t + C
c
2 25 5 x2 x+2 (1(( ))
/ x/2 x) 2+)3+ 3+ +5 5
+ (1

Example 6: Evaluate :
= sinx − 2cosec x − 6 tan−1 (sinx) + Cc

x
4e + 6e −x ∫ cosec
22
∫ cosecx.x.log
n
n (cosx++ cos2x
(cosx ) dx for sin x > 0
cos2x) dx
Example 4: Evaluate : ∫ 9ex − 4e−x dx
Sol: By substituting cos2 x − sin2 x in place of . cos2x .
we can reduce the given integration as the sum of two
Sol: By partial fractions we can reduce the given fraction
integrations and then by integrating them separately
as a sum of two fractions and then by integrating them
we will obtain the result.
we will get the result.
x −x ∫∫cosec
2
nn((cosx
cosec2x.x.log cosx++ cos2x ))
cos2x dx
dx
4e + 6e
I= ∫ 9ex − 4e−x dx
 2 
nncos sin2xxdx
2 2
== ∫∫cosec
cosec2xxlog cosxx++ coscos2xx−−sin dx
x −x x −x
Let 4e + 6e = A 9e − 4e + B  (  d
 dx
x
9e − 4e  )
−x 

( 

)
 

nn sinx(cot cot2xx−−1)1) dx
2 2
=
= ∫ ∫ cosecx2 xlog
cosec
 
sinx(cotxx++ cot

dx
By comparing the coefficients of ex and e− x , we get
== ∫∫cosec cosec22x.x.log
nsinx
nsinxdx dx++ ∫ ∫cosec 22
cosecx. x.log
n[cot
2
cot2xx−−1]dx
n[cotxx++ cot 1]dx
−19 35
A= ,B=
36 36 = I1 + I2

A(9ex − 4e− x ) + B(9ex + 4e− x ) I1I1==∫∫cosec 2


cosec2x.x.log
nsinxdx
nsinxdx
∴I= ∫ 9ex − 4e− x
dx
= ( −( −cot
cotx).
x).lognsinx−−∫∫( −( −cot
nsinx cotx).cot
x).cotxdx
xdx
x −x
9e + 4e
= A ∫ dx + B ∫
x −x
dx = Ax + Blog 9ex − 4e− x + cC = −−cot
cotx.x.log
nsinx (cosec22xx−−1)dx
nsinx++∫∫(cosec 1)dx
9e − 4e
19 35 = −−cot
cotx.x.log
nsinx
nsinx−−cot
cotxx−−xx++CC11
= − x+ log 9ex − 4e− x + Cc
36 36 22
=
=I2I2 ∫∫cosec
cosec x.x.log
n[cot
n[cotxx++ cot22xx−−1]dx
cot 1]dx

1 + x2 dx Put cot x=t; − cosec2 xdx =


dt
Example 5: Evaluate : ∫ 1 − x2 for x > 0
−−∫∫log
I2I2= 2
n[t++ t t2−−1]dt
n[t
1 + 3x2 + x 4 = 1]dt
(Integrate by parts)
Sol: Dividing by x2 in the numerator and denominator
and then putting x – 1/x = t we can solve the given 1 + (t / t2 − 1)
==−t. n  t + t2 − 1  + ∫ t.
−t.log dt
problem.   t + t2 − 1
I= ∫
1 + x2 dx
= ∫
(1 / x2 ) + 1 dx ( ) t
1 − x2 1 + 3x2 + x 4
==−t. n  t + t2 − 1  + ∫
−t.log dt
((1 / x) − x ) ( x − (1 / x)) + 5
2
  t2 − 1
 1 
Put x – 1/x = t ;  1 +  dx = dt ==−t. n  t + t2 − 1  + t2 − 1 + C2
−t.log
 
 x2 
dt
I = −∫ − cot x n  cot x + cot2 x − 1  + cot2 x − 1 + C2
==
−cotxlog
t t2 + 5  

Put t2 + 5 =z 2 ⇒ 2t dt = 2z dz sinx
Example 7: Evaluate : ∫ sin3 x + cos3 xdx
M a them a ti cs | 22.31

Sol: By taking cos3 x common from the denominator 1 m m


and then by putting tan x = t we can solve it.
= cosm x sinnx + Im−1,n−1 − Im,n
n n n
Integrand contains odd powers in sin x and cos x. So, ⇒ (m + n)I
= cosm x sinnx + mIm−1,n−1
m,n
put tan x = t.
1 sinx n
⇒I= ∫ cos3 x 1 + tan3 xdx Example 9: Evaluate : I = 
∫  x + 1 + x2  dx

tanx.sec2 x Sol: Simply putting x + 1 + x2 = t and integrating we
= ∫ 1 + tan x3
dx (put tan x = t)
can solve the given problem.

t 1 dt 1 t +1 Let x + 1 + x2 = t, then
= ∫1+ t 3
dt = −
3 ∫ 1 + t 3 ∫ t2 − t + 1
+ dt
 x  t
1 +  dx =
dt ⇒ dx = dt
 
1 1 (2t − 1) + 3  1 + x2  1 + x 2
= − log t + 1 + ∫ dt
3 6 t2 − t + 1
As 1 + x2 + x =t
1 1
= − log t + 1 + log t2 − t + 1 +
3 6 1 1 1 + x2 − x
= =
1 dt t 1 + x2 + x 1
2 t − (1 / 2) 2 + (3 / 4)
( ) 1 t2 + 1
⇒ 2 1 + x2 =t + =
1 1 t t
= − log t + 1 + log t2 − t + 1 +
3 6  t2 + 1  dt
Thus I = ∫ tn  
1 2  t − (1 / 2)   2t  t
tan−1   + C  
2 3  3 /2  1 n− 2 2 1 n n− 2 1  tn+1 tn−1 
2∫ 2∫
= t (t + 1)dt = (t +t )dt =  + +C
2  n + 1 n − 1 
1 1 − tanx + tan2 x 1  2 tanx − 1 
= log + tan−1  +C
6
(1 + tanx )
2
3  3  Where t =x + 1 + x2

Example 8: If Im,n = ∫ cosm x.cosnx dx, show that Example 10: Evaluate:
(m + n)I
= m,n cosm x sinnx + mIm−1,n−1 2sin3 ( x / 2 ) dx
I= ∫ for cos x > 0
Sol: By using integration by parts and by taking cosm x ( cos(x/ 2)) cos3 x + 3cos2 x + cos x
as the first term we can prove the given equation.
Integrating by parts, Sol: Here we can reduce the given fraction by using the
x x
sinnx formula sinx = 2sin cos and then by putting
Im,n = cosm x + 2 2
n cos x = t we can solve it.
m
n∫
cosm−1 x sinx sinnx dx

… (i) ( 2sin ( x / 2) cos ( x / 2) ) ( 2sin2 ( x / 2) ) dx
I= ∫
But cos(n-1)x=cos(nx-x) (2cos (x / 2))
2
cos3 x + 3cos2 x + cos x
= cosnx cos nx + sin nx sin x (1 − cos x)sinx dx
⇒ sinx sin nx = cos(n-1)x – cosnx cosx ... (ii)
= ∫
(1 + cos x) cos3 x + 3cos2 x + cos x
From (i) and (ii):
[Put cos x = t]
1
Im,n = cosm x sinnx +
n ( t − 1) dt
m
⇒I= ∫
cosm−1 x[cos(n − 1)x − cosnx cos x]dx (1 + t) t3 + 3t2 + t
n∫
2 2 . 3 2 | Indefinite Integration

(t 2
)
− 1 dt Example 12: Evaluate : I = ∫
dx
= ∫ (1 + t)2 t t + 3 + (1 / t)
1 + x2 + x + 2

Sol: We can reduce the given fraction as


(
t2 1 − (1 / t2 ) dt ) dx
= ∫ t(t2 + 2 t+ 1) t + (1 / t) + 3 ∫ and then by putting
( x + (1 / 2))
2
1+ + (7 / 4)
(1 − (1 / t )) dt 2
1 7 π π
= ∫ (t + (1 / t) + 2) =x+ tan θ : − < θ < ; and using appropriate
( t + (1 / t)) + 3 2 2 2 2
integration formula we can integrate the given fraction.
1 dx
Put t + +3 =z2 : z > 0 ; I=∫
t
1 + ( x + (1 / 2) ) + (7 / 4)
2

 1
Then  1 −  dt =
2zdz
 t2  1 7 π π
Put =
x+ tan θ : − < θ < ; then
2 2 2 2
2zdz dz z −1
⇒ I= ∫ (z2 − 1)z = 2∫
2
z −1
= log
z +1
+ Cc
=dx
7
sec2 θ d θ
2

cos x + sec x + 3 − 1 7 sec2 θdθ


⇒ I log + Cc ⇒ I= ∫ 2 1 + ( 7 / 2)sec θ
cos x + sec x + 3 + 1
7 dθ
2 ∫ cos θ cos θ + ( 7 / 2)
=
Example 11: Evaluate:
( )
(sinx − cos x) dx  1 1 
I= ∫ = ∫  cos θ − cos θ + ( dθ
7 / 2) 
(sinx + cos x) sinx cos x + sin2 x cos2 x 

dθ 7
Sol: Here by putting sin x + cos x = t we can integrate = log sec θ + tan θ − ∫ ; a=
the given fraction using the appropriate formula. a + cos θ 2

Let sin x + cos x = t = I log sec θ + tan θ − I 1 ….(i)

⇒ (cos x – sin x) dx =dt dθ


Where, I 1 = ∫ a + cos θ
Also, t2 =+
(sinx cosx)2 =
1 + 2sinx cos x
θ 1 − t2
t2 − 1 Put tan = t ; cos θ =
∴ sinx cos x = 2 1 + t2
2
dt
⇒ I = −∫ 2dt 1
(
t (t2 − 1) / 2 1 + (t2 − 1) / 2 )( ( )) I 1= ∫ 1 + t2 a +
((1 − t ) / (1 + t ))
2 2

dt t3dt dt
= −∫ = −2∫ = 2∫
t ((t2 − 1)(t2 + 1)) / 4 t4 t4 − 1 a(1 + t ) + 1 − t2
2

Put t 4 - 1 = z 2 : z > 0 2 dt
= ∫
a − 1 ( (a + 1) / (a − 1) ) + t2
1 2z dz dz
⇒ I = −2∫ = −∫
4 (z 2 + 1)z 1 + z2  a−1 
2 a−1
= tan−1  t  + Cc
a−1 a+1  a+1 
= − tan−1 z + Cc = − tan−1 t 4 − 1 + Cc  

2  a−1 θ
= − tan−1 (1 + sin2 x)2 − 1 + C
c = tan−1  tan  + Cc.…(ii)
 a+1 2 
a2 − 1 
= − tan−1 sin2 2 x + 2sin2 x + Cc From (i) and (ii), we get I.
M a them a ti cs | 22.33

JEE Main/Boards

Exercise 1 Q.18 If f ' (x)= x −


1
and f(1) =
1
, find f(x).
x 2 2
sec x
Q.1 ∫ dx Q.19 For any natural number evaluate m
sec x + tanx

∫ (x )( )
1/m
  3m
1 2 1 + x2m + xm 2x2m + 3xm + 6 dx, x〉 0
Q.2 ∫  1 + − +5 + a dx
x
 1+x 2 2 
 1 − x x x2 − 1  x3 + 3x + 2
−1  sin2x   π π
Q.20 ∫ (x2 + 1)2 ( x + 1 )dx
Q.3 ∫ tan  1 + cos2x dx : x ∈  − 2 , 2 
dx
Q.4 ∫
1 + tanx
dx
Q.21 ∫ sinx + sec x
x + logsec x

2cos x − 3sinx cos2x


Q.5 ∫ 3cos x + 2sinxdx Q.22 ∫ sinx
dx : cos x > 0

2x − 1 x2 + 1(log(x2 + 1) − 2logx
Q.6 ∫ dx
x2 − x − 1
Q.23 ∫ x4
dx

dx sinx
Q.7 ∫ 1 − 3x − 5 − 3x
Q.24 ∫ sinx − cos x dx
2 x3 3
Q.8 ∫x e cos(ex )dx  1 2a − x 
−1
Q.25 ∫ x sin 
2 a
dx

sec2 (2 tan−1 x)  
Q.9 ∫ 1 + x2
dx
4
Q.26 ∫ sec x cosec2 xdx
dx
Q.10 ∫ (2sinx + 3cos x)2

Q.27 ∫ :a>b>0
( a + b cos θ )
3/5 3 2
Q.11 ∫ cos x sin xdx

logx dx
Q.12 ∫
2
dx Q.28 Evaluate ∫
x x x4 − 1

−1 x
Q.13 ∫ sin a+ x
dx : a > 0
Q.29 ∫
dx
sin3 x sin ( x + α )
x 2 + sin2x
Q.14 ∫e 1 + cos2x
dx dx
Q.30 ∫
Q.15 ∫
dx (1 + x ) x − x2
x 6 (logx ) + 7logx + 2
2
  cos8x − cos7x
Q.31 ∫ 1 + 2cos5x
dx
x2 + 1
Q.16 ∫ (x + 3)(x − 1)2 dx
x3 + 1
Q.32 ∫ dx
1 x ( x − 1)
3
Q.17 ∫ dx
1 − tanx
2 2 . 3 4 | Indefinite Integration

dx  x x
Q.33 ∫3 11
2n  sec + tan  + Cc
(C) 2log
2 2
sin x.cos x 
ex n 1 + sinx + Cc
(D) log
Q.34 Evaluate ∫ dx
e2x − 4
Sin2x
Q.5 ∫ dx is equal to
log x sin x + cos4 x
4
Q.35 Evaluate ∫ (1 + x)3 dx
(A) cot −1 (cot2 x) + Cc (B) − cot −1 (tan2 x) + Cc
f(x)
Q.36 Evaluate ∫ x3 − 1 dx , where f(x) is polynomial of (C) tan−1 (tan2 x) + Cc (D) − tan−1 (cos2x) + Cc
the second degree in x such that f(0) = f(1) = 3f(2) = −3 dθ
Q.6 The value of integral ∫ cos3 θ sin2θ
can be

Exercise 2 expressed as irrational function of tan θ as

Single Correct Choice Type 2 2 


(A)  tan θ + 5  tan θ + c
5  
3/2
 x 
Q.1 ∫  1 + x5  dx equals- (B)
2
5
(
tan2 θ + 5 ) tan θ + c

(A)
2 x5
+ Cc (B)
2 x
+ Cc
(C)
5
2
(
tan2 θ + 5 ) tan θ + c
5 1 + x5 5 1 + x5
2
( )
tan θ
2 1 (D) tan2 θ + 5 +c
(C) + Cc (D) None of these 5
5 1 + x5
dx
8 8
Q.7 ∫ a + bx2 a,b≠0 and a/b > 0
cos x − sin x
Q.2 dx equals-
1 − 2sin2 x cos2 x 1 b b b
(A) tan−1 x + c (B) tan−1 x +c
ab a a a
sin2x sin2x
(A) − +c (B) +c a a b
2 2 (C) tan−1 x +c (D) ab tan−1 x +c
b b a
cos2x cos2x
(C) + c (D) − +c
3 1 1 − x7
Q.8 ∫x dx equals
Q.3 Identify the correct expression ( 1 + x7 )
= =xx2 2log
(A) x ∫ logxdx
nxdx x22 +
nx −−x + Cc nx
(A) logx
22 7
(( ))
nn 11++xx7 ++cc (B) logx
nx++ log
77
nx nx
22
nn
− − log
44
(( ))
1 −1 x−7 x7+ c+ c

(B) xx∫∫log
nnxx=
dx xexx ++CCc
dx xe
=
x
nx
(C) logx
22
nx−− log
77
7
((
))
nn 11++xx7 ++cc (D) logx
nx
22
nx++ log
44
7
nn11−−xx7 ++cc (( ))
(C) x ∫ e =dx xex + Cx
cx

dx 1 x
(D) ∫= tan−1   + C
c log | x |
2
a +x 2 a a Q.9 ∫x 1 + log | x |
dx equal

Q.4 Primitive of 1 + 2 tanx ( sec x + tanx ) w.r.t. x is (A)


2
1 + log| x |(log | x | − 2) + c
3
n( sec
n
(A) log ( secxx++tanx
tanx) )++log
ncos
ncosxx++CC
c
2
(B) log ( sec
nn ( secx x++tanx
tanx) +) +log
nsec
nsecx x++CcC (B)
3
1 + log| x |(log | x | + 2) + c
M a them a ti cs | 22.35

1
(C) 1 + log| x |(log | x | − 2) + c cot x − tanx
3 Q.15 ∫ dx equals
2(cos x + sinx)
(D) 2 1 + log| x |(3log | x | − 2) + c
(A) sec−1 (sin x + cos x ) +cC

xx44++11 BB (B) sec−1 (sin x − cos x ) + Cc


Q.10 If ∫ ∫ dx = = AAlog
dx nnxx++ ++cc ,
22 22
x(x
x(x ++1)1) 11++xx22
(C) log c
ln [(sinx + cos x) + sin2x ] + C
Where c is the constant of integration then:
(A) A=1; B=-1 (B) A=-1; B=1 (D) log
ln [(sinx − cos x) + sin2 x ] + c

(C) A=1; B=1 (D) A=-1; B=-1

x 3x Previous Years’ Questions


Q.11 ∫ 4 sin x cos cos dx equals
2 2

(A) cos
cos Xx −
1 1 (x2 − 1)dx
cos2 x + cos 3x + c Q.1 The value of ∫  (2006)
2 3
x3 2x 4 − 2x2 + 1
1 1
(B) cos
cos Xx − cos2 x − cos 3x + c
2 3 2 1 2 1
(A) 2 2 − + + c (B) 2 2 + + + Cc
2 4 2
1 1 x x x x4
(C)cos
cosXx + cos2 x + cos 3x + c
2 3
1 2 1
1 1 (C) 2− + + Cc (D) None of these
(D)cos
cosXx + cos2 x − cos 3x + c 2 x 2
x4
2 3

Q.12 ∫ sin x. cos x. cos 2x. cos 4x. cos 8x. cos16x dx 4ex + 6e− x
equals Q.2 If ∫ dx = Ax + B log (9e2x − 4) + Cc, then
9ex − 4e− x
sin16x cos32x A=……….., B=……… and C=……… (1989)
(A) + c (B) − +c
1024 1024
cos32x cos32x 1 sinx
(C) + c (D) − +c Q.3 Integrate or (1978)
1096 1096 1 − cot x sinx − cos x 

x
F(x) , then ∫ x3 f(x2 )dx is equal to
Q.13 If ∫ f(x) dx = Q.4 Integrate the curve  (1978)
1 + x4

(A)
1
2
( )
(F(x))2 − ∫ (f(x))2 dx (B)
1 2 2
2
(
x F(x ) − ∫ (f(x2 ) d(x2 ) ) Q.5 Integrate
sinx⋅ sin2 x⋅ sin3 x + sec2 x ⋅ cos2 2x + sin4 x ⋅ cos4 x. 
1  1 2   (1979)
(C)  F(x) − ∫ (F(x ) dx  (D) None of these
x2
2  2 
Q.6 Integrate  (1979)
(a + bx)2
4ex + 6e− x
Q.14 If ∫ Ax + B log (9e2x − 4) + C then
dx =
9ex − 4e− x
Q.7 Evaluate ∫( tanx + cot x)dx.  (1988)
A, B and C are
(x + 1)
3 36 3 Q.8 Evaluate ∫ dx.  (1996)
(A)=
A =,B =,C log 3 + cons tant x(1 + xex )2
2 35 2
3 35 −3 Q.9 Integrate the following  (1997)
(B)=
A =,B =,C log 3 + cons tant
2 36 2 1/2
 1− x  dx
3 35 3 ∫ 
(C) A =
− , B =, C = 1log 3 + cons tant  1+ x  x
2 36 2  
(D) None of these
2 2 . 3 6 | Indefinite Integration

sinxdx x+
1
Q.10 The value of 2 is  (2008) e x
 π  1
sin  x −  Q.14 The integral  1 + x −  dx is equal to  (2014)
 4  x
1 1
x+
(A) ( x + 1 )
 π  π x+
(A) x + log cos  x −  + c (B) x − log sin  x −  + c x + c (B) −xe x +c
 4   4 1
1 x+
(C) ( x − 1 )
x+
x + c (D) xe x +c
 π  π
(C) x + log sin  x −  + c (D) x − log sicos  x −  + c
 4  4
 dx
Q.15 The integral ∫ equals  (2015)
(x )
3/ 4
2 4
dy x +1
Q.11 If = y + 3 > 0 and y ( 0 ) = 2 , then y(log2) is
equal to  dx
1/ 4
(2011)  x4 + 1 
( )
1/ 4
(A)   + c (B) x 4 + 1 +c
(A) 5 (B) 13 (C) -2 (D) 7  x4 
 
1/ 4
 x4 + 1 
( )
1/ 4
Q.12 If the integral (C) − x + 1 4
+ c (D) −  4  +c
 x 
55tanx
tanx  
∫∫tanx
tanx−−22
dx xx++aalog
dx =
= nnsinx
sinx−−2cos
2cosxx ++kk
2x12 + 5x9
then a is equal to  (2012) Q.16 The integral ∫ dx is equal to  (2016)
(x )
3
5 3
(A) -1 (B) -2 (C) 1 (D) 2 + x +1

x10 x5
( )
Q.13 If ∫ f ( x ) dx = ψ ( x ) , then ∫ x5 f x3 dx is equal to (A)
( 5
2 x + x +1 3
)
2
+ c (B)
( 5
2 x + x +1 3
)
2
+c
 (2013)
1 3
3
3 2
( ) 3
(A)  x ψ x − ∫ x ψ x dx  + c 
 ( ) (C)
−x10
+ c (D)
−x5
+c
( ) ( )
2 2
2 x5 + x3 + 1 2 x5 + x3 + 1
(B)
1 3
3
( )
x ψ x3 − ∫ x2 ψ x3 dx + c( )
(C)
1 3
3
( )
x ψ x3 − ∫ x2 ψ x3 dx + c ( )
(D)
1 3
3 
( ) ( )
x ψ x3 − ∫ x2 ψ x3 dx  + c

JEE Advanced/Boards

Exercise 1
 x  x  e  x 
dx Q.4   +    logxdx
∫ nx dx
Q.1 (i) ∫ ;  e   x  
cot(x/ 2) ⋅ cot(9 x/ 3) ⋅ cot(x/ 6)  

cos(x − a)
tan(log x) tan(log(x / 2))tan(log2) Q.5 ∫ dx
(ii) ∫ dx sin(x + a)
x
dx
Q.2 ∫ x5 + 3x 4 − x3 + 8x2 − x + 8
(x − α ) (x − α )(x − β) Q.6 ∫ dx
x 2 +1

(
log log ( (1 + x) / (1 − x) ) ) dx Q.7 ∫
( x + 1)dx
Q.3 ∫ 1 − x2 x( 3 x + 1)
M a them a ti cs | 22.37

−1 x cos2 x
Q.8 ∫ sin dx Q.23 ∫ dx
a+ x 1 + tanx

−1
xInx Q.24 ∫ log x.sin x dx
Q.9 ∫ dx
(x2 − 1)3/2
(x2 + 1)ex
Q.25 Evaluate ∫ (x + 1)2
dx
log6 6 ((sinx)6cos x ) cos x
Q.10 ∫ dx
sinx
esin z 3
Q.26 Evaluate ∫ cos2 x (x cos x − sin x)dx
 x2 + 1 log(x2 + 1) − 2logx  
Q.11    
∫ x 4  dx dx
  Q.27 Evaluate ∫
1 − 2x − x2
sinx
Q.12 If f(x) = the antiderivative of
sin2 x + 4 cos2 x dx
Q.28 ∫ dx
sec x + cosecx
 1   1  
 g ( x )  + c is then g (x) is equal to
−1
  tan  
 3  3   2x2 + 3x + 4dx
Q.29 Evaluate ∫
cot x dx dx
Q.13 ∫ Q.30 Evaluate ∫ x(xn + 1)
(1 − sinx)(sec x + 1)
cos x − sinx
3x2 + 1 Q.31 ∫ dx
Q.14 ∫ dx 7 − 9 sin2x
(x2 − 1)3
cot x − tanx
(ax2 − b)dx Q.32 ∫ dx
Q.15 ∫ 1 + 3sin2x
x c2 x2 − (ax2 + b)2
4x5 − 7x 4 + 8x3 − 2x2 + 4x − 7
Q.33 ∫ dx
m tan−1x x2 (x2 + 1)2
e
Q.16 Evaluate ∫ (1 + x2 )3/2 dx Q.34 ∫
dx
cos3 x − sin3 x

−1 x x2
Q.17 Evaluate ∫ sin a+ x
dx Q.35 ∫
(x cos x − sinx)(x sinx + cos x)
dx

Q.18 cos θ + sin θ


Q.36 ∫ cos2θ.In dθ
cos θ − sin θ
(e x
− e− x 
)cos  e

x
+ e− x
+
π
(
+ e
4
x
+ e− x
) cos  e x
− e− x
+
π

4
∫ dx
x Q.37 Match the columns
2
x +x
Q.19 ∫ dx
(e + x + 1)2
x
Column I Column II

 (x2 + 1) + x 4 + 1 
ecos x (x sin3 x + cos x) (A) ∫
x4 − 1
dx (p) log
In   ++Cc
Q.20 ∫ dx  x 
sin2 x x2 x 4 + x2 + 1  

5x 4 + 4x5 2
(B) ∫ x − 1 dx 1 1  x 4 x+41+−1 −2x2 x 
Q.21 ∫ dx (q) C C− − In
log  2 
2 2 In
  (x −21) 
(x5 + x + 1)2 x 1 + x4   (x − 1)  

a2 sin2 x + b2 cos2 x 1 + x2
Q.22 ∫ dx (C) ∫ dx (r) C − tan−1 

1+
1 
−1 
4 2 4 2
a sin x + b cos x (1 − x2 ) 1 + x 4  4
x 
 
2 2 . 3 8 | Indefinite Integration

Column I Column II sec x + tanx − 1


Q.5 ∫ dx equals
tanx − sec x + 1
1
(D) ∫ dx x 4 + x2 + 1
(s) +
+Cc
(1 + x )4
1+x −x4 2
x (A) log [(sec x + tan x)] + log sec x + c
(B) log [sec x-tan x] – log cos x + c
Exercise 2 (C) log (sec x + tan x) – log sec x + c
D) –log (sec x + tan x) + log cos x + c
Single Correct Choice Type

tan−1 x − cot −1 x (x2 − 3)


Q.1 If ∫ dx is equal to : Q.6 ∫ ex dx
−1 −1
tan x + cot x (x + 3)2
44 −1−1 22 x 6
(A) x tan
x tan x+x + log
n(1
n(1
++x2x)2−) −
x+ C Cc
x+ (A) ex ⋅ + Cc (B) ex (2 − ) + Cc
ππ ππ x+3 x+3
44 −1−1 22
(B) x tan
x tan x−x − log
n(1 + +x2x)2+) +x +
n(1 x+ c
CC 6 3
ππ ππ (C) ex (1 − ) + c (D)
C ex + Cc
x+3 x+3
44 −1−1 22
(C) x tan
x tan n(1+ +x2x)2+) +x x+ +CC
x x+ + log
n(1 c
ππ ππ
44 22 1 − cos x
(D) x tan −1−1
x tan n(1+ +x2x)2−) −x x+ +CcC
x x− − log
n(1 Q.7 ∫ dx where 0 < α < x < π, equals
ππ ππ cos α − cos x

 α x  cos(x/ 2) 
x2 − 4 2n  cos − cos  + c
(A) 2log (B) 2cos−1  +c
Q.2 ∫ dx equals  2 2   cos(α / 2) 
x 4 + 24x2 + 16
 αα xx  cos(x/ 2) 
 x2 + 4   x2 + 4  (C) 22 22log
nncos cos ++cc (D) −2sin1 
cos −−cos +c
1 1  22 22  cos(α / 2) 
(A) tan−1   + Cc (B) − cot −1   + Cc
4  4x  4  x 
   
3x 4 − 1
1  4x + 4  1 2 x +4 2 Q.8 Primitive of w.r.t. x is :
(C) − cot −1   + Cc (D) cot −1   + Cc (x 4 + x + 1)2
4  x  4  x 
   
x x
(A) + c (B) − +c
4 4
x + x +1 x + x +1
( x − 1)
2

Q.3 ∫ dx equals
x + 2x2 + 1
4 x +1 x +1
(C) + c (D) − +c
4 4
3 5 3 x + x +1 x + x +1
x x x +x +x+3
(A) +x+ + Cc (B) + Cc
3 2
x +1 3 x2 + 1 ( ) 3x 3x
Q.9 If ∫ e cos 4x dx= e (A sin 4x + B cos 4x) + c
x5 + 4x3 + 3x + 3 Then
(C) +C
c (D) None of these
(
3 x +1 2
) (A) 4A=3b (B) 2A=3B
(C) 3A=4B (D) 4B+3A=1
x4 − 4
Q.4 ∫ dx equals
x2 4 + x2 + x 4 pxp + 2q−1 − qxq−1
Q.10 The evaluation of ∫ dx is
x2p + 2q + 2xp + q + 1
4 + x2 + x 4
(A) c
+C (B) 4 + x2 + x 4 +Cc xp xq
x (A) − + Cc (B) + Cc
xp + q + 1 xp + q + 1
4 + x2 + x 4 4 + x2 + x 4
(C) + Cc (D) + Cc xq xp
2 2x (C) − + Cc (D) +C
c
xp + q + 1 xp + q + 1
M a them a ti cs | 22.39

= g(x), then ∫ f −1 (x)dx is equal to −


Q.11 If ∫ f(x)dx ex e− x
Q.3 Let I =
∫ dx, J =
∫ dx
e4x + e2x + 1 e−4x + e−2x + 1
(A) g−1 (x) (B) xf −1 (x) − g(f −1 (x))
Then, for an arbitrary constant c, the value of J-l equals
−1 −1 −1
(C) xf (x)g (x) (D) f (x)  (2008)

1 e4x − e2x + 1 1 e2x + ex + 1


x (A) log + c   (B) log +c
Q.12 Primitive of 3 w.r.t x is − 2 e4x + e2x + 1 2 e2x − ex + 1
(x − 1)4
4

1 1 1 e2x − ex + 1 1 e4x + e2x + 1


3 1 3 3 1 3 (C) log + c   (D) log +c
2 e2x + ex + 1 2 e4x − e2x + 1
(A) 1 + 4  + Cc (B) −  1 + 4  + Cc
4 x −1 4 x −1
1 1 Q.4 ƒ(x) is the integral of
4 1 3 4 1 3
(C)  1 + 4  + Cc (D) − 3  1 + 4  + Cc 2sinx − sin2x
3 x −1  x −1 , x ≠ 0 find lim ƒ'(x).  (1979)
x3 x →0

Q.13 If ∫ eu . Sin 2x dx can be found in terms of known Q.5 Evaluate the following integrals  (1980)
functions of x then u can be :
(A) x (B) sin x (C) cos x (D) cos 2x
1  x2
(i) ∫ 1 + sin  x dx (ii) ∫ dx
2  1−x

Q.6 Evaluate ∫(elog x + sinx)cos x dx.  (1981)


Previous Years’ Questions
cos3 x + cos5 x (x − 1)ex
Q.1 The value of the integral ∫ dx is Q.7 Evaluate ∫ dx .  (1983)
2
sin x + sin x 4 (x + 1)3
 (1995) dx
−1
Q.8 Evaluate ∫ .  (1984)
(A) sinx − 6 tan (sinx) + c x (x + 1)3/ 4
2 4

(B) sinx − 2(sinx)−1 + c


1− x
(C) sinx − 2(sinx)−1 − 6 tan−1 (sinx) + c Q.9 ∫ dx.  (1985)
1+ x
(D) sinx − 2(sinx)−1 + 5 tan−1 (sinx) + c
sin−1 x − cos−1 x
x Q.10 Evaluate ∫ dx. (1986)
Q.2 Let ƒ(x)
= for n > 2 and sin−1 x + cos−1 x 
(1 + xn )1/n

g(x) fofo....of (x). Then, ∫ xn−2g(x)dx equals  (2007) (cos2x)1/2


  Q.11 Evaluate ∫ dx. (1987)
f occurs n times sinx 

1
1 1− Q.12 Find the indefinite integral
(A) (1 + nxn ) n + c
n(n − 1)
 1 log(1 + 6 x ) 
1 ∫ 3 x + 4 x
 +
3
x + x 
 dx  (1992)
1 1− 
(B) (1 + nxn ) n + c
n−1

1
1 1+
(C) (1 + nxn ) n + c
n(n + 1)

1
1 1−
(D) (1 + nxn ) n + c
n+1
2 2 . 4 0 | Indefinite Integration

x3 + 3x + 2 sec2 x
Q.13 Integrate ∫ (x2 + 1)2 (x + 1) dx  (1999) Q.17 The integral ∫ 9
dx equals
( sec x + tanx ) 2

−1
 2x + 2  (for some arbitrary constant K)  (2012)
Q.14 Evaluate ∫ sin 
 2
 dx. 

(2000)
 4x + 8x + 13 
1 1 1 2
(A) −  − ( sec x + tanx )  + K
Q.15 For any natural number m, evaluate  (2002) ( sec x + tanx )
11
2 11 7 

∫(x3m + x2m + xm )(2x2m + 3xm + 6xm )1/m dx, x > 0. 1 1 1 2


(B)  − ( sec x + tanx )  + K
11 11 7
ex e− x ( sec x + tanx ) 2  
Q.16 Let I ∫=
= dx, J ∫ e−4x + e−2x + 1 dx .
e4x + e2x + 1 1 1 1 2
(C) −  + ( sec x + tanx )  + K
11 11 7
Then, for an arbitrary constant C, the value of J − I ( sec x + tanx ) 2  
equals  (2008)
1 1 1 2
1  e4x − e2x + 1  1  e2x + ex + 1  (D)  + ( sec x + tanx )  + K
(A) log  +c (B) log  +c ( sec x + tanx )
11
2 11 7 
2  e4x + e2x + 1  2  e2x − ex + 1 
 

1  ex − e− x − 1  1  e4x + e2x + 1 
(C) log  +c (D) log  +c
2  ex + e− x + 1 
  2  e4x − ex + 1 

PlancEssential Questions
JEE Main/Boards JEE Advanced/Boards

Exercise 1 Exercise 1
Q.14 Q.19 Q.23 Q.25 Q.3 Q.11 Q.18 Q.20

Q.29 Q.32 Q.35 Q.38 Q.30 Q.37 Q.38

Exercise 2 Exercise 2
Q.4 Q.10 Q.13 Q.15 Q.1 Q.5 Q.7 Q.10

Q.12
Previous Years’ Questions
Q.5 Q.7 Q.9 Previous Years’ Questions
Q.1 Q.2 Q.4 Q.10

Q.12
M a them a ti cs | 22.41

Answer Key

1 1
JEE Main/Boards Q.17
2
x − log cos x − sinx + A
2

Exercise 1 x2 1
Q.18 ƒ(x)= + −1
Q.1 tan x – sec x + A 2 x

ax 1
Q.2 x + tan−1 x − 2sin−1 x + 5 sec−1 x + +A Q.19 (2x3m + 3x2m + 6xm )m+1/m
loga 6(m + 1)

x2 1 1 3 x
Q.3 +A Q.20 log(x2 + 1) − log(x + 1) + tan−1 x +
2 4 2 2 2
x +1

Q.4 log x + log(sec x) + A 1 3 +p



Q.21 log + tan−1 (q) + c
2 3 3 −p
Q.5 log 3cos x + 2sinx + A
1 2 + 1 − tan2 x
⇒ − log | cot x + cot2 x − 1 | +
Q.22 log +c
2 2 − 1 − tan2 x
Q.6 2 x2 − x − 1 + A
3
1 1  2   1  2
1  Q.23 −  1 +  log  1 + 2  −  + Cc
Q.7 (1 − 3x)3/2 + (5 − 3x)3/2  + A 3 x2    x  3
18  

1 3
1 1
Q.8 sin(ex ) + A Q.24 x + log sinx − cos x + Cc
3 2 2

Q.9
x
+ Cc x2 −1  1 2a − x  a2  1 x x x2 
2 Q.25 sin  + sin − 1− +c
1+ x 2 2 a  2  2a 2a 4a2 
 
1
Q.10 − +A
2(2 tanx + 3) 1
Q.26 tan3 x + 2 tanx − cot x + Cc
3
5 5
Q.11 − cos8/5 x + cos18/5 x + A
8 18 2  a−b 
=Q.27 tan−1  tan(θ / 2) + c1
a2 − b2  a + b 
− logx 1
Q.12 − +A
x x 1 1 1
=
Q.28 ∫ 2 dθ= θ + c= sec−1 (x2 ) + c
2 2
 x x
Q.13  tan−1  (a + x) − a + Cc −2
 a  a = cos α + cot x sin α + c
  Q.29
sin α
x
Q.14 e tanx + A
 x 1 
Q.30= 2  − +c
 1− x 1 − x 
Q.15 log 2logx + 1 − log 3logx + 2 + A 

1 1 1
2∫
Q.31= (cos 3x − cos 2x)dx = sin 3x − sin 2x + c
5 3 1 3 2
Q.16 log x + 3 + log x − 1 − +A
8 8 2(x − 1) 1 1
Q.32=− log | x | +2 log | x − 1 | + − +c
x − 1 (x − 1)2
2 2 . 4 2 | Indefinite Integration

3 3 1
Q.33 2logt − log(2t − 1) − c, where t = x + x2 − x + 1
+C
2 2 (2t − 1)

= log(ex + e2x − 4 ) − log 2 +=


Q.34 c log(ex + e2x − 4 ) + c'

1 log x 1 x 1
Q.35 I =
− + log + +c
2 (1 + x)2 2 1 + x 2(1 + x)

x2 + x + 1 2 2x + 1
Q.36 log + tan−1 +c
x −1 3 3

Exercise 2
Single Correct Choice Type

Q.1 A Q.2 B Q.3 C Q.4 B Q.5 C Q.6 C

Q.7 A Q.8 C Q.9 A Q.10 C Q.11 B Q.12 B

Q.13 B Q.14 D Q.15 A

Previous Years’ Questions


3 35 1 x
Q.1 C Q.2 A =
− , B= and C ∈ R Q.3 log(sinx − cos x) + + c
and
2 36 2 2

1 cos 4x cos2x cos6x 3x sin 4x sin8x


Q.4 tan−1 (x2 ) + c Q.5 − − + + sin2x + tanx − 2x + − +
2 16 8 24 128 128 1024

1  a2   tanx − cot x 
Q.6  a + bx − 2alog(a+ bx) − + c  Q.7 2 tan−1  +c
b3  a + bx 


 2 

dt xex x
xe 11
Q.8 ∫ 2 = ln
logn ++ ++cc Q.9 2[cos−1 x − log| 1 + 1 − x | − 1 log| x |] + c
xx xx
t (t − 1) 11++xexe 11++xe xe 2

Q.10 C Q.11 D Q.12 D Q.13 C Q.14 D Q.15 D

Q.16 A

JEE Advanced/Boards

Exercise 1
 
 x   x   x   

 n sec xx   x
3ln sec x   x
6ln sec x   sec(logx)  − x tan log | 2 | + c
22n2nsec
Q.1 (i) 2log  sec
 − −
 −
3ln
3log
3ln sec
sec
 2     3 − −
 −
6ln
6log
6ln sec
sec
 3     6 + +

C
 6 
+
C C
+ c (ii) log
 2 2 3 6   x  
       sec  log  
  2
M a them a ti cs | 22.43

x x
−2 x −β 11++xx 111+++xxx 111+++xxx  1 + x  x e
Q.2 ⋅ ++Cc nn
Q.3 log nnn
log  ––log
nnn
 –+nc   Q.4   −   ++Cc
α −β x −α 11−−xx 111−−−xxx 111−−−xxx  1 − x  e x

 cos x x 
 cos
Q.5 cos
cosa.arc
a.arccos
cos  − sina
− sina .nnsinx
. log  sinx 2 2
 + + sinsinx −x sin
2 2 
− sin + Cc
a a+ C
cosa
cosa   

x 4x 4 3 3 2 2 11 2 2 −1
tt44 tt22 11 2 −1 

Q.6 + x+ x− x− x+ 5x n(x
n(x+ 1)
+ + log
+ 5x + 1) + 3tan−x1 +x +
+ 3tan C
+Cc Q.7 66 −− ++tt++ log n(1++tt2))−−tan
n(1 tan−1tt ++CC where
where tt =
=xx1/6
1/6
44 22
 44 22 22 
+ c where t =x1/6

 x  logx
Q.8 (a + x)tan−1  − ax  + c Q.9 sec−1 x − +c
 a  x2 − 1

3/2
1  log2 (sinx)  x  1 1   1  2
Q.10  + log  tan + cos x   + c Q.11 – 1 + 2  log  1 + 2  –  + c
6  log36  2   3 x    x  3

dt 1 sec x 1 1 x x 1 1 2 x2 x x x
∫ f(x)dx
Q.12 = ∫= tan−1 +c Q.13 ln tan
ln tan+ +tantan
log + tan
+ tan+ C+ Cc
t2 + 3 3 3 2 2 2 24 4 2 2 2 2

x  ax2 + b 
Q.14
C− + C Q.15 sin−1  +k
(x2 − 1)2  cx 
 
−1 x −1
em tan  1 x  em tan x (m + x)
Q.16 1
= m ⋅ + = +c +c
m2 + 1  x2 + 1

x2 + 1  (m2 + 1) x2 + 1

 −1 x  a + x  x
=Q.17 a tan


a a 
− +c
a  (
Q.182 2 cos (e− x
)(
) sin(e x ) + cos (e x
) c
) +C

 x +1  1
Q.19 −log
− ln  + 1 − + Cc Q.20C − ecos x (x + cosec x) + c
x
 e   x + 1 + 1
 x 
 e 
x +1 x5 1   a2 tanx  
Q.21 C − or C
c+ Q.22  x + tan−1    + Cc
5
x + x +1 x
x + x +1 a + b 
2 2  b2  
 

1 1 x x1 1  1 − 1 − x2 
Q.23 log
ln(cos
ln(cos
x +xsinx)
+ sinx)
+ ++ +(sin2x
(sin2x
+ cos2x)
+ cos2x) c
+ C+ C Q.24 (xlogx − x)Sin−1 x + 1 − x2 logx − log   + Cc
4 4 2 28 8  x 
 

 x −1   1 
= ex f(x)=
Q.25 + c ex  +c = ez f(z) +=
Q.26 c ez  sin−1 z − c esin x (x − sec x) + c
 +=
 x +1  
 1 − z2 

−1  1 + x  1 1  11   
 x x π π
=Q.27 sin   + c Q.28 sinx
sinx− −
cos
cos
x−x − log
nntan
tan
  + +  + +C C c
 2  2 2  22 2 2 8 8
  

 2x2 + 3x + 4 
4x + 3 23 4x + 3
Q.29 I
= 2x2 + 3x + 4 + log  +  +c
8 16 2  4 2 
 
2 2 . 4 4 | Indefinite Integration

1 z −1 1 xn 11 (4(4++3sinx
3sinx++3cos
3cosx)x)
=Q.30 log = +c log + c Q.31 n
n
log ++CCc
n z n n
x +1 24
24 44−−3sinx
3sinx−−3cos
3cosx)x)

−1
 2 sin2x  7 6x
Q.32 tan   + Cc Q.33 4logx
4nx + + 6 tan−1 (x) + + Cc
 sinx + cos x  x 1 + x2
 
22 −1−1 11 22++sinx
sinx++cos
cosxx x sinx + cos x
Q.34 tan (sinx
tan (sinx++cos
cosx)x)++ nn
log ++CC
c Q.35 log
n +c
33 33 22 22−−sinx
sinx−−cos
cosxx x cos x − sinx

1 11  cos
 cos θ+ + sin
θ sin θ θ  1 11 2
Q.36 (sin2
(sin2 ))nn  cos θ + sin θ− −−n(sec
θ)θlog
n n(sec
2θ2)2θ+θ) )C
++CcC = +c Q.37 A → s; B → p; C → q; D → r
2 22
(sin2 θ
cos
 
cosθ θ
− − sin
sin θ θ  2 2 logn(sec
cos2θ
  cos θ − sin θ  2

Exercise 2
Q.1 D Q.2 A Q.3 D Q.4 A Q.5 A Q.6 C Q.7 B

Q.8 B Q.9 C Q.10 C Q.11 B Q.12 B Q.13 A

Previous Years’ Questions

Q.1 C Q.2 A Q.3 C Q.4 1

x x  2 1 
Q.5 (i) 4 sin − 4 cos + c (ii) −2  1 − x − (1 − x)3/2 + (1 − x)5/2  + c
4 4  2 5 

cos2x ex (x 4 + 1)1/ 4
Q.6 x sinx + cos x − +c Q.7 +c Q.8 − +c
4 (x + 1)2 x

2
Q.9 −2 1 − x + cos−1 x + x(1 − x) + c Q.10 [ x − x2 − (1 − 2x)sin−1 x ] − x + c
π

1 2 + 1 − tan2 x
Q.11 − log| cot x + cot2 x − 1 | + log +c
2 2 − 1 − tan2x

Q.12 3 x2/3 − 12 x7/12 − 4 x1/2 − 12 x5/12 + 1 x1/3 − 4x1/ 4 − 7x1/6 − 12x1/12 + (2x1/2 − 3x1/3
2 7 3 5 2
−6x + 11)ln(1 + x ) + 12ln(1 + x1/2 ) − 3[ln(1 + x1/6 )]2 + c
1/6 1/6
−6x1/6 +11)log(1 + x1/6) + 12log(1 + x1/2)−3[log(1 + 1/6)]2 + c

1 1 3 x  2x + 2  3
Q.13 − log| x + 1 | + log| x2 + 1 | + tan−1 x + + c Q.14 (x + 1)tan−1  2
 − log(4x + 8x + 13) + c
2 4 2 2
x +1  3  4

1
Q.15 ⋅ (2x3m + 3x2m + 6xm )(m+1)/m + c Q.16 C Q.17 C
6(m + 1)
M a them a ti cs | 22.45

Solutions

JEE Main/Boards ⇒ (2x – 1)dx = dt


1
dt – 2
Exercise 1 ∫ t
= ∫ t 2 dt = 2 t + c = 2 x – x – 1 + c

sec x sec x(sec x − tanx) dx


Sol 1: ∫ sec x + tanx dx = ∫ sec2 x − tan2 x
dx Sol 7: ∫ 1 − 3x − 5 − 3x
= ∫ (sec2 x − sec x tanx)dx = tanx – secx + C
1 − 3x + 5 − 3x
= ∫ (–4)
dx

  1 1
1 + 1 – 2 1 x = – ∫ 1 − 3xdx – ∫ 5 − 3xdx
Sol 2: ∫  1 + x2 2
+ 5
2
+ a

dx
4 4
 1 − x | x | x − 1 
1 2
( ) 1 2
( )
3/2 3/2
dx 2dx dx = × 1 – 3x + × 5 − 3x
+ ∫ ax dx 12 3 12 3
= ∫ dx + ∫ 1 + x2 – ∫ 2
+ 5∫
2
1−x | x | x −1
1 
( 1 − 3x ) + (5 − 3x )  + c
3/2 3/2
=  
ax 18 
= x + tan–1x – 2sin–1x + 5sec–1x + +c
loga
2 x3 3
Sol 8: ∫x e cos(ex )dx
sin2x
Sol 3: = tanx 3
ex = t ⇒
3
3x2ex dx = dt
1 + cos2x
1 1 1 3
 sin2x 
−1 \ ∫ cos tdt = sint + c = sinex + c
∫ tan  1 + cos2x  dx =
−1
∫ tan tanxdx 3 3 3

x2 sec2 (2 tan−1 x)
= ∫ xdx = 2
+c Sol 9: ∫ dx
1 + x2
1 + tanx 2
2tan–1x = t ⇒ dx = dt
Sol 4: ∫ x + logsec x dx 1 + x2

Put x + logsecx = t 1 1 1
∫ sec2 tdt = tant + c = tan(2 tan−1 x) + c
2 2 2
 1 
⇒ 1 + sec x tanx  dx =
dt
 sec x  1 2 tan(tan−1 x) x
= +c = +c
2 1 + tan (tan x)
2 −1
1 + x2
dt
∴∫ = logt + c = log(x + logsecx) + c
t
dx sec2 x
Sol 5: Put 3cosx + 2sinx = t
Sol 10: ∫ (2sinx + 3cos x)2 = ∫ (2 tanx + 3)dx
dt Put 2tanx + 3 = t ⇒ 2sec2x dx = dt
–3sinx + 2cosx =
dx
1 dt 1 1 1
2 ∫ t2
dt ∴ = – +c = – × +c
∴I = ∫ t = logt = log(3cosx + 2sinx) 2t 2 (2 tanx + 3)

3/5
2x − 1 Sol 11: ∫ cos x sin3 xdx
Sol 6: ∫ dx
x2 − x − 1 3/5
x(1 – cos2 x)sinxdx
= ∫ cos
Put x2 – x – 1= t
2 2 . 4 6 | Indefinite Integration

∫ ( cos )
3/5
= x – cos13/5 x sinxdx  x x
=  tan−1  (a + x) − a +c
 a  a
Put cosx = t; –sinxdx = dt  

∫ (t )
13/5 5 18/5 5 8/5
= – t3/5 dt = t – t +c  2 + sin2x 
x
18 8 Sol 14: ∫e  dx
 1 + cos2x 
5 5
= (cos x)18/5 – (cos x)8/5 + c 1 + cos2x = 2cos2x
18 8
2 + sin2x = 2 + 2sinx cosx
logx
Sol 12: ∫ x2
dx  2 + 2sinx cos x 
⇒ ∫ ex  x
(sec2 x + tanx)dx
 2cos2 x
 =

∫e
1  dlogx 1 
logx ∫ dx – ∫  ∫ dx  dx
x2
 dx x 2
 tanx = t ⇒ sec2xdx = dt
x
 1 1  1
= (logx)  –  – ∫ ×  –  dx + c
∴This is of form ∫e (f(x) + f '(x)dx = exf(x)
 x x  x ∴I = extanx + c
1 1 1
= – logx + ∫ dx + c = – (logx + 1) + c dx
x 2 x
x Sol 15: ∫ x[6(logx)2 + 7logx + 2]
−1 x
Sol 13: ∫ sin a+ x
dx , a > 0 1
logx = t; dx = dt
x
x = atan2θ ⇒ dx = 2atanqsec2qdq dt dt
∫ (6t2 + 7t + 2) = ∫ (6t2 + 3t + 4t + 2)
−1 atan2 θ
∫ sin asec2 θ
2a tanθ sec2θ dq dt  3 2 
= ∫ (3t + 2)(2t + 1) =– ∫  (3t + 2) – (2t + 1)  dt
= 2a∫ (sin−1 sin θ)tan θ sec2 θ dθ
1 1
tanθ = t ⇒ sec2qdθ = dt = –3 ∫ dt + 2∫ dt
3t + 2 2t + 1
= 2a∫ t tan−1 tdt
1 22
= – 3 log(3t + 2) + log
n(2t+ +
n(2t 1)1)+ +c c
 1 1  3 22
= 2a tan−1 t ∫ tdt – ∫ × t2dt 
 2 1+t 2
 2  2t + 1  2  2logx + 1 
=  + cc = log
n(2t + 1) n(2t + 1) + c  + c
 
2  3t + 2  2  3logx + 2 
t2 1
= 2a (tan–1 t) – t − tan−1 t   + c
 2 2    = log | 2log x + 1| – log | 3 log x + 2 | + c

1 1 
= at2tan–1t +  tan−1 t – t  2a + c
2 2  x2 + 1
Sol 16: ∫ (x + 3)(x – 1)2 dx
x
\t = tanθ = 5 1 3 1 1
a ∫ 8 × (x + 3) dx + ∫ 8(x − 1) dx + 2 ∫ (x – 1)2 dx
I=

x x 1 x 1 x 5 55 3 33 1 11
∴I = a tan−1 +  tan−1 –  2a + c = n(x
log
n(x +3)
+ +3)
n(x n(x
+ ++ n(x
3) log – –1)
n(x –1)
– ––
1) + +c+cc
a a  2 a 2 a  8
8 8 8
8 8 2(x – –1)
2(x2(x –1)
1)
1
−1 x 1 x 1 x Sol 17: ∫ (1 − tanx ) dx
= x tan +  tan−1 –  2a + c
a  2 a 2 a 
Put tanx = t ⇒ sec2xdx = dt
x x x
= xtan–1 a + atan
–1
a –a a +c dt dt
or dx = =
2
1 + tan x 1 + t2
M a them a ti cs | 22.47

1 2
2∫
= (cos2θ + 1)dθ + tan–1x
∴I = ∫ (1 – t)(1 + t2 ) dt
1 1 1 t +1 1 11 1 −1 −1  
=  tan + x+2x)2–) tan
tan (1(1
−1
– tan−x1 −x −log
n(x + 1)
n(x  + c+ c
+ 1)
= ∫
2 (1 – t)
dt + ∫
2 t2 + 1
dt 2 22 2  

1 11 1 1 −1−1 11 −1−1 2 11 −1−1 11


= – 1log n(1+ +t2t)2+) + tan = tan (1(1++xx2) )–– tan
tan tan xx–– log
n(x
n(x++1)1)++cc
n(1––t)t) + log
n(1 n(1 tan t +t +c c 44 22 22
22 44 22

1 11    1 11   1 11 2 2 2 x xx
= nn
log n 
 + ++nlog
n
 1 1+1+tan
n +tan
tanx x+x+++ +c+cc
2 22  1 1−1−tanx
tanx  2 22 2 22 1
 − tanx  Sol 21: I = ∫ sin x + sec x dx
1 1   sec secxx  xx
nn
= log  + + + +c c
2 2  1 1− −tanx
tanx  2 2 2 cos x (cos x + sin x) + (cos x − sin x)
⇒ ∫ 2 + 2 sin x cos x dx =
∫ 2 + sin 2x
dx
x x 11
= – – log
nn
| cos
| cos
x –x –
sinx
sinx| +| c+c
22 22 cos x + sin x cos x − sin x
⇒ ∫ 2 + 2 sin x
dx + ∫
2 + 2 sin 2x
dx
1
Sol 18: f’(x) = x –
x2 1
⇒ ∫ 2 + [1 − (sin x + cos x)2 ] × d(sin x − cos x)
 1  x2 1
f(x) = ∫ f '(x)dx
= ∫  x2 
 x −  dx = + +c
2 x 1
1 1
+ ∫ 2 + [(sin x + cos x)2 − 1] × d(sin x + cos x)
f(1) = + 1 + c = ⇒ c = –1
2 2
1
x2 1 ⇒ ∫( × d(sin x − cos x)
∴f(x) = + –1 3) − (sin x − cos x)2
2
2 x
1
Sol 19: ∫ (x3m + x2m + xm )(2x2m + 3xm + 6)1/mdx + ∫ (1)2 + (sin x + cos x)2 × d(sin x + cos x)
Put xm = t and integrate.
1 1
x3 + 3x + 2 ⇒ ∫( 2
3) − p 2
dp + ∫
1 + q2
2
dq
Sol 20: ∫ (x2 + 1)2 (x + 1) dx

x3 + 3x + 2 = x3 + x + 2x + 2 1 3 +p
⇒ log + tan−1 (q) + c
= x(x2 + 1) + 2(x + 1) 2 3 3 −p

x(x2 + 1) + 2(x + 1) Where p =


sin x − cos x & q =
sin x + cos x
I= ∫ (x2 + 1)2 (x + 1) dx
x 1
= ∫ + 2∫ dx
2
(x + 1)(x + 1) (1 + x2 )2 Sol 22:
2
(x + 1) − 1  1  cos 2x cos2 x − sin2 x
= ∫ (x2 + 1)(x + 1) dx + 2∫  1 + x2  dx =I ∫ =
sin x
dx ∫ =
sin2 x
dx ∫ cot2 x − 1dx

On putting
1 dx 1
= 2∫ dx – ∫ +∫ dx cot x= sec θ & − cosec2 dx= sec θ tan θ dθ
(1 + x2 )2 (x2 + 1)(x + 1) (1 + x2 )

Put x = tanq sec θ tan θ


We =
get, I ∫ sec2 θ − 1 × dθ
dx = sec2qdq − cosec2 x
sec θ tan2 θ
2
sec θ 1  (x − 1) 1  −∫
= dθ
= 2∫ dθ + tan−1 x + ∫  − dx 1 + sec2 θ
4
sec θ 2  (1 + x ) x + 1 
2
2 2 . 4 8 | Indefinite Integration

(cosx + sinx)dx = dt
sin2 θ 1 − cos2 θ
= −∫ dθ = − ∫ dθ
cos θ + cos3 θ cos θ(1 + cos2 θ) 1 1
⇒ x + log(sinx − cos x) + c
2 2
(1 + cos2 θ) − 2 cos2 θ cos θ ∴ 1 1
= ∫ cos θ(1 + cos θ) 2
= − ∫ sec θdθ + 2∫
2
1 + cos θ

∫ t ∫=
dt =
t
dtlogt
nt=
= nt=n(sinx
n(sinx
log – cos
– cos
x) x)

d(sin θ) −1
 1 2a − x 
⇒ − ∫ sec θdθ + 2∫
2
1 + cos θ
Sol 25: ∫ x sin  .
2 a
dx

 
d(sin θ)  1 2a − x  
⇒ − ∫ sec θ dθ + 2∫    dsin−1  
2 − sin2 θ −1 1 2a − x 2 a
sin  .  ∫ xdx − ∫   xdx  dx

2 a   dx  
1 2 + sin θ     
⇒ − log | sec θ + tan θ | +2 × log +c   
2 2 − sin θ
x2  1 2a − x 
⇒ sin−1  
1 2 + 1 − cos2 θ 2 2 a 
⇒ − log | sec θ + sec2 θ − 1 | + log +c  
2 2 − 1 − cos2 θ  
 
2
x 1 1 –1 
1 2 + 1 − tan2 x – ∫ × × ×  dx
− log | cot x + cot2 x − 1 | +

2
log +c
 2  2a − x  2a 2 2a − x 
1−
2 − 1 − tan2 x   
  4a2  

x2  1 2a − x 
Sol 23: ∫
(
x2 + 1 log(x2 + 1) – logx2 )dx ⇒
2
sin−1 
2
 a



x4
2a x2 1
 1  1 
+
8a ∫ ×
2a – x
dx
 1 + 2 log  1 + 2   4a2 – 2a + x
 x  x  
=∫ dx
x3
1
1+
1 2
= t ⇒ – dx = dt
Sol 26: ∫ cos4x sin2 x dx
2
x x3
1 1 t3/2 1 t3/2  (sin2 x + cos2 x)2
– ∫ t logtdt – (logt)
=
2 2  3/2
–∫ × dt 
t 3 / 2 
∫ sin2 x cos4 x
dx

1 2 2 1/2   sin4 x + cos4 x + 2sin2 x cos2 x 


=
2 3
3/2
 – (log)t + ∫ t dt 
3
= ∫  sin2 x cos4 x
 dx

  

   sin2 x 1 2 
1 2 
3/2 3/2
1  1 4 1 
= – log  1 +  1 +  +  1 +  + c   = ∫  cos4 x + sin2 x + cos2 x  dx
2 3  x2   x2  9 x2    
  
2
1 1
3/2
  1  2 = ∫ tan x sec2 xdx + ∫ cosec2 xdx + 2∫ sec2 xdx
= – 1 +  log  1 + 2  –  + c
3 x2    x  3 tan3 x
= – cotx + 2tanx + c
3
sinx
Sol 24: ∫ sinx − cos x dx 1
Sol 27: I
= ∫ (a + b cos θ)2 dθ
1 sinx − cos x + sinx + cos x 1 1 dt
2∫
= dx = ∫ 1dx + ∫
(sinx − cos x) 2 2 t sin θ
Let P =
(a + b cos θ)
Put sinx – cosx = t
M a them a ti cs | 22.49

dP (a + b cos θ)cos θ − sin θ(0 − b sin θ) sec θ tan θ sec θ tan θ


= =dx = dθ dθ
dθ (a + b cos θ)2 2x 2 sec θ
Now, from (i)
a cos θ + b(cos2 θ + sin2 θ) a cos θ + b  
= dx  1  sec θ ⋅ tan θdθ
(a + b cos θ)2 =
(a + b cos θ)2 ∫ 4 ∫ 2  2 sec θ
x x −1  sec θ sec θ − 1 
Y −a
Let a + b cos θ= Y ⇒ cos θ=  1  sec θ tan θ
= b ∫  
 sec θ ⋅ tan θ  2 sec θ

 Y −a  
a +b
dP  b  aY + b2 − a2 1 1 1
sec−1 (x2 ) + c
= =
dθ  2
= ∫ 2 dθ= 2
θ + c=
2
 Y − a  bY 2
 a + b  
  b 
dx
Sol 29: I = ∫
dP a  1   b2 − a2  1 sin3 x sin (x + α )
⇒ =  + 
dθ b  Y   b  Y 2
1
= ∫ 3
dx
2 sin x[sin x cos α + sin α cos x]
dP a  1   b2 − a2  1 
⇒=  + 
dθ b  a + b cos θ   b   a + b cos θ  1
 = ∫ 4
sin x(cos α + cot x sin α )
dx
Integrating,

a 1 b2 − a2 1 1 1
∫ b ∫ (a + b cos θ) sin α ∫ cos α + cot x sin α
=P dθ + dθ =− d(cos α + cot x sin α )
b a + b cos θ

−(b2 − a2 ) 1 a 1 1 1

b ∫ (a +=
b cos θ)2

b ∫ a + b cos θ
dθ − P =−
sin α ∫ t
dt ; where t = cos α + cot x sin α

1 b a 1  −2
⇒ ∫
2  b ∫ a + b cos θ
= dθ dθ =
− P cos α + cot x sin α + c
2 2
(a + b cos θ) (a − b )   sin α

b a sin θ  1
=  I1 −
b (a b cos )
+c Sol 30: I = ∫ dx
a − b2
2
 + θ  (1 + x ) x − x2

1 put x= sin2 θ & dx= 2sin θ cos θ dθ


Where I1
= ∫ a + b cos θdθ
2sin θ cos θdθ 1 − sin θ
⇒ I
= ∫ 2
= 2∫
4 cos2 θ

2  a−b  (1 + sin θ) sin θ − sin θ
tan−1  tan(θ / 2) + c1
a2 − b2  a + b 
 x 1 
=
⇒ 2(tan θ − sec θ) + c = 2  − +c
 1− x 1 − x 

Sol 28: [Here, x4 =
−1 (x2 )2 − 1 , which is of the form,
cos 8x − cos 7x
x2 − a2 hence substitution x = sec θ may be tried]
2 Sol 31: I = ∫ 1 + 2 cos 5x
dx

dx dx 5x 5x
Now, ∫ = ∫  …(i) 1
2sin
2
cos 8x − 2 sin cos 7x
2
x x4 − 1 x (x2 )2 − 1 = ∫ dx
2 5x 5x
sin + 2 cos 5x sin
Let x2= sec θ, then 2x dx= sec θ tan θ dθ 2 2
2 2 . 5 0 | Indefinite Integration

 21x 11x   19x sin 9x  Sol 34: [Here e2x − 4 =− (ex )2 − 22 , which is of the
 sin − sin  −  sin − 
1  2 2   2 2 
= ∫ form x2 − a2 , hence substitution= ex 2sec θ may be
2 15 x
sin tried]
2
ex ex
15x 15x Now, ∫ dx = ∫ dx …(i)
2sin cos 3x − 2sin cos 2x e2x − 4 (ex )2 − 22
1 2 2
= ∫ dx
2 15 x
sin ex 2 sec θ, then ex=
Let = dx 2sec θ tan θdθ
2
1 1 1 Now from (i),
= ∫ (cos 3x − cos 2x)dx = sin 3x − sin 2x + c
2 3 2 ex 2sec θ tan θ 2sec θ tan θ
∫ 2x dx
= ∫ = dθ ∫ 2 tan θ

e −4 4 sec2 θ − 4
x3 + 1
Sol 32: I = ∫ x(x − 1)3 dx = ∫ sec θ=
dθ log | sec θ + tan θ | +c  …..(ii)

x3 + 1 A B C D ex
=+ + + ex 2 sec θ
 = ∴ sec
= θ
x(x − 1)3 x x − 1 (x − 1)2
(x − 1)3 2

put x =1 ⇒ D =2 e2x e2x − 4


∴ tan=
θ sec2 θ −=
1 1
−=
4 2
put x =
0 ⇒ A=
−1

put x =
−1 & x =
2 ⇒ B=
2 & c=
1 ex ex e2x − 4
From (ii), ∫ dx =
log
2
+
2
+c
−1 2 1 2 22x − 4
∴=I ∫ dx + ∫ dx + ∫ dx + ∫ dx
x x −1 (x − 1)2 (x − 1)3  ex + e2x − 4 
= log  +c [ ex + e2x − 4 > 0]
1 1  2 
=− log | x | +2 log | x − 1 | + − +c  
x − 1 (x − 1)2
= log(ex + e2x − 4 ) − log 2 +=
c log(ex + e2x − 4 ) + c'
1
Sol 33: I = ∫ 2
dx
 1   1 
x + x − x +1
2  ∫
Sol 35:
= I (log x) ⋅  − − −  dx
2 
 2(1 + x)   2(1 + x) x 
put t = x + x2 − x + 1 [Taking u = log x]
t2 − 1 2t2 − 2t + 2 1 log x 1 dx
⇒ x
= = and dx dt =− ⋅ + ∫ ….(i)
2t − 1 (2t − 1)2 2 (1 + x)2 2 x(1 + x)2

t2 − t + 1 1 1+x−x 1 1
⇒ 2∫
I= dt Now, = = −
t(2t − 1)2 x(1 + x)2
x(1 + x)2 x(1 + x) (1 + x)2
t2 − t + 1
A B C
let =+ + 1+x−x 1 1 1 1
t(2t − 1) t 2t − 1 (2t − 1)2
2 = − =− −
x(1 + x) (1 + x)2 x 1 + x (1 + x)2
Solving by partial fraction method, we get
dx 1 1 1 
A = 1, C =
3
and B = −
3 ∴ ∫ x(1 + x)2 =∫  x − 1 + x − (1 + x)2 dx
2 2  

3 3 1 1 x 1
=I 2 log t − log (2t − 1) − +c = log x − log(1 + x) + = log +
2 2 (2t − 1) 1+x 1+x 1+x

where t = x + x2 − x + 1 [Here x > 0 a log x occurs in the integrand]


1 log x 1 x 1
∴ From (i), I =
− + log + +c
2 (1 + x)2 2 1 + x 2(1 + x)
M a them a ti cs | 22.51

Sol 36: Let f(x) = ax2 + bx + c Exercise 2


f(0) = c = –3
Single Correct Choice Type
f(1) = a + b + c = – 3
∴ 3/2
or a + b = 0 a = 1, b = –1  x 
f(2) = 4a + 2b + c = – 1
Sol 1: (A) ∫  1 + x5  dx

or 4a + 2b = 2 3/2
x x −6
∴f(x) = x – x – 3
2 ∫ 3/2
dx = ∫ 3/2
dx
15/2  1   1 
x 1 + 5  1 + 5 
x2 − x − 3 x(x − 1) 3  x   x 
∫ dx = ∫ dx – ∫ dx
x3 – 1 (x − 1)(x2 + x + 1) (x3 − 1) 1
Put 1 + =t
x5
x 3
= ∫ (x2 + x + 1) dx – ∫ (x3 − 1) dx ⇒ –5x–6dx = dt or x–6dx = –
1
dt
5
  1 dt 1 1
  ∴I= – ∫
5 t 3/2
– × (–2)
=
5 t
+c
1 2x + 1 dx  3
∫ dx – ∫ –∫ dx
2  (x + x + 1)
2 2
 
2  (x 3
− 1)
  1  3  2 1 2 x5
  x +  +  
  ∴I= +c= +c
 2   2   5 5 1 + x5
 1
1+
1 x5
xx++ 1
11 2 11 2 2 −1 22
= log n(x
n(x2++xx++1)1)–– ×× tan tan−1 (sin8 x − cos8 x)
22 22 33 33 Sol 2: (B) − ∫ dx
22 1 − 2sin2 x cos2 x
 1 x+2 
– ∫  −  dx (sin2 x − cos2 x)(sin2 x + cos2 x)(sin4 x + cos4 x)
 x − 1 (x + x + 1) 
2 = –∫ dx
1 − 2sin2 x cos2 x
(sin2 x − cos2 x) (sin2 x + cos2 x)2 − 2sin2 x cos2 x)
1 11 2 2 2 1 1 1 −1 −2x 2x1+ 1
+2x +1 = –∫ dx
= n(xn(x
log + x+++x1)
n(x +x 1)
+– 1) tan1−1 – –n(x
– – tantan –n(x – 1)
–n(x
1)
log(x –−1)1)
2 22 3 33 3 33 1 − 2sin2 x cos2 x
1 11 2x  2x
+2x1+ 1
+1 3 33    1
+ +∫+ ∫ ∫ 2 2 + ++ 2 2  dx
 dx  dx = – ∫ (sin2 x − cos2 x)dx = – ∫ (– cos2x)dx = sin2x + c
2 22x x+xx+++x1+
2
x1+ (x 2
1 (x+(xx+++x1)
+x 1)
+ 1) 2

1 2x + 1 Sol 3: (C)
= log(x2 + x + 1) – tan− – log(x
n(x + 1)
− 1)
3 3   nx)x − x ×11 1dx  
(A) xx∫ ∫xlog
∫nxdx
=
= =xx(x((logx)x
nxdx
nxdx nx)x − ∫−−∫x∫×x ×dxdx
(nx)x    = x logx – x + cx
2 2

  xx x   
3  2x + 1 
+ tan−1   x ∫x ∫ | x| x|dx
(B)=
=
2
|dx x x2log
2
| –x2++cxcx
nn| x| x| –x
3  3 

2 x
(C) x ∫ e= dx x ex + c  = xex + cx
2
(x + x + 1) 2 2x + 1
 
= log
n + tan−1 +c
| x −1 | 3 3 dx
(D) ∫
a2 + x2

x = atanθ, dx = asec2qdq

sec2 θdθ
⇒I= ∫ =
asec θ ∫ sec θdθ
x x2 + a2
= log|secθ + tanq| + c = log + +c
a a
2 2 . 5 2 | Indefinite Integration

 x 
Sol 4: (B) ∫ 1 + 2 tanx(secx + tanx)dx dx 1 −1  
Sol 7: (A) ∫ a 2 =
a
tan  a 
b + x   
= ∫ 1 + 2 tanxsecx + 2 tan2 x dx b×  b 
b  b

= ∫ tan2 x + sec2 x + 2 tanx sec xdx  b


1
= tan−1  x +c
ab  a
 
= ∫ (tanx + sec x)dx = ∫ tanxdx + ∫ sec xdx
= –log|cosx| + log|secx + tanx| + c 1 + x7 − 2x7
Sol 8: (C) I = ∫ x(1 + x7 )
dx
= log|secx| + log|secx + tanx| + c
1 2x6 2x6
2sinx cos x 2
2 tanx sec x = ∫ x 1 + x7 dx = logx –
− ∫ 1 + x7 dx
Sol 5: (C) I = ∫ sin4 x + cos4 x dx = ∫ tan4 x + 1
dx
Let tan2x = t let1 + x7 = t ⇒ 7x6dx = dt
∴ I = 2tanx sec2x dx = dt 2 1 2
I = logx – ∫
7 t
dt = logx – nt + c
7
1
∴I= ∫ t2 + 1 dt 2
= logx – n(1 + x7 ) + c
7
∴ I = tan–1(t) + c = tan–1(tan2x) + c
1
 1  Sol 9: (A) Let log|x| = t ⇒ dx = dt
x
tan–1(tan2x) = tan–1  2  = cot (cot x)
–1 2
 cot x  t 1
tan–1(tan2x) =
π
– cot–1(tan2x) + c
∴I = ∫ 1+t
dt = ∫ 1+t –
1+t
dt
2
= –cot – (tan2x) + c1 (t + 1)3/2
= × 2 – (t + 1)1/2 × 2 + c
3
1  t +1  2
Sol 6: (C) ∫ cos3 θ dθ = 2(t + 1)1/2  − 1  + c = (t + 1)1/2 (t – 2) + c
2sin θ cos θ  3  3
2
((log ) 1/2 (log|x| – 2) + c
1/2
1 = n | x|x|| ++11)
= ∫ 2 cos7/2 θ sin1/2 θ
dθ 3

Dividing and multiplying by cos4θ Sol 10: (C)


sec 4 θ x 4 + 2x2 + 1 – 2x2 (x2 + 1)2 – 2x2
I= ∫ 2 tan 1/2
θ
I= ∫ dx = ∫ dx
x(x2 + 1)2 x(x2 + 1)2
let tan1/2θ = t 1 2x 2x
1
= ∫ x – (x2 + 1)2 dx = log|x| – ∫ (x2 + 1)2 dx
∴ × sec2 θdθ =dt
2 tan θ
let x2 + 1 = t ⇒ 2xdx = dt
2
∴I= ∫ 2 sec θdt 1

∴I = log|x| – ∫ t2 dt
t5 
= ∫ (
2 1 + t 4 dt =) 2 ×t +  + c

 5  = log|x| +
1
+ c = log|x| +
1
+c
t 1 + x2
=
5
2
(
5 tan θ + tan2 θ × tan θ + c ) A = 1, B = 1

I= 2
(tan2 θ + 5) tan θ + c
Sol 11: (B) I = ∫ 2sinx(cos2x + cos x)dx
5 2
= ∫ 2sinx(2cos x − 1 + cos x)dx
M a them a ti cs | 22.53

Let cosx = t ⇒ –sinxdx = dt 2sinxcosx = (cosx+sinx)2 – 1


3
2 4t And d(cosx + sinx) = (–sinx + cosx)dx
∴I= ∫ 2(1 − t − 2t )dt = 2t – t2 –
3
+c
4 cos3 x ∴ let cosx + sinx = t
= 2cosx – cos2x – +c
3 1
 4 cos3 x − 3cos x 
∴I= ∫ dt
t2 − 1 (t)
= cosx –   – cos2x + c
 3 
  ∴ I = sec–1(t) + c = sec–1(cosx+sinx) + c
1 1
= cosx – cos3x – cos2x – +c
3 2
Previous Years’ Questions
1 cos2x
= cosx – cos3x – +c
3 2
(x2 − 1)dx
1
Sol 1: (C) Let I = ∫ , dividing numerator
Sol 12: (B) I = ∫ sin2xcos2xcos4xcos18xcos16xdx x3 2x 4 − 2x2 + 1
2
and denominator by x5
1 cos32x
=
32 ∫ sin32xdx = – 1024 + c 1 1 
 3 − 5  dx
x x 
=∫ 
3 2 2 1
Sol 13: (B) I = ∫ x f(x )dx 2− 2 + 4
x x
let x2 = t
1 2 1  4 4
Put 2 − + t ⇒  −  dx =
= dt
2∫
∴I = tf(t)dt 3
x 2
x 4
x x5 
t 1
( ∫ f(t)dt ) dt
1

2∫
= f(t)dt – ∫ 1 dt 1 t 2
2
4∫ t 4 1
∴I
= = · +c

1 2 2 2
= x F(x ) − ∫ f(x2 )d(x2 )
2  
1 2 1
= 2− + +c
2 x 2
x4
4e2x + 6
Sol 14: (D) I = ∫ 9e2x − 4dx
4ex + 6e− x
⇒ 4e + 6 = a(9e – 4) + b× 18 × e
2x 2x 2x
Sol 2: Given, ∫ 9ex − 4e−x dx = Ax + B log (9e 2x
– 4) + c

⇒ 9a + 18b = 4 4e2x + 6
–4a = 6
LHS = ∫ 9e2x − 4 dx
3
∴a=– Let 4e2x + 6 = A (9e2x – 4) + B(18e2x)
2
27 35 ⇒ 9A + 18B = 4
18b = 4 + ⇒b=
2 36 and –4A = 6
3 35
3 35 18e2x ⇒A= – and B =
∴I = ∫ 2 36 9e2x − 4 dx
– dx + 2 36

3x 35 A(9e2x − 4) + B(18e2x )
= – – 3x+ + 35log
n(9e2x
n(9e2x− −4)4)+ +c c
∴∫ dx
2 2 3636 9e2x − 4
3 35 1
A= – and B = 9e2x − 4
= A ∫ 1dx + B ∫ dt, where t =
2 36 t
=Ax + B log (9e2x–4) + c
cos x − sinx
Sol 15: (A) I = ∫ 2sinx cos x ( cos x + sinx )
dx
3 35
= – x + log(9e2x − 4) + c
2 36
2 2 . 5 4 | Indefinite Integration

3 35 3 35
∴ A= – , B= ∴ A= – , B=
2 36 2 36
and C = any real number and C = any real number
4
sinx and I3 = ∫ sin x cos4 x dx
Sol 3: Let I = ∫ sinx − cos x dx
1 3x sin 4x sin8x
Again, let sin x = A (cos x + sin x) + B (sin x – cos x),
=
128 ∫ (3 − 4 cos 4x + cos8x)dx = –
128 128
+
1024
then A + B = 1 and A – B = 0
∴I = I1 + I2 + I3
1 1
⇒A= ,B=
2 2 cos 4x cos2x cos6x
= − – + + sin2x + tanx − 2x
1 1 16 8 24
(cos x + sinx) + (sinx − cos x) 3x sin 4x sin8x
∴I = ∫ 2 2 dx + − +
(sinx − cos x) 128 128 1024

1 cos x + sinx 1 x2
= ∫
2 sinx − cos x
dx + ∫ 1dx + c
2
Sol 6: Let I =
(a + bx)2
Put a + bx = t
1 1
= log(sinx − cos x) + x + c
2 2 ⇒ bdx = dt
2
xdx  t −a
Sol 4: Let I = ∫ 1 + x4  
b  dt 1  t2 − 2at + a2 
∴I = ∫ · = ∫   dt
t 2 b b3 t2 
1 2x  
= ∫
2 1 + (x2 )2
dx
1  2a a2  1  a2 
Putx2 = u ⇒ 2xdx = du
= ∫ 
b3 
1 − + 
t t2 
dt = 
b3 
t − 2a log t −  +c
t 
1 du 1 1 −1 2
tan−1 (u) + c = tan (x ) + c
2 ∫ 1 + u2 2
=∴ I = 1  a2 
2 =  a + bx − 2a log (a + bx) − + c
3 a + bx 
b  
Sol 5: Let I1 = ∫ sinx sin2x sin3x dx
tanx + 1
1
= ∫ (sin 4x + sin2x − sin6x)dx
Sol 7:
= Let I ∫( tanx + cot x )dx = ∫ tanx
dx
4
Put tanx = t2 ⇒ sec2x dx = 2t dt
cos 4x cos2x cos6x
= − − + 2t
16 8 24 ⇒ dx = dt
1 + t4
2 2
I2 = ∫ sec2 x ·cos2 2xdx = ∫ sec x (2cos x − 1)2 dx
t2 + 1 2t t2 + 1
=
= ∫ (4 cos2 x + sec2 x − 4)dx = ∫ (2cos2x + sec2 x − 2)dx
∴I ∫ 2 t4 + 1
= · dt 2 ∫ t 4 + 1 dt
t
1 1
= sin2x + tanx – 2x 1+ 2
1+
= 2∫ t dt = 2∫ t2 dt
1 2

( 2)
2x 2x 2  1 2
A(9e − 4) + B(18e ) t + −2+2
∴∫ dx t2 t −  +
2x
9e −4  t
1 1
1 Put t – = u ⇒ 1+ dt = du
9e2x − 4
= A ∫ 1dx + B ∫ dt, where t = t t2
t
du
∴I = 2∫
= Ax + B log (9e2x – 4) + c u + ( 2)2
2

3 35 2  u 
=– x+ log (9e2x − 4) + Cc ⇒I= tan−1  +c
2 36
2  2
M a them a ti cs | 22.55

 tanx − cot x   1 
= 2 tan−1  +c I = 2 cos−1 x − log 1 + 1 − x − log | x | + c
 2   2 
 

(x + 1)  π π
sin  x − +  dx
Sol 8: ∫ x(1 + xex )2 dx sinxdx  4 4
Sol 10: (C) 2 = 2∫
 π  π
ex (x + 1) sin  x −  sin  x − 
This can be rewritten as ∫ 2ex (1 + xex )2 dx  4  4

 π  π π
let 1 + xex =t ⇒ ex (1 + x)dx =dt = 2 ∫  cos + cot  x −  sin  dx
 4  4 4
dt
Now integration becomes ∫ t2 (t − 1)  π  π
∫ dx + ∫ cot  x − 4  dx =x + n sin  x − 4  + c
=
1 A Bt + C
⇒ =+ (using partial fraction)
t (t − 1) t − 1
2
t2 dy dy
Sol 11: (D) = y +3⇒ = dx
dx y +3
⇒ 1= t2 (A + B) + (C − B)t − C
dy dy
Comparing, we get C = -1, B =- 1 and A = 1 = y +3⇒ = dx
dx y +3
Now our integration becomes log ( y + 3) =x + c
dt 1 t +1 1 1 2
x =0⇒ y =2
∫ t2 (t − 1)= ∫ t − 1 dt − ∫ t 2
dt
= ∫ t − 1 dt − ∫ t dt − ∫ t dt ⇒ log5= 0 + c
−2 +1
t t −1 1 c = log5
= log (t − 1) − log(t) − = + C log + + Cc
−2 + 1 t t
log ( y + 3) =x + log5
In5
x
Putting t= 1 + xe , we get y + 3 = ex − log5 ⇒ y + 3 elog2 + log5
x x y + 3 = 10 ⇒ y = 7
dtdt xexe 11
∫ ∫t2t(t2 (t− −1)=1)= lnlog
ln
x x
1 1+ +xexe
++ + +c c
x x
1 1+ +xexe
Sol 12: (D)
1
1 − x dx 2 5sinx
Sol 9: Let I = ∫   · ∫ sinx − 2cos xdx
1 + x  x
 
 2 ( cos x + 2sinx ) + ( sinx − 2cos x ) 
Put x = cos θ ⇒ dx = –2cos θ sin θ d θ
2
⇒ ∫ dx
 sinx − 2cos x 
1  
 1 − cos θ  2 −2cos θ ·sin θ
∴ I ∫  1 + cos θ  · cos2 θ dθ  cos x + 2sinx 
= ∫  sinx − 2cos 
 dx + ∫ dx + k
θ θ θ θ
sin 2sin · 2sin · cos = 2log sinx − 2cos x + x + k
2· −2sin θ 2 2 2
= ∫ dθ = − ∫ dθ
θ cos θ θ ∴a=2
cos cos · cos θ
2 2
θ
2sin2
2 dθ = −2 1 − cos θ dθ
Sol 13: (C) ∫ f ( x ) dx = ψ ( x )
= −2∫ ∫ cos θ
cos θ
( )
I = ∫ x5 f x3 dx

= 2∫ (1 − sec θ)dθ = 2[ θ − log | sec θ + tan θ |] + c dt 1


Put x3 =t ⇒ x2dx = = ∫ tf ( t ) dt
3 3
 1 1 
⇒ I = 2 cos−1 x − log + −1 +c 1
 x x  = tψ ( t ) dt 
3 
2 2 . 5 6 | Indefinite Integration

JEE Advanced/Boards
=
1 3
3 
 ( ) 
1
3
( )
x ψ x3 − 3∫ x2 ψ x3 dx  + c =x3 ψ x3 dx + c ( )
  x + 1  1
Exercise 1
  1  x+ 
Sol 14: (D)
= I ∫ e x  + x  1 −  e x  dx
  x2   x x x
1
  Sol 1: (i) ∫ tan 2 tan 3 tan 6 dx
x+
= x.e x +c
  x x 
  tan + tan  
As ∫ ( xf ' ( x ) + f ( x ) ) dx =xf ( x ) + c x
= ∫ tan 1 − 

3 6 

dx
2 x
 tan 
 2 
dx
Sol 15: (D) ∫  x x x
( )
3/ 4
x2 x 4 + 1 = ∫  tan 2 − tan 3 − tan 6 dx
dx 1 x x x
∫ 3/ 4
⇒ 1+
x 4
t4
= = ∫ tan 2 dx – ∫ tan 3 dx – ∫ tan 6 dx
 1 
x3  1 + 4 
 x  x x x
= 2logsec – 3logsec – 6logsec + c
1 2 3 6
−4 =dx =4t3dt
5
x  x
tan(logx)tan  log  tan(log2)
dx  2
(ii)
x3
= t3dt ∫ x
dx

1/ 4 1
−t3dt  1  Put logx = t ⇒ dx = dt
∫ t3
=−t + c =−  1 +


x4 
+c x

∫ tant tan(t(t − log2)tan(log2)dt


n2)tan(n2)dt

2x12 + 5x3 ∫ tant


= ∫ tant – tan(t
– tan(t − n2)
(t − log2)
n2) tan( n2)
dtdt
−tan(log2)
− −tan( n2)
Sol 16: (A) ∫ dx
( x5 + x3 + 1 ) = logsect – logsec(t – log2) – xtan(log2) + c
 2 5 
 3+ 6 sec(logx)
x x  = log − x tanlog | 2 | + c
∫ 3
dx  x
sec  log 
 1 1  2
1 + 2 + 5  
 x x 
Sol 2: Put x = asec2θ – btan2q
1 1
Let 1 + + t
=
x2 x5 2(α − β)sec2 θ tan θdθ

dt −2 5 (α − β)tan2 θ (α − β)tan2 θ(α − β)sec2 θ
= −
dx x3 x6
2sec2 θ tan θ
−dt 1 1 x10 = ∫ tan2 θ × (α + β)tan θ sec θdθ
∫ t3 = 2t2 + c= + c= +c
( )
2 2
 1 1  2 x5 + x3 + 1
2 1 + 2 + 5  2 sec θ
 x x  = ∫
α − β tan2 θ

Put tanθ = t sec2qdθ = dt


dt
Or secqdθ =
1 + t2
M a them a ti cs | 22.57

2 dt 2 t –3dt cos(x − a)
( α − β) ∫ t 2 1 + t 2 (α − β) ∫ t –2 + 1 ∫
∴I = = Sol 5: dx
sin(x + a)

∴1 + t–2 = u ⇒ –2t–3dt = du
cos x cosa − sinx sina
1 ∫ sinx cosa + cos x sina
dx
Or t–3dt = – du
2
1 2 –du –1 1 − tanx tana
= ×
2 ( α − β) ∫ =
( α − β)
×2 u = ∫ tanx + tana
dx = ∫ cot(a + x) dx
u

–2 1
= 1+ x5 + 3x 4 – x3 + 8x2 – x + 8
( α − β) t2 Sol 6: ∫ (x2 + 1)
dx

–2 1 –2 sec2 θ
= 1+ = (x3 + 3x2 – 2x + 5)(x2 + 1) x+3
( α − β) tan2 θ (α − β) tan2 θ = ∫ 2
(x + 1)
dx + ∫
x2 + 1
dx

–2 (α − β)sec2 θ –2 (x − β) x4 1
= = = + x3 – x2 + 5x + log(x
n(x22++4)4)++6 tan−1‒1 x + c
3tan x+c
α − β (α − β)tan2 θ α − β (x − α ) 4 2

 1 + x  ( x + 1) (x)1/2 + 1
Sol 7: ∫ = ∫ x1/2 dx
n  n    dx
  1 − x 
x(3 x + 1) (x1/3
+1)
Sol 3: ∫ 1 − x2 Put x1/6 = t ⇒ dx = 6t5dt
1 + x 
n 
Put log  =t (t3 + 1)t5 (t3 + 1)t2
1 − x  6∫ dt = 6 ∫ dt
t3 (t2 + 1) (t2 + 1)
 1 − x  1 − x + 1 + x 
⇒   ×  dx =
dt
1 + x  (1 − x)2   (t3 – t + 1)(t2 + 1)
= 6∫ 
t −1 
2
+  dt
 2   (t + 1) (t2 + 1) 
Or  dx = dt
2 
1 − x   t 4 t2 
dt = 6  – + t  + 3log(1 + t2) – 6tan–1t + c
∴∴I I=
= lnln( t()t ) dt
∫∫log  4 2 
22
Where t = x1/6
1
= ∴ I∫ 1. lnln( t( t) )dtdt
=∫log
2 2
−1 x
Integration by parts,
Sol 8: ∫ sin a+ x
dx , a > 0

1  d   x = atan2q
=I log(t)∫ 1dt −  ∫ (Iog(t) ) ∫ 1dt  dt 
2  dt   dx = 2atanqsec2qdq
1 atan2 θ
= t(logt − 1) + c −1
2 ∫ sin asec2 θ
2a tanθ sec2θ dq

1   1 + x     1 + x   
log   log  log    − 1 + c 2a∫ (sin−1 sin θ)tan θ sec2 θ dθ
2   1 − x     1 − x   
tanθ = t ⇒ sec2qdθ = dt

x x
x x
e e
x x 2a∫ t tan−1 tdt
nx  and d   =   [−logx]
Sol 4: d   =   [logx] [– nx]
e
  e
  x
  x  1 1 
2a tan−1 t ∫ tdt – ∫ × t2dt 
x x x x x x  2 1+t 2

 x x   e e  x e
∴ ∫ ∫   logxdx
nxdx+++∫ ∫   nxdx
nxdx nxdx =   –   + c
logxdx
e
  e  x
  x  e x
2 2 . 5 8 | Indefinite Integration

 t2 1 
= 2a (tan–1 t) – t − tan−1 t   + c (
x2 + 1 log(x2 + 1) – logx2 )dx
 2 2 

Sol 11: ∫ x 4

1 1 
= at2tan–1t +  tan−1 t – t  2a + c  1  1 
2 2   1 + 2 log  1 + 2  
 x  x  

∴ t = tanθ =
x ∫ x3
dx
a
1 2
1+ =t⇒– dx = dt
x x 1 x 1 x x 2
x3
∴ I = a tan−1 +  tan−1 –  2a + c
a a  2 a 2 a 
1 1 t3/2 1 t3/2 
– ∫ t logtdt – (logt)
= –∫ × dt 
2 2  3/2 t 3 / 2 
x 1 x 1 x
= x tan−1 +  tan−1 –  2a + c
a  2 a 2 a 
1 2 3/2 2 1/2 
=  – (logt)t + ∫ t dt 
x x 2 3 3 
x
= xtan–1 a + atan–1 a – a +c
a  
1 2 
3/2 3/2
1  1 4 1 
  = – log  1 +  1 +  +  1 +  + c  
−1 x x 2 3  x2   x2  9 x2  
=  tan  (a + x) − a +c   
 a  a
  3/2
1 1   1  2
 x  x = – 1 +  log  1 + 2  –  + c
= x  x tan−1 − a  + atan−1 +c 3 x2    x  3
 a  a

sinx tanx sec x


xlogx Sol 12: f(x)
= =
Sol 9:∴ ∫ dx sin x + 4 cos2 x
2
tan2 x + 4
(x2 − 1)3/2
 xlogx  tanx sec x
xnx  1 1 =
∴ ∫  2 3/2 

− dx + ∫

dx sec2 x + 3
 (x − 1)
2 3/2 2
1)  x x − 1  x x2 − 1
 Putting sec x = t, dx sec x tan x = dt so
logx  logx
∴ ∫  −d  + sec–1x + c = − + sec–1x + c
 2  dt 1 sec x
 x −1 x2 − 1 tan−1
∫ f(x)dx
= ∫=
t2 + 3 3 3
+c

log6 [(sinx)6cos x ]1/6 cos x


Sol 10: ∫ sinx
dx
cos2 x (1 + sinx)
1
Sol 13: ∫ sinx(1 − sinx)(1 + cos x)dx = ∫ sinx(1 + cos x) dx
log (sinx) + log6 6cos x  cos x
6 6 
∫ sinx
dx
 x x 
2
x 
2

 sin + cos   tan + 1 


2 2 1 2
 1   = ∫   dx = ∫   dx
  log(sinx) 1 cos x   x x 2 x 4 x 2 x
6 2sin cos × 2cos tan cos
∫   (log6)sinx + 6 sinx  cos x  dx 2 2 2 2 2
  
   x x
Let tan = t ⇒ sec2 dx = 2dt
 1 1 cos x 1 cos2 x  2 2
= ∫ log(sinx) + dx
 6 log6 sinx 6 sinx 
1 ( t+ 1)
2
1  1
= ∫ dt = ∫  t + 2 +  dt
1 log2 (sinx) 1 2 t 2  t
= + ∫ (cosecx − sinx)dx
6 log36 6
tt22 11
== ++tt++ log
nn||tt||++CcC
1  log2 (sinx) x  44 22
=  + ntan + cos x  + c
6  log36 2 
M a them a ti cs | 22.59

3x2 + 1 emz 1
= cos z + ∫ emz sin z dz
Sol 14: ∫ (x2 − 1)3 dx
m m

 3(x2 – 1) + 4  (3x2 + 1)(x2 − 1) emz 1  emz emz 


∫  (x2 − 1)3 dx or ∫ (x2 − 1)4
dx = cos z + 
m m  m
sin z − ∫
m
cos z dz 

 

 3x2 (x2 − 1) + (x2 − 1)  3x 4 − 2x2 − 1 emz emz 1


= ∫  (x2 − 1)4
 dx =

∫ 2
(x − 1) 4
dx = cos z + sin z − 1
 m m 2
m2
 x 4 + 1 – 2x2 – 4x 4 + 4x2   1  emz
= ∫ 

(x2 − 1)4
 dx Or,  1 + =  1 (m cos z + sin z)
   m2  m2

 (x2 − 1)2 − 2.2x2 (x2 − 1)  emz (m cos z + sin z)


= ∫ –  (x2 − 1)4
 dx =∴1 +c
  m2 + 1
Or,
 x  x
= ∫ –d   =– 2 +c −1 x −1
2 2  em tan  1 x  em tan x (m + x)
 (x − 1)  (x − 1)2 =1 m ⋅ + = +c +c
m2 + 1  x2 + 1

x2 + 1  (m2 + 1) x2 + 1
(ax2 − b)dx
Sol 15: ∫
x c2 x2 − (ax2 + b)2 x
Sol 17: [Here occurs, ∴ put
= x a tan2 θ ]
a+ x
(ax2 − b) acx2 – bc
∫ ⇒∫ dx Put
= x a tan2 θ, then=
dx 2a tan θ sec2 θdθ
2 (ax2 + b)2 2 2 (ax2 + b)2
cx 1− c x 1−
c2 x 2 c2 x 2 Now,
−1
2acx2 − (acx2 + bc) =I ∫ sin (sin θ)2a tan θ sec2 θd=
θ 2a∫ θ ⋅ (tan θ sec2 θ)dθ
= ∫ dx
(ax2 + b)2  sec2 θ
(cx2 ) 1 − sec2 θ 
c2 x 2 = 2a θ − ∫1 dθ  + c
 2 2 
2
2acx2 – (ax2 + b)c 1 [ ∫ tan θ sec θd=θ ∫ zdz, where= z sec θ]
= ∫ (cx)2
×
2
dx
 ax2 + b 
1− = a[θ sec2 θ − tan θ] + c
 cx 
 
 x a+ x x
= a tan−1  − +c
2
ax + b  (2ax)cx − c(ax + b)  2
a a  a 
Put =t =   dx 
cx  (cx)2 
 
 ax2 + b   x x π
dt dsin  e +e +
∴∫ = sin t + c = sin  cx  + c
–1 –1


4
1 − t2   Sol 18:
dx
1  π   e x e– x 
Sol 16: Put z = tan‒1x, then dz = dx and x = tan z = cos  e x
+ e– x
1+x 2 +  – 
 4  2 x 2 x 
mz
 
e mz
=Now, I ∫=2
dz ∫ e cos z dz
Also
d 
sin  e x
− e– x
+
π

1 + tan z dx  4
emz emz  π   e x 2e– x 
= cos z − ∫ ( − sin z)dz x
– e– x
m m = cos  e +  + 
 4  2 x 2 x 

2 2 . 6 0 | Indefinite Integration

  x π  5x 4 + 4x5
dsin  e + e

– x
+  
4 
Sol 21: ∫ (x5 + x + 1)2 dx
2∫ 
∴I =
  π 
 +dsin  e x − e– x +   5x5 + 5x 4 + x + 1 – x5 – x – 1
  4   = ∫ (x5 + x + 1)2
dx

  π  π 
= 2 sin  e x
+ e– x
+  + sin  e
x
– e– x
+  (x5 + x + 1) + (5x 4 + 1)(x + 1)
  4  4  = ∫– (x5 + x + 1)2
dx


= 2sin  e

x
+
π
 cos e
4 ( )
– x
=
 x +1  x +1
∫  x5 + x + 1  x5 + x + 1
–d  =  – + c or
x5
+c
x5 + x + 1
= 2  sin e
 ( ) + cos (e )  cos (e ) + c
x x – x

 a2 tan2 x + b2 
Sol 22: ∫  a4 tan2 x + b4  dx

 
(x2 + x)
Sol 19: ∫ (ex + x + 1)2 dx 1 b2 (a2 + b2 ) + a2 (b2 + a2 )tan2 x

(a2 + b2 )
∫ a4 tan2 x + b 4
dx

 x(ex + x + 1) − xex 
= ∫  (ex + x + 1)2  dx
 1
 2 2 2x 
 a b (1 + tan + 1 dx
  ⇒ ∫
a2 + b2  a4 tan2 x + b 4 
 
 x xex 
= ∫  ex + x + 1 (ex + x + 1)2  dx
 –
1  a2b2 + b 4 + (a2b2 + a4 )tan2 x 
  ⇒ ∫
a2 + b2  a4 tan2 x + b 4
 dx

  

  
 1 × x − 1  x   
= ∫   x   dx  
  x 2
 1 +  x + 1   e  x +1 e  1   a2  2 1 
1 + x  2 ∫ 2 
   ⇒   sec x × + 1  dx
  ex    e  
2
a + b  b  a 4 
2 
  4 tan x + 1  
x +1 –x  b  
Put +1 =t ⇒ dx = dt
x
e ex 2
a 2
 dt 1  1   sec x
1 1 b
 
∫  – +  dt = – logt –
t t2  t
+c ⇒ 2
a + b2
x+
a2 + b2
∫ 2
dx
 a2 
1 +  2 tanx 
 x +1  1 b 
= –log  + 1 – +c  
x  x + 1 
 e 
 x + 1 a2 a2
 e  Put tanx = t ⇒ sec2 xdx = dt
b2 b2
1 1 dt
ecos x (x sin3 x + cos x) ∴ x+ ∫ 1 + t2
Sol 20: ∫ sin2 x
dx 2
a +b 2 2
a +b 2

cos x  cos x  1  a2 
∫ e  sin2 x + x sinx  dx Or  x + tan−1 tanx  + c
2 2 2
a + b  b 
cos x
∫e (cot x cosecx + x sinx)dx
cos2 x cos3 x
⇒– ∫ e cos x
(1 − cos tx cosecx − x sinx − 1)dx
Sol 23: ∫ 1 + tanxdx = ∫ sinx + cos x dx
= – ∫ ecos x (1 − cosecx cot x) + (x + cosecx)ecos x (– sinx)dx
  1 3cos x 1 cos3x
4 ∫ sinx + cos x
= dx + ∫ dx
4 sinx + cos x
= – ∫ decos x (x + cosecx) = –ecosx(x + cosecx) + c
M a them a ti cs | 22.61

1 cos x cos2x − sinx sin2x 3 cos x x


= ∫ dx + ∫ dx Again, ∫ dx =− 1 − x2
4 (sinx + cos x) 4 sinx + cos x 1−x 2

1   sin2x cos x − cos2x sinx   ∴ from (i),


= ∫
4 
(cos2x − sin2x) + 
 sinx + cos x
 dx
  1 − 1 − x2 
=I (x log x − x)sin−1 x + 1 − x2 log x − log  +c
3 cos x  x 
4 ∫ sinx + cos x
+ dx  

1 1 (x2 + 1)ex (x2 − 1) + 2


= ∫ (cos2x − sin2x)dx + ∫ dx Sol 25: ∫ dx = ∫ ex dx
4 4 (x + 1)2 (x + 1)2
1 cos xdx 3 cos xdx
− ∫ + ∫  x2 − 1
4 sinx + cos x 4 sinx + cos x 2  x x −1 2 
=∫ ex  +  dx =∫ e  +  dx
 (x + 1) 2
(x + 1)   x + 1 (x + 1)2 
1 x 1 cos xdx
= (sin2x + cos2x) + + ∫
8 4 2 sinx + cos x x x −1
=∫ e [f(x) + f '(x)]dx, where f(x) =
x +1
1 x
= (sin2x + cos2x) +  x −1 
8 4 = ex f(x)=+ c ex  +c
1  cos x + sinx + cos x − sinx   +1
x
+ ∫  dx
4  cos x + sinx 
f(z)
Sol 26: [Here e occurs, where f(x) = sin x
1 x 1   cox − sinx  
= (sin2x + cos2x) + + ∫ 1 +   dx
8 4 4   cos x + sinx   ∴ Put z = f(x) = sin x]
Put z = sin x, then dz = cos x dx
11 xx 11
= (sin2x
(sin2x++cos2x)
cos2x)++ ++ log
n(cos
n(cosxx++sinx)
sinx)++cc
88 22 44 esin x 3
=Now, I ∫ cos3 x (x cos x − sin x)dz
Sol 24:
sin x
xlog x x = ∫e (x − tan x sec2 x)dz
I =(x log x − x)sin−1 x − ∫ dx + ∫ dx ...(i)
1 − x2 1 − x2  −1
z z 1 
[Integrating by parts taking sin x as u] ‒1
= ∫e sin z − ⋅
2
 dz [ sin=
x z]
 1 − z 2 1 − z 
x log x
Now in order to evaluate ∫ dx
z
 −1 1 1 z 
1 − x2 = ∫e sin z + − −
2 3/2
 dz
 1 − z2 1 − z 2 (1 − z ) 
Put x= sin θ, then dx= cos θ dθ
 1   1 z 
xlog x z
 sin−1 z −
∴ ∫ dx
= ∫ sin θ log sin θ dθ = ∫e 
+
 

2 3/2
(1 − z ) 
 dz
1 − x2 1 − z2   1−z
2

= − cos θ log sin θ − ∫ − cos θ cot θdθ


z 1
sin−1 z −
∫ e [f(z) + f '(z)]dz, where f(z) =
=
cos2 θ 1 − z2
= − cos θ log sin θ + ∫ dθ
sin θ  1 
= ez f(z) +=
c ez  sin−1 z − c esin x (x − sec x) + c
 +=
 
 1 − z2 
1 − sin2 θ
= − cos θ log sin θ + ∫ dθ
sin θ dx dx
Sol 27: I
= ∫= ∫
2 2
= − cos θ log sin θ + ∫ (cosec θ − sin θ)dθ 1 − (x + 2x) 2 − (x + 2x + 1)

dx dx
= − cos θ log sin θ + log | cosec θ − cot θ | + cos=
θ ∫=2
∫ ...(i)
2 − (1 + x) (2) − (1 + x)2
2
 1 − 1 − x2 
=− 1 − x2 log x + log   + 1 − x2 Let z = 1 + x, then dz = dx
 x 
 
2 2 . 6 2 | Indefinite Integration

From (i), z 2 a2
∫ z 2 + a2 dz
= z + a2 + log  z + z 2 + a2  + c
dz 1 + x  2 2  
−1 z
=I ∫ = sin = + c sin−1  +c
( 2)2 − z 2 2  2   3
 −x +  2
4  3 23
=  x +  +
dx  sinx cos x  2  4 16
Sol 28: ∫ sec x + cosecx = ∫  sinx + cos x  dx
 2 
23 3  3 23 
+ log  x + +  x +  + +c
1  1 + 2sinx cos x − 1  32  4  4 16 
2 ∫  sinx + cos x 
=   dx  

4x + 3
1 (sinx + cos x)2 − 1 =I 2x2 + 3x + 4
= ∫ dx 8
2 (sinx + cos x)
 2x2 + 3x + 4 
23 4x + 3
1 1 1 + log  + +c
= ∫ (sinx + cos x)dx – ∫ dx 16 2  4 2 
2 2 2 1 (sinx + cos x)
 
2
1 1 dx z xn + 1 , then dz = nxn−1dx
Sol 30: Let =
=
2
[sinx − cos x] –
2 2
∫  π dx dx
sin  x +  Now, I
= ∫= ∫ nxn−1 ⋅ x
 4
(
x xn + 1 ) (x n
+1 )
x π
sec2  +  1 dz 1 dz
 …(i)
n ∫ xn xn + 1 n ∫ (z − 1)z
1 1  2 8  dx = =
= [sinx − cos x] –
2 2 2
∫ x π
2 tan  + 
( )
2 6 1 A B A(z − 1) + Bz …(ii)
Let =+ =
11 11 xx ππ z(z − 1) z z − 1 z(z − 1)
= [sinx
[sinx−−cos
cosx]x]–– ntan
ntan ++ ++cc
log
22 22 22 22 88
1
∴ A(z − 1) + Bz = …(iii)
11  1 xx ππ
sinx−−cos
= [sinx cosx]x–– ntan
ntan ++  ++cc
log
Putting Z = 0 we get, ‒A = 1
22  2 22 22 88
∴ A =‒1
 3  Putting Z = 1, we get B = 1
Sol 29=I ∫ 2x2 + 3x + 4 dx
= ∫ 2  x2 + x + 2 dx
 2 
1 dz 1  1 1 
∴ From =
I ∫ = ∫ − + dz
3 n z(z − 1) n  z z − 1 
= 2 ∫ x2 + x + 2dx
2
1
= [ − log | z | + log | z − 1 |] + c
2 2 n
3 3 3
= 2 ∫ x2 + 2 ⋅ x ⋅ +   −   + 2dx
4 4 4 1 z −1 1 xn
= log = +c log +c
n z n xn + 1
2
 3 23
= 2∫  x +  + dx ...(i)
 4  16 cos x − sinx
Sol 31: ∫ 16 − 9 (1 + sin2x ) dx
3 23 3
Let, z =
x + , then dz =
dx. Let = a
=
4 16 4
(cos x − sinx)
Then from
= (i), I 2 2
2 ∫ z + a dz ∫ 16 − 9(sinx + cos x)2 dx
Now, Let 3(sinx + cosx) = t
M a them a ti cs | 22.63

⇒ (3cosx – 3sinx)dx = dt ⇒ dx = sec2qdq

1 dt 11 11 44++t t  11 44++3cos
3cosxx++3sinx
3sinx 7 1 − tan2 θ
∫ = . . log
nn =  log
nn  = 4logx + +6tan x + 6 ∫
–1 dθ
3 16 − t 2 2.4
2.4 33 44−−t t 24
24 44−−3cos
3cosxx−−3sinx
3sinx x (1 + tan2 θ)

7
= 4logx + + 6tan–1x + 6 ∫ cos2θdθ
co tx − tanx x
Sol 32: ∫ 1 + 3sin2x
dx
7 1
= 4logx + + 6tan–1x + 6 sin2θ
x 2
cos x − sinx
2∫ dx
( )
sin2x (1 + 3sin2x) ∵sin2θ =
2 tan θ
1 + tan θ 2
=
2x
1 + x2
  7 6x
(cos2x cos x + sin2 x sinx) + (cos2x sinx − sin2x cos x) 
2∫  dx ∴I = 4logx + + 6tan–1x + +c

 (
sin2x (1 + 3sin2x) ) 

x 1 + x2

dx
 cos2x(ssinx + cos x) − sin2x(cos x − sinx) 
2∫  dx
Sol 34: ∫ cos3 x − sin3 x
 2sin2x(1 + 3sin2x) 
dx
1 ∫ (cos x − sinx)(1 + cos x sinx)
⇒ ∫ (sinx + cos x)2 + 2sin2x
(sinx + cos x)2 (cos x − sinx)2 + 2sinx cos x
 (sinx + cos x)2.cos2x − 2sin2 x(cos x − sinx) 
= ∫ (cos x − sinx)(1 + cos x sinx) dx
×∫  dx
 (sinx + cos x)2 2sin2x  (cos x − sinx) sinx cos x
 
∫ (1 + cos x sinx) dx + 2∫ (cos x − sinx)(1 + cos x sinx) dx
 
1   2sin2x  dx
⇒ ∫ 2 cos x − sinx 2 (cos x − sinx)
 2sin2x   sinx + cos
= 
x ∫ dx + ∫ dx
1 +  3 1 + (sinx + cos x)2 3 2 − (sinx + cos x)2
 (sinx + cos x)2 
2 1 2 + sinx + cos x
 2 sin2x  = tan−1 (sinx + cos x) + log +c
= tan  −1
+c 3 3 2 2 − sinx − cos x
 sinx + cos x 
 
x2
5 4
4x – 7x + 8x − 2x + 4x − 7 3 2 Sol 35: ∫ (x cos x − sinx)(x sinx + cos x)dx
Sol 33: ∫ x2 (x2 + 1)2
dx

(x cos x − sinx)2 + (x sinx + cos x)2


5 4
4x − 7x + 8x – 2x + 4x − 7 3 2 ∫ (x cos x − sinx)(x sinx + cos x)
dx

x2 (x2 + 1)2
x cos x − sinx x sinx + cos x
A B Cx + D Ex2 + Fx + G
= + + +
∫ x sinx + cos x dx + ∫ x cos x − sinx dx
x x2 x2 + 1 (x2 + 1)2
–1
5 2
(4x − 7x + 8x − 2x + 4x – 7) 3 2
+ ∫ (x cos x − sinx)(x sinx + cos x) dx
∫ x2 (x2 + 1)2
dx
 x cos x + sinx − sinx  x sinx + sinx − cos x
= ∫  dx + ∫ dx
4  7  6 6(1 – x )   x sinx +2cos x  x cos x − sinx
= ∫  x  x2  x2 + 1 (1 + x2 )2 
 + – + +  dx
  cos2 x + sin2 x − 1
+ ∫ dx
 1 − x2  (x sinx + cos x)(x cos x − sinx)
7
= 4logx + + 6tan–1x + 6 ∫   dx
x  (1 + x2 )2  x sinx + cos x
  n
= log +c
Put x = tanq x cos x − sinx
2 2 . 6 4 | Indefinite Integration

 cos
cosθθ++sin
sinθθ  1
Sol 36: ∫∫cos2
cos2θθlog
nn   ddθθ Put x + =t
 cos
cosθθ−−sin
sinθθ  x
 cos θ + sin θ  ⇒(1 – x–2)dx = dt
⇒ n 
⇒log  ∫ cos2θdθ
 cos θ − sin θ  dt

 dn  cos
cosθθ ++ sin
sinθθ 
 

t2 − 2
dlog cos θ − sin θ  
cos θ − sin θ Put t = 2 secq
− ∫∫
− 
d θ
 ∫∫
cos2
cos2θdθddθθ
θd θ
 dθ 
 2 sec θ tan θdθ
 
∫ 2 tan θ
= log|secθ + tanq|
11  cos
cosθθ++sin
sinθθ    22 sin2
sin2θθ 
⇒⇒ sin2
sin2θθlog
n
n   ––∫ ∫  ×× ddθθ
22  cos
cosθθ− −sin
sinθθ  cos2
cos2θθ 22   x2 + 1 x4 + 1
= log
n + +c
11  cos
cosθθ++sin
sinθθ   2 sin2θ  2x 2x
⇒ sin2
sin2
θθ nn 
log  ––∫ ∫ tan2θd×θ dθ
22  cos
cosθθ−−sin
sinθθ  cos2θ 2 
x2 + 1 x4 + 1
111  cos
cos
cos
θθ+θ+sin
+sin
sin
θθθ 1 1 2 sin2θ  Or log
n + +c
⇒ sin2
sin2
sin2
θθ θnnn
log  ––∫– log
n(sec2
n(sec2
× θθ)d)dθθ dθ x x
222  cos
cos
cos
θθ−θ−sin
−sin
sin
θθθ 22cos2θ 2 
(1 + x2 )
 1 − tan θ   (1 − tan θ × sec2 θ  (C) ∫ dx
=    − (1 + tan θ)sec2 θ  1  1
 1 + tan θ   (1 − tan θ) 2  x  − x  x2 + 2
2
 x  x
(1 − tan θ) 2sec2 θ 2(1 + tan2 θ) 2 (1 + x –2 )
= × = =
(1 – tan θ) (1 – tan θ)2 2
(1 − tan θ) cos2θ ∫ 2
dx
1   1
 − x x −  + 2
 x   x
Sol 37: A → s; B → p; C → q; D → r 1
x– = t ⇒ (1 + x–2)dx = dt
x −14 x
(A) ∫ dx dt
3 2 1 = –∫
x x + +1
x2 t t2 + 2

x − x –3 t= 2 tanθ
∫ 1
dx
x2 + +1 dt = 2 sec2θ dq
x2
1 2 sec2 θdθ 1
Put x2 + +1=t = –∫
x 2
2 tan θ 2 sec θ

2
∫ cosecdθ
 2 dt  1  dt 11
 2x − 3  dx = or  x −  dx = = nncosec
log cosecθθ−−cot
cotθθ++cc
 x  2 
3
x  2 22

1 dt 1 x 4 + x2 + 1 11 xx44 ++11−− xx
x2 + ⇒ −− log
nn ++cc
2∫ t
= t +c = + 1+ c = +c
x2 x 22 (x22 −−1)
(x 1)

x2 − 1
(B) ∫ dx
1 + x 4 + x2
x 1 + x4 (D) ∫ dx
2 –2
1 + x4
x −1 (1 − x )
∫ 1
dx ⇒ ∫
2
dx 



x 2
x + 2  1 1 1 1  –4 
x2 x +  − 2 = ∫  × × 
 x  1  1  x5 
  1 
 1 + 4  2  1 + 4 – 1 2 1+
 x   x  x4
 
M a them a ti cs | 22.65

 1   4
– tan−1  1+ –1+ c ⇒  1 −  dx = dt
 x4   x2 
 
dt 1 t
  ∴∫ = tan−1 + c
1 2
t (4) 2 4 4
dtan−1  1+ 4 –1
∴  x 
   4
dx 1 x+ 
−1
∴I = tan  x +c
1  1  4  4 
= × 1+ –1  
    
 1 x4
1+ 1 + 4 – 1  
 x  1  x2 + 4 
  Or I = tan−1   +c
4  4x 
 
  
  
 1  1  1 –4
= × (x − 1)2 x2 − 1 − 2x + 2
 x   1  1
 1 +   2 1 + 4 − 1  2 1 + 4
x5 Sol 3: (D) ∫ x 4 + 2x2 + 1 dx = ∫ (x2 + 1)2
dx
 4  x  x
(x2 − 1) –2x 2dx
= ∫ 2
dx + ∫ (x2 + 1)2 dx + ∫ (1 + x2 )2
 1
Exercise 2 x2  x + 
 x

Single Correct Choice Type Put x2 + 1 = t and Put x = tanθ


dx = sec2qdq
−1 −1
tan x − cot x
Sol 1: (D) ∫ tan−1 x + cot−1 xdx  1 
1 − 2 
x  dt sec2 θdθ
π = ∫ dx + ∫ – + 2∫
∴ 2
t2 sec 4 θ
tan–1x + cot–1x =  1
2 x + 
 x
 −1 x π
 2 tan −  1
2 2  dx = 4 tan−1 xdx – dx Put x + =u
π∫ ∫
I = ∫ x
 π 
  du 1
 2  = ∫ + + 2∫ cos2 θdθ
2 2
u x +1
4   dtan−1 x   
= ∫ tan−1 x ∫ dx − ∫    ∫ dx  dx  –x + c 1 1 2
1  (x2 + 1) 2 ∫
π   dx   = – + + (cos2θ + 1)dθ
     
x + 
4 4  x   x
= x tan−1 x − ∫   dx − x + c
π x  1 + x2  (1 − x)
1
= + sin2θ + θ + c
44 22 x +1 2
2

= tan−1−1xx–– log
xxtan n(1++xx2 2) ) –x + c
n(1
ππ ππ (1 − x) 1 2× x
= + + tan−1 x + c
1+x 2 2 1 + x2
x2 − 4 x2 − 4
Sol 2: (A) ∫ x 4 + 24x2 + 16 dx = ∫  16 
dx
=
(1 − x)
+
x
+ tan−1 x + c =
1
+ tan−1 x + c
x2  x2 + 24 + 2  1+x 2
1 + x2 1+x 2
 x 

 4   4  x4 − 4
1 – 2  1 − 2 
 x 
Sol 4: (A) ∫ dx
 x  x2 x 4 + x2 + 4
= ∫
 2 16 
dx = ∫ 2
dx
x + + 8 + 16
 4 2 x4 − 4
   x +  + (4)
 x2   x ∫ 4
dx
2 2
x ×x x +1+
4 x2
Put x + =t
x
2 2 . 6 6 | Indefinite Integration

 4 x
2 sin
x − 3  2
 x  I= ∫
∫ 4
dx
2cos2
α
− 2cos2
x
x2 + 1 + 2 2
x2
x
2 sin
2
Put x2 +
4  8 
+ 1 = t ⇒  2x −  dx =
dt
= ∫ 2
dx
x 2
 x3   x
α  cos 
2 cos 1− 2 
1 dt 1 4 2  cos α 
x2 + 1 +
2∫ t 2
= = .2 t + c = +c  
x2  2

 x
 x 4 + x2 + 4   – sin 
  1 2
=   +c × 2x  
x 2  cos α 
 
 2  dx
= ∫ 2
 sec x + tanx − 1   x
Sol 5: (A) ∫   dx  cos 
 tanx − sec x + 1  1− 2
 cos α 
sec x + tanx − (sec2 x − tan2 x)  
2
= ∫ (tanx − sec x + 1)
dx 
x
cos
(sec x + tanx)[1 − sec x + tanx] 3 =t ∴I= 2
= ∫ [tanx − sec x + 1]
dx Let
α ∫– dt
cos 1 − t2
2
= ∫ (sec x + tanx)dx  x
 cos 
= log|secx + tanx| + logsecx + c 2
= 2cos–1(t) + c = 2cos–1   +c
 cos α 
 
s = 1 + 2x + 3x2 + 4x3 + ………….  2
sx = x + 2x2 + 3x3 + ………… 3x 4 – 1
s(1 – x) = 1 + x + x2 + x3 + ……….
Sol 8: (B) I = ∫ (x 4 + x + 1)2 dx
1 1
s(1 – x) = ∴s= 4x 4 + x – (x 4 + x + 1)
1−x (1 − x)2 = ∫ (x 4 + x + 1)2
dx
1 (1 − x)−1
∴∫ dx = + c = (1 – x)–1 + c
(1 − x)2 –1 × –1 x(4x2 − 1) 1
= ∫ (x 4 + x + 1)2 – (x 4 + x + 1) dx
(x2 − 3) x (x2 − 9 + 6) 1
Sol 6: (C) ∫e
x

(x + 3)2
dx = ∫e (x + 3)2
dx Let ∫ x 4 + x + 1 = I1
 x −3 6 
= ∫e
x
 +  dx (4x3 + 1) 4x3 + 1
2  ∴I = x ∫ – ∫∫ dx – I1
 x + 3 (x + 3)  (x 4 + x + 1)2 (x 4 + x + 1)2
∫ e ( f(x) + f '(x)) dx
x
= = exf(x) + c
(x 4 + x + 1)−1 (x 4 + x + 1)−1
(x − 3)
=x×
–1
– ∫ –1
dx – I1
= ex +c
(x + 3) –x
= + c + I1 – I1
4
(x + x + 1)
1 − cos x
Sol 7: (B) I = ∫ cos α − cos x
dx
3x
Sol 9: (C) ∫e cos 4xdx

Let 3x =t
M a them a ti cs | 22.67

1 t 4t  
1/3
∴I =
3 ∫ e cos dt
3 1/3  
 x   x 
1 t
= ∫=
4t 1 t 4t 4 4t  I = ∫  4  dx = ∫  4 
dx
3
e cos dt
3 3 ∫ e  cos − sin  dt
 3 3 3
4 
 (x − 1)   16  1  
 x 1 − 4  
  x  
4 t  4π 
+
9 ∫e  sin  dt
 3  = ∫
1 1
dx
4/3 5
 1  x
1  t 4t  4 t  4t 4 4t  1 − 4 
 x 
= ∫
3 
e cos  + ∫ e  sin + cos  dt
3 9  3 3 3
1 4
16 4t Let 1 – = t ∴ dx = dt
– ∫ et cos dt x 4
x5
27 3
25 1 4t 4 4t 1 1 1 t –1/3 3
I = et cos + et sin + c ∴I = ∫ t 4/3 × 4 dt =
4
×
1
+ c = – t–1/3 + c
4
9 3 3 9 3 –
3
et  4t 4t  −1/3 1/3
I=  3 + 4 sin  +c 3  1  3  x4 
25  3 3 =– 1 – 4  +c=–   +c
4  x  4  x 4 − 1 
∴3A = 4B
u
Sol 13: (A) I = ∫e sin2xdx
Sol 10: (C) When u = x
p + xp + 2q−1 + qxp + 2q−1 − q(xq−1 + xp + 2q−1 ) x x
I= ∫ p+q 2
dx I= ∫ e sin2xdx= ∫ e (sin2x + 2cos2x − 2cos2x)dx
(x + 1)
x
(p + q)xp + q−1 xq − qxq−1 (xp + q ) + 1 = ∫e (sin2x + 2cos2x) − 2∫ ex cos2x
= ∫ dx
(xp + q + 1)2 x
= ∫e (sin2x + 2cos2x)
uv '− vu'
It is of the form –2∫ ex (cos2x − 2sin2x + 2sin2x)dx
2
u
∴ Where u = x p+q
+ 1 and v = –xq ∴ 5 I = ∫ ex (sin2x + 2cos2x)dx

v –xq –2∫ ex (cos2x − 2sin2x)dx


∴I = +c = +c
u xp + q + 1 = exsin2x – 2excos2x + c

Sol 11: (B) Let f–1(x) = t When u = sinx


sin x
∴f(f–1(x)) = x I= ∫ 2e cos x sinxdx

∴f(t) = x Put sinx = t ∴ cosxdx = dt


−1
∴ ∫ f (x)dx ∴I = 2 ∫ tet dt which is solvable

∫ tdx = ∫ tf '(t)dt
= tf(t) – ∫ f(t)dt = tf(t) – g(t)
Previous Years’ Questions
= f–1(x) (x – g)(f–1(x))
cos3 x + cos5 x

1/3

Sol 1: (C) Let I = ∫ sin2 x + sin4 x dx
 x 
Sol 12: (B)   dx
(cos2 x + cos4 )·cos x dx
( )
4
 x4 − 1



= ∫ (sin2 x + sin4 x)
Put sinx = t ⇒ cosx dx = dt
2 2 . 6 8 | Indefinite Integration

[(1 − t2 ) + (1 − t2 )2 ] Put ex = u ⇒ exdx = du


∴ I= ∫ t2 + t 4
dt
(u2 − 1)
∴ J – I =∫ du
1 − t2 + 1 − 2t2 + t 4 1 + u2 + u4
⇒ I= ∫ t2 + t 4
dt
 1   1 
2 − 3t2 + t 4 1 − 2  1 − 2 
⇒ I= ∫ t2 (t2 + 1)
dt ....(i)
= ∫ 
u 
du = ∫ 
u 
du
1 2  1
2
1+ 2 +u
Using partial fraction for, u u +  − 1
 u
y2 − 3 y + 2 A B 1
=1 + + ( where y = t2 ) Put u + =t
y (y + 1) y y +1 u

⇒ A = 2, B = –6  1  dt
⇒ 1 − dt = ∫
 du =
 u2  2
t −1
y2 − 3 y + 2 2 6
∴ =1 + −
y (y + 1) y y +1 1 t −1 1 u2 − u + 1
= log
= +c log +c
2 t +1 2 u2 + u + 1
∴ Eq. (i) reduces to,
 2 6  2 1 e2x − ex + 1
I= ∫  1 + t2 − 1 + t2  dt = t−
t
− 6 tan−1 (t) + c =
2
log
e2x + ex + 1
+c

2
= sin x – – 6 tan–1(sinx) + c
sinx  2sinx − sin2x 
Sol 4: Given, f(x) = ∫   dx
 x3 
x
Sol 2: (A) Given,f(x) = for n ≥ 2
(1 + xn )1/n On differentiating w.r.t. x, we get

f(x) x 2sinx − sin2x 2sinx  1 − cos x 


∴ff(x) = = f’(x) = =  
[1 + f(x) ] n 1/n
(1 + 2x ) n 1/n
x 3 x  x2 
x
and fff(x) =  2 x 
(1 + 3xn )1/n  sinx   2sin 2 
x lim f '(x) = lim2   
∴g(x) = fofo...of
   (x) = x →0 x →0  x   x2 
n times (1 + nxn )1/n 



xn−1dx 1 n2 xn−1dx  
=Let I ∫=
n− 2
x g (x)dx ∫ = ∫
n2 (1 + nxn )1/n

sin2 x 
(1 + nxn )1/n  2  =1
= 4 · 1 · lim  
 4× x 
x →0 2
d  
(1 + nxn )   2 
1dx   
= ∫
n2 (1 + nxn )1/n
dx

1 x
∴ I=
1 1−
(1 + nxn ) n + c Sol 5: (i) Let I = ∫ 1 + sin dx
2
n(n − 1)

x x x x
ex ⇒I= ∫ cos2 + sin2 + 2sin cos dx
Sol 3: (C) Since, I = ∫ dx 4 4 4 4
e4x + e2x + 1
e3x  x x
J =∫ dx ⇒I= ∫  cos 4 + sin 4  dx
1 + e2x + e4x
x x
(e3x − ex ) = 4sin – 4cos + c
∴J – I = ∫ dx 4 4
1 + e2x + e4x
M a them a ti cs | 22.69

1/ 4
x2  1  (x 4 + 1)1/ 4
(ii) Let I = ∫ 1−x
dx = − 1 +


x4 
+c= −
x
+c

Put 1 – x = t2 ⇒ –dx = 2t dt
1− x
(1 − t2 )2 · ( −2t)
Sol 9: Let I = ∫ 1+ x
dx
∴I = ∫ dt
t
 2t3 t5  Put x = cos2 θ ⇒ dx = – 2sin θ cos θ d θ
2 4
= −2∫ (1 − 2t + t ) dt = −2  t − + +c
 3 5  1 − cos θ
 =∴I ∫ 1 + cos θ
· ( −2sin θ cos θ)dθ
 2 1 
= −2  1 − x − (1 − x)3/2 + (1 − x)5/2  θ θ
 3 5  = − ∫ 2 tan · sin θ cos θ dθ = −2∫ 2sin2 · cos θ dθ
2 2

Sol 6:=
Let I ∫ (e
log x
+ sinx)cos x dx = −2∫ (1 − cos θ) cos θ dθ = −2∫ (cos θ − cos2 θ) dθ

= ∫ (x + sinx)cos x dx = −2∫ cos θ dθ + ∫ (1 + cos2 θ) dθ


1
=∴I ∫ x cos x dx + 2 ∫ (sin2x) dx = −2sin θ + θ +
sin2 θ
+c
cos2x 2
= (x ·sinx − ∫ 1·sinx dx) − +c
4 = −2 1 − x + cos−1 x + x(1 − x) + c
cos2x
= xsinx + cosx – +c
4
sin−1 x − cos−1 x
(x − 1)ex
Sol 10: Let I = ∫ sin−1 x + cos−1 x
dx
Sol 7: Let I = ∫ (x + 1)3 dx
π 
sin−1 x −  − sin−1 x 
 x + 1 − 2  x  2  dx
⇒ I = ∫ e dx = ∫
3  π
 (x + 1) 
2
 1 2  x 2  π
∫  (x + 1)2 − (x + 1)3  e
−1
= dx = ∫  2sin x −  dx
  π  2

1 1 4
sin−1 x dx − x + c
π∫
x
dx − 2∫ ex · = ...(i)
= ∫e ·
(x + 1)2 (x + 1)3
dx


Applying integration by parts, Now, ∫ sin−1 x dx

 1 −2  Put x= sin2θ ⇒ dx = sin2θ


=  ·ex − ∫ ex · dx 
2
 (x + 1) (x + 1)3  = ∫ θ ·sin2θ dθ
1 ex θ cos2θ 1
−2∫ ex · dx = + c = − + ∫ cos2θ dθ
(x + 1)3 (x + 1)2 2 2
θ 1
= − cos2θ + sin2θ
dx dx 2 4
Sol 8: Let I ∫=
= ∫
x (x + 1)3/ 4
2 4

3/ 4
1  1 1
x 2 · x3  1 + 4  = − θ(1 − 2sin2 θ) + sin θ 1 − sin2 θ
 x  2 2
4 1 1
Put 1 + x–4 = t ⇒ – dx = dt = − sin−1 x (1 − 2x) + x 1−x ...(ii)
x5 2 2 
1 dt 1 t1/ 4 From Eqs. (i) and (ii), we get
− ∫
∴I = − ·
= +c
4 t3/ 4 4 1/4
2 2 . 7 0 | Indefinite Integration

4  1 1   1 
I=  − (1 − 2x)sin
−1
x+ x − x2  − x + c Now, I1 = ∫   dx
3 4
π  2 2   x+ x
2 Put x = t12 ⇒ dx = 12t11dt
= x − x2 − (1 − 2x)sin−1 x  − x + c

π 
t11 t8 dt
∴ I1 = 12∫ dt = 12∫
cos2x t4 + t 3 t +1
Sol 11: Let I = ∫ sinx
dx
dt
= 12∫ (t7 − t6 + t5 − t 4 + t3 − t2 + t − 1)dt + 12∫
2
cos x − sin x 2 t +1
= ∫ = dx ∫ cot2 x − 1 dx
sin2 x  t8 t7 t6 t5 t 4 t3 t2 
= 12  − + − + − + − 1  + 12 log
In (t(t++1)1)
Put cot x = sec θ ⇒ –cosec2xdx = sec θ tan θ d θ 8 7 6 5 4 3 2 
 

sec θ · tan θ  In (1 + 6 x ) 
∴I
= ∫ sec2 θ − 1 · dθ And I2 = ∫   dx
2 3
−(1 + sec θ)  x + x 
sec θ · tan2 θ Put x = u6 ⇒ dx = 6u5 du
= −∫ dθ
2
1 + sec θ
log (1 + u) 5 log (1 + u)
2 =∴ I2 ∫ =6u du ∫ ⋅ 6u5 du
sin θ u2 + u3 u2 (1 + u)
= −∫ dθ
cos θ + cos3 θ
u3  u3 − 1 + 1 
6∫ In(1 + u)du= 6 ∫ 
+ u)du
log(1  In(1
log(1+ u)du
+ u)du
1 − cos2 θ (u + 1)  u+1 
= −∫ dθ  
cos θ(1 + cos2 θ)
 1 
= 6 ∫  u2 − u + 1 − log(1
 In(1 + u)du
+ u)du
(1 + cos2 θ) − 2cos2 θ  u+1
= −∫ dθ
cos θ (1 + cos2 θ) In(1 + u)+ u)
log(1
= 6 ∫ (u2 − u + 1) log
In(1(1++u)du 6 ∫6 ∫
u)du− − du du
(u +(u1)+ 1)
cos θ II I|
= − ∫ sec θ dθ + 2∫ dθ
1 + cos2 θ  u3 2 
= 6  − u + u  log(1
In (1 ++u)u)
 3 2 
cos θ
= − log | sec θ + tan θ | + 2∫ dθ
2 − sin2 θ 2u3 − 3u2 + 6u 1 2
−∫ du − 6 [In(1 + u)]
[log(1 + u)]2
u+1 2
dt
= − log | sec θ + tan θ | + ∫ 2 − t2 , where sin θ = t = (2u3 – 3u2 + 6u) log (1 + u)
 11u 
= − log | sec θ + tan θ | +2 ·
1
log
2 + sin θ
+c − ∫  2u2 − 5u +  du − 3[In(1
[log(1+ u)]2 2
+ u)]
 u+1
2 2 2 − sin θ
= (2u3 – 3u2 + 6u) log (1 + u)
− log cot x + cot2 x − 1
=
2u3
2u3 55 22 
=
−− −− uu ++11u
11u−−11
11log
InIn(u
(u++1)
1) [log(1
−−3[In(1 +22u)]2
+ u)]
1 2 + 1 − tan2 x
=  33 22  3[In(1 + u)]
+ log +c  
2 2 − 1 − tan2 x
3 2/3 12 7/12 12
∴=I x − x + 2x1/2 − x5/12 + 3x1/3 − 4x1/ 4
2 7 5
 1 In(1 + 6 x )  – 6x1/6 – 12x1/12 + 12 log (x1/12+1)
12: Let I
Sol= ∫  3 x + 4 x + 3
x + x 
 dx
 + (2x1/2 – 3x1/3 + 6x1/61111/1) log(1+ x1/6)
∴ I = I 1 + I2 ,
 2 2 1/2 5 5 1/31/3 1/61/6 
− −  x1/2
x − − x x 11x 11x − −1111 In(1+ +x1/6
In(1
log x1/6))
 1  In(1 + 6 x ) 3 3 2 2 
=where I1 ∫=
3  dx , I2 ∫ dx
4 3
 x+ x x+ x
M a them a ti cs | 22.71

– 3 [log (1 + x1/6)]2 + c ∴ From Eq. (i)

3 2/3 12 7/12 4 1/2 12 5/12 1 1 3 x


= x − x + x − x I=– log |x+1| + log |x2 +1| + tan–1x + +c
2 7 3 5 2 4 2 2
x +1
1
+ x1/3 − 4x1/ 4 − 7x1/6 − 12x1/12  2x + 2 
2 Sol 14: Let I = ∫ sin−1   dx
 2 
(2x1/2
++(2x 1/2
3x1/3
−−3x 1/3
6x1/6
−−6x 1/6
++11) In(1++xx1/6
11)log
In(1 1/6
))  4x + 8x + 13 
 2x + 2 
+ 12log(1 + x1/2) − 3[log(1 + x1/6)]2 + c −1   dx
= ∫ sin
 (2x + 2)2 + 9 
 
x3 + 3x + 2 x3 + 2x + x + 2 Put 2x + 2 = 3 tan θ ⇒ 2dx = 3sec2qdθ
Sol 13: =
2 2 2 2
(x + 1) (x + 1) (x + 1) (x + 1)
−1
 3tan θ  3
2 = ∴I ∫ sin   · sec2 θ dθ
x(x + 1) + 2(x + 1) x 2  2  2
= = +  9 tan θ + 9 
(x2 + 1)2 (x + 1) (x2 + 1)(x + 1) (x2 + 1)2
−1  3tan θ  3 2
Again,
x
=
Ax + B
+
C = ∫ sin 
 3sec θ
 · sec θ dθ
 2
(x + 1)(x + 1) (x + 1) (x + 1)
2 2

−1  sin θ  3 2
⇒ x = (Ax + B) (x + 1) + C (x2 + 1) = ∫ sin 
 cos θ · sec θ
 · sec θ dθ
 2
Putting x = –1,we get –1 = 2C ⇒ C = –1/2
3
sin−1 (sin θ)·sec2 θ dθ
2∫
Equation coefficient of x2, we get =

0 = A + C ⇒ A = –C = 1/2
3 3
= ∫ θ ·sec2 θ dθ = [θ · tan θ – ∫ 1· tan θ dθ]
Putting x = 0,we obtain 2 2
0 = B + C ⇒ B = –C = 1/2
3
= [θ tanθ − log sec θ ] + c
3
x + 3x + 2 x +1
1 2 2
= +−
2 2
(x + 1) (x + 1) 2(x + 1) 2(x
2 + 1) (x + 1)2
2
 2
3  −1  2x + 2   2x + 2   2x + 2  
= tan  ·
   − log 1 +   +c
x3 + 3x + 2 2  3   3   3   1
∴I = ∫ dx  
(x2 + 1)2 (x + 1)  2x + 2  3   2x + 2 
2
= (x + 1)tan−1   − log  1 +   + c
1 dx 1 x +1 dx  3  4   3   1
=– ∫  
2 x + 1 2 ∫ x2 + 1
+ dx + 2∫
(x2 + 1)2
 2x + 2  3
1 1 1 = (x + 1)tan−1  2
 − log(4x + 8x + 13) + c
⇒ I = – log |x+1| + log |x +1| + tan x + 2I1 ...(i)
2 –1
 3  4
2 4 2
dx  3 
where I1 = ∫  let log3 + c1 = c
(x2 + 1)2  2 

Put x = tan θ ⇒ dx = sec2 θ d θ


Sol 15: For any natural number m, the given integral
sec2 θdθ 2 1 can be written as,
I1 ∫ ∫ cos θ= 2∫

= = dθ (1 + cos2θ)dθ
(tan2 θ + 1)2
3m
I= ∫ (x + x2m + xm )(2x3m + 3x2m + 6xm )1/m dx
1 1  1 1 tan θ
= θ + sin2θ = θ+ ·
2 2  2 2 (1 + tan2 θ) 3m
⇒I
= ∫ (2x + 3x2m + 6xm )1/m (x3m−1 + x2m−1 + xm−1 ) dx
1 1 x
= tan−1 x + ·
2 2 (1 + x2 ) Put 2x3m + 3x2m + 6xm = t
2 2 . 7 2 | Indefinite Integration

⇒ (6mx3m–1 + 6mx2m–1 + 6mxm–1)dx = dt 1  1 t2 


=
−  + 
11  11 7 
1
+1 r 2 
dt 1 tm
1/m 1 1 1 2
=∴ I ∫=
t ·
6m 6m  1  =
−  + ( sec x + tanx )  + k .
11 7
( sec x + tanx )
11/2
 + 1  
m 
(m+1)
1
= · (2x3m + 3x2m + 6xm ) m +c
6(m + 1)

Sol 16: (C)

(
ex e2x − 1 ) (z 2
−1 ) where z = ex
=J−I ∫ e=4x
+ e2x + 1
dx ∫ z 4 + z + 1 dz
  1 1 
 1 −1 − 2  dx  dx
1  e e+x + e−ex−−x 1− 1 
x
  z z2  = 1 In
 x
In
= log 
J −J −
I= ∫ ∫ z +z +1 1− 1− 1 2 2  e e+x +e−ex−+x 1+ 1 
I=
 
  z z 

1  exex++e−ex− x−−11 
∴ J − I = 1In
log
In    + c.
22  exex++e−ex− x++11 
 

sec2 x
Sol 17: (C) I = ∫ 9
dx
( sec x + tanx ) 2

Let sec x + tanx =


t
⇒ sec x − tanx =
1/t
sec x ( sec x + tanx ) dx =
dt
S
dt 1  1 
sec x=
dx , t += sec x
t 2 t

 1
t +  1  −9 2 −12 
1  t  dt
I= ∫
2
=
9 t

2 
t + t 2 dt

t 2

 −9 +1 −13 +1 
1t 2 t 2 
=  
2 9 13
− + 1 − 2 + 1
 2 

 −7 −11 
1 t 2 t 2 
=  + 
2 7 11 
− −
 2 2 

1 −7 1 −11
− t 2 − t 2
=
7 11

1 1 1 1
=
− −
7 2 11 11 2
7
t t
2017-18 100 &
op kers
Class 12 T
By E ran culty
-JE Fa r
IIT enior emie .
S fP r es
o titut
Ins

MATHEMATICS
FOR JEE MAIN & ADVANCED
SECOND
EDITION

Exhaustive Theory
(Now Revised)

Formula Sheet
9000+ Problems
based on latest JEE pattern

2500 + 1000 (New) Problems


of previous 35 years of
AIEEE (JEE Main) and IIT-JEE (JEE Adv)

5000+Illustrations and Solved Examples


Detailed Solutions
of all problems available

Topic Covered Plancess Concepts


Tips & Tricks, Facts, Notes, Misconceptions,
Definite Integration Key Take Aways, Problem Solving Tactics

PlancEssential
Questions recommended for revision
23. D E F I N I T E I N T E G R AT I O N

1. INTRODUCTION
Let f(x) be a continuous function defined on a closed interval [a, b] and ∫ f(x)dx
= F(x) + c then
b b

∫ f(x)dx
= [F(x)]ba or ∫ f(x)dx
= F(b) − F(a) is called the definite integral of f(x) within limits a and b. The interval
a a
[a, b] is called the range of integration. Every definite integral has a unique solution.

b π /2
Note: = F(b) − F(a) also represents the net area of the curve f(x) with x-axis.
∫ f(x)dx ∫ sin2 x dx 
a 0

π /2 π /2 π /2
 1 − cos2x  1 sin2x  1 π  π
Sol: ∫ sin2 x dx = ∫   dx = x −  =  − 0 =
0 0  2  2  2 0 2  2  4

1
Illustration 1: If ∫0 (3x
2
0, find the value of k.
+ 2x + k)dx = (JEE MAIN)

∫ 3x + 2x + k dx is already given i.e. 0 hence by using simple integral


2
Sol: Here the answer of the definite integral

0
formulas we can solve it and by comparing it to 0, we will obtain the value of k.
1 2
Here, we have, ∫0 (3x + 2x + k)dx =
0
1
 x3 x2  1
3 + 2 + kx  = 0 ;  x3 + x2 + kx  =0
 0
 3 2  0
(1 + 1 + k) – (0 + 0 + 0) = 0 ; 2 + k = 0 ⇒ k = –2
π
4
Illustration 2: Evaluate: ∫ (2sec
2
x + x3 + 2)dx.  (JEE MAIN)
0
b b b
Sol: As we know ∫ {f ( x ) ± g ( x )} dx = ∫ f ( x ) dx ± ∫ g ( x ) dx . Hence by using this method we can solve the given
definite integral. a a a
π π π π
4 4 4 4
We have, ∫ (2sec
2
x + x3=
+ 2)dx 2 ∫ sec2 x dx + ∫ x3 dx + 2 ∫ dx
0 0 0 0

π/ 4
π/ 4  x4   π   ( π / 4)4  π 
= 2 tanx  +  + 2[x]0π / 4 = 2  tan − tan0  +  − 0 + 2  − 0
0
 4  0  4   4   4 
2 3 . 2 | Definite Integration

 π4  π π4 π
= 2(1 − 0) +  − 0  + =2 + +
 45  2 1024 2
 

2. PROPERTIES OF DEFINITE INTEGRALS

Property 1
b b b

∫=
f(x)dx ∫=
f(t)dt ∫ f(u)du
a a a

Here x is a dummy variable; it can be replaced by any other variable t, u,……..


π /2 π /2 π /2

∫= ∫ sint dt ∫=
sin(x)dx = sinudu
0 0 0
10 10 10
This is similar to the summation property ∑
= r2 ∑=
t2 ∑
= u2 ……..
=T 1=T 1=U 1

Property 2
b a

∫ f(x)dx = −∫ f(x)dx
a b

i.e. the interchange of limits of a definite integral changes only its sign.

Property 3
b c b

∫=
f(x)dx ∫ f(x)dx + ∫ f(x)dx (a < c < b)
a a c

Generally, this property is used when the integrand has two or more rules in the integration interval
b c1 c b
⇒ ∫a f(x)dx
= ∫ f(x)dx + ∫ 2 f(x)dx + ....... + ∫cn f(x)dx where a < c1 < c2 < ……. cn < b.
a c1

4 2x + 8, 1 ≤ x ≤ 2
Illustration 3: Evaluate: ∫1 f(x)dx, where f(x) = 6x, (JEE MAIN)
2≤x≤4

b c b
Sol: Here as we know, ∫=
f(x)dx ∫ f(x)dx + ∫ f(x)dx where (a < c < b) . Hence by using this property and solving by
a a c
using the integral formula we can solve it.
4
We have, I = ∫1 f(x)dx
2 4 2 4
= ∫1 f(x)dx + ∫2 f(x)dx = ∫1 (2x + 8)dx + ∫2 6x dx
2 4
 x2 + 8x  + 3x2  = (2)2 + 8(2) − (1)2 − 8(1) + 3(4)2 − 3(2)2 
=
 1  2    
= 11 + 36 = 47.
2
Illustration 4: Evaluate : ∫ | 1 − x |dx  (JEE MAIN)
0
M a them a ti cs | 23.3

1 − x, when 0 ≤ x ≤ 1
Sol: Here |1 – x| =  therefore, similar to the problem above, we can solve it.
x − 1, when 1 ≤ x ≤ 2
1 − x, when 0 ≤ x ≤ 1
|1 – x| = 
x − 1, when 1 ≤ x ≤ 2
1 2 1 2
 x2   x2 
∴ I =∫ (1 − x)dx + ∫ (x − 1)dx = x −  +  − x  = (1/2 – 0) + (0 + 1/2) = 1
0 1  2   2 1
0

Property 4
a a

∫ f(x)dx
= ∫ f(a − x)dx
0 0

Property 5
b b

∫ f(x)dx= ∫ f(a + b − x)dx


a a
b
f(x) b−a
Application: ∫ f(x) + f(a + b − x) dx = 2
a

PLANCESS CONCEPTS

With the help of the above property, the following integrals can be obtained.
π /2 π /2 π /2 π /2

∫ f(sinx)dx = ∫ f(cos x)dx ; ∫ f(tanx)dx = ∫ f(cot x)dx


0 0 0 0

π /2 π /2 1 1

∫ f(sin2x)sinx dx = ∫ f(sin2x)cos x dx ; ∫ f(logx)dx


= ∫ f[log(1 − x)]dx
0 0 0 0

π /2 π /2
sinn x cosn x π
∫ sinn x + cosn x dx
= ∫ cosn x + sinn x dx 4
=
0 0

π /2 π /2 π /2 π /2
tann x cotn x π 1 1 π
∫ 1 + tann x dx
= ∫ 1 + cotn x dx 4 ; =
= ∫ 1 + tann x dx ∫ 1 + cotn x dx 4
=
0 0 0 0

π /2 π /2 π/ 4
secn x cosecn x π π
∫ secn x + cosecnx dx
= ∫ cosecnx + secn xdx 4 ;
= ∫ log(1 + tanx)dx =
8
log2
0 0 0

π /2 π /2

∫ log cot x dx
= ∫ log tanx dx 0
=
0 0

Shrikant Nagori (JEE 2009 AIR 30)

1 −1 1
Illustration 5: Prove that ∫0 cot (1 − x + x2 )dx =2∫ tan−1 x .dx  (JEE MAIN)
0
a a
a b
Sol: As we know cot −1   = tan−1   and ∫ f(x)dx
= ∫ f(a − x)dx by using these two formulae we can solve the
b
  a 0 0
given problem.
2 3 . 4 | Definite Integration

 1+ x − x 
1 −1 2 1 −1  1  1 −1 1 −1  x + (1 − x) 
∫0 cot=
(1 − x + x )dx ∫ tan
0
=  2
1 − x + x 
∫0 tan=
  ∫ tan
1 − x (1 − x )  0

 − x (1 − x) 
1
 dx
 
1 −1 1 −1 1 −1
= ∫0 tan x dx + ∫ tan (1 −=
0
x)dx 2∫ tan x .dx
0

 −1  a + b  −1 −1 
 tan  =  tan a + tan b 
  1 − ab  
1
1 
Illustration 6: Find the value of ∫ log  − 1  dx  (JEE MAIN)
0  x 
a a
1 − x 
Sol: Here log  =  log (1 − x ) − log ( x ) and ∫ f(x)dx
= ∫ f(a − x)dx by using these two formulae we can solve it.
 x  0 0
1 1 1 1 1 1 1
1 − x 
∫ log  x  dx = ∫ log (1 − x)dx − ∫ log (x)dx = ∫ log 1 − (1 − x )dx − ∫ =
logx dx ∫ logx dx − ∫ logx dx
0 0 0 0 0 0 0
1 1
= ∫ log (x)dx − ∫ log (x)dx = 0
0 0

π /2
asinx + bcos x
Illustration 7: Evaluate: ∫ dx  (JEE MAIN)
0
sinx + cos x
a a π /2
asinx + bcos x
Sol: As ∫ f(x)dx
= ∫ f(a − x)dx therefore we can write ∫ sinx + cos x
dx in the form of
0 0 0

π /2
asin( π / 2 − x) + bcos( π / 2 − x)
∫ sin( π / 2 − x) + cos( π / 2 − x)
dx and then adding these two equations we can solve the given problem.
0
π /2
asinx + bcos x
I= ∫ sinx + cos x
dx  … (i)
0

π /2 π /2
asin( π / 2 − x) + bcos( π / 2 − x) acos x + bsinx
I ∫ sin(π / 2 − x) + cos(π / 2 − x) dx
= ∫ sinx + cos x
dx  … (ii)
0 0

Adding (i) and (ii),


π /2 π /2
(a + b)(sinx + cos x)
∴ 2I = ∫ sinx + cos x
dx = ∫ (a + b)dx = (a + b)π / 2 ⇒ I = (a + b) π/4
0 0

π /2
sin2 x 1
Illustration 8: Show that ∫ = dx log( 2 + 1)  (JEE ADVANCED)
0
sinx + cos x 2
Sol: This problem is similar to the problem above.
π /2 sin2 x
Let I = ∫0 dx  … (i)
sinx + cos x
By property 4, we have

π /2 sin2 ( ( π / 2) − x ) π /2 cos2 x
I ∫0 sin ((π / 2) − x ) + cos ((π / 2) − x ) dx
= ∫0 sinx + cos x
dx  … (ii)

Adding (i) and (ii), we get


M a them a ti cs | 23.5

2
π /2 sin x + cos2 x 1 π /2 dx 1 π /2 1
2I = ∫0 sinx + cos x
dx ⇒ I = ∫
2 0 sinx + cos x
=
2 2
∫0
(1 / 2)sinx + (1 / 2)cos x
dx

1 π /2 1 1 π /2 1
= ∫ dx = ∫ dx
2 2 0 cos( π / 4)sinx + sin( π / 4)cos x 2 2 0 sin ( x + ( π / 4) )
π /2
1 π /2  π 1   x π  
=
2 2
∫0 cosec  x=

+  dx
4
logtan  +  
2 2   2 8  0

1  π π π  1  tan(3π / 8)  1  cot( π / 8) 
= log tan  +
=  − log tan  log 
=  log  
2 2  4 8 8  2 2  tan( π / 8)  2 2  tan( π / 8) 
2 π 1
= =logcot log( 2 + 1)
2 2 8 2
3π / 4 tanx
Illustration 9: Evaluate : ∫−π / 4 1 + dx  (JEE ADVANCED)
tanx
sinx b b
Sol: By putting tanx = and using the property = ∫ f(a + b − x)dx , we can solve the given problem.
∫a f(x)dx
cos x a

3π / 4 tanx 3π / 4 sinx
Let I = ∫−π / 4 1 + dx ⇒ I= ∫−π / 4 dx  … (i)
tanx cos x + sinx
b b
On applying = ∫ f(a + b − x)dx we get
∫a f(x)dx a

3π / 4 sin ( (3π / 4) − ( π / 4) − x )
I= ∫−π / 4 dx
cos ( (3π / 4) − ( π / 4) − x ) + sin ( (3π / 4) − ( π / 4) − x )

3π / 4 sin ( ( π / 2) − x )
= ∫−π / 4 dx
cos ( ( π / 2) − x ) + sin ( ( π / 2) − x )

3π / 4 cos x
= ∫−π / 4 dx  … (ii)
sinx + cos x
Adding (i) and (ii), we get

3π / 4 sinx 3π / 4 cos x 3π / 4 sinx + cos x


2I ∫−π / 4 dx + ∫ dx = ∫−π / 4 dx
−π / 4
sinx + cos x sinx + cos x sinx + cos x

3π / 4  3π  π    3π π  π
= ∫−π / 4 dx= [x]3−ππ //=
4
4  −  − =   + =π ⇒ I =
 4  4   4 4  2

π /2  4 + 3sinx 
Illustration 10: The value of ∫0 log   dx is 
 4 + 3cos x 
(JEE ADVANCED)

π /2
 4 + 3sinx 
Sol: Similar to the problems above, we can write ∫ log  4 + 3cos x  dx as
0

π /2  4 + 3sin ( ( π / 2) − x ) 
∫0 log   dx and then by adding these two equations we can solve the given problem.
 4 + 3cos ( ( π / 2) − x ) 
 
2 3 . 6 | Definite Integration

π /2  4 + 3sinx 
Let I = ∫0 log   dx
 4 + 3cos x 

On applying property 5, we get

π /2  4 + 3sin ( ( π / 2) − x ) 
I= ∫0 log   dx
 4 + 3cos ( ( π / 2) − x ) 
 

π /2  4 + 3 cos x  π /2  4 + 3 sin x 
= ∫0 log   dx = − ∫0 log   dx = −I ⇒ I = 0
 4 + 3 sin x   4 + 3 cos x 

π /2  4 + 3sinx 
Thus, ∫0 log   dx = 0
 4 + 3cos x 

π /2 dx
Illustration 11: I = ∫0  (JEE ADVANCED)
4 + 5sinx

x
2 tan
2 x
Sol: Let sinx = and then by putting tan = t , we can solve the given problem.
x 2
1 + tan2
2
π /2 dx π /2 sec2 ( π / 2)dx
I= ∫0 = ∫0
4 + 5 (2 tan(x/ 2) / 1 + tan2 (x/ 2)) 4 + 4 tan2 ( π / 2) + 10 tan( π / 2)

x 1 x
Let tan = t ⇒ sec2 = dt
2 2 2
1
1 2dt 1 1 dt 1 1 1 1 1  t + (1 / 2)  1
⇒ ∫0 2
=
4 + 4t + 10t 2 ∫ (
=
)( )
0 t + (1 / 2) t + 2 3 ∫ (
0 t + (1 / 2)

) ( )
t +
=
2
dt
3
ln
 t + 2
= 
0 3
log2

π /3
dx
Illustration 12: Evaluate : ∫  (JEE ADVANCED)
π /6 1 + tanx

sinx b b
Sol: Let tanx = and then using property = ∫ f(a + b − x)dx , we can solve the given problem.
∫a f(x)dx
cos x a

π /3 π /3
dx cos x
∫ = ∫ dx  … (i)
π /6 1 + tanx π /6 sinx + cos x
π /3
cos( π / 2 − x) ∴
= ∫ sin( π / 2 − x) + cos( π / 2 − x)
dx [ here a + b = π/2]
π /6

π /3
sinx
= ∫ cos x + sinx
dx  … (ii)
π /6

π /3
π /3 π π π π
∴ 2I = ∫ 1dx = [x]π /6 = 3 − 6 = 6 ⇒I=
12
π /6
M a them a ti cs | 23.7

Property 6
a
 a

∫ f(x)dx =  ∫0
2 f(x) dx if f( −x) =f(x) (even function)
−a 
0 if f( −x) =−f(x) (odd function)

Note: This property is to be used if the integrand is either an even or odd function of x

π /2
Illustration 13: ∫−π /2 cos
2
x dx is equal to (JEE MAIN)

π /2 π /2
Sol: As ∫−π /2 cos
2
x dx = 2∫ cos2 x dx , therefore using property 7 we can solve it.
0

π /2 π /2 π /2
 sin2x  π
Here I = 2 ∫ cos2 x dx { f( −x) =f(x)} ; ∫ (1 + cos2x)dx =
x +  =
0 0  2 0 2

1 x3 sin(1 + x2 )
Illustration 14: ∫−1 dx is equal to  (JEE ADVANCED)
1 + x2
 a a
Sol: Here by using the property ∫ f(x)dx =  ∫0
2 f(x) dx if f( −x) =f(x) (even function)
−a 
0 if f( −x) =−f(x) (odd function)

x3 sin(1 + x2 ) x3 sin(1 − x2 )
Here f(x) = & f(–x) = –
1 + x2 1 + x2

f(x) = – f(x)
∴I=0
 a
f(x)dx =  ∫0
2a 2 f(x)dx, if f(2a − x) =
f(x)
Property 7: ∫0

0, if f(2a − x) =−f(x)

Note: The above property is used to halve the limits

2π sin2θ
Illustration 15: Evaluate : ∫0 dθ  (JEE MAIN)
a − b cos θ
 a
f(x)dx =  ∫0
2a 2 f(x)dx, if f(2a − x) =
f(x)
Sol: Let ∫0 . Hence by using this property we can solve the given problem.

0, if f(2a − x) =−f(x)

2π sin2θ sin2θ
=Let I ∫0 a − b cos θ
dθ → Let f(θ) =
a − b cos θ

sin2(2π − θ) − sin2θ
f(2π − θ) = = = −f(θ)
a − bcos(2π − θ) a − bcos θ

By property 7, we have
2π sin2θ
\ ∫0 a − b cos θ
dθ =0
2 3 . 8 | Definite Integration


Illustration 16: Evaluate ∫0 x sin4 x cos6 x dx  (JEE ADVANCED)

Sol: Similar to the problem above.


2π 2π
=I ∫0 x sin4 x cos6 x=
dx ∫0 (2π − x) sin4 x cos6 x dx

2π π
2I = 2π∫ sin4 x cos6 x dx ; I = 2π∫ sin4 x cos6 x dx ;
0 0

π /2 π /2
I= 4 π∫ sin4 x cos6 x dx ; I= 4 π∫ cos4 x sin6 x ;
0 0

2π π /2 dt
(sin2x)4 dx ⇒ 2x = t ⇒ dx =
16 ∫0
⇒ I=
2

 π 1 3π 3π2
=⇒ I
π π 4
16 ∫0
= sin t dt
π π /2 4
8 ∫0
sin t dt
π  1 π /2
⇒ I=  ∫
8 2 0
( )
sin4 t + sin4 t dt  = . . =
 8 2 8 128
na a
Property 8: If f(x) = f(x + a) (i.e. f(x) is a function with period a), then ∫0 f(x)dx = n∫ f(x)dx
0


Illustration 17: Evaluate: ∫0 sin8 x dx  (JEE MAIN)

Sol: Here sin8 (π – x) = sin8x, therefore by using this property, we can solve the given problem.
π /2
π 8 7.5.3.1 π 35π
I = 4 ∫= sin8 x dx ∫ sin x dx 8=
8= .
0
0
8.6.4.2 2 32


Illustration 18: Evaluate: ∫0 cos5 x dx  (JEE ADVANCED)

Sol: Let I = ∫0 cos5 x dx
Let f(x) = cos5x
f(2π – x) = cos5 (2π – x) = cos5x = f(x)
2π π
Then ∫0 cos5 x x dx = 2∫ cos5 x dx
0

Now, f(π – x) = cos (π – x) = (– cos x)3 = – cos5x


5

π 5
= – f(x) ; ∫0 cos x dx = 0

Hence ∫0 cos5 x dx = 0

Property 9
a+nT T

∫ f(x)dx = n∫ f(x)dx (if f(x + T) = f(x), and nÎN i.e. f(x) is a function with period T)
a 0
b +nT T b

∫ (n − m)∫ f(x)dx + ∫ f(x)dx


f(x)dx = m,n ∈ I
a+mT 0 a

200 π
Illustration=
19: I ∫ 1 + cos x dx (JEE MAIN)
0
M a them a ti cs | 23.9

200 π
x x
Sol: I = 2 ∫ cos
2
dx
2
=t
0

100 π π
⇒I= 2 2 ∫ | cos t | dt = 200 2 ∫ | cos t | dt = 400 2
0 0

h(x)
d
Property 10: ∫ f(t) dt = h’(x) f(h(x)) – g’(x) f(g(x))
dx g(x)

h(x)
d
Corollary (1):
dx ∫ f(t) dt = h’ (x) f(h(x)) [a is any constant independent of x]
a
x
d
dx ∫a
Corollary (2): f(t) dt = f(x)

b b
Property 11: ∫ f(x) dx ≤ ∫ | f(x) | dx
a a

b
Property 12: If f(x) ≥ 0 on [a, b], then ∫ f(x)dx ≥ 0
a

This property is also called the domination law.


There are a few more properties which might be helpful in solving problems
b b±c
1. Shift property: ∫ f(x)dx = ∫ f(x)dx
a a± c
b −b
2. Reflection property: ∫ f(x)dx =
− ∫ f( −x)dx
a −a
b b/k

∫ f(x)dx k
3. Expansion/Contraction property: = ∫ f(x)dx ∀ k > 0
a a/k

PLANCESS CONCEPTS

β dx
∫α (x − α )(x − β)
= π if (β > α )

β π
∫α (x − α )(x − β)dx= (β − α )2
8

b x−a π
∫a b−x
dx = (b − a)
2
x
If f(t) is an odd function, then φ(x) = ∫a f(t)dt is an even function.

x
If f(x) is an even function, then φ(x) = ∫a f(t)dt is an odd function.

Every continuous function defined on [a, b] is integrable over [a, b]


Every monotonic function defined on [a, b] is integrable over [a, b]
2 3 . 1 0 | Definite Integration

PLANCESS CONCEPTS

Change of variables: If the function f(x) is continuous on [a, b] and the function x = φ(t) is continuously
differentiable on the interval [t1, t2] and a = φ(t1), b = φ(t2), then
b t2
= ∫ f(φ( t ))φ '( t )dt .
∫a f(x)dx t1

Nitish Jhawar (JEE 2009 AIR 7)

3. SOME SPECIAL INTEGRALS

3.1 Walli’s Formula


π /2 π /2
n n (n − 1) (n − 3)....2
∫ sin x dx
= ∫ cos x dx
=
n(n − 2)....1
(if n is odd positive integer)
0 0

(n − 1) (n − 3).....1  π 
=   (if n is even positive integer)
n(n − 2).....2 2
π /2
Illustration 20: Evaluate ∫ cos7 x dx (JEE MAIN)

0

Sol: By using Walli’s formula we can solve the given problem.

6.4.2 16
=I =
7.5.3 35

3.2 Gamma Function


π /2 Γ ( (m+ 1) / 2 ) Γ ( (n+ 1) / 2 )
∫ sinm x cosn x dx =
0 2Γ ( (m+ n + 2) / 2 )

where Γ(n) is called the gamma function


OR
π /2
((m − 1) (m − 3).....(2 or 1)) (n − 1) ((n − 3)...(2 or 1))
∫ sinm cosn x dx =
0
(m + n) (m + n − 2).......(2 or 1)

(if m and n both are not simultaneously even positive integers)

((m − 1) (m − 3)....(1))((n − 1)(n − 3)....(1))  π 


  (if m and n are both even positive integers)
(m + n)(m + n − 2)....(2) 2

π /2
Illustration 21: Evaluate I = ∫ sin4 x cos5 x dx. (JEE MAIN)
0 
Sol: Using the gamma function formula i.e.
M a them a ti cs | 23.11

π /2 Γ ( (m+ 1) / 2 ) Γ ( (n+ 1) / 2 )
∫ sinm x cosn x dx =
0 2Γ ( (m+ n + 2) / 2 )

We can solve it.

r ( (4 + 1) / 2 ) Γ ( (5 + 1) / 2 ) Γ (5 / 2 ) Γ(3) ((3 / 2).(1 / 2))(=2.1 ) 8


=I = =
2Γ ( (4 + 5 + 2) / 2 ) 2Γ (11 / 2 ) 2 ( (9 / 2).(7 / 2).(5 / 2).(3 / 2).(1 / 2) ) 315

4. NEWTON LEIBNITZ FORMULA


In calculus, Leibnitz’s rule for differentiation under the integral sign named after Gottfried Leibnitz tells us that if
y1
we have an integral ∫ f(x, y)dy then for x in (x0, x1) the derivative of this integral is thus expressible as
y0

d  y1  y1
 ∫y f(x, y)dy  = ∫y0 fx (x, y)dy
dx  0 
provided that f and its partial derivative fx are both continuous over a region in the form [x0. x1] × [y0, y1].

5. SUMMATION OF SERIES BY INTEGRATION (LIMIT AS A SUM)


To find the sum of an infinite series with the help of definite integration, the following formula is used
n−1 1
 r 1
lim
n→ ∞
∑ f  n  n = ∫ f(x)dx
r =0   0

The following method is used to solve the questions on summation of series.


1 r
(i) After writing (r – 1)th or rth term of the series, express it in the form f  .
n n
n−1
1 r
Therefore the given series will take the form as lim
n→ ∞
∑nfn
  r =0
r
(ii) Now write ∫ in place of lim ∑ and x in place of and dx in place of n. We get summation in the form of
n→∞ n
1
integral ∫ f(x)dx .
0
b
b−a  b − a
Also we can write
= ∫ f(x)dx lim [f(a) + f(a + h) + .... + f(a + (n − 1 ) h)]  where h = 
n→ ∞ n  n 
a

 1 1 1
Illustration 22: Evaluate lim  + + ..... +  (JEE MAIN)
n→ ∞ n + 1 n + 2 2n 

n−1 1
 r 1
Sol: By using the summation of series by integration formula i.e lim
n→ ∞
∑ f  n  n = ∫ f(x)dx we can solve it.
r =0   0
n 1
1  1 1 1
Limit = lim ∑ n + r= lim ∑  . = ∫ 1 + x dx= [log(1 + x)]10 = log2
n→ ∞
r =1
n→ ∞  1 + (r / n) n  0

1100 + 2100 + 3100........n100


Illustration 23: lim  (JEE MAIN)
n→ ∞ n101
Sol: By observing the given problem, we can say that it’s a sum of an infinite series so by using the summation of
series by integration formula we can solve it.
2 3 . 1 2 | Definite Integration

100 n 100
r1001 r 1 r 1 100 1
Tr =
101
= × 
n n
; S = lim
n→ ∞ n
∑ n  ; = ∫0 x dx =
101
n r =1  

 n n 1 
Illustration 24: Find the value of lim  + + ..... +  (JEE ADVANCED)
n → ∞ (n + 1)

2
(n + 2)2 4n 

n 1 1
Sol: Here tr = = , therefore similar to the problem above, we can solve it.
(n + r)2 n [1 + (r / n)]2
n
1 1 1 1
Therefore the given series = lim
n→ ∞
∑ [1 + (r / n)]2 .
n ∫0 (1 + x)2
= dx
r =1

1 1
1  1  −1 1
Given series = ∫ (1 + x)2 dx = − 1 + x  = 2 + 1 = 2
0 0

Evaluate the following definite integrals as the limit of sums.

b
Illustration 25: ∫a cos x dx  (JEE ADVANCED)

b
b−a b−a
Sol: Here
= ∫ f(x)dx lim [f(a) + f(a + h) + .... + f(a + (n − 1 ) h)] where f(x) = cos x and h =
a
n→ ∞ n n
b
b−a
∴ ∫=
cos x dx lim [cosa + cos(a + h) + .... + cos(a + (n− 1)h)]
n→ ∞ n
a

b − a  cos ( a + ((n − 1) / 2 ). h) . sin (nh / 2 ) 


= lim . 
n→ ∞ n  sin(h/ 2) 
 n − 1 (b − a)   n . (b − a) 
cos  a + .  . sin  
b −a  2 n   2n 
= lim   .
n→ ∞  n  b −a
sin  
 2n 
b − a cos ( a + (1 − (1 / n)) / 2) (b − a) ) . sin ( (b − a) / 2 )
= lim 2 . .
n→ ∞ 2n sin ( (b − a) / 2n)

= lim 2 .
( )
cos a + (1 − (1 / n) ) ( (b − a) / 2 ) . sin ( (b − a) / 2 )
n→ ∞ sin ( (b − a) / 2n) / ( (b − a) / 2n)

b +a b −a
= 2cos   sin   = sin b – sin a
 2   2 

2
Illustration 26: ∫1 (x
2
+ x)dx  (JEE ADVANCED)

Sol: Similar to the problem above.


b − a 2 −1 1
=h = =
n n n
b
b−a
∫ f(x)dx nlim
=
→∞ n
[f(a) + f(a + h) + .... + f(a + (n− 1)h)]
a
M a them a ti cs | 23.13

2 2 1
∫1 (x + x)dx = lim [f(1) + f(1 + h) + .... + f(1 + (n− 1)h)]
n→ ∞ n

1 2
= lim [(1 + 1) + {(1 + h)2 + (1 + h)} + .... + {(1 + (n− 1)h)2 + (1 + (n− 1)h)}
n→ ∞ n

1
= lim [12.n + h(1 + 2 + ... + (n− 1)) + 1 . n + 2h(1 + 2 + ... + (n− 1)) + h2 (12 + 22 + ...(n− 1)2 )]
n→ ∞ n

1
Here h =
n
1  1 (n − 1)(n) 2 n(n − 1) 1 (n − 1)n(2n − 1) 
= lim n+ +n+ . + 
n→ ∞ n  n 2 n 2 n2 6 

= lim 1 +
(1 − (1 / n)) (1) + 1 + 2 (1 − (1 / n)) + (1 − (1 / n)) (1) ( 2 − (1 / n)) 
n→ ∞  2 2 6 

1 1 23
= 1+ +1+1+ =
2 3 6

6. INTEGRAL WITH INFINITE LIMITS


If a function f(x) is continuous for a ≤ x < ∞, then by definition,
∞ b
∫a f(x)dx = blim  … (i)
→ ∞ ∫a
f(x)dx

If there exists a finite limit on the right-hand side of (i), then the improper integral is said to be convergent;
otherwise it is divergent.
Geometrically, the improper integral (i) for f(x) > 0, is the area of the figure bounded by the graph of the function
y = f(x), the straight line x = a, and the x-axis. Similarly, we can define
b b ∞ a ∞
∫−∞ f(x)dx = alim and ∫=
→− ∞ ∫a ∫−∞ f(x)dx + ∫a f(x)dx
f(x)dx f(x)dx
−∞

7. IMPORTANT RESULTS
b π /2
If f(x) ≥ 0 and a < b, then ∫ f(x)dx ≥ 0, e.g. ∫ sinx dx =
1
a 0
a 0
If f(x) ≥ 0 and a < b, then ∫ f(x)dx ≤ 0, e.g. ∫ cos x dx =
−1
b π /2
a 0
If f(x) ≤ 0 and a < b, then ∫ f(x)dx ≥ 0, e.g. ∫ sinx dx =
1
b π /2
x 1 2 3 x

∫ [x]dx
= ∫ (0)dx + ∫ (1)dx + ∫ 2dx + ... + ∫ [x]dx, where [ ] denotes the greatest integer of x.
0 0 1 2 [x]
π /2 π /2
π
∫ log(sinx)dx = ∫ log(cos x)dx = − log2
2
0 0
π /2 π /2

∫ log(tanx)dx
= = ∫ (cot x)dx 0
0 0
2a a a a a

∫ f(x)dx= ∫ f(x)dx + ∫ f(2a − x)dx= ∫ f(x)dx + ∫ f(a + x)dx


0 0 0 0 0
2 3 . 1 4 | Definite Integration

b 1

∫ [x]dx= (b − a)∫ x dx, where [ ] denotes  the fractional part of x.


a 0
5 1
5
e.g., ∫= ∫ x dx
[x]dx 5=
0 0
2
f(b) b
Integral of an inverse function is given by ∫ f −1 (y)dy = bf(b) − af(a) − ∫ f(x)dx
f(a) a

Derivation of the given formula is given in the solved examples

8. GEOMETRICAL APPLICATION
y = f2(x)
The area of the figure bounded by the graphs of two continuous functions y = f1(x) and

y = f2(x), f1(x) ≤ f2(x), and two straight lines x= a and x = b is determined by the formula
b
=S ∫a (f2 (x) − f1 (x))dx . It is sometimes convenient to use formulae analogous to x.with

respect to y, i.e., regarding x as a function of y. In particular, the area bounded by the curve
d
x =f(y), the y-axis and the two abscissae y = c and y = d is given by ∫c f (y)dy. The area of
y = f1(x)
the figure bounded by the graphs of two continuous functions x = f1(y) and f2(y) (with f1(y)
Figure 23.1
d
≤ f2(y)), and the two straight lines y = c, y = d is given by ∫c (f2 (y) − f1 (y))dy
b
From the view of geometry we get an important inequality as if m ≤ f(x) ≤ M for a ≤ x ≤ b, then m(b – a) ≤ ∫ f(x)dx
≤ M(b – a) a

FORMULAE SHEET
Important results
b b b b b a
1. ∫ {f(x) ± g ( x ) ± h ( x )} dx= ∫ f(x)dx ± ∫ g(x)dx + ∫ h(x)dx 2. ∫ f(x)dx = −∫ f(x)dx
a a a a a b

b c b a a
3. ∫=
f(x)dx ∫ f(x)dx + ∫ f(x)dx (a < c < b) 4. ∫ f(x)dx
= ∫ f(a − x)dx
a a c 0 0

a
 a b b

5. ∫ f(x)dx =  ∫0
2 f(x) dx if f( −x) =f(x) (even function) 6. ∫ f(x)dx= ∫ f(a + b − x)dx
−a  a a
0 if ( −x) =−f(x) (odd function)

 a h(x)
d
8. ∫ f(t) dt = h’(x) f(h(x)) – g’(x) f(g(x))
f(x)dx =  ∫0
2a 2 f(x)dx, if f(2a − x) =
f(x)
7. ∫0 dx g(x)

0, if f(2a − x) =−f(x)

a+nT T 10. If f(x) = f(x + a) then


9. ∫ f(x)dx = n∫ f(x)dx (if f(x + T) = f(x), and n∈N i.e. f(x) is a na
∫0 f(x)dx = n∫ f(x)dx
a
a 0 0
function with period T)
M a them a ti cs | 23.15

b b b b/k
11. 12. =
∫ f(x)dx k ∫ f(x)dx ∀ k > 0
∫ f(x) dx ≤ ∫ | f(x) | dx a a/k
a a

d  y1  y1
13.  ∫y f(x, y)dy  = ∫y0 fx (x, y)dy (Leibnitz formula)
dx  0 

Definite integral of rational functions

∞ dx π xp −1dx
∞ π
1. ∫0 = 2.
x2 + a2 2a ∫0 1 + x sin(pπ) , 0 < p < 1
=

2 π /2 π /2 2 π π / 2 p > 0
∫ sin x dx
3. =
0 ∫=
0
cos x dx
4 4.
∞ sin(px) 
∫0 =
x
dx =
0 p 0
−π / 2 p < 0

2
∞ sin px πp 2x dx 2π
5. ∫0 x 2
=
2
6. ∫0 a + bsinx
=
a2 − b2

∞2 ∞ 2 1 π ∞ sinx ∞ cos x π
∫ sin ax dx
7. =
0 ∫0=
cos(ax )dx
2 2a
8. =
∫0 x dx ∫=
0
dx
2
x

∞ tanx π
9. ∫0 x
dx =
2

Advanced formulas

π 0 m,n int egers and m ≠ n


1. ∫ sin(mx) . sin(nx)dx = 
0
π / 2 m,n int egers and m = n

π 0 m,n int egers and m ≠ n


2. ∫ cos(mx) . cos(nx)dx = 
0
π / 2 m,n int egers and m = n

π 0 m,n int egers and m + n odd


3. ∫ sin(mx).cos(nx)dx =  2 2
0
2m / (m − n ) m,n int egers and m + n even

π /2 π /2 1.3.5....2m − 1 π
4. = 2m
∫ sin x dx ∫= cos x dx 2m
0 0 2.4.6....2m 2

Definite integrals of exponential functions

∞ a ∞ b
1. ∫ e−ax cos bx dx = 2. ∫ e−ax sin bx dx =
2 2
0
a +b 0
a + b2
2

∞ 2 1 π ∞ Γ(n + 1)
3. ∫ e−ax dx = 4. ∫ xne−ax dx =
0 2 a 0
an+1
2 3 . 1 6 | Definite Integration

m+1 ∞ x dx π2
Γ  6. ∫ =
2 2  0
ex − 1 6
5. ∫ xme−ax dx = 

0
2a(m+1)/2

∞ xn−1 1 1 1  ∞ x dx π2
7. ∫ dx =
Γ(n)  + + + .....  8. ∫ =
0 x
e −1 1n
2n
3n

0
ex + 1 12

∞ xn−1 1 1 1  ∞e
− ax
− e−bx 1  b2 + p2 
9. ∫ dx =
Γ(n)  − + − .......  10. ∫ dx = ln  
0 x n n n
e +1 1 2 3  0 x sec (px) 2  a2 + p2 

− ax − ax
∞e − e−bx b a ∞e (1 − cos x) a
11. ∫ = dx arctan − arctan 12. ∫0 dx = arccot a − ln(a2 + 1)
0 x csc(px) p p x 2 2

Solved Examples

JEE Main/Boards π
2 θ
π
= a∫ 2sin = dθ a∫ (1 − cos θ)dθ
0
2 0
Example 1: Evaluate:
a
dx
a
a−x = a(θ − sin θ)0π = a( π) = aπ.
(i) ∫ (ii) ∫ a+ x
dx
(a2 / 4) − ( x − (a / 2) )
2
0 −a π /2
sinx
Example 2: Evaluate ∫ sinx + cos x
dx
dx x 0
Sol: (i) As we know ∫ = sin−1 , therefore by a a
a2 − x2 a
using this formula we can solve the given problem.
Sol: Let ∫ f(x)dx
= ∫ f(a − x)dx .
0 0
(ii) Put x = a cos θ : θ ∈ [0, p] and solve it using the π /2
sinx
appropriate formula. By using this we can write ∫ sinx + cos x
dx
0
a
dx π /2
(i) ∫ sin ( π / 2) − x 
0 (a / 4) − ( x − (a / 2) )
2 2 as ∫ sin ( π / 2) − x  + cos ( π / 2) − x 
dx and by adding
0

a a
 x − (a / 2)   −1 2x − a  we can get the result.
=  sin−1  ; =  sin 
 (a / 2) 0  a 0 π /2 sin ( π / 2) − x 
I= ∫ dx
π sin ( π / 2) − x  + cos ( π / 2) − x 
= [sin 1–sin (–1)] = 2 sin (1) = 2 × = π . (ii)
–1 –1 –1 0
2
π /2
Then dx = –a sin θ dθ. Hence, cos x
= ∫ cos x + sinx
dx
0
a 0
a−x 1 − cos θ
∫ a+ x
dx = ∫ 1 + cos θ
( −asin θ)dθ π /2
sinx + cos x
π /2
π
−a π =
∴ 2I ∫ =
sinx + cos x
dx ∫ dx
=
2
0 0
π 2
2sin (θ / 2) θ θ π
= a∫ . 2sin cos dθ ∴ I=
0 2cos (θ / 2)2 2 2 4
M a them a ti cs | 23.17

1 3 3
1  1 9
2 −∫3 ∫=
Example 3: Evaluate ∫ log  − 1  dx = = | z | dz | z | dz .
0 x  0
2
2 4
1 − x 
Sol: Here log  =  log (1 − x ) − log ( x ) and (iii) I
= ∫ f(x)dx + ∫ f(x)dx
a a  x  1 2
∫ f(x)dx
= ∫ f(a − x)dx by using these two formulae we
2 4
0 0
can solve it. = ∫ (4x + 3)dx + ∫ (3x + 5)dx
1 2
1
1   3x2 
4
=I ∫ log  − 1  dx 2
x  = (2x + 3x)12 + + 5x 
0  2 
 2
(Put x = cos2t: cos t > 0; then dx = –2 cos t sin t dt)
= 9 + 28 = 37.
0
− ∫ log (sec2 t − 1) . 2cos t sint dt
= 1.7
2
π /2 Example 5: Evaluate I = ∫ [x ]dx, where [x] is the
π /2 π /2 greatest integer function 0

= ∫ log(tan2 t) . sin2t dt = 2 ∫ sin2t . log(tan t)dt


0 0 Sol: [x2] takes constant values 0, 1, 2 in intervals (0, 1),
π /2 (1, 2), ( 2, 3) respectively. By substituting these
=2 ∫ log(cot t) . sin2t dt values we will get the required result.
0
1 2 1.7
π /2
I = ∫ [x2 ]dx + 2
∫ [x ]dx + ∫ [x
2
]dx
∴ 2I 2
= ∫ log(tan t . cot t)=
× sin2t dt 0
0 1 2
0
1 2 1.7

Example 4: Evaluate:
= ∫ 0 dx + ∫ 1 dx + ∫ 2 dx
0 1 2
π
(i) I = ∫ | cos x | dx = 0 + ( 2 − 1) + 2(1.7 − 2) = 2.4 − 2
0

1 Example 6: Let f(x) be an odd function in the interval


(ii) I
= ∫ | 2x + 1 | dx  T T x
−2  − ,  with period T, prove that F(x) = ∫ f(t)dt is a
 2 2 a
4 4x + 3, 1 ≤ x ≤ 2 periodic function with period T.
(iii) I = ∫ f(x)dx, where f(x) =
3x + 5, 2 < x ≤ 4
1
Sol: As f(x) is an odd function.
Sol: (i) Here |cos(π – x)| = |cos x| hence | cos x | =cos x x+T x x+T

therefore using the formula ∫ cos x = sinx we can solve F(x+T)= ∫ ∫ f(t)dt +
f(t)dt = ∫ f(t)dt =
F(x) + I(x)
a a x
it. T
x+T 2
(ii) By putting 2x + 1 =we
z can solve it. where I(x) = ∫=
b c b
f(t)dt ∫=
f(t)dt 0 (since f is an odd
x T

(iii) As ∫=
f(x)dx ∫ f(x)dx + ∫ f(x)dx (a < c < b) 2
a a c function). Hence F(x) is a periodic function with period T.
By using this formula we can obtain the result. π
2
π /2 Example 7: Evaluate ∫ θ sin θ cos2 θ dθ 
(i) I = 2 ∫ | cos x |dx 0
0 a a
π /2
π /2
Sol: As we know, ∫ f(x)dx
= ∫ f(a − x)dx , hence by using
= 2 ∫ cos x=
dx 2(sinx)=
0 2(1)
= 2 0 0
0 this formula we can evaluate it.
1
π
(ii)
= I ∫ | 2x + 1 | dx (put 2x +=
1 z) 2
Let I = ∫ θ sin θ cos2 θ dθ
−2
0
2 3 . 1 8 | Definite Integration

π 3π
2 2
= ∫ (π − θ)sin (π − θ)cos (π − θ)dθ 2I = π ∫
4
dθ π
; Put θ = + y
0
π
1 + sin θ 2
π
2 2 4
= ∫ (π − θ)sin θ cos θ dθ π π
0
4 4
dy dy
π
2 2
π
2 2 = π∫ 2π ∫
=
=π∫ sin θ cos θ dθ − ∫ θ sin θ cos θ dθ π
1 + cos y 0
1 + cos y

0 0 4
π 2 π
 sin2θ  π/ 4
= π∫   dθ − I π 4
y  y π
sec2 dy = π tan 
2 ∫0
0
2  I= = π tan
2  2 0 8
π π
π 2 π  1 − cos 4θ 
⇒ 2I
=
40∫ sin= 2θdθ
4 ∫0 

2
 dθ

π
JEE Advanced/Boards
π sin 4θ  π2
= θ − = 1
8 4 0 8 x2
Example 1: Show that 1 < ∫e dx < e.
π2 0
∴ I= 2
16 Sol: ex is an increasing function in [0, 1]. Further, e0 ≤
n−1
1 n+r  2
Example 8: Evaluate lim ∑n   ex ≤ e1 ∀ x ∈ [0, 1]
n→∞
r =1 n−r 
1 1 1
2
Sol: Here by using the limit as a sum method we can ∴ ∫ 1 dx < ∫ ex dx < ∫ edx
solve the given problem. 0 0 0
1
n−1 2
1 n+r  or 1 < ∫ ex dx < e.
lim ∑n  
n→∞
r =1 n−r  0

x2 2
1 t − 5t + 4
= lim ∑
n−1
1 1+r /n
= ∫
1+x
dx
Example 2: If F(x) = ∫ 4 + e2t
dt, find the critical
r =1 n 1 − r / n 1−x
n→∞ 0
0 points of F(x).
1
1+x 1 dx 1 x dx
= ∫ dx = ∫0 +∫ Sol: By using Leibnitz rule we can write
2 2 0
0 1−x 1−x 1 − x2
x2 2
t − 5t + 4
= [sin–1x – 1 − x2 ]10
F(x) = ∫ 4 + e2t
dt,
0
π
= [sin–1 1 – 0] – [sin–1 0 – 1] = +1 (x2 )2 − 5x2 + 4
2 as F'(x) = . (2x) = 0.
2
3π 4 + e2x
4
θ By Leibnitz Rule,
Example 9: Integrate
= : I ∫ 1 + sin θ dθ (x2 )2 − 5x2 + 4
π F'(x) = . (2x)
4 2
a a 4 + e2x
Sol: As ∫ f(x)dx F’(x) = 0
= ∫ f(a − x)dx hence we can
0 0
3π 3π
⇒ (x4 – 5x2 + 4) x = 0
4 4
θ π−θ ⇒ (x2 – 4) (x2 – 1) x = 0
write ∫ 1 + sin θ
dθ as ∫ 1 + sin θ
dθ and then
π π ⇒ x = 0, ±1, ±2
4 4
π These are the critical points of F(x).
by putting θ= + y we can solve the given problem.
2
3π 3π
4 4
θ π−θ
=I ∫ =
1 + sin θ
dθ ∫ 1 + sin θ dθ
π π
4 4
M a them a ti cs | 23.19

π /2
 2(2 + h) (4 + h) 2(2 + h) 
Example 3: Evaluate: ∫ log sin x dx = lim  + (2a + 1) + 2(a2 + a) 
n→∞  6 2 
0
π /2
8 38
Sol: We can write ∫ log sin x dx =
3
+6+4 =
3
0
π /2 b
π 
As ∫ log sin  − x  dx and then by adding these two (ii) I = ∫ sin x dx
0 2  a

integration we can obtain the result.  n 


nh = b – a ; I= lim h  ∑ sin(a + rh) 
π /2 π /2
π  h→0  
 r =1 
I= ∫ log sin x dx = ∫ log sin  2 − x  dx
0 0 h
 n h 
π /2
= lim 2  ∑ 2sin sin(a + rh) 
 
= ∫ log cos x dx h→0 h
sin  r =1
2 
0 2
π /2 π /2 h
∴ 2I = ∫ (log sinx + log cos x)dx = ∫ log(sinx cos x)dx  n  h  h 
= lim 2  ∑ cos  a + hr −  − cos  a + hr +  
0 0
h→0 h   2  2  
π /2 π /2 π /2 sin  r =1
 sin2x  2
= ∫ log 
 2 
 dx = ∫ log sin2x dx − ∫ log2 dx h
0 0 0
  h  h 
π
π /2 = lim 2  cos  a +  − cos  a + nh +  
h 2 2 
= − log2 +
2 ∫ log sin2x dx (Put 2x =
t) h→0
sin   
0 2
π = cos a – cos b
π 1
= − log2 + ∫ log sint dt
2 20 3
Example 5: Evaluate =
I ∫ (| x − 2 | +2[x])dx, where [x] is
π /2
π 1 −1
= − log2 + (2)
2 2 ∫ log sint dt. the greatest integer function.
0
Sol: By putting x − 2 =and
y it is negative in interval -3
π π to -1 and positive in interval 0 to 1.
∴ 2I = – log2 + I ⇒ I =− log2
2 2 3

3
I1= ∫ | x − 2 |dx ; Put x − 2= y
−1
2
Example 4: Evaluate: (i)
= I ∫ (x + x)dx 1 −1 1

b
1
∫ | y |dy =∫ −y dy + 2∫ y dy
−3 −3 0
(ii) I = ∫ sin x dx as limit of a sum.
a 1
= − [y 2 ]−−31 + [y 2 ]10 = 4 + 1 = 5
Sol: By using the limit as a sum method we can solve 2
the problems above. 3

(i) f(x) = x2 + x, a = 1, b = 3, nh = 3 – 1 = 2
I2 = ∫ [x] dx
−1
n
0 1 2 3
=I lim h∑ f(a + rh)
n→∞
r =1
= ∫ −dx + ∫ 0 dx + ∫ dx + ∫ 2 dx = –1 + 0 + 1 + 2 = 2
−1 0 1 2
n
= lim h∑ ((a + rh) + (a + rh)) 2
∴ I = I1 + 2I2 = 9
n→∞
r =1

 n  x log x
= lim h  ∑ r 2h2 + rh(2a + 1) + (a2 + a) 
Example 6: Show that I = ∫ (1 + x2 )2 dx = 0
n→∞   0
 r =1 
Sol: By splitting the given integration into two intervals
 n(2 + h) (4 + h) n(2 + h)  i.e. from 0 to 1 and then 1 to ꝏ we can solve the given
lim h  + (2a + 1) + n(a2 + a)  problem.
n→∞  6 2 
2 3 . 2 0 | Definite Integration

∞ 1 ∞ b
x log x x log x x log x
∫ (1= + x2 )2
dx ∫ (1 + x2 )2 dx + ∫
(1 + x2 )2
dx Example 8: Evaluate ∫ (px + q)dx as a limit of a sum
0 0 1 a

Put x = 1/y in the second integral Sol: Here as f(x) = px + q, therefore using the limit as
∞ 0 1 sum method we can solve the given problem.
x log x y 4 log y y log y
∴ ∫ (1 + x2 )2 dx = ∫ y3 (1 + y 2 )2 dy = −∫
2 2
dy b

1 1 0 (1 + y ) =I ∫ (px + q)dx
a
1 1
x log x y log y
Thus I = ∫ (1 + x2 )2 dx − ∫ (1 + y 2 )2 dy =
0 = lim h[f(a) + f(a + h) +…..+ f(a + (n – 1)h)]
h→0
0 0
= lim h[(pa + q) + {p(a + h) + q} +……+
1 h→0
3 4
x  2x  {p(a + (n – 1)h) + q}]
Example 7: If I = ∫ cos−1   dx, then find
4
1 1−x  1 − x2  = lim h[p(a + a +…….+ a) + ph(1 + 2 +……+ (n – 1))

its value. 3 h→0

+ q(1 + 1 +……+ 1)]


2x   −2x  −1 
Sol: We can write cos  2 
as cos−1  
1 − x   1 − x2   1 
= lim h pna + pnh(n − 1) + qn
h→0  2 
 2x 
=  π − cos−1  and then by solving we will get the
 1 + x2   1 
= lim hpna + pnh(hn − h) + qnh  …..(i)
result. h→0  2 
1 1
Since, h = (b – a)/n, or nh = b – a, we obtain from (i)
3 4 3
x 2x  −1  x4 −2x 
−1   p 
I= ∫ 1−x 4
cos   dx =
 1 − x2 
∫ 1−x 4
cos   dx
 1 − x2 
I = lim (pa + q)(b − a) + (b − a)(b − a − h)

1

1 h→0  2 
3 3
p
1 = (pa + q) (b – a) + (b – a)2
3
2
x4
 −1 2x 
= ∫  π − cos
1−x  4  dx
1 + x2  =
p
(b – a) (2a + b – a) + q(b – a)

1 2
3
p
1 1 = (b2 – a2) + q (b – a).
3 4 3 2
x x4
π ∫
2I = 2π ∫
dx = dx π
4 4 1 − cos nπ
1 1−x 0 1−x
− Example 9: If Un = ∫ 1 − cos x
dx where n is a positive
3 0
1 integer or zero, then show that Un+2 + Un = 2Un+1.
3
 1  π /2
I = π( −1) ∫ 1 −  dx sin2 nθ nπ
0  1 − x4  Hence show that ∫ 2
sin θ
dθ =
2
0
1

π π 3
1 1 Sol: Here Un = Un+2 – Un+1 therefore by substituting
=– + ∫ 1 − x2 + 1 + x2 dx n+2 and n+1 in place of n and solving we will get the
3 2 0
required result.
1 π
1 − cos nx
=–
π π π π 3
1 1  Un = ∫ 1 − cos x
dx
+ . +
3 2 6 4
∫ +
1−x 1+x
dx 0
0 ∴ Un+2 – Un+1
1
π
π π2 π  |1 + x | 3 {(1 − cos(n + 2)x)} − {1 − cos(n + 1)x}
=– + +  log  = ∫ (1 − cos x)
dx
3 12 4  | 1 − x | 0 0
π
 3 +1 = cos(n + 1)x − cos(n + 2)x
π2 π π ∫ dx
= − + log   (1 − cos x)
12 3 4  3 −1  0
 
M a them a ti cs | 23.21

π 2sin (n + (3 / 2) ) x sin(x/ 2)  x 
= ∫ know   is an odd function
0 2sin2 (x / 2)  2 − cos 2x 

( )
π sin n + (3 / 2) x π/ 4
x
∫ sin(x/ 2) dx 
⇒ Un+ 2 − Un+1 = …..(i) therefore ∫ 2 − cos2x
dx = 0.
0 −π / 4

Similarly π
π/ 4
1
sin (n + (1 / 2) ) x
π
Therefore 0 +
4
2 ∫ 2 − cos2x
dx
0
⇒ Un+1 ∫ sin(x/ 2) dx 
− Un = ….(ii)
0  
x
from (1) and (2), we get This is because   is an odd function,
 2 − cos 2x 
(Un+2 – Un+1) – (Un+1 – Un)
 1 
whereas   is an even function
π sin (n + (3 / 2) ) x − sin (n + (1 / 2) ) x  2 − cos 2x 
= ∫ sin(x/ 2) π/ 4
0 π dx
π
2cos(n + 1)x sin(x/ 2)  sin(n + 1)x 
π
=
2 ∫ 2 − ((1 − tan x) / (1 + tan2 x))
2
0
= ∫ sin(x/ 2)
dx = 2 
 (n + 1) 0
 =0
0
π/ 4 π/ 4
π (1 + tan2 x)dx π sec2 x dx
∴ Un+2 + Un = 2Un+1 =
2 ∫ 2(1 + tan2 x) − (1 − tan2 x)
=
2 ∫ 1 + 3tan2 x
0 0
Hence proved
Now Un+2 – Un+1 =Un+1 – Un. Now let tan x = t ∴ sec2x dx = dt
1
Similarly implies π2
π dt π
( )
1
⇒ ∫
= = tan−1 3t
Un+2 – Un+1 = Un+1 – Un = Un – Un – 1 = …….. = U1 – U0 2 0 1 + 3t2 2 3 0 6 3
∴ Un – Un–1 = U1 – U0 = π – 0
Example11: Show that
⇒ Un = π + Un–1
1
π dx π
= π + π + Un–2 < ∫ <
6 0 4 − x 2 − x3 4 2
= 2π + Un–2

Un = nπ + U0 ……(3) [ U0 = 0] Sol: Since 0 < x < 1
Un = np 1 1 1
so < <
π /2 π /2
sin2 nθ 1 − cos2nθ 4 − x2 4 − x 2 − x3 4 − 2x2
Hence ∴ = ∫ sin2 θ dθ ∫ 1 − cos2θ

Hence by using the property:
0 0
dx b b
Put 2θ = x ∴ dθ =
2 If f(x) ≤ g(x) on [a, b], then ∫ f(x)dx ≤ ∫ g(x)dx we
π /2 π a a
sin2 nθ 1 1 − cosnx
Hence ∫ sin2 θ
dθ =
2 ∫0 1 − cos x
dx can solve the given problem.
0
Integrate the above relation
1 1
= U=
n nπ {from (1)} 1 1 1
2 2 dx dx dx
∫ < ∫ < ∫
π/ 4
x + ( π / 4) 0 4 − x2 0 4 − x2 − x2 0 4 − 2x2
Example 10: Solve ∫ 2 − cos2x dx. 1 1
−π / 4  −1 x  1 dx 1  −1 x 
 sin  <
2 0 ∫0 <  sin
2

2 0
π/ 4
x + ( π / 4)  4 − x 2 − x3
Sol: By splitting ∫ dx
2 − cos2x 1
−π / 4 π dx π
6
< ∫ 2 3
<
4 2
.
π/ 4
x π 1
π/ 4 0 4−x −x
= ∫ 2 − cos2x
dx +
4 ∫ 2 − cos2x dx and as we Hence proved.
−π / 4 −π / 4
2 3 . 2 2 | Definite Integration

JEE Main/Boards

Exercise 1 x2
Q.13 If f(x) = ∫ 1 + t2 dt , then find the value of f'(x).
1/2 0
dx
Q.1 ∫ π /2
π + 4x2
x − x2 Q.14 Evaluate
1/ 4
∫ dx.
−π /2 − cos ( | x | +( π / 3) )
π /2
dx
Q.2 ∫ (4 sin x + 5cos2 x)
2
x
log t
0 Q.15 If f(x) = ∫ l + t dt then prove that
1
π /2
sin2 x 1 1
Q.3 ∫ 1 + sinx cos x
dx (lnx)2 2. .
f(x) + f   = (logx)
x 2
0  
1
2t
Q.4 ∫ | 5x − 3 | dx Q.16
0
∫ | logx | dt
1
x
3
2x + 1, 1 ≤ x ≤ 2
Q.5 ∫ f(x)dx, where f(x) =  x2 + 1, x sin (n + (1 / 2) ) x
2≤x≤3
1  Q.17 ∫ 2sin(x/ 2)
dx, n ∈ N.
0
π/ 4
Q.6 ∫ | sinx |dx x
−π / 4 Q.18 If F(x) = ∫ (3sint + 4 cos t)dx . Find the
5x
π
x 4
Q.7 ∫ dx  5π 4 π 
(1 + sin2 x) least value of F(x) on the interval  , .
0
4 3 
2 π
2 4
Q.8 Evaluate using limit of a sum: ∫ (x + 1)dx
Q.19 If IA = ∫ tan
n
θ dθ , n ∈ N, then find n(In–1 + In+1)
0
and IB. 0
π /2
Q.9 Evaluate: ∫ | sinx − cos x | dx
0
Q.20 If ‘‘a’’ is a positive integer, solve for ‘‘a’’
a
Q.10 If f and g are continuous function on [0, a]  2  cos3x 3   −a3
satisfying f(x) = f(a – x) and g(x) + g(a– x) = 2 then, ∫   4 4
a + cos x 

+ asinx − 20 cos x 

dx ≤
3
.
0
show that
a
Q.21 If f(x) = sin x, then find its mean value on (–2, 0).
∫ f(x)g(x)dx = ∫ f(x)dx.
0
π
1
100 π Q.22 Evaluate I = ∫ dx.
Q.11 Evaluate: ∫ 1 − cos2x dx 0 x + a2 − x2
0
a a2
 x  dx n(n − 1)(4 π + 1)
Q.12 (i) Show that if f(t) is an odd function then ∫ f(t)dx Q.23 Show
= that I ∫=
  6
, where
0
is an even function w.r.t. x. 0
x [x] is the greatest integer function.
(ii) Can ∫ f(t)dt be an odd function if f(t)dt is an even
nx +λ
a

function?
Q.24 Show that I = ∫ | sinx | dx = 2n + 1 − cos λ , n∈N,
0 ≤ λ < π. 0
M a them a ti cs | 23.23

x
π π sin2x,sin ( ( π / 2)cos x ) 8 Q.4 ∫0 | cos x |dx equals
Q.25 Show that I = ∫ 2x − π
dx =
x2
.
0 (A) 1 (B) 2 (C) 3 (D) 4

Q.26 Let f and g be function satisfying the following 2


Q.5 ∫−2 | 2x + 3 |dx equals
conditions:
(i) f(0) = 1 (ii) f(x) = g(x), g’(x) = f(x) 25 25 25
(A) (B) 0 (C) (D)
(iii) g(0) = 0 (iv) g(x) ≥ 0 ∀ x ∈ R 2 4 3

Find f(1). 2
2
Q.6 ∫ |1− x | dx =
Q.27 Show that −2

π (A) 2 (B) 4 (C) 6 (D) 8


(i) ∫ log(1 + cos x)dx =
π log(1/ 2) ;
0
x2 2
t − 5t + 4
(ii) ∫
π /3
dx
=
π Q.7 The point of extremum of ∫ 2 + et
dt are
0
π /6 1 + cot x
12
(A) x = –2 (B) x = 1
Q.28 Prove that (C) x = 0 (D) All of the above
π
dx π π
∫ 1 − 2acos x + x2 − 1 − a2 or
2
a −1
; a > 0, Q.8 The point of intersection
0
x x
According as a < 1 or a > 1. F1(x) = ∫ (2t − 5) dt and F2 (x) = ∫ 2t dt, are -
a 2 0

∫ x dx
Q.29 (i) Evaluate lim 0  6 36  2 4 1 1  1 1 
n→0 α sin α (A)  ,  (B)  ,  (C)  ,  (D)  , 
 5 25  3 9 3 9  5 25 
a
dy
(ii) If y = x ∫ ln
logdxdx, Find at x = e.
x
dx Q.9 If f and g are continuous function on [0, a) satisfying
a
Q.30 Find the intervals of increase of f(x) defined by f(x) f(x) = f(a – x) and g(x)+g(a–x)=2,then I = ∫ f(x)g(x)dx =
α a 0 0
2 2
= ∫ (t + 2t) (t − 1)dt. (A) ∫ f(x)dx (B) ∫ f(x)dx
0 0 a
a

Exercise 2 (C) 2∫ f(x)dx (D) None of these


0

Single Correct Choice Type log5


ex ex − 1
Q.10 The value of integral ∫ dx =
1 1 − 2x, x < 0 0 ex + 3
Q.1 ∫ f(x) dx is equal to where f(x) = 
−1
1 + 2x, x ≥ 0 (A) 3 + 2p (B) 4 – p
(A) 4 (B) –4 (C) 2 (D) –2 (C) 2 + p (D) None of these
π
1 |x|
Q.2 ∫−1 e dx equals Q.11 The value of the integral ∫ sin mx sinnx dx for m
≠ n (m, n ∈ I), is - −α
(A) 2e (B) 2e – 1 (C) 2e – 2 (D) e – 2
(A) 0 (B) p (C) π/2 (D) 2p
1
Q.3 ∫0 [x]dx equals ; where [·] is G.I.F.

(A) 0 (B) 2 (C) 3 (D) 1


2 3 . 2 4 | Definite Integration

e Q.20 If [x] stands for the greatest integer


Q.12 ∫ | logx |dx = 10
[x2 ]
1/e function, the value of ∫ [x2 − 28x + 196] + [x2 ] dx is
1  1 4
(A) 1 − (B) 2  1 − 
e  e (A) 0 (B) 1 (C) 3 (D) None of these
(C) e – 1
–1
(D) None of these
3

π Q.21 The value of ∫ (| x − 2 | +[x])dx is ([x]


dx
Q.13 ∫ 1 − 2a cos x + a2 = −1
stands for greatest integer less than or equal to x)
0

π (A) 7 (B) 5 (C) 4 (D) 3


(A) (B) π(1 – a2)
2
2(1 − a )
π /2
π sin2 x
(C)
1− a
(D) None of these Q.22 ∫ sinx + cos x
dx is equal to
0
π
1 (A) (B) 2 log( 2 + 1)
2
Q.14 ∫ (1 − x)9 dx =
0 1
1 11 (C) log( 2 + 1) (D) None of these
(A) x (B) (C) (D) 2 2
10 10
π /2
π Q.23 If u10 – ∫ x10 sinx dx then the value of
dx
Q.15 ∫ 3
=
u10 + 90 u8 is
0
 x + x2 + 1 
0
 
 
8 9 9 9
3 1 3 π π π π
(A) (B) (C) − (D) None of these (A) 9   (B)   (C) 10   (D) 9  
8 8 8 2 2 2 2

Q.16 If [x] denotes the greatest integer less Q.24 For any integer n, the integral
5
π
than or equal to x, then the value ∫ [| x − 3 |]dx is - sin2 x
cos3 (2n + 1)x dx has the value
1
∫e
0
(A) 1 (B) 2 (C) 4 (D) 8
(A) p (B) 1 (C) 0 (D) None of these
π /2
sinx 2
Q.17 ∫ e− cos x dx is equal to - π /2

−π /2 1 + cos x
2 Q.25 The value of ∫ sin(log(x + x2 + 1))dx is
−π /2
(A) 2e–1 (B) 1 (C) 0 (D) None of these (A) 1 (B) –1 (C) 0 (D) None of these

Q.18 The value of Q.26 The value of α ∈ (-π, 0) satisfying


1/2 2α
  x + 1 2  x − 1 2 
1/2
∫−1/2  x − 1   x + 1  
   +   − 2 dx equal
sin α + ∫ cos2x dx = 0 is
α
  (A) –π/2 (B) –p (C) –π/3 (D) 0
(A) log (4/3) (B) 2 log (4/3) x4

(C) 4 log (4/3) (D) –4 log (4/3) Q.27 If f(x) = ∫ sin t dt, then f–1(x) equals
x2

Q.19 Let f(x) = x – [x], for every real number (A) sin x2 – sin x (B) 4x3 sin x2 – 2x sin x
1 (C) x4 sin x2 – x sin x (D) None of these
x, where [x] is integral pat of x. Then ∫ f(x)dx is
−1

(A) 1 (B) 2 (C) 0 (D) ½


M a them a ti cs | 23.25

π
4
Q.2 For any integer n, the integral
Q.28 ∫ x sinx cos x dx = x cos2 x
0 ∫0 e cos3 (2n + 1)x dx has the value  (1985)
π π π
(A) (B) (C) − (D) None of these (A) p (B) 1 (C) 0 (D) None of these
10 5 5

Q.29 If f(x) = ae2x + bex + cx, satisfies the conditions f(0) Q.3 Let f: R → R be a differentiable function and
f(x) 2t
= –1, f’(log 2) f(1) = 4. Then, the value of lim ∫ dt is (1990)
log 4 x→ 1 4 x −1
39
= 31, ∫ (f(x) − cx)dx = , then (A) 8f’ (1) (B) 4f’ (1) (C) 2f’ (1) (D) f’ (1)
0
2

(A) a = 5, b = 6, c = 3 (B) a = 5, b = –6, c = 3 π /2 dx


Q.4 The value of ∫0 is  (1993)
1 + tan3 x
(C) a = –5, b = 6, c = 3 (D) None of these
(A) 0 (B) 1 (C) π/2 (D) π/4
π/ 4 x 2
e sec x dx
Q.30 ∫ e2x − 1
is equal to
Q.5 The value of

∫0 [2sinx]dx where [·] represents the
−π / 4
greatest integral function, is  (1995)
(A) 0 (B) 2 (C) e (D) None of these
5π 5π
(A) − (B) – p (C) (D) –2p
a 3 3
Q.31  1 + x2  dx is equal to
∫ loga  x +  x 1
−1 Q.6 then the value of f(1) is
∫0 f(t)dt + x + ∫x t f(t)dt,
(A) 2 logaa (B) 0  (1998)
1 1
(C) loga2 + log a (D) None of these (A) (B) 0 (C) 1 (D) –
2 2

sin2 x Q.7
3π / 4 dx
is equal to  (1999)
Q.32 The value of ∫ (x / π) + (1 / 2) dx, where [x] ∫π / 4 1 + cos x
  −2
1 1
= the greatest integer less than or equal to x, is (A) 2 (B) –2 (C) (D) –
2 2
(A) 1 (B) 0 (C) 4 – sin 4 (D) None of these
Q.8 If for a real number y, [y] is the greatest integer
x less than or equal to y, then the value of the integral
2
Q.33 If f(x) = ∫ log(1 + t )dt then the value of f’(1) is 3π /2
[2sinx]dx is  (1999)
equal to 0 ∫π /2
π π
(A) 2 (B) 0 (C) 1 (D) None of these (A) –p (B) 0 (C) − (D)
2 2
x
dx π cos2 x
Q.34 ∫ 1 + 3cos x is equal to Q.9 The value of ∫−π 1 + ax dx, a > 0 , is  (2001)
0
π π
(A) p (B) 0 (C) (D) None of these (A) p (B) ap (C) (D) 2p
2 2
π
Q.10 Let f : (0, ∞) → R and F(x) = ∫0 f(t)dt, If
Previous Years’ Questions F(x2) = x2 (1 + x), then f(4) equals  (2001)
5
Q.1 The value of the integral (A) (B) 7 (C) 4 (D) 2
4
π /2 cot x
∫0 dx is  (1983) x
cot x + tanx Q.11 Let f(x) = ∫1 2 − t2 dt. Then, the real value of x if it

(A) π/4 (B) π/2 (C) p (D) None of these satisfies x2 – f’(x) = 0 are  (2002)
1 1
(A) ±1 (B) ± (C) ± (D) 0 and 1
2 2
2 3 . 2 6 | Definite Integration

Q.12 Let T > 0 be a fixed real number. Suppose, f is x

a continuous function such that for all x ∈ R, f(x + T) Q.20 ∫ cot x dx, . denotes the greatest integer
0
= f(x). If I =
T 3+3T
function, is equal to  (2009)
∫0 f(x)dx, then the value of ∫3 f(2x)dx,
π π
 (2002) (A) (B) 1 (C) -1 (D) −
3 2 2
(A) I (B) I (C) 3I (D) 6I
2
Q.21 Let p ( x ) be a function defined on R such that
x2 +1 − t2 p (=
x ) p (1 − x ) for all p ( 0 ) = 1 p (1 ) = 41 .Then
Q.13 If f(x) = ∫x2
e dt, then f(x) increases in (2003) 1

(A) (2, 2) (B) No value of x (C) (0, ∞) (D) (– ∞, 0) ∫ p ( x ) dx equals.  (2010)


0

1 1−x (A) 21 (B) 41 (C) 42 (D) 41


Q.14 The value of the integral ∫0 1+x
dx is (2004)
1 8log (1 + x )
π π Q.22 The value of . is 
(A) + 1 (B) − 1 (C) –1 (D) 1 ∫ 1 + x2
dx (2011)
2 2 0
π π
(A) log2 (B) log2 (C) log2 (D) π log2
Q.15 Match the conditions expressions in column I with 8 2
statement in column II (2007)
x
Q.23 If g ( x ) = ∫0 cos 4t dt , then g ( x + π ) equals (2012)
Column I Column II g(x)
(A) (B) g ( x ) + g ( π )
1 dx 1 2 g ( π)
(A) ∫−1 1 + x2 (p) log  
2 3 (C) g ( x ) − g ( π ) (D) g ( x ) .g ( π )
π /3
1 dx 2 dx
(B) ∫0 (q) 2log  
3
Q.24 Statement-I : The value of the integral ∫
1 − x2 π /6 1 + tanx
is equal to π / 6 .
3 dx π
(C) ∫2 1 − x2 (r) b b
3 Statement-II: ∫ f ( x ) dx= ∫ f ( a + b − x ) dx  (2013)
a a
2 dx π
(D) ∫1 (s) (A) Statement-I is true; statement-II is true; statement-II
x x −12 2 is a correct explanation for statement-I.
(B) Statement-I is true; statement-II is true; statement-II
2 2 is not a correct explanation for statement-I.
Q.16The value of ∫−2 | 1 − x | dx is….. (1989)
(C) Statement-I is true; statement-II is false.
3π / 4 x (D) Statement-I is false; statement-II is true.
Q.17The value of ∫π / 4 1 + sinx
dx.....  (1993)
1
 1  e+
Q.25 The integral ∫  1 + x −  e x dx is equal to (2014)
3 x  x
Q.18The value of ∫2 dx is…..  (1994)
5−x + x x+
1
x+
1
(A) ( x + 1 ) e x + c (B) −xe x +c
1 1
sinx cos x
Q.19 Let = ∫ x
dx and J = ∫ x
dx .
(C) ( x − 1 )
x+
1
x + c (D) xe
x+
1
x +c
0 0

Then which one of the following is true?  (2008) 4


logx2
Q.26 The integral ∫ logx2 + log 36 − 12x + x2 dx
(A) I >
2
3
2
and J > 2 (B) I < and J < 2
3
2 ( )
is equal to  (2015)
2 2
(C) I < and J > 2 (D) I > and J < 2 (A) 1 (B) 4 (C) 1 (D) 6
3 3
M a them a ti cs | 23.27

JEE Advanced/Boards

Exercise 1 Q.7 Let h(x) = (fog) (x) + K where K is any


d sinx
1 constant. If (h(x)) = – then
tan−1 x −1 dx −2
Q.1 ∫e sin (cos x)dx. cos (cos x)
0
compute the value of j(0) where j(x)
f(x)
Q.2 Prove that : f(t)
∫ dt, where f and g are trigonometric functions.
1 g(t)
(β − α )2 x g(x)
(i) ∫ (x − α )(β − x)dx −
8
0
π /2
a 1 − sin2x
(ii) ∫
x−α
dx = (β − α )
π Q.8 ∫ 1 + sin2x
dx
0
β−x 2 0

a 2
dx π Q.9 If the value of the definite integral I = ∫ (3x2 − 3x + 1)
(iii) ∫x = where α, β > 0
0 (x − α )(β − x) αβ 0
cos(x3 – 3x3 + 4x – 2) dx can be expressed in the form as
b
x dx π p(sin q where p, q ∈ N, then find (p + q).
(iv) ∫ = ( α + β) where α < β
0 (x − α )(β − x) 2
3
nπ 2x7 + 3x6 − 10x5 − 7x3 − 12x2
Q.3 (i) Let β(Π) = ∫ 1 − sint dt.
Q.10 ∫ x2 + 2
dx .
− 2
0
Find the value of β(2) – β(1).
Q.11 For a ≥ 2, if the value of the definite integral
(ii) Determine a positive integer n ≤ 5, such that
a
1 dx x
x
∫ e (x − 1)
n
dx = 16 – 6e. ∫ a2 + (x − (1x))2 equals
5050
. Find the value of a.
0 0
π /2
 x x
Q.4 (i) ∫ ex cos(sinx)cos2 + sin(sinx)sin2  dx 2
x2 − x
2 2
π
0  Q.12 ∫ .
x2 + 4
∫ {(1 + x)e } ln x dx.
x −x −2
(ii) + (1 − x)e
0 π/ 4 2
 cos x 
Q.13Let u = ∫   dx and
∞ ∞ ∞  sinx − cos x 
x2 x dx dx 0
Q.5 If P = ∫ 1 + x 4 dx ;Q = ∫ 1 + x 4 and R = ∫ 1 + x4 π/ 4 2
0 0 0  sinx + cos x  v
v= ∫   dx . Find the value of
then prove that 0  cos x  u

π π/ 4
x dx
(i) Q = ,
4 Q.14 ∫ cos x(cos x + sinx)
.
0
(ii) P = R 1
sin−1 x
π
Q.15 ∫ x2 − x + 1 dx
0
(iii) P − 2 Q + R =
2 2
1+ 5
2
(x2 − 1)dx
u 2
x2 + 1
 1
Q.6 ∫ = where u and v are Q.16 ln  1 + x −  dx
1x
2
2x − 2x + 1 v
4 2 ∫ 4
x − x +1  2 x
1
(1000)u
in their lowest form. Find the value of
V
2 3 . 2 8 | Definite Integration

1π π /2
cos x
Q.17 Lim n2 ∫ (2010 sinx + 2012cos x) | x | dx . Q.29 Evaluate ∫ dx .
x →0
−1 π 0
1 + cos x + sinx

Q.18 Find the value of the definite integral π


ln(I + ax)
π Q.30 ∫ dx, a ∈ N.
1 + x2
∫| 2 sinx + 2cos x | dx. 0

0
ln3
π
(cos x + cos2x + cos3x) + 2 2
ex + 1
Q.19 If ∫ Q.31 ∫ dx .
(sinx + sin2x + sin3x) dx2 0 e2x + 1
0

π  a π /2
has the value equal to  + w  . w are positive integer.
k  Q.32 If ∫ x dx = 2a ∫ sin3 x dx , find the value of
Find the value of (k2 + w2). a+1 0 0

∫ x dx .
1 a
1−x dx
Q.20 ∫
0
1 +x x + x 2 + x3 Q.33 Let α, β be the distinct positive
1 roots of the
equation tan x = 2x then evaluate ∫ (sinα x . sinβ x)dx,
π /2 independent of α and β. 0
asinx + bcos x
Q.21 ∫ dx .
π  p+q
0 sin  + x 
4  Q.34 Show that ∫ = 2q + sinp where q ∈ N
| cos x | dx
0
π π
Q.22 A continuous real function f satisfies f(2x) = 3 f(x) &– <p< .
2 2
∀ x ∈ R.
π
Q.35 Show that the sum of the two integrals
If ∫ f(x)dx = 1 , then compute the value of definite
0 2 −π 2/3
2 (x −2x)2
integral ∫ f(x)dx. ∫ e(x +1) dx + 3 ∫e dx is zero.
1 −1 1/3

3 Q.36 Let F(x) = max (sin px, cos px). Find the value of
Q.23 The value of ∫−1 {lx − 2l + [x]} dx , where [x]
π
10

denotes the greatest integer less than or equal to x is.


4 2
∫ F(x)dx.
−10

0
2x
Q.24 ∫ sin−1 dx . π /2  1 + sinx + 1 − sinx 
1 1 + x2 Q.37 ∫ tan−1   dx .
0  1 + sinx − 1 − sinx 
1
(ax + b)sec x tanx
Q.25 ∫ dx (a,b > 0) Q.38 Comment upon the nature of roots of the quadratic
0 4 + tan2 x 1

π
equation x2 + 2x = k + ∫ | t + k | dt dependent on the
(2x + 3)sinx
Q.26 ∫ (1 + cos2 x) dx . value of k ∈ R.
0

1
(2x232 + x998 + 4x1668 sinx691 )
π /2
cos x Q.39 ∫
Q.27 Evaluate ∫ cos x + sinx
−1 1 + x666
0

nπ π 
x | sinx | x2 sin2x . sin  cos x 
Q.28 If ∫ I + | cos x |
dx (n ∈ N) is equal to 100 π log 2,
Q.40 π∫
π
2  dx
0
2x − π
then the value of n. 0
M a them a ti cs | 23.29

π
(x − x )
1/3 x
3
Q.41 Evaluate
1
Q.5 Solve ∫ a2 cos2 x + b2 sin2 x dx
∫ x4
dx 0
1/3
2
π π2
(A) (B)
n=1  k −1  2ab 4ab
1
Q.42 Lim
x →∞ n2
∑ k ∫ (x − k)(k + 1 − x)dx 
 k  π2 π
k =0
(C) (D)
3ab 5ab
π /2
cos x + 4
Q.43 Let I = ∫ 3sinx + 4 cos x + 25
dx and π/ 4
sec x
0 Q.6 ∫ 1 + 2sin2 x
is equal to -
π /2 0
sinx + 3
I= ∫ 3sinx + 4 cos x + 25
dx .
1 π 1 π
0 (A) log( 2 + 1) + (B) log( 2 + 1) −
c c 3 2 2 3 2 2
If 25 I = aπ + b ln where a, b, c and d ∈ N and is
d d
π π
not a perfect square of a rational then find the value of (C) 3 log( 2 + 1) − (D) 3 log( 2 + 1) +
(a + b + c + d). 2 2 2 2
1
x2
Q.44 Let y = f(x) be a quadratic function with f(2) = 1.
Q.7 If ∫e (x − α )dx = 0, then
0
Find the value of the integral (A) 1 < α < 2 (B) α < 0
2 +π
x −2
(C) 0 < α < 1 (D) None of these
∫ f(x).sin 
 2 
 dx .
2 −π π /2
Q.8 ∫ {x − [sinx]}dx is equal to -
b

Exercise 2 (A)
π2
(B)
π2
−1 (C)
π2
−2 (D) None of these
8 8 8
Single Correct Choice Type 100

2
Q.9 The value of the integral ∫ sin{x − [x]}π dx is -
Q.1 ∫ | x2 + 2x − 3 |dx equals b
0
100 200
(A) (B) (C) 100 p (D) 200 p
(A) 5/3 (B) 7/3 (C) 4 (D) 0 π π

π /2
 xlogx
π Q.10 The value of the integral ∫ (1 + x2 )2 dx is -
Q.2 The correct evaluation of ∫ sin  x −  dx is -
 4 0
0
(A) 1 (B) 0 (C) 2 (D) None of these
(A) 2 + 2 (B) 2 − 2 (C) −2 + 2 (D) 0 1/n
 1  2  3  n 
π Q.11 lim  1 +   1 +   1 +  ......  1 +   is
Q.3 The correct evaluation of ∫ | sin4 x |dx is -
n→∞ 
 n  n  n  n 
0 equal to -
8π 2π 4π 3π
(A) (B) (C) (D) (A) e/4 (B) 4/e (C) 2/e (D) None of these
3 3 3 8

1.5 Q.12 The solution of the equation


2
Q.4 ∫ [x ]dx, where [·] denotes the greatest integer x
∫log2
1
dx =
π
0 x 6
function, equals - e −1

(A) 2 + 2 (B) 2 − 2 (A) x = log 4 (B) x = log 2

(C) −2 + 2 (D) −2 − 2 1


(C) x = log   (D) None of these
4
2 3 . 3 0 | Definite Integration

Q.13 The value of Q.3 Let f be a non-negative function defined


x
 1 4 9  on the interval [0, 1] . If ∫0 1 − (f '(t))2 dt =
lim  + + + ... + n terms  is -
n→∞  1 + n3 3 3 x
8+n 27 + n  0 ≤ x ≤ 1 and f(0) = 0, then 
∫0 f(t)dt, (2009)

1 1 1 1 1 1
(A) log2 (B) 0 (C) log3 (D) None of these (A) f   < and f   >
3 3
2 2 3 3

Q.14 lim 1 1 1 1


n→∞ (B) f   > and f   >
2 2 3 3
 n2 n2 n2 
 2 2 3/2
+ + ... +  is 1 1 1 1
 (n + 1 ) (n2 + 22 )3/2 [n2 + (n − 1)2 ]3/2  (C) f   < and f   <
2 2 3 3
equal -
1 1 1 1
1 1 (D) f   > and f   <
(A) − (B) (C) 2 (D) None of these 2 2 3 3
2 2
  Q.4 The value of
Q.15 lim  n + n
+
n
+ .... +
n

n→∞
 n2 (n + 4)3 (n + 8)3 [n + 4(n − 1)]3  log3 x sinx2
∫ log2 sinx2 + sin(log6 − x2 ) dx is - (2011)
is equal -
1  1
(A) 5 − 5  (B) 5 − 5  (C) 5 − 5  (D) 0 1 3 1 3 3 1 3
10     5  (A) log (B) log (C) log (D) log
4 2 2 2 2 6 2
1
  π  2π   3π   nπ   6
Q.16 lim tan   tan   tan   ... tan    n
n
n→∞ 
  2n   2n   2n   2n   Q.5 Let Sn = ∑ n2 + kn + k 2 and
k =0
is equal -
n−1
(A) 0 (B) 1 (C) –1 (D) 2 n
Tn = ∑ n2 + kn + k 2 , for n = 1, 2, 3 ….. , then  (2008)
k =0

π π
Previous Years’ Questions (A) Sn <
3 3
(B) Sn >
3 3
π π
1/2 
1 + x  (C) Tn < (D) Tn >
Q.1 The integral ∫  [x] + log   dx equals ( 2002) 3 3 3 3
1/2
  1 − x 
1 1
sinnx
(A) − (B) 0 (C) 1 (D) log   π
2 2
Q.6 If In = ∫−π (1 + πx )sinx dx, n = 0, 1, 2,…, then (2009)
10
1
Q.2 If I(m,n) = ∫ tm (1 + t)n dt, then the expression for
0
(A) In = In + 2 (B) ∑ I2m+1= 10π
m=1
I(m,n) in terms of I(m + 1, n – 1) is (2003) 10
(C) ∑ I2m = 0 (D) In = In+1
2n n m=1
(A) − I(m + 1,n − 1)
m+1 m+1 1
x 4 (1 − x)4
(B)
n
I(m + 1,n − 1)
Q.7 The value(s) of ∫ 1 + x2
dx is (are)  (2010)
0
m+1
22 2 71 3π
2n n (A) −π (B) (C) 0 (D) −
(C) + I(m + 1,n − 1) 7 105 15 2
m+1 m+1
m Paragraph for Q.8
(D) I(m + 1,n − 1)
m+1
Read the following passage and answer the questions.
For every function f(x) which is twice differentiable,
these will be good approximation of
M a them a ti cs | 23.31

b b −a π /3 π + 4x3
∫a f(x)dx =  2  {f(a) + f(b)}, Q.14 Evaluate ∫−π /3 2 − cos (| x | +(π / 3)) dx.  (2004)

for more accurate results for c∈(a, b),


c−a b−c Q.15 Evaluate
F(c)
= [f(a) − f(c)] + [f(b) − f(c)]
2 2 π |cos x|  1  1 
∫0 e  2sin  cos x  + 3cos  cos x   sinx dx  (2005)
a+b  2  2 
When c =
2
1
b b−a (5050)∫ (1 − x50 )100 dx
∫a f(x)dx
=
4
{f(a) + f(b) + 2f(c)}dx  (2006) Q.16 The value of
1
0
(2006)
∫0 (1 − x50 )101 dx
π /2 
Q.8 Good approximation of ∫ sinx dx,is (2003)
0  0x f '(t)
Q.17 Let g ( x ) = ∫ 1 + t2 dt , then which of the following
(A) π/4 (B) π( 2 + 1) / 4
0
is true? (2008)
(C) π( 2 + 1) / 8 (D) π/8
(A) g ( x ) is positive on ( −∞ ,0 ) and negative on ( ∞ ,0 )
Q.9 If f’’(x) < 0, “x∈ (a, b), and (c,f(c)) is point of maxima
(B) g ( x ) is negative on ( −∞ ,0 ) and positive on ( 0,∞ )
where c∈(a, b), then f’(c) is - (2009)
(C) g ( x ) changes sign on both ( −∞ ,0 ) and ∴
f(b) − f(a)  f(b) − f(a) 
(A) (B) 3   (D) g ( x ) does not change sign on ( ∞ , ∞ )
b−a  b−a 
 f(b) − f(a)  1
(C) 2   (D) 0
 b−a  Q.18 ∫ g' ( x ) dx =  (2008)
−1

(A) 2g ( −1 ) (C) −2g (1 ) (D) 2g (1 )


1
∫ f(x)dx − ((t − a) / 2){f(t) + f(a)} (B) 0
Q.10 If lim a = 0, then
t →a (t − a)3
degree of polynomial function f(x) at most is -  (2002) Q.19 The total number of distinct x ∈ 0,1  for which
x
t2
(A) 0 (B) 1 (C) 3 (D) 2 ∫ 1 + t 4 dt= 2x − 1 is  (2016)
0

Q.11 For any real number x, let [x] denotes the largest Q.20 Let f R → R be a continuous function which satisfies
integer less than or equal to x. Let f be a real valued x
function defined on the interval f ( x ) = ∫ f ( t ) dt . Then the value of f(In5) is  (2009)
0
x − [x] if [x] is odd
[–10, 10] by f(x) = 
1 + [x] − x if [x] is even Q.21 Let f be a non-negative function defined on the
x x
( )
2
π2 10 interval 0,1 ∫ 1 − f ' (=
t ) dt ∫ f ( t ) dt 0 ≤ x ≤ 1 and
Then the value of f(x) cospxdx is  ……. (2010)
10 ∫−10 0 0
f ( 0 ) = 0 then  (2009)
tx log
Q.12 For x > 0, let f(x) = ∫ dt . Find the function 1 1 1 1
1 1+ t (A) f   < and f   >
2
  2 3 3
f(x) + f(1/x) and show that f(e) +f(1/e) = 1/2. Here,
ln t = loget  (2000) 1 1 1 1
(B) f   > and f   >
2 2 3 3
Q.13 If f is an even function, then prove that 1 1 1 1
(C) f   < and f   <
π /2
f(cos2x)cos x dx = 2 ∫
π/ 4
f(sin2x)cos x dx  (2003) 2 2 3 3
∫0 0
1 1 1 1
(D) f   > and f   <
2 2 3 3
2 3 . 3 2 | Definite Integration

Q.22 Match the statements/expressions in column I with the open intervals in column II.  (2009)

Column I Column II
(A) Interval contained in the domain of definition of non-zero
 π π
(p)  − , 
 2 2
5  π
(B) Interval containing the value of the integral ∫ ( x − 1)( x − 2)( x − 3)( x − 4 ) dx (q)  0,


2
1

2 π π
(C) Interval in which at least one of the points of local maximum of cos x + sin lies
(r)  , 
8 2
(D) Interval in which tan
−1
( sinx + cox ) is increasing 
(s)  0,
π

 2

(
(t) −π, π )

Q.23 Match the statements in column I with those in column II.  (2010)

Column I Column II

8
x−
x − 2 y −1 z +1 3 y=+ 3 z −1
(A) A line from the origin meets the lines = = and = at P and Q
1 −2 1 2 −1 1 (p) -4

respectively. If length PQ = d Then d2 is

3  
(B) The values of x satisfying tan
−1
( x + 3) − tan−1 ( x − 3) =
sin−1  
5
(q) 0
 
        
(C) Non-zero vectors a,b and c satisfy a.c
= 0 b− a . b− c = ( )( )
0 and possible values of are (r) 4

 9x  x
f 0 = 9 and f x
(D) Let f be the function on  −π, π  given by = sin  ( ) () 2
 / sin   x ≠ 0
π   2 (r) 5
2
The value of ∫ f ( x )dx is
π −π
(s) 6

x
1 tlog(1 + n) log3
Q.24 The value of lim ∫ dt is  (2010) x sinx2
Q.26 The value of dx is
x → 0 x3
0 t +44
sinx 2
+ sin(log6 − x 2
)

log2
1 1 1
(A) 0 (B) (C) (D) (2011)
12 24 64
11 33 11 33 3 1133
4 (A) log
InIn (B) log
InIn (C) log
In (D) log
In In
1 4 (1 − x ) 44 22 22 22 2 6622
x
Q.25 The value (s) of ∫ dx is (are)  (2010)
0 1 + x2 ππ/2
/2
 2 ππ++xx
22 2 Q.27 The value of the integral ∫∫ xx2++log
nn  cos
ππ−−xx
cosxdx
xdx
(A) −π (B) −π/2
−π
7 105 is  /2
(2012)
2 2 2
π π π
71 3π (A) 0 (B) −4 (C) + 4 (D)
(C) 0 (D) − 2 2 2
15 2
M a them a ti cs | 23.33

π /2 1
 −1   12 + 9x 2 
∫ ( 2cosecx )
17
Q.28 The following integral dx is equal Q.31 If α =∫  e9x +3 tan x    dx
   1 + x2 
π/ 4 0  
to  (2014)
Where tan−1 takes only principal values, then the value
 3π 
(
log 1 + 2 ) of  loge 1 + α −  is  (2015)
( ) 4 
16
(A) u −u 
∫ 2 e +e du
0
Q.32 The option(s) with the values of a and L that satisfy
(
log 1 + 2 ) the following equation is(are)
( )
16
(B) ∫ 2 eu + e−u du 4π
0
∫e
t
(sin at + cos at )
6 4

(
log 1 + 2 ) 0
= L?  (2015)
(e )
16 π
(C) ∫
u −u

0
−e du
∫e
t
(sin at + cos at )
6 4

0
(
log 1 + 2 ) e4 π − 1 e4 π + 1
( ) (A)=
a 2,L (B)=
a 2,L
16 = =
(D) ∫ 2 e −eu −u
du eπ − 1 eπ + 1
0
e4 π − 1 e4 π + 1
a 4,L
(C)= = a 4,L
(D)= =
Q.29 Match the following: (2014) eπ − 1 eπ + 1

List I List II
Q.33 The correct statement(s) is(are)  (2015)
(i) The number of polynomials f x with non ( ) (p) 8
(A) f ' (1 ) < 0
negative integer coefficients of degree ≤ 2
1 (B) f ' ( 2 ) < 0
satisfying f 0 = 0 and( ) ∫ f ( x ) dx = 1 ,is (C) f ' ( x ) ≠ 0 for any x ∈ (1,3)
0

(ii) The number of points in the interval (q) 2 (D) f ( x ) = 0 for some x ∈ (1,3)

 − 13, 13  at which
 
= f x sin x2 + cos x2 () ( ) ( ) Q.34 Let f : R → R be a function defined by
attains its maximum value, is   x  x ≤ 2 
f ( x ) =    where  x  is the greatest integer
2 (r) 4  0 x > 2
3x2
(iii) ∫ dx equals
−2 (1 + e ) x less than or equal to x.

(s) 0 If I =
2
( )
xf x2
dx, then the value of (4I - 1) is
 1/2 1 + x   ∫ 2 + f ( x + 1 )dx
 ∫ cos2.x.log   dx  −1
  1 − x  
 −1/2  (2015)
(iv) equals
1/2
1 + x  π
∫ cos2x.log  1 − x dx  2
x2 cos x
0 Q.35 The value of ∫ 1 + ex
dx is equal to  (2016)
π

Codes: i ii iii iv 2

(A) r q s p π2 π2
(A) −2 (B) +2
(B) q r s p 4 4
π π
(C) r q p s (C) π2 − e 2 (D) π2 + e 2
(D) q r p s

1  2 
Q.30 The value of 3 d
∫  dx2 1 − x
0
4x

2
( )dx is __________
 (2014)
2 3 . 3 4 | Definite Integration

PlancEssential Questions
JEE Main/Boards JEE Advanced/Boards
Exercise 1 Exercise 1
Q.3 Q.8 Q.12 Q.2 Q.7 Q.10
Q.17 Q.21 Q.23 Q.15 Q.22 Q.27
Q.26 Q.28 Q.32 Q.34 Q.44

Exercise 2 Exercise 2
Q.9 Q.12 Q.17 Q.2 Q.7 Q.10
Q.20 Q.23 Q.29 Q.12 Q.15
Q.32 Q.34

Previous Years’ Questions Previous Years’ Questions


Q.4 Q.8 Q.11 Q.1 Q.4 Q.6
Q.7 Q.10 Q.15

Answer Key

JEE Main/Boards
Exercise 1

π π π
Q.1 Q.2 Q.3
6 4 5 3 3
13 34
Q.4 Q.5 Q.6 2 − 2
10 3
π2 14
Q.7 Q.8 Q.9 2( 2 − 1)
2 2 3

Q.11 200 2 Q.12 (ii) Not necessary Q.13 2x 1 + x 4

4π 1 2 π
Q.14 tan−1   Q.16 2 − + 2e log2 Q.17
3 2 e 2

3 1 5 1
Q.18 −2 3 + Q.19 1, − log2 Q.20 a = 1, 2, 3 or 4
2 2 12 2
M a them a ti cs | 23.35

π e2 + 1
Q.21 –1 Q.22 Q.26
4 2e
1
Q.29 (i) ; (ii) 1+e Q.30 (–∞, –2) ∪ (–1, 0) ∪ (1, ∞)
2

Exercise 2
Single Correct Choice Type

Q.1 A Q.2 C Q.3 C Q.4 B Q.5 A Q.6 B

Q.7 D Q.8 A Q.9 A Q.10 B Q.11 A Q.12 B

Q.13 C Q.14 B Q.15 A Q.16 B Q.17 C Q.18 C

Q.19 A Q.20 C Q.21 A Q.22 C Q.23 C Q.24 C

Q.25 C Q.26 C Q.27 B Q.28 B Q.29 B Q.30 A

Q.31 B Q.32 C Q.33 C Q.34 C

Previous Years’ Questions


Q.1 A Q.2 C Q.3 A Q.4 D Q.5 A Q.6 A

Q.7 A Q.8 C Q.9 C Q.10 C Q.11 A Q.12 C

Q.13 D Q.14 B Q.15 A → s ; B → s ; C → p ; D → r Q.16 4 Q.17 π( 2 − 1)

1
Q.18 Q.19 B Q.20 D Q.21 A Q.22 D Q.23 B C
2
Q.24 D Q.25 D Q.26 C

JEE Advanced/Boards
Exercise 1

π2 π 1
Q.1 − (1 + log2) + Q.3 (a) 4 (b) n = 3
8 4 2
1 π/2
Q.4 (i) [e (cos 1 + sin 1)–1] (ii) e1+e +e1–e + e–e – ee + e – e–1 Q.6 125
2

Q.7 1 – sec(1) Q.8 ln 2 Q.9 4

π 16 2
Q.10 − Q.11 2525 Q.12 4 2 − 4ln 1 + 2
2 2 5
π π2
Q.13 4 Q.14 log2
ln2 Q.15
8 6 3
π
Q.16 log2
ln2 Q.17 2012 Q.18 2 6
8
π π(a + b)
Q.19 153 Q.20 Q.21
3 2 2
2 3 . 3 6 | Definite Integration

π
Q.22 5 Q.23 90 Q.24
3

(aπ + 2b)π π( π + 3) 1 π /2 1π  π


Q.25 Q.26 Q.27 = x  =  − 0 =
3 3 2 2  0 22  4

1 π  −1 1
Q.28 10 Q.29  − log 2 Q.30 tan
tan−(a)(a). ln. ln1 1
log ++a2a2
2 2 

1  π1  π   9
Q.31  + ln3log3− ln2
+ ln3− −log2
ln2 Q.32 Q.33 0
2  62  6   2

3π2
Q.36 5 Q.37 Q.38 Real and distinct ∀ k ∈ R
16
π+4
Q.39 Q.40 8 Q.41 6
666
π /2
π
Q.42
16
Q.43 62 Q.44 I = 8 as ∫ y sin y dy = 1
0

Exercise 2
Single Correct Choice Type

Q.1 C Q.2 B Q.3 D Q.4 B Q.5 A Q.6 A

Q.7 C Q.8 A Q.9 B Q.10 B Q.11 B Q.12 A

Q.13 A Q.14 B Q.15 A Q.16 B

Previous Years’ Questions


Q.1 A Q.2 A Q.3 C Q.4 A Q.5 A, D Q.6 A, B, C

Q.7 A Q.8 C Q.9 A Q.10 B Q.11 4

11 11 22 11 π/ 4 4π 1
Q.12 f(e) (lne)2=
f(e)++ff == (loge)
(lne) = Q.13 I = 2 ∫ f(sin2t)cos t dt Q.14 tan−1  
ee 22 22 0 3 2

24  1 e 1 
Q.15  ecos   + sin   − 1  Q.16 5051 Q.17 B Q.18 D Q.19 A
5  2 2 2 

Q.20 0 Q.21 C Q.22 A → p, q, s; B → p, t; C → p, q, r, t; D → s Q.23 A

Q.24 B Q.25 A Q.26 A Q.27 B Q.28 A

Q.29 D Q.30 2 Q.31 9 Q.32 A, C Q.33 A, B, C Q.34 ‒2

Q.35 A
M a them a ti cs | 23.37

Solutions

JEE Main/Boards π /2
sin2 x
Sol 3: ∫ 1 + sinx cos x
dx

Exercise 1
0

π 
π /2 sin2  − x  π /2
1/2
dx
1/2
dx  2  dx or I = cos2 x
Sol 1: ∫ = ∫ I= ∫ 1 + sinx cos x ∫ 1 + sinx cos x
dx
1/ 4 x − x2 1/ 4 1  1
2 0 0
− x − 
4  2 π /2
1
π /2
sec2 xdx
1/2
∴ 2I = ∫ dx = ∫
 1 0
1 + sinx cos x 0 1 + tan2 x + tanx
x −   –1 / 4 
2
= sin−1  = sin–10 – sin–1  1 / 2  x
dt
x
dt
1/2   = lim ∫ = lim ∫
2 2
01+ t +t
x →∞ x →∞ 2
0  1  3
1/ 4
 t +  +  
 2   2 
1 π ∞
= sin–1  2  = 2 2t + 1
  6 = tan−1
3 3 0
π /2 π /2
dx dx
Sol 2: ∫ = ∫ 2 π π 2π
2
4 sin x + 5cos x 2
4 + cos2 x =  − =
0 0 3 2 6  3 3
π /2 π /2
dx 2dx π
= ∫ = ∫ ∴I=
9 cos2θ 1 – tan θ 2
3 3
0 + 0
9+
2 2 1 + tan θ 2

π /2 3/5 1
2
sec θdθ Sol 4:
= 2 ∫ ∫ (3 − 5x)dx + ∫ (5x − 3)dx
0 10 + 8 tan2 θ 0 3/5

3/5 1
π/ 4
2sec2 θdθ
π /2
2cosec2 θ 5 2 5x2
= = 3x – x + − 3x
∫ 10 + 8 tan2 θ
+ ∫ 2
dθ 2 0 2
0 π / 4 10 cot θ+8 3/5

π/ 4 π /2 9 5 9 5  5 9 9
sec2 θdθ cosec2 θ = − × +  – 3 –  × – 
= ∫ + ∫ dθ 5 2 25  2   2 25 5 
0 5 + 4 tan2 θ π / 4 5cot
2
θ+4
9 9 1 9 9
1 0 1 1 = – – – +
dt dt dt dt 5 10 2 10 5
= ∫ 5 + 4t2 + ∫ – 5t2 + 4 = ∫ 5 + 4t2 + ∫ 5t2 + 4
0 1 0 0 18 9 1 9 1 13
= – – = − =
1 1 5 5 2 5 2 10
1 1 t 1 1 t
= × tan−1 + × tan−1
4 5 /2 5 /20 5 2/ 5 2/ 5 0 2 3
Sol 5: ∫ (2x + 1)dx + ∫ (x2 + 1)dx
1 2 1 5 1 2
= tan−1 + tan−1 3
2 5 5 2 5 2 2 x 3
= x2 + x + +x
1 3 2
1  −1 2 2  8 
= tan + cot −1  = (4 + 2 – 2) + (9 + 3) –  + 2 
2 5 5 5 3 
1 π π
= × = 8 8 34
2 5 2 4 5 = 4 + 12 – 2 – = 14 – =
3 3 3
2 3 . 3 8 | Definite Integration

π/ 4 π/ 4  n(n + 1)(2n + 1) 
Sol 6: sinxdx = 2  – cos x
π/ 4  = lim h3 × + hn
∫ sinxdx = 2 ∫  0   n→∞  6 
– π/ 4 0

  1   1   8 (n)(n + 1)(2n + 1) 2 
= 2 –  = lim  + × n
− 1  = 2 1 –  =2− 2 n→∞  n3 6 n 
  2    2
8 ×1× 2 14
= +2 =
π
x
π
π−x 6 3
Sol 7: ∫ (1 + sin2 x) dx = ∫ (1 + sin2 x) dx =I
0 0 π /2

π π /2
Sol 9: ∫ sinx − cos x dx
1 1
∴2I = π∫ dx = 2π ∫ dx 0
2
0 1 + sin x 0 1 + sin2 x π/ 4 π /2

π /2 π /2
= ∫ (cos x − sinx)dx + ∫ (sinx − cos x)dx
1 2
∴I= π ∫ dx = π ∫ dx 0 π/ 4

0 1+
1 − cos2x 0
3 − cos2x
π/ 4 π /2
2 = sinx + cos x + (–cosx – sinx
0 π/ 4
π /2 π /2
dx sec2 xdx
= 2π ∫ = 2 π ∫
(1 − tan2 x) 2  1 1    1 1 
0 0 2 + 4 tan x =  + – 1  – 1 −  +
3−  
1 + tan2 x  2 2    2 2  
 
π /2 1
π  dt
π /2
cosec2 xdx  = 2 –1–1+ 2 = 2 2 –2
π sec2 xdx
= ∫
2  0 1 2 π∫/ 4 1
= ∫ – 
2 1
0 + tan2 x +t cot2 x + 1 
2  2 2  Sol 10: f(x) = f(a – x)
 
1 
  g(x) + g(a – x) = 2
 0
π  1 −1 t  dt 
=  tan −∫  a a
2  1 1  1
  1 t2 + 1 
2
∫ f(x)g(x)dx = ∫ f(a − x)g(a − x)dx =I
 2 2 0  0 0
 
a a
π dt 
1
=  2 tan−1 2 + 2∫  ∴2I = ∫ f(x)g(x) + f(a − x)g(a − x)dx = ∫ f(x) × 2 dx
2  2
0 t + 2
 0 0

1 a
π 2 t  I=
=  2 tan−1 2 +
2
tan−1  ∫ f(x)dx
2 2 0 0

π −1 1  100 π
=  2 tan 2 + 2 tan−1 
2 2 Sol 11: ∫ 1 − cos2xdx
0
π π π2
= × 2× = 100 π 100 π
2 2 2 2 2sin2 xdx = sin2 xdx
∫ 2 ∫
0 0
2
2
Sol 8: ∫ (x + 1)dx Qsin (π– x) = sin x
2 2

0
π π
∴ 2
b−a 2−0 2 I = 100 2 ∫ sin xdx = 100 2 ∫ | sinx | dx
h= = =
n n n 0 0

Also sin ( π − x ) =| sinx |


n n
∴I = lim ∑ hf(a + rh) = lim ∑ h((rh) + 1)
2
n→0 n→0
r =1 r =1 π /2
n ∴I = 200 2 ∫ | sinx | dx
= lim ∑ (r 2h3 + h) 0
n→0
r =1
= 200 2 ( – | cos x |)
π /2
= 200 2
0
M a them a ti cs | 23.39

Sol 12: (i) f(–t) = –f(t) π /3


π − 4x3
I= ∫  
dx
– π /3 2 − cos | x | + π
x
g(x) = ∫ f(t)dt 
3


a
π /3
–x –a –x dx
g(–x) = ∫=
f(t)dt ∫ f(t)dt + ∫ f(t)dt
∴2I = 2π ∫  
– π /3 2 − cos | x | + π
a a –a  
 3
Qf(t) = odd function
π /3
dx
–a ⇒I = 2π ∫
So ∫ f(t)dt = 0 x π
0 (1 − tan2  + 
a
2− 2 6
–x –x  2x π 
 1 + tan  +  
∴ ∫ f(t)dt = ∫ f(t)dt   2 6 
a –a
x π
Put t = –p π /3 sec2  +  dx
=
2π 2 6
x
∴ 3 ∫ 1 x π
= – ∫ f(–p)dp f(–p) = –f(p) 0 + tan2  + 
a 3 2 6
x π
x
Put tan  +  = t
= ∫ f(p)dp 2 6
a
1 x π
∴g(–x) = g(x) sec2  +  dx =
dt
2 2 6
(ii) f(t) = f(–t) 3
4π dt 4π 3
∴I = ∫ = × 3 tan−1 3t
x –x 3 1 2 3 1/ 3
g(x)= 1/ 3 +t
∫ f(t)dt ; g(–x) = ∫ f(t)dt 3
a a
4 π  −1 4 π  −1  3 − 1  
Put t = –p = tan 3 − tan−1 1 = tan  
3  3  1 + 3 × 1 
x x x
= – ∫ f(–p)dp = – ∫ f(p)dp = – ∫ f(t)dt 4π 1
−a −a −a
= tan−1
3 2
–a
∴ g(–x) = ∫ f(t)dx
x x
logt
∴ g(x) + g(–x) =
x –a –a Sol 15: f(x) = ∫ 1 + t dt
∫ f(t)dt + ∫ f(t)dt = ∫ f(t)dt 1
a x a
1 1
x nx)2 2
To Prove. f(x) + f   = ((logx)
∴ It is not necessary that if f(t) is even then ∫ f(t)dt is x 2
odd a 1 1
Put t = ⇒ dt = – dp
p p2
x2
1
Sol 13: f(x) = 1 + t2 dt log
∫  1
x
p  1 
a =f 
x
∫  1
 − 2  dp
 p 
1 11 +   
f’(x) = 1 + x 4 dx2 = 2x 1 + x 4 p
 
x x
π /3  1 logp logt
π + 4x3dx =f  ∫= dp ∫ t(t + 1) dt
Sol 14: I = ∫ x p(1 + p)

– π /3 2 − cos | x | + π
 1 1
 
 3
2 3 . 4 0 | Definite Integration

x x
 1 logt Sol 18: F(x) =
∴ f(x) + f   =∫ dt ∫ (3sint + 4 cos t)dt
x 1 t 5π / 4

x x
x logt = 3(– cos t) + 4 sint
= (log)2 − ∫ dt 5π / 4 5π / 4
1 t
∴ 2I = (logx)2   1   1 
= 3  –  cos x –   + 4 sinx − 
1 1   2    2
nx)2 2
∴ f(x) + f   = ((logx)
x
  2 3 4
= –3cosx + + 4sinx –
2 2
1 2e
Sol 16: –  4 sinx − 3cos x  1
∫ logxdx + ∫ logxdx = 
5
5 –
1/e 1   2

= – ( xlog x − x )
1
+ (xlog x − x)
2e  5π 4 π 
1/e 1 From interval  ,  sinx < cosx
4 3 
 1 1 1  4π
= – 0 − 1 −  log −   ∴We get min value of x =
3
 e e e 

+ [(2elog2e – 2e) – (0 – 1)] 3 3 1 3 1


∴Min value = – 4 × + – = –2 3–
2 2 2 2 2
1 1
=1– – + 2elog2 + 2e – 2e + 1
e e
π/ 4
2 Sol 19: In = tann θdθ
=2–
e
+ 2elog2 ∫
0

 1 π/ 4

Sol 17: ∫
sin  n +  x

π
2
dx n ∈ N
In–1 + In+1 = ∫ ( tan n−1
θ + tann+1 θ dθ )
0
x
0 2sin
2 π/ 4
= ∫ (tann−1 θ)sec2 θdθ
 1 x
2sin  n +  x cos = sin(nx + 2) + sin(nx) 0
 2  2
tanθ = t ⇒ sec2qdθ = dt
π
1 sinnx + sin(nx + x) 1
=
2 ∫0
dx 1
tn 1 n−1
sinx = ∫t =
dt =
0
n n
π 0
1 sin(n + 1)x + sinnx
= ∫ dx 1
20 sinx ∴ n(In–1 + In+1) = n × =1
n
If n is odd π/ 4
π I7 = ∫ tan7 θdθ
1 sinnx − sin(n + 1)x
I= ∫ dx 0
20 sinx
π/ 4 π/ 4
sinnx
π
π = ∫ tan5 θ sec2 dθ – ∫ tan3 θ sec2 θdθ
∴ 2I = ∫ dx = π ⇒I=
0
sinx 2 0 0

π/ 4 π/ 4
If n is even + tan θ sec2 θdθ –
π
∫ ∫ tan θdθ
0 0
sin(n + 1)x π
2I = ∫ dx = π; I =
0
sinx 2 11 11 11 5 1
= – – + + – –log
nn2 2 = – log2
66 44 22 12 2
M a them a ti cs | 23.41

π /2  2  cos3x 3   = ∑ (n − 1)(2n − 1) = ∑ (2n2 − 3n + 1)


a  + cos x   a2
Sol 20: ∫   4 4   dx ≤ –
3
0  + asinx − 20 cos x  2n(n + 1)(2n + 1) 3n(n + 1)
  = − +n
6 2
π /2
a2
∫{ a2 cos3 x + asinx − 20 cos x dx ≤ – } 3 =
n(n + 1)  4n + 2


− 3 + n =
n(n + 1)(4n − 7)
+n
0 2  3  6

 1 π /2 3 π /2   4n2 − 3n − 7 + 6  n(4n2 − 3n − 1)
= a2  + sin3 x + ( + sinx)  = n  =
 12 0 4 0
 
 6 
 6
π /2 π /2 a2 n(n − 1)(4n + 1)
+ a(– cos x) – 20 sinx ≤– =
0 0 3 6
 1 3 a2
= a2  – +  + a – 20 ≤ – nπ+λ
 12 4  3 Sol 24: ∫ | sinx | dx = 2n + 1 – cosλ
⇒ a2 + a – 20 ≤ 0 0

∴ n ∈ N, 0 ≤ λ < p
(a+5) (a–4) ≤ 0 a ∈ [–5, 4]
∴a is +ve interger λ nπ+λ
LHS = ∫ | sinx | dx + ∫ | sinx | dx
So a = 1, 2, 3 or 4 0 λ
π
λ
Sol 21: f(x) = sin x = – cos x + n∫ | sinx | dx
0
0
Mean value of sin x from [–2, 0) π /2
= –(cosλ – 1) + 2n ∫ sinxdx
0
sinx –1[0 + 2] 0
∴ ∫ dx = = –1
–2
2 2 = 2n + 1 –cosλ

a
1 π 
Sol 22: I = x sin2x.sin  cos x 
∫ dx
Sol 25: I = ∫
π
2 dx  ….. (i)
0 x + a2 − x2
x = acosq 0
2x − π
dx = –asinqdq π 
π (π – x)(– sin2x)sin  (– cos x)  dx
a π /2 2 
–asin θdθ
∫ acos θ + asin θ = ∫


sin θ 
 dθ
I= ∫ 2( π − x) – π
….. (ii)
π /2 0  cos θ + sin θ  0 
On adding (i) and (ii)
π /2
 cos θ 
= ∫   dθ π 
0  cos θ + sin θ  π (2x − π)sin2x sin  cos x 
 2  dx
2I = ∫ (2x − π)
1 
π /2
1 π π 0
⇒I=  ∫ dθ  =   =
2
 0  2 2 4 π
π 
= ∫ sin2x sin  2 cos x  dx
0
1 4 9 n2 π /2
Sol 23: I = ∫ 0dx + ∫ 1dx + ∫ 2dx.... + π 
∫ (n − 1)dx or I = ∫ sin2x sin  cos x  dx
0 1 4 (n−1)2 0 2 
n n2 π
Let cos x = t
∴∑ ∫ (n − 1)dx 2
n= 0
(n−1)2 π 2
– sinxdx = dt or sinxdx = – dt
∑ (n − 1) (n )
n2 2 π
= ∑ (n − 1)x (n−1)2 = 2
− (n − 1) 2
2 3 . 4 2 | Definite Integration

0 π
2 2t Sol 28: dx
I = – ∫ 2 × sintdt
π π /2 π
∫ 1 − 2acosx + a2
0

π
8
π /2
8 
π /2  dx
= ∫ t sintdt = t(– cos t)
π /2
+ ∫ cos tdt  = ∫  x
π2 0 π2  0
0  0 2a  1 − tan2 
2
8  π /2  8 1 + a2 −  
= 0 + sint = 2 x
2   1 + tan
π  0  π2 2
x
Sol 26: Let f(x) = K1ex + K2e–x π sec2 dx
= ∫ 2
g(x) + f’(x) = K1ex – K2e–x 2 
0 (1 + a ) 1 + tan 2 x   2 x
  – 2a  1 − tan 
 2   2
∴g’(x) = K1ex + K2e–x = f(x)
x
∴f(0) = 1 ⇒ K1 + K2 = 1 π sec2 dx
= ∫ 2
Also g(0) = 0 ⇒ K1 – K2 = 0 x
0 (1 + a2 − 2a) + tan2 (1 + a2 + 2a)
1 2
K 1 = K2 =
2 x
Putting =t
e +ex x 2
∴f(x) = x
2 sec2 dx = 2dt
2
1
e+ 2
∴f(1) = e = e +1 ∞ ∞
dt 1 dt
2 2e ∫ (1 + a)2 t2 + (1 − a)2 =
1+a 2 ∫ 2
0 0 2 1 −a
π t + 
1 + a
Sol 27: (i) ∫ log(1 + cos x)dx
0 ∞

π
1 1+a t
= ∫ log(1 − cos x)dx = I = tan−1
×
0 (1 + a ) | 1 − a |
2 1−a
π 1+a 0
∴ 2I = ∫ log(1 − cos2 x)dx
π 1
0 = If a < 1
π π /2 1−a 2 2

= 2 ∫ logsinxdx = 4 ∫ logsinxdx π 1
0 0 = if a > 1
 –π 
=4×  a −1 2
2
 log2
 2  α

2I = –2plog2 ∫ xdx α2
1 Sol 29: (i) lim 0
= lim
∴ I = –plog2 = plog α→0 α sinx α→0 2α sin α
2
π /3
dx
π /3
dx 1 1 1
(ii) = lim =
∫ = ∫ 2 α→0  sin α  2
π /6 1 + cot x π /6 π   
1 + cot  − x   α 
π /3 2 
dx x
= ∫ ∫ ntdt
π /6 1 + tanx (ii) y = x 1

π /3  xx 
tanx + 1  π π π logy = ∫∫log
ntdt logx
ntdt nx
nx
∴ 2I = ∫  dx = – =  
 1 + tanx  3 6 6 11 
π /6  
x
π 1 dy 1
logtdt
ntdt +(+nx)(
(logx)(logx)
y dx x 1∫
∴I= = nx)
12
M a them a ti cs | 23.43

x
–3/2 2
= –  x + 3x 
∫ log tdt  2
dy 1
x  + x2 + 3x
= x 1 log2 x + ∫ logtdt  –2 –3/2
dx  x1 
9 9    9 9 
e = –  – – (4 − 6) +  4 + 6 –  –  
dy ∫ log tdt  1
e  4 2    4 2 
= e1 log2 e + ∫ logtdt 
dx x = e  e1  9 9 9 25
= – 2 + 10 + = +8=
4 4 2 2
 1 
= e(eloge − e − ( −1)) log2 e + (eloge − e ( −1)) 2 2
 e  2 2
Sol 6: (B) ∫ | 1 − x | dx =2∫ 1 − x dx
1  –2 0
= e  + 1 =e + 1 1 2 
e  2 2
= 2  ∫ (1 − x )dx + ∫ (x − 1)dx 
 0 1 
x
2 2
Sol 30: f(x) = ∫ (t + 2t)(t − 1)dt 
x3
1
x3
2

1 = 2 x − + −x 
 3 3 
f’(x) = (x2 + 2x) (x2 – 1) > 0  0 
1

x(x + 2)(x – 1)(x + 1) > 0  1 8  1 


= 2 1 − +  − 2  −  − 1  
 3  3   3 
∴x ∈ (–∞, –2) ∪ (–1, 0) ∪ (1, ∞)
2 2 2
= 2 + +  = 4
3 3 3
Exercise 2
 x 4 − 5x2 + 4) 
Sol 7: (D) f’(x) =   × 2x = 0
Single Correct Choice Type  2 
 2 + ex 
0 1
x = 0 or (x2 – 4)(x2 – 1) = 0
Sol 1: (A) ∫ (1 − 2x)dx + ∫ (1 + 2x)dx
–1 0 ∴x = 0, x = ±2, x = ±1
0 1 x x
= x − x2 + x + x2 = 0 – [–1 – 1] + [1 + 1] = 4 Sol 8: (A) F1(x) = = t2 − 5t
–1 0 ∫ (2t − 5)dt 2
2
0 1 = x – 5x – (4 – 10) = x – 5x + 6
2 2

−x x
Sol 2: (C) ∫ e dx + ∫ e dx x
–1 1 F2(x) = ∫ 2tdt = x2
0 1 0
= –e–x + ex – [1– e+1] + [e1 – 1] 6 36
–1 0 ∴x = x2 – 5x + 6 ⇒ x =
2
,y=
5 25
= e+1 + e1 – 2 = 2e – 2

Sol 9: (A) f ( x=
) f (a − x )
3 1 2 3
Sol 3: (C) ∫ [x]dx = ∫ 0dx + ∫ 1dx + ∫ 2dx
0 0 1 2
g ( x ) =−
2 g(a − x )
=0+1+2=3
a a

π /2 π
I= ∫ f ( x ) g ( x ) dx = ∫ f ( a − x ) . ( 2 − g ( a − x ) ) dx
Sol 4: (B) ∫ cos xdx + ∫ – cos xdx 0 0
a a
0 π /2
⇒ 2 ∫ f ( a − x ) dx − ∫ f ( a − x ) .g ( a − x ) .dx
π /2 π
= sinx − sinx = 1 – [0 – 1] = 2 0 0
0 π /2
Put a – x = t
− dx = dt
–3/2 2
0 0
Sol 5: (A) ∫ –(2x + 3)dx + ∫ (2x + 3)dx
⇒ − 2 ∫ f ( t ) dt − ∫ −f ( t ) .g ( t ) dt
–2 –3/2
a a
2 3 . 4 4 | Definite Integration

0 0 π sec2 ( x / 2 ) .dx
⇒ − 2 ∫ f ( t ) .dt + ∫ f ( t ) .g ( t ) .dt ⇒ ∫
a a 0 (1 + a ) (1 + tan ( x / 2)) − 2a (1 − tan x / 2)
2 2 2

a a
⇒ I 2 ∫ f ( t ) .dt − ∫ f ( t ) .g ( t ) dt
=
0 0
Put tan ( x / 2 ) = t
a a a ∞
1 dt
=I 2 ∫ f ( t ) .dt −=
I ⇒ 2I 2 ∫ f (=
t ) .dt I ∫ f ( x ) dx . sec2 ( x / 2 ) dx = dt ⇒ 2 ∫
0 0 0
2 2
0 1−a + 1+a
2
t2 ( ) ( )

(1 + a) t  ⇒ 2 tan−1 ∞ − tan−1 0
log5
ex ex − 1 ⇒
2
1−a
. tan−1 .
(1 − a)  1 − a
( )
Sol 10: (B) ∫ ex + 3
dx
0
0
ex + 3 = t 2 π π
⇒ . ⇒
1−a 2 1−a
exdx = dt
8
t−4 π/ 4
sec x
1
∫ t
dt Sol 14: (B) ∫ dx = ∫ (1 − (1 − x)9 dx
4
0 1 + 2sin2 x 0
t = 4 sec2θ; dt = 8sec2qtanqdq 1
1
9
= ∫ x dx =
π/ 4 π/ 4 10
2 tan θ × 8 sec2 θ tan θ 0
∫ dθ = ∫ 4(sec2 θ − 1)dθ
2
0 4 sec θ 0 ∞
dx
Sol 15: (A) ∫
(x + )
π/ 4 π/ 4 3
= 4 tan θ – 4θ =4–p 0
x2 + 1
0 0
x = tanq
Sol 11: (A) π /2
sec2 θdθ
π /2
cos θ
π π ∫ (tan θ + sec θ) 3
= ∫ (1 + sin θ)3

∫ sinmx sinnxdx = 2 ∫ sinmx sinnxdx 0 0

–π 0 1 + sinθ =t
π
= ∫ [cos(m − n)x − cos(m + n)x]dx = 0 cosqdθ = dt
2 2
0 dt 1 1 1  3
∫ t3 = – = –  − 1 =
1 e
1 2t 2
1
2 4  8
Sol 12: (B) – ∫ logxdx + ∫ logxdx
1/e 1 5
Sol 16: (B) ∫  x − 3 dx
= –  xlogx − x  + xlogx − x
1 e
 1/e  1
 1
2 3 4 5

 1 1 1 
=– (–1) –  log −   + ( eloge − e ) − ( −1)
∫ 1dx + ∫ 0dx + ∫ 0dx + ∫ 1dx
1 2 3 4
 e e e 
1+1=2
 1 1
= 1 + –  – +e–e+1
 e e
π /2
sinx 2
Sol 17: (C) I = ∫ e− cos x dx
2
2  1 −π /2 1 + cos x
=2– = 2 1 – 
e  e π /2
− sinx 2
I= ∫ e– cos x dx
2
π – π /2 1 + cos x
dx
Sol 13: (C) ∫ 1 − 2a cos x + a2 ⇒2I = 0 ⇒I = 0
0
1/2
  x + 1 2  x − 1 2
1/2 
1 − tan ( x / 2)
2 Sol 18: (C)=I ∫  x − 1   x + 1  
   +   − 2 dx
⇒ Put cos x = –1/2  
1 + tan2 ( x / 2 )
M a them a ti cs | 23.45

2/2 π/ 4
1/2
x +1 x −1 2  π
= ∫ sec  x −  dx
=I 2 ∫ −
x −1 x +1
dx
2 0  4
0

1/2 π/ 4
1/2 
 x +1 x −1  4x 1  π  π 
= 2 ∫  −  dx = 2 ∫ dx = log sec  x −  + tan  x −  
 x −1 x +1 0 x2 − 1 2   4   4 
0 0

= –4 log (x2 − 1)
1/2

0
= –4 log
3
4
2I =
2 
2 
log ( 2 +1  ⇒ I =
 )1
2
log ( 2 +1 )
1 π /2
Sol 19: (A) ∫ {x − [x]}dx Sol 23: (C) µ10 = ∫ x10 sinxdx
–1 0
1 π /2
x2 0 1 
= –  ∫ (–1)dx + ∫ 0dx  µ8 = ∫ x8 sinxdx
2  –1 0  0
–1
π /2 π /2
= 0 – [–(0 + 1) + 0] = 1 µ10 = x10 (– cos x) – ∫ (– cos x)10x9 dx
0
0

10 10 π /2
[x2 ] π
Sol 20: (C) I = ∫ [(x − 14)2 ] + [x2 ] dx µ10 =   (0) + ∫ cos x10.x9 dx
4 2 0

10
[(14 − x)2 ]  π /2 
I= ∫ [x2 ] + [(x − 14)2 ] dx = 10  ∫ cos x.x9 dx 
4  0 
10  π /2 π /2 
∴2I = ∫ dx = 10 – 4 = 6 ⇒ I = 3 = 10  x9 sinx − ∫ 9x8 sinxdx 
4  0
0 
9
π
3 = 10 ×   – 90µ8
Sol 21: (A) ∫ (| x − 2 | +[x]) dx 2
9
–1 π
∴µ10 + 90µ8 = 10  
2 3 0 2
= ∫ (2 − x)dx + ∫ (x − 2)dx + ∫ (–1)dx
–1 2 –1 π
sin2 x
Sol 24: (C) ∫e cos3 (2n + 1)xdx
1 2 3
0
+ ∫ 0dx + ∫ 1dx + ∫ 2dx π
sin2 x
cos3 ( (2n + 1)π − (2n + 1)x ) dx
0 1 2
I= ∫e
2 3
x2 x2 0
= 2x − + – 2x – 1(0+1) +1 + 2 π
2 2 = – ∫ sin2 x cos3 ( (2n + 1)x ) dx
–1 2
0
 1 9 
= [4 – 2] –  –2 −  +  – 6  – (2 – 4) + 2 ∴2I = 0 ⇒I = 0
 2 2 

π /2
5 3
=2+ − +4=6+1=7 Sol 25: (C) I = ∫ sinlog  x + x2 + 1  dx
2 2  
– π /2

π /2 π /2
sin2 x cos2 x π /2
Sol 22: (C) ∫ dx = ∫ dx = ∫ sinlog  x2 + 1 – x  dx
0
sinx + cos x 0
sinx + cos x  
– π /2

π /2 π /2 π /2
1 1 dx 1
∴ 2I = ∫ sinx + cos x
dx =
2
∫  π
= ∫ sinlog dx
0 0 cos  x −  – π /2 x2 + 1 + x
 4
2 3 . 4 6 | Definite Integration

π /2 = 15a + 6b = 39 ….. (iii)


= – ∫ sinlog  x2 + 1 + x  dx = –I
– π /2
  ⇒ 9a = 45
a = 5; b = – 6; c = 3
∴2I = 0 ⇒ I=0
π/ 4
π/ 4
e− x sec2 xdx ex sec2 x
2α Sol 30: (A) I = ∫ e–2x − 1
= ∫ 1 − e2x
dx
Sol 26: (C) sinα + ∫ cos2xdx =0 – π/ 4 – π/ 4
α
π/ 4  ex sec2 x ex sec2 x 
1

∴ 2I = ∫  2x −  dx =
0
⇒ sinα + sin2x =0  e2x − 1 
– π/ 4  e −1
2 α
I=0
1
⇒ sinα + [sin4α – sin2a] = 0
2 a
Sol 31: (B) I =  1 + x2 
⇒ sinα + cos3α sinα= 0 ∫ log  x + 
–1
⇒ sinα = 0 or cos3α = −1    
–1 1
⇒ α = nπ, or 3α = (2n+1) π = log   = log  
 2   2 
x− 1+x   1+x −x
π
∴α = – a
3 =  1 + x2 − x 
∫ – log  

–1
a a
Sol 27: (B) f’(x) = sin x 4 dx 4 – sin x2 dx2 2 2
∴ 2I = ∫ log(x + 1 – x ) = ∫ log1 = 0
= 4x3sinx2 – 2xsinx –1 –1

0 2
π π sin2 x sin2 x
Sol 28: (B) 4 4 Sol 32: (C)∫ 1 dx + ∫ 1 dx
∫ x sinx cos xdx = ∫ ( π − x)sinx cos xdx
–2 – 0
0 0
2 2
π 0 2
2 1 − cos2x 2
∴2I = π ∫ sinx cos4 xdx =– ∫ dx + ∫ (1 − cos2x )dx
0 2 –2 1 20

2
Let cosx = t 0 2
1 1
–sinxdx = dt = –1[2] + sin2x + 1[2] – sin2x
2 –2
2 0
–1
2I = –π 4 1 1
∫t dt =–2+ [0 + sin4] + 2 – [sin4] = 4 − sin 4
1 2 2
1
4 π
2I = π ∫t dt = [1 + 1] x

–1
5 Sol 33: (C) f(x) = ∫ log(1 + t2 )dt
0
π
∴I =
5 f’(x) = log(1 + x ) 2

1
Sol 29: (B) f(0) = a + b = –1 f”(x) = × 2x
1 + x2
f’(x) = 2ae2x + bex + c  ….. (i) 2
∴f”(1) = =1
f’(log2) = 8a + 2b + c = 31 ….. (ii) 2
log 4 log 4

(ae )
π
∫ ( f(x) − (x)) dx = ∫
2x
+ bex dx Sol 34: (C)
dx
∫ 1 + 3cos x
0 0 0

a 2x log 4 a  39
= e + bex = 8a + 4b –  + b  = π
dx
π
dx
2 0 2  2 = ∫ 1 + 3cos( π−x) = ∫ 1 + 3– cos x
0 0
M a them a ti cs | 23.47

π
3cos x π /2 cos3 x
⇒ I= ∫0 dx  …..(i)
= ∫ 1 + 3cos x dx
cos3 x + sin3 x
0
π π 
cos3  − x 
∴2I = ∫ dx =π π /2  2 
0 ⇒I=∫ 0
3π  3π 
dx
π cos  − x  + sin  − x 
∴I =
2 2  2 

π /2 sin3 x
⇒ I= ∫0 dx  …..(ii)
sin3 x + cos3 x
Previous Years’ Questions
π /2
On adding Eqs (i) and (ii), we get 2I = ∫0 1 dx
π /2 cot x π π
Sol 1: (A) Let I = ∫0 cot x + tanx
dx  ….(i) =2I [x]
= π /2
0
2
⇒I=
4
1 2A
π /2 tanx Now, ∫0 f(x)dx =
⇒I= ∫0 cot x + tanx
dx  ….(ii) π

1
  πx   2A
On adding Eqs. (i) and (ii), we get ⇒ ∫0 A sin  2  + B dx =
π
π /2
2I = ∫0 I dx 1
 2A πx  2A
⇒ − cos + Bx  =
π  π 2 0 π
∴I =
4
2A 2A
⇒ B+ =⇒B=0
π π
π cos2 x
Sol 2: (C) Let I = ∫0 e . cos3 {(2n + 1)x} dx
a Sol 5: (A) It is a questions of greatest integer function.
Using ∫0 f(x)dx We have subdivide the interval π to 2π as under keeping
in view that we have to evaluate [2 sin x]
0, f(a − x) =−f(x)
 Y
=  a/2
2∫0 f(x)dx, f(a − x) =
f(x)
1,/2
cos2 x 3
Again, let f(x) = e .cos {(2n + 1)x}
O
cos2 x X’ X
∴ f(π – x) = (e ) {– cos3 (2n + 1)x} = – f(x) o o
30 30
(0,) (0,2)

∴I=0
-1,3/2
f(x) -1/2,7/6 -1/2,11/6
f(x) 2t ∫4 2t dt Y’
Sol 3: (A) lim
x →1 ∫4 x −1
dt = lim
x →1 x −1 π 1
We known that, sin = ,
6 2
(using L’ Hospital’s rule)
 π 7π 1
sin  π +  = sin =−
= lim 2f(x). f '(x) = 2f(1) . f’(1)  6 6 2
x →1 1
∴ 11π  π 1
= 8f’(1) [ f(1) = 4] sin
π
= sin  2π −  = − sin = −
6  6 6 2
π /2 1
Sol 4: (D) Let I = ∫0 dx 9π 3π
1 + tan3 x sin = sin = −1
6 6
π /2 1
=∫ dx Hence, we divide the interval π to 2π as
0
sin3 x
1+
cos3 x
2 3 . 4 8 | Definite Integration

 7π   7π 11π   11π  Sol 8: (C) The graph of y = 2sin x for π/2 ≤ x ≤ 3π/2 is
 π,  ,  ,  , ,2π  given in figure. From the graphs, it is clear that
 6   6 6   6 
 1  1  1   2, if x = π/2
sin x =  0, −  ,  −1, −  ,  − ,0  
 2  2  2   1, if π / 2 < x ≤ 5π / 6

[2sinx]  0, if
= 5π / 6 < x ≤ π
2sin x = (0, –1), (–2, –1), (–1, 0) −1,if π < x ≤ 7π / 6

[2sinx] x = –1 −2,if 7π / 6 < x ≤ 3π / 2
7 π /6 11 π /6
= ∫π [2sinx]dx + ∫ [2sinx]dx Y
7 π /6


+ ∫11π /6 [2sinx]dx 2
7 π /6 11 π /6 2π 1
=∫ −1 dx + ∫ −2dx + ∫ −1 dx
π 7 π /6 11 π /6

π  4π  π 10π 5π X
/2 5/6 7/6 3/2
= – − 2 − =− =−
6  6  6 6 3 -1

x 1
-2
Sol 6: (A) Given, ∫0 f(t)dt= x + ∫ t f(t)dt
x

On differentiating both sides w.r.t. x, we get 3π /2

f(x) 1 = 1 – xf(x) . 1
Therefore, ∫π /2 [2sinx]dx

⇒ (1 + x) f(x) = 1 =
5 π /2 π 7 π /6
∫π /2 dx + ∫
5 π /6
0dx + ∫
π
( −1)dx
1
⇒ f(x) = 3π /2
1+x +∫ ( −2)dx
7 π /6
1 1
⇒ f(1) = = 5 π /6 7 π /6 3π /2
1+1 2 =  x  +  −x  +  −2x 
π /2 π 7 π /6

3π / 4 dx  5π π   7 π 
Sol 7: (A) Let I = ∫π / 4  …..(i) = −  + − + π
1 + cos x  6 2   6 
3π / 4 dx
⇒ I= ∫π / 4 1 + cos( π − x)  −2.3π 2.7π 
+ + 
3π / 4 dx  2 6 
I= ∫π / 4  ….(ii)
1 − cos x 5 1  7 7 
= π  −  + π 1 −  + π  − 3
On adding Eqs. (i) and (ii), we get  6 2   6   3 

3π / 4  1 1  5 −3  1 7−9 π
2I = ∫  +  dx = π  + π −  + π  = −
π/ 4
 1 + cos x 1 − cos x   6   6  3  2

3π / 4  2  cos2 x
∫π / 4
π
⇒ 2I = dx Sol 9: (C) Let I =  ….(i)
 2 
 1 − cos x  ∫−π 1 + ax dx
3π / 4 2
−π cos ( − x)
⇒ I= ∫π / 4 cosec2 x dx = [ − cot x]3ππ/ 4/ 4 = ∫π 1 + a− x
d( −x)

 3π π cos2 x
=  − cot + cot  = – (–1) + 1 = 2 π
ax dx  ….(ii)
 4 4 
⇒ I= ∫−π 1 + ax
On adding Eqs. (i) and (ii), we get
M a them a ti cs | 23.49

 1 + ax
π  x2 +1 − t2
2I = ∫   cos2 x dx Sol 13: (D) Given, f(x) = ∫ e dt
−π  x  x2
1 + a 
On differentiating both sides using Newton’s Leibnitz
π 2 π 1 + cos2x formula , we get
= ∫=
−π
cos x dx 2∫
0 2
dx
2 +1)2 d 2  −(x2 )2  d 2 
π f’(x) = e−(x  (x + 1) −e  (x )
= ∫0 (1 + cos2x)dx  dx   dx 
2 +1)2 2 )2
π π
= e−(x . 2x − e−(x . 2x
= ∫=
0
1dx ∫ cos2x dx
0
4 + 2x2 +1) 2 +1
x π /2 = 2xe−(x (1 − e2x )
=  x  + 2∫ cos2x dx
0 0 2 +1 4 + 2x2 +1)
[where, e2x > 1, ∀ x and e−(x > 0∀x]
=π+0
∴ f’(x) > 0
⇒ 2I = π ⇒ I = π/2
which shows 2x < 0 or x < 0
π ⇒ x ∈ (–∞, 0)
Sol 10: (C) Given, F(x) = ∫0 f(t)dt
By Leibnitz rule, 1 1−x 1 1−x
Sol=
14: (B) I ∫= dx ∫ dx
F’(x) = f(x)  …(i) 0 1+x 0
1 − x2
But F(x2) = x2 (1 + x) = x2 + x3 (given) 1 1 1 x
⇒ F(x) = x + x
∫0 2
dx − ∫
0
dx
1 − x2
3/2
1−x
3 1/2 1 0t
⇒ F’(x) = 1 + x =I sin−1 x  + ∫ dt
2  0 1 t
3 1/2 (where, t2 = 1 – x2 ⇒ t dt = –x dx)
⇒ f(x) = F’(x) = 1 + x [from Eq. (i)]
2
0 π
3 (sin−1 1 − sin−1 0) + t  = − 1
I=
⇒ f(4) = 1 + (4)1/2 1 2
4
3 1 dx
⇒ f(4) = 1 + ×2=4 Sol 15: (A) Let I =
2 ∫−1 1 + x2
x Put x = tan θ ⇒ dx = sec2θ dq
Sol 11: (A) Given, f(x) = ∫1 2 − t2 dt
π/ 4 π
2 ∴ I 2∫ =
= dθ
⇒ f’(x) = 2−x 0 2
1 dx
Also x2 – f’(x) = 0 (B) Let I = ∫0
2
1 − x2
∴x = 2
2−x
Put x = sinθ ⇒ dx = cosθ dq
⇒ x4 = 2 – x2 ⇒ x4 + x2 – 2 = 0
π /2 π
⇒ x = ±1
∴ I
= ∫0 1dθ
=
2
3
3 dx 1   1 + x 
3+3T 1 (C) ∫2 1 − x2 = log  
Sol 12: (C) ∫ f(2x)dx put 2x = y ⇒ dx = dy 2   1 − x 2
3 2

1 6 + 6T 6I 1  4   3  1   2 
= 2 log  −2  − log  −1   = log   
2 ∫6
∴ f(y)dy
= = 3I
2      2   3 

2 dx 2 π π
(D) ∫1 = sec−1 x  = − 0 =
2  1 3 3
x x −1
2 3 . 5 0 | Definite Integration

2 2 π
Sol 16: ∫−2 | 1 − x | dx =
2
( −2 + 2 2)

−1 2 1
= ∫−2 (x − 1)dx + ∫ (1 − x2 )dx π( 2 − 1)
=
−1

2
+ ∫ (x2 − 1)dx 3 x
1 Sol 18: Let I = ∫2 5−x + x
dx  … (i)
−1 1 2
 x3   x3   x3 
=  − x  + x −  +  − x  3 2+3−x
 3  −2  3 
−1  3 1 ⇒I = ∫2 dx
(2 + 3) − (5 − x) + 2 + 3 − x
 1 8   1 1
=  − + 1 + − 2  + 1 − + 1 − 
 3 3   3 3 3 5−x
⇒I = ∫2 x + 5−x
dx  … (ii)
8 1 
+  − 2 − + 1
 3 3  On adding Eqs. (i) and (ii), we get
=4 3 x + 5−x
⇒ 2I = ∫2 5−x + x
dx
3π / 4 x
Sol 17: Let I = ∫π / 4 dx  … (i) 3 1
1 + sinx ⇒ 2I
= ∫=
2
1dx 1 ⇒ I=
2
 π 3π 
 + − x
3π / 4 4 4 
⇒ I= ∫π / 4  π 3π 
dx Sol 19: (B)
1 + sin  + − x
4 4  1
sinx
1
x
1
2 2
I =∫ dx < ∫ dx =∫ xdx = x3/2 2
=
 f(x)dxb b  x x 3 2 3
 ∫a ∫a f(a + b − x)dx 
= 0 0 0

2
3π / 4 π−x ⇒I<
3
⇒ I= ∫π / 4 1 + sin( π − x)
dx
1 1
cos x 1
J =∫ dx < ∫ dx |01 = 2
3π / 4 π 3π / 4 x x x
⇒ I= ∫π / 4 1 + sinx
dx − ∫
π / 4 1 + sinx
dx 0 0

∴ J ≤ 2.
dx 3π / 4
π∫
⇒ 1= − 1 [from Eq. (i)]
π / 4 1 + sinx x
Sol 20: (D) Let ∫ cot x dx  … (i)
π 3π / 4 dx 0
⇒ 1= ∫
2 π / 4 (1 + sinx) x x

π 3π / 4 (1 − sinx)
= ∫ cot ( π − x )dx, ∫ − cot x dx  ....(2) ... (ii)
0 0
⇒ 1= ∫ dx
2 π / 4 (1 + sinx)(1 − sinx) Adding (1) and (2)
x x x
π 3π / 4 (1 − sinx)
2I = ∫ cotx dx + ∫  − cot x dx =∫ ( −1 ) dx
2 ∫π / 4 1 − sin2 x
= dx
0 0 0
  x  +  −x  =−1if x ∉ Z 
π 3π / 4  1 sinx   
2 ∫π / 4  cos2 x cos2 x 
 −  dx  = 0if x ∈ Z 

π 3π / 4 =  −x  0x = −π
(sec2 x − sec x. tanx)dx
2 ∫π / 4
=
π
π 3π / 4 ∴= −
= tanx − sec x  2
2 π/ 4

π
= [ −1 − 1 − ( − 2 − 2)]
2
M a them a ti cs | 23.51

( x ) p' (1 − x )
Sol 21: (A) p'= π /3
tanx dx
= ∫
π /6 1 + tanx
⇒ p(x) =−p (1 − x ) + c
π /3
at x = 0
2I = ∫ dx
Now p ( 0 ) =−p (1 − x ) + 42
π /6

1 π π π
I
⇒ =  − = , statement-1is false
⇒ p ( x ) + p (1 − x ) =
42 2  3 6  12
1 1 b b
=I ∫ p ( x ) dx ∫ p (1 − x ) dx ∫ f ( x ) dx= ∫ f ( a + b − x ) dx it is property
0 0 a a
1
=2I ∫ ( 42 ) dx=⇒I 21.   x _ 1 
  x 
1
1  e+ x 
0 e +  1 −  e  dx
Sol 25: (D) ∫   x2  

Sol 22: (D) I = 8 ∫
1 log
(1 + x ) dx x+
1

1+x 2 = x.e x +c
0

∫ ( xf ' ( x ) + f ( x ) ) dx ( x ) + c
π
As
4 log (1 + tan θ )
= 8∫ sec θdθ (let x= tan θ )
2

0 1 + tan2 θ
4
π π π logx2
Sol 26: (C) I = ∫ logx2 + log 36 − 12x + x2 dx
=
4  π 
∫ log  1 + tan  4 − θ=
 dθ
4 4

∫ log2dθ − ∫ log (1 + tan θ )dθ 2 ( )


0  0 0 4
2 logx
dx ..(i) ... (i)
2 ∫2 logx + log ( 6 − x )
π I=
= 8log2 −1
4
2I = 2π log2 4 log ( 6 − x ) b b 
I = π log2 I
= ∫ logx ( 6 − x ) + log dx  ∫ f ( x ) dx
= ∫ f ( a + b − x ) dx  ...(ii)
2  a a 
x
Equation (i) & (ii) gives
Sol 23: (B, C) g ( x ) = ∫ cos 4t dt
0
( ) dx 4=
4 logx + log 6 − x

⇒ g' ( x ) = cos 4x ⇒ g(x) =


sin 4x
+k
= ∫ logx + log ( 6 − x )= ∫ dx 2
4 2 2

sin 4x Hence I = 1

= g(x)  g ( 0 ) 0 
=
4 
= g(x) + g(π
= ) g ( x ) − g ( π )  g ( π=) 0 ( )
JEE Advanced/Boards
π /3
dx
Sol 24: (D) I = ∫
π /6 1 + tanx Exercise 1
π /3 1
dx tan−1 x
= ∫ π 
Sol 1: ∫e sin−1 (cos x)dx.
π /6
1 + tan  − x  0
2  1
−1  π 
π /3 ∫ (tan x)sin−1  sin  − x   dx
tanx dx  2 
= ∫ 0

π /6 1 + tanx 1
π 
−1
= ∫  2 tan x − x tan−1 x  dx
0 
2 3 . 5 2 | Definite Integration

π /2 π /2
π  2sec2 θdθ
1 1
1 1 2x 2dθ 1
= 
2
x tan−1
x
0
– ∫ dx  –
2 0 1 + x2  ∫ x tan
−1
xdx = ∫ α cos2 θ + β sin2 θ
=
β ∫ α
  0 0 0 + tan2 θ
β
 1 1 
ππ ππ 11    x2 1 x2 + 1 − 1  Put tanθ = t
n2  - (tan−1 x)
2 ∫0 1 + x2
=   –– log
n2 − dx
22 44 22    2  ∞
 0 

π2 π 1 π 1  1 2 dt 2 1 t
–1 
1
= × tan−1
= – log
8 4
n2 – × +
2 4 2 (x)0 – tan 0 
  β ∫0 α 2 β α α
+t
β β β 0
ππ22 ππ ππ 11 11 ππ
= –– log
n2
n2–– ++ –– ×× 2 π π
88 44 88 22 22 44 = × =
αβ 2 αβ
2
π2 π π π 1 1
= – – ((((log2)
(( n2)
n2) ++ 1)
+ 1) 1) +
+
8 8 4 4 2 2 π /2
(α cos2 θ + β sin2 θ) × (β – α )sin2θdθ
(iv) I= ∫
0
(β – α )sin θ cos θ
Sol 2: (i) Put x = acos2θ + bsin2θ π /2
2
dx = 2(β – α)sinθcosθ dθ =2 ∫ (α cos θ + β sin2 θ)dθ
0
b
π /2
I= ∫ (x − α )(β − x)dx
∫ ((β − α ) sin θ + α)dθ
2
=2
α
0
π /2 π /2
π (β − α )
= ∫ (β − α )cos2 θ(β – a)sin2 θ = 2α × +2 ∫ (1 − cos2θ)dθ
0 2 0
2

× (β – α)sin2θdθ π /2
β−α
= aπ+2   ∫ (1 − cos2θ)dθ
(β − α )2
π /2
2  2  0
= ∫ sin 2θdθ
2 0
β−α π β α π
= aπ + × × 2 =  +  π = ( α + β)
(β − α ) 2 π /2 2 2  2 2  2
= ∫ (1 − cos 4θ)dθ
4 0
2π π
(β − α ) π (β – α )2 π
2
= × = Sol 3: (i) ∫ 1 − sintdt – ∫ 1 − sintdt
4 2 8 0 0

2π π

∫( )
β
(x − α ) 1
(ii) I= = 1 − sint + 1 + sint dt – ∫ 1 − sintdt
∫ (β − α )
dx
2 0 0
α

π /2 π π
(β − α )cos2 θ
= ∫ × (β – α )sin2θdθ = ∫ 1 − sint + 1 + sintdt – ∫ 1 − sintdt
0 (β − α )sin2 θ 0 0

π /2 π
cos θ =
= 2(β – α) ∫ sin θ
× sin θ cos θdθ ∫ 1 − sintdt
0 0
π
t t
π /2
 1 + co2θ  = ∫ sin 2 + cos 2 dt
= 2(β – α) ∫   dθ 0
0  2 
π
π t t
= (β – α) = –2cos + 2sin
2 2 20
π /2
2(β − α )sin θ cos θdθ = –2[0 – 1] + 2[1 – 0] = 4
(iii) I = ∫ 2 2
(α cos θ + β sin θ) × (β – α )cos θ sin θ
0
M a them a ti cs | 23.53

1 e

∫ {(1 + x)e }
x
(ii) x
∫ e (x − 1)
n (ii) + (1 − x)e–x logxdx
nxdx
0 1

logx = t
 1 1 
= (x − 1)n ex – n∫ (x − 1)n−1 ex dx  x = et ⇒ dx = et dt
 0
0 
1

∫ {{1 + e }} e { }
 n−1 x
1  t et
+ 1 − et e−e dt
t
(x − 1) e 0 
= - (–1)n – n   0
1
 n− 2 x 
 −(n − 1)∫ (x − 1) e dx  1
 et t   t t   t
 0  = ∫  e + e–e  t +  et ee − et e−e  t  e dt
0    
= –(–1)n + n(–1)n-1
 n− 2 x
1  ∞
(x − 1) e 0  dx
+ n(n-1)  1

Sol 5: R = ∫ 1 + x4
0
 n−3 x 
 −(n − 2)∫ (x − 1) e dx  1 1
 0  Put x = ⇒ dx = – dt
t t2
= –(–1)n + n(–1)n–1 – n(n–1)(–1)n–2 0 ∞
–t2 x2
= ∫ dt = ∫ 1 + x2 =P
1
n−3 x ∞1+ t2 0
– n(n–1)(n–2) ∫ (x − 1) e dx
0 ∞
1 + x2
Taking n = 3 ∴2I = 2P = ∫ 1 + x 4 dx
1
x 0
= –(–1)3 + 3(–1)2 – 3(3 – 1)(–1)1 – 3(2) (1) ∫ e dx ∞
0 1 + x –2
= +1 + 3 + 6 – 6(e1 – 1) = ∫ 2
dx
0  1
= 16 – 6e 1 −  + 2
 x
 x  ∞
dt 1
cos(sinx)cos2 = (Put x – = t)
Sol 4: (i)
π /2 

ex  2  ∫
∫ x
 dx –∞
2
t +2 x
0  + sin(sinx)sin−2 
 2  1 1

π /2
∴2I = tan−1
1 cos(sinx)[cos x + 1]  2 2
∫ ex   dx
–∞
2 0  + sin(sinx)[1 − cos x]
1 π π π
=  +  =
1
π /2
 cos(sinx) + sin(sinx)  2 2 2  2
∫ ex   
2 0  + cos x[cos(sinx) − sin(sinx)] π
∴I =
2 2
Put cos(sinx) + sin(sinx) = t

xdx
(–sin(sinx)cosx + cos(sinx)cosx)dx = dt ∫ 1 + x4
π /2 0
1
∫ ex {f(x) + f '(x)}dx Put x2 = t ⇒ 2xdx = dt
2 0
∞ ∞
1 dt 1 π
1 x
π /2
1 x π /2 = ∫ = tan−1 t =
e {cos(sinx) + sin(sinx)} 201+t 2 2
2
= e f(x) = 0
2 0
2 0
π π 2π
∴P + R – 2Q = + –
1 2 2 2 2 4
= eπ /2 (cos1 + sin1) – e°(cos0)
2  π π π
= – =
1 2 2 2 2 2
= eπ /2 (cos1 + sin1) – 1
2 
2 3 . 5 4 | Definite Integration

2
(x2 − 1) Put x3 – 3x2 + 4x – 2 = t
Sol 6: ∫ 2 1
dx 2
2
1
x5 2 – 2
+ 4
I= ∫ cos tdt = sint = sin2 + sin2 = 2sin2
–2
x x –2

2 1  4 4 ∴p = q = 2 ⇒ p + q = 4
Put 2 – + t ⇒  −  dx =
= dt
3
x2 x 4
x x5 
2
25 3x6 − 12x2 + 1
1 16
dt 1
25
1 5  1 Sol 10: I = ∫ x2 + 2
dx
4 ∫ t
=
4
×2 t
1
16 =  − 1 =
2 4  8
– 2
1
2
1000 3x6 + 6x 4 − 6x 4 − 12x2 + 1
∴ = 125 = ∫ dx
8 – 2
x2 + 2

Sol 7: h(x) = fog(x) + k 2


3x 4 (x2 + 2) − 6x2 (x2 + 2) + 1
= ∫ dx
dh(x) – sinx x2 + 2
= f ' {g(x)} g'(x) = – 2
dx cos2 (cos x) 2
1
f(x)
f(t) = ∫ 3x 4 − 6x2 + dx
2
j(x) = ∫ dt – 2
x +2
g(x)
g(t)
sinx dt  5 2
x
2
h(x) = – ∫ 2
dx =
∫ cos2 t = tant = 2× 3  x 2 3 
– x +2×
1
tan–1
cos (cos x) 5 3 0  2
2 0
= tan(cosx) + c  

∴f(x) = tanx, g(x) = cosx 4 2 4 2  π –16 2 π


= 6 − + = +
tan x
 5 3  2 2 5 2 2
tant
J(x) = ∫ dt
cos x
cos t
∞ ∞
dx 1 x
0 Sol 11: ∫ a2 + x2 = tan−1
sint a a0
j(0) = ∫ cos2 t dt 0

1
1π π
= ⇒ a = 2525
cost = u ⇒ –sintdt = du a 2 5050
1 cos1
du –1  1 
= ∫ u2 =
u 1
=– 
cos1
− 1 = 1 – sec1 2
x2 − x
2
x2 + x
  Sol 12:
0
∫ dx = ∫ dx
–2 x2 + 4 –2 x2 + 4
π /2 π /2 2 2 2
1 − sin2x  1 − tanx  x2 x2
Sol 8: ∫ dx = ∫  dx ∴I =
1 + sin2x

 1 + tanx  ∫ dx = 2 ∫ dx
0 0 –2 x2 + 4 0 x2 + 4
π /2 π/ 4
 π  π 2 
= 4
∫ tan  x −  dx = 2logsec
 4
2nsec  x − 
 40
= 2∫  x2 + 4 −

dx

0 0 x2 + 4 
= 2n 2 = log2 2
x 2 
 x + 4 + 2log x + x2 + 4 
2 = 2 2 
Sol 9: 2I = ∫ (3x2 − 3x + 1)cos(x3 − 3x2 + 4x – 2)dx  −8log x + x2 + 4 
0   0
2
+ ∫ (3x2 − 9x + 7)cos(x3 − 3x2 + 4x − 2)dx = 2 8 − 4log 2 + 2 2 + 4log2
0
2
2 3 2
2I = 2 ∫ (3x – 6x + 4)cos(x – 3x + 4x − 2)dx = 4 2 − 4log 1 + 2
0
M a them a ti cs | 23.55

2
  ⇒ dx =  1 − sin2 t  2sintdt = sin2tdt
π/ 4    
1  cos x 
2 ∫0  
Sol 13: u = dx π /2
π t sin2tdt
sin
 
 
x + 
4   ∫ sin t − sin2 t + 1
4
0

2 π 
 π  π /2 π /2  − t  sin2tdt
π / 4  cos  − x  t sin2tdt 2 
=
1   4   dx = ∫ = ∫
2 ∫0   0 1 − sin2 t cos2 t 0 1 − sin2 t cos2 t
cos x
 
  π
π /2
sin2tdt
∴2I = ∫
 π 
2 2 0 1 − sin2 t cos2 t
π / 4  cos  − x 
4 
v=2 ∫  π /2 π /2
π sin2tdt π sin2tdt
0
 cos x  2I= ∫ = 4. ∫
  2 0
2
sin 2t 2 0 3 + cos2 2t
  1−
4
v 2  –1  dt
= = 4
∴  1
u 1/2 2I= 2π ∫   –
 1  3 + t2   2
π/ 4 1
1 xdx π dt π 1
1
t
Sol 14: ∫
2 0 cos x cos  π − x 
∴Ι= ∫
2 –1 3 + t 2
=
2
× tan−1
  3 3 –1
4 
π  1 π π π π2
π/ 4  − x  dx =  +  =
1 4  2 3 6 6  6 3
=
2
∫ π 
0 cos  − x  cos x
 4  1 +1 +5 5
π 22
x2x2+ +1 1  1 
π/ 4 dx Sol 16: ∫∫ nn1 1+ +x x− − 1 dx
log
∴ 2I =
1 4  1   x x  dx

2 0 cos x cos  π − x 
1 +1 +5 5x 2 2 x2 2+ 1− 1   
x  x + 2 2 − 1  
  22  xx  
4 
1 +1 +5 5
π/ 4
π dx 22
11++x2x2     11  
I=
8 2
∫ π 
= ∫∫  
log
nn 1 1++ x x−−   dx
     x x  
22  dx
0 cos x cos  − x  11   1 
1 
  x x−−  ++11 
4     x x   
π/ 4
 
π dx
= ∫
8 2
cos x + cos x sinx 1  1 
0 x– = t ⇒  1 +  dx =
dt
x  x2 
π/ 4
π sec2 xdx
= ∫ 1
8 1 + tanx log(1 + t)
0 = ∫ dt
1 11 0 (t2 + 1)
π dt ππ π
= ∫ = log
n(1++t)t) = log2
n(1 n2 t = tanθ
8 01+t 88 00
8
ππ/ 4/ 4

1
= ∫∫ log
n(1
n(1++tan
tanθθ)d)dθθ
sin−1 x
Sol 15: ∫ x2 − x + 1 dx 00

0 ππ/ /44
 11−−tan
tanθθ
= ∫∫ log
nn11++ ddθθ
Let sin−1 x = t 00  11++tan
tanθθ
π / π4π
/ 4/ 4
1 1
× dx =
dt = ∫ ∫∫log
n(2)
n(2)
−−
n(2)−log
n(1n(1
++
n(1tan
+tan
θ)θθ)d)θddθθ
tan
1−x 2 x 0 00
2 3 . 5 6 | Definite Integration

ππ//44 Sol 19: cosx + cos3x = 2cos2xcosx


ππ π
∴2I = ∫∫ log
n2d = ln2
n2dθθ= og2⇒
n2 I = log2
n2
44 8 sinx + sin3x = 2sin2xcosx
00
π
1/n ∴I= ∫ (2cos x + 1)2 cos2 2x + sin2 2x dx
2  
Sol 17: lim n
n→∞ ∫ (2010 sinx + 2012cos x) | x | dx 0
–1/n π

1/n
= ∫ | 2cos x + 1 | dx
2 0
= lim n ∫ (2012cos x) | x | dx
n→∞ 2 π /3 π
–1/n
= ∫ (2cos x + 1)dx + ∫ (–2cos x − 1)dx
1/n 0 2 π /3
= 2012 lim 2n2 ∫ x cos xdx
n→∞ 2 π /3 2π π  2π 
0 = 2sinx + – 2sinx − n−
0 3 2 π /3  
3 

 1/n
1/n 
= 2012×2 lim n2  x sinx – ∫ sinxdx   3   4π   3
0 = 2 + – π – 20 − 
 
n→∞
0  2 
 2   3  
1 1 1  π π π
= 2012×2 lim n2  sin + cos − 1 = 3+ 3+ = 2 3+ = 12 +
n→∞  n n n  3 3 3
 1  ∴w = 12, k = 3
1
 sin cos − 1 
⇒ k2 + w2 = 9 + 144 = 153
= 2012 × 2 lim  n + n 
n→∞  1 1 
 n 2 
 n  1
(1 − x)(1 + x) dx
 1
Sol 20: ∫ x(1 + x)(1 + x) 1
0
= 2012 × 2 1 −  = 2012 +1+ x
 2 x

 
π 1  2 
1−x  1  dx
Sol 18: ∫ 2 sinx + 2cos x dx = ∫ 2
 1  
0 x 1
  x + 1  (1 + x)  x + x + 1
0

2 sinx + 2cosx > 0   


1
⇒ tanx > – 2 x –2 − 1 dx
= ∫ 1  1
 + x + 1
0
∴ x < π – tan–1 2 x
+ x +1
  x
π –tan−1 2 1 dt
3
= ∫
0
( 2 sinx + 2cos x dx ) Put
x
+x + 1 = t ⇒ – ∫
∞ (t + 1) t

π –tan−1 2 Put t = tan2θ ⇒ dt = 2tanθsec2θdθ


+ ∫ ( 2 sinx + 2cos x dx ) π /3
2 tan θ sec2 θ
π /2
π
=– ∫ dθ = 2 ∫ dθ
π− tan–1 2 π− tan−1 2 π /2 sec2 θ tan π /3
= – 2 cos x + 2sinx
0 0
π π π
π− tan–1 2 π− tan−1 2 = 2× –  =
+ 2sinx – 2 cos x 2 3 3
π π

=– 2[– cos tan−1 2 − 1] + 2sintan−1 2 π /2


 asinx + bcos x 
Sol 21: ∫   2dx
0  sinx + cos x 
+ 2 [sintan−1 2] – 2  – cos tan−1 2 + 1
 
π /2
(acos x + bsinx) 2
1 2 6 2 = ∫ dx
= 2 2× + 4× = = 2 6 sinx + cos x
0
3 3 3
M a them a ti cs | 23.57

π (a + b)π  π /3 
∴ 2I= 2(a + b) ⇒ I= π /3 π /3
2 2 2 = – 2 θ tan θ − ∫ tan θdθ  + π tan θ π / 4
π/ 4
 π/ 4 
1
Sol 22:  π/ 4 
∫ f(x)dx = 1 +2 θ tan θ
π/ 4
– ∫ tan θdθ 
0 0
 0 
1 1
f(2x)
⇒∫ dx = 1 ⇒ ∫ f(2x)dx = 3
3  π π   111 2 22 
0 0 = – 2  × 3 –   + log
n2
n2––– ln2]
n2 og
n2]
n2]  
π /2
(a + b)(sinx + cos x)  3 4   222 
∴2I ∫ (sinx + cos x)
2dx
0 ππ 11 
dt + π( 3 − 1) + 2  −− log
n2
n222
Put 2x = t ⇒ dx = 44 22 
2
2 2 2π π π
dt =– + + log2 + 3π–π+ – log2
∫ f(t) =3⇒ ∫ f(t)dt = 6 3 2 2
1
2 1
2 π
=
∴ ∫ f(t)dt = 6 – 1 = 5 3
1
π
(ax + b)sec x tanx
3 0 Sol 25: ∫ dx
Sol 23: ∫−1 {lx − 2l + [x]} dx = ∫−1 {lx − 2l + [x]} dx + 0 4 + tan2 x

−1 2 2I  π sec x tanx 
∫0 {lx − 2l + [x]} dx
= ∫1 {lx − 2l + [x]} dx = ∫ dx 
aπ + 2b  0 4 + tan2 x 
3  

+ ∫2 {lx − 2l + [x]} dx
–1 –1
dt 1 −1 t
0 1 = ∫ 3 + t2 = tan
∫−1 (2 − x − 1)dx + ∫0 (2 − x + 0)dx +
1 3 31

2 3
1  π π 1 2π (aπ + 2b)π
∫1 (2 − x + 1)dx + ∫2 (x − 2 + 2)dx + = π − −  = × ∴I =
3 6 6  3 3 3 3
0 1 2 3
x2 x2 x2 x2
=x − + 2x − + 3x − + π
(2x − 3)sinx
2
−1
2
0
2
1
2
2
Sol 26: ∫ (1 + cos2 x) dx
0

 1  1  1 9 π
−  −1 −  +  2 −  + (6 − 2) −  3 −  + − 2
= (2π + 6)sinx
 2  2  2 2 2I = ∫ dx
0 1 + cos2 x

 1  1  1 9 I
π
sinx
–1
–dt
1
dt
−  −1 −  +  2 −  + (6 − 2) −  3 −  + − 2
= =
 2  2  2 2 π+3 ∫ 1 + cos2 x dx = ∫ 1 + t2 = ∫ 1 + t2
0 1 1
=7 1 π π π
= tan−1 t = + ⇒ I = (π + 3)
−1 4 4 2
Sol 24: x = tanq
cos x
dx = secθdq Sol 27: Let f ( x ) =  ….(i)
cos x + sin x
π /3
 –1 2 tan θ  2
∫  sin

 sec θdθ
1 + tan θ 
0 π 
cos  − x 
2
π/ 4 π /3
π  
= 2θ sec2 θdθ + ( π − 2θ)sec2 θdθ Then, f  − x  =
∫ ∫ 2  π  π 
0 π/ 4
cos  − x  + sin  − x 
 2   2 
2 3 . 5 8 | Definite Integration

π /2
sin x 1  x
=  …(ii) = 2  x + 2 log cos 2 
sin x + co s x  0

1  π π 
  cos x + sinx =  + 2 log cos  − ( 0 + 2 log 1 ) 
) + f  2π − x 
Now, f ( x= = 1 2  2 4 
  cos x + sin x
1 π 1  1 π 1 1 π 
1
π2
 π  1
π /2 =  + 2 log =  + log  = 2  2 − log 2
f ( x ) + f  −=
2 2 2  2 2 2  
=∴I
2 ∫  2
x   dx
 2 ∫ x dx
0 0

1 π /2 1π  π a
= x  =  − 0 = log(1 + ax)
2  0 22  4 Sol 30: ∫ dx
0 1 + x2

nπ ax = tanθ ⇒ abx = sec2θ dq


nπ | sinx |
Sol 28: 2I = ∫ dx
1 + (cos x) tan−1 a2
0 alog(1 + tan θ)
∫ × sec2 θ = dθ
π
sinx
π /2
sinx 0 (a2 + tan2 θ)
2I = n2π ∫ dx = 2n2π ∫ dx
0
1 + cos x 0
1 + cos x
 tan−1 a2 
 tan−1 a2 2 
0 sec θ
–dt 1 log(1+tanθ)
n(1 + tan θ) ∫ dθ 
= 2n2π ∫ = 2n2π log
n(t + 1) = 2n2p log2  a2 + tan2 θ 
1
1+t 0
a 0
0 
 tan−1 a2 tan−1 θ 
∴I = n2p log2 = 100p log2  sec θ2 2
sec θ 
 – ∫ ∫ a2 + tan2 θ dθ
∴n = 10  0
(1 + tan θ ) 0 

 tan−1 a2
 1 −1 tan θ
π /2
cos x log(1+tanθ)
a n(1 + tan θ × tan

Sol 29: ∫ 1 + cos x + sinx
dx

a α 0
0
tan−1 a2 
π /2
cos x sec2 θ 1 tan θ 
– ∫  × tan−1  dθ
= ∫ (1 + cos x ) + sinx dx
0
 1 + tan θ a
 a 
0

x x 2I = log(1 + a2)tan–1a
π /2 cos − sin2 2

= ∫ 2 2 dx I = tan–1a log 1 + a2
0 2cos2 x x x
+ 2sin cos
2 2 2
n3 n3 n3
x
π /2 1 − tan2
2
ex + 1 2
ex dx 2
1
2 dx s Sol 31: ∫ dx = ∫ + ∫ dx
= ∫ x 0 e2x
+1 0
2x
e +1 0
2x
e +1
0 2 + 2 tan
2 e2x = t
x
[Dividing numerator and denominator By cos2 ] 2e2x dx = dt
2
 x  x 1
π /2  1 − tan   1 + tan  dx = dt
1  2  2 2t
=
2 ∫ x
dx
0 1 + tan log3 3
2 1 1
+
2 1∫ (t + 1)t
tan −1
e x 2 dt
π /2 0
1  x
=
2 ∫  1 − tan  dx
2
 1  1
3
0 π 1 
= tan–1 3 – + ∫  −  dt
4 2  1  t t + 1 
M a them a ti cs | 23.59

1 1 π π  1  1 1 
=   + log3
+n3 −n2
−−log2
n3 n2 = sin α sin β  −  =0
2 2 6 6  2  2αβ 2αβ 

a π /2
p + qπ
Sol 32: Given, ∫ x dx = 2a ∫ sin3 x dx
a 0
Sol 34: ∫ | cos x | dx
a 0
 x3/2  π /2
3 sin x − sin3 x p p + qπ
⇒  = 2a ∫ dx
 3 / 2  0 0
4 ∫ cos x + ∫ | cos x | dx
0 p

 sin
= 3x 3 sin x − 4 sin3 x  qπ
  p
π /2
= sinx +
0 ∫ (cos x)dx
2  3/2 a   cos 3x   0
⇒ a 0
−= 3 ( − cos x ) −  − 
3  2
  3 0 qπ
= sinp + q×2 ∫ (cos x)dx
0
2 3/2 a   π  1 3π 
⇒ a = −3  cos − cos 0  +  cos − cos 0   = 2q + sinp
3 2  2  3 2 
2
2 a 1  2/3 9 x − 2 
 −3 ( 0 − 1 ) + ( 0 − 1 ) 
–5
⇒ a3/2 = (x +5)2 e  3  dx
3 2 3 
Sol 35: ∫e dx + 3 ∫
–4 1/3
2 3/2 4a
⇒ a = ⇒ a a − 2a = 0  2
3 3 Let x + 5 = t and 3  x −  = t
 3
⇒a ( )
a − 2 = 0 ⇒ a = 0 or a = 2 ⇒ a = 0 or a = 4 0
t 2 t
0
2
= ∫ e dt + (–1)∫ e dt =
0
When a = 0: 1 1
1
a+1 1  x2  1 1 Put t = –2
∫=
x dx ∫=x dx   =
2
( 1 − 0) =
2
a 0  2  0 1
t2 z
0
2

When a = 4:
∫ e dt + (–1)∫ e dz = 0
0 1

a+1 5 5
 x2  1 9
∫=
x dx ∫=x dx =  ( 25 − 16 ) =
2 Sol 36: sin π x > cos π x
0 4  2  4 2
π π
2nπ + < px < 2nπ +
4 4
Sol 33: tan x = 2x
1 1
2n + < x < 2n +
1
1 4 4
2 ∫0
[cos(α − β)x − cos(α + β)x]dx
10
π

1  sin(α − β)x
1 1
sin ( α + β ) x 

4 2 –10
∫ F(x)dx

2  ( α − β) 0 ( α + β) 
 0
 π
1
= × 2 × 10 ∫ f(x)dx
4 2
1  sin(α − β) sin ( α − β ) 
0
 − 
2 α −β ( α + β)  5π 
1/ 4 1 
  =  ∫ cos πxdx + ∫ sin πxdx 
sinα = 2acosa 2  0 1/ 4 

sinβ = 2bcosb 5π  1  π  1 π 
=  sin − 0  − cos π − cos  

sin α
sin β sin α

  sin αisnβ cos α cos β 
sin β   + 2 π  4  π 4 
1  2β 2α 2β 2α

=  − 
2  α −β   ( α + β) 
   
   
2 3 . 6 0 | Definite Integration

5π  1 1  2 π 
=  +  =5 π [x − (x − π)2 ]sin2x sin  cos x  dx
2  2π 2π  Sol 40: 2I = π ∫ 2 
0
2x − π
π /2  1 + sinx − (1 − sinx) π
tan−1  π 
Sol 37: ∫  1 + sinx + (1 – sinx) − 2 1 − sin2 x
dx = p2 ∫ sin2x sin  cos x  dx
0
0 2 
π /2
2sinx
π /2
2sinx π π
= tan−1 tan−1 Let cos x = t ⇒ – sinxdx = dt
∫ 2 − 2cos x
dx = ∫ 2 − 2cos x
dx 2 2
0 0
– π /2
2 2
π /2
π x
π /2
π x = – π2 × ∫ 2. t sintdt
= ∫ tan−1 tan  −  dx = ∫  −  dx π π /2
π
0 2 2 0 2 2
π /2

π2 1 π2 1 π2 π2 3π2 2I = 8 ∫ t sintdt
= − × × = − = – π /2
4 2 4 2 4 16 16
 π /2
π /2 
1 I = 4  −t cos t + ∫ cos tdt  = 4[+2] = 8
– π /2
 
Sol 38: x2 + 2x = k + ∫ t + k dt – π /2
0
t = k = 0 ⇒ dt = dU 1/3
 3 1 
(x − x )
1/3
k +1 1 3 1 
x  2 − 1 
1 x 
kt ∫ udu = (k + 1)2 − k 2  Sol 41: ∫ dx = ∫ 
k
2 
1/3 x4 1/3 x 4

2k + 1 1/3
=  1 
2 1  2 − 1
x
4k + 1  4k + 1  dx = ∫   dx
x2 + 2x = ⇒ x2 + 2x –   =0 1/3 x 3
2  2 
1 2 1 1
4 + 2(4k + 1) Put −1 =t, then − dt or
dx = dx = − dt
⇒ x = –2 ± x 2
x 3 3
x 2
2a
1 1
⇒ x = real and distinct When x = 1,t = − 1 = 0 and when x = ,t = 9 − 1 = 8
2
1 3
0
 
(x − x )
1 1/3
2x332 + x998 + 4x1668 sinx691 1 3 0
Sol 39: I = ∫ dx 1 1/3 1  t 4/3 
–1 1 + x666 Now, ∫ x4
dx =− ∫ t dt =
2 8
− 
2 4 

1/3
1 1  3  8
2x332 + x998 2x332 + x998
I= ∫ dx = 2 ∫ dx 3 3 3
1 + x666 1 + x666 =− 0 − 8 4/3  =−  −24  = − ( −16 ) =6
–1 0
8  8   8
 1  x332  
= 2 ∫  + x332  dx 
 0  1 + x 666  
  1  n−1
k +1 
Sol 42: lim  ∑ k ∫ (x − k)(k + 1 − x)dx 
n→∞ n2  
1
1
x332 k = 0 k
= 2 + 2∫ dx
333 333 2 x–k=t
0 1 + (x )
1 1 2
1
∫ t(1 − t)dt =
 1  dt   1 2
=
2
+ 2 ∫  
∫   − (t − 1) dx
2
0 0
333  0  1 + t2   333 1
 1
2 t − 
1 1 1  1 1 2
2 2 1
tan−1 t =
2  π π + 4 = t −  −  t −  + sin−1 
= + 1 +  = 2 2 4  2 8 1
333 333 0 333  4  666
2
0
M a them a ti cs | 23.61

π /2  π /2
π /2 
π 1 π 1 π = 4 ∫ tsint dt = 4  t(– cos t) + ∫ cos tdt 
– π /2
= × + × = −π /2  – π /2 
2 8 2 8 8
= 4 sint
π /2 
π1 n−1 1
π π 1 π =8
 – π /2 
∴ lim 2 ∑ k × = ∫ 8 xdx = 8 × 2 = 16 
n→∞ n
k =0 8 0

π /2
sinx + 3 3
Exercise 2
Sol 43: I = ∫ 5sin(x + α ) + 25
dx cosα =
5
0
Single Correct Choice Type
π /2
4 cos x + 3sinx + 25 π 2
4I + 3J = ∫ dx =
0
4 cos x + 3sinx + 25 2 Sol 1: (C) ∫ (x + 3)(x − 1) dx
0
π /2
3cos x − 4 sinx 1 2
3I – 4J = ∫ 4 cos x + 3sinx + 25
dx = ∫ (x + 3)(1 − x)dx + ∫ (x + 3)(x − 1)dx
0 0 1
π /2 1 2
= log
n(4 cos x + 3sinx + 25) = – ∫ (x2 + 2x − 3)dx + ∫ (x2 + 2x − 3)dx
0

28 1 1
= log(28) – log(29) = log
29 x 3
1  x
3
2
28 28 = –  + x2 − 3x  +  + x2 − 3x 
 3   3
0 1
16I + 9I = 2π + 3log
3n = 2π + 3log 
29 29
a + b + c + d = 2 + 3 + 28 + 29 = 62 1  8 1 
= –  + 1 − 3 +  + 4 − 6 −  + 1 − 3  
3   3  3 
Sol 44: f(x) = ax2 +bx + c 5 2 5
= + + =4
f’(x) = 2ax + b 3 3 3
f’(2) = 4a + b = 1 π /2
1 1
f’(2) = 4a + b = 1 Sol 2: (B) ∫ 2
sinx −
2
cos x dx
0
2 +π 2 +π
x −2 (x − 2)
∫ f(x)sin  2  dx = – ∫ f(4 − x)in 2 dx 1 
π/ 4 π /2 
2 −π 2 −π = 
2  0
∫ (cos x − sinx)dx + ∫ (sinx − cos x)dx 

π/ 4
2 +π
(x − 2)
2I = – ∫ f(x) − f(4 − x) sin dx 1  π/ 4 π /2 
2 = sinx + cos x + (–cox − sinx)
2 −π 
2 0 π /4 

2 +π ax 2 + bx + c  
  x −2
= – ∫ sin   dx 1  1 1    1 1  
 2 
− a(4 − x) + b(4 − x) + c   2  =  + − 1  – 1 −  +  
2 −π 
  
2   2 2    2 2   
2 +π
a(x − 4 + x)(x + 4 − x)   x − 2 
= ∫   sin   dx 1  2 2 −2
2 −π 
+b(x − 4 + x)  2  = 2 −1 −1 + 2 = =2– 2
2   2
2 +π
x −2
= ∫ ( a(2x − 4)4 + 2bx − 4b ) sin   dx
2  π /2
2 −π 4  (4 − 1)(4 − 3)  π 3π
Sol 3: (D) 2 ∫ (sin x)dx = 2 ×  × =
2 +π 0  4 × (4 − 2)  2 8
(x − 2)
= ∫ (8ax + 2bx − 4)sin dx
2 −π
2
1 2 1.5

2 +π Sol 4: (B) =
∫ 0dx ∫ 1dx + ∫ 2dx
(x + 2)  x − 2  x−2
= 4 ∫ sin  dx =t 0 1 2
2 −π
2  2  2
= 2 – 1 + 2(1.5 – 2)=2– 2
2 3 . 6 2 | Definite Integration

π
x π/ 4
sec x
Sol 5: (A) I = ∫ dx  …(i)
0 a2 cos2 x + b2 sin2 x
Sol 6: (A) ∫ 1 + 2sin2 x
dx
0

Then I = ∫
π
(π − x) dx
π/ 4
dx
= ∫
0 a cos ( π − x ) + b sin ( π − x )
2 2 2 2
0 cos x + 2sin2 x cos x

 a a  π/ 4
cos x dx
 ∫ f ( x )=
dx ∫ f ( a − x ) dx  = ∫
 0 0  0 cos x + 2sin2 x cos2 x
2

π
π−x π/ 4
cos xdx
Or I = ∫ a2 cos2 x + b2 sin2 x dx  …(ii) = ∫
0
0 (1 − sin x)(1 + 2sin2 x)
2

Adding (i) and (ii), we get 1/ 2


dt
π
x +π− x
= ∫ (1 − t )(1 + 2t2 )
2
0
2I = ∫ a2 cos2 x + b2 sin2 xdx
0
1/ 2
1  1 2 
π
=
3 ∫ 
1 − t 2
+ dt
1 + 2t2 
1 0
=π ∫ a2 cos2 x + b2 sin2 x dx
0 1/ 2 1/ 2
1 1 1 1
= ∫ dt + ∫ dt
π /2 3 1−t 2 3 1 2
1 0 0 +t
= 2π ∫ 2 2
a cos x + b sin x 2 2
dx 2
0 1/ 2
1 1+t 1 1 1/ 2
 2a a  = n + × tan−1 2t
 ∫ f ( x ) dx 2 ∫ f ( x ) dx,
= = if f ( 2a − x ) f ( x )  6 1−t 0 3 1/ 2 0
 0 0 
π /2
sec2 x 1 2 +1 2 π 1 π 
= n + × = n( 2 + 1) +
∴ 2I =2π ∫ a2 + b2 tan2 x
dx
6 2 −1 3 4 3 

2 2
0

[Dividing num. and denom. By cos² x]


1
x2
π /2
sec x 2 Sol 7: (C) ∫e (x − α )dx = 0
Or I = π ∫ a2 + b2 tan2 x
dx 0
0
For this integral to be zero
Let tan x = z . Then, sec x dx = dz 2
If α < 0 then x – α when x ∈ (0, 1) > 0
π π ∴It is not possible that integral reduce to zero
Also x = 0 ⇒ z = tan 0 = 0 and x → 2 ⇒ z → tan 2
If 2 > α > 1 then x – a
or z → ∞
when x ∈ (0, 1) < 0 function gives negative value and so

π dz

dz cannot reduced zero.
∴ I =π ∫ = ∫ ∴If 0 < α < 1, fn can take both positive and negative
(a / b)
2 2 2
0 a +b z b2 0
2
+ z2
valuce and it is possible that integral reduced to zero

1  −1  z  
π
= × tan   π /2
b (a / b)   a / b 0
∫ {x – [sinx]} dx
2
Sol 8: (A)
∞ 0

⇒I
=
π
ab
 −1  bz  
tan =
 a
 0

π
ab
tan−1 ∞ tan−1 0 ( ) [ ] → greatest integer function
 π
[sinx] = 0 sinx ∈ [0, 1) i.e. x ∈ 0, 
π π  π2  2
=  − 0  = 2ab π /2 2
π /2
π2
ab  2  = ∫ (x − 0)dx = x =
0 2 8
0
M a them a ti cs | 23.63

100 2t
Sol 9: (B) ∫ sin ( x[x]) πdx Put ex – 1 = t2 ⇒ exdx = 2tdt or dx = dt
1 + t2
0
ex –1 ex −1
Since x – [x] has a period of 1 2 π
∫ dt = 2 tan−1 t =
1 1+t 2 1 6
1
100  1
∴I = 100 ∫ sin πxdx =  – cos πx 0 
π    π π
0 2 tan−1 t –  =
 4 6
100 200
=
π
( –(–1 – 1) ) =
π π π
∴tan–1t = ⇒ ∴t = tan = 3
3 3

xlogx ∴ex – 1 = 3 ⇒ x = log4
Sol 10: (B) ∫ (1 + x2 )2 dx
0 2
r
 
x = tanθ ⇒ dx = sec2qdq n
r2 n
n 1
Sol 13: (A) lim ∑ = lim ∑ ×
π /2
n→∞
r =1 r
3 3
+n n→∞
r =1  r
3 n
tan θ log(tan θ)sec2 θdθ   +1
I= ∫ n
sec 4 θ  
0
1 1
x21 n2
π /2 = ∫ dx = n(x3 + 1) =
= ∫ tan θ log(tan θ)cos2 θdθ 3
0 x +1
3 0
3
0

π /2  n n2  1
= ∫ sin θ cos θ log(tan θ)dθ Sol 14: (B) lim  ∑ − 
 
 r =1 (n + r )  2n 
n→∞  2 2 3/2
0
n 1
π /2 1 1 1
= lim ∑ × = ∫ dx
= ∫ cos θ sin θ logcot θdθ n
( )
n→∞ 3/2 3/2
r =1   2 0 1 + x2
0  1 +  r  
π /2

  n  
∴ 2I = ∫ sin θ cos θ logtan θ + logcot θ  dθ
0
Put x = tanθ ⇒ dx = sec2θdq
π/ 4
π /2 π sec2 θdθ π/ 4 1
= ∫ cos θdθ = sin θ =
= ∫ sin θ cos θ logtan− logtan θ  dθ = 0 4 ∫ sec3 θ 0
0
2
0

n
n n
Sol 15: (A) lim ∑
nn
  r r  11 –
lim∑
Sol 11: (B) log I = lim ∑log
n
n 1
 1++n  ××n n→∞
n→∞
r =1  
n→∞r =1 n  n r =1 [n + 4(r − 1)]3 (n + 4n)3
1 n
1 1 1
log I = ∫ log(1
n(1 + x)dx
+ x)dx lim ∑ × –
n→∞
r =1 3 n (5) × n1/2
3/2
0  4r 
1 1 + 
1 x  n
logI = x log (1 + x) – ∫ 1 + x dx
0 1 1
0 1
1
= ∫ dx = ∫ (1 + 4x)–3/2 dx
1 (1 + 4x)3
= log2 – ∫ 1 − n(1 + x)10
dx = log2 – [1] + log 0 0

0
1+x
1 1
1/2
4 4 (1 + 4x)−1/2 1 1 1 
= 2log2 – loge = log ⇒I= = × = –  
e e –1 / 2 4 2  1 + 4x 
0 0

Sol 12: (A)


x


1
dx
1 1
=– 
2 5
 1
− 1=
 10
5− 5 ( )
log2 ex − 1
2 3 . 6 4 | Definite Integration

 n  πr   1 2n n
Sol 16: (B) log I = lim  ∑ logtan    ∴ I(m,n)= − . I(m + 1,n − 1)
n→∞   2n   n m+1 m+1
 r =1
1
π  x
= ∫ logtan  x  dx 1 − (f '(t))2 dt
0 2 
Sol 3: (C) Given ∫0
x
π 2 = ∫ f(t)dt, 0 ≤ x ≤ 1
x = t ⇒ dx = dt 0
2 π
π /2
Differentiating both sides w.r.t.x by using Leibnitz rule,
we get
⇒ ∫ logtantdt =0
0
1 − (f '(x))2 =
f(x)
∴I=e =1 0

⇒ f '(x) =± 1 − (f(x))2
Previous Years’ Questions f '(x)
⇒∫ dx = ± ∫ dx
1/2  1 + x  1 − (f(x))2
Sol 1: (A) ∫−1/2  [x] + log  1 − x   dx
⇒ sin–1 (f(x)) = ± x + c
1/2 1/2 1 + x  Put x = 0
= ∫−1/2 [x]dx + ∫−1/2 log  1 − x  dx ⇒ sin–1 (f(0)) = c
1/2 ⇒ c = sin–1 (0) = 0 (∴ f(0) = 0)
= ∫−1/2 [x]dx + 0
∴ f(x) = ± sin x
 1 + x   but f(x) ≥ 0, ∀ x ∈ [0, 1]
 log   is an odd function
 1 − x   ∴ f(x) = sin x

=
0 1/2 As we know that,
∫−1/2 [x]dx + ∫0 [x]dx
sin x < x ∀ x > 0
0 1/2
= ∫−1/2 (−1)dx + ∫0 (0)dx
1 1 1 1
∴sin   < and sin   <
0 2 2 3 3
=  x 
−1/2
1 1 1 1
⇒ f   < and f   <
 1 1 2 2 3 3
= −0 +  =−
 2 2

1 m Sol 4: (A) x2 = t ⇒ 2x dx = dt
Sol 2: (A) Here, I(m, n) = ∫ t (1 + t)ndt reduce into
0
log 3
I(m + 1, n – 1) [we apply integration by parts taking 1 sin t
I= ∫
2 log 2 sin t + sin (log 6 − t)
dt
(1 + t)n as first and tm as second function] ∴
1
 tm+1  log 3
= (1 + t)n .
∴ I(m,n)  1 sin (log 6 − t)
2 log∫ 2 sin (log 6 − t) + sin t
m + 1  I= dt
 0

1 tm+1
− ∫ n(1 + t)(n−1) . dt log 3
0 m+1 1 1 3
2 log∫ 2
=2I 1dt ⇒ I log
4 2
2n n 1
= − ∫ (1 + t)(n−1) . tm+1dt
m+1 m+1 0
M a them a ti cs | 23.65

n
n ∴ In+2 = In . … (iii)
Sol 5: (A, D) Gives, Sn = ∑ n2 + kn + k 2
k =0
π sinnx
Since, In = ∫0 dx ⇒ I1 = π and I2 = 0
  sinx
n   n
1 1 1 ∴ From Eq. (iii) I1 = I3 = I5 = …. = p
∑ n  k k 2  < nlim ∑
k 0n
→∞
k 0=
 1 + +  and I2 = I4 = I6 =…..= 0
 n n 
2
10 10
  ⇒ ∑ I2m+1= 10π and ∑ I2m = 0
  m=1 m=1
 1 
 2  ∴ Correct options are A, B, C.
 k k  
1 + n + n 
    4
1x (1 − x)4
1 Sol 7: (A) Let I = ∫0 dx
1 1  2 −1  2  1   1 + x2
==
∫0 1 + x + x2 dx  3 tan  3  x + 2  
 0 4
1 (x − 1)(1 − x)4 + (1 − x)4
=∫ dx
2 π π π 0
(1 + x2 )
= . −  =
3  3 6  3 3
1 1 (1 + x2 − 2x)2
π = ∫0 (x2 − 1)(1 − x)4 dx + ∫ dx
i.e. Sn < 0
(1 + x2 )
3 3
π 1
 4x2 
Similarly, Tn > 2
− 1)(1 − x)4 + (1 + x2 ) − 4x + 4 −
3 3
= ∫0 (x  dx
(1 + x2 ) 

π sinnx 1 4 
Sol 6: (A, B, C) Given In = ∫−π (1 + πx )sinx dx  … (i) = 2
− 1)(1 − x)4 + (1 + x2 ) − 4x + 4 −
∫0  (x  dx
1−x  2

b b
Using ∫a f(x)dx
= ∫ f(b + a − x)dx
a =
1 6
∫0  x − 4x51 + 5x 4 − 4x2 + 4 −
4
 dx 2
1+x 
π πx sinnx
we get In = ∫−π (1 + πx )sinx dx  … (ii) 1x
4x6 5x5 4x37 
1

= ∫  − + − + 4x − 4 tan−1 x 
0 7 6 5 3
  0
Adding Eqs. (i) and (ii), we get

sinnx π sinnx 1 4 5 4 π  22
π = − + − + 4 − 4 − 0 = −π
=2In ∫=
−π sinx
dx 2∫
0 sinx
dx 7 6 5 3  4  7

sinnx = (cosec x.cot x + sec2x – cos x)


( f(x) = is an even function)
sinx . (cos3x – cos x) . cos x
π sinnx  sin2 x + cos3 x − cos3 x.sin2 x 
⇒ In = ∫0 sinx
dx = − 
 sin2 x.cos2 x 
π sin(n + 2)x − sinnx
Now, In+ 2 − In = . cos2x . sin2x
∫ 0 sinx
dx
= – sin2x – cos3x (1 – sin2x)
π 2cos(n + 1)x.sinx = – sin2x – cos5x
= ∫0 sinx
dx
π /2 π /2
∴ ∫0 − ∫ (sin2 x + cos5 x)dx
f(x)dx =
0
π
= 2∫ cos(n + 1)x dx
0
π
 sin(n + 1)x 
2=
  0
 (n + 1)  0
2 3 . 6 6 | Definite Integration

 ⇒ f’’(a) = 0, ∀ a ⇒ R
m+1 n+ 2 
 π /2  ⇒ f(x) must have maximum degree 1
 m n 2 2 
 ∫0
sin x.cos x dx =
m+n+2 
 2 
 2  x − [x] if [x] is odd
Sol 11: Given, f(x) = 
 3 1 1 + [x] − x if [x] is even
6 1
 . . 
π /2 
− 2 2 + 2 2 
∴ ∫ f(x)dx =
 f(x) and cos px both are periodic with period 2 and both
0
 2 2 7  are even.
 2
2  10 10
∴ ∫ πxdx 2 ∫ f(x)cos πxdx
f(x)cos=
−10 0
π /2
Sol 8: (C) ∫0 sinx dx
y
π   π 
−0  0 + 
2 π 2
=  sin0 + sin   + 2sin  
4  2  2 
   x
  
–10 –9 –2 –2 0 1 2 9 10
π
= (1 + 2)
8
3
= 10 ∫ f(x)cos πx dx
Sol 9: (A) F’(c) = (b – a) f’(c) + f(a) – f(b) 0

F’’(c) = f’’(c) (b – a) < 0 1


Now, ∫ f(x)cos πx dx
⇒ F’(c) = 0 0
f(b) − f(a)
⇒ f '(c) = 1 1
b−a ∫ (1 − x)cos πx dx =
= − ∫ ucos πudu and
0 0
t (t − a)
∫a f(x)dx − 2 {f(t) + f(a)} 2 2 1
Sol 10: (B) Given, lim =0 ∫ f(x)cos πx dx = ∫ (x − 1)cos πx dx = – ∫ ucos πudu
t →a (t − a)3 1 1 0

Using L’ Hospital’s rule 10 1


40
a +h h
∴ ∫ f(x)cos πx dx =
−20 ∫ ucos πudu =
π2
∫af(x)dx − {f(a + h) + f(a)}
2
−10 0
⇒ lim =0
h→0 h3 π2
10

10 −∫10
1 ⇒ f(x)cos πx dx =
4
f(a + h) − {f(a + h) + f(a)}
2
h x ln t
− {f '(a + h)} Sol 12: f(x) (given)
⇒ lim
2 =0
= ∫1 1 + t dt for x > 0
h→0 3h2
1/x lnt
Now, f(1 / x) = ∫1 dt
Again, using L’Hospital’s rule 1+t
Put t = 1/u
1 1
f '(a + h) − f '(a + h) − f '(a + h)
2 2 ⇒ dt = (–1/u2)du
h
− f ''(a + h) x ln(1 / u) ( −1)
2 =0 ∴ f(1 / x) = ∫1 1 + 1 / u . du
⇒ lim u2
h→0 6h
h x ln u x lnt
− f ''(a + h) = ∫1 u(u + 1) du = ∫1 t(1 + t) dt
⇒ lim 2 =0
h→0 6h
M a them a ti cs | 23.67

 1 x log t x log t
π /3 π dx
∴ I 2∫ +0
Now, f(x) + f =
 
x
∫1 (1 + t) dt + ∫1 (1 + t) =
dt 0  π
2 − cos  | x | + 
 3
x (1 + t)log t x xlog t  
= ∫1 dt + ∫1 dt
t(1 + t) t  3 
 x dx
is odd
  π 
1 x 1  2 − cos  | x | +  
= (logt)2  = (logx)2 3
2 1 2   
Put x = e π /3 dx
I = 2π ∫
11  11 11
0 2 − cos(x + π / 3)
2
∴ f(e)
f(e) ++ ff = = (lne)
 (lne)2==2
(loge)
 ee  22 22
Put x +
π
= t ⇒ dx = dt
3
Hence proved. t
sec2 dt
2 π /3 dt 2 π /3 2
∴ I = 2π ∫π /3 = 2π ∫
2 − cos t π /3 t
Sol 13: Let I =
π /2
f(cos2x)cos x dx  … (i) 1 + 3tan2
∫0 2
t t
π /2  π  π  Put tan = u ⇒ sec2 dt = 2du
2 2
=I ∫0 f  cos2  − x   cos  − x  dx
  2   2 
3 2du 4π
using a f(x)dx a  ⇒ I = 2π ∫ = [ 3 tan−1 3u] 13
 ∫0 = ∫0 f(a − x)dx  1/ 3
1 + 3u 2 3
3
π /2
I= ∫0 f(cos2x)sinx dx  … (ii) 4π 4π 1
= (tan−1 3 − tan−1 1) = tan−1  
3 3 2
On adding Eqs. (i) and (ii), we get
π /2 π /3 π + 4x3 4π 1
2I ∫0 f(cos2x)(sinx + cos x)dx ∴∫ dx = tan−1  
−π /3  π
2 − cos  | x | + 
 3 2
π /2  3
2∫ f(cos2x)[cos(x − π / 4)]dx
0

π Sol 15: Let


Put –x + = t ⇒ –dx = dt
4 π |cos x |  1  1 
=I ∫0 e  2sin  cos x  + 3cos  cos x   sinx dx
 2  2 
−π / 4  π 
∴ 2I=– 2 ∫ f  cos  − 2t   cos t dt
π/ 4
 2  π |cos x | 1 
⇒ I= ∫0 e .sinx.2sin  cos x  dx
 2 
π/ 4
∴ 2I = 2∫ f(sin2t)cot t dt
−π / 4
π 1 
π/ 4
+ ∫ e|cos x| .3cos  cos x  .sinx dx … (i)
∴ I = 2∫ f(sin2t)cos t dt
0
2  
0
⇒ I = I1 + I2
Sol 14:  using 2a f(x)dx 
 ∫0 
π /3 π dx π /3 x3 dx  
Let I ∫−π /3 +4 ∫−π /3  0, f(2a − x) =−f(x) 

 π  π =  a 
2 − cos  | x | +  2 − cos  | x | +   2∫ f(x)dx, f(2a − x) = +f(x)  
 3  3   0 

0, f( −x) =−f(x) where I1 = 0 [ f(π – x) = –f(x)] …(ii)
a 
Using ∫−a f(x)dx =  a
2∫0 f(x)dx, f( −x) =f(x) and
π /2 cos x 1 
I2 = 6 ∫ e .sinx.cos  cos x  dx
0 2  
2 3 . 6 8 | Definite Integration

1 t A circle C of radius 1 is inscribed in an equilateral triangle


Now, I2 = 6 ∫ et .cos   dt PQR. The points of contact of C with the sides PQ, QR,
0
2
RP are D, E, F, respectively. The line PQ is given by the
(Put cos x = t ⇒ – sin x dx = dt)
3 3 3
 t 1 t 
1
equation 3x + y − 6 = 0 and the point D is  2 , 2 
= 6 et cos   + ∫ et sin dt   
 2
  2 2 0 Further, it is given that the origin and the centre of C are
1
  t  1 t et t  on the same side of the line PQ.
= 6 et cos   +  et sin − ∫ cos dt  
 2 2  2 2 2  
0 x
t2
1 Sol 19: (A) Let f ( x=
) ∫ 1 + t 4 dt= 2x − 1
 t 1 t I
=6 et cos + et sin  − 2 0

 2 2 2 0 4 4
x2 1 + x 4 − 2 ⇒ −2x + x − 2 < ∀x ∈ R
f (x) =
x4 + 1
24  1 e 1 
=  ecos   + sin   − 1   … (iii)
f ( 0 ) > 0, f (1 ) < 0
5  2 2 2 
From Eqs. (i), we get ∴ One solution in (0, 1)

24  1 e 1  x
=I  ecos   + sin   − 1  Sol 20: f ( x=
) ∫ f ( t ) dt ⇒ f ( 0=) 0
5  2 2 2  0

f ( x ) f ( x ) , x > 0 ⇒ f ( x ) = kex , x > 0


Also=
1
Sol 16: Let=
I2 ∫0 (1 − x
50 101
) dx,
 f ( 0 ) = 0 and f ( x ) is continuous
using integration by parts
1
⇒ f ( x ) = 0∀x > 0
= (1 − x50 )101 . x  ∴ f ( In5 ) =
0
 0
1
+ ∫ (1 − x50 )100 50. x 49 . x dx
0 Sol 21: (C) f ' =± 1 − f2
1
= 0 − ∫ (50)(101)(1 − x50 )100 ( − x50 )dx ⇒ f (x) =
sinx or f ( x ) =
− sinx (not possible)
0
⇒ f (x) =
sinx
1 50 101 1
= –50(101) ∫0 (1 − x ) dx +(50)(101) ∫0 (1 − x
50 100
) dx Also x > sinx∀ > 0 .

= 5050I2 + 5050I1
Sol 22: A → p, q, s; B → p, t; C → p, q, r, t; D → s
∴ I2 + 5050I2 = 5050I1
dy
(A) ( x − 3 )
2
+y =0
(5050)I1 dx
∴ = 5051
I2 dx dy 1
∫ = ∫ y
⇒ =In y + c
( x − 3)
2 x −3

Sol 17: (B) g ( x ) =


( )
f ' ex ex
So domain is R − {3} .
1 + e2x
Hence positive for ( 0,∞ ) and negative for ( −∞ ,0 ) (B)
x= Put
t + 3x = t + 3
2 2
Consider the line
∫ ( t + 2 )( t + 1 ) t ( t − 1 )( t − 2 ) dt = ∫ t ( t − 1 )( t − 4 ) dt =
2 2
0
x +1 y + 2 z +1 x −2 y +2 z −3
L1 : = = ,L2 : = = 5  1
2
3 1 2 1 2 3 (C) f ( x ) =−  sinx − 
4  2
1 1
Sol 18: (D) Hence ∫ g' ( x =) g (1 ) − g ( −1=) 2g (1 ) Maximum value occurs when sinx =
2
−1
(D) f ( x ) > 0 if cos > sinx
M a them a ti cs | 23.69

x y z Sol 25: (A)


Sol 23: (A) Let the line be = = intersects the lines
( ) ( )
  2 2
 
a b c x 4  1 + x2
− 4x 1 + x2 + 4x2 
4  1 + x2  −2x 
x 4 (1 − x )
1
1 1   1
x 
⇒ S.D = 0 ⇒ a + 3b + 5c = 0 and ∫ 1 + x2
= ∫= =∫  
2 2
0 0 1+x
1+x 0

( )
2
4 
2
3a + b − 5c = 0 ⇒ a : b : c :: 5r : −5r : 2r  2   2
1 x  1+x −4
4  1 + x  −2x 
( )
1
1 x4 1 − x 1  
x  

on solving with given lines we get points of intersection =∫ ∫= ∫
2
0 1 + x 0 1 + x2 0 1+
 10 10 8 
P ≡ (5, −5,2 ) and Q ≡  , − ,  ⇒ PQ2 =d2 =6 1
 3 3 3  4x2 
(B) (p, r)
= ∫x
42
(
 1 + x − 4x +

)
1 + x2 
dx
0
t an−1 ( x + 3) − tan−1 ( x − 3) =
sin−1 ( 3 / 5 )  4x6 
= ∫  x6 + x 4 − 4x5 +  dx
⇒ tan−1
( x + 3) − ( x − 3) = tan−1
3

6
=
3  1 + x2 

(
1+ x −92
) 4 x −8 4
2
Now on polynomial division of x6 by 1 + x2 , we obtain
∴ x2 − 8 =8
Or x = ±4
 6 
+ x 4 − 4x5 + 4  x 4 − x2 + 1 −
∫ x 
1 
(

1 + x2  
)
(C) (q, s)
    2


(
= ∫  x6 − 4x5 + 5x 4 − 4x2 + 4 −
dx
4 
 dx
1 + x2 
)

( )
As a = µ b + 4 c ⇒ µ b =− 4 b. c and b = 4 a. c
 
 x7 4x6 5.x5 4x3 
2     = − + − + 4x  01−4 tan− x x 
and b + b. c − d. c =  
0  7 6 5 3 
   
Again, as 2 b + c = b − a 1 4 4   π 1  22
  =  − + 1 + 4  − 4   =  + 3 − =
π −π
2
Solving and eliminating b. c and eliminating a 7 6 3  4 7  7

2
( )
We get 2 µ2 − 10µ b = 0 ⇒ µ = 0 and 5. Sol 26: (A) x2 =
t ⇒ 2xdx =
dt

x / 2( ) )dx ( x / 2)dx In3


x sin9 x / 2
2π sin9sin9
π 1 sint
(D) 2=
x 2
I sin9 ∫ ( 2= × 2∫ ( x / 2 ) I= ∫
2 In2 sint + sin ( In6 − t )
dt
π −∫x sin π ∫0 sin0( xsin
π − x sin ( x /dx2=)
/ 2( )
=I × 2π / 2)
x dx
( x / 2)
x / 2 = θ ⇒ dx = 2bθ
x / 2 = θ ⇒ dx = 2bθ 1
In3 sin ( In6 − t )
x = πθ = π / 2 and I = ∫ dt
x = πθ = π / 2 2 In2 sint ( In6 − t ) + sint
π /2
8 8sin9sin9
π /2
θ (θsin7
θ − sin7
θ − sin7
( sin7θ − sin5θ )
+ θ − sin5θ )

π ∫0 π 0 sin θsin θ
+ x2 =
sin θsin θ t ⇒ 2xdx =
dt

( sin5( sin5 θ ) θ()sin3


( sin3θ − sin θ ) sin θ d16 In3
θ − sin3 1 1 3
+ +
θ − sin3
+ + θ − sin θ ) + sin+θ dθ = θ =2I ∫ 1dt=
⇒I In
2 In2 4 2
sin θsin θ sin θsin θ sin sin π
π /2 π /2
8
∫ ( cos8θ + cos6θ + cos 4θ + cos2θ + 1) + π ∫ dθ Sol 27: (B)
0 0
 2  π + x 
16  sin8θ sin6θ sin 4θ sin2θ   x + n    cos xdx
=  + + +  
π /2
 π − x 
π  8 6 4 2  ∫ π /2  π+x 
8 π /2 8 π  =−π /2
2 ∫ x2 cos xdx + 0  n   is an odd function 
+ θ  = 0 ×  − 0 = 4 0   π − x  
π π 2 
 π /2 π /2   2  π /2

xn (1 + x ) 1 1 1  (
= 2  x2 sinx ) ∫  2  π − 0  − 4 x sinxdx
2x sinxdx=
  4




Sol 24: (B) lim = lim ×=  0 0  0
x →0
( 4
x + 4 × 3x) 2 x → 0 4 3 12
π2  π /2  π2
− 4 ( −x cos x )
π /2
= + ∫ cos xdx=
 −4
2  0
0  2
2 3 . 7 0 | Definite Integration

π
1  1 1 
( ) ( ) 
2 
( −2x )
4 5
− ∫ 2x (1 − x ) dx 
5
Sol 28: (A) ∫ ( 2cosecx ) dx
17 =  4x3 × 5 1 − x2 − 12   x2 1 − x2
 0  
π
 0 0 
1
( )
4
= 0 − 0 − 12 0 − 0  + 12∫ 2x 1 − x2 dx
Let 0

(( ))
π π 1
2cosecx,x,xx= = π ⇒ =InIn11++ 22 , x, x= = π ⇒
( )
n −n
een ++ee−n = =2cosec ⇒uu=log ⇒uu= =00  2
6 
44 22  1−x   1
= 12 ×  −  = 12 0 + = 2
 6   6
en and
⇒ cosec x + cot x =   0
u −u
e −e
x − cot x =e−n ⇒ cot x =
2 1
−1   12 + 9x 2 

(e u
) dx =
−e −u
−2cosecx cot xdx Sol 31: α =∫  e9x +3 tan x  
0    1 + x2 

 dx

⇒ −∫ (e + e ) u
(e − e ) du−u
17
u −u
Put 9x + 3tan−1 x =

t ⇒ 9 +
3 
 dx =
1 + x2 
dt

2cosecx cot x
4 3π
t
9+  3π 
0 (
log 1+ 2 ) ⇒
= α ∫ e= dt e 4 − 1 ⇒  loge 1 + α −  = 9
4 
−2
= ∫ (e
u
)
+ e−u du = u u
∫ 2(e + e )du
0 

(
log 1+ 2 )
((sin at + cos at ))dt =
π
t 6 4
Sol 32: (A, C) Let ∫e A
0
1 2π
Sol 29: (D) (p) f ( x ) =
ax + bx, ∫ f ( x ) dx =
1 =I
2

0
∫e
t
(sin at + cos at ) dt
6 4

⇒ 2a + 3b =
6 Put t = π + x
dt = dx
⇒ ( a,b ) ≡ ( 0,2 ) and ( 3,0 )
For a = 2 as well as a = 4
π

(q) f ( x )
=
 π
2 cos  x2 − 
= (
I ex ∫ ex sin6 ax + cos4 at dt= e2π A ⇒ I = eπ A )
 4 0
π

π π Similarly ∫e e
π π
(sin6 at + cos4 at)dt =
e2π A
x2 − = 2nπ ⇒ x2= 2nπ + 0
4 4
A + e A + e2π A + e3π A e4 π − 1
π
π 9π =L = ∴ For both a = 2, 4
⇒ x =± ,± as x ∈  − 3, 13  A eπ − 1
4 4  

(r)
2
3x2 3x2  2
2 ( x ) F ( x ) + xF'(( x )
Sol 33: (A, B, C) (A) f '=
∫  1 + ex 1 + e− x
 +
 ∫ 3x dx =
dx = 8
0  0
(1) F (1) + F' (1)
f=
1/2
1 + x 
(s) ∫ cos2x In   dx = 0 as it is an odd function (1 ) F (1 ) < 0
f '=
−1/2 1 − x 
(B) f ( 2 ) = 2F ( 2 )
1
d2 F ( x ) is decreasing and F(1) = 0
Sol 30: (2) ∫ 4x
0
3

dx2
(1 − x )dx 2

Hence F(2) < 0

⇒ f ' (2) < 0


51 1

=  4x3

d
dx
(
1 − x2  − ∫ 12x2
0
 0
d
dx
1 − x2 dx ) ( )
( x ) F ( x ) + xF' ( x )
(C) f=

F ( x ) < 0∀x ∈ (1,0 )


M a them a ti cs | 23.71

F' ( x ) < 0∀x ∈ (1,3)


Hence f ( x ) < 0∀x ∈ (1,3)

0 1 2
x.0 x.0 x.1 1
Sol=
34: I ∫ 2 + 0 dx + ∫ 2 + 1 dx + ∫ 2 + 0=
dx + 0
−1 0 1
4
⇒ 4I − 1 =0

π /2
x2 cos x
Sol 35: (A) I = ∫ dx  … (i)
−π /2 1 + ex

π /2
x2 cos x
I= ∫ 1
dx  … (ii)
−π /2 1 +
ex
π /2
x2 cos x.ex
= ∫ 1 + ex
dx
−π /2

(i) and (ii)


π /2
2I ∫ x2 cos x dx
−π /2

π /2
I= ∫ x2 cos x dx (even fn)
0
π /2
= x2 .sin x π /2
0 − ∫ 2x sin x dx
0

π2  π /2 
− 2 ( −x cos x ) − ∫ ( − cos x ) dx 
π /2
=
4  0
0 
π2 π2 π2
= − 2 0 + sin x π /2 
0   = − 2 1 = − 2
4  4 4
2017-18 100 &
op kers
Class 12 T
By E ran culty
-JE Fa r
IIT enior emie .
S fP r es
o titut
Ins

MATHEMATICS
FOR JEE MAIN & ADVANCED
SECOND
EDITION

Exhaustive Theory
(Now Revised)

Formula Sheet
9000+ Problems
based on latest JEE pattern

2500 + 1000 (New) Problems


of previous 35 years of
AIEEE (JEE Main) and IIT-JEE (JEE Adv)

5000+Illustrations and Solved Examples


Detailed Solutions
of all problems available

Topic Covered Plancess Concepts


Tips & Tricks, Facts, Notes, Misconceptions,
Differential Equations Key Take Aways, Problem Solving Tactics

PlancEssential
Questions recommended for revision
24. DIFFERENTIAL
E Q U AT I O N S

1. INTRODUCTION
An equation containing an independent variable, dependent variable and differential coefficients is called a
differential equation.
2 3 3
dy  d2 y   dy   d4 y  dy
(i) = sin x (ii)   + x  = 0 (iii)  4  −4 5 cos 3 x
=
dx  dx2   dx   dx  dx
   

2. ORDER OF DIFFERENTIAL EQUATION


The order of a differential equation is the order of the highest derivative occurring in the differential equation. For
example, the order of the above mentioned differential equations are 1, 2, and 4 respectively.

3. DEGREE OF DIFFERENTIAL EQUATION


The degree of a differential equation is the degree of the highest order derivative when differential coefficients
are free from radicals and fractions. For example the degrees of above differential equations are 1, 2, and 3
respectively.

Table 24.1: Degree of differential equation

Differential Equation Order of D.E. Degree of D.E

dy
+ 4y = sin x 1 1
dx
4
 d2 y   dy 5
 2  +  − y = ex 2 4
 dx   dx 
 

d2 y dy
− + 3y =
cos x 2 1
dx 2 dx

dy x4 − y 4
= 1 1
dx xy x2 + y 2 ( )
2 4 . 2 | Differential Equations

Differential Equation Order of D.E. Degree of D.E


2
dy  dy 
y =+
x a2   + b2
dx  dx 
2 1 2
 dy  dy
⇒ (x – a ) 
2 2
 –2xy +(y2 – b2)=0
dx
  dx

3/2 2 3
  dy  
2
d y   dy    d2 y 
2 2
= 1+   ⇒ 
 − 1+  0
= 2 2
 dx2    dx  
dx2   dx     

4. CLASSIFICATION OF DIFFERENTIAL EQUATIONS


Differential equations are first classified according to their order. First-order differential equations are those in
which only the first order derivative, and no higher order derivatives appear. Differential equations of order two or
more are referred to as higher order differential equations.
A differential equation is said to be linear if the unknown function, together with all of its derivatives, appears in
the differential equations with a power not greater than one and not as products either. A nonlinear differential
equation is a differential equation which is not linear.
e.g. y’ + y = 0 is a linear differential equation,
y” + yy’ + y2 = 0 is a non linear differential equation,

Procedure to form a differential equation that represents a given family of curves


Case I:
If the given family F1 of curves depends on only one parameter then it is represented by an equation of the form
F1(x, y, a) = 0  … (i)
For example, the family of parabolas y2 = ax can be represented by an equation of the form
f(x, y, a): y2 = ax
Differentiating equation (i) with respect to x, we get an equation involving y’, y, x and a.
g(x, y, y’, a) = 0  … (ii)
The required differential equation is then obtained by eliminating a from equation (i) and (ii) as
F(x, y, y’) = 0  … (iii)

Case II:
If the given family F2 of curves depends on the parameters a, b (say) then it is represented by an equation of the
form F2(x, y, a, b) = 0  … (iv)
Differentiating equation (iv) with respect to x, we get an equation involving y’, x, y, a, b.
g(x, y, y’, a, b) = 0  … (v)
Now we need another equation to eliminate both a and b. This equation is obtained by differentiating equation (v),
wrt x, to obtain a relation of the form h(x, y, y’, y”, a, b) = 0  … (vi)
The required differential equation is then obtained by elimination a and b from equations (iv), (v) and (vi) as F(x, y,
y’, y”) = 0  … (vii)
M a them a ti cs | 24.3

Note: The order of a differential equation representing a family of curves is the same as the number of arbitrary
constants present in the equation corresponding to the family of curves.

5. FORMATION OF DIFFERENTIAL EQUATIONS


If an equation is dependent and dependent variables having some arbitrary constant are given, then the differential
equation is obtained as follows:
(a) Differentiate the given equation w.r.t. the independent variable (say x) as many times as the number of
arbitrary constants in it.
(b) Eliminate the arbitrary constants.
dy d2 y
(c) Hence on eliminating arbitrary constants results a differential equation which involves x, y, , …….
m dx dx2
d y
(where m=number of arbitrary constants).
dxm

Illustration 1: Form the differential equation corresponding to y2 = m(a2 – x2), where m and a are arbitrary
constants.  (JEE MAIN)

Sol: Since the given equation contains two arbitrary constant, we shall differentiate it two times with respect to x
and we get a differential equation of second order.
We are given that y2 = m(a2 – x2) … (i)
Differentiating both sides of (i) w.r.t. x, we get
dy dy
2y = m(–2x) ⇒ y = –mx  … (ii)
dx dx
2
d2 y  dy 
Differentiating both sides of (ii) w.r.t. to x, we get y +   = –m … (iii)
 dx 
2
dx
 d2 y  dy 2  dy
From (ii) and (iii), we get, x  y +   = y
 dx   dx
 dx 2

This is the required differential equation.

Illustration 2: Form diff. equation of ax2 + by2 = 1  (JEE MAIN)

Sol: Similar to the above problem the given equation contains two arbitrary constants, so we shall differentiate it
two times with respect to x and then by eliminating a and b we get the differential equation of second order.

dy
ax2 + by2 = 1 ⇒ 2ax + 2by =0 ⇒ a + b (yy” + (y’)2) = 0
dx
2
y ⇒ d2 y  dy  y dy
Eliminating a and b we get y’ = yy” + (y’)2 y +   – =0
x dx2  dx  x dx

Illustration 3: Form the differential equation corresponding to y2 = a(b2 – x2), where a and b are arbitrary
constants.  (JEE MAIN)

Sol: Similar to illustration 1.


We have, y2 = a(b2 – x2) … (i)
2 4 . 4 | Differential Equations

In this equation, there are two arbitrary constants a, b, so we have to differentiate twice, Differentiating the given
dy dy
equation (i) w.r.t. ‘x’. We get 2y = –2x.a ⇒ y = –ax  … (ii)
dx dx
2
d2 y dy dy d2 y  dy 
Differentiating (ii) with respect to x, we get y + . −a ⇒ y
= +  = −a  … (iii)
dx2 dx dx dx2  dx 
Substituting the value of a in (ii), we get
2 2
dy  d2 y  dy   dy d2 y  dy  d2 y  dy  dy
y
=  y +    2 x ⇒ y
= xy +   ⇒ xy + x  − y 0
=
dx  dx
2
 dx   dx dx2  dx  dx 2
 dx  dx

Illustration 4: Find the differential equation of the following family of curves: xy = Aex + Be–x + x2  (JEE MAIN)

Sol: Here in this problem A and B are the two arbitrary constants, hence we shall differentiate it two times with
respect to x and then by eliminating constant terms we will get the required differential equation.
Given: xy = Aex + Be–x + x2  … (i)
dy
Differentiating (i) with respect to ‘x’, we get x + y = Aex – Be–x + 2x
dx
Again differentiating with respect to ‘x’, we get

d2 y dy dy d2 y dy
x +1 + 1. = Aex + Be–x + 2 ⇒ x +2 = xy – x2 + 2
dx2 dx dx dx 2 dx

Illustration 5: Prove that x2 – y2 = c(x2 + y2)2 is a general solution of the differential equation
(x3 – 3xy2)dx = (y3 – 3x2y)dy  (JEE ADVANCED)

Sol: Here only one arbitrary constant is present hence we shall differentiate it one time with respect to x and then
by substituting the value of c we shall prove the given equation.
Let us find the differential equation for x2 – y2 = c(x2 + y2)2
... (i)

Differentiating (i), with respect to ‘x’, we get 2x – 2y


dy
dx
( 

)
=c.2 x2 + y 2  2x + 2y
dy 
 
dx 
... (ii)

Substituting the value of c from (i) in (ii), we get

x2 − y 2
⇒ x=
−y
dy
(x 2
) 
+ y 2  2x + 2y
dy 
 ⇒ (x + y )
2 2

x − y
dy 
 = (x – y )
2 2

 2x + 2y
dy 

dx
(x ) dx dx dx 
2
2
+ y2     
dy dy
⇒ [2y(x2 – y2) + y(x2 + y2)] = x(x2 + y2) – 2x(x2 – y2) ⇒ (3x2y – y3) = 3xy2 – x3
dx dx
⇒ (x3 – 3xy2)dx = (y3 – 3x2y)dy As this equation matches the one given in the problem statement. Hence the given
equation is the solution for the differential equation.
Hence proved.

Illustration 6: Find the differential equation of the family of curves y = ex(acosx + bsinx) (JEE ADVANCED)

Sol: Since given family of curves have two constants a and b, so we have to differentiate twice with respect to x.
We have, y = ex(acosx + bsinx)  … (i)
M a them a ti cs | 24.5
Differentiating (i) with respect to x, we get
dy
=ex(acosx + bsinx) + ex(–asinx + bcosx) = y + ex (–asinx + bcosx)
dx
dy
⇒ – y = ex(–asinx + bcosx) … (ii)
dx
Differentiating (ii) with respect to x, we get
d2 y dy dy
– = ex(–asinx + bcosx) + ex(–acosx – bsinx) = – y – ex (acos x + bsinx)
dx 2 dx dx
d2 y dy dy ∴ d2 y dy
⇒ − = – y – y [ ex(a cosx + bsinx) = y] ⇒ −2 + 2y = 0
dx 2 dx dx dx 2 dx

This is the required differential equation.

Illustration 7: Find the differential equation of all circles which pass through the origin and whose centers lie on
the y axis.  (JEE ADVANCED)

Sol: As circles passes through the origin and whose centers lie on the y axis hence g = 0 and point (0, 0) will satisfy
general equation of given circle.
The general equation of a circle is
x2 + y2 + 2gx + 2fy + c = 0  … (i)
Since it passes through origin (0, 0), it will satisfy equation (i)
⇒ (0)2 + (0)2 + 2g.(0) + 2f.(0) + c = 0 ⇒ c=0
⇒ x2 + y2 + 2gx + 2fy = 0
This is the equation of a circle with center (–g, –f) and passing through the origin.
If the center lies on the y-axis, we have g = 0,
⇒ x2 + y2 + 2.(0).x + 2fy = 0 ⇒ x2 + y2 + 2fy = 0 … (ii)
Hence, (ii) represents the required family of circles with center on y axis and passing through origin.
Differentiating (ii) with respect to x, we get
  dy  
 x + y.   
2x + 2y
dy
+ 2f
dy 
= 0 ⇒ f = –   dx  

dx dx   dy  
  dx  

Substituting this value of f in (2), we get


  dy  
 x + y.   
x2 + y2 – 2y   dx   = 0 ⇒ (x2 + y2) dy – 2xy – 22  dy  = 0 ⇒ (x2 – y2) dy – 2xy = 0
2y  
  dy   dx dx
     dx 
dx
   
This is the required differential equation.

PLANCESS CONCEPTS

Curves representing the solution of a differential equation are called integral curves.
Nitish Jhawar (JEE 2009, AIR 7)
2 4 . 6 | Differential Equations

6. SOLUTIONS OF DIFFERENTIAL EQUATIONS


Finding the dependent variable from the differential equation is called solving or integrating it. The solution or the
integral of a differential equation is, therefore, a relation between the dependent and independent variables (free
from derivatives) such that it satisfies the given differential equation.
Note: The solution of the differential equation is also called its primitive.
There can be two types of solution to a differential equation:
(a) General solution (or complete integral or complete primitive)
A relation in x and y satisfying a given differential equation and involving exactly the same number of arbitrary
constants as the order of the differential equation.
(b) Particular solution
A solution obtained by assigning values to one or more than one arbitrary constant of general solution

dy
Illustration 8: The general solution of x2 dx = 2 is  (JEE MAIN)

Sol: First separate out x term and y term and then integrate it, we shall obtain result.
dy 2 2 2
= ⇒ dy = dx Now integrate it. We get y = – +c
dx x 2
x 2 x

Illustration 9: Verify that the function x + y = tan–1y is a solution of the differential equation y2y’ + y2 + 1 = 0
 (JEE MAIN)
Sol: By differentiating the equation x + y = tan–1y with respect to x we can prove the given equation.
We have, x + y = tan–1y ... (i)
Differentiating (i), w.r.t. x we get

dy 1 dy dy  1 + y 2 − 1 
1+ = ⇒ 1+   =0
dx 1 + y 2 dx dx  1 + y 2 
dy
⇒ (1 + y2) + y2 dx = 0 ⇒ y2y’ + y2 + 1 = 0

 dy 
Illustration 10: Show that the function y = Ax +  2x + 2y  is a solution of the differential equation
 dx 
2
d y dy
x2 2 + x dx – y = 0  (JEE MAIN)
dx
B
Sol: Differentiating y = Ax + twice with respect to x and eliminating the constant term, we can prove the given
equation. x

dy
We have, y = Ax + ⇒ xy = Ax2 + B  ... (i)
dx
dy
Differentiation (i) w.r.t. ‘x’. we get ⇒ x + 1.y = 2Ax ... (ii)
dx
Again differentiating (ii) w.r.t., ‘x’, we get
dy
x +y
d2 y dy dy d2 y dy dx d2 y dy
⇒ x. + + = 2A ⇒ x +2 = ⇒ x2 +x −y =0
dx 2 dx dx dx 2 dx x dx 2 dx

dy
Which is same as the given differential equation. Therefore y = Ax + is a solution for the given differential
equation. dx
M a them a ti cs | 24.7

dy
x2 + 1 log  x2 + 1  show that (x2 + 1)
Illustration 11: If y. = + xy + 1 = 0 (JEE MAIN)
  dx

x2 + 1 log  x2 + 1 − x  one time with respect to x, we


Sol: Similar to the problem above, by differentiating y. =
 
will prove the given equation.

We have, y. 2
x= + 1 log  x2 + 1   ... (i)
 
Differentiating (i), we get

(1 / 2)  2 x/ x2 + 1  – 1
dy 1 2x dy x x − x2 + 1
x2 +1 + y=  +  ⇒ = x2 + 1 ;
dx 2 2 dx x2 + 1 x2 + 1  x2 + 1 − x 
2
x +1 x +1 − x
 
2
dy x − x +1 dy dy
(x2 +1) + xy = ; (x2 + 1) + xy = –1; (x2 + 1) + xy + 1 = 0
dx x2 + 1 – x dx dx

Illustration 12: Show that y = acos(logx) + bsin(logx) is a solution of the differential equation:
d2 y dy
x2 2 + x dx + y = 0  (JEE ADVANCED)
dx

Sol: As the given equation has two arbitrary constants, hence differentiating it two times we can prove it.
We have, y = acos(logx) + bsin(logx) ... (i)
dy asin(logx) bcos(logx)
Differentiating (i) w.r.t ‘x’. we
= get ; – +
dx x x
dy
x = –asin(logx)+bcos(logx)  ... (ii)
dx
Again differentiating with respect to ‘x’, we get

d2 y dy acos (logx ) bsin (logx )


x = + −
dx2 dx x x
d2 y dy d2 y dy d2 y dy
⇒ x2 +x = –[acos(logx) + bsin(logx)] ⇒ +x = –y ⇒ +x +y=0
dx 2 dx dx 2 dx dx 2 dx
Which is same as the given differential equation
Hence, y = acos(logx) + bsin(logx) is a solution of the given differential equation.

7. METHODS OF SOLVING FIRST ORDER FIRST DEGREE DIFFERENTIAL


EQUATION

7.1 Equation of the Form dy/dx = f(x)


To solve this type of differential equations, we integrate both sides to obtain the general solution as discussed
below
dy
= f (x) ⇒ dy = f(x)dx
dx
Integrating both sides we obtain
= ∫ dy ∫ f ( x ) dx + c ⇒ y= ∫ f(x)dx + c
2 4 . 8 | Differential Equations

dy 2
Illustration 13: The general solution of the differential equation = x5 + x2 – is  (JEE MAIN)
dx x

Sol: General solution of any differential equation is obtained by integrating it hence for given equation we have to
integrate it one time to obtain its general equation.
dy 2
We have: = x5 + x2 –
dx x
 2 1 x 6 x3
Integrating, y = ∫  x5 + x2 –  dx + c = ∫ x5
dx + ∫ x 2
dx − 2 ∫x dx + c
C ⇒ y = + – 2log|x| + c
 x 6 3
Which is the required general solution.

d2 y
Illustration 14: The solution of the differential equation cos2x = 1 is  (JEE MAIN)
dx2

Sol: By integrating it two times we will get the result.


d2 y d2 y
cos2x = 1 ⇒ = sec2x
2 2
dx dx
dy
On integrating, we get = tanx + c1
dx
Integrating again, we get y = log(secx) + c1x + c2

7.2 Equation of the form dy/dx = f(x) g(y)


To solve this type of differential equation we integrate both sides to obtain the general solution as discussed below
dy
=f(x)g(y) ⇒ g(y)−1 dy =f(x)dx
dx
Integrating both sides, we get ∫ (g(y))−1 dy = ∫ f(x)dx

Illustration 15: The solution of the differential equation log(dy/dx) = ax + by is  (JEE MAIN)
dy
Sol: We can also write the given equation as = eax + by . After that by separating the x and y terms and integrating
dx
both sides we can get the general equation.
dy dy 1 1
= eax + by ⇒ =eax + by ⇒ e–bydy = eax dx ⇒ − e−by = eax + c
dx dx b a
dy
Illustration 16: The solution of the differential equation =ex+ y + x2ey is  (JEE MAIN)
dx

Sol: Here first we have to separate the x and y terms and then by integrating them we can solve the problem above.
dy
The given equation is = ex+y + x2ey
dx


dy
dx
= ex.ey + x2ey ⇒ e–ydy = (ex + x2)dx, Integrating, ∫e
−y
( )
dy =∫ ex + x2 dx + c

e− y x3 1 1 1 x3
⇒ + ex + + c ⇒ − = ex + x3 + c ⇒ ex + + =C
−1 3 ey 3 ey 3
M a them a ti cs | 24.9

7.3 Equation of the Form dy/dx = f (ax+by+c)


dy 1  dy 
To solve this type of differential equation, we put ax + by + c = v and
=  − 0
dx b  dx 
dy
\ = dx
a + bf ( v )
dy
So solution is by integrating ∫
a + bf ( v ) ∫
= dx

dy 2
Illustration 17: (x + y)2 dx = a  (JEE MAIN)

dy dt
Sol: Here we can’t separate the x and y terms, therefore put x + y = t hence = − 1 . Now we can easily
separate the terms and by integrating we will get the required result. dx dx

 dt  dt a2 a2 + t2 t2dt
Let x + y = t ⇒ t2  dx − 1  = a2 ; = +1 = ⇒ ∫ =x+c
  dx t2 t2 t2 + a2
dy x+y
⇒ t – a tan–1 dx = x + c ⇒ y − atan−1 =c
a

dx x + y −1
Illustration 18: =  (JEE MAIN)
dx x + y +1
Sol: Put x + y +1 = t2 and then solve similar to the above illustration.
let x + y +1 = t2
 dt  t2 − 2 2tdt t2 + t − 2 2t2
⇒  2t − 1 = ⇒ = ⇒ ∫ (t− 1)(t+ 2) dt= x+c
 dx  t dx t

 1 4  2ln | t − 1 | 8ln | t + 2 |
⇒ 2∫  1 + − x +c
 dt = ⇒ 2t + − x+c
=
 3(t − 1) 3(t + 2)  3 3

2ln | x + y + 1 − 1 | 8ln | x + y + 1 + 2 |
⇒ 2 x + y +1 + − = x+c
3 3

dy
Illustration 19: = cos(10x + 8y). Find curve passing through origin in the form y = f(x) satisfying differential
dx
equations given  (JEE MAIN)

Sol: Here first put 10x + 8y = t and then taking integration on both sides we will get the required result.
Let 10x + 8y = t
dy dt dy dt
⇒ 10 + 8 = ⇒ – 10 = 8cost ⇒ ∫ 8 cos t + 10 ∫ dx =x+c
dx dx dx
dp 1 + p2 dy dt 2dp
=p tant / 2 = ⇒=
dx 2(1) dx dx 1 + p2
2dp
1 + p2 2dp dp
∴∫ + 10 = ∫ =∫ =x+c
 1 − p2  1p2 + 18 p2 + 9
8
 1 + p2   tan(t / 2) 
⇒ tan−1(P/ 3) =x + c ⇒ tan−1 
   =x + c
 3 
 tan(t / 2) 
⇒ tan−1(P/ 3) =x + c ⇒ tan−1   =x + c ⇒ 3tan(x =
+ c) tan(10x + 84)
 3 
⇒ 3tan(x =
+ c) tan(10x + 84)
2 4 . 1 0 | Differential Equations

7.4 Parametric Form


Some differential equations can be solved using parametric forms.
Case I:
x = rcosθ y = rsinθ
Squaring and adding x2 + y2 = r2  ... (i)
−y
tanθ = ∫e dy =∫ (ex + x2 )dx + c  ... (ii)
xdx + ydy = rdr  ... (iii)
e− y x3
sec2θ dθ = = ex + + c ⇒ xdy – ydx = x2sec2θ dθ x = rcosθ; xdy – ydx = r2dθ
–1 3
Case II:
If x = rsecθ, y = rtanθ
x – y = r 
2 2 2
... (i)
1 1
=ex + x3 + c = sinq  ... (ii)
e 3
y

⇒ xdx – ydy = rdr; xdy – ydx = cosθ x2 dq ⇒ xdy – ydx = r2secqdq

Illustration 20: Solve xdx + ydy = x(xdy – ydx) (JEE MAIN)

Sol: By substituting x = r cosθ and y = r sinθ the given equation reduces to rdr = rcosθ(r2dθ). Hence by separating
and integrating both sides we will get the result.
Let x = rcosθ, y = rsinq
Hence the given equation becomes rdr = rcosθ(r2dθ)

dr 1 1 y
⇒ − = +c
∫ r=
2 ∫ cos θdθ − = sin θ + c
r

x + y2
2
x2 + y 2
dy
x+y 2 2
Illustration 21: Solve dx = 1 − x − y  (JEE ADVANCED)
dy x2 + y 2
x −y
dx
Sol: Similar to the problem above, by substituting x = r cosθ and y = r sinθ the given equation reduces to

r dr 1 − r2
= . Hence by integrating both sides we will get the result.
r 2 dθ r
dy
x+y 2 2
dx = 1 − x − y ⇒ xdx + ydy 1 − x2 − y 2
=
dy x2 + y 2 xdy − ydx x2 + y 2
x −y
dx
Let x = rcosθ, y = rsinq

rdr 1 − r2 dr
= ⇒ ∫ = θ + c ⇒ sin–1r = θ + c
r dθ2 r 1−r 2

y
⇒ sin−1=
x2 + y 2 sin−1 +c
x2 + y 2
M a them a ti cs | 24.11

xdx + ydy ydx − xdy


Illustration 22: =  (JEE ADVANCED)
2
x +y 2 x

Sol: Similar to the above illustration.


Let x = rcosθ, y = rsinq

rdr r2 dr
⇒ − = ⇒ ∫ sec θdθ + ∫ =0
2
r dθ r cos θ r
⇒ log(secθ + tanθ) + logr = c ⇒ x2 + y 2 + y  x2 + y 2  + Cx =
0
 

7.5 Homogeneous Differential Equations


A differential equation in x and y is said to be homogeneous if it can be y = f(x)
dy f ( x, y )
put in the form = , where f(x, y) and g(x, y) are both homogeneous P(x,y)
dx g ( x, y )
function of the same degree in x and y.
dy f ( x, y )
To solve the homogeneous differential equation = ,
dx g ( x, y )
dy dy
substitute y = vx and so = v+x
dx dx
dy dx dv
Thus differential reduces to the form v + x = f(v) ⇒ = x
dx x f(v) − v
dy dv
Therefore, solution=
is ∫ ∫ f (v) − v + c Figure 24.1
x

Illustration 23: Find the curve passing through (1, 0) such that the area bounded by the curve, x-axis and 2
ordinates, one of which is constant and other is variable, is equal to the ratio of the cube of variable ordinate to
variable abscissa. (JEE MAIN)
x
y3
Sol: By differentiating ∫ ydx = , we will get the differential equation.
c
x
x
y3 x,3y 2 y '− y 3 ,1 dy x2 + y 2
A = ∫ ydx = ⇒y= ⇒ x2 = 3xyy’ – y2 ⇒ =
c
x x2 dx 3xy
(On differentiating the first integral equation w.r.t x)
dt 1 + v2 3v 1 3
Put y = vx; v + x = ⇒ ∫ 1 − 2v 2 dv = ∫ x dx ⇒ − log 1 − 2v 2 = logx + logc ⇒ (x2 – 2y2)3 = cx2
dx 3v 4

Given this curve passes through (1, 0). So, c=1 Hence the equation of curve is (x2 – 2y2)3 = cx2

dy y y
Illustration 24: The solution of differential equation = + tan is  (JEE MAIN)
dx x x

Sol: Here by putting y = xv and then integrating both sides we can solve the problem.
dy dv
Put y = xv ⇒ = v+x
dx dx
dv dv
Hence the given equation becomes x + v = v + tanv ⇒ x =tanv
dx dx
y
sin  

dv
=
dx
⇒ log sinv = logx + logc ⇒
sin v
=c⇒  x  = c ⇒ cx = sin  y 
 
tan v x x x x
2 4 . 1 2 | Differential Equations

dy y 2 − 2xy − x2
Illustration 25: Solve = given y at x = 1 is –1 (JEE ADVANCED)
dx y 2 + 2xy − x2

Sol: Similar to the problem above, by putting y = vx, we can solve it and then by applying the given condition we
will get the value of c.
Let y = vx

dv  v 2 – 2v – 1  dv (v 3 + v 2 + v + 1)
⇒ v+x =   ⇒x = –
dx  v 2 + 2v – 1  dx v 2 + 2v – 1
 
v 2 + 2v − 1 2v(v + 1) – (v 2 + 1)
⇒ ∫ dv = c – logx ⇒ ∫ dv = c – logx
( v + 1) ( v2 + 1) (v + 1)(v 2 + 1)

⇒ log 
(
 v2 + 1 x 
)
 = logc ⇒
(v 2
−1 x ) =c⇒
x2 + y 2
=c
 v +1 
 
( v + 1) y+x

⇒ k(x2 + y2) = x + y
Given at x = 1, y = – 1 ⇒ 2k = 0. Hence the required equation is x + y = 0

2
 dy  dy
Illustration 26: Solve y   + 2x – y = 0 given y at x = 1 is 5 (JEE ADVANCED)
dx
  dx

−b ± b2 − 4ac dy −2x ± 4x2 + 4y 2


Sol: As we know, when ax2 + bx + c =0 then x = . Hence from given equation =
2a dx 2y
so by putting y = vx and integrating both side, we will get the result.

 dy  dy
Given y   + 2x –y=0
 dx  dx

dY −2x ± 4x2 + 4y 2 dy −x ± x2 + y 2
⇒ = ⇒ =
dX 2y dx y
Let y = vx

dv ± v 2 + 1 − 1 dv ± v2 + 1 − 1 − v2
⇒ x
= −v ⇒x =
dx v dx v

vdv vdv
⇒ ∫ = logx + C ⇒ ∫ = logx + C
± v + 1   v 2 + 1 + 1 

2 2 2
± v + 1 − (1 + v )
 

⇒ – ln   v 2 + 1 + 1  = logx + C ⇒ x   v 2 + 1 + 1  = c
   
dy 7X – 3Y
Given at x = 1, y = v = = ⇒ C=  6 + 1
dx –3X + 7Y

⇒  y 2 + x2 + =
x  6 +1

This is the required equation.


Note: The obtained solution has 4 equations.
M a them a ti cs | 24.13

7.6 Differential Equations Reducible to Homogenous Form


dv a1 x + b1 y + c1 a b
A differential equation of the form = , where 1 ≠ 1 can be reduced to homogeneous form by
dx a2 x + b2 y + c θ2 b2
adopting the following procedure
dy dy
Put x = X + h, y = Y + k, so that =
dx dx

dY a1 X + b1 Y + (a1h + b1k + c1 )
The equation then transforms to =
dX a2 X + b2 Y + (a2h + b2k + c2 )

Now choose h and k such that a1h + b1k + c1 = 0 and a2h + b2k + c2 = 0. Then for these values of h and k the
equation becomes

dy a1 X + b1 Y
=
dx a2 X + b2 Y
This is a homogeneous equation which can be solved by putting Y = vX and then Y and X should be replaced by
y – k and x – h.

dy ax + by + c a b
Special case: If = and = = m say, i.e. when coefficient of x and y in numerator and
dx a' x + b' y + c' a' b'
denominator are proportional, then the above equation cannot be solved by the method discussed before because
the values of h and k given by the equation will be indeterminate. In order to solve such equations, we proceed as
explained in the following example.

dy 3x − 6y + 7
Illustration 27: Solve =  (JEE MAIN)
dx x − 2y + 4

Sol: Here the coefficient of x and y in the numerator and denominator are proportional hence by taking 3 common
from 3x – 6y and putting x – 2y = v and after that by integrating we will get the result.

dy 3x − 6y + 7 3 ( x − 2y ) + 7 dy dy
= = ; Put x – 2y = v ⇒ 1 – 2 =
dx x − 2y + 4 x − 2y + 4 dx dx

dv  3v + 7 
Now differential equations reduces to 1 − 2
= 
dx  v+4 
dv  v+2  2 
⇒ = −5   ⇒ ∫  1 + v + 2  dv =
−5∫ dx
dx v+4

⇒ v + 2log v + 2 =−5x + c ⇒ 3x − y + log x − 2y + 2 =c

Illustration 28: Solution of differential equation (3y – 7x + 7)dx + (7y – 3x + 3) dy = 0 is (JEE MAIN)

Sol: By substituting x = X + h, y = Y + k where (h, k) will satisfy the equation 3y – 7x + 7 = 0 and 7y – 3x + 3 =0 we


can reduce the equation and after that by putting Y = VX and integrating we will get required general equation.
dy 7x − 3y − 7
The given differential equation is =
dx −3x + 7y + 3
Substituting x = X + h, y = Y + k, we obtain

dY (7X – 3Y) + (7h − 3k – 7)


=  ... (i)
dX (–3X + 7Y) + (–3h + 7k + 3)
2 4 . 1 4 | Differential Equations

Choose h and k such that 7h – 3k – 7 = 0 and –3h + 7k + 3 = 0.


This gives h = 1 and k = 0. Under the above transformations, equation (i) can be written as

dY dV dY 7X − 3Y
Let Y = VX so that = V+X , we get =
dX dX dX –3X + 7Y

dV −3V + 7 dV 7 − 7V 2 dX 7 2V 3
V+X = ⇒X = ⇒ –7 = . dV − dV
dX 7V − 3 dX 7V − 3 X 2 V2 − 1 2
V −1
Integrating, we get
7 3 V −1
–7logX = log(V2 – 1) – log – logC ⇒ C = (V + 1)5 (V – 1)2X7 ⇒ C = (y + x – 1)5 (y – x + 1)2
2 2 V +1
Which is the required solution.

7.7 Linear Differential Equation


A differential equation is linear if the dependent variable (y) and its derivative appear only in the first degree. The
general form of a linear differential equation of the first order is
dy
+ Py = Q ….. (i)
dx
where P and Q are either constants or functions of x.

This type of differential equation can be solved when they are multiplied by a factor, which is called integrating
factor.
Pdx  dy 
Multiplying both sides of (i) by e∫ , we get e∫ 
pdx
+ Py  = Qe∫
pdx

 dx 
On integrating both sides with respect to x, we get

ye∫ ∫ Qe∫ + c which is the required solution, where c is the constant and e∫
Pdx Pdx pdx
= is called the integrating factor.

dy 1 ey
Illustration 29: Solve the following differential equation: + =  (JEE MAIN)
dx x x

dy e− y 1
Sol: We can write the given equation as e− y + = . By putting e–y = t, we can reduce the equation in the
dx x x
dt
form of + Pt = Q hence by using integration factor we can solve the problem above.
dx

dy 1 ey dy e− y 1
We have, + = ⇒ e− y + =  ... (i)
dx x x dx x x
dy dt t 1 dt 1 1
Put e–y = t. so that in equation (i), we get – + =⇒ − 
− t= ... (ii)
dx dx x x dx x x
This is a linear differential equation in t.
 1
1 1 ∫  − dx −1 1
Here, P = − and Q = − ∴ I.F. = e∫
Pdx
= e  x  = e− log x = elog x =
x x x
dy 3x – 6y + 7 3(x − 2y) + 7
∴ The solution of (ii) is, t.(I.F.)
= = =
dx x − 2y + 4 x − 2y + 4

1 1 1 t 1 e− y 1
t =∫  −  dx + C ⇒ = +c ⇒ = +C
x x x x x x x
M a them a ti cs | 24.15

x
y(x)
Illustration 30: The function y(x) satisfy the equation y(x) + 2x ∫ 1 + x2 dx = 3x 2
+ 2x + 1. Prove that the substitution
x 0
y(x)
z(x) = ∫1 dx converts the equation into a first order linear differential equation in z(x) and solve the original
0 + x2
equation for y(x)  (JEE MAIN)
y (x)
Sol: By putting z’(x) = we will get the linear differential equation in z form and then by applying integrating
1 + x2
factor we get the result.
d(x)
Let z’(x) = ⇒ z’(x) × (1 + x2) + 2x(z(x)) = 3x2 + 2x + 1
1 + x2
dz 2x 3x2 + 2x + 1
⇒ + z =  ... (i)
dx 1 + x2 x2 + 1
This is a first order linear differential equation in z.
2x
∫ dx
∴ I.F. = e∫ = e ∫ ( Q × I.F ) dx + c
Pdx 1 + x2 = 1 + x2 ∴ Solution of (i) is z(I.F.) =

x3 + x 2 + x x 4 x3 x 2
⇒ z (1 + x2) = ∫ (x2 + 1)dx + C ⇒ z (1 + x2) = + + + C and y = 3x2 + 2x + 1 – 2xz
x2 + 1 4 3 2

Illustration 31: Solve the differential equation ysin2x.dx – (1 + y2 + cos2x)dy = 0 (JEE MAIN)
dt
Sol: Similar to illustration 28, by putting –cos 2x = t, we can reduce the equation in the form of + Pt = Q hence
by using integration factor we can solve the problem given above. dx

We have, ysin2x.dx – (1 + y2 + cos2x)dy = 0


dx cos2x 1 + y 2
⇒ sin2x. − = ... (i)
dy y y
dx dt dt 2  1 + y2 
Putting –cos 2x = t so that 2sin2x = in equation (i), we get + t=
2 
dy dy dy y  y 
 
2 1 + y2
Here, P = and Q = 2
y y
2
∫y
∫ Pdy e=
dy
∴ I.F. = e= y2 ∴ The solution is t.(I.F.) = ∫ (Q × I.F.)dy + C

1 + y2 2 y4
⇒ t.y2 = 2 ∫ .y dy = 2 ∫ y + y 3 dy ⇒ t.y2 = y2 + +C
y 2
y2
On putting the value of t, we get –.cos2x = 1 + + Cy −2
2
dx
Illustration 32: Solve ylogy + x – logy = 0  (JEE MAIN)
dy
dx
Sol: By reducing the given equation in the form of Q we can solve this as similar to above illustrations.
+ Px =
dy
dx dx x 1
We have, ylogy +x –logy = 0 ⇒ + =
dy dy y log y y
This is a linear differential equation in x.
1
1 1 ∫ dy
Here P = , Q = ; I.F. = e y log y = elog(logy) = logy
y log y y
2 4 . 1 6 | Differential Equations

1 1
The solution is, x(I.F.) = ∫ ( Q × I.F. ) + C ; xlogy = ∫ y (log y ) dy + c =
2
(log y)2 + C
1 1
x= log y + C
2 log y

dx
Illustration 33: Solve (x + 2y3) = y (JEE ADVANCED)
dy
dx
Sol: By reducing given equation in the form of + Px = Q and then using the integration factor we can solve this.
dy
dx dx x + 2y 3 x dx 1
(x + 2y3) =y⇒ = = + 2y 2 ⇒ 2y 2
− x=
dy dy y y dy y
1
– ∫ dy 1
y
I.F = e = ;
y
1
Solutions is x. = y2 + C
y

Alternate method: xdy + 2y3dy = ydx

ydx − xdy x x


⇒ 2ydy = ⇒ 2ydy = d   ⇒ y2 = +C
y 2
y y

Illustration 34: Let g(x) be a differential function for every real x and g’(0) = 2 and satisfying g(x+y) = eyg(x) +
2exg(y) ∀ x and y. Find g(x) and its range.  (JEE ADVANCED)

g ( x + h) − g ( x )
Sol: By using g’(x) = lim and solving we will get g(x).
b →0 h
g ( x + h) − g ( x )
g’(x) = lim
b →0 h
e g(x) + 2ex g(h) − g(x)
h
eh − 1 g(h)
⇒ g’(x) = lim ⇒ g’(x) = g(x) lim + 2ex lim ⇒ g’(x) = g(x) + 2ex
h→0 h h→0 h h→0 h

At x = 0, g(x) = 0 ⇒ g(0) = 0
dy
– y = 2ex ⇒ I.F. = e–x
dx
Solution is y.e–x = 2x + C
g(0) = 0 ⇒ C = 0 ⇒ g(x) = 2xex
g’(x) = 2ex + 2xex = 2ex(x + 1)
g’(x) = 0 at x = -1; g(–1) = –2/e
 2 
⇒ Range of g(x) =  − e , ∞ 
 

dy
Illustration 35: Find the solution of (1 – x2) + 2xy = x 1 − x2  (JEE ADVANCED)
dx
dy
Sol: By reducing given equation in the form of + Py = Q and then by using integration factor i.e.
dx
 dy 
e∫ + Py  = Qe∫ we can solve the problem.
Pdx pdx

 dx 
M a them a ti cs | 24.17

2x
dy 2x x 1 − x2 ∫ 2 dx 1
e∫
Pdx
+ y = ; I.F. =
= e=1− x
dx (1 − x )
2
1−x 2
1 − x2

1 x 1 x −1 −2x
2 ∫
Solution
= is y. y ∫ dx + c = ∫ (1 – x2 )3/2 dx + C = dx + c
( )
3/2
1 − x2 1 − x2 1 − x
2
1 − x2
-1
1 1
y = +c
1 − x2( ) 1 − x2 -1

Figure 24.3

PLANCESS CONCEPTS

Every linear differential equation is of degree 1 but every differential equation of degree 1 is not linear
Shivam Agarwal (JEE 2009, AIR 27)

7.8 Equations Reducible to Linear form

(a) Bernoulli’s Equation


dy
A differential equation of the form + Py = Qyn, where P and Q are function of x and y is called Bernoulli’s
dx
equation. This form can be reduced to linear form by dividing yn and then substituting y1–n = v
dy
Dividing both sides by yn, we get, y–n dx + P.y–n+1 = Q
dy dv dv
Putting y–n+1 = v, so that , (1 – n)y–n dx = , we get + (1 – n)P.y = (1 – n)Q
dx dx
Which is a linear differential equation
dy
(b) If the given equation is of the form + P. f(y) = Q.g(y), where P and Q are functions of x alone, we divide the
dx
– ln(1–x2 ) 1
e∫
Pdx
equation by f(y), and then we get= e=
1 − x2
f (y)
Now substitute = v and solve.
g( y )

dy
Illustration 36: Solve = xy + x3 y 2  (JEE MAIN)
dx
1 dy 1 −1
Sol: By rearranging the given equation we will get – x = x3 and then by putting = t and using the
y 2 dx y y
integration factor we can solve it.
dy dy 1 dy 1
= xy + x3 y 2 ⇒ x3 y 2
− xy = ⇒ – x = x3
dx dx y 2 dx y
−1 dy
put = t ⇒ + tx = x3
y dx
2 /2 2 /2 x2 /2
This is a linear differential equation with I.F. = ex ⇒ t ex 3
∫ e x dx
=
2 4 . 1 8 | Differential Equations

Illustration 37: Find the curve such that the y intercept of the tangent is proportional to the square of ordinate of
tangent (JEE MAIN)
dy −1
Sol: Here X = 0 and Y= y – mx i.e. x – y = –ky2. Hence by putting = 1 and applying integration factor we
dx y
will get the result.
dy
X = 0 ⇒ Y= y – mx ⇒ x – y = –ky2
dx
1 dy 1 1 –k
⇒ – . =
y 2 dx y x x

−1 dt t –k
Put =t ⇒ + =
y dx x x

⇒ I.f. = x
−x
⇒ Solution is t.x = –kx + C ⇒ = –kx + C
y

7.9 Change of Variable by Suitable Substitution

Following are some examples where we change the variable by substitution.

dy
Illustration 38: Solve ysinx = cos x(sinx – y 2 )  (JEE MAIN)
dx
dt
Sol: Here by putting y2 = t, the given equation reduces to + ( 2cot x ) t = 2cosx and then using the integration
dx
factor method we will get result.
dy
ysinx = cos x(sinx – y 2 )
dx
1 dt
Let y2 = t ⇒ sinx = cosx (sinx – t)
2 dx
dt dt
= 2cos x − (2cot x)t
⇒ ⇒ + ( 2cot x ) t = 2cosx
dx dx

I.F. = sin2x
2
⇒ Solution is tsin2x = ∫ 2cos x.sin xdx

2sin3 x
y 2=
sin2 x +c
3

dy
= ex − y (ex − ey ) 
Illustration 39: Solve (JEE MAIN)
dx

Sol: Simply by putting ey = t and using the integration factor we can solve the above problem.
dy
( )
2
dy ex − ex e y
= ex − y (ex − ey ) ⇒ e= y
dx
dx
dy
Put ey = t ⇒ + tex = (ex)2;
dx
x
I.F. = e∫
edx
= ee
x x
Solution is tee = ∫ (ex )2 .ee dx
M a them a ti cs | 24.19

PLANCESS CONCEPTS

If we can write the differential equation in the form


f(f1(x, y) d(f1(x, y)) + φ(f2(x, y)d(f2(x,y)) + …… = 0, then each term can be easily integrated separately. For
this the following results must be memorized.
(i) d(x + y) = dx + dy (ii) d(xy) = xdy + ydx

 x  ydx − xdy  y  xdy − ydx


(iii) d   = (iv) d  =
y y2 x x2

 x2  2xydx − x2dy  y 2  xydx − y 2dx


(v) d   = (vi) d  =
 y  y2  x  x2
   
 x2  2xy 2dx − 2x2 ydy  y 2  xydx − 2xy 2dx
(vii) d   = (viii) d  =
 y2  y4  x  x4
   
 x  ydx − xdy  y  xdy − ydx
(ix) d  tan−1  = (x) d  tan−1  =
 y  x2 + y 2  x x2 + y 2

xdy + ydx   x   ydx − xdy


(xi) d[log(xy)] = (xii) d  log    = ‘
xy  y  xy

  y   xdy − ydx
1
2
(  xdx + ydy
(xiii) d  log x2 + y 2  =
 x2 + y 2
)
(xiv) d log    =
  x  xy

 1  xdy + ydx  ex  yex dx − ex dy


(xv) d  −  = (xvi) d =
 xy  x2 y 2  y  y2
 
 ey  xey dy − ey dx
(xvii)
d = (xviii) d(xmyn) = xm–1yn–1 (mydx + nxdy)
 x  x2
 
dt t 1 x + y  xdy − ydx
(xix) d + (xx) d  log = 2
dx x 2 x−y x − y2
1 −n
d f ( x, y )  f ' ( x, y ) 1 1 1  1  dx dy
(xxi)  = (xxii) d  −  = d   − d   = −
1−n
( f ( x, y ) ) y x y  x  x2 y 2
n

Shrikant Nagori (JEE 2009, AIR 30)

8. EXACT DIFFERENTIAL EQUATION


The differential equation Mdx + Ndy = 0, where M and N are functions of x and y, is said to be exact if and only if
∂M ∂N
=
∂y ∂x

Rule for solving Mdx + Ndy = 0 when it is exact


(a) First integrate the terms in M w.r.t. x treating y as a constant.
(b) Then integrate w.r.t. y only those terms of N which do not contain x.
2 4 . 2 0 | Differential Equations

(c) Now, sum both the above integrals obtained and quote it to a constant i.e. ∫ Mdx + ∫ Ndy =
k , where k is a constant.
(d) If N has no term which is free from x, the ∫ Mdx = c (y constant)

Following exact differentials must be remembered:

xdy − ydx y ydx − xdy x


(i) xdy + ydx = d(xy) (ii) = d   (iii) = d 
x y
2 2
x y
xdy + ydx dx + dy xdy − ydx  y
(iv) = d(logxy) (v) = dlog(x + y) (vi) = d  n 
xy x+y xy  x
ydx – xdy  x xdy − ydx  y ydy − xdx  y
(vii) = d  n  (viii) = d  tan−2  (ix) = d  tan−1 
xy  y
2
x +y 2
 x 2
x +y 2
 x

 ex  yex dy − ex dy
(x) d  =
 y  y2
 

9. ORTHOGONAL TRAJECTORY
Definition 1: Two families of curves are such that each curve in either family is orthogonal (whenever they intersect)
to every curve in the other family. Each family of curves is orthogonal trajectories of the other. In case the two
families are identical then we say that the family is self-orthogonal
Slope –1/dy/dx
= -1/dy/dx
Slope =

Slope = dy/dx
Slope = dy/dx

Figure 24.4: Orthogonal trajectories

PLANCESS CONCEPTS

Orthogonal trajectories have important application in the field of physics. For example, the equipotential
lines and the streamlines in an irrotational 2D flow are orthogonal.
Ravi Vooda (JEE 2009, AIR 71)

9.1 How to Find Orthogonal Trajectories


Suppose the first family of curves F(x, y, c) = 0  ... (i)
To find the orthogonal trajectories of this family we proceed as follows. First, differentiate (i) w.r.t. x to find
G(x, y, y’, c) = 0  ... (ii)
Now eliminate c between (i) and (ii) to find the differential equation H(x, y, y’) = 0  ... (iii)
M a them a ti cs | 24.21

The differential equation for the other family is obtained by replacing y’ with –1/y’. Hence, the differential equation
the orthogonal trajectories is H(x, y, –1/y’) = 0  ... (iv)
General solution of (iv) gives the required orthogonal trajectories.

Illustration 40: Find the orthogonal trajectories of a family of straight lines through the origin.  (JEE MAIN)
Sol: Here as we know, a family of straight lines through the origin is given by y = mx.
Hence by differentiating it with respect to x and eliminating m we will get an ODE of this family and by putting –1/y’
in place of y’ we will get an ODE for the orthogonal family.
The ODE for this family is xy’ – y = 0
The ODE for the orthogonal family is x + yy’ = 0
Integrating we find x2 + y2 = c, which are family of circles with center at the origin.

10. CLAIRAUT’S EQUATION


The differential equation
y = mx + f(m),  ... (i)

dy
where m = is known as Clairaut’s equation.
dx

To solve (i), differentiate it w.r.t. x, which gives

dy dm df(m)
=m+x +
dx dx dx
dm dm
⇒ x + f '(x) =0
dx dx
dm
either = 0 ⇒ m = c ... (ii)
dx

or x + f’(x) = 0  ... (iii)

PLANCESS CONCEPTS

•• If m is eliminated between (i) and (ii),the solution obtained is a general solution of (i)
•• If m is eliminated between (i) and (iii), then the solution obtained does not contain any arbitrary
constants and is not the particular solution of (i). This solution is called singular solution of (i)
Chinmay S Purandare (JEE 2012, AIR 698)
2 4 . 2 2 | Differential Equations

PROBLEM SOLVING TACTICS

Think briefly about whether you could easily separate the variables or not. Remember that means getting all the x
terms (including dx) on one side and all the y terms (including dy) on the other. Don’t forget to convert y’ to dy/dx
or you might make a mistake.
If it’s not easy to separate the variables (usually it isn’t) then we can try putting our equation in the form y’ + P(x)y = Q(x).
In other words, put the y’ term and the y term on the left and then you may divide so that the coefficient of y’ is 1.
 ex  xey dy − ey dx
Then we can use the trick of the integrating factor in which we multiply both sides by . d   = . This
 x  x2
 
makes things much simpler, but it’s best to see why from doing problems, not from memorizing formulas.

FORMULAE SHEET

(a) Order of differential equation: Order of the highest derivative occurring in the differential equation

(b) Degree of differential equation: Degree of the highest order derivative when differential coefficients are free
from radicals and fractions.
dy
(c) General equation : = f ( x ) ⇒ y = ∫ f ( x ) dx + c
dx
dy
(d) = f(ax + by + c) , then put ax + by + c = v
dx
dy
(e) If =f(x)g(y) ⇒ g(y)−1 dy =f(x)dx then ∫ (g(y))−1 dy = ∫ f(x)dx
dx

(f) Parametric forms


y
Case I: x = rcosθ, y = rsinθ ⇒ x2 + y2 = r2; tanθ = ; xdx + ydy = rdr; xdy – ydx = r2dq
x
y
Case II: x = rsecθ, y = rtanθ ⇒ x2 – y2 = r2; = sinθ; xdx – ydy = rdr; xdy – ydx = r2secqdq
x
dy f ( x, y ) dx dv
(g) If = , then substitute y = vx ⇒ ∫ =∫ +c
dx g ( x, y ) x f (v) − v

dv a1 x + b1 y + c1
(h) If = , then substitute x = X + h, y = Y + k
dx a2 x + b2 y + c2

dY a1 X + b1 Y + (a1h + b1k + c1 )
⇒ =
dX a2 X + b2 Y + (a2h + b2k + c2 )

choose h and k such that a1h + b1k + c1 = 0 and a2h + b2k + c2 = 0.

dy
ye∫ ∫ Qe∫
Pdx Pdx
(i) If the equation is in the form of + Py = Q then= +c
dx
M a them a ti cs | 24.23

Solved Examples

JEE Main/Boards xdy − ydx  y


(y – b)2 = 2(y – b) = d  tan−1  (x – a)
x +y2 2
 x
Example 1: Find the differential equation of the family
of curves y = Aex + Be–x Differential equation is,
Sol: By differentiating the given equation twice, we will dy
2(x – a) = y – b.
get the result. dx
dy
= Aex − Be− x Example 4: Show that the function y = bex + ce2x is a
dx
solution of the differential equation.
d2 y
⇒ =Aex + Be–x = y d2 y dy
dx2 −3 +2y = 0
dx 2 dx
Example 2: Find differential equation of the family of Sol: Differentiating given equation twice we can obtain
curves y = c(x – c)2 , where c is an arbitrary constant. the required differential equation.
Sol: By differentiating the given family of curves and y = bex + ce2x
then eliminating c we will get the required differential
dy
equation. ⇒ = bex + 2ce2x = y + ce2x
dx
y = c(x – c)2
d2 y
dy ⇒ =bex + 4ce2x = y + 3ce2x
⇒ = 2(x – c)c dx2
dx
x−c y d2 y dy
By division, = ⇒ −3 + 2y =
0
2 dy / dx dx 2 dx
2y
or c=x– dy 1 − y2
dy / dx Example 5: Solve: + =0
dx 1 − x2
Eliminating c, we get
2 Sol: By separating x and y term and integrating both
 dy 
  = 4c (x – c) = 4cy
2 2 sides we can solve it.
dx
 
dy dx
∫ = –∫
 2y 
= 4y 4y x −  1 − y2 1 − x2
 dy / dx 
3
⇒ sin–1y = – sin–1x + c or sin–1y + sin–1x = c
 dy   dy 
⇒ =
  4y  x − 2y 
 dx   dx  Example 6: Find the equation of the curve that passes
through the point P(1, 2) and satisfies the differential
Example 3: Find the differential equation of all equation
parabolas which have their vertex at (a, b) and where –2xy
the axis is parallel to x-axis. ∫ f(x)dx + C = x2 + 1 : y > 0
Sol: Equation of parabola having vertex at (a, b) and Sol: By integrating both sides we will get general
axis is parallel to x-axis is (y – b)2 = 4L(x – a) where L is equation of curve and then by substituting point (1, 2)
a parameter. Hence by differentiating and eliminating L in that we will get value of constant part.
we will get required differential equation.
dy 2xy dy 2x
∴ 2(y –b)
dy
= 4L dx
= −
x2 + 1
⇒ ∫ y
= −∫
2
x +1
dx
dx
On eliminating L, we get ⇒ log|y| = –log(x2 + 1) + logc0
2 4 . 2 4 | Differential Equations

⇒ log(|y| (x2 + 1)) = logc0 dy dy


Example 9: Solve: tanx : 0 < x <
+ sec x =
dx dx
⇒ |y|(x2 + 1) = c0
dy
As point P(1, 2) lies on it, Sol:The given equation is in the form of + px = q
dx
hence by using integration factor method we can solve it.
2(1 + 1) = c0 or c0 = 4
–2xy dy 2xy
I.F. = = =–
∴ Curve is y(x2 + 1) = 4 x =1 2 dx x2 + 1
= secx + tanx
Example 7: Solve:
dy
=
(x − y) + 3
dx 2 ( x − y ) + 5 Solution is y(secx + tanx)

Sol: By putting x – y = t and integrating both sides we = ∫ tanx ( sec x + tan) dx


will obtain result.
∫ ( sec )
2
= ∫ sec x tanx dx + x − 1 dx
dy dt
Put x – y = t; then, 1 – = = secx + tanx – x + c
dx dx
or (y – 1) (secx + tanx) = c – x
Differential equation becomes
dy t+3 dt t+3 t+2
1− = or =1– = Example 10: Solve:
dx 2t + 5 dx 2t + 5 2t + 5
2t +5 dt sinx.cosy.dx + cosx.siny.dy = 0
⇒ ∫ dx = ∫ t + 2 dt = 2t + ∫1+2 given, y =
π
when x = 0.
4
⇒ x + c = 2t + log|t + 2| = 2(x – y) + log|(x – y + 2)|
Sol: Here by separating variables and taking integration
Example 8: x dy + y(x + y)dx = 0: xy > 0
2 we will get the general equation and then using the
given values of x and y we will get value of constant c.
Sol: We can write the given equation as We have,
3
 dy   dy  dy sinx.cosy.dx + cosx.siny.dy = 0
=
  4y  x − 2y  =–
dx
   dx  dx On separating the variables, we get
and then by substituting y = vx and integrating we will dt

dx
get required general equation.
Integrating both sides, we get
dy y (x + y)
=– (Put y = vx) sinx sin y
dx x2 ∫ cos x dx + ∫ cos y dy =
0

dy [Dividing by cosx cosy], we get


⇒ v+x = –v(1 + v)
dx
⇒ log|secx| + log|secy| = logC
dv dv dx
⇒ =–2v – v2 or ∫ =∫
dx v ( v + 2) x ⇒ log|secx| |secy| = logC
⇒ secx.secy = C ... (i)
dx 1  1 1  π
⇒ −∫ = ∫  −  dv On putting y = , x = 0 in (i),
x 2  v v +2 4
π
d2 y dy we have C = sec0.sec
⇒ –log|x| = –3 0 (log|v| – log|v+2| + c0
+ 2y = 4
2 dx
dx
y 2
⇒ C (1). ( 2) = 2
x Substituting the value of C in (i) we get
V 2 yx2
or x = c or x = c or =c
V+2 y y + 2x 1 1
+2 secx. = 2 ⇒ cosy = sec x
x cos y 2
 1 
⇒ y = cos–1  sec x 
 2 
M a them a ti cs | 24.25

Example 11: Solve the differential equation x 1


⇒ = |logx| + c
dy y 2
= x2e−3y , given that y =0 for x =0.
dx
Example 13: Solve the following differential equation
Sol: Similar to the problem above we can solve it.
dy
cos2x + y = tanx
dy dx
Here, = x2e−3y  ….. (i)
dx
Sol: Here by reducing the given equation in the form of
On separating the variables, we have
dy
⇒ e3ydy=x2dx + py = q and then using integration factor we will
dx
Integrating both sides, we get get the result.
dy
∫e
3y
= ∫ x2dx We have, cos2x + y = tanx
dx
e3y x3 dy
⇒ = +c  ….. (ii) ⇒ + y.sec2x = tanx.sec2x
3 3 dx
2x
putting: y = 0 for x = 0, in (ii), we obtain I.F. = e∫
sec
= etanx
0
e 1
3
=0+C⇒
3
= C [e0 = 1] y.(I.F.) = ∫ Q. ( I.F. ) dx + c
2
On substituting the value of C in (ii), we get y.etanx = ∫ tanx sec xetan x dx = ∫ tet dt
∴ e3y = x3 + 1  tanx = t 
tet − ∫ et dt + c  2 
which is the required particular solution of (i) sec xdx = dt 
= tet – et + c
Example 12: Solve the following differential equation:
dy = tan x etanx – etanx + c
2x2 dx – 2xy + y2 = 0 y = tanx – 1 + ce–tanx

Sol: Here by rearranging the given equation we will dy


Example 14: Solve x – y = x2
dx
get elog(sec x + tan x) = ∫ tanx(sec x + tanx)dx . Now by
Sol: As similar to the problem above, we can reduce the
substituting y = vx and then integrating we can solve
dy
the illustration above. given equation as therefore by using integration
dx
dy
2x2 = 2xy – y2 factor we can solve this.
dx
dy
dy 2xy − y 2 We have, x −y =x2
⇒ =  ….. (i) dx
dx 2x2
dy dy 1
Put y = vx so that = v + x
dv
in (i), we get
⇒ x
− y= ... (i)
dx dx dx x
This is a linear differential equation in y
dv 2x ( vx ) − ( vx ) 2
⇒ v+x = 2xy − y 2
dx 2x2 Here, P = – and Q = x
2x2
dv v2 1
⇒ v+x =v– ∫ − dx
I.F. = e∫
pdx
dx 2 Now, =e x

xdv –v 2 dv dx −1 1
= ⇒ = e–logx = elog = x–1 =
dx 2 v 2 2x x
Integrating, we have ∴ The solution of (i) is


1
=
1
|logx| + c y(I.F.) = ∫ ( Q × IF. ) dx + C =x+C
4 4
⇒ y = x2 + Cx
2 4 . 2 6 | Differential Equations

Example 15: Solve the following differential equation: Integrating, we get


dv 1 –v
+ y = cosx – sinx. logx – e (sinv + cosv) = c
dx 2
dy 1 –y/x  y y
Sol: Here given equation is in the form of Q,
+ Py = or logx = c + e .
dx  sin − 4 cos 
2  x x
where P = 1 and Q = cosx – sinx hence by using
integration factor we will get result. Example 2: Solve:
Given differential equation is xdy – ydx = xy3(1 + logx)dx
dy
+ y = cosx – sinx ... (i) Sol: We can reduce the given equation in the form of
dx
x x
The given differential equation is a linear differential – d   = x2(1 + logx)dx. Hence by integrating L.H.S.
y y
equation x
with respect to and R.H.S. with respect to x we will
y
dy
On comparing with + Py = Q get the solution.
dx
∴ P = 1, Q = cosx – sinx ydx − xdy
– = xy(1 + logx)dx
y2
I.F. = e∫ = ex
pdx
x
or –d   = xy (1 + logx)dx
∴ required solution of (i) is y
y (I.F.) = ∫ Q. ( I.F ) dx + c or –
x x
d   = x2(1 + logx)dx
y y
⇒ y.ex = ∫ (cos x − sinx)ex dx + c
x x
⇒ y.ex = x x Integrating, − ∫ d  
∫ cos xe dx − ∫ sinx.e dx + c y y
Integrating by parts, we get
∫ x (1 + logx ) dx
2
=
∫ e dx − ∫ ( − sinx ) e dx 2 x dx + c
x x
⇒ y.ex = cosx – ∫ sec
x x
⇒ y.ex = excosx + ∫e sinxdx – ∫e sinxdx + Cc 2
1 x x3 x3 1
or –   = (1 + logx) − ∫ . dx
y.ex = excosx + c 2 y  3 3 x
∴ y = cosx + ce–x 2
−1  x  x3 x3
  = (1 +log x) − +c
2 y 3 9
JEE Advanced/Boards
Example 1: Solve Example 3: Find the equation of the curve passing
through (1, 2) whose differential equation is
 y /x y y
 xe – y sin  dx +xsin dy =0; x>0 y(x + y3)dx = x(y3 – x)dy
 x  x

Sol: Simply by putting y = vx and integrating we can Sol: Similar to example 2 we can solve the problem
solve the problem above. above by reducing the given equation as –
 y y y ydy y y 1
 e x − sin  + sin =0 d  + d ( xy ) =
0.
 x x xdx x  x  x y2
2
 
Put y = vx (xy + y4)dx = (xy3 – x2)dy
 dv 
∴ (ev – vsinv) + sinv  v + x  = 0 or y3(ydx – xdy) + x(ydx + xdy) = 0
 dx 
dx xdy − ydx
⇒ ∫ + e− v sinvdv = or –x2y3 + xd(xy) = 0
x ∫
0
x2
M a them a ti cs | 24.27

y y 1 dy
or – d  + d ( xy ) =
0 Given y = 1 and = 0 at x = 1
x  x  x2 y 2 dx
⇒ C1 = 0 and C2 = 2
Integrating, we get
2
Therefore, the required solution is y = x log x – x + 2
1y 1
–   − = Ac
=
2 x  xy Example 6: By the elimination of the constant h and k,
find the differential equation of which (x–h)2+(y–k)2=a2,
or y3 + 2x – 2cx2y = 0
is a solution.
As it passes through (1, 2), condition is
Sol: Three relations are necessary to eliminate two
5
8 + 2 + 4c = 0 ⇒ c = – constants. Thus, besides the given relation we require
2
two more and they will be obtained by differentiating
Thus curve is y3 + 2x – 5x2y = 0 the given relation twice successively.
Thus we have
Example 4: Form the differential equation representing
the family of curves y = Acos2x + Bsin2x, where A and dy
(x – h) + (y – k) =0 ... (i)
B are arbitrary constants. dx
2
d2 y  dy 
Sol: Here we have two arbitrary constants hence we 1 + (y – k) +  = 0 ... (ii)
dx2  dx 
have to differentiate the given equation twice.
From (i) and (ii), we obtained
The given equation is:
2
y = Acos2x + Bsin2x  ... (i)  dy 
1+ 
Diff. w.r.t. x, y–k=–  dx 
dy d2 y
= –2Asin2x + 2Bcos2x
dx dx2
d2 y   dy 2  dy
Again diff. w.r.t. x, = –4Acos2x – 4Bsin2x 1 +   
2
dx   dx   dx
= –4(Acos2x + Bsin2x) = –4y [Using (i)] x–h=  
2
2
d y
d y
Hence + 4y = 0, which is the required differential dx2
dx2
equation. Substitute these values in the given relation, we
obtained
3 2
  dy 2   d2 y 
Example 5: The solution of the differential equation x 1 +    = 2
a  
  dx    dx2 
d2 y dy    
= 1, given that y = 1, = 0, when x = 1, is
dx 2 dx which is the required differential equation.
d2 y
Sol: By integrating x = 1 twice we will get its
dx2 Example 7: Form the differential equations by elimi-
general equation and then by substituting given values nating the constant(s) in the following problems.
dy
of x, y and we will get the values of the constants. (a) x2 – y2 = c(x2 + y2)2, (b) a(y + a)2 = x3
dx

d2 y d2 y 1 Sol: Given equations have one arbitrary constant,


x =1⇒ = hence by differentiating once and eliminating c and a
dx2 dx2 x
we will get the required differential equation.
dy
⇒ = logx + C1 (a) The given equation contains one constant
dx
Again integrating Differentiating the equation once, we get
2x – 2yy’ = 2c(x2 + y2) (2x + 2yy’)
y = xlogx – x + C1x + C2
2 4 . 2 8 | Differential Equations

x2 − y 2 dx 1
But c = Let P= ∫ = log{(x–y)2 –1}
( x − 3) 2
(x )
2
2
+ y2
dx
Substituting for c, we get ∴ P= ∫ ( x − 3y )

(x – yy’) =
(x 2
+ y2 )( x 2
− y2 ) .2(x + yy’) dP
=
1
 ... (i)
dx ( x − 3y )
(x )
2
2 2
+y
1
or (x2 +y2) (x – yy’) = 2(x2 – y2)(x+yy’) Also P = log{(x – y)2 – 1}
2
⇒ yy’[(x2 + y2) + 2(x2 – y2)]
( x − y ) 1 − dy
 

⇒ x(x2 + y2)–2x(x2 – y2) dP  dx  
∴ = ... (ii)
⇒ yy’(3x – y ) = x(3y – x )
2 2 2 2
dx
{( x − y ) 2
−1 }
Hence, y’ =
(
x 3y 2 − x2 ) Given y(x – y)2 = x
y ( 3x 2
−y ) 2
Differentiating both sides w.r.t. x
(b) The given equation contains only one constant. 1 − 2y ( x − y )
dy
Differentiating once, we get ∴ =  ... (iii)
dx ( x − y )( x − 3y )
2a(y + a)y’ = 3x2  ... (i)
From (ii) and (iii)
Multiplying by y + a, we get
2a(y + a)2y’ = 3x2(y + a) dP (x − y){1 − (1 − 2y(x − y) / (x − y)(x − 3 y))}
=
Using the given equation, we obtain dx {
(x − y)2 − 1 }
2x y’ = 3x (y + a)
3 2
or 2xy’ = 3y + 3a
1 =
( x − y )( x − 3y ) − 1 + 2y ( x − y )
or a= (2xy’ – 3y)
3 ( x − 3y ){( x − y 2 ) − 1}
Substituting the value of a in (i) we obtain

2  1  =
{( x − y ) − 1}
2

( x − 3y ) {( x − y ) − 1}
(2xy’ – 3y)  y + (2xy – 3y) y ' = 3x2 2
3  3 
2
(2xy’–3y)(2xy’)y’ = 3x2 dP 1
9 ⇒ =
dx ( x − 3y )
Cancelling x, we obtain
8x(y’)3 – 12y(y’)2 – 27x = 0 It is true from (i)

Example 8: If y(x – y) = x, then show that 2


Hence
dx
∫ ( x − 3y=)
1
2 {
n (x − y) − 1
2
}
dx 1
∫ ( x − 3y ) = 2 log[x–y)2 – 1]
Example 9: Solve: cos(x + y)dy = dx

Sol: As given y(x – y)2 = x, therefore by differentiating Sol: Simply by putting x + y = t we can reduce the given
dy dt
it with respect to x we will get the value of . After equation as = sect + 1 and then by separating the
dx dx
dx 1 variable and integrating we can solve the problem
that differentiate both sides of equation ∫ = given above.
( x − 3y ) 2
log[x–y)2 – 1] w.r.t. x and then by substituting the value We have cos(x + y)dy = dx
dy dy
of we can prove it. ⇒ = sec(x + y)
dx dx
M a them a ti cs | 24.29

∫ ( sec )
dy dt 2
On putting x + y = t so that 1 + =
dx dx

= ∫ dx t − tantsect dt

dy dt ⇒ x = tant – sect
or = − 1 we get
dx dx
⇒ x = tan(x + y) – sec(x + y) + C
dt
−1 =sec
dx
Example 11: Solve the equation:
dt
⇒ = 1 + sect dy y y
dx = + x sin
dx x x
dt cost
=dx ⇒ dt = dx
1 + sec t cos t + 1 Sol: Simply by putting y = vx and integrating we
can obtain the general equation of given differential
cos t
∫ cos t + 1 dt = ∫ dx equation.
We have,
 1 
⇒ ∫ 1 −  ' dt = x + C dy y y
 cos t + 1  = + x sin  ... (i)
dx x x
 1 
∫ 1 − 2cos2 (t / 2) − 1 + 1  dt = x + C Put y = vx, so that
 
dy dv
= v+x
 1 2 t dx dx
∫  1 − 2 sec  dt = x + C
2 dy
On putting the value of y and in (i), we get
dx
t
⇒ t – tan =x+C dv
2 v+x = v + xsinv
dx
x+y
x + y – tan =x+C dv dv
2 ⇒ x ⇒ = sin v
dx dx
x+y
y – tan =C Separating the variables, we get
2
dv
 dy  = dx ⇒ ∫ cosecv dv = ∫ dx
Example 10: Solve: sin  dx  = x + y
–1 sin v
 
v
⇒ log tan = x + C  ... (ii)
Sol: Similar to example 9. 2

 dy  On putting the value of v in (ii), we have


dy
We have, sin  dx  = x + y ⇒
–1
= sin(x + y) y
  dx logtan =x+C
2x
Putting x + y = t, so that
This is the required solution
dy dt dy dt
1+ = ⇒ =
dx dx dx dx
Example 12: Solve:
dy dt
Now, substituting x + y = t and = − 1 in (i), x  x 
dx dx
we get 2ye y dx +  y − 2xe y  dy =
0
 
 
dt dt dt
= sint ⇒ = sint + 1 ⇒ dx = dy 2xex/ y
dx dx 1 + sint Sol: We can reduce the given equation as =
dx 2yex/ y
Integrating both sides, we get and then by putting x = vy and integrating we can
dt obtain general equation.
= ∫ dx ∫ 1 + sin2 t dt + c
We have,
1 − sint 1 − sint x  x 
⇒ ∫ dx = ∫ 1 − sin2 t dt +C= ∫ cos2 t
dt
2ye y dx +  y − 2xe y  dx =
0
 
 
2 4 . 3 0 | Differential Equations


x
dx 
x  y2
⇒ 2ye . y = 1+
+ y − 2xe y 0 dy x2 
dy   or = ... (i)
  dx 2y
dy 2xex/ y x
⇒ =  ... (i)
dx 2yex/ y Equation (i) is a homogeneous differential equation.
dy dv
Clearly, the given differential equation is a homogeneous So we put y = vx and = v+
differential equation. As the right hand side of (i) is dx dx
x y dy
expressible as a function of   . So, we put Substituting the value of and in equation (i), we
y x dx
get
dt dx
= v ⇒ x = vy and
dx dy dv 1 + v2
v+x =
dv dx 2v
=v+y in (i), we get
dy dv 1 − v 2
or x =  ... (ii)
dv 2vev − 1 dx 2v
v+y =
dy 2ev Separating the variables in equation (ii), we get
v
dv 2ve − 1
y
⇒= −v 2v dx 2v dx
dy 2ev dv = or dv = −  ... (iii)
1−v 2 x 2
v −1 x
dv 1
⇒ y = − Integrating both sides of equation (iii), we get
dy 2ev 2v 1
⇒ 2yevdv = – dy ∫ v 2 − 1 dv = – ∫ x dx
1
⇒ 2evdv = – dy , y ≠ 0 or log|v2 – 1| = –log|x| + log|C1|
y
Integrating both sides, we get or log|(v2 – 1)(x)| = log|C1|  ... (iv)

v 1 y  y2 
∫ e dv – ∫ y dy + logC
2= logc Replacing v by
x
in equation (iv), we get 
 x2
− 1

 
⇒ 2ev = –log|y| + logc x = ±C1

c or (y2 – x2) = ±C1x or x2 – y2 = Cx


⇒ 2ev = log
y
x dy x + 2y − 3
c  x Example 14: Solve: =
⇒ 2e = log
y
 v =  dx 2x + y − 3
y  y
Sol: Simply by putting x = X + h; y = Y + k were (h, k) will
Example 13: Show that the family of curves for which satisfy the equations x + 2y – 3 =0 and 2x + y – 3 =0 we
the slope of the tangent at any point (x, y) on it is can solve the problem.
x2 + y 2 dy x + 2y − 3
, is given by x2 – y2 = cx =
2xy dx 2x + y − 3

Sol: Here by reading the above problem, we get that Put: x = X + h; y = Y + k


dy x2 + y 2 ⇒ dx = dX; dy = dY
= . Hence by putting y = vx and then
dx 2xy dy dY
integrating both sides we can prove the given equation. ⇒ =
dx dX
We have slope of the tangent Given equation reduces to
x2 + y 2 dy x2 + y 2 dy ( x + h) + 2 ( Y + k ) − 3 X + 2Y + (h + 2k − 3 )
= ⇒ = = = ... (i)
2xy dx 2xy dx 2 ( X + h) + ( Y + k ) − 3 2X + Y + ( 2h + k − 3 )
M a them a ti cs | 24.31

Choose h and k such that Substituting the value of X and Y in (iv), we get
h + 2k – 3 = 0; and 2h + k – 3 = 0 x+ y −2
⇒ = c2
(x − y)
3
⇒ h=1;k=1

Equation (i) becomes [ X = x – h = x – 1; Y = y – 1]

dY X + 2Y
=  ... (ii) Example 15: Solve the following differential equation:
dX 2X + Y
dy 2
(x2 – 1) + 2xy =
Put: Y = VX dx 2
x −1
dY dV dy
⇒ = V+X Sol: First reduce this into the form of Q and
+ Py =
dX dX dx
then using the integration factor i.e. e∫
Pdx
we can solve
Now equation (ii) becomes:
this.
dV X + 2VX 1 + 2V
V+X = = dy 2
dX 2X + VX 2+V We have, (x2 – 1) + 2xy =
dx 2
x −1
dV 1 + 2V 1 − V2 dy 2x 2
⇒ X= V
−= ⇒ + y= …..(i)
dX 2+V 2+V dx x2 – 1 (x − 1)2 
2

Separating the variables, we have


This is a linear differential equation in y.
2+V dX 2x 2
⇒ dV = Here, P = ,Q=
1−V 2 X
(x )
2 2
x −1 2
−1
Integrating, we get 2x
∫ dx
2 –1)
I.F. = e∫
Pdx x2 −1
2 VdX =e = elog(x = x2 – 1
∫ 1 − V2 dV + ∫ 1 − V2 dV =
∫X
∴ The solution of (i) is
1 1+V 1
⇒ 2. log
2
– log(1 − V 2 ) = logX + logc
1−V 2
y(I.F.) = ∫ ( Q × I.F ) dx + C
∫ (x )
2 2
y.(x2 – 1) = −1 . dx + C
1 + V 
( )
2
⇒ 2log   – log(1 – V )= 2logcX
2
x2 − 1
 1 − V 
1
  1 + V 2 = 2∫ dx + C
1  2
x −1
⇒ log   ×  = log(cX)2
 1 − V  1 − V 2 
 
1 x −1
2 = 2. log +C
1 + V  1 2 x +1
⇒   × = (cX)2
1 − V  1 − V2 ( ) ⇒ y(x2 – 1) = log
x −1
+C
1+V x +1
⇒ = c2 X 2  ... (iii)
(1 − V )
3
 1  x −1 
⇒ y=   log + C
2
 x −1 x +1 
Putting the value of V in (iii), we have
X+Y
⇒ X 2 = c2 X 2  ... (iv)
(X − Y)
3

 Y
∴ V =
 X
2 4 . 3 2 | Differential Equations

JEE Main/Boards

Exercise 1 dy
Q.13 = ex–y + x3e–y
dx
Q.1 Write the order and degree of the differential
dy 1 + cos2y
 dy  Q.14 + 0
=
equation x – cos   =0 dx 1 − cos2x
 dx 
dy 1 − y2 dy
Q.2 Solve the differential equation + 0
= Q.15 x + x2 + y 2 + x2 y 2 + xy =0
dx 1 − x2 dx

Q.3 Write the order and degree of the differential dy x2 y


Q.16 =
dy  dy  dx x3 + y 3
equation + sin   = 0
dx  dx 
dy
Q.17 = sin3x.cos3x + xex
Q.4 How will you proceed to solve the differential dx
dy
equation = 1 + x + y + xy?
dx Q.18 (1 – x2)dy + xydx = xy2dx

Q.5 Find the integrating factor for solving the differential


equation dy
Q.19 x + y = y2
dx
(1 + y2) dx = (tan–1y – x) dy
Q.20 (x – y3)dy + ydx = 0
dr
Q.6 Solve the differential equation = cosθ
dθ dy  dy 
Q.21 y – x = a  y 2 + x2  , where x = a, y = a.
dy dx  dx 
Q.7 To solve the differential equation + 2y = 6ex,
how will you proceed? dx
dy 2
Q.22 xlogx + y = log x
dx x
Q.8 Prove that, the differential equation that represents
all parabolas having their axis of symmetry coincident
Q.23 edy/dx = x + 1, y(0) = 4
with the axis of x is yy 2 + y12 =
0.
π
Q.24 y’ + 2y2 = 0, y(0) =
Q.9 Form the differential equation representing the 2
family of curves y = Acos2x + Bsin2x, where A and B
are constants. π
Q.25 xy’ + y = xcosx + sinx, y   = 1
2
B
Q.10 Prove that, the function y = Ax + is a solution
x dy dy
Q.26 y2 + x2 = xy , given that when x = 1, y = 1
d2 y dy dx dx
of the differential equation: x2 +x −y = 0
dx 2 dx

Q.11 Prove that, the differential equation of which Q.27 (1+sinx2x)dy + (1+y2)cos x dx=0,given that when
π
x= ,y=0
dy 2
1 + 8y2tanx = cy2 is a solution is cos2x = 4y3
dx
Q.28 xydy = (y + 5)dx, given that y(5) = 0
Q.12 Form the differential equation of the family of
curves y = AeBX Q.29 (x + 2)dx = (x2 + 4x + 9)dy, given that y(0) = 0
M a them a ti cs | 24.33

Exercise 2 Q.6 The solution of the differential equation


d2 y
(x + 2y3) = y is:
Single Correct Choice Type dx2
x x
Q.1 The general solution of the differential equation, y’ (A) = y+c (B) = y 2 + c
+ yφ’(x) – φ(x).φ’(x) = 0 where φ(x) is a known function is: y 2 y

(A) y = ce–φ(x) + φ(x) – 1 x2 y


(C) = y 2 + C (D) = x2 + c
y x
(B) y = ce+φ(x) + φ(x) – 1
(C) y = ce–φ(x) – φ(x) + 1
Q.7 A normal is drawn at a point P(x, y) of a curve.
(D) y = ce–φ(x) + φ(x) + 1 It meets the x-axis and y-axis in the points A and B
1 1
dy respectively such the + = 1, where ‘O’ is the
Q.2 The differential equation y + x = C, where C is OA OB
dx origin. The equation of such a curve passing through
any arbitrary constant represents:
(5, 4) denotes:
(A) A set of circles with centre on x-axis.
(A) A line. (B) A circle.
(B) A set of circles with centre on y-axis.
(C) A parabola. (D) Pair of straight line.
(C) A set of concentric circles.
(D) A set of ellipses. Q.8 The latus rectum of the conic passing through the
origin and having the property that normal at each
Q.3 The differential equation of all parabolas having point (x, y) intersects the x-axis at ((x + 1), 0) is:
their axis of symmetry coinciding with the axis of x is: (A) 1 (B) 2 (C) 4 (D) None of these
2
d2 y dy d2 y  dy 
(A) y + 0
= (B) x +  = 0 Q.9 The solution of the differential equation
dx  dx 
2
dx2 dy
2 dy π
d2 y dy  2x2y = tan(x2y2) – 2xy2, that given y(1) = is
(C) y +  = 0 (D) None of these dx 2
 dx 
2
dx (A) sinx2y2 = ex–1 (B) sin(x2y2) = x

Q.4 The solution of the differential equation, (C) cosx2y2 + x = 0 (D) sin(x2y2) = e.ex

1 + y2 
 dy 
xy  =
 dx 
 2 (
 1= x + x
 1 + x 
2
) Q.10 A wet porous substance in the open air loses its
moisture at a rate proportional to the moisture content. If
a sheet hung in the wind loses half its moisture during the
given that when x = 1, y = 0 is: first hour, then the time when it would have lost 99.9% of
π its moisture is: (weather condition remaining same)
(A) log 1 + y 2 = logx + tan–1x –
2 (A) More then 100 hours
1 + y2 π
(B) log = 2tan–1x – (B) More than10 hours
x2 2
(C) Approximately 10 hours
1 + y2 π
(C) log = – 2tan–1x (D) Approximately 9 hours
x2 4
(D) None of these c
Q.11 If y = (where c is an arbitrary constant)
log x
Q.5 Given, y = 1 + cosx and y = 1 + sinx are solution of is the general solution of the differential equation
d2 y dy y x x
the differential equation +y = 1, then its solution = + φ   then the function φ   is
dx2 dx x y y
will be also:
x2 x2 y2 y2
(A) y = 2(1 + cosx) (B) y = 2 + cosx +sinx (A) (B) − (C) (D) –
2
(C) y = cosx – sinx (D) y = 1 + cosx + sinx
y y2 x2 x2
2 4 . 3 4 | Differential Equations

Q.12 A tank contains 10000 liters of brine in which Q.2 A solution of the differential equation
10 kg of salt is dissolved initially at t = 0. Fresh brine  dy 
2
dy
containing 20 gms of salt per 100 liters keeps running   −x +y =0 is: (1999)
into the tank at the rate of 50 liters per minute. If the  dx  dx
mixture is kept stirring uniformly, then the amount of (A) y = 2 (B) y = 2x
salt (in kgs) present in the tank at the end of 10 minutes,
is (Assume that there is no overflow of brine is the bank) (C) y = 2x – 4 (D) y = 2x2 – 4

dy
(A) 11.5 (B) 11.15 (C) 10.1 (D) 10.5 Q.3 If y(t) is a solution of (1 + t) 1 and
− ty =
dt
y(0) = –1, then y(1) is equal to:- (2003)
Q.13 Which of the following differential equation is not 
1 1 1 1
of degree 1? (A) − (B) e + (C) e – (D)
2 2 2 2 2
(A) x3y2+ (x+x)2 y1 + exy3=sinx
2 + sin x  dy 
(B) y1/2
2 + (sinx)y1 + xy = x
Q.4 If y = y(x) and   = –cosx, y(0) = 1, then
y + 1  dx 
(C) y1 + y = x + 1 π
y   equals  (2004)
2
(D) None of these
1 2 1
(A) (B) (C) – (D) 1
dy xy + y 3 3 3
Q.14 If = , then the solution of the differential
dx xy + y
equation: dy
Q.5 If = y(log y − log x + 1) , then the solution of
(A) y = xe + c x
(B) y = e + c
x dx

(C) y = Axe x
(D) y = x + A
the equation is (2005)

y y
Q.15 The degree of the differential equation (A) log   = cx (B) log   = cy
2 −3 x x
 d3 y  d2 y dy
 3 + 4 −3 +5 =0 is
 dx  dx 2 dx y y
  (C) y log   = cx (D) x log   = cy
x x
(A) 1 (B) 2 (C) 3 (D) None of these
Q.6 A right circular cone with radius R and height H
Q.16 The differential equation for all parabolas each contains a liquid which evaporates at a rate proportional
of which has a latus rectum ‘4a’ and whose axes are to its surface area in contact with air (proportionality
parallel to x-axis is: constant = k > 0). Find the time after which the cone is
(A) of degree 2 and order 1 (B) of order 2 and degree 3 empty. (2003)

3
d2 x d2 x dy  Q.7 If length of tangent at any point on the curve y =
(C) 2a = 1 (D) 2a +  = 0
dy 2 dy 2
 dx  f(x) intercepted between the point and the x-axis is of
length 1. Find the equation of the curve. (2005)

Previous Years’ Questions Q.8 If a curve y = f(x) passes through the point (1, –1)
and satisfies the differential equation,
Q.1 The order of the differential equation whose general  1
x +c y(1 + xy) dx = xdy, then f  −  is equal (2016)
solution is given by y = (c1 + c2)cos(x + c3) – c 4 e 5 .  2
where c1, c2, c3, c4, c5 are arbitrary constants, is  (1998) 4 2 4 2
(A) − (B) (C) (D) −
5 5 5 5
(A) 5 (B) 4 (C) 3 (D) 2
M a them a ti cs | 24.35

Q.9 Let y(x) be the solution of the differential equal (x Q.12 Solution of the differential equation cos x dy =
dy π
log x) = + y 2xlogx, (x ≥ 1) . Then y(e) is equal to y(sin x – y) dx, 0 < x <  (2010)
dx 2
 (2015)
(A) y sec x = tan x + c (B) y tan x = sec x + c
(A) e (B) 0 (C) 2 (D) 2e (C) tan x = (sec x + c)y (D) sec x = (tan x + c)y

dy
Q.10 If y = sec(tan-1x), then at x = 1 is equal to : Q.13 The differential equation which represents the
dx c x
 (2013) family of curves y = c1e 2 , where c1 and c2 are arbitrary
1 1 constants is (2009)
(A) (B) (C) 1 (D) 2
2 2 (A) y’ = y2 (B) y” = y’ y

d2 x (C) yy” = y’ (D) yy” = (y’)2


Q.11 equals (2011)
dy 2
Q.14 The solution of the differential equation
−1
 d2 y   dy −3  d y   dy 
2 −2
dy x + y
(A) −   (B)   = satisfying the condition y (1) = 1 is  (2008)
 dx2   dx   dx2   dx  dx x
   
−1 (A) y = log x + x (B) y = x log x + x2
 d2 y   dy −3  d2 y 
(C) −   (D)  
 dx2   dx   dx2  (C) y = xe(x−1) (D) y = x log x + x
   

JEE Advanced/Boards

Exercise 1
 y y  y y  dy
2
Q.6 Solve:  x cos + y sin  y =  y sin x − x cos x  x dx
dy x + xy  x x  
Q.1 (i) Solve =
dx x2 + y 2
Q.7 Find the curve for which any tangent intersects
(ii) (x3 – 3xy2)dx = (y3 – 3x2y)dy
the y-axis at the point equidistant from the point of
tangency and the origin
Q.2 Find the equation of a curve such that the projection
of its ordinate upon the normal is equal to its abscissa. Q.8 Solve: (x – y)dy = (x + y + 1)dx

Q.3 The light rays emitting from a point source situated dy x + 2y − 3


at origin when reflected from the mirror of a search light Q.9 Solve: =
dx 2x + y − 3
are reflected as beam parallel to the x-axis. Show that
the surface is parabolic, by first forming the differential dy y − x + 1
equation and then solve it. Q.10 Solve: =
dx y + x + 5

Q.4 The perpendicular from the origin to the tangent at dy x + y +1


any point on a curve is equal to the abscissa of the point Q.11 Solve: =
dx 2x + 2y + 3
of contact. Find the equation of the curve satisfying the
above condition and which passes through (1, 1) 2 ( y + 2)
2
dy
Q.12 Solve: =
dx
( x + y − 1)
2
Q.5 Use the substitution y2 = a – x to reduce the
dy
equation y3. + x + y2 =0 to homogeneous form and
dx
hence solve it.
2 4 . 3 6 | Differential Equations

Q.13 Show that the curve such that the distance dy


Q.2 y = f(x) satisfies the differential equation −y =
between the origin and the tangent at an arbitrary dx
point is equal to the distance between the origin and cosx – sinx with the condition that y is bounded when
y
± tan−1 x → +∞. The longest interval in which f(x) is increasing
the normal at the same point, x2 + y 2 =
ce x
in the interval
dy π π  π  π 5π   π
Q.14 If solution of differential equation
– y = 1 – e–x (A)  ,  (B)  0,  (C)  ,  (D)  0, 
dx 3 2  2 2 6   6
and y(0) = y0 has a finite value. When x → ∞, then
find y0. Q.3 The real value of m for which the substitution,
dy
y = um will transform the differential equation 2x4y +
Q.15 Let y = y(t) be a solution to the differential dx
y y4 = 4x6 into a homogeneous equation is:
equation y’ + 2t y = t2, then find lim
t→∞ t
(A) m = 0 (B) m = 1
dy x 1
Q.16 Solve: + y= (C) m = 3/2 (D) m = 2/3
dx 1 + x2 2x 1 + x2 ( )
dy Q.4 The solution of the differential equation
Q.17 Solve: (1 – x2) + 2xy = x(1 – x2)1/2
dx dy 1 1
x2 .cos − y sin =–1 , Where y → –1 as x → ∞ is
dx x x
Q.18 (i) Find the curve such that the area of the
trapezium formed by the co-ordinate axes, ordinate of 1 1 x +1
(A) y = sin –cos (B) y =
an arbitrary point and the tangent at this point equals x x 1
x sin
half the square of its abscissa. x
1 1 x +1
(ii) A curve in the first quadrant is such that the area of (C) y = cos + sin (D) y =
x x 1
the triangle formed in the first quadrant by the x-axis, x cos
a tangent to the curve at any of its point P and radius x
vector of the point P is 2 square units. If the curve Q.5 The equation of a curve for which the product of
passes through (2, 1) find the equation of the curve. the abscissa of point P and the intercept made by a
normal at P on the x-axis equals twice the square of the
dy radius vector of the point P, and passes through (1, 0) is:
Q.19 Solve: x(x –1) –(x–2)y = x3(2x – 1)
dx (A) x2 + y2 = x4 (B) x2 + y2 = 2x4
(C) x2 + y2 = 4x4 (D) None of these
Exercise 2
Q.6 The order and the degree of the differential
Single Correct Choice Type equation whose general solution is, y = c(x – c)2, are
respectively:
 π
Q.1 A curve passes through the point  1,  , and its (A) 1, 1 (B) 1, 2 (C) 1, 3 (D) 2, 1
 4
y y
slope at any point is given by − cos2   , Then the Q.7 the degree of the differential equation
x x 2
d2 y  dy   d2 y 
curve has the equation, y is equal to: + 3  = x n   is
dx2  dx   dx2 
 
 e
(A) y = xtan–1   n x  (A) 1 (B) 2 (C) 3 (D) None of these
 

(B) y = xtan–1(log+2) Q.8 Orthogonal trajectories of family of parabolas


y2 = 4a(x + a) where ‘a’ is an arbitrary constant is
1  e
(C) y = tan–1   n x  (A) ax2 = 3cy (B) x2 + y2 = a2
x  
x

(D) None of these (C) y = ce 2a (D) axy = c2
M a them a ti cs | 24.37

Q.9 If the function y = e4x + 2e–x is a solution of the Previous Years’ Questions
d3 y
dy
– 13
3 dx Q.1 Le f(x) be differentiable on the interval (0, ∞) such
differential equation dx = K, then the value
y t2 f(x) − x2 f(t)
of K is:- that f(1) = 1, and lim = 1 for each x > 0.
t→x t−x
(A) 4 (B) 6 (C) 9 (D) 12 Then f(x) is: (2007)
1 2x2 1 4x2
(A) + (B) − +
x dy f (y / x) 3x 3 3x 3
Q.10 Solution set of the equation −y =x.
dx f '(y / x) 1 2 1
x (C) − + (D)
y x x2 x
(A) f   = cy (B) f   = cx
y x
dy 1 − y2
Q.2 The differential equation = determines
y dx y
(C) f   = cxy (D) None of these
x a family of circles with (2007)
(A) Variable radii and a fixed center at (0, 1)
dy x2 + 2xy + y 2
Q.11 = . Let C1 and C2 be two of it’s (B) Variable radii and fixed center at (0, –1)
dx x2 − 2xy + 2y 2
(C) Fixed radius of 1 and variable center along the x-axis
solutions. C1 passes through, A(1, 2), and line through
origin and A meets C2 at B. Then slope of the tangent (D) Fixed radius of 1 and variable center a long the y-axis
to the curve C2 at B is:
5 9 9 Q.3 Let y = f(x) be a curve passing through (1, 1) such
(A) (B) (C) – (D) None of these that the triangle formed by the coordinates axes and the
9 5 5
tangent at any point of the curve lies in the first quadrant
 dy  and has area 2 unit, from the differential equation and
Q.12 The solution of the differential equation log  
determine all such possible curves. (1995)
= 4x – 2y – 2, y = 1 when x = 1 is:-  dx 
Q.4 A and B are two separate reservoir of water.
(A) 2e2y=
+2
e4x + e2 Capacity of reservoir A is double the capacity of
reservoir B. Both the reservoirs are filled completely
(B) 2e2y=
−2
e4x + e4 with water, their inlets are closed and then the water
is released simultaneously from both the reservoirs.
(C) 2e2y=
+2
e4x + e4 The rate of flow of water out of each reservoir at any
instant of time is proportional to the quantity of water
(D) 3e2y=
+2
e3x + e4 in the reservoir at the time. One hour after the water
1
is released the quantity of water in reservoir A is 1
2
Multiple Correct Choice Type times the quantity of water in reservoir B. After how
many hours do both the reservoirs have the same
Q.13 The general solution of the differential equation, quantity of water? (1997)
 dy  y
x  = y n   is: Q.5 Let u(x) and v(x) satisfy the differential equation
 dx  x
du dv
+ p ( x)u =
f ( x ) and + p ( x ) =,
g ( x ) where P(x),
(A) y = xe1 –cx (B) y = xe1 + cx dx dx
f(x) and g(x) are continuous functions. If u(x1) > v(x1)
(C) y = xe.xecx (D) y = xecx for some x1 and f(x) > g(x) for all x > x1, prove that any
where c is an arbitrary constant. point (x, y) where x > x1 does not satisfy the equations
y = u(x) and y = v(x)  (1997)

Q.6 A curve passing through the point (1, 1) has the


property that the perpendicular distance of the origin
from the normal at any point P of the curve is equal
to the distance of P from the x-axis. Determine the
equation of the curve.  (1999)
2 4 . 3 8 | Differential Equations

Q.7 A country has food deficit of 10%. Its population Q.13 The function y = f (x) is the solution of the
grows continuously at a rate of 3% per year. Its annual dy xy x 4 + 2x
food production every year is 4% more than that of the differential equation + = in (-1, 1)
dx x2 − 1 1 − x2
last year. Assuming that the average food requirement
3
per person remains constant, prove that the country 2
will become self-sufficient in food after n years, where satisfying f(0) = 0. Then f(x)dx is 
n is the smallest integer bigger than or equal to
∫ (2014)
3

ln10 − ln9 2
 (2000)
ln (1.04 ) − ( 0.03) π 3 π 3
(A) − (B) −
3 2 3 4
Q.8 Let f: R → R be a continuous function, which satisfies
π 3 π 3
x
(C) − (D) −
f(x) = ∫ f ( t ) dt . Then the value of f(log 5) is …….  (2009) 6 4 6 2
0
x  π
Q.14 A curve passes through the point  1,  ,
Q.9 If the function f(x) = x3 + e 2 and g(x) = f–1(x), then
 6
Let the slope of the curve at each point (x, y) be
the values of g’(1) is ……………….. (2009)
y y
+ sec   , x > 0 , x > 0. Then the equation of the
x x
Q.10 Let y’(x) + y(x)g’(x) = g(x)g’(x), y(0) = 0, x ∈ R, where curve is (2013)
df ( x )
f’(x) denotes and g(x) is a given non-constant y 1
dx (A) sin =  logx +
x
  2
differentiable function on R with g(0) =g(2) = 0. Then the
y
value of y(2) is ………….. (2011) (B) cosec =  logx + 2
x

Q.11 A solution curve of the differential equation  2y 


(C) sec  =  logx + 2
dy 2  x 
(x2 + xy + 4x + 2y + 4) - y = 0, x > 0, passes through
dx  2y  1
the point (1, 3). Then the solution curve dy dx  (2016) (D) cos  =  logx +
 x  2
(A) Intersects y = x + 2 exactly at one point.
(B) Intersects y = x + 2 exactly at two points Q.15 If y(x) satisfies the differential equation y’ - ytanx
= 2x secx and y(0) = 0, then  (2012)
(C) Intersects y = (x + 2)2
π π2  π  π2
(D) Does NOT intersect y = (x + 3)2 (A) y   = (B) y '   =
4 8 2  4  18
Q.12 Consider the family of all circles whose centers
 π  π2  π  4 π 2π2
lie on the straight line y = x. If this family of circles (C) y   = (D) y '  =  +
is represented by the differential equation Py” + 3 9 3 3 3 2
Qy’ + 1 = 0, where P, Q are functions of x, y and y’
 dy d2 y  Q.16 Let f: [1, ∞) → [2, ∞) be a differentiable function
 here
= y' = , y"  , then which of the following
 dx dx2 
x
 such that= = 3xf(x) − x3 for all x ≥ 1 ,
f(1) 2. If 6 ∫ f(1)dt
1
statements is (are) true ? (2015)
then the value of f(2) is (2011)
(A) P = y + x
(B) P = y − x Q.17 Let f be a real-valued differentiable function on
R (the set of all real numbers) such that f(1) = 1. If the
(C) Q = 1 + y1 + y12 y-intercept of the tangent at any point P(x, y) on the
(D) P − Q = x + y − y’ − (y’)2 curve y = f(x) is equal to the cube of the abscissa of P,
then the value of f(–3) is equal to (2010)
M a them a ti cs | 24.39

Q.18 Interval contained in the domain of definition of 1 2 3 1


non-zero solutions of the differential equation (x – 3)2 Statement-II: y(x) is given by = − 1−
y x x2
y’ + y = 0 (2009)
 (2008)
Q.19 Let a solution y = y(x) of the differential equation (A) Statement-I is True, statement-II is True; statement-II
2 is a correct explanation for statement-I
x x2 − 1 dy − y =
y 2 − 1 dx 0 satisfy
= y(2)
3 (B) Statement-I is True, statement-II is True; statement-II
is NOT a correct explanation for statement-I.
 π
= sec y(x) sec  sec−1 x −  and
Statement-I: y(x) =
 6  (C) Statement-I is True, statement-II is False
(D) Statement-I is False, statement-II is True

PlancEssential Questions
JEE Main/Boards JEE Advanced/Boards

Exercise 1 Exercise 1
Q.9 Q.14 Q.20 Q.21 Q.3 Q.6 Q.14 Q.18

Q.26 Q.19

Exercise 2 Exercise 2
Q.3 Q.4 Q.8 Q.10 Q.1 Q.4 Q.5 Q.9

Q.11 Q.16 Q.14 Q.11 Q.13

Previous Years’ Questions Previous Years’ Questions


Q.3 Q.5 Q.8 Q.2 Q.4 Q.7
Q.10
2 4 . 4 0 | Differential Equations

Answer Key

JEE Main/Boards
Exercise 1
Q.1 Order = 1; Degree = 1 Q.2 sin–1y + sin–1x = c

Q.3 Order = 1, degree is not defined Q.4 Separate the variables after factorizing
tan−1 y
Q.5 e Q.6 r = sin θ + c

Q.7 (I.F.)y = ∫ (I.F.).Q Q.9 y” = –4Acos2x – 4Bsin2x = -4y


2
d2 y
 dy  x4
Q.12 y =  Q.13 ey = ex + +c
 dx  4
2
dx
1  1 + x2 – 1 
Q.14 tany – cotx = c Q.15 x2 + 1 + y 2 + 1 + log  +c =0
2  1 + x2 + 1 
 
x3 (cos x)6 (cos x)4
Q.16 log|y| = +c Q.17 y = – + ex(x + 1) + c
3y 3 6 4

1
Q.18 –[logy – log(y – 1)] = – log(1 – x2) + c Q.19 y – 1 = xy
2
 x  2
y3 2 y
  a + 1 
Q.20 x = + Q.21 =  1   2 
4 y 
1   x +   a − 1 
y−  a
a 
–2log | x | 2
Q.22 ylog|x| = − +c Q.23 y = (x + 1)log|x + 1| – x + 4
x x
1 2
Q.24 = 2x + Q.25 y = sinx
y π
y π
Q.26 =log|y| + 1 Q.27 tan–1(sinx) + tan1y =
x 4
1
Q.28 y – 5log|y + 5| = log|x| – 6log5 Q.29 y = log(x2 + 4x + 9) – log3
2

Exercise 2
Single Correct Choice Type
Q.1 A Q.2 A Q.3 C Q.4 B Q.5 D Q.6 B
Q.7 B Q.8 B Q.9 A Q.10 C Q.11 D Q.12 C
Q.13 D Q.14 C Q.15 B Q.16 C

Previous Years’ Questions


dy
Q.1 C Q.2 C Q.3 A Q.4 A Q.5 A Q.6 T =
dx
d2 y dy
Q.7 x2 +x = ±x + c Q.8 C Q.9 C Q.10 A Q.11 C
dx 2 dx

Q.12 D Q.13 D Q.14 D


M a them a ti cs | 24.41

JEE Advanced/Boards
Exercise 1
 1 x + 2y 
Q.1 (i) c(x – y)2/3 (x2 + xy + y2)1/6 = exp  tan−1  where exp x = e
x
(ii) (x2 + y2)2 = (x2 – y2)c
 3 x 3 

y 2 ± y y 2 − x2 c2
Q.2 = log  y ± y 2 − x2  . , where same sign has to be taken
x2   x3
1 a
Q.4 x2 + y2 – 2x = 0 Q.5 log | x2 + a2 | – tan−1   = c, where a = x + y2
2 x
y
Q.6 xycos = c Q.7 x2 + y2 = cx
x
 1
 y+ 
c tan−1  2 
 x+ 1 
 
Q.8 e  2
Q.9 (x + y – 2) = c(y – x)3

−1 y +3 4
Q.10 tan + logc (y + 3)2 + (x + 2)2 =
0 Q.11 x + y + = ce3(x–2y)
x+2 3
y +2
–2 tan−1 1 1
Q. 12 c = e x −3 = (y +2) Q.14 Q.15
2 2

2 1  1  2 1 1 + x2 − 1
Q.16 y 1 + x =c + log  tan arctanx  another form is y 1 + x =c + log
2  2  2 x

Q.17 y = c(1 – x2) + 1 − x2 Q.18 (i) y = cx2 +x (ii) xy =2

Q.19 y(x – 1) = x2(x2 – x + c)

Exercise 2

Single Correct Choice Type


Q.1 A Q.2 B Q.3 C Q.4 A Q.5 A Q.6 C
Q.7 D Q.8 C Q.9 D Q.10 B Q.11 B Q.12 C

Multiple Correct Choice Type


Q.13 B, C

Previous Years’ Questions


1
Q.1 A Q.2 C Q.3 x + y = 2, xy = 1 Q.4 log3/ 4   Q.6 x2+y2=2x
2
Q.8 0 Q.9 2 Q.10 0 Q.11 A, C Q.12 B, C Q.13 B

Q.14 A Q.15 A, D Q.16 6 Q.17 9 Q.19 C


2 4 . 4 2 | Differential Equations

Solutions

JEE Main/Boards dy
Sol 7: + 2y = 6ex
dx
Exercise 1
This is a linear equation
 dy  dy
= cos−1 x ∴ Integrating factor = e∫
2 dx
Sol 1: x = cos   ⇒ = e2x
 dx  dx
dy
∴ Degree = 1, order = 1 ∴ e2x dx + 2e2xy = 6e3x

2x 6 3x
dy 1 − y2 ⇒ ∫ d(e y) = ∫ 6e3x dx ⇒ e2xy = e +c
Sol 2: =– 3
dx 1 − x2
∴ y = 2ex + ce–2x
dy dx
⇒ ∫– = ∫ Sol 8: Ellipse with their axis coincide with x-axis
1 − y2 1 − x2

x2 y2 2x 2yy '
⇒ –sin–1y = sin–1x + c or sin–1x + sin–1y = c + 1 ⇒
= + 0
=
2
a2
b 2
a b2

dy  dy  b2 x –b2  y − xy ' 
Sol 3: + sin   = 0 ∴ y’ = – ⇒ y” =  
dx  dx  a2 y a2  y 2 

Highest order derivative = 1 –b2  x  –b2  b2 x 


⇒ yy” = 1 – y ' = +  y'
Degree is not defined as differential coefficient is not a2  y  a2  a2 y 
free from radical and fraction.
–b2
∴ yy” + (y’)2 =
a2
dy dy
Sol 4: = 1 + x + y(1 + x) or = (1 + x) (1 + y) For parabola
dx dx
Equation will be y2 = 4ax
(Separation of variables method)
2a
dy ⇒ 2yy’ = 4a or y’ =
⇒∫ = ∫ (1 + x)dx y
(1 + y)
or 2a = yy’
x2
⇒ log(1 + y) = x + +c
2 ⇒ (y’)2 + yy” = 0

Sol 5: (1 + y2)dx = (tan–1y – x)dy Sol 9: y = Acos2x + Bsin2x

dx 1 tan−1 y y’ = –2Asin2x + 2Bcos2x


+ x =
dy (1 + y 2 ) 1 + y2 y” = –4Acos2x – 4Bsin2x = –4y
 1 
∫   dy
 B
∴ Integrating factor = e  1 + y2  −1 y
= etan Sol 10: y = Ax +
x
B
y’ = A –
dr x2
Sol 6: = cosq
dθ 2B
y” =
x3
∫=
dr ∫ cos θdθ
∴ x2y” + xy’ – y
r = sinθ+ c
M a them a ti cs | 24.43

x2 (2B)  B   B dy
= + xA −  –  Ax +  ⇒ xy = – (1 + x2 )(1 + y 2 )
x 3
 x2   x dx

2B B B y (1 + x2 )
= + Ax – – Ax – =0 ⇒∫ dy = ∫ dx
x x x 1 + y2 x
Hence proved.
⇒ take 1 + y2 = t, differentiating both sides
2ydy = dt
Sol 11: 1 + 8y2tanx = cy2
y2(c – 8tanx) = 1 1 dt 1 1/2
1 + y2
2∫ t 2
I1 = = 2t = t
=
dy
∴ 2y(c – 8tanx) + y2(–8sec2x) = 0
dx
1 + x2 (1 + x2 )
1 dy ∫ dx = ∫ dx
2y = 8y2sec2x x x 1 + x2
2 dx
y
dy dy 1 x
∴ = 4y3sec2x or cos2x = 4y3 =
dx dx ∫ 2
dx + ∫ dx
x 1 + x 1 + x2
Sol 12: y = AeBx x
I3 = ∫ dx
= 1 + x2
⇒ y’ = ABe Bx
1 + x 2

⇒ y” = AB e 2 Bx
1
⇒ yy” = A2B2e2Bx = (y’)2 I2 = ∫ dx
x 1 + x2
2
d2 y  dy 
∴y =  Put x = tanθ; dx = sec2θ dq
 dx 
2
dx
sec2 θdθ
dy = ∫ tan θ sec
= ∫ cosecθdθ = log |cosecθ – cotq|
Sol 13: = ex – y + x3e–y θ
dx
2
1 − cos θ 1  1 − cos θ 
⇒ ∫ ey=
dy x 3
∫ (e + x ) dx = log = log  
sin θ 2  sin θ 
x4
⇒ ey = ex + +c
4 1 (1 − cos θ)2 1  1 − cos θ 
= log = log  
2 (1 − cos θ) 2
2
 1 + cos θ 
dy 1 + cos2y
Sol 14: + =0
dx 1 − cos2x  1 + x2 − 1 
1  sec θ − 1  1
dy dx or I2 = log   = log  
⇒∫ =∫ 2  sec θ + 1  2  1 + x2 + 1 
1 + cos2y cos2x − 1  

dy dx   2  
⇒∫ = –∫ 2   x2 + 1  + 1 log  1 + x − 1  + c 
2cos y 2 2
2sin x ∴ y = + 1 –  
  2  1 + x2 + 1  
   
⇒ ∫ sec2 y dy = – ∫ cosec2 x dx
1  1 + x2 – 1 
⇒ tany = cotx + c or x2 + 1 + y 2 + 1 − log  +c =0
2  1 + x2 + 1 
 
∴ tany – cotx = c

dy
Sol 15: 1 + x2 + y 2 + x2 y 2 + xy =0
dx
dy
(1 + x2 )(1 + y 2 ) + xy =0
dx
2 4 . 4 4 | Differential Equations

dy x2 y 1
Sol 16: = ⇒ –[logy – log(y – 1)] = – log(1 – x2) + c
dx x3 + y 3 2

  x 2  dy
    Sol 19: x + y = y2
dy   y   dx
= 
dx   x 3  dy
 y  +1 ⇒ x = (y 2 − y)
   dx

dy dv dy dx 1 1 
Put x = vy ⇒ 1 = v +y ⇒∫ = ∫ ⇒ –∫ −  dy =logx + c
dx dx 2
y −y x  y (y − 1) 

dy 1  x dv  v2 ⇒ log(y – 1) – log(y) = logx + logc


=  1 −  =
dx v  v dx  v3 + 1 ⇒ (x – 1) = xy

–x dv v3 –1 Sol 20: (x – y3)dy + ydx = 0


= –1=
v dx v 3 + 1 3
v +1
⇒ xdy + ydx = y3dy
(v 3 + 1) dx v3 ⇒ ∫ d(xy) = ∫ y 3dy
⇒∫ dv = ∫ ⇒ + logv = logx + c
v x 3 y4 y3 c
⇒ xy = + c or x = +
3 4 4 y
1 x 
⇒   + logx – logy = logx + c
3 y 
dy  dy 
x3 Sol 21: y – x = a  y 2 + x2 
x 3 dx  dx 
3y3
∴ logy + c = ⇒ y = Ce
3y 3 dy
⇒ (ax2 + x) = y – ay2
dx
dy dy dx
Sol 17: = sin3x cos3x + xex ∴∫
dx y − ay 2
= ∫ ax2 + x
dy = ∫ (sin3 x cos3 x + xex )dx 1 1
2
⇒ ∫ y(1 – ay) dy = ∫ x(ax + 1) dx
= ∫ sinx(1 – cos x)cos3 xdx + xex – ex
1 a  1 a 
Put cosx = t ⇒ –sinxdx = dt ⇒ ∫ + ∫
 dy =  −  dx
 y 1 − ay   x ax +1
= – ∫ (1 – t2 )(t3 )dt + x(ex) – ex    
  1 
5 3
= ∫ (t – t )dt + e (x – 1) 1 1  1
⇒ ∫ –
x

y 
=
1 
dy ∫  x  1   dx
 −
t6 t 4  y −   x +  
= − + ex(x – 1) + c   a   a
6 4
(cos x)6 (cos x)4  1  1
= – + ex(x + 1) + c logy – log  y –  = logx – log  x +  + logc
6 4  a   a

 1  1
Sol 18: (1 – x2)dy + xydx = xy2dx loga – log  a–  = loga log.  a +  + logc
 a  a
(1 – x2)dy = (xy2 – xy)dx = x(y2 – y)dx
a2 + 1
dy x logc = log
∫ (y 2 – y) = ∫ 1 − x2 dx a2 − 1
 x  2
1 1  1 –2x ⇒
y
=   a + 1 
1  
–∫ − dy = – ∫ 1 − x2 dx 1   x +    a2 − 1 
 y (y − 1)  2 y−  a 
a  
M a them a ti cs | 24.45

dy 2 1 1 1
Sol 22: xlogx + y = log x ∴c= = ∴y=
dx x π π  1
2× 2 x + 
2  π
dy 1 2
⇒ + y= 1 2
dx xlogx x2 or = 2x +
1 y π
∫ x log x dx
I. F. = e = e n nx =logx
dy
2 Sol 25: x + y = xcosx + sinx
∴(logx)y = ∫ x2 logxdx dx

1 dy  1  sinx
Put logx = t ⇒ dx = dt Or +   y =cos x +
x dx  x  x
–t
∴(logx)y = 2 ∫e tdt
I. F. = e
1
∫ x dx
=x
u = –t
d(xy)  sinx 
∴ =  cos x + x
dt = – du =+ 2 ∫ euudu = +2[eu(u – 1)] + c dx  x 

∴(logx)y = +2[e–logx(–logx – 1)] + c


∫ d(xy)
∴= ∫ (x cos x + sinx)dx
2
∴(logx)y = – (log | x | +1) + c xy =
x ∫ x cos xdx + ∫ sinxdx
 dx 
dy /dx
= x ∫ cos xdx – ∫  ∫ cos xdx  dx + ∫ sinxdx
Sol 23: e = x + 1, y(0) = 4  dx 

dy = xsinx – ∫ sindx + ∫ sinxdx +c


= log(x + 1) ⇒=
∫ dy ∫ n(x + 1)dx
dx ∴ xy = xsinx + c
 dn(x + 1)  π
∫ dy = log(x + 1) ∫ 1dx – ∫  dx ∫ 1.dx  dx y  = 1
2
x π π
y = xlog(x + 1) – = +c ⇒c=0
∫ x + 1dx + c 2 2

= xlog(x + 1) – x + log(x+1) + c ∴ y = sinx + 0

Or y = (x + 1) log(x+1) – x + c ∴ y = sinx

y(0) = 4 dy dy
Sol 26: y2 + x2 dx = xy
∴4=c dx
∴ y = (x + 1) log(x + 1) – x + 4 y
2

 
dy y2 x
Sol 24: y’ + 2y2 = 0 =
∴ =
dx xy − x 2 y
dy   − 1
= –2y 2 x
dx Let y = vx
1
∴∫– dy = ∫ dx dy dv
2y 2 ∴ = v+x
dx dx
1 1 1 dv v2 dv v2 v
⇒–  – = x + c or =x+c ∴v+ x = or x = –v =
2 y  2y dx v −1 dx v − 1 v −1

π v −1 dx
y(0) = ∴∫ dv = ∫
2 v x
y y  y y 
⇒ v – logv = logx + c or – –log
nn    = logx + c
x x  x x 
2 4 . 4 6 | Differential Equations

when x = 1, y = 1 1 dt 1 11
2∫ t
∴y= = log
nt +t c+ c or y = logn(x22++4x
n(x 4x++9)9)++cc
1 – log1 = log1 + c or c = 1 2 22
yy for x = 0, y = 0
∴= log
= | y| y| +| 1+1
nn
xx 1
∴c=– log9
or y = xlog(ey) 2
22
1 1 (x(x + +4x4x+ +9)9)
∴y= log
nn
22 99
Sol 27: (1 + sin2x)dy + (1 + y2)cosxdx = 0
1
dy cos x Or y = log(x2 + 4x + 9) – log3
⇒∫ = –∫ dx 2
1 + y2 1 + sin2 x

tan–1y = –
dt
(Putting sinx = t) Exercise 2
∫ 1 + t2
∴ cosxdx = dt Single Correct Choice Type

∴ tan–1y = – tan–1sinx + c
Sol 1: (A) y’ + y φ’(x) – φ(x) φ’(x) = 0
π
At x = , y = 0 This is a linear equation
4
φ(x)
I. F. = e∫
φ '(x)dx
π =e
∴ c = tan 1 = –1
4 ∴ ∫ d(eφ(x) .y) φ(x)
= ∫ e φ(x)φ '(x)dx
π
∴ tan–1sinx + tan–1y = ∴ eφ(x)y = tet – ∫ et tdx
4
Let φ(x) = t
Sol 28: xydy = (y + 5)dx
φ’(x)dx = dt
 y  1 t
⇒   dy = dx
eφ(x)y = tet – ∫ e dt +c= tet – et + c = (φ(x) – 1)eφ(x) + c
 y + 5 x
∴ y = (φ(x) –1) + ce–φ(x)
 5  or y = ce– φ(x) + φ(x) – 1
⇒ ∫ 1 −  dy = logx + c
 y +5
dy
Sol 2: (A) y +x =c ⇒ ∫ ydy
= ∫ (c − x)dx
⇒ y – 5log(y + 5) = logx + c dx
since for x = 5, y = 0 y2 x2 x2 y2
= cx – ⇒ – cx + =0
⇒ 0 – 5log5 = log5 + c ⇒ c = –6log5 2 2 2 2
∴ y = 5log(y + 5) + logx – 6log5 ⇒ x2 – 2cx + y2 = 0 or (x – c)2 + y2 = c2
 y +5 x ∴ Circle with centre at (c, 0) and radius c.
y = 5log   + log  
 5  5
Sol 3: (C) Parabola equation y2 = 4ax
or y – 5 log | y + 5 | = log |x| – 6 log 5
dy
∴ 2y = 4a
dx
2
Sol 29: (x + 2)dx = (x2 + 4x + 9)dy  dy  d2 y
or   + y =0
 dx  dx2
(x + 2) 1 2x + 4
∴ dy = ∫ (x2 + 4x + 9) dx ⇒y=
2 ∫ (x2 + 4x + 9)
dx
dy (1 + y 2 )
Sol 4: (B)=
xy (1 + x + x2 )
Put x2 + 4x + 9 = t dx (1 + x )
2

∴(2x + 4)dx = dt y (1 + x2 ) + x
⇒∫ dy = ∫ dx
1 + y2 (1 + x2 )x
M a them a ti cs | 24.47

1 1 1 y2 x2
⇒ log(1 + y 2 )= ∫ dx + ∫ dx ⇒ = –y x– +c
2 x 1 + x2 2 2
1 ⇒(y – 1)2 + (x – 1)2 – 2 = 2c
⇒ log(1 + y 2 ) = log x + tan−1 x + c
2
∴(y – 1)2 + (x – 1)2 = 2 + 2c
For x = 1, y = 0
π at x = 5, y = 4
∴ c = –tan–11 = –
4 23
∴42 + 32 = 2 + 2c or c =
π 2
or log(1 + y2) = 2logx + 2tan–1x –
2 ∴(y – 1)2 + (x – 1)2 = (5)2
2
(1 + y ) π
or log = 2tan–1x – This is circle with centre (1, 1) and radius 5.
x 2 2

dy
Sol 5: (D) y = 1 + cosx Sol 8: (B) X-intercept of normal = y +x =x+1
dx
y = 1 + sinx dy y2
∴y = 1 ; ⇒ = x + c ⇒ y2 = 2(x + c)
using option we can see that dx 2
∴ This curve pass through origin
y = 1 + cosx + sinx is satisfying the equation
dy So c = 0
= cos x − sinx
dx ∴ y2 = 2x

d2 y d2 y ∴ Latus rectum = 2
= –sinx – cosx∴ +y=1
dx2 dx2
dy
Sol 9: (A) 2x2y + 2xy2 = tan[(xy)2]
dy dx
Sol 6: (B) (x + 2y3) =y
dx Put xy = t
dy dy
x + 2y 3 y
= dy dt
dx dx ∴x +y =
dx dx
⇒ ydx – xdy = 2y3dy
 dy  dt
∴ 2xy  x + y  = tanx2y2 or 2t = tant2
ydx − xdy  dx  dx
⇒ = 2ydy
y2 2t
x ∴∫ dt = ∫ dx
∴ = y2 + c tant2
y
Put t2 = u
Sol 7: (B) Equation of normal at P(x, y) 2tdt = du
dx du
(Y – y) = – (X – x) ∴∫ =x+c
dy tanu
dy or logsinu = x + c or logsinxy = x + c
OA = x-intercept = x + y
dx
dx π
OB = y-intercept = y + x for x = 1, y =
dy 2
dy π
1+ ∴ logsin = c + 1; c = –1
1 1 dx = 1 2
∴ + 1 or
=
dy dx dy ∴ sin(xy)2 = ex–1
x+y y+x y +x
dx dy dx

dy
⇒(y – 1) = (1 – x)or ∫ (y − 1)dy =∫ (1 − x)dx
dx
2 4 . 4 8 | Differential Equations

dm Sol 13: (D) (A) order 2 degree 1


Sol 10: (C) = cm
dt (B) y2 = (x – xy – (sinx)y1)2
∴ logm = c1t + c2
∴ order 2 degree 1
at t = 0, m = maximum = M
(C) y1 + y = (x + 1)2
∴ logm = c2
∴ Order 1 degree 1
M
at t = 1 hr, moisture content remains
2 dy xy + y
Sol 14: (C) =
M dx xy + x
∴ log = c1 + c2
2
dy y  x + 1 
M ∴ =  
∴ log – logM = c1 dx x  y + 1 
2
1 (y + 1) (x + 1)
∴ c1 = log ∫ y
dy = ∫
x
dx
2
After t = x hr moisture content remains y + logy = x + logx + c

0.1M ⇒ logyey = logxeyA [ c = logA]
100 – 9Sol.9 99 = 0. 1% =
100 ∴ yey = Axex or y = Axex–y
0.1M
∴ log =xc1 + c2
100 Sol 15: (B) Degree is 2
0.1M 1
∴ log – logm = xc1 = xn
100 2 Sol 16: (C) y2 = 4xa
0.1 dy
log ∴ 2y = 4a
100 dx
∴x= ≈ 96 ≈ 10 hr
1 dy dy
log
2 ∴ y2 = 2y x or y = 2x
dx dx
 x  –y 2 ∴ Order 1 degree 1
Sol 11: (D) Considering option taking φ   =
y x2 dx 1 d2 x 1
2 or = y or =
dy y –y dy 2a dy 2 2a
= –
dx x x2
dv
y = vx ⇒v + x = v – v2
dx
dv
= – v2 ⇒
1 1 Previous Years’ Questions
⇒ x
dx ∫ – v 2 dv = ∫ x dx
1 Sol 1: (C) Given,y = (c1 + c2)cos(x + c1) –
⇒ = logx + c
v
dy  d2 y  dy 2 
 
x x y
= y 2 +   x  … (i)
or = logcx
ncx or y = dx dx
y logcx  dx 
  
2
dy d2 y  dy 
⇒ y=(c1 + c2)cos(x + c3) – c ex.=
y xy +x 
Sol 12: (C) In 10 min. total litres run into tank = 50 × dx dx 2
 dx 
10 = 500 lt 2
d2 y  dy  dy
In 100 litres there is 20 gm salt Now, let c1 + c2 = A, c3 = B,c4 xy +x  −y 0 =c
=
dx 2 dx
  dx
20 × 500
∴ in 500 litres we have = 100 gm = 0. 1 kg ⇒ y = Acos(x + B) – cex … (ii)
100
Initially we have 10 kg of salt
dy
∴ Total salt after 10 minutes = 10 + 0. 1 = 10.1 kg On differential w.r.t. x, we get x  … (iii)
dx
M a them a ti cs | 24.49

Again differentiating w.r.t. x, we get dy


∴ IF = = e–t + log(1 + t)
2
d y dy dy dx
x + 1. + 1. = –Acos(x + B) – cex  … (iv)
dx 2 dx dx = e–t.(1 + t)
d2 y
dy Required solution is
⇒ x +2 … (v)
dx dx 
2
dy
ye–1(1 + t) = (1 + t)dt + c
 xy =Aex + Be− x + x2  dx
⇒  
⇒ Aex + Be− x =
xy – x2  d2 y dy dy
= − =
dx 2 dx dx
Again differenting w.r.t. x, we get
⇒ ye–t(1 + t) = –e–t + c
dy  dy  Since, y(0) = –1
=c.2(x2 + y 2 )  2x + 2y   … (vi)
dx  dx  ⇒–1 e0(1 + 0) = –e0 + c
d3 y dy d2 y c = 0;
⇒ + = + y [from Eq. (v)]
dx3 dx dx2 −1
Putting t = 1, we get y (1) =
2
Which is a differential equation of order 3
dy − cos x(y + 1)
Sol 4: (A) Given, =
Sol 2: (C) (a) y = 2 dx 2 + sinx

dy − cos x
 dy  ⇒ = − dx
⇒ x−y  y + 1 2 + sinx
 dx 
On integrating both sides
On putting in equation, (i),
02 – x(0) + y = 0 ⇒ log(y + 1) =− log (2 + sin x) + log c,

⇒ y = 0 which is not satisfied. When, x = 0, y = 1 ⇒ c = 4


4
 dy  ⇒ y +1=
(b) y = 2x ⇒  2x + 2y  2 + sin x
 dx 
π 4
∴ y  = −1
on putting equation (i),
2 3
(2)2 – x . 2 + y = 0 π 1
⇒ y  =
⇒ 4 – 2x + y = 0 2 3
⇒ y = 2x – 4 which is not satisfied.
dy
(c) y = 2x2 – 4 Sol 5: (A) x = y(log y − log x + 1)
dx
dy
dx dy  y   y 
=
∴    log + 1 
dx  x   x 
On putting in equation (i),
y dy dt
(4x)2 – x . 4x + y = 0 Put = t ⇒ y = xt ⇒ t x
=+
x dx dx
⇒ y = 0 which is not satisfied. ∴ t log dx =
x dt
Therefore, C is the answer. dt dx
⇒ =
t log t x
dy
Sol 3: (A) Given, and y(0) = –1
dx ⇒ log log t =
log x + log c
Which represents linear differential equation of first y
order ⇒ log   =
cx
x
2 4 . 5 0 | Differential Equations

Sol 6: Given, liquid evaporates at a rate proportional to dy d2 y dy


its surface are. ⇒ =± +x +y=
0
dx dx 2 dx
B
⇒ ∝ –S … (i) dy asin(logx) bcos(logx)
x =
⇒ + = ±x + c
dx x x
d2 y dy
We know, volume of cone = +x Put y = sinq
dx 2 dx
dy
⇒ dy = cosqdq ∴x = ±x + c
dx
d2 y dy acos(logx) b sin(logx)
R ⇒x 1.
+= – − = ±x + c
dx 2 dx x x

Again put cosθ = t ⇒ –sinqdθ = dt

t2
–∫ dt = ±x + c
H 1 + t2
d2 y dy
h ∴– x2 +x = ±x + c
 dx 2 dx

d2 y dy
⇒ t – log x2 +x = ±x + c
dx 2 dx
and surface area = pr2
d2 y dy
1 ⇒ x2 +x =±x + c
or V = πr 2h and S = pr2 … (ii) dx 2 dx
3
dy d2 y dy dy
where tanθ = and x + 1. + =tanθ  … (iii) Sol 8: (C) y(1 + xy) dx = xdy ⇒ ydx – xdy + xy2dx = 0
dx dx 2 dx dx
x x x2
From equation (ii) and (iii), we get y 2d   + xy 2dx =
0 ⇒ + c 
= … (i)
y y y
1 3 Since, (1,-1) satisfies the above equation
V= πr cot θ and S = pr2  … (iv)
3 1 1
−1 + =c ⇒ c =−
2 2
On substituting equation (iv) in equation (i), we get
1
Put in (i) x = −
1 dr 2
cot θ3r 2 = –kpr2 1 1
3 dt −
2 + 4 =− 1 ⇒ −1 =−1 − 1
T
d2 y dy y 2 2 2y 2 8
⇒ cotθ x2
dx2
+x
dx
−y=0 = –k ∫ dt
0 1 5 4
⇒ = ; ⇒ y=
⇒ cotq(0 – R) = –k(T – 0) 2y 8 5

⇒ Rcosθ = kT Sol 9: (C)


⇒ H = kT [from Equation (iii)]
dy y
+ = 2 at x = 1; y =0
+ 1 log  x2 + 1 − x 
⇒ T = y. x=
2
dx xlogx
 
1 log(logx)
dy=I.F ∫ = dx e= logx
∴ Required time after which the cone is empty, T = e xlogx
dx
⇒ y(logx) = ∫ 2(logx)dx
dy
Sol 7: Since, the length of tangent = =1
dx ⇒ y(logx)= 2[xlog− x] + c
x − x2 + 1 dy At x=1, c=2 x=e
⇒ = 1 ∴
2
x +1 − x dx
y = 2(e – e) + 2 ⇒ y = 2
M a them a ti cs | 24.51

Sol 10: (A) Sol 13: (D) y = c1e


c2 x
 … (i)
y = sec (tan-1 x)
c2 x
Let tan-1 x = q y ' = c2c1e

x = tan q y’ = c2y  … (ii)


y “ = c2y ‘
From (ii)
2
1+x x y'
c2 =
y
( y ')
2

So, y " = ⇒ yy " = (y')2
1 y

⇒ y = sec q Sol 14: (D) Y = vx

⇒=
y 1 + x2 dy dy
= v+x
dx dx
dy 1
⇒ = .2x dv
dx 2 1 + x2 v+x 1+v
=
dx
At x = 1 dx
⇒ dv =
dy 1 x
Therefore, =
dx 2 v logx + c
∴=
y
⇒ = logx + c
Sol 11: (C) x
 
  Since, y (i) = 1, we have
d  dy  d  1  1 d  dy 
  = = −
2 dy  dx 
y = x log x + x
dy  dx  dy   dy    dy   
   
dx  
   dx 
 dy 
−2
1 d  dy   d2 y   dy −3 JEE Advanced/Boards
=−    = −  
 dx   dy  dx  dx   dx2   dx 
 
  Exercise 1
 dx 

dy x2 + xy
Sol 12: (D) cos x dy = y(sin x – y) dx Sol 1: (i) =
dx x2 + y 2
dy y
= y tanx − y 2 (secx) 1+
dx dy x
=
1 dy 1 dx y
2
− tanx = − secx 1+ 
y 2 dx y x

1 1 dy dt y
Let =t ⇒ − = Put =v
y y 2 dx dx x
dy dt dv 1 + v
− − t (tanx) =
− sec x ⇒ + (tanx)t =
sec x ∴v+x =
dx dx dx 1 + v 2

∫ tan x dx dv 1 − v 3 (1 + v 2 ) dx
=I.F. e= sec x ∴x =
dx 1 + v 2
⇒ ∫ (1 − v3 ) dv = ∫ x
Solution is t ( I.F ) = ∫ ( I.F ) sec x dx
1  1  –3v 2
1 ⇒∫ dv +  –  ∫ dv = logx
sec
= x tan x + c (1 − v 3 )  3  (1 − v 3 )
y
2 4 . 5 2 | Differential Equations

1  1 v+2  1 3 dt
⇒– log(1 – v 4 ) – ∫ logx + c
3 ∫  (1 − v) (v 2 + v + 1) 
⇒  + dv 4 2 (1 − t2 )

1 3 1 1 + t 
– 1 log(1 –= ⇒– log(1 – v4) – × log   = logx + c
v 3 ) logx + c 4 2 2 1 − t 
3

1 3  x2 + y 2 
1 1  2v + 1 + 3  – log(x4 – y4) + logx – n   = logx + c
= – log(1 – v) + ∫   dv 4 4  x2 − y 2 
3 6  v2 + v + 1 
3/ 4
 x2 + y 2 
1 ∴ log   log(x4 – y4)1/4 = logc
= log(x3 – y 3 ) – logx + logx  x2 − y 2 
 
3
Or (x2 + y2) (x2 – y2)–1/2 = c
1 3 dv Or (x2 + y2)2 = (x2 – y2)c
– log(1 – v) ∫
3 6 2 2
 1  3
 v +  +   Sol 2: Projection of ordinate on normal
 2   2 
ycosq
1 1 1
= log(x3 − y 3 ) + log(x − y) – logx + c ycosθ = x
p(x, y)
3 3 3  1 x 

v+  cosθ =
1 1 1 2  2 y
⇒ log(y2 + xy + x2) – logx + × tan–1  
6 3 2 3 3
 
 2 

1 1 1
= log(x3 – y3) + log(x – y) – logx + c
3 3 3 x2 y 2 – x2
1 – sin2θ = ⇒ sinθ =
y2 y2
1 −1 (2y + x)
⇒ tan
3 3x dy y 2 − x2 y
2
∴ tanθ
= = =   –1
= log(x3 – y3)1/3 (x – y)1/3 (y2 + xy + x2)–1/6 + c dx x x
1  2y + x  y = vx
tan−1  
∴(x – y)2/3 (y2 + xy + x2)1/6 = e 3  3x 
dv
v + x= v2 − 1
2 dx
y
1 − 3  1
(ii)
dy
= x ∴ ∫ dv = logx + c
dx  y 3 y v2 − 1 – v
  – 3   2 
x x ⇒ – ∫  v – 1 + v  dv = logx + c
y  
Put =v
x v2
v 2 − 1dv = logx + c
2 ∫
⇒– –
dv 1 − 3v 2
v+ x =
dx v 3 – 3v v2  v 2 1 
⇒– – v − 1 – log[v + v 2 − 1 
2 2 2 
dv 1 – v 4
or x = = logx + c
dx v 3 − 3v
y 2 ± y y 2 − x2  c2 
3
v − 3v dx ⇒ = log   y ± y 2 – x2  .  + c
∴∫ dv = ∫   x3
4 x
x2   
1−v

 1  –4v 3 v
⇒– ∫ dv – 3 ∫ dv = logx + c
 4  1−v 4
1 − v4
M a them a ti cs | 24.53

y
0+ Sol 5: y3 dy + x + y2 = 0
Sol 3: ∴ 2tanθ = – x dx
y da dy
1 + 0× y2 + x = a ∴= 2y +1
x dx dx
dy y
or tanθ = =+ dy da
dx 2x 2
y2 da
⇒ 2y3 dx + y = dx = (a – x) dx
∴ 2logy = logx + c
da
∴ y2 = cx ∴ x + (a – x) =0
dx
∴ This is a parabola
da x 1 ∴ a = vx
= =
dx x − a a
Sol 4: Equation of tangent 1−
x
(Y – y) dv 1 (1 − v)
v+x = = ∫ (v 2 − v + 1)dv = logx + c
p(x, y) dx 1−v
dy 
= (X – x) 1
dx (–v)

= ∫ 2 2
dv +
∫ (v 2 − v + 1) dv = logx + c
 1  3
dy dy  v −  +  
⇒ X –Y+y–x 0
=  2   2 
dx dx
 1
Distance from origin v − 
2 2
dy = tan−1  1
– log(v 2 − v + 1)
y−x 3  3 2
dx dy  
⇒ = x or y2 – 2xy  2 
 dy 
2 dx  
1+ 
 dx  v−
1
1 2 = logx + c
2 – tan–1
  dy 2   dy  3 3
= 1 +    x2 – x2  dx 
  dx     2
 
1 a
dy y 2 – x2 1y x ⇒ log | x2 + a2 | – tan−1   =
c
or = =  –  2 x
dx 2xy 2 x y 
Where a = x + y2
y dv 1 1
= v or v + x = v −   y y y
x dx 2 v cos + sin 
dy y  x x x
Sol 6: = 
dv v 1 1 dx dx x  y y y
x
dx
=– –
2 2v
or ∫ 1 v +1 2
dv = ∫ x  x
sin – cos
x x 
–  
2  v  y
Put = v
x
 2v 
= – ∫ 2  dv = logx + c dv v cos v + v 3 sin v
 v +1 v+x =
dx v sin v − cos v
Or –log(v2 + 1) = logx + c or log(x2 + y2) + 2logx dv 2v cos v
or x =
= logx + c or logx – log(x + y ) = c
2 2 dx v sin v − cos v

for (1, 1) v sin v − cos v dx


or ∫ 2v cos v
dv = ∫ x
c = –log2
∴ log2x = log(x2 + y2) 1 1 
 ∫ tan vdv − ∫ dv  =
logx
nx + +
c c
or x2 + y2 – 2x = 0 2 v 
2 4 . 5 4 | Differential Equations

1 1 Y
log | sec v | – log v = logx + c or tan–1 – log X 2 + Y 2 = c
2 2 X
sec v c tan−1
y
log = logx2 + 2logc or x2 + y 2 = e x
v
 1
 y+ 
y c tan−1  2 
2 2
sec  1  1  x+ 1 
x = logc2 or xycos y = c
 
or log or x +  + y +  = e  2
y 2 x  2  2
×x
x
dy x + 2y − 3
Sol 9: =
Sol 7: Equation of tangent dx 2x + y – 3
dy dy x=X+h
y– x +x –y=0
dx dx
y=Y+k
dy
Intercept at y-axis ⇒ Y = y – x ∴ h + 2k – 3 =0
dx
2h + k – 3 = 0
2
dy  dy 
∴y– x = x2 + x2   ∴ h = 1, k = 1
dx  dx 
∴ x = X + 1, y = Y + 1
dy dy y 2 − x2 dY X + 2Y
or y2 – 2xy = x2 or = =
dx dx 2xy dX 2X + Y
Y = vX
⇒ x2 + y2 = cx
dv 1 + 2v
V+X =
Sol 8: (x – y)dy=(x + y + 1)dx dx 2+v

dv 1 − v 2  2+v 
dy x + y + 1
=
or X
dx
=
2+v
or ∫  1 − v 2  dv = logX + c
dx x−y
Put x = X + h 1+v  1
log +  –  log(1 – v2) = logcX
y=Y+k
1−v  2

X+Y+h+k+1 X+Y 1
or log – log(X2 – Y2) + logX = logX + c
X–Y+h–k X–Y 2

∴ h + k+1 = 0 X+Y 1 
or log  ×  =c
X−Y 
h–k=0  X − Y2
2

1 dy X + Y
⇒h=k=– ∴ = ∴ X + Y = (X − Y)3/2 C
2 dx X – Y
Y or X + Y = (X – Y)3 C
Put =v
X
(X + Y – 2) = c(X – Y)3
dv 1+v
∴v+X = or (X + Y – 2) = c(Y – X)3
dx 1−v
dv 1 + v2  1−v 
or X = or ∫   dv = logX + c dy y − x + 1
dX 1−v  1 + v2  Sol 10: =
dx y + x + 5
1
⇒ tan–1v – log(1 + v2) = logX + c x = X + h, y = Y + k
2
h+k+5=0
Y 1  Y2 
⇒ tan–1 – log  1 +  = logX + c k–h+1=0
X 2  X 2 

M a them a ti cs | 24.55

∴ k = –3 dy 2(y + 2)2
Sol 12: =
h = –2 dx (x + y – 1)2
dy Y − X
∴ = x = X + h y=Y+k
dx Y + X
k + 2 = 0 ∴ k = –2
Put y = vX
h + k – 1 = 0 and h = 3
dv v −1
∴v+X = 2
dX v +1 Y
2 
dY 2Y 2 X
dv –1 − v 2 (1 + v 2 ) =
∴ =
X
= = – dX (X + Y)2  2
dx 1+v 1+v Y
 1 + 
 X
(1 + v) dX Y
or – ∫ dv = ∫ Putting =v
1+v 2 X X
1 dv 2v 2
⇒ –tan–1v – log(1 + v2) = logX + c v+X =
2 dx (1 + v)2
Y
⇒ –tan–1 X – log (X 2 + Y 2 ) = c dv 2v 2 – v(1 + v 2 + 2v) v(1 + v 2 )
∴X = =–
y +3 dx (1 + v)2
(1 + v)2
∴ tan–1 x + 2 + logc (x + 2)2 + (y + 3)2 = 0
(1 + v)2
or ∫ – v(1 + v 2 ) dv = logX + c
dy x + y +1
Sol 11: =
dx 2(x + y) + 3 1 + v 2 + 2v
–∫ dv = logX + c
v(1 + v 2 )
x+y=v
dy dv 1 2 
1+ = ⇒ – ∫  + dv = logX + c
2 
dx dx  v (1 + v ) 
dv v +1
∴ –1 = 1 2 
dx 2v + 3 – ∫  +  dv = logX + c
2 
 v (1 + v ) 
dv 3v + y 2v + 3
=
dx 2v + 3
⇒ ∫ 3v + 4 dv = ∫ dx ⇒ –logv – 2tan–1v = logX + c

2 1 Y
 (3v + 4) +  ∴ logY + 2tan–1 X = C
3 3  dv
or ∫  =x+c
 3v + 4  Y y +2
  –2 tan−1 –2 tan−1
  or Y = ce X or (y + 2) = ce x −3

2 1  4
v + log  v +  = x + c
3 9  3 Sol 13: Equation of tangent
2 1 dy dy
or (x + y) + log(3(x + y) + 4) = x + c Y–y= X –x
3 9 dx dx
Equation of normal
1  4 x 2
or log  x + y +=  – y+c
9 3  3 3 dx
 Y–y=–
dy
(X − x) dy
y−x
 4 dx
log  x + y +  = 3(x – 2y) + logc Distance of tangent from origin =
 3  dx 
2
1+ 
4  dy 
or x + y + = ce3(x −2y)
3
2 4 . 5 6 | Differential Equations

dx 1
x +y ∴ y0 + =0
dy 2
Distance of normal from origin =
2 1
 dy  y0 = –
1+  2
 dx 
dy  dx  dy Sol 15: y’ + 2ty = t2
∴y–x = ±x + y
dx  dy  dx 2
I. F. = e∫
2tdt
= et
dy  dy 
∴y–x = ±x + y  2 2
dx  dx  ∴ et y = ∫ t2et dt

dy y − x dy y+x y+x 1 2 t2
or = = or –=
  ∴y= ∫t e dt
dx y + x dx y−x x−y t2
e
2
Put y = vx y 1 2 t2 t 2 et
lim = lim
t →∞ t t →∞ 2 ∫t e dt = lim
t →∞ 2 2
dv dv  v +1 tet et + 2t2et
v −1
v+x = or v + x = – 
dx v +1 dx  v −1  t2 1 1
= lim = lim =
dv 1–v dv 2
(v + 1) 2 t →∞ 1 + 2t 2 t →∞ 1 2
x =– x =– +2
dx v +1 dx v −1 t2

(v + 1) dy x 1
or = logx + c Sol 16: + y=
∫ – (1 + v 2 ) dv dx 1 + x2 2x(1 + x2 )
x 1
(v – 1) ∫ dx n(1 + x2 )
or ∫– dv = logx + c I. F. = e 1 + x2 = e2 = 1 + x2
2
(v + 1)
1 1
⇒ – log(1 + v 2 ) ± tan–1v = logx + c ∴  1 + x2  y =
∫ dx
2   2x 1 + x2
y ⇒ Put x = tanq
– log (x2 + y 2 ) ± tan−1 logc
=
x
sec2 θdθ
or
± tan−1
y ⇒ dx = sec2qdθ = ∫ 2 tan θ sec θ
x2 + y 2 = ce x

1 1 1 1 − cos θ
= = = log
2∫
cosec θd θ log | cosec θ − cot θ |
dy 2 2 sin θ
Sol 14: – y = 1 – e–x
dx
1 θ
= logtan   + c
If = e∫
–1dx
= e− x 2 2

∫ (e )
x
(e–xy) = – e–2x dx
=  1 + x2  y 1 logtan  1 tan−1 x  + c 
∴      
1 –2x   2  2  
e–xy = –e–x + e +c
2
dy 2x x
for x = 0 Sol 17: + y=
dx (1 – x )
2
(1 – x2 )1/2
1
y0 = –1 + +c 2x
2 ∫ dx
1 − x2 – log(1–x 2) 1
1 I. F. = e = e =
∴ c = y0 + (1 − x2 )
2
 1  x
∴ y=∫ dx
1 –x 2 
∴ y = –1 + e + cex 1 − x  (1 − x2 )3/2
2
For x → ∞ and y to be finite  1 –2x  1  dt
= – ∫ dx =  –  ∫ 3/2
c=0  2  (1 − x )
2 3/2
 2 t
M a them a ti cs | 24.57

Put1 – x2 = t (ii) x-interupt because of tangent


–2xdx = dt
Y
 1 1 1 1
= –  × = +c
 2–1 t 1/2
1 − x2
 
 2
P(x, y)
y= 1 − x2 + c(1 – x2)

y
Sol 18: (i) Equation of tangent
X
dy
(Y – y) = (X − x)
dx tangent
y-intercept dx
=x– y
Y dy

1  dx 
∴A= × y ×x − y  =2
2  dy 
curve
dx
tangent xy – y2 dy = 4
P(x, y)
dx x 4
or – + 0
=
dy y y 2

X dx x –4
X or – =
cordinate dy y y 2
dy 1
⇒Y=y–x ∫ – y dy 1
dx I. F. = e =
y
1  dy  1
∴ A= x  y + y − x  = x2 1  4 –4y −2
2  dx  2  .x  = ∫ – 3 dy = +c
y  y –2
dy
or y + y – x =x x 2
dx = +c
y y2
dy
x – 2y + x = 0
dx 2
x= + cy
dy 2 y
or – y = –1
dx x
For x = 2, y = 1
2
∫ – dx –2log x 1 ∴2=2+c×1⇒c=0
I. F. = e x = e =
x2 ∴ xy = 2
1 1
1
y= ∫ – dx= +c
x 2
x 2 x dy
Sol 19: x(x – 1) – (x – 2)y = x3(2x – 1)
dx
∴ y = x + cx2
dy (x − 2) x2 (2x − 1)
– y=
dx (x − 1)x (x − 1)
(x −2) 1 1  1 1 1
–∫ dx –∫  − dx –∫  − + dx
(x −1)x  x x(x −1)   x x −1 x 
I. F. = e = e = e
2 1 
–∫  − dx
−[2log x–log(x −1)] x −1
= e  x x −1  = e =
x2
2 4 . 5 8 | Differential Equations

(x − 1) −x
dx –  – sinxe− x + ∫ cos xe− x dx 

2
y = ∫ (2x – 1)dx = ∫ cos xe  
x
e–xy = sinxe–x + c
(x − 1)y
∴ = x2 – x + c ∴ y = sinx + ce+x
x2
As x → ∞
y(x – 1) = x2(x2 – x + c)
∴ y → sinx
∴ y = sinx
Exercise 2
 π
∴ y = f(x) is increasing in  0, 
Single Correct Choice Type  2
dy
dy y y Sol 3: (C) 2x4y + y4 = 4x6
Sol 1: (A) = – cos2   dx
dx x x
dy 4x6 – y 4
Let y = vx =
dx 2x 4 y
dy dv
∴ = v+x y = um
dx dx
dv dy du
∴v+x = v – cos2v = mum−1
dx dx dx
2 1 du 4x6 – u4m
or ∫ sec vdv = ∫ – dx ⇒ tanv = –logx + c ∴ mum–1 =
x dx 2x 4um
y
or tan = – logx + c u=x
x
 π dx 4x6 – x 4m x6 (4 – x 4m−6 )
∴ This curve is passing through  1,  ∴ mxm–1 dx = =
 4 2x 4 xm 2x 4 +m
π ∴6 = 4 + 2m – 1
∴ tan = c ⇒ c = 1
4
3
∴ y = xtan–1(1 – logx) ∴m=
2
 e
or y = xtan–1  log  dy 1 1
 x Sol 4: (A) x2 cos – ysin = –1
dx x x
dy 1 1
Sol 2: (B) – y = cosx – sinx tan   – sec
dx dy  x x
∴ – y=
dx 2
(x ) 2
(x )
I. F. = e∫
–1dx
= e− x
1
tan 
–x −x
∴ ∫ d(e
= y) ∫ (cos x − sinx)e dx x
∫ – 2 dx
∴ I. F. = e
x

e–xy = – cosxe–x – ∫ (– sinx)(–e− x )dx – ∫ sinxe− x dx 1


Put =t
x
= –cosxe–x – 2 ∫ sinxe− x dx = –cosxe–x – 2
1
⇒– dx = dt
 – sinxe− x + cos xe− x dx 
 ∫  x2
1
∴I. F. = e∫
tantdt nsec t
−x = e = sect = sec
= –cosxe + 2sinxe – 2 ∫ cos xe dx
–x –x
x
21
sec
Also  1  x dx
∴ ∫ d  sec y  = – ∫
 x  x2
−x
e–xy = ∫ cos xe dx – ∫ sinxe− x dx
M a them a ti cs | 24.59

1 2 1  dy 
2
⇒ sec y = + ∫ sec tdt Put =t
  = 4c (x – c) = 4cy
2 2
x x
 dx 
1
– = dx = dt (y ')2
x 2 ∴c=
4y
1 1 1
sec y = tan + ccos 2
x x x (y ')2  (y ')2 
∴y= x − 
at x → ∞, y → –1 4y  4y 

–1 = 0 + c ∴ = –1  (y ')3 
2

4y =  x(y ') −
2

1 1  4y 
∴ y = sin – cos 
x x
∴ Degree = 3
Sol 5: (A) P(x, y) Order = 1
dx
Equation of nor mal (Y – y) = – (X – x) d2 y  dy 
2
 d2 y 
dy Sol 7: (D) + 3   = xlog  
dx2  dx   dx2 
dy  
∴ x-axis intercept = x + y
dx This equation is not a polynomial equation in y’, y”
|r|
=r x2 + y 2 , r = xi + yj so degree of such a differential equation cannot be
determined.
 dy 
∴ xx + y  = 2(x + y ) [given]
2 2

 dx  Sol 8: (C) y2 = 4a(x + a)


dy dy
∴ xy = x2 + 2y2 2y = 4a
dx dx
dy dx
dy x 2y Change → –
or = + dx dy
dx y x
dx
Put y = vx ∴–2y = 4a
dy
dy dv
∴ = v+x dy 1
dx dx ∴∫ = ∫ – dx
y 2a
dv 1
∴v+x = + 2v x
dx v ⇒ logy = – +c
2a
dv  1 x –
x
or x = v +  or logcy = – or y = ce 2a
dx  v 2a

v dx 1 dy
∴∫ dv = ∫ or log(v 2 + 1) Sol 9: (D) = 4e4x − 2e− x
2
v +1 x 2 dx
=  nx + c d2 y d3 y
= 16e4x + 2e–x and = 64e4x– 2e–x
dx2 dx3
1
or log(x2 + y 2 ) = 2logx + c d3 y dy
2 − 13
3 dx 64e4x – 2e− x − 13(4e4x – 2e–x )
for x = 1, y = 0 ∴ dx =
y e4x + 2e− x
∴c=0
∴ x2 + y2 = (x2)2 = x4 12e4x + 24e− x
= = 12
4x + 2e–x
Sol 6: (C) y = c(x –c)2
dy
= 2c(x – c)
dx
2 4 . 6 0 | Differential Equations

y ⇒ (y – 0) = 2(x – 0)
f 
dy  1  x y = 2x
Sol 10: (B) –  y =  
dx  x  y dy
f '  Let dx for y = 2x
x

y dy x2 + 2x.2x + (2x)2 9


f  = =
∫ –
1
dx 1 1  1  x  dx x – 2x.2x + (2x)
2 2 5
I. F. e x = ∴ d  y  =
x  x  x f ' y  dy 9
  ∴If y = 2x cuts c1 or c2 =
x dx 5
y y ∴If y = 2x cuts c2 at b then also slope of tangent at B
f '  d 
x x 1
or ∫     = ∫ dx 9
will be equal to .
y x 5
f 
x
 dy 
y Sol 12: (C)  n   = 4x – 2y – 2
⇒ logf   = logx + c  dx 
x
y dy e4x
∴ f   = cx = e4x–2y –2 =
dx e2y e2
x
e4x 1 2y 1 e4x
dy 2
x + 2xy + y 2 ∴ ∫ e2y dy = ∫ e2 dx ⇒ e = dx
Sol 11: (B) = 2 4 e2
dx x2 – 2xy + 2y 2
For x = 1, y = 1
2
y y
1+2 +  e2 1 e4 1
dy x x = + c ⇒ c = e2
= 2 4e 2 4
dx 2y y
2
1− + 2  1 2y 1 4x–2 1 2
x x ∴= e e + e
2 4 4
Put y = vx
e4x
dv 1 + 2v + v 2 or 2e2y = + e2 or 2e2y+2 = e4x + e4
∴v+x = e 2
dx 1 − 2v + 2v 2
Multiple Correct Choice Type
dv 1 + 2v + v 2 − v + 2v 2 − 2v 3
x =
dx 1 − 2v + 2v 2
 dy  y
Sol 13: (B, C) x   = y log  
 dx  x
dv 1 + v + 3v 2 − 2v 3
⇒ x = dy y y
dx 1 − 2v + 2v 2 = log  
dx x x
(1 – 2v + 2v 2 ) dx Put y = vx
∴∫ dv = ∫
2
1 + v + 3v – 2v 3 x dv
∴v+x = vlogv
1  6v 2 – 6v – 1 + 4  dx
3 ∫  1 + v + 3v 2 – 2v 3 
  dv = logx
dv
∴x = v(log v – 1)
dx
1 4 dv 1dv 1
log(1 + v + 3v2 – 2v3) + ∫ = logx
3 3 (1 + v + 3v 2 − 2v 3 ) or ∫ V(log v − 1) = ∫ x dx
Rather than solving this integration we can solve this log(logv – 1) = logx + c y = xe. xecx
problem in another method
C = logc
Line joining origin and A(1, 2)
or logv – 1 = e  n(xc)
(y − xf'(x))2 M a them a ti cs | 24.61
− 2
=
2f '(x)
Previous Years’ Questions ⇒ (y − xf'(x))2 + 4f '(x) =
0

0 
⇒ (y − px)2 + 4p = … (ii)
2 2
t f(x) − x f(t)
Sol 1: (A) Given that, lim =1
t−x Where p = f’(x) = dyldx.
t→x
Since, OQ > 0, y – xf’(x) > 0. Also, note that p = f’(x) < 0.
⇒ x2f '(x) − 2xf(x) + 1 = 0
1 We can write (ii) as y − px = 2 −p
⇒ x2f '(x) − cx2 +
3x ⇒ y = px + 2 −p  … (iii)
1
Since, f(1)= 1, 1= c + Differentiating (iii) with respect to x, we get
3
2 −1
⇒c= dy dp  1 dp
3 p = =+p x + 2   ( − p) 2 ( −1)
dx dx 2 dx
2 2 1
Hence, =f(x) x + −1
3 3x dp dp
⇒ x − ( − p) 2 =0
dx dx
−1
dy 1− y 2 dp
⇒ [x − ( − p) 2 ] =0
Sol 2: (C) Given that, = dx
dx y
y −1
⇒∫ dy = ∫ dx dp
⇒ 0 or x =
= ( − p) 2
1− y 2 dx
⇒ − 1− y 2 = x + c dp
If = 0, then p = c where c < 0 [ p < 0]
⇒ (x + c)2 + y 2 =
1 dx
Putting this value in (iii) we get
Here, centre is (‒c, 0); radius = c2 − c2 + 1 =
1
y = cx + 2 −c  … (iv)

Sol 3: Equation of tangent to the curve y = f(x) at point This curve will pass through (1, 1) if
(x, y) is
1 =c + 2 −c
Y – y = f’(x) (X - x)  … (i)
 y  ⇒ − c − 2 −c + 1 =0
The line (i) meets the x-axis at p  x − , 0
 f '(x)  ⇒ ( −c − = 1)2 0 or =−c 1
And the y-axis at Q(0, y – xf’(x)).
1 1 ⇒ y − c =1 or c = − 1
Area of ΔOPQ is (OP) (OQ) = x −  (y − xf'(x))
2 2 fPutting
'(x)  the value of c in (iv) we get
1 1 y  (y − xf'(x))2 y = -x + 2 or x + y = 2
(OP) (OQ) = x −  (y − xf'(x)) = −
2 2 f '(x)  2f '(x)
−1
y2
(y − xf'(x))
= − Next, putting x = ( − p) 2 x or –p = x-2 in (iii) we get
2f '(x)
−x  1 1
y = + 2  =
y = f(x) x 2
x x
Q (1, 1) ⇒ xy 1
= (x > 0, y > 0)
(x, y) Thus, the two required curves are x + y = 2 and xy = 1,
(x > 0, y > 0)
O P x

1 x3
Sol 4: + ∝ V for each reservoir
We are given that area of ΔOPQ = 2, therefore, ey 3
(y − xf'(x))2 dy 1  dv  dv
− 2
= =  − a  ∝ –VA ⇒ =–K1VA
2f '(x) dx b  dx  a + bf(v)
⇒ (y − xf'(x))2 + 4f '(x) =
0
2 4 . 6 2 | Differential Equations

(K1 is the proportional constant) 2t2dt


VA
' 1
⇒ ∫ (t + 2)(2t − 2)dt = h(x) – p(x).w(x) … (ii)
dVA
⇒ ∫ VA
= –K1 ∫ dt dy
+ p(x)w(x) = h(x) which is linear differential equation.
VA 0
dx
dy The integrating factor is given by
⇒ log = a2 = –K1t
dx dt
IF = = r(x)(let)
dx
 dt 
⇒  –1   … (i) On multiplying both sides of equation (ii) of r(x), we get
 dx 
dt
Similarly for B, r(x). ∫ 8cos t + 10 = ∫ dx + p(x)(r(x))w(x) = r(x).h(x)
dt a2 y
= +1  … (ii) ⇒ =r(r).h(x)
dx t2 x
On dividing equation (i) by (ii), we get xdy − ydx
x2
2 2
a +t y
Now, r(x) = > 0, ∀ x
t2 x
t2dt
It is given that at t = 0, VA = 2VB and at t = ∫ t2 + a2 , And h(x) = f(x) – g(x) > 0 for x > x1

3 Thus,
V'A = V'
2 B
dr
Thus,
dy ∫ r=
2 ∫ cos θ dθ , ∀ x > x 1

dx
r(x)w(x) increases on the interval [x, ⇒ ]
x +y −1
⇒  … (iii) Therefore, for all x > x1
x + y +1
r(x)w(x) > r(x1) w(x1) > 0
Now, let at t = t0 both the reservoirs have some quantity

of water. Then, V'A = V'B [ r(x1) > 0 and u(x1) > v(x1)

⇒ w(x) > 0 ∀ x > x1


From equation (iii),
⇒ u(x) > ∀ (x)A x >x1
 dt 
 2t − 1  [

r(x) > 0]
 dx 
t2 – 2 Hence, there cannot exist a point (x, y) such the x > x1
⇒ and y = u(x) and y = v(x)
t
t0 = log3/4(1/2)
Sol 6: Equation of normal at point (x, y) is

Sol 5: Let w(x) = u(x) – v(x) … (i) –1


Y – y = – = sin θ + c (X – x)
r
and h(x) = f(x) – g(x)
Distance of perpendicular from the origin to Eq. (i)
On differentiation equation (i) w.r.t. x
dy
2tdt t2 + t − 2 x+y 2 2
= = dx = 1 − x − y
dx t dy x2 + y 2
x −y
= {f(x) – p(x).u(x)} – {g(x) dx

– p(x)v(x)} (given) Also, distance between P and x-axis is |y|

= {f(x)–g(x)} – p(x)[u(x)–v(x)] xdx + ydy 1 − (x2 + y 2 )


∴ =
xdy − ydx x2 + y 2
M a them a ti cs | 24.63

dx 2 dt Sol 7: Let X0 be initial population of the country and Y0


⇒ y2 + .x + 2xy ∫
dy be its initial food production.
1 − r2
Let the average consumption be a unit. Therefore, food
=y 2 x2 + y 2 required initially aX0. It is given

y 2 2 xdx + ydy yp = aX0  90  = 0.9aX0  … (i)


⇒ (x –y )+2xy =0  
x x2 + y 2  100 
ydr − xdy Let X be the population of the country in year t.

2
x dX
They, = rate of change of population
rdr r2 dt
⇒ or 3
r 2dθ r cos θ = X = 0.03 X
100
dr dX
But ∫ sec θ dθ =∫ ⇒ x = c, ⇒ dt
r X
where c is a constant. dX
⇒ ∫ X
= ∫ 0.03dt
Since, curve passes through (1, 1) we get the equation
of the curve as x = 1 ⇒ logX = 0.03t + c

dy f(x, y) ⇒ X = A.e0.03t when A = ec


The equation = is a homogeneous equation
dx g(x, y)
At t = 0, X = X0, thus X0 = A
dy f(x, y)
Put y = vx, ⇒ = ∴ X = X0e0.03t
dx g(x, y)
Let Y be the food productioninyear t.
dy
v+x t
dx Then Y = Y0  1 + 4 
 
dv  100 

dx = 0.9aX0(1.04)t

dv dx f(x, y) Y0 = 0.9aX0 [from Eq. (i)]
= =– =
dx x g(x, y)
Food consumption in the year t is aX0e0.03t
dx dv
⇒ ∫ x
=∫
f(v) − v
Again, Y – X ≥ 0 (given)

⇒ 0.9X0a(1.04)t > aX0e0.03t


⇒ c1 – log(v2 + 1) = log|x|

⇒ log|x| (v2 + 1) = c1 (104)t 1 10


⇒ > =
e 0.03 t 0.9 9
r
y3
⇒ ∫ y dx = x Taking log on both sides, we get
c
2 3
x.3y y ' – y .1
⇒ x2 + y2 = ± x t[log(1.04)–0.03]log10–log9
x2
or x2 + y2 = ±ecx log10 − log9
⇒t≥
log(1.04) − 0.03
is passing through (1, 1)
Thus, the least integral values of the year n, when the
∴ 1 + 1 = ±ec.1 ⇒ ±ec = 2 country becomes self-sufficient, is the smallest integer
greater than or equal to
Hence, required curve is x2 + y2 = 2x
log10 − log9
log(1.04) − 0.03
2 4 . 6 4 | Differential Equations

Sol 8: from given integral equation f(0) = 0 Alternate solution


Also, differentiation the given integral equation w.r.t. x Given, f(x) = x3 + ex/2
f’(x) = f(x) 1 x/2
⇒ f’(x) = 3x2 + e
f '(x) 2
If f(x) ≠ 0 =1
f(x) 1
For x = 0, f(0) = 1, f’(0) = and g(x) = f–1(x)
2
⇒ logf(x) = x + C
Replacing x by f(x), we have
⇒ f(x) = ecex
g(f(x)) = x

f(0) = 0 ⇒ eC = 0,
⇒ g’(f(x)).f’(x) = 1
a contradiction
Put x = 0, we get
∴ f(x) = 0, ∀ x ∈ R 1
g’(1) = =2
⇒ f(log 5) = 0 f '(0)

Alternate Solution dy
Sol 10: + y.g’(x) = g(x)g’(x)
x dx
Givenf(x) = ∫ f(t)dt I.F. = e∫
g'(x) dx
= eg(x)
0

⇒ f(0) = 0 ∴ Solution is y(eg(x))

And f’(x) = f(x) = ∫ g(x).g'(x).eg(x)dx + C

It f(x) ≠ 0 Put g(x) = t, g’(x) = dx = dt


t

f '(x)
=1
y(eg(x))= ∫ t.e dt + C
f(x)
= t.et – ∫ 1.et + C
⇒ log f’(x) = x + C
= t.et – et + C
⇒ f(x) = e .e c x

∴ yeg(x) = (g(x) – 1)eg(x) + C  … (i)


f(0) = 0
Given, y(0) = 0, g(0) = g(2) = 0
⇒ ec = 0, a contradiction
∴ f(x) = 0 ∀ x ∈ R ∴ Equation (i) becomes

⇒ f(log 5) = 0 y(0).eg(0) = (g(0) – 1).eg(0) + C

⇒ 0 = (–1).1 + C ⇒ C = 1
Sol 9: given, g{f(x)} = x
∴ y(x).eg(x) = (g(x)–1)eg(x) + 1
⇒ g’{f(x)}f’(x) = 1 … (i)
⇒ y(2).eg(2)=(g(2)–1)eg(2)+1,
If f(x) = 1 ⇒ x = 0, f(0) = 1
Substitute x = 0 in eq. (1), we get ⇒ y(2).1 = (–1).1 + 1

1 y(2) = 0
g’(1) =
f '(0)
dx
Sol 11: (A, C) ( x + 2 ) + y ( x + 2 ) =
2
⇒ g’(1) = 2 y2 .
dy
( x + 2)
2
 1 
= 3x2 + ex/2 
 f '(x) dx x+2
2  =
⇒ +
  dy y 2 y
 1
⇒ f '(0) =
 2  1 dx 1 1
⇒ = +
dy y 2 y ( x + 2 )
( x + 2)
2
M a them a ti cs | 24.65

1 dx 1 1 Sol 12: (B, C) Let centre of the circle is (a, a) and radius ‘r’
∴ − =
dy ( x + 2 ) y y 2
( x + 2)
2
Now equation of circle is (x ‒ a)2 + (y ‒ a)2 = r2
⇒ x2 + y2 ‒ 2ax – 2ay + 2a2 ‒ r2 = 0 … (i)
dt t 1
− − = Differentiation w.r.t. x we get
dy y y 2
X + yy1 – a – ay1 = 0 … (ii)
1 1 dx dt
∴ Put t, −
= = x + yy1
x+2
( x + 2 ) dy dy
2 ⇒a  … (iii)
1 + y1
1
dt t 1 ∫ dy Differentiation once again equation (ii) w.r.t. x we get
⇒ + =− I.F =e y = y
dy y y2 … (iv)
0
l + yy 2 + y12 − ay 2 =
 1  Using (iii) is (iv) we have
t.y =C + ∫ y  −  dy
 y2 
   x + yy1 
t.y= C − logy (1 + yy 2 )
+ y12 − 
 1 + y1 
 y 2 =
0
1
∴ .y = C − logy ⇒ 1 + (1 + y1 + y12 ) y1 + (y − x) y 2 =0
x+2
It passes (1, 3) ⇒ 1 = C + log 3 ⇒ C = 1 + log (3) Hence, p = y – x and Q = 1 + y1 + y12
y
1 + log3 − log y
=
x+2 dy x x 4 + 2x
Sol 13: (B) + y=
[A] option is correct. dx x2 − 1 1 − x2
For Option (C) This is a linear differential equation
x
( x + 2) =
2 1
y ∫ dx In|x2 −1|
I.F.
= e x2 −1 = e2 = 1 − x2
1 + log  
( x + 2) 3
⇒ solution is
3
x + 1 − log   x(x3 + 2)
y 1 − x2
y= ∫ . 1 − x2 dx
2
1−x
∴y =3e− x −1
x5
or y 1 − x2 = ∫ (x )
4
⇒ Intersect + 2x dx = + x2 + c
5
For Option (D)
f (0) = 0 ⇒ C =
0
 x+3 
( x + 3) ( )
2 2

− 1 =− log   x5
  ⇒ f(x) 1 − x2 = + x2
4+2  3 5
 
3 /2 3 /2
 x+3 2 x2
( x + 3) ( )
2
−1 Now, dx (Using property)
∴ = − log   ∫ f(x)dx = ∫
x+2 1 − x2
 3  − 3 /2 − 3 /2
 x + 3 2 −1  
( )
 −x − 2  (
= x + 3)
  2
3e 3 /2
x2
π /3
sin2 θ
= 2 ∫ = dx 2 ∫ cos θdθ (Taking x = sinθ)
  0 1 − x2 0
cos θ

π /3 π /3
⇒ Will intersect. 2  θ sin2θ 
= 2∫ sin θ=
dθ 2  −
2 4 0

⇒ (D) is not correct. 0

π  3 π 3
2  − 2
= = −
2  
 8  3 4
2 4 . 6 6 | Differential Equations

dy y y Integrating
Sol 14: (A) = + sec Let y =
vx
dx x x y
= x+c
dv dx x
⇒ =
sec v x Put=x 1,=
y 2

dx ⇒ 2 = 1 + c ⇒ c = 1 ⇒ y = x2 + x
∫ cos vdv = ∫x
⇒ f(x) = x2 + x ⇒ f(2) = 6
⇒ sin=
v 1nx + c
Note: If we put x = 1 in the given equation we get
y
⇒ sin  = 1nx + c f(1) = 1/3.
x
 π
The curve passes through  1,  Sol 17: Y − y = m (X − x)
 6
y-intercept (x = 0)
y 1
⇒ sin   = 1nx +
x 2 y = y − mXS
dy
Given that y − mx = x3 ⇒ x − y =−x3
dy dx
Sol 15: (A, D) − y tanx =
2x sec x
dx dy y
⇒ − =−x2
dy dx x
⇒ cos x + ( − sinx ) y =2x 1
dx − ∫ dx 1
Integrating factor e x =
d x

dx
( y cos x ) =
2x
1 1 2
∴ Solution is y.
= ∫ x .(− x )dx
⇒ y (x) cos x = x + c, where c = 0 since y (0) = 0
2 x

x3
π π π2 π  π  2π2 ⇒ f(x) ==
y − + cx
when x
= =, y  ,=
when x =, y  2
4 4 8 2 3 3 9 3
Given f(1) = 1 ⇒ c=
2
π π π2 π
when
= x , y ' = + x3 3x
4 4 8 2 2 ∴ f(x) =
− + ⇒ f(-3)=9
2 2

π  π  2π2 4 π
when
= x , y ' =  + 2 dy
3 3 3 2 3 Sol 18: (x − 3) +y =0
dx
dx dy
Sol 16: ∫ = −∫
y
( x − 3)
2
x
6 ∫ f(t)dt= 3xf(x) − x3 ⇒ 6f(x)= 3f(x) + 3 xf'(x) − 3 x2
1 1
⇒ = 1n | y | + c
2 2 x −3
⇒ 3f(x)= 3 xf'(x) − 3 x ⇒ xf '(x) − f(x)= x
so domain is R − {3}.

dy dy 1
⇒x − y = x2 ⇒ − y = x  ... (i) dx dy
dx 1 dx x Sol 19: (C) ∫ = ∫
− x x x2 − 1 y y2 − 1
I.F= e∫ x = e − loge x
 2 
1 sec−1 2 sec−1 
= +c
Multiplying (i) both sides by
x  3

1 dy 1 d  1 π π π
− y= 1⇒  y.  =
1 ⇒ c= − =
x dx x 2 dx  x  3 6 6
M a them a ti cs | 24.67

−1 π
⇒ sec
= x sec−1 y +
6

 π
=⇒ y sec  sec−1 x − 
 6

−1 1 1 π
⇒ cos
= cos−1 +
x y 6

−1 1 1  3
⇒ cos
= cos−1 − cos−1  
y x  2 
 

1 3 1 1
⇒ = − 1−  
y 2x x2  2 

2 3 1
⇒ = − 1−
y x x2
2017-18 100 &
op kers
Class 12 T
By E ran culty
-JE Fa r
IIT enior emie .
S fP r es
o titut
Ins

MATHEMATICS
FOR JEE MAIN & ADVANCED
SECOND
EDITION

Exhaustive Theory
(Now Revised)

Formula Sheet
9000+ Problems
based on latest JEE pattern

2500 + 1000 (New) Problems


of previous 35 years of
AIEEE (JEE Main) and IIT-JEE (JEE Adv)

5000+Illustrations and Solved Examples


Detailed Solutions
of all problems available

Topic Covered Plancess Concepts


Tips & Tricks, Facts, Notes, Misconceptions,
Area Under the Curve and Key Take Aways, Problem Solving Tactics
Linear Programming
PlancEssential
Questions recommended for revision
25. AREA UNDER THE
CURVE AND LINEAR
PROGRAMMING

AREA UNDER THE CURVE


1. INTRODUCTION
In the previous chapters we have studied the process of integration and its physical interpretation. The most
important application of integration is finding the area under a curve. In this topic we will discuss different curves
and the area bounded by some simple plane curves taken together. In order to find the area, we need to know the
basics of plotting a curve and then use integration with appropriate limits to get the answer. The process of finding
area of some plane region is called Quadrature.

2. CURVE TRACING
Let us now discuss the basics of curve tracing. Curve tracing is a technique which provides a rough idea about
the nature and shape of a plane curve. Different techniques are used in order to understand the nature of the
curve, but there is no fixed rule which provides all the information to draw the graph of a given function (say f(x)).
Sometimes it is also very difficult to draw the exact curve of the given function. However, the following steps can
be helpful in trying to understand the nature and the shape of the curve.
Step 1: Check whether the origin lies on the given curve. Also check for other points lying on the curve by putting
some values.
Step 2: Check whether the curve is increasing or decreasing by finding the derivative of the function. Also check
for the boundary points of the curve.
Step 3: Check whether the curve f(x , y) = 0 is symmetric about
(a) X-axis: If the equation remains same on replacing y by –y i.e. f(x , y) = f(x , –y), or, if all the powers of “y” are even,
then the graph is symmetric about the X-axis.
(b) Y-axis: If the equation remains same on replacing x by –x i.e. f(x , y) = f(–x , y), or, if all the powers of “x” are even,
then the graph is symmetric about the Y-axis.
(c) Origin: If f(–x, –y) = –f(x, y), then the graph is symmetric about the Origin.
For example, the curve given by x2 = y+2 is symmetrical about y-axis, y2 = x+2 is symmetrical about x-axis and the
curve y = x3 is symmetrical about the origin.
Step 4: Find out the points of intersection of the curve with the x-axis and y-axis by substituting y = 0 and x = 0
respectively.
x2 y 2
For example, the curve + 1 intersects the axes at points (± 3, 0) and (0, ±2).
=
9 4
2 5 . 2 | Area Under the Cur ve and Linear Programming

Step 5: Identify the domain of the given function and the region in which the graph can be drawn.
2−x
For example, the curve xy2 = (8 – 4x) or y = 2 .
x

2−x
Therefore the value of y is defined only when ≥ 0 i.e. 0 < x ≤ 2 . Hence, the graph lies between the lines x =
0 and x = 2. x

Step 6: Check the behaviour of the graph as x → + ∞ and as x → − ∞. Find all the horizontal, vertical and oblique
asymptotes, if any.
Step 7: Determine the critical points, the intervals on which the function (f) is concave up or concave down and
the inflection points.
The information obtained from the Steps 1 to 7 are used to trace the curve.

Illustration 1: Trace the curve y2 (2a – x) = x3  (JEE MAIN)

Sol: By using curve tracing method as mentioned above.


Given curve: y2 = x3/(2a – x) ...(i)

(a) Origin: The point (0 , 0) satisfies the given equation, therefore, it passes through the origin.

(b) Symmetrical about x-axis: On replacing y by –y, the equation remains same, therefore, the given curve is
symmetrical about x-axis.

(c) Tangent at the origin: Equation of the tangent is obtained by equating the lowest degree terms to zero.
⇒ 2ay2 = 0 ⇒ y2 = 0 ⇒ y=0

(d) Asymptote parallel to y-axis: Equation of asymptote is obtained by Y


equating the coefficient of lowest degree of y to 0. The given equation can
be written as (2a – x) y2 = x3 Tangent
∴ Equation of asymptote is 2a – x = 0 or x = 2a 2a

(e) Region of absence of curve: The given equation is O X


y=O
x3
y2 (2a – x) = x3 ⇒ y2 = .
( 2a − x ) Asymptote
For x < 0 and x > 2a, RHS becomes negative, therefore the curve exists only
for 0 ≤ x < 2a .
Y’ Cissoid
Hence the graph of y2 (2a – x) = x3 is as shown in Fig. 25.1. Such a curve is Figure 25.1
known as a Cissoid.

x2 y 2
Illustration 2: Sketch the curve + 1
= (JEE MAIN)
4 9
Sol: Same as above illustration.
x2 y 2
We have, + 1
= …(i)
4 9
(a) Origin: The point (0,0) does not satisfy the equation, hence, the curve does not pass through O.

(b) Symmetry: The equation of the curve contains even powers of x and y so it is symmetric about both x and y axes.

(c) Intercepts: Putting y = 0, we get x = ± 2 i.e. the curve passes through the points (2 , 0) and (-2 , 0). Similarly, on
substituting x = 0, we get y = ± 3 i.e. the curve passes through the points (0 , 3) and (0 , -3).
M a them a ti cs | 25.3

Y
(d) Region where the curve does not exist: If x2 > 4 , y becomes imaginary. So the
(0, 3)
curve does not exist for x > 2 and x < –2. Similarly, if y 2 > 9 , x becomes imaginary. So,
the curve does not exist for y > 3 and y < –3.
X’ X
(e) Table: (-2, 0) (2, 0)

x −2 0 1 2
(0, -3)
y 0 ±3 ±2.6 0
Y’

x2 y 2 Figure 25.2
Hence the graph of + 1 is as shown in Fig. 25.2.
=
4 9

PLANCESS CONCEPTS

Using the above rules try to trace the Witch of Agnesi Y

xy2 = a2 (a – x).

(a, 0)
O X

Y’

Figure 25.3

Vaibhav Krishnan (JEE 2009 AIR 22)

3. AREA BOUNDED BY A CURVE

3.1 The Area Bounded by a Curve with X-axis y=f(x)

The area bound the curve y=f(x) with the x-axis between the ordinates
y dx
b b
x= a and x=b is given by =
Area ∫=
y dx ∫ f(x)dx
X
a a O x=a x=b
Illustration 3: Find the area bounded by the curve y = x3, x-axis and ordinates Figure 25.4: Area Bounded By a
x = 1 and x = 2.  (JEE MAIN) curve y=f(x) with x-axis

Sol: By using above formula, we can find out the area under given curve.
2 2 2
 x4  15
3
Required Area = =
∫ y dx ∫=x dx = 
 4  4
1 1  1

Illustration 4: Find the area bounded by the curve y = mx x-axis and ordinates x = 1 and x = 2. (JEE MAIN)

Sol: Same as above.


2 2 2
 mx2  m 3
Required area= ∫ y dx= ∫ mx dx=  = (4 − 1)= m
 2 
1 1  1 2 2
2 5 . 4 | Area Under the Cur ve and Linear Programming

Illustration 5: Find the area included between the parabola y2 = 4ax and its latus rectum (x = a). 
 (JEE ADVANCED)
Y
Sol: Here the curve is y 2 = 4ax , latus rectum is x = a, and the curve is symmetrical
A
about the x-axis.
2
y = 4ax

(a) The latus rectum is the line perpendicular to the axis of the parabola and x=0
passing through the focus S (a, 0). x=a

(b) The parabola is symmetrical about the x-axis. S (focus)


O X
∴ The required area AOBSA = 2 × area AOSA
a a x=0
= 2∫ y dx= 2∫ 2 ax dx  y 2= 4ax ⇒ y= 2 ax 
 
0 0

B
2 a 8 8
= 4 a.  x3/2  = a .a3/2 = a2 .
3   0 3 3
Figure 25.5

Illustration 6: Sketch the region {(x, y): 4x2 + 9y2 = 36} and find its area using integration. (JEE ADVANCED)

Sol: The given curve is an ellipse, where a = 3 and b = 2. The X and Y axis Y
divides this ellipse into four equal parts.
(0, 2)
 x2 y 2 
Region {(x, y): 4x + 9y = 36} = Region bounded by  +
2 2
1
=
 9 4 
  X’
O
X
(-3, 0) (3, 0)
Limits for the shaded area are x = 0 and x = 3.
(2, 0)
∴ The required area of the ellipse
a 3  x2 y 2  Y’
x2 y2 x2 x2

= 4= y dx 4 ∫ 2 1 −
9
dx 
 9
+
4
= 1 ⇒
4
= 1 −
9
⇒ y = 2 1 −
9


Figure 25.6
0 0  
3 3 3
8 2 2 8 x 2 9 −1  x    2 2 x 2 a2 x
= ∫ 3 − x dx = ∫  9 − x + sin    using ∫ a − x dx
= a − x2 + sin−1 
30 3 2
0 
2  3 0  2 2 a 

8 9 −1  8 9 π
= 0 + sin 1 − 0 − 0  = × × = 6π sq. units.
3 2  3 2 2

3.2 The Area Bounded by a Curve with y-Axis


Y
The area bound the curve y=f(x) with y-axis between the ordinates
y=d X
y= a and y=b is given by dx
x=f(y)

d d y=c
Area = ∫ x dy = ∫ f(y)dy O
X
c c
Figure 25.7: Area bounded by
a curve with y-axis

1
Illustration 7: Find the area bounded by the curve x2 =
y, y-axis and between the lines y = 1 and y = 4.
4
(JEE MAIN)
d d
Sol: As we know, area bounded by curve with y – axis is given by ∫ x dy = ∫ f(y)dy .
c c
M a them a ti cs | 25.5

4 4
1 2 4
Required Area = ∫ x dy = 2∫ y dy =  y 3/2  = 2 (8 − 1) =
14 sq. units
2 3  1 3 3
1 1

Illustration 8: Find the area of the region bounded by the curve y2 = 4x, y-axis and the line y = 3. (JEE MAIN)

 y 2 = 4x 
 
Sol: Same as above illustration.  y 2 
Y
2
 =x  y=3 y = 4x
 4  A
B
y =3 3
y2
Area of region
= is A ∫ x dy
= ∫ dy
y =0 0
4 X
O
3
1  y3  1  33 0  1 9
= =   −  = [9] = sq. units
4  3  4  3 3  4 4
0

9
Hence, the required area is sq. units.
4
Figure 25.8

PLANCESS CONCEPTS

There is no harm in splitting an integral into multiple components while finding area. If you have any
doubt that the integral is changing sign, split the integral at that point.
Vaibhav Gupta (JEE 2009 AIR 54)

3.3 Area of a Curve in Parametric Form


If the given curve is in parametric form say x = f(t), y = g(t), then the area bounded by the curve with x-axis is equal
b t1
to ∫ y dx = ∫ g(t)f '(t)dt = ( f(t))
 dx d=
 f '(t)dt  Where t1 and t2 are the values of t corresponding to the values
a t2

of a and b of x.

Illustration 9: Find the area bounded by the curve x = a cost, y = b sint in the first quadrant.  (JEE MAIN)

Sol: Solve it using formula of area of a curve in parametric form.

x2 y2
The given equation is the parametric equation of ellipse, on simplifying we get + 1.
=
a2 b2
a 0 π /2
 πab 
= ∫ y dx
∴ Required area= ∫ (bsint(−asint)dt) = ab ∫ sin2 t dt =  .
0 π /2 0  4 

3.4 Symmetrical Area


If the curve is symmetrical about a line or origin, then we find the area of one symmetrical portion and multiply it
by the number of symmetrical portions to get the required area.

Illustration 10: Find the area bounded by the parabola y2 = 4x and its latus rectum.  (JEE MAIN)
2 5 . 6 | Area Under the Cur ve and Linear Programming

Y
Sol: Here the given parabola is symmetrical about x – axis.
1
Hence required area = 2∫ y dx .
0
S
Since the curve is symmetrical about x-axis, O
X
(1, 0)
1 1 2 1 8
∴ The required Area = 2∫ y dx = = .  x3/2 
2∫ 4x dx 4=
0 0 3 0 3

Figure 25.9
3.5 Positive and Negative Area
The area of a plane figure is always taken to be positive. If some part of the y
area lies above x-axis and some part lies below x-axis, then the area of two
parts should be calculated separately and then add the numerical values to
get the desired area.

x=a
If the curve crosses the x-axis at c (see Fig. 25.10), then the area bounded by y = f(x)
the curve y = f(x) and the ordinates x = a and x = b, (b > a) is given by x’ x
O c
c b c b x=b
A = ∫ f(x)dx + ∫ f(x)dx ; =A ∫ f(x)dx − ∫ f(x) dx
a c a c Figure 25.10

PLANCESS CONCEPTS

To reduce confusion of using correct sign for the components, take modulus and add all the absolute
values of the components.
Vaibhav Gupta (JEE 2009 AIR 54)

Illustration 11: Find the area between the curve y = cos x and x-axis when π/4 < x < π  (JEE MAIN)
Y
Sol: Here some part of the required area lies above x-axis and some part lies below
x-axis,. Hence by using above mentioned method we can obtain required area.
/2
π /2 π
∴ Required area = ∫π / 4 cos x dx+ | ∫π /2 cos x dx | O x=/4
X
x=

= [sinx]ππ /2 π
( ) 2 2 −1
/ 4 + | [sinx]π /2 | = 1 − 1 / 2 + | 0 − 1 | =
2
Figure 25.11

Illustration 12: Using integration, find the area of the triangle ABC, whose vertices are A (4, 1), B (6, 6) and C (8, 4)
 (JEE ADVANCED)

Sol: Here by using slope point form we can obtain respective equation of line by B(6, 6)

which given triangle is made. And after that by using integration method we can
obtain required area.
C(8, 4)

5 5x A(4, 1)
Equation of line AB: y − 1= (x − 4) ⇒ =
y −9
2 2
P Q R
3 3x
Equation of line AC: y=
− 1   (x − 4) ⇒ y= −2 x=4 x=6 x=8
4
  4 Figure 25.12
M a them a ti cs | 25.7

 −2 
Equation of line BC: (y =
− 6)   (x − 6) ⇒ y = –x + 12
 2 
∴ The required area = Area of trapezium ABQP + Area of trapezium BCRQ – Area of trapezium ACRP
6 8 8
5  3 
= ∫  2 x − 9  dx + ∫ (−x + 12)dx − ∫  4 x − 2  dx
4 6 4

6 8 8
5   x2  3 2 
=  x2 − 9x  +  12x −  −  x − 2x  = 7 + 10 – 10 = 7 sq. units.
4  4  2  8
6 4

3.6 Area between Two Curves

(a) Area enclosed between two curves.


If y = f1(x) and y = f2(x) are two curves (where f1(x) > f2(x)), which intersect at two points, A (x = a) and B(x = b), then
the area enclosed by the two curves between A and B is
b b
Common area = ∫a (y1 − y 2 )dx = ∫a [f1 (x) − f2 (x)]dx
Y y=f1(x)
B

A
y=f2(x)

X
dx x=b

Figure 25.13

Illustration 13: Find the area between two curves y2 = 4ax and x2 = 4ay.  (JEE MAIN)

Sol: By using above mentioned formula of finding the area enclosed Y


between two curves, we can obtain required area.
2
x =4ay

Given, y2 = 4ax … (i)


x2 = 4ay … (ii)
Solving (i) and (ii), we get x = 4a and y = 4a.
X
4a O x=4a
4a  x2   x3/2 x3 
So required area = ∫0  4ax −  dx
= 2 a − 
 4a   3 / 2 12a 
  0

4 a 64a3 16 2
2

=
y =4ax
| 4a |3/2 − = a
3 12a 3 Figure 25.14

(b) Area enclosed by two curves intersecting at one point and the X-axis. Y

If y = f1(x) and y = f2(x) are two curves which intersect at a point P (α, β) and meet
x-axis at A (a, 0) and B (b, 0) respectively, then the area enclosed between the P(a,b)
x)

curves and x-axis is given by


f1 (
Y=

Y=
f 2(

α b
x)

Area = ∫ f1 (x)dx + ∫ f2 (x)dx O A(a,0) (a,0) B(b,0)


X
a α

Figure 25.15
2 5 . 8 | Area Under the Cur ve and Linear Programming

(c) Area bounded by two intersecting curves and lines parallel to y-axis.
The area bounded by two curves y = f(x) and y = g(x) (where a ≤ x ≤ b), when they intersect at x = c ∈ (a, b), is given
b c b
by
= A ∫ | f(x) − g(x) | dx ⇒ =
A ∫ (f(x) − g(x))dx + ∫ (g(x) − f(x))dx
a a c

Y y=f(x) y=g(x)

X’ X
O x=a x=c x=b
Y’
Figure 25.16

Illustration 14: Draw a rough sketch of the region enclosed between the circles x2 + y2 = 4 and (x – 2)2 + y2 =4. Using
method of integration, find the area of this enclosed region  (JEE ADVANCED)

Sol: By solving given equations simultaneously, we will be get intersection points of circles and then by using
integration method we can obtain required area.
The figure shown alongside is the sketch of the circles
x2 + y2 = 4  … (i)
and, (x – 2) + y = 4
2 2
… (ii)
From (i) and (ii), we have (x – 2)2 - x2 = 0 Y

⇒ (x – 2 – x)(x – 2 + x) = 0 ⇒ x = 1 … (iii) A(1,-3)


(1) (2)
Solving (i) and (iii), we get y= ± 3
C
Therefore, the circles (i) and (ii) intersect at A(1, 3) and B(1, − 3) . X’ O (2,0)
X

Area of enclosed region = Area OACBO = 2 Area OACO


B(1,-3)
= 2 [Area OAD + Area ACD]
Y’
1 2
= 2∫ 4 − (x − 2)2 dx + 2∫ 4 − x2 dx Figure 25.17
0 1

2 1
= 2∫ 4 − x2 dx + 2∫ 4 − (x − 2)2 dx
1 0

2 1
 x 4 − x2 4 x  (x − 2) 4 − (x − 2)2 4  x − 2   x 2 a2 x
2 2 2
=2 + sin−1  +2  + sin−1     a − x dx ⇒ a − x + sin−1 
 2 2 2  2 2  2 
   2 2 a 
 1  0

 3  π   3 π  8π
= 2 π − − 2  + 2 − − 2  + π = − 2 3 sq. units
   
 2  6   2 6  3

Illustration 15: Using integration, find the area of the region given below:
{(x, y): 0 ≤ y ≤ x2 + 1, 0 ≤ y ≤ x + 1, 0 ≤ x ≤ 2}  (JEE ADVANCED)
M a them a ti cs | 25.9

Sol: Same as above illustration, by solving given equation y = x2 + 1 and Y

1
x+
y = x + 1 we will be get their points of intersection and after that using
2
y=x +1

y=
integration method and taking these points as limit we can obtain required
F

area.
(1, 2)
C
The region is shaded as shown in the Fig. 25.18. B x=2
A
Given, y=x +1
2
… (i) (0, 1)

y = x + 1  … (ii) X
O D E
X

On solving (i) and (ii), we have x2 + 1 = x + 1 Figure 25.18


⇒ x = 0, 1 and y = 1, 2
∴ The shaded region can be divided into two parts OABCDO and CDEFC.
Limits for the area OABEO are x = 0 and x = 1.
Limits for the area EBDFE are x = 1 and x = 2.
Area of the shaded region = Area OABEO + Area EBDFE.
1 2
1 2 2  x3   x2  1  4 1  23
= ∫ (x + 1)dx + ∫ (x + 1)dx =  + x  +  + x  =  + 1  +  + 2 − − 1  = sq. units
 3  0  2 1  3  2 2  6
0 1

Illustration 16: Find the area of the following region: [(x, y): y2 ≤ 4x, 4x2+4y2 ≤ 9]  (JEE ADVANCED)

Sol: Similar to above problem, Here the required area is equal to Area AOBA + Area ACBA.
Given y2 = 4x … (i)
2
3
4x2 + 4y2 = 9 ⇒ x2 + y2 =    … (ii)
2
Y
1  1 
Curves (i) and (ii) intersect at A  , 2  and B  , − 2 
 2   2 
9
=
2

A
4y

2
1 y = 4x
Limits for the area OAB are x = 0, x =
+
4x 2

2
1 3 C
Limits for the area ACB are
= x = ,x . X’

( (
X
2 2 O x=1 3
,0
2 2
The required area = Area AOBA + Area ACBA
B
 1/2 3/2 9 
= 2  ∫ y1 dx + ∫ y 2 =
dx  2  ∫
1/2 3/2
4x dx + ∫ − x2 dx 
 0 1/2   0 1/2 4  Y’

1/2 3/2 Figure 25.19


2  x 9 9  x 
= 4  x3/2  + 2 . − x2 + sin−1  
3 0  2 4 8  3 / 2 1/2

8 1  1 9 1  4 1 9π 1 1 9 1
= . + 0 − +  sin−1 1 − sin−1   = − +  − sin−1  = + cos−1 .
3 2 2  2 4 3  3 2 2 42 3 3 2 4 3

Illustration 17: Draw a rough sketch and find the area of the region bounded by the two parabolas y2 = 8x and x2 = 8y,
by using method of integration. (JEE MAIN)
2 5 . 1 0 | Area Under the Cur ve and Linear Programming

Sol: As the given two equation is the equation of parabola


which intersect at O(0, 0) and A(8, 8), and the required area
2
Y

is equal to Area OBADO – Area OADO. A(8, 8)

Given parabolas are y2 = 8x


1
… (i) 2
x = 8y B
and, x2 = 8y … (ii)
The curves (i) and (ii) intersect at O(0, 0) and A(8, 8).
X’ X
∴ Required Area = Area OBADO – Area OADO O D

8
= ∫ (y1 − y 2 )dx
0

8
8 x2   x3/2 1 x3  64 2
y = 8x
= ∫  8x −  dx= 2 2 . −  = sq. units.
0 8  3 / 2 8 3  3 Y’
  0
Figure 25.20

Illustration 18: Find the area between the curves y = 2x, x + y = 1 and x-axis.  (JEE MAIN)
1 2
Sol: Here y = 2x and x + y = 1 is a two line intersect at p  ,  , therefore using integration method we can obtain
required area. 3 3

Given y = 2x … (i)


and, x + y = 1  … (ii)
Solving (i) and (ii), we get x + 2x = 1 ⇒ x = 1/3.
Line (i) intersects with the x – axis at the origin and the line (ii) intersects with the x – axis at x = 1.

P(x = 1/3)
x+
2x

y=
=

1
Y

O A
X
X=0 X=1/3 X=1

Figure 25.21

1
1/3 1 1/3  x2 
So required area = ∫0 =  x2  +  x − 
2x dx + ∫ (1 − x)dx
1/3  0  2 
 1/3

1 1  1 1  1
= + − −  = sq. units
9  2   3 18  3

Illustration 19: Using the method of integration, find the area of the region bounded by lines: 2x + y = 4, 3x – 2 y
= 6 and x – 3y + 5 = 0 (JEE ADVANCED)

Sol: Same as above problem.


Given equation of the lines are 2x + y = 4  … (i)
M a them a ti cs | 25.11

3x – 2y = 6 … (ii)
x – 3y + 5 = 0 … (iii)
Solving (i) and (ii), we get (2, 0)  Y
Solving (ii) and (iii), we get (4, 3)
0

2x
Solving (i) and (iii), we get (1, 2) + 5= (4,3)

+
- 3y

y=
X
4 x +5 4  3x − 6 

4
2 (1,2)
∴ Required Area = ∫1   dx − ∫1 (4 − 2x)dx − ∫2   dx

6
3   2 

y=
-2
4 4
1  x2  1  3x2 

3x
=  + 5x  − [4x − x2 ]12 −  − 6x  X
3  2 1 2  2  2 O (2,0)

1 1  1 Figure 25.22
= (8 + 20) −  + 5   − [(8 − 4) − (4 − 1)] − [(24 − 24) − (6 − 12)]
3 2  2

7
= = sq. units.
2

SKETCH OF STANDARD CURVES

2 2 2
Y x +y =a
x y
b + =1 (0 ,0)
a b

O a X

2
Y x = 4by
2
y = 4ax (b > 0)

2 (a > 0)
y = -4ax X
X

2
x = -4by

2 2
Y x +y Y
2 =1
2 2
2
a b x -y
2 2 =1
a b
X
O
X
O
2 5 . 1 2 | Area Under the Cur ve and Linear Programming

Y X Y
1 y = sinx 3
2
xy = c  2
 O x
-
O 2 -1 2
y = cosx

y=tanx Y
Y
y=cotx
y = logex


O  O 1
X
2
y=tanx y=cotx -

x
y=e
1

- O X

4. STANDARD AREAS
2
x = 4by
Y 2
y = 4ax

4.1 Area Bounded by Two Parabolas


X
Area between the parabolas y2 = 4ax and x2 = 4; a > 0, b > 0, is O k

16ab
| A |=
3
Figure 25.23

Illustration 20: Find the area bounded by y = x and x = y . (JEE MAIN)

Sol: By using above mentioned formula.


Y xx =
= yy
Area bounded is shaded in the figure
yy =
= xx
1 1
Here, a = and b =
4 4

\Using the above formula, Area = (16 ab)/ 3


X
O

Figure 25.24
16 × (1 / 4) × (1 / 4) 1
= =
3 3
M a them a ti cs | 25.13

4.2 Area Bounded By Parabola and a Line Y y = mx


2
y = 4mx
8a2
Area bounded by y = 4ax and y = mx; a > 0, m > 0 is A =
2

3m3
O
Area bounded by x2 = 4ay and =
y mx; a > m > 0 x=c
X

8a2
is =
y mx; a > m > 0 A =
3m3
Figure 25.25
Illustration 21: Find the area bounded by, x2 = y and y = |x|. (JEE MAIN)

8a2
Sol: Using above formula, i.e. A = Y
3m3

Area bounded is shaded in the Fig. 25.26.


Here, a = 1/4, m = 1 2
1
2×8×  x
 8a2  4 1 x=-c O x=c
\Using the above formula, Area =
= 2  =
 3m 
3
3 × (1)3 3
  Figure 25.26
Illustration 22: Find the area bounded by y = x and x = | y |. 
2
(JEE MAIN)
Y
Sol: Here, a = 1/4, m = 1, and required area is divided in to two equal parts at above and
below x – axis. x=c

 8a2 
Hence required area will be 2  . X
 3m3  O
 
 8a2  2 × 8 × (1 / 4 )
2 x=-c
1
\Using the above formula, Area = = 2  =
 3m3  3 × (1)3 3
 
Figure 25.27

4.3 Area Enclosed by Parabola and It’s Chord Y

Area between y = 4ax and its double ordinate at x = a is


2
D A
2
Area of AOB = (area ABCD)
3
O X

B
C
x=a

Figure 25.28

Illustration 23: Find the area bounded by y = 2x – x2, y + 3=0. (JEE MAIN)

Sol: Here first obtain area of rectangle ABCD and after that by using above
mentioned formula we will be get required area. D C
Solving y = 2x – x2, y + 3 = 0, we get x = –1 or 3 -1
X
3
Area (ABCD) = 4 × 4 = 16.
y = -3
A B
2 32
∴ Required area = × 16 =
3 3 Figure 25.29
2 5 . 1 4 | Area Under the Cur ve and Linear Programming

4.4 Area of an Ellipse (0, b)

x2 y2
For an ellipse of the form + 1 is
= A = πab (a, 0)
a2 b2

Figure 25.30
PLANCESS CONCEPTS

Try to remember some standard areas like for ellipse, parabola. These results are sometimes very helpful.

Vaibhav Gupta (JEE 2009 AIR 54)

5. SHIFTING OF ORIGIN
Area remains unchanged even if the coordinate axes are shifted or rotated or both. Hence shifting of origin /
rotation of axes in many cases proves to be very convenient in finding the area.
For example: If we have a circle whose centre is not origin, we can find its area easily by shifting circle’s centre.
Illustration 24: The line 3x + 2y = 13 divides the area enclosed by the curve 9x2 + 4y2 – 18x – 16y – 11 = 0 into two
parts. Find the ratio of the larger area to the smaller area.  (JEE ADVANCED)

Sol: Given 9x2+ 4y2 – 18x – 16y – 11 = 0 … (i)

and, 3x + 2y = 13 … (ii)

9(x2 – 2x) + 4 (y2 – 4y) = 11;

⇒ 9[(x – 1)2 – 1] + 4 [(y – 2)2 – 4] = 11

⇒ 9(x – 1)2 + 4(y – 2)2 = 36

(x − 1)2 (y – 2)2 X2 Y2
⇒ + 1⇒
= + 1
= (where X = x – 1 and Y = y – 2)
4 9 4 9 y
Hence 3x + 2y = 13 (0,3) 3x + 2y = 6
A1
⇒ 3(X + 1) + 2(Y + 2) = 13 Q
A2
⇒ 3X + 2Y = 6 P
x
O
X Y (2, 0)
⇒ + =1
2 3
∴ Area of triangle OPQ = 1/2 × 2 × 3 = 3
Figure 25.31
Also area of ellipse = π (semi major axes) (semi minor axis) = π.3.2 = 6π
6π 3π
A1 = − area of ∆OPQ = − 3
4 2

 6π  9π
A2 =3   + area of ∆OPQ = + 3
 4  2

A2 +3
2 3π + 2
Hence,
= =
A1 3π π−2
−3
2
M a them a ti cs | 25.15

6. DETERMINATION OF PARAMETERS
In this type of questions, you will be given area of the curve bounded between some axes or points, and some
parameter(s) will be unknown either in equation of curve or a point or an axis. You have to find the value of the
parameter by using the methods of evaluating area.

4
Illustration 25: Find the value of c for which the area of the figure bounded by the curves y = ; x = 1 and
9 x2
y = c is equal to .  (JEE MAIN)
4

Sol: By using method of evaluating area we can find out the value of c. Y
2
.c
1 1 c
 4  9  4 9 y=c
A= ∫ c − 2  dx = ;  cx +  =
x 4  x 2 4
2/ c 
c
9 9
(c + 4) − (2 c + 2 c ) =; c − 4 c + 4 =
4 4 X
O
9 3 3 x=1
⇒ ( c − 2)2 =⇒ ( c − 2) = or –
4 2 2 Figure 25.32
Hence c = (49/4) or (1/4)
Illustration 26: Consider the two curves:
C1 : y = 1 + cos x, and C2 : y = 1 + cos(x – α) for α ∈ (0, π/2) and x ∈ [0, p].
Find the value of α, for which the area of the figure bounded by the curves C1, C2 and x = 0 is same as that of the
area bounded by C2, y = 1 and x = π. For this value of α, find the ratio in which the line y = 1 divides the area of the
figure by the curves C1, C2 and x = π.  (JEE ADVANCED)

Sol: Solve C1 and C2 to obtain the value of x, after that by following given condition we will be obtain required
value of α.
Solving C1 and C2 , we get
α
1 + cos x = 1 + cos(x – α) ⇒ x = α – x ⇒ x =
2
According to the question, C1
A1 + A2 = A
α /2 π 2
∫ (cos x − cos(x − α ))dx = − ∫ (cos(x − α ))dx A1 C2
0 π

2 1
π y=1
α /2 +α
⇒ sinx − sin(x − α )=
 sin(x − α ) 2 A2
0 π
/2
O    
 α  α  π +
⇒ sin − sin  −   − [0 − sin( −α )] = sin   − sin( π − α ) 6 2 2
 2  2  2
Figure 25.33
α α π
⇒ 2sin − sin α =1 − sin α . Hence, 2sin = 1 ⇒ α=
2 2 3

7. AREA BOUNDED BY THE INVERSE FUNCTION


The area of the region bounded by the inverse of a given function can also be calculated using this method. The
graph of inverse of a function is symmetric about the line y = x. We use this property to calculate the area. Hence,
area of the function between x = a to x = b, is equal to the area of inverse function from f(a) to f(b).
2 5 . 1 6 | Area Under the Cur ve and Linear Programming

Illustration 27: Find the area bounded by the curve g(x), the x-axis and the lines at y = –1 and
x3 x2 13x
y = 4, where g(x) is the inverse of the function f(x) = + + +1 .  (JEE MAIN)
24 8 12

Sol: Here f(x) is a strictly increasing function therefore required area will be
2 0 Y
A=∫ (4 − f(x))dx + ∫ (f(x) + 1)dx

0 −2 6 f(x)

x3 x2 13x
Given f(x) = + + +1
24 8 12
f(1) = 6
⇒ f(0) = 1; f(2) = 4 and f(–2) = –1 (inflection
(0, 2)
point )
x2 x 13
Also, f ' (x) = + + , -1
8 4 12 O X
f(-1) = 2
i.e. f(x) is a strictly increasing function. -2

2 0 Figure 25.34
∴ A= ∫ (4 − f(x))dx + ∫ (f(x) + 1)dx
0 −2

2  0  3 
x3 x2 13x x x2 13x
A= ∫  24 8 12 
 4 − − − − 1  dx + ∫  24 8 + 12 + 1 + 1  dx
 +
0  −2  
 24 23 13.22     24 23 13.22   16
∴ A=  3.2 − − −  − (0) + (0) −  − + − 2.2  =
 24.4 8.3 12.2   24.4 8.3 12.2  3
     

Illustration 28: Let f(x) = x3 + 3x + 2 and g(x) is the inverse of it. Find the area bounded by g(x), the x-axis and the
ordinate at x = –2 and x = 6.  (JEE ADVANCED)

6
Sol: Let A = ∫ f −1 ( x ) dx Y
−2 4
Substitute x = f (u) or u = f −1 ( x )
f −1 ( 6 )
= ∫ u f −1 (u) du (0, 1)

f −1 ( 2 )
-2
O 2
X
-1
f −1 ( 6 )
= ∫ (
4 3u2 + 3 du ) Figure 25.35
f −1 ( 2 )

2 and f (1 ) = 6
We have, f ( −1 ) =
1 1

∫ u (3 u ) ∫ (3 u )
2 3
= + 3 du 2
= + 3u du
−1 0
1
3  9
=  u4 + 3u2  = Sq. units.
 2 0 2
M a them a ti cs | 25.17

8. VARIABLE AREA
If y = f(x) is a monotonic function in (a, b), then the area of the function y = f(x) bounded by the lines at x = a,
a+b
x = b, and the line y = f(c), [where c ∈ (a, b)] is minimum when c = .
2
c b Y
Proof: A = ∫ f(c) − f(x)dx + ∫ (f(x) − f(c))dx
y=f(x)
f(b)
a c
(0,f(c)) y=f(c )
c b f(a)
= f(c) (c − a) − ∫ (f(x))dx + ∫ (f(x))dx − f(c)(b − c)
a c
x
b c O x=a c x=b
= {(c – a) – (b – c)} f(c) + ∫ (f(x))dx − ∫ (f(x))dx
c a
Figure 25.36

b c
A = [2c − (a + b)] f(c) + ∫ (f(x))dx − ∫ (f(x))dx
c a
dA a+b a + b dA
For maxima and minima = 0 ⇒ f ‘ (c) = [2c – (a + b)] = 0 (as f ‘ (c) = 0) hence c = also c < , <0
dc 2 2 dc
a + b dA a+b
and c > , > 0 Hence A is minimum when c =
2 dc 2

9. AVERAGE VALUE OF A FUNCTION


In this section, we would study the average of a continuous function. This concept of average is frequently applied
in physics and chemistry.
b
1
b − a ∫a
Average of a function f(x) between x = a to x = b is given by y av = f(x)dx

PLANCESS CONCEPTS

(a) Average value can be positive, negative or zero .


b
1
b ∫0
(b) If the function is defined in (0, ∞), then y av = Lim f(x)dx provided the limit exists
b→∞
1
 1 b 2 2
(c) Root mean square value (RMS) is defined as ρ = ∫ f (x)dx 
 b − a a 

(d) If a function is periodic then we need to calculate average of function in particular time
period that is its overall mean.
Vaibhav Krishnan (JEE 2009 AIR 22)

Illustration 29: Find the average value of y2 w.r.t. x for the curve ay = b a2 − x2 between x = 0 & x = a. Also find
the average value of y w.r.t. x2 for 0 ≤ x ≤ a.  (JEE MAIN)
b
1
b − a ∫a
Sol: As average of a function f(x) between x = a to x = b is given by y av = f(x)dx
2 5 . 1 8 | Area Under the Cur ve and Linear Programming

a
b2 b2 2 2b2
Let f(x) = y =
2 (a2 − x2 ) Now f(x)
= |av x2 )dx
∫ (a − =
a2 a2 (a − 0) 0 3

a2 a2 a2
1 2 b 2 2 2 b 2 2ba3
Again yav w.r.t. x as f(x) |av
2
=
(a2 − 0)
∫ y d(x =
)
a2a
∫ a − x dx=
a3
∫ 2t dt= 3
0 0 0

10. DETERMINATION OF FUNCTION


Sometimes the area enclosed by a curve is given as a variable function and we have to find the function. The area
dAax
function Aax satisfies the differential equation = f(x) with initial condition Aaa = 0 i.e. derivative of the area
dx
function is the function itself. Thus we can easily find f(x) by differentiating area function.

PLANCESS CONCEPTS

Aax
If F(x) is integral of f(x) then, = ∫ f(x)dx
= [F(x) + c]

And since, Aaa= 0= F(a) + c ⇒ c = – F(a).

Aax F(x) − F(a) . Finally by taking x = b we get, =


∴ = Aba F(b) − F(a)
Note that this is true only if the function doesn’t have any zeroes between a and b.
If the function has zero at c then area = |F(b) – F(c)| + |F(c) – F(a)|
Vaibhav Gupta (JEE 2009 AIR 54)

Illustration 30: The area from 0 to x under a certain graph is given to be A = 1 + 3x − 1 , x ≥ 0 ;


(a) Find the average rate of change of A w.r.t. x and x increases from 1 to 8.
(b) Find the instantaneous rate of change of A w.r.t. x at x = 5.
(c) Find the ordinate (height) y of the graph as a function of x.
(d) Find the average value of the ordinate (height) y, w.r.t. x as x increases from 1 to 8.  (JEE ADVANCED)

Sol: Here by differentiating given area function we can obtain the main function.

A(8) − A(1) 3
(a) A(1) = 1, A(8) = 4 ; =
8 −1 7

dA 1. 3 3
(b)
= =
dx x =5 2 1 + 3x x =5 8

3
(c) y =
2 1 + 3x
8 8
1 3 1 3 3
(d) = ∫ dx = ∫ dx
(8 − 1) 1 2 1 + 3x 7 1 2 1 + 3x 7
M a them a ti cs | 25.19

Illustration 31: Let C1 & C2 be the graphs of the function y = x2 & y = 2x, 0 ≤ x ≤ 1 respectively. Let C3 be the graphs
of a function y = f(x), 0 ≤ x ≤ 1, f(0) = 0. For a point P on C1, let the lines through P, parallel to the axes, meet C2 &
C3 at Q & R respectively (see figure). If for every position of P(on C1), the area of the shaded regions OPQ & ORP
are equal, determine the function f(x).  (JEE ADVANCED)

Sol: Similar to the above mentioned method. C2:y=2x


h2 h
 y 2 (1/2,1) C1:y=x
2

∫ y −
2


dy =∫ (x − f(x))dx differentiate both sides w.r.t. h (0,1)
(1,1)
0 0 C1

 h2 
Q 2

 h −  2h = h2 – f(h)
P(h,h )
 2 

 h2 
f(h) = h2 –  h −  2h O
 2 
 (0,0) R

C3:y=f(x)
= h2 – h(2h – h2) = h2 – 2h2 + h3
Figure 25.37
f(h) = h – h 3 2

f(x) = x3 – x2 = x2(x – 1)

11. AREA ENCLOSED BY A CURVE EXPRESSED IN POLAR FORM


r = a (1 + cosθ) (Cardioid)
2π 2π
1 2 a2 θ
= 4
2 ∫0 ∫ 4 cos
A =r d θ dθ
2 0
2

θ
Substitute = t , dθ = 2dt
2
π
3πa2
A= a2 ∫ 4 cos4 t dt= 8 ×
0
16 Figure 25.38

Illustration 32: Find the area enclosed by the curves x = a sin3t and y = acos3t.  (JEE MAIN)

Sol:
2 2 2
a3 and dx = 3asin2 t.cos t.dt
x3 + y3 =
a π /2
A = 4 ∫ y dx ; A = 4a2 ∫ 3cos
3
t sin2 t cos t dt
0 0
a
π /2 2 2
1.3.1 π 12a π 3πa
=A 12a2 ∫ sin2 t cos=
4
t dt (12a2 ) . = . =
0
6.4.2 2 32 8

Figure 25.39
2 5 . 2 0 | Area Under the Cur ve and Linear Programming

Linear Programming

1. INTRODUCTION
Linear Programming was developed during World War II, when a system with which to maximize the efficiency of
resources was of utmost importance.

2. LINEAR PROGRAMMING
Linear programming may be defined as the problem of maximising or minimising a linear function subject to linear
constraints. The constraints may be equalities or inequalities. Here is an example.
Find numbers x1 and x2 that maximize the sum x1 + x2 subject to the constraints x1 ≥ 0, x2 ≥ 0, and
x1 + 2x2 ≤ 4
4x1 + 2x2 ≤ 12
−x1 + x2 ≤ 1
Here we have two unknowns and five inequalities (constraints). Notice that these constraints are all linear functions of
the variables. The first two constraints, x1 ≥ 0 and x2 ≥ 0, are special. These are called no negativity constraints and are
often found in linear programming problems. The other constraints are called the main constraints. The function to
be maximised (or minimized) is called the objective function. In the above example the objective function is x1 + x2 .

3. GRAPHICAL METHOD
As we have only two variables, we can solve this problem by plotting the constraints with x1 and x2 as axes. The
intersection region of these inequalities is called feasible region for the objective function. This is the region which
satisfies all the constraints. Now from this feasible region we have to select point(s) such that objective function is
maximized or minimized.

Theorem 1: Let R be the feasible region (convex polygon) for a linear programming problem and let Z=ax + by
be the objective function. When Z has an optimal value (maximum or minimum), where the variables x and y are
subject to constraints described by linear inequalities, this optimal value must occur at a corner point (vertex) of
the feasible region.

Theorem 2: Let R be the feasible region for a linear programming problem and let Z=ax + by be the objective
function. If R is bounded, then the objective function Z has both a maximum and a minimum value on R and each
of these occurs at corner point (vertex) of R.

Remark: If R is unbounded, then a maximum or a minimum value of the objective function may not exist. However,
if it exists it must occur at a corner point of R. (By Theorem 1).
So for the above example
6 4x1 + 2x2 = 12
Corner point (x1, x2) Z (= x1 + x2) value
-x1 + x2 = 1
0,1 1
3,0 3
2
8/3,2/3 10/3 1 -x1 + 2x2 = 4
2/3,5/3 7/3
3 4

Figure 25.40
M a them a ti cs | 25.21

Hence (8/3, 2/3) is the optimal solution.


Note that z has also minimum value in the feasible region at (0, 1).
This method of solving is generally called as corner point method. Note that a function can have more than one
optimal points.

4. MODELS
There are few important linear programming models which are more frequently used and some of them we
encounter in our daily lives.
(a) Manufacturing/Assignment problems: In these problems, we determine the number of units of different
products which should be produced and sold by a firm when each product requires a fixed manpower,
machine hours, labour hours per unit of product, warehouse space per unit of the output. In order to make
maximum profit.
Example: There are I persons available for J jobs. The value of person i working 1 day at job j is aij , for i =
1,……,I, and j = 1,…..,J. The problem is to choose an assignment of persons to jobs to maximize the total value.
An assignment is a choice of numbers, xij , for i = 1,……,I, and j=1,……,J, where xij represents the proportion of
person i’s time that is to be spent on job j. Thus,
J
∑ xij ≤ 1 For i = 1,……, I  ... (i)
j=1
I
∑ xij ≤ 1 For j=1,……, J  ... (ii)
i=1

And xij ≥ 0 for i = 1,……,I, and j=1,……,J  ... (iii)

Equation (i) reflects the fact that a person cannot spend more than 100% of his time working, (ii) means that
only one person is allowed on a job at a time, and (iii) says that no one can work a negative amount of time
I J
on any job, Subject to (i), (ii) and (iii) , we wish to maximize the total value of ∑∑ aij xij
=i 1 =j 1
(b) Diet problems: In these problems, we determine the amount of different kinds of nutrients which should
be included in a diet so as to minimise the cost of the desired diet such that it contains a certain minimum
amount of each nutrients.
Example: There are m different types of food, F1 ,........,Fm , that supply varying quantities of the n nutrients
, N1 ,........,Nn , that are essential to good health. Let c j be the minimum daily requirement of nutrient, N j
contained in one unit of food Fi . The problem is to supply the required nutrients at minimum cost.
Let y i be the number of units of food Fi to be purchased per day. The cost per day of such a diet is
b1 y1 + b2 y 2 + ......... + bm ym  … (i)
The amount of nutrient Nj contained in this diet is
a1 j y1 + a2 j y 2 + ......... + amj ym

For j = 1,…….., n. We do not consider such a diet unless all the minimum daily requirements are met, that is, unless

a1 j y1 + a2 j y 2 + ......... + amj ym ≥ c j For j = 1,…….., n  … (ii)



Of course, we cannot purchase a negative amount of food, so we automatically have the constraints

y1 ≥ 0, y 2 ≥ 0,................, ym ≥ 0  ... (iii)

Our problem is: minimize (i) subject to (ii) and (iii). This is exactly the standard minimum problem.
2 5 . 2 2 | Area Under the Cur ve and Linear Programming

(c) Transportation problems: In these problems, we determine a transportation schedule in order to find the
cheapest way of transporting a product from plants/factories situated at different locations to different
markets.
Example: There are I ports, or production plants, P1 ,..........PI , that supply a certain commodity, and there are
J markets, M1 ,..........MJ , to which this commodity must be shipped. Port Pi possesses an amount si of the
commodity (i=1,2,……I), and market M j must receive the amount rj of the commodity ( j = 1,……..J). Let bij be
the cost of transporting one unit of the commodity from port Pi to market M j . The problem is to meet the
market requirements at minimum transportation cost is
I J
∑∑ y ijbij  ... (i)
=i 1 =j 1
J
The amount sent from port Pi is ∑ yij ≤ yij and since the amount available at port Pi is si , we must have
j=1
J
∑ y ij ≤ si for i = 1,……,I  ... (ii)
j=1
I
The amount sent to market M j is ∑ yij , and since the amount required there is rj , we must have
i=1
I
∑ y ij ≤ rj for j = 1,……,I  ... (iii)
i=1
It is assumed that we cannot send a negative amount from PI to M j , we have
y ij ≥ 0 for I = 1,……..I and j =1,……..J.  ... (iv)
Our problem is minimize (i) subject to (ii), (iii) and (iv).

FORMULAE SHEET
b b
(a) Area bounded by a curve with x – axis: =
Area ∫=
y dx ∫ f(x)dx
a a
d d
(b) Area bounded by a curve with y – axis: Area = ∫ x dy = ∫ f(y)dy
c c
b t1
(c) Area of a curve in parametric form: Area = ∫ y dx = ∫ g(t)f '(t)dt
a t2
c b
(d) Positive and Negative Area: A = ∫ f(x)dx + ∫ f(x)dx ;
a c

(e) Area between two curves:


(i) Area enclosed between two curves intersecting at two different points.
b b
Area = ∫a (y1 − y 2 )dx = ∫a [f1 (x) − f2 (x)]dx
(ii) Area enclosed between two curves intersecting at one point and the x – axis.
α b
Area = ∫ f1 (x)dx + ∫ f2 (x)dx
a α

(iii) Area bounded by two intersecting curves and lines parallel to y – axis.
c b
Area = ∫ (f(x) − g(x))dx + ∫ (g(x) − f(x))dx
a c
M a them a ti cs | 25.23

(a) Standard Areas:


16ab
(i) Area bounded by two parabolas y2 = 4ax and x2 = 4by; a > 0, b > 0 : Area =
3
8a2
(ii) Area bounded by Parabola y2 = 4ax and Line y = mx : Area =
3m3
x2 y 2
(iii) Area of an Ellipse + 1 : Area = πab
=
a2 b2

Solved Examples

JEE Main/Boards is above the curve y = x2 y ≤ x ⇒ area is below the line


y=x
Example 1: Find area bounded by y = 4 – x2, x-axis and
the lines x = 0 and x = 2. Y

Sol: By using the formula of Area Y


Bounded by the x – axis, we can N(1, 1)
obtain
4

Required Area.
2 2
2
= ∫y =
dx ∫ (4 − x )dx
O
X
O M
0 0 2
2 1
 x3  8 16 1  x 2 x3  1
=  4x −  = 8 − = sq. units Area = ∫ (x − x )dx =2
 −  = sq. units
 3  3 3  2 3  6
 0 0  0

Example 2: Find the area bounded by the curve Example 4: Find the area of the region enclosed by
y2 = 2y – x and the y-axis. π
Y B y = sin x, y = cos x and x-axis, 0 ≤ x ≤ .
2
Sol: Here given equation is the 2 Y
equation of parabola with vertex
t Sol: Find point of intersection
(1, 1) and curve passes through is P. Therefore after obtaining
the origin. the co-ordinates of P and then P
integrating with appropriate
Curve is y2 – 2y = –x or (y – 1)2 = limits, we can obtain required
– (x – 1) Area. X
X O L /2
It is a parabola with O 2
At point of intersection P,
Vertex at (1, 1) and the curve passes through the origin. x=
π
as ordinates of y = sin
At B, x = 0 and y = 2 4
x and ; y = cos x are equal
Area
π 1 
2 2  y3 
2
4 Hence, P is  .  Required area
= ∫ x= 2
dy ∫ (2y − y ) dy =  y 2 −  = sq. units 4 2
 3  3
0 0  0 π/ 4 π /2
cos x dx = ( − cos x ) + ( sinx )
π/ 4 π /2
= ∫ sinx dx + ∫ 0 π/ 4
Example 3: Find the area of the region {(x, y): x ≤ y ≤ x}
2 0 π/ 4

 1   1 
Sol: Consider the function y = x2 and y = x Solving = − + 1  + 1 −  =2 − 2 sq. units
them, we get x = 0, y = 0 and x = 1, y = 1;x2 ≤ y ⇒ area  2   2
2 5 . 2 4 | Area Under the Cur ve and Linear Programming

Example 5: The area bounded by the continuous curve Example 8: Find the area bounded by the curves {(x, y) :
y = f(x), (lying above the x-axis), x-axis and the ordinates y ≥ x2, y ≤ |x|}
x = 1 and x = b is (b – 1) sin (3b + 4). Find f(x)
Sol: Here the region is symmetric about y-axis, the
Sol: Using Leibniz rule, we can solve given problem. required area is 2[area of shaded region in first
b quadrant].
∫ f(x)dx =
(b − 1)sin(3b + 4)
The curves intersect each other at x = 0 and x = ±1 as
1
Apply Leibniz Rule: differentiate both sides w.r.t. ‘‘b’’, shown in figure. The points of intersection are (–1, 1),
f(b) = sin (3b + 4) + 3(b – 1) cos (3b + 4) (0, 0) and (1, 1).
⇒ f(x) = sin (3x + 4) + 3(x – 1) cos (3x + 4) Since, the region is symmetric about y-axis, the required
area is 2[area of shaded region]
Example 6: Find the area bounded by the curve y = k sin 1 1
1 1  1
x and y = 0 from x = 0 to x = 2π. Hence, Area = 2∫ (x − x )dx = 2  x2 − x3  =
2
sq.
units. 0 2 3 0 3

Sol: Here the area of OAB is above the x-axis (y = 0) and


thus it is positive while the area BCD is below x-axis (y =
0) and in negative but equal in quantity. Example 9: Draw a rough sketch of the curve y = sin2x,
 π
Y x∈ 0,  . Find the area enclosed between the curve,
A  2 π
x-axis and the line x = .
2
B D π π π π
O  2
X Sol: Here by substituting x = 0, , , , we will
6 4 3 2
y = Sin x
get respective values of y. hence by plotting these
y = k sin x values we can draw the given curve.
π π
Some points on the sin2x graph are :
Area OAB = ∫ y dx= ∫ k sinx dx= k[ − cos x]0π
0 0
π π π π
= k[– cos p] – k [– cos 0] x 0
6 4 3 2
= k[– (–1)] – k [–(1)] = k + k = 2k y 0 0.25 0.5 0.75 1
∴ Total area = 4k sq. units. By plotting points and joining them, we trace the curve.
Area bounded by curve y = sin2x between x = 0 and
Example 7: Find the area bounded by the curve π
x= Y
x = a (θ – sin θ), y = a (1 – cos θ), 0 ≤ θ ≤ 2π, with x-axis. 2 A

Sol: Substitute the value of y and dx and integrate.


B
O X
θ= 2 π θ= 2 π /2
dx
∫ y dx
Area = = ∫ y dθ

=θ 0 =θ 0

2π Y’
= ∫ a(1 − cos θ)a(1 − cos θ)dθ π π π
2 2
0 12
= ∫ y=
dx ∫ sin2 x=
2 ∫0

dx (1 − cos2x)dx
= a2 ∫ (1 − 2cos θ + cos
2
θ)dθ 0 0
π
0
1 sin2x  2 1  π  
2π = x −  =  − 0  − (0 − 0)
 1  2 2  0 2  2  
= a2 ∫ 1 − 2cos θ + 2 (1 + cos2θ)
0 π
= sq.units

4
3 1 
= a2  θ − 2sin θ + sin2θ  = 3πa2 sq. units.
2 4 0
M a them a ti cs | 25.25

JEE Advanced/Boards Example 2: Let An be the area bounded by the curve


π
y = (tan x)n : n ∈ N and the lines x = 0, y = 0 and x = .
Example 1: A tangent is drawn to x2 + 2x – 4ky + 1 4
Prove that for n > 2, An + An–2 = and deduce
3 = 0 at a point whose abscissa is 3. The tangent is n−1
perpendicular to x + 3 = 2y. Find the area bounded by 1 1
< An < .
the curve, this tangent, x-axis and line x = –1 2n + 2 2n − 2

Sol: As we know multiplication of slopes of two Sol: We can write (tan x)n as tann−2 x (sec2 x − 1) .
perpendicular line is – 1, by using this, we can obtain π/ 4
the value of k and will get standard equation. After that tann−2 x (sec2 x − 1)dx we
using integration with respective limit, we will be get
Therefore by solving An= ∫
0
required area. can prove given equation.
Y
π/ 4
An = ∫ tann x dx : n > 2
0
A O B C
X π/ 4
3
= ∫ tann−2 x (sec2 x − 1)dx
-1
0
π /2
 tann−1 x 
 or An =   − An − 2
 n − 1  0
1
∴ An + An–2 =  … (i)
n−1
tann x ≤ tann–2 x
π
P (as 0 ≤ tan x ≤ 1 for 0 ≤ x ≤ )
4
dy x + 1 ⇒ An < An–2
x2 + 2x – 4ky + 3 = 0; = Tangent is perpendicular
dx 2k 1
to x + 3 = 2y ∴ An + An < An + An–2 = by (1)
n−1
x +11 1
∴ An <  … (ii)
∴   = −1 at x = 3 2(n − 1)
2k  2 
⇒ 1/k = –1 ⇒ k = –1 Similarly An + 2 < An

∴ Curve becomes (x + 1)2 = –4(y + 1/2) which is a ⇒ An + 2 + An < An + An


parabola with vertex at V(–1, –1/2). 1
or < 2An by (1)
Coordinates of P are (3, –9/2). (n + 2) − 1
Equation of tangent at P is y + 9/2 = –2(x – 3) 1
⇒ < An  … (iii)
2n + 2
B is (3/4, 0), C is (3, 0)
1 1
⇒ < An <
Required Area = Area (ACPV) – Area of triangle BPC. 2n + 2 2n − 2
3
x2 + 2x + 3 1
= ∫ dx − (BC)(CP) Example 3: A(a, 0) and B(0, b) are points on the ellipse
−4 2
−1 x2 y 2
3
+ =1 . Show that the area between the arc AB
1  x3  1 3 9 a2 b2
1
=  + x2 + 3x  −  3 −   
 3  and chord AB of the ellipse is ab (π – 2).
4  −1 2  4 2 4
Sol: Area between the chord and ellipse = Area
1  1  81 109 bounded by curve AB - Area of ∆ OAB.
=  27 + − (1 − 3)  − = sq. units.
4 3  16 48
b
Equation of line AB is : y =
− (x − a)
a
2 5 . 2 6 | Area Under the Cur ve and Linear Programming

b 2 1 
Equation of curve AB=
is y a − x2  , 0  on the x-axis.
a 2 
dy
Area of bounded region is It will have a turning points where =0
B dx
a
b  b  dy 1 3
∫  a a2 − x2 −  − (x − a)   dx ∴ = ( −3 − 4x) = 0 Þx = –
0  a  dx 2 4
d2 y 3
b a2 π a2  ( π − 2) Also = −4 . That is, it is a max. at x =
= 0 + −  = ab O A dx 2 4
a  4 2  4
Also it cuts y-axis where
Y
x = 0, then y = 1. Thus the
Alternate method:
shape of the curve is as B(0,1)
Area between the chord and ellipse = Area bounded by shown in the figure.
curve AB - Area of ∆ OAB
The required area is ABC. X
1 1
= πab − ab =
( π − 2) ab sq. units It is given by
A(-2,0) O

4 2 4 1/2 1/2
1
∫ y dx
= ∫ (2 − 3x − 2x2 )dx
Example 4: Find the area of the region bounded −2 −2
2
1 1/2
y = + 1, x =1 and tangent drawn at the point 1 3 2 2x3 
x = 2x − x − 
1 2  2 3 
P(2, 3/2) to the curve y = + 1. −2
x
1  1 31
2 3
Y 21
Sol: Here first obtain = 2   −   −    –
2 2 22 32 
equation of tangent and
N
M P(2.3/2)  
then use the formula for
area. 1 3 2 2 3 
2( −2) − ( −2) − ( −2 )
2 2 3
Equation of tangent at
y=1 
A
1 X
1  13  1  14  125
P(2, 3/2) to y = + 1 is O 1 2
=   −  −  = sq. units.
x 2  24  2  3  48
3 1
y− = − (x − 2) or x +
2 4
Example 6: Find the area of the region bounded by the
4y = 8.
curve x2 = 4y and x = 4y – 2.
Required area is area of region PMN
Sol: Solving given equation simultaneously, we will
2
 1  8−x get the point of intersection. Using these points as the
Area = ∫   + 1  −  dx
x 4  limits of integration, we calculate the required area.
1  
Y
2
 1 x2  5
=  lnx − x +  = ln2 − sq. units
 4 2  8
 1 (2, 1)
(-1, 1/4)
X
O
Example 5: Find the area of the region bounded by the 2
x x+2
1 The curve intersect each other, where = , or x2
x-axis and the curve y = (2 – 3x – 2x2). 4 4
2 – x – 2 = 0, or x = –1, 2
Sol: Here the curve will intersect the x-axis when y = 0, Hence, the points of intersection are (–1, 1/4) and (2, 1).
therefore by substituting y = 0 in the above equation we The region is plotted in figure. Since, the straight line
will get the points of intersection of curve and x – axis. x = 4y – 2 is always above the parabola x2 = 4y in the
1 interval [–1, 2], the required area is given by
⇒ 2 – 3x – 2x2 =0 or (2 + x) (1– 2x) = 0 or x = –2, x =
2 2
Thus, the curve passes through the points (–2, 0) and Area = ∫ [f(x) − g(x)]dx
−1
M a them a ti cs | 25.27

Sol: By solving these


2 2 Y
 x + 2 x2  1 1 2 1 3
Area = ∫  4 − 4  dx =  x + 2x − x 
4 2 3  −1 two equation
−1   P(a, a)
 simultaneously,
we can obtain their
B
1  8 1 1  9 intersection points and A
=  2 + 4 −  −  − 2 +   = sq. units.
4  3 2 3  8 then by subtracting
area of parabola from
O C X
area of circle we will
Example 7: Find by using integration, the area of the
be get the result.
ellipse ax2 + 2hxy + by2 = 1.
Y Solving the two equation, simultaneously we see that
Sol: The equation the two curves intersect at (0, 0), (a, a) and (a, –a). We
can be put in the form have to find the area of the region OAPBO, where P is
by2 + 2hxy + (ax2 – 1) = 0
L
y1 the point of intersection (a, a)
dx a
Cut an elementary strip. X
M O x y N Required area = ∫ [f(x) − g(x)]dx
Let the thickness of
2
0
strip = dx a a
= ∫ (2ax − x2 ) dx − ∫ ax dx
If y1, y2 be the values of y corresponding to any value 0 0
at x.
a a
Length of strip = y1 – y2 (2ax − x2 ) dx [a2 − (a − x)2 ] dx
Now, ∫ = ∫
0 0
2 2 2 2
= h x − b(ax2 − 1) = b − (ab − h2 )x2 To evaluate this integral, we substitute a – x = a sin θ
b b
and obtain
ab – h2 being positive here, since the conic is an ellipse. a 0

The extreme values of x, are given by ∫ x2 )dx


(2ax − = ∫ (acos θ)(−acos θ)dθ
0 π /2

b
y1 – y2 = 0, i.e., x = ± 2
π /2
2 21 π πa2
ab − h2 =∫ a cos =θ d θ a =
22 4
0
b/ab −h2 )
a
Hence, the area required = ∫ (y1 − y 2 )dx
Also =
a
2
a x3/2
2a2
− b/ab −h2 )
∫ (ax) dx =
3 3
0 0

b/(ab −h2 )
2  π 2
= ∫ b − (ab − h2 )x2 dx ∴ Required area = a2  −  sq. units
b  4 3
− b/(ab −h2 )

Example 9: Prove that the area of the region bounded


and putting (ab − h2 ) x = b sin θ , this becomes 5
by the curve a4y2 = x5(2a – x), is times to that of the
4
2
π /2
π circle whose radius is a.
∫ cos2 θ dθ = sq. units.
2
(ab − h ) −π /2 (ab − h2 )
Sol: The curve is a loop lying between the line x = 0 and
x = 2a and is symmetrical about the x-axis. Therefore
the required area Y
Example 8: Find the area of region lying above x-axis,
and included between the circle x2 + y2 = 2ax and the 2a

parabola y2 = ax. = 2 ∫ y dx
0
2a X
2 5/2
O x = 2a
=
a2
∫ x 2a − xdx
0
2 5 . 2 8 | Area Under the Cur ve and Linear Programming

To evaluate this integral, we put x = 2a sin2q. When, curves is not same.


1 4 5 
x = 0, θ = 0 and when x = 2a, θ = π ∴ Required area = 2  1 (x2 − 4)dx + (52 − x2 ) dx 
2 ∫
 2 4
∫ 
π /2 4
2 5/2 5
=
a2 0
∫ (2a) sin θ. 2a cos θ.4a sin θ cos θ dθ 4 5
2  x3  x 2 25 −1 x 
=  − 4x   + 2  25 − x + sin 
π /2 2 4  3 
  2 2 5 4
5.3.1.1 π 5a π 2
= 64a2 ∫ sin6 θ cos2 θ dθ = 64a2 . =
8.6.4.2 2 4
0 1  64  8 
=  − 16  −  − 8   +
5 2  3  3 
= x area of the circle whose radius is a.
4
 25 −1   25 −1 4  
2  0 + sin 1  −  6 + sin 
Example 10: Find the area bounded by the curves x2 +  2   2 5 
y2 = 25, 4y = | 4 – x2 | and y = 0.
1  32  4
=   + 25sin−1 1 – 12 − 25sin−1
Sol: Here x2 + y2 = 25 represent circle with centre at 2 3  5
origin and radius 5 unit. Therefore the required area
= 2 area ABC  16  π 4
=  − 12  + 25 − 25sin−1
 14 5   3  2 5
= 2  ∫ (4 − x2 )dx + ∫ 252 − x2 dx 
 2 4 4  20 π 4  −1 4 20 
= − + 25  −= sin−1   25cos −  sq. units.
Note: Here the portion is Y 3 2 5  5 3 
also bounded by two D
curves but we do not apply
B B(4,3)
A = ∫ [f(x) − g(x)]dx rule.
(-4,3) E
X
C’ A’ A C
Reason: Range of (-5,0) (-2,0) (2,0) (5,0)
integration of both the

JEE Main/Boards

Exercise 1 Q.3 Find the area bounded by the curve y = sinx, x-axis
and between x = 0, x = π.
Q.1 Write an expression for finding the area of the
shaded portion. Q.4 Write an expression for finding the area of the
shaded portion.
2
Y y=x +3 Y

3 x+y=5

X X
0 2 4 0

Q.2 Find the area bounded by the curve y = cos x, x-axis Q.5 Write an expression for finding the area bounded
and between x = 0, x = π. by the curve x2 = y and the line y = 2.
M a them a ti cs | 25.29

Q.6 Write an expression for finding the area of a circle Exercise 2


x2 + y2 = a2, above x-axis.
Single Correct Choice Type
Q.7 On sketching the graph of y = |x – 2| and evaluating
3 3 Q.1 The area of the figure bounded by the curve y = ex,
∫−1 | x − 2 | dx, what does ∫−1 | x − 2 | dx, represent on y = e–x and the straight line x = 1 is
the graph? 1 1
(A) e + (B) e −
e e
Q.8 Draw the rough sketch of the curve=y 3x + 4 and 1
find the area under the curve above x-axis and between (C) e + −2 (D) None of these
e
x = 0 and x = 4.

3 Q.2 The area bounded in the first quadrant by the


Q.9 Find the area under the curve y = above normal at (1, 2) on the curve y = 4x, x-axis & the curve
3
(1 − 2x)
x-axis and between x = –4 an x = –1. is given by
10 7 4 9
(A) (B) (C) (D)
Q.10 Find the area bounded by the curves y = 6x – x 2 3 3 3 2
and y = x2 – 2x.
Q.3 The area of the figure bounded by the curves
Q.11 Draw a rough graph of f(x)= x + 1 in the interval y = lnx and y = (lnx)2 is
[0, 4] and find the area of the region enclosed by the (A) e + 1 (B) e – 1 (C) 3 – e (D) 1
curve, x-axis and the lines x = 0 and x = 4.
Q.4 The area bounded by the curves y = x2 + 1 & the
Q.12 Find the area of the region bounded by the curve tangents to it drawn from the origin is:
xy – 3x – 2y – 10 = 0; x-axis and the lines x = 3, x = 4.
(A) 2/3 (B) 4/3 (C) 1/3 (D) 1

Q.13 Find the area bounded by the curve y = x sin x2, πx


Q.5 The area bounded by x2 + y2 – 2x= 0 & y = sin
π 2
x-axis and between x = 0 and x = . in the upper half of the circle is
2
π 4 π 2 8 π 2
(A) − (B) − (C) π − (D) −
Q.14 Using integration, find the area of the region 2 π 4 π π 2 π
bounded by the following curves, after making a rough
sketch: Q.6 Consider the region formed by the lines x = 0, y =
y = |x + 1| + 1, x = –2, x = 3, y = 0. 0, x = 2, y = 2. Area enclosed by the curves y = ex and
y = lnx, within this region is removed, then the area of
Q.15 Draw the rough sketch of y = sin 2x and determine the remaining region is
the area enclosed by the curve, the x-axis and the lines (A) 2(1 + 2  n2) (B) 2(2  n2 – 1)
x = π/4 and x = 3π/4.
(C) (2  n2 – 1) (D) 1 + 2  n2
Q.16 Find the area of the following region: {(x, y) :
x2 + y2 ≤ 2ax, y2 ≥ ax, x ≥ 0, y ≥ 0}. Q.7 The area bounded by the curves y = x(1 – lnx);
x = e–1 and positive x-axis between x = e–1 and x = e is
Q.17 Find the area bounded by the curve y2 = 4a2(x – 3)  e2 − 4e−2   e2 − 5e−2 
and the lines x = 3, y = 4a. (A)   (B)  
 5   4 
   
Q.18 Make a rough sketch of the region given below
 4e2 − e−2   5e2 − e−2 
and find its area using integration. {(x,y): 0 ≤ y ≤ x2 + 3 ; (C)   (D)  
 5   4 
0 ≤ y ≤ 2x + 3, 0 ≤ x ≤ 3}.    

Q.19 Determine the area enclosed between the curve


y = 4x – x2 and the x-axis.
2 5 . 3 0 | Area Under the Cur ve and Linear Programming

Q.8 The positive values of the parameter ‘a’ for which Q.15 The area of the region for which 0 < y < 3 – 2x – x2
the area of the figure bounded by the curve y = cos ax, & x > 0 is
π xπ 3 3
y ==0, x = ,x is greater than 3 are (A) ∫ (3 − 2x − x2 )dx (B) ∫ (3 − 2x − x2 )dx
6a 2a
1 0
(A) f (B) (0, 1/3)
1
(C) (3, ∞) (D) None of these (C) ∫ (3 − 2x − x2 )dx (D) None of these
0

Q.9 The value of ‘a’ (a > 0) for which the area bounded Q.16 The graphs of f(x) = x2 and g(x) = cx3
x 1 1 1 
by the curves y =+ , y = 0, x = a and x = 2a has (c > 0) intersect at the points (0, 0) &  ,  . If the
6 x2
 c c2 
the least value, is
region which lies between these graphs & over the
(A) 2 (B) 2 (C) 21/3 (D) 1 interval [0, 1/c] has the area equal to 2/3 then the value
of c is
Q.10 The ratio in which the area enclosed by the curve
(A) 1 (B) 1/3 (C) 1/2 (D) 2
 π
y = cos x  0 ≤ 0 ≤  in the first quadrant is divided by
 2
Q.17 The curvilinear trapezoid is bounded by the
the curve y = sin x, is
curve y = x2 + 1 and the straight lines x = 1 and x =
2. The co-ordinates of the point (on the given curve)
(A) ( 2 − 1) : 1 (B) ( 2 + 1) : 1
with abscissa xÎ[1, 2] where tangent drawn cut off from
(C) 2 :1 (D) 2 +1 : 2 the curvilinear trapezoid are ordinary trapezium of the
greatest area, is
Q.11 The area bounded by the curve y = f(x), the (A) (1, 2) (B) (2, 5)
x
co-ordinate axes & the line x = x1 is given by x1. e 1 .
Therefore f(x) equals  3 13 
(C)  ,  (D) None of these
(A) ex (B) x ex (C) xex – ex (D) xex + ex 2 4 

Q.18 In the given figure, if A1 is the area of the ∆AOB


Q.12 The area bounded by the curves y =− −x and and A2 is the area of the parabolic region AOB then the
x =− − y where x, y ≤ 0 A
ratio 1 as a → 0 is
(A) Cannot be determined A2
Y
(B) Is 1/3
2
A y=a
B
(C) Is 2/3
2
(D) Is same as that of the figure bounded by the curves y=x

y= −x ; x ≤ 0 and x =− y ; y ≤ 0
0
Q.13 The area from 1 to x under a certain graph is given
by A = (1 + 3x)1/2 – 1, x ≥ 0.. The average value of y w.r.t. (A) 1 (B) 8/9 (C) 3/4 (D) 2/3
x as x increases from 1 to 8 is
(A) 3/7 (B) 1/2 (C) 3/8 (D) 4/7
Previous Years’ questions
Q.14 The slope of the tangent to a curve Q.1 The area of the quadrilateral formed by the
y = f(x) at (x, f(x)) is 2x + 1. If the curve passes through tangents at the end points of latus rectum to the ellipse
the point (1, 2) then the area of the region by the curve, x2 y 2
the x-axis and the line x = 1 is + = 1, is  (2003)
9 5
(A) 5/6 (B) 6/5 (C) 1/6 (D) 1
(A) 27/4 sq. unit (B) 9 sq. unit
(C) 27/2 sq. unit (D) 27 sq. unit
M a them a ti cs | 25.31

Q.2 The area bounded by the curves= y x ,2y =


+3 x 5 15 9 7
(A) (B) (C) (D)
and x-axis in the 1st quadrant is  (2003) 64 64 32 32
(A) 9 sq. unit (B) 27/4 sq. unit
Q.10 The area (in square units) bounded by the curves
(C) 36 sq. unit (D) 18 sq. unit
=y + 3 0, x-axis, and lying in the first
x ,2y − x =
Q.3 The area enclosed between the curves y = ax2 and quadrant is  (2013)
x = ay2 (a > 0) is 1 sq unit. Then, the value of a is (2004) 27
(A) 36 (B) 18 (C) (D) 9
1 1 1 4
(A) (B) (C) 1 (D)
3 2 3 Q.11 The area of the region described by A

Q.4 The area of the equilateral triangle, in which three {


= (x, y) : x2 + y 2 ≤ 1 and y 2 ≤ 1 − x is } (2014)
coins of radius 1 cm are placed, as shown in the figure, π 4 π 4
(A) + (B) −
is  (2005) 2 3 2 3

sq.cm
(A) (6 + 4 3)sq cm π 2 π 2
(C) − (D) +
2 3 2 3
sq.cm
(B) (4 3 − 6)sq cm
x −1 y +1 z −1
sq.cm
(C) (7 + 4 3)sq cm Q.12 If the lines = = and
2 3 4
x −3 y −k z
= = intersect, then k is equal to  (2012)
sq.cm
(D) 4 3 sq cm 1 2 1

2 9
Q.5 The area enclosed within the curve |x| + |y| = 1 (A) -1 (B) (C) (D) 0
is ……..  (1981) 9 2

y
Q.13 The area bounded between the parabolas x2 =
Q.6 The area of the triangle formed by the positive 4
x-axis and the normal and the tangent to the circle x2 + and x2 = 9y and the straight line y = 2 is  (2012)
y2 = 4 at (1, 3) is …….. (1989) 10 2 20 2
(A) 20 2 (B) (C) (D) 10 2
3 3
Q.7 The area bounded by the curves y = (x – 1)2,

y = (x + 1)2 and y =
1
is  (2005) Q.14 The area bounded by the curves y = cos x and
4 3π
y = sinx between the ordinates
= x 0=
and x
1 2 2
(A) sq. unit (B) sq. unit
3 3 (2010)
1 1 (A) 4 2 + 2 (B) 4 2 − 1
(C) sq. unit (D) sq. unit
4 5
(C) 4 2 + 1 (D) 4 2 − 2
Q.8 The area (in sq. units) of the region

{( x, y ) : y 2 2 2
}
≥ 2x and x + y ≤ 4x, x ≥ 0, y ≥ 0 is: (2016) Q.15 The area of the region enclosed by the curves
1
8 4 2 =y x,= x e,= y and the positive x-axis is (2011)
(A) π − (B) π − x
3 3
3
π 2 2 4 (A) 1 sq. unit (B) sq. units
(C) − (D) π − 2
2 3 3
5 1
(C) sq. units (D) sq. units
Q.9 The area (in sq. units) of the region described by 2 2

{( x, y ) : y 2
}
≤ 2x and y ≥ 4x − 1 is :  (2015)
2 5 . 3 2 | Area Under the Cur ve and Linear Programming

JEE Advanced/Boards

Exercise 1 curve y = f(x) and compute the area enclosed by the


curve and the pair of tangents.
Q.1 In the adjacent figure, graphs of two functions
y = f(x) and y = sinx are given. y = sinx intersects, Q.4 Show that the area bounded by the curve
y = f(x) at A(a, f(a)); B(π, 0) and C(2π, 0). Ai (i = 1, 2, 3) logx − c
is the area bounded by the curve y = f(x) and y = sin y= , the x-axis and the vertical line through
x
x between x = 0 and x = a; i = 1, between x = a and
the maxima point of the curve is independent of the
x = π; i = 2, between x = π and x = 2π; i = 3.
constant c.
Y
y = f (x) Q.5 Consider the curve y = xn where n > 1 in the 1st
A2 quadrant. If the area bounded by the curve, the x-axis
A (2,0)
A1 and the tangent line to the graph of y = xn at the point
y=sinx B
 C (1, 1) is maximum then find the value of n.
O x=a x
y=sinx
Q.6 For what value of ‘a’ is the area of the figure
1 1
A3 bounded by the lines= y = ,y , x = 2 &
x 2x − 1
4
x = a equal to ln ?
5
If Ai = 1 – sin a + (a – 1) cos a, determine the function
f(x). Hence determine ‘a’ and A1. Also calculate A2 and A3. 1
Q.7 For the curve f(x) let two points on it are
1 + x2
Q.2 The figure shows two regions in the first quadrant.  1  1 
A(α,f(α)), B  − ,f '  −   (α > 0). Find the minimum
2 2  α  α 
Y y=sin x P(t, sin t ) area bounded by the line segments OA, OB and f(x),
where ‘O’ is the origin.
A(t)
X’ X
O Q.8 Find the area bounded by the curve y = sin−1 x and
π
Y’ the lines x = 0, y = .
2
2
Y P(t, sin t )
Q.9 If f(x) is monotonic in (a, b) then prove that the area
bounded by the ordinates at x = a ; x = b ; y = f(x) and
A(t) a+b
X’ X y = f(c), c ∈ (a, b) is minimum when c = . Hence
O 2
x3
Y’ if the area bounded by the graph of f(x) = − x2 + a,
3
A(t) is the area under the curve y = sin x2 from 0 to t the straight lines x = 0, x = 2 and the x-axis is minimum
and B(t) is the area of the triangle with vertices O, P and then find the value of ‘a’.
A(t)
M(t, 0). Find Lim .
t → 0 B(t) Q.10 Let ‘c’ be the constant number such that c > 1.
If the least area of the figure given by the line passing
Q.3 A polynomial function f(x) satisfies the condition
through the point (1, c) with gradient ‘m’ and the
f(x + 1) = f(x) + 2x+ 1. Find f(x) if f(0) = 1. Find also the
parabola y = x2 is 36 sq. units find the value of (c2 + m2).
equations of the pair of tangents from the origin on the
M a them a ti cs | 25.33

Q.11 Let C1 & C2 be two Q.17 Find the values of m(m > 0) for which the area
curves passing through the Y
C2
C bounded by the line y = mx + 2 and x = 2y – y2 is,
origin as shown in the figure. C1 (i) 9/2 square units & (ii) minimum. Also find the
A curve C is said to ‘‘bisect minimum area.
the area’’ the region between A
C1 & C2, if for each point P of B Q.18 Find the area bounded by the curve y = x e–x ;
C, the two shaded regions A xy = 0 and x = c where c is the x-coordinate of the
& B shown in the figure have X curve’s inflection point.
equal area. Determine the
O

upper curve C2, given that the bisecting curve C has the Q.19 Find the area of the region
equation y = x2 & that the lower curve C1 has the
equation y = x2/2. {( x, y ) : y 2
≤ 4 x, 4x2 + 4y 2 ≤ 9 }
Q.12 Consider one side AB of a square ABCD, (read in Q.20 For what value of ‘a’ is the area bounded by the
order) on the line y = 2x – 17 and the other two vertices curve y = a2x2 + ax + 1 and the straight line y = 0, x = 0
C, D on the parabola y = x2. & x = 1 the least?

(i) Find the minimum intercept of the line CD on y-axis. 1


Q.21 Consider two curves C1: y = and C2: y = ln x
(ii) Find the maximum possible area of the square ABCD. x
on the xy plane. Let D1 denotes the region surrounded
(iii) Find the area enclosed by the line CD with minimum by C1, C2 and the line x = 1 and D2 denotes the region
y-intercept and the parabola y = x2. Consider the two surrounded by C1, C2 and the line x = a. If D1 = D2. Find
curves C1 : y = 1 + cos x & C2 : y = 1 + cos (x – α) for the value of ‘a’.
α ∈ (0, π/2) ; x ∈ [0, p]. Find the value of α, for which
the area of the figure bounded by the curves C1, C2 &
x = 0 is same as that of the figure bounded by C2, y = 1 & x Exercise 2
= π. For this value of α, find the ratio in which the line y = 1
divides the area of the figure by the curves C1, C2 & x = π. Single Correct Choice Type

Q.1 The area bounded by the curve y = x2 – 1 & the


Q.13 Draw the rough sketch of y 2= x + 1 and
straight line x + y = 3 is
y 2 =−x + 1 and determine area enclosed by the two
curves. 9 7 17 17 17
(A) (B) 4 (C) (D)
2 2 6
Q.14 Let f : (0, ∞) → R be a continuous and strictly
x Q.2 The area bounded by the curve y = e–x & the lines
3 2
increasing function such that f (x) = ∫ t f (t)dt, ∀ x > 0. y = e–4 & x = 1 is given by
0
Find the area enclosed by y = f(x), the x-axis and the e3 − 4 e3 + 4
(A) (B)
ordinate at x = 3. e4 e4

Q.15 For what values of a Î[0, 1] does the area of the e3 + 1


(C) (D) None of these
figure bounded by the graph of the function y = f(x) e4
and the straight lines x = 0, x = 1 & y = f(a) is at a
minimum & for what values it is at a maximum if f(x) = y
Q.3 Area common to the curve = 9 − x 2 & x2 + y2
= 6x is
1 − x2 . Find also the maximum & the minimum area.

 π π+ 3 π− 3
(A) (B)
Q.16 Let f(x) = sin x ∀ x ∈ 0,  4 4
 2
π    3 3
f(x) + f(π – x) = 2 ∀ x ∈  , π  and f(x) = f(2π – x) ∀ x 3
2  (C) 3  π +  (D) 3  π + 
 4  4 
∈ (π, 2π).  

If the area enclosed by y = f(x) and x-axis is aπ + b, then


find the value of (a2 + b2).
2 5 . 3 4 | Area Under the Cur ve and Linear Programming

3 Q.11 The ratio in which the curve y = x2 divides the


Q.4 The area bounded by y = 2 – |2 – x| & y = is
|x|  πx 
region bounded by the curve ; y = sin   & the x-axis
4 + 3n3 4 − 3n3  2 
(A) (B) as x varies from 0 to 1, is
2 2
3 1 (A) 2 : p (B) 1 : 3
(C) + n3 (D) + n3
2 2 (C) 3 : p (D) (6 – π) : p

Q.5 The area bounded by the curves y = sin x & y = cos Q.12 Area of the region enclosed between the curves
x between x = 0 & x = 2 π is
x = y2 – 1 and
= x | y | 1 − y 2 is

(A) ∫ (sinx − cos x)dx (B) 2 2 sq. unit (A) 1 (B) 4/3 (C) 2/3 (D) 2
0

(C) 0 (D) 4 2 sq. unit Q.13 Let y = g(x) be the inverse of a bijective mapping f :
R → R, f(x) = 3x3 + 2x. The area bounded by the graph
of g(x). The x-axis and the ordinate at x = 5 is
Q.6 If f(x) = –1 + |x – 2|, 0 ≤ x ≤ 4 ; g(x) = 2 – |x|, –1 ≤ x
≤ 3. (A) 5/4 (B) 7/4 (C) 9/4 (D) 13/4

Then the area bounded by y = gof(x); x = 1, x = 4 and


x-axis is
Previous Years’ Questions
(A) 7/2 sq. units (B) 9/4 sq. units
(C) 9/2 sq. units (D) None of these Q.1 The area of the region between the curves
1 + sinx 1 − sinx
y= and y = and bounded by the
Q.7 Area enclosed between the curve y = sec x, y = –1 cos x cos x
cosec–1 x and the line x = 1 is π
lines x = 0 and x = is  (2008)
4
(A) ln (3 + 2 2) (B) ln (3 + 2 2) − 1
2 −1 t
(C) ln (3 + 2 2) − π / 2 (D) None of these
(A) ∫0 dt
(1 + t ) 1 − t2
2

Q.8 The area of the closed figure bounded by y = x, 2 −1 4t


(B) ∫ dt
0
y = –x the tangent to the curve=
y 2
x − 5 at the point (1 + t ) 1 − t2
2

(3, 2) is
2 +1 4t
15
(C) ∫0 dt
(A) 5 (B) (1 + t ) 1 − t2
2
2
35 2 +1 t
(C) 10 (D) (D) ∫ dt
2 0
(1 + t ) 1 − t2
2

Q.9 The line y = mx bisects the area enclosed by the


3 Q.2 Let the straight line x = b divide the area enclosed
curve y = 1 + 4x – x2 & the lines x = 0, x = & y = 0, by y = (1 – x)2, y = 0 and x = 0 into two parts R1(0 ≤ x
then m is equal to 2
1
13 6 3 ≤ b) and R2(b ≤ x ≤ 1) such R1 – R2 = . Then, b equals
(A) (B) (C) (D) 4  4 (2011)
6 13 2
3 1 1 1
(A) (B) (C) (D)
Q.10 The area common to y ≥ x & x > − y and the 4 2 3 4
curve x2 + y2 = 2 is
π 1 3π 1 π
(A) + (B) (C) (D)
4 3 2 3 2
M a them a ti cs | 25.35

Q.3 Let f : [–1, 2] → [0, ∞) be a continuous function such  4a2 4a 1 f( −1) 3a + 3a 
2
2    
that f(x) = f(1 – x) for all xÎ[–1, 2], Let R1 = ∫−1 x f(x)dx  
Q.12 If  4b2 4b 1  f(1)  = 3b 2
+ 3b ,
and R2 be the area of the region bounded by y = f(x), x  2
4c

4c 1 f(2)  3c2 + 3c 
     
= –1, x = 2 and the x-axis. Then, (2011)
f(x) is a quadratic function and its maximum value
(A) R1 = 2R2 (B) R1 = 3R2
occurs at a point V. A is a point of intersection of y = f(x)
(C) 2R1 = R2 (D) 3R1 = R2 with x-axis and point B is such that chord AB subtends
a right angled at V. Find the area enclosed by f(x) and
chord AB.  (2005)
Q.4 Find the area bounded by the curve x2 = 4y and the
straight line x = 4y – 2. (1981)
Q.13 A curve passes through (2, 0) and the slope of
Q.5 Find the area bounded by the x-axis, part of the (x + 1)2 + y − 3
tangent at point P(x, y) equals . Find the
 8  (x + 1)
curve = y  1 +  and the ordinates at x = 2 and
 x2  equation of the curve and area enclosed by the curve
x = 4. If the ordinate at x = a divides the area into two and the x-axis in the fourth quadrant.  (2004)
equal parts, find a.  (1983)
Q.14 Area of the region
Q.6 Find the area of the region bounded by the x-axis
π π
{( x, y ) ∈ R 2
:y≥ x + 3 , 5y ≤ x + 9 ≤ 15 } is equal to
and the curves defined by y = tan x, − ≤ x ≤ and  (2016)
3 3
π π 1 4 3 5
y = cot x, ≤ x ≤ (1984) (A) (B) (C) (D)
6 2 6 3 2 3

Q.7 Sketch the region bounded by the curves Q.15 Let S be the area of the region enclosed by
2 2
y
= 5−x and y = |x – 1| and find its area. (1985) y e− x =
= x 0 , and x = 1 . Then 
, y 0,= (2012)

1 1
Q.8 Find the area bounded by the curves x2 + y2 = 4, (A) S ≥ (B) S ≥ −
e e
x2 = – 2y and x = y.  (1986)
1 1  1 1  1 
(C) S ≤ 1 +  (D) S ≥ + 1 − 
4 e 2 e 2
Q.9 Find the area of the region bounded by the curve C :
π
y = tan x, tangent drawn to C at x = and the x-axis.
 4 (1988)

Q.10 Find all maxima and minima of the function y = x


(x – 1)2, 0 ≤ x ≤ 2.
Also, determine the area bounded by the curve y=x(x – 1)2,
the y-axis and the line x = 2. (1989)

Q.11 Compute the area of the region bounded by the


logx
curves y = ex log x and y = where log e = 1
ex
 (1990)
2 5 . 3 6 | Area Under the Cur ve and Linear Programming

PlancEssential Questions
JEE Main/Boards JEE Advanced/Boards

Exercise 1 Exercise 1
Q.4 Q.10 Q.17 Q.1 Q.5 Q.12
Q.14 Q.16 Q.20
Exercise 2 Q.21
Q.2 Q.6 Q.10
Q.12 Q.15 Q.17
Exercise 2
Q.18 Q.2 Q.3 Q.7
Q.11 Q.13
Previous Years’ Questions
Previous Years’ Questions
Q.2 Q.4 Q.7
Q.1 Q.3 Q.7
Q.10 Q.11 `Q.13

Answer Key

JEE Main/Boards

Exercise 1

74 3
Q.1 sq.units Q.2 2 sq. units Q.3 2 sq. units Q.4 ∫1 (5 − y)dy
3

2 a 112
Q.5 2∫0 y dy Q.6 ∫−2 a2 − x2 dx Q.7 5
 sq. units Q.8 sq. units
9

2 64 28
Q.9 sq. units Q.10 sq. units Q.11 sq. units
27 3 3

1
Q.12 (3 + 16 log 2) sq. units Q.13 sq. units Q.14 13.5 sq. units Q.15 1 sq. units
2

a2 16a 50 32
Q.16 (3π – 8) sq. units Q.17 sq. units Q.18 sq. units Q.19 sq. units
12 3 3 3
M a them a ti cs | 25.37

Exercise 2
Single Correct Choice Type

Q.1 C Q.2 A Q.3 C Q.4 A Q.5 A Q.6 B

Q.7 B Q.8 B Q.9 D Q.10 C Q.11 D Q.12 B

Q.13 A Q.14 A Q.15 C Q.16 C Q.17 C Q.18 C

Previous Years’ Questions


Q.1 D Q.2 A Q.3 A Q.4 A Q.5 2 sq. units

Q.6 2 3 sq. units Q.7 A Q.8 A Q.9 C Q.10 D Q.11 A

Q.12 C Q.13 C Q.14 D Q.15 B

JEE Advanced/Boards

Exercise 1

Q.1 f(x) = x sin x, a = 1 ; A1 = 1 – sina ; A2 = π – 1 – sina ; A3 = (3π – 2) sq. units Q.2 2/3

2 1
Q.3 f(x) = x2 + 1;y = ± 2x ; A = sq. units Q.4 Q.5 2 + 1 Q.6 a = 8
3 2
( π − 1) 2
Q.7 Q.8 2 Q.9 a = Q.10 104 Q.11 (16/9)x2
2 3
32 3 8
Q.12 (i) 3 ; (ii) 1280 sq. units ; (iii) sq. units Q.13 3 Q.14 Q.15= sq. units
3 2 3
7
3m + 2m2 +  2 9π 9 1
Q.16 4 Q.17 A 6 Q.18 1 – 3e–2 Q.19=  + − sin−1  sq. units
m3  6 8 4 3 
Q.20 a = –3/4 Q.21 e

Exercise 2
Single Correct Choice Type

Q.1 D Q.2 A Q.3 D Q.4 B Q.5 D Q.6 C Q.7 C


Q.8 A Q.9 A Q.10 A Q.11 D Q.12 D Q.13 B

Previous Years’ Questions


Q.1 B Q.2 B Q.3 C Q.4 9/8 sq. units Q.5 2 2

1 5π 1 1  1
Q.6 loge 3 sq. units Q.7 − sq. units Q.8 − π sq. units Q.9  log 2 −  sq. units
2 4 2 3  4
e2 − 5
Q.10 10/3 sq. units Q.11 sq. units Q.12 125/3 sq. units Q.13 4/3 sq. units
4e
Q.14 3/2 sq. units Q.15 A, B, D
2 5 . 3 8 | Area Under the Cur ve and Linear Programming

Solutions

JEE Main/Boards 3
Sol 7: ∫ x − 2 dx is the area under curve |x – 2|
Exercise 1 −1

where x ∈ [–1, 3]
4 4
x3 2 3
Sol 1: Area = ∫ x2 + 3 = + 3x 2 3
x2 x2
2
3 A = ∫ 2 − x + ∫ x − 2 = 2x − + − 2x
2
−1 2
2 2
−1 2
64 8 56 74 1 9
= + 12 − − 6 ⇒ Area = + 6 = sq. units ⇒ A = 2 + 2 + + – 6 – 2 + 4 = 5 sq. units
3 3 3 3 2 2
π /2 y
π /2
Sol 2: 2 ∫ cos xdx = 2 sinx 
0
= 2[1 – 0] = 2 sq. units
0

π
π
Sol 3: ∫ sinxdx =  − cos x  = − cos π + cos0 = 1 + 1 -1 2 3
x
0
0

= 2 sq. units

y = 3x+4
Sol 4: x + y = 5 ⇒ x = –y + 5
3
2 3  y2  9 1 Sol 8: y = 3x + 4 
⇒ ∫ x = ∫ ( − y + 5 ) dy =  − + 5y  = − 15 − + 5
4 1  2 1 2 2
4
= 6 sq. units
3
Expression = ∫ (5 − y ) dy
1
4
2 ( 3x + 4 )
3/2 
4 4
  0
Sol 5: y = x2 ; A = ∫=
y dx ∫ 3x + 4 dx = 
Y=2
0 0 9x3

Y=0 2  3/2 2
= 16 − 43/2  =  43 − 23 
9   9 
2 2 56 × 2 112
x= y A = 2∫ ydy = 64 − 8=
 = sq. units
9 9 9
0

3
Sol 6: Sol 9: y = above x axis & x ∈ [–4, –1]
(1 − 2x )
3

−1
−1  
3dx 3 1
∫ =  × 
-a a
−4 (1 − 2x )
3 
 (1 − 2x )
+2
( )( ) 
−2 −2
−4

−1
 
3 1 31 1 3× 8 2
=  = 
 − = = sq. units
4  1 − 2x  4 3 4 81 27
( ) ×
2 2 2
a 9 
a2 − x2 dx   −4
Area = ∫
−a
M a them a ti cs | 25.39

Sol 10: dt
xdx =
2
π /2 π /2
dt  cos t  −0 + 1 1
∫ sint = −  = = sq. units
3 4
0
2  2 0 2 2

Sol 14:

4

∫ ( 6x − x )
2
Area = − x2 + 2x dx
0

4
 4 2x3  2 64
= 8x − 2x 2
=  4x2 −  = 64 – × 64 = sq. units
 0  3  3 3
 0
f(x) = 1+ x
3 3 −1
Sol 11: f(x) = 1 + x 
∫ x + 1 + 1= ∫ x + 1 + 1 + ∫ −1 – x + 1
1 −2 −1 −2
4 4 4
 2x3/2  3 −1
∫ y dx= ∫ x + 1=  + x a x2  x2 
 3 = + 2x
0 − 
0 0  0
2  2  −2
−1
2 16 28
= × 23 + 4 = + 4 = sq. units 9 1 1 7 27
3 3 3 = + 6 − + 2 − + 2 = + 10 = sq. units
2 2 2 2 2

Sol 12: xy – 3x – 2y – 10 = 0
Sol 15:
3x + 10
y=
x−2
y



2
x
3 4
π /2
2 π /2
Area = 2 ∫ sin2x dx
=
2
 − cos2x 
π/ 4
π/ 4

= |+1 – 2| = 1 sq. units


4 4
3x + 10 3x − 6 + 16  16 
A= ∫ x − 2
dx A =
x−2
= ∫  3 + x − 2  dx
3 3 Sol 16:
4
= 3x + 16n ( x − 2 ) 
3

12 + 16  n2 − 9 − ln 1

= (3 + 16log 2) sq units.

π /2
Sol 13: ∫ y dx = ∫ x sinx2dx
0 x2 – 2ax + y2 ≤ 0

Substituting x2 = t y2 – ax ≥ 0
2 5 . 4 0 | Area Under the Cur ve and Linear Programming
2
y = 4x - x
a a a Sol 19: y = 4x – x2
A = ∫  2ax − x2 − ax  dx ⇒ A = ∫ 2ax − x2 dx − ∫ ax dx 4
   x3 
∫( )
0 0
0
4x − x2 dx = 2x2 − 
a  3 
2 3/2 0
a a2x 2
= π −
4 3 64 32
0 = 32 − = sq. units
3 3
πa2 2 a 3/2 πa2 2a2 a2
=
4

3
a =−
4 3
=
12
(3π − 8 ) sq. units
Exercise 2
Sol 17:
Single Correct Choice Type
4a
Sol 1: (C)
3 7

1
y = 4a ⇒ 16a2 = 4a2(x – 3) ⇒ x = 7
4a 4a  4a
y2   y3 
∫ x dy
== ∫  4a2 + 3  dy = 12a2 + 3y  1

∫ (e )
1
A= x ex + e− x 
− e− x dx =
0 0    0  0
0
( 4a)
3
64 1 1
= + 12a = a + 12a =e+ –1–1=e+ –2
12a 2 12 e e

 16 + 36  52a Sol 2: (A)


⇒  a =
 3  3 (1, 2)

52a 16a
A= − 3 × 4a = sq. units
3 3

y x2 + 3 and
Sol 18: The points of intersection of =
y 2x + 3 are (0, 3) and (2, 7).
=

Area = A1 + A2

1
1  x3/2 
 0 4
∫ 4x dx 2=
A1 = =
3/2 3
0
2
y −2
Equation normal = −1
x −1
⇒y–2=1–x
3
27 2
3
A1 = ∫ 2x + 3 = x + 3x = + 9 − 4 − 6 = 18 – 10 = 8 x+y=3 …… (i)
2
2 3
3 3
 x2  9 1
2
x3 8
2
A2 = ∫ ( 3 − x ) dx =3x −  = 9 – –3+
A2 = 2 2  2 2
∫x +3 =
3
+ 3x = + 6
3
1  1
0 0
=6–4=2
8 50
A1 + A2 = + 6 + 8 = sq. units 4 10
3 3 Area = 2 + =
3 3
M a them a ti cs | 25.41

Sol 3: (C) Sol 6: (B)



y=2

1 e

x=2

e 2 2
 2
A= ∫  lnx − (lnx )  dx ∫ ln=
dx x ln x − x Area = ∫ lnx dx + ∫ lny dy
1 1 1
2
A = (xlnx – x) – lnx ( xlnx − x ) − ∫ (lnx − 1) ==
2
2∫ lnx 2  xlnx − x  = 2[2ln2 – 2 + 1]
  1
1
= xlnx – x – x(lnx)2 + xlnx + xlnx – x – x
e
= 4ln2 – 2 = 2(2ln2 – 1)
 −x lnx 2 + 3xlnx − 3x 
 ( )  1
1
Sol 7: (B) y = x(1 – lnx) & x =
= –e + 3e – 3e + 3 = 3 – e e
1
Between x = &e
Sol 4: (A) e

2x

1
1 e
e
1 1 1

( )
3
= 2∫
A 2
x + 1 − 2x dx ∫ = 2 ⇒ (x − 1)
= 2 (x − 1) dx 2
=
2
0 0
3 3
0
x2 1  2 x2 
Sol 5: (A) (x – 1)2 + y2 = 1
I= ∫ ( x − xlnx ) dx = −  x lnx − 
2 2  2 
 πx  3x2 x2
If sin   I= − lnx
nx
 2  2 2
(1, 0) 2 e
 ⇒A=
I  1
e

3e2 e2  3 1  1 2 5
A= − − +  = e − 2
2 4 2  4e2 2e2  4 4e
 πx 
1 − ( x − 1 ) − sin
2
A= ∫  2 
 dx
0
Sol 8: (B)
2
 πx 
=  2x − x2 − sin 
 2 0
2
 sin−1 x − 1 + x − 1 2x − x2 
= 
( ) ( ) 2 πx
+ cos 
 2 π 2

π 2 π 2 π 4
 0
) )
2a
= + ( −1) + − = −
2×2 π 2.2 π 2 π
2 5 . 4 2 | Area Under the Cur ve and Linear Programming

π /2a x1
2
∫ ( cosax ) dx = a sinax  π /6a
π /2a
Area =
∫ f ( x ) = x1e 1 ⇒ f(x) = xe + ex
x x

π /6a 0

2 1 1  1
= 1 −  = a > 3 = a ∈  0,  Sol 12: (B)
a 2  3

x 1
Sol 9: (D) y = + ;y = 0
6 x2

2a 2a
x 1   x2 1 
A = ∫  +  dx =
 − 
a 
6 x2   12 x  a 0 0

2
a 1 a 1 a 1 2 2
Area = ∫ − −x − −x2 ( )= ∫− −x + x2
−1
= − − + = + −1
3 2a 12 a 4 2a  3 2 −x 3/2 
0

=  +
x ( )  = 1−2 = 1
a 1 3 3  3 3 3
Aleast when A’ = − =0⇒a=1   −1
2 2a2
x
Sol 13: (A) ∫ f(x)dx = 1 + 3x − 2
Sol 10: (C) 1

3
f(x) =
2 1 + 3x
O /2
3 3
x = 1, y = and x = 8, y =
4 10
8

 π/ 4 
∫ ydx 1 + 3.8 − 2 3
π/ 4
Area1 =  ∫ sinx  2 = 2  − cosx  A= 1
= =
0 ∆x 8 −1 7
 0 
dy
 1
= 2 −
 2

+ 1 =

( 2 −1 ) 2=
2− 2 Sol 14: (A) = 2x + 1
dx
y = x2 + x + c
π /2
π /2
(1, 2); c = 0
Area2 = ∫ cos x −=
area1 sinx 
0
−2+ 2
0

=1–2+ 2 = 2 –1

ratio is
2 ( 2 −1 )= 2 2 1
2 −1

y = x2 + x + 1
Sol 11: (D)
1 1
 x3 x 2  5
A = ∫ x + x=  +  =
2

0  3 2  0 6
M a them a ti cs | 25.43

Sol 15: (C) 3


For areamax = A’ = 3 – 2c = 0 ⇒ C =
2
 3 13 
 , 
2 4 
-3 -1 1 2
Sol 18: (C) A1 = a3
× a × 2a =
2
a
0 ≤ y < 3 – 2x – x2 a  x3  2a3
2
⇒ ==
∫ x = 
0 < y < 4 – (x + 1)2 −a  3  −a 3
1 2a3 4a3
2
= ∫ (3 − 2x − x )dx ⇒2 = 2a × a2 – =
3 3
0
A1 3
⇒ =
A2 4
Sol 16: (C)

Previous Years’ Questions

x2 y 2
Sol 1: (D) Given, + 1
=
1 9 5
O
C To find tangents at the end points of latus rectum, we
find ae.
1 i.e. ae = a2 − b2 = 4= 2
c

∫( )
2
x2 − cx3 dx =  4 5
0
3 and b2 (1 − e2 )= 5  1 − =
 9 3
1/c
 x3 cx 4  1 1 2 By symmetry, the quadrilateral is a rhombus.
⇒  −  = 3− 3 =
 3 4  3c 4c 3
0 y
A
1 2 1
⇒ = ⇒c=
12c3 3 2
2 2
L(ae, b (1-e ))
Sol 17: (C)
2 x’ O B x
(e, c +1) D

L’
O 1
C C

y’

y − c2 − 1 So, area is four times the area of the right angled triangle
= 2c
x−c formed by the tangent and axes in the 1st quadrant.
⇒ y = 2cx – c2 + 1  5
∴ Equation of tangent at  2,  is
x = 1, y = 2c – c2 + 1  3
x = 2, y = 4c – c2 + 1 2 5 y x y
x+ . =1⇒ + =1
1 9 3 5 9/2 3
⇒ Area of trapezoid = 6c − 2c2 + 2 × 1 = 3c – c2 + 1
2 
2 5 . 4 4 | Area Under the Cur ve and Linear Programming


x y ⇒ x = a (ax2)2
+ =1
9/2 3 1 1
⇒ x = 0, ⇒ y = 0,
∴ Area of quadrilateral ABCD a a
1 1
1 9  ∴ Point of intersection are (0, 0) and  , 
= 4 (area of ∆AOB)= 4  . .3  = 27 sq. units a a
2 2  Thus, required area OABCO = Area of curve OCBDO –
area of curve OABDO
Sol 2: (A) To find the area between the curves,
1/a  x 
=y + 3 x and x-axis in the 1st quadrant (we can
x ,2y = ⇒∫  − x2  dx =
1 (given)
plot the above condition as) ; 0  a 
 
1/a
 1 x3/2 ax3 
y’ ⇒ . −  1
=
 a 3 / 2 3 
0

2 1
B y= x ⇒ − 1
=
2
3a 3a2
1 1 ∴
x-3 ⇒ a2 = ⇒ d= ( a > 0)
y= 3 3
2
x’ x
O A(3,0) (9,0) Sol 4: (A) Since, tangents drawn from external point to
the circle subtends equal angle at the centre

y ∴ ∠O1BD = 30°
A
Area of shaded portion OABO
9 9 x − 3 
= ∫0 x dx − ∫   dx
3
 2  O3
9 9
 x3/2  1  x2 
=   −  − 3x 
3/2  
 0 2  2 3
O1 O3
2  1  81  9   1 cm 1 cm
=  .27  −  − 27  −  − 9  
3  2  2  2   B
30o 30o
C
1
= 18 – (18) = 9 sq. unit 3 cm D 2 cm E 3 cm
2

Sol 3: (A) As from the figure, area enclosed between O1D


In DO1BD, tan 30° =
the curves is OABCO. BD
Thus, the point of intersection of ⇒ BD = 3 cm

y = ax2 and x =2 ay2 2 2


Also, DE = O1O2 = 2 cm and EC = 3 cm
y = ax and x = ay
Y Now, BC = BD + DE + EC = 2 + 2 3
a
⇒ Area of ∆ABC
2
y=x
B
(1/a, 1/a) 3 3
C = = (BC)2 . 4(1 + 3)2
4 4
A
X’ O D
X = (6 + 4 3) sq. cm.

Sol 5: The area formed by |x| + |y| = 1 is square shown


as below,
Y’
M a them a ti cs | 25.45

y 1 1
and (x + 1)2 = ⇒–
4 2
-x + y = 1 x+y=1
1 1   1 1
x’
O
x ∴ Q  ,  and R  − , 
-1 1  2 4   2 4
x+y=1 x-y=1 ∴ Required area
1/2
y’ 1/2  1
2
 (x − 1)3 1 
= 2∫ (x − 1) −  dx = 2  − x
2
0
 4  3 4 
∴ Area of square = ( 2) = 2 sq. units 0

 1 1  1  8 1
Sol 6: Equation of tangent at the point (1, 3) to the = 2 − − −  − − 0  = = sq. unit
 8.3 8  3  24 3
curve
x2 + y2 = 4
Sol 8: (A) Region ( x, y ) : y 2 ≥ 2x
4,
is x + 3y =
x2 + y 2 < 4x and x ≥ 0, y ≥ 0
whose x-axis intercept (4, 0)
Y
y (2, 2)

A
P(1, 3)
O (2, 0) X
x’ x
(0, 0) O A(4, 0)

2
1
Area = π ( 2 ) − ∫ 2x dx
y’ 2

4 0
Thus, area of ∆ formed by (0, 0) (1, 3) and 2
 3 
(4, 0)  x2  2 2  2
3
= π− 2  = π− 2 
0 0 1  3  3  
 
1 1  
2 0
= 1 3=
1 | (0 − 4 3) | = 2 3 sq
sq.unit
units
2 2
4 0 1 2 2
= π− ×2 2
3

Sol 7: (A) The curves y = (x – 1)2, y = (x + 1)2 and y = 8


= π− sq. units
1/4 are shown are 3

Sol 9: (C) Region ( x, y ) : y < 2x and y ≥ 4x − 1


y y = (x + 1)
2 4
y = (x - 1) 2

y = 4x - 1
2
1/4 P y = 2x
R Q ( 1 ,1
2 (
y = 1/4
x
-1 -1/2 O 1/2 1
O

Where points of intersection are


( 18 , -12 (
1 1
(x – 1)2 = ⇒x=
4 2
2 5 . 4 6 | Area Under the Cur ve and Linear Programming

1  y + 1 y2  2m + 1 = n + 3 ⇒ 2m − n = 2
Area
= ∫  4 − 2  dy
−1/2   3m − 1 = 2n + k ⇒ 3m − 2n = k + 1
1
 y 2 y y3  1 1 1 1 1 1  = 9 4m + 1 =n ⇒ 4m − n =−1
=  + −  =  + − − + − 
3 4 6 32 8 48  32
 8 4 6  −1/2  On solving these equations, we get
3 9
Sol 10: (D) The point of intersection x − 3 =2 x m =− ,n =−5 ⇒ K =
2 2
⇒ ( x − 3) =
2
4x
⇒ x2 − 10x + 9 =0 Sol 13: (C) The required area
⇒x=
1,9
2 5 y  2
2  y 
⇒y=
1,3
= 2 ∫ 3 y−
  dy  = 2 ∫
 0 2
dy  = 5 ∫ y dy
3  0  2  
 
 0
3  3 
2y + 3 − y 2 dy =  y 2 + 3y − y 
Area ∫  3 
0  0 2
 y 3/2  10  3/2  10 × 2 2 20 2
=5   = = 2 =
= 9 + 9 − 9  = 9 sq. units 3   3 3
 3 / 2  0

Sol 11: (A)


Sol 14: (D)
2 2
x +y =1 y

y = sin x

5
4
x
O 
4

y = cos x

2
y =1-x π/ 4 5/ 4
Area = ∫ ( cos x − sinx )dx + ∫ ( sinx − cos x )dx
Area = Area of half circle 0 π/ 4
1 3π
 1 − x 3/2 
 ( ) 
1 1 2
1 − x dx = 2 π (1 ) + 2  − 3 / 2 
2
+2∫ + ∫ ( cos x − sinx )dx
0   0 5π
4
π/ 4
π 2 π 4 5π / 4
= + 2  = + = sinx + cos x  +  − cos x − sinx 
0 π/ 4
2 3 2 3
3π /2
+ sinx + cos x 
5π / 4
x −1 y +1 z −1
Sol 12: (C) = = = m
2 3 4 1 1  −1 1 1 1 
= + −1 +  − − − 
⇒ Any point on this line is ( 2m + 1,3m − 1, 4m + 1 ) 2 2  2 2 2 2
 1 1 
+  −1 + 0 + + 
x −3 y −k 2 2 2
Similarly for = = = n 
1 2 1
= 4 2 −2
(n + 3,2n + k,n)
It they intersect, then
M a them a ti cs | 25.47

= –2π – (π) = –3p


Sol 15: (B) y=x
⇒ A3 = (3π – 2) sq units.
xsinx = sinx ⇒ x = 1
⇒ a = 1 ⇒ A2 = π – 1 – sina
⇒ A1 = 1 – sina
y=1/x
t
1
2 × t sint2
O
x=1/e Sol 2: A = ∫ sinx dx , B = 2
x=1 0
t
2
1 1/e
1 A
∫ sinx dx
Area ∫ xdx + ∫ x
dx Now, Lim = 0
0 1 t →0 B 1
× t sint2
1 2
 x2  1/e 1
=   + Inx  = + In 1 / e 0 sint2
2
  0
1 2 form ⇒ Lim =
0 t 1
( 2t ) cos t2 + sint2
t →0

1 3 2 2
= +1 = sq. units
2 2 [L’ Hospital’s rule]

2sint2 2 tant2
= =
JEE Advanced/Boards 2t2 cos t2 + sint2 2t2 + tant2

Exercise 1 2 tant2
t2 2 2
a
= = =
tant 2 2+1 3
Sol 1: A1 = ∫ sinx − f(x) 2+
t2
0
a a
a
= – cos x − ∫ f(x) =
1 − cosa − ∫ f(x) Sol 3: f(x + 1) = f(x) + 2x + 1
0
0 0
f(0) = 1
π π π
π
⇒ A2= ∫ f(x) − sinx = ∫ fx + cos x= ∫ f(x) − 1 − cosa f(1) = 2⇒ f(x) = x2 + 1
a 2
a a a f(1) = 2 f(x) = x + 1
2π 2π 2π

⇒ A3 = ∫ sinx − f(x) = – cos x π
− ∫ f(x) =−2 − ∫ f(x) 2
π π π (t1t + 1)
⇒ A1 = 1 – sina + acosa – cosa
a
= 1 – cosa – ∫ f(x) dx
0
a
⇒ –acosa + sina = ∫ f(x) dx = sina – acosa
0
⇒ f(x) = xsinx f(–1) = 2;f(2) = 5;f(–2) = 5
π π y = x2 + 1
∫ f ( x ) dx =
π
∫ x sinx dx =
 −x cos x + sinx 
a
( )
Equation of tangent at P t,t2 + 1 y – (t2 + 1) = 2t(x – t)
a a

= π – [–acosa + sina] = π + acosa – sina Since, it passes through (0, 0)

⇒ A2 = π + acosa – sina – 1 – cosa –t2 – 1 = 2t(–t)

2π π ⇒ t2 = t ⇒ t = ±1

∫ ∫ x sinx dx =
f(x)dx =  −x cos x + sinx 
π ⇒ y = ±2x
π a
2 5 . 4 8 | Area Under the Cur ve and Linear Programming

∫ (x )
Area = 2 2
+ 1 − 2x dx ⇒ > 1 i. e. n = 2 +1
0
1
2 2 2
1 Sol 6:
= 2∫ ( x − 1 ) dx = ( x − 1 ) = 0 + 1 =sq. units
2 3

0
3 0
3 3

lnx − c
Sol 4: y = = 0 ⇒ x = ec 1
x

1
x   − (lnx − c)
x
f’(x) =   =0
x2
⇒ 1 – lnx + c = 0
a a
⇒ lnx = c + 1 1 1  ln(2x − 1) 
A= ∫ x − 2x − 1 = lnx −
 2

2
⇒ x = ec+1 2

ec +1 ec +1 ec +1
ln(2a − 1) ln3
lnx − c lnx c = lna – − ln2 +
dx b2 − 4ac 2 2
⇒ ∫ x
dx =∫
x
dx − ∫ x
ec c ec
3a 3a 4
e
= ln × 2 = ln − ln
2(2)(2a − 1) 4a − 2 5
ec +1
 (lnx)2  ec +1
=   − c lnx  ⇒ 15a = 16a – 8 ⇒a=8
ec
 2  ec
1
(c + 1)
2
− c2 2c + 1 1 Sol 7: f(x) =
= − c c + 1 − c  = − c  = 1 + x2
2 2 2  1  1 
We have, A(α, f(α)) and B  − ,f  −  
 α  α 
Sol 5: y = xn x
OA → y =  ……… (i)
α(1 + α2 )
y – 1 = n(x – 1)
−α3
1 1 OB → y = (x)
A
= n
∫ x dx − ∫ n(x − 1) + 1dx 1 + α2
α 0  
0 1  1 x  1 α3
1−
n ∫  1 + x2 − α(1 + α2 )  dx + ∫  1 + x2 + 1 + α2 x  dx
1
1 0  1 
 xn+1   x −1 2  −

=
  0 
− n
( ) + x
α
α 0
n+1  2   x2   α3 x 2 
 1− 1 = tan−1 x −  + tan−1 x + 
n  2α(1 + α2 )  0  2(1 + α2 )  − 1
α
1  n 1 1 1 α
= − 1 − −1 +  = − = tan–1α –
n+1  2n2 n  n + 1 2n
2(1 + α2 )
n−1   1 α 
⇒= + 0 + 0 − tan−1  −  −
2n(n + 1) 
  α  2(1 + α2 ) 
dA
= 0 ⇒2n(n + 1) – (n – 1) (4n + 2)=0
dn  1 (α + α )
= tan–1α – tan–1  − α  –
⇒ 2n2 + 2n – 4n2 – 2n + 4n + 2 = 0   2(1 + α2 )
⇒ –2n2 + 4n + 2 = 0 α2 − 1 (α + α )
= tan–1 1 − 1 −
⇒ n2 – 2n – 1 = 0 2(1 + α2 )

2± 8 π (α + α ) π 2α
n= = 1± 2 = − = −
2 2 2(1 + α ) 2 2(1 + α2 )
2
M a them a ti cs | 25.49

16 8  4
A’ = 0 ⇒ α = 1 = − + 2a=  2a − 
12 3  3
π 1
A= − 4 2
2 2 Is minimum ⇒ 2a = ⇒a=
3 3
−1
Sol 8: The curve is y = sin x, i.e., x = sin y. This is a
Sol 10: Sol.10
π π
standard curve. Lines x = 0, y = and y = − are the
2 2
π
y-axis and two lines parallel to the x-axis at a distance ,
2
one above and the other below the x-axis respectively.
Hence, the shaded part is the required area ∆. By
symmetry of the curve and the lines,
y – c = m(x – 1)
ar (OABO) = ar (OCDO)
∫ (x )
2
A= − m(x − 1) − c dx
∴∆= 2 × ar ( OABO )
b
π π  x3 m(x − 1)2 
2 2 =
 − − cx 
∫ ( x ) dy 2 ∫ sin y dy,
= 2=
curve
 3 2  a
0 0
∴  b3 − a3 m  
( b − 1 ) − ( a − 1 )  − c (b − a ) 
2 2
( The equation of the curve is x = sin y) =  −
π
 3 2   
∴ ∆= 2  − cos y  2= 2 0 + 1= 2
0
(b − a) (b2 + a2 + ab )
=
3
Sol 9: m 2
− b − a − 2b + 2a − c(b − a)
2
2 
c + mx – m = x2
⇒ x2 – mx + m – c = 0
a c b x1 + x2 = m
x1x2 = m – c

⇒ x1 – x2 = m2 − 4m + 4c
c  b

  ( )
A =  ∫ f(c) − f(x)  dx + ∫ f ( x ) − f ( c ) dx A  m2 − m + c m 
a  c ⇒ (b - a)  − m − 2  − c 
 3 2 
c b
= f(c)(c − a) − ∫ f(x) dx + ∫ f(x) dx − f(c)(b − c)  m2 − m + c m2 3c 3m 
a c ⇒ (b - a)  − − + 
c b  3 2 3 3 
A = f(c) [c – a – b + c] – ∫ f(x) dx + ∫ ( f(x)) dx  m2 2c 2m 
a c
⇒ (b – a)  − − + 
This is minimum when 2c – a – b = 0  6 3 3 
a+b 1 2
⇒c= A= + [m + 4c – 4m]3/2
2 6
2 3 2
x   x 4 x3 
∫ f(x) = ∫ 3
 − x 2
+ a  dx=  − + ax  m2 − 4m + 4c =
36

0   12 3  0
B2 − 4AC =
0
2 5 . 5 0 | Area Under the Cur ve and Linear Programming

(ii) For maximum Area of sq. ABCD


16 − 4 ( 4c − 36 ) =
0
Length 2 5 1 + c must be maximum
⇒ c = 10 ⇒ m = 2
For c = 63 (From Previous questions)
⇒ c2 + m2 = 104
( )
2
Area = 2 5 1 + c
Sol 11: A1 + A2 for any point P(t , t2)
(2 )
2
= 5 1 + 63 = 4 × 5 × 64
k t t

∫ C2dx − C + ∫ t dx − C= ∫ ( C − C1 ) dx
2
= 1280 sq. units.
0 k 0
(iii) Area bounded by y = 2x + 3 and
k k t 3
x  3  −x  3 3

∫ ( 2x + 3 − x )dx
t
2
∫ C2dx −  3  +  3  + t (t − k) = Area = 2

0  0  k 6 −1
3
k  x3 
k 3 k 3 t3 3 2 t3 =  x2 + 3x − 
∫ 2
C dx −
3
+ −
3 3
+ t − t k =
6  3 
−1
0
1
k
t3 2 = 9+ 9 − 9 −1 + 3 −
3
∫ C2dx =− 2 + t k
0 1 32
= 11 − = sq. units
Let C2 = f(x) l x 2 2 3 3
t/ λ
t3
∫( ) Sol 13:
λ2 x2 dx =+ t2k
0
2
2
t/ λ
a +1
λ  x3 
2
t3 t3 λ 2 t3 t3 t3
 0
=− + ⇒ =− +
3 2 λ 3 λ3 2 λ a

10 1 1 2 4
⇒ = − = ⇒ λ=
2 λ 3λ 3λ 3 Area of rectangle = a(a2 + 1)
a2 +1 a2 +1
16x2  2(y − 1)3/2 
⇒ c2 = A= ∫ y − 1 dy =
 
9 1  3 1
Sol 12: (i) Let equation of CD be y = 2x+c 2 3 2
= a − 0 = a3
3 3
For intersection with y = x2
2 3 1 2 4 3
⇒ x= 2x + c ; x − 2x − c =
2 2
0 == a a(a + 1) = a= a3 + a
3 2 3
2 and x1 x2 = −c
⇒ x1 + x2 =
a3 = 3a⇒ a = 3,– 3,0
Length of CD= x1 + x2 5 ⇒ a= 3
= 2 5 1+c x
2
c + 17
Sol 14: f3(x) = ∫ tf (t) dt
Length of AC
= BD
= 0
5
⇒ f’(x) 3f2(x) = xf2(x)
Given ABCD is square than,

c + 17 ⇒ f2(x)[3f’(x) – x] = 0
2 5 1+ c =
5 x x2
⇒ f’(x) = ⇒ f(x) = +c
⇒ c2 − 66 c + 189 =
0 3 6
⇒c=
3 ,63 x2
⇒ f(x) = +c
Therefore, least value of c is 3. 6
M a them a ti cs | 25.51

π /2 π
A= 2 ∫ sinx + 2 ∫ 2 − sinx
0 π /2
3 x 2
 x2   t2 
 + c =
 6 
∫  6 + c  dx
t  = 2  − cos x 
π /2
0
+ 2 2x + cos x 
π
π /2
0  

x6 3cx 4 3x2c2 = 2[+1] + 2[2π – 1 – p] = 2π = aπ + b


+ c3 + +
216 36 6 a = 2, b = 0

 t4 x
ct2 
x  t5 ct3  a2 + b2 = 4
2
2
= ∫ +c +  t dx = ∫  36
 + c t +  dx
3 
 36
0
3  0
Sol 17: –1 + x = 2y – y2 – 1
x
 t6 c2 t2 ct 4  x6 c2 x2 cx 4 x – 1 = –(y – 1)2
=  + +  = + + ⇒c=0
 216 2 12  216 2 12
0 y= 1 − x + 1 = mx + 2
2
x 1 – x = (mx + 1)2
⇒ f(x) =
6 1 – x = m2x2 + 1 + 2mx
3 3
 x3  3
Hence, ⇒ m2x2 + (2m + 1)x = 0
∫ f(x)dx
= = 
0   0 2
18
− ( 2m + 1 )
⇒ x1 = 0, x2 =
m2
Sol 15: Given curves are y = x + 1 2
…. (i)
− ( 2m + 1 )
and y =−x + 1 or y =− ( x − 1 )
2 2
…. (ii) x1x2 = 0, x1 + x2 =
m2
Curve (i) is the parabola having axis y = 0 and vertex
x2
(-1, 0).
A= ∫ (1 + 1 − x − mx − 2 dx )
Curve (ii) is the parabola having axis y = 0 and vertex x1
(1, 0) x
 (1 − x)3/2 mx2 2
(1) - (2) 2x = 0 x = 0 A = x − − − 2x 
 3/2 2  x
From (i), x = 0 ⇒ y =±1 1
x
 −2(1 − x) 1 − x mx2 2
Required area = area ACBDA =  − − x
 3 2  x
1 Y 1

∫ ( x1 − x2 ) dy
)
= C (0,1
2 m 
( )
3/2 
= − (1 − x2 ) − (1 − x1 )  −  x22 − x12  − ( x2 − x1 ) 
3/2

y=1
−1
3  2 
1

∫ (1 − y ) − (y )
− 1  dy
2 2
= 2 m
= − (1 − x ) − 1 − x2 − x
3/2
 A O B
−1
(-1, 0) (1, 0) 3  2
1
= 2 ∫ 1 − y 2 dy ( ) D(
0,-1
y = -1 2 2
= − (1 − x)3/2 −
mx2
−x
−1 ) 3 3 2
1
 y3   1   1  3/2
= 2  y − = 2 1 −  −  −1 +   2 2
= − 1 +
( 2m + 1)  –
2
m  2m + 1   2m + 1 
 3   3   3    + 
−1 3 3 m2  2  m2   m2 

8
= sq. units 2 2 2 1
3 = − (m + 1)3 + +
3 3m 3 m m2
y
Sol 16:
m 4 1 4  2 2 1 2 1
1 −  + + = + + − −
2  m2 m4 m3  3 m m2 m 2m3

O   3 2
2 2
2 5 . 5 2 | Area Under the Cur ve and Linear Programming

region R2 is the interior of the circle (ii). Therefore,


2 2 2 2 2 3 2 7 R1 ∩ R 2 is the shaded region.
− − − − − = − − −
m 2 3 3m m m
3 2
m2 m 6m3
Putting the value of y2 from (i) in (ii), we get
7 1 9
3m + 2m2 + 4x2 + 16x − 9 = 0 ⇒ x = −
A= 6 2 2
m3 1
From (i), x = ⇒ y =
± 2
2
Sol 18: 9
And x = − is not possible
2
1  1 
Thus, A =  , 2  and=
C  , − 2
2  2 

Required area = 2 area OABD


2

f’(x) = –xe–x + e–x = 2 ∫ ( x1 − x2 )dy


0
f”(x) = xe–x – e–x – e–x
2 2
= (x – 2)e–x = 0 1 y2
32 − ( 2y ) dy − 2
2
= 2 ∫2 ∫ 4
dy
⇒ x = 2 inflection point 0 0

2 2 2 2
2 1 1  y3 
−x  −xe− x − e− x 
∫ xe =  0
=
2 ∫
2 2
3 − z dz −
2
 
 3  0
0 0

= –2e – e + 1 = 1 – 3e
–2 –2 –2
[Putting z = 2y in first integral]

{( x, y ) : y ≤ 4x, 4x + 4y ≤ 9}
2 2
Sol 19: Let
= R 2 2 2
1  2 9 − z2 9 z 1
=  + sin−1  − .2 2
2  2 2 3 6
= {( x, y ) : y 2
≤ 4x} ∩ {( x, y ) : 4x + 4y ≤ 9} = R ∩ R
2 2
1 1
 0

1  2 2 2 2
Where
= R1 {( x, y ) : y 2
≤ 4x and
= R2 } {( x, y ) : 4 x 2
+ 4y 2 ≤ 9 } =+  2
2 
9
2
sin−1 −
3  6
Equation of the given curves are
2 9 2 2 2 9 1
y 2 = 4x = + sin−1 = + cos−1
… (i) 6 4 3 6 4 3
9
and 4x2 + 4y 2 = … (ii)
2 3 2 2
Y sin−1 = θ, then sin θ = and
3 3
2 2
4x + 4y = 9 A 1
2
y = 4x cos θ= 1 − sin2 θ=
3
y= 2
X 2 9 π 1
O
D B y=0 Required area = +  − sin−1 
6 4 2 3

 −1 π 
 cos x= − sin−1 x 
 2 
Curve (i) is a parabola having axis y = 0 and vertex (0, 0).  2 9π 9 1
=  + − sin−1  sq. units.
3  6 8 4 3 
Curve (ii) is a circle having centre at (0, 0) and radius . 
2
Clearly region R1 is the interior of the parabola (i) and
Sol 20:
M a them a ti cs | 25.53

Single Correct Choice Type

Sol 1: (D)
b b

∫( x2 − 1 − 3 + x dx) ∫ (x )
2
A= = + x − 4 dx
a a

b
 x3 x 2 
=  + − 4x 
y = a2x2 + ax + 1  3 2  a
2
 1 3 b3 − a3 b2 − a2
y =  ax +  + = + − 4(b − a)
 2  4 3 2

1 1 + 4 ( −1) 5 9 17 17
3
2
 1 = 17 + −4 = 17 − =
⇒A= ∫ 
 ax +  +  dx 3 2 3 2 6

0 
2 4

1
For point of intersection
 a2 x3 ax2  a2 a
⇒A=  + + x = − + + 1 x2 – 1 = 3 – x
 3 2  0 3 2
⇒ x2 + x – 4 = 0
2a 1
A least if A’=0 ie + =0 ⇒ b + a = –1 ⇒ ab = –4
3 2
⇒b–a= 1 + 16 ⇒ b – a = 17
3
a= −
4
Sol 2: (A) e–x and linearly = e–4 & x = 1
Sol 21:

∫ e − e−4  dx
−x
K a 
1   1 1
∫  x − nx  dx =∫  nx − x  dx = –e–4 – 4e–4 – (–e–1 – e–4)
1 K

1 = K  nK = –5e–4 + e–4 + e–1

KK = e 1 4 e3 − 4
= e–1 – 4e–4 = − =
K a e e4 e4
nx − xnx + x=
  xnx − x − nx 
1 K

 nK – 1 + K – 1 Sol 3: (D) y = 9 − x2 x2 – 6x + y2 = 0
= a  na – a –  na – 1 + K +  nK
(a – 1) = (a – 1) (  na)
(a – 1) [1 –  na] = 0
Either a = 1 or a = e
∴a=e

Exercise 2
2 5 . 5 4 | Area Under the Cur ve and Linear Programming

3/2
 9 − x2 − 6x − x2  dx
Area = 2 ∫  

0
3/2
 2 π/ 4 2π
 9 − x − 9 − ( x − 3)  dx = sin+ cos  + sin+ cos  +
2
= 2 ∫  
0 5π / 4
0
5π / 4
3/2
 − sin− cos 
π/ 4
 −1 x 2 
 9 sin 3 + x 9 − x 
= 2  – = 2 –1+1+ 2 – [– 2 – 2 ]= 4 2 sq. units
 2 
  0

 −1 x − 3 
3/2
Sol 6: (C)
 9 sin + (x − 3) 6x − x2 
2 3 
 2 
  0

π 3 9 π 3 9
= 9. + 9 − + 9. + 9−
6 2 4 6 2 4

π x 3 π 9 3  3 3
= 9. + 3×3 = 3 + = 3 π + 
3 2 6 2 2  f(x) = –1 + |x – 2|

g(x) = 2 – |x|
Sol 4: (B) f(x) =1 – x ∈ (0, 2)
x – 3 ∈ (2, 4)
g(x) =2 – x ∈ (0, 3)
2 4
2 + x ∈ (–1, 0)
g(f(x)) =2 – f (x) f (x) ∈ (0, 3)
2 3
 3  3
Area = ∫  x −  dx + ∫  4 − x −  dx =2 + f(x)f(x) ∈ (–1, 0)
x x
3 2
=2 – (1–x), x ∈ (0, 1) = 1 + x
2 3
 x2   x2 
=  − 3nx  +  4x − − 3nx  =2 + (1–x), x ∈ (1, 2) = 3 – x
 2  3  2  2
=2 + (x–3), x ∈ (2, 3) = x – 1
3 9 =2 – (x–3), x ∈ (3, 4) = 5 – x
= 2 – 3  n2 – + 3  n 3 + 12 – – 3  n3 – 8
2 2
+ 2 + 3  n2
1 15 1 3 4 − 3n3
= + − 6 + 3n = 2 − n3 =
2 2 3 2 2
1 2 3 4

Sol 5: (D)
3 9
Area = 1 + 2  × 1 =
2 2
O

Sol 7: (C)
sin x

Area = 
π/ 4 5π / 4

∫ ( cos x − sin x ) dx + ∫ ( sinx − cos x ) dx /2


x=0
0 π/ 4

+ ∫ ( cos x − sinx ) dx
5π / 4
M a them a ti cs | 25.55

π/ 4 1
Area = 2 ∫ ( sec x − 1) dx 1  2x3/2 
 =
2
0
Area1 = ∫ x dx = 
 3  0 3
0
π/ 4 π
= 2 n ( sec x + tanx )  − 1
0 2  2 − x2  dx
Area2 = ∫  

= 2ln ( 2 +1 −
π
2
)= ln 3 + 2 2 − (
π
2
) 0
1
 −1 x 
 sin + x 2 − x2 
Sol 8: (A) = 2 2  = π +1
a=1  2  4
 
 0
(3, 2)
π 2  π 1
Area = +1− =  + 
4 3  4 3

Sol 11: (D)

Eq. of tangent
y − 2 2×3 3
= = ⇒ 2y – 4 = 3x – 9
x −3 2 4 2
3x – 2y = 5  ... (i)
1 1
1
5
  3x − 5   πx 2 πx  2 2
A = × 2×1 + ∫ x −    dx A1 = ∫ sin dx = − cos  = − ( −1 ) =
2  2  2 π 2 0 π π
1 0

5 1
 −x2 + 10x  1 1
= 1+  = 1 + (16) = 5 A2 = ∫x
2
dx =
4 4 3
 1 0
1
3 6−π
Sol 9: (A) y = 1 + 4x – x2 Ratio = =
2 1 π

⇒ y – 5 = –(x – 2)2 π 3
3/2
2
Area = ∫ 1 + 4x − x Sol 12: (D) A = 
∫  y
2
− 1− | y | 1 − y 2  dy
0 
3/2
 x3  3 9 27 9 39 1 0
2
=  x + 2x −  = = 6− =  2 2   2 2 

 3  2
+ 2× −
4 24 8 8
= ∫  y − 1 − y 1 − y 3/2
 dy1+ ∫  y − 1 + y 1 − y  dy
  3/2 0
0 0  −1 
 2   2 
 3
y  1−y    3
y  1−y  
1 3 3m 1 39
Area of ∆ = × × = × =  −y+   + −y−  
2 2 2 2 8 3 3  3 3 
 2    
39 13  2 0   −1
9m = ⇒m=
2 6
1  1  1  1 
= −1 − 0 − 0 +  + 0 − 0 − −  − + 1
Sol 10: (A) 3  3  3  3 
1 1
=–1+ − − 1 = –2; Area = 2
3 3
2 5 . 5 6 | Area Under the Cur ve and Linear Programming

Sol 13: (B) f(x) = 3x3 + 2x 1 1


3 3 1
⇒ − {(1 − b) − 1} + {0 − (1 − b) } =
g(x) = f–1(x) 3 3 4
5 1 1
 3x 4  3  7 2 1 1 1
∫=g(x) ∫=f(x)  + x2  =  + 1 = ⇒ – (1 − b)3 =− + =−
0 0  4  0  4  4 3 3 4 12
1 1 1
⇒ (1 − b)3 =⇒ (1 − b) = ⇒ b =
8 2 2
Previous Years’ Questions 2
Sol 3: (C) R1 = ∫−1 x f(x)dx  ... (i)
π/ 4  1 + sinx 1 − sinx 
Sol 1: (B) Required area = ∫
 −  dx Using,
b b
0 
 cos x cos x  ∫a f(x)dx
= ∫ f(a + b − x)dx
a
 1 + sinx 1 − sinx 
=  > > 0 2
 cos x cos x 
R1 = ∫−1 (1 − x)f(1 − x)dx,
  [given, f(x) = f(1 – x)]
x x
 2 tan 2 tan  2
2 2 ∴ R1 = ... (ii)
 1+
 2 x
1−
x


∫−1 (1 − x)f(x)dx 
π/ 4 
1 + tan 1 + tan2 
= ∫  2 2 Given, R2 is area bounded by
−  dx
2 x x
0
 1 − tan 1 − tan2 
  f(x), x = –1 and x = 2
2 2
 2 x 2 x
 2
 1 + tan 1 + tan  ∴ R2 = ∫−1 f(x)dx  ... (iii)
 2 2 
  Adding Eqs. (i) and (ii), we get
x x
π/ 4
 1 + tan 1 − tan 
2 − 2 2
= ∫ 
0  x x
 dx

2R1 = ∫−1 f(x)dx  ... (iv)
 1 − tan 2 1 + tan 
 2  ∴ From Eqs. (iii) and (iv), we get

x x x 2R1 = R2
1 + tan − 1 + tan π/ 4
2 tan
π/ 4 2 2 dx = 2
= ∫0 ∫0 x
dx
x 1 − tan 2 Sol 4: The point of intersection of the curves x2 = 4y
1 − tan2
2 2 and x = 4y – 2 could be sketched as, are x = –1 and
x = 2.
x 1 x
Substitute tan = t ⇒ sec2 dx = dt ∴ Required area
2 2 2
2
tan
π
4t dt 2 x + 2   x2   1  x2 x3 
= = ∫−1  4  −  4   dx =  + 2x − 
∫0 8
 

  4  2 3 
(1 + t2 ) 1 − t2 −1

π 1  8 1 1 
As tan= 2 −1 =  2 + 4 −  −  − 2 +  
8 4  3 2 3 

2 −1 4t dt y
So, ∫0 2 2
2
(1 + t ) 1 − t x =4y

Sol 2: (B) Here, area between 0 to b is R1 and b to 1


to R2.
b 2 1 1
∴ ∫0 (1 − x) dx − ∫ (1 − x)2 dx = x
b 4
1 2
b 1
 (1 − x)3   (1 − x)3  1
⇒  −  = y’
 −3   −3 
 0  b 4
M a them a ti cs | 25.57

1  10  −7   1 9 9 y
=  −    = . = sq unit.
4  3  6  4 2 8
y = cot x y = tan x

a 8  4 8 
Sol 5: Here, ∫2  1 + x2  dx =∫a  1 + x2  dx /2 -
x’ x
O /4
-/2 -/4
a 4
 8  8
⇒  x −  = x − 
 x 2  x a

y’
 8  8
⇒  a −  − (2 − 4) = (4 − 2) −  a − 
 a   a
 1   3 1 
=  log − 0  +  log − log 
y
 2    2 2 

3 1 3
P(1, 3) = log − 2log log
=
2 2 2
x’ x
(0,0) O A(4,0) 1 1
⇒ − log = loge 3 sq. units
2 2

y’ Sol 7: Given curves =


y 5 − x2 and y = |x – 1| could be
sketche as shown whose point of intersection are.
Thus, area of ∆ formed by (0, 0) (1, 3) and (4, 0) y

0 0 1
1 1
= 1 3 =
1 (0 − 4 3) = 2 3 sq unit
2 2 y=x-1
4 0 1 y = -x + 1
8 8
⇒ a− +2 = 2−a+
a a
x’ x
16 5-1 1 2 5
⇒ 2a − 0
=
a
y’
⇒ 2(a2 – 8) = 0
5 – x = (x – 1)
2 2

∴ a = ± 2 2 , (neglecting –ve sign) ⇒5 – x2 = x2 – 2x + 1


a = 2 2. ⇒ 2x2 – 2x – 4 = 0
⇒ x = 2, – 1
 π π
tanx, − ≤ x ≤ ∴ Required area
Sol 6: Given, y =  3 3
cot x, π ≤ x ≤ π 2 1 2
 =∫ 5 − x2 dx − ∫ ( −x + 1)dx − ∫ (x − 1)dx
6 2 −1 −1 1
1 2
which could be plotted as, y-axis. 2  2   2 
x 5  x   −  −x + x  −  x − x 
=  5 − x2 + sin−1  
π/ 4 π /3  2  −1  2 1
∴ Required area = ∫0 (tanx)dx + ∫
π/ 4
(cot x)dx  2 2  5   −1 

=  − log | cos x |


π/ 4
+ logsinx 
π /3  5 −1 2   5 −1  −1  
0 π/ 4 =  1 + sin  −  −1 + sin   
 2 5  2  5 
2 5 . 5 8 | Area Under the Cur ve and Linear Programming

y y = tanx
 1 1   1 
− − + 1 + + 1 − 2 − 2 − + 1
 2 2   2 

5  −1 2 1  1
=  sin + sin−1 − O
A
1
2 5 5 2 x’ x
B /4 /2-1

5 −1  2 1 1 4 1
= sin  1− + 1− −
2  5 5 5 5  2

5 1 5π 1
= sin−1 (1) − = − sq. units y’
2 2 4 2
π 
⇒ (2x – y) =  − 1 
Sol 8: Given curves are x2 + y2 = 4, x2 = – 2y  2 

Thus, the required area ∴ Required area is OABO


π/ 4
=∫ (tanx)dx − area of ∆ALM
0
2
y=x
π/ 4 1
= log | sec x | − BL . AL
2 2
x +y =4
0 2
O 2
x’ x 1π π−2
-2 - 2 2 = log 2 −  − .1
22 4 

 1
=  log 2 −  sq. unit
-2  4
2
y’ x = - 2y dy
Sol 10: y = x (x – 1)2 ⇒ = x . 2 (x – 1) + (x – 1)2
dx
2 0 2 −x2 2
= ∫− 4 − x2 dx − ∫− x dx − ∫0 dx 1 2 4
2 2
2 ∴ Maximum at x = 1/3 ⇒ ymax =  −  =
3 3 27
 x2 
0 2 Minimum at x = 1
2 x3
= 2∫0 4 − x2 dx −   −
 2  3 2
 − 2 0 y 2
y = x(x-1)
2
x 4 x 2 max
= 2  4 − x2 − sin−1  − 1 − 4
 2 2 2 0 3
27
5 1 x’ x
= (2 − π) − = − π sq. units O 1/3 1 min
3 3

dy
Sol 9: y = tan x ⇒ = sec2 x
dx y’
 dy 
∴   =2 = (x – 1) (2x + x – 1)
 dx x = π
4 = (x – 1) (3x – 1)
π 
Hence, equation of tangent at A  , 1  is + – +
4 
y −1 π 1/3 1
= 2 ⇒ y – 1 = 2x –
x−π/4 2
⇒ ymin = 0
M a them a ti cs | 25.59

Now, to find the area bounded by the curve y = x(x – y


1)2, the y-axis and line x = 2 y=
log x
ex
y y = ex log x
B
C
x’ x
2 O 1/e 1
4
27
x’ x
O 1 A
x=2 y’
1  log x 
∴ The required area = ∫1/e  ex
− ex log x  dx

y’ 1 1
1  (logx)2   x2  e2 − 5
=   − e  (2logx − 1) = sq. units
∴ Required area e  2   4 1/e 4e
1/e
2
= Area of square OABC – ∫ y dx
0
 4a2 4a 1 f( −1) 3a + 3a 
2
2
0
2 2
= 2 × 2 − ∫ x(x − 1) dx = 4 − ∫ x − 2x + x dx
0
( 3 2
) 
Sol 12: Given,  4b2

4b 1

f(1)

=

3b 2
+ 3b


 
 x 4 2x3 x2 
2  2  f(2)  3c2 + 3c 
4  −
=− +  =4 −  16 − 16 + y  4c 4c 1
    
 4 3 2   
0 4 3 2
⇒ 4a2 f(–1) + 4a f(1) + f(2) = 3a2 + 3a, … (i)
10
= sq. units 4b f(–1) + 4b f(1) + f(2) = 3b + 3b
2 2
... (ii)
3
and 4c2 f(–1) + 4cf(1) + f(2) = 3c2 + 3c … (iii)
Sol 11: Both the curves are defined for x > 0. Both are Where f(x) is quadratic expression given by,
positive when x > 1 and negative when 0 < x < 1
f(x) = ax2 + bc + c and (i), (ii) and (iii)
We know, lim (logx) → −∞ ⇒ 4x2 f(–1) + 4x f(1) + f(2) = 3x2 + 3x
x → 0+
logx or {4 f(–1) –3}x2 + {4 f(1) –3} x + f{2} = 0 … (iv)
Hence, lim → −∞ , This, y-axis is asymptote of
x→ 0 + ex As above equation has 3 roots a, b an c
second curve. ∴ above equation is identity in x.

And lim ex log x [(0) x ∞ form] i.e., Coefficients must be zero.


x → 0+
1 ⇒ f(–1) = 3/4, f(1) = 3/4, f(2) = 0 … (v)
e  ∴
elogx  ∞  x =0 f(x) = ax2 + bx + c
= lim  − form  = lim
x→ 0 1 / x  ∞
+  x → 0+  1 
− 2  ∴ a = –1/4 , b = 0 and c = 1, using Eq. (v)
 x 
4 − x2
(using L‘Hospital’s rule) Thus, f(x) = shown as,
4
Thus, the first curve starts from (0, 0) but does not y
include (0, 0).
(0, 1) V
Now, the given curves intersect, therefore
logx
ex logx = A
ex x’
(-2, 0) (2, 0)
x
i.e. (e2x2 – 1) log x = 0
2
1 B(2t, -t + 1)
⇒ x = 1, (since x > 0)
e
y
2 5 . 6 0 | Area Under the Cur ve and Linear Programming

Let A( −2,0),B = (2t, −t2 + 1) v y – 3 = (x + 1)2 + c(x + 1), which passes through (2, 0).

Since, AB subtends right angle at vertex V(0, 1) ⇒ –3 = (3)2 + 3c


⇒ c = –4
1 −t 2
⇒ . = −1 ⇒ t= 4 ∴ Required curve y = (x + 1)2 – 4(x + 1) + 3
2 2t
⇒ y = x2 – 2x
∴B(8, –15)  x3  2
2

∴ Required area = ∫ (x − 2x)dx = − x2  2


Equation of chord AB is 0  3 
 0
−(3x + 6)
y=
2 8 4
= − 4 = sq. units
∴ Required area 3 3

8  4 − x2 3x + 6  Sol 14:
= ∫−2  4 + 2  dx
{( x, y ) ∈ R }
  2
Region
= :y≥ x + 3 ,5y ≤ x + 9 ≤ 15
8
 x 3
3x  2
= x − + + 3x  Plotting all the curves
 12 4 
 −2

 128  2 
= 8 − + 48 + 24 −  −2 + + 3 − 6   6
(-3 , ) B(1, 2)
 3  3  (-4, 1) F
5
C
125
= sq. units
3 D E(-3, 0)
A

Sol 13: Here, slope of tangent


Area = Area ( ABFE ) − Area ( AEB )
dy (x + 1)2 + y − 3 dy (y − 3)
= ⇒ = (x + 1) + ,X
dx (x + 1) dx (x + 1) + Area (DEFC ) - Area (D E C )

Substitute x + 1 = X and y – 3 = Y 1 −3

dy dY
=
32
5 −∫3
− ( −x − 3 dx + ) 11
10 −∫4
− ( )
x + 3 dx =
3
2
sq. units
⇒ =
dx dX

dY Y dY 1 Sol 15: (A, B, D)


∴ = X+ ⇒ − Y=
X
dX X dX X y
S T
y=1
1
∫ − x dX − log X 1
=I.F e= e=
X
y y= 1 U W
e

y= 1 R 2

e y=e-x
2
Q
y = x - 2x
O V P
x

x’
2
x Form figure x = 1 / 2 x= 1 x=1
O 2
S > Area ( OP QR )
y’ 1 1
⇒ S > 1× ⇒S>
∴ Solution is, e e
1 1 Y
Y.
= ∫ X. X dX + c ⇒ = X+c S > Area (P VUW ) + Area ( OS T V )
X X
M a them a ti cs | 25.61

 1  1 1
< 1 −  +1×
 12  e 2

1 1  1 
< + 1 −  D is correct
12 e 2

Now, e− x ≤ e− x if x ∈ ( 0, 1 )
2

1 1
−x − x2  1
∫ e dx ≤ ∫ e dx ⇒  1 − e  ≤ S B is correct
0 0  

1 1 1  1
1− > 1 +  and S > 1 −
e 4 e e

1 1 
⇒S> 1 + 
4 e

(B) and (D) Correct


2017-18 100 &
op kers
Class 12 T
By E ran culty
-JE Fa r
IIT enior emie .
S fP r es
o titut
Ins

MATHEMATICS
FOR JEE MAIN & ADVANCED
SECOND
EDITION

Exhaustive Theory
(Now Revised)

Formula Sheet
9000+ Problems
based on latest JEE pattern

2500 + 1000 (New) Problems


of previous 35 years of
AIEEE (JEE Main) and IIT-JEE (JEE Adv)

5000+Illustrations and Solved Examples


Detailed Solutions
of all problems available

Topic Covered Plancess Concepts


Tips & Tricks, Facts, Notes, Misconceptions,
Vectors Key Take Aways, Problem Solving Tactics

PlancEssential
Questions recommended for revision
26. V E C TO R S

1. INTRODUCTION TO VECTOR ALGEBRA

1.1 Scalars and Vectors


Scalar: A scalar is a quantity that has only magnitude but no direction. Scalar quantity is expressed as a single
number, followed by appropriate unit, e.g. length, area, mass, etc. In linear algebra, real numbers are called scalars.
Vector: A vector is a quantity that has both magnitude and direction, e.g. displacement, velocity, etc.

1.2 Representation of Vectors


(a) A vector is represented diagrammatically by a directed line segment or an arrow. A directed line segment has
both magnitude (length) and direction. The length is denoted by V .

(b) If P and Q are the given two points, then the vector from P to Q is denoted by PQ , where P is called the tail
and Q is called the nose of the vector.

1.3 Vector Components


In a two-dimensional coordinate system, any vector can be resolved into x-component and y-component

v = vx , vy
Let us consider the figure shown (adjacent) here. In this figure, the components can be quickly read. The vector in

the component form is v = 4,5 .
The relation between magnitude of the vector and the components of the vector
can be calculated by using trigonometric ratios.

Adjacent side v
= cos θ = x V Vy
Hypotenuse v

Opposite side v y 
= sin θ =
Hypotenuse v
V
v x = v cos θ; v y = v sin θ Figure 26.1

If v x and v y are the known lengths of a right triangle, then the length of the hypotenuse, V, is calculated by using
the Pythagorean theorem

v
= v 2x + v 2y
2 6 . 2 | Vectors

2. TYPE OF VECTORS

2.1 Null Vector/Zero Vector


A zero vector or null vector is a vector that has zero magnitude, i.e. initial and terminal points are coincident, so that
its direction is in indeterminate form. It is denoted by φ .

2.2 Unit Vector


A unit vector is a vector of unit length. A unit vector is sometimes denoted by replacing the arrow on a vector with
“^”.
Unit vectors parallel to x-axis, y-axis and z-axis are denoted by î , ˆj and k̂ , respectively.

 V
Unit vector Û parallel to V can be obtained as Û = .
|V|
  
Illustration 1: Find unit vector of i − 2J + 3k  (JEE MAIN)
 
a
ˆ
Sol: Here unit vector of a is given by a =  .
    |a|
a = i − 2J + 3k

 1   
( )
  a
ˆ ˆ ˆ
If a = ax i + ay j + azk then it’s magnitude a = ax + ay + az ⇒ a = 14 ⇒ â=
2 2 2 = i − 2J + 3k
14 14

2.3 Collinear or Parallel Vectors


Two or more vectors are said to be collinear, when they are along the same lines or parallel lines irrespective of their
magnitudes and directions.

2.4 Like and Unlike Vectors


Vectors having the same direction are called like vectors. Any two vectors parallel to one another, having unequal
magnitudes and acting in opposite directions are called unlike vectors.

2.5 Co-Initial Vectors


All those vectors whose terminal points are same, are called co-terminal vectors.

2.6 Co-Terminal Vectors


Vectors that have the same initial points are called co-initial vectors.

Illustration 2: Which are co-initial and equal vectors in the given rectangle diagram? (JEE MAIN)

Sol: By following above mentioned conditions we can obtain co-initial and equal vectors.

b
A B

 
a d

C  D
c

Figure 26.2
M a them a ti cs | 26.3

     
Here, a and b are co-initial vectors, b and c , a and d are equal vectors.

2.7 Coplanar Vectors


Vectors lie on the same plane are called coplanar.

2.8 Negative vector


A vector that points to a direction opposite to that of the given vector is called a negative vector.

2.9 Reciprocal of a Vector



A vector having the same direction as that of a given vector a , but magnitude equal to the reciprocal of the given


vector is known as the reciprocal of a and is denoted by a−1 .

2.10 Localized and Free vectors


A vector drawn parallel to a given vector through a specified point unlike free vector in space is called a localized
vector. For example, the effect of force acting on a rigid body depends not only on the magnitude and direction but
also on the line of action of the force. A vector that depends only on its length and direction and not on its position
in the space is called a free vector, e.g. gravity. In this chapter, we will deal with free vectors, unless otherwise stated.
Thus a free vector can be determined in space by choosing an arbitrary initial point.

   
llustration 3: Let a= ˆi + 2ˆj and =
bˆ 2 ˆi + ˆj . Is a = b ? Are the vectors a and bˆ equal?  (JEE MAIN)

Sol: Two vectors are equal if their modulus and corresponding components both are equal.
   
We have a = 12 + 22 = b
5 and = 22 + 12 . So, a = b . But, the two vectors are not equal, since their

corresponding components are distinct.

Illustration 4: Find a vector of magnitude 5 units which is parallel to the vector 2iˆ − ˆj.  (JEE MAIN)

a
Sol: As we know aˆ =  , therefore required vector will be 5aˆ .
|a|
 ˆ ˆ 
Let a= 2i − j. Then, a = 22 + ( −1)2 = 5

1  1 2 ˆ 1 ˆ

∴ Unit vector parallel to a = aˆ =  . a =
a 5
(
2iˆ − ˆj = )5
i−
5
j.

 2 ˆ 1 ˆ
So, the required vector is 5aˆ =5  i− j  =2 5iˆ − 5ˆj .
 5 5 

2.11 Position Vector


A vector that represents the position of a point P in space with respect to an arbitrary reference origin O is called
a position vector (p.v.). It is also known as location vector or radius vector and usually denoted as x, r or s; it
corresponds to the displacement from O to P.

r = OP.
2 6 . 4 | Vectors

Illustration 5: Show that, the three points A(-2,3,5), B(1,2,3) and C(7,0,-1) are collinear.  (JEE MAIN)

 
Sol: By obtaining AB and BC , we can conclude that given points are collinear or not.
We have
    ∧ ∧ ∧   ∧ ∧ ∧  ∧ ∧ ∧
AB = OB − OA =  i + 2 j + 3k  −  −2 i + 3 j + 5k  = 3 i − j − 2k
   
   
    ∧ ∧ ∧   ∧ ∧ ∧  ∧ ∧ ∧  ∧ ∧ ∧
BC = OC − OB =  7 i + 0 j − k  −  i + 2 j + 3k  = 6 i − 2 j − 4k = 2  3 i − j − 2k 
     
     
 
Therefore, BC = 2AB .
 
This shows that the vectors AB and BC are parallel. But, B is a common point. So, the given point A, B and C are
collinear.

2.12 Equal Vectors


Two vectors having the same corresponding components and direction and represent the same physical quantity
are called equal vectors.

 
Illustration 6: Find the values of x, y and z, so that the vectors a = xiˆ + 2ˆj + zkˆ and b = 2iˆ + yjˆ + kˆ are equal.
 (JEE MAIN)
Sol: Two vectors are equal, if their corresponding components are equal.
 
Note that two vectors are equal, if their corresponding components are equal. Thus, the given vectors a and b will
be equal, if and only if x = 2, y = 2, z = 1.

Illustration 7: Find the vector joining the point P (2, 3, 0) and Q (-1, -2, -4) directed from P to Q.  (JEE MAIN)

Sol: By subtracting the component of P from Q we will get PQ .
Since the vector is to be directed from P to Q. Clearly, P is the initial point and Q is the terminal point. So, the
required vector joining P and Q is the vector PQ given by
   
PQ = OQ − OP = ( −1 − 2 ) ˆi + ( −2 − 3) ˆj + ( −4 − 0 ) Kˆ i.e. PQ = − 3iˆ ± 5ˆj − 4kˆ

( ) ( ) ( )
Illustration 8: Show that, the points A 2iˆ − ˆj + kˆ , B ˆi − 3ˆj − 5kˆ , C 3iˆ − 4ˆj − 4kˆ are the vertices of a right-angled
triangle. (JEE MAIN)
 2  2  2
Sol: Here if AB = BC + CA then only the given points are the vertices of right angled triangle. We have

AB =(1 − 2) ˆi + ( −3 + 1) ˆj+ ( −5 − 1)kˆ =−ˆi − 2ˆj − 6kˆ
 
BC = (3 − 1) ˆi + ( −4 + 3) ˆj+ ( −4 + 5)kˆ = 2iˆ − ˆj + kˆ and CA =(2 − 3) ˆi + ( −1 + 4) ˆj+ (1 + 4)kˆ =−ˆi + 3ˆj + 5kˆ
 2  2  2
Moreover, AB =41 =6 + 35 =BC + CA

Hence, it is proved that the points form a right-angled triangle.


M a them a ti cs | 26.5

3. RESULTANT OF VECTORS
When two or more vectors are added, they yield the resultant vector. If vectors A and B are added together, the
  
result will be vector R, i.e. R= A + B . Same technique can also be applied for multiple vectors.

4. VECTOR ADDITION
4.1 Triangular Law of Addition
It states that if two vectors can be represented in magnitude and direction by the two sides of a triangle taken in
the same order, then their resultant is represented by the third side of the triangle, taken in the opposite direction
of the sequence.

4.2 Parallelogram Law of Addition


It states that if two vectors can be represented in magnitude and direction by the two adjacent sides or a
parallelogram, then their resultant is represented by the diagonal of the parallelogram.

4.3 Addition in Component Form


Consider two vectors A and B
A=
< a1 , b1 , c1 >
B=
< a2 , b2 , c2 >
Then, A + B =< a1 + a2 , b1 + b2 , c1 + c2 >

4.4 Properties of Vector Addition


The properties of vector addition are listed as follows:
(a) π / 2 Commutative
(b) π/3 Associative

(c) π / 4 Null vector is an additive identity
(d) ˆ and Bˆ
A Additive inverse
(e) π

(f) ˆ − Bˆ
A
(g) π/2

4.5 Vector Subtraction


   
Subtraction is taken as an inverse operation of addition. If u and v are two vectors, the difference u − v of two
     
vectors is defined to be the vector added to v to get u . In order to obtain u − v , we put the tails of u and v
   
together, the directed segment from the nose of v to the nose of u is a representative of u − v .

 ∧ ∧ ∧  ∧ ∧ ∧
Illustration 9: If a =i + 2 j + 3k and b = 2 i + 4 j − 5k represent two adjacent sides of a parallelogram, find the
unit vectors parallel to the diagonals of the parallelogram. (JEE MAIN)

Sol: As mentioned above, if two vector quantities are represented by two adjacent sides or a parallelogram then
the diagonal of parallelogram will be equal to the resultant of these two vectors.
   
Let ABCD be a parallelogram such that, AB / / b and BC / / b .
2 6 . 6 | Vectors

Then,
        
AB + BC = AC ⇒ AC = a + b = 3iˆ + 6ˆj − 2kˆ and AB + BD = AD
     
⇒ BD = AD − AB ⇒ BD = b − a = ˆi + 2ˆj − 8kˆ 
  a
Now, AC = 3iˆ + 6ˆj − 2kˆ ⇒ AC = 9 + 36 + 4 = 7  D C




b
And BD =ˆi + 2ˆj − 8k.ˆ

a

a

  

b
⇒ BD = 1 + 4 + 64 = 69 b b

 AC 1
∴ Unit Vector along AC=  =
AC 7
( )
3iˆ + 6ˆj − 2kˆ

 A B
 BD 1    
∴ Unit vector along BD=  =
BD 69
( )
i + 2J − 8k .
a
Figure 26.3

      


Illustration 10: ABCDE is a pentagon. Prove that the resultant of the forces AB, AE, BC, DC, ED and AC is 3AC .
 (JEE MAIN)

Sol: By using method of finding resultant of vector we can prove required result. D

Let R be the resultant force


      E
C
∴R = AB + AE + BC + DC + ED + AC

     
(
  
) (
∴R= AB + BC + AE + ED + DC + AC

) A B
= AC + AC + AC = 3AC. Hence proved. Figure 26.4

Illustration 11: ABCD is a parallelogram. If L and M are the middle points of BC and CD, respectively express
      3 
AL and AM in terms of AB and AD , also show that AL + AM = AC  (JEE MAIN)
2
Sol: By using mid – point formula and method of finding resultant of vector we can prove given relation.
 
Let b and a be the position vectors of points B and D, respectively be referred to A as the origin of reference.
      
Then AC = AD + DC = AD + AB ∴DC = AB 
 
      B L C
=d + b ∴AB =b, AD =d
 
i.e. the position vector of C referred to A is d + b
M
   1     1 
AL = p.v. of L, the midpoint of BC . AM= a + d + b  = AD + AB
2  2
    1      
 A D
3 3 3 3
∴ AL + AM = b + d + b = b + d= (b + d)= AC
2 2 2 2 2 Figure 26.5

5. SCALAR MULTIPLE OF A VECTOR


  
If a is the given vector, then k a is a vector, whose magnitude is k times the magnitude of a and whose direction

is the same or opposite as that of a according to whether k is positive or negative.
M a them a ti cs | 26.7

6. SECTION FORMULA
 
(a) If a and b are the position vectors of two points A and B, then the position vector of a point which divides A
 
and B in the ratio m:n is given by r =
(na + mb) .
(m + n)
 

(b) Position vector of the midpoint of AB =
(
a+b
.
)
2

PLANCESS CONCEPTS

  
• If a, b and c are the position vectors of the vertices of any ∆ABC . Then the position vector of
  
a+b+c
centroid G will be .
3

The position vector of incenter of triangle with position vectors of triangle ABC, are A ( a ),
• 
  
   aa + bb + cc
B( b ), C( c ) is r = .
a+b+c
Anurag Saraf (JEE 2011, AIR 226)

Illustration 12: If ABCD is a quadrilateral and E and F are the mid points of AC and BD, respectively, prove that
    
AB + AD + CB + CD = 4EF.  (JEE MAIN)

Sol: By using mid-point theorem we can prove given relation.


Since F is the midpoint of BD. Applying the midpoint theorem in triangle ABD, C
  
we have ⇒ AB + AD = 2AF  … (i) D

Applying the midpoint theorem in triangle BCD, we have E


  
⇒ CB + CD = 2CF 
… (ii)
Adding equations (i) and (ii), we obtain
      A
(
⇒ AB + AD + CB + CD= 2 AF + CF ) B
   Figure 26.6
Now applying the midpoint theorem in triangle CFA, we have AF + CF =
2EF
      
( )
⇒ AB + AD + CB + CD= 2 AF + CF= 4EF Hence proved.
2 6 . 8 | Vectors

     


Illustration 13: If G is the centroid of the triangle ABC, show that GA + GB + GC = 0 and conversely GA + GB + GC = 0,
then G is the centroid of triangle ABC. (JEE ADVANCED)
  
a+b+c
Sol: As G is the centroid of triangle ABC, hence G = . Therefore A
   3
by obtaining GA , GB and GC we can prove this problem.
  
Let the position vector of the vertices be a, b and c , respectively.
   F 2 E
a+b+c G
So, the position vector of centroid, G, is .
     3
    a + b + c 2a − b − c
GA = OA – OG = a − =
   3 3 1
 2b − a − c  2c − a − b
Similarly, GB =
= . GC B  C
D
3 3 
a
   1       b 
⇒ GA + GB + GC=
3
(2a − 2a + 2b + 2b + 2c − 2c = 0 ) c
Figure 26.7
  
Conversely if GA + GB + GC = 0
  
           OA + OB + OC
( ) ( )
⇒ (OA − OG) + OB − OG + OC − OG =0 ⇒ OA + OB + OC =3OG ⇒ OG =
3
Hence, G is the centroid of the points A, B and C.
 
Illustration 14: Find the values of x and y, for which the vectors a = ( x + 2 ) ˆi − ( x − y ) ˆj + kˆ , b =( x − 1 ) ˆi + ( 2x + y ) ˆj + 2kˆ
are parallel (JEE MAIN)

Sol: Two vectors are parallel if ratio of there respective components are equal.
  x+2 y−x 1 −20
a and b are parallel if = = ⇒ x=-5, y=
x − 1 2x + y 2 3

Illustration 15: If ABCD is a parallelogram and E is the midpoint of AB, show by vector method, that DE trisects and
is trisected by AC. (JEE MAIN)

Sol: By using section formula, we can solve this problem. D C


   
Let AB = a and BC = b M
K
       
B
Then BC = AD =b and AC = AB + AD =+ a b A E
Figure 26.8
Also, let K be a point on AC, such that AK:AC = 1:3
    
⇒ AK = 1 AC ⇒ AK = 1 a + b 
3 3
( ) … (i)
 1 
Let E be the midpoint of AB, such that AE = a
2

Let M be the point on DE such that DM: ME = 2:1
   
 AD + 2AE b + a
∴=AM =  … (ii)
 
1+2 3
Comparing equations (i) and (ii), we find that
=
 b+a  ( )
AK = AM , and thus we conclude that K and M coincide, i.e.
   3
DE trisects AC and is trisected by AC . Hence proved.
M a them a ti cs | 26.9

7. LINEAR COMBINATION OF VECTORS

7.1 Collinear and Non-Collinear Vectors


    
Let a and b be non-zero vectors. These vectors are said to be collinear if there exists λ ≠ 0 such that α + λb + γc = 0 .
         
Given a finite set of vectors a, b, c....., then the vector r = xa + yb + zc + ... is called a linear combination of a,b,c.....,
for any scalar x, y, z …… ∈ R .

7.2 Collinearity of Three Points


  
a,b, and c be collinear. Then there exists λ, γ both not being 0
Let three points with position vectors (non-zero) a,b,
 
such that α + λb + γc = 0

7.3 Coplanar Vectors


      
and
Let a,b, a,b, be non-zero, non-collinear vectors. Then, any vector r coplanar with a,b can be uniquely expressed
 
as a linear combination of a,b , i.e. there exist some unique x, y ∈ R, such that.

PLANCESS CONCEPTS

        
•• If a, b, c are non-zero, non-coplanar vectors, then xa + yb + zc= x'a + y 'b + z'c ⇒ x= x', y= y ', z= z'
   
•• Let a, b, c be non-zero, non-coplanar vectors in space. Then any vector r can be uniquely expressed
      
as a linear combination of a, b, c or there exists some unique x, y, z ∈ R, such that xa + yb + zc =
r.

Vaibhav Krishan (JEE 2009, AIR 22)

7.4 Linear Dependency of Vectors


  
{  
}

A set of vectors v1 , v 2 ,.... vp is said to be linearly independent if the vector equation x1 v1 + x2 v 2 + .... + xp vp = 0

{  
}

has only a trivial solution. The set v1 , v 2 ,....vp is said to be linearly dependent if there exists weights c1 ,......,cp , not
  
all 0, such that c1 v1 + c2 v 2 + .... + cp vp =0

PLANCESS CONCEPTS

•• Two non-zero, non-collinear vectors are linearly independent.


•• Any two collinear vectors are linearly dependent.
•• Any three non-coplanar vectors are linearly independent.
•• Any three coplanar vectors are linearly dependent.
•• Any four vectors in three -dimensional space are linearly dependent.

Nitish Jhawar (JEE 2009, AIR 7)


2 6 . 10 | Vectors

       
Illustration 16: The
 position
 vectors of three points A = a − 2b + 3c , B = 2a + 3b − 4c and C = −7b + 10c . Prove
that the vectors AB and AC are linearly dependent. (JEE MAIN)
 
Sol: Here obtain AB and AC to check its linear dependency.
          
Let O be the point of reference, then, OA =−a 2b + 3c, OB =2a + 3b − 4c, and OC = −7b + 10c
          
( ) (
⇒ AC =OC − OA = −7b + 10c − a − 2b + 3c =−a − 5b + 7c )
           
( ) (
AB =OB − OA = 2a + 3b − 4c − a − 2b + 3c =a + 5b − 7c )

∴ AC = λ AB, where λ = −1.
 
Hence AB and AC are linearly dependent.

        
Illustration 17: Prove that the vectors 5a + 6b + 7 c, 7 a − 8b + 9 c and 3a + 20b + 5c are linearly dependent
  
and a, b, c, being linearly independent vectors. (JEE MAIN)

Sol: We know that if these vectors are linearly dependent, then we can express one of them as a linear combination
of the other two.
Now, let us assume that the given vectors are coplanar, and then we can write
        
( ) ( )
5a + 6b + 7c=  7a − 8b + 9c + m 3a + 20b + 5c , where  and m are scalars.
  
Comparing the coefficients of a,bandc on both sides of the equation
5 7 + 3m 
= ....(i) ... (i)
6=−8 + 20m  ....(ii) ... (ii)
7 9 + 5m 
= ....(iii) ... (iii)

From equations (i) and (iii), we get


1
4 = 8 ⇒  = = m , which evidently satisfies equation (ii) too.
2
Hence, the given vectors are linearly dependent.

           
Illustration 18: Prove that the four points 2a + 3b − c,a − 2b + 3c, 3a + 4b − 2 c and a − 6b + 6 c are coplanar.
 (JEE MAIN)
  
Sol: Let the given four points be P, Q, R and S respectively. These points are coplanar, if the vectors PQ, PR and PS
are coplanar. These vectors are coplanar if one of them can be expressed as a linear combination of other two.
So, let
  
PQ
= xPR + yPS
        
(
⇒ −a − 5b + 4c= x a + b − c + y −a − 9b + 7c
 
) ( )
   
⇒ −a − 5b + 4c= ( x − y ) a + ( x − 9y ) b + ( −x + 7y ) c
⇒ x − y =−1, x − 9y =−5, −x + 7y =4
1 1
Solving the first two of these three equations, we get x =
− ,y =
2 2
On substituting the values of x and y in the third equation, we find that the third equation is satisfied. Hence, the
given four points are coplanar.
M a them a ti cs | 26.11

        
Illustration 19: Show that, the vectors 2a − b + 3c, a + b − 2c and a + b − 3c are non-coplanar vectors.
 (JEE MAIN)

Sol: If vectors are coplanar then one of them can be expressed as a linear combination of other two otherwise they
are non-coplanar. Assume the given vectors are coplanar.
Then one of the given vectors is expressible in terms of the other two.
        
Let 2a − b + 3c = x(a + b − 2c) + y( a + b − 3c) for some scalars x and y.
     
⇒ 2a − b + 3c = (x + y) a + (x + y) b + ( −2x − 3y) c ⇒ 2 = x + y, −1 = x + y and 3 = 2x − 3y,
Clearly, the first two equations contradict each other. Hence, it is proved that the given vectors are not coplanar.

8. SCALAR OR DOT PRODUCT


 
The scalar product of two vectors a = ( a1 , a2 , a3 ) and b = (b1 , b2 , b3 ) is written using a dot as an operator (∙)
between the two vectors. The component form of the dot product is as follows:
 
a ⋅=b a1 b1 + a2 b2 + a3 b3 … (i)

And in geometrical form
   
a ⋅ b | a || b | cos θ 
= … (ii)
where θ is the angle between the two vectors and 0 ≤ θ ≤ π .
From equation (i), it can also be written as

a.b a b + a2b2 + a3b3
cos
= θ = 1 1 ,
   
ab ab
 
which can be used to find the angle between two vectors. If a and b are perpendicular then
 
θ = 90° ⇒ cos θ = 0 ⇒ a.b = 0

PLANCESS CONCEPTS
   
•• a⋅b ≤ a b
 
•• a ⋅ b > 0 ⇒ Angle between a and b is acute.
 
•• a ⋅ b < 0 ⇒ Angle between a and b is obtuse.
Shivam Agarwal (JEE 2009, AIR 27)

Geometrical Interpretation of Dot Product



The scalar product is used to determine the projection of r vector along the given direction.
   B
( )
P
ON is the component of vector OB = b in the direction of vector
      M
a⋅b
OA ( a ) ;ON
= = bcos θ . Thus the projection of b along â =  â
a P1 P2

  a.b
 
∴ ON =   a 
  2  O
a  N A
Figure 26.9
2 6 . 12 | Vectors

 
   a.b   a.b 
Projection of a along b =    b̂ ∴ OM =   b
 b  
2

  b
 

8.1 Properties of Scalar Product


The properties of scalar product are listed as follows:
     2  
(a) a, b are vectors and a.b is a number (b) a. a = a (c) a.b = b. a
            
(d) a.(b + c) = a.b+ a. c (e) (ca).b = c(a.b) (f) 0. a = 0
           
(g) a.b a b cos θ (h) a.b =0 ⇔ a =0 or b =0 or a ⊥ b
=

 
Illustration 20: Find the angle ‘ θ ’ between the vectors a = ˆi + ˆj − kˆ and vectors b = ˆi − ˆj + kˆ  (JEE MAIN)
 
 a.b
Sol: The angle θ between the two vectors a and b is given by cos θ =  
ab

( )( )
Now a.b = ˆi + ˆj − kˆ . ˆi − ˆj + kˆ =1 − 1 − 1 =−1
−1  −1 
Therefore, we have cos θ = . Hence, the required angle is θ =cos−1  
3  3 

Illustration 21: Find the length of the projection of vector a = 2iˆ + 3ˆj + 2kˆ on vector bˆ =ˆi + 2ˆj + kˆ .  (JEE MAIN)
  1 
Sol: The projection of vector a on the vector b is given by  a.b .
b
( )
1   ( 2 ⋅ 1 + 3 ⋅ 2 + 2 ⋅ 1 ) 10 5
b
( )
 a.b
= = =
6 3
6
(1 ) + ( 2 ) + (1 )
2 2 2

      
Illustration 22: Let a, b, c be the vectors of lengths 3, 4 and 5, respectively. Let a be perpendicular to (b+ c) , b
     
 
( )
to ( c + a ) and c to a + b . Then, find the length of the vector a + b + c . ( ) (JEE MAIN)

Sol: By using property of scalar product of vector we can solve this illustration.
  
Given= a 3,= b 4,= c 5
  2   2 2 2         
∴ a+b+c = ( a + b + c ) . ( a + b + c ) ( ) ( )
= a + b + c + a b + c + b ( c + a ) + c a + b = 9 + 16 + 25 + 0 + 0 + 0
  
⇒ a+b+c =5 2

   
Illustration 23: Let a = 4iˆ + 5ˆj − k, b = ˆi − 4ˆj + 5kˆ and c = 3iˆ + ˆj − kˆ . Find a vector d , which is perpendicular to both
   
a and b , and satisfying d . c = 21.  (JEE MAIN)

Sol: If two vector are perpendicular then their product will be zero.
  
ˆ Since d is perpendicular to both a and b . Therefore,
Let d = xiˆ + yjˆ + zk.
 
( )( )
d.a = 0 ⇒ xiˆ + yjˆ + zkˆ . 4iˆ + 5ˆj − k = 0 ⇒ 4x + 5y − z = 0  .....(i) ... (i)

( )( )
d.b = 0 ⇒ xiˆ + yjˆ + zkˆ . ˆi − 4ˆj + 5kˆ = 0 ⇒ x − 4y + 5z = 0  ......(ii) ... (ii)

( )( )
d.c = 21 ⇒ xiˆ + yjˆ + zkˆ . 3iˆ + ˆj − kˆ = 21 ⇒ 3x + y − z = 21  ......(iii) ... (iii)
M a them a ti cs | 26.13

Solving equations (i), (ii) and (iii), we get x = 7, y = z = −7



Hence, d = 7iˆ − 7 j − 7kˆ

     
Illustration 24: Three vectors a, b and c satisfy the condition a + b + c =0 . Evaluate the quantity
     
µ= a.b + b.c + c.a, if a= 1, b= 4 and c= 2.  (JEE MAIN)

Sol: Simply using property of scalar product we can calculate the value of µ .
             
Since a + b + c = ( )
0, we have a . a + b + c = 0 ⇒ a.a + a.b + a.c =
0 . Therefore, a.b + a.c =
2
−a =−1

  2  
Similarly a.b + b.c =
−b = −16 , a. c + b.c =
−4 .
   −21
On adding these equations, we have 2(a.b + b.c + a.c) = −21 or 2µ = −21, i.e., µ =
2
 2 2
Illustration 25: Prove, Cauchy–Schawarz inequality, (a.b)2 ≤ a b , and hence show that
2 (JEE ADVANCED)
( a1b1 + a2b2 + a3b3 ) ≤ ( a1 + a2 + a3 ) (b + b2 + b3 )
2 2

2 2
Sol: As we know cos2 θ ≤ 1 , solve it by multiplying both side by a b .
We have, cos2 θ ≤ 1
2 2 2 2   2 2 2
⇒ a b cos2 θ ≤ a b ⇒ a.b ≤ a b ( )
 
Let a = a1ˆi + a2ˆj + a3kˆ and b = b1ˆi + b2ˆj + b3k.
ˆ Then,
  2 2
a.b = a1b1 + a2b2 + a3b3 , a = a12 + a22 + a32 and b = b12 + b22 + b32 .
 2 2

( a.b ) ( )(b )
2
≤ a b ⇒ ( a1b1 + a2b2 + a3b3 ) ≤ a12 + a22 + a32
2 2
1 + b22 + b32

     
Illustration 26: If a, b, c are three mutually perpendicular vectors of equal magnitude, prove that a + b + c is
  
equally inclined with vectors a, b and c . (JEE ADVANCED)
     
Sol: Here use formula of dot product to solve the problem. Let a= b= c = λ (say). Since a,b,c are mutually
  
perpendicular vectors, We have a.b= b.c= c.a = 0
  2            2 2 2   
Now, a + b + c =a.a + b.b+ c. c + 2a.b+ 2b. c + 2 c . a = a + b + c = 3λ2 ∴ a + b + c = 3λ
     
Let a + b + c makes angles θ1 , θ2 , θ3 with a, b and c , respectively. Then,
   
cos θ=
(
a. a + b + c
=
)
  
a.a + a.b + a.c
=
2
a λ 1  1 
∴ θ1 = cos−1 
1            = = 
a a+b+c a a+b+c a a+b+c 3 3  3

 1  −1  1 
Similarly, θ2 =cos1   and θ3 =cos   ∴ θ1 =θ2 =θ3
 3  3
     
Hence, a + b + c is equally inclined with a, b and c.

Illustration 27: Using vectors, prove that cos(A+B) = cosA cosB – sinA sinB (JEE ADVANCED)

Sol: From figure, using vector method we can easily prove that cos(A+B)=cosA cosB – sinA sinB.
2 6 . 14 | Vectors

Let OX and OY be the coordinate axes and let ˆi and ˆj be unit vectors along OX and OY, respectively.
Let ∠XOP
= A and ∠ XOQ = B. Draw PL ⊥ OX and QM ⊥ OX Y
   
Therefore, the angle between OP and OL is A & OQ and OL is B.

In ∆OLP,= OL OP cos A and = LP OP sinA. i


 
= ( OP cos A=
Therefore,OL ) i and LP ( OP sinA ) −ˆj
ˆ ( ) B M L
   A
O
Now, OL + LP = OP X’
j
X

⇒ OP OP ( cos A ) ˆi − ( sinA ) ˆj
= ....(i) ... (i)
 
 
In ∆OMQ,OM
= OQ cosB and = MQ OQ sinB. P
  Y’
Therefore,| OM (=
= OQ cosB ) i, MQ ( OQ sinB ) ˆj
ˆ
Figure 26.10

 OQ  cos β ˆi + sin β ˆj 
OQ OQ (( cos β )) ˆi + (( sin β )) ˆj
⇒= OQ ...(ii) ...(ii)
⇒= ...(ii)
 
From (i) and (ii), weget
From
 (i) and (ii), weget
OP.OQ = OP ( cos A ) ˆˆi − ( sinA ) ˆˆj .OQ ( cosB ) ˆˆi + ( sinB )=
  ˆj OP.OQ cos A cosB − sinA sinB 
OP.OQ = OP ( cos A ) i − ( sinA ) j .OQ ( cosB ) i + ( sinB )= ˆj OP.OQ cos A cosB − sinA sinB 
 
       
But,=   
OP.OQ OP OQ cos(A = + B) OP.OQ cos(A + B)
But,= OP.OQ OP OQ cos(A = + B) OP.OQ cos(A + B)
∴ OP.OQ
= cos(A + B) OP.OQ[cos A cosB − sinA sinB]
∴ OP.OQ
= cos(A + B) OP.OQ[cos A cosB − sinA sinB]
⇒ cos(A = + B) cos A cosB − sinA sinB
⇒ cos(A = + B) cos A cosB − sinA sinB

 
Illustration 28: Find the values of c for which the vectors
= a ( clog2 x ) ˆi − 6ˆj + 3kˆ and
= b (log2 x ) ˆi + 2ˆj + ( 2clog2 x ) kˆ
made an obtuse angle for any x ∈ ( 0, ∞ ) . (JEE ADVANCED)
 
a.b
Sol: For obtuse angle cos θ < 0 , therefore by using formula cos θ =   , we can solve this problem.
  ab
Let θ be the angle between the vectors a and b . Then,
 
a.b
cos θ =  
ab 
a.b
For θ to be an obtuse angle, we must have ⇒ cos θ < 0,for all x ∈ ( 0, ∞ ) ⇒   < 0,for all x ∈ ( 0, ∞ )
ab
 
⇒ a.b < 0, for all x ∈ ( 0, ∞ ) ⇒ a.b < 0, for all x ∈ ( 0, ∞ ) ⇒ c (log2 x ) − 12 + 6c (log2 x ) < 0, for allx ∈ ( 0, ∞ )
2

⇒ cy 2 + 6cy − 12 < 0, for all y ∈ R, where y =− log2 x [ x > 0 ⇒ y =log2 x ∈ R]

⇒ c < 0 and 36c2 + 48c < 0 [ ax2 + bx2 + c > 0 for all x ⇒ a < 0 and Discriminant < 0]
⇒ c < 0 and c(3c + 4) < 0
4  −4 
⇒ c < 0 and − < c < 0 ⇒ c∈ ,0 
3  3 

Illustration 29: D is the midpoint of the side BC of a triangle ABC, show that AB2 + AC2= 2 AD2 + BD2  ( )
 A (JEE MAIN)
Sol: By using the formula of resultant vector we will get the required result.
 
Given D is midpoint of BC ⇒ BD = DC
     2
(
We have AB = AD + DB ⇒ AB2 = AD + DB ) B D
C
 
AB2 = AD2 + DB2 + 2AD ⋅ DB ...(i) Figure 26.11
M a them a ti cs | 26.15

    


( )
2
Also we have AC = AD + DC ⇒ AC2 = AD + DC
 
AC2 = AD2 + DC2 + 2AD ⋅ DC ...(ii)
Adding equations (i) and (ii), we get
  
( ) (
AB2 + AC2 = 2AD2 + 2BD2 + 2AD. DB + DC = 2 DA2 + DB2 )
9. VECTOR OR CROSS PRODUCT
     
Let a and b be two vectors. The vector product of these two vectors can be calculated as a ×=

b ( )
a b sin θnˆ

, where θ is the angle between the vectors a and b , ( 0 ≤ θ ≤ π ) and n̂ is the unit vector at right angles to both
     
a and b , i.e. n̂ is vector normal to the plane that contains a and b . a , b and n̂ are three vectors which form a
right-handed set.
The convention is that we choose the direction specified by the right-hand screw rule. Imagine a screw in your

right hand. If you turn a right-handed screw from a to b , the screw advances along the unit vector n̂ . It is very
important to realize that the result of a vector product is itself a vector.

a
b


b
a

n
Figure 26.12

Let us see how the order of multiplication matters from the definition of the right-hand screw rule:
       
( ) ( )
The vector given by b × a points in the opposite direction to a × b . So, a × b = ( ) (
− b×a . )
ˆi ˆj kˆ
 
We can define vector product in terms of matrix notation as a × b = a1 a2 a3
b1 b2 b3
   
and n terms of components as =a a1 , a2 , a3 =
,b b1 , b2 , b3 ⇒ a ×=b a2b3 − a3b2 ,a3b1 − a1b3 ,a1b2 − a2b1
 
axb
From the definition, the angle can be calculated as sin θ =  
ab
   
If a and b are parallel then θ = 0° ⇒ sin θ = 0 and axb = 0

9.1 Properties of Vector Product


The properties of vector product are listed as follows:
   
a,b and a × b are all vectors in three dimensions.
   
(a) a × b ⊥ a and b
2 6 . 16 | Vectors

   
(b) a=
× b a b sin θ

(c) ˆi × ˆj= k,
ˆ ˆj × kˆ = ˆi,kˆ × ˆi = ˆj
       
(d) a × b = 0 ⇔ a = 0 or b = 0 or a  b
   
(e) a × b =− b × a
     
(f) ( ca) × b =a × cb =c a × b( ) ( )
      
(g) (
a× b + c = a×b + a× c)
     
(h) (
a. b × c = a × b .c ) ( )
     
× b a b sin θ , denotes the area of parallelogram, in which a and b are the two
Geometrical interpretation of a=
adjacent sides.

Vector area of the plane figure


Considering the boundaries of closed, bounded surface, which has been described in a specific manner and that

do not cross, it is possible to associate a directed line segment c , such that


C
outward in the plane

C
not possible
Figure 26.13

(a) c is the number of units of area enclosed by the plane figure.

(b) The support of c is perpendicular to the area and outside the surface.

(c) The sense of description of the boundaries and the direction of c is in accordance with the R.H.S. screw rule.

Vector area of a triangle 


B(b)
 
If a b are the position vectors, then the vector area of a triangle is given by the formula
 1  
=∆
2
(a×b ) O

A(a)
   Figure 26.14
If a, b,c are the position vectors, then the vector area of ∆ABC is given by the formula
 1   
=
∆  c − b × a − b 
( ) ( ) 
A(a)
2 

 1   
=
∆  a × b + b × c + ( c × a ) 
( ) ( )  
2   B(b) C( c )
Figure 26.15
M a them a ti cs | 26.17

PLANCESS CONCEPTS

        
(i) If three points with position vectors a, b and c are collinear, then a × b + b × c + c × a =0
  
(ii) Unit vector perpendicular to the plane of the ∆ABC , when a,bandc are the p.v. of its angular point is
     
a×b + b× c + c × a .
n̂ = ±
2∆
Nitish Jhawar (JEE 2009, AIR 7)

10. ANGULAR BISECTOR


D C
As discussed earlier, the diagonal of a parallelogram is not
necessarily the bisector of the angle formed by two adjacent sides.
D’ C’
However, the diagonal of a rhombus bisects the angle formed
    C’’
between two adjacent sides. Consider vectors
= AB a= and AD b
forming a parallelogram ABCD as shown in the figure. 

Consider the two unit vectors along the given vectors, forming a A B’ B
rhombus AB’C’D’. Figure 26.16

       
 a  b a b a b
Now, AB
=  andAD
=  . Therefore AC'=  +  Then, any vector along the internal bisector is λ   +  .
a b a b a b
 
   
a b
Similarly, any vector along the external bisector is λ   −   .
a b
 

Illustration 30: Find a vector of magnitude 9, which is perpendicular to both the vectors 4iˆ − ˆj + 3kˆ and −2iˆ + ˆj − 2kˆ .
 (JEE MAIN)

Sol: By using property of vector product, we can solve this problem. Let a =4iˆ − ˆj + 3kˆ and bˆ =−2iˆ + ˆj − 2k.
ˆ Then,

ˆi ˆj kˆ
   
a × b = 4 − 1 3 =( 2 − 3 ) ˆi − ( −8 + 6 ) ˆj + ( 4 − 2 ) kˆ =−ˆi + 2ˆj + 2kˆ ⇒ a × b = ( −1)
2
+ 22 + 22 =3
−2 1 − 2
 
 a×b  9
( )
∴ Required vector =9     = −ˆi + 2ˆj + 2kˆ =−3iˆ + 6ˆj + 6kˆ
 a × b  3


Illustration 31: Find the area of a parallelogram, whose adjacent sides are given by the vectors a = 3iˆ + ˆj + 4kˆ and
bˆ = ˆi − ˆj + k.
ˆ
 (JEE MAIN)
   
Sol: The area of a parallelogram with a and b as its adjacent sides is given by a × b .
ˆi ˆj kˆ
 
Now, a × b = 3 1 4 = 5iˆ + ˆj − 4k.
ˆ
1 −1 1
 
Therefore, a × b= 25 + 1 + 16= 42 ; Hence, the required area is 42 .
2 6 . 18 | Vectors

         π
Illustration 32: Let a,b,c be the unit vectors such that a ⋅ b = a ⋅ c = 0 and the angle between b and c is ,
   6
Prove that a = (
±2 b × c . ) (JEE MAIN)

        
Sol: Here a ⋅ b = a ⋅ c = 0 , therefore a is perpendicular to the plane of b and c and it is parallel to b × c .
   
We have a ⋅ b = a ⋅ c = 0
      
⇒ a ⊥ b and a ⊥ c ⇒ a is perpendicular to the plane of b and c.
     
( )
⇒ a is parallel to b × c. ⇒ a =λ b × c for some scalar λ.

   λ 
⇒ a =λ b c sin
π
⇒1=  a =b =c 
6 2  

⇒ λ =2 ⇒ λ = ±2
     
(
∴ a =λ b × c ) (
⇒ a =±2 b × c . )
           
Illustration 33: If a, b, c are three non-zero vectors, such that = b × c a, prove that a, b, c are
a × b c and =
  
mutually at right angles=and b 1= and c a . (JEE MAIN)

Sol: Use property of vector or cross product to prove this illustration.


   =        
We
a × bhave,
= a × b=
c and b ×cc and b×c a
a =
   

   ⇒ c ⊥ a,c ⊥  a ⊥ b,a ⊥ c  ⇒ a ⊥ b,b ⊥ c and c ⊥ a.
 b and 
⇒ c ⊥ a,c ⊥ b and a ⊥ b,a ⊥ c ⇒ a ⊥ b,b ⊥ c and c ⊥ a.
  
⇒ a,b,c are mutually perpendicular lines.
           
Again a × =
b c and b × =
c a ⇒ a × b= c and b × c= a
  π    π     
⇒ a b sin = c and b c sin = a [ a ⊥ b and b ⊥ c]
2 2
     
⇒= a b c and= b c a
2        
⇒=b c c Putting
= a b c
a b c in=
 
2 
=⇒b 1  c ≠ 0 
 

⇒b = 1
     
Putting
= b 1=in a b c, we obtain
= a c.

a b c
Illustration 34: Prove by vector method, that in a ∆ABC, = =  (JEE MAIN)
sinA sinB sinC

1   1   1  


Sol: As area of triangle ABC is equal to AB × AC = BC × BA = CA × CB , therefore by using cross product
method we can prove this problem. 2 2 2
     
Let=BC a,= CA b,= AB c. Then

1   1   1  


The area of ∆ABC = AB × AC = BC × BA = CA × CB ⇒ bc sinA =ca sinB =ab sinC
2 2 2
sinA sinB sinC
Dividing the above expression by abc, we get = =
a b c
M a them a ti cs | 26.19

 
Illustration 35: Given the vectors a = pˆ + 2qˆ and b =
2pˆ + qˆ , where p and q are unit vectors forming an angle of
30° . Find the area of the parallelogram constructed on these vectors. (JEE MAIN)
   
Sol: Simply by applying cross product between a and b , we have a × b =(pˆ + 2qˆ ) × ( 2pˆ + qˆ ) =−3 (pˆ × qˆ ) .
  π 3
⇒ a × b= 3 (pˆ + qˆ )= 3 pˆ qˆ sin =
6 2

      


Illustration 36: Let OA = a,OB = 10a + 2b and OC = b, where O is the origin. Let p denote the area of
 
the quadrilateral OABC and q denote the area of the parallelogram with OA and OC as adjacent sides.
Prove that p = 6q.  (JEE MAIN)

Sol: We have to obtain the area of quadrilateral and parallelogram using cross product method to get the required
result.
We have, p = area of the quadrilateral OABC
       
= 1/ 2 OB × =
AC 1/ 2 OB × OC − OA ( )

(
= 1/ 2 10a + 2b × b − a ) ( )
       
( ) ( ) (
= 1/ 2 10 a × b − 10 ( a × a ) + 2 b × b − 2 b × a )
     
= 1/ 2 10 ( a × b ) − 0 + 0 + 2 ( a × b )= 6 ( a × b )  ... (i)
 
and q = area of the parallelogram with adjacent sides OA and OC
   
=OA × OC =a × b ( )

..(ii)  ... (ii)

From equations (i) and (ii), we get p = 6q.

Illustration 37: Given that D, E, F are the midpoints of the sides of a triangle ABC, using the vector method, prove
1
∆DEF
that area of = (area of ∆ABC)  (JEE MAIN)
4
 
Sol: Taking A as the origin, let the position vectors of B and C be b and c respectively.

1   1  1   
Then, the position vector of D, E and F are
2
( 2
)
b + c , c and b respectively. Therefore first obtain DE and DF ,
2

and after that by applying formula of vector area of triangle DEF we can obtain the required result.

 1  1   −b 
Now, DE = c − b + c =
2 2
(
2
) A(origin)

 1  1   
−c
and DF = b − b + c = ( )  
2 2 2

b
2
F
(( E
c
2 ((

1    −b −c 
∴ Vector area of ∆DEF =
2
(
DE × DE = 
 2
) × 
2   
B(b) C( c )
D
1   1 1  1 1 
1 
  1  1
=
8
( ) (
8 4 2
)(
b × =c = b × c =AB ×AC AB ()
= × AC
4 2  4  4
)
(vector
= area of ∆area
(vector ABC)of ∆ABC)
Figure 26.17
1 1
Hence,area of ∆=
DEFof ∆=
Hence,area (area of (area
DEF ∆ABC)of ∆ABC)
4 4
2 6 . 20 | Vectors

Illustration 38: Given that P, Q are the midpoints of the non-parallel sides BC and AD of a trapezium ABCD. Show
that area of ∆APD = ∆CQB . (JEE MAIN)
   
Sol: Use formula of vector area of triangle to solve this problem.
= Let AB b= and AD d Now DC is parallel to
   
AB ⇒ there exists a scalar t, such that. =
DC t=AB tb
    
∴ AC =AD + DC =d + tb D C
AB + DC Q P
From geometry we know that QP = QP =
2
   
  b + d + tb d   A B
Now AP and AQ are and  , respectively.
2
Now,2 ∆APD = 2 AP × AD
Figure 26.18
  1     1  
Now,2∆APD =AP × AD=
2
(
b + d + tb × d=
2
)( 1 + t) b × d ( )
1     1  
=
2
(  2 (
b + d + tb × d= )
1 + t ) b × d
     d ( ) 
Also 2∆CQB = BC × BQ =  −b + d + tb  ×  −b + 
 2
   
  b × d tb × d 1  
(
= − d×b −
2
) +
2
= (1 + t ) b × d = 2∆APD ⇒ ∆APD = ∆CQB
2

11. TRIPLE PRODUCT OF VECTORS


Two types of triple products are listed below:
Vector triple product ⇒ ( a × b ) × c
Scalar triple product ⇒ ( a × b ) .c h

c
C
b 
11.1 Scalar Triple Product O
a A
The scalar triple product has an interesting geometric Figure 26.19
interpretation:
 
We know that a ×= b ( )
a b sin θnˆ = (area of the parallelogram defined by a and b)
   
( ) 
Thus, a × b .c = (area of the parallelogram) n̂. c = (area of the parallelogram) n̂ c cos φ
   
But c cos θ = h= height of the parallelepiped normal to the plane containing a and b . ( φ is the angle between c
and n̂).
     
( )
So, a × b .c = volume of the parallelepiped defined by a , b and c . Thus, the following conclusions are arrived:
  
( )
(a) If any two vectors are parallel, then a × b .c = 0 (zero volume)
  
(
(b) If the three vectors a co-planar, then a × b .c = 0 (zero volume) )
  
( )
(c) If a × b .c = 0, then either

 
(i) a = 0, or (ii) b = 0 or (iii) c = 0 or
(iv) two of the vectors are parallel or (v) the three vectors are co-planar
        
( ) ( )
(d) a × b .c = a. b × c = b. ( c × a )= The same volume.
   
(e) (a × b) . c is also known as box product, which is represented as [ ab c ].
         
Also a + b c d = a c d + b c d
     
M a them a ti cs | 26.21

        
(f) If a,b,c are non-coplanar, then [ a,b,c ]>0, for right-handed system and [ a,b,c ] < 0, for left handed system.
  
(g) If O is the origin and a,b,c are the position vectors of A, B and C, respectively, of the tetrahedron OABC, then
1   
the volume is given by the formula V = a b c  .
6
Reciprocal system of vectors
           
(a) If a,b,c and a',b' ,c' are the two sets of non-coplanar vectors, such that a.a', = b.b' = c.c' = 1,
           
a.b'
= a.c' = 0, b.a'
= b.c' = 0 and c.a'
= c.b'= 0,
     
Then a,b,c and a',b' ,c' constitute a reciprocal system of vectors.
(b) Reciprocal system of vectors exists only in the case of dot product.
     
(c) a',b' ,c' can be defined in terms of a,b,c as
     
     
= a' =
b×c
   ;b' =
a b c 
c×a
   ;c'
a b c 
a×b
  
a b c 
a b c  ≠ 0
  ( )
     

Note:
              
(i) ( )
a × a' + b × b' + c × c' = 0 ⇒ a × b × c + b × ( c × a ) + c a × b = 0 ( )
     
(ii) a ⋅ a' =b ⋅ b' =c ⋅ c' =1
     
( )(
(iii) a + b + c . a' + b' + c' = )3
      1      
V then a' b' c' =
(iv) If a b c  = ⇒ a b c  a' b' c' =
1
  V  

  
Illustration 39: If  , m, n three non-coplanar vectors, then prove that
    
⋅a  ⋅b 
         
  ( )
  m n a × b = m ⋅ a
 
m ⋅ b m . 
  
(JEE ADVANCED)
n⋅a n⋅b n

Sol: Use scalar triple product method as mentioned above to solve this problem.
  
Let,  = 1ˆi +  2ˆj +  3k,
ˆ m = m1ˆi + m2ˆj + m3k, ˆ n = n1ˆi + n2ˆj + n3k,
ˆ
 
and a = a1ˆi + a2ˆj + a3k, ˆ b = b1ˆi + b2ˆj + b3kˆ

1 2 3 ˆi ˆj kˆ
    
 
Now   m n = m1 m2 ( )
m3 and = a × b = a1 a2 a3
n1 n2 n3 b1 b2 b3

1  2  3 ˆi ˆj kˆ 1ˆi +  2ˆj +  3kˆ ∑ 1a2 ∑ 1b1


    
  ( )
  m n a × b = m1 m2 m3 a1 a2 a3 = m1ˆi + m2ˆj + m3kˆ ∑ m1a1 ∑ m1b1
n1 n2 n3 b1 b2 b3 n1ˆi + n2ˆj + n3kˆ ∑ n1a1 ∑ n1b1

Now, .a = (  ˆi +  ˆj +  kˆ ) . (a ˆi + a ˆj + a kˆ ) = ∑  a
1 2 3 1 2 3 1 2 etc.
2 6 . 22 | Vectors

         
  ⋅ a  ⋅b  ⋅ a  ⋅b 
              
(
∴   m n a × b = )
m m⋅ a m⋅b =
    
m⋅ a m⋅b m ,
    
n n⋅ a n⋅b n⋅ a n⋅b n

Hence proved.

Illustration 40: Find the volume of a parallelepiped, whose sides are given by −3iˆ + 7ˆj + 5k,
ˆ −5 ˆi + 7 ˆj− 3kˆ and
7 ˆi − 5ˆj − 3kˆ  (JEE MAIN)
     
Sol: We know that, the volume of a parallelepiped, whose three adjacent edges are a,b,c is a b c  .
 
ˆ =−5 ˆi + 7 ˆj− 3kˆ and c =7 ˆi − 5ˆj − 3kˆ
Let a =−3iˆ + 7ˆj + 5k,b
     
We know that, the volume of a parallelepiped, whose three adjacent edges are a,b,c is a b c 

−3 7 5
  
Now, a b c  =
  −5 7 −3 = −3 ( −21 − 15 ) − 7 (15 + 21 ) + 5 ( 25 − 49 ) =−
108 252 − 120 =
−264
7 −5 −3
  
So, the required volume of the parallelepiped= a b c  = −264 = 264 cubic units.

     
Illustration 41: Simplify a − b b − c c − a  (JEE ADVANCED)

Sol: Here by using scalar triple product we can simplify this.


         
a − b b − c c − a =
  {(
a − b × ) (
b )}
 
− c . ( c − a ) [by def.]
         
( )
= a × b − a × c − b × b + b × c . ( c − a ) [by dist.law]
         
( )
= a × b + c × a + b × c . ( c − a)  b × b = 0 
 
                 
( ) ( ) ( ) ( )
= a × b .c − a × b .a + ( c × a ) .c − ( c × a ) . a + b × c . c− b × c .a [by dist. law]
     
= a b c  − 0 + 0 − 0 + 0 − b c a
             
 b c a  = a b c  
= a b c  − b c a = a b c  − a b c  = 0
    

   
Illustration 42: Find the volume of the tetrahedron, whose four vertices have position vectors a,b,c and d ,
respectively. (JEE MAIN)
... (i)
1       
Sol: Here volume of tetrahedron is equal to a − d b − d c − d .
6
   
Let, four vertices be A, B, C, D with p.v. a,b,c and d respectively.
        
( ) ( )
∴ DA = a − d ,DB = b − d ,DC = c − d ( )
1       1       
Hence volume=
6
a−d b−d c − d=
6
( ) (
a−d . b−d × c−d
 ) ( )
1                
=
6
( )
a − d . b × c − b × d + c × d=

1 
6 
a{ b

c    
 
 −  a b d +  a c }
d − d b c 

1             
=
6 
{
a b c  − d b c  −  a d c  − a b d . }
M a them a ti cs | 26.23

         
Illustration 43: Let u and v be unit vectors and w is a vector, such that u ×=
v + u w and=
w × u v , then find the
  
value of u v w  .  (JEE ADVANCED)

      
Sol: Here as given u ×= v + u w and= w × u v , solve it using scalar triple product.
      
Given, u ×= v + u w and= w ×u v
     
⇒ (u × v + u) × u = w × u
        
⇒ (u × v )=×u + u x u v = (as, w × u v)
           
⇒ (u.u) v − ( v.u) u += u=
× u v (using = = u.u
u.u 1and
1 andu=
=×uu× u 0,0,since unit
since unitvector)
vector)
      
⇒ v − ( v.u) u = v ⇒ (u.v ) u = 0
 
=
=⇒ u.v 0 (as;uu ≠≠ 0)
(as; 0) .......(i)
.......(i)
     
∴ u v w  = u. ( v × w )
        
= u. v × (u × v + u)( (givenw
(given )
w ==uu××vv ++u) u)
            
( ) (
= u. v × (u × v ) + v × u= u. ( v.v ) u − ( v.u) v + v × u )
    
= u. (u − 0 + v × u) (as u.v= 0 from (i))
   
= (u.u) − u. ( v × u) =
= = 1 −−00 = =11
  
∴ u v w  = 1

11.2 Vector Triple Product


     
( )
Definition: a × b × c is a vector, which is coplanar to a and b and perpendicular to c .
           
( )
a × b Hence (
× c = xaa+× yb )
b × c = xa ...(i)
+ yb ...(i)[Linear Combination
[Linear Combination
of a and b] of
 a and b] ... (i)
     
( )
× c x ( a.c ) + y b.c 
c. a × b = ( ) ... (ii)
 
= 0 x ( a.c ) + y b.c ( ) ....(ii)

x y
 = − = λ
b.c a.c
 
( )
∴ x = λ b.c and y = −λ ( a.c )
      
( ) ( )
Substituting the values of x and y we get, a × b × c = λ b.c a − λ ( a.c ) b
  
This identity must hold true for all values of a,b,c
  
Substitute= a ˆi;b
= ˆjandc= kˆ

(ˆi × ˆj) × ˆi = λ (ˆj.iˆ ) ˆi − λ (ˆi.iˆ ) ˆj


ˆj = −λˆj ⇒ λ = −1
      
⇒ a×b = ( )
× c ( a.c ) b − b.c a ( )   
   a×b × a
Note: Unit vector coplanar with a and b perpendicular to a is ±   
( )
a×b × a ( )
2 6 . 24 | Vectors

      
{ (
Illustration 44: Prove that a × b × c × =
d )} (b.d) ( a × c ) − (b.c )( a × d)  (JEE MAIN)

Sol: By using vector triple product as mention above.


     
{ 
} {( ) ( ) }
We have, a × b × ( c × d) =a × b.d c − b.c d
    
=

{( )
a × b.d c − a b.c d
  
( )}
  
[by distributivelaw]

( ) ( )( )
= b.d ( a × c ) − b.c a × d

 
Illustration 45: Let a =aiˆ + 2ˆj − 3k,b ˆ and c =2iˆ + ajˆ − kˆ . Find the value (s) of a, if any, such that
ˆ =ˆi + 2ajˆ − 2k,
   
{( ) ( )} 
a × b × b × c × ( c × a) =
0.  (JEE MAIN)

Sol: Here use vector triple product to obtain the value of a.


                 
{( ) ( )}
a×b × b× c =
 
× ( c × a ) a b c  b = {( ) ( ) }
× ( c × a ) a b c  a.b c − b.c a ,
            
{( ) ( )}  
Given a × b × b × c × ( c × a ) = ( ) ( )
0 ⇒ a.b c = b. c a or a b c  = 0
     
( ) ( )
a.b c = b. c a leads to three different equations which do not have a common solution.
a 2 −3
  
a b c  = 0 ⇒ 1 2a −2 = 0 ⇒ 9a − 6 = 0 ⇒ a = 2
  3
2 a −1

      
Illustration
 46: Solve for r , from the simultaneous equations r × b = c × b, r.a = 0 , provided a is not perpendicular
to b . (JEE MAIN)
   
Sol: As given r × b = c × b , solve this using vector triple product to get the result.
   
Given r × b = c × b
     
⇒ ( r − c ) ×=b 0 ⇒ ( r − c ) and b are collinear
     
∴ r − c = kb ⇒ r = c + kb ....(i)
   
= r.a 0

⇒= (
c + kb .a 0 )    
a.c   a.c  a × (c× b)
⇒ k =−   putting in eq. (i) we get r =c −   b =   .
a.b a.b a.b

        
Illustration 47: If x × a + kx = b , where k is a scalar and a,b are any two vectors, then determine x in terms of a, b
and k. (JEE MAIN)
    
Sol: Here as given x × a + kx = b , Apply cross product of a with both side and solve using vector triple product.
   
x × a + kx = b  ....(i) ... (i)
      
( )
⇒ a × ( x × a) + k ( a × x ) = a × b
       
⇒ ( a.a ) x − ( a.x ) a + k ( a × x ) =a × b  ...(ii) ... (ii)
     
(i) ⇒ a. ( x × a ) + k ( a.x ) = a.b
 
⇒ k ( a.x ) = a.b  … (iii)
M a them a ti cs | 26.25

Substituting the values from equations (i) and (iii) in equation (ii), we get,
  1       
( ) ( )
⇒ ( a.a ) x − a.b a + k kx − b =a × b
k

  1        a.b  
( ) (
k
) ( )
⇒ a2 + k 2 x = a × b + a.b a + kb =

⇒x
1
a + k2
2 ( )
kb + a × b +
 k 
a

12. APPLICATION OF VECTORS IN 3D GEOMETRY


 a b c
(a) Direction cosines of r = aiˆ + bjˆ + ckˆ are given by  ,  ,  .
r r r
  
aa + bb + cc
(b) Incentre formula: The position vector of the incentre of ∆ABC is
a+b+c
(c) Orthocentre formula: The position vector of the orthocenter of
  
atanA + b tanB + c tanC
∆ABC is
tanA + tanB + tanC

(d) The vector equation of a straight line passing through a fixed point with position vector a and parallel to a
   
given vector b is given by r = a + λb .
 
(e) The vector equation of a line passing through two points with position vectors a and b is given by
   
(
r= a+λ b−a . )

(f) Perpendicular distance of a point from a line: Let L be the foot of perpendicular drawn P ( α ) on the line
   
r = a + λb , where r is the position vector of any point on the give line. Therefore, let the position vector
 
L be a + λb . P()
       
(a − α) × b        ( a − α ) .b  
PL =  and PL = a − α + λb = ( a − α ) −  2 b
b  b 
 
 A   B
  
The length PL is the magnitude of PL , and the required length of perpendicular. r = a + b L = (a + b)

Figure 26.20

    
()
(g) Image of a point in a straight line: If Q β is the image of P in r = a + λb , then P()

   
   2 ( a − α ) .b   
β= 2a −  2 b − α
 b  A B
    
L = (a + b)
  
L = (a + b)


Q()Image

Figure 26.21
2 6 . 26 | Vectors

    
(h)
 Shortest distance  between two skew lines: Let l1 and I2 be two lines whose equations are l1 : r= a1 + λb1 and l2 : r= a2
   
l1 : r= a1 + λb1 and l2 : r= a2 + λb2 , respectively.
       
Then, shortest distance = is given by PQ =  
( )
b1 × b2 . ( a2 − a1 ) b1 b2 a2 − a1 

 

b1 × b2 b1 × b2

Shortest distance between two parallel lines: The shortest distance between the two given parallel lines
  
      ( a2 − a1 ) × b
r= a1 + λb and r= a2 + µb is given by d =  .
b
     
If the lines r= a1 + λb1 and r= a2 + µb2 intersect, then the shortest distance between them is zero.
   
Therefore, b1 b2 a2 − a1  = 0
           
(i) If the lines r= a1 + λb1 and r= a2 + λb2 are coplanar, then a1 b1 b2  = a2 b1 b2  and the equation of
     
the plane containing them is given by  r b1 b2  = a1 b1 b2  .
       
( )
( j) The vector equation of a plane through the point A ( a ) and perpendicular to the vector a × b + c = a × b + a × c
  
is given by ( r − a ) .n = 0.

(k) Vector The vector equation of a plane normal to unit vector n and at a distance d from the origin is given by

r.nˆ = d.
  
(l) The equation of the plane passing through a point having position vector a and parallel to b and c is given
         
by r = a + λb + µc ⇒  r b c = a b c  , where λ and µ are scalars.
      
(m) The vector equation of plane passing through a point a , b , c is given by r =(1-s-t) a +s b +t c
         
(
or r ⋅ b × c + c × a + a × b =  )
a b c 

    
(n) The equation of any plane through the intersection of= planes r.n1 d= 1 and r.n2 d2 is r. (n1 + λn2 =
) d1 + λd2 ,
where λ is an arbitrary constant.

  a.n − d
(o) The perpendicular distance of a point having position vector a from the plane r.n = d is given by p =  .
  n
  n1 .n2
(p) The angle θ between the planes
= r1 .nˆ 1 d= ˆ
1 and r2 .n2 d2 is given by cos θ = ±  
n1 n2
  
(q) The perpendicular distance of a point P( r ) from a line passing through a and parallel to b is given by

   1/2
( r − a) × b     2 
P=     2 

= ( r − a ) − 
( r − a ) .b  
  
b  b  

 
   
(r) The equation of the planes bisecting the angles between the planes
= r1 .n1 d=
1 and r2 .n2 d2 is
   d d
r. (n1 ± n2 ) =  1 ±  2
n1 n2

  
(s) The perpendicular distance of a point P( r ) from a plane passing through a point a and parallel to points b
   

and c is given by PM =
( r − a) . b × c
 
( )
b×c
M a them a ti cs | 26.27

   
(t) The perpendicular distance of a point P( r ) from a plane passing through the points a , b and c is given by
       
P=
(
( r − a) . b × c + c × a + a × b
     
)
b× c + c × a + a×b

   
( )
(u) Angle between a line and the plane: If θ is the angle between a line r= a + λb and the plane r.n = d , then

b.n
sin θ =   .
b n
   
(v) The equation of sphere with center at C( c ) and radius ‘a’ is r − c =
a. If center is the origin then r = a.

   a.n − d
(w) The plane r.n = d touches the sphere= r − a R,if=  R , i.e. the condition of tangency.
n
 
(x) If a and b are the position vectors of the extremities of a diameter of a sphere, then its equation is given by
    2       2  2  2
( r − a) . r=( ) (
− b 0 or r − r. a + b + a.b = )
0 or r − a + r − b = a − b .

FORMULAE SHEET

(a) OP = x î +y ˆj
 y
(b) OP
= x2 + y 2 and direction is tan=
θ
x

Vector a
(c) Unit vector Û = = 
Its modulus a

(d) Properties of vector addition:

         
i. a + b = b + a commutative ( ) ( )
(a) a + b + c = a + b + c Associative
     
ii. a + 0 =a Null vector is an additive identity (b) a + ( −a ) =0 Additive inverse

      
( )
iii. c a + b = ca + cb (c) ( c + d) a =ca + da
   
iv. ( cd) a = c ( da ) (d) 1 × a =a

(e) Section formula:


 
(i) If a and b are the position vectors of two points A and B, then the position vector of a point which divides
 


AB in the ratio m:n is given by r =
na + mb( .
)
(m + n)
 


(ii) Position vector of mid-point of AB =
a+b
.
( )
2
2 6 . 28 | Vectors

  
(f) Collinearity of three points: If a, b, and c are the position vectors (non-zero) of three points and given they
  
are collinear then there exists λ , γ both not being 0 such that a + λb + γc

    
(g) Coplanar vectors: Let a, b be non-zero, non-collinear vectors. Then, any vector r coplanar with a, b can be
    
expressed uniquely as a linear combination of a, b i.e. there exist some unique x, y ∈ R, such that xa + yb = r

(h) Product of two vectors:

(i) Scalar Product (dot product)



If a.b = a1b1 + a2b2 + a3b3
 
a.b
Note : • cos θ =  
ab
 
• a and b are perpendicular if θ= 90°

(ii) Properties of scalar product:

      
i. a.b = b. a ii. ma.nb = a .mnb
= mna.b ( )
    
( )
iii. a. b + c = a.c + a.b
( 
iv. a + b )
2   
=a2 + 2.a.b + b2

v. If ˆi
= (=
1,0,0 ) , ˆj ( 0,1,0
= ) ,kˆ ( 0,0,1 ) then ˆi.=
ˆj ˆj.k
=ˆ k.i
ˆ=ˆ 0

 
(iii) Vector (cross) Product of two vectors:
= Let a (=a1 , a2 , a3 ) ,b (b1 , b2 , b3 ) be two vectors then the cross
   
product of a × b is devoted by a × b and defined by

   a2 a3 a1 a2 
a ×=
b ( a1 , a2 , a3 ) × (b1 , b2 , b=
3)  =  ( a2b3 − a3b2 ,a3b1 − a1b3 ,a1b2 − a2b1 )
 b2 b3 b1 b2 

OR
ˆi ˆj kˆ
 
a ×=
b a1 a2 a= 3 ( a2b3 − a3b2 ) ˆi + ( a3b1 − a1b3 ) ˆj + ( a1b2 − a2b1 ) kˆ
b1 b2 b3
   
a × b = a × b sin θ nˆ

 
Note: (i) θ being angle between a & b
       
(ii) If θ =0 , The = a × b 0 and a & b are parallel if a × b =0.
a × b 0 i.e. =
M a them a ti cs | 26.29

(iv) Properties of cross product

         
0 ⇒ a 0=
=
i. a × b = or b 0 or a  b ii. a × b =−b × a

          
( )
iii. a × b + c = a × b + a × c (
iv. (na ) × b= n a × b )
 
    vi. a × b is a Area of parallelogram with sides
v. a × b is perpendicular to both a and b 

a and b .

 
(v) Scalar Triple Product: If a = a1ˆi + a2ˆj + a3k,b ˆ  = c ˆi + c ˆj + c kˆ .
ˆ = b ˆi + b ˆj + b k,c
1 2 3 1 2 3
        
( )
Then a. b × c = b. ( c × a ) = c. a × b ( )
a1 a2 a3
  
(
a. b × c = )
b1 b2 b3
c1 c2 c3
   
( )
a. b × c is also represented as a b c 

  


=a b c  =
b c a  c a b 
     
 
a b c  = − a c b 
   
  
• If any of the two vectors are parallel, then a b c  = 0
     
• a b c  is the volume of the parallelepiped whose coterminous edges are formed by a b c
 
     
• If a b c are coplanar, a b c  = 0

1         
• a × b + b × c + c × a = area of triangle having a ,b, c as position vectors of vertices of a triangle.
2

(vi) Vector Triple Product:


      
a × b ×=
c( ) ( a.c ) b − ( a.b ) c
      
(a×b = ×c ) ( a.c ) b − (b.c ) a
  

   a×b × a
Unit vector coplanar with a and b perpendicular to a is ±    .
( )
a×b × a ( )
2 6 . 30 | Vectors

Solved Examples

JEE Main/Boards Example 2: If ABCDE is a pentagon, prove that the


      
resultant of AB, AE,BC,DC,ED and AC is 3AC
Example 1: Show that the points A, B & C with position
vector 2iˆ − ˆj + k,
ˆ ˆi − 3ˆj − 5kˆ and
and 3iˆ − 4ˆj − 4kˆ respectively, Sol: By using resultant vector formula, we can obtain
required result.
are the vertices of a right angled triangle. Also find the
remaining angles of the triangle. If R be the resultant vector then
E
     
R = AB + AE + BC + DC + ED + AC
Sol: We have,      
( ) ( )
A D
 = AB + BC + AE + ED + DC + AC
( ) (
AB = ˆi − 3ˆj − 5kˆ − 2iˆ − ˆj + kˆ =−ˆi − 2ˆj − 6kˆ )     B C
 = AC + AC + AC = 3AC
( ) (
BC = 3iˆ − 4ˆj − 4kˆ − ˆi − 3ˆj − 5kˆ = 2iˆ − ˆj + kˆ and )

( ) (
CA = 2iˆ − ˆj + kˆ − 3iˆ − 4ˆj − 4kˆ =−ˆi + 3ˆj + 5kˆ ) Example 3: Prove that the straight lines joining the
mid points of the sides of a quadrilateral ABCD, taken
  
Since AB + BC + CA in order, form a parallelogram.

( ) ( ) (
= −ˆi − 2ˆj − 6kˆ + 2iˆ − ˆj + kˆ + −ˆi + 3ˆj + 5kˆ = 0 ) Sol: Let the position vectors of A, B, C and D be
   
a, b, c and d . Hence position vectors of P, Q, R and S
So, A, B and C are the vertices of a triangle.
(the mid points of AB, BC, CD & DA respectively) are
 
Now, BC.CA        
a+b b+c c +d d+a
, , and respectively.
= (2iˆ − ˆj + kˆ ) + ( −ˆi + 3ˆj + 5kˆ ) =−2 − 3 + 5 =0 2 2

2

2
   c + b a + b R
π PQ
= −
D C
BC ⊥ CA ⇒ ∠BCA = 2 2
2  
 c−a
Hence, ABC is a right angled triangle. Since a is the PQ
= PO + OQ = S Q
  2
angle between the vectors AB andAC . Therefore,  
 a + d c + d a − c
  RS = − =
AB.AC 2 2 2 A P B
CosA =    
AB AC  c − a
⇒ SR =
  2
( )(
−ˆi − 2 j − 6k . ˆi − 3ˆj − 5kˆ )  
PQ is parallel and equal to SR . Hence PQRS is a
( −1) + ( −2) + ( −6 ) 12 + ( −3) + ( −5 )
2 2 2 2 2
parallelogram.

−1 + 6 + 30 35 35
= = = Example 4: Write an equation for the plane that
1 + 4 + 36 1 + 9 + 25 41 35 41
contains the points (2, 0, -3), (-4, -5, 2), and (0, 3, -4) in
 the form ax+by+cz = d.
35 −1 BA
A cos
= , cosB  
41 BA BC 
Sol: Let v = ( −4, −5,2 ) − ( 2,0, −3 ) = ( −6, −5,5 ) and

( )(
ˆi + 2ˆj + 6kˆ . 2iˆ − ˆj + kˆ ) ω = ( 0,3, −4 ) − ( 2,0, −3) = ( −2,3, −1 ) .
   
v ×=ω i (5 − 15 ) − j ( 6 + 10 ) +
12 + 22 + 62 22 + ( −1 ) + (1 )
2 2

k̂ ( −18 − 10 ) =−
( 10, −16, −28 )
2−2+6 6 6 We can choose n̂ to be any vector in the same direction
⇒ cosB
= = B cos−1
⇒=  
41 6 41 41 as v × ω so let n̂ = (5, 8, 14). Then the plane has the
M a them a ti cs | 26.31

form 5x + 8y + 14z = d. Substituting the point (2,0,-3) 1


2 (3+2t) + 1 (-3 + t) - (-3 -t) = 6 + 6t = 3 ⇒ t = - .
for (x,y,z) and solving for d gives d=10+0+0(-42)=-32. 2
So the plane has the equation 5x + 8y + 14z = −32 . So the point in the plane closest to (3,-3,-3) is
 1  7 5
Example 5: Find a vector that is perpendicular to the c  −  =  2, − , −  .
 2   2 2
vector (1, 2, 3) with the same length. Also, find a plane
perpendicular to (1, 2, 3) that passes through the point The distance between the point and the plane is thus
(3, 2, 1). 2 2
2 1  1 3
1 +   + −  = .
Sol: By using formula of perpendicular vector we can 2  2 2
obtain the result.
 
A vector v is perpendicular to (1, 2, 3) if v .(1,2,3)=0. 
There are infinite number of possibilities to choose Example 8: Determine if the three vectors = a (1, 4, −7 )
  
from, but one possible choice for v is (2,-1, 0). However, b= ( 2, −1, 4 ) and c =
( 0, −9,18 ) lie in the same plane
we want this vector to have length (1,2,3) = 14 . or not.
Since ( 2, −1,0 ) =
5 ,we need to rescale our vector to Sol: Three vectors lies in the same plane if volume of
the parallelepiped formed by these three vectors is
14
be ( 2, −1,0 ) . zero.
5
If (1, 2, 3) is a normal vector to a plane then the plane So, as we noted prior to this example all we need to do
will have the form x+2y+3z=d. since the plane passes is compute the volume of the parallelepiped formed
through the point (3, 2, 1), we substitute these values for by these three vectors. If the volume is zero, then they
x, y, and z to get 3+4+3=10=d so our plane equation is lie in the same plane and if the volume isn’t zero they
x + 2y + 3x = 10 . don’t lie in the same plane.

   1 4 −7
Example 6: Write an equation for the plane that
contains the point (1, 0, 3) and the line (-3,-2,-2) +
(
a⋅ b X c =)
2 −1 4 =−18 + 126 − 144 + 36 = 0
0 −9 18
t(1, 2,-1) in the form ax+by+cz=d.
=0
So, the volume is zero and so they lie in the same plane.
Sol: Since the plane contains the line (-3,-2,-2)+
t(1,2,-1) we know that one tangent vector to the plane is

v =(1,2,-1). We can get a second tangent vector by
finding the vector between (-3,-2,-2) and (1, 0, 3).
JEE Advanced/Boards

So let ω =(4,2,5). Then Example 1: If O be the circumcenter; G, the centroid
    
and H, the orthocenter of triangle ABC, prove that O, G,
ω i(10 + 2) − j(5 + 4) + k(2 − 8) =(12,-9,-6)
v ×=
 H are collinear and G divides OH in the ratio 1:2
So we can choose n = ( 4, −3, −2 ) and our plane has
the form 4x-3y-2z=d. Plugging in (1, 0, 3) for (x,y,z) and   
Sol: Consider position vector of A, B, C be taken as a, b, c .
solving for d yields 4x ‒ 3y ‒ 2z = ‒2
And then use geometry of triangle to solve this
problem.
Example 7: Find the minimum distance between the
point (3,-3,-3) and the plane 2x+y-z=3. Let O, the circumcenter of the ∆ ABC be chosen as
  
origin and position vector of A, B, C be taken a, b, c .
Sol: The point in the plane closest to (3,-3,-3) lies on
a line that is perpendicular to the plane and passes Hence position vector of G the centroid is
  
through (3,-3,-3). Since (2, 1,-1) is a normal vector to  a + b + c
the plane, we will use it as the direction of this line. Thus OG =  ..........(i) … (i)
3
a parameterized form of the line is
Since O is circumcenter
c (t) = (3, −3, −3) + t ( 2,1, −1) = (3 + 2t, −3 + t, , −3 − t )      
2 2
∴ OA = OB = OC= OA= OB= OC2 or a=
2 2
b= c2
We substitute this into the plane equation to find its
intersection with the plane and get: a2 − b=
2
0, b2 − c=
2
0, c2 − a=
2
0
2 6 . 32 | Vectors

    2 2
 2  2 1   1  
Or (a + b).(a − c) =
     
0 ⇒ BE = CF ⇒
2
c − 2b ( ) = b − 2c
2
( )
( )(
Or a + b + c − c . a − b =0  ) … (ii)
1  2 1  2    
( ) = (b − 2c)
2 2
Let P be the point whose position vector is ⇒ c − 2b = b − 2c ⇒ c − 2b
A 4 4
       
( )(
⇒ c − 2b  c − 2b =− ) (
b 2c  b − 2c )( )
M
F 2   2 2   2
H ⇒ c − 4b.c + 4 b =b − 4b.c + 4 c
E
2 2 2 2
⇒3b =3c ⇒ b =c
O G
C
B D ⇒ AB=AC
       Hence, triangle ABC is an isosceles triangle.
(
a + b + c ∴ OP − OC . OA − OB = 0 )( )   
 
Or CP ⊥ BA Example 3: D, E, F are points dividing side BC, CA, AB
of a triangle ABC in the ratio 2:3, 1:2 and 3:1 respectively.
  
In similar manner we can show that BP is perpendicular Show that the lines AD, BE, CF are concurrent and hence
to AC and AP is perpendicular to CB. find the position vector of their point of intersection.
Hence P is the orthocentre which is H.
Sol: By using section formula we can obtain required
     
OP = OH = a + b + c = 3OG  ... (iii) result.
   
    OG 1 If d, e, f are position vector of points D, E & F
∴ OH = 3OG or GH = 2.OG or  = respectively then, by section formula
GH 2
 
Above show that O, G, H are collinear and G divides OH  2c + 3b
in the ratio 1:2 d=
5  …. (i)
 
 2c + a
Example 2: Prove using vectors: If two medians of a e=  ….(ii)
3
triangle are equal, then it is isosceles.  
 3b + a
f=  …(iii)
Sol: Using mid – point formula of vector, we can solve this 4

Let ABC be a triangle and let BE and CF be two equal A(a)
medians. Taking A as the origin, let the position vectors 3
  2
of B and C be b and c respectively. Then
F P E
1
Position vector of E = c and 1 1
2
1  

C( c )
Position vector of F = b B(b) 2 D 3
2
   
( )
A(origin)
Equation of line AD is r =+a t d−a
   
( )
 
Equation of line BE is r =+
b m e−b
(( b
2
F E ((
c
2  
For intersection of AD and BE we need that
     

B(b)
D

C( c ) ( ) ( )
“ a + t d − a =b + m e − b ” be true for some 0<t, m<1.
 1    1        
∴ BE =
2
(
c − 2b ,CF =
2
) b − 2c ( ) 
a+t
( 2c + 3b − 5a) 
= b+m
( 2c + a − 3b )
  5 3
Now, BE= CF= BE = CF
M a them a ti cs | 26.33

m 3t 2t 2m 5 5
∴ 1−t = ; = 1−m ; = Then the area of the triangle with sides (3, 2) and  , 
3 5 5 3 2 2
5 1 is one half the area of the parallelogram with sides (3, 2)
t
∴= ,m
=
6 2 5 5
and  ,  . So, the area of the triangle is
The existence of t and m assures the intersection of 2 2
 
AD and BE . 1 5 5  1  5 5
(3,2,0)X=
 , ,0   0,0, 
= .
2 2 2  2  2 4
The point of intersection is
  
  5   (
a + 3b + 2c ) Example 6: Find the minimum distance between the
6
(
r =a + d − a = ) 6
point (4, 2,-3) and the line (1, 0, 2) + t (-1,-1, 2).

Example 4: Find a parametric form for the line passing Sol: Let v(t) represents the vector from the point
through the point (1,2) in the direction (3,4), which we (4,2,-3) and line (1,0,2) + t(-1,-1,2) = (1-t,-t,2+2t) at any
will call c1(t). Set c1(t) equal to (x,y) and eliminate t to t ∈ .

get the line into y = mx + b form. Now find a different So, v(t) = (4, 2,-3)-(1-t,-t, 2+2t) = (3+t, 2+t,-5-2t). We
parametrization c2(t) of the same line such that 
c2(0) =(-2,-2) and c2(2)= (-5,-6). want to find the t such that v(t) is perpendicular to the

line, which is when v(t) . (-1,-1, 2) = 0.
Sol: c1(t) = (1,2) + t (3, 4) = (1 + 3t, 2 + 4t). Setting (x, y)
= (1 + 3t, 2 + 4t) yields x = 1 + 3t and y = 2 + 4t. (3 + t, 2 + t, -5 - 2t) . (-1, -1, 2) = -15 - 6t = 0
Solving the former equation for t yields t = (x-1)/3. 5  5
⇒ t = − . So the length of v  −  should represent
Substituting this into the second equation then gives 2  2
4 2 the minimum distance from (4, 2,-3) and the line.
us = y x+ .
3 3 
 1
v ( −5 / 2 ) = (1 / 2, −1 / 2,0) = .
Let c2 (t)= p + t v . c2 will then be a parameterization of 2
the same
 line given by c1 if p is a point on the same line
and v is in the same direction as (3,4) (i.e. some scalar Example 7: Prove that, in any triangle ABC
multiple of (3,4)). Since c2(0) = (-2,-2) we will choose
(i) c2 = a2 + b2 − 2accosC=
(ii) c bcos A + acosB
p= (-2,-2) (you can check that this point indeed lies on
the line parameterized by c1). Then
 Sol: By using simple scalar product method we can
c2 (2) =( −2, −2) + 2v =( −5, −6) , so we get that prove given relation.
   
v= ( −3 / 2, −2) , which is indeed a scalar multiple (i) In ∆ ABC, AB + BC + CA =0
of (3,4). So   
 −3  or, BC + CA = −AB ….(i)
c2 (t) =( −2, −2) + t  , −2  is a different
 2  Squaring both sides
parameterization of the line parameterized by c1.  2  2    2
( ) ( ) ( )( ) ( )
BC + CA + 2 BC . CA = AB

  -c C
Example 5: Find the vector projection of (3, 2) onto
(-1,-1). Then find the area of the triangle with one
2 2
( )
⇒ a + b + 2 BC.CA = c 2

side vector (3, 2) and another side the result of this ⇒ c = a + b = 2abcos ( π − C )
2 2 2

projection. A B
⇒ c2 = a2 + b2 − 2ab cosC
Sol: Use projection method to obtain vector projection         
of (3, 2) and area of triangle will be half of the area of
parallelogram.
( )
(ii) BC + CA. .AB = −AB.AB ⇒ BC.AB + CA.AB

=−c2 − accosB − bccos A


−5 5 5
proj( −1,−1) (3,2)= ( −1, −1)=  ,  .
2 2 2 ⇒ acosB + bcos A =
c
2 6 . 34 | Vectors

Example 8: In any triangle, show that the perpendicular  1  


bisectors of the sides are concurrent.
∴ OD = (
2
b+c , )
 1  
Sol: By using formula of Dot product and Mid – point
we can solve this problem.
OE
=
2
( )c+a ,
 1  
Let ABC be the triangle and D, E and F are respectively
  
and = OF ( 2
a+b )
middle points of sides BC CA and AB . Let the Also
         
perpendicular through D and E meet of O join OF . We BC = c − b, CA =a − c and AB = b−a
 
are required to prove that OF is ⊥ to AB . Let the      
Since, OD ⊥ BC, 1 / 2(b + c).(c − b) =
0
position vectors of A, B, C with O as origin of reference
   ⇒ b2 = c2  … (i)
be a, b and c respectively.     
A
Similarly, OE ⊥ CA,1 / 2(a + c).(a − c) =
0

⇒ a2 =
c2  … (ii)

F
a
From (i) and (ii) we have b2 − a2 =
0
E
       

b
O 
c
( )(
a+b . b+a = ) (
0 ⇒1/2 b+a . b−a =)(
0 )
 
B C ⇒ OF ⊥ AB
D
Hence proved.

JEE Main/Boards


JEE Main/Boards Q.3 The points A and B have position vectors a and
 
b relative to an origin O, where a = 4iˆ + 3ˆj − 2kˆ and
Exercise 1 
b= −7iˆ + 5ˆj + 4kˆ

Q.1 The line L1 passes through the points (2,-3, 1) and (i) Find the length of AB.
(-1,-2,-4). The line L2 passes through the point (3, 2,-9) (ii) Use a scalar product to find angle OAB.
and is parallel to the vector 4iˆ − 4ˆj + 5kˆ .
   Q.4 The position vectors of the points P and Q with
(i) Find an equation for L1 in the form r= a + tb
respect to an origin O are 5iˆ + 2ˆj − 9kˆ and 4iˆ + 4ˆj − 6kˆ
(ii) Prove that L1 and L2 are skew. respectively.
(i) Find the vector equation for the line PQ
Q.2 Two lines have vector equations
The position vector of the point T is ˆi + 2ˆj − kˆ
 4   −8   −2   −9 
          (ii) Write down a vector equation for the line OT and
r= 2  + t  1  and r =
 a  + s 2  , show that OT is perpendicular to PQ.
 −6   −2   −2   −5 
       
It is given that OT intersects PQ.
Where 'a' is a constant. (iii) Find the position vector of the point of intersection
(i) Calculate the acute angle between the lines. of OT and PQ.
(ii) Given that these two lines intersect, find the point (iv) Hence find the perpendicular distance from O to
of intersection. PQ, giving your answer in an exact form.
M a them a ti cs | 26.35

 
Q.5 ABCD is a parallelogram. The position vectors of A, Q.11 If r and s are non zero constant vectors and the
B and C are given respectively by  
   scalar b is chosen such that r + bs is minimum, then
a = 2iˆ + ˆj + 3kˆ , b= 3iˆ − 2ˆj , c = ˆi − ˆj − 2kˆ
 2  
(i) Find the position vector of D. show that the value of bs + r + bs is equal to | r |2 .

(ii) Determine, to the nearest degree, the angle ABC.


Exercise 2
Q.6 The position vectors of three points A, B and
C relative to an origin O are given respectively by Single Correct Choice Type
  
OA = 7iˆ + 3ˆj − 3kˆ , OB = 4iˆ + 2ˆj − 4kˆ , OC = 5iˆ + 4ˆj − 5kˆ .     
Q.1 Let a , b , c be three vectors such that a= c= 1;
(i) Find the angle between AB and AC.      
b = 4 and b × c =15 . If b − 2c = λa then a value of
(ii) Find the area of triangle ABC.
λ is
Q.7 Two lines have vector equations (A) 1 (B) -1 (C) 2 (D) -4

( )
r = ˆi − 2ˆj + 4kˆ + λ 3ˆi + ˆj + akˆ and

 Q.2 Vector r which is equally inclined to coordinate
r = −8iˆ + 2ˆj + 3kˆ + µ(iˆ − 2ˆj − k)
ˆ ,

Where 'a' is a constant axes such that r = 15 3 is

(i) Given that the lines are skew, find the value that a (A) i + j + k (
(B) 15 i + j + k )
cannot take.
(ii) Given instead that the lines intersect, find the point
( )
(C) 7 i + j + k (D) None of these

of intersection.   
Q.3 For 3 vectors u, v, w , which of the following
Q.8 Lines, L1, L2 and L3 have vector equations expressions is ≠ to any remaining three.
      
L1 : r = (5iˆ − ˆj − 2k)
ˆ + s( −6iˆ + 8ˆj − 2k)
ˆ ,

( )
(A) u. v × w ( )
(B) v × w .u
L2 : r = (3iˆ − 8ˆj) + t(iˆ + 3ˆj + 2k)
ˆ      
 (C) v. (u × w ) (D) ( w × u) .v
L3 : r = (2iˆ + ˆj + 3k)
ˆ + u(3iˆ + cjˆ + k)
ˆ .

(i) Calculate the acute angle between L1 and L2.       


Q.4 If a + b + c =0, =
a 3,=
b 5 &=
c 7 then ∠θ
(ii) Given that L1 and L3 are parallel, find the value of c.  
between a and b is
(iii) Given that L2 and L3 intersect, find the value of c
(A) 40° (B) 30° (C) 150° (D) None of these

Q.9 Given that u =ˆi − 2ˆj + 3k; ˆ   
  ˆ ˆ Q.5 If 2 out of 3 vectors a , b , c are unit vectors,
v = 2iˆ + ˆj + 4k;
ˆ w =i + 3 j + 3kˆ and         
    
(u ⋅R − 10) ˆi + (vR − 20) ˆj + (w ⋅ R − 20)kˆ =0.
a+b+c= ( )
0 and 2 a.b + b.c + c.a + 3 = 0, then third
 vector is length-
Find the unknown vector R.
(A) 3 (B) 1 (C) 2 (D) None of these
  
Q.10 The base vectors a1 , a2 , a3 are given in terms of     
   Q.6 Let a + b is orthogonal to b and a + 2b is
base vectors b1 , b2 , b3 as, 
    orthogonal to a , then
a1 = 2b1 + 3b2 − b3 ;    
        (A) a = 2 b (B) a = 2 b
a2 =b1 − 2b2 + 2b3 & a3 = −2b1 + b2 − 2b3 .
          
If F = 3b1 − b2 + 2b3 . then express F in terms of a1 , a2 , (C) a = b (D) 2 a = b

a3 .
2 6 . 36 | Vectors

  
Q.7 Magnitude of projection of vector ˆi + 2ˆj + kˆ on Q.4 If a,b,c are non-coplanar unit vectors such that
   
  
vector 4iˆ + 4ˆj + 7kˆ is a × (b × c) =
(b + c) , then the angle between a and b
2
(A) 3 (B) 3 6 (C) 6 /3 (D) None of these is (1995)
3π π π
(A) (B) (C) (D) π
Q.8 Magnitude of moment of force −2i + 6j − 8k acting 4 4 2
at point 2i − j + 3k about point i + 2j − k    
Q.5 If a and b are two unit vectors such that  a + 2b
 
(A) 211 (B) 0 (C) 54 (D) None of these and 5   a − 4b are perpendicular to each other, then the
 
angle between a and b is (2002)

Q.9 If a & b are unit vectors represented by O A and 1 2
 (A) 45° (C) cos−1   (D) cos−1  
(B) 60°
OB, then unit vector along bisector of ∠AOB is scalar 3 7
multiple of        
Q.6 Let V = 2i + j − k and W = i + 3k . If U is a unit
(A) a − b (B) a × b (C) b × a (D) None of these vector, then the maximum value of the scalar triple
  
          product [ U V W ] is  (2002)
Q.10 If 2a + 4b c d = λ a c d + µ b c d
     
(A) -1 (B) 10 + 6 (C) 59 (D) 60
then λ + µ =
(A) 6 (B) -6 (C) 10 (D) None of these Q.7 The unit vector which is orthogonal to the vector
3iˆ + 2ˆj + 6kˆ and is coplanar with the vectors 2iˆ + ˆj + kˆ
Previous Years’ Questions and ˆi − ˆj + kˆ is  (2004)
 
2iˆ − 6ˆj + kˆ 2i − 3 j
Q.1 The volume of the parallelepiped whose sides are (A) (B)
   41 13
given by OA= 2i − 3j , OB = i + j − k ,OC= 3i − k , is
 (1983) 3ˆj − kˆ 4iˆ + 3ˆj − 3kˆ
(C) (D)
4 2 10 34
(A) (B) 4 (C) (D) None of these
13 7
Q.8 Two adjacent sides of a parallelogram ABCD are
  
Q.2 A vector a has components 2p and 1 with respect given by AB =2iˆ + 10ˆj + 11kˆ and AD =−ˆi + 2ˆj + 2kˆ . The
to a rectangular Cartesian system. This system is rotated side AD is rotated by an acute angle α in the plane
through a certain angle about the origin in the counter  of the parallelogram so that AD becomes AD’. If AD’
clockwise sense. If, with respect to the new system, a makes a right angle with the side AB, then the cosine of
has components p+1 and l, then (1986) the angle α is given by (2010)
1 8 17 1 4 5
(A) p = 0 (B) p=1 or p= − (A) (B) (C) (D)
3 9 9 9 9
1
(C) p = -1 or p = (D) p = 1 or p = -1 
3  
ˆ b = ˆi − ˆj + kˆ and c = ˆi − ˆj − kˆ be
Q.9 Let a = ˆi + ˆj + k,
  
Q.3 Let a ,b, c be distinct non-negative numbers. If the three vectors. A vector v in the plane of a and b ,
 1
ˆ i + kˆ and ci + cjˆ + bkˆ lie in a plane,
vectors ai + ajˆ + ck, whose projection on c is , is given by (2011)
then c is  (1993) 3
  
(A) The Arithmetic Mean of a and b. Q.10 Let a,b and c be three unit vectors such that
   3    
(B) The Geometric Mean of a and b.
( )
a× b× c −
2 
( )
b + c . If b is not parallel to c then the

(C) The Harmonic Mean of a and b.
angle between a and b is:  (2016)
(D) Equal to zero.
π 2π 5π 3π
(A) (B) (C) (D)
2 3 6 4
M a them a ti cs | 26.37

    
Q.11 Let a. b and c be three non-zero vectors such that Q.15 If the vectors a = i − j + 2ˆ + 4ˆ j + kˆ and c = λˆi + ˆj + µkˆ
   1   are mutually orthogonal then ( λ , µ ) =
( )
no two of them are collinear and a. × b × c − b c a If
3
(2010)
  (A) (2, ‒3) (B) (‒2, 3) (C) (3, ‒2) (D) (‒3, 2)
θ is the angle between vectors b and c then a value
of sinθ is  (2015)  
Q.16 Let a= ˆj − kˆ . Then vector b satisfying
    
− 2 2 2 3 2 3 a×b + c =0 and a.b is (2010)
(A) (B) (C) − (D)
3 3 3 3
(A) 2iˆ − ˆj + 2kˆ (B) ˆi − ˆj + 2kˆ
     2
Q.12 If a × bb × c × a =λ abc  then λ is equal to
    (C) ˆi + ˆj − 2kˆ (D) −ˆi + ˆj − 2kˆ
 (2014)
 1  1 ˆ ˆ
(A) 1 (B) 3 (C) 0 (D) 1 Q.17 If a=
10
( 3iˆ + kˆ ) and b=
7
(2i + 3 j − 6kˆ ) , then
       
Q.13 If the vectors AB= 3j + 4k and AC = 2j + 4k
are the sides of a triangle ABC, then the length of the
( ) ( ) (
the value of 2a − b .  a × b × a + 2b  is
  ) (2011)

median through A is  (2013)


(A) -3 (B) 5 (C) 3 (D) -5
(A) 72 (B) 33 (C) 45 (D) 18  
Q.18 The vector a and b are not perpendicular and
     
Q.14 Let a and b be two unit vectors. If the vectors a and b are two vectors satisfying: b × c = b × d and
   
c= a + 2b and d = 5a − 4b are perpendicular to each a.d = 0 . Then the vector is equal to (2011)
 
other then the angle between a and b is  (2012)   a.c     b.c  
(A) c +     b (B) b +     c
π π π π  a.b 
(A) (B) (C) (D)  a.b 
6 2 3 4  
  a.c     b.c  
(C) c −     b (D) b +     c
 a.b   a.b 

JEE Advanced/Boards

   
Exercise 1 Q.5 What will be the angle between a and b if a & b
       
(a × b) + (a  b)
2 2
are unit vectors such that a + 3b is ⊥ to 7a − 5b ?
Q.1 What will be the value of 2  2 ?
2a b ˆ and Bˆ are inclined at π then
Q. 6 If the unit vectors A

ˆ − Bˆ /2 ?
what will be the value of A
Q.2 What will be the area of the triangle determined by
the vectors 3i+4j and -5i+7j?
Q.7 A particle acted upon by forces 3i + 2ˆj + 5k and
Q.3 What will be the value of a if points whose position 2i + ˆj + 3k is displaced from a point P to a point Q

vectors are 60i + 3j, 40i − 8j, ai − 52 j are collinear? whose respective position vectors are 2i + j + 3k and
4i + 3ˆj + 7k . What will be the work done by the force?
Q.4 What will be the angle between diagonals which
  Q.8 A force F = 6i + λj + 4k acting on a particle displaces
adjacent sides of llgm are along a = i + 2j & b= 2i + j ? it from A (3,4,5) to B(1,1,1). If the work done is 2 units,
then What will be the value of λ ?
2 6 . 38 | Vectors

Q.9 What will be the length of longer diagonal Q.6 If P and Q be two given points on the curve y=x+1/x
    such that OP.I = 1 and OQ.I = -1 where I is a unit vector
of llgm constructed on 5a + 2b & a − 3b . Given
    along the x-axis, then the length of vector 2OP +3OQ is
= b 3= & a 2 2 and angle between a & b is π / 4
(A) 5 5 (B) 3 5 (C) 2 5 (D) 5
Q.10 The vectors i + xjˆ + 3k is rotated through an
angle θ and doubled in magnitude, then it becomes Q.7 Let A, B, C be three vectors such that A (B + C) + B.
C = 0 and lAl = 1, lBl = 4, lCl =8, then lA+B+Cl equals
( )
4i + 4x − 2 j + 2k . Find x ?
(A) 13 (B) 81 (C) 9 (D) 5

Exercise 2 ˆ and Bˆ are inclined at an angle


Q.8 If the unit vectors A

Single Correct Choice Type ˆ − Bˆ ≤ 1 , then for θ ∈ 0, π  , θ , may lie in the
2θ and A  
interval
Q.1 Moment of couple formed by forces 5i + j & − 5i + j
acting at [9,-1, 2] and [3,-2, 1]
(A)  π / 6, π / 3 (B)  π / 6, π / 2 
(A) −i + 5j + k (B) i − j − 5k
(C) 5π / 6, π  (D)  π / 2,5π / 6 
(C) 2i − 2j − 10k (D) −2i − 2 j + 10k
Q.9 If unit vectors ˆ and Bˆ
A such that STP
   ˆ Bˆ A
ˆ × Bˆ = ˆ and Bˆ are inclined
Q.2 Let a, b, c be vectors such that A 1 / 4 then A
        
( ) ( ) (
a. b + c + b. c + a + c. a + b =0 and ) (A) π / 6 (B) π / 2 (C) π / 3 (D) π / 4
  
a 1,=
= b 4, = c 8 then a + b + c equals ˆ and Bˆ unit vectors then greatest value of
Q.10 If A

(A) 13 (B) 81 (C) 9 (D) None of these ˆ − Bˆ + A


A ˆ − Bˆ is

(A) 2 (B) 4 (C) 2 2 (D) 2


Q.3 Position vectors of A and B are 2i + 2 j + k
and 2 i + 4 j + 4 k . Length of internal bisector of
∠BOA of ∆AOB is Previous Years’ Questions
 
136 136 20 Q.1 (i) If C be a given non zero scalar and A and B be
(A) (B) (C) (D) None of these  
9 9 3 given non-zero vectors such that A ⊥ B , find the vector
     
         X which satisfies the equation= A  X c and = A×X B .
Q.4 If a + 2b + 3c =
0 , then a xb + b x c + c x a is equal to 
(ii) A vector A has components A1 , A2 , A3 in a right-
   
( )
(A) 6 b × c (B) 6 a × b ( ) handed rectangular Cartesian coordinate system oxyz.
  The coordinate system is rotated about the x-axis
(C) 6 ( c × a) (D) None of these π
through an angle . Find the components of A in the
2
        new coordinate system, in terms of A1 , A2 , A3 . (1983)
Q.5 Value of a − b, b − c, c − a where=
a 1,=
b 2, and
   
c = 3 is Q.2 If vectors a, b, c are coplanar, show that
  
(A) 1 (B) -6 (C) 0 (D) None of these a b c
      
a⋅a a⋅b a⋅c = 0 (1989)
     
b ⋅a b ⋅b b ⋅c
M a them a ti cs | 26.39

   
Q.3 Let A = 2i + k , B = i + j + k , and C = 4i − 3j + 7k . (A) There is exactly one choice for such v
     
Determine a vector R satisfying R × B = C × B and (B) There are infinitely many choices for such v
  (C) If u lies in the xy-plane then u = u
R⋅A = 0.  (1990) 1 2

(D) If u lies in the xz-plane then 2 u1 = u3


   
Q.4 If a, b, c, d are four distinct vectors satisfying the    
        Q.11 Let ∆PQR be a triangle. Let = a QR,
= b RP and
conditions a × b = c × d and a × c = b × d then prove      
        | a | 12,= | b | 4 3 and b ⋅ c 24, then
that a ⋅ b + c ⋅ d ≠ a ⋅ c + b ⋅ d.  (2004) c = PQ . If = =
which of the following is (are) true (2015)

Q.5 Incident ray is along the unit vector v and the reflected | c |2 
(A) − |a| = 12
ray is along the unit vector . The normal is along unit vector 2
 in terms of a and v . 
a outwards. Express w (2005) | c |2 
(B) − |a| = 30
 2
Q.6 Let A be vector parallel to line of intersection of    
(C) | a × b + c × a | = 48 3
planes P1 and P2 through origin. P1 is parallel to the
 
vectors 2j + 3k and 4j − 3k and P2 is parallel to j − k (D) a ⋅ b =− 72
and 3 i + 3 j , then the angle between vector A and
2iˆ + ˆj − 2kˆ is (2006)   
Q.12 Let a = −i − k,b
 = −i + j and c =+
i 2j + 3k be three
π π π 3π     
(A) (B) (C) (D) given vectors. If r is a vector such that r × b = c × b and
2 4 4 4    
r. a = 0 then the value of r. b is (2011)
Q.7 The vector(s) which is /are coplanar with vectors  
i + j + 2k and i + 2j + k , are perpendicular to the vector Q.13 If a and b are vectors in space given by
i + j + k is/are  i − 2j   2i + j + 3k
(2011) a= b and b = then the value of
5 14
(A) j − k (B) −i + j (C) i − j (D) −j + k            
 
(2 a + b) . (a × b ) × (a − 2 b ) (2 a + b ) . (a × b ) × (a − 2 b )
Q.8 If a and b are vectors in space by is  (2010)
 i − 2j  2i + j + 3k
a= and b = , then the value of   
5 14 Q.14 Let a, b and c be three non-coplanar unit vectors
      such that the angle between every pair of them is
( ) ( ) (
2a + b ⋅ 
  )
a × b × a − 2b  is (2010) π       
If a × b + b × c = pa + qb + rc where p, q and r are
3
 
 

i 2 j + 3k p2 + 2q2 + r 2
Q.9 Let a =− i − k , b =− i + j , and c =+ be scalars, then the value of is  (2014)
     q2
three given vectors. If r is a vector such that r × b = c × b,
     
r ⋅a =0 , then the value of r ⋅ b is…… (2011) Q.15 Let PR = 3i + j − 2k and SQ = i − 3j − 4k determine

diagonals of a parallelogram PQRS and PT =+ i 2j − 3k
Q.10 Let u = u1 i + u2 j + u3 k be a unit vector in R2 and be another vector. Then the volume of the parallelepiped
1 ˆ ˆ    
=ŵ
6
( )
i + j + 2kˆ . Given that there exists a vector v determined by the vectors PT,PQ and PS is  (2013)
  (A) 5 (B) 20 (C) 10 (D) 30

in R such that u × v =
2 1 and w ˆ = (uˆ × v ) = 1 . Which of

the following statements (s)is (are) correct?  (2016)


2 6 . 40 | Vectors

Q.16 A line I passing through the origin is perpendicular (C) i → r, ii → s, iii → p, iv → q


to the lines
(D) i → p, ii → s, iii → r, iv → q
( ) ( ) ( )
 : 3 + t i + −1 + 2t j + 4 + 2t k − ∞ < t < ∞
1
  
 2 : ( 3 + 2x ) j + ( 3 + 2s ) j + ( 2 + s ) k,
   −∞ < s < ∞
Q.19 If a, b and c are unit vectors satisfying
 2  2  2   
Then, the coordinate (s) of the point(s) on  2 at a
a, −b + b − c + c−a = 9 then 2a + 5b + 5c is (2012)
distance of 17 from the point of intersection of  and
 
 2 is (are)  (2013) Q.20 If a and b are vectors such that
   
7 7 5
(A)  , ,  (B) ( −1, −1,0 )
 3 3 3 
( ) (
a + b = 29 and a × 2i + 3j + 4k = 2i + 3j + 4k × b
 
)
(C) (1,1,1 )
7 7 8
(D)  , , 
( )(
 
then a possible value of a + b . −7i + 2 j + 3k

) is
9 9 9  (2012)
(A) 0 (B) 3 (C) 4 (D) 8
Q.17 Consider the set of eight vectors
V= {  
}
ai + b j + ck : a, b, c ∈ {−1, 1} . Three non-coplanar
   
 = i − j + k and c = i − j + k be
Q.21 Let a = i + j + k,b
  
vectors can be chosen from V in 2n ways. Then p is (2013) three vectors. A vector v in the plane of a and b whose
 1
projection on c is , is given by (2011)
Q.18 Match list I with list II and select the correct answer 3
using the code given below the lists :  (2013)
(A) ˆi + 3ˆj + 3kˆ (B) −3iˆ − 3ˆj + kˆ

List I List II (C) 3iˆ − ˆj + 3kˆ (D) ˆi − 3ˆj − 3kˆ

(i) Volume of parallelepiped by vectors (p) 100


   Q.22 The vector(s) which is/are coplanar with vectors
a, b and c is 2. Then the volume of the i + j + 2 k and i + 2 j + k , and perpendicular to the
parallelepiped determined by vectors
      vector i + j + k is/are to 
( ) ( )
2 a × b ,3 b × c and c × a is( )      
(2011)
 
(ii) Volume of parallelepiped determined (q) 30 (A) j − k (B) − i + j (C) i − j (D) − j + k
  
by vectors a , b and c is 5. Then the
volume of the parallelepiped determined
     
( )( )
by vectors 3 a + b , b _ + c and 2 c + a ( )
is
(iii) Area of a triangle with  adjacent (r) 24
sides determined by vectors a and b
is 20. Then the area of the triangle with
adjacent sides determined by vectors
   
( )
2a + 3b and a − b is( )
(iv) Area of a parallelogram with adjacent (s) 60
 
sides determined by vectors a and b is
30. Then the area of the parallelogram
with adjacent sides determined by
  
( )
vectors a + b and a is

(A) i → s, ii → q, iii → r, iv → p
(B) i → q, ii → r, iii → p, iv → s
M a them a ti cs | 26.41

PlancEssential Questions
JEE Main/Boards JEE Advanced/Boards
Exercise 1 Exercise 1
Q.2 Q.4 Q.7 Q.8 Q.2 Q.9 Q.12 Q.15
Q.10
Exercise 2
Exercise 2 Q.1 Q.3
Q.3 Q.8 Q.10
Previous Years’ Questions
Previous Years’ Questions Q.1 Q.5 Q.6 Q.8
Q.2 Q.8 Q.9 Q.10

Answer Key

JEE Main/Boards
Exercise 1

Q.1 (i) r = (2iˆ − 3ˆj + kˆ  or ) ( )


– ˆi − 2ˆj − 4kˆ + t 3iˆ – ˆj + 5kˆ

Q.2 (i) 15° (15.38.....),0.268 rad (ii) a = 1 and intersection is (−20, 5, −12)

Q.3 (i) 161 (ii) 43°

( )
Q.4 (i) r= (either point) + t ˆi − 2ˆj − 3kˆ or – ˆi + 2ˆj + 3kˆ , (ii) s (ˆi + 2 ˆj − k)
ˆ (iii) 3iˆ + 6ˆj − 3kˆ (iv) 54

Q.5 (i) 2ˆj + kˆ (ii) 86° Q.6 (i) 45.3° (ii) 3.54 Q.7 (i) A cannot be 2. (ii) −5iˆ − 4ˆj
   
Q.8 (i) 68.5° (ii) c=-4 (iii) c=-3 Q.9 −ˆi + 2ˆj + 5kˆ Q.10 F = 2a1 + 5a2 + 3a3

Exercise 2
Single Correct Choice Type

Q.1 D Q.2 B Q.3 C Q.4 B Q.5 B Q.6 A

Q.7 D Q.8 B Q.9 A Q.10 A


2 6 . 42 | Vectors

Previous Years’ Questions


Q.1 B Q.2 B Q.3 B Q.4 A Q.5 B Q.6 C

Q.7 C Q.8 A Q.9 3iˆ − ˆj + 3kˆ Q.10 C Q.11 D Q.12 A

Q.13 B Q.14 C Q.15 D Q.16 D Q.17 D Q.18 C

JEE Advanced/Boards
Exercise 1

1 41 π
Q.1
= Q.2 Q.3 ‒40 Q.4 90° and 90° Q.5 Q.6 1
2 2 3
−2
Q.7 48 units Q.8 ‒10 Q.9 593 Q.10 2 or
3

Exercise 2
Single Correct Choice Type

Q.1 A Q.2 C Q.3 A Q.4 A Q.5 C Q.6 D


Q.7 C Q.8 C Q.9 C Q.10 C

Previous Years’ Questions

   
  c    1   
2
 A 
2
 A 
( )
Q.1 (i) X =   A −    A × B (ii) (A2 i − A1 j + A3 k)
 Q.3 −i − 8j + 2k

   

( )
 =v − 2 aˆ ⋅ vˆ a
Q.5 w Q.6 B, D Q.7 A, D Q.8 5 Q.9 9

Q.10 B, C Q.11 A, B, C Q.12 9 Q.13 5 Q.14 4 Q.15 C

Q.16 B, D Q.17 5 Q.18 C Q.19 3 Q.20 C Q.21 C

Q.22 A, D
M a them a ti cs | 26.43

Solutions

JEE Main/Boards Scalar product of any two vector


cos θ =
Product of their moduli
Exercise 1
( 4 ˆi + 3 ˆj − 2kˆ ) . (11 ˆi − 2 ˆj − 6kˆ )
Sol 1: (i) For (either point) + t(diff b/w vectors)
42 + 32 + 22 112 + 22 + 62
r= (2i − 3 j + k or − i − 2 j − 4k) + t(3i − j + 5k)
(ii) L(2)(r) = 3i + 2 j − 9k + s(4i − 4 j + 5k) 44 − 6 + 12 50
= = = 43°
L(1) and L(2) must be of form r = a +tb 29 161 29 161

2 + 3t =3 + 4s, −3 − t =2 − 4s,1 + 5t =−9 + 5s


Sol 4: (i) For (either point) + t(diff between position
(t,s) =( + / −3,2) or ( − / +1,1) or ( − / +9, −7) vectors)
Or (+/-4,2) or (0, 1) or (-/+8, -7) (ii) r = s (i+2j-k) or (i + 2j – k) + s (i + 2j – k)
Evaluate scalar product of i + 2j – k and their dir vect in (i)
Sol 2: (i) Angle between the lines
Show as (1 × 1 or 1) + (2x − 2 or − 4) + ( −1x − 3 or 3)
−8 × 9 + 1 × 2 + ( −2 ) × ( −5 ) 84
cos θ = = = 0.9641 = 0 and
64 + 1 + 4 81 + 4 + 25 69 110
(iii) Obtain t = -2 or s = 3 (possibly – 3 or 2 or – 2)
=⇒ θ cos
= ( 0.9641) 15.38 degree
−1
Check if t = 2, 1 or -1

(ii) Let P be the point of intersection Subst. into eqn AB or OT and to produce 3i + 6j -3k

x−4 y −2 z+6 (iv) OC is to be found, where C is their point of
Equation of lines are = = = r1
−8 1 −2
intersection
Point P be ( −8r1 + 4,r1 + 2, − 2r1 − 6 )  ….(i) 
OC = 54
x+2 y −a z+2
Similarly for second line = = = r2
−9 2 −5
Sol 5: (i) OD = OA + AD or OB + BC + CD AEF
The point P be ( −9r2 − 2,2r2 − a, − 5r2 − 2 )  …(ii)
AD = BC or CD = BA
From (i) and (ii), we get 
OD= 2 ˆi + kˆ
−8r1 + 4 =−9r2 − 2
(ii) AB.CB = |AB||CB| cos θ ⇒ cos θ= 86°
r1 + 2 = 2r2 + a
       
−2r1 − 6 =−5r2 − 2 Sol 6: (i) Work out b − a or a − b or c − a or a − c
r1 3,=
On solving, we get= r2 2 and a = 1 = ± ( −3i − j − k) or ± ( −2i + j − 2k)

The points of intersection is (-20, 5, -12) Use cosine rule and find angle as 45.3o
1  
    (ii) Use of AB × AC sin θ
Sol 3: (i) Find a − b or b − a irrespective of label 2
(expect 11 ˆi − 2 ˆj − 6kˆ or −11 ˆi + 2 ˆj + 6kˆ ) =
1
2
( )
11 ( 3) sin 45.3° 3.54
=

Magnitude of vector = 161


    Sol 7: (i) Produce at least 2 of the 3 relevant eqns in
(ii) Using (AO or OA) and (AB or BA) λ and µ
Solving we get
2 6 . 44 | Vectors

1st solution: λ = −2 or µ = 3 ⇒ R1 − 2R 2 + 3R 3 =
10  … (i)
2nd solution: µ = 3 or λ = −2 2R1 + R 2 + 4R3 =
20  … (ii)
Substitute their λ and µ into 3 eqn and find ‘a’
rd
R1 + 3R 2 + 3R3 =
20  … (iii)
We get a=2 but a cannot be 2
On solving, we get R1 =
−1,R 2 =
2,R3 =
5
(ii) Subst their λ or µ (& pass a) into either line eqn 
R =− ˆi + 2ˆj + 5kˆ
Point of intersection is −5iˆ − 4ˆj
   
Sol 10: a1 = 2b1 + 3b2 − b3
−6 × 1 + 8 × 3 − 2 × 2   
Sol 8: (i) cos θ =

a2 =b1 − 2b2 + 2b3
36 + 64 + 4 1 + 9 + 4
   
14 a3 = 3b1 − b2 + 2b3
= = 68.47
104 14    
On solving, we get F = 2a1 + 5a2 + 3a3

(ii) Since, and are parallel   2 2 2 


−6 × 3 + 8 × C − 2 × 1 Sol 11: r + bS =r + b2 s + 2br.s
=
cos φ = 1
104 9 + C2 + 1       
r + bs is minimum when r .s =
− r s and r =
−bs
⇒ 8C=
− 20 104 10 + C2 2  2 2 2 2  
Then, bs + r + bs = b2 s + r + b2 s − 2b r s
2
(
⇒ ( 8C − 20 )= 104 10 + C2 ) 2 2 2 2
= 2b2 s + r − 2b2 s = r
⇒ 64C2 + 400 − 320C
= 1040 + 104C2

⇒ 40C2 + 320C + 640 =


0
Exercise 2
⇒ C2 + 8C + 16 =
0
Single Correct Choice Type
⇒ (C + 4 ) =
2
0 ⇒C=−4
   1  
Sol 1: (D) (b − 2 c) = λa or | b − 2c |= 1
x −3 y +3 z −0 λ
(iii) L2 ≡ = = =m  
1 3 2    
or (b − 2c)2 = (λ )2 or (b − 2c).(b − 2c) = λ2
     
Any point (m + 3, 3m − 8, 2m) or b.b − 2b.c − 2c.b + 4c.c
 
x − 2 y −1 z −3 ⇒ 16 − 4. | b | . | c | .cos θ + 4 = λ2
L3 ≡ = = =n
3 C 1 15 1
sin
= θ ⇒ cos
= θ
Any point ( 3n + 2, Cn + 1, n + 3) 4 4
1
If L2 and L3 intersect, then ⇒ 16 − 4 × × 4 × 1 + 4 =λ2
4
m + 3 = 3n + 2 ...(i) 2
⇒ λ = 16 ⇒ λ = ±4
3m − 8 = Cn + 1 ...(ii)
2m= n + 3 ...(iii)
Sol 2: (B) l2 + m2 + n2 =
1
On solution, we get C = -3
or cos2 α + cos2 β + cos2 γ =1

Sol 9: Let R = R1ˆi + R 2ˆj + R 3kˆ 2 1
or 3cos
= θ 1 or=
cos θ
     
( ) ( ) (
u. R − 10 i + V . R − 20 ˆj + W . R − 20 kˆ =
0 ) ∴ The desired vector is
3

(R1 − 2R2 + 3R3 − 10 ) ˆi + ( 2R1 + R2 + 4R3 − 20 ) ˆj  1 ˆ 1 ˆ 1 ˆ


15 3  i+ j+  15(iˆ + ˆj + k)
k= ˆ
+ (R1 + 3R 2 + 3R 3 − 20 ) kˆ = 0  3 3 3 
M a them a ti cs | 26.45

Sol 3: (C) Hint: Scalar Triple product Sol 9: (A)


y 
   B(b)
Sol 4: (B) a + b + c =
0

>
>
(a-b)

>
>
 (a+b)
 
| a | 3,=
= | b | 5,=|c| 7

2 2 2 A(a)
(5) + (3) − (7) 25 + 9 − 49
=cos θ =
2×5×3 30 O x
−15 1
= = − ⇒ θ = 150 or − 30    
30 2 Sol 10: (A) 2a + 4bcd = λ a c d + µ b c d
R
     
           

a =3

b =5   ( ) ( )
2a + 4b.cd = 2a + 4b. c × d = 2a. c × d + 4b. c × d ( )
 
= 2 a c d + 4 b c d ⇒ λ= 2, µ= 4 ⇒ λ + µ = 6
   

 Q
P c =7
Previous Years’ Questions
 
Sol 5: (B) Let |a =| 1= | b |
Sol 1: (B) The volume of parallelopiped
   
2(a b + b.c + c.a) + 3 =0
2 −3 0
        
(a + b + c).(a + b + c) = 0 =  a b c  = 1 1 −1
 
 2     3 0 −1
⇒ 1 + 1+ | c | +2(a b + b.c + c.a) =0
 
⇒ | c | + 2−3 = 0 ⇒ | c | =1 =2(–1) + 3(–1+3) = –2+6 = 4

     
Sol 6: (A) (a + b).b =⇒0 2 a.b + 2 b.b =
0 a 2piˆ + ˆj
Sol 2: (B)=
      y
Similarly (a + 2b).a =0 ⇒ a.a + 2a.b =0 y’
   
⇒| a |2 =+2 | b |2 ⇒| a |=2 | b | x’

  a.b 
Sol 7: (D) Projection of a on b = 
|b| 
 ˆ ˆ ˆ  O x
Let a =i + 2 j + k and b =4iˆ + 4ˆj + 7kˆ
 New vector
a.b
∴ Projection =  
|b| =a 4P2 + 1 cos αi + 4P2 + 1 sin αj
(iˆ + 2ˆj + k).(4i
ˆ ˆ + 4ˆj + 7k)
ˆ 4+8+7 19
= = ⇒ 4p2 + 1cos α = p + 1
(4)2 + (4)2 + (7)2 16 + 16 + 49 9
p +1
⇒ cos α =
Hence, the correct option is d. 4p2 + 1

Sol 8: (B) And 4p2 + 1 sin α =1


ˆi ˆj kˆ 1
⇒ sin α =
−2 6 −8= ˆi(24 − 24) − ˆj( −8 + 8) + k(6
ˆ − 6)
= 0 4p2 + 1
1 −3 4 (p + 1)2 1
⇒ sin2 α + cos2 α = 1 = +
2 2
4p + 1 4p + 1
∴ Magnitude = 0 2 2
⇒ 4p + 1= p + 1 + 2p + 1

⇒ 3p2 − 2p − 1= 0 ⇒ 3p2 − 3p + p − 1= 0
2 6 . 46 | Vectors

⇒ 3p(p − 1) + 1(p − 1) =0  
Sol 7: (C) As we know, a vector coplanar to a, b and
  
orthogonal to c is λ {(a × b) × c}
⇒ (p − 1)(3p + 1) =
0
∴A vector coplanar to (2iˆ + ˆj + k),
ˆ (iˆ − ˆj + k)
ˆ and
−1
⇒p=
1,
3 Orthogonal to 3iˆ + 2ˆj + 6kˆ

Sol 3: (B) Since, three vectors are coplanar. = λ[{(2iˆ + ˆj + k)


ˆ × (iˆ − ˆj + k)}
ˆ × (3iˆ + 2ˆj + 6k)]
ˆ

a a c = λ[(2iˆ − ˆj − 3k)
ˆ × (3 ˆi + 2ˆj + 6k)]
ˆ

1 0 1 =0 = λ(21 ˆj − 7k)
ˆ
c c b (21ˆj − 7k)
ˆ (3ˆj − k)
ˆ
∴ Unit vector = + = +
Applying C1 → C1 – C2 (21)2 + (7)2 10

0 a c 
Sol 8: (A) AB =2iˆ + 10ˆj + 11kˆ
⇒ 1 0 1 =0

0 c b AD =−ˆi + 2ˆj + 2kˆ
 
⇒ –1(ab – c2) = 0 ⇒ ab = c2 Angle ‘ θ ’ between AB and AD is
 
  AB· AD −2 + 20 + 22 8
   b+ c cos( θ ) =   = =
Sol 4: (A) Since, a × (b × c) = | AB | |AD | (15)(3) 9
2
     1  1  
⇒ ( a · c )b – ( a ·=
b)c b+ c Sol 9: V = i + j + k + λ ( i − j + k )
2 2

On equating the coefficient of c , we get = (1 + λ ) i + (1 − λ ) j + (1 + λ ) k
  1  1
a·b = – Projection on C is
2 3
  1  
⇒ | a | | b | cos θ = − V.C 1
2  =
C 3
1 3π
∴ cos θ = − ⇒ θ=
2 4 (1 + λ ) − (1 − λ ) − (1 + λ ) = 1
    3 3
Sol 5: (B) Since, ( a + 2 b ) · (5a − 4b) =
0
    ⇒ 1 + λ − 1 + λ −1 − λ =1 ⇒ λ = 2
⇒ 5 |a |2 + 6 a · b – 8 |b |2 = 0 
V = 3iˆ − ˆj + 3kˆ
   
⇒ 6 a · b = 3 [  |=a | |= b| 1]
   3  
1
⇒ cos θ = ⇒ θ =60º
Sol 10: (C) a × b × c −( )
2
b+c ( )
2
      3  
3 b + c

Sol 6: (C) Given, v =
 ˆ
2iˆ + ˆj – kˆ and w = i + 3kˆ ⇒ a.c
( ) ( )
b − a . b c
⇒ a.c b − a . b c = 2 b + c=
2
( )
      3   3
∴ [u v w = ] u·[(2iˆ + ˆj – k)
ˆ × (iˆ + 3k)]
ˆ 3 3
⇒ a.c
⇒ a.c = = 2
and
and aa .b.b == −
− 2
  ˆ 2 2
= u·(3iˆ − 7ˆj – =
ˆ | u|
k) | 3i − 7 ˆj − kˆ | cos θ   3
3
 ⇒ a
⇒ ab b cos
cos θθ==− − 2
which is maximum, if angle between u and 3iˆ − 7ˆj – kˆ 2
is 0 and maximum value= | 3iˆ − 7ˆj – kˆ | = − 3
59 cos θ
⇒ cos
⇒ θ==− 3
2
2
5
5ππ

⇒θθ= =6
6
M a them a ti cs | 26.47

   1    
( )
Sol 11: (D) a × b × c = b c a
3
Sol 16: (D) a = ˆj − kˆ and c = ˆi − ˆj − kˆ

    1    Let b = b1ˆi + b2ˆj + b3kˆ
( )
⇒ ( c. a) b − c. b a =
3
b c a  
a × b+ c=
 
0, a × b =

−c

  1 1
⇒ c. b = − b c ⇒ cos θ = − ˆi ˆj kˆ
3 3
0 1 − 1 =− ˆi + ˆj + kˆ
1 8
⇒ sin2 θ = 1 − cos2 θ = 1 − = b1 b2 b3
9 9
2 3 ˆi (b + b ) − ˆj (b ) + k ( − b ) =− ˆi + ˆj + kˆ
⇒ sin θ = ± 3 2 1 1
3
22 23 b3 + b2 = ‒1 … (i)
⇒ sin
But sinθθ==±
33 b1 = ‒1  … (ii)
 
a⋅b = 3
      2
Sol 12: (A) a × b.b × c c × a =
λ a b c  b2 – b3 = 3 … (iii)
   
Solve (i) and (iii)
We know that
        2 2b2 = 2 b1 = 2 b3 = –2
a × b b × c c × a =
a b c 
    ∴ b1 = –1 b2 = 1 b3 = –2
⇒ λ =1 
Hence b =− ˆi + ˆj − 2kˆ

  
Sol 13: (B) The length of median through A
 
Sol 17: (D) 2a − b ⋅ ( 
) {( a × b) × ( a + 2b)}
3iˆ + 4kˆ + 5iˆ − 2ˆj + 4kˆ 8iˆ − 2ˆj + 8kˆ
(2a − b) ⋅ {a ⋅ ( a + 2b) b − b ⋅ ( a + 2b) a  }
AB + AC     
== = =
2 2 2
3i + 4k + 5i − 2 j + 4k
== 4iˆ − ˆj + 4kˆ 2   
() ( )
2 2
2 − 5 ( a) b
= +5 a⋅b −5
=
8i − 2 j + 8k
= 16 + 1 + 16 = 33
Length
=
2
= 4i − j + 4k      
 Sol 18: (C) b × c = b × d
Sol 14: (C) c = a + 2b and d = 5a − 4b
     
( ) ( )
 
c⊥ d ⇒ a × b × c =a × b × d
 
           
c. d = 0
2     2 ( ) ( ) ( ) ( )
⇒ a .c b − a .b c = a .d b − a.b d
⇒ 5 a − 4a. b + 10a .b − 8 b = 0         
  ⇒ ( a .c ) b − ( a .b ) c =
− ( a.b ) d
⇒ 5 + 6a .b − 8 = 0
   
⇒ 6 a .b = 3    a.c  
∴ d = c −    b
  1  a.b 
⇒ a .b =  
2
1
⇒ cos φ =
2
π
JEE Advanced/Boards
⇒φ=
3
Exercise 1
       
 
Sol=
15: (D) a.b 0=b. c 0=c.a 0
( a × b ) + ( a.b )
2 2

⇒ 2λ= +4+µ 0 λ=− 1 + 2µ 0 Sol 1:  


2a2b2
Solving we get : λ = −3, µ = 2
2 6 . 48 | Vectors

     2 2 

(=   
a × b ) . ( a × b ) + ( a.b ) . ( a.b ) a × b + a.b b 3 ˆi and=
Sol 9: Let =

a 2 ˆi + ˆj ( )
   
2 2 2 2 The two diagonals will be 6 a − b and 4 a + 5b
2a .b 2a .b
 
2 2 2 2 Length of 6 a − b = 9 ˆi + 12 ˆj = 15
a . b .sin2 θ + a . b .cos2 θ
=  
2 2 Length of 4 a + 5b
2a .b
=8ˆj + 23iˆ = ( 23) + ( 8 ) = 593
2 2

1 ˆ ˆ
Sol 2: Area
=
2
( ) (
3i + 4 j × −5iˆ + 7ˆj =
1
2
)
21 Kˆ + 20 kˆ =
41
2 Sol 10: 2 × ˆi + xjˆ + 3kˆ = 4iˆ + ( 4x − 2 ) ˆj + 2kˆ

16 + ( 4x − 2 ) + 4
2
{
λ ( 40 − a) ˆi + 44ˆj
Sol 3: 20iˆ + 11jˆ = } or,2. 12 + x2 + 9=

∴ λ ( 40
= − a) 20 and=
λ ( 4 ) 11 ( )
∴ 4 10 + x2 = 20 + 16x2 + 4 − 16x

⇒ a =−40 or, 40 + 4x2 =24 + 16x2 − 16x

  or,12x2 − 16x − 16 =
0 or,3x2 − 4x − 4 =
0
Sol 4: Note that since a = b hence the parallelogram
will be a rhombus. 4 ± 16 + 4.4.3 4 ± 8 −2
x
∴= = = 2 or
6 6 3
   
( )
Sol 5:  a + 3b is perpendicular to 7 a − 5b ( )
    Exercise 2
∴ ( a + 3b ) . ( 7a − 5b ) =
0
2   2
⇒ 7. a − 5 a.b + 21 a.b − 15 b =

0 ⇒ 16 a.b =
8 (9iˆ − ˆj + 2kˆ ) − (3iˆ − 2ˆj + kˆ )= (6iˆ + ˆj + kˆ )

Sol 1: (A) r=
 
∴ Moment of couple = r × F = ( 6iˆ + ˆj + kˆ ) × (5iˆ + ˆj )
 1   1 1 π
∴ a.b= ⇒ a . b .cos θ= ⇒ cos θ= ⇒ θ=
2 2 2 3 ˆi ˆj kˆ
= 6 1 1 = ˆi ( 0 − 1) − ˆj ( 0 − 5) + kˆ ( 6 − 5)

( ) ( Aˆ − Bˆ ) 5 1 0
2
ˆ − Bˆ = A
Sol 6: A ˆ − Bˆ
=−ˆi + 5ˆj + kˆ
2 2
ˆ .Bˆ = 1 + 1 − 2 1 1 cos π = 2 + 2 = 4
ˆ + Bˆ − 2 A
= A
     
ˆ − Bˆ =
ˆ − Bˆ
A (
Sol 2: (C) a + b + c . a + b + c )( )
⇒ A 2 ⇒ 1
=
2 2 2 2         
( )
= a + b + c + a b + c + b (a + c) + c a + b ( )
 
(1) + ( 4 ) + ( 8 )
2 2 2
Sol 7: F1 = 3 ˆi + 2 ˆj + 5kˆ and F2 = 2 ˆj + ˆj + 3kˆ = + 0= 81
     
∴ F = F1 + F2 = 5iˆ + 3ˆj + 8kˆ ∴ a+b+c =9



∆xx == ( 4i4iˆˆ ++ 33ˆˆjj ++ 7k
7kˆ ) −
ˆ ( 2iˆˆ + ˆˆj + 3kˆˆ ) =
 
− 2i + j + 3k =
ˆ 2ˆˆj + 4kˆˆ
2i
2iˆ +
+ 2 j + 4k
Sol 3: (A) Let M be the point of intersection of internal

∴ Work
∴ Work done
done == F.
F.∆∆xx == 10 +6
10 + + 32
6+ 32 == 48
48 units bisector with AB.
Hence,(d).
Hence,(d).
AM 1
∴ =
 MB 2
Sol 8: ∆x =−2iˆ − 3ˆj − 4kˆ 
 
Work done= F.∆x ∴ OM =
( ) (
 1 2iˆ + 4 j + 4kˆ + 2 2iˆ + 2ˆj + kˆ ) =
6iˆ + 8ˆj + 6kˆ
⇒ 2 = −12 − 3λ − 16 ⇒ 30 = −3λ ⇒ λ = −10 3 3
 2
8 72 + 64 136
∴ OM = 4+  +4 = =
3
  9 9
M a them a ti cs | 26.49

       1
Sol 4: (A) a + 2b + 3c =
0 Sol 9: (C) A B A × B =
4
     
⇒ a × b + 2b × b + 3c × b = 0     1        1
       
⇒ a × b + 3c × b = 0 ⇒ a × b = 3b × c ... (i)
(
⇒ A ⋅ B× A×B =
4 )
⇒ A  B ⋅B A − A ⋅B ( ) ( )B = 4
           2
1 1
Similarly, a + 2b+ 3c =
 
0 ⇒ A ⋅  A − A ⋅ B B =
  4 ( )
⇒ A⋅A − A⋅B = ( ) 4
      2 
a × a + 2b × a + 3c × a = 0 ⇒ c × a = a × b
3   1 3   3
( )
2
    ⇒ A⋅B =1− = ⇒ A⋅B=
2
( ) (
= × 3 b × c =2 b × c
3
) ... (ii) 4 4 2
        3 π
⇒a×b + b× c + c ×= a 6 b ×c ( ) ⇒ cos=
θ
2
⇒=
θ
3

               
( ( ) {) ( )
Sol 5: (C) a − b b − c c − a=  =a −ab− b. b. ×bc−−cb ×× (ac+−ca×) a Sol 10: (C) A − B + A + B=
} 2 − 2 cos θ + 2 + 2 cos θ
             
( ){
     
} ( )
= a − b . b × c − b × a + c × a = a. b × c − b. ( c × a )= 0 = ( 2 1 − cos θ + 1 + cos θ )
( )
= a. b × c − b. ( c × a )= 0  θ θ  θ
= 2.  2 sin + 2cos  =2  sin + cos 
θ
Sol 6: (D) A general point on the curve will have vector  2 2  2 2
 1 Greatest value is 2 2
xiˆ + yjˆ = xiˆ +  x +  ˆj
   x
 OP. I = 1

  1   Previous Years’ Questions
∴ xi +  +  ˆj .iˆ = 1 ⇒ x = 1

  x       
 Sol 1:
= A. X C and =
A×X B
∴ OP =+ ˆi 2ˆj

  Let A = A ˆi + A ˆj + A kˆ
Again, OQ. I =−1 ⇒ x =−1 1 2 3
  
   (i) A × X =B
∴ OQ =− i − 2 j             
 
∴ 2OP + 3OQ =−ˆi − 2ˆj ( ) ( ) ( )
A × A × X =A × B ⇒ A. X A − A. A X = A ×B
    
∴ 2OP + 3OQ = ( −1 ) + ( 2 ) = 5
2 2  2      CA A × B
⇒ A X = CA − A × B ⇒ X=  − 
2 2
Hence,(d). A A
     
( )
Sol 7: (C) A ⋅ B ⋅ C + BC A + B =
0 ( ) (ii) If coordinate system is rotated about the x-axis
⇒ A.B + A.C + B.C =
0 π
through an angle , then
2
A + B + C = A + B + C + 2 ( A.B + B.C + C.A )
2 2 2 2
x- component = A2
= 1 + 16 + 64 + 0 = 81 y – component = A1
⇒ A +B + C =9 z – component = A3

A = A2ˆi − A1ˆj + A3kˆ
Sol 8: (C) Given
[New coordinates system]
A −B ≤ 1
A −B ≤ 1        
∴ 122 + 122 − 2cos2θ ≤ 1 a b c a+b+c b c
∴ 1 + 1 − 2cos2θ ≤ 1           
1
⇒ 2 − 2cos2θ ≤ 1 ⇒ 2cos2θ ≥ 1 ⇒ cos2θ ≥ 1
⇒ 2 − 2cos2θ ≤ 1 ⇒ 2cos2θ ≥ 1 ⇒ cos2θ ≥ 2
Sol 2: a.a

a.b a.c=
   (
a. a + b + c
   
) a.b a.c
  
5π 5π
∴ 5π ≤ 2θ ≤ 2π ⇒ 5π ≤ θ ≤ π
2 b.a b.b b.c
(
b. a + b + c ) b.b b.c
∴ 3 ≤ 2θ ≤ 2π ⇒ 6 ≤ θ ≤ π
3 6
2 6 . 50 | Vectors

 
1 b c ⇒ 1 + 1 – ŵ · v̂ = λ2 ⇒ 2 – 2cos 2 θ = λ2 ⇒=
λ 2sin θ
     
= ( )
a+b+c . a

a.b
 
a.c
 where 2θ is the angle between v̂ and ŵ .
b b.b b.c Hence, ŵ – v̂ = 2sin θ · â
         = 2cos (90º – θ ) â = –(2 â · v̂ ) â
   ( )( ) ( )( ) ( )( )
 a.b b.c − b.b a.c − a.b b.c + 
= ( a+b+c ) .          

ŵ = v̂ – 2( â · v̂ ) â
( )( ) ( )( ) ( )( )
 b.b a.c + a.c b.b − b.c a.b

 
          Sol 6: (B, D) Let vector AO be parallel to line of
= ( ) ( )( ) ( )( )
a + b + c  b.b a.c − a.b b. c  = 0
  intersection of planes P1 and P2 through origin
  
⇒ if a + b + c =0 [coplanar condition] Normal to plane P1 is

n1 =[(2ˆj + 3k)
ˆ × (4ˆj − 3k)]
ˆ = −18iˆ
         
(
Sol 3: R × B = C × B ⇒ A × R × B = A × C × B
           
) ( ) Normal to plane P2 is

( ) ( ) ( ) ( )
⇒ A.B R − A .R B = A.B C − A.C B

n2 = (ˆj − k)
ˆ × (3iˆ + 3ˆj) = 3iˆ − 3ˆj − kˆ
     
( 2 + 0 + 1)R − 0 = ( 2 +0 +1) C − ( 8 + 0 + 7 )B
  
∴ OA is parallel to ± ( )
n1 × n2 = 54ˆj − 54 kˆ
⇒ 3R = 3C − 15B ⇒ R = C − 5B
=4iˆ − 3ˆj + 7kˆ − 5iˆ − 5ˆj − 5kˆ =−ˆi − 8ˆj + 2kˆ
∴ Angle between 54 (ˆj − k)
ˆ and 2iˆ + ˆj − 2kˆ is ( )
 54 + 108  1 π 3π
cos θ = ±  =± ∴ θ= ,
     3·54 · 2  2 4 4
Sol 4: Given, a × b = c × d  
     
and a × c = b × d Sol 7: (A, D) Let, a = ˆi + ˆj + 2kˆ , b =ˆi + 2ˆj + kˆ and
        
⇒ a×b − a× c = c × d − b× d c = ˆi + ˆj + kˆ 
 
      ∴ A vector coplanar to a and b , and perpendicular to c
⇒ a × (b − c ) = ( c − b ) × d         
      r= λ ( a × b ) × c = λ { (a· c)v – (b· c)a }
⇒ a × (b − c ) − ( c − b) × d = 0
      =λ { (1 + 1 + 4)(iˆ + 2ˆj + k)
ˆ – (1 + 2 + 1)(iˆ + ˆj + 2 k)
ˆ }
⇒ a × (b − c ) − d × (b − c) = 0
        = λ { 6iˆ + 12ˆj + 6kˆ − 6iˆ − 6ˆj − 12 kˆ } = λ { 6ˆj − 6 kˆ } = 6λ {ˆj − kˆ }
⇒ ( a − d) × (b − c ) = 0 ⇒ ( a − d) || (b − c )
            1 ⇒
For λ = (a) is correct.
∴ ( a − d)·(b − c ) ≠ 0 ⇒ a·b + d·c ≠ d·b + a·c 6
1
and λ = − ⇒ (d) is correct.
Sol 5: Since, v̂ is unit vector along the incident ray and 6
ŵ is the unit vector along the reflected ray.
Sol 8: From the given information, it is clear that
 ˆi − 2ˆj
- a w a=
5
v
    
⇒ | a | = 1, | b | =1, | a · b | = 0
 Mirror
     
Now, (2a + b) · [(a × b) × ( a − 2b)]
         
= (2a + b) · [a2b − (a·b)· a + 2b2 · a − 2(b · a)· a]

Hence, â is a unit vector along the external bisector of        


= [2a + b] · [b + 2a] = 4a2 + b2 = 4·1 + 1 = 5 [as a.b =0]
v̂ and ŵ .
∴ ŵ – v̂ = λ â
On squaring both sides, we get
2
⇒ c = 144 − 96
M a them a ti cs | 26.51
⇒ c =
4 3
    2
Sol 9: r × b = c × b c  48
      ⇒ −a = − 12 = 24 − 12 = 12
⇒ (r − c) × b = 0 ⇒ r − c =λb 2 2
   
or r = c + λb  ...(i) Given b.c = 24
 
   − b c cosp = 24
Given, r · a = 0 , taking dot product with a for Eq. (i)
      −4 3 × 4 3 cosp =
24
⇒ r= · a a · c + λa · b
  −1
−a · c   cosp =
∴ λ =   ( r · a = 0)  ...(ii) 2
a·b
Since b = c
From Eqs. (i) and (ii), we get
  ∠PQR =∠PRQ ⇒ ∠QPR =1200
  a·c  
r= c −   b , taking dot with b , we get    
a·b and ∠PQR =
∠PRQ =300 ⇒ a × b + c × a =48 3
   
    a·c   And a × b =-72
r=·b c·b −   (b·b)
a·b
   
( −1 − 3) Sol 12: r × b = c × b
= (–1+2) – (1 + 1) where,
(1) Taking cross with a
      
 a =−ˆi − kˆ 
  ⇒ a × (r × b) = a × (c × b)
ˆ ˆ         
 b =− i + j
 ˆ ˆ
 = 1+8 = 9

c =i + 2 j + 3kˆ 
( ) ( )
⇒ a.b r− a.r b=

ax cxb ( )
  ⇒ r= − 3iˆ + 6ˆj + 3k
 
 r. b = 3 + 6 = 9
Sol 10: (B, C) Given: w. (uˆ × v ) =
1

⇒ w (uˆ × v ) cos θ= 1 ⇒ cos θ= 1      

w ⊥ uˆ × vˆ ⇒ w ⊥ uˆ and w ⊥ vˆ and uˆ × vˆ =1
( ) (
Sol 13: 2a + b .  a × b × a − 2b  ) ( )
  ii −−22jj  2i 2i ++ jj ++ 3k
3k
Angle between u and v can change to have initially=
=a
a =
= b
b
5
5 14
14
many of vectors v as w ˆ ⊥ vˆ        
If u lies in xy plane then u = u ˆi + u ˆj

⇒ (
2a +
2a +b ) ( ) ( )
b aa ×× aa ×× bb −− (aa××bb )××bb
 
1 2                
ˆ ˆ =0 ⇒ u1 + u2 =0 ⇒ u1 =u2
⇒ w.u = ( )( ) ( ) ( )
2a × b  a. a b − a.b

a.b aa ++ 22 a. ( )bb aa
a. bb bb ++22 bb

     
( )
=2a + b . b + 2a 

{ } 

a.a. bb =
=00
Sol 11: (A, C, D) In ∆PQR    22 
  
22
   = 2a b + 4 a + b − 22 b. b. aa
−a = b + c
      2 2

(
⇒ a. a = b + c . b + c )( ) P = 4a +b
= 4 +1 = 5
2 2 2  
⇒ a = b + c + 2b. c
4 3       
2 Sol 14: Given: a × b + b × c = pa + qb + rc
( 4 3 ) + c + 2 × 24
2 2
a
⇒=
         
⇒ (12
= ) ( 4 3 ) + c + 2 × 24
2 2 2
Q
12
R
( ) ( )
⇒ a. a × b + a. b × c =p + q a. b + r a. c ( ) ( )
  
2
⇒ c = 144 − 96
( )
⇒ a. b × c = P + +
q r
2 2
...(i)

     
⇒ c =
4 3 ( ) ( ) p
Similarly, b. a × b + b. b × c = + q +
2
r
2
2 p r
c  48 ⇒ +q+ = 0 ...(ii)
⇒ −a = − 12 = 24 − 12 = 12 2 2
2 2 p q
and + + r= a (b × c ) ...(iii)
2 2
2 6 . 52 | Vectors
p r
⇒ +q+ = 0 ...(ii)
2 2
p q
Sol 17: V = ai + b j + ck : a,b,c ∈ {−1,1}
  
and + + r= a (b × c ) ...(iii)
2 2
Total number of selection = 8C
3
From (i), (ii) and (iii)
No. of coplanar vectors = 6 × 4 = 24
p2 + 2q2 + r 2
P =−q =r ⇒ =4 Total number of non co-planar vet
q2
= 8 c3 − 24 = 32 = 2p
  = P= 5
Sol 15: (C) ⇒ x + y = 3i + j − 2k
         2
and x − y =i − 3j − 4k Sol 18: (C) (i) 2a × b 3b × c c × a = 6 a b c 
R

y Q  
On solving we get   
( 2) 24 a.=
2
 = 6= b. c 2

x = 2i − j − 3k 
x 
 x            

y =+i 2j + k ( )
(ii) 3 a + b b + c 2c + a = 6 a + b b + c c + a
S  P

( ))
Volume of parallelopiped      
( ) (
y
= 6 a + b.  b + c × c + a 
 
2 −1 −3          
= 1 2 1 ( )
= 6 a × b b × c + b × a + c × c + c=

× a 12a. (b × c )

1 2 3      
= 2 ( 6 − 2) + 1 (3 − 1) − 3 ( 2 − 0 ) = 12 a b c  = 12 × 5 = 60
  ( a b c  = 5
  )
= 8 + 2 = 10 1    
(iii)
2
(
2a + 3b × a − b ) ( )
Sol 16: (B, D) Vector perpendicular to 1 and  2 is
1       
given by
=
2
(
2a × a − 2a × b + 3b × a − 3b × b )
ˆi ˆj kˆ
P 1   1 1   


2 2 1 2 = 5a × b = × 5 × 40 = 100  a × b = 20 
2 2 2 
1 2 2 17
1      
= ˆj ( 4 − 2 ) − ˆj ( 4 − 1) + kˆ ( 4 − 2 )   
Q
( )
(iii) a × b × a = a × a + b × a= b × a
= 2iˆ − 3ˆj + 2kˆ  
= 30  a ×=
b 30 
The eq. of line ⊥ to 1 and  2  

x−0 y −0 z−0  2  2  2
= = = γ Sol 19: a − b + b − c + c − a =9
2 −3 2
⇒ Q ≡ ( 2γ , −3γ ,2γ )  2 2 2   2
⇒ 3 a + b + c  − a + b + c = 9
2γ − 3 −3γ + 1 2γ − 4  
The point Q lies on  2 , =
then = =   2
1 2 2 ⇒ 3 (1 + 1 + 1 ) − a + b + c =9
⇒ γ =1
  2   
⇒ Q ≡ ( 2, −3,2 ) ⇒ a+b+c = 0 ⇒a+b+c= 0
Distance of P from Q in 17   
⇒ b + c =−a
( 2 − 3 − 2) + ( −3 − 3 − 25) + ( 2 − 5 − 2 )   
2 2 2   
PQ 2 = 17 =
−10
Now, 2a + 5b + 5c = 2a + 5 b + c ( )
⇒ S =−2,   
9 = 2a − 5a = 3 a = 3
7 7 8
⇒ P ≡ ( −1, −1,0 ) and  , , 
9 9 9
M a them a ti cs | 26.53


Sol 20: (C) Let c = 2iˆ + 3ˆj + 4kˆ
  
  
a× c = c ×b ⇒ a + b × c = ( 0 )
  
( )
⇒ a + b || c
  
Let ( a + b ) =
λc
  
⇒ | a + b |=
| λ || c |

⇒ 29 = | λ | . 29 ⇒ λ = ±1


(
∴ a + b =± 2iˆ + 3ˆj + 4kˆ )
 
( )( )
± ( −14 + 6 + 12 ) =
Now, a + b . −7iˆ + 2ˆj + 3kˆ = ±4

  
Sol 21: (C) Any vectors v coplanar with a and b is
given by
  
=
=v ma
v ma + nb + nb

= m
= (
ˆ ˆˆj + kˆˆ + n
m ˆii ++ j+k +n ) (ˆˆii ++ ˆˆjj −− kkˆˆ )
== ((m
m+ n)) ˆii +
+n
ˆ (m − n) ˆˆj + (m + n) kˆˆ
+ (m − n) j + ( m + n) k  ... (i)

  v.c 1
Projection to v on c is given by  =
c 3
⇒ (m + n) − (m − n) − (m + n) =
1
⇒ m+n−m+n−m−n =1
⇒ m +1= n
⇒ m =n − 1
Substituting in (i)

( 2n − 1) ˆi − ˆj + 2n − 1kˆ
for n = 2
3iˆ − ˆj + 3kˆ

Sol 22: (A, D) Let r the vector coplanar with i + j + 2k
and i + 2 j + k then

r= m ( i + j + 2k ) + n ( i + 2 j + k )
= (m + n) i + (m + 2n) j + ( 2m + n) k
 
r ⊥ c, then

m + n + m + 2n + 2m + n =0
⇒ m+n = 0
⇒ r= ( 0 ) i + ( 0 + n) j + (n + 0 ) k
= nj + mk
2017-18 100 &
op kers
Class 12 T
By E ran culty
-JE Fa r
IIT enior emie .
S fP r es
o titut
Ins

MATHEMATICS
FOR JEE MAIN & ADVANCED
SECOND
EDITION

Exhaustive Theory
(Now Revised)

Formula Sheet
9000+ Problems
based on latest JEE pattern

2500 + 1000 (New) Problems


of previous 35 years of
AIEEE (JEE Main) and IIT-JEE (JEE Adv)

5000+Illustrations and Solved Examples


Detailed Solutions
of all problems available

Topic Covered Plancess Concepts


Tips & Tricks, Facts, Notes, Misconceptions,
3D Geometry Key Take Aways, Problem Solving Tactics

PlancEssential
Questions recommended for revision
27. 3D GEOMETRY

1. COORDINATE OF A POINT IN SPACE y

Let P be a point in the space. If a perpendicular from that point is dropped to the xy- y1 P (x1, y1, z1 )
plane, then the algebraic length of this perpendicular is considered as z-coordinate.
x
From the foot of the perpendicular, drop a perpendicular to x and y axes, and algebraic O
z1
lengths of perpendicular are considered as y and x coordinates, respectively. x1
Z
Figure 27.1
2. VECTOR REPRESENTATION OF A POINT IN SPACE
If (x, y, z) are the coordinates of a point P in space, then the position vector of the point P w.r.t. the same origin is

OP = xiˆ + yiˆ + zkˆ .

3. DISTANCE FORMULA
If (x1, y1, z1) and (x2, y2, z2) are any two points, then the distance between them can be calculated by the following

formula: (x1 − x2 )2 + (y1 − y 2 )2 + (z1 − z 2 )2

3.1 Vector Method


→ →
If OA and OB are the position vectors of two points A (x1, y1, z1) and B (x2, y2, z2), then AB
= | OB − OA |

⇒ AB = |(x2 i + y2j + z2k) – (x1i + y1j + z1k)| ⇒ AB = (x2 − x1 )2 + (y 2 − y1 )2 + (z 2 − z1 )2

3.2 Distance of a Point from Coordinate Axes


Let PA, PB and PC be the distances of the point P(x, y, z) from the coordinates axes OX, OY and OZ, respectively.
Then PA = y 2 + z 2 , PB = z 2 + x2 , PC = x 2 + y 2

Illustration 1: Show that the points (0, 7, 10), (–1, 6, 6) and (–4, 9, 6) form a right-angled isosceles triangle. 
 (JEE MAIN)

Sol: By using distance formula we can find out length of sides formed by these points and if it satisfies Pythagoras
theorem then these points form a right angled triangle.
2 7 . 2 | 3D Geometr y

Let A ≡ (0, 7, 10), B ≡ (–1, 6, 6), C ≡ (–4, 9, 6)AB2 = (0 + 1)2 + (7 – 6)2 + (10 – 6)2 = 18
∴ AB =
3 2 Similarly BC = 3 2 and AC = 6; Clearly AB2 + BC2 = AC2 and AB = BC
Hence, ∆ABC is isosceles right angled.

Illustration 2: Find the locus of a point which moves such that the sum of its distance from points A(0, 0, –α) and
B(0, 0, α) is constant. (JEE MAIN)

Sol: Consider the point whose locus is required be P(x, y, z). As sum of its distance from point A and B is constant
therefore PA + PB = constant = 2a.
Let P(x, y, z) be the variable point whose locus is required
Given that PA + PB = constant = 2a(say)

∴ (x − 0)2 + (y − 0)2 + (z + α )2 + (x − 0)2 + (y − 0)2 + (z − α )2 =


2a

⇒ x2 + y 2 + (z + α )2 = 2a − x2 + y 2 + (z − α )2

⇒ x2 + y 2 + z 2 + α2 + 2zα = 4a2 + x2 + y 2 + z 2 + α2 − 2zα − 4a x2 + y 2 + (z − α )2

z2 α2 x2 + y 2 z2
⇒ 4zα − 4a2 = −4a x2 + y 2 + (z − α )2 ⇒ + a2 − 2zα = x2 + y 2 + z 2 + α2 − 2zα ⇒ + 1
=
a2 a2 − α2 a2

4. SECTION FORMULA
If a point P divides the distance between the points A(x1, y1, z1) and B(x2, y2, z2) in the ratio of m:n, then the
 mx + nx1 my 2 + ny1 mz 2 + nz1  A P B
coordinates of P are  2 , , 
 m+n m+n m + n 
m:n
 x + x2 y1 + y 2 z1 + z 2  1:1
Note: Midpoint  1 , , 
 2 2 2 
A P B

5. DIRECTION COSINES AND DIRECTION RATIOS


(a) Direction cosines: If α, β, γ are the angles which the line makes with the positive directions of the axes x, y
and z coordinates, respectively, then cosα, cosβ, cosγ are called the direction cosines (d.c.s) of the line. The
direction cosines are usually denoted by (l, m, n), where l = cosα, m = cosβ and n = cos γ.
z
(b) If  , m, n are the direction cosines of a line, then l2 + m2 + n2 = 1

(c) Direction ratios: If the intercepts a, b, c are proportional to the direction cosines  ,
P
(d) m, n, then a, b, c are called the direction ratios (d.r.s).
O y
(e) If  , m, n are the direction cosines and a, b, c are the direction ratios of a vector, then
x
a b c
= ,m = ,n = or Figure 27.2
a2 +b2 + c2 a2 +b2 + c2 a2 +b2 + c2
−a −b −c
 = ,m = ,n
a2 + b2 + c2 a2 + b2 + c2 a2 + b2 + c2

(f) If OP = r, where O is the origin and l, m, n are the direction cosines of OP, then the coordinates of P are (lr,
mr, nr) If direction cosines of the line AB are l, m, n, |AB| = r, and the coordinates of A is (x1, y1, z1), then the
coordinates of B are (x1 + rl, y1 + rm, z1 + rn)
M a them a ti cs | 27.3

Illustration 3: Let α, β, γ be the angles made with the coordinate axes. Prove that sin2α + sin2β + sin2γ = 2 
 (JEE ADVANCED)
Sol: Here line makes angles α, β, γ with the co-ordinates axes, hence by using its direction cosine we can prove
given equation.
Since a line makes angles α, β, γ with the coordinates axes, cos α, cos β, cos γ, are direction cosines.
∴ cos2α + cos2β + cos2γ = 1
⇒ (1 – sin2α) + (1 – sin2β) + (1 – sin2γ) = 1 ⇒ sin2α + sin2β + sin2γ = 2

Illustration 4: Find the direction cosines l, m, n of a line using the following relations: l + m + n = 0 and 2mn +
2ml – nl = 0. (JEE ADVANCED)

Sol: By solving these two equations simultaneously, we will be get l : m : n.

Given,  + m + n = 0 … (i) 2mn + 2m l – n l = 0 …(ii)


From equation (i), n = –( l + m) Substituting n = –( l + m) in equation (ii), we get,

–2m(l + m) + 2m l + (l + m) l = 0 ⇒ –2ml –2m2 + 2ml + l 2 + ml = 0


2
  l −1 ± 1 + 8 −1 ± 3
⇒ l2 + m l – 2m2 = 0 ⇒   + −2 = 0 ⇒ = = = 1, −2
m m m 2 2
 
Case
= I: When
= Inthiscase
1:= In1:=
this case
Inthiscase
m  m  From equation (1), 2 l + n = 0
m m
⇒ n = –2 l
∴ l : m : n = 1 : 1 : –2 ∴ Direction ratios of the line are 1, 1, –2
1 1 −2 1 1 −2 1 1 2
∴ Direction cosines are ± ,± ,± = , , or − ,− ,
2
1 + 1 + ( −2)2 2 2 2
1 + 1 + ( −2) 2 2 2
1 + 1 + ( −2) 2 6 6 6 6 6 6

Case II: When = −2: Inthiscase  =
−2m
m
From equation (i), –2m + m + n = 0 Þ n = m ∴  : m : n = –2m:m:m
∴ Direction ratios of the line are –2, 1, 1

∴ Direction cosines are given by

−2 1 1 −2 1 1 2 −1 −1
, , = , , or , ,
2
( −2) + 1 + 12 2 2 2
( −2) + 1 + 1 2 2
( −2) + 1 + 1 2 2 6 6 6 6 6 6

6. ANGLE BETWEEN TWO LINE SEGMENTS


If a1, b1, c1 and a2, b2, c2, are the direction ratios of any two lines, respectively, then a1i + b1j + c1k and a2i + b2j + c2k
are the two vectors parallel to the lines, and the angle between them is given by the following formula:
a1a2 + b1b2 + c1c2
cos θ =
a12 + b12 + c12 a22 + b22 + c22

(a) The lines are perpendicular if a1a2 + b1b2 + c1c2 = 0


a1 b1 c1
(b) The lines are parallel if = =
a2 b2 c2
(c) Two parallel lines have same direction cosines, i.e. l1 = l2, m1 = m2, n1 = n2
2 7 . 4 | 3D Geometr y

Illustration 5: Prove that the lines, whose direction cosines given by the relations a2 l + b2m + c2n = 0 and mn + nl
1 1 1
+ lm = 0, are perpendicular if + + 0 and parallel, if a  b ± c =
= 0  (JEE ADVANCED)
2 2
a b c2
Sol: Here if two lines are perpendicular then, 1  2 + m1m2 + n1n2 = 0 and if they are parallel then,

= 2 , m1
1 = m
=2 , n1 n2

Given that, a2 l + b2m + c2n = 0 … (i)

and mn + n l + l m = 0 ... (ii)


1 1
Eliminating m from equations (i) and (ii), we have − (a2  + c2n)n + n − (a2  + c2n) =
0
2 2
b b
2
   
0 ⇒ a2
⇒ a2n + c2n2 − b2n + a2 2 + c2 n = + c2 − b2 + a2 + c2 =0
n n n 2 n
2
 
(
⇒ a2   + a2 − b2 + c2   + c2 =
n n
0 ) ... (iii)

1  2
Let , be the roots of the equation (iii).
n1 n2
 1  2 c2 1  2 n1n2 1  2 m1m2 n1n2
∴ Product of roots , = ⇒ = ⇒ = = [By symmetry]
n1 n2 a2 1/a 2
1/c 2
1/a 2
1/b 2
1 / c2

1  2 m1m2 n1n2 1  2 + m1m2 + n1n2


⇒ = = =
1 / a2 1 / b2 1 / c2 1 / a2 + 1 / b2 + 1 / c2

For perpendicular lines 1  2 + m1m2 + n1n2 =


0

1 1 1

2
+
2
+ 0
= Two lines are parallel= 2 , m1
if 1  = m
=2 ,n1 n2
a b c2
1 2
⇒ = ⇒ roots of equation (iii) are equal ⇒ (a2 – b2 + c2)2 – 4a2c2 = 0 ⇒ a2 – b2 + c2 = ± 2ac
n1 n2
b2 ⇒ (a ± c)2 =
⇒ a2 + c2 ± 2ac = b2 ⇒ (a ± c) =±b ⇒ ab±c =0

Note: In the above result, the two signs are independent of each other. So, the total cases would be
( a + b + c= 0,a + b − c= 0,a − b + c= 0,a − b − c= 0).

7. PROJECTION OF A LINE SEGMENT ON A LINE


If (x1, y1, z1) and (x2, y2, z2) are the coordinates of P and Q, respectively, then the projection of the line segments PQ

on a line having direction cosines  , m, n is |  (x2 – x1) + m(y2 – y1) + n(z2 – z1)|.
 
 ab
Vector form: Projection of a vector a on another vector b is =  . In the above case, we replace 2 6 with
|b|

PQ as(x − x )iˆ + (y − y )ˆj + (z − z )kˆ and ⇒ (x − 1)2 =
2 1 2 1 2 111 with ˆi + mjˆ + nkˆ .

where  | r |,m | r |&n | r | are the projections of r in the coordinate axes OX, OY and OZ, respectively.
r | r | (ˆi + mjˆ + nk)
= ˆ

Illustration 6: Find the projection of the line joining the coordinates (1, 2, 3) and (–1, 4, 2) on line having direction
ratios 2, 3, –6.  (JEE MAIN)
M a them a ti cs | 27.5

Sol: Here projection of line joining (1, 2, 3) and (–1, 4, 2) on the line having direction ratios 2, 3, –6 is given by
(x2 − x1 ) + m(y 2 − y1 ) + n(z 2 − z1 ) .
B
Let A ≡ (1,2,3),B ≡ ( −1, 4,2) . Direction ratios of the given line PQ are 2, 3, –6 A

2 3 6
∴ Direction cosines of PQ are , ,−
7 7 7 90o 90o
P L M Q
Projection of AB on PQ = (x2 − x1 ) + m(y 2 − y1 ) + n(z 2 − z1 )
Figure 27.3
2 2 6 −4 + 6 + 6 8
= ( −1 − 1) + (4 − 2) − (2 − 3) = =
7 7 7 7 7

8. PLANE
If a line joining any two points on a surface entirely lies on it or if a line joining any two points on a surface is
perpendicular to some fixed straight line, then the surface is called a plane. This fixed line is called the normal to
the plane.

8.1 Equation of a Plane


(a) Normal form: The equation of a plane is given by lx + my + nz = p, where  , m, n are the direction cosines
of the normal to the plane and p is the distance of the plane from the origin.
(b) General form: The equation of a plane is given by ax + by + cz + d = 0, where a, b, c are the direction ratios
of the normal to the plane.
(c) The equation of a plane passing through the point (x1, y1, z1) is given by a(x – x1) + b(y – y1) + c (z – z1) = 0,
where a, b, c are the direction ratios of the normal to the plane.
(d) Plane through three points: The equation of a plane through three noncollinear points is given by
x y z 1
x − x3 y − y3 z − z3
x y1 z1 1
(x1, y1, z1), (x2 , y 2 , z 2 ), (x3 , y 3 , z3 ) is 1 0 ≡ x1 − x3
= y1 − y 3 z1 − z3 =0
x2 y2 z2 1
x 2 − x3 y 2 − y3 z 2 − z3
x3 y3 z3 1
x y z
(e) Intercept form: The equation of a plane cutting the intercepts a, b, c on the axes is given by + + =1.
 a b c
(f) Vector form: The equation of a plane passing through a point having a position vector a and unit vector
   
normal to plane is (r − a).nˆ =0 ⇒ r.nˆ =a.nˆ
(g) The equation of any plane parallel to the given plane ax + by + cz + d = 0 is given by ax + by + cz + λ = 0
(same direction ratios), where λ is any scalar.
  
(h) The equation of a plane passing through a given point a and parallel to two vectors b and c is given by
      
r ⋅ (b × c) =a ⋅ (b × c) where r is a position vector of any point on the plane.

8.2 Plane Parallel to a Given Plane


The general equation of the plane parallel to the plane ax + by + cz + d = 0 is ax + by + cz + k = 0, where k is any
scalar.
| d1 − d2 |
Distance between two parallel planes ax + by + cz + d1 = 0 and ax + by + cz + d2 = 0 is given by
a2 + b2 + c2
Illustration 7: Find the distance between the planes 2x – y + 2z = 4 and 6x – 3y + 6z = 2. (JEE MAIN)

| d1 − d2 |
Sol: Here if two planes are parallel then the distance between them is equal to .
a2 + b2 + c2
2 7 . 6 | 3D Geometr y

Given planes are 2x – y + 2z – 4 = 0  … (i)


and 6x – 3y + 6z – 2 = 0 … (ii)
a1 b1 c1
We find that = = . Hence, planes (i) and (ii) are parallel
a2 b2 c2
Plane (ii) may be written as 2x – y + 2z – 2/3 = 0 … (iii)
4 − (2 / 3) 10 10
∴ Required distance between the planes
= = =
22 + ( −1)2 + 23 3.3 9

8.3 Plane Passing Through the Line of Intersection of Planes


Let π1 and π2 be the two planes represented by equations
 
r=⋅ nˆ 1 d1 and r = ⋅ nˆ 2 d2 , respectively. The position vector of any point on the 3

line of intersection must satisfy both equations.


   2
If t is the position vector of a point on the line, then t= ⋅ nˆ 1 d1 and t =
⋅ nˆ 2 d2

Therefore, for all real values of λ, we have t ⋅ (nˆ 1 + λnˆ 2= ) d1 + λd2
 1

Because t is arbitrary, it satisfies for any point on the line. Hence, the equation
 Figure 27.4
r ⋅ (nˆ 1 + λnˆ 2=
) d1 + λd2 represents a plane p3 which is such that if any vector
X satisfies the equations of both the planes π1 and π2, it also satisfies the
equation of plane p3.

8.4 Cartesian Form


ˆ n = A ˆi + B ˆj + C kˆ and r = xiˆ + yjˆ + zkˆ .
In a Cartesian system, let n1 = A1ˆi + B1ˆj + C1k, 2 2 2 2

On substituting above values in vector equation we get,


x(A1 + λA2 ) + y(B1 + λB2 ) + z(C1 + λC2=
) d1 + λd2 or (A1 x + B1 y + C1 z – d1) +  (A2 x + B2 y + C2 z – d2) = 0

Illustration 8: Show that the points (0, –1, 0), (2, 1, –1), (1, 1, 1), (3, 3, 0) are coplanar.  (JEE MAIN)

Sol: Equation of any plane passing through (x1 , y1 , z1 ) is given by a(x − x1 ) + b(y − y1 ) + c(z − z1 ) =
0 , by using this
formula we can obtain respective equation of plane.
Let A ≡ (0, –1, 0), B ≡ (2, 1, –1), C ≡ (1, 1, 1) and D ≡ (3, 3, 0)
Equation of a plane through A(0, –1, 0) is a(x – 0) + b(y + 1) + c(z – 0) = 0
⇒ ax + by + cz + b = 0 …(i)
If plane (i) passes through B(2, 1, –1) and C(1, 1, 1)
Then 2a + 2b – c = 0 ...(2) and a + 2b + c = 0 …(iii)
a b c a b c
From equations (ii) and (iii), we have = = or = = = k (say)
2 + 2 −1 − 2 4 − 2 4 −3 2
Substituting values of a, b, c in equation (i), equation of the required plane is 4kx – 3k (y + 1) + 2kz = 0
⇒ 4x – 3y + 2z – 3 = 0 …(iv)
Thus, point D(3, 3, 0) lies on plane (iv).
Because the points on the plane passes through A, B, C, the points A, B, C and D are coplanar.

Illustration 9: Find the equation of the plane upon which the length of normal from the origin is 10 and direction
ratios of this normal are 3, 2, 6. (JEE ADVANCED)
M a them a ti cs | 27.7

Sol: Let p be the length of perpendicular from the origin to the plane and  , m, n be the direction cosines of this
normal. The equation is given by

 x + my + nz = p … (i)
From the data provided, p = 10 and the direction ratios of the normal to the plane are 3, 2, 6.
3 2 6
∴ Direction cosines of normal to the required plane are= =,m = ,n
7 7 7
3 2 6
Substituting values of  , m, n, p in equation (i), equation of the required plane is x+ y+ z =
10
7 7 7
⇒ 3x + 2y + 6z = 70

x y z
Illustration 10: A point P moves on a plane + + = 1. A plane through P and perpendicular to OP meets the
a b c
coordinate axes in A, B and C. If the planes through A, B and C parallel to the planes x = 0, y = 0, z = 0 intersect in
Q, find the locus of Q. (JEE ADVANCED)

Sol: Similar to above problem.


x y z
Given plane is + + = 1 … (i)
a b c
h k 
Let P ≡ (h, k,  ), Then, + + =1 … (ii)
a b c

(OP) = h2 + k 2 + 2
h k 
Direction cosines of OP are , ,
2 2 2 2 2 2
h +k + h +k + h + k 2 + 2
2

∴ Equation of the plane through P and normal to OP is

h k 
x+ y+ = h2 + k 2 + 2
2 2 2 2 2 2 2 2 2
h +k + h +k + h +k +

 h2 + k 2 + 2   h2 + k 2 + 2   h2 + k 2 + 2 
⇒ hx + ky + z = (h2 + k 2 + 2 ), A ≡  ,0,0  , B =  0, ,0  , C ≡  0,0 
 h   k    
     

 h2 + k 2 + 2   h2 + k 2 + 2   h2 + k 2 + 2 
⇒ A = ,0,0  , B =0, ,0  , C  0,0 
 h   k    
     
h2 + k 2 + 2 h2 + k 2 + 2 h2 + k 2 + 2
Let Q ≡ (α , β, =
γ ), then α = ,β = ,γ  … (iii)
h k 

1 1 1 h2 + k 2 + 2 1
Now =
+ + =  … (iv)
2 2 2 2 2 2 2
α β γ (h + k +  ) (h + k 2 + 2 )
2

h2 + k 2 + 2
From equation (iii), h =
α

h h2 + k 2 + 2 k h2 + k 2 + 2  h2 + k 2 + 2
∴ = Similarly = and
a aα b bβ c cγ

h2 + k 2 + 2 h2 + k 2 + 2 h2 + k 2 + 2 h k 
∴ + + = + + = 1  [from equation (ii)]
aα bβ cγ a b c
2 7 . 8 | 3D Geometr y

1 1 1 1 1 1 1
or, + + = = + +  [from equation (iv)]
aα bβ cγ h + k + 
2 2 2
α 2
β2
γ2
1 1 1 1 1 1
∴ Required locus of Q(α, β, γ) is + = = + + .
ax by cz x2 y 2 z 2

8.5 Plane and a Point


(a) A plane divides the three-dimensional space into two equal segments. Two points A(x1 y1 z1) and B(x2 y2 z3) lie
on the same sides of the plane ax + by + cz + d = 0 if the two expressions ax1 + by1 + cz1 + d and ax2 + by2
+ cz3 + d are of same sign, and lie on the opposite sides of plane if both of these expressions are of opposite
sign.

(b) Perpendicular distance of the point (x’, y’, z’) from the plane ax + by + cz + d = 0 is given by
ax'+ by '+ cz'+ d
.

a2 + b2 + c2
 
(c) The length of the perpendicular from the point having a position vector a to the plane r.n = d is given by

| a.n − d |
P= 
|n|

(d) The coordinates of the foot of the perpendicular from the point (x1, y1, z1) to the plane ax + by + cz + d =0 are
x'− x1 y '− y 1 z'− z1 (ax1 + by1 + cz1 + d)
given by = = = −
a b c a2 + b2 + c2

(e) If P’(x’, y’, z’) is the image of a point P(x1, y1, z1) w.r.t. the plane ax + by + cz + d = 0, then

x'− x1 y '− y 1 z'− z1 (ax1 + by1 + cz1 + d)


= = = −2
a b c a2 + b2 + c2

PLANCESS CONCEPTS

The distance between two parallel planes ax + by + cx + d = 0 and ax + by + cx + d’ = 0 is given by


| d − d' |
a2 + b2 + c2
If a variable point P moves so that PA2 – PB2 = K, where K is a constant and A and B are the two points,
then the locus of P is a plane.
Vaibhav Gupta (JEE 2009 AIR 54)

Illustration 11: Show that the points (1, 2, 3) and (2, –1, 4) lie on the opposite sides of the plane x + 4y + z – 3 = 0.
 (JEE MAIN)

Sol: Substitute given points in to the given equation of plane, if their values are in opposite sign then the points
are on opposite sides of the plane.
Since 1 + 4 × 2 + 3 – 3 = 9 and 2 – 4 + 4 – 3 = –1 are of opposite sign, the points are on opposite sides of the plane.
M a them a ti cs | 27.9

8.6 Angle Between Two Planes


Let us consider two planes ax + by + cz + d = 0 and a’x + b’y + c’z + d = 0. Angle between these planes is the
angle between their normals. Let (a, b, c) and (a’, b’, c’) be the direction ratios of their normals of the two planes,
respectively, and the angle θ between them is given by
aa'+ bb'+ cc'
cos θ = .
a + b2 + c2 a'2 + b'2 + c'2
2

a b c
The planes are perpendicular if aa’ + bb’ + cc’ = 0 and the planes are parallel if = = .
a' b' c'
 
    n1 .n2
In vector form, if θ is the angle between the=planes r .n d= 1 and r .n2 d2 , then cos θ =   . The planes are
    | n1 | ⋅ | n2 |
perpendicular if n1 ⋅ n2 = 0 and the planes are parallel if n1 = λn2 .

8.6.1 Angle Bisectors


(a) Equations of the planes bisecting the angle between the two given planes a1x + b1y + c1z + d1 = 0 and a2x +
b2y + c2z + d2 = 0 are
a1 x + b1 y + c1 z + d1 a2 x + b2 y + c2 z + d2
= ±
a12 + b12 + c12 a22 + b22 + c22

(b) Equation of bisector of the angle containing the origin is given by


a1 x + b1 y + c1 z + d1 a2 x + b2 y + c2 z + d2
= [where d1 and d2 are positive]
a12 + b12 + c12 a22 + b22 + c22

(c) In order to find the bisector of acute/obtuse angle, both the constant terms should be positive. If
a1 a2 + b1 b2 + c1 c2 > 0 Þ then the origin lies in the obtuse angle
a1 a2 + b1 b2 + c1 c2 < 0 Þ then the origin lies in the acute angle
now apply step (ii) according to the question.

8.7 Family of Planes


(a) The equation of any plane passing through the line of intersection of nonparallel planes or through the given line is
a1x + b1y + c1z + d1 = 0 and a2x + b2y + c2z + d2 = 0, i.e. P1 = 0 and P2 = 0
a1x + b1y + c1z + d1 + λ(a2x + b2y + c2z + d2) = 0, i.e. P1 + λP2 = 0
   
(b) The equation of plane passing through the intersection of the planes
= r .n1 d=
1 & r .n2 d2 is
r .(n1 + λn2=
) d1 + λd2 , where λ is an arbitrary scalar.

Illustration 12: The plane x – y – z = 4 is rotated through 90° about its line of intersection with the plane x + y +
2z = 4. Find its equation in the new position.  (JEE MAIN)

Sol: As the required plane passes through the line of intersection of given planes, therefore its equation may be
taken as x + y + 2z – 4 + k(x – y – z – 4) = 0
⇒ (1 + k) x + (1 – k) y + (2 – k) z – 4 – 4k = 0 … (iii)
Thus, planes (i) and (iii) are mutually perpendicular.
∴ (1 + k) – (1 – k) – (2 – k) = 0 ⇒ 1 + k – 1 + k – 2 + k = 0 ⇒ k = 2/3
Substituting k = 2/3 in equation (iii), we get, 5x + y + 4z = 20. This is the required equation of the plane in its new
position.
2 7 . 1 0 | 3D Geometr y

Illustration 13: Find the equation of the plane through the point (1, 1, 1) and passing through the line of intersection
of the planes x + y + z = 6 and 2x + 3y + 4z + 5 = 0 (JEE MAIN)

Sol: Similar to above illustration.


Given planes are x + y + z – 6 = 0 … (i)
and 2x + 3y + 4z + 5 = 0  … (ii)
Given point is P(1, 1, 1).
Equation of any plane passing through the line of intersection of the planes (i) and (ii) is
x + y + z – 6 + k (2x + 3y + 4z + 5) = 0  … (iii)
If plane (iii) passes through a point P, then 1 + 1 + 1 – 6 + k (2 + 3 + 4 + 5) = 0
3
⇒ k=
14
From equation (i), the required plane is 20x + 23y + 26z – 69 = 0

Illustration 14: If the planes x – cy – bz = 0, cx – y + az = 0 and bx + ay – z = 0 pass through a straight line, then
find the value of a2 + b2 + c2 + 2abc. (JEE ADVANCED)

Sol: Here the plane passing through the line of intersection of planes x – cy – bz = 0 and cx – y + az = 0 is same as
the plane bx + ay – z = 0. Hence by using family of planes we can obtain required result.
Given planes are x – cy – bz = 0  … (i)
cx – y + az = 0  … (ii)
bx + ay – z = 0  … (iii)
Equation of any plane passing through the line of intersection of the planes (i) and (ii) may be written as
x – cy – bz + λ(cx – y + az) = 0 ⇒ x(1 + lc) – y(c + λ) + z(–b + aλ) = 0  … (iv)
If planes (3) and (4) are the same, then equations (iii) and (iv) will be identical.

1 + cλ −(c + λ ) −b + aλ
∴= = ;
b a −1
(i) (ii) (iii)
From equations (i) and (ii), a + acλ = –bc – bl
(a + bc)
⇒ λ=−  … (v)
(ac + b)

From equations (ii) and (iii), c + λ = −ab + a2 λ


(ab + c)
⇒ λ=−  … (vi)
1 − a2
−(a + bc) −(ab + c)
From equations (v) and (vi), we have, =
ac + b (1 − a2 )
⇒ a – a3 + bc – a2bc = a2bc + ac2 + ab2 + bc ⇒ 2a2bc + ac2 + ab2 + a3 – a = 0
⇒ a2 + b2 + c2 + 2abc = 1

Illustration 15: Through a point P(h, k,  ), a plane is drawn at right angles to OP to meet the coordinate axes in A,
p5
B and C. If OP = p, show that the area of ∆ABC is . (JEE ADVANCED)
2hk
Sol: Here line OP is normal to the plane, therefore  x + my nz = p. where  , m and n are direction cosines of given
plane.
M a them a ti cs | 27.11

OP = h2 + k 2 + 2 = p
h k 
Direction cosines of OP are , ,
h2 + k 2 + 2 h2 + k 2 + 2 h2 + k 2 + 2
Since OP is the normal to the plane, therefore, equation of the plane will be
h k 
x+ y+ z= h2 + k 2 + 2
2 2 2 2 2 2 2 2 2
h +k + h +k + h +k +

⇒ hx + ky +  z = h2 + k2 +  2 = p2

 p2   p2   p2 
=∴ A = ,0,0  ,B =0, ,0  ,C  0,0, 
 h   k    
    
 
AB × AC
Thus, area of ∆ABC =
2

 p2 p2   p2 p2   p4 p4 ˆ p4 ˆ 
 ˆi − ˆj  ×  ˆi − kˆ   ˆj + k+ i
 h k   h    h kh k 
= =
2 2

1 8 1 1 1  P8 p5
=
2
p 
2 2
h 
+
2 2
hk
=+
2 2 
k  
1
2 h2 2 k 2
( =
2
+h +k ) 2 2
2hk

9. TETRAHEDRON
Volume of a tetrahedron given the coordinates of its vertices A(x1, y1, z1), B(x2, y2, z2), C(x3, y3, z3) and D(x4, y4, z4) can
be calculated by

x1 y1 z1 1
x − x1 y 2 − y1 z 2 − z1
1 x y2 z2 1 1 2
V= 2 = x3 − x1 y 3 − y1 z3 − z1
6 x3 y3 z3 1 6
x 4 − x1 y 4 − y1 z 4 − z1
x4 y4 x4 1

PLANCESS CONCEPTS

Four points (xr, yr, zr); r = 1, 2, 3, 4; will be coplanar if the volume of the tetrahedron with the points as
vertices is zero. Therefore, the condition of coplanarity of the points

x2 − x1 y 2 − y1 z 2 − z1
(x1, y1, z1), (x2, y2, z2), (x3, y3, z3) and (x4, y4, z4) is x3 − x1 y 3 − y1 z3 − z1 =0.
x 4 − x1 y 4 − y1 z 4 − z1

Vaibhav Gupta (JEE 2009 AIR 54)


2 7 . 1 2 | 3D Geometr y

Centroid of a Tetrahedron
Let A(x1, y1, z1), B(x2, y2, z2), C(x3, y3, z3) and D(x4, y4, z4) be the vertices of a tetrahedron.
 Σx Σy Σz 
The coordinate of its centroid (G) is given as  i , i , i 
 4 4 4 
Illustration 16: If two pairs of opposite edges of a tetrahedron are mutually perpendicular, show that the third pair
will also be mutually perpendicular.  (JEE MAIN)

Sol: If two lines are perpendicular then summation of product of their respective direction ratios is equals to zero.
Let OABC be the tetrahedron where O is the origin and coordinate of A, B, C be (x1, y1, z1), (x2, y2, z2), (x3, y3, z3),
respectively.
Let OA ⊥ BC and OB ⊥ CA. We have to prove that OC ⊥ BA
A(x1, y1, z1 )
Direction ratios of OA are x1 – 0, y1 – 0, z1 – 0 or x1, y1, z1

Direction ratios of BC are (x3 – x2), (y3 – y2), (z3 – z2)


O (0, 0, 0)
OA ⊥ BC
⇒ x1 (x3 − x2 ) + y1 (y 3 − y 2 ) + z1 (z3 + z 2 ) =
0 ... (i) B C
Figure 27.5
Similarly, OB ⊥ CA
⇒ x2 (x1 − x3 ) + y 2 (y1 − y 3 ) + z 2 (z1 − z3 ) =
0 ... (ii)
On adding equations (1) and (2), we obtain the following equation:
x3 (x1 − x2 ) + y 3 (y1 − y 2 ) + z3 (z1 − z 2 ) =
0

∴ OC ⊥ BA [ direction ratios of OC are x3, y3, z3 and that of BA are (x1 – x2), (y1 – y2), (z1 – z2)]

10. LINE

10.1 Equation of a line


A straight line in space will be determined if it is the intersection of two given nonparallel planes and therefore, the
equation of a straight line is present as a solution of the system constituted by the equations of the two planes, a1x
+ b1y + c1z + d1 = 0 and a2x + b2y + c2z + d2 = 0. This form is also known as non-symmetrical form.

(a) The equation of a line passing through the point (x1, y1, z1) with a, b, c as direction ratios is
x − x1 y − y1 z − z1
= = = r. This form is called symmetrical form. A general point on the line is given by (x1 +
a b c
ar, y1 + br, z1 + cr).

(b) Vector equation of a straight line passing through a fixed point with position vector a and parallel to a given
   
vector b is r = a + λb , where λ is a scalar.

(c) The equation of the line passing through the points (x1, y1, z1) and (x2, y2, z3) is
x − x1 y − y1 z − z1
(d) = = .
x2 − x1 y 2 − y1 z 2 − z1
     
(e) Vector equation of a straight line passing through two points with position vectors a and b is r = a + λ(b − a).

(f) Reduction of Cartesian form of equation of a line to vector form and vice versa is as

x − x1 y − y1 z − z1
(g) = = ⇔ r (x1ˆi + y1ˆj + z1k)
= ˆ + λ(aiˆ + bjˆ + ck)
ˆ
a b c
M a them a ti cs | 27.13

PLANCESS CONCEPTS

Straight lines parallel to coordinate axes


Straight lines Equations
(i) Through origin y = mx, z = nx
(ii) x-axis y = 0, z = 0
(iii) y-axis x = 0, z = 0
(iv) z-axis x = 0, y = 0
(v) Parallel to x-axis y = p, z = q
(vi) Parallel to y-axis x = h, z = q
(vii) Parallel to z-axis x = h, y = p
The number of lines which are equally inclined to the coordinate axes are 4.
Vaibhav Krishnan (JEE 2009 AIR 22)

Illustration 17: Find the equation of the line passing through the points (3, 4, –7) and (1, –1, 6) in vector form as
well as in Cartesian form.  (JEE MAIN)

r (x1ˆi + y1ˆj + z1k)


Sol: Here line in vector form is given by= ˆ + λ(aiˆ + bjˆ + ck)
ˆ and in Cartesian form is given by
x − x1 y − y1 z − z1
= = .
a b c
 →  →
Let A ≡ (3, 4, –7), B ≡ (1, –1, 6); Now, a = OA = 3iˆ + 4ˆj − 7k,b
ˆ = OB = ˆi − ˆj + 6kˆ
  
Equation (in vector form) of the line passing through A(a) and B( b ) is r =a + t(b − a)
      
⇒ r = 3i + 4 j − 7k + t( −2 i − 5 j + 13k)  … (i)
x −3 y − 4 z+7 x −3 y − 4 z +7
Equation in Cartesian form is = = ⇒ = =
3 − 1 4 + 1 −7 − 6 2 5 −13

x −1 y − 2 z − 3 x − 4 y −1
Illustration 18: Show that the two lines = = and = = z intersect. Find also the point of
intersection of these lines. 2 3 4 5 2 (JEE MAIN)

Sol: The given lines will intersect if any point on respective lines coincide for some value of λ and r.
x −1 y − 2 z −3
Given lines are = =  .… (i)
2 3 4
z − 4 y −1 z − 0
and = =  .… (ii)
5 2 1
Any point on line (1) is P(2r + 1, 3r + 2, 4r + 3) and any point on line (2) is Q(5λ + 4, 2λ + 1, λ)
Lines (i) and (ii) will intersect if P and Q coincide for some value of λ and r.
⇒ 2r + 1 = 5λ + 4 ⇒ 2r – 5λ = 3 … (iii)
⇒ 3r + 2 = 2λ + 1 ⇒ 3r – 2λ = –1 … (iv)
⇒ 4r + 3 = λ ⇒ 4r –λ = –3 … (v)
Solving equations (iii) and (iv), we get r = –1, λ = –1; these obtained values of r and λ clearly satisfy equation (v)
⇒ P ≡ (–1, –1, –1). Hence, lines (i) and (ii) intersect at (–1, –1, –1)
2 7 . 1 4 | 3D Geometr y

Illustration 19: Find the angle between the lines x – 3y – 4 = 0, 4y – z + 5 = 0 and x + 3y – 11 = 0, 2y – z + 6 = 0.


 (JEE MAIN)
x − 4 y − 0 z −5
Sol: = =  … (i)
3 1 4
x − 11 y − 0 z − 6  … (ii)
= =
−3 1 2
a = 3, b = 1, c = 4 ∴ a1 =
−3,b1 =
1, c1 =
2

aa1 + bb1 + cc1 =−9 + 1 + 8 =0 ⇒ cos θ =0 θ =90

Illustration 20: Find the equation of the line drawn through point (1, 0, 2) to meet at right angle with the line
x +1 y −2 z +1
= = .  (JEE ADVANCED)
3 −2 −1

Sol: If two lines are perpendicular then summation of product of their direction ratios are equal to zero. Hence by
obtaining direction ratio of these line, we will be get the result.
x +1 y −2 z +1
Given line is = =  … (i)
3 −2 −1
Let P ≡ (1, 0, 2); coordinates of any point on line (i) may be written as Q≡ (3r – 1, –2r + 2, –r –1).

Direction ratios of PQ are 3r – 2, –2r + 2, –r –3


x +1 y −2 z +1
Direction ratios of the line = = are 3, –2, –1
3 −2 −1
Since PQ ⊥ to the line ⇒ 3(3r – 2) – 2(–2r + 2) – 1 (–r – 3) = 0
1 1 7
⇒ 9r – 6 + 4r – 4 + r + 3 = 0 ⇒ 14r = 7 ⇒ r = ∴ Direction ratios of PQ are − ,1, −
2 2 2
x −1 y − 0 z − 2
Hence, equation of the line PQ is = =
−1 2 −7

Illustration 21: Find the equation of the line of intersection of the planes 4x + 4y – 5z = 12, 8x + 12y – 13z = 32 in
the symmetric form (JEE ADVANCED)

Sol: Consider the line of intersection meet the xy-plane at P(α, β, 0), therefore obtain the value of α and β and
direction ratios of line of intersection to solve the problem.
Given planes are 4x + 4y – 5z – 12 = 0 … (i)
and 8x + 12y – 13z – 32 = 0  … (ii)

Let  , m, n be the direction ratios of the line of intersection. Then,


 m n  m n
4  + 4m – 5n = 0 and 8  + 12m – 13n = 0 ⇒ = = ⇒ = =
−52 + 60 −40 + 52 48 − 32 2 3 4
Direction ratios of the line of intersection are 2, 3, 4.
Let the line of intersection meet the xy-plane at P(α, β, 0)
Then P lies on planes (i) and (i) ⇒ 4α + 4β – 12 = 0 ⇒ α+β–3=0
and 8α + 12β – 32 = 0 … (v)
or 2α + 3β – 8 = 0 … (vi)
α β 1
Solving equations (v) and (vi), we get = = ⇒ α = 1, β = 2
−8 + 9 −6 + 8 3 − 2
M a them a ti cs | 27.15

x −1 y − 2 z − 0
Hence, equation of the line of intersection in symmetrical form is = =
2 3 4

10.1 Coplanar Lines


Coplanar lines are lines that entirely lie on the same plane.

x −α y −β z − γ x − α ' x − β' z − γ '


(i) If = = and = = , are the lines, then the condition for intersection/coplanarity
 m n ' m' n'
α − α ' β − β' γ − γ ' x −α y −β z − γ
is  m n = 0 and the plane containing the aforementioned lines is  m n = 0.
' m' n' ' m' n'

(ii) Condition of coplanarity if both lines are in general form.


Let the lines be ax + by + cz + d = 0 = a’x + b’y + c’z + d’ and ax + by + gz + δ = 0 = α’x+ β’y + γ’z + δ’ = 0
a b c d
a' b' c' d'
If ∆ = = 0 , then they are coplanar.
α β γ δ
α ' β' γ ' δ'

10.2 Skew Lines


Skew lines are two lines that do not intersect and are not parallel.

α '− α β '− β γ '− γ


If ∆  m n ≠ 0, then the lines are skew.
' m' n'

Shortest distance
x −α y −β z − γ x − α ' x − β' z − y '
Let the equation of the lines be = = and = =
 m n ' m' n'

α '− α β '− β γ '− γ


(α − α ')(mn' − m'n) + (β − β ')(n '− n'  ) + ( γ −  ')(m'−  'm)
S.D. = =  m n ∑ (nm'− m'n)2
∑ (mn'− m'n)2 ' m' n'

Vector form
         
For lines a1 + λb1 and a2 + λb2 to be skew, the following condition should be satisfied: (b1 × b2 ) ⋅ (a1 − a2 ) ≠ 0 or 
b1 b

PLANCESS CONCEPTS

Shortest distance between two skew lines is perpendicular to both the lines.
Anvit Tawar (JEE 2009 AIR 9)
2 7 . 1 6 | 3D Geometr y

10.3 Intersecting Lines


Two or more lines that intersect at a point are called intersecting lines, and their shortest distance between the two
lines is
    x2 − x1 y 2 − y1 z 2 − z1
(b1 × b2 ) ⋅ (a2 − a1 )        
zero, i.e.   = 0 ⇒ (b1 × b2 ) ⋅ (a1 − a2 ) =0 ⇒ [b1b2 (a2 − a1 )] =0 ⇒ a1 b1 c1 0
=
| b1 × b2 |
a2 a2 c2

10.4 Parallel Lines


T(a2)
Parallel lines are lines that never intersect, and are coplanar. 2

8 1 16 8 1 16
Let − , , and , , be the two parallel lines.
3 3 3 3 3 3
   P
Let the lines be given by r= a1 + λb  … (i) 1
  
r= a2 + µb  … (ii) Figure 27.6
 x y −1 z
where a1 is the position vector of a point S on = = and a2 is the position vector of a point T on  2 . As
2 3 5
1 and  2 are coplanar, if the foot of the perpendicular from T on the line 1 is P, then the distance between the
→ 
lines 1 and  2 = |TP|. Let θ be the angle between the vectors ST and b .
 →  →
Then= b × ST (| b ||ST |sin θ)nˆ  … (iii)
where n̂ is the unit vector perpendicular to the plane of the lines a + bm + cn = 0 and ax1 + by1 + cz1 + d ≠ 0 . But
→  
ST
= a2 − a1
   
Therefore, from equation (iii), we get b × (a2 − a1 ) = | b |PTnˆ (as PT = ST sin θ)   
    → b × (a2 − a1 )
⇒ | b × (a2 − a1 )=| | b |PT ⋅ 1(as| nˆ=| 1) Hence, the distance between the given parallel lines is=
d | PT
=|  .
|b|

10.5 Angular Bisector


If a(x − α ) + b(y − β) , m1, n1 and +c(z − γ ) =0, , m2, n2 are the direction cosines of the two lines inclined to each other
at an angle θ, then the direction cosines of the
1 +  2 m + m2 n + n2
(a) Internal bisectors of the angle between the lines are , 1 and 1 .
2cos(θ / 2) 2cos(θ / 2) 2cos(θ / 2)
1 −  2 m1 − m2 n1 − n2
(b) External bisectors of the angle between the lines are , and .
2sin(θ / 2) 2sin(θ / 2) 2sin(θ / 2)

10.6 Reduction to Symmetric Form


Let the line in nonsymmetrical form be represented as a1x + b1y + c1z + d1 = 0, a2x + b2y + c2z + d2 = 0.
To find the equation of the line in symmetrical form, (i) its direction ratios and (ii) coordinate of any point on it must
be known.

(a) Direction ratios: Let  , m, n be the direction ratios of the line. Since the line lies on both planes, it must
be perpendicular to the normal of both planes. So a1  + b1m + c1n = 0, a2  + b2m + c2n = 0. From these
equations, proportional values of  , m, n can be found by using the method of cross-multiplication, i.e.
 m n
= =
b1c2 − b2c1 c1a2 − c2a1 a1b2 − a2b1

M a them a ti cs | 27.17

Alternate method

i j k
The vector a1 b1 c1 = i(b1c2 − b2c1 ) + j(c1a2 − c2a1 ) + k (a1b2 − a2b1 ) will be parallel to the line of intersection of the
a2 b2 c2
two given planes. Hence,  : m : n = (b1c2 – b2c1): (c1a2 – c2a1):(a1b2 – a2b1).

(b) Coordinate of any point on the line: Note that as  , m, n cannot be zero simultaneously, so at least one
must be nonzero. Let a1b2 – a2b1 ≠ 0, so that the line cannot be parallel to xy-plane, and will intersect it. Let it
intersect xy-plane at the point (x1, y1, 0). These x − x1a1x1 + b1yy−1 +
y1 d1 = 0 and
z − 0a2x1 + b2y1 + d2 = 0. Solving these, we
get a point on the line. Thus, we get theb= following =
equation: or
1 c2 − b2 c1 c1a2 − c2a1 a1b2 − a2b1

x − x1 y − y1 z−0 x − (b1d2 − b2d1 / a1b2 − a2b1 ) y − (d1a2 − d2a1 / a1b2 − a2b1 ) z−0
= = oror = =
b1c2 − b2c1 c1a2 − c2a1 a1b2 − a2b1 b1c2 − b2c1 c1a2 − c2a1 a1b2 − a2b1
x − (b1d2 − b2d1 / a1b2 − a2b1 ) y − (d1a2 − d2a1 / a1b2 − a2b1 ) z−0
= =
Note: If  ≠ 0,b1take
c2 − ba2cpoint
1
on yz-plane as (0,
c a y− , z
c
1 2 1 21 1
)
a and if m ≠ 0,
a take a point on xz-plane as (x1, 0, z1).
1 2 − a2b1
b

Alternate method:

a1 b1
If ≠ , then put z = 0 in both equations and solve the equation a1x + b1y + d1 = 0, a2x + b2y + d2 = 0 or put y = 0
a2 b2

and solve the equation a1x + c1z + d1 = 0 and a2x + c2z + d2 = 0.

10.7 Point and Line


Foot Length and Equation of Perpendicular from a Point to a Line
x −a y −b z −c
Cartesian form: Let equation of the line be = = = r (say) …(i)
 m n
and A(α, β, γ ) be the point. Any point on line (i) is P(lr + a, mr + b, nr + c). If it is the foot of the perpendicular from
point A on the line, then AP is perpendicular to the line.

⇒  (  r + a – α) + m(mr + b – β) + n(nr + c – γ) = 0, i.e. r = [(α – a)  + (β – b) m + (γ – c)n]/ l2 + m2 + n2

Using this value of r, we get the foot of the perpendicular from point A on the given line. Because the foot of the

AP
perpendicular P is known, the length of the perpendicular = (r + a − α2 ) + (mr + b − β)2 + (nr + c − γ )2 is given
x−α y −β z−γ
by the equation of perpendicular as = =
r + a − α mr + b − β nr + c − γ

Illustration 22: Find the coordinates of the foot of the perpendicular drawn from the point A(1, 2, 1) to the line
joining B(1, 4, 6) and C(5, 4, 4). (JEE ADVANCED)

Sol: Using section formula we will get co-ordinates of the foot D, and as AD is perpendicular to BC therefore
 
AD.BC : = 0.
A(1, 2, 1)
Let D be the foot of the perpendicular drawn from A on BC,
and let D divide BC in the ratio k:1. Then, the coordinates
 5k + 1 4k + 4 4k + 6 
of D are  , ,  …(i)
 k +1 k +1 k +1 
k 1
 B(1, 4, 6) D C(5, 4, 4)
Now, AD = Position vector of D – Position vector of A
Figure 27.7
2 7 . 1 8 | 3D Geometr y

 5k + 1   4k + 4   4K + 6   4k  ˆ  2k + 2  ˆ  3k + 5  ˆ
= − 1  ˆi +  − 2  ˆJ +  − 1  Kˆ = i +  j+  k
 k +1   k +1   K +1  k +1  k +1   k +1 

and BC = Position vector of C – Position vector of B = (5iˆ + 4ˆj + 4k)
ˆ − (iˆ + 4ˆj + 6k)
ˆ = 4iˆ + 0ˆj − 2kˆ
     4k  ˆ  2k + 2  ˆ  3k + 5  ˆ  ˆ ˆ
since AD ⊥ BC ⇒ AD.BC = 0 ⇒  i +  j+ ˆ =
 k  .(4i + 0 j − 2k) 0
 k + 1   k + 1   k + 1  
 4k   2k + 2   3k + 5  16k (3k + 5)
⇒ 4  + 0  − 2 =0 ⇒ +0−2 =0
 k + 1   k + 1   k + 1  k + 1 k +1

⇒ 16k − 6k − 10 =
0 ⇒ k=1

Substituting k = 1 in equation (i), we obtain the coordinates of D as (3, 4, 5).

10.8 Vector Form


    
Equation of a line passing through a point having position vector α and perpendicular to the lines r1= a1 + λb1 and r1= a2 + λ
           
r1= a1 + λb1and
and r1= a2 + λb2 is parallel to b1 × b2 . So the vector equation of such line is r = α + λ(b1 × b2 ). The equation of the
   
    (a − α
  )b
perpendicular passing through α is r = α + µ  (a − α −    b. 
  | b |2  
   

10.9 Image w.r.t. the Line


x − x2 y − y2 z − z2
Let L ≡ = = be the given line.
a b c
Let (x’, y’, z’) be the image of the point P(x1, y1, z1) w.r.t the line L. Then

(i) a(x1 − x') + b(y1 − y ') + c(z1 − z') =


0

x1 − x' y1 − y ' z1 − z'
− x2 − y2 − z2
(ii) 2 = 2 = 2 = λ
a b c
From (ii), the value of x’, y’, z’ in terms of λ can be obtained as x’ = 2aλ + 2x2 – x1, y’ = 2bα – 2y2 – y1,z’ = 2cλ + 2z2 – z1
On substituting values of x’, y’, z’ in (i), we get λ and on re-substituting value of λ, we get (x’ y’ z’).

x +1 y −3 z + 2
Illustration 23: Find the length of the perpendicular from P(2, –3, 1) to the line = =  (JEE MAIN)
2 3 −1
Sol: Here Co-ordinates of any point on given line may be taken as Q ≡ (2r – 1, 3r + 3, –r – 2), therefore by using
distance formula we can obtain required length.
x +1 y −3 z + 2
Given line is = = …(i) P = (2, –3, 1)
2 3 −1
Coordinates of any point on line (i) may be written as Q ≡ (2r – 1, 3r + 3, –r – 2)
Direction ratios of PQ are 2r – 3, 3r + 6, –r – 3.
Direction ratios of AB are 2, 3, –1.
−15
Since PQ ⊥ AB ⇒ 2(2r – 3) + 3(3r + 6) – 1(–r – 3) = 0 ⇒ r=
14
 −22 −3 −13  531
⇒ Q= , ,  ⇒ PQ = units
 7 14 14  14
M a them a ti cs | 27.19

10.10 Plane Passing Through a Given Point and Line 


A(a2)
   
Let the plane pass through the given point A( a ) and line r = b + λc . For any position R(r)
  
of point R (r) on the plane, vectors AB,RA and c are coplanar.
    
Then  r − a b − a c  = 0 , which is the required equation of the plane. B(b) c

Figure 27.8
Angle between a plane and a line:
Angle between a line and a plane is complementary to the angle made by the line with the normal of plane. Hence,
x − x1 y − y1 z − z1
if θ is the angle between the line = = and the plane
 m n
 a + bm + cn 
ax + by + cz + d = 0, then sin θ = 
 (a2 + b2 + c2 ) (2 + m2 + n2 ) 
 

Vector form:
 
      b ⋅n 
If θ is the angle between the line r= (a + λ b) and r ⋅ n= d then sin θ=    
  | b || n | 
 m n 
Line and plane are perpendicular if = = , i.e. b × n = 0.
a b c
 
Line and plane are parallel if a  + bm + cn = 0, i.e. b ⋅ n = 0.

PLANCESS CONCEPTS

Condition for a Line to Lie on a Plane


x − x1 y − y1 z − z1
(i) Cartesian form: Line = = would lie on a plane ax + by + cz + d = 0 if,
 m n
ax1 + by1 + cz1 + d = 0 and a  + bm + cn = 0
     
⋅ nˆ d if b =
(ii) Vector form: Line r = a + λb + would lie on the plane r = ⋅ nˆ 0 & a=⋅ nˆ d.
The number of lines which are equally inclined to the coordinate axes is 4.
1
If  , m, n are the d.c.s of a line, then the maximum value of lmn = .
3 3
Akshat Kharaya (JEE 2009 AIR 235)

Illustration 24: Find the shortest distance and the vector equation of the lines of shortest distance between the
             
lines given by r = 3i + 8 j + 3k + λ (3i − j + k) and r =−3i − 7 j + 6k + µ( −3i + 2 j + 4k) . (JEE ADVANCED)

Sol: Consider LM is the shortest distance between given lines therefore LM is perpendicular to these lines, hence
by obtaining their direction ratios and using perpendicular formula we will get the result.
      
Given lines are r= 3i + 8 j + 3k + λ (3i − j + k)  … (i)
      
and r = −3i − 7 j + 6k + µ ( −3i + 2 j + 4k)  … (ii)
Equations of lines (i) and (ii) in Cartesian form are
x −3 y −8 z −3 x+3 y +7 z−6
AB : = = = λ and CD : = = = µ
3 −1 1 −3 2 4
Let L ≡ (3λ + 3, −λ + 8, λ + 3) and M ≡ ( −3µ − 3, 2µ − 7, 4µ + 6)
2 7 . 2 0 | 3D Geometr y

Direction ratios of LM are 3λ + 3µ + 6, − λ − 2µ + 15, λ − 4µ − 3 since LM ⊥ AB


⇒ 3(3λ + 3µ + 6) − 1( −λ − 2µ + 15) + 1(λ − 4µ − 3) = 0
or, 11λ + 7µ =0  … (v)
Again LM ⊥ CD
∴ −3(3λ + 3µ + 6) + 2( −λ − 2µ + 15) + 4(λ − 4µ − 3) = 0
or, −7λ − 29µ =0  … (vi)
Solving equations (v) and (vi), we get λ= 0, µ= 0 ⇒ L ≡ (3, 8, 3), M ≡ (–3, –7, 6)

Hence, the shortest distance LM = (3 + 3)2 + (8 + 7)2 + (3 − 6)= 2


270
= 3 30 units
      
Vector equation of LM is r = 3i + 8 j + 3k + t ( −6 i + 15 j − 3k).

11. SPHERE
11.1 General Equation
The general equation of a sphere is given by x2 + y2 + z2 + 2ux + 2vy + 2wz + d = 0, where
(–u, –v, –w) is the center and u2 + u2 + w2 − d is the radius of the sphere.
A L B
90o
11.2 Diametric Form
90o
If (x1, y1, z1) and (x2, y2, z2) are the coordinates of the extremities of a diameter of a C M D
sphere, then its equation is given by (x – x 1) (x – x 2) + (y – y 1) (y – y 2) + (z – z1)
Figure 27.9
(z – z2) = 0.

11.3 Plane and Sphere


If the perpendicular distance of the plane from the center of the sphere is equal to the
radius of the sphere, then the plane touches the sphere. The plane lx + my + nz
= p touches the sphere x2 + y2 + z2 + 2ux + 2vy + 2wz + d = 0, if (u  + vm + wn + p)2 = (l2 + m2 + n2) (u2 + v2
+ w2 – d).

11.4 Intersection of Straight Line and a Sphere


Let the equations of the sphere and the straight line be
x2 + y2 + z2 + 2ux + 2vy + 2wz + d = 0 … (i)
x −α y −β z − γ
and = = = r, (say)  … (ii)
 m n
Any point on the line (ii) is (α + lr, β + mr, γ + nr). If this point lies on the sphere (i), then we have

(α + r)2 + (β + mr)2 + ( γ + nr)2 + 2u(α + r) + 2v(β + mr) + 2w( γ + nr) + d = 0

⇒ r 2 [2 + m2 + n2 ] + 2r[(u + α ) + m(v + β) + n(w + γ )] + (α2 + β2 + γ 2 + 2uα + 2vβ + 2wγ + d) = 0  … (iii)

This is a quadratic equation in r and thus two values of r are obtained. Therefore, the line (ii) intersects the sphere
(i) at two points which may be real, coincident and imaginary, according to roots of (iii).

If  , m, n are the actual d.c.s of the line, then l2 + m2 + n2 = 1 and then the equation (iii) can be simplified.
M a them a ti cs | 27.21

11.5 Orthogonality of Two Spheres


Let the equation of the two spheres be x2 + y2 + z2 + 2ux + 2vy + 2wz + d = 0 … (i)
and x + y + z + 2u’x + 2v’y + 2w’z + d’ = 0
2 2 2
… (ii)

If the sphere (i) and (ii) cut orthogonally, then 2uu’ + 2vv’ + 2ww’ = d + d’, which is the required condition. If the
spheres x2 + y2 + z2 = a2 and x2 + y2 + z2 +2ux + 2vy + 2wz + d = 0 cut orthogonally, then d = a2.
r1r2
Two spheres of radii r1 and r2 cut orthogonally, then the radius of the common circle is .
r12 + r22
Illustration 25: A plane passes through a fixed point (a, b, c). Show that the locus of the foot of the perpendicular
to it from the origin is the sphere x2 + y2 + z2 – ax – by – cz = 0 (JEE ADVANCED)

Sol: Consider P(α, β, γ ) be the foot of perpendicular from origin to, therefore by getting the direction ratios of OP
we will get the required result.
Let the equation of the variable plane be O(0, 0, 0)

 x + my + nz + d = 0 … (i)

Plane passes through the fixed point (a, b, c) P(, , )

∴  a + mb + nc + d = 0 … (ii)
Figure 27.10
Let P(α, β, γ ) be the foot of the perpendicular from the origin to plane (i)

Direction ratios of OP are α – 0, β – 0, γ – 0, i.e. α, β, g

From equation (i), it is clear that the direction ratios of the normal to the plane, i.e. OP are  , m, n, and α, β, γ are
α β γ 1
the direction ratio of the same line OP ∴ = = = (say) ;  =kα ,m =kβ,n =kγ  … (iii)
 m n k
Substituting values of  , m, n from equation (iii) in equation (ii), we get, kaα + kbβ + kcγ + d = 0  … (iv)
Since α, β, γ lies on plane (i) ∴ α + mβ + nγ + d = 0  … (v)
Substituting values of  , m, n from equation (iii) in equation (v), we get kα2 + kβ2 + kγ 2 + d =0 ... (vi)
2 2 2
[substituting value of d from equation (iv) in equation (vi)] or α + β + γ − aα − bβ − cγ = 0

Therefore, locus of foot of the perpendicular from the point P(α, β, γ) is x2 + y 2 + z 2 − ax − by − cz =


0

   
Illustration 26: Find the equation of the sphere if it touches the plane r ⋅ (2 i − 2 j − k) =0 and the position vector
of its center is 3iˆ + 6ˆj − 4k.
ˆ
 (JEE ADVANCED)
 
Sol: Here equation of the required sphere is | r − c | = a where a is the radius of the sphere.
   
Given plane is r ⋅ (2 i − 2 j − k) =
0 … (i)
→   
Let H be the center of the sphere, then OH = 3i + 6 j − 4k = c(say)

Radius of the sphere = length of perpendicular from H to plane (i)


| c ⋅ (2i − 2 j − k) | | (3i + 6 j − 4k) ⋅ (2i − 2 j − k) | | 6 − 12 + 4 | 2
=    =    = = = a (say)
| 2i − 2 j − k | (2 i − 2 j − k) 3 3
 
Equation of the required sphere is | r − c | = a
      2    2 4
⇒ | x i + yj + zk − (3i + 6 j − 4k)
=| or | (x − 3)i + (y − 6) j + (z + 4)k
= | ⇒ (x – 3)2 + (y – 6)2 + (z + 4)2
3 9
= 4/9 or 9(x2 + y2 + z2 – 6x – 12y + 8z + 61) = 4 ⇒ 9x2 + 9y2 + 9z2 – 54x – 108y + 72z + 545 = 0
2 7 . 2 2 | 3D Geometr y

Illustration 27: Find the equation of the sphere with the points (1, 2, 2) and (2, 3, 4) as the extremities of a diameter.
Find the coordinates of its center. (JEE MAIN)

Sol: Equation of the sphere having (x1, y1, z1) and (x2, y2, z2) as the extremities of a diameter is (x – x1) (x – x2) + (y –
y1) (y – y2) + (z – z1) (z – z2) = 0.
Let A ≡ (1, 2, 2), B ≡ (2, 3, 4)
Equation of the sphere having (x1, y1, z1) and (x2, y2, z2) as the extremities of a diameter is
(x – x1) (x – x2) + (y – y1) (y – y2) + (z – z1) (z – z2) = 0

Here x1 = 1, x2 = 2, y1 = 2, y2 = 3, z1 = 2, z2 = 4

∴ Required equation of the sphere is (x – 1) (x – 2) + (y – 2) (y – 3) + (z – 2) (z – 4) = 0 or


x2 + y2 + z2 – 3x – 5y – 6z + 16 = 0

Center of the sphere is the midpoint of AB

3 5 
∴ Center is  , ,3  .
2 2 

Illustration 28: Find the equation of the sphere passing through the points (3, 0, 0), (0, –1, 0), (0, 0, –2) and whose
center lies on the plane 3x + 2y + 4z = 1  (JEE ADVANCED)

Sol: Consider the equation of the sphere be x2 + y2 + z2 + 2ux + 2vy + 2wz + d = 0 .


As the sphere passes through these given points hence these points will satisfy equation of sphere.  ... (i)
Let A ≡ (3, 0, 0), B ≡ (0, –1, 0), C ≡ (0, 0, –2) since sphere (i) passes through A, B and C.
∴ 9 + 6u + d = 0 … (ii)
1 – 2v + d = 0 … (iii)
4 – 4w + d = 0 … (iv)
Since the center (–u, –v, –w) of the sphere lies on plane 3x + 2y + 4z = 1∴ – 3u – 2v – 4w = 1  ....(v)
(ii) – (iii) ⇒ 6u + 2v = –8 … (vi)
(iii) – (iv) ⇒ –2v + 4w = 3 … (vii)
−2v − 8
From (vi), u =  … (viii)
6
From (vii), 4w = 3 + wv
2v + 8
Substituting the values of u, v and w in equation (v), we get + 2v − 3 − 2v =1
2
⇒ 2v + 8 – 4v – 6 – 4v = 2 ⇒ v = 0
0−8 4
From equation (viii), u = = − ; From equation (ix), 4w = 3 ∴ w = 3/4
6 3
From equation (iii), d = 2v – 1 = 0 – 1 = –1 From equation (i), the equation of the required sphere is

0−8 8 3
x2 + y 2 + z2 − − x + z −1 =
0 or 6x2 + 6y2 + 6z2 – 16x + 9z – 6 = 0
6 3 2
M a them a ti cs | 27.23

FORMULAE SHEET

 (a) Distance between the points (x1, y1, z1) and (x2, y2, z2) is (x2 − x1 )2 + (y 2 − y1 )2 + (z 2 − z1 )2

 (b) Coordinates of the point dividing the distance between the points (x1, y1, z1) and (x2, y2, z2) in the ratio m:n are
 mx2 + nx1 my 2 + ny1 mz 2 + nz1 
 , , 
 m+n m+n m + n 

 (c) If A(x1, y1, z1), B(x2, y2, z2) and C(x3, y3, z3) are vertices of a triangle, then its centroid is

 x1 + x2 + x3 y1 + y 2 + y 3 z1 + z 2 + z3 
 , , 
 3 3 3 

 (d) If A(x1, y1, z1) and B (x2, y2, z2) are the two points, the point which divides the line segment AB in ratio λ:1 is

 λx2 + x1 λy 2 + y1 λz 2 + z1 
 λ + 1 , λ + 1 , λ + 1 
 

 (e) If (x1, y1, z1) and (x2, y2, z2) are the two points on the line with x2 – x1, y2 – y1, z2 – z1 as direction ratios, then their
d.c.s are
x2 − x1 y 2 − y1 z 2 − z1
± ,± ,±
2 2
Σ(x2 − x1 ) Σ(x2 − x1 ) Σ(x2 − x1 )2

 (f) If  , m, n are d.c.s of a line, then l2 + m2 + n2 = 1. Thus, if a line makes angles α, β, γ with axes, then
cos2 α + cos2 β + cos2 γ =1 and sin2 α + sin2 β + sin2 γ = 2

a b c
 (g) If a, b, c are the d.r.s of a line, then the d.c.s of the line are ± ,± ,±
2 2
Σa Σa Σa2
 
 (h) If p(x, y, z) is a point in space such that OP = r has d.c.s  , m, n, then
  
(a)  | r |,m | r |,n | r | are the projections on x-axis, y-axis and z-axis, respectively.
  
(b) = x = | r |, y m= | r |, z n | r |
 
(c) r =| r | (l ˆi + mjˆ + nk) ˆ and rˆ = ˆi + mjˆ + nkˆ

  |r |
Moreover, if a, b, c are d.r.s of a vector = r , then r (aiˆ + bjˆ + ck).
ˆ
2 2 2
a +b +c

 (i) Length of projection of the line segment joining (x1, y1, z1) and (x2, y2, z2) on a line with d.c.s  , m, n is |
 (x2 − x1 ) + m(y 2 − y1 ) + n(z 2 − z1 ) |

 ( j) If θ is the angle between two lines having direction ratios a1, b1, c1 and a2, b2, c2 then
a1a2 + b1b2 + c1c2
cos θ = ±
Σa12 Σa22

a1 b1 c1
 (k) Two lines are parallel if = = and two lines are perpendicular if a1a2 + b1b2 + c1c2 =
0
a2 b2 c2

 (l) Cartesian equations of a line passing through (x1, y1, z1) and having direction ratios a, b, c are
x − x1 y − y1 z − z1
= = = t
a b c
2 7 . 2 4 | 3D Geometr y

    
 (m) Vector equation of a line passing through the point A(a) and parallel to vector b is r = a + λb for scalar λ.

 (n) Cartesian equation of a line passing through two points having coordinates (x1, y1, z1) and (x2, y2, z2) is
x − x1 y − y1 z − z1
= = .
x2 − x1 y 2 − y1 z 2 − z1
     
 (o) Vector equation of a line passing through two points having position vectors a and b is r = α + λ(b − a)
  
      | b × (a − a ) |
2
 (p) Distance between the parallel lines r= a1 + λb and r= a2 + µb is  1
|b|
     
(q) Shortest distance (S.D.) between two lines with equations; r= a1 + λb1 and r= a2 + µb2 is
     
| (b1 × b2 ) ⋅ (a2 − a1 ) | b1 .b2
  . If θ is the angle between the lines, then cos θ =  
| b1 × b2 | | b1 || b2 |

x − x1 y − y1 z − z1
(r) The length of perpendicular from the point (α , β, γ ) to the line = = , m, n being d.cs)
( ,m,n
 m n

is given by (α − x1 )2 + (β − y1 )2 + ( γ − z1 )2 − [(α − x1 ) + m(β − y1 ) + n( γ − z1 )]2

     
(s) If a and b are the unit vectors along the sides of an angle, then a + b and a − b are the vectors, respectively,
along the internal and external bisector of the angle. In fact, the bisectors of the angles between the lines,
 
      a b 
r = xa and r = yb are given by r = λ   +   ; λ ∈ R
| a | |b |
 
(t) Equation of plane passing through the point (x1, y1, z1) is a(x − x1 ) + b(y − y1 ) + c(z − z1 ) =
0.

(u) Equation of plane passing through three points (x1, y1, z1), (x2, y2, z2) and (x3, y3, z3) is
x − x1 y − y1 z − z1
x2 − x1 y 2 − y1 z 2 − z1 =
0
x3 − x1 y 3 − y1 z3 − z1
x y z
(v) Equation of a plane making intercepts a, b, c on axes is + + =1
a b c
  
(w) Vector equation of a plane through the point a and perpendicular to the unit vector nˆ is (r − a).nˆ =
0
  nˆ ⋅ nˆ
(x) If θ is the angle between the two planes r= .nˆ 1 d1 and r=⋅ nˆ 2 d2 , then cos θ = 1 2
| nˆ 1 || nˆ 2 |
x − x1 y − y1 z − z1
(y) Equation of a plane containing the line = = and passing through the point (x2, y2, z2) not
a b c
x − x1 y − y1 z − z1
on the line is a b c =0
x2 − x1 y 2 − y1 z 2 − z1

x − x1 y − y1 z − z1 x − x2 y − y 2 z − z2
(z) Equation of a plane through the line = = and parallel to the line = =
a1 b1 c1 a2 b2 c2
x − x1 y − y1 z − z1
is a1 b1 c1 = 0
a2 b2 c2
M a them a ti cs | 27.25

x − x1 y − y1 z − z1
(aa) If θ is the angle between the line = = and the plane Ax + By + Cz + D =
0 , then
a b c
| aA + bB + cC |
sin θ =
a + b2 + c 2 A 2 + B2 + C2
2

| ax1 + by1 + cz1 + d |


(ab) Length of perpendicular from (x1, y1, z1) to the plane ax + by + cz + d =0 is
a2 + b2 + c2

(ac) The equation of a sphere with center at the origin and radius ‘a’ is | r | = a or x2 + y 2 + z 2 =
a2

(ad) Equation of a sphere with center (α , β, γ ) and radius ‘a’ is (x − α )2 + (y − β)2 + (z − γ )2 = a2


      
(ae) Vector equation of the sphere with center c and radius ‘a’ is | r − c | = a2
a or (r − c) ⋅ (r − c) =

(af) General equation of sphere is x2 + y 2 + z 2 + 2ux + 2vy + 2wz + d =whose


0 center is ( −u, − v, −w) and radius is

u2 + v 2 + w2 − d
(ag) Equation of a sphere concentric with x2 + y 2 + z 2 + 2ux + 2vy + 2wz + d =is
0

x2 + y 2 + z 2 + 2ux + 2vy + 2wz + λ = 0 , where λ is a real number.

Solved Examples

JEE Main/Boards Let P(x, 0, 0) be a point on X-axis such that distance of


P from the point (1, –2, 3) is 2 6
Example 1: Find the coordinates of the point which
divides the join of P(2, –1, 4) and Q(4, 3, 2) in the ratio 2 ⇒ (1 − x)2 + ( −2 − 0)2 + (3 − 0)2 =2 6
: 3 (i) internally (ii) externally
⇒ (x − 1)2 + 4 + 9 =24 ⇒ (x − 1)2 =
11
Sol: By using section formula we can obtain the result. ⇒ x − 1 =± 11 ⇒ x = 1 ± 11
Let R(x, y, z) be the required point
2 × 4 + 3 × 2 14 2 × 3 + 3 × ( −1) 3 Example 3: If a line makes angles α , β, γ with OX, OY,
=(i) x = = ; y = OZ, respectively, prove that sin2 α + sin2 β + sin2 γ = 2
2+3 5 2+3 5
2 × 2 + 3 × 4 16 Sol: Same as illustration 2.
=z =
2+3 5 Let  ,m,n be the d.c.’s of the given line, then
 14 3 16   = cos α ,m = cos β,n = cos γ
So, the required point is R  , , 
 5 5 5  cos2 α + cos2 β + cos2 γ =1
2× 4 − 3× 2 2 × 3 − 3 × ( −1)
(ii) x = = −2 ; y = = −9 ⇒ (1 − sin2 α ) + (1 − sin2 β) + (1 − sin2 γ ) =
1
2−3 2−3
⇒ sin2 α + sin2 β + sin2 γ = 2
2× 2 − 3× 4
=z = 8
2−3 Example 4: Projections of a line segment on the
Therefore, the required point is R(–2. –9, 8) axes are 12, 4 and 3 respectively. Find the length and
direction cosines of the line segment.
Example 2: Find the points on X-axis which are at a
distance of 2 6 units from the point (1, –2, 3) Sol: Let  ,m,n be the direction cosines and r be the
length of the given segment, then r,mr,nr are the
Sol: Consider required point is P(x, 0, 0), therefore by projections of the segment on the axes.
using distance formula we can obtain the result.
2 7 . 2 6 | 3D Geometr y

Let  ,m,n be the direction cosines and r be the It will pass through B(3, 4, 2) and C(7, 0, 6) if
length of the given segment, then r,mr,nr are the
a(3 − 2) + b(4 − 2) + c(2 + 1) =
0 or
projections of the segment on the axes; therefore
=r 12,=mr 4, = nr 3 a + 2b + 3c = 0 .… (ii)
Squaring and adding, we get & a(7 − 2) + b(0 − 2) + c(6 + 1) = or
0
2 2 2 2 2 2 2 2 0
5a − 2b + 7c = .… (iii)
r ( + m + n ) = 12 + 4 + 3 ⇒ r =
169
⇒ r = 13 ⇒ length of segment = 13 Solving (ii) and (iii) by cross multiplication, we get
And direction cosines of segment are a b c or a = b = c = λ (say)
12 12 4 4 3 3 = =
= = , m= = and n= = 14 + 6 15 − 7 −2 − 10 5 2 −3
r 13 r 13 r 13
⇒ a =5λ ,b = 2λ ,c =−3λ
Example 5: Find the length of the perpendicular from
Substituting the values of a, b and c in (i), we get
the point (1, 2, 3) to the line through (6, 7, 7) and having
5λ(x − 2) + 2λ(y − 2) − 3λ(z + 1) =0
direction ratios (3, 2, –2).
or,5(x − 2) + 2(y − 2) − 3(z + 1) =
0
Sol: By using distance formula i.e. |  (x2 – x1) + m(y2 – y1) ⇒ 5x + 2y – 3z = 17,
+ n(z2 – z1)| , we can obtain required length.
Which is the required equation of the plane.
Direction cosines of the line are
3 2 −2 Example 7: Find the angle between the planes x + y +
, ,
2 2
3 + 2 + ( −2)2 2 2
3 + 2 + ( −2) 2
3 + 22 + ( −2)2
2 2z = 9 and 2x – y + z = 15
a1a2 + b1b2 + c1c2
3 2 −2 Sol: By using formula cos θ =
i.e. , , a12 + b12 + c12 a22 + b22 + c22
17 17 17
we can obtain the result.
∴ AN = Projection of AP on AB
The angle between x + y + 2z = 9 and 2x – y + z = 15
3 2 ( −2) is given by
= (6 − 1) + (7 − 2) + (7 − 3)
17 17 17
a1a2 + b1b2 + c1c2
cos θ =
15 + 10 − 8 17
= = = 17 a12 + b12 + c12 a22 + b22 + c22
17 17
P(1, 2, 3) (1)(2) + 1( −1) + (2)(1) 1 π
=cos θ = ⇒θ =
2 2 2 2
1 + 1 + 2 2 + ( −1) + 1 2 2 2 3

B
A(6, 7, 7) N d.r’.s(3, 2, -2) Example 8: Find the distance between the parallel
planes 2x – y + 2z + 3 = 0 and 4x – 2y + 4z + 5 = 0
Also, AP = (6 − 1)2 + (7 − 2)2 + (7 − 3)2
Sol: By making the coefficient of x, y and z as unity we
= 25 + 25 + 16 = 66 will be get required result.
Let P(x1, y1, z1) be any point on 2x – y + 2z + 3 = 0, then,
∴ PN = AP2 − AN2 = 66 − 17 = 49 = 7 unit
2x1 – y1 + 2z1 + 3 = 0  … (i)

Example 6: Find the equation of the plane through the The length of the perpendicular from
points A(2, 2, –1), B(3, 4, 2) and C(7, 0, 6) P(x1 , y1 , z1 )to 4x − 2y + 4z + 5 =0 is

Sol: As we know, equation of a plane passing through


the point (x1, y1, z1) is given by 4x1 − 2y1 + 4z1 + 5 2(2x1 − y1 + 2z1 ) + 5
= =
2 2 2
a(x – x1) + b(y – y1) + c (z – z1) = 0 . 4 + ( −2) + 4 36

The general equation of a plane through (2, 2, –1) is


2( −3) + 5 1
a(x − 2) + b(y − 2) + c(z + 1) =
0  .… (i) = =
6 6
M a them a ti cs | 27.27

Example 9: The equation of a line are 6x – 2 = 3y + 1 JEE Advanced/Boards


= 2z – 2. Find its direction ratios and its equation in
symmetric form. Example 1: Find the equations of the bisector planes of
the angles between the planes 2x – y + 2z + 3 = 0 and
Sol: The given line is 6x – 2 = 3y + 1 = 2z – 2 3x – 2y + 6z + 8 = 0 and specify the plane bisecting the
acute angle and the plane bisecting obtuse angle.
 1  1
⇒ 6  x −  = 3  y +  = 2(z − 1)
 3  3 Sol: As we know, Equation of the planes bisecting the
1 1 angle between two given planes a1x + b1y + c1z + d1 =
x− y+
⇒ 3 = 3 = z −1 0 and a2x + b1y + c2z + d2 = 0 are
1 2 3
a1 x + b1 y + c1 z + d1 a2 x + b2 y + c2 z + d2
= ±
[We make the coefficients of x, y and z as unity]
a12 + b12 + c12 a22 + b22 + c22
This equation is in symmetric form. Thus the direction
The two given planes are
ratios of the line are 1, 2 and 3 and this line passes
1 1  2x – y + 2z + 3 = 0 … (i)
through the point  , − ,1  .
3 3  and 3x – 2y + 6z + 8 = 0  … (ii)
Example 10: Find the image of the point (3, –2, 1) in The equations of the planes bisecting the angles
the plane 3x – y + 4z = 2. between (i) and (ii) are

Sol: Consider Q be the image of the point P(3, –2, 1) 2x − y + 2z + 3 3x − 2y + 6z + 8


= ±
in the plane 3x – y + 4z = 2. Then PQ is normal to the 2 2
2 + ( −1) + 2 2
32 + ( −2)2 + 62
plane hence direction ratios of PQ are 3, –1, 4.
⇒ 14x − 7y + 14z + 21 =
± (9x − 6y + 18z + 24)
Let Q be the image of the point P(3, –2, 1) in the
plane 3x – y + 4z = 2. Then PQ is normal to the plane. Hence the two bisector planes are
Therefore direction ratios of PQ are 3, –1, 4. Since PQ 5x − y − 4z − 3 =
0 … (iii)
passes through P(3, –2, 1) and has direction ratios
3, –1, 4. Therefore equation of PQ is and 23x − 13y + 32z + 45 =
0 … (iv)

x −3 y + 2 z −1 Now we find angle θ between (i) & (iii)


= = = r (say)
3 −1 4 We have,
Let the coordination of Q be (3r + 3, –r, –2. 4r + 1). 5(2) + ( −1)( −1) + 2( −4) 1
cos θ = =
Let R be the mid-point of PQ. Then R lies on the plane 2
2 + ( −1) + 22 2 2 2
5 + ( −1) + ( −4) 2 42
3x – y + 4z = 2. The coordinates of R are π
Thus the angle between (i) & (iii) is more than .
 3r + 3 + 3 −r − 2 − 2 4r + 1 + 1  4
Therefore, (iii) is the bisector of obtuse angle between
 , , 
 2 2 2  (i) and (ii) and hence (iv) bisects acute angle between
them.
 3r + 6 −r − 4 
or  , ,2r + 1 
 2 2  Example 2: Find the distance of the point (1, –2, 3) from
the plane x – y + z = 5 measured parallel to the line
 3r + 6   −r − 4  whose direction cosines are proportional to 2, 3, –6.
3 −  + 4(2r + 1) =
2
 2   2 
⇒ 13r = –13 ⇒ r = –1 Sol: By using distance formula we can obtain required
length,
So, the coordinates of Q are (0, –1, –3)
Equation of line through (1, –2, 3) parallel to the line
with d.r.’s 2, 3, –6 is
x −1 y + 2 z −3
= = = r  …(i)
2 3 −6
Any point on it is (1 + 2r, − 2 + 3r, 3 − 6r)
Line (i) meets the plane x – y + z = 5.
1
If 1 + 2r – (–2 + 3r) + (3 – 6r) = 5 ; i.e. if r =
7
2 7 . 2 8 | 3D Geometr y

If PQ is perpendicular to (i) and (ii), we have


∴ Point of intersection is  9 , − 11 , 15 
7 7 7  3(4r2 – 3r1 – 3) + 2(3r2 – 2r1 + 2) + 5(–2r2 – 5r1 – 2) = 0
whose distance from (1, –2, 3) is & 4(4r2 – 3r1 – 3) + 3(3r2 – 2r1 + 2) – 2(–2r2 – 5r1 – 2) = 0
2 2 2 i.e. 8r2 – 38r1 – 15 = 0 & 29r2 – 8r1 – 2 = 0
9   11   15 
 − 1 +  − + 2 +  − 3 r2 r1 1
7   7   7  Solving them, = =
76 − 120 −435 + 16 1038
4 9 36 49 44 419
= + += = 1 ⇒ r2 =
− , r1 =

49 49 49 49 1038 1038
 1257 838 2095 
Example 3: Show that the lines ∴ Points P and Q are  − + 1, − − 1, − + 1
 1038 1038 1038 
x −1 y +1 z −1
= =  … (i)  176 132 88 
3 2 5 and  − − 2, − + 1, − 1
 1038 1038 1038 
x + 2 y −1 z +1
and = =  … (ii) We can find the distance PQ by distance formula which
4 3 −2
is the shortest distance.
do not intersect. Also find the shortest distance
between them. Example 4: Find the angle between the lines whose
direction ratios satisfy the equations :
x2 − x1 y 2 − y1 z 2 − z1
3 + m +=
5n 0, 6mn − 2n + 5=
m 0
Sol: If 1 m1 n1 ≠0
2 m2 n2 aa'+ bb'+ cc'
Sol: Here, cos θ = .
then the lines do not intersect each other. And using 2
a +b +c2 2 2 2
a' + b' + c'2

distance formula we will be get required shortest


distance. The given equations are 3 + m + 5n =
0 3 … (i)

Points on (i) and (ii) are (1, –1, 1) and (–2, 1, –1) 0 
and 6mn − 2n + 5m = … (ii)
3 2 5 From (i), we have m =−3 − 5n  … (iii)
respectively and their d.c.’s are , ,
38 38 38 Putting in (ii), we get
4 3 −2
and , , respectively. 6( −3 − 5n) n − 2n + 5( −3 − 5n) =
0
29 29 29
⇒ 30n2 + 45n + 15 2 =
0
−2 − 1 1 + 1 −1 − 1 ⇒ 2n2 + 3n + 2 =
0 ⇒ (n +  )(2n +  ) =
0
x2 − x1 y 2 − y1 z 2 − z1
3 2 5 ⇒ Either  =
−n or  =
−2n
∴ 1 m1 n1
= 38 38 38
2 m2 n2 If  = −n , then from (iii), m = –2n
4 3 −2
If  = −2n , then from (iii), m = n
29 29 29
Thus the direction ratios of two lines are
−3 2 −2
1 1 –n, –2n, n and –2n, n, n
= × 3 2 5
38 29 i.e. 1, 2, –1 and –2, 1, 1
4 3 −2
∴ If θ is the angle between the lines, then
1 1
= × , [ −3( −4 − 15) + 2(20 + 6) −2(9 − 8)] ≠ 0 1.( −2) + 2.1 + ( −1).1 −2 + 2 − 1 −1
38 29 = cos θ = = .
1 + 4 + 1. 4 + 1 + 1 6. 6 6
Hence the given lines do not intersect.
Any point P on (i) is (1 + 3r1, 2r1, –1, 5r1 + 1) and a point Example 5: Find the equation of the plane through the
on (ii) is Q(4r2 – 2, 3r2 + 1, –2r2 – 1) intersection planes 2x + 3y + 4z = 5, 3x – y + 2z = 3
and parallel to the straight line having direction cosines
∴ Direction ratios of PQ are
(–1, 1, –1).
(4r2 – 3r1 – 3, 3r2 – 2r1 + 2, –2r2 – 5r1 – 2)
M a them a ti cs | 27.29

Sol: By using formula of family of plane, we will get the x +1 y +3 z +5


result. = = 3 5 7 0
Equation of plane through the given planes is 1 4 7
2x + 3y + 4z − 5 + λ(3x − y + 2z − 3) = 0
or (x + 1)(35 − 28) − (y + 3)(21 − 7) + (z + 5)(12 − 5) =o
0 r
i.e. (2 + 3λ )x + (3 − λ )y + (4 + 2λ )z +( −5 − 3λ ) =0
x − 2y + z =0
This plane is parallel to the given straight line.
⇒ − 1(2 + 3λ ) + 1(3 − λ ) + ( −1)(4 + 2λ ) =0 Example 7: Find the equation of the plane passing
⇒ −2 − 3λ + 3 − λ − 4 − 2λ =0 through the lines of intersection of the planes 2x –
y = 0 and 3z – y = 0 and perpendicular to the plane
1
⇒ 6λ = −3 ⇒ λ = − 4x + 5y – 3z = 8.
2
∴ Equation of required plane is Sol: Here by using the family of plane and formula of
two perpendicular plane we will get the result.
1 7 7
x + y + 3z − =0 ⇒ x + 7y + 6z =7
2 2 2 The plane 2x – y + k(3z – y) = 0
⇔ 2x − (1 + k) y + 3kz =
0 is perpendicular to the plane
Example 6: Prove that the lines 4x + 5y − 3z = 8
x +1 y +3 z +5 x−2 y −4 z−6 ⇒ 2.4 − (1 + k).5 + 3k( −3) =
0 ⇒ 14k =
3
= = and = =
3 5 7 1 4 7 3
⇒k =
are coplanar. Also, find the plane containing these two 14
lines. Thus the required equation is
 3 
2x − y +   (3z − y) =0 ⇔ 28x − 17y + 9z =
0
Sol: As similar to example 3.  14 
x − x1 y − y1 z − z1
We know the lines = = Example 8: Show that the lines
1 m1 n1
x − x2 y − y 2 z − z2 x +5 y + 4 z −7
and = = are coplanar if = = ,3x + 2y + z − 2 = 0 = x − 3y + 2z − 13
2 m2 n2 3 1 −2
are coplanar and find the equation to the plane in
x2 − x1 y 2 − y1 z 2 − z1
which they lie.
1 m1 n1 =0
2 m2 n2 Sol: By using the condition of coplanarity of line, we
will get given lines are coplanar or not. And after that
and the equation of the plane containing these two by using general equation of the plane we can obtain
lines is required equation of plane.
x − x1 y − y1 z − z1 The general equation of the plane through the second
1 m1 n1 = 0 line is
2 m2 n2
3x + 2y + z − 2 + k(x − 3y + 2z − 13) =0
Here x =
−1, y1 =
−3, z1 =
−5, x2 =
2, y 2 =
4, z 2 =
6,
⇔ x(3 + k) + y(2 − 3k) + z(1 + 2k) −2 − 13k =0 ;
1 3,m
= = 1 5,n
= 1 7,=
 2 1,m
= 2 4,n
= 2 7 K being the parameter

x2 − x1 y 2 − y1 z 2 − z1 3 7 11 This contains the first line only if


1 m1 n1 = 3 5 7 9
3(3 + k) + (2 − 3k) − 2(1 + 2k) =
0 ⇒k =
2 m2 n2 1 4 7 4
Hence the equation of the plane which contains the
= 21 − 98 + 77 = 0 two lines is
So, the given lines are coplanar, The equation of the 21x − 19y + 22x − 125 =
0
plane containing the lines is
This plane clearly passes through the point ( −5, −4,7)
2 7 . 3 0 | 3D Geometr y

JEE Main/Boards

Exercise 1 Q.15 Find the intercepts cut by the plane 3x – 2y + 4z


– 12 = 0 on axes.
3 −2 6
Q.1 Direction cosines of a line are , , , find its
7 7 7 Q.16 Direction ratios of a line are 1, 3, –2. Find its
direction ratios.
direction cosines.

Q.2 Find the direction ratios of a line passing through


Q.17 Find the direction cosines of y-axis.
the points (2, 1, 0) and (1, –2, 3).
Q.18 Find the direction ratio of the line
Q.3 Find the angle between the lines
x y z x −1 y +5 z −3 x + 2 2y − 1 3 − z
= = and = = . = = .
1 2 0 3 2 1 1 3 5

Q.4 Find the equation of a line parallel to the vector Q.19 Find the angle between the planes
3iˆ − ˆj − 3kˆ and passing through the point ( −1,1,1).
r.(iˆ − 2ˆj= ˆ 1andr.(3iˆ − 6ˆj=
− 2k) ˆ 0.
+ 2k)

Q.5 Write the vector equation of a line whose Cartesian 


x + 3 y −1 z +1 Q.20 Find the angle between the line = r (2iˆ − ˆj + 3k)
ˆ + λ(3iˆ − 6ˆj + 2
equation is = = .  
2 4 5 r (2iˆ − ˆj + 3k)
= ˆ + λ(3iˆ − 6ˆj + 2k)
ˆ and the plane r.(iˆ + ˆj + k)
ˆ +3

Q.6 Write the Cartesian equation of a line whose vector


 Q.21 Find the direction cosines of the two
equations is =r (3iˆ + 2ˆj − 5k)
ˆ + λ( −2iˆ + ˆj + 3k).
ˆ
lines which are connected by the relations
+ 3n 0 and72 + 5m2 −=
 − 5m = 3n2 0.
Q.7 Find the value of p, such that the line
x y z x y z Q.22 Prove that, the line passing through the point (1,
= = and = =
1 3 2p −3 5 2 2, 3) and (–1, –2, –3) is perpendicular to the line passing
through the points (–2, 1, 5) and (3, 3, 2).
are perpendicular to each other.
Q.23 Find the coordinates of the foot of the
Q.8 Write the Cartesian equation of the plane
 perpendicular drawn from the point (1, 2, 1) to the line
r(3iˆ + 2ˆj + 5k)
ˆ =7.
joining the points (1, 4, 6) and (5, 4, 4).

Q.9 Write the vector equation of plane 3x − y − 4z + 7 =0.


Q.24 If a variable line in two adjacent positions has
direction cosines  , m, n and  + δ  ,m + δ m,n + δ n,
Q.10 Find the vector, normal to the plane
 prove that the small angle δθ between two position is
r.(3iˆ − 7k)
ˆ +5 =0.
given by (δθ)2 = (δ )2 + (δm)2 + (δn)2 .
Q.11 Find the direction ratios of a line, normal to the
plane 7x + y – 2z = 1. 1 +  2 +  3 m1 + m2 + m3 n1 + n2 + n3
Q.25 Verify that , ,
3 3 3
x + 2 y −1 z
Q.12 Find the angle between the line = = can be taken as direction cosines of a line equally
and the plane 3x − 2z + 4 =
0. 4 −5 7
inclined to three mutually perpendicular lines with
direction cosines 1 ,m1 ,n1 ;  2 ,m2 ,n2 and  3 ,m3 ,n3
Q.13 Find the distance of the plane x + y + 3z + 7 = 0
from origin.
Q.26 Find the equations of line through the point (3, 0, 1)
and parallel to the planes x + 2y = 0 and 3y – z = 0.
Q.14 Find the distance of the plane 3x – 3y + 3z = 0
from the point (1, 1, 1).
M a them a ti cs | 27.31

Q.27 Find the equations of the planes through the Q.39 Find the length and coordinates of the foot of
intersection of the planes x + 3y + 6 = 0 and 3x – y – perpendicular from points (1, 1, 2) to the plane 2x – 2y
4z = 0 whose perpendicular distance from the origin is + 4z + 5 = 0.
equal to 1.
Q.40 Find the vector equation in the scalar product
Q.28 Find the equation of the plane through the points form, of the plane passing through the points (1, 0, –1),
(–1, 1, 1) and (1, –1, 1) and perpendicular to the plane (3, 2, 2) and parallel to line
x + 2y + 2z = 5.
r = ˆi + ˆj + λ(iˆ − 2ˆj + 3k).
ˆ

Q.29 Find the distance of the point (–1, –5, –10) from
the plane x – y + z = 5 measured parallel to the line Q.41 Find the distance between the parallel planes
2x – y + 3z – 4 = 0 and 6x – 3y + 9z + 13 =0.
x −2 y +1 z −2
= = .
3 4 12 Q.42 Prove that, the equation of a plane. Which meets
the axes in A, B, and C and the given centroid of triangle
Q.30 Find the vector and Cartesian forms of the x y z
equation of the plane passing through (1, 2, –4) and ABC is the point (α , β, γ ), is + + = 3.
 α β γ
parallel to the line r = ˆi + 2ˆj − 4kˆ + λ(2iˆ + 3ˆj + 6k)
ˆ and
 ˆ ˆ
r = i − 3 j + 5kˆ + µ(iˆ + ˆj − k).
ˆ Q.43 Find the equation of the plane passing through
the origin and the line of intersection of the planes x –
Q.31 If straight line having direction cosines given 2y + 3z + 4 = 0 and x – y + z + 3 = 0.
by a + bm + cn = 0 and fmn + gn + hm = 0 are
f g h Q.44 Prove that, the line 2x + 2y – z – 6 = 0, 2x + 3y – z –
perpendicular, then prove that + + = 0.
a b c 8 = 0 is parallel to the plane y = 0. Find the coordinates
of the point where this line meets the plane x = 0.
Q.32 Prove that, the lines x = ay + b, z = cy + d and x
= a’y + b’, z = c’y + d’ are perpendicular to each other,
Q.45 Find the equation of the plane through the line
if aa’ + cc’ + 1 = 0.
ax + by + cz + d = 0, a’x + b’y + c’z + d’ = 0 and parallel
x y z
Q.33 Find the equation of the plane passing through to the line = = .
 m n
the intersection of the planes 4x – y + z = 10 and x +
y – z = 4 and parallel to the line with direction ratios Q.46 Find the equation of a plane parallel to x-axis and
2, 1, 1. Find also the perpendicular distance of (1, 1, 1) has intercepts 5 and 7 on y and z-axis, respectively.
from this plane.
Q.47 A variable plane at a constant distance p from
Q.34 The foot of the perpendicular drawn from the origin meets the coordinate axes in points A, B and C,
origin to the plane is (2, 5, 7). Find the equation of plane. respectively. Through these points, planes are drawn
parallel to the coordinate planes, prove that locus of
Q.35 Find the equation of a plane through (–1, –1, 2) 1 1 1 1
point of intersection is + + =.
2 2 2
and perpendicular to the planes 3x + 2y – 3z = 1 and x y z p2
5x – 4y + z = 5.
Q.48 Find the value of λ , for which the points
Q.36 Find the angle between the lines whose with position vectors ˆi − ˆj + 3kˆ and 3iˆ + λˆj + 3kˆ are

direction cosines are given by equations  + m + n =0; equidistant from the plane r.(5iˆ + 2ˆj − 7k)
ˆ +9 = 0.
2 + m2 − n2 = 0
Q.49 Find the equation of a plane which is at a distance
Q.37 Find the equation of the line which passes through of 7 units from the origin and which is normal to the
(5, –7, –3) and is parallel to the line of intersection of the vector 3iˆ + 5ˆj − 6kˆ
planes x – 3y – 5 = 0 and 9y – z + 16 = 0.
Q.50 Find the vector equation of the plane,
r = ˆi − ˆj + λ(iˆ + ˆj + k)
ˆ + µ(4iˆ − 2ˆj + 3k)
ˆ in the scalar
Q.38 Prove that, the plane through the points (1, 1, 1),
(1, –1, 1) and (–7, 3, –5) is perpendicular to xz-plane. product from.
2 7 . 3 2 | 3D Geometr y

Exercise 2 Q.8 The symmetric form of the equation of the line x +


y – z = 1, 2x – 3y + z = 2 is
Single Correct Choice Type x +1 y −2 z +1 x y z −1
(A) = = (B) = =
3 −2 −1 2 3 5
Q.1 The sum of the squares of direction cosines of a
straight line is x y −1 z x −1 y z
(C)
= = (D) = =
2 3 5 2 3 5
(A) Zero (B) Two
(C) 1 (D) None of these x −1 y −3 z − 4
Q.9 The line = = is parallel to the plane
1 2 3
Q.2 Which one of the following is best condition for the (A) 2x + y + 2z + 3 = 0 (B) 2x – y – 2z = 3
plane ax + by + cz + d = 0 to intersect the x and y axes
at equal angle (C) 21x – 12y + z = 0 (D) 2x + y – 2z = 0

(A) |a| = |b| (B) a = –b


Q.10 The vertices of the triangle PQR are (2, 1, 1),
(C) a = b (D) a2 + b2 = 1 (3, 1, 2) and (–4, 0, 1). The area of the triangle is

Q.3 The equation of a straight line parallel to the x-axis 38


(A) (B) 38 (C) 4 (D) 2
is given by 2
x −a y −b z −c x −a y −b z −c
(A) = = (B) = = Q.11 Equation of straight line which passes through the
1 1 1 0 1 1
point P(1, 0, –3) and Q(–2, 1, –4) is
x −a y −b z −c x −a y −b z −c
(C) = = (D) = = x −2 y +1 z − 4 x −1 y z+3
0 0 1 1 0 0 (A) = = (B) = =
−3 1 −1 3 1 1
Q.4 A straight line is inclined to the axes of x and z at x −1 / 2 y −1 z + 4 x −1 y z+3
(C) = = (D) = =
angles 45° and 60° respectively, then the inclination of −3 1 −1 −3 1 −1
the line to the y-axis is
Q.12 A point moves so that the sum of the squares
(A) 30° (B) 45° (C) 60° (D) 90° of its distances from the six faces of a cube given by
x =±1, y = ±1, z = ±1 is 10 units. The locus of the point
Q.5 The coordinates of the point of intersection is
x +1 y +3 z + 2
of the line = = with the plane (A) x2 + y + z2 = 1 (B) x2 + y2 + z2 = 2
1 3 −2
3x + 4y + 5z = 5 (C) x + y + z = 1 (D) x + y + z = 2
(A) (5, 15, –14) (B) (3, 4, 5)
(C) (1, 3, –2) (D) (3, 12, –10) Q.13 The points (0, –1, –1), (–4, 4, 4), (4, 5, 1) and
(3, 9, 4) are
Q.6 Perpendicular is drawn from the point (0, 3, 4) to (A) Collinear (B) Coplanar
the plane 2x – 2y + z = 10. The coordinates of the foot
of the perpendicular are (C) Forming a square (D) None of these

8 1 16 8 1 16 Q.14 The equation of the plane containing the line


(A) − , , (B) , ,
3 3 3 3 3 3
x −α y −β z − γ
= = is a(x − α ) + b(y − β) +c(z − γ ) =0,
8 1 16 8 1 16  m n
(C) ,− , (D) , ,−
3 3 3 3 3 3 where a + bm + cn is equal to
(A) 1 (B) –1 (C) 2 (D) 0
Q.7 The equation of the plane through the line of
intersection of the planes 2x + y – z – 4 = 0 and 3x + 5z
– 4 = 0 which cuts off equal intercepts from the x-axis Q.15 The reflection of the plane 2 x + 3 y + 4 z − 3 = 0
and y-axis is in the plane x − y + z − 3 = 0 is the plane
(A) 3x + 3y – 8z + 8 = 0 (B) 3x + 3y – 8z – 8 = 0 (a) 4x− 3y + 2z −15 = 0 (b) x− 3 y + 2z − 15 = 0
(C) 3x – 3y – 8z – 8 = 0 (D) x + y – 8z – 8 = 0 (c) 4x + 3y− 2z + 15 = 0 (d) None of these
M a them a ti cs | 27.33

Previous Years’ Questions Q.10 The distance of the point (1, 0, 2) from the point
x −2 y +1 z −2
of intersection of the line − − and the
x − 4 y −2 z −k 3 4 12
Q.1 The value of k such that = = lies
1 1 2 plane x − y + z =16, is:  (2015)
in the plane
2x − 4y + z =7, is  (2003) (A) 8 (B) 3 21 (C) 13 (D) 2 14

(A) 7 (B) –7 (C) No real value (D) 4 Q.11 The equation of the plane containing the line
2x − 5y + z + 3; x + y + 4 z =5 and parallel to the plane
   x −3 y −k z
Q.2 If the lines r= a2 + µb and = = x + 3y + 6z = 1 is  (2015)
1 2 1
intersect, then the value of k is  (2004) (A) x + 3y + 6z =
−7 (B) x + 3y + 6z =
7
 9 2 3 (C) 2x + 6y + 12z =
−13 (D) 2x + 6y + 12z =
13
(A) a2 (B) (C) − (D) −
2 9 2

x y z Q.12 The number of common tangents to the circles


Q.3 A variable plane + + = 1 at a unit distance  (2015)
a b c
from origin cuts the coordinate axes at A, B and C. (A) Meats the curve again in the second in the second
quadrant
1 1 1
Centroid (x, y, z) satisfies the equation + + =K.
x 2
y 2
z2 (B) Meats the curve again in the third quadrant
The value of K is  (2005) (C) Meets the curve again in the fourth quadrant
(A) 9 (B) 3 (C) 1/9 (D) 1/3 (D) Does not meet the curve again

Fill in the Blanks for Q.4 and Q.5 x −1 y −3 z − 4


Q.13 The image of the line = = in the
3 1 −5
Q.4 The area of the triangle whose vertices are A(1, –1, 2), plane 2x − y + z + 3 is the  (2014)
B(2, 1, –1), C(3, –1, 2) is …  (1983)
x + 3 y −5 z − 2 x + 3 y −5 z − 2
(A) = = (B) = =
Q.5 The unit vector perpendicular to the plane 3 1 −5 −3 −1 5
determined by P(1, –1, 2), Q(2, 0, –1) and R(0, 2, 1)
x + 3 y −5 z − 2 x −3 y −5 z − 2
is…………..  (1983) (C) = = (D) = =
3 1 −5 −3 −1 −5
Q.6 A plane is parallel to two lines whose direction
ratios are (1, 0, –1) and (–1, 1, 0) and it contains the Q.14 The angle between the lines whose direction
point (1, 1, 1). If it cuts coordinate axes at A, B, C. Then cosines satisfy the equations l+m+n=0 and l2=m2+n2 is
find the volume of the tetrahedron OABC.  (2004)  (2014)
π π π π
(A) (B) (C) (D)
Q.7 Find the equation of the plane containing the line 3 4 6 2
2x – y + z – 3 = 0, 3x + y + z = 5 and at a distance of
Q.15 If the lines
1
from the point (2, 1, –1).  (2005) x −2 y −3 z − 4 x −1 y − 4 z − 5
6 = = and = =
1 1 −k k 2 1
x −3 y +2 z + 4
Q.8 If the line = = lies in the plane, are coplanar, then k have  (2013)
2 −1 3
(A) Exactly one value (B) Exactly two value
x + my − z = 9 , then 2 + m2 is equal to:  (2016)
(C) Exactly three values (D) Any value
(A) 18 (B) 5 (C) 2 (D) 26
Q.16 An equation of a plane parallel to the plane
Q.9 The distance of the point (1, −5,9 ) from the plane x − 2y + 2z − 5 =and
0 at a unit distance from the origin
x − y + z =measured
5 along the line x= y= z is (2016) is  (2012)
10 20 (A) x − 2y + 2z − 3 =0 (B) x − 2y + 2z + 1 =0
(A) 10 3 (B) (C) (D) 3 10
3 3
(C) x − 2y + 2z − 1 =0 (D) x − 2y + 2z + 5 =0
2 7 . 3 4 | 3D Geometr y

y −1 z −3 (A) Statement-I is true, statement-II is true; statement-II


=
Q.17 If the angle between the line x = and is not a correct explanation for statement-I
2 λ
 5 (B) Statement-I is true, statement-II is false.
4 is cos−1 
the plane x + 2y + 3z =  , then λ equals
 14 
  (C) Statement-I is false, statement-II is true
 (2011)
3 2 5 2 (D) Statement-I is true, statement-II is true, statement-II
(A) (B) (C) (D)
2 5 3 3 is a correct explanation for statement-I

Q.18 Statement–I: The point A (1,0,7 ) is the mirror


x y −1 z − 2
image of the point B (1,6,3) in the line
= =
1 2 3
x y −1 z − 2
Statement-II: The line:
= = bisects the line
1 2 3
segment joining A (1,0,7 ) and B (1,6,3) (2011)


JEE Advanced/Boards

Exercise 1 
Q.5 For real number x, y the vector =
 
p xa + y c satisfies
  
the condition 0 ≤ p ⋅ a ≤ 1 and 0 ≤ p ⋅ b ≤ 1. Find the
Q.1 Points X and Y are taken on the sides QR and RS  
maximum value of p. c

respectively, of parallelogram PQRS, so that QX = 4 XR
 
and RY = 4 YS . The line XY cuts the line PR at Z. prove Q.6 For the maximum value of x andy, find the linear
 
  21   combination of p in terms of a and b .
that PZ =   PR .
 25 
Q.7 If O be the origin and the coordinates of P be (1, 2,
Q.2 Given three points on the xy plane on O(0, 0), -3), then find the equation of the plane passing through
A(1, 0) and B(–1, 0). Point P is moving on the plane P and perpendicular to OP.
   
satisfying the condition (PA ⋅PB) + 3(OA ⋅ OB) =0 . If the
 
maximum and minimum values of | PA || PB | are M and Q.8 Given non zero number x1 , x2 , x3 ; y1 , y 2 , y 3 and
m, respectively then find the value of M2 + m2. z1 , z 2 andz3 (i) Can the given numbers satisfy

x1 x2 x3  x1 x2 + y1 y 2 + z1 z 2 =
0
Instruction for questions 3 to 6. 
   y1 y2 y 3 = 0 and x2 x3 + y 2 y 3 + z 2 z3 =
0
Suppose the three vectors, a,b,c on a plane satisfy the 
z1 z2 z3  x3 x1 + y 3 y1 + z3 z1 =0
condition that
     (ii) If x1 > 0 and y1 < 0 for all I = 1, 2, 3 and
|a | = | b | = | c | = | a + b |= 1;c is perpendicular to a and
 P = (x1 , x2 , x3 ); Q(y1 , y 2 , y 3 ) and O(0, 0, 0) can the
b ⋅ c > 0, then
triangle POQ be a right angled triangle?
  
Q.3 Find the angle formed by 2a + b and b .
Q.9 ABCD is a tetrahedron with pv’s of its angular
points as A( −5,22,5);B(1,2,3);C(4,3,2) and D( −1,2, −3).
Q.4 If the
 vector c is expressed as a linear combination If the area of the triangle AEF where the quadrilaterals

λa + µb then find the ordered pair ABDE and ABCF are parallelogram is S, then find the
1 −  2 m1 − m2 n1 − n2 value of S.
, and .
2sin(θ / 2) 2sin(θ / 2) 2sin(θ / 2)
M a them a ti cs | 27.35

Q.10 If x, y are two non-zero and non-collinear vectors Q.5 A line makes angle α , β, γ , δ with the four diagonals
satisfying [(a − 2)α2 + (b − 3)α + c]x + [(a − 2)β2 + (b − 3) β + c]y of a cube then cos2 α + cos2 β + cos2 γ + cos2 δ is equal
to
+[(a − 2) γ 2 + (b − 3) γ + c](x × y) =0
(A) 1 (B) 4/3 (C) ¾ (D) 4/5
where α , β, γ are three distinct real numbers, then find
the value of (a2 + b2 + c2 ).
Q.6 A variable plane passes through a fixed point (a, b,
c) and meets the coordinate axes in A, B, C. The locus of
Q.11 Find the distance of the point (-1, -5, -10)
the point common to plane through A, B, C parallel to
from the point of intersection of the line
coordinate planes is

(
r= 2 ˆi − ˆj + 2kˆ + λ 3 ˆi + 4 ˆj + 2kˆ and the plane) (A) ayz + bzx + cxy = xyz (B) axy + byz + czx = xyz
(
 ˆ ˆ ˆ
r. i − j + k = 5) (C) axy + byz + czx = abc (D) bcx + acy + abz = abc

Q.12 Find the equations of the straight line passing Q.7 The equation of the plane bisecting the acute angle
through the point (1, 2, 3) to intersect the straight between the planes
line x + 1 = 2(y − 2) = x + 4 and parallel to the plane
2x – y + 2z + 3 = 0 and 3x – 2y + 6z + 8 = 0
x + 5y + 4z =0.
(A) 23x – 13y + 32z + 45 = 0
Q.13 Find the equations of the two lines through the (B) 5x – y – 4z = 3
x −3 y −3 z
origin which intersect the line = = at an (C) 5x – y – 4z + 45 = 0
π 2 1 1
angle of . (D) 23x – 13y + 32z + 3 = 0
3

Q.8 The shortest distance between the two straight


Exercise 2
x − 4 /3 y +6 /5 z −3/2
lines = = and
Single Correct Choice Type 2 3 4
5y + 6 2z − 3 3x − 4
= = is
Q.1 If P(2, 3, –6) and Q(3, –4, 5) are two points, the 8 9 5
direction cosines of line PQ are
(A) 29 (B) 3 (C) 0 (D) 6 10
1 7 11 1 7 11
(A) − ,− ,− (B) ,− ,
171 171 171 171 171 171 Q.9 The equation of the straight line through the origin
1 7 11 1 7 11 parallel to the line (b + c)x + (c + a)y + (a + b) z = k
(C) , ,− (D) − ,− ,
171 171 171 171 171 171 = (b – c)x + (c – a)y + (a – b)z is
x y z
(A) = =
Q.2 The ratio in which yz-plane divide the line joining 2
b −c 2 2
c −a 2
a − b2
2

the points A(3, 1, –5) and B(1, 4, –6) is x y z


(B) = =
(A) –3 : 1 (B) 3 : 1 (C) –1 : 3 (D) 1 : 3 b c a
x y z
(C) = =
2 2 2
Q.3 The value of λ for which the lines a − bc b − ca c − ab
3x + 2y + z + 5 = 0 = x + y − 2z − 3 and 2x − y − λz = 0 (D) None of these
=7x + 10y − 8z are perpendicular to each other is
(A) –1 (B) –2 (C) 2 (D) 1 Assertion Reasoning Type

Q.4 The ratio in which yz-plane divides the line joining Q.10 Consider the following statements
(2, 4, 5) and (3, 5, 7)
Assertion: The plane y + z + 1 = 0 is parallel to x-axis.
(A) –2 : 3 (B) 2 : 3 (C) 3 : 2 (D) –3 : 2
Reason: Normal to the plane is parallel to x-axis.
2 7 . 3 6 | 3D Geometr y

(A) Both A and R are true and R is the correct Q.5 Consider three planes
x −3 y −8 z −3
(B) Both A and R are true and R is not a correct AB : = = = λ
explanation of A 3 −1 1
x+3 y +7 z−6
(C) A is true but R is false CD : = = = µ and
−3 2 4
(D) A is false but R is true L ≡ (3λ + 3, −λ + 8, λ + 3)
Let L1, L2, L3 be the lines of intersection of the planes P2
and P3, P3 and P1, P1 and P2, respectively.
Previous Years’ Questions
Statement-I : At least two of the lines L1, L2 and L3 are
Q.1 A plane passes through (1, –2, 1) and is perpendicular non-parallel.
to two planes 2x – 2y + z = 0 and x – y + 2z = 4, then
the distance of the plane from the point (1, 2, 2) is  Statement-II : The three planes do not have a common
 (2006) point  (2008)
(A) 0 (B) 1 (C) 2 (D) 2 2
Paragraph for Q.6 to Q.8
Q.2 Let P(3, 2, 6) be a point in space and Q be a point
 Read the following passage and answer the questions.
on the line r= (iˆ − ˆj + 2k)
ˆ + µ( −3iˆ + ˆj + 5k)
 ˆ . Then the
Consider the lines
value of µ for which the vector PQ is parallel to the
x +1 y + 2 z +1 x −2 y +2 z −3
plane x – 4y + 3z = 1 is  (2009) L1 : = = , L2 : = =
3 1 2 1 2 3
1 1 1 1  (2008)
(A) (B) − (C) (D) −
4 4 8 8
Q.6 The unit vector perpendicular to both L1 and L2 is

Q.3 A line with positive direction cosines passes through − ˆi + 7 ˆj + 7kˆ − ˆi − 7 ˆj + 5kˆ
(A) (B)
the point P(2, –1, 2) and makes equal angles with the 99 5 3
coordinate axes. The line meets the plane 2x + y + z =
9 at point Q. The length of the line segment PQ equals − ˆi + 7 ˆj + 5kˆ 7 ˆi − 7 ˆj − kˆ
(C) (D)
 (2009) 5 3 99
(A) 1 (B) 2 (C) 3 (D) 2
Q.7 The shortest distance between L1 and L2 is
17 41 17
(A) 0 (B) (C) (D)
For the following question, choose the correct answer 3 5 3 5 3
from the codes (A), (B), (C) and (D) defined as follows.
(A) Statement-I is true, statement-II is also true; Q.8 The distance of the point (1, 1, 1) from the plane
statement-II is the correct explanation of statement-I passing through the point (-1, -2, -1) and whose normal
is perpendicular to both the lines L1 and L2 is
(B) Statement-I is true, statement-II is also true;
statement-II is not the correct explanation of statement-I. 2 7 13 23
(A) (B) (C) (D)
(C) Statement-I is true; statement-II is false. 75 75 75 75

(D) Statement-I is false; statement-II is true


Match the Columns
Match the condition/expression in column I with
Q.4 Consider the planes 3x – 6y – 2z = 15 and 2x + y
statement in column II.
–2z = 5.
Statement-I: The parametric equations of the line of Q.9 Consider the following linear equations ax + by +
intersection of the given planes are x = 3 + 14t, y = 1 cz = 0, bx + cy + az = 0, cx + ay + bz =0  (2007)
+ 2t, z =15t.
Column I Column II
Statement-II: The vectors 14iˆ + 2ˆj + 15kˆ is parallel to
the line of intersection of the given planes. (2007) (A) a + b + c ≠ 0 and (p) The equations
2 2 2 represent planes meeting
a + b + c = ab + bc + ca
only at a single point
M a them a ti cs | 27.37

(q) The equation represent Q.14 In R3, consider the planes P1 : y=0 and P2 : x+z= 1.
(B) a + b + c =0 and
Let P3 be a plane, different from P1 and P2, which passes
the line x = y = z
a2 + b2 + c2 ≠ ab + bc + ca through the intersection of P1 and P2. If the distance of
(r) The equations the distance of the point (0, 1, 0) from P3 is 1 and the
(C) a + b + c ≠ 0 and
represent identical planes distance a point (α, β, γ) from p3 is 2, then which of the
a2 + b2 + c2 ≠ ab + bc + ca following relations is (are) true?  (2015)
(D) a + b + c =0 and (s) The equations represent (A) 2α + β + 2γ + 2 = 0 (B) 2α − β + 2γ + 4 = 0
2 2 2 the whole of the three
a + b + c ab + bc + ca
dimensional space (C) 2α + β − 2γ − 10 = 0 (D) 2α − β + 2γ − 8 = 0

Q.10 (i) Find the equation of the plane passing through Q.15 In R3 let L be a straight line passing through
the points (2, 1, 0), (5, 0, 1) and (4, 1, 1). the origin. Suppose that all the points on L
are at a constant distance from the two planes
(ii) If P is the point (2, 1, 6), then the point Q such that
P1 : x +=
2y − z + 1 0 and P2 : 2x=− y + z − 1 0 . Let M
PQ is perpendicular to the plane in (a) and the mid
be the locus of the feet of the perpendiculars drawn
point of PQ lies on it.  (2003)
from the points on L to the plane P1. Which of the
following points lie (s) on M?  (2015)
Q.11 T is a parallelepiped in which A, B, C and D are vertices
of one face and the face just above it has corresponding  5 2  1 1 1
(A)  0, − , −  (B)  − , − , 
vertices A’, B’, C’, D’, T is now compressed to S with face  6 3   6 3 6
ABCD remaining same and A’, B’, C’, D’ shifted to A’’, B’’, C”,
D” in S. the volume of parallelepiped S is reduced to 90%  5 1  1 2
− ,0,
of T. Prove that locus of A” is a plane. (2003) (C)  6 6  (D)  − ,0, 
 3 3

Q.12 Consider a pyramid OPQRS located in the first Q.16 From a point p ( λ , λ , λ ) perpendiculars PQ and
octant ( x ≥ 0, y ≥ 0, z ≥ 0 ) with O as origin, and OP and PR are drawn respectively on the lines = y x,=z 1 and
OR along the x-axis and the y-axis, respectively. The y= −1 . If p is such that ∠QPR is a right angle,
−x, z =
base OPQR of the pyramid is a square with OP = 3 .The then the possible value(s) of λ is(are)  (2014)
point S is directly above the mid-point T of diagonal (A) 2 (B) 1 (C) -1 (D) − 2
OQ such that TS = 3 .Then  (2016)
π Q.17 Perpendiculars are drawn from points on the line
(A) The acute angle between OQ and OS is
3 x + 2 y + 1 z to the plane x + y + z =
= = 3. The feet of
(B) The equation of the plane containing the triangle 2 −1 3
OQS is x ‒ y = 0
perpendiculars lie on the line  (2013)
(C) The length of the perpendicular from p to the plane
3 x y −1 z − 2 x y −1 z − 2
containing the triangle OQS is (A)
= = = =
(B)
2 5 8 −13 2 3 −5
(D) The perpendicular distance from O to the straight
x y −1 z − 2 x y −1 z − 2
(C)
= = = =
(D)
15 4 3 −7 2 −7 5
line containing RS is
2
y z
L1 : x 5,=
Q.18 Two lines= and
Q.13 Let P be the image of the point ( 3,1,7 ) with 3 − α −2
respect to the plane x − y + x =.Then
3 equation of the y z
L2 : x =
α, = are coplanar.
plane passing through P and containing the straight −1 2 − α
x y z The α can take value(s)  (2013)
line = = is  (2016)
1 2 1
(A) 1 (B) 2 (C) 3 (D) 4
(A) x + y − 3z =0 (B) 3x + z =0
(C) x − 4y + 7z =0 (D) 2x − y =0 x −1 y z+3 x−4 y +3 z+
Q.19 Consider the lines L1 : = = ,L2 : = =
2 −1 1 1 1 2
x −1 y z+3 x−4 y +3 z+3
L1 : = = ,L2 : = = and the planes
2 −1 1 1 1 2
2 7 . 3 8 | 3D Geometr y

P1 : 7x + y +=
2z 3,P2 : 3x + 5y −=
6z 4.
x
Let ax + by + cz =
d be the equation of the plane passing Q.22 If f ( x )= t2
(t − 2) ( t − 3) dt for all x ∈ ( 0, ∞ ) then
through the point of intersection of lines L1 and L2.and
∫e
0
perpendicular to planes P1 and P2  (2012)
(A) f has a local maximum at x = 2
Match List I with List II and select the correct answer
using the code given below the list:  (2013) (B) f is decreasing on (2, 3)

List I List II
(C) There exists some c ∈ ( 0, ∞ ) such that f’(c)=0
p. a= 1. 13 (D) f has local minimum at x = 3
q. b= 2. -3
r. c= 3. 1 Q.23 If the distance between the plane Ax − 2y + z =d
s. d= 4. -2 and the plane containing the lines
Codes:
x −1 y − 2 z −3 x −2 y −3 z − 4
= = and = = is 6 , then d is
p q r s 2 3 4 3 4 5
x −1 y − 2 z −3 x −2 y −3 z − 4
(A) = = 3 2and = 4 = 1 is 6 , then d is
2 3 4 3 4 5
(B) 1 3 4 2 (2010)
(C) 3 2 1 4
Q.24 If the distance of the point P (1, −2,1 ) from the
(D) 2 4 1 3 plane x + 2y − 2z =
α , where α > 0, is 5, then the foot
of the perpendicular from P to the plane is  (2010)
Q.20 The point P is the intersection of the straight line
8 4 7  4 4 1
joining the point Q ( 2,3,5 ) and R (1, −1, 4 ) with the plane (A)  , , −  (B)  , − , 
5x − 4y − z =.1 If S is the foot of the perpendicular 3 3 3  3 3 3
drawn from the point T ( 2,1, 4 ) to QR, then the length  1 2 10  2 1 5
(C)  , ,  (D)  , − , 
of the line segment PS is  (2012)  3 3 3  3 3 2
1
(A) (B) 2 (C) 2 (D) 2 2
2 Q.25 Two adjacent sides of a parallelogram ABCD are
 
given by AB = 2i + 10j + 11k and AD = −i + 2j + 2k
Q.21 The equation of a plane passing through the
The side AD is rotated by an acute angle a in the plane
line of intersection of the planes x + 2y + 3z =2 and of the parallelogram so that AD becomes AD'. If AD’
2 makes a right angle with the side AB, then the cosine of
x − y + z =and3 at a distance from the point
3 the angle a is given by  (2010)
(3,1, −1 ) is  (2012)
8 17 1 4 5
(A) 5x − 11y + z =
17 (B) 2x + y= 3 2 − 1 (A) (B) (C) (D)
9 9 9 9
(C) x + y + z =3 (D) x − 2y =
1− 2
M a them a ti cs | 27.39

PlancEssential Questions
JEE Main/Boards JEE Advanced/Boards

Exercise 1 Exercise 1
Q.5 Q.10 Q.23 Q.2 Q.5 Q.8
Q.29 Q.36 Q.40 Q.10 Q.13
Q.42 Q.47 Q.49
Exercise 2
Q.50
Q.2 Q.5 Q.6
Exercise 2 Q.7 Q.9
Q.2 Q.8 Q.12
Previous Years’ Questions
Q.13 Q.14
Q.3 Q.5 Q.6
Previous Years’ Questions Q.9 Q.11
Q.3 Q.6

Answer Key

JEE Main/Boards
Exercise 1
 7 
Q.1 <3, –2, 6> Q.2 <1, 3, –3> Q.3 cos−1  
 70 
x −3 y −2 z +5
Q.4 r = ( −ˆi + ˆj + k)
ˆ + λ(3iˆ − ˆj − 3k)
ˆ Q.5 r = ( −3iˆ + ˆj − k)
ˆ + λ(2iˆ + 4ˆj + 5k)
ˆ Q.6 = =
−2 1 3

Q.7 –3 Q.8 3x + 2y +5z = 7 Q.9 r(3iˆ − ˆj − 4k)


ˆ +7 =0

−1  −2 
Q.10 3iˆ − 7ˆj Q.11 <7, 1, –2> Q.12 sin  
 90 13 
7 1
Q.13 Q.14 Q.15 4, –6, 3
11 3

 1 3 −2 
Q.16   Q.17 <0, 1, 0> Q.18 <2, 3, –10>
 14 14 14 
2 7 . 4 0 | 3D Geometr y

−1  11  −1  −1   1 2 3   −1 1 2 
Q.19 cos   Q.20 sin   Q.21  . , ,  , , 
 21  7 3  14 14 14   6 6 6 
x −3 y z −1
Q.23 (3, 4, 5) Q.26 = = Q.27 x – 2y – 2x – 3 = 0; 2x + y – 2z + 3 = 0
−2 1 3

Q.28 2x + 2y – 3z + 3 = 0 Q.29 13 units Q.30 9x – 8y + z + 11 = 0


3 2
Q.33 5y − 5z − 6 =0, Q.34 2x + 5y + 7z = 78 Q.35 5x + 9y + 11z – 8 = 0
5
π x −5 y + 7 z + 3  1 25 1  13 6
Q.36 4 Q.37 = = Q.39  − , , −  ,
3 1 9  12 12 6  12
25 14
Q.40 r(4iˆ − ˆj − 2k)
ˆ =6
Q.41 42 Q.43 x + 2y – 5z = 0
a + bm + cn
Q.44 (0, 2, –2) Q.45 (ax + by + cz + d) − (a'x + b' y + c'z + a') =
0
(a'1 + b'm + c'm)

Q.46 7y + 5z = 35 Q.48 λ = 3, − 6 Q.49 r(3iˆ + 5ˆj − 6k)


ˆ − 7 70 =
0

Q.50 r(5iˆ + ˆj − 6k)


ˆ =4

Exercise 2
Single Correct Choice Type

Q.1 C Q.2 A Q.3 D Q.4 C Q.5 A Q.6 B


Q.7 B Q.8 D Q.9 C Q.10 A Q.11 D Q.12 B
Q.13 B Q.14 D Q.15 A

Previous Years’ Questions


(2iˆ + ˆj + k)
ˆ 9
Q.1 A Q.2 B Q.3 A Q.4 13 sq units Q.5 ±
sq.unit Q.6 cu unit
6 2

Q.7 2x – y + z – 3 = 0 and 62x + 29y + 19z – 105 = 0 Q.8 C Q.9 A Q.10 C
Q.11 B Q.12 B Q.13 A Q.14 A Q.15 B Q.16 A
Q.17 D Q.18 B

JEE Advanced/Boards
Exercise 1
π  1 2   
Q.2 34 Q.3 Q.4  ,  Q.5 3 p 2(a + b)
Q.6=
2  3 3

Q.7 x + 2y − 3z = 14 Q.8 No, No Q.9 110 Q.10 13 Q.11 13

x −1 y − 2 z −3 x y z x y z
Q.12 = = Q.13 = = or = =
2 2 −3 1 2 −1 −1 1 −2
M a them a ti cs | 27.41

Exercise 2
Single Correct Choice Type

Q.1 B Q.2 A Q.3 D Q.4 A Q.5 B Q.6 A

Q.7 A Q.8 C Q.9 C

Assertion Reasoning Type

Q.10 C

Previous Years’ Question

Q.1 D Q.2 A Q.3 C Q.4 D Q.5 D Q.6 B

Q.7 D Q.8 C

Q.9 A → r; B → q; C → p; D → s Q.10 (a) x + y – 2z = 3 (b) Q(6, 5, –2) Q.12 B, C, D

Q.13 C Q.14 B, D Q.15 A, B Q.16 C Q.17 D Q.18 A, D

Q.19 A Q.20 A Q.21 A Q.22 B, C Q.23 6 Q.24 A

Q.25 B

Solutions

JEE Main/Boards Sol 5: r = − 3 ˆi + ˆj − kˆ + λ (2iˆ + 4ˆj + 5k)


ˆ

Exercise 1
Sol 6: xiˆ + yjˆ + zkˆ = (3 − 2λ )iˆ + (2 + λ )ˆj + ( −5 + 3λ )kˆ
3 −2 6 x −3 y −2 z +5
Sol 1: l = m= n= = =
7 7 7 −2 1 3
Direction ratios are <3, –2, 6>
Sol 7: cos θ = 0 = –3.1 + 3.5 + 2p.2
Sol 2: [2, 1, 0] & [1, –2, 3] ⇒ 12 + 4p = 0 ⇒ p = –3
Direction ratios = 2 – 1, 1 + 2, 0 – 3 = <1, 3, –3>
Sol 8: (xi + yj + zk) . (3i + 2j + 5k) = 7
x y z x −1 y +5 z −3 3x+ 2y + 5z = 7
Sol 3: = = and = =
1 2 0 3 2 1
<1, 2, 0> and <3, 2, 1> Sol 9: 3x – y – 4z = –7; r(3i – j – 4k) = –7
1.3 + 2.2 + 0.1 7  7  r(3iˆ − ˆj − 4k)
ˆ + 7 =0
cos θ = = ⇒ θ =cos−1  
5 14 70  70 
x +1 y −1 z −1 Sol 10: 3x – 7y = –5
Sol 4: = = = t
3 −1 −3 Direction ratios of normal to plane are (3, –7, 0) the
vector along that normal is 3iˆ − 7ˆj .
(
r = ˆi + ˆj + kˆ + t 3iˆ – ˆj – 3kˆ )
2 7 . 4 2 | 3D Geometr y

Sol 11: 7x + y – 2z = 1 
Sol 20: r = 2i – j + 3k + λ (3i – 6j + 2k) and Plane

Direction ratios of vector normal to the plane are r . (i + j + k) = 3
7i + j – 2k = 0 3.1 − 6.1 + 2.1 −1
Sinθ = =
(7, 1, -2) 3 9 + 36 + 4 7 3

Sol 12: Direction ratios of line <4, -5, 7> Sol 21: l - 5m + 3n = 0 ; 7l 2 + 5m2 − 3n2 =
0
Direction ratio of line perpendicular to plane <3, 0, –2> l 5m − 3n
=

4 × 3 + ( −5) × (0) + 7 × ( −2) −2 ⇒ 7(25m2 + 9n2 − 30mn) + 5m2 − 3n2 =


0
sinθ = =
16 + 25 + 49 9 + 0 + 4 90 13 ⇒ 180m2 + 60n2 − 210mn =
0
⇒ 6m2 − 7mn + 2n2 =
0
Sol 13: x + y + 3z + 7 = 0
⇒ 6m2 − 4mn − 3mn + 2n2 =
0
0 + 0 + 3(0) + 7 7
Distance from origin is =
1+1+ 9 11 ⇒ 2m (3m – 2n) – n(3m – 2n) = 0
n 2n
⇒ m = or m =
Sol 14: 3x – 3y + 3z = 0 2 3
3(1) − 3(1) + 3(1) 3 1
Distance from (1, 1, 1) is = = n 2n m
9+9+9 3 3 3 If m = , l = –m, if m = , l=
2 3 2
The following ratio are
Sol 15: 3x – 2y + 4z = 12
Intercept on x-axis (y, z = 0, 0)x = 4 −1 1 2 +1 2 3
, , or , ,
Intercept on y-axis (z, z = 0, 0)y = –6 6 6 6 14 14 14
Intercept on z-axis (x, y = 0, 0)z = 3
Sol 22: Line through the points
Sol 16: <a, b, c> = <1, 3, –2> x −1 y −2 z −3
= = = λ
 1 3 −2  2 4 6
< l , m, n> =  , , 
 1+9+4 1+9+4 1+9+4  x −3 y −3 z −3
= = = λ
5 2 −3
 1 3 −2 
=  , ,  2.5 + 4.2 + 6.( −3)
 14 14 14  cos θ = =0
56 38
θ= 90°
Sol 17: Direction cosines of y-axis = <0, 1, 0>
x −5 y−4 z−4
x+2 2y − 1 3−z Sol 23: = =
Sol 18: = = 4 0 −2
1 3 5
1 equation of line = λ
y−
⇒ x+2 = 2 = z −3
1 3 −5 Let foot of ⊥ is ( α , β, γ )
2 α = 5 + 4λ ; β = 4 ; γ = 4 − 2λ
3
Direction ratio are 1, , − 5 or 2, 3, − 10 ⇒ (α − 1). 4 + (β − 2) . 0 + ( γ − 1) . ( −2) = 0
2
−1
⇒ (4 + 4λ )4 − 2(3 − 2λ ) =0 ⇒ 20λ + 10 =0 ⇒ λ =
  2
Sol 19: r . (i – 2j – 2k) = 1; r . (3i – 6j + 2k) = 0
 −1 
⇒ α = 5 + 4  = 3 β =4 = 4
3.1 + ( −6).( −2) + (2)( −2) 3 + 12 − 4 11  2 
cos θ = = =
1 + 4 + 4 9 + 36 + 4 3.7 21
 −1 
⇒ γ = 4 − 2 = 5
 11   2 
θ =cos−1  
 21  (3, 4, 5)
M a them a ti cs | 27.43

Sol 24: cos (δθ) Sol 27: x + 3y + 6 = 0, 3x – y – 4z = 0

l . (l + δl ) + m . (m + δm) + n . (n + δn) x + 3y + 6 + λ (3x – y – 4z) = 0


=
l 2 + m2 + n2 (l + δl )2 + (m + δm)2 + (n + δn)2 x(1 + 3λ ) + y (3 – λ ) + z (-4 λ ) + 6 = 0
6
 neglecting  Distance from origin = =1
 2 2 2 (1 + 3λ ) + (4λ )2 + (3 − λ )2
2
δl , δm , δn 
36 = 1 + 9λ2 + 6λ + 16λ2 + 9 + λ2 − 6λ
l 2 + m2 + n2 + l δl + mδm + nδn
= 36 = 26λ2 + 10
2 2 2 2 2 2
(l + m + n ) (l + 2l δl + m + 2l δm + n + 2l δn)
λ = ±1
2
1 − (δθ) 1 + l δl + mδm + nδn Planes are ⇒ 4x + 2y – 4z + 6 = 0 (λ =1)
=
2 1
−2x + 4y + 4z + 6 =0 (λ = −1)
(δθ)2 = −2(l δl + mδm + nδn)  … (i)
Sol 28: ax + by + cz = 1  … (i)
l 2 + m2 + n2 = (l + δl )2 + (m + δm)2 + (n + δn)2
(–1, 1, 1) lies on (1)
⇒ (δl )2 + (δm)2 + (δn)2 =−2l δl − 2mδm − 2nδn  … (ii)
⇒ –a+b+c=1
(δθ)2 = (δl )2 + (δm)2 + (δm)2
(1, –1, 1) lies on (1)

Sol 25: y ⇒ +a – b + c = 1 ⇒ c = 1
If ⊥ to x + 2y + 2z =
5
y1 P (x1, y1, z1 )
a•1+b•2+2•c=0
x
O
z1 a + 2b = –2
x1
Z a–b=0
 l1 + l2 + l3   m1 + m2 + m3   n1 + n2 + n3 
  l1 +   m1 +   .n1 −2
 3   3   3  a=b=
3
Equation of plane is–2x – 2y + 3z = 3.
1 + l1l2 + l1l3 + m1m2 + m3m1 + n1m2 + n1n3 1
= =
3 3
Sol 29: P (–1 + r cos α , – 5 + r cos β , – 10 + r cos γ)
1
Similarly dot product with l2 and l3 gives as result
3 are coordinates of point at distance r from (–1, –5, –10)
i.e. it makes same angle with (l1 ,m1 ,n1 ) (l2 ,m2 ,n2 ) and along α , β, γ
(l3 ,m3 ,n3 ) Point P lies on the given plane
x–y+z=5
Sol 26: x + 2y = 0 … (i)
– 1 + r cos α + 5 – r cos β + r cos γ – 10 = 5
3y – z = 0
r cos α – r cos β + r cos γ = 11
2z
2y – = 0 … (ii) 11 11.13
3 r= = = 13 units
3 − 4 + 12 11
The line will be across (a1 , b1 , c1 ) × (a2 , b2 ,c2 )
13
(1 2 0) × (0 3 –1)
Sol 30: ax + by + cz = 1
i j k
(1, 2, –4)
1 2 0 = i (–2) –j (–1) + k(3) = –2i + j + 3k
a + 2b – 4c = 1  … (i)
0 3 −1
This plane is parallel
x −3 y − 0 z −1
Equation of line will be = = r1 = i + 2j + 4k + λ (2i + 3j + 6k)
−2 1 3
r2 = i – 3j + 5k + λ (i + j – k)
2 7 . 4 4 | 3D Geometr y

⇒ 2a + 3b + 6c = 0 Distance from (1, 1, 1)

⇒ a+b–c=0 −5 + 5 + 6 6 3 2
= = =
25 + 25 5 2 5
⇒ b = –8c
⇒ a = 9c Sol 34: Ratios of line perpendicular to plane is {(2 – 0),
⇒ 9c – 16c – 4 c = 1 (5 – 0), (7 – 0)}
−1 +8 −9 Equation of plane is 2x + 5y + 7z = k
⇒ c= ,b= ,a =
11 11 11 (2, 5, 7) lies on the plane
Equation of plane is –9x + 8y – z = 11 or
2.2 + 5.5 + 7.7 = k = 78

⇒ r . (–9i + 8j – k) = 11
2x + 5y + 7z = 78
Sol 31: al + bm + cn =
0  … (i)
Sol 35: Direction ratios of line ⊥ to the given planes
and f mn + gnl + hlm =
0 ... (ii)
3x + 2y – 3y = 1; 5x – 4y + z = 5
f g h
⇒ + + =0  ... (iii) i j k
l m n
3 2 −3 =i(2 – 12) – j (3 + 15) + k (– 12 – 10)
Comparing (i) and (iii)
5 −4 1
a 2 b 2 c 2
l = m = n = λ
f g h = −10i − 18 j − 22k
f f Plane will be 10x + 18y + 22z = k
⇒ l2 = λ ⇒ l =± λ
a a Passes through (–1, –1, 2)
Similarly
2 . (22) – 28 = k ∴ K = +16
g g
m2 = λ ⇒ m =± λ 10x + 18y + 22z – 16 = 0
b h
⇒ 5x + 9y + 11z − 8 =0
2 h h
n = λ ⇒ n =± λ
c c
Sol 36: l + m + n =0 and l2 + m2 =
n2
Since, lines are ⊥
⇒ n =− (l + m)
cos=
θ l1 l2 + m1 m2 + n1 n= 0
⇒ l2 + m2 = (l + m) =l2 + m2 =l2 + m2 + 2m.n
2 2

f g h
− λ − λ − λ =0
a b c ⇒ m.n =
0
f g h  f g h ⇒
= m 0 or=n 0
⇒ λ  + +  =0 ⇒ + + =0
a b c  1 1 
a b c   1
⇒ (l,m,n) ≡  −
1  ,− , 0
,0 ,  or 
 2 2  2 2 
x −b z−d
Sol 32: =y= … (i) ⇒ Angle = π
a c  4
x − b' z − d'
=y= … (ii)
a' c'  Sol 37:
These 2 are perpendicular if aa’ + cc’ + 1 = 0
i j k
Sol 33: 4x – y + z – 10 + λ (x + y – z – 4) = 0 1 −3 0 =i(3) – j (–1) + k (9) =3i + j + 9k
0 +9 −1
⇒ x(4 + λ ) + y( −1 + λ ) + z(1 − λ=
) 10 + 4λ x −5 y+7 z+3
= =
⇒ (4 + λ ) • 2 + (λ − 1) •1 + (1 − λ ) •1 = 0 3 1 9

⇒ 8 − 1 + 1 + 2λ =0 ⇒ λ = −4 Sol 38: ax + by + cz = 1
⇒ −5y + 5z = −6 , equation of plane a + b + c = 1⇒ a + c = 1 – b
M a them a ti cs | 27.45

a – b + c = 1⇒ b = 0 Sol 41: distance between 2x – y + 3z = 4

–7a + 3b – 5 +5a = 1 −13


2x – y + 3z =
3
b = 6 + 2a/3,a = –3,c = 4 13
4+
3 25 25 14
–3x + 4z = 1 → ratio → [-3, 0, 4] Distance, d = = =
4 +1+ 9 3 14 42
xz plane → ratio → [0, 1, 0]
–3.0 + 0.1 + 4.0 = 0 Sol 42: ax + by + cz = 1
Hence given plane is perpendicular to xz plane. 1   1   1
A  ,0,0  , B  0, ,0  , C  0,0, 
(1,1,2) a   b   c

α −1 β −1 γ −2 1 1 1
Sol 39: = = = 3α , = 3β , = 3γ
2 −2 4 a b c

(a,b,g)
x y z
+ + =3
2x α β γ
−(2 − 2 + 8 + 5) −13
= = Sol 43: x − 2y + 3z + 4 + λ(x − y + z + 3) = 0
4 + 4 + 16 24
−4
13 1 13 25 Through origin 3λ + 4 =0 ; λ=
α =1− =− , β=1+ = 3
12 12 12 12
 4  4  4
13 −1 ⇒ x  1 −  + y  −2 +  + z  3 −  =0
γ =2– =  3   3   3 
6 6
2−2+ 8 +5 13  −x 2y 5z
Length = = ⇒ r − + 0
=
24 24 3 3 3
⇒ x + 2y – 5z = 0
Sol 40: ax + by + cz = 1
(1, 0, –1) ⇒ a – c = 1 Sol 44: 2x + 2y − z − 6 + λ(2x + 3y − z − 8) =0
(3, 2, 2) ⇒ 3a + 2b + 2c = 1 x(2 + 2λ ) + y(2 + 3λ ) + z( −1 − λ ) − 6 − 8λ =0 equation of
plane
It is parallel to 1, −2,3
xz plane 0,1,0 any point on the line is (α ,2,2α − 2)
⇒ a – 2b + 3c = 0
Direction ratios of line
⇒ 4a + 5c = 1
⇒ 4 + 4c + 5c = 1 i j k
2 2 −1 = i( −2 + 3) − j( −2 + 2) + k(6 − 4)
−1
⇒ c= 2 3 −1
3
2 = i + 2k = 1,0,2
⇒ a=
3
2 3 This is parallel to plane y = 0 as
⇒ − = +2b
3 3 (1, 0, 2) . (0, 1, 0) = 0
−1
b= α =0 i.e. (0, 2, –2)
6
y
Eq. of plane 2x − –z=3 Sol 45: The equation of Plane
2
4x – y – 2z = 6 ax + by + cz + d + λ ( a' x + b' y + c' z + d' ) = 0  … (i)

⇒ r ⋅ (4iˆ − ˆj − 2k)
ˆ =6 ⇒ ( a + λa' ) x + (b + λb ) y + ( c + λ c' ) z + d + λd' = 0
x y z
Which parallel to line = =
l m n
2 7 . 4 6 | 3D Geometr y

⇒ ( a + λ a' ) l + (b + λ b' ) m + ( c + λ c' ) n = 0 Sol 50: r = i − j + λ(i + j + k) + µ(4i − 2 j + 3k)

al + bm + cn i j k
⇒− =λ
a'l + b'm + c'n B = 1 1 1 = i(5) – j(–1) + k(– 2 – 4) = 5i + j – 6 k
Substituting in (i) 4 −2 3

Plane pass through (1, –1, 0)


( ax + by + cz + d) − a'lal++b'm
bm + cn
+ c'n
( a' x + b' y + c' z + d) =
0 
Equation of plane r • (5i + j – 6k) = z

Sol 46: ax + by + cz = 1 5(1) + 1(–1) – 6 (0) – z = 4



1 1 The equation of plane ⇒ r • (5i + j – 6k) = 4
= 5, = 7{given intercepts}
b c
<a, b, c> • <1, 0, 0> = 0 Exercise 2
a=0
Single Correct Choice Type
y z
+ =1 ; 7y + 5z = 35
5 7 Sol 1: (C) l 2 + m2 +=
n2 cos2 α + cos2 β + cos=
2
γ 1

Sol 47: ax + by + cz = 1 Sol 2: (A) ax + by + cz + d = 0 to intersect x and y axis


1 at equal angle
= P ……(i)
a + b2 + c2
2 | tan α=
| | tan β | ⇒ | a | = | b |

1   1   1
A  ,0,0  ; B  0, ,0  ; C  0,0,  Sol 3: (D) Parallel to x-axis i.e. <1, 0, 0>
a   b   c
x−a y −b z−c
1 1 1 = =
x= ,y= ,c= 1 0 0
a b z
1 1 1 1 1 1
= a2 + b2 + c2 = + + from (i) Sol 4: (C) cos α = cos
= γ cos60
= °
P2 x2 y2 z2 2 2
2 2 2
l +m +n =
1
Sol 48: i – j + 3k from 5x + 2y – 7z + 9 = 0
1 1 1
⇒ + + m2 =
1 ⇒ m2 =
5 − 2 − 21 + 9 9 2 4 4
⇒ =
49 + 4 + 25 78 1
⇒ m = = cosβ
2
⇒ (3i + λj + 3k) from 5x + 2y – 7z + 9 =0 ⇒ β= 60°
15 + 2λ − 21 + 9 3 + 2λ
⇒ = ⇒ 3 + 2λ =9 x +1 y +3 z + 2
49 + 4 + 25 78 Sol 5: (A) = = = λ
1 3 −2
⇒ λ =3 or –6
⇒ ( −1 + λ , − 3 + 3λ , − 2 − 2λ )

Sol 49: Normal to vector 3i + 5j – 6k ⇒ 3 ( −1 + λ ) + 4 ( 3λ − 3 ) + 5 ( −2 − 2λ ) =5


3x +5y – 6z = k ⇒ 5λ − 3 − 12 − 10 =5 ⇒ 5λ =30

at 7 units from origin ⇒ x=6


k (5, 15, –14)
= 7 ; k = ± 7 70
36 + 25 + 9
 Sol 6: (B)
r ⋅ (3iˆ − 5ˆj − 6k)
ˆ = ± 7 70
x−0 y −3 z−4 (0 − 6 + 4 − 10)
= = = −
2 −2 1 9
M a them a ti cs | 27.47

⇒ x = y − 3 = z – 4 = 12 = 4 4 × 74 − 144 296 − 144 152 38


= = = =
2 −2 3×3 3 16 16 16 4
8 8 4
⇒x= ,y=3– ,z=4+ 38
3 3 3 ∆=
8 1 16 2
⇒ , ,
3 3 3
x+2 y −1 z+4
Sol 11: (D) = =
Sol 7: (B) 2x + y – z – 4 + λ (3x + 5z – 4) = 4 3 −1 1
−1 x −1 y z+3
2 + 3λ = 1 ⇒ λ = or = =
3 3 −1 1
5 4
⇒ 2x – x + y – z – z – 4 + =0 Sol 12: (B) Let P(x, y, z) be any point on the locus, then
3 3 the distances from the six faces are
⇒ 3x + 3y – 8z – 8 = 0
|x + 1|, |x− 1|, |y + 1|, |y− 1|, |z + 1|, |z− 1|

i j k According to the given condition

Sol 8: (D) 1 1 −1 = i(–2) – j(3) + k(–3 –2) |x + 1|2 + |x − 1| + |y + 1|2 + |y − 1|2 + |z + 1|2 + |z− 1|2
2 −3 1 = 10
⇒ 2(x2 + y2 + z2) = 10 − 6 = 4
= –2i – 3j – 5k
⇒ x2 + y2 + z2 = 2
It passes through (1, 0, 0)
y z
Equation of line is
x −1
= = Sol 13: (B) If ( x1 , y1 , z1 ) , ( x2 , y 2 , z2 ) , ( x3 , y3 , z3 ) and
2 3 5
( x 4 , y 4 , z 4 ) are coplanar, then
x −1 y − 3 z − 4
Sol 9: (C) Line = = is parallel to plane x2 − x1 y 2 − y1 z 2 − z1
1 2 3
ax + by + cz = 1 x3 − x1 y 3 − y1 z3 − z1 =0
x 4 − x1 y 4 − y1 z 4 − z1
If a + 2b + 3c = 0
Only C satisfies the condition −4 − 0 4 + 1 4 + 1 −4 5 5
4 −0 5+1 = 1+1 ⇒ 4=6 2 0
Sol 10: (A) a = 49 + 1 + 1 = 51 ; 3−0 9 +1 4 +1 3 10 5

=−4 ( 30 − 20 ) − 5 ( 20 − 6 ) + 5 ( 40 − 18 ) =−40 − 70 + 110 =0


b= 1+ 0 +1 = 2 ;c = 36 + 1 + 0 = 37

2 + 51 + 37 Sol 14: (D) The plane y + z + 1 =0


s=
2
Since the plane does not have any intercepts on x-axis,
s(s – a) (s – b) (s – c) therefore it is parallel to x-axis.
51 + 37 + 2  2 + 37 − 51  Then normal to plane can not be parallel to x-axis.
⇒  
2  2 
Sol 15: (A) Using the fact that reflection of a' x + b' y +
 2 + 51 − 37   37 + 51 − 2  c'z + d' = 0 in the plane ax+ by + cz + d = 0 is given by
   
2 2 2 (aa' + bb' + cc') (ax+ by + cz + d)
   
= (a2 + b2 + c2) (a' x + b' y + c' z + d')
37 + 2 + 2 74 − 51 31 − (37 + 2 − 2 74 
=    We get the required equation as
16
2 (2 + 3 + 4) (x−y + z−3) = (1 + 1 + 1)(2x−3y + 4z−3)
2 74 − 12 12 + 2 74 
   6 (x− y + z − 3) = 2x− 3y + 4z −3
=
16 4x− 3y + 2z− 15 = 0
2 7 . 4 8 | 3D Geometr y

Previous Years’ Questions Sol 4: Area of ∆ABC =


1 → →
(AB × AC), where
2
Sol 1: (A) Given equation of straight line → →
AB =ˆi + 2ˆj − 3kˆ and AC = 2iˆ + 0ˆj + 0kˆ
x − 4 y −2 z −k
= = ˆi ˆj kˆ
1 1 2 → →
Since, the line lies in the plane 2x – 4y + z = 7 ∴ AB × AC = 1 2 −3 = 2( −3ˆj − 2k)
ˆ
2 0 0
∴ Point (4, 2, k) must satisfy the plane.
⇒8–8+k=7 ⇒ k=7 1 → →
⇒ Area of triangle
= (AB × AC)
2
Sol 2: (B) Since, the lines intersect they must have a 1
= .2. 9 +
= 4 sq.units
13 sq units
point in common 2
x −1 y +1 z −1
i.e., = = = λ Sol 5: A unit vector perpendicular to the plane
2 3 4
→ →
x −3 y −k z (PQ ×PR)
and = = = µ determined by P, Q, R = ±
1 2 1 → →
| PQ ×PR |
⇒ x = 2λ + 1, y = 3λ − 1, z = 4λ + 1 → →
where PQ = [iˆ + ˆj] − 3kˆ and PR =−ˆi + 3ˆj − kˆ
and x = µ + 3, y = 2µ + k, z = µ are same
ˆi ˆj kˆ
⇒ 2λ + 1 = µ + 3, 3λ − 1 = 2µ + k, 4λ + 1 = µ → →
∴ PQ ×=
PR 1 1 −3
On solving Ist and IIIrd terms, we get,
−1 3 −1
3
λ=− and µ = −5
2 = ˆi( −1 + 9) − ˆj( −1 − 3) + kˆ (3 + 1) = 8iˆ + 4ˆj + 4kˆ
 3 9
∴ k = 3λ − 2µ − 1 ⇒ k = 3  −  − 2( −5) − 1 = → →
 2 2 ⇒ | PQ × PR =
| 4 4 +1 +=
1 4 6
9
∴ k= → →
2 ∴ Unit vector = ± (PQ × PR) = 4(2iˆ + ˆj + k)
ˆ (2iˆ + ˆj + k)
ˆ
± =
±
→ → 4 6 6
Sol 3: (A) Since, x + y + z = | PQ × PR |
1
a b c y
cuts the coordinate axes at Sol 6: Let the equation of plane through
B (0, b, 0) (1, 1, 1) having a, b, c as DR’s of normal to plane,
A(a, 0, 0), B(0, b, 0), C(0, 0, c) a(x − 1) + b(y − 1) + c(z − 1) =
0 and plane is parallel to
A x straight line having DR’s.
and its distance from origin (a, 0, 0)
=1 C (1, 0, –1) and (–1, 1, 0)
1 z (0, 0, c)
∴ =1 ⇒ a – c = 0 and –a + b = 0
1 1 1 ⇒a=b=c
2
+ 2
+
a b c2
∴ Equation of plane is x – 1 + y – 1 + z – 1 = 0
1 1 1
or + = 1 + … (i) x y z
a b 2
c2 2 or + + =1. Its intercept on coordinate axes are
 3 3 3
where P is centroid of triangle
A(3, 0, 0), B(0, 3, 0), C(0, 0, 3)
a b c
=
∴ x = ,y = ,z … (ii) Hence, the volume of tetrahedron OABC
3 3 3
3 0 0
∴ From Eqs. (i) and (ii), we get 1  1 27 9
= [ab c]
= = 0 3 0 = cu units
1 1 1 1 1 1 6 6 6 2
+ + 1 or
= + + =9 =K 0 0 3
2 2 2 2 2
9x 9y 9z x y z2

∴K =9
M a them a ti cs | 27.49

Sol 7: Equation of plane containing the lines 3r + 2 − 4r + 1 + 12r + 2 − 16 =0


2x − y + z − 3 =0 and 3x + y + z =
5 is ⇒ 11r − 11 = 0
(2x − y + z − 3) + λ(3x + y + z − 5) =0 ⇒r = 1
The point in (5,3,14 )
⇒ (2 + 3λ ) x + (λ − 1) y + (λ + 1) z − 3 − 5λ =0

(5 − 1) + (3 − 0 ) + (14 − 2)
2 2 2
Since, distance of plane from (2, 1, –1) to above plane Distance =
is 1 / 6. = 16 + 9 + 144
6λ + 4 + λ − 1 − λ − 1 − 3 − 5λ 1 = 169 13
=
∴ =
2 2
(3λ + 2) + (λ − 1) + (λ + 1) 2 6
Sol 11: (B) Let the two lines in a same plane interest at
⇒ 6(λ − 1)2= 11λ2 + 12λ + 6
P ( x, y,0 ) , then 2x=
− 5y 3 and =x+y 5
24
⇒ λ= 0, −
5 On solving, we get P ≡ ( 4,1,0 )
∴ Equations of planes are 2x − y + z − 3 =0 Any plane || to x + 3y + 6z =
1 is
and 62x + 29y + 19z − 105 =
0 x + 3y + 6z =
λ
P ( 4,1,0 ) must
4 +satisfies
3 + 0 =it,
λ then
x −3 y +2 z + 4
Sol 8: (C) The line = = lies in the plane,
2 −1 3 4 + 3 + 0 =λ ⇒ λ =7
then point ( 3, −2, −4 ) lies on the plane ⇒ λ =7
The eq. to required plane
5 …(i)
⇒ 3 − 2m = ⇒ x + 3y + 6z = 7
−8 × 2 − 5
And line is ⊥ to normal of plane Sol 12: (B) The parallel planes 2x +16 4 +=
y ++2z 16
8
−8 × 2 − 521
⇒ 2 − m =
3 …(ii) =
and 4x + 2y + 4z = −5 16 + 4 + 1636
From (i) and (ii) −8 × 2 − 5 21 21 7
 = 1 and m = − 1 Distance = = = =
16 + 4 + 16 36 6 2
⇒ 2 + m2= 12 + ( −1)2 = 2 21 21 7
= = =
36 of point (1,3,
Sol 13: (A) Image 6 42) is
Sol 9: (A) The eq of line passes through (1, − 5, 9 ) 21 7
x − 1 y= −3 = z − 4 −2 ( 2 − 3 + 4 + 3 )
along x= y= z is = =6 2= = −2
2 −1 1 4 +1+1
x −1 y +5 z − 9 ⇒ ( −3, 5, 2 )
= = = r
1 1 1
Since line is parallel to plane direction, ratio will not
The point on line (r + 1, r − 5, r + 9 ) change
This point also lies on the given plane x + 3 y −5 z − 2
Eq. of imaged line = =
r +1−r +5+r + 9 = 5 3 1 1
r = −10 Sol 14: (A)  + m + n =0 ⇒ n =− (  + m)
The point in ( −9, − 15, − 1 ) 2
Substituting in = m2 + n2
Distance between (1, −5,9 ) and ( −9, −15, −1 )
2 = m2 + (  + m)
2

102 + ( −10 ) + (10=


) 10 3 unit
2 2
= ⇒ 2 = m2 + 2 + m2 + 2m
⇒ 2m2 + 2m =
0
x −2 y +1 z −2 ⇒ 2m (m + 1 ) =
0
Sol 10: (C) = = = r
3 4 12 ⇒m=
0,1
The point of interstation ( 3r + 2, 4r − 1, 12r + 2 ) −1 1
if =
m 0,= ,n
=
Lies on plane, then 2 2
if m= 1, = 0= n (not possible)
⇒ 2 = m2 + 2 + m2 + 2m
⇒ 2m2 + 2m =
0
2⇒
7 .2m
50 m(
| +3D )
1 Geometr
= 0 y

⇒m=
0,1
−1 1 Sol 18: (B) Statement-I: Since mid point of A(1, 0, 7)
if =
m 0,= ,n
= and B(1, 6, 3) is which lies on the line, therefore point B
2 2
is image of A about line
if m= 1, = 0= n (not possible)
Statement-II: Since it given that the line only bisects
Therefore direction cosine the line joining A and B, therefore not the correct
explanation.
 1 1   1 1 
− ,0,  or  − , ,0   1
2 2  2 2  1   1 1 
 − ,0,  or  − , ,0 
 1  1   1   1  1  2 2  2 2 
cos φ =  −  −  + (0)  +  (0) =
 2  2  2  2 2
 1   −1   1 
⇒φ=
π cos θ =  −   + (0)   + (0)
3  2  2   2
x −2 y −3 z − 4 1 π
Sol 15: (B) The lines = = and = ⇒9=
1 1 −k 2 3

x −1 y − 4 z −5
= = are coplanars, then
k 2 1
JEE Advanced/Boards
1 1 −k
k 2 1 =0
1 −1 1
Exercise 1
 
⇒ k (k + 3 ) =
0 Sol 1: Let point P be taken as origin and q, s are the
⇒ K = 0, −3 position vectors of Q and S points respectively.
  
⇒ PR =q + s
Two values exist.
1 Y 4
Sol 16: (A) Eq. of plane parallel to x − 2y + 2z − 5 =0 is R
  1
x − 2y + 2z =
λ s z
1 1 X
⊥ distance from origin is 1, 4
P  Q
0−0+0−λ λ 
(o)
then =1 ⇒ 1 ⇒ λ = ±3
=
1+4+4 3
    
Eq. of plane x − 2y + 2z =
±3 q + 4 ( q + s ) 5q + 4 s
P.V. of X =
=
5 5
    
1 + 4 + 3λ 4s + q + s q + 5 s
Sol 17: (D) sin θ = P.V. of Y =
=
1+4+9 1+4+λ 5 5
5 + 3λ  ... (i) PZ 1 YZ
= = and = µ
 14 5 + λ Let,
ZR λ ZX

5  4  q 
Given cos θ = µq+ s  +  + s 
14 q+ s  5  5 
P.V. of
= P =
λ +1 µ +1
5 3
⇒ sin =
θ 1 − cos2 θ= 1− =
14 14 1
µ+
1 5 … (i)
3 5 + 3λ ⇒ =
From (i) = λ +1 µ +1
14 14 5 + λ
1
µ+
2 2 2
(
⇒ 3 5 + λ = (5 + 3λ ) ⇒ 9 5 + λ = 25 + 9λ + 30λ ) ⇒
1
= 5 
λ +1 µ +1
… (ii)

2
⇒ 30λ =20 ⇒ λ =
3
M a them a ti cs | 27.51

From (i) & (ii), we get


1 2 ⇒  1 , 2 
⇒λ= ,µ=  
4  3 3
µ= 4, λ= 3 3
21
    
PZ 21 PZ 21 Sol 5: P = x a + y c ⇒ p= y c
⇒ =⇒ =
ZR 4 PR 25 
0 ≤ p.a =x ≤ 1 x ∈ [0,1]
  
Sol 2: p (x ,y) 0 ≤ p.b = xa.b ≤ 1 x ∈ [ −2,0] ⇒ x =0
     
PA • PB + 3OA • OB = 0 p. c = y
 
 a 2b
(x − 1)(x + 1) + y 2 + 3( −1) =0 c
= +
3 3
x2 + y 2 =
4  
  p.a 2p.b
  p.=c +
PA ⋅ PB 3 3
 
   a + 2b 
p xa + y 
= 
(x − 1)2 + y 2 (x + 1)2 + y 2
 3 
(5 − 2x) (5 + 2x) = 25 − 4x2 y 2y
Sol 6: For max. x and y; x + =⇒ y = 3 x
3 3
Max is 25 = 5 = M
x = 1; y = 3
  
Min = 9 =3 ⇒ p = 2a + 2b
2 2
M +m =
34
Sol 7: The coordinates of the points, O and P, are (0, 0,
     0) and (1, 2, -3) respectively.
Sol 3: | a | = | b | = | c | = | a + b | = 1
Therefore, the direction ratios of OP are (1 - 0) = 1, (2 -
      −1 0) = 2 and (-3-0) = -3
⇒ | a |2 + | b |2 +2a ⋅ b =
1; a ⋅ b =
2 It is knows that the equation of the plane passing
θ 120°
=
through the point ( x1 y1 z1 ) is
  
(
∠ 2a + b & b ) a ( x − x1 ) + b ( y − y1 ) + c ( z − z1 ) =
0 , where a, b and c
⇒ (2a + b) (b) = |2a + b| |b| cos θ1 are the direction ratio of normal.
Here, the direction ratios of normal are 1, 2 and -3 and
⇒ 2a . b + | b |2 = 4a2 + b2 + 4a . b | b | cos θ1
the point P is (1, 2, -3).
⇒ −1 + 1 = cos θ1 × k Thus, the equation of the required plane is
⇒ cos θ1 =0 1 ( x − 1 ) + 2 ( y − 2 ) − 3 ( z + 3) =
0
π ⇒ x + 2y − 3z − 14 =0
⇒ θ1 =
2
  
 Sol 8: (i) A = [x1 y1 z1 ] ; B = [x2 y 2 z 2 ] ; C = [x3 y 3 z3 ]
Sol 4: c =λa + µb
    
| c |2 =λ2 + µ2 + 2λµa . b =1 (
A B×C = )0 all are coplanar
2
⇒ λ + µ − λµ =12      
A × B = 0 = B × C = C × A i.e. all are mutually ⊥ which
    simultaneously is not possible.
⇒ c . a = λ + µ (a . b) = 0
µ (ii) P = (x1 , y 2 , x3 ) Q = (y1 y 2 y 3 ) O (0,0,0)
⇒λ− =0
2 In ∆POQ; OP = x1i + x2 j + x3k
µ
⇒ λ = ⇒ u = 2λ OQ = y1i + y 2 j + y 3k
2
OP . OQ = x1 y1 + x2 y 2 + x3 y 3 x1 > 0 y1 > 0
⇒ λ2 + 4λ2 − 2λ2 =1
OP . OQ < 0 [i.e. it can never be zero]
2 7 . 5 2 | 3D Geometr y

Sol 9: A = ( −5,22,5) ; B = (1,2,3) ; C = (4,3,2) Substituting this value in equation (i), we obtain the

D=
( −1,2, −3) equation of the line as r = 2 ˆi − ˆj + 2kˆ

This means that the position vector of the point of


and ∆AEF = S 
   intersection of the line and the plane is r = 2 ˆi − ˆj + 2kˆ
DE
= DA + DB
This shows that the point of intersection of the given
OE – OD = OA – OD + OB – OD line and plane is given by the coordinates (2, -1, 2). The
 point is (-1, -5, -10).
OE = −5i + 22 j + 5k + i + 2 j + 3k − ( −i + 2 j − 3k)
 The distance d between the points, (2, -1, 2) and
OE = −3i + 22 j + 11k (-1, -5, -10), is
  
BF
= BA + BC
( −1 − 2) + ( −5 + 1) + ( −10 − 2)
2 2 2
d=
   
OF = OA + OC − OB = 9 + 16 + 144 = 169 = 13

OF =−5i + 22 j + 5k + 4i + 3 j + 2k − i − 2 j − 3k
x −1 y − 2 z −3
OF = −2i + 23 j + 4k Sol 12: = =
a b c
1     1
Area = AE × AF = (2i + 6k) × (3i + j − k) Parallel to the plane x + 5y + 4z =
0
2  2 
ˆi ˆj kˆ ⇒ a + 5b + 4c = 0
1
1
2 0 6 = ˆi( −6) − ˆj( −2 − 18) + k(2)
ˆ =−6iˆ + 20ˆj + 2kˆ ⇒ ( −1 + 2λ , 2 + λ , −4 + 2λ ) = ⇒ (1 + ka,2 + kb,3 + kc )
2 2
3 1 −1
ka + 2 2kb 7 + kc
⇒ = =
1 440 2 2 2
= 36 + 400 + 4 = = 110 ⇒ S =110
2 4 2 7 5
⇒ = =
2b − a 2b − c a−c
(
Sol 10: (a − 2)α2 + (b − 3)α + c x + ) ⇒ 10b =7a − 2c  ……..(i)
(
⇒ (a − 2)β2 + (b − 3)β + c y + ) ⇒
7a − 10b
=
−a − 5b
⇒ ( (a − 2)γ 2
+ (b − 3)γ + c ) ( x × y ) =0 2 4
−3a
⇒a–2=b–3=c=0 ⇒ a=b;c= a = 2b = 2c = –3
2
⇒a=2;b=3;c=0 x y z
Sol 13: = = = λ
⇒ a2 + b2 + c2 =
13 a b c

λa = 3 + 2k ; λb = 3 + k ; λc =k
Sol 11: The equation of the given line is
 1 2a + b + c
r= 2iˆ − ˆj + 2kˆ + λ(3iˆ + 4ˆj + 2k)
ˆ
 … (i) = 3 (a2 + b2 + c2 ) = 2(2a + b − c)2
2 2 2 2
The equation of the given plane is 6 a +b +c


( )
r. ˆi − ˆj + kˆ =5 … (ii)
3 + 2k 3 + k k
= =
a b c
Substituting the value of from equation (i) in equation 3a − 3b 3c 3c
(ii), we obtain. = =
2b − a b − c a − 2c
( )( )
2 ˆi − ˆj + 2kˆ + λ 3 ˆi + 4 ˆj + 2kˆ  ˆi − ˆj + kˆ =5
  ⇒ a = 1, b = 2, c = –1 or a = –1 b = 1, c = –2
⇒ ( 3 λ + 2 ) ˆi + ( 4 λ − 1 ) ˆj ( 2 λ + 2 ) kˆ  . ( ˆi − ˆj + kˆ ) =5
 
⇒ ( 3 λ + 2 ) − ( 4λ − 1 ) + ( 2λ + 2 ) =5
⇒ λ =0
M a them a ti cs | 27.53

Exercise 2 h − a −b −c
−a k − b −c = 0
Single Correct Choice Type −a −b l − c

Sol 1: (B) Direction cosines of PQ (2, 3, –6) (3, –4, 5)


(h − α ) (k − b)(l − c) − bc  + b  −al + ac − ac  – c [ab +
Ratios = 2 – 3, 3 + 4, –6 – 5 = –1, 7, –11 ak – ab] = 0
−1 7 −11 (h − α ) [kl − kc − bl ] −bal − cak =
0
Direction cosines = , ,
171 171 171
hk l − hkc − hbl − akl + akc + abl − bal − cak =
0
1 −7 11
or , , ayz + bzx + cxy =
xyz
171 171 171
2x − y + 2z + 3 (3x − 2y + 6z + 8)
m+3 4m + 1 −6m − 5 Sol 7: (A) = −
Sol 2: (A) α = , β= , γ=

Potrebbero piacerti anche